You are on page 1of 948

Critical Care Pharmacy

Evolution and Validation,


Practice Standards, Training,
and Professional Development
Bradley A. Boucher, Pharm.D., FCCP, FNAP, MCCM
University of Tennessee Health Science Center College of Pharmacy
Memphis, Tennessee

Curtis Haas, Pharm.D., FCCP


University of Rochester
Rochester, New York
Critical Care Pharmacy Evolution and Validation, Practice Standards, Training, and Professional Development

Critical Care Pharmacy


Evolution and Validation,
Practice Standards, Training,
and Professional Development
Bradley A. Boucher, Pharm.D., FCCP, FNAP, MCCM
University of Tennessee Health Science Center College of Pharmacy
Memphis, Tennessee

Curtis Haas, Pharm.D., FCCP


University of Rochester
Rochester, New York

ACCP Updates in Therapeutics® 2022: Critical Care Pharmacy Preparatory Review and Recertification Course

3
Critical Care Pharmacy Evolution and Validation, Practice Standards, Training, and Professional Development

Learning Objectives CE Continuing education


CPD Continuing professional development
1. Describe landmark events in the evolution of critical CPE Continuing pharmacy education
care pharmacy as a specialty, including summarizing ICU Intensive care unit
key published documents and evidence validating PDP Personal development plan
critical care pharmacy as a specialty. SCCM Society of Critical Care Medicine
2. Identify the core knowledge area for pharmacists
caring for critically ill patients.
3. Summarize the findings from key studies document- Self-Assessment Questions
ing the association of critical care pharmacy services Answers and explanations to these questions can be
with favorable health care outcomes. found at the end of the chapter.
4. List the criteria for credentialing and training phar-
macists who provide critical care services at the 1. Which is the journal that, in 1982, was the first to
desired and optimal levels as outlined in the 2011 publish a critical care therapeutics column?
American College of Clinical Pharmacy (ACCP) A. Drug Intelligence and Clinical Pharmacy.
critical care “PRN Opinion Paper,” in addition to B. Pharmacotherapy.
critical care training opportunities and growth. C. Chest.
5. Outline the standards of practice for clinical phar- D. Heart & Lung.
macy in the critical care practice environment
including using a standard process of care. 2. The scope of pharmacy practice within the inten-
6. Develop an approach to conducting a gap analysis sive care unit (ICU) was outlined by two task forces
relative to the principles and values of team-based focused on models of critical care delivery, the
care in a local critical care practice environment. definition of an intensivist, and the practice of criti-
7. Differentiate between the conventional and nontra- cal care medicine within three different proposed
ditional pathways of training to obtain knowledge, models in 2001. Which best depicts the professional
skills, and attitudes for critical care pharmacy prac- organization that formed these two task forces?
tice, including the key features of a mentor-mentee
A. Institute of Medicine.
(protege) relationship in the training of critical care
B. American College of Clinical Pharmacy
pharmacists.
(ACCP).
8. Develop an approach to lifelong professional learn-
C. American College of Critical Care Medicine
ing to maintain competency in critical care pharmacy
(ACCM).
practice using the principles of continuing profes-
D. Clinical Pharmacy and Pharmacology Section
sional development, including the many educational
of the Society of Critical Care Medicine
components or techniques that can be incorporated
(SCCM).
into a personal development plan.
9. Identify the avenues and processes for contributing
3. Which best depicts the medical event that has been
to the critical care body of knowledge as a presenter,
documented to be associated with a lower mortal-
author, or peer reviewer.
ity in ICUs with clinical pharmacists compared with
ICUs without clinical pharmacists?
Abbreviations in This Chapter A. Corrected QT (QTc)-interval prolongation.
B. Preventable adverse drug interactions.
ACCM American College of Critical Care Medicine C. Drug-drug interactions.
ACCP American College of Clinical Pharmacy D. Thromboembolic/infarction-related events.
ACLS Advanced cardiac life support
ASHP American Society of Health-System 4. Which best describes what is deemed a core knowl-
Pharmacists edge base area for pharmacists caring for critically
BPS Board of Pharmacy Specialties ill patients?

ACCP Updates in Therapeutics® 2022: Critical Care Pharmacy Preparatory Review and Recertification Course

4
Critical Care Pharmacy Evolution and Validation, Practice Standards, Training, and Professional Development

A. Nephrology. 9. Which statement is most accurate concerning the


B. Dermatology. mentor-mentee relationship as it pertains to the train-
C. Rheumatology. ing and development of critical care pharmacists?
D. Obstetrics. A. Formal mentoring relationships are restricted to
residency and fellowships.
5. Which most accurately reflects the journal that pub- B. Voluntary relationships that evolve and develop
lished a landmark study documenting a decrease in through mutual interests have the greatest like-
preventable adverse drug reactions after the inclu- lihood of success.
sion of pharmacists on interdisciplinary medical C. Mentored training programs are the only reli-
rounds? able pathway for the nonconventional training
A. New England Journal of Medicine. of critical care pharmacists.
B. Lancet. D. Most successful critical care pharmacists have a
C. Journal of the American Medical Association. single relevant mentor-mentee relationship dur-
D. Annals of Internal Medicine. ing their training and development.

6. There were eight American Society of Health- 10. Which is the most accurate description of the rela-
System Pharmacists (ASHP)-accredited critical care tionship between the continuing pharmacy education
pharmacy residencies in 2001. Which annual abso- (CPE) and the continuing professional development
lute value represents the approximate increase in (CPD) of clinical pharmacists?
ASHP-accredited critical care residencies between A. CPE and CPD are two distinctly different pro-
2001 and 2021? cesses for continuing development.
A. 5. B. CPD is an individualized, self-directed, and
B. 8. iterative process of development that replaces
C. 12. traditional CPE.
D. 15. C. CPE is strictly a didactic process, whereas CPD
incorporates many different learning strategies
7. Which best reflects the current conventional or and techniques.
preferred postgraduate training pathway to clini- D. CPD is an individualized, self-directed process
cal pharmacy practice in an ICU providing level I that typically incorporates relevant CPE as one
services? of the learning strategies.
A. Postgraduate year 1 (PGY1) residency with
focused critical care rotations. 11. Which statement is most accurate relative to the
B. PGY1 residency followed by on-the-job men- recently published standards of care and standard-
tored training. ized process of care for clinical pharmacy when
C. PGY1 and PGY2 critical care residency. considering critical care pharmacy practice?
D. PGY1 and critical care traineeship. A. ICUs are highly individualized practice
environments that cannot easily conform to
8. When considering the five principles of team-based broad-based, discipline-wide standards.
health care delineated in the Institute of Medicine B. Critical care pharmacists have knowledge and
discussion paper, which of the other four principles skill sets that are specific to their practice style
is effective communication most tightly linked to? and environment and are not consistent with the
A. Shared goals. standards.
B. Clear roles. C. The standards of care and standardized process
C. Mutual trust. of care are very consistent with critical care
D. Measurable processes and outcomes. pharmacy practice standards and expectations.

ACCP Updates in Therapeutics® 2022: Critical Care Pharmacy Preparatory Review and Recertification Course

5
Critical Care Pharmacy Evolution and Validation, Practice Standards, Training, and Professional Development

D. The standard process of care, which has evolved


around the “provider status” efforts, is primar-
ily applicable to the ambulatory, primary care
environment of practice.

12. Which is the best example of an audience that has


not traditionally been an important focus of critical
care pharmacists’ educational and teaching efforts?
A. Patients and families.
B. Pharmacy students.
C. Critical care physicians.
D. Critical care nurses.

13. As part of the reflection stage of the CPD process,


which would be the best example of an episodic
opportunity for self-assessment?
A. An annual 360-degree peer evaluation provid-
ing feedback that your coworker believes you
do not contribute adequately on departmental
initiatives.
B. Recognition that the usual approach to training
and assessing a challenging student on rotation
was ineffective.
C. A self-evaluation of the past year’s accomplish-
ments for your direct supervisor.
D. An annual performance evaluation with specific
goals for the coming year.

14. Which would be considered the most valid reason


for recommending the rejection of a manuscript as a
scientific reviewer?
A. Poor syntax and word choices.
B. A methodological flaw that results in incorrect
data.
C. A disagreement concerning the statistical anal-
ysis presented in the manuscript.
D. Results presented in the abstract that are incon-
sistent with results presented in a table.

ACCP Updates in Therapeutics® 2022: Critical Care Pharmacy Preparatory Review and Recertification Course

6
Critical Care Pharmacy Evolution and Validation, Practice Standards, Training, and Professional Development

BPS Critical Care Pharmacy Specialist Examination Content Outline

This chapter covers the following sections of the Critical Care Pharmacy Specialist Examination Content Outline:
1. Domain 1: Clinical Knowledge and Application
a. Task 3: 1
b. Task 5: 3
2. Domain 2: Practice Management, Policy, and Quality Improvement
a. Task 1: 1, 2
b. Task 2: 1
c. Task 5: 2
3. Domain 3: Evidence-Based Medicine, Scholarship, Education, and Professional Development
a. Task 2: 2, 3
b. Task 3: 1, 2
c. Task 5: 1, 2
d. Task 6: 1
e. Task 7: 1-4

ACCP Updates in Therapeutics® 2022: Critical Care Pharmacy Preparatory Review and Recertification Course

7
Critical Care Pharmacy Evolution and Validation, Practice Standards, Training, and Professional Development

I.  LANDMARK EVENTS IN CRITICAL CARE MEDICINE/PHARMACY

A. First ICU: Three-Bed Neurosurgical Unit in Baltimore, Maryland (1930s) (Ann Pharmacother 2006;40:612-8)

B. First Pharmacists Assigned to ICUs in a Limited Number of Hospitals – Occurred in the late 1960s (Ann
Pharmacother 2006;40:612-8)

C. Establishment of several Critical Care Pharmacists ICU Practices (in or around the 1970s; clinical research
conducted in a wide array of therapeutic specialty areas [e.g., pharmacokinetics, infectious diseases, nutrition
support, ACLS]) (Ann Pharmacother 2006;40:612-8; Practice of Critical Care Pharmacy, Rockville, MD:
Aspen Publications, 1985)
1. Cardiovascular ICUs
2. Pediatric/neonatal ICUs
3. Medical ICUs
4. Emergency medicine
5. Trauma
6. Surgical ICUs
7. Neurosurgical ICUs

D. Formation of SCCM with 100 Members (1970)


1. Multidisciplinary model stressed by founding SCCM President Max Harry Weil, M.D.
2. Subsequent inclusion of pharmacists as permanent members of the SCCM governing council

E. Emergence of Critical Care Specialty Journals and Publications


1. Heart & Lung (1972)
2. Intensive Care Medicine (1972)
3. Critical Care Medicine (1973)
4. Critical Care Clinics (1984)
5. Critical care therapeutics column in Drug Intelligence and Clinical Pharmacy (1982)
6. First critical care pharmacy textbook: Critical Care Pharmacy (1985)

F. Formation of Critical Care Pharmacists Specialty Groups


1. SCCM Clinical Pharmacy and Pharmacology Section (1989)
2. ACCP Critical Care Practice and Research Network (PRN) (1992)
3. Neurocritical Care Society (NCS), Pharmacy Section (circa 1999)

II.  VALIDATION OF CRITICAL CARE PHARMACY AS A SPECIALTY

A. Chapter Titled “Role of the Pharmacist in Caring for the Critically Ill Patient,” Published in The Pharmacologic
Approach to the Critically Ill Patient, 3rd ed. Baltimore, MD: Williams & Wilkins, 1994:156-66.

ACCP Updates in Therapeutics® 2022: Critical Care Pharmacy Preparatory Review and Recertification Course

8
Critical Care Pharmacy Evolution and Validation, Practice Standards, Training, and Professional Development

B. Publication of “Position Paper on Critical Care Pharmacy Services” (Crit Care Med 2000;28:3746-50);
establishment of three levels of pharmacy services (fundamental, desirable, optimal) for the provision of
pharmaceutical care in critically ill patients; major update in 2020 with recommendations simplified to
essential and desirable. Statements and recommendations categorized as follows: Patient Care (n=34),
Quality Improvement (n=21), Research/Scholarship (n=9), Training Education (n=10), and Professional
Development (n=8) (Crit Care Med 2020;48:1375-82) (see Table 1 for details).

C. SCCM Recognition of Critical Care Pharmacists as an Integral Member of the Multidisciplinary Team
Together with Physicians, Nurses, and Respiratory Therapists (Crit Care Med 2001;29:2007-19)

D. Scope of Pharmacy Practice Within ICU Outlined by Two ACCM Task Forces: Models of critical care
delivery and definition of an intensivist and the practice of critical care medicine within three different
proposed models (critical care pharmacy and pharmacist services deemed “essential” within level I critical
care centers as endorsed by ACCM (Crit Care Med 2003;31:2677-83) (see specifics that follow)

E. Awarding of FCCM Status by ACCM: 88 pharmacists from an overall total of around 1326; seven of total
101 MCCMs (Master Critical Care Medicine) (2021); awarding FNCS status by the Neurocritical Care
Society (n=5) Receipt of Several Honors and Awards by Critical Care Pharmacists Within SCCM, ACCM,
ACCP, and NCS
1. SCCM/ACCM: Distinguished Investigator Award, Shubin-Weil Master Clinician, Excellence in Bedside
Teaching Award, Distinguished Service Award
2. ACCP: Russell R. Miller Award, Paul F. Parker Medal, Clinical Practice Award, Robert M. Elenbaas
Service Award, Education Award
3. Creation of the Joseph F. Dasta Critical Care Pharmacy Outcomes Research Grant by the Clinical
Pharmacy and Pharmacology Section of SCCM (2000); Barry A. Shapiro Memorial Award for
Excellence in Critical Care Management

F. Leadership Roles in Major Multidisciplinary and Pharmacy Organizations


1. SCCM: President Judith Jacobi (2010)
2. ACCP: Presidents Robert Elenbaas, Barbara Zarowitz, Bradley Boucher, Curtis Haas, Judith Jacobi,
Brian Erstad
3. Neurocritical Care Society: President Gretchen Brophy (2017-2018)
4. ASHP: President: Thomas J. Johnson (2020)

G. Standards for Neurologic Critical Care Units: Inclusion of dedicated pharmacists on the neurocritical care
team (Neurocrit Care 2018;29:145-60)

III.  CRITICAL CARE PHARMACY GROWTH

A. SCCM Membership: About 17,754; over 2058 with pharmacy credentials. ACCP Critical Care PRN: about
1917 (2021)

B. Board Certified Critical Care Pharmacists (BCCCP: Around 3300 (2021)

ACCP Updates in Therapeutics® 2022: Critical Care Pharmacy Preparatory Review and Recertification Course

9
Critical Care Pharmacy Evolution and Validation, Practice Standards, Training, and Professional Development

C. Provision of Direct Patient Care Services: 62.2% of ICUs responding to hospital survey sent to pharmacy
directors identified by the American Hospital association as having an ICU (Ann Pharmacother
2006;40:612-8)

IV.  STUDIES DOCUMENTING THE ASSOCIATION OF CRITICAL CARE PHARMACY SERVICES


WITH FAVORABLE HEALTH CARE OUTCOMES

A. Reduction in Drug Costs in ICU with the Inclusion of a Pharmacist as a Member of the Multidisciplinary
Team
1. Medical-surgical ICU: Annual savings of $67,664 in 1994 dollars (Crit Care Med 1994;22:1044-8)
2. Several other studies in a wide range of ICU settings (see Intensive Care Med 2003;29:691-8)
3. Burn ICU: Annual savings of $22,162 in 2003 dollars (J Burn Care Res 2006;310-13)
4. Neurosurgical ICU: Reduction in pharmacy acquisition costs from $4833 to $3239 per patient after
the addition of a pharmacist to the neurosurgery team; reduction in ICU days from 8.56 to 7.24 days
(p=0.003) (Neurosurgery 2009;65:946-50; discussion 950-1)
5. Scoping review conducted for cost avoidance generated by clinical pharmacists on interventions
performed in and ICU or emergency department identified 38 distinctive categories (Pharmacotherapy
2019;39:215-31)

B. Reduction in Adverse Drug Effects/Drug-Drug Interactions


1. Decrease in preventable adverse drug effects after the inclusion of a pharmacist on interdisciplinary
medical ICU rounds: 66%, (p<0.001) (JAMA 1999;282:267-70); supported by meta-analysis of three
studies (odds ratio (OR) 0.23, 95% confidence interval (0.11, 0.48) (J Crit Care 2015;30:1101-06).
Reduced; preventable and nonpreventable adverse drug events (OR 0.26, 95% CI (0.15, 0.44), p<0.0001
and OR 0.47, 95% CI (0.28, 0.77, p=0.003, respectively) (Crit Care Med 2019;47:1243-50).
2. Decreased incidence of QTc-interval prolongation with ICU monitoring by a pharmacist using a standard
algorithm: 19% versus 39% (p=0.006) (Ann Pharmacother 2008;42:475-82)
3. Reduction in drug-drug interactions by 65% (p<0.01) in medical ICUs with a pharmacist (J Crit Care
2011;26:104.e101-106)

C. Improvement in Infectious Diseases Morbidity, Mortality, and Costs (Crit Care Med 2008;36:3184-9)
1. Mortality higher in ICUs without clinical pharmacists than in ICUs with clinical pharmacists: 23.6% for
nosocomial-acquired infections in ICUs without clinical pharmacists (p<0.001), 16.2% for community-
acquired infections in ICUs without clinical pharmacists (p=0.008), 4.8% for sepsis in ICUs without
clinical pharmacists (p≤0.008)
2. Length of stay longer for ICUs without clinical pharmacists than for ICUs with clinical pharmacists:
7.9% for nosocomial-acquired infections in ICUs without clinical pharmacists (p<0.001), 5.9% for
community-acquired infections in ICUs without clinical pharmacists (p=0.03), 8.1% for sepsis in ICUs
without clinical pharmacists (p<0.001)
3. Medicare billings increased in ICUs without clinical pharmacists compared with ICUs with clinical
pharmacists: 12% for nosocomial-acquired infections in ICUs without clinical pharmacists, 11.9% for
community-acquired infections in ICUs without clinical pharmacists, 12.9% for sepsis in ICUs without
clinical pharmacists (p<0.001)

ACCP Updates in Therapeutics® 2022: Critical Care Pharmacy Preparatory Review and Recertification Course

10
Critical Care Pharmacy Evolution and Validation, Practice Standards, Training, and Professional Development

D. Improvement in Thromboembolic and Infarction-Related Event (TIE) Clinical and Economic Outcomes
(Pharmacotherapy 2009;29:761-8)
1. Mortality increased in ICU patients with TIE without clinical pharmacy services compared with ICU
patients with clinical pharmacy services: 37%, (p<0.0001).
2. Bleeding complications increased by 49% (p<0.001), with 39% more patients receiving transfusions
(p=0.006) in ICUs without clinical pharmacy services.
3. Length of ICU stays and costs were significantly higher in patients with TIE in ICUs without clinical
pharmacy services.

E. Meta-Analysis of 14 Studies of Critical Care Pharmacists as Member of Multidisciplinary Team (Crit Care
Med 2019;1243-50) Reduced mortality: OR 0.78, 95% CI (0.73-0.83, p<0.00001), LOS reduction: mean -1.33
days, 95% CI (-1.75, -0.90), p<0.00001)

F. Impact of ICU Protocols on Patient Outcomes


1. Significant improvement in sedation and analgesia monitoring targets with the use of protocol versus
empiric therapy (p≤0.01); no difference in length of ICU stay (Pharmacotherapy 2000;20:662-72)
2. Pharmacist-enforced ICU sedation protocol reduced mechanical ventilator duration as well as ICU,
hospital length of stay (p<0.001) (Crit Care Med 2008;36:427-33)
3. Improvement Mortality with Inclusion of Clinical Pharmacist Following Multicomponent Intervention
in Tertiary Care Medical ICU (also included increase in ICU beds, larger rooms, 24-hour critical care
specialist coverage, decrease in respiratory therapist/patient ratio) (Crit Care Med 2011;39:284-93)
a. ICU Mortality decrease from 18.4% to 14.9% (p=0.006), hospital mortality decrease 25.8% to
21.7% (p=0.005)
b. Increase in median ICU length of stay; no difference in hospital length of stay
c. Increase in median 28-day ventilator-free days in mechanically ventilated patients
d. Mean decrease in daily dosing of fentanyl and lorazepam
4. Pharmacist management of pain, agitation, and delirium in ICU through multidisciplinary bundle; 46%
reduction in continuous sedation, reduction in ICU, hospital length of stay (Am J Health-Syst Pharm
2017;74:253-62)
5. Pharmacist involvement in multidisciplinary initiative to reduce sepsis-related mortality in ICU (i.e.,
“Code Sepsis”); reduction in mean time from sepsis screen to antibiotic administration from 427 minutes
to 31 minutes (Am J Health-Syste Pharm 2016;73(3):143-9)
6. Leadership role in developing clinical practice guidelines for preventing and managing pain, agitation/
sedation, delirium, immobility, and sleep disruption in adult critically ill patients (PADIS Guidelines)
(Crit Care Med 2018;46:e825-73)
7. Before and after study of critical care pharmacist interventions on drug therapy and clinical strategies:
Reduction in hospital length of stay (mean 3.7 days, p<0.001), ICU length of stay (1.4 days, p<0.01),
duration of mechanical ventilation (1.2 days, p<0.01), hospital costs per stay (2560 euros, p<0.001); no
impact on mortality. (Crit Care Med 2018;46:199-207)
8. Retrospective cohort study of albumin use in critically ill patients at single academic medical center:
Reduction in inappropriate albumin use (50.9%, p<0.001), total annual cost savings greater than
$355,000. (Ann Pharmacotherapy 2020;54:105-112).

ACCP Updates in Therapeutics® 2022: Critical Care Pharmacy Preparatory Review and Recertification Course

11
Critical Care Pharmacy Evolution and Validation, Practice Standards, Training, and Professional Development

V.  PRACTICE STANDARDS FOR CRITICAL CARE PHARMACY

A. Standards of Practice for Clinical Pharmacists (Pharmacotherapy 2014;34:794-7): The Standards of Practice
for Clinical Pharmacists were published by ACCP, incorporating a standardized process of care endorsed
by all major pharmacy organizations. This document defines expectations of clinical pharmacists delivering
comprehensive medication management in team-based, collaborative practice settings, including the ICU.
1. Qualifications
a. Licensed pharmacists
b. Advanced education, training, and experience
i. Advanced, accredited residency in critical care pharmacy (PGY2);
ii. Fellowship in critical care research; or
iii. Equivalent, relevant clinical experience
c. Clinical and personal competencies to practice in a team-based collaborative environment
d. Board certification
2. Process of care
a. Assess the patient.
b. Evaluate medication therapy.
c. Develop and implement therapeutic plan.
d. Provide follow-up evaluation and monitoring.
e. Document clinical activities.
i. Medication history
ii. Problem list and assessment
iii. Plan of care and follow-up
3. Collaborative, team-based care and privileging
4. Professional development and maintenance of competence
a. Board certification and recertification
b. CPE
c. Maintenance of licensure
d. Participation in formal and informal development activities
5. Professionalism and ethics – Demonstrate the traits of:
a. Responsibility
b. Commitment to excellence
c. Respect for others
d. Honesty and integrity
e. Care and compassion for others
f. High ethical standards
g. Legal and regulatory compliance
6. Research and scholarship
7. Other
a. Education and training
b. Mentorship
c. Management and leadership
d. Policy and service development and implementation
e. Consultation

ACCP Updates in Therapeutics® 2022: Critical Care Pharmacy Preparatory Review and Recertification Course

12
Critical Care Pharmacy Evolution and Validation, Practice Standards, Training, and Professional Development

B. Scope of Critical Care Pharmacy Services (Crit Care Med 2020;48:1375-82; Position Paper on Critical Care
Pharmacy Services [a joint effort of SCCM, ACCP, and ASHP])
1. The task force provided 82 recommendation statements across five distinct domains (see Table 1 for
details):
a. Patient Care (n=34)
b. Quality Improvement (n=21)
c. Research/Scholarship (n=9)
d. Training/Education (n=10)
e. Professional Development (n=8)
2. Gradations of pharmacy practice
a. Essential: These are practice recommendations that are considered vital to the provision of pharmacy
care to ICU patients.
b. Desirable: Offers recommendations that are more specialized and specific to the ICU beyond the
essential recommendations
c. Essential versus desirable recommendation statements are further categorized based on the level of
ICU care being provided (Level I, II or III).

C. Critical Care Pharmacist as Educator: The critical care pharmacist has several educational missions and
obligations (Pharmacotherapy 2011;31:135e-175e; Ann Pharmacother 2006;40:612-8; Pharmacotherapy
2002;22:1484-8; Crit Care Med 2000;28:3746-50 ; Crit Care Med 2020;48:1375-82), and teaching methods
and techniques vary depending on intended audience and content. The clinical pharmacist must develop
comfort and expertise with a wide range of teaching styles and techniques to be successful as an educator in
the ICU setting.
1. Pharmacy students and residents: Content has to be at a level appropriate to learners who may or may
not have a primary interest in critical care. Active learning strategies must be incorporated with didactic
approaches that are more traditional. For this audience, the clinical pharmacist has primary responsibility
for assessment/grading.
a. Clinical practice training
i. Role modeling (I do, you watch)
ii. Coaching (I do, you help … then … you do, I help)
iii. Mentoring (You do, I watch)
b. Case-based teaching (point-of-care teaching)
c. Hands-on demonstrations of equipment, technology, and devices used in the ICU
d. Clinical conferences/topic discussions
e. Assigned readings
f. Journal club
g. Quality improvement projects
h. Writing assignments
i. Case reports
ii. Guideline/protocol development
iii. Pharmacy and therapeutics (P&T) monographs
i. Drug information response
j. Medication use evaluations
2. Critical care team: More heavily focused on the specifics of critical care therapeutics. Content and
sophistication will vary depending on audience (e.g., physicians vs. nurses). Audience is assumed to
have a primary interest in critical care.
a. Case-based, point-of-care teaching (bedside rounds)
b. Didactic teaching

ACCP Updates in Therapeutics® 2022: Critical Care Pharmacy Preparatory Review and Recertification Course

13
Critical Care Pharmacy Evolution and Validation, Practice Standards, Training, and Professional Development

i. Teaching rounds/conferences
ii. In-service education
iii. Grand rounds
iv. Basic science lectures
c. Critical care–specific journal club
d. Collaboration on guidelines/protocols
e. Quality improvement projects
3. Pharmacist colleagues: May not have a primary focus or interest in critical care. Content may be focused on
specific pharmacotherapeutic issues (e.g., pharmacokinetic principles in the critically ill) that often arise
during cross-coverage. May include pharmacists taking a nonconventional path to critical care practice.
a. Didactic lectures (e.g., clinical conferences, topic-specific lectures)
b. Hands-on demonstration of equipment, technology, and devices used in the ICU
c. Case-based, point-of-care teaching
d. Journal club
e. Competency-based programs – Lead to credentialing according to demonstrated skills
4. Other trainees: Often includes a mix of backgrounds and interests (e.g., critical care fellows, anesthesia
residents, medicine residents, emergency department (ED) residents, fourth-year medical students,
nursing students, advanced-practice provider (APP) students, and dietary students). Content has to be
appropriate for the predominant audience and baseline understanding of the topic.
a. Didactic lectures
i. Teaching rounds
ii. Clinical conferences
b. Point-of-care teaching – Bedside rounds
5. Patients and families: Education of patients and family members has not been a traditional realm of
clinical pharmacist involvement in the ICU because of an assumption that patients were not awake and
alert enough and that patients were almost never discharged from the ICU. However, with an increasing
emphasis on patient and family satisfaction and a greater involvement of the patient and family in
decision-making, the need to educate around pharmacotherapy in the ICU has increased. An emerging
area of critical care pharmacist involvement is patient and family care and education in post-ICU clinics
to address the challenges associated with post-ICU care for those who survive their ICU admission. The
SCCM-sponsored THRIVE initiative promotes the growth of these interprofessional clinics.
a. Techniques
i. Simple language, basic content
ii. Teach-back technique to assess understanding
iii. Frequent reinforcement
iv. Motivational interviewing techniques
v. Open-ended questions to understand what is important to the patient and family
b. Content
i. Medications being initiated in the ICU
ii. Why the medication is being used – Have goals different from home medications
iii. What to expect – Effects, adverse effects, changes in patient interaction, etc.
iv. Expected duration of new medications
v. What factors are monitored to see whether medications are helping or hurting

ACCP Updates in Therapeutics® 2022: Critical Care Pharmacy Preparatory Review and Recertification Course

14
Critical Care Pharmacy Evolution and Validation, Practice Standards, Training, and Professional Development

D. Critical Care Services (Crit Care Med 2003;31:2677-83): ACCM recommended critical care services and
personnel according to the level of care being provided. ICUs were defined as levels I, II, and III.
1. Levels of ICU services
a. Level I
i. Comprehensive critical care for a wide variety of patient populations with a high level of
specialization
ii. Requires broad range of comprehensive support, including pharmacy services, respiratory
therapy, clinical nutrition, pastoral care, and social services
iii. Often fulfills an academic mission
b. Level II
i. Comprehensive critical care but may not provide care for certain patient populations
ii. Must have transfer protocols in place for patients with special needs
iii. Comprehensive support services must be available.
iv. May or may not have an academic mission
c. Level III
i. Provides stabilization, but has limited ability to provide comprehensive critical care
ii. Must have transfer protocols in place for patients requiring level I and II critical care services
iii. Support services are often limited in scope.
2. Critical care pharmacy services (level I and II ICUs)
a. Reiterates pharmacist and pharmacy services defined in 2000 guideline (Crit Care Med
2000;28:3746-50)
b. Emphasizes the importance of clinical pharmacists as required members of the patient care team
c. Qualifications and competence of the critical care pharmacist in ICU therapeutics are defined as
essential. Acknowledges several pathways, including advanced degrees, residency, fellowship, and
other specialized practice experiences.
d. ICUs with an academic mission should provide protected time for pharmacist participation in
scholarly activities and appropriate knowledge and skills to provide education to critical care nurses,
physician trainees, and physicians.
e. Nonacademic centers should provide time for maintenance of competence and maintain current
certification.

E. Principles and Values of Team-Based Health Care


1. SCCM and ACCP have long promoted the team-based care model for critical care as a standard,
including clinical pharmacists as essential staff.
2. An Institute of Medicine discussion paper delineated the core principles and values of highly functioning
interprofessional health care teams (see www.iom.edu/Global/Perspectives/2012/TeamBasedCare.
aspx).
a. Definition of team-based care: Team-based health care is the provision of health services to
individuals, families, and/or their communities by at least two health providers who work
collaboratively with patients and their caregivers—to the extent preferred by each patient—to
accomplish shared goals within and across settings to achieve coordinated, high-quality care.
b. Five personal values of effective members of high-functioning teams:
i. Honesty: Includes effective, transparent communication. Essential to building mutual trust.
ii. Discipline: Each team member carries out roles and responsibilities in a highly disciplined
approach, even when inconvenient or difficult.
iii. Creativity: Maintains excitement around addressing new and difficult challenges. Sees
opportunity in both successes and failures.

ACCP Updates in Therapeutics® 2022: Critical Care Pharmacy Preparatory Review and Recertification Course

15
Critical Care Pharmacy Evolution and Validation, Practice Standards, Training, and Professional Development

iv. Humility: Equal respect of all members, regardless of level of training or role – Not tied to
traditional hierarchical thinking in health care. Recognizes that all members of the team are
susceptible to mistakes.
v. Curiosity: Dedicated to reflection and continuous improvement
c. Five principles of team-based health care
i. Shared goals: Clearly articulated, understood, and supported goals are established by the team
that are consistent with the patient and family wishes. The patient and family are actively
involved in establishing the goals of care as members of the team.
ii. Clear roles: Each team member’s functions, responsibilities, and accountabilities are clearly
established and understood by the team. Efficiency and logical division of labor are achieved.
Although autonomy is important, flexibility of roles and collaboration exist as needed.
iii. Mutual trust: Establishing and maintaining trust, as well as openness to address questions
about or breaches of trust, are essential. Mutual trust permits individual team members to
function to their highest potential and rely on other team members to follow through on their
commitments.
iv. Effective communication: Tightly linked to mutual trust. The team has consistent channels for
candid and complete communication by all team members and in all situations.
v. Measurable processes and outcomes: The team develops and implements accurate and timely
measures of successes and failures and uses the results to track and improve performance.
Measures fall into two categories: Process/outcome measures and measures of team function.
This principle is typically the most challenging for a team to implement and sustain effectively.
3. Critical care teams – Gap analysis: When considering the core principles of team-based care, critical
care team members should evaluate their team structure and performance against these five principles.
Effective teams are much more than patient care rounds by a mix of health care professionals. Common
questions to consider when evaluating potential gaps should include:
a. Shared goals
i. Are the patient and family goals for critical care routinely incorporated into the care plan?
ii. Are the patient and family viewed as active members of the team during the establishment of goals?
iii. Are there clearly articulated and understood goals that are agreed on by all members of the
team during the provision of care to all ICU patients and the work of the team in the care of
that patient?
iv. Is progress toward the goals routinely reevaluated in light of the changing course and evolving
perspective of the patient and family? Are goals adjusted or refined throughout the dynamic
course of the critical care admission as needed?
v. Are there adequate organizational resources and commitments to permit effective establishment
of shared goals in the treatment of ICU patients?
b. Clear roles
i. Are each team member’s functions, responsibilities, and accountabilities clearly defined? Can
each team member articulate and understand the role of the other team members?
ii. Are the roles and responsibilities of each team member focused on the shared goals of the team
and patient?
iii. Is there clear respect for the contributions of each team member from a nonhierarchical,
interdependent perspective?
iv. Is each team member introduced (and reintroduced) to the patient and family, including a lay
description of each member’s role and responsibility?
v. Does each team member go about his or her responsibilities with a reasonable degree of
autonomy?
vi. Is there a clear team leader? Does the leadership role vary according to individual circumstances,
problems, or environment?

ACCP Updates in Therapeutics® 2022: Critical Care Pharmacy Preparatory Review and Recertification Course

16
Critical Care Pharmacy Evolution and Validation, Practice Standards, Training, and Professional Development

vii. Does the team have a reasonable balance between autonomous functions and collaboration?
viii. Are there adequate organizational resources and commitments for professional development,
team education, facilitation of communication, and restructuring of care processes to support
the effective division of labor?
c. Mutual trust
i. Does an environment of mutual trust and support exist among the ICU team? Can breaches of
trust be openly discussed and addressed between team members without a detrimental impact
on professional or personal relationships?
ii. Does the hiring process include a focus on the personal and professional values that support an
environment of mutual trust? Do members of the ICU team participate in the hiring process
across traditional departmental siloes?
iii. Is the team effective at establishing and maintaining mutual trust with patients and families?
Are effective communication skills used to explain the process of establishing goals, sharing
information on progress, and incorporating effective negotiation and conflict resolution skills?
iv. Does the team regularly participate in non–patient care activities (e.g., social interaction) that
allow team members to develop greater trust and know each other at many levels?
v. Is there adequate organizational support of the elements necessary to establish mutual trust
among teams?
d. Effective communication
i. Has the team established a high priority for open, direct, clear, consistent, professional
communication between team members?
ii. Does communication take advantage of all potential modes and technologies of communication
for efficiency and convenience?
iii. Do members of the team use effective listening skills, recognizing that deep listening to
the input of all team members, including patients and families, is an essential component of
effective communication?
iv. Are signs of tension and unspoken conflict in the communication process regularly recognized
and addressed to improve team communication skills and effectiveness?
v. Does effective communication occur across the team regardless of traditional hierarchical
structures in health care?
vi. Are the organizational elements for effective communication available to the team?
e. Measurable processes and outcomes
i. Has the team identified and implemented reliable, timely, and ongoing measures of team
performance?
ii. Are these measures focused on both process/outcomes of care provision and team function or
effectiveness?
iii. Are measures of patient and family satisfaction included in the assessment process?
iv. Are measures of team member satisfaction included in the team assessment?
v. Does the team regularly report its measures of success and failure, both internally to the team
and to others in the organization?
vi. Are performance data regularly used for process improvement with respect to both patient care
and team function?
vii. Does the team use any standardized tools to assess team function and quality?
viii. Are organizational resources and commitment adequate to permit teams to adequately measure
quality of patient care and team function?

ACCP Updates in Therapeutics® 2022: Critical Care Pharmacy Preparatory Review and Recertification Course

17
Critical Care Pharmacy Evolution and Validation, Practice Standards, Training, and Professional Development

F. Other Standards
1. The Joint Commission
a. Medication management chapter
i. High-alert, hazardous medication standards
ii. Look-alike/sound-alike medications
iii. Monitoring of medication response
iv. Adverse drug event detection, evaluation, and reporting
v. Duplicate therapy
b. National Patient Safety Goals
i. Two-factor patient identification
ii. Medication reconciliation
iii. Safe medication use and labeling
iv. Anticoagulation management and education
v. Antimicrobial stewardship program
2. Centers for Medicare & Medicaid Services (CMS) conditions for participation (42 CFR 482)
a. Quality assurance and performance improvement programs (§482.21)
i. Medical errors
ii. Adverse events
b. Preparation and administration of medications (§482.23)
c. Medical records requirements (§482.24)
d. Pharmaceutical services (§482.25)
i. Policies and procedures to minimize drug errors
ii. Adverse drug reaction and medication error detection and reporting
iii. Drug information standards

VI.  TRAINING OF CRITICAL CARE PHARMACISTS

A. Positions and Policy


1. ACCP position
a. ACCP clarified its position concerning qualifications of clinical pharmacists providing direct
patient care in a 2013 Board of Regents commentary (Pharmacotherapy 2013;33:888-91): Clinical
pharmacists providing direct patient care “should possess the education, training, and experience
necessary to function effectively, efficiently, and responsibly in this role. Therefore, ACCP believes
that clinical pharmacists engaged in direct patient care should be board certified (or board eligible
if a Board of Pharmacy Specialties [BPS] certification does not exist in their area of practice) and
have established a valid collaborative drug therapy management (CDTM) agreement or have been
formally granted clinical privileges by the medical staff or credentialing system within the health
care environment in which they practice.”
b. Board certification
i. ACCP considers BPS certification the cornerstone of eligibility for direct patient care.
ii. Eligibility
(a) Graduate of an accredited school of pharmacy
(b) Pharmacy licensure
(c) Postgraduate residency training in area of specialization or 3–4 years of relevant experience
with at least 50% of time practicing in the specialty area

ACCP Updates in Therapeutics® 2022: Critical Care Pharmacy Preparatory Review and Recertification Course

18
Critical Care Pharmacy Evolution and Validation, Practice Standards, Training, and Professional Development

iii. ACCP has expressed that postgraduate residency training is the preferred training pathway for
clinical pharmacists providing direct patient care in previous position statements and a white
paper.
2. ASHP policy
a. Policy 2027: “Pharmacists who provide direct patient care should have completed an ASHP-
accredited residency or have attained comparable skills through practice experience.”
b. ASHP has no policy directly related to the provision of direct patient care in a specialty practice
area.

B. Potential Workforce Demands


1. Hospital data
a. 6090 hospitals in the United States – 919,559 staffed beds (2019 American Hospital Association
survey data; https://www.aha.org/statistics/fast-facts-us-hospitals)
b. According to 2018 AHA survey data, there are 107,276 adult, pediatric, and neonatal ICU beds in
community hospitals across the United States (www.aha.org/statistics/fast-facts-us-hospitals).
2. Critical care pharmacists
a. No accurate database to indicate the number of pharmacists spending 50% or more of time in
critical care
b. Direct patient care by pharmacist provided in 62.2% of ICUs in the United States in 2006. This
represents primarily fundamental-level services (Ann Pharmacother 2006;40:612-8). A more recent
survey of 1220 institutions revealed 70.8% of ICUs had direct clinical ICU pharmacy services,
which is slightly less than an increase of 9% compared with 2006 (Crit Care Explorations 2021;3:1-
11).
c. A recent survey of 185 critical care pharmacists indicated that the pharmacist/patient ratio is
variable, with 84% of respondents caring for more than 15 patients. In addition, 30% of respondents
expressed concern about safety relative to workload, and 50% indicated they felt overworked. This
survey suggests that ICUs with pharmacist coverage may be understaffed (JACCP 2020;3:68-74).
d. A review of ACCP, SCCM, ASHP, and American Pharmacists Association (APhA) membership
records identified 2928 individual pharmacists indicating specialization in critical care at the time
of the petition for recognition of critical care as a specialty (https://www.accp.com/docs/positions/
petitions/Final_CRITICAL_CARE_PETITION_For_BPS_Post.pdf).
e. A survey sent to those 2928 pharmacists yielded 504 responses (https://www.accp.com/docs/
positions/petitions/Final_CRITICAL_CARE_PETITION_For_BPS_Post.pdf).
i. Out of the responses, 476 reported that they practiced in critical care (94%).
ii. Of those 476, 91% responded that their practice met the definition of critical care pharmacy as
a specialty.
iii. Among the respondents, 74% indicated they spent at least 50% of their time practicing in the
ICU.
iv. More than 80% of respondents completed residency or fellowship training in critical care.
f. A survey of employers of ICU pharmacists yielded 204 responses (https://www.accp.com/docs/
positions/petitions/Final_CRITICAL_CARE_PETITION_For_BPS_Post.pdf).
i. Collectively employed 1034 full-time equivalent (FTE) critical care pharmacists
ii. Recruited 256 critical care pharmacists during the previous 3 years
iii. Estimated a need to hire 234–243 critical care pharmacists in the next 3 years
iv. Of the respondents, 99.5% estimated the demand for critical care pharmacists to grow or remain
stable at their site during the next 5 years.
g. Critical Care Societies Collaborative (CCSC) – http://ccsconline.org/workforce
i. Collaborative effort of several stakeholder organizations in critical care to define the workforce
shortage in critical care and advocate for federal action to address the problem

ACCP Updates in Therapeutics® 2022: Critical Care Pharmacy Preparatory Review and Recertification Course

19
Critical Care Pharmacy Evolution and Validation, Practice Standards, Training, and Professional Development

ii. Most of this work has focused on intensivist and ICU nurse shortages, but there is also
recognition of shortages of other professionals, including critical care pharmacists.
h. Current and objective quantification of critical care pharmacist shortage or demand is unavailable.

C. Training Recommendations and Capacity


1. Minimum requirements for all levels of ICU service (I–III)
a. Graduate of Accreditation Council for Pharmacy Education (ACPE)-accredited school or college
of pharmacy
b. Licensure and registration by a state board of pharmacy
2. Conventional or preferred postgraduate training
a. PGY1 pharmacy practice residency based in a hospital
b. PGY2 critical care residency or fellowship
3. Nontraditional alternative paths: There is no widely accepted or clearly defined alternative pathway
to specialty experience and competence in critical care pharmacy. Some potential pathways and
components of a self-directed training program are outlined in the text that follows. The extent and
variety of experiences needed may be determined by the practice setting, level of care to be provided,
baseline knowledge, availability and willingness of qualified mentors, and other personal and professional
skills of the individual. Although many potential paths are defined later, those that provide continued,
practical experience during a prolonged period in a supervised or mentored environment are considered
of greatest value in developing competency in the ICU setting.
a. Mentored or supervised clinical practice experience without residency
i. Clinical practice experience must be hands-on and team based under supervision.
ii. Mentors may be PGY2- or fellowship-trained critical care pharmacists, clinical pharmacists
with equivalent experience, critical care faculty from affiliated schools of pharmacy, intensivist
physicians, and/or other critical care professionals.
iii. Several mentors may best meet the variety of needs of the mentee pharmacist.
iv. Reinforced by frequent reading and analysis of the critical care primary and secondary literature,
journal club participation, and frequent critical discussions of the clinical implications of the
primary literature
v. Normally, expect at least 3–4 years of mentored/supervised experience to gain competency for
independent clinical practice (optimal services) in level I and II ICUs. Shorter periods may be
adequate to provide lower levels of service to level II and III ICUs.
b. PGY1 with supervised/mentored ICU clinical practice experience
i. Mentored clinical experiences similar to those described earlier
ii. PGY1 with critical care experiences during residency may be adequate to provide fundamental
and desirable services to level II and III ICUs.
iii. Normally, expect 2–3 years of mentored/supervised experience to gain competency for
independent clinical practice (optimal services) in level I and II ICUs.
c. Critical care traineeship (https://www.ashpfoundation.org/leadership-development/traineeships)
i. Offered through the ASHP Foundation
ii. Four-month distance education component – Independent reading, web-based education, and
teleconference case studies
iii. Two-week on-site experiential training
iv. Post-experiential training activities
v. Is not a comprehensive training program but can be a valuable component of a training program
for nontraditional-path clinical pharmacists
d. Other potential components of a nonconventional training program: Actual program structure will
vary depending on the available resources, practice environment, baseline knowledge and skills of
the pharmacist, and institutional support.

ACCP Updates in Therapeutics® 2022: Critical Care Pharmacy Preparatory Review and Recertification Course

20
Critical Care Pharmacy Evolution and Validation, Practice Standards, Training, and Professional Development

i. Graduate degree (e.g., master’s degree)


ii. Continuing education (CE) programming – Live, web based, print
iii. Attendance at national and regional critical care meetings – CE, networking, research
presentations
iv. Fundamental Critical Care Support course completion (www.sccm.org/Fundamentals)
v. ACLS, advanced trauma life support (ATLS), and/or pediatric advanced life support (PALS)
training and certification
vi. Regular participation in the SCCM Clinical Pharmacy and Pharmacology Section national
journal club
vii. SCCM Clinical Pharmacy and Pharmacology Section mentor program – Long-distance
mentoring program
viii. Self-arranged experiential rotations at peer institutions under the supervision of a qualified
critical care pharmacist
ix. Visiting professor or scholar programs to bring specialized expertise to the clinical site for on-
site experiential training and didactic teaching
x. Policy, guideline, and protocol development for critical care pharmacotherapy–related issues
under the supervision of qualified peers
xi. Critical care pharmacy service or program development, implementation, and outcome
measurement under the supervision of qualified peers
4. PGY2 residency and fellowship programs
a. First critical care pharmacy residency described: 1981 (The Ohio State University)
b. ASHP critical care pharmacy residency standards published in 1990
c. 168 ASHP-accredited critical care residencies in 2021; increased from eight in 2001 and from 39
in 2005
d. Most PGY2 critical residents somewhat or very satisfied (91% and 76%, respectively) with their
program and mentorship according to 2012 survey
e. Critical care pharmacy research training: Long history of fellowship training; however, the ACCP
fellowship directory lists three fellowship programs with a primary or secondary focus on critical
care.

D. Mentoring and Critical Care Pharmacy Training (Am J Pharm Educ 2013;77:1-7; Am J Pharm Educ
2003;67:1-7)
1. Mentor-protégé relationship
a. Symbiotic, nurturing relationship between two adults
b. Assist each other in meeting shared career objectives
c. Attributes of a successful mentor-protégé relationship (see Box 1)
d. Mentor typically 15–20 years older than protégé
2. Mentor should fulfill five functions:
a. Teaching – New knowledge, skills, and attitudes
b. Sponsoring – Helps protégé reach career goals, assists in networking, vouches for abilities, offers
protection from threats
c. Encouraging – Affirming, challenging, inspiring
d. Counseling – Listening, probing, advising during difficult challenges
e. Befriending – Acceptance, understanding, and trust
3. Phases of the mentor-protégé relationship:
a. Initiation phase
i. Weeks to months in duration
ii. Begin work together
iii. Mentor coaches protégé, and protégé may provide technical assistance.

ACCP Updates in Therapeutics® 2022: Critical Care Pharmacy Preparatory Review and Recertification Course

21
Critical Care Pharmacy Evolution and Validation, Practice Standards, Training, and Professional Development

b. Cultivation phase
i. 2–5 years in duration
ii. Both individuals realize personal and professional benefits.
iii. Deeply intimate and personal bonds are formed.
c. Separation phase
i. Typically months in duration
ii. Protégé no longer requires guidance and begins to seek more autonomy.
iii. Mentor may think they have been deserted, whereas protégé may believe they are being held
back.
iv. Resentment or hostility may lead to end of relationship.
d. Transformation phase
i. Years in duration (lifelong)
ii. Peer relationship evolves.
iii. Mutual sense of gratitude and appreciation
4. Voluntary versus arranged relationships
a. Increasingly, organizations are establishing mentoring programs with assigned mentors.
b. Successful mentoring relationships are voluntary and based on mutual respect.
c. Successful and powerful people are not necessarily good mentors.
d. The factors that lead to mentor-protégé relationships are unclear and may be difficult to create
through assignment of mentors.
e. Factors that contribute to successful mentorship:
i. Common interests
ii. Common purpose
iii. Desire on the part of the mentor to participate
iv. Mentor and protégé must be able to spend time together.
v. Persistent and regular interaction between mentor and protégé
f. Formal mentoring programs can be successful, but less so than voluntary relationships.
5. Mentoring and critical care training
a. Beyond formal residency/fellowship programs, mentor-protégé relationships are essential to the
formal development of critical care pharmacists.
b. Developing critical care pharmacists should seek out mentors with similar interests and purpose
who can help them fill gaps in their knowledge, skills, and attitudes relative to critical care practice.
c. Over time, critical care pharmacists may have several mentor-protégé relationships to meet evolving
educational and experiential needs.
d. Experienced and successful critical care pharmacists should volunteer to mentor junior pharmacists,
residents, and students and take their roles as mentors seriously by being kind, helpful, supportive,
and encouraging.
e. SCCM Clinical Pharmacy and Pharmacology Section mentoring services: Practice, education,
administration, scholarship

ACCP Updates in Therapeutics® 2022: Critical Care Pharmacy Preparatory Review and Recertification Course

22
Critical Care Pharmacy Evolution and Validation, Practice Standards, Training, and Professional Development

VII.  CONTINUING PROFESSIONAL DEVELOPMENT (https://www.acpe-accredit.org/continuing-


professional-development/; Resident Survival Guide. Lenexa, KS: ACCP, 2011:123-37; Am J Health Syst
Pharm 2004;61:2069-76)

A. General Considerations
1. Lifelong learning by health care professionals is both a necessity and an obligation to several stakeholders.
2. CPD is a multifaceted, self-directed, holistic, outcomes-focused approach to lifelong learning.
3. CPD is a career-long iterative process with continuous cycles, rather than a start and a finish.
4. Sustained career growth and success are more dependent on CPD than on early career education and
training.
5. CPD should be closely integrated into daily practice and the work environment for success and
sustainability.

B. Stakeholders in CPD: Stakeholders may have a role in contributing to lifelong learning, benefiting from the
sustained competency of the clinical pharmacist, or both.
1. Pharmacist-learner (self)
a. Most at stake
b. Primarily responsible for developing a self-directed, structured approach to learning and assessment
c. Must develop an approach that is flexible, integrated, and capable of being sustained throughout
decades of practice
d. Must be prepared to commit personal time to CPD
2. Employer
a. Has both an obligation to and an expectation of the clinical pharmacist relative to CPD
b. Provision of resources
i. Travel funding
ii. Access to electronic databases and literature
iii. Environment that promotes sharing and learning (clinical conferences, journal club, open
discussion and debate among colleagues, etc.)
iv. Protected time to pursue educational opportunities
c. Establish a credentialing and privileging process that incorporates CPD expectations.
d. Aligning personal development goals with institutional priorities is mutually beneficial and may
increase employer support.
e. Employer benefits from sustained and expanded competencies of clinical pharmacist and should
incorporate into hiring, retention, and promotion decisions
3. Colleagues
a. Contribute to lifelong learning of the clinical pharmacist
i. Case-based discussion and debate on daily rounds
ii. Drug-related questions
iii. Interdisciplinary teaching rounds
iv. Clinical conferences, journal clubs
v. Inclusion in collaborative scholarly activities
b. Benefit from lifelong learning of the clinical pharmacist
i. Greater quality and sophistication of contributions to team-based care of critically ill patients
ii. Educational offerings by the clinical pharmacist
iii. Collaboration around scholarly activities
iv. ICU-related treatment guidelines and protocols developed by or in collaboration with the
clinical pharmacist

ACCP Updates in Therapeutics® 2022: Critical Care Pharmacy Preparatory Review and Recertification Course

23
Critical Care Pharmacy Evolution and Validation, Practice Standards, Training, and Professional Development

4. Students, residents, and fellows


a. Contribute to lifelong learning of the clinical pharmacist
i. Assisting in identifying gaps in their own knowledge
ii. Creating incentive to maintain competency through CPD
iii. Regularly challenging applicability and relevance of professional knowledge and skills
b. Benefit from current, relevant knowledge and skills being incorporated into:
i. Teaching
ii. Role modeling/coaching
iii. Mentoring
c. CPD is a lifelong obligation of pharmacists who accept responsibility for training future clinical
pharmacists.
d. The best trainees seek out the most competent teachers, preceptors, and mentors.
5. Patients
a. Greatest beneficiary of clinical pharmacist CPD
b. Providing the best possible care to ICU patients should be the biggest motivator for the clinical
pharmacist to pursue CPD.
c. Well-informed patients will seek out the most competent and capable health care professionals.

C. CPD Process: The CPD process is structured around four essential steps. A potential fifth step is documentation
of the process, but that should be an integral part of each step, not a separate process.
1. Reflection
a. Self-assessment process
b. Evaluation and feedback from others
i. Coworkers
ii. Colleagues
iii. Employer
iv. Patients
v. Pharmacy trainees (students/residents)
c. Personal SWOT (strengths, weaknesses, opportunities, and threats)
i. Assessment of internal strengths and weaknesses related to knowledge, skills, experiences, and
behaviors
ii. Assessment of external environmental factors for opportunities and threats
iii. Goal is to identify learning needs and opportunities that exist to address those needs.
d. Reflection should be both scheduled and episodic.
i. Annual performance evaluation/self-evaluation (scheduled)
ii. Some set or chosen anniversary date (scheduled)
iii. After the care of a complex or difficult patient (episodic)
iv. After an interaction with a challenging student or resident (episodic)
e. Result of reflection is to identify two or three specific, well-defined, and achievable learning needs.
2. Plan
a. Develop a personal development plan (PDP) to address the needs and opportunities identified
during reflection.
b. Includes learning objectives that are specific, measurable, achievable, relevant, and timed (SMART)
c. Identifies resources needed to address the PDP
d. Evaluates the availability and access to needed resources and modifies plan accordingly
e. The PDP should be regularly reassessed and adjusted as needed.

ACCP Updates in Therapeutics® 2022: Critical Care Pharmacy Preparatory Review and Recertification Course

24
Critical Care Pharmacy Evolution and Validation, Practice Standards, Training, and Professional Development

3. Act
a. Develop an action plan to implement the PDP.
b. The action plan will need to incorporate a variety of learning strategies and methods (see text that
follows).
c. Incorporating the action plan into the daily practice activities is key to success and sustainability.
CPD should not be considered an additional burden.
4. Evaluate
a. Evaluate the effectiveness of the action plan for achieving the learning objectives of the PDP.
i. Did the activities provide adequate content, depth, and hands-on experiences to truly address
the learning objectives and meet the needs identified during reflection?
ii. Did the activities stay focused on the learning objectives, and were timelines adhered to
adequately?
iii. Were all competencies adequately addressed?
iv. How did the CPD activities affect the pharmacist-learner and possibly the patient (often very
challenging to measure)?
b. Evaluation is expected to lead to the next round of reflection and restart the continuous and iterative
process of CPD.
5. Portfolio
a. Process of documenting the CPD progression
b. Although the format may be standardized by employer, regulatory authorities, CE providers, or
others, the content should be individualized to reflect the needs, actions, and assessments of the
pharmacist-learner.
c. Is a dynamic, living document that reflects the continuous, iterative nature of CPD
d. Examples of CPD portfolio formats/templates:
i. https://www.acpe-accredit.org/pdf/CPD_Portfolio.pdf
ii. www.ncbop.org/CE/CPDLearningPortfolio.pdf
iii. www.ocpinfo.com/practice-education/qa-program/learning-portfolio/

D. CPD Learning Strategies and Methods


1. CPE
a. CPD is not a replacement for CPE, but CPE should be one component of a PDP.
b. ICU pharmacists should focus on CPE programming that meets their defined educational needs and
incorporates several different techniques that will help meet the full range of competencies needed
for clinical practice in the ICU.
c. Accreditation standards maintain minimum quality assurance of CPE activities.
d. CPE credits are the most widely used “currency” by regulatory bodies, accrediting agencies, and
other organizations as a proxy for professional competency, and this is unlikely to change in the
near or intermediate term.
e. Traditional didactic lecture-style CPE activities have several limitations toward achieving CPD
learning objectives.
i. Often non-curricular
ii. Limited influence on changing practice
iii. Educational outcomes may not align with the individual’s needs.
iv. Content is sponsor or speaker driven.
v. Opportunity for bias (or perception of bias), depending on source of support
vi. CE efforts are often fragmented across professions (not interdisciplinary).

ACCP Updates in Therapeutics® 2022: Critical Care Pharmacy Preparatory Review and Recertification Course

25
Critical Care Pharmacy Evolution and Validation, Practice Standards, Training, and Professional Development

f. CPE providers are expanding the diversity of educational methodologies and techniques to include
interaction, experiential learning, simulation, discussion and debate, and role-playing, among others.
g. Limited evidence suggests that live CE over print, multimedia format, and a series of programs on
a curricular theme is the most effective CE method.
2. Short courses or seminars
a. Certificate or credentialing programs
i. ACLS
ii. ATLS
iii. PALS
iv. Emergency neurological life support (ENLS)
b. Structured curricular programs
i. ACCP Academies
ii. Fundamental Critical Care Support (FCCS) and Pediatrics Fundamental Critical Care Support
(PFCCS) course
iii. Pharmacotherapy of Neurocritical Care Series (PONS) offered by the Neurocritical Care
Society
3. Membership and participation in national organizations
a. ACCP; Critical Care PRN
b. SCCM; Clinical Pharmacy and Pharmacology Section
c. ASHP
d. Several specialty organizations related to critical care (American College of Chest Physicians,
American Trauma Society, Neurocritical Care Society, American Burn Association, etc.)
4. Primary and secondary literature
a. Reading, analyzing, and applying the relevant literature should be central to any strategy of
professional development.
b. No gold standard strategy for staying current with the literature
c. Many “foraging” strategies will need to be considered and used.
i. Review table of contents of high-impact journals in critical care (e-mail or rich site summary
[RSS] push technology) (e.g., Critical Care Medicine, Intensive Care Medicine, Chest,
American Journal of Respiratory and Critical Care Medicine, Journal of Trauma and Acute
Care Surgery, Journal of Critical Care).
ii. Topic alerts (e-mail or RSS) for critical care articles from high-impact multispecialty journals
(e.g., New England Journal of Medicine, Annals of Internal Medicine, JAMA, The BMJ, Lancet)
iii. Scan high-impact pharmacy specialty journals for critical care articles (e.g., Pharmacotherapy,
Annals of Pharmacotherapy, American Journal of Health-System Pharmacy).
iv. Use of saved search strategies with automatic e-mail alerts on a scheduled interval (e.g.,
PubMed, PubCrawler, Ovid Medline)
v. Subscribe to a medical information alert service with high and transparent standards for
validity, relevance, and contextual interpretation of the data (e.g., Essential Evidence Plus, FPIN
[Family Physicians Inquiries Network] Clinical Inquiries, BMJ Clinical Evidence, Cochrane
for Clinicians).
vi. Scan review journals relevant to critical care (e.g., Critical Care Clinics).
vii. Identify high-quality, relevant, and contemporary clinical practice guidelines for critical care
therapeutics (e.g., National Guideline Clearinghouse, PubMed Clinical Queries, MD Consult).
viii. Use up-to-date systematic reviews (e.g., Cochrane Database of Systematic Reviews, Agency for
Healthcare Research and Quality [AHRQ] Evidence-Based Practice Center Evidence-Based
Reports).

ACCP Updates in Therapeutics® 2022: Critical Care Pharmacy Preparatory Review and Recertification Course

26
Critical Care Pharmacy Evolution and Validation, Practice Standards, Training, and Professional Development

ix. Selective use of other resources (e.g., evidence-based summaries such as Bandolier, Clinical
Evidence), critically appraised topics, point-of-care review services (e.g., UpToDate, Medscape),
SCCM CPP Section Pharmacotherapy Literature Updates, and meta-search engines (e.g., Trip
database)
d. The tools and resources available for staying current with the literature is a rapidly evolving, dynamic
market. The individual pharmacist will need to stay current to maximize use of the literature and
will need to adapt his or her strategy over time.
5. Discussion and debate with colleagues, mentors, and other content experts
a. Therapeutic dilemmas
b. Complex cases
c. Primary literature
d. Guidelines
6. Journal clubs/clinical conferences
7. Interdisciplinary, patient care rounds – Daily interactive discussions of diagnostics, disease states,
therapeutics, monitoring, technology in the ICU, ethics, communication with patients and families, etc.
8. Guideline and protocol development for the ICU
a. Translation of evidence to best practices
b. Benchmarking with peer institutions
c. Consensus building
d. Project management – Implementation and measurement of outcomes
9. Point-of-care learning
a. Refers to day-to-day learning opportunities
b. Uncommon disease state or unexpected adverse drug reaction prompts reading and learning.
c. Complex drug information questions from colleagues

VIII.  CORE KNOWLEDGE BASE AREAS FOR PHARMACISTS CARING FOR CRITICALLY ILL
PATIENTS (2017 BOARD OF PHARMACY SPECIALTIES CRITICAL CARE CONTENT OUTLINE)

A. Pulmonary
B. Infectious Diseases
C. Cardiology
D. Endocrinology
E. Hematology
F. Oncology
G. Neuroscience
H. Nephrology
I. Hepatology
J. Psychiatry
K. Nutrition
L. Immunology

ACCP Updates in Therapeutics® 2022: Critical Care Pharmacy Preparatory Review and Recertification Course

27
Critical Care Pharmacy Evolution and Validation, Practice Standards, Training, and Professional Development

M. Gastroenterology
N. Surgery
O. Trauma
P. Burn
Q. Toxicology
R. Transplantation
S. Supportive Care
T. Medical Emergencies
Also: Pathophysiology, Epidemiology, and Pharmacology.

IX.  DISSEMINATION OF CRITICAL CARE KNOWLEDGE

A. Reasons to Disseminate Knowledge


1. Recognition by peers
2. Promotion and tenure
3. Ethical obligation of research
4. Grantsmanship success
5. Giving back to the discipline – Critical care pharmacy is a new and evolving specialty.
6. Travel support

B. Venues for Disseminating Knowledge


1. Peer-reviewed publications (see text that follows for greater detail)
a. Traditional, print journals
b. Open-access (electronic) journals
2. Non–peer-reviewed publications
a. Textbook chapter
b. Pre-print publications
c. Commentary/editorial
d. Newsletter
e. Guideline
f. Compendia
g. CE material
h. Blogs
3. Abstract (see text that follows for greater detail)
a. Poster
b. Platform
c. Regional, national, international meetings
d. Virtual poster sessions

ACCP Updates in Therapeutics® 2022: Critical Care Pharmacy Preparatory Review and Recertification Course

28
Critical Care Pharmacy Evolution and Validation, Practice Standards, Training, and Professional Development

4. Presentation
a. CE
i. Live/lecture – Local, regional, national, international venues
ii. Webinar
iii. Recorded/archived
b. Seminar or conference

C. Publication and Peer-Review Process


1. Categories of publications (will vary by journal)
a. Original research
b. Systematic review (e.g., meta-analysis)
c. Expert review
d. Brief reports (e.g., preliminary or pilot data)
e. Case reports
f. Practice or educational insights (typically must include assessment of outcomes)
2. Selecting a target journal
a. Quality and importance of the publication
i. First-tier journals – Highly important, innovative, and/or high quality
ii. Second- and third-tier journals – To be considered when paper unlikely to be accepted in, or
rejected by, first-tier journal
b. Target audience and scope of the journal relative to content of publication – Looking for good match
on both
c. Seek input from coauthors, peers, colleagues concerning appropriate journal to target
d. Publication fees (if applicable)
3. Preparing the manuscript
a. Comply with journal requirements.
i. Historically published with first issue of each volume
ii. Today, easiest to access online
iii. Manuscript format
iv. Margins, font, type size
v. Abstract format
vi. Word limits
vii. Reference style and limits
viii. Figures and tables
b. Succinct, focused, non-repetitive – Economy of words
c. Common weaknesses to avoid (original research) – In manuscript preparation, not underlying
research
i. Abstract does not match body of manuscript
ii. Introduction fails to sell the importance and relevance of the objective(s)
iii. Poorly worded or unclear study objective(s)
iv. Methods without results; results without methods
v. Unnecessary duplication of results in tables and body of manuscript
vi. Rambling, unfocused discussion
vii. Failure to adequately address weaknesses of the study (all studies have them)
viii. Conclusions that reach beyond the data
ix. Several tables that can be consolidated
x. Unneeded figures (usually, simplistic presentations of data that can be presented parenthetically)
xi. Failure to cite the literature correctly or according to journal’s requirements
xii. Exceeding word count limits – Both in abstract and in manuscript

ACCP Updates in Therapeutics® 2022: Critical Care Pharmacy Preparatory Review and Recertification Course

29
Critical Care Pharmacy Evolution and Validation, Practice Standards, Training, and Professional Development

4. Using citation manager software (EndNote, Reference Manager, RefWorks, etc.)


a. May contain templates consistent with many biomedical journal requirements
b. Actively cite the literature while writing
c. Automatically re-sort the references during revisions
d. Direct download of citations during literature searches
e. Can include PDF files and your notes in citation file
f. Develop libraries of commonly used citations
g. Overall, can ease the writing and formatting process for publication
5. Submitting the manuscript
a. Greatly simplified by web-based submission
b. Follow download instructions carefully.
c. Cover letter
i. Communication to the editor
ii. Declare category of publication (though now part of submission template).
iii. Indicate corresponding author (also part of template).
iv. Some journals encourage a brief explanation of why paper is being submitted to the journal –
Relevance, importance, target audience. However, this is declining.
d. Copyright release
i. Electronic methods are increasingly used.
ii. Each author must sign/submit.
iii. Provide assurance that part or all of content has not been previously published and is not
currently under consideration by another publisher. Usually excludes abstracts.
6. Conflicts of interest: All authors must provide conflict-of-interest statements.
7. Review and revision process
a. Editorial review
i. The editor or a member of the editorial board may review initially.
ii. Looking for relevance to journal, general quality of the manuscript, composition, and readability
iii. Failure to get past the editor’s desk results in rejection
b. Peer or scientific review
i. Sent to peers with content expertise for review and critique
ii. Typically, sent to two to five reviewers (varies by journal and internal criteria)
iii. Most journals request a review to be returned in 10–30 days.
iv. Reviewers are asked to focus on the quality of the research or content and importance (including
relevance to the journal’s audience), not copyediting details.
v. Typical recommendation categories are as follows: Accept, minor revision, major revision, or
reject.
vi. Reviewers provide detailed comments and critique to be shared with the authors, and, often,
comments to the editor that are not shared with the authors.
c. Editor response
i. The editor (or designee) uses reviewer input to formulate a response to the authors.
ii. If minor or major revision is requested, details of the required revisions are provided, often
including the reviewers’ specific comments. The editor often adds requests for revision.
iii. A timeline for response is included. If manuscript is not resubmitted by deadline, opportunity
to publish is usually surrendered.
iv. A rejection decision is usually final.
d. Revision process
i. The authors need to respond to each request. The authors need not agree with each request, but
each must be responded to and defended if not revised as advised.

ACCP Updates in Therapeutics® 2022: Critical Care Pharmacy Preparatory Review and Recertification Course

30
Critical Care Pharmacy Evolution and Validation, Practice Standards, Training, and Professional Development

ii. Common format is a letter to the editor restating each request, with the response to the request
immediately following.
iii. A manuscript incorporating all revisions is submitted with the letter. Some journals request a
“track changes” version of the manuscript to ease the rereview process.
iv. All authors must review the revisions and indicate their agreement with all changes.
v. Revised manuscripts are often returned to the original peer reviewers for a second review,
especially with major revisions. That may result in another round of revision and review.
8. After acceptance
a. At some point (timelines vary by journal), the corresponding author will receive the galley proof.
This is a copyedited, typeset version of the paper that will look like the final publication.
b. The galley proof will come with comments that must be addressed.
c. Deadline for galley submission may be as short as 48 hours.
d. The galley proof must be read very carefully and compared with the manuscript to ensure that
copyediting changes do not affect the meaning, tables are formatted as intended, figures and legends
are correct, and references are in the correct order and format (there are often errors with references
– better with electronic confirmation).
e. Ideally, all authors should review the galley proof; however, that may not be practical. All authors
should approve a review by the corresponding author.
9. Timelines (highly variable by journal)
a. Important competitive metric for biomedical journals
b. From submission to response – Can be 2–4 months
c. Reply to decision – Can be 1 week to 3 months (depends on whether rereview occurs)
d. Decision to publication – Can be an additional 3–6 months
e. Sometimes an important consideration when selecting a journal – Manuscript can be tied up for
long periods.
f. Open-access and e-journals have a speed advantage – May have fees.

D. Abstracts and Scientific Presentations


1. Selecting a meeting
a. Quality and relevance of presentation
b. Prestige of the meeting relative to authors’ career goals
c. Membership and desire to support an organization
d. Availability of travel funding
e. Priorities of key coauthors
f. Location of the meeting (unfortunate, but true …)
g. Encore presentations permitted
2. Developing the abstract
a. Succinct and effective
i. Greatest impact in the least space – No unnecessary words
ii. Use of identifiable abbreviations, but not to excess
iii. If allowed, use of tables to present results (many disallow)
b. Title must be brief, be on point, and capture the reader.
c. Clearly stated purpose/objective (minimize introductory material). May need to limit to primary
objective.
d. Methods and analysis are concise but of adequate detail to permit review.
e. Results may need to be limited to the primary end point.
f. Conclusion is a single brief sentence directly tied to the objective(s).
g. Must meet word or character count limit (tricks and tips depend on organization)

ACCP Updates in Therapeutics® 2022: Critical Care Pharmacy Preparatory Review and Recertification Course

31
Critical Care Pharmacy Evolution and Validation, Practice Standards, Training, and Professional Development

h. Revise, revise, revise with input from all authors – Eliminate unnecessary words and content.
i. Some organizations may permit students or residents to submit abstracts without data or with partial
data.
3. Abstract submission
a. Greatly simplified by electronic submission
b. Must meet deadline – Most websites shut down after deadline.
c. Must carefully follow online instructions
d. Pay careful attention to the abstract categories (e.g., clinical practice, original research) to be sure
the abstract is submitted under the proper category.
e. Word limit usually controlled by software – Difficult to cheat
f. If platform presentations are an option, usually need to indicate consideration for platform, if that
is the goal
4. Platform versus poster
a. Platform slots are intended for presentations that have high-quality content and that are relevant and
effective.
b. Usually based on reviewer scores
c. Many organizations may accept a platform submission as a poster presentation if it was not scored
high enough to be accepted as a platform; others may just reject it.
d. Authors must be realistic concerning the quality of their abstract when considering submission for
a platform, given the meeting, audience, and likely competing research.
5. Review process
a. Typically reviewed by three to five reviewers
b. Review uses relatively limited scoring criteria, given the brevity of an abstract, combined with a
recommendation of accept or reject.
c. Reviews are compiled into an overall score, and recommendation is provided to the authors.
d. There is no opportunity or time for revision and resubmission. Decisions are final.
e. Reviewer comments may or may not be shared with the authors.
f. Platform versus poster decisions may also be an outcome of the review process.
g. Review process may also be used for determining abstracts to be considered for awards.
6. Poster presentation
a. Format (size and/or style) is often specified by the organization.
b. Many tips and tricks are available for developing effective posters. Some key issues are:
i. Avoid wordy posters – Nobody wants to read them.
ii. Use tables, figures, and concise bullet lists as much as possible.
iii. Use a font size that can easily be read from about 5–6 feet (e.g., 24 point or greater).
iv. Ease of readability is more important than aesthetics – Consider dark letters on a white
background.
v. Use a logical flow from left to right and from the introduction to the conclusions.
vi. Unless required, do not reprint an abstract on a poster – It is unnecessary and uses valuable
space.
vii. Most institutions have requirements to use logos – Comply with the requirements or potentially
run into last-minute challenges with printing.
viii. Large-print formats have eased the production and transport of posters; however, review proofs
carefully for content changes before printing.
ix. Commercial printers who will ship to the meeting site are a great alternative if last-minute
challenges develop.

ACCP Updates in Therapeutics® 2022: Critical Care Pharmacy Preparatory Review and Recertification Course

32
Critical Care Pharmacy Evolution and Validation, Practice Standards, Training, and Professional Development

c. If there are walk-rounds, be fully prepared to present the key points of your poster in about 5 minutes
to allow time for questions. It is important to confirm the time allotment set by each organization
because this may vary.
d. Consider having small, legible versions of the poster at the poster session for those who want a copy
to review. In addition, have business cards available.
e. Plan to have at least one author stay for the duration of the poster session.
f. Virtual poster sessions are very similar in submission, review, and acceptance process. Presentations
are virtual and may involve the abstract only or a more detailed “poster,” with interactive sessions
scheduled with either random viewers or scheduled peer reviewers.
7. Platform presentation
a. Considered an honor of recognition for high-quality, innovative, or impactful work
b. Presentation is usually limited to 10 minutes, with 5–10 minutes left for questions.
c. Typical format is a brief slide presentation focused on the most important aspects of the work. Time
does not allow a detailed description of all aspects of the project.
d. May involve peer review/judging if awards are involved
e. Feedback in verbal or written format is often provided to the presenter.
f. May also require a poster presentation during one of the poster sessions (varies by organization)
g. Repeated practice with coauthors, peers, and colleagues, followed by critique and revision, is highly
recommended.

E. Participation in the Peer-Review Process (Eos 2011;92:233-40; PLoS Comput Biol 2006;2:e110)
1. Reasons to participate
a. Professional obligation
i. Authors “take” from the process, so they should “give back.”
ii. Contribute expertise to improving the biomedical literature
b. Professional service to an organization or journal
c. Recognition, tenure, and promotion – Professional service
d. Some enjoy reviewing the “raw” product of the biomedical literature.
e. Educational opportunity for trainees
2. Reasons to decline invitation to participate
a. Conflict of interest
i. Former trainee is author
ii. Collaborator or coworker is author
iii. Financial conflict of interest with the subject
b. Lack of expertise in the subject matter
c. Lack of time to meet the deadline because of other commitments
3. Getting on the list of potential reviewers
a. Publishing in the journal
b. Being recommended by a peer to the editor
c. Being a recognized expert (nationally, internationally) in a relevant field
d. Publishing in the field in peer journals
e. Volunteering through the journal’s website or a general call for reviewers – Not an option for all
journals
4. Review process
a. Invitation
i. Normally sent by e-mail with a response link
ii. Includes an abstract of the paper and often identifies the authors and institution
iii. Deadline for submission of review is provided so that reviewer can gauge availability of time.

ACCP Updates in Therapeutics® 2022: Critical Care Pharmacy Preparatory Review and Recertification Course

33
Critical Care Pharmacy Evolution and Validation, Practice Standards, Training, and Professional Development

iv. Usually a short timeline to respond to invitation


v. If decline, most journals ask for recommendation of a peer to review (building their reviewer
database)
vi. Once accepted, online access to content is provided (manuscript, review form, instructions to
reviewers, etc.).
b. Tips for the review
i. Biomedical literature review skills are beyond the scope of this chapter.
ii. Review is expected to focus on scientific quality and importance of the paper.
iii. Grammar, sentence structure, and word choices are normally better handled by the copyeditors
because of their greater expertise, unless it is critical to the scientific meaning of the paper.
iv. Connect the dots to find common errors (original research and systematic reviews).
(a) Objective/purpose and conclusions must match.
(b) Methods must be directly related to the objective/purpose.
(c) Methods must be valid, widely accepted, and/or state of the art, including statistical
handling of the data.
(d) Widely accepted guidelines according to study design should be complied with (e.g.,
CONSORT, STROBE, PRISMA, STARD).
(e) Every method described must have results reported.
(f) Every result reported has to be tied to a method description.
(g) Conclusions must be limited to and supported by the study results.
(h) A fair and complete discussion of study weaknesses must be presented.
c. Recommendation to editor
i. Reason for rejection
(a) Fatal flaw – No amount of rewriting or reanalysis of the data will make it worthy of
publication – Poor-quality research or serious errors in methods
(b) Extent of revision required is so extensive that it is equivalent to starting over
(c) Valid research that is not important, either because of lack of relevance to the target journal
or because manuscript is reporting a well-known finding with no new information
(d) Republication of all or an extensive portion of the content (may be justifiable [e.g., portions
of the methods section when it is a legitimate secondary publication of a previously
published study])
(e) Serious ethical violations
ii. Revision (minor or major revision)
(a) Usually for publications believed to have adequate quality and importance, but there are
weaknesses that need to be corrected or addressed
(b) Major revisions usually require a second review – The journal may ask whether you will
serve as a continuing reviewer, or it may be assumed.
(c) Minor revisions may not require rereview; depends on reasons for revision
(d) Most journals will share the recommendation to the authors, together with the other
reviewers’ comments to author – Can be very instructive for future reviews
iii. Accept (without revision)
(a) Unusual with first submission and review
(b) Is a truly exceptional manuscript
d. Submission of the review
i. Electronic submission according to the journal
ii. Format is often dictated by the journal.
iii. Meet the deadline – Delayed reviews prolong the timeline and create workflow challenges for
the journal.

ACCP Updates in Therapeutics® 2022: Critical Care Pharmacy Preparatory Review and Recertification Course

34
Critical Care Pharmacy Evolution and Validation, Practice Standards, Training, and Professional Development

iv. Comments to the authors


(a) Should be clear, concise, and factual. Should not be abrasive or a personal attack.
(b) Comments should be clearly referenced to the location in the manuscript (e.g., line number,
page/paragraph/sentence).
(c) Are usually anonymous, but some journals provide option to be identified
(d) For manuscripts with a fatal flaw, comments can be limited to that flaw.
v. Comments to the editor
(a) Should not require extensive comments beyond those to the authors
(b) Is an opportunity to further explain the rationale for the recommendation or ethical concerns
(c) Are attributed to the reviewer, but not shared with the authors
vi. Miscellaneous
(a) Journal may ask whether you believe an editorial is needed, and a proposed author
(b) There may be a question about concerns with ethics, animal treatment, or human subjects’
protection.
5. Rereview process
a. Response content
i. Cover letter detailing response to comments/requests of editor and reviewers
ii. Revised manuscript (with or without “track changes”)
b. Review process
i. Should restrict comments to responses in first review. Finding an entirely new set of criticisms
to original content is considered “not playing fair.”
ii. Confirm that the revisions have not materially altered the meaning of other parts of the
manuscript.
iii. If the authors have argued not to make a recommended revision, evaluate the validity of the
response.
iv. Recommendations are the same options as for the primary review.
v. If major revisions are still needed, the editor may decide to reject, or it may undergo another
round of reply and review.
vi. Deadline for the review is often shorter than for the primary review.

ACCP Updates in Therapeutics® 2022: Critical Care Pharmacy Preparatory Review and Recertification Course

35
Critical Care Pharmacy Evolution and Validation, Practice Standards, Training, and Professional Development

Table 1. Critical Care Pharmacy Services in Level 1 Critical Care Centers (Crit Care Med 2020;48:1375-82)
Patient Care
Essential:
• Rounding with interdisciplinary team
• Assists with informed pharmacotherapy decisions
• Provision of drug information to the critical care team
• Provision of drug therapy education to critical care team
• Assists with prevention of inappropriate drug therapy
• Provides clinical consultation for pharmacotherapeutic issues related to critical illness
• Medication consults are available 24 hr/d, 7 d/wk
• Provision of pharmacokinetic monitoring and therapeutic adjustments for targeted drugs
• Review of medication history for determination if maintenance therapy should be continued
• Medication reconciliation services for ICU patients upon admission, transfer, and discharge
• Prospective review of all orders for verification of appropriateness
• Patient and/or caregiver medication therapy education
• Performance of independent patient assessments, e.g., pain/agitation, delirium, nutrition
• Response to cardiac live support by ACLS certified pharmacists 24 hr/d, 7 d/wk
• Response to other resuscitation and time-dependent emergencies in the hospital
• Provision of stewardship of antimicrobials and other selected medications
• Collaboration with other pharmacists within the hospital as needed
• Provision of nutrition therapy plan and modifications in concert with clinical dieticians
• Use of medical record for communication with other healthcare professionals
• Documentation of patient care and economic impact of services
• Documentation of all clinical activities
• Pharmacy liaison with other interdisciplinary team members relative to medication policies, procedures,
guidelines, and pathways
• Utilizes pharmacoeconomic analyses to evaluate new critical care pharmacotherapeutic agents
• Proactive in designing, prioritizing, and promoting new clinical pharmacy programs
• Evaluation of clinical pharmacy services for stakeholder satisfaction, significance, and economic value
• Participation in planning and implementation of processes for disaster and mass casualties
• Majority of time dedicated to critical care services
• Patient care time devoted to care of the critically ill patients
• Decentralized clinical services 24 hr/d
• Development of critical care pharmacists “teams”
• Appropriate pharmacist-to-patient ratio based on patient acuity and complexity

Desirable:
• Provision of medication management problems documentation and progress notes daily
• Preparation and presentation of drug monographs to the P&T Committee
• Provision of telemedicine services if onsite critical care pharmacists absent

ACCP Updates in Therapeutics® 2022: Critical Care Pharmacy Preparatory Review and Recertification Course

36
Critical Care Pharmacy Evolution and Validation, Practice Standards, Training, and Professional Development

Table 1. Critical Care Pharmacy Services in Level 1 Critical Care Centers (Crit Care Med 2020;48:1375-82)
(continued)
Quality Improvement
Essential:
• Medication safety leaders for critically ill patients
• Assists with management and reduction of ADEs
• Reporting of ADEs to institutional committees and national programs
• Evaluation of availability of critical medications through automated dispensing cabinets
• Team member in the design process for building new and remodeled critical care areas
• Implements and maintains policies and procedures related to safe and effective medication use
• Coordination of development and implementation of ICU-focused drug therapy guidelines, order sets, and/or
care ICU pathways
• Evaluation of impact of ICU-focused drug therapy guidelines, order sets, and/or care ICU pathways
• Leads or provides critical care pharmacotherapy to hospital committees
• Service on hospital committees
• Contribution to hospital newsletter and drug monographs
• Identifies and evaluates drug minimization opportunities
• Identifies local quality metrics for CQI
• Participates in quality assurance programs
• Shares responsibility for hospital performance for quality and process measure compliance
• Collaborates with other healthcare professions in preparing for accreditation
• Assessment of adequacy of pharmacy space and facilities
• Implementation of safety technology
• Presence of contemporary medication use systems within institution
• Presence of contemporary information management systems within institution

Desirable:
• Use of real-time dashboard or analytics monitoring of quality metrics and drug utilization
Research/Scholarship
Essential:
• Participates as key investigator for critical care research
• Representation on the IRB as applicable

Desirable:
• Active involvement with critical care pharmacotherapy research
• Contributes to the pharmacy and medical literature
• Report of research results at professional and scientific meetings
• Participates in research design and data analysis
• Secures research funding
• Collaboration in multicenter research projects
• Peer reviewer of the medical literature

ACCP Updates in Therapeutics® 2022: Critical Care Pharmacy Preparatory Review and Recertification Course

37
Critical Care Pharmacy Evolution and Validation, Practice Standards, Training, and Professional Development

Table 1. Critical Care Pharmacy Services in Level 1 Critical Care Centers (Crit Care Med 2020;48:1375-82)
(continued)
Training/Education
Essential:
• Provision of training and mentoring of pharmacy students, residents, and fellows
• Supports postgraduate residencies and/or fellowship training in critical care pharmacy practice
• Evaluation and documentation of trainee outcomes
• Provision of education to health professional students and trainees
• Development and implementation of training programs for ICU personnel

Desirable:
• Project advisor for pharmacy students, residents, and fellows
• Provision of formal education at accredited educational sessions
• Active participation in interdisciplinary simulation activities
• Certified instructor for certification classes
• Identifies and educates medical and community groups relative to pharmacist role on healthcare team
Professional Development
Essential:
• Maintains master of critical care pharmacotherapy knowledge
• Maintains certification in life-support courses
• Seeks board certification when eligible
• Involvement in nonpatient care activities
• Provision of formal accredited educational sessions
• Membership in critical care and pharmacy organizations
• Availability of protected time for education, administrative, and research/scholarship activities
• Mechanisms in place for career development
ACLS = advanced cardiac life support; ADE = adverse drug events; CQI = continuous quality improvement; ICU = intensive care unit; IRB = institutional review board;
P&T = pharmacy and therapeutics.

ACCP Updates in Therapeutics® 2022: Critical Care Pharmacy Preparatory Review and Recertification Course

38
Critical Care Pharmacy Evolution and Validation, Practice Standards, Training, and Professional Development

Box 1. Attributes of Successful Mentor-Protégé Relationships


Mentor Qualities
Strong interpersonal skills
Technical competence/expertise
Knowledge of organization and profession
Status/prestige within the organization and profession
Willingness to be responsible for someone else’s growth and development
Ability to share credit
Patience
Protégé Qualities
Self-perceived growth needs
A record of seeking/accepting challenging assignments
Receptivity to feedback and coaching
Willingness to assume responsibility for own growth and development
Ability to perform in more than one skill area
Relationship Qualities
Voluntary
Mutual benefits perceived and derived from the relationship
No conflicts of interest/competition between mentor and protégé
Not confined to professional or business interests
Adapted from: Haines ST. The mentor-protégé relationship. Am J Pharm Educ 2003;67:Article 82, p 2.

ACCP Updates in Therapeutics® 2022: Critical Care Pharmacy Preparatory Review and Recertification Course

39
Critical Care Pharmacy Evolution and Validation, Practice Standards, Training, and Professional Development

REFERENCES

General/Critical Care Pharmacy Validation 13. Hammond DA, Gurnani PK, Flannery AH, et
1. American College of Clinical Pharmacy (ACCP). al. Scoping review of interventions associated
Standards of practice for clinical pharmacists. with cost avoidance able to be performed in the
Pharmacotherapy 2014;34:794-7. intensive care unity and emergency department.
2. Beardsley JR, Jones CM, Williamson J, et al. Phamacotherapy 2019;39:215-31.
Pharmacist involvement in a multidisciplinary 14. Harris IM, Phillips B, Boyce E, et al. Clinical phar-
initiative to reduce sepsis-related mortality. Am J macy should adopt a consistent process of direct
Health Syst Pharm 2016;73:143-9. patient care. Pharmacotherapy 2014;34:e133-e148.
3. Board of Pharmacy Specialties (BPS). Critical 15. Haupt MT, Bekes CE, Brilli RJ, et al. Guidelines
Care Content Outline, 2017. Available at https:// on critical care services and personnel: recommen-
www.bpsweb.org/wp-content/uploads/CritCare_ dations based on a system of categorization of three
ContentOutlineForPublication20170913.pdf. levels of care. Crit Care Med 2003;31:2677-83.
Accessed November 15, 2021. 16. Horn E, Jacobi J. The critical care clinical pharma-
4. Brilli RJ, Spevetz A, Branson RD, et al. Critical cist: evolution of an essential team member. Crit
care delivery in the intensive care unit: defining Care Med 2006;34(3 suppl):S46-51.
clinical roles and the best practice model. Crit Care 17. Jacobi J. Critical care pharmacy practice. In:
Med 2001;29:2007-19. DiPiro JT, ed. Encyclopedia of Clinical Pharmacy.
5. Buckley MS, Knutson, KD, Agarwal S, et al. New York: Marcel Dekker, 2003:233-9.
Clinical pharmacist-led impact on inappropriate 18. Joint Commission of Pharmacy Practitioners.
albumin use and costs in the critically ill. Ann Pharmacists’ Patient Care Process. May 2014.
Pharmacother 2020;54:105-12. Available at http://jcpp.net/patient-care-process/
6. Dasta JF. Critical care. Ann Pharmacother Accessed August 28, 2019.
2006;40:736-7. 19. Kane SL, Weber RJ, Dasta JF. The impact of
7. Dasta JF, Jacobi J. The critical care pharmacist: critical care pharmacists on enhancing patient out-
what you get is more than what you see. Crit Care comes. Intensive Care Med 2003;29:691-8.
Med 1994;22:906-9. 20. Kim MM, Barnato AE, Angus DC, et al. The effect
8. Dasta JF, Jacobi J, Armstrong DK. Role of the of multidisciplinary care teams on intensive care
pharmacist in caring for the critically ill patient. unit mortality. Arch Intern Med 2010;170:369-76.
In: Chernow B, ed. The Pharmacologic Approach 21. Lat I, Paciullo C, Daley MJ, et al. Position paper
to the Critically Ill Patient, 3rd ed. Baltimore, MD: on critical care pharmacy services (execu-
Williams & Wilkins, 1994:156-66. tive summary): 2020 update. Crit Care Med
9. Dasta JF, Segal R, Cunningham A. National sur- 2020;48:1375-82.
vey of critical-care pharmaceutical services. Am J 22. Leape LL, Cullen DJ, Clapp MD, et al. Pharmacist
Hosp Pharm 1989;46:2308-12. participation on physician rounds and adverse
10. Devlin JW, Skrobik Y, Gélinas C, et al. Clinical drug events in the intensive care unit. JAMA
practice guidelines for the prevention of and man- 1999;282:267-70.
agement of pain, agitation/sedation, delirium, 23. Lee H, Ryu K, Sohn Y, et al. Impact on patient
immobility, and sleep disruption in adult critically outcomes of pharmacist participation in multi-
ill patients. Crit Care Med 2018;46:e825-73. disciplinary critical care terms. Crit Care Med
11. Erstad BL. A primer on critical care pharmacy ser- 2019;47:1243-50.
vices. Ann Pharmacother 2008;42:1871-81. 24. Leguelinel-Blance G, Nguyen TL, Louart B, et al.
12. Erstad BL, Haas CE, O’Keeffe T, et al. Impact of quality bundle enforcement by a critical
Interdisciplinary patient care in the intensive care care pharmacist on patient outcome and cost. Crit
unit: focus on the pharmacist. Pharmacotherapy Care Med 2018;46:199-207.
2011;31:128-37. 25. Louzen P, Jennings H, Ali M, Kraisinger M.
Impact of pharmacist management of pain,

ACCP Updates in Therapeutics® 2022: Critical Care Pharmacy Preparatory Review and Recertification Course

40
Critical Care Pharmacy Evolution and Validation, Practice Standards, Training, and Professional Development

agitation, and delirium in the intensive care unit intervention in a tertiary care medical intensive
through participation in multidisciplinary rounds. care unit. Crit Care Med 2011;39:284-93.
Am J Health-Syst Pharm 2017;74:253-62. 37. Newsome AS, Smith SE, Jones, TW, et al. A sur-
26. MacLaren R, Bond CA. Effects of pharmacist vey of critical care pharmacists to patient ratios
participation in intensive care units on clinical and practice characteristics in intensive care units.
and economic outcomes of critically ill patients J Am Coll Clin Pharm 2020;3:68-74.
with thromboembolic or infarction-related events. 38. Ng TM, Bell AM, Hong C, et al. Pharmacist moni-
Pharmacotherapy 2009;29:761-8. toring of QTc interval-prolonging medications in
27. MacLaren R, Bond CA, Martin SJ, et al. Clinical critically ill medical patients: a pilot study. Ann
and economic outcomes of involving pharmacists Pharmacother 2008;42:475-82.
in the direct care of critically ill patients with 39. Papadopoulos J, Rebuck JA, Lober C, et al. The
infections. Crit Care Med 2008;36:3184-9. critical care pharmacist: an essential intensive care
28. MacLaren R, Devlin JW, Martin SJ, et al. Critical practitioner. Pharmacotherapy 2002;22:1484-8.
care pharmacy services in United States hospitals. 40. Patel NP, Brandt CP, Yolwer CJ. A prospective
Ann Pharmacother 2006;40:612-8. study of the impact of a critical care pharmacist
29. MacLaren R, Plamondon JM, Ramsay KB, et al. A assigned as a member of the multidisciplinary burn
prospective evaluation of empiric versus protocol- care team. J Burn Care Res 2006;27:310-13.
based sedation and analgesia. Pharmacotherapy 41. Preslaski CR, Lat I, MacLaren R, et al. Pharmacist
2000;20:662-72. contributions as members of the multidisciplinary
30. MacLaren R, Roberts RJ, Dzierba AL, et al. ICU team. Chest 2013;144:1687-95.
Characterizing critical care pharmacy services 42. Rivkin A, Yin H. Evaluation of the role of the criti-
across the United States. Critical Care Explorations cal care pharmacist in identifying and avoiding
2021;3:e0323. or minimizing significant drug-drug interactions
31. Majerus TC, Dasta JF, eds. Practice of Critical Care in medical intensive care patients. J Crit Care
Pharmacy. Rockville, MD: Aspen Publications, 2011;26:104.e101-106.
1985. 43. Rudis MI, Brandl KM. Position paper on criti-
32. Marshall J, Finn CA, Theodore AC. Impact of cal care pharmacy services. Society of Critical
a clinical pharmacist-enforced intensive care Care Medicine and American College of Clinical
unit sedation protocol on duration of mechani- Pharmacy Task Force on Critical Care Pharmacy
cal ventilation and hospital stay. Crit Care Med Services. Crit Care Med 2000;28:3746-50.
2008;36:427-33. 44. Schumock GT, Butler MG, Meek PD, et al.
33. Mitchell P, Wynia M, Golden R, et al. Core Evidence of the economic benefit of clinical phar-
Principles & Values of Effective Team-Based macy services: 1996-2000. Pharmacotherapy
Health Care. Discussion Paper. Washington, DC: 2003;23:113-32.
Institute of Medicine, 2012. Available at https:// 45. Schumock GT, Meek PD, Ploetz PA, et al.
nam.edu/wp-content/uploads/2015/06/VSRT- Economic evaluations of clinical pharmacy ser-
Team-Based-Care-Principles-Values.pdf Accessed vices—1988-1995. The Publications Committee
November 15, 2021. of the American College of Clinical Pharmacy.
34. Moheet AM, Livesay SL, Abdelhak T, et al. Pharmacotherapy 1996;16:1188-208.
Standards for neurologic critical care units: 46. Touchette DR, Doloresco F, Suda KJ, et al. Economic
a statement for healthcare professionals from evaluations of clinical pharmacy services: 2006-
the Neurocritical Care Society. Neurocrit Care 2010. Pharmacotherapy 2014;34:771-93.
2018;29:145-60. 47. Wang T, Benedict N, Olsen, et al. Effect of criti-
35. Montazeri M, Cook DJ. Impact of a clinical phar- cal care pharmacist’s intervention on medication
macist in a multidisciplinary intensive care unit. errors: A systematic review and meta-analysis of
Crit Care Med 1994;22:1044-8. observational studies. J Crit Care 2015;30:1101-06.
36. Netzer G, Xinggang L, Shanholtz C, et al. Decreased 48. Weant KA, Armitstead JA, Ladha AM, et al.
mortality resulting from a multicomponent Cost effectiveness of a clinical pharmacist on a

ACCP Updates in Therapeutics® 2022: Critical Care Pharmacy Preparatory Review and Recertification Course

41
Critical Care Pharmacy Evolution and Validation, Practice Standards, Training, and Professional Development

neurosurgical team. Neurosurgery 2009;65:946- Available at https://www.acpe-accredit.org/


50; discussion 950-1. continuing-professional-development/. Accessed
September 4, 2018.
Training 2. Haas CE. Chapter 10. Lifelong learning as a pro-
1. American College of Clinical Pharmacy (ACCP). fessional obligation. In: Murphy JE, ed. Resident
Board of Regents commentary. Qualifications Survival Guide. Lenexa, KS: ACCP, 2011:123-37.
of pharmacists who provide direct patient care: 3. Rouse MJ. Continuing professional develop-
perspectives on the need for residency train- ment in pharmacy. Am J Health Syst Pharm
ing and board certification. Pharmacotherapy 2004;61:2069-76.
2014;33:888-91.
2. American College of Clinical Pharmacy, Dissemination of Knowledge in Critical Care
American Pharmacists Association, American 1. Bourne PE, Korngreen A. Ten simple rules for
Society of Health-System Pharmacists. A Petition reviewers. PLoS Comput Biol 2006;2:e110.
to the Board of Pharmacy Specialties Requesting Available at https://journals.plos.org/ploscomp-
Recognition of Critical Care Pharmacy Practice as biol/article?id =10.1371/journal.pcbi.0020110
a Specialty. 2012. Available at https://www.accp. Accessed November 15, 2021.
com/docs/positions/petitions/Final_CRITICAL_ 2. Browner WS. Publishing and Presenting Clinical
CARE_PETITION_For_BPS_Post.pdf. Accessed Research, 3rd ed. Philadelphia: Lippincott,
October 24, 2016. Williams and Wilkins, 2012.
3. American Society of Health-System Pharmacists 3. Nicholas KA, Gordon W. A quick guide to writing
(ASHP). Supplemental standard and learning a solid peer review. Eos 2011;92:233-40.
objectives for residency training in critical-care
pharmacy practice. American Society of Hospital
Pharmacists. Am J Hosp Pharm 1990;47:609-12.
4. ASHP Foundation. Critical Care Traineeship.
Available at http://www.ashpfoundation.
o r g / M a i n M e n u C a t e g o r i e s / Tr a i n e e s h i p s /
CriticalCareTraineeship. Accessed October 24,
2016.
5. Dager W, Bolesta S, Brophy G, et al. An opinion
paper outlining recommendations for training,
credentialing, and documenting and justifying
critical care pharmacy services. Pharmacotherapy
2011;31:135e-175e.
6. Haines ST. The mentor-protégé relationship. Am J
Pharm Educ 2003;67:1-7.
7. Metzger AH, Hardy YM, Jarvis C, et al. Essential
elements for a pharmacy practice mentoring pro-
gram. Am J Pharm Educ 2013;77:1-7.
8. Murphy JE, Nappi JM, Bosso JA, et al. American
College of Clinical Pharmacy’s vision of the
future: postgraduate pharmacy residency training
as a prerequisite for direct patient care practice.
Pharmacotherapy 2006;26:722-33.

Continuing Professional Development


1. Accreditation Council for Pharmacy Education.
Continuing Professional Development web page.

ACCP Updates in Therapeutics® 2022: Critical Care Pharmacy Preparatory Review and Recertification Course

42
Critical Care Pharmacy Evolution and Validation, Practice Standards, Training, and Professional Development

ANSWERS AND EXPLANATIONS TO SELF-ASSESSMENT QUESTIONS

1. Answer: A rheumatology (Answer C), and obstetrics (Answer D)


The journal Drug Intelligence and Clinical Pharmacy are not considered core knowledge areas and are there-
(now Annals of Pharmacotherapy) was the first to pub- fore incorrect.
lish a critical care therapeutics column in 1982, which
was a landmark event relative to the evolution of critical 5. Answer: C
care pharmacy (Answer A). Although the other journals The landmark study documenting a decrease in pre-
listed—Pharmacotherapy (Answer B), Chest (Answer ventable adverse drug reactions after the inclusion of
C), and Heart and Lung (Answer D)—publish critical pharmacists on interdisciplinary medical rounds was
care therapeutics articles, Annals of Pharmacotherapy published in the Journal of the American Medical
was the first to incorporate a critical care therapeutics Association by Dr. Lucian Leape and colleagues
column into its publication. (Answer C). This highly publicized article published in a
mainstream medical journal by a physician remains one
2. Answer: C of the foundational studies documenting the association
In 2001, ACCM, which exists within the organiza- of critical care pharmacy services with favorable health
tional framework of SCCM, formed the two task forces care outcomes. The other mainstream medical journals
focused on models of critical care delivery, the definition listed, New England Journal of Medicine (Answer A),
of an intensivist, and the practice of critical care medi- Lancet (Answer B), and Annals of Internal Medicine
cine within three different proposed models (Answer C). (Answer D), have not published similar articles on pre-
Neither the Institute of Medicine (Answer A) nor ACCP ventable adverse drug reactions after the inclusion of
(Answer B) was involved in formulating the levels of pharmacists on interdisciplinary medical rounds.
critical care delivery. Although the Clinical Pharmacy
and Pharmacology Section of SCCM (Answer D) may 6. Answer: B
have contributed to this document, it is not mentioned in As stated, there were eight ASHP-accredited critical care
the publication. pharmacy residencies in 2001. In 2021, ASHP notes 168
ASHP-accredited critical care pharmacy residencies.
3. Answer: D Assuming linear growth, the 160-residency increase
In a 2009 study by MacClaren and Bond, mortality over 20 years equals an increase of eight residencies
increased in thromboembolic and infarction-related per year (Answer B is correct). Although this represents
events in ICU patients without clinical pharmacy services significant growth, more than 2200 pharmacists would
compared with ICU patients with clinical pharmacy ser- be needed to provide critical care pharmacy services,
vices: 37%, p<0.0001 (Answer D). Although the impact assuming 30 patients/pharmacists in the more than
of clinical pharmacists in affecting QTc-interval prolon- 67,000 adult ICU beds in the United States as of 2009.
gation (Answer A), preventable adverse drug interactions Answer A (5 residencies/year), Answer C (12 residencies/
(Answer B), and drug-drug interactions (Answer C) has year), and Answer D (15 residencies/year) are incorrect.
been evaluated, differences in mortality have not been
documented. Therefore, these answers are incorrect. 7. Answer: C
The preferred and recommended pathway to training
4. Answer: A in critical care pharmacy is a PGY1 pharmacy practice
The core knowledge areas for pharmacists caring for residency, followed by a PGY2 critical care residency.
critically ill patients include pulmonary, cardiology, This is especially true for the provision of desirable-to-
psychiatry, oncology, neuroscience, nephrology, hepatol- optimal pharmacy services in ICUs providing level I and
ogy, nutrition, gastroenterology, surgery, trauma, burn, II services. Critical care fellowship training is an option
pharmacology, transplantation, supportive care, medical that would also be considered preferred; however, the
emergencies, immunology, endocrinology, hematology, intent is for a greater research and academic focus. The
nephrology, toxicology, and surgery. Therefore, nephrol- demands of the workplace often exceed the supply of
ogy (Answer A) is correct. Dermatology (Answer B), PGY2-trained critical care pharmacists. And although

ACCP Updates in Therapeutics® 2022: Critical Care Pharmacy Preparatory Review and Recertification Course

43
Critical Care Pharmacy Evolution and Validation, Practice Standards, Training, and Professional Development

many of the alternative pathways available to gaining 10. Answer: D


experience, knowledge, and skills in critical care phar- Continuing pharmacy education should be included
macy have been successful, these are not considered as an important strategy in a CPD PDP, and therefore
preferred pathways, given the high degree of variability options A and B are incorrect. The self-directed learner
and inconsistency of resources to support them, there- should select CPE programs that are relevant to their
fore options A, B and D are non-preferred alternative PDP, incorporate active learning strategies, are prefer-
pathways and are incorrect. ably curricular based, and are free of commercial or
other bias. Continuing professional development is not
8. Answer: C an alternative to CPE; rather, it is an individualized, self-
Even though the principles of team-based health care directed, continuous, and iterative process intended to
are all interdependent, effective communication is most address specific learning objectives developed over time
tightly linked to mutual trust. Open and frank commu- by the pharmacist-learner. Continuing pharmacy educa-
nication and the willingness to state your beliefs and tion is an important component of this process, but it
challenge those of your teammates require a high level should not be the only learning strategy. Both CPD and
of mutual trust to keep the relationship and conversa- CPE can include multiple learning strategies, so option
tion professional and nonpersonal. Without mutual trust, C is incorrect.
communication can be more guarded, ineffective, and
political. Options A, B and D are relevant principles of 11. Answer: C
team-based healthcare, but not as tightly linked to effec- The recently published Standards of Practice for
tive communication, and are incorrect. Clinical Pharmacy, which includes a standardized pro-
cess of care endorsed by all major pharmacy practitioner
9. Answer: B organizations, is intended to be applicable to any prac-
Although structured mentor-mentee programs can be tice environment, regardless of acuity or complexity.
successful, there is a greater probability of success with Like other professions (e.g., medicine, nursing, physi-
relationships that are voluntary and that evolve from cal therapy), clinical pharmacy must define and apply
mutual interests and a perceived opportunity to have a standards of care to create consistent expectations by all
mutually beneficial relationship. Mentors must be will- stakeholders. It is often argued that clinical pharmacists
ing to serve in this role, which can require a great deal in different complex or unique practice environments
of time and effort; they wish to work with mentees who cannot possibly conform to a standard of care; how-
are highly motivated with a track record of accepting ever, a thoughtful review of the Standards of Practice
challenges. Moreover, mentees must be responsive to for Clinical Pharmacy reveals that it can be easily incor-
feedback and teaching. Mentees seek out mentors with porated into any practice environment, and therefore
shared interests, a record of sharing their time and options A, B and D are incorrect.
expertise, and the necessary prestige and position in the
organization to promote and create opportunities for 12. Answer: A
them. In an arranged relationship, it is less likely that Clinical pharmacists practicing in the ICU have long
all of these factors will come together to lead to a highly had a broad educational role that includes students and
productive relationship. Mentoring relationships can residents in their own profession, residents and students
exist both within and outside formal training programs in other professions, and colleagues on the critical care
like residencies and fellowships so option A is incorrect, team, as well as coworkers in the pharmacy department,
and clinical pharmacists will often have several mentors among the target audiences. Clinical pharmacists have
through the different stages of their career to address used many different strategies and techniques to teach
different and evolving needs as they mature in their these diverse audiences across different learning envi-
practice and scholarly activities so option D is incor- ronments. Although there are exceptions, the frequent
rect. Finally, although mentored training programs are a or regular inclusion of patients and families in their edu-
viable option for the nonconventional training of critical cational activities is a more recent development. Many
care pharmacists, they are not an exclusive pathway so factors have led to this change, including a greater focus
option C is incorrect. on patient- and family-centered care, greater inclusion of

ACCP Updates in Therapeutics® 2022: Critical Care Pharmacy Preparatory Review and Recertification Course

44
Critical Care Pharmacy Evolution and Validation, Practice Standards, Training, and Professional Development

patients and families as members of the team, increased


demand for inclusion by patients and families, inclusion
of patient satisfaction scores in pay-for-performance
metrics (including pain control and understanding of
medications), and greater patient awareness and inter-
action in the ICU with changes in sedation goals. It is
anticipated that clinical pharmacists in the ICU will
increasingly be directly involved in patient and fam-
ily discussions and education. Options B, C and D all
represent traditional audiences for clinical pharmacist
educational efforts and are therefore incorrect.

13. Answer: B
In the context of CPD, episodic opportunities for reflec-
tion refer to spontaneous, unscheduled events that
contribute to the self-assessment of learning and train-
ing needs that can be incorporated into the overall PDP.
Scheduled reflection usually involves a predictable cycle
like performance evaluations, annual self-evaluations,
peer feedback as part of annual assessment, or a deci-
sion to schedule reflection around some set anniversary
(e.g., hire dates, birthdays, end of academic year), and
therefore options A, C and D are incorrect. Examples
of opportunity for episodic reflection may include the
challenges of managing a very difficult case, post-
event debriefings for code responses, experiences with
a difficult student or resident, or a request to develop a
treatment guideline that is outside the clinician’s usual
area of expertise.

14. Answer: B
A methodological flaw that results in the collection and
reporting of incorrect data would be considered a “fatal
flaw” that no amount of rewriting or reanalysis could
correct. Poor writing can be corrected with revision by
the authors or during copyediting, but if the study is well
conducted and has value, this would not necessarily be
a reason to recommend rejection so option A is incor-
rect. Disagreements concerning statistical analysis are
not uncommon; however, the authors may have a valid
explanation or be able to revise the statistical analysis
if it is not a major departure from the original intent of
the study meaning option C is incorrect. It is also not
uncommon for the abstract to disagree in some way with
the body of the manuscript, and there is an opportu-
nity to correct that during revisions so option D is also
incorrect.

ACCP Updates in Therapeutics® 2022: Critical Care Pharmacy Preparatory Review and Recertification Course

45
Supportive and
Preventive Medicine
Jerry Altshuler, Pharm.D., BCPS, BCCCP
Hackensack Meridian Health
JFK Medical Center
Comprehensive Pharmacy Services
Edison, New Jersey

Sara Radparvar, Pharm.D., BCPS, BCCCP


The Mount Sinai Hospital
New York, New York
Supportive and Preventive Medicine

Supportive and
Preventive Medicine
Jerry Altshuler, Pharm.D., BCPS, BCCCP
Hackensack Meridian Health
JFK Medical Center
Comprehensive Pharmacy Services
Edison, New Jersey

Sara Radparvar, Pharm.D., BCPS, BCCCP


The Mount Sinai Hospital
New York, New York

ACCP Updates in Therapeutics® 2022: Critical Care Pharmacy Preparatory Review and Recertification Course

49
Supportive and Preventive Medicine

Learning Objectives C. If appropriate, initiate sedation interruption,


and ensure that patients’ heads are elevated 45
1. Identify the key components of intensive care medi- degrees above the bed, if not contraindicated.
cine that can be applied to all critically ill patients. D. Assess the need for VTE prophylaxis in patients
2. Recommend therapeutic options to prevent stress- admitted to the ICU, and initiate an insulin infu-
related mucosal disease. sion to maintain a blood glucose of 120 mg/dL.
3. Recommend therapeutic options to prevent venous
thromboembolism in a critically ill patient. 2. Regarding pharmacologic prophylaxis for stress-
4. Compare therapeutic options for patients with hepa- related mucosal injury, which is most appropriate?
rin-induced thrombocytopenia. A. Sucralfate neutralizes gastric pH.
5. Discuss medications that can be used to comfort a B. Proton pump inhibitors (PPIs) are superior to
critically ill patient at the end of life. histamine-2 receptor antagonists (H2RAs) in
preventing clinically significant bleeding.
C. Tolerance will occur with continued H2RA
Abbreviations in This Chapter administration.
D. Antacids are effective when used up to three
aPTT Activated partial thromboplastin time times daily.
CDI Clostridioides (formerly Clostridium)
difficile infection 3. A 66-year-old man is admitted to the ICU with
DOAC Direct oral anticoagulant abdominal pain, nausea, and altered mental status.
DVT Deep venous thrombosis He has a history of alcoholic cirrhosis, atrial fibril-
H2RA Histamine-2 receptor antagonist lation, and erosive esophagitis. He is intubated
HIT Heparin-induced thrombocytopenia and stabilized on the ventilator. A nasogastric tube
ICU Intensive care unit (NGT) is placed, and the patient is tolerating enteral
NGT Nasogastric tube tube feedings. Which would be best to recommend
PF4 Platelet factor 4 for preventing stress-related bleeding?
PPI Proton pump inhibitor
A. Pantoprazole 40 mg intravenously twice daily.
SRMD Stress-related mucosal disease
B. Ranitidine 50 mg intravenously three times
SUP Stress ulcer prophylaxis
daily.
VTE Venous thromboembolism
C. Famotidine 20 mg twice daily by NGT.
D. Omeprazole suspension 20 mg once daily by
NGT.
Self-Assessment Questions
Answers and explanations to these questions may be
4. A 51-year-old woman is admitted to the ICU for
found at the end of this chapter.
hypovolemic shock secondary to severe dehydra-
tion. She reports a 5-day history of diarrhea and
1. On rounds, you have a “checklist” of interventions
malaise. She has no recent history of illnesses or
that will benefit all critically ill patients in an inten-
contact with health care personnel. Her medical his-
sive care unit (ICU). Which interventions would be
tory includes hypothyroidism and gastroesophageal
most effective?
reflux disease. Her medications include levothyrox-
A. Initiate stress ulcer prophylaxis (SUP) in ine 25 mcg orally daily and famotidine 20 mg orally
patients who are admitted to the ICU, and if at bedtime. Recently, her primary care physician
appropriate, discontinue sedation. changed famotidine to omeprazole 20 mg orally
B. Initiate enteral nutrition, when appropriate, and at bedtime for increased gastroesophageal reflux
mechanical venous thromboembolism (VTE) disease symptoms. While she is in the ICU, testing
prophylaxis. for Clostridioides (formerly Clostridium) difficile
infection (CDI) returns positive. Which would be
most appropriate regarding PPI use and CDI?

ACCP Updates in Therapeutics® 2022: Critical Care Pharmacy Preparatory Review and Recertification Course

50
Supportive and Preventive Medicine

A. PPIs are a potential risk factor for CDI by pro- 7. A 34-year-old man (weight 70 kg) is admitted to the
ducing hypochlorhydria and increasing the host surgical ICU for acute respiratory failure from pan-
susceptibility to infections. creatitis. He has no pertinent medical history. His
B. Prospective randomized controlled trials have current medications include norepinephrine at 0.07
shown that the risk of CDI is associated with mcg/kg/minute, dexmedetomidine at 0.7 mcg/kg/
PPI use. hour, ampicillin/sulbactam 3 g intravenously every
C. There is no association between PPI use and 6 hours, famotidine 20 mg intravenously twice daily,
CDI risk. and heparin 5000 units subcutaneously three times
D. Studies reporting on CDI and PPI use have used daily. On day 3 of his ICU admission, the team sus-
the same definition of CDI and implemented pects heparin-induced thrombocytopenia (HIT). His
the same infection control practices. platelet count (Plt) was 360,000/mm3 on admission
and is 180,000/mm3 today. The 4Ts score is used to
5. A 50-year-old woman (weight 70 kg) is admitted determine the probability of HIT. The score is cal-
to the ICU for worsening mental status. Her medi- culated as 3 equals low risk. The team would like
cal history is significant for hypertension, tobacco to send the heparin–platelet factor 4 (PF4) immu-
use, and osteoporosis. The next morning, she is noassay and initiate argatroban. Which is the most
intubated and stabilized on a ventilator. An NGT is appropriate response?
placed. Her current medications include ceftriaxone A. Discontinue all heparin products, but do not ini-
2 g intravenously every 12 hours, vancomycin 1250 tiate argatroban.
mg intravenously every 12 hours, acyclovir 800 mg B. Discontinue all heparin products, and initiate
intravenously every 8 hours, famotidine 20 mg by argatroban.
NGT twice daily, and a bowel regimen. Serum cre- C. Send the heparin-PF4 immunoassay, and con-
atinine (SCr) is normal. Which would be the most tinue low-dose unfractionated heparin until the
appropriate VTE prophylaxis for this patient? results return.
A. Intermittent pneumatic compression devices. D. Do not send the heparin-PF4 immunoassay, and
B. Enoxaparin 40 mg subcutaneously daily. continue low-dose unfractionated heparin.
C. Unfractionated heparin continuous infusion to
maintain a therapeutic activated partial throm- 8. Which would be the most important group of con-
boplastin time (aPTT). siderations in a critically ill patient approaching the
D. No VTE prophylaxis at this time. end of life?
A. Pain management, tight glucose management,
6. A 34-year-old woman (weight 65 kg) is admitted and control of secretions.
to the ICU with several upper extremity fractures, B. Routine vital sign checks, discontinuation
a closed-head injury, and a grade 4 liver laceration of unnecessary medications, and control of
after a motor vehicle crash. Her medical history is secretions.
nonsignificant. She is admitted to the ICU on a ven- C. Pain management, control of secretions, and
tilator after surgery. Her current laboratory values discontinuation of unnecessary medications.
are as follows: sodium (Na) 145 mEq/L, potassium D. Discontinuation of unnecessary medications,
(K) 3.1 mEq/L, chloride 97 mEq/L, carbon diox- insertion of a Foley catheter, and treatment of
ide 18 mEq/L, blood urea nitrogen (BUN) 70 mg/ nausea and vomiting.
dL, and SCr 3.5 mg/dL. Which would be the most
appropriate VTE prophylaxis on the day of admis-
sion for this patient?
A. Provide intermittent pneumatic compression
devices.
B. Give dalteparin 5000 units subcutaneously daily.
C. Give fondaparinux 2.5 mg subcutaneously daily.
D. No VTE prophylaxis is indicated at this time.

ACCP Updates in Therapeutics® 2022: Critical Care Pharmacy Preparatory Review and Recertification Course

51
Supportive and Preventive Medicine

BPS Critical Care Pharmacy Specialist Examination Content Outline

This chapter covers the following sections of the Critical Care Pharmacy Specialist Examination Content Outline:
1. Domain 1: Clinical Knowledge and Application
a. Task 1: 1, 3
b. Task 2: 1-4
c. Task 3: 1-7
d. Task 4: 1-7
e. Task 5: 1, 2, 5, 6
f. Task 6: 1-4, 6
g. Task 7: 1-3, 7
2. Domain 2: Practice Management, Policy, and Quality Improvement
a. Task 1: 2, 3
b. Task 2: 2, 4
3. Domain 3: Evidence-Based Medicine, Scholarship, Education, and Professional Development
a. Task 1: 3
b. Task 3: 1
c. Task 4: 1, 2

ACCP Updates in Therapeutics® 2022: Critical Care Pharmacy Preparatory Review and Recertification Course

52
Supportive and Preventive Medicine

I.  KEY ASPECTS IN THE GENERAL CARE OF ALL CRITICALLY ILL PATIENTS

A. The FAST-HUG mnemonic emphasizes important aspects of ICU medicine that can be applied at least daily
to all critically ill patients to ensure safe, effective, and efficient care (Crit Care Med 2005;33:1225-9).

Table 1. Key Elements of the FAST-HUG Approach


Element Importance Considerations
Malnutrition can lead to
impaired immune function,
• Initiate enteral feeding (preferred to parenteral nutrition)
in turn leading to increased
as soon as possible, typically within the first 24–48 hr
susceptibility to infection,
Feeding after stabilization
inadequate wound healing,
• (See Fluids, Electrolytes, Acid-Based Disorders, and
bacterial overgrowth in the GI
Nutrition Support chapter for a more in-depth review)
tract, and increased propensity
for decubitus ulcers
• Pain should regularly be assessed with a validated tool
such as the behavioral pain scale (BPS) or the Critical-
Care Pain Observation Tool
• Preemptive analgesia should be considered for invasive
Analgesia
Analgesic and sedative or potentially painful clinical procedures
administration optimizes • (See Management of Pain, Agitation, and
patient comfort and minimizes Neuromuscular Blockade in Adult Intensive Care Unit
the acute stress response Patients chapter for a more in-depth review)
(hypermetabolism, increased • Sedation should be assessed and reassessed with a
oxygen consumption, validated tool such as the Richmond Agitation-Sedation
hypercoagulability, and Scale or the Sedation Agitation Scale
alterations in immune function) • Maintain light levels of sedation
Sedation
• If appropriate, execute daily sedative interruption
• (See Management of Pain, Agitation, and
Neuromuscular Blockade in Adult Intensive Care Unit
Patients chapter for a more in-depth review)
• Initiate appropriate prophylaxis, considering VTE and
bleeding risks
Thromboembolic Most ICU patients have at least • Mechanical prophylaxis (graduated compression
prophylaxis one risk factor for VTE stockings or intermittent pneumatic compression
devices) is an alternative nonpharmacologic option in
patients at high risk of bleeding
Elevating the head and thorax
above bed to a 30–45 degree
• Where appropriate, ensure patient position periodically
Head of bed angle reduces the occurrence
throughout the day, especially after procedures that
elevation of GI reflux and nosocomial
require the patient to lie flat
pneumonia in patients who are
receiving mechanical ventilation
Critically ill patients develop
Stress Ulcer stress-related mucosal damage, • Consider discontinuing acid-suppressive medications
prophylaxis potentially leading to clinically when risk factors are no longer present
significant bleeding

ACCP Updates in Therapeutics® 2022: Critical Care Pharmacy Preparatory Review and Recertification Course

53
Supportive and Preventive Medicine

Table 1. Key Elements of the FAST-HUG Approach (continued)


Element Importance Considerations
Glycemic control is necessary in
critically ill patients to decrease • Maintaining blood glucose at 140–180 mg/ dL should be
the incidence of complications considered in the acutely ill patient when blood glucose
Glycemic control such as decreased wound concentrations are ≥ 150 mg/dL
healing, increased infection • (See Fluids, Electrolytes, Acid-Based Disorders, and
risk, and increased risk of Nutrition Support chapter for a more in-depth review)
polyneuropathy
GI = gastrointestinal; ICU = intensive care unit; VTE = venous thromboembolism.

B. Updated Mnemonic: FAST-HUGS BID (Crit Care Med 2009;37:2326-7; author reply 2327)
S = spontaneous breathing trial
B = bowel regimen
I = indwelling catheter removal
D = de-escalation of antimicrobials

C. MAIDENS: Additional Mnemonic with a Medication Focus (Can J Hosp Pharm 2011;64:366-9)
M = medication reconciliation
A = antibiotics or anti-infectives
I = indications for medications
D = drug dosing
E = electrolytes, hematology, and other laboratory results
N = no drug interactions, allergies, duplications, adverse effects
S = stop dates

D. Daily Checklists
1. Checklists provide a framework of standardization and regulation of interventions in a systematic
manner, allowing individuals to assess the presence or absence of the items.
2. Provides structure to important ICU-related interventions in an effort to reduce errors of omission and
increase compliance with evidence-based practices to improve outcomes in the ICU patient population
(N Engl J Med 2009;360:491-9; N Engl J Med 2006;355:2725-32)

ACCP Updates in Therapeutics® 2022: Critical Care Pharmacy Preparatory Review and Recertification Course

54
Supportive and Preventive Medicine

Patient Case

1. A 68-year-old man (weight 85 kg) is admitted to the ICU for the management of severe hypoxemic respira-
tory failure associated with community-acquired pneumonia. He is endotracheally intubated and placed on
mechanical ventilation. His medical history consists of Child-Pugh class B cirrhosis secondary to alcohol
abuse, heart failure, and myocardial infarction. His laboratory values show white blood cell count (WBC)
15 x 103 cells/mm3, Plt 150,000/mm3, BUN 15 mg/dL, SCr 1.1 mg/dL, K 4.5 mEq/L, international normal-
ized ratio (INR) 1.0, aspartate aminotransferase (AST) 58 IU/mL, and alanine aminotransferase (ALT) 49
IU/mL. His current medications include azithromycin 500 mg intravenously daily, ceftriaxone 1 g intrave-
nously daily, vancomycin 1250 mg intravenously every 12 hours, heparin 5000 units subcutaneously every
8 hours, fentanyl drip at 50 mcg/hour, midazolam drip at 1 mg/hour titrated to a Richmond Agitation-
Sedation Scale (RASS) of 0 to -1, and a regular insulin drip at 1.5 units/hour titrated to maintain blood
glucose at 140–180 mg/dL. Currently, on day 3 of his ICU stay, the patient’s head is 30 degrees above the
bed, his RASS is documented as -4, he is on minimal ventilator settings, and an NGT is placed. As the
clinical pharmacist rounding on this patient, you go through the FAST-HUG mnemonic. Which are the best
recommendations for the team?
A. Initiate enteral nutrition by NGT, add SUP, and discontinue fentanyl and midazolam drips.
B. Initiate enteral nutrition by NGT, discontinue deep venous thrombosis (DVT) prophylaxis, and transi-
tion the insulin drip to sliding scale.
C. Transition the insulin drip to sliding scale, add SUP, and discontinue fentanyl and midazolam drips.
D. Discontinue fentanyl and midazolam drips, discontinue DVT prophylaxis, and add SUP.

II.  STRESS ULCER PROPHYLAXIS

A. Epidemiology of Stress-Related Mucosal Disease (SRMD)


1. Endoscopic evidence of superficial mucosal damage occurs in 75%–100% of patients within 1–2 days
after ICU admission.
2. Mortality from stress-related bleeding is 50%–70% in the critically ill population, with a 20% mortality
rate attributable to SRMD.
3. Clinically significant stress-related bleeding has decreased during the past decade because of factors
that include early resuscitation and use of SUP.

B. Characteristics of SRMD
1. Several superficial erosive lesions occurring early in the course of critical illness, potentially progressing
to deep ulcers
2. Stress ulcers are diffuse and unamenable to endoscopic therapy; they generally heal over time, without
intervention, as the patient’s clinical status improves.

Table 2. Stress Ulcers vs. Peptic Ulcers


Stress Ulcers Peptic Ulcers
Several superficial lesions at the proximal stomach bulb; Few deep lesions in the duodenum; typically involves
involves superficial capillaries; results from splanchnic a single vessel; results from break in gastric, duodenal,
hypoperfusion or esophageal lining from the corrosive action of pepsin

ACCP Updates in Therapeutics® 2022: Critical Care Pharmacy Preparatory Review and Recertification Course

55
Supportive and Preventive Medicine

C. Pathophysiology of SRMD (N Engl J Med 2018;378:2506-16)


1. Decreased gastric blood flow and mucosal ischemia are the primary causes of stress ulcer–related
bleeding.
2. Reduced splanchnic blood flow is caused by mechanisms common to critical illness:
a. Hypovolemia
b. Reduced cardiac output
c. Proinflammatory mediator release
d. Increased catecholamine release
e. Visceral vasoconstriction
3. Additional factors leading to stress ulcer–related bleeding:
a. Decreased gastric mucosal bicarbonate production
b. Decreased gastric emptying of irritants and acidic contents
c. Acid back-diffusion
d. Reperfusion injury that may occur after restoration of blood flow after prolonged periods of
hypoperfusion

D. Risk Factors for Stress-Related Bleeding (N Engl J Med 1994;330:377-81)


1. Independent risk factors for SRMD and bleeding are respiratory failure requiring mechanical ventilation
for 48 hours or longer OR coagulopathy (Plt less than 50,000 cells/mm3, INR greater than 1.5, or aPTT
greater than 2 times the control).
a. Patients with at least one risk factor had a 3.7% incidence of bleeding compared with a 0.1%
incidence if risk factors were absent.
b. Most of the 2252 patients enrolled in this study were cardiothoracic patients, potentially making
extrapolations to other ICU settings inaccurate.
c. Prophylaxis was discouraged unless patients had head injury, burns over greater than 30% of total
body surface area, organ transplantation, evidence of peptic ulcer disease, or upper gastrointestinal
(GI) bleeding within the previous 6 weeks. Overall, 674 of the 2252 enrolled patients (29.9%)
received prophylaxis.
2. Variables associated with the risk of GI bleeding while receiving prophylaxis:
a. Renal failure (Crit Care Med 1999;27:2812-7)
b. Age (50 or older), male sex, acute respiratory failure, myocardial infarction, acute kidney injury,
neurologic injury, sepsis, shock, acute or chronic hepatic failure, and coagulopathy (JAMA Intern
Med 2014;174:564-74)
c. Severity of illness, liver disease, and renal replacement therapy (Intensive Care Med 2015;41:833-
45)
3. Other risk factors for SRMD and bleeding (Am J Health Syst Pharm 1999;56:347-79; J Trauma
1995;39:289-94):
a. Spinal cord/head trauma
b. Thermal injury affecting more than 35% of total body surface area
c. History of GI bleed within the past year
d. Postoperative transplantation
e. Ulcerogenic medications (nonsteroidal anti-inflammatory drugs, aspirin, corticosteroids)
f. Enteral nutrition has been suggested as a protective variable (Crit Care Med 2010;38:2222-8; Crit
Care 2018;43:108-13)

E. Pharmacologic Therapy for Preventing Stress Ulcers


1. Antacids
a. Dose-dependent neutralization of gastric acid

ACCP Updates in Therapeutics® 2022: Critical Care Pharmacy Preparatory Review and Recertification Course

56
Supportive and Preventive Medicine

b. Not recommended for routine use because of frequency of administration (up to every hour),
adverse effects (diarrhea, constipation, electrolyte abnormalities), and interactions (interferes with
the absorption of some drugs)
2. Sucralfate (Carafate)
a. Complexes with albumin and fibrinogen to form a viscous, adhesive substance that adheres to
ulcers with a gastric pH less than 4
b. Not recommended for routine use because of adverse effects (constipation, aluminum toxicity,
hypophosphatemia) and interactions by chelation
3. H2RAs
a. Competitive blockade of histamine receptors on the basolateral membrane of the parietal cells. In
addition, H2RAs inhibit gastrin secretion to reduce acid production; however, they do not reliably
inhibit vagally induced acid secretion.
b. Alternative pathways (e.g., gastrin, acetylcholine) can be up-regulated in response to the use of
H2RAs, leading to a decrease in gastric pH and explaining the tolerance related to H2RAs. Tolerance
can occur as early as day 2 of H2RA use (World J Gastrointest Pharmacol Ther 2014;5:57-62; Am
J Gastroenterol 1999;94:351-7).
c. In animal models, H2RAs may also attenuate reperfusion injury by decreasing interleukin-6 and
neutrophil activation, reducing inflammation by enhancing cell-mediated immunity, and acting as
a weak free radical scavenger.
d. Dose-dependent increase in gastric pH
e. Previous studies of SRMD-related bleeding used either continuous infusion H2RAs or combined
H2RAs with intermittent antacids to maintain a pH greater than 4. Current practice is to use
intermittent administration of H2RAs without pH monitoring.
f. All H2RAs require renal dosing adjustment.
g. Adverse effects
i. Mental status changes such as confusion, hallucinations, agitation, and headaches (mainly
associated with cimetidine, but reported with famotidine and ranitidine in patients with severe
renal failure)
ii. Thrombocytopenia (occurs over several days from hapten formation; may occur within hours
if patient is sensitized)
iii. Rapid infusion–related hypotension
iv. Prolongation of corrected QT interval has been reported with famotidine and ranitidine.
v. Sinus bradycardia
vi. Risk of nosocomial pneumonia
vii. Increase in SCr with cimetidine – Competitively inhibits tubular secretion of creatinine
h. Drug interactions
i. Cimetidine inhibits cytochrome P450 (CYP) isoenzymes 3A4, 2D6, 2C9, 2C19, and 1A2.
ii. pH-dependent interactions
i. Available agents: Cimetidine (continuous infusion is the only H2RA approved by the U.S. Food and
Drug Administration [FDA] for SUP), famotidine, nizatidine, and ranitidine
4. PPIs
a. Prodrugs activated in the acidic environment of the stimulated parietal cell inhibiting both
histamine-induced and vagally mediated gastric acid by binding and inhibiting active proton pumps
b. Dose-dependent increase in gastric pH, with maximal activity reached 3 days after initiation
c. Most trials evaluated the effectiveness of enteral PPIs.
d. Despite short elimination half-lives, PPIs suppress acid secretion for 20 hours or more, permitting
once-daily dosing without requiring gastric pH monitoring.
e. Tachyphylaxis does not occur with PPIs.

ACCP Updates in Therapeutics® 2022: Critical Care Pharmacy Preparatory Review and Recertification Course

57
Supportive and Preventive Medicine

f. Rebound acid hypersecretion may occur after discontinuation; however, clinical relevance is
unknown.
g. Adverse effects
i. Diarrhea, abdominal pain, constipation, nausea
ii. Headaches
iii. Rash
iv. Interstitial nephritis
v. Hypomagnesemia (3 months or more of therapy)
vi. Neurologic effects with high-dose intravenous omeprazole (hearing and vision disturbances)
vii. Hypophosphatemia and metabolic alkalosis when administered with sodium bicarbonate
viii. Vitamin B12 deficiency
ix. Increased risk of fractures (hip, waist, and spine)
x. CDI (definitive cause-effect relationship is not well established)
xi. Risk of nosocomial pneumonia
h. Drug interactions
i. All agents are hepatically metabolized by CYP isoenzymes 3A4 and 2C19.
ii. Omeprazole is an inhibitor of 3A4, 2C19, 2C9, and 1A2.
iii. Lansoprazole may induce CYP1A2.
iv. Clinically significant PPI interactions with clopidogrel through CYP2C19 inhibition have not
consistently been shown to be important.
v. pH-dependent interactions
i. Available agents: Dexlansoprazole, esomeprazole, lansoprazole, omeprazole (immediate-release
capsule FDA approved for SUP), pantoprazole, and rabeprazole

F. Stress-Related Bleeding
1. Endoscopically evident mucosal damage and occult bleeding rates are reported from historical data;
more contemporary data are lacking (Table 3).
2. Clinically important GI bleeding
a. Most trials define clinically important GI bleeding as overt bleeding accompanied by one of the
following:
i. Decrease in blood pressure of 20 mm Hg within 24 hours before or after GI bleeding episode
ii. Decrease in blood pressure of 10 mm Hg and increase in heart rate of 20 beats/minute or more
on orthostatic change.
iii. Decrease in hemoglobin of 2 g/dL and transfusion of 2 units of blood within 24 hours of
bleeding OR failure of the hemoglobin concentration to increase after transfusion by at least
the number of units transfused minus 2 g/dL
b. Antacids, sucralfate, H2RAs, and PPIs have all reduced clinically significant SRMD-related
bleeding compared with placebo.
c. Randomized placebo-controlled trial showed significantly less clinically important GI bleeding
with H2RAs than with sucralfate (relative risk [RR], 0.44; 95% confidence interval [CI] 0.21–
0.92; p=0.02) (N Engl J Med 1998;338:791-7). A recent meta-analysis suggests the risks of overt or
clinically relevant bleeding are similar between sucralfate and H2RAs; however, these conclusions
are limited by considerable changes in practice, such as ventilator bundles and early nutrition, since
the studies were performed (J Crit Care 2017;40:21-30).
d. Three meta-analyses favored PPIs to H2RAs for GI bleeding; however, the studies that were
included lacked methodological quality and had unexpectedly high baseline bleeding rates, a
disproportionate number of risk factors between patient groups, inconsistent definitions of bleeding,
and different routes and dosing of agents (Crit Care Med 2010;38:1197-205; Am J Gastroenterol
2012;107:507-20; Crit Care Med 2013;41:693-705).

ACCP Updates in Therapeutics® 2022: Critical Care Pharmacy Preparatory Review and Recertification Course

58
Supportive and Preventive Medicine

e. A retrospective cohort including more than 35,000 mechanically ventilated patients reported an
odds ratio (OR) of GI bleeding that was greater with PPIs than with H2RAs (2.24; 95% CI, 1.81–
2.76) (JAMA Intern Med 2014;174:564-74).
f. A meta-analysis including seven randomized controlled trials showed no difference in the risk
of GI bleeding in patients receiving SUP compared with those who did not receive SUP when
standard, early enteral nutrition was initiated (Crit Care 2018;22:20).
g. Network meta-analyses involving 57 trials provide moderate-quality evidence of a significant
decrease in the risk of bleeding for three comparisons: PPIs versus H2RAs (OR for bleeding 0.4; CI,
0.2–0.7), PPIs versus no prophylaxis (0.2; CI, 0.1–0.6), and PPIs versus sucralfate (0.3; CI, 0.1–0.7)
(Intensive Care Med 2018;44:1-11).
h. The largest randomized controlled trial to compare stress ulcer prophylaxis in a modern-day setting
with placebo (SUP-ICU trial) enrolled 3298 patients to receive 40 mg of intravenous pantoprazole
daily or placebo. Early enteral nutrition was provided to most patients. Although the primary end
point of mortality did not differ significantly, there was a reduction in the secondary outcome of
clinically important GI bleeding (2.5% vs. 4.2%; RR, 0.58; 95% CI, 0.40–0.86) (N Engl J Med
2018;379:2199-208).
i. A recently published large cluster crossover randomized open-label trial evaluated PPI versus H2RA
in over 26,500 critically ill patients requiring mechanical ventilation. There was no difference in
the primary endpoint of 90 day mortality (RR, 1.05; 95% CI, 1.00–1.10; p=0.54). Of note, this
associated CI could not rule out a potential signal of harm from the use of PPI compared to H2RA.
The secondary endpoint of clinically important bleeding was reduced with PPI versus H2RA from
1.3% to 1.8% (RR, 0.73; 95% CI, 0.57–0.92; p=0.009). Additional secondary endpoints, including
CDI and ventilator-associated conditions, were not different. A major limitation of this study was
the capability of clinicians to prescribe acid-suppressing medications outside of the designated
medication to which the cluster was randomized (units were assigned a default medication for
6 month blocks). This resulted in only 63.4% of patients in the H2RA groups receiving H2RA
exclusively and 82.5% of patients in the PPI group receiving PPI exclusively. (JAMA 2020;323:
616-26).

Table 3. Categories of Stress-Related Bleeding


Incidence in
Outcome Definition
ICU Patients
Endoscopically evident mucosal
75%–100% Superficial lesions identified on endoscopy
damage
Occult bleeding 15%–50% Presence of guaiac-positive stools or nasogastric aspirate
Appearance of coffee grounds in nasogastric aspirate,
Overt or clinically evident bleeding 5%–25%
hematemesis, melena, or hematochezia
Presence of overt bleeding with hemodynamic instability
Clinically important bleeding 1%–5%
and/or blood transfusion within 24 hr of the event

G. Infectious Complications
1. Increases in gastric pH promote bacterial overgrowth, potentially leading to infectious complications.
2. Both H2RAs and PPIs cause changes in gastric pH; however, PPIs have a greater propensity to maintain
a sustained higher pH.
3. PPIs may also have immunosuppressive effects through the inhibition of neutrophils.
4. Pneumonia

ACCP Updates in Therapeutics® 2022: Critical Care Pharmacy Preparatory Review and Recertification Course

59
Supportive and Preventive Medicine

a. Meta-analyses have shown lower pneumonia rates with sucralfate than with H2RAs alone or H2RAs
combined with antacids (JAMA 1996;275:308-14; Crit Care Med 1991;19:942-9).
b. Meta-analyses have failed to show an association between H2RAs and PPIs on the risk of pneumonia
(Crit Care Med 2013;41:693-705; Am J Gastroenterol 2012;107:507-20; Crit Care Med 2010;38:1197-
205); however, a more recent meta-analysis showed an increased risk of pneumonia in patients
receiving SUP and early enteral nutrition (Crit Care 2018;22:20).
c. In two large pharmacoepidemiologic cohorts, an increase in the propensity-adjusted odds of
pneumonia occurred with PPIs compared with H2RAs (OR 1.2; CI, 1.03–1.41) in mechanically
ventilated patients (JAMA Intern Med 2014;174:564-7) and in patients admitted for cardiac surgery
after propensity matching (OR 1.19; CI, 1.03–1.38) (BMJ 2013;347:f5416).
d. Many of the trials included in these analyses had varying definitions of pneumonia.
5. CDI
a. A pharmacoepidemiologic cohort study found that CDI rates were significantly higher with PPIs
than with H2RAs (3.8% vs. 2.2%; p<0.001) (JAMA Intern Med 2014;174:564-7).
b. A large retrospective study found PPI use to be an independent risk factor for developing CDI in
medical ICU patients (OR 3.11; 95% CI, 1.11–8.74) (J Crit Care 2014;29:696).
c. No prospective trials have been large enough to evaluate the risk of CDI in ICU patients.
d. Many published trials have different definitions of CDI, unclear association of antisecretory therapy
initiation and CDI diagnosis, and variable infection control practices.
6. In the SUP-ICU trial, there was no difference in infectious adverse events (new-onset pneumonia or
CDI).
7. SUP duration should be evaluated daily, and SUP should be continued only as long as one or more risk
factors are present.

H. Pharmacoeconomics
1. According to the landmark trial comparing H2RAs with sucralfate, H2RAs may be more cost-effective
because of a reduced incidence of bleeding without an increase in pneumonia rates (N Engl J Med
1998;338:791-7).
2. Cost-effectiveness models have compared H2RAs with PPIs with respect to clinically important
bleeding and adverse effects (ventilator-associated pneumonia [VAP] and CDI).
a. Use of PPI therapy for SUP resulted in a $1250 net cost savings per patient compared with
H2RAs. Univariate sensitivity analysis showed that PPI therapy was not as cost-effective when the
probability of VAP rates was altered (Value Health 2013;16:14-22).
b. Use of H2RA therapy for SUP resulted in a $1095 net cost savings compared with PPIs. Univariate
sensitivity analysis showed that assumptions of pneumonia and bleeding rates were the primary
drivers of incremental costs (Crit Care Med 2014;42:809-15).
3. Initiating SUP in patients at risk and appropriately discontinuing SUP when a patient no longer has any
of the risk factors for stress-related bleeding is the best practice for cost minimization.

I. Guideline Recommendations
1. In 1999, the first guideline from the American Society of Health-System Pharmacists was published
(Am J Health Syst Pharm 1999;56:347-79). The guideline recommended that institutions decide on
H2RAs, antacids, or sucralfate according to safety profile, costs, and ease of administration.
2. In 2008, the Eastern Association for the Surgery of Trauma published a guideline recommending
cytoprotective agents, H2RAs, or PPIs; antacids were not recommended (www.east.org).
3. In 2014, the Danish Society of Intensive Care Medicine and the Danish Society of Anaesthesiology
and Intensive Care Medicine published guidelines suggesting PPIs as the preferred agent (Dan Med J
2014;61:C4811).

ACCP Updates in Therapeutics® 2022: Critical Care Pharmacy Preparatory Review and Recertification Course

60
Supportive and Preventive Medicine

4. In 2016, the Surviving Sepsis guidelines recommended PPIs or H2RAs in patients at risk (Crit Care Med
2017;45:486-552).
5. The 2020 clinical practice guideline for gastrointestinal bleeding prophylaxis for critically ill patients
recommends stress ulcer prophylaxis with a gastrointestinal bleeding risk of at least 4% based on
several well-studied risk factors. Proton pump inhibitors are recommended over H2RAs, and sucralfate
is not recommended. (BMJ 2020;368:l6722).

Patient Case

Questions 2–4 pertain to the following case.


A 45-year-old woman is admitted to the ICU for severe respiratory failure from community-acquired pneumo-
nia. She is endotracheally intubated and placed on mechanical ventilation. An NGT is placed to begin enteral
nutrition. Three days later, she is receiving norepinephrine and vasopressin infusions and appropriate antimicro-
bial agents. Her WBC is 20 x 103 cells/mm3, Plt 45,000/mm3, BUN 70 mg/dL, SCr 4.5 mg/dL (baseline 0.9 mg/
dL), K 4.5 mEq/L, INR 1.4, AST 30 IU/mL, and ALT 46 IU/mL.

2. Which best reflects this patient’s risk factors for stress-related bleeding?
A. Mechanical ventilation.
B. Mechanical ventilation, coagulopathy, and acute kidney injury.
C. Mechanical ventilation, coagulopathy, acute kidney injury, and shock.
D. Mechanical ventilation, coagulopathy, acute kidney injury, shock, and enteral nutrition.

3. Which would be most appropriate for preventing stress-related bleeding?


A. Sucralfate 1 g four times daily by NGT.
B. Magnesium hydroxide 30 mL every 4 hours by NGT.
C. Pantoprazole 40 mg intravenously twice daily.
D. Famotidine 20 mg intravenously daily.

4. One week later, the patient’s respiratory status has greatly improved. She has been off sedation and vaso-
pressors for the past 4 days, working with physical therapy, and is now extubated. Her only medications
include ceftriaxone, heparin subcutaneously, and SUP. Her current laboratory values are as follows: WBC 6
x 103 cells/mm3, Plt 256,000/mm3, BUN 10 mg/dL, SCr 1.1 mg/dL, K 4.0 mEq/L, INR 0.8, AST 15 IU/mL,
and ALT 10 IU/mL. Which would be the most appropriate recommendation regarding this patient’s SUP
regimen?
A. SUP should be continued until hospital discharge.
B. SUP should be continued until ICU discharge.
C. SUP should be discontinued now.
D. SUP should be discontinued once the patient is off antimicrobials.

III.  PROPHYLAXIS AGAINST DEEP VENOUS THROMBOSIS OR PULMONARY EMBOLISM

A. Epidemiology
1. Reported occurrence of DVT is 10%–80% (J Crit Care 2002;17:95-104). Precise incidence in the
critically ill population is challenging because of inconsistencies in patient populations, different
diagnosis strategies, and variable study methodologies.

ACCP Updates in Therapeutics® 2022: Critical Care Pharmacy Preparatory Review and Recertification Course

61
Supportive and Preventive Medicine

2. DVT rates in the absence of prophylaxis vary, depending on the patient population.
a. In the absence of prophylaxis: 30% in medical-surgical patients, 50%–60% in trauma patients, up
to 80% in orthopedic surgical patients, and 20%–50% in neurosurgical patients (Arch Intern Med
2001;161:1268-79)
b. A randomized controlled trial of medical-surgical ICU patients receiving pharmacologic prophylaxis
found proximal DVT rates of 5%–6% using ultrasonography twice weekly for detection (N Engl J
Med 2011;364:1305-14).

B. Risk Factors
1. Malignancy, previous VTE, immobility, known thrombophilia, recent (1 month or less) surgery or
trauma, older age (70 or older), heart or respiratory failure, sepsis, obesity (body mass index of 30
kg/m2 or more), pregnancy, erythropoiesis-stimulating agents with a hemoglobin of 12 g/dL or more,
hormonal therapy, recent transfusions of concentrated clotting factors, central venous lines, and long-
distance travel (Chest 2012;141:S195-226)
2. Additional VTE risk factors in critically ill patients: A single-center prospective cohort (n=261)
identified four independent risk factors for ICU-acquired VTE: personal or family history of VTE
(multivariate hazard ratio [HR] 4.0; 95% CI, 1.5–10.3; p=0.004), end-stage renal failure (HR 3.7; 95%
CI, 1.2–11.1; p=0.02), platelet transfusion (HR 3.2; 95% CI, 1.2–8.4; p=0.02), and vasopressor use (HR
2.8; 95% CI, 1.1–7.2; p=0.03) (Crit Care Med 2005;33:1565-71).
3. In the critically ill population, there are no validated risk assessment models to estimate the risk of
VTE.

C. Prevention of VTE in the General Critically Ill Patient Population


1. Routine ultrasound screening is not recommended (Chest 2012;141:S195-226).
2. Prophylactic use of inferior vena cava filters is not recommended (Chest 2012;141:S195-226).
3. Mechanical VTE prophylaxis should be used in a critically ill patient if the patient is bleeding or at
high risk of bleeding. Once the bleeding risk abates, pharmacologic VTE prophylaxis can be initiated.
a. Intermittent pneumatic compression devices and graduated compression stockings (GCS)
significantly reduce the risk of symptomatic VTE compared with no prophylaxis (Chest
2012;141:S195-226).
b. A Cochrane review evaluated the results from 19 studies of GCS (Cochrane Database Syst Rev
2014;12:1-72).
i. DVT rates were 9% in the GCS group and 21% in the control group (OR 0.33; p<0.00001), and
pulmonary embolism rates were 2% in the GCS group and 5% in the control group (OR 0.38;
p=0.04).
ii. The patient population was largely limited to those undergoing orthopedic and general surgery.
c. Data are limited in medical critically ill patients.
d. The PREVENT trial randomized 2003 critically ill patients to receive either pharmacological
prophylaxis alone or pharmacological prophylaxis with intermittent pneumatic compression
devices. A majority of included patients were medical (>75%). The addition of mechanical
prophylaxis did not result in a difference in the incidence of proximal lower-limb DVT (adjusted
relative risk: 0.89; 95% CI, 0.58–1.36), pulmonary embolism (aRR 0.92; 95% CI, 0.32–2.06), and
the composite of lower limb DVT/pulmonary embolism (aRR 1.11; 95% CI, 0.85–1.44) (N Engl J
Med 2019;380:1305-15).
4. Either low-dose unfractionated heparin or low-molecular-weight heparin should be initiated in a
critically ill patient over no prophylaxis.

ACCP Updates in Therapeutics® 2022: Critical Care Pharmacy Preparatory Review and Recertification Course

62
Supportive and Preventive Medicine

5. The 2018 American Society of Hematology guidelines recommend continuing VTE prophylaxis only
while patients remain admitted in the hospital, over continuing extended prophylaxis (Blood Adv
2018;2:3360-92).

Table 4. Randomized Trials of VTE Prophylaxis in Critically Ill Patients


Major
Screening
Citation Study Type Population Intervention VTE Rates Bleeding
Methods
Rates
LDUH 5000 Daily DVT: 13% in LDUH
119 medical-
Crit Care Med units SC 125I-labeled group vs. 29% in
Single-center surgical ICU NR
1982;10:448-50 twice daily fibrinogen placebo group;
patients
vs. placebo leg scanning p<0.05
Proximal DVT:
14.7% in LDUH 0.6% in
Enoxaparin
Venography group vs. 6.2% in LDUH
30 mg twice
344 patients on days enoxaparin group; group vs.
N Engl J Med daily vs.
Single-center with major 10–14 and p=0.012 2.9% in
1996;335:701-7 LDUH 5000
trauma US if VTE PE: 0% in LDUH enoxaparin
units twice
suspected group vs. 0.8% in group;
daily
enoxaparin group; p=0.12
p=NR
5.6% in
221 MV Nadroparin
Am J Respir DVT: 15.5% in nadroparin
patients (weight Weekly
Crit Care Med Multicenter, nadroparin group group and
with COPD based) SC US and
2000;161:1109- double-blind vs. 28.2% in placebo 2.7% in pla-
(169 patients daily vs. venography
14 group; p=0.045 cebo group;
evaluated) placebo
p=0.28
DVT: 5.6% in LDUH
group vs. 5.9% in
LDUH 5000
1935 patients enoxaparin group
units SC
Thromb with severe vs. 7.0% in placebo
twice daily
Haemost Multicenter, sepsis US on day group; p=NS
vs. enoxa- NR
2009;101:139- double-blind receiving 4–6 PE: 0.4% in LDUH
parin 40 mg
44 drotrecogin group vs. 0.4% in
SC daily vs.
alfa (activated) enoxaparin group
placebo
vs. 0.8% in placebo
group; p=NS
2.7% in
LDUH 5000
156 surgi- US 5–7 days LDUH
units SC DVT: 2.7% in LDUH
Blood Coagul Single- cal patients after surgery group vs.
twice daily group vs. 1.2% in
Fibrinolysis center, undergoing and when 1.2% in
vs. enoxapa- enoxaparin group;
2010;21:57-61 double-blind major elective clinically enoxapa-
rin 40 mg SC p=0.51
surgery indicated rin group;
daily
p=0.48

ACCP Updates in Therapeutics® 2022: Critical Care Pharmacy Preparatory Review and Recertification Course

63
Supportive and Preventive Medicine

Table 4. Randomized Trials of VTE Prophylaxis in Critically Ill Patients (continued)


Major
Screening
Citation Study Type Population Intervention VTE Rates Bleeding
Methods
Rates
3746 medical- LDUH 5000 Proximal DVT: 5.8%
5.6% in
surgical units SC US 2 days in LDUH group vs.
LDUH
ICU patients twice daily after admis- 5.1% in dalteparin
N Engl J Med group vs.
Multicenter, expected to vs. dalteparin sion, twice group; p=0.57
2011;364:1305- 5.5% in
double-blind remain in the 5000 weekly, and PE: 2.3% in LDUH
14 daltepa-
ICU ≥ 3 days international as clinically group vs. 1.3% in
rin group;
(90% medical, units SC indicated dalteparin group;
p=0.98
76% MV) daily p=0.01
COPD = chronic obstructive pulmonary disease; DVT = deep venous thrombosis; LDUH = low-dose unfractionated heparin; MV = mechanically ventilated; PE =
pulmonary embolism; NR = not reported; NS = not significant; SC = subcutaneously; US = ultrasonography.

D. Prevention of VTE in the Non-orthopedic Surgical Patient (Chest 2012;141:S227-77)

Table 5. VTE Prophylaxis Recommendations in Trauma Patients


Risk Level for VTE Risk of Bleeding Prophylaxis
Low-moderate Low LMWH,a LDUH,a or IPCD (all preferred to no prophylaxis)
Highb Low LMWHa or LDUHa with elastic stockings or IPCD
a
If LDUH or LMWH is contraindicated, mechanical prophylaxis with IPCD is preferred to no prophylaxis in the absence of lower-extremity injury.
b
Includes acute spinal cord injury, traumatic brain injury, and spinal surgery from trauma; pharmacologic prophylaxis should be initiated as soon as possible, typically
24–48 hr after the event, but this may depend on the extent of bleeding on head computed tomography.
IPCD = intermittent pneumatic compression device; LMWH = low-molecular-weight heparin.

Table 6. VTE Prophylaxis Recommendations in the General and Abdominal-Pelvic Surgical Patient
Risk Level for VTE Risk of Bleeding Prophylaxis
Very low Low Early ambulation
Low Low IPCD
Low LMWH, LDUH, or IPCD
Moderate
High IPCD
Low LMWH or LDUH with elastic stockings or IPCD
Low with contraindications to
Low-dose aspirin, fondaparinux, or IPCD
High LMWH or LDUH
IPCD until risk of bleeding abates; then pharmacologic
High
prophylaxis should be initiated

ACCP Updates in Therapeutics® 2022: Critical Care Pharmacy Preparatory Review and Recertification Course

64
Supportive and Preventive Medicine

Table 7. Available Agents and Dosing


Dose in Patients with
Dose in Patients with Renal Impairmenta
Normal Renal Function
Enoxaparin 40 mg SC daily 30 mg SC daily
Specific dosage adjustments have not been recommended; accumulation
Dalteparin 5000 units SC daily did not occur in critically ill patients with severe renal insufficiency.
(Arch Intern Med 2008;168:1805-12)
5000 units SC every 8–12 hr: Choosing between every 8 hr and every 12 hr should be based on the
LDUH
patient’s risk of thrombosis and bleeding
2.5 mg SC once daily for Contraindicated; however, doses of 2.5 mg SC every 48 hr have been
Fondaparinux
patients weighing ≥ 50 kg used (Pharmacotherapy 2017;37:1241-48).
a
Estimated CrCl 20–30 mL/min/1.73 m2.
CrCl = creatinine clearance.

E. Considerations for Critically Ill Patients


1. Inability to communicate symptoms (impaired consciousness) and altered physical examination (edema)
make the diagnosis of symptomatic VTE challenging in the critically ill population. Routine screening
for VTE with ultrasonography is not recommended.
2. Dosing frequency of low-dose unfractionated heparin (twice vs. thrice daily): Meta-analyses suggest
no difference in thrombosis or major bleeding rates with twice-daily regimens compared with thrice-
daily regimens in nonsurgical hospitalized patients (Chest 2011;140:374-81). Most of these patients
were not critically ill and had few comorbidities. A more recent observational study using the Premier
Healthcare Database compared thrice-daily with twice-daily dosing in 30,800 critically ill patients. No
significant differences occurred in VTE rates (6.16 vs. 6.23, p=0.8) or bleeding (0.23 vs. 0.33, p=0.084)
in the propensity-matched cohort. VTE and bleeding events were both identified using ICD-9 coding
(Pharmacotherapy 2019;39:232-41).
3. Bioavailability of subcutaneously administered drugs is reduced in critically ill patients with the
concomitant use of vasoactive drugs or the presence of edema, thereby potentially reducing the effect.
4. A high proportion of critically ill patients have acute kidney injury, which may limit the use of low-
molecular-weight heparin. Critically ill patients (n=138) administered prophylactic subcutaneous
dalteparin with an estimated creatinine clearance (CrCl) of less than 30 mL/minute/1.73 m 2 were
evaluated in a prospective study. No evidence was found of accumulation or an increased risk of
bleeding (Arch Intern Med 2008;168:1805-12).
5. Bleeding
a. Bleeding rates in critically ill patients vary, depending on the type of pharmacologic prophylaxis.
b. Patients at high risk of bleeding are often excluded from studies.
c. Patients at high risk of bleeding with a moderate to high risk of VTE may be considered for
mechanical VTE prophylaxis; however, pharmacologic prophylaxis should be reassessed when the
bleeding risk is no longer present.
6. Limited evidence exists to guide dosing in the critically ill population with obesity. An inverse
relationship between body weight and anti-factor Xa (anti-Xa) concentration may exist in patients
with obesity; however, the risk of VTE and the optimal anti-Xa concentrations to achieve are unclear.
Some clinicians may choose heparin 7500 units subcutaneously every 8 hours over heparin 5000 units
subcutaneously every 8 hours in patients with BMI greater than 40 kg/m2; however, several studies,
which were limited by sample size, have failed to demonstrate a clinical benefit (Pharmacotherapy
2016;36:740-8; Hosp Pharm 2016;51:376-81).

ACCP Updates in Therapeutics® 2022: Critical Care Pharmacy Preparatory Review and Recertification Course

65
Supportive and Preventive Medicine

F. Oral Anticoagulants for VTE Prophylaxis


1. The only study evaluating direct oral anticoagulants (DOACs) for VTE prophylaxis in critically ill
patients was a substudy of extended duration (35-42 days) of betrixaban versus shorter duration (10 ±
4 days) low-molecular-weight heparin in critically ill medical patients. Of the 7513 patients in the full
cohort, 703 critically ill patients were included in the substudy. Extended-duration betrixaban reduced
the rate of VTE at 35-42 days from 7.95% to 4.27% (p=0.042). Major bleeding in the two groups was
1.14% and 3.13% (p=0.07). These findings should currently be treated as exploratory, given that the
full study had predefined statistical stipulations that were unmet (Intensive Care Med 2019;45:477-87).
Betrixaban was removed from the market in April 2020 because of a lack of commercial success.
2. Rivaroxaban is noninferior to standard treatments in other settings such as orthopedic surgery.
3. Rivaroxaban 10 mg orally once daily was compared with enoxaparin 40 mg subcutaneously daily
for 10 days, followed by placebo in acutely ill hospitalized patients. The rates of asymptomatic or
symptomatic VTE, pulmonary embolism, or death were comparable; however, the bleeding rates were
increased in the rivaroxaban group (N Engl J Med 2013;368:513-23). A subsequent study evaluating
rivaroxaban 10 mg/day orally with placebo in patients at high risk of VTE demonstrated a decrease in
rates of symptomatic VTE (0.18 vs 0.42%; HR 0.44, 95% CI, 0.22-0.89); however, low overall VTE and
bleeding event rates limit widespread use. Further studies must identify the populations most likely to
benefit from extended prophylaxis (N Engl J Med. 2018;379:1118-27).
4. Low-molecular-weight heparin is preferred to vitamin K antagonists such as warfarin and DOACs for
prophylaxis (Blood Adv 2018;2:3360-92).

G. Heparin-Induced Thrombocytopenia (Chest 2012;141(2 suppl):495S-530S)


1. HIT is a severe, immune-mediated reaction that potentially leads to life-threatening complications
such as myocardial infarction, skin necrosis, stroke, and VTE (around 50%–75% of patients with HIT
develop symptomatic thrombosis).
2. A rare manifestation is delayed-onset HIT, affecting patients exposed to heparin in the recent past
(prior 2 weeks) who present with a new thrombosis and low platelet count.
3. Frequency of HIT
a. Higher in patients receiving unfractionated heparin than in patients receiving low-molecular-
weight heparin, occurring in 1%–5% of patients versus less than 1%, respectively
b. Occurs in less than 1% of ICU patients
c. Higher risk in cardiac or orthopedic surgical patients receiving unfractionated heparin (15%) than
in medical patients (0.1%–1%)
4. Alternative causes of thrombocytopenia in critically ill patients include extracorporeal devices, intra-
aortic balloon pumps, sepsis, disseminated intravascular coagulation, bleeding, and medications.
Platelet counts may decrease after cardiac bypass surgery and subsequently recover; however, a
secondary decrease in platelet count may signal potential HIT.
5. Clinical diagnosis of HIT
a. Suspected when a patient has a decrease in absolute platelet count to less than 150,000/mm3 or a
relative decrease of at least 50% from baseline, skin lesions at injection sites, or systemic reactions
after intravenous boluses
b. Typical onset is 5–10 days after heparin exposure, though onset can be delayed and can occur up
to 3 weeks after therapy cessation.
c. Recent heparin exposure may result in rapid-onset HIT, occurring within hours after rechallenge.
d. Patients with recent unfractionated heparin/low-molecular-weight heparin exposure and a new
thrombosis should have their platelet count checked before anticoagulant therapy is initiated.

ACCP Updates in Therapeutics® 2022: Critical Care Pharmacy Preparatory Review and Recertification Course

66
Supportive and Preventive Medicine

6. Probability of HIT
a. 4T score
i. Segregates patients into low, intermediate, and high clinical probability on the basis of four
criteria
ii. Four clinical features incorporated: (1) thrombocytopenia, (2) timing of thrombocytopenia, (3)
presence of thrombosis or other clinical sequelae, and (4) other causes of thrombocytopenia
iii. A low probability 4T score has a high negative predictive value (0.998), whereas an intermediate
or high probability score has a low positive predictive value (0.64 and 0.14, respectively) (Blood
2012;120:4160-7).
b. HEP (HIT Expert Probability) score
i. Incorporates more clinical features than the 4T score
ii. More complex (time-consuming) and evaluation studies limited (not in ICU patients)
7. Laboratory testing
a. Antigen assays
i. Depends on the detection of antibody binding by enzyme-linked immunosorbent assays or
particle-based immunoassays
ii. A recently approved automated latex immunoturbidimetric assay has been made available. This
assay does not require sample batching as is common with the enzyme-Linked immunosorbent
assays; thus, results can be obtained within 20 minutes.
iii. Antibody present if sample from patient binds to the heparin-PF4–coated wells, leading to a
color-producing reaction. A higher antibody concentration leads to greater color production
and a higher optical density reading. Optical density readings of 0.4 or greater are considered
positive and indicative of HIT antibodies.
iv. High sensitivity (greater than 90%) and low to moderate specificity
(a) Clinically insignificant HIT antibodies are often detected among patients who received
heparin 5–100 days earlier.
(b) Detects a range of immunoglobulin (Ig) A and IgM antibodies that are not pathogenic
b. Functional assays
i. Examples: Heparin-induced platelet aggregation (HIPA) and C14 serotonin release assay
ii. Detect platelet activation in the presence of heparin. Patient serum is mixed with washed
platelets from healthy volunteers and low and high concentrations of heparin. In the presence
of HIT antibodies, platelets are activated in low concentrations of heparin and detected using
radioactive serotonin (serotonin release assay) or visually (HIPA).
iii. High sensitivity and specificity
iv. Technically challenging and not readily available
8. Treatment of HIT
a. Immediately discontinue all sources of heparin, and initiate an alternative non-heparin anticoagulant.
b. Parenteral direct thrombin inhibitors are the agents of choice for anticoagulation in acute HIT
because they have no cross-reactivity with heparin. Some studies support the use of the factor Xa
inhibitor fondaparinux for the treatment of HIT, though there are reports of fondaparinux-induced
HIT.
c. Parenteral direct thrombin inhibitors are associated with a higher rate of major bleeding
complications than is unfractionated heparin.
d. Initiate warfarin once the platelet count has recovered and is within normal limits (at least
150,000/mm3) and after at least 5 days of therapy with an alternative anticoagulant. Alternatively,
conservative warfarin dosing may begin once the platelet count is recovering. If a patient is
receiving warfarin at the time of HIT diagnosis, reversing with vitamin K is recommended.

ACCP Updates in Therapeutics® 2022: Critical Care Pharmacy Preparatory Review and Recertification Course

67
Supportive and Preventive Medicine

e. DOACs are another option that significantly simplify HIT management. Currently, data analyses
supporting DOACs for HIT are limited to small observational cohort studies. DOACs are a
recommended therapeutic option in the American Society of Hematology 2018 guidelines for VTE
management in HIT.
f. Argatroban dosing in the critically ill population (Crit Care 2010;14:R90; Ann Pharmacother
2007;41:749-54)
i. Mean dose in critically ill patients was 0.24 ± 0.16 mcg/kg/minute and was 0.22 ± 0.15 mcg/
kg/minute in critically ill patients with multiple organ dysfunction.
ii. In patients with severe liver impairment, consider 0.5 mcg/kg/minute.
iii. The target aPTT is 1.5–3 times baseline.
g. Bivalirudin dosing in the critically ill population (Pharmacotherapy 2006;26:452-60)
i. Dose reduced to 0.05–0.1 mg/kg/hour, depending on renal function and bleeding risks
ii. The target aPTT is 1.5–2.5 times baseline.

Table 8. Parenteral Agents for the Treatment of HIT

Argatroban Bivalirudin (Angiomax) Fondaparinux (Arixtra)

FDA approved for the Yes (percutaneous coronary


Yes No
treatment of HIT intervention with HIT)
Mechanism of action Direct thrombin inhibitor Direct thrombin inhibitor Factor Xa inhibitor
Elimination half-life 40–50 min 25 min 17–20 hr
80% enzymatic
Elimination Hepatobiliary Renal
20% renal
Unlabeled dose for HIT: Unlabeled dose for
2 mcg/kg/min
0.15–0.2 mg/kg/hr HIT: 5–10 mg SC daily
Dosing (see above for dosing in the
(see above for dosing in the (depending on weight); 2.5
critically ill population)
critically ill population) mg/day for prophylaxis
Monitoring aPTT aPTT Anti-Xa concentration
Effect on INR Excessive Moderate None
aPTT = activated partial thromboplastin time; HIT = heparin-induced thrombocytopenia.

Patient Cases

5. A 93-year-old man (weight 45 kg) confined to his bed is admitted from a nursing home with a chronic
obstructive pulmonary disease exacerbation requiring mechanical ventilation. He has a history of diabetes
mellitus and heart failure. His laboratory values are all within normal limits except for BUN 35 mg/dL and
SCr 2.8 mg/dL (baseline 0.5). Which would be the most appropriate recommendation for VTE prophylaxis
in this patient?
A. Intermittent pneumatic compression devices.
B. Enoxaparin 30 mg subcutaneously once daily.
C. Heparin 5000 units subcutaneously twice daily.
D. Fondaparinux 2.5 mg subcutaneously daily.

ACCP Updates in Therapeutics® 2022: Critical Care Pharmacy Preparatory Review and Recertification Course

68
Supportive and Preventive Medicine

Patient Cases (continued)

Questions 6 and 7 pertain to the following case.


A 55-year-old man (weight 60 kg) with a medical history of diabetes mellitus, hyperlipidemia, and a DVT 4 months
ago secondary to lower-extremity trauma is admitted today to the ICU for acute respiratory failure from influenza
virus. His current laboratory values are as follows: WBC 13.1 x 103 cells/mm3, Plt 250,000/mm3, BUN 13 mg/dL,
SCr 0.9 mg/dL, INR 1.2, AST 22 IU/mL, and ALT 11 IU/mL. His current medication regimen includes fentanyl
and midazolam boluses for pain and agitation, piperacillin/tazobactam, vancomycin, regular insulin infusion, SUP,
and a heparin drip. Five days later, the patient remains intubated on the same medications. At this time, his Plt
has decreased to 112,000/mm3, and his BUN and SCr have increased to 45 mg/dL and 2.7 mg/dL, respectively.
The team sends a heparin-PF4 immunoassay; however, the results will not return for 48 hours.

6. Which would be the best course of action?


A. Discontinue the heparin drip, and initiate an argatroban continuous infusion at 0.25 mcg/kg/minute.
B. Do nothing because the patient has several other reasons to be thrombocytopenic.
C. Discontinue the heparin drip, and initiate fondaparinux at 10 mg subcutaneously daily.
D. Do nothing until the heparin-PF4 immunoassay results return.

7. Three days later, both the heparin-PF4 immunoassay and the serotonin release assay return positive, and the
patient has a new DVT. The team wants to initiate warfarin. The patient’s current Plt is 130,000/mm3. Which
would be the most appropriate response?
A. Discontinue argatroban and initiate warfarin at 5 mg orally daily.
B. Discontinue argatroban and initiate warfarin at 10 mg orally daily.
C. Warfarin should never be used in patients with HIT.
D. Warfarin should not be initiated right now.

IV.  END-OF-LIFE CARE

A. Clinicians commonly provide end-of-life and palliative care in ICUs.

B. The World Health Organization describes palliative care as “an approach that improves the quality of life
of patients and their families facing the problems associated with life-threatening illness, through the
prevention and relief of suffering by means of early identification and impeccable assessment and treatment
of pain and other problems, physical, psychosocial, and spiritual” (Global Atlas of Palliative Care at the End
of Life).

C. Goals of Palliative Care


1. Provides relief from pain and other distressing symptoms.
2. Affirms life, and regards dying as a normal process.
3. Intends neither to hasten nor postpone death.
4. Integrates the psychological and spiritual aspects of patient care.
5. Offers a support system to help patients live as actively as possible until death.
6. Offers a support system to help the family cope during the patient’s illness and in their own bereavement;
uses a team approach to address the needs of patients and their families, including bereavement
counseling, if indicated.

ACCP Updates in Therapeutics® 2022: Critical Care Pharmacy Preparatory Review and Recertification Course

69
Supportive and Preventive Medicine

7. Enhances quality of life, and may also positively influence the course of illness.
8. Is applicable early in the course of illness, in conjunction with other therapies that are intended to
prolong life, such as chemotherapy or radiation therapy, and includes those investigations needed to
better understand and manage distressing clinical complications.

D. Categories of Support
1. Pain management is of paramount importance for comfort and reduction of distress. Providers and
families can collaborate to identify the sources of pain and relieve them with drugs and other forms of
therapy.
2. Symptom management involves treating symptoms other than pain such as nausea, thirst, bowel and
bladder problems, depression, anxiety, dyspnea, and secretions.
3. Emotional and spiritual support is important for both the patient and the family in dealing with the
emotional demands of critical illness.

E. General Considerations
1. Minimize or discontinue the use of uncomfortable or unnecessary procedures, tests, or treatments.
2. Minimize or discontinue the use of routine vital sign checks, patient weights, cardiac or other electronic
monitoring, fingersticks, and intermittent pneumatic compression devices.
3. Consider discontinuing routine blood tests, radiologic imaging, and other diagnostic procedures.
4. Consider discontinuing all medications not necessary for patient comfort.
5. Neuromuscular blocking agents should be discontinued, and their effects allowed to reverse, to best
assess the patient’s comfort level before withdrawal of life support. If this is not possible due to the
delay that paralysis cessation would contribute to withdrawal of life support, best efforts should be
made to detect discomfort.

F. Symptom Management
1. Pain
a. No evidence supports that unconscious patients do not experience pain.
b. Opioids are the treatment mainstay for patients with pain at the end of life.
c. Administer an opioid as an intravenous bolus dose, and begin an intravenous continuous infusion,
adjusting rates to maintain comfort; avoid using subcutaneous or enteral, unless intravenous access
is unavailable, because the onset is delayed and absorption may be unpredictable (Crit Care Med
2019;47:1619-26).
d. Bolus and titrate infusion to control labored respirations; specific dosages of medications are less
important than the goal of symptom relief. Optimal dose is determined by assessing the patient and
rapidly increasing the dose as needed until symptoms are no longer present. Dose is determined by
symptom relief and adverse effects (excessive sedation, respiratory depression).
e. Suggested goals include keeping the respiratory rate at or below 30 breaths/minute and keeping the
patient pain free. Pain assessment should include using the visual analog scale, the behavioral pain
scale, or the Critical-Care Pain Observation Tool (see the Management of Pain, Agitation, Delirium
and Neuromuscular Blockade in Adult Intensive Care Unit Patients chapter for further details on
these scales).
f. Never use neuromuscular blocking agents to treat pain.
g. Morphine is most commonly used; hydromorphone and fentanyl are alternatives.
h. In addition, opioids reduce dyspnea.
i. Tolerance may develop over time.
j. Pain can be improved with correct dosing and titration without causing respiratory depression or
hastening death (Chest 2004;126:286-93; Crit Care Med 2004;32:1141-8; JAMA 1992;267:949-53).

ACCP Updates in Therapeutics® 2022: Critical Care Pharmacy Preparatory Review and Recertification Course

70
Supportive and Preventive Medicine

2. Dyspnea
a. Common symptom in patients at the end of life
b. Individualize therapy based on underlying cause, patient’s level of consciousness, and level of
sedation.
c. Oxygen may be used for patients with hypoxia. Corticosteroids, bronchodilators and diuretics may
also be useful (Crit Care Med 2008;36:953-63).
d. Opioids are the first-line therapy. Opioids reduce oxygen consumption, ventilation, and perception
of dyspnea (J Palliat Med 2012;15:106-14).
e. No benefit with benzodiazepines unless anxiety is present (Cochrane Database Syst Rev
2010;1:CD007354)
3. Anxiety/agitation/delirium
a. Symptoms at the end of life can relate to acute or chronic anxiety, delirium, or terminal delirium.
b. Nonpharmacologic treatments for agitation and anxiety can include frequent reorientation to the
environment and reduction in noise and other bothersome or stimulating environmental factors.
c. Haloperidol is the agent of choice for delirium because of its proven efficacy, the availability of an
intravenous injection, and the agent’s less sedative effects compared with other agents (Crit Care
Med 2008;36:953-63). Intravenous haloperidol may be used without electrocardiographic (ECG)
monitoring because the benefits outweigh the risks of prolonged corrected QT interval, given the
goals of care.
d. Benzodiazepines (midazolam and lorazepam):
i. Benzodiazepines are the agents of choice for anxiety. Dose is determined by assessing the
patient and increasing the dose as needed (lower initial doses and titration with frequent
assessment).
ii. Determining what would be perceived as an acceptable level of sedation with the patient and/
or family or surrogate decision-maker is important before initiating sedatives.
iii. Tolerance may develop over time.
4. Fever
a. Acetaminophen is an effective therapy for improving comfort and decreasing the incidence of
fever. If the patient cannot swallow, this agent may be administered per rectum. If neither enteral
nor rectal access are available, acetaminophen may be administered intravenously.
b. A nonsteroidal anti-inflammatory drug may be used when acetaminophen is ineffective.
c. Dexamethasone, which is also known to have antipyretic properties, can be considered.
5. Nausea and vomiting
a. Underlying causes such as medications, uremia, ascites, gastroparesis, and intestinal or gastric
obstruction should be treated or eliminated, if possible.
b. Agents to consider include metoclopramide, haloperidol, risperidone, ondansetron, and
dexamethasone.
c. Lorazepam can be considered as an adjunct, especially with anticipatory vomiting.
d. Use of more than one agent from different classes may be necessary for symptom relief.
6. Cough
a. Excessive coughing can lead to exacerbation of dyspnea and spells of nausea and vomiting, in
addition to disturbing sleep and exacerbating pain.
b. Non-opioid antitussives such as benzonatate and dextromethorphan may be considered.
c. All opioids have intrinsic antitussive action by inhibiting the brain stem cough center; however, if
the patient is receiving an opioid for other reasons, adding another opioid has not shown additional
benefit.
d. For refractory cough, consider nebulized lidocaine.

ACCP Updates in Therapeutics® 2022: Critical Care Pharmacy Preparatory Review and Recertification Course

71
Supportive and Preventive Medicine

7. Secretions
a. Near the end of life, the ability to clear oral and tracheobronchial secretions diminishes.
b. Secretions are usually too low in the tracheobronchial tree for gentle oral suctioning to help, and
suctioning can be disturbing.
c. The treatment mainstay includes anticholinergic and antimuscarinic medications.
i. Scopolamine and atropine cross the blood-brain barrier and can be more sedating than
glycopyrrolate.
ii. Glycopyrrolate (0.1 mg intravenously every 4 hours) or atropine (1% ophthalmic solution 2
drops sublingually every 4 hours as needed) should be used for acute symptoms.
iii. The scopolamine patch is more gradual in onset (12 hours).
iv. More than 1 scopolamine patch may be used for unrelieved symptoms.

Patient Case

8. An 88-year-old woman is admitted to the ICU for decompensated heart failure, acute kidney injury, and
uncontrollable pain from the rib fractures she had 1 month ago from a fall. This is her fourth admission to
the ICU in the past 5 months. Speaking with her, you find that she wishes not to be resuscitated or intubated
but only to be comfortable. Her blood pressure is currently 119/70 mm Hg, heart rate 120 beats/ minute, and
respiratory rate 55 breaths/minute. Her pain is 9/10 using the BPS. In a meeting with the patient’s family, all
members agree that they do not want to see her suffer any longer. It is decided to initiate a morphine drip
at 2 mg/hour. Titration parameters include giving a bolus dose equivalent to the current rate and increasing
the infusion by 25% to maintain a score of 3 (no pain) using the BPS. Her laboratory values are all within
normal limits, including BUN 10 mg/dL and SCr 0.6 mg/dL. The nurse taking care of the patient believes
that the titration parameters are too aggressive. Which would be the most appropriate change in titration
parameters?
A. Change the parameters to increase the morphine drip when the patient has signs of discomfort, such as
an increase in blood pressure or heart rate.
B. Discontinue titration parameters, keeping the morphine infusion at the current rate.
C. Discontinue titration parameters, keeping the morphine infusion at the current rate and adding a mid-
azolam infusion at 2 mg/hour.
D. Do not change the titration parameters at this time; however, assess the patient’s response after the first
dose increase.

ACCP Updates in Therapeutics® 2022: Critical Care Pharmacy Preparatory Review and Recertification Course

72
Supportive and Preventive Medicine

REFERENCES

Key Aspects in the General Care of All Critically Ill prophylaxis in critically ill patients: a meta-analy-
Patients sis. Am J Gastroenterol 2012;107:507-20.
1. Haynes AB, Weiser TG, Berry WR, et al. A sur- 6. Bateman BT, Bykov K, Choudhry NK, et al. Type
gical safety checklist to reduce morbidity and of stress ulcer prophylaxis and risk of nosocomial
mortality in a global population. N Engl J Med pneumonia in cardiac surgical patients: cohort
2009;360:491-9. study. BMJ 2013;347:f5416.
2. Mabasa VH, Malyuk DL, Weatherby EM, et al. 7. Buendgens L, Bruensing J, Matthes M, et al.
A standardized, structured approach to identify- Administration of proton pump inhibitors in
ing drug-related problems in the intensive care critically ill medical patients is associated with
unit: FASTHUG-MAIDENS. Can J Hosp Pharm increased risk of developing Clostridium difficile-
2011;64:366-9. associated diarrhea. J Crit Care 2014;29:696.
3. Pronovost P, Needgam D, Berenholtz S, et al. An 8. Cook D, Guyatt G, Marshall J, et al. A comparison
intervention to decrease catheter-related blood- of sucralfate and ranitidine for the prevention of
stream infections in the ICU. N Engl J Med upper gastrointestinal bleeding in patients requir-
2006;355:2725-32. ing mechanical ventilation. Canadian Critical Care
4. Rhodes A, Evans LE, Alhazzani W, et al. Trials Group. N Engl J Med 1998;338:791-7.
Surviving Sepsis Campaign: international guide- 9. Cook D, Heyland D, Griffith L, et al. Risk fac-
lines for management of sepsis and septic shock: tors for clinically important upper gastrointestinal
2016. Crit Care Med 2017;44:486-552. bleeding in patients requiring mechanical venti-
5. Vincent JL. Give your patient a fast hug (at least) lation. Canadian Critical Care Trials Group. Crit
once a day. Crit Care Med 2005;33:1225-9. Care Med 1999;27:2812-7.
6. Vincent WR III, Hatton KW. Critically ill patients 10. Cook DJ, Fuller HD, Guyatt GH, et al. Risk fac-
need “FAST HUGS BID” (an updated mnemonic). tors for gastrointestinal bleeding in critically ill
Crit Care Med 2009;37:2326-7; author reply 2327. patients. Canadian Critical Care Trials Group. N
Engl J Med 1994;330:377-81.
Stress Ulcer Prophylaxis 11. El-Kersh K, Jalil B, McClave SA, et al. Enteral
1. Alhazzani W, Alenezi F, Jaeschke RZ, et al. Proton nutrition as stress ulcer prophylaxis in critically
pump inhibitors versus histamine 2 receptor antag- ill patients: a randomized controlled exploratory
onists for stress ulcer prophylaxis in critically ill study. J Crit Care 2018;43:108-13.
patients: a systematic review and meta-analysis. 12. Guillamondegui OD, Gunter OL, Bonadies JA,
Crit Care Med 2013;41:693-705. et al. Practice Management Guidelines for Stress
2. Alhazzani W, Alshamsi F, Belley-Cote E, et al. Ulcer Prophylaxis 2008. Available at www.east.
Efficacy and safety of stress ulcer prophylaxis org. Accessed June 6, 2016.
in critically ill patients: a network meta-anal- 13. Huang HB, Jiang W, Wang CY, et al. Stress ulcer
ysis of randomized trials. Intensive Care Med prophylaxis in intensive care unit patients receiv-
2018;44:1-11. ing enteral nutrition: a systematic review and
3. ASHP Therapeutic Guidelines on Stress meta-analysis. Crit Care 2018;22:20.
Ulcer Prophylaxis. Am J Health Syst Pharm 14. Krag M, Marker S, Perner A. Pantoprazole in
1999;56:347-79. patients at risk for gastrointestinal bleeding in the
4. Barkun AN, Adam V, Martel M, et al. Cost- ICU. N Engl J Med 2018;379:2199-208.
effectiveness analysis: stress ulcer bleeding 15. Krag M, Perner A, Wetterslev J, et al. Prevalence
prophylaxis with proton pump inhibitors, H2 and outcome of gastrointestinal bleeding and use
receptor antagonists. Value Health 2013;16:14-22. of acid suppressants in acutely ill adult intensive
5. Barkun AN, Bardou M, Pham CQ, et al. Proton care patients. Intensive Care Med 2015;41:833-45.
pump inhibitors vs. histamine 2 receptor antag- 16. Lin P, Chang C, Hsu P, et al. The efficacy and safety
onists for stress-related mucosal bleeding of proton pump inhibitors vs histamine-2 receptor
antagonists for stress ulcer bleeding prophylaxis

ACCP Updates in Therapeutics® 2022: Critical Care Pharmacy Preparatory Review and Recertification Course

73
Supportive and Preventive Medicine

among critical care patients: a meta-analysis. Crit for venous thromboprophylaxis. N Engl J Med
Care Med 2010;38:1197-205. 2019;380:1305-15.
17. MacLaren R, Campbell J. Cost-effectiveness of 2. Attia J, Ray JG, Cook DJ, et al. Deep vein throm-
histamine receptor-2 antagonist versus proton bosis and its prevention in critically ill adults. Arch
pump inhibitor for stress ulcer prophylaxis in criti- Intern Med 2001;161:1268-79.
cally ill patients. Crit Care Med 2014;42:809-15. 3. Beiderlinden M, Treschan TA, Görlinger K, et
18. MacLaren R, Reynolds PM, Allen RR. Histamine-2 al. Argatroban anticoagulation in critically ill
receptor antagonists vs proton pump inhibitors on patients. Ann Pharmacother 2007;41:749-54.
gastrointestinal tract hemorrhage and infectious 4. Cade JF. High risk of the critically ill for venous
complications in the intensive care unit. JAMA thromboembolism. Crit Care Med 1982;10:448-50.
Intern Med 2014;174:564-74. 5. Chi G, Gibson CM, Kalayci A, et al. Extended-
19. Madsen KR, Lorentzen K, Clausen N, et al. duration betrixaban versus shorter-duration
Guideline for stress ulcer prophylaxis in the inten- enoxaparin for venous thromboembolism prophy-
sive care unit. Dan Med J 2014;61:C4811. laxis in critically ill medical patients: an APEX
20. Marik PE, Vasu T, Hirani A, et al. Stress ulcer trial substudy. Intensive Care Med 2019;45:477-87.
prophylaxis in the new millennium: a system- 6. Cohen AT, Spiro TE, Büller HR, et al. Rivaroxaban
atic review and meta-analysis. Crit Care Med for thromboprophylaxis in acutely ill medical
2010;38:2222-8. patients. N Engl J Med 2013;368:513-23.
21. McRorie JW, Kirby JA, Miner PB. Histamine2- 7. Cook D, Crowther M, Meade M, et al. Deep
receptor antagonists: rapid development of venous thrombosis in medical-surgical critically
tachyphylaxis with repeat dosing. World J ill patients: prevalence, incidence, and risk factors.
Gastrointest Pharmacol Ther 2014;5:57-62. Crit Care Med 2005;33:1565-71.
22. PEPTIC Investigators for the Australian and New 8. Cook D, Meade M, Guyatt G, et al. Dalteparin ver-
Zealand Intensive Care Society Clinical Trials sus unfractionated heparin in critically ill patients.
Group, Alberta Health Services Critical Care N Engl J Med 2011;364:1305-14.
Strategic Clinical Network, and the Irish Critical 9. Cuker A, Arepally GM, Chong BH. American
Care Trials Group; Young PJ, Bagshaw SM, Forbes Society of Hematology 2018 guidelines for
AB, et al. Effect of stress ulcer prophylaxis with management of venous thromboembolism: hep-
proton pump inhibitors vs histamine-2 recep- arin-induced thrombocytopenia. Blood Adv
tor blockers on in-hospital mortality among ICU 2018;2:3360-92.
patients receiving invasive mechanical ventilation: 10. Cuker A, Gimotty PA, Crowther MA, et al.
the PEPTIC Randomized Clinical Trial. JAMA Predictive value of the 4Ts scoring system for
2020;323:616-26. diagnosis and management of heparin-induced
23. Rhodes A, Evans LE, Alhazzani W, et al. thrombocytopenia: a systemic review and meta-
Surviving Sepsis Campaign: international guide- analysis. Blood 2012;120:4160-7.
lines for management of sepsis and septic shock: 11. De A, Roy P, Garg VK, et al. Low-molecular-
2016. Crit Care Med 2017;44:486-552. weight heparin and unfractionated heparin in
24. Simons RK, Hoyt DB, Winchell RJ, et al. A risk prophylaxis against deep vein thrombosis in criti-
analysis of stress ulceration after trauma. J Trauma cally ill patients undergoing major surgery. Blood
1995;39:289-93. Coagul Fibrinolysis 2010;21:57-61.
25. Ye Z, Reintam Blaser A, Lytvyn L, et al. 12. Douketis J, Cook D, Meade M, et al. Prophylaxis
Gastrointestinal bleeding prophylaxis for criti- against deep vein thrombosis in critically ill
cally ill patients: a clinical practice guideline. BMJ patients with severe renal insufficiency with the
2020;368:l6722. low-molecular-weight heparin dalteparin: an
assessment of safety and pharmacodynamics: the
Prophylaxis Against Deep Venous Thrombosis or DIRECT study. Arch Intern Med 2008;168:1805-12.
Pulmonary Embolism 13. Fraisse F, Holzapfel L, Couland JM, et al.
1. Arabi YM, Al-Hameed F, Burns KEA, et al. Nadroparin in the prevention of deep vein
Adjunctive intermittent pneumatic compression thrombosis in acute decompensated COPD. The

ACCP Updates in Therapeutics® 2022: Critical Care Pharmacy Preparatory Review and Recertification Course

74
Supportive and Preventive Medicine

Association of Non-University Affiliated Intensive 24. Wahby KA, Riley LK, Tennenberg SD. Assessment
Care Specialist Physicians of France. Am J Respir of an extended interval fondaparinux dosing regi-
Crit Care Med 2000;161:1109-14. men for venous thromboembolism prophylaxis in
14. Geerts W, Cook D, Selby R, et al. Venous throm- critically ill patients with severe renal dysfunc-
boembolism and its prevention in critical care. J tion using antifactor Xa levels. Pharmacotherapy
Crit Care 2002;17:95-104. 2017;37:1241-48.
15. Gould MK, Garcia DA, Wren SM, et al. Prevention
of VTE in nonorthopedic surgical patients: End-of-Life Care
Antithrombotic Therapy and Prevention of 1. Brody H, Campbell ML, Faber-Langendoen K, et
Thrombosis, 9th ed: American College of Chest al. Withdrawing intensive life-sustaining treatment
Physicians Evidence-Based Clinical Practice – recommendations for compassionate clinical
Guidelines. Chest 2012;141(2 suppl):227S-77S. management. N Engl J Med 1997;336:652-7.
16. Kahn SR, Lim W, Dunn AS, et al. Prevention 2. Chan JD, Treece PD, Engelberg RA, et al. Narcotic
of VTE in nonsurgical patients: Antithrombotic and benzodiazepine use after withdrawal of life
Therapy and Prevention of Thrombosis, 9th ed: support: association with time to death? Chest
American College of Chest Physicians Evidence- 2004;126:286-93.
Based Clinical Practice Guidelines. Chest 3. Global Atlas of Palliative Care at the End of
2012;141(2 suppl):195S-226S. Life. Available at https://www.who.int/nmh/
17. Kiser TH, Fish DN. Evaluation of bivalirudin Global_Atlas_of_Palliative_Care.pdf. Accessed
treatment for heparin-induced thrombocytopenia September 1, 2021.
in critically ill patients with hepatic and/or renal 4. Kamal AJ, Maguire JM, Wheeler JL, et al.
dysfunction. Pharmacotherapy 2006;26:452-60. Dyspnea review for the palliative care profes-
18. Linkins LA, Dans AL, Moores LK, et al. Treatment sional: treatment goals and therapeutic options. J
and prevention of heparin-induced thrombocyto- Palliat Med 2012;15:106-14.
penia: Antithrombotic Therapy and Prevention of 5. Laserna A, Durán-Crane A, López-Olivo MA, et
Thrombosis, 9th ed: American College of Chest al. Pain management during the withholding and
Physicians Evidence-Based Clinical Practice withdrawal of life support in critically ill patients
Guidelines. Chest 2012;141(2 suppl):495S-530S. at the end-of-life: a systematic review and meta-
19. Phung OJ, Kahn SR, Cook DJ, et al. Dosing analysis. Intensive Care Med. 2020;46:1671-82.
frequency of unfractionated heparin thrombopro- 6. Simon ST, Higginson IJ, Booth S, et al.
phylaxis: a meta-analysis. Chest 2011;140:374-81. Benzodiazepines for the relief of breathless-
20. Reynolds PM, Van Matre ET, Wright GC, et al. ness in advanced malignant and non-malignant
Evaluation of prophylactic heparin dosage strate- diseases in adults. Cochrane Database Syst Rev
gies and risk factors for venous thromboembolism 2010;1:CD007354.
in the critically ill patient. Pharmacotherapy 7. Treece PD, Engelberg RA, Crowley L, et al.
2019;39:232-41. Evaluation of a standardized order form for the
21. Sachdeva A, Dalton M, Amaragiri SV, et al. withdrawal of life support in the intensive care
Graduated compression stockings for prevention unit. Crit Care Med 2004;32:1141-8.
of deep vein thrombosis. Cochrane Database Syst 8. Truog RD, Campbell ML, Curtis JR, et al.
Rev 2014;12:1-72. Recommendations for end-of-life care in the
22. Saugel B, Phillip V, Moessmer G, et al. Argatroban intensive care unit: a consensus statement by the
therapy for heparin-induced thrombocytopenia American College of Critical Care Medicine. Crit
in ICU patients with multiple organ dysfunc- Care Med 2008;36:953-63.
tion syndrome: a retrospective study. Crit Care 9. Wilson WC, Smedira NG, Fink C, et al. Ordering
2010;14:R90. and administration of sedatives and analgesics dur-
23. Shorr AF, Williams MD. Venous thromboembo- ing the withholding and withdrawal of life support
lism in critically ill patients: observations from from critically ill patients. JAMA 1992;267:949-53.
a randomized trial in sepsis. Thromb Haemost
2009;101:139-44.

ACCP Updates in Therapeutics® 2022: Critical Care Pharmacy Preparatory Review and Recertification Course

75
Supportive and Preventive Medicine

ANSWERS AND EXPLANATIONS TO PATIENT CASES

1. Answer: A 4. Answer: C
The mnemonic FAST-HUG stands for Feeding, Once the risk factors are no longer present, SUP should
Analgesia, Sedation, Thromboembolic prophylaxis, promptly be discontinued (Answer C is correct). This
Head of bed elevation, stress Ulcer prophylaxis, and patient no longer has risk factors (mechanical ventila-
Glycemic control. Using this mnemonic as a checklist tion, coagulopathy acute kidney failure, and severe
every day for each critically ill patient will help maxi- sepsis). In addition, there is no evidence that SUP
mize therapeutic interventions and promote patient should be continued until hospital or ICU discharge or
safety. This patient would benefit from receiving enteral when antimicrobial therapy is complete (Answers A, B,
nutrition (an NGT is already placed and he has a work- and D are incorrect).
ing GI tract), interrupting the sedative (current RASS
score is above the designated goal), and adding SUP 5. Answer: C
(risk factors include mechanical ventilation) (Answer This patient has several risk factors for VTE, includ-
A is correct). Critically ill patients with risk factors for ing immobility and respiratory failure, making heparin
VTE should remain on VTE prophylaxis (Answers B 5000 units subcutaneously twice daily appropriate for
and D are incorrect); moreover, sliding-scale insulin VTE prophylaxis (Answer C is correct). Neither enoxa-
should be initiated when the patient is not critically ill, parin nor fondaparinux is appropriate for this patient,
adding another reason why Answer B is incorrect and who has acute kidney injury with an estimated CrCl of
making Answer C incorrect. less than 20 mL/minute/1.73 m2 (Answers B and D are
incorrect). Intermittent pneumatic compression would
2. Answer: C be insufficient in a patient with no contraindication to
Two independent risk factors for SRMD are respiratory pharmacologic prophylaxis (Answer A is incorrect).
failure requiring mechanical ventilation for 48 hours
or longer and coagulopathy (Plt less than 50,000/mm3, 6. Answer: A
INR greater than 1.5, or aPTT greater than 2 times the Diagnosing HIT is difficult in a critically ill patient
control). This patient has both of these risk factors. In because there are many alternative causes of thrombo-
addition, she has septic shock, as evidenced by end- cytopenia. Clinical assessment is essential in diagnosing
organ dysfunction and acute kidney injury (Answer C is HIT because of the immediate need for treatment and
correct). Answers A, B, and D are incorrect because this the delay in laboratory testing (Answers B and D are
patient has four risk factors for developing stress-related incorrect). Clinically, this patient has had a greater than
mucosal damage. 50% decrease in Plt within 5 days of receiving heparin.
The calculated 4Ts score is 5 (2 points for Plt decrease
3. Answer: D by greater than 50%, 2 points for a clear onset at days
Antacids are not recommended for routine use because 5–10, and 1 point for other possible causes of throm-
of their frequency of administration, adverse effects, bocytopenia. This score is an intermediate probability
and interactions (Answer B is incorrect). In a large for HIT. In managing suspected HIT, first ensure that
randomized controlled trial, sucralfate was inferior all forms of heparin are discontinued, including flushes
to H2RAs in preventing clinically significant bleed- and heparin-coated catheters. Next, initiate an alterna-
ing from SRMD and is generally not recommended tive form of anticoagulation. Direct thrombin inhibitors
because of its adverse effect profile (Answer A is incor- are the agents of choice for anticoagulation in acute
rect). Proton pump inhibitors are no better than H2RAs HIT because they have no cross-reactivity with heparin
in preventing SRMD and are associated with increased (Answer A is correct). Factor Xa inhibitors have been
infectious complications, including pneumonia and CDI used to manage HIT; however, they would not be best
(Answer D is correct). Meta-analyses have favored PPIs in this patient, who has acute kidney injury (Answer C
to H2RAs for GI bleeding; however, the individual tri- is incorrect).
als included lacked methodological quality (Answer C
is incorrect).

ACCP Updates in Therapeutics® 2022: Critical Care Pharmacy Preparatory Review and Recertification Course

76
Supportive and Preventive Medicine

7. Answer: D
Warfarin can be initiated (Answer C is incorrect) once
the Plt has recovered to at least 150,000/mm3 and after at
least 5 days of therapy with an alternative anticoagulant
(Answer D is correct). Because this patient’s Plts have
not reached 150,000/mm3 and only 3 days of argatroban
have been completed, warfarin therapy should not be
initiated at this time (Answers A and B are incorrect).
Argatroban should be continued, and warfarin may be
considered at low doses (maximum 5 mg) as the Plt con-
tinues to recover (Answer B is incorrect).

8. Answer: D
Up to 50% of seriously ill hospitalized patients have
moderate or severe pain. Opioids are the treatment main-
stay for patients with pain and dyspnea at the end of life.
Assessing pain in the ICU can be particularly challeng-
ing because many patients have impaired cognition and
communication. Vital signs alone should not be used for
pain assessment (Answer A is incorrect). Evidence sug-
gests that pain can be improved with correct dosing and
titration (Answers B and C are incorrect) without caus-
ing respiratory depression or hastening death (Answer
D is correct).

ACCP Updates in Therapeutics® 2022: Critical Care Pharmacy Preparatory Review and Recertification Course

77
Supportive and Preventive Medicine

ANSWERS AND EXPLANATIONS TO SELF-ASSESSMENT QUESTIONS

1. Answer: C 4. Answer: A
The FAST-HUG mnemonic can serve as a checklist Proton pump inhibitors are potent inhibitors of gastric
for every patient admitted to the ICU. Every patient acid production and are the drug of choice for gastro-
should be assessed for a sedation interruption to min- esophageal reflux disease. To date, the only prospective
imize sedative exposure and maintain a light level of randomized controlled trial to evaluate CDI risk with PPI
sedation (Answer C is correct). To decrease the risk of use found no difference between PPI and placebo; how-
nosocomial pneumonia, each patient should have his or ever, this was an underpowered secondary end point,
her head elevated 30–45 degrees above the head of the and several cohort studies have found an association.
bed (Answer C is correct). Enteral nutrition should be (Answer C is incorrect). All published trials assessing
initiated as soon as possible – typically, once the patient the risk of CDI with PPI use have been limited by the
is stabilized; however, thromboprophylaxis should be inconsistent definitions of CDI and the variable infection
initiated in every patient, using pharmacologic agents control practices (Answer D is incorrect). Gastric juice is
preferentially to mechanical prophylaxis (Answer B strongly bactericidal for microorganisms. Proton pump
is incorrect). Stress ulcer prophylaxis should only be inhibitors are commonly used to increase the gastric
initiated in patients who have risk factors and should pH; therefore, they act as a potential risk factor for CDI
be discontinued once the risk factors no longer exist (Answer A is correct).
(Answer A is incorrect). Insulin infusions should be ini-
tiated only if blood glucose readings are not 140–180 5. Answer: B
mg/dL (Answer D is incorrect). Low-dose unfractionated heparin or low-molecular-
weight heparin should be initiated for VTE prophylaxis
2. Answer: C in a critically ill patient over no prophylaxis (Answer
Sucralfate forms a protective barrier over the surface D is incorrect). Intermittent pneumatic compression
of the stomach, reducing exposure to acidic gastric devices would be insufficient prophylaxis in a patient
contents; therefore, sucralfate does not affect gastric with several risk factors for VTE (Answer A is incor-
pH (Answer A is incorrect). Compared with H2RAs, rect). A continuous infusion of heparin is inappropriate
PPIs appear more effective at reducing gastric acidity, for preventing VTE (Answer C is incorrect). Enoxaparin
but no well-conducted trial has shown PPIs superior in may be used for VTE prophylaxis in a critically ill
preventing clinically significant bleeding (Answer B is patient with stable renal function (Answer B is correct).
incorrect). Tolerance to any H2RA may occur, but not to
PPIs (Answer C is correct). Antacids have some effect 6. Answer: A
on reducing stress ulceration, provided the gastric pH This patient had a closed-head injury, placing her at
is kept around 3.5, but frequent dosing (up to every 2 high risk of VTE (Answer D is incorrect). She is at high
hours) is required to achieve this goal, making their use risk of major bleeding and acute kidney injury; there-
impractical (Answer D is incorrect). fore, a low-molecular-weight heparin or a factor Xa
inhibitor would not be best for her (Answers B and C
3. Answer: D are incorrect). Mechanical prophylaxis with intermit-
The patient has an indication for SUP (mechanical ven- tent pneumatic compression devices is preferred to no
tilation). He has an NGT in place and is tolerating tube prophylaxis in the absence of lower-extremity injury
feedings, indicating a functioning gut; therefore, intra- until the bleeding risk is no longer present (Answer A
venous therapy is not required (Answers A and B are is correct).
incorrect). The patient has erosive esophagitis, for which
a PPI will be more effective than an H2RA (Answer C 7. Answer: D
is incorrect). Omeprazole suspension is effective in pre- Clinical assessment is essential in diagnosing HIT
venting SRMD; therefore, an omeprazole suspension because of the immediate need for treatment and the
would be most appropriate for this patient (Answer D delay in laboratory testing. Although this patient’s Plt
is correct). did decrease by 50%, the characteristic onset of the Plt

ACCP Updates in Therapeutics® 2022: Critical Care Pharmacy Preparatory Review and Recertification Course

78
Supportive and Preventive Medicine

decrease in HIT is 5–10 days after heparin initiation.


Clinical prediction rules to help determine the probabil-
ity of HIT (e.g., 4Ts score) have been developed. Patients
with a low 4Ts score (0–3) have a very low probability
of HIT (Answer D is correct). Direct thrombin inhibi-
tors are the agents of choice for anticoagulation in acute
HIT because they have no cross-reactivity with heparin.
Initiating these agents in those with a low probability of
HIT could lead to an unnecessary increase in bleeding
risk (Answer B is incorrect). If HIT were highly sus-
pected in this patient, the first step would be to ensure
that all forms of heparin are discontinued, includ-
ing flushes and heparin-coated catheters (Answer C is
incorrect). The next step would be to initiate an alterna-
tive form of anticoagulation (Answer A is incorrect).

8. Answer: C
General considerations in the critically ill patient at
the end of life include minimizing uncomfortable or
unnecessary procedures, tests, and treatments, includ-
ing fingersticks, Foley catheters, and routine vital signs
(Answers A, B, and D are incorrect). Symptom man-
agement of pain and anxiety, fever, cough, secretions,
nausea and vomiting, and delirium should be considered
in the dying patient (Answer C is correct).

ACCP Updates in Therapeutics® 2022: Critical Care Pharmacy Preparatory Review and Recertification Course

79
Neurocritical Care
Gretchen M. Brophy, Pharm.D., FCCP, FCCM, FNCS,
MCCM, BCPS
Virginia Commonwealth University
Richmond, Virginia

Eljim P. Tesoro, Pharm.D., FNCS, FCCM, BCCCP


University of Illinois at Chicago
Chicago, Illinois
Neurocritical Care

Neurocritical Care
Gretchen M. Brophy, Pharm.D., FCCP, FCCM, FNCS,
MCCM, BCPS
Virginia Commonwealth University
Richmond, Virginia

Eljim P. Tesoro, Pharm.D., FNCS, FCCM, BCCCP


University of Illinois at Chicago
Chicago, Illinois

ACCP Updates in Therapeutics® 2022: Critical Care Pharmacy Preparatory Review and Recertification Course

83
Neurocritical Care

Learning Objectives Self-Assessment Questions


Answers and explanations to these questions may be
1. Identify pertinent pathophysiologic and laboratory found at the end of this chapter.
changes that acutely occur after neurologic injuries
and require therapeutic intervention. 1. A 56-year-old woman is hospital day 4 after
2. Describe monitoring devices commonly used in her acute aneurysmal subarachnoid hemorrhage
neurocritical care patients that help develop and (SAH). She is oriented and following commands.
optimize treatment strategies. Laboratory values reveal a serum sodium of 128
3. Develop an evidence-based treatment strategy for mmol/L. Other serum chemistry values include
neurocritical care patients that optimizes patient out- potassium (K) 3.9 mEq/L, chloride 103 mEq/L,
comes and reduces the risk of adverse drug effects bicarbonate 27 mEq/L, blood urea nitrogen (BUN)
and drug interactions. 10 mg/dL, and serum creatinine (SCr) 1.0 mg/dL.
4. Recommend a monitoring plan to assess response to Her urinary output is 1–2 mL/kg/hour, and her fluid
therapeutic regimens and specific therapeutic goals balance has been +435 mL during the past 24 hours
for neurocritical care patients. (currently receiving 0.9% sodium chloride at 125
5. Reassess and develop new plans of care for neu- mL/hour). Which is the best initial therapy for this
rocritical care patients according to therapeutic and patient’s hyponatremia?
adverse outcomes and progress toward therapeutic A. Tolvaptan 20 mg orally daily.
goals. B. 1.5% sodium chloride infusion at 125 mL/hour.
C. Water restriction to less than 1.5 L/day.
D. No treatment indicated right now.
Abbreviations in This Chapter
2. A 27-year-old woman is admitted with an acute
ADH Antidiuretic hormone ventriculoperitoneal shunt failure and associated
CNS Central nervous system infection. She has no significant medical history
CPP Cerebral perfusion pressure and no allergies to medications. A lumbar punc-
CSF Cerebrospinal fluid ture reveals cerebrospinal fluid (CSF) white blood
CSWS Cerebral salt-wasting syndrome cell count (WBC) 34 × 103 cells/mm3, red blood
EEG Electroencephalogram cell count (RBC) 1 × 103 cells/mm3, protein 78
GCS Glasgow Coma Scale mg/dL, and glucose 21 mg/dL. The cultures grow
ICH Intracerebral hemorrhage methicillin-resistant Staphylococcus epidermidis
ICP Intracranial pressure (MRSE). Her shunt is externalized. Despite 4 days
ICU Intensive care unit of intravenous vancomycin (most recent vancomy-
MAP Mean arterial pressure cin trough was 17.7 mcg/mL), the CSF continues to
MRSE Methicillin-resistant Staphylococcus grow MRSE. Which is the most appropriate intra-
epidermidis ventricular antimicrobial regimen to initiate for this
PSH Paroxysmal sympathetic hyperactivity patient’s refractory ventriculitis?
RSE Refractory status epilepticus
A. Give vancomycin 10 mg intraventricularly
SAH Subarachnoid hemorrhage
daily.
SCI Spinal cord injury
B. Give gentamicin 5 mg intraventricularly daily.
SIADH Syndrome of inappropriate antidiuretic
C. Give ampicillin 50 mg intraventricularly daily.
hormone
D. No antimicrobials should be given intraventric-
TBI Traumatic brain injury
ularly for this patient.
VTE Venous thromboembolism
3. A 25-year-old man is admitted after a two-story fall
from a ladder. The initial computed tomography
(CT) scan of his brain reveals a large right tempo-
ral subdural hematoma, an overlying skull fracture,

ACCP Updates in Therapeutics® 2022: Critical Care Pharmacy Preparatory Review and Recertification Course

84
Neurocritical Care

and a left temporal contusion. His post-resuscitation C. Initiate nicardipine to reduce blood pressure to
Glasgow Coma Scale (GCS) score is E1-M4-V1T. systolic blood pressure (SBP) less than 140 mm
An intracranial pressure (ICP) monitor is placed Hg, followed by alteplase 0.9 mg/kg intrave-
with an opening pressure of 32 mm Hg and a cere- nously (10% bolus dose, 90% infusion up to 90
bral perfusion pressure (CPP) of 53 mm Hg. Serum mg maximum).
laboratory values include sodium (Na) 141 mEq/L, D. Initiate vitamin K 10 mg intravenously × 1.
K 3.6 mEq/L, BUN 8 mg/dL, SCr 1.1 mg/dL, glu-
cose 178 mg/dL, WBC 14.8 × 103 cells/mm3, pH 5. An 18-year-old man is admitted to the intensive care
7.46, and partial pressure of carbon dioxide (Pco2) unit (ICU) after falling from a tree. Initial trauma
34 mm Hg. Which supportive care issue is most rel- screening reveals a C3–C4 fracture and dislocation
evant to the appropriate treatment of a patient with a with an incomplete spinal cord injury (SCI) at the
severe traumatic brain injury (TBI)? corresponding levels (he has some sensory func-
A. Avoid enteral nutrition for the first 5–7 days tion bilaterally). The fracture has been reduced, and
because of the lack of gastrointestinal (GI) tol- he arrives in the ICU 6 hours after injury. Which
erance in severe TBI. is the most appropriate statement related to initiat-
B. Maintain CPP at 60–70 mm Hg to optimize per- ing high-dose methylprednisolone therapy for this
fusion and reduce complications. patient’s SCI?
C. Provide dextrose 5% or other dextrose-contain- A. May be used because he has an incomplete
ing intravenous fluids to compensate for the injury with some sensory function.
patient’s increased metabolic needs. B. Should be used because it will augment spinal
D. Initiate high-dose methylprednisolone therapy perfusion.
within 8 hours of injury to reduce cerebral C. Should not be used because of the potential for
edema. adverse effects and questionable benefit.
D. Should not be used because the patient is out-
4. A 69-year-old woman presents to the emergency side the treatment window.
department with a 30-minute history of difficulty
with word finding and left upper-extremity weak- 6. A 27-year-old woman presents with fever, agitation,
ness. Her NIH Stroke Scale score is 13. A head CT hypertension, and muscle rigidity. Her drugs-of-
scan reveals no acute abnormalities. The patient’s abuse screen is negative, and serotonin syndrome is a
home medications include lisinopril, carvedilol, possible diagnosis. Which home medication is most
warfarin, and atorvastatin. Her medical history likely a causative agent for serotonin syndrome?
includes hypertension, atrial fibrillation, and tran- A. Buspirone.
sient ischemic attacks (diagnosed 6 months ago). B. Levetiracetam.
Serum laboratory values include Na 145 mEq/L, K C. Cyproheptadine.
4.0 mEq/L, BUN 18 mg/dL, SCr 1.2 mg/dL, glucose D. Bupropion.
132 mg/dL, WBC 8.7 × 103 cells/mm3, hematocrit
(Hct) 38.9%, platelet count (Plt) 355,000/mm3, and
international normalized ratio (INR) 1.5. Her vital
signs include blood pressure 167/98 mm Hg, heart
rate 132 beats/minute, oxygen saturation (Sao2) 98%,
and respiratory rate 14 breaths/minute. Which is the
most appropriate next step in this patient’s care?
A. Initiate aspirin 324 mg orally × 1.
B. Initiate alteplase 0.9 mg/kg intravenously
(10% bolus dose, 90% infusion up to 90 mg
maximum).

ACCP Updates in Therapeutics® 2022: Critical Care Pharmacy Preparatory Review and Recertification Course

85
Neurocritical Care

7. A 58-year-old woman with a Hunt and Hess grade


4 SAH resides in your ICU. She is day 6 after her
SAH. Her current medications include 0.9% nor-
mal saline at 100 mL/hour, nimodipine 60 mg by
feeding tube every 4 hours, norepinephrine 0.05
mcg/kg/minute (5 mcg/minute), famotidine 20 mg
intravenously every 12 hours, docusate 250 mg by
tube every 12 hours, and morphine as needed for
headache. Current laboratory values include Na
144 mEq/L, K 4.1 mEq/L, SCr 0.6 mg/dL, serum
osmolality 322 mOsm/L, and Hct 32.3%. Her blood
pressure is 167/99 mm Hg, heart rate 133 beats/min-
ute, respiratory rate 18 breaths/minute, Sao2 99%,
and central venous pressure 5 mm Hg. Her most
recent transcranial Doppler velocities are mean
middle cerebral artery 125/135 (right/left), with a
corresponding Lindegaard ratio of 3.5/3.7 (right/
left). Her ICP is currently 24 mm Hg. She has a
Licox monitor placed in the hemisphere ipsilateral
to her aneurysm, which currently reveals a partial
pressure of brain tissue oxygen of 14%. She is
intubated and on the ventilator. Her GCS score has
decreased from 10 to 8 during the past hour. Which
bedside treatment would be most appropriate for
this patient?
A. Verapamil 2.5 mg intra-arterially × 1.
B. 3% sodium chloride 2.5 mL/kg intravenously
× 1.
C. 20% mannitol 0.25 g/kg intravenously × 1.
D. 1 unit of packed RBCs.

8. Which sedative is most desirable for a patient with a


diagnosis of SAH currently having vasospasm?
A. Lorazepam 1 mg/hour.
B. Midazolam 4 mg/hour.
C. Morphine 2 mg/hour.
D. Propofol 25 mcg/kg/minute.

ACCP Updates in Therapeutics® 2022: Critical Care Pharmacy Preparatory Review and Recertification Course

86
Neurocritical Care

BPS Critical Care Pharmacy Specialist Examination Content Outline

This chapter covers the following sections of the Critical Care Pharmacy Specialist Examination Content Outline:
1. Domain 1: Clinical Knowledge and Application
a. Task 1: 1, 3
b. Task 2: 2–4, 7
c. Task 3: 1–7
d. Task 4: 1–5, 8
e. Task 5: 5–7
f. Task 6: 1–6
g. Task 7: 1–7

Systems and patient care problems:


– Hyponatremia/hypernatremia
– Status epilepticus
– CNS infections/intraventricular antibiotic administration
– Intracranial pressure treatment
– Paroxysmal sympathetic hyperactivity
– Acute ischemic stroke
– Intracerebral hemorrhage
– Subarachnoid hemorrhage
– Interventional endovascular management
– Acute spinal cord injury
– Brain tumors
– Critical illness polyneuropathy
– Guillain-Barre syndrome
– Myasthenia crisis
– Serotonin syndrome
– Neurologic monitoring devices

2. Domain 2: Practice Management, Policy, and Quality Improvement


a. Task 5: 1
b. Task 6: 3
3. Domain 3: Evidence-Based Medicine, Scholarship, Education, and Professional Development
a. Task 1: 3

ACCP Updates in Therapeutics® 2022: Critical Care Pharmacy Preparatory Review and Recertification Course

87
Neurocritical Care

I.  HYPONATREMIA

A. Epidemiology: Hyponatremia (Na less than 135 mEq/L) is common in patients with a neurologic injury
(12%–43%).

B. Diagnosis/Pathophysiology
1. Laboratory tests (serum sodium) are needed to diagnose hyponatremia.
2. Urine sodium, urine osmolality, serum osmolality, and measurement of intravascular volume may also
help determine the specific pathogenesis for hyponatremia.

C. Causes
1. Consideration of iatrogenic hyponatremia
2. Typically caused by an increase in salt-free water or loss of serum sodium

D. Differentiating Between Syndrome of Inappropriate Antidiuretic Hormone (SIADH) and Cerebral Salt-
Wasting Syndrome (CSWS)
1. Typically made by assessing intravascular volume. Patients with SIADH tend to be euvolemic or
hypervolemic with hyponatremia because of excessive antidiuretic hormone (ADH) release, whereas
patients with CSWS tend to be hypovolemic with hyponatremia because of inappropriate urinary
excretion of sodium and extracellular fluid.
2. Measure intravascular volume using a central venous pressure catheter or similar invasive monitoring.
3. Noninvasive hemodynamic monitoring devices
4. Monitor fluid balance, weights, skin turgor
5. Echocardiogram to estimate ventricular filling pressures

Table 1. Differential Diagnosis for SIADH and CSWSa


Serum
Serum Sodium Urine Sodium Urine Osmolality Intravascular
Osmolality
(mEq/L) (mEq/L) (mOsm/L) Volume Status
(mOsm/L)
SIADH < 135 < 285 > 25 > 200 Euvolemia
CSWS < 135 < 285 > 25 > 200 Hypovolemia
a
Note: Medications, particularly diuretics, may alter serum or urine measurements of osmolality or Na concentration.
CSWS = cerebral salt-wasting syndrome; SIADH = syndrome of inappropriate antidiuretic hormone.

E. Clinical Impact
1. Hyponatremia may result in increased brain edema and elevated ICP.
2. May cause neurologic symptoms such as delirium, agitation, tremor, seizure, or coma

ACCP Updates in Therapeutics® 2022: Critical Care Pharmacy Preparatory Review and Recertification Course

88
Neurocritical Care

Table 2. Hyponatremia Clinical Symptoms by Severity of Hyponatremia


Serum [Na+] 130–135 Serum [Na+] Serum [Na+]
mEq/L 120–130 mEq/L < 120 mEq/L
• Asymptomatic • Malaise • Headache
• Headache • Unsteadiness • Restlessness
• Nausea • Headache • Lethargy
• Vomiting • Nausea • Seizures
• Fatigue • Vomiting • Brain stem herniation
• Confusion • Fatigue • Respiratory arrest
• Anorexia • Confusion • Death
• Muscle cramps • Anorexia
• Depressed reflexes • Muscle cramps

F. SIADH: Increased secretion of ADH (or vasopressin) results in increased water retention at the renal distal
tubules.

Table 3. Typical Causes of SIADH


TBI Medications
Brain tumor Desmopressin/vasopressin
Stroke Selective serotonin reuptake inhibitors
Brain infection Tricyclic antidepressants
SAH Carbamazepine
Intracerebral hemorage Oxcarbazepine
Pneumonia/tuberculosis Chlorpropamide
Lung cancer Cyclophosphamide/iphosphamide
Methylenedioxymethamphetamine
Nicotine
Opioids
Phenothiazine antipsychotic medications
Nonsteroidal anti-inflammatory drugs
MDMA (ecstasy)
MDMA = 3,4-methylenedioxymethamphetamine; SAH = subarachnoid hemorrhage; TBI = traumatic brain injury.

G. Treatment – Updated guidelines for diagnosing and managing hyponatremia are available (Intensive Care
Med 2014;40:320-31).

Table 4. Treatment Strategies for SIADH


Fluid Vasopressin
PO Sodium IV Sodium Demeclocycline
Restriction (V)-Antagonists
Restriction
of free water Inhibition of renal
Inhibition of
results in V2 vasopressin
ADH activity,
increased impact receptors
Mechanism Na supplemen- Na possibly because
of insensible (conivaptan V1A
of action tation supplementation of inhibition of
losses, + V2 receptors;
aquaporin water
permitting Na tolvaptan V2
channels
concentration to only)
rise

ACCP Updates in Therapeutics® 2022: Critical Care Pharmacy Preparatory Review and Recertification Course

89
Neurocritical Care

Table 4. Treatment Strategies for SIADH (continued)


Fluid Vasopressin
PO Sodium IV Sodium Demeclocycline
Restriction (V)-Antagonists
Conivaptan:
20–40 mg IV daily
4–16 g/day (1 0.9%–3% at 300 mg every 12 Tolvaptan:
Dose < 1500 mL/day g = 17 mEq of 0.5–1.5 mL/ hr up to 1200 mg/ 15–60 mg PO
Na) kg/hr day daily (Intensive
Care Med
2014;40:320-31)
Modest, more
Modest, more Modest, prompt
Modest, delayed effective for Modest, delayed
Efficacy effective for CSWS over the first 24
(over 2+ days) CSWS (over 2+ (over 1 wk)
(over 2+ days) hr
days)
Thirst, GI upset,
Common Thirst;
diarrhea, Fluid overload, hepatotoxicity,
adverse Thirst conivaptan
nausea, hyperchloremia nephrotoxicity,
effects Infusion pain
vomiting photosensitivity
Cost; drug-drug
≤ 3% sodium
Difficult interactions are
chloride may be
to ensure common; risk of
given through
adherence; overcorrection of
peripheral IV line; Chelation
caution for Na > 8 mEq/24
Poor The smallest- occurs with
permitting hours; tolvaptan
Common palatability bore IV in the coadministered
hypovolemia should not be
considerations when taken by largest available cations;
in patients used > 30 days
mouth vessel should be nephrotoxicity has
with cerebral in patients with
used. Central been reported
perfusion needs liver disease due
line remains the
such as SAH, to risk of life-
preferred access, if
TBI threatening liver
available
injury
ADH = antidiuretic hormone; IV = intravenous(ly); PO = oral(ly).

1. Treatment of SIADH can be challenging in neurocritical care patients such as those with SAH and TBI.
2. Treatment of choice is fluid restriction, which is typically not feasible in patients with SAH or TBI.
3. Priority on maintaining euvolemia to optimize CPP, particularly when treating elevated ICP or cerebral
vasospasm
4. Hypertonic sodium solutions are often needed to raise serum sodium.
5. A practice-based study evaluated the impact of various hyponatremia treatments on the serum sodium
concentration (Neurocrit Care 2017;27:242-8). Hypertonic saline solutions were commonly used and
most effectively increased the serum sodium.

H. Cerebral Salt-Wasting Syndrome (CSWS)


1. Etiology is largely unknown, but speculation typically focuses on the increased secretion of natriuretic
peptides, causing loss of sodium at the renal distal tubules.
2. May also be associated with relative adrenal insufficiency
3. Typical causes include TBI, SAH, and brain tumor.

ACCP Updates in Therapeutics® 2022: Critical Care Pharmacy Preparatory Review and Recertification Course

90
Neurocritical Care

4. Fludrocortisone 0.1–0.4 mg/day may help reduce sodium loss in CSWS (Arch Intern Med 2008;168:
325-6).
a. Hypokalemia: Consider potassium supplementation.

I. Considerations for Rapid Correction of Hyponatremia


1. Recommended increase in serum sodium concentration is 0.5 mEq/L/hour or less (Box 1).
2. Patients with chronic hyponatremia may need to be corrected more slowly because of the equilibration
of brain electrolytes with chronic hyponatremic state (N Engl J Med 2000;342:1581-9).
3. Patients with acute hyponatremia may tolerate quicker correction.
4. Patients with severe neurologic symptoms because of hyponatremia appear to tolerate rapid correction
of 4–6 mEq/L (with subsequent cautious elevations in serum sodium as described earlier) (Crit Care
Med 2017;45:1762-71).

Box 1. Common Equations Used for Sodium Correction in Hyponatremia


Na requirement (mmol) = total body water (0.6 × kg)a × (desired Na − current Na)
Infusion rate (mL/hr) = (Na requirement × 1000)/(infusion Na concentration × time)
a
0.6 for men, 0.5 for women; infusion Na concentration in millimoles per liter and time in hours.
Na = sodium.
Information from: Adrogue HJ, Madias N. Hyponatremia. N Engl J Med 2000;342:1581-9.

5. Quicker correction in patients with severe symptoms (coma, seizures) may be prudent – Up to 1–2
mEq/L/hour for the first few hours
6. Rapid correction necessitates frequent serum sodium monitoring (e.g., every 4 hours) to avoid
overcorrection or too-rapid correction.
7. Too-rapid correction of serum sodium may result in osmotic demyelination syndrome; a routine
approach should be to limit Na increase to not more than 12 mEq/L in the first 24 hours or less than 18
mEq/L in the first 48 hours (Intensive Care Med 2014;40:320-31).

II.  HYPERNATREMIA

A. Epidemiology
1. Hypernatremia (Na greater than 150 mEq/L) is also common in patients with neurologic injury.
2. SAH up to 22%
3. TBI up to 21%
4. Hypernatremia and hyperchloremia are increasingly associated with an increased risk of acute kidney
injury; thus, careful monitoring is warranted.

B. Diagnosis/Pathophysiology: Laboratory tests (serum sodium) are needed to diagnose hypernatremia.


Urine sodium, urine osmolality, urine specific gravity, urine volume, and serum osmolality may also help
determine the specific pathogenesis.

C. Typical Causes
1. Consideration of iatrogenic hypernatremia
2. Diabetes insipidus

ACCP Updates in Therapeutics® 2022: Critical Care Pharmacy Preparatory Review and Recertification Course

91
Neurocritical Care

a. Decreased secretion of ADH or vasopressin results in decreased retention of water at the renal
distal tubules.
b. Characterized by voluminous (greater than 250 mL/hour), dilute urinary output

D. Treatment
1. Hypotonic solutions for free-water replacement
a. Dextrose 5% in water
b. 0.45% sodium chloride
c. Water supplementation orally or by feeding tube
2. Vasopressin analogs
a. Supplementation of ADH to normal functional concentrations
b. Titrate therapy to normalized urinary output, serum sodium correction, and urine specific gravity.
c. Desmopressin
i. Intravenously or subcutaneously: 0.5–4 mcg every 8–12 hours (usual starting dose 1–2 mcg)
ii. Intranasally: 10–40 mcg/day divided into two or three doses (usual starting dose 10 mcg)
iii. Orally: 50–800 mcg divided into two doses (usual starting dose 50 mcg)
iv. May be dosed as needed, depending on initial laboratory values
d. Patients after pituitary removal may more commonly require long-term therapy.
3. Arginine vasopressin – Continuous infusion 1–15 units/hour (usual starting dose 1 unit/hour; titrate to
urinary output)
4. Considerations for rapid correction of hypernatremia
a. Recommended decrease in serum sodium concentration is 0.5 mEq/L/hour or less.
b. Too-rapid correction of serum sodium may result in cerebral edema.
c. In general, neurocritical care patients should receive minimal amounts of dextrose or free water–
containing fluids to avoid the risk of cerebral edema.

III.  STATUS EPILEPTICUS

A. Epidemiology – Accounts for 150,000 admissions in the United States annually

B. Status Epilepticus – Continuous seizures for 5 minutes or more OR intermittent seizures without regaining
consciousness in between seizures (N Engl J Med 1990;323:497-502; Neurocrit Care 2012;17:3-23)
1. Generalized convulsive status epilepticus: Generalized convulsions present with onset of seizure
activity – Usually clinically evident
2. Nonconvulsive status epilepticus: Typically occurs in comatose patients with no overt convulsions
present (seizures noted with electroencephalogram [EEG] monitoring only)

C. Refractory Status Epilepticus (RSE) – Status epilepticus that persists after standard treatment (e.g., a
benzodiazepine followed by another anticonvulsant medication)

D. Super-refractory status epilepticus – Failure to wean continuous anesthetic agent after 24–48 hours of burst
suppression or termination of continuous EEG seizures

E. Diagnosis/Pathophysiology – Diagnostic tests


1. Laboratory tests often show electrolyte abnormalities (particularly sodium, magnesium, and
phosphorus).

ACCP Updates in Therapeutics® 2022: Critical Care Pharmacy Preparatory Review and Recertification Course

92
Neurocritical Care

2. EEG monitoring is necessary to identify and characterize seizures.


a. Continuous monitoring is preferred in patients with status epilepticus to capture intermittent or
fluctuating seizure patterns (Neurocrit Care 2012;17:3-23).
b. Typical recommended duration is at least 48 hours, and monitoring should be initiated as soon as
possible after suggestion or diagnosis of seizure.

F. Causes

Table 5. Typical Etiologies of Status Epilepticus


Cause of Status Epilepticus Approximate % of Patients
Epilepsy 33–55
Miscellaneous 12–24
Stroke 14–22
Anticonvulsant nonadherence 20
Drug withdrawal 10–14
Brain tumor 10
Metabolic 10
TBI 7
Drug toxicity 5
CNS infection 3
CNS = central nervous system; TBI = traumatic brain injury.

G. Clinical Impact
1. Mortality rate ranges from 9% (primarily in patients with preexisting epilepsy/anticonvulsant
medication nonadherence) to 30% (in patients with a concomitant pathology such as TBI or stroke).
a. Mortality in nonconvulsive status epilepticus is about double that in more overt seizures.
b. Older adults have a higher mortality rate.
2. Discharge disposition: 14%–18% of patients presenting to the emergency department in status
epilepticus ultimately have a resultant neurologic deficit.

H. Agent Selection (Neurocrit Care 2012;17:3-23; Epilepsy Curr 2016;16:48-61)


1. First-line therapy (emergency)
a. Benzodiazepine therapy preferred; appropriate dosing is essential – Do NOT underdose first-line
therapy
b. Lorazepam 0.1 mg/kg intravenously (maximum 4 mg/dose) OR
c. Midazolam 5–10 mg (or 0.2 mg/kg, maximum dose 10 mg) intramuscularly (preferred for patients
with no intravenous access) OR
d. Diazepam intravenously (0.15–0.2 mg/kg, maximum 10 mg/dose) or rectally (0.2–0.5 mg/kg,
maximum 20 mg/dose) – Not recommended as first-line therapy for hospitalized patients because
of short duration of seizure control
2. Second-line therapy (urgent): Initiate an anticonvulsant after benzodiazepine therapy if seizures persist
or if a maintenance therapy needs to be initiated to prevent future seizures.
a. Can add anticonvulsants if the patient does not respond to treatment and is not intubated
b. Valproate 40 mg/kg (maximum dose 3000 mg) intravenously
c. (Fos)phenytoin 20 mg/kg intravenously
d. Phenobarbital 20 mg/kg intravenously
e. Levetiracetam 1–3 g intravenously or 60 mg/kg with a maximum of 4500 mg

ACCP Updates in Therapeutics® 2022: Critical Care Pharmacy Preparatory Review and Recertification Course

93
Neurocritical Care

3. Third-line therapy (refractory)


a. If seizures persist after first- and second-line therapy, RSE should be treated aggressively and in a
timely manner with continuous infusion anesthetic agents. The patient should be intubated before
starting therapy.
b. RSE agents should be titrated to burst suppression (the almost total elimination of electrical activity
on EEG, except for short “bursts” of cortical activity [typically therapy is titrated to 2–5 bursts/
minute]) or termination of electrographic seizure activity, depending on provider preference.
c. Treat RSE for 24–48 hours before trying to taper these continuous agents. In addition, optimize
maintenance anticonvulsant therapy before wean.
d. Limited data exist supporting any one agent over another.
e. Continuous infusion anesthetics for RSE
i. Midazolam high-dose infusion 0.05–2 mg/kg/hour (target burst suppression)
ii. Pentobarbital infusion (loading dose about 25 mg/kg total, continuous infusion 0.5–5 mg/kg/
hour [target burst suppression])
iii. Propofol infusion 20–200 mcg/kg/minute (target burst suppression)
f. Other options for RSE in nonintubated patients
i. Valproate 20–40 mg/kg intravenously (if not already given)
ii. Lacosamide 200–400 mg intravenously
iii. Topiramate 200–400 mg orally/nasogastrically
iv. Other anticonvulsants listed earlier if not already given
4. Super-refractory status epilepticus
a. Treatment is typically reinitiation of continuous anesthetic agents used for RSE.
b. Ketamine, an N-methyl-d-aspartate (NMDA) receptor antagonist, may also be considered.
Ketamine may cause emergence psychosis and hallucinations, for which midazolam may help.
c. Patients should have a workup for causes of super-refractory status epilepticus (e.g., NMDA
receptor antibody encephalitis, paraneoplastic syndromes, infectious encephalitis).
5. Nonpharmacologic therapies if medical therapy fails
a. Ketogenic diet: Limiting medications and nutrition to low-carbohydrate content; more likely to
be successful in pediatric patients than in adult patients because of difficulty achieving ketosis in
adults. Exact mechanism of action is unknown, but may increase sequestration of glutamate and
decrease reactive oxygen species. Benefits typically occur within 1–3 months. Must modify diet
targeting fat to carbohydrate plus protein in a 4:1 or 3:1 ratio
b. Vagus nerve stimulation
c. Surgical management
d. Hypothermia

I. Monitoring
1. Continuous EEG monitoring is necessary for status epilepticus and RSE.
2. Proactively monitor serum concentrations for agents such as phenytoin and valproic acid to ensure
adequate concentrations and mitigate the risk of toxicity.

ACCP Updates in Therapeutics® 2022: Critical Care Pharmacy Preparatory Review and Recertification Course

94
Neurocritical Care

Table 6. Characteristics of Agents for Status Epilepticus


Antiepileptic Common
Dosing Considerations
Drug Adverse Effects
0.1 mg/kg IV (slow IV push)
IV formulation contains propylene
Lorazepam (maximum 4 mg/dose); up to 8 Sedation, hypotension
glycol
mg total
Midazolam 5–10 mg IM, 0.2–0.3 mg/kg
Sedation, hypotension Short duration with IV bolus
(intermittent) intranasally (up to 10 mg)
0.15 mg/kg IV (slow IV push);
IV formulation contains propylene
typically up to 10 mg total
Diazepam Sedation, hypotension glycol; rectal administration may be
0.2 mg/kg intranasally (20 mg
considered
maximum)
20 mg PE/kg IV (not to exceed Hypotension,
Fosphenytoin 150 mg PE/min); arrhythmia, Several drug-drug interactions
may also give IM hepatotoxicity
Several drug-drug interactions,
Hypotension, IV formulation contains propylene
arrhythmia, glycol and ethanol. Addition of
20 mg/kg IV (not to exceed 50
Phenytoin phlebitis, purple parenteral phenytoin to dextrose
mg/min)
glove syndrome, and dextrose-containing solutions
hepatotoxicity should be avoided because of lack of
solubility and resultant precipitation
Hyperammonemia,
40 mg/kg IV (not to exceed 6 Many drug-drug interactions, avoid
Valproic acid thrombocytopenia,
mg/kg/min) in patients with a TBI
hepatotoxicity
1–3 g IV (not to exceed 5 mg/
kg/min) (NCS [Brophy 2012])
Sedation/paradoxical Renally eliminated, limited drug-
Levetiracetam or 60 mg/kg as a single dose
excitation, irritability drug interactions
(maximum 4.5 g) (AES [Glauser
2016])
200–400 mg IV (typically over Dizziness,
Lacosamide Limited drug-drug interactions
15–30 min) bradyarrhythmia
PO/enteral loading dose of 400
mg QID × 2 days, tapering to
Topiramate Metabolic acidosis No IV formulation available
200 mg BID by 200 mg/day
every 2 days or as tolerated
Sedation,
20 mg/kg (not to exceed 100 IV formulation contains propylene
Phenobarbital hypotension,
mg/min) glycol
respiratory depression
Sedation,
10 mg/kg (typically over
hypotension,
15–30 min, depending on
respiratory IV formulation contains propylene
blood pressure); typically need
Pentobarbital depression, glycol; target is burst suppression;
additional 5 to 10 mg/kg boluses
constipation, several drug-drug interactions
to full loading dose of 25–30
cardiac depression,
mg/kg; 1 to 3 mg/kg/hr infusion
immunosuppression

ACCP Updates in Therapeutics® 2022: Critical Care Pharmacy Preparatory Review and Recertification Course

95
Neurocritical Care

Table 6. Characteristics of Agents for Status Epilepticus (continued)


Antiepileptic Common
Dosing Considerations
Drug Adverse Effects
Midazolam Sedation,
Potential tachyphylaxis, target is
high-dose 0.05–2 mg/kg/hr hypotension,
burst suppression
infusion respiratory depression
Sedation,
20–200 mcg/kg/min, titrate by
hypotension,
5 mcg/kg/min; use caution Target is burst suppression; provides
respiratory
Propofol when administering doses > 1.1 kcal/mL; hypertriglyceridemia at
depression,
80 mcg/kg/min for extended higher doses
pancreatitis, propofol
periods
infusion syndrome
May be more effective in prolonged
Excitation, refractory status epilepticus. May
hypertension, possible cause hypotension in patients with
Ketamine 0.5–10 mg/kg/hr
neurotoxicity, decreased shock index; consider
hallucinations IV fluid volume with high-dose
infusions
AES = American Epilepsy Society; BID = twice daily; IM = intramuscular(ly); NCS = Neurocritical Care Society; PE = phenytoin equivalents; QID = four times daily.
Brophy GM, Bell R, Claassen J, et al. Guidelines for the evaluation and management of status epilepticus. Neurocrit Care 2012;17:3-23; Glauser T, Shinnar S, Gloss D,
et al. Evidence-based guideline: treatment of convulsive status epilepticus in children and adults: report of the Guideline Committee of the American Epilepsy Society.
Epilepsy Curr 2016;16:48-61.

Patient Case

1. A 37-year-old man is admitted to the emergency department with meningitis. On admission, his GCS score
decreases from E3-M6-V1T to E1-M5-V1T over 10 minutes, and his nurse notices facial twitching. An EEG
is ordered. Current medications include famotidine 20 mg intravenously every 12 hours, heparin 5000 units
subcutaneously every 8 hours, docusate 100 mg nasogastrically twice daily, and a supplemental vitamin
infusion for potential alcohol withdrawal. Which is the best acute therapy for this patient’s suspected sei-
zure activity?
A. Fosphenytoin 20 mg PE/kg intravenously × 1.
B. Valproic acid 20 mg/kg intravenously × 1.
C. Midazolam 10 mg intramuscularly × 1.
D. Levetiracetam 1 g intravenously × 1.

IV.  CENTRAL NERVOUS SYSTEM INFECTION: INTRAVENTRICULAR ANTIBIOTIC


ADMINISTRATION

A. Case Selection
1. Recommended in adult patients with CSF shunt or ventriculostomy infections for difficult-to-eradicate
pathogens or for patients who cannot undergo the surgical component of therapy
2. May also be reasonable in patients receiving systemic therapy who have not cleared CSF cultures after
3–4 days
3. Not recommended for neonatal or infant central nervous system (CNS) infection

ACCP Updates in Therapeutics® 2022: Critical Care Pharmacy Preparatory Review and Recertification Course

96
Neurocritical Care

B. Appropriate Dosing
1. Intravenous plus intraventricular is probably superior to intravenous or intraventricular alone.
2. Use preservative-free formulations.
3. Do not use diluents containing dextrose, whenever possible.
4. Do not use medications known to lower the seizure threshold (e.g., β-lactams).
5. Daily dosing is usually necessary; may need to adjust according to the amount of CSF drainage from
external ventricular drain

Table 7. Various Antimicrobials and Doses for Intraventricular Administration


Daily Dose/Volume Approximate
Antimicrobial Common Adverse Effects
(adults) Osmolality (mOsm/kg)
Headache, mental status changes,
Vancomycin 10–20 mg/1 mL of NS 291
possible hyponatremia
Gentamicin 4–8 mg/1 mL of NS 293 Seizures
Tobramycin 4–8 mg/1 mL of NS 283 Seizures
Amikacin 30 mg/1 mL of NS 383 Seizures
Hypotonia, seizures, meningeal
Polymyxin B 5 mg/1 mL of NS 10
inflammation
Colistimethate 10 mg/3 mL of NS 367 Meningeal inflammation
Amphotericin B
0.5 mg/3 mL of SWI 256 (in dextrose 5%) Nausea, vomiting
deoxycholate
Daptomycin 5 mg/2 mL of NS 364 Unknown
NS = normal saline; SWI = sterile water for injection.
Cook AM, Mieure KD, Owen RD, et al. Intracerebroventricular administration of medications. Pharmacotherapy 2009;29:832-45; Mueller SW, Kiser TH, Anderson
TA, et al. Intraventricular daptomycin and intravenous linezolid for the treatment of external ventricular-drain-associated ventriculitis due to vancomycin-resistant
Enterococcus faecium. Ann Pharmacother 2012;46:e35.

V.  INTRACRANIAL PRESSURE TREATMENT

A. General Concepts
1. Elevated ICP decreases tissue perfusion and tissue oxygenation and worsens neurologic outcome.
2. Monro-Kellie doctrine: ICP equals cerebral blood volume (10%) plus CSF (10%) plus brain tissue
(80%). Each therapy targeted at decreasing ICP acts on one or more of these components.
3. Most practitioners use a stepwise approach to treating elevated ICP, including the following interventions:
a. Head-of-bed elevation (30–45 degrees): Optimizes venous return from the brain, reducing venous
pooling
b. Osmotherapy (mannitol or hypertonic saline)
c. Acute hyperventilation: Reduction in Pco2 to around 32 mm Hg causes a compensatory
vasoconstriction, which reduces cerebral blood volume (chronic hyperventilation should be avoided
because of complications such as stroke).
d. Drainage of CSF by a ventriculostomy
e. Sedation with or without neuromuscular blockade (avoid benzodiazepines, when possible)
f. Maintenance of CPP at 60–70 mm Hg
g. Surgical decompression or hemicraniectomy (depending on the clinical scenario)
h. Pharmacologic coma (pentobarbital)
i. Hypothermia (33°C–36°C) – Clinicians should consider the risk-benefit of moderate hypothermia
versus targeted temperature management for control of ICP.

ACCP Updates in Therapeutics® 2022: Critical Care Pharmacy Preparatory Review and Recertification Course

97
Neurocritical Care

B. Treatment Thresholds
1. Recommendations are to treat sustained ICP greater than 22 mm Hg as measured by external ventricular
drain, intraparenchymal catheter, or bolt (Neurosurgery 2017;80:6-15). If subarachnoid hemorrhage,
symptom-based dosing over ICP targets is suggested (Neurocrit Care 2020;32;647-66).
2. Specific ICP threshold may have interpatient variability.
3. CPP ideally within 60–70 mm Hg
a. Increasing the threshold may be necessary in patients who have refractory elevations in ICP or who
have a history of uncontrolled hypertension before admission.
b. Targeting CPP greater than 80 mm Hg routinely is also associated with an increased incidence of
acute respiratory distress syndrome and mortality in patients with a TBI; thus, patients must be
selected carefully when targeting higher ranges of CPP (Crit Care Med 1999;27:2086-95).
4. Osmotherapy

Table 8. Comparison of Osmotherapy Agents


Mannitol Hypertonic Saline
Acute increase in cerebral blood flow Acute increase in cerebral blood flow
results in cerebral vasoconstriction results in cerebral vasoconstriction
(because of autoregulation), leading to (because of autoregulation), leading to
decreased cerebral blood volume decreased cerebral blood volume
Mechanism of action
Increase in serum osmolality creates Increase in serum osmolality creates
osmotic gradient to pull extracellular osmotic gradient to pull extracellular
fluid from brain fluid from brain
Osmotic diuretic
3%: 2.5–5 mL/kg over 15 min
0.5–1 g/kg over 15 min (0.2-micron filter) 7.5%: 1–2 mL/kg over 15 min
Typical dose
Up to 1.6 g/kg if acute herniation 23.4%: 30 mL over 15 min (may infuse
over 5–10 minutes in acute herniation)
Serum: Osmolality, BMP (Na, K, SCr,
Serum: Osmolality, Na, SCr, K
Monitoring values BUN, glucose), osmolar gap
Urine: Urinary output
Urine: Urinary output
Hyper/hyponatremia Hypernatremia
Hypokalemia Hypokalemia
Adverse effects Renal failure Hyperchloremic acidosis
Hypovolemia Renal failure
Rebound cerebral edema (?) Osmotic demyelination syndrome (?)

5. Metabolic acidosis may occur after several hypertonic saline doses as the result of hyperchloremia.
a. In patients with acidemia, which complicates ventilator management or other aspects of care,
a combination of sodium chloride and sodium acetate may be considered to maintain the
hyperosmolarity of the solution but reduce chloride provision (Crit Care Med 1999;26:440-6).
b. Sodium bicarbonate 8.4% may also be considered for acute ICP elevations when other osmotherapy
options are not immediately available, such as in patient care areas that do not typically care for
neurologic ICU patients or patients with a TBI (Neurocrit Care 2010;13:24-8).

ACCP Updates in Therapeutics® 2022: Critical Care Pharmacy Preparatory Review and Recertification Course

98
Neurocritical Care

6. Monitoring osmolar changes with osmotherapy: The traditional serum osmolality threshold was 320
mOsm/L when using mannitol.
a. Theory was that serum osmolality values greater than 320 were associated with renal dysfunction.
b. Osmolar gap appears to be a more appropriate and accurate method of evaluating renal dysfunction
risk with mannitol.
c. Approximates the mannitol concentration
d. Goal osmolar gap is less than 55 mOsm/kg.
e. Calculation of osmolar gap (Box 2)

Box 2. Calculation of Osmolar Gap

Osmolar gap = measured osmolality − estimated osmolality


Osmolar gap = measured osmolality − [(2 × Na) + (BUN/2.8) + (glucose/18)]

C. Metabolic Suppression
1. Mechanism of action: Suppression of electrical activity in brain (i.e., “burst suppression”) causes
reduced cerebral metabolic rate of oxygen (CMRO2).
2. Reduced CMRO2 leads to decreased cerebral blood volume.
3. Pentobarbital sodium is usually used in the United States (thiopental in Europe).
4. Risks may outweigh benefit, at least for certain conditions such as large hemispheric infarction.
5. Pentobarbital is no more effective and is potentially more harmful for first-line therapy for ICP control
than mannitol – Early studies led clinicians to begin using pentobarbital only in patients with refractory
ICP elevations (Can J Neurosurg 1984;11:434-40).
6. Typical pentobarbital dosage
a. 25–30 mg/kg intravenous loading dose. Usually given as 10 mg/kg × 1 dose, followed by 5 mg/kg
every hour × 3 or 4 doses to avoid hypotension with large bolus dose
b. 1- to 5-mg/kg/hour infusion after loading dose
7. Titration
a. Titrated to goal ICP (usually less than 20 mm Hg)
b. Burst suppression on continuous EEG (target usually is 2–5 bursts/minute) is a surrogate end point
for need of additional pentobarbital doses.
c. A bolus dose is required concomitantly with an infusion titration because of its long half-life and
rapid redistribution.
8. Monitoring
a. ICP
b. EEG and burst occurrence per minute
c. Serum concentrations do not correlate well with ICP response and should not be used to titrate
infusion. May be useful when therapy has been discontinued as part of brain death examination (to
rule out continued intoxication from pentobarbital)
d. Due to the risk of metabolic acidosis from propylene glycol accumulation, serum chemistry, pH,
and osmolality should be monitored during prolonged treatment.

ACCP Updates in Therapeutics® 2022: Critical Care Pharmacy Preparatory Review and Recertification Course

99
Neurocritical Care

9. Adverse effects
a. Hypotension as the result of several different causes
i. Propylene glycol diluent
ii. Direct vasodilator
iii. Reduction in sympathetic tone because of metabolic suppression
iv. Cardiac depressant (particularly with high doses and duration greater than 96 hours)
b. Bradycardia
c. Decreased GI motility and ileus
i. Difficulty with enteral nutrition
ii. Caloric needs are usually around 80%–90% of basal energy needs, so a lower flow rate for
enteral nutrition is permissible.
iii. Ideally, would use an elemental or semi-elemental nutrition product because stooling is rare
on pentobarbital infusion
d. Infection (particularly pneumonia)
e. Immunosuppression
f. Withdrawal seizures may occur.
g. Metabolic acidosis due to propylene glycol toxicity

D. Sedation – Mechanism of action: Decreased systemic oxygen delivery needs; reduced coughing, reduced
agitation, decreased CMRO2
1. Propofol is typically the preferred sedative – Quick onset, short acting, less accumulation with prolonged
duration
a. Patients with a TBI (and other neurologic injuries) require frequent, accurate neurologic
examinations to evaluate the evolution of the neurologic injury.
b. Hypotension risk may be harmful in specific patient types (e.g., aneurysmal SAH/vasospasm, TBI,
SCI).
c. In contrast to midazolam, propofol is advantageous because of its short half-life and relative lack
of residual sedative effects.
d. Although dexmedetomidine has a short half-life similar to propofol, propofol has the added
advantage of reducing CMRO2.
2. Benzodiazepines
a. Not preferred because of duration of action
b. Also associated with delirium and cognitive impairment
c. Potential for withdrawal effect, seizures
3. Dexmedetomidine
a. Little evidence to support use in neurocritical care
b. Hypotension risk may be harmful in specific patient types (e.g., aneurysmal SAH/vasospasm, TBI,
SCI).
c. May be particularly helpful in patients with paroxysmal sympathetic hyperactivity (PSH)
(“storming”)

ACCP Updates in Therapeutics® 2022: Critical Care Pharmacy Preparatory Review and Recertification Course

100
Neurocritical Care

E. Neuromuscular Blockade
1. Mechanism of action: Decreased systemic oxygen delivery needs; reduced coughing
a. Neuromuscular blockers have no intrinsic value for reducing ICP, but they may be helpful in select
patients with specific issues that exacerbate ICP elevations.
i. Prevention of cough, ventilator dyssynchrony (both increase ICP)
ii. Control Pco2 (increased Pco2 may also increase ICP)
b. Prevention of shivering during therapeutic hypothermia or targeted temperature management
c. Reduces intrathoracic pressure
d. May be essential in patients requiring high positive end-expiratory pressure (increased intrathoracic
pressure may increase ICP)
2. Various agents may be useful – Depends on patient organ function, prescriber preference
a. Vecuronium (particularly if normal organ function)
b. Cisatracurium (particularly if end-organ dysfunction)
c. Avoid atracurium, if possible, because of hypotension risk. Laudanosine, a hepatically metabolized
product of Hofmann elimination, is a CNS stimulant that can accumulate with prolonged use.
3. Monitor by train-of-four (goal 1-2/4 twitches with no clinical evidence of neuromuscular function [e.g.,
overbreathing the ventilator rate]).

Patient Case

2. A 25-year-old man is admitted after a two-story fall from a ladder. The initial CT scan of his brain reveals
a large right temporal subdural hematoma, an overlying skull fracture, and a left temporal contusion. His
post-resuscitation GCS is E1-M4-V1T. An ICP monitor is placed with an opening pressure of 32 mm Hg,
and CPP is 53 mm Hg. Serum laboratory values include Na 139 mEq/L, K 3.6 mEq/L, BUN 42 mg/dL, SCr
2.4 mg/dL, glucose 178 mg/dL, WBC 14.8 × 103 cells/mm3, pH 7.46, and Pco2 34 mm Hg. Which is the best
initial therapy for this patient’s elevated ICP?
A. Mannitol 20% 1 g/kg intravenously × 1.
B. 23.4% sodium chloride 30 mL intravenously × 1.
C. Pentobarbital 10 mg/kg intravenously × 1.
D. Midazolam 10 mg intravenously × 1.

F. TBI Guidelines (J Neurotrauma 2007;24:S1-S95; Neurosurgery 2017;80:6-15). The fourth edition of


these guidelines was published in 2017; these are “living guidelines” that will be updated continuously.
Unfortunately, the newest guidelines reflect only recommendations with high levels of evidence. Therefore,
the 2007 guidelines may be needed to review practical recommendations (based on all levels of evidence)
for complications in patients with a TBI.
1. Seizure prophylaxis
a. Recommended as an option for prevention of early posttraumatic seizures (first 7 days after event)
b. Phenytoin/fosphenytoin is the most commonly recommended agent (because of support for use
from prospective clinical trials) (N Engl J Med 1990;323:497-502).
c. Levetiracetam is also commonly used, despite a paucity of data (monitor for pronounced behavioral
adverse effects in patients with neurologic injury).
d. Valproic acid is as effective as phenytoin, but a trend toward increased mortality was observed in
a prospective clinical trial; thus, it is usually avoided if possible (J Neurosurg 1999;91:593-600).
e. Use of anticonvulsants for prevention of late seizures (after 7 days) has not been proven effective
(not recommended).

ACCP Updates in Therapeutics® 2022: Critical Care Pharmacy Preparatory Review and Recertification Course

101
Neurocritical Care

2. CPP modulation
a. CPP = mean arterial pressure (MAP) minus ICP.
b. Surrogate for global cerebral perfusion
c. Recommended goal is 60–70 mm Hg.
d. Ideal CPP may have interpatient variability because of the patient’s medical history and unique
characteristics of the TBI.
e. Patients with a history of poorly treated hypertension (right shift of autoregulatory curve) may
require a higher CPP.
3. Fluid resuscitation with or without vasopressor therapy
a. Norepinephrine or phenylephrine is the preferred vasopressor for this indication.
b. Routine targeting of CPP greater than 80 mm Hg is no more effective than targeting of lower CPP
and may result in increased complications (acute respiratory distress syndrome) and pulmonary
edema (Crit Care Med 1999;27:2086-95).
4. Supportive care – Venous thromboembolism (VTE) prophylaxis
a. Patients with a TBI have an increased risk of VTE because of:
i. TBI-related coagulopathy
ii. Delay in initiation of pharmacologic VTE prophylaxis
iii. Immobility
iv. Concomitant injuries (in polytrauma)
b. Mechanical prophylaxis should be initiated as soon as possible.
c. Pharmacologic prophylaxis should be initiated after intracranial bleeding is stabilized.
i. Typically, 24–48 hours after event
ii. May depend on coagulopathy on admission, extension of bleeding on CT scan, and other
factors
iii. Unfractionated heparin (every 8 hours) or low-molecular-weight heparin may be used for
pharmacologic prophylaxis. Low-molecular-weight heparin may be preferred in patients with
polytrauma, particularly long bone or pelvic fractures.

G. Nutrition Support
1. Initiating nutrition support within 48 hours improves immune competence and may improve neurologic
outcome (Crit Care Med 1999;27:2525-31).
2. Gastric feeding is not well tolerated in patients with a TBI, particularly during the first 5–7 days and
particularly in those with an elevated ICP (causes decreased gastric motility). Postpyloric feeding
access should be established as soon as possible.
3. Metabolic needs are elevated after a TBI (typically proportional to the severity of injury).
a. Patients with a TBI typically require 120%–160% of basal metabolic needs.
b. Metabolic cart/direct calorimetry can be used to better evaluate caloric needs.

H. Prevention of Stress-Related Mucosal Bleeding


1. Patients with a TBI have an increased risk of stress-related mucosal bleeding.
a. Hypotension associated with a TBI or trauma
b. Hypersecretion of acid associated with neurologic injury (Cushing ulcers)
c. Potential for coagulopathy
d. Need for mechanical ventilation

ACCP Updates in Therapeutics® 2022: Critical Care Pharmacy Preparatory Review and Recertification Course

102
Neurocritical Care

2. Almost all patients with a severe TBI should receive prophylaxis for stress-related mucosal bleeding.
a. Histamine-2 receptor antagonists (H2RAs) have traditionally been the preferred agents.
b. Proton pump inhibitors (PPIs) also raise gastric pH and permit hemostasis in areas of gastritis.
c. Recent meta-analyses have suggested that PPIs are superior to H2RAs, but a well-powered clinical
trial has not been completed in the ICU or the neurologic ICU population.
d. Which agent to select may depend on:
i. Medications taken at home before admission
ii. Presence of GI bleeding on admission
iii. Risk of Clostridium difficile infection

I. Glycemic Control
1. Hyperglycemia is associated with increased mortality in TBI.
2. Potential mechanisms
a. Glucose toxicity in neurons
b. Surrogate for severity of injury
c. Exacerbation of cerebral edema
3. Avoid administering dextrose 5% and other hypotonic glucose-containing fluids.
4. Glycemic goals
a. Prevent hyperglycemia (greater than 180 mg/dL)
b. 140–180 mg/dL seems reasonable to avoid hypoglycemia.
c. Caution should be used with glucose values in the low-normal range because of the risk of
hypoglycemia.
d. Hypoglycemia is associated with a worse outcome in TBI.
i. Glucose obligate substrate for neurons
ii. Threshold for glucose needs may be altered in TBI.
iii. May increase seizure risk

J. Steroids
1. High-dose methylprednisolone plays no role in the treatment of inflammation or edema associated with
TBI.
2. Large, prospective, randomized clinical trial showed increased mortality in steroid group compared
with placebo (CRASH) (Lancet 2005;365:1957-9).

K. Pharmacokinetic Alterations
1. Altered volume of distribution: Patients with a TBI have increased volume of distribution because of
the following:
a. Fluid resuscitation
b. Reduced plasma protein binding (particularly with albumin as a negative acute-phase reactant)
c. Transient increased permeability of blood-brain barrier
2. Hepatic metabolism induction
a. TBI increases hepatic metabolic capacity (extent to which is likely proportional to severity of
injury).
b. Results in more effective clearance of hepatically metabolized medications
c. Increased dosing requirement for commonly used agents such as phenytoin, midazolam
d. Induction subsides over time (usually 1–3 months, but varies by patient).
e. Hypothermia during TBI may also reduce the induction of hepatic metabolism/cause metabolic
rate of medications to be less than baseline.

ACCP Updates in Therapeutics® 2022: Critical Care Pharmacy Preparatory Review and Recertification Course

103
Neurocritical Care

3. Augmented renal clearance


a. Increase in glomerular filtration rate (e.g., CrCL greater than 120 mL/min)
b. Fluid resuscitation
c. Increased endogenous catecholamines and glucocorticoid response
d. Results in more effective clearance of renally eliminated medications
e. Increased dosing requirement for commonly used agents such as vancomycin, aminoglycosides,
β-lactams (J Trauma Acute Care Surg 2016;81:1115-21; Neurocrit Care 2015;23:374-9; Clin
Pharmacokinet 2010;49:1-16; Neurocrit Care 2018 Jul 26 [Epub ahead of print]; Pharmacotherapy
2019;39:346-54)
f. Augmented renal clearance tends to subside over time (usually after first 7–10 days, but varies by
patient).

VI.  PAROXYSMAL SYMPATHETIC HYPERACTIVITY (I.E., “STORMING”)

A. Epidemiology
1. About 8%–33% incidence in survivors of acquired brain injury
2. Commonly associated with TBI (specifically diffuse axonal injury and high-burden parenchymal
lesions), but may occur with other CNS insults

B. Diagnosis
1. Typically, three or more symptoms (J Neurotrauma 2014;31:1515-20)
a. Fever
b. Tachycardia
c. Hypertension
d. Tachypnea
e. Dyspnea
f. Diaphoresis
g. Muscle rigidity (posturing)
2. Common triggers
a. Pain
b. Bladder distension
c. Turning
d. Tracheal suctioning
e. Typically unprovoked (hence “paroxysmal”)

C. Pathophysiology largely unknown, but thought to be caused by somatosympathetic activation and heightened
activity of brain stem after brain injury

D. PSH Assessment Measure: New tool developed to determine the possibility of PSH at that specific time. Has
two components – Diagnosis Likelihood Tool and Clinical Feature Scale (J Neurotrauma 2014;31:1515-20)

ACCP Updates in Therapeutics® 2022: Critical Care Pharmacy Preparatory Review and Recertification Course

104
Neurocritical Care

Table 9. Preventive and Abortive Therapies for Paroxysmal Sympathetic Hyperactivity


Preventive Therapies Abortive Therapies
Baclofen IT (titrated according to patient response) Baclofen IT (titrated according to patient response)
Bromocriptine 1.25 mg enterally BID (up to 40 mg/day) Dantrolene 0.25–2 mg/kg IV every 6–12 hr
Clonidine 0.1–0.3 mg enterally three times daily Dexmedetomidine 0.2–1.5 mcg/kg/hr IVa
Gabapentin 100–300 mg three times daily (up to 4800 Diazepam 5–10 mg IV PRN
mg/day) Fentanyl 25–100 mcg IV PRN
Propranolol 20–60 mg enterally every 4–6 hr Morphine 2–8 mg IV PRNa
Propranolol 1–3 mg IV every 4 hra
a
Most common treatments.
IT = intrathecal; PRN = as needed.
Rabinstein AA, Benarroch EE. Treatment of paroxysmal sympathetic hyperactivity. Curr Treat Options Neurol 2008;10:151-7; Thomas A, Greenwald BD. Paroxysmal
sympathetic hyperactivity and clinical considerations for patients with acquired brain injuries: a narrative review. Am J Phys Med Rehabil 2018 Jun 22. [Epub ahead
of print]

VII.  ACUTE ISCHEMIC STROKE

A. Epidemiology
1. Fifth leading cause of death and No. 1 cause of disability in the United States, with around 800,000
strokes in the United States annually
2. 85% of strokes in the United States are ischemic.

B. Diagnosis/Pathogenesis
1. Diagnostic tests
a. Neurologic examination
b. Vital signs
c. NIH Stroke Scale (greater than 25 is severe, range 1–42)
d. Imaging and other tests (Stroke 2013;44:870-947)
e. Noncontrast CT scan or magnetic resonance imaging (MRI) of the brain (to rule out bleeding)
f. CT angiography (if intra-arterial thrombolysis or thrombectomy is contemplated)
g. CT or MRI perfusion and diffusion imaging may be considered for patients outside the thrombolysis
window.
h. Chest radiography (if lung disease is suspected)
i. Lumbar puncture (if SAH is suspected and CT scan is negative for blood)
j. EEG (if seizures are suspected)
2. Laboratory tests
a. Blood glucose
b. INR, activated partial prothrombin time (consider thrombin time, anti–factor Xa [anti-Xa] activity
for newer oral anticoagulants)
c. Complete blood cell count (CBC)
d. Tests for hypercoagulable state

C. Causes
1. Cardioembolic (29.1%)
2. Large-artery atherosclerosis (16.3%)
3. Lacunar infarcts (15.9%)
4. Unknown (36.1%)
5. Other (2.6%)

ACCP Updates in Therapeutics® 2022: Critical Care Pharmacy Preparatory Review and Recertification Course

105
Neurocritical Care

D. Treatment Considerations (Stroke 2013;44:870-947)


1. 1. Thrombolysis
a. Alteplase 0.9 mg/kg (maximum 90 mg) within 4.5 hours of symptom onset; 10% of total dose as
intravenous bolus, followed by 90% as 60-minute intravenous infusion.
b. Tenecteplase 0.25 mg/kg intravenous bolus (maximum 25 mg) within 4.5 hours of symptom
onset may also be considered for off-label use in patients who are also eligible for mechanical
thrombectomy.
c. In patients with acute ischemic stroke who awake with stroke symptoms or have unclear time of
onset more than 4.5 hours from last known well, or at baseline state can undergo MRI to help select
those who can benefit from intravenous alteplase administration within 4.5 hours of stroke symptom
recognition.

Table 10. Typical Inclusion/Exclusion Criteria for IV Alteplase for Ischemic Strokea
Patient Selection Criteria Patient History Excludes All Contraindications
Onset of symptoms < 4.5 hr from drug Intracranial or intraspinal surgery within 3 mo
administration Head trauma or stroke < 3 mo
Baseline head CT excludes intracerebral Active internal or intracerebral bleeding
hemorrhage (ICH) or other risk factors Symptoms suggestive of SAH
Age ≥ 18 yr Any history of ICH
Intracranial neoplasm, arteriovenous malformation, or aneurysm
Vital signs and laboratory values: Arterial puncture at noncompressible site within 1 wk
INR ≤ 1.7 Current SBP > 185 mm Hg or DBP > 110 mm Hg
Plt ≥ 100,000/mm3 Current use of anticoagulant agents with evidence of elevated
Blood glucose > 50 mg/dL sensitive laboratory tests, including direct-acting oral anticoagu-
Blood pressure control (SBP < 185 mm Hg, lants and therapeutic doses of low-molecular-weight heparin
DBP < 110 mm Hg)
Additional exclusion criteria for 3- to 4.5-hr window:
NIHSS > 25
Current treatment with PO anticoagulants (regardless of INR)
Evidence of ischemic injury > 1/3 of MCA territory
a
Exclusions are primarily based on the risk of systemic bleeding or hemorrhagic conversion of stroke.
DBP = diastolic blood pressure; MCA = middle cerebral artery; NIHSS = NIH Stroke Scale (score); SBP = systolic blood pressure; tPA = tissue plasminogen activator.
Hacke W, Kaste M, Bluhmki E, et al. Thrombolysis with alteplase 3 to 4.5 hours after acute ischemic stroke. N Engl J Med 2008;359:1317-29. Demaerschalk BM,
Kleindorfer DO, Adeoye OM, et al. Scientific rationale for the inclusion and exclusion criteria for intravenous alteplase in acute ischemic stroke: a statement for
healthcare professionals from the American Heart Association/American Stroke Association. Stroke 2016;47:581-641. Powers WJ, Rabinstein AA, Ackerson T, et al.
2018 guidelines for the early management of patients with acute ischemic stroke: a guideline for healthcare professionals from the American Heart Association/American
Stroke Association. Stroke 2018;49:e46-e99. Powers WJ, Rabinstein AA, Ackerson T, et al. Guidelines for the early management of patients with acute ischemic stroke:
2019 update to the 2018 guidelines for the early management of acute ischemic stroke: a guideline for healthcare professionals from the American Heart Association/
American Stroke Association. Stroke 2019;50:e344-e418.

2. Permissive hypertension
a. Reduction in blood pressure after thrombolysis or recanalization is reasonable within the first 24
hours after the onset of stroke.
b. Reduce blood pressure cautiously to avoid hypotension or underperfusion of infarcted area (less
than 15% blood pressure lowering).
c. Resumption of home blood pressure medications is reasonable 24 hours after the onset of stroke.
d. Recommended to treat blood pressure in patients who do not receive thrombolytics if it exceeds
SBP greater than 220 mm Hg or diastolic blood pressure (DBP) greater than 120 mm Hg
3. Seizure prophylaxis. Use of anticonvulsant medications for seizure prophylaxis is not indicated after
ischemic stroke.

ACCP Updates in Therapeutics® 2022: Critical Care Pharmacy Preparatory Review and Recertification Course

106
Neurocritical Care

4. Mechanical thrombectomy and neuroendovascular interventions for ischemic stroke


a. Neuroendovascular devices may be used to remove or disrupt clot to facilitate recanalization.
b. No difference in safety outcomes when comparing usual care with usual care plus mechanical
thrombectomy
5. Many clinical trials support mechanical thrombectomy in patients with acute ischemic stroke.
a. Increases the likelihood of independence or improved modified Rankin Scale score compared with
standard therapy
b. Does not increase the risk of intracerebral hemorrhage (ICH)
c. Extends the treatment window both for patients receiving tissue plasminogen activator (tPA) within
the therapeutic window and for those who are ineligible for tPA
6. Blood pressure management after thrombectomy
a. A retrospective, multicenter study of blood pressure after thrombectomy suggested that higher blood
pressures were associated with more complications and worse outcomes (Stroke 2019;50:2448-54).
b. The cohort with an SBP of 121–140 mm Hg appeared best in this study.
c. The optimal blood pressure after thrombectomy is not well defined and needs further, high-quality
study.
7. Secondary prevention
a. Initiating aspirin (325 mg × 1; then 81–325 mg/day), a high-intensity statin, and an intensive blood
pressure regimen is necessary.
i. Ideally, give as soon as feasible after the onset of stroke.
ii. Aspirin should not be initiated within 24 hours of alteplase.
b. Clopidogrel 75 mg daily is also an option in patients whose aspirin therapy has failed or who have
an aspirin allergy.
c. Aggrenox (dipyridamole/aspirin) and ticagrelor plus aspirin (less than 100 mg) are also effective
alternatives to aspirin for secondary prevention.
d. Control/modification of other disease states is often necessary.
e. Hypertension: Threshold to treat high blood pressure is greater than 130/80 mm Hg; typical blood
pressure goal is less than 130/80 mm Hg (Stroke 2021;52:e364-467).
f. Atrial fibrillation
i. Rate or rhythm control
ii. Anticoagulation (warfarin, direct-acting oral anticoagulants)
iii. Typically, anticoagulant therapy is delayed until at least 5–14 days after stroke to reduce the
risk of hemorrhagic conversion.
iv. Avoid using loading doses of warfarin.
g. Carotid artery stenosis: Stent versus endarterectomy (usually for patients with greater than 70%
blockage and/or clinically evident symptoms)
i. Aspirin 81–325 mg daily after endarterectomy
ii. Dual antiplatelet therapy should be given for 3 months if stent placement is required, followed
by one antiplatelet agent thereafter.
h. Control of diabetes mellitus
i. Identification and treatment of inherited or acquired hypercoagulable states

ACCP Updates in Therapeutics® 2022: Critical Care Pharmacy Preparatory Review and Recertification Course

107
Neurocritical Care

VIII.  INTRACEREBRAL HEMORRHAGE

A. Epidemiology. Around 50,000 cases in the United States annually


1. Diagnosis/pathogenesis
a. Neurologic examination
b. Vital signs
c. NIH Stroke Scale and/or GCS score
2. Imaging and other tests
a. CT or MRI scan of the brain
b. CT angiography or contrast-enhanced CT (to help identify patients at risk of hematoma expansion
and to evaluate for underlying structural lesions)
c. Medication history to identify agents that might produce coagulopathy
d. Laboratory tests
e. Blood glucose
f. INR
g. CBC

B. Causes
1. Chronic/poorly treated hypertension
2. Oral anticoagulant use
3. Cocaine/other stimulant use
4. Ischemic stroke with hemorrhagic transformation
5. Chronic alcohol intake
6. Brain tumor
7. Arteriovenous malformation
8. Amyloid angiopathy

C. Clinical Impact – Death or major disability occurs in around 50% of patients.

D. Treatment Considerations (Stroke 2015;46:2032-60)


1. Coagulopathy reversal
2. Prompt reversal is necessary. Reversal of laboratory values does not mean that hemostasis will occur in
all cases, especially with the newer oral anticoagulants.
3. rFVIIa is not recommended for reversal of anticoagulation because it only replaces one clotting factor
and it does not appear to be effective for dabigatran reversal. However, if no other reversal agents are
available, rFVIIa may be an option for patients who cannot accept human blood products.
4. Reversal of antiplatelet agents is somewhat controversial.
a. Two prospective studies suggest that platelet transfusion for patients taking antiplatelet agents with
a new ICH is harmful (Lancet 2016;387:2605-13; J Neurosurgery 2013;118:94-103).
i. Both studies included patients with a small, stable ICH. Primarily basal ganglia hemorrhages,
which are traditionally nonoperative
ii. Patients receiving dual antiplatelet therapy were underrepresented in both studies.
b. Routine administration of platelet transfusion for any patient with ICH who took antiplatelet agents
before admission is not advisable.
c. Platelet transfusion is likely still indicated for patients who:
i. Have an urgent need for surgery or a ventriculostomy and have abnormal platelet function tests
(if available)
ii. Have a large ICH, receiving antiplatelet agents and requiring neurosurgery
iii. Have acute neurologic decline with an intracranial hemorrhage

ACCP Updates in Therapeutics® 2022: Critical Care Pharmacy Preparatory Review and Recertification Course

108
Neurocritical Care

Table 11. Anticoagulant Reversal Options


Anticoagulant Reversal Agent and Dose Adverse Effects
4F-PCC 25 units/kg (INR < 4), 35 units/kg (INR
Thrombosis, anaphylactoid reaction
4–6), or 50 units/kg (INR > 6) + vitamin K 10 mg
Warfarin (vitamin K), pulmonary edema, or
IV
transfusion-related reaction (FFP)
Alternative: FFP 10–15 mL/kg + vitamin K 10 mg IV
Andexanet: Rebound anti-Xa thera-
Andexanet alfa (Andexxa) (see dosing information
peutic concentrations within 2 hr of
PO factor Xa that follows)
infusion completion
inhibitorsa or 4F-PCC 25–50 units/kg or activated 4F-PCC
Thrombosis; limited data for reversal,
(FEIBA) 25–50 units/kg
especially with FEIBA
Idarucizumab (Praxbind) 5 g IV × 1; repeat dose may
Idarucizumab approved in late 2015 –
be necessary in patients with high DTI exposure or
antibody specifically for dabigatran
poor renal function
PO DTIsa Thrombosis; limited data analyses
If hemostasis is not achieved, consider 4F-PCC
show minimal efficacy for reversal
25–50 units/kg or activated 4F-PCC (FEIBA)
with PCC and FEIBA
25–50 units/kg
Platelet transfusion infusion (usefulness is uncertain
Pulmonary edema or transfusion-
PO antiplatelets – see NCS guidelines)
related reaction
Consider desmopressin 0.4 mcg/kg IV
Unfractionated Protamine 1 mg (max 50 mg) for each 100 units of
Hypotension, hypersensitivity
heparin heparin (infused within the past 2–3 hr)
Low-molecular- Protamine 1 mg (max 50 mg) for each 1 mg of enoxa-
Hypotension, hypersensitivity
weight heparins parin (within 8 hr of last dose)
a
Reversal options have not been tested in patients with ICH and have had varying degrees of coagulopathy reversal in experimental animal and human models.
DTI = direct thrombin inhibitor; FEIBA = factor eight inhibitor bypassing activity; FFP = fresh frozen plasma; 4F-PCC = 4-factor prothrombin complex concentrate.

Table 12. Andexanet Alfa Dosing Based on Factor Xa Inhibitor Dose


Factor Xa Inhibitor Last Dose < 8 hr or Unknown Since Last Dose ≥ 8 hr Since Last Dose
≤ 5 mg Low dose Low dose
Apixaban
> 5 mg or unknown High dose Low dose
≤ 10 mg Low dose Low dose
Rivaroxaban
> 10 mg or unknown High dose Low dose
Low dose = 400 mg at a target rate of 30 mg/min, then 4 mg/min for up to 120 min; high dose = 800 mg at a target rate of 30 mg/min, then 8 mg/min for up to 120 min.

E. Blood Pressure Management


1. Prompt control of blood pressure is essential.
2. Historically, caution may have been used in rapidly reducing blood pressure in patients with chronic
hypertension because of concerns regarding accommodations in cerebral autoregulation.
3. However, there may be some benefit of rapidly reducing the blood pressure to approximately 140-150
mm Hg (and thus reducing the risk of hemorrhage expansion). This may outweigh concern for cerebral
autoregulation issues and potential for ischemia, but patients with initial SBP >180 mm Hg may have a
greater risk of renal dysfunction.
4. INTERACT-2 – Large, prospective, randomized trial that compared blood pressure control within 1
hour (N Engl J Med 2013;368:2355-65). SBP less than 140 mm Hg was as safe and effective as SBP less
than 180 mm Hg (and may have improved functional outcomes).

ACCP Updates in Therapeutics® 2022: Critical Care Pharmacy Preparatory Review and Recertification Course

109
Neurocritical Care

5. ATACH-2 – Large, prospective, randomized trial comparing blood pressure control within 4½ hours in
patients with SBP >180 mm Hg. Targeting an SBP goal of 110–139 mm Hg was as effective as targeting
an SBP goal of 140–179 mm Hg but was associated with increased renal adverse events at 3 months.
(Mean minimum SBP in the first 2 hours after randomization was 128.9 mm Hg in the intensive-
treatment group and 141.1 mm Hg in the standard-treatment group.) (N Engl J Med 2016;375:1033-43).
6. Evidence also suggests that blood pressure fluctuations in the first 4–6 hours post-ICH are associated
with poor functional outcomes (Stroke 2018;49:348-54).

F. Seizure Prophylaxis. Use of anticonvulsants for seizure prophylaxis is not indicated after ICH.

G. VTE prophylaxis should be initiated about 24 hours after symptom onset or surgical evacuation.

Patient Case

Questions 3 and 4 pertain to the following case.


A 61-year-old man is admitted with acute onset of difficulty speaking, confusion, and right-sided weakness.
His NIH Stroke Scale score is 20. A head CT scan reveals a right parietal intracerebral hemorrhage (ICH).
The patient’s home medications include hydroxychloroquine, ibuprofen as needed, warfarin, amlodipine, and
donepezil. His medical history includes a deep venous thrombosis (1 year ago), hypertension, early dementia,
and arthritis. Serum laboratory values include Na 140 mEq/L, K 3.6 mEq/L, BUN 27 mg/dL, SCr 1.8 mg/dL,
glucose 289 mg/dL, Hct 36.7%, Plt 245,000/mm3, and INR 6.8. His vital signs include blood pressure 163/101
mm Hg, heart rate 99 beats/minute, Sao2 97%, and respiratory rate 20 breaths/minute.

3. Which is the most appropriate initial therapy in addition to vitamin K for this patient’s care?
A. Reinitiate amlodipine.
B. Give 4-factor prothrombin complex concentrate (4F-PCC) 50 units/kg intravenously × 1.
C. Give 6-pack infusion of platelets.
D. Give recombinant factor VIIa (rFVIIa) 90 mcg/kg intravenously × 1.

4. For this 61-year-old patient with ICH, which is the most appropriate initial antihypertensive therapy?
A. Clevidipine 1-mg/hour infusion to keep SBP 180 - 220 mm Hg.
B. Nicardipine 5-mg/hour infusion to keep SBP 140 - 150 mm Hg.
C. Labetalol 10 mg intravenously as needed to keep SBP 160 - 200 mm Hg.
D. Esmolol 50-mcg/kg/minute infusion to keep SBP 120 - 130 mm Hg.

IX.  SUBARACHNOID HEMORRHAGE

A. Epidemiology
1. Occurs in around 15 of 100,000 people in the United States
2. 60%–70% female, typical age 40–60 years

B. Diagnosis/Pathogenesis
1. Serial neurologic examination
2. Vital signs
3. NIH Stroke Scale and/or GCS score

ACCP Updates in Therapeutics® 2022: Critical Care Pharmacy Preparatory Review and Recertification Course

110
Neurocritical Care

Table 13. SAH Severity Scale Scores


Score Range Comments
Hunt and Hess 0 (no symptoms) to 5 (moribund) Best correlated with risk of mortality
World Federation
Integrates risk of mortality and
of Neurological 1 (GCS score 15, no deficit) to 5 (GCS score 3–6)
motor dysfunction
Surgeons
1 (no blood visualized) to 4 (diffuse SAH, ICH, Best correlated with risk of
Fisher
or intraventricular hemorrhage present) vasospasm

C. Imaging (Stroke 2009;40:994-1025)


1. CT scan of brain
2. Lumbar puncture when CT scan of brain is negative for blood
3. Digital subtraction (“conventional”) angiography
4. May use CT angiography or magnetic resonance angiography if conventional angiography is unavailable
5. Transcranial Doppler, often daily during peak vasospasm risk period

D. Medication History – To identify agents that might produce coagulopathy or hypertension

E. Laboratory and Other Tests


1. INR
2. CBC
3. Troponin
4. ECG (electrocardiogram)
5. Echocardiogram

F. Causes – Typically caused by cerebral aneurysm. Modifiable risk factors for SAH:
1. Hypertension
2. Smoking
3. Illicit drug use

G. Clinical Impact
1. Sudden death: Around 20% of patients die before hospitalization.
2. Vasospasm and delayed ischemic neurologic deficits
a. Presence of blood in subarachnoid space elicits a chemical meningitis-type inflammatory response
and results in hemolysis of subarachnoid blood.
b. Vasospasm (persistent vasoconstriction) occurs, reducing distal cerebral blood flow.
i. Typical course is 3–14 days.
ii. Vasospasm risk peaks at around 7–10 days.
c. Several mechanisms of pathogenesis
i. Inflammatory infiltration
ii. Endothelin activation
iii. Liberation of hemoglobin results in the scavenging of nitric oxide.
d. Vasospasm is one of the main factors resulting in death or disability after an acute SAH, aside from
initial ictus.

ACCP Updates in Therapeutics® 2022: Critical Care Pharmacy Preparatory Review and Recertification Course

111
Neurocritical Care

H. Treatment Considerations (Stroke 2012;43:1711-37)


1. Surgical management to prevent rebleeding and ‘secure’ aneurysm
a. Craniotomy and aneurysm clipping
b. Endovascular aneurysm coiling
2. Agents for preventing vasospasm or delayed ischemic neurologic deficits
a. Craniotomy and aneurysm clipping
b. Endovascular aneurysm coiling
3. Nimodipine (Br Med J 1989;298:636-42)
a. Lipophilic dihydropyridine calcium channel blocker
b. “Cerebrovascular-specific”
c. 60 mg orally or by tube every 4 hours × 21 days (dose reduction 30 mg every 2 hours may be
considered if 60 mg dose intolerable): The clinical impact of dose reducing or discontinuing
nimodipine because of hypotension is not well understood but may be associated with unfavorable
outcomes (Neurocrit Care 2016;25:29-39).
d. Only U.S. Food and Drug Administration (FDA) label-approved medication to reduce delayed
ischemic neurologic deficits associated with SAH
e. Clinical trials did not show a large effect of nimodipine on the occurrence of vasospasm (though
the effects of delayed ischemic neurologic deficits were significantly less).
f. Possibly neuroprotective
g. Administration issues in patients who require enteral doses – Black box warning against inadvertent
intravenous administration
h. Nursing should be prohibited from extracting the gel inside the nimodipine capsule for bedside
administration (may increase the risk of inadvertent intravenous administration, incomplete
extraction from the capsule) (Neurocrit Care 2015;22:89-92).
i. There is a commercially available liquid product that is for use in patients with swallowing
difficulty or feeding tubes. Pharmacy compounding of nimodipine syringes from liquid capsules
is not recommended due to risk of medication dosing and administration errors.
4. Antifibrinolytic agents
a. Use of antifibrinolytic agents such as aminocaproic acid or tranexamic acid has been evaluated
after SAH.
b. Older data analyses before the advent of endovascular interventions suggested that rebleeding is
less common with these agents (primarily aminocaproic acid) but that stroke may be more common.
c. More recent data analyses suggest that a short infusion (less than 72 hours) reduces rebleeding but
probably does not affect long-term outcomes.
i. The 2012 American Heart Association SAH guidelines moderately support this strategy in
early SAH management (Stroke 2012;43:1711-37).
ii. For example, use of 1000 mg of tranexamic acid intravenously every 6 hours until the aneurysm
is secured
5. Statins
a. Preservation of nitric oxide balance as heme is liberated during SAH hemolysis.
b. Phase II data with pravastatin and simvastatin
c. Phase III trial (STASH) showed no benefit of applying statins in aneurysmal SAH (Lancet Neurol
2014;13:666-75).
d. Abrupt withdrawal of statins in patients who were taking statins before SAH may result in a
withdrawal effect and increase the risk of vasospasm.
6. Other studied agents
a. Magnesium – No usefulness in attaining magnesium concentrations 3–4 mEq/L. Maintaining
magnesium at normal concentrations (i.e., preventing hypomagnesemia) is advisable.

ACCP Updates in Therapeutics® 2022: Critical Care Pharmacy Preparatory Review and Recertification Course

112
Neurocritical Care

b. Clazosentan – No usefulness in blocking endothelin-1


c. Albumin – Not beneficial and may be associated with an increase in pulmonary edema
7. Treatment of vasospasm
a. Intra-arterial therapies (see text that follows)
b. Triple-H therapy (hypertension, hypervolemia, hemodilution)
i. No longer recommended in the traditional format. Euvolemia is better than hypervolemia –
Similar outcomes in clinical trials, less pulmonary edema
ii. Hemodilution has not been shown beneficial.
c. Hyperperfusion therapy
i. Better descriptor for the goal of therapy
ii. Vasospasm causes distal vasoconstriction to the point of ischemia.
iii. Maximizing cerebral blood flow mitigates ischemia.
d. Contemporary therapy includes:
i. Euvolemia
ii. Vasopressors (blood pressure targets are ill defined, but are also typically patient- and
symptom-dependent)
iii. Inotropes (milrinone)
iv. Superselective intra-arterial vasodilators (e.g., verapamil, nicardipine)
8. Headache
a. “Worst headache of my life” is the typical chief concern for an aneurysmal SAH.
b. Multimodality treatment will be needed to improve patient satisfaction during hospital admission
(which may be up to 14–21 days).
c. Optimal choice of agents is not well known.
i. Common analgesics such as acetaminophen, nonsteroidal anti-inflammatory drugs (if
aneurysm is secured)
ii. Other therapies such as magnesium, corticosteroids, or gabapentin may be helpful.
iii. Agents that contain caffeine are often used for headache but may not be appropriate, given the
risk of cerebral vasospasm after SAH.
iv. Opioids may be needed, but use should be limited to avoid sedation and inability to monitor
neurologic examination.
v. Nonpharmacologic therapies like warm compresses on the back of the neck, minimization of
noise, and minimization of bright light may also be useful.
9. Seizure prophylaxis
a. Use of anticonvulsants for seizure prophylaxis is controversial after SAH.
b. One small study suggests that 3 days of prophylactic phenytoin after aneurysm clipping helps
reduce the seizure rate.
c. SAH guidelines permit prophylaxis, according to this study.
d. Evidence suggests that phenytoin use in aneurysmal SAH is associated with worse neurocognitive
outcomes and increased in-hospital complications.
10. Role of nicotine replacement therapy (NRT) – Smoking is a common risk factor for developing cerebral
aneurysms.
a. Nicotine administration to naive users causes vasoconstriction, whereas administration to
heavy users causes little reaction. Nicotine withdrawal in heavy users causes acute, temporary
vasodilation, which may lead to headaches.
b. Seder et al.: Neurocrit Care 2011;14:77-83: Evaluated safety of NRT in active cigarette smokers
admitted with an aneurysmal SAH and included 128 who received NRT and 106 who did not.
Despite its vasoactive properties, NRT administered among active smokers with acute aneurysmal
SAH appeared safe, with similar rates of vasospasm and delayed cerebral ischemia and a slightly
higher rate of seizures.

ACCP Updates in Therapeutics® 2022: Critical Care Pharmacy Preparatory Review and Recertification Course

113
Neurocritical Care

c. Panos et al.: Am J Health Syst Pharm 2010;67:1357-61: No difference in unfavorable discharge


disposition among neurosurgery ICU patients who were smokers treated with NRT (n=114), smokers
not treated with NRT (n=113), and nonsmokers not treated with NRT (n=113). Primary admitting
diagnosis for neurosurgery patients included SAH, ICH, other trauma, and elective neurosurgery.

X.  INTERVENTIONAL ENDOVASCULAR MANAGEMENT

A. Intra-arterial Therapies (Pharmacotherapy 2010;30:405-17)


1. Administration technique -- typically administered during cerebral angiography
2. Catheter advanced to vessels with lesion/vasospasm, and drug is infused locally (“superselective
infusion”)

B. Vasodilators
1. Calcium channel blockers
a. Typically used for cerebral vasospasm associated with SAH
b. Direct, local infusion typically results in immediate vasodilation.
c. Usually effective in proximal and distal vessels
2. A trial is ongoing to determine the optimal intra-arterial drug treatment regimen (nicardipine vs.
verapamil vs. nicardipine plus verapamil plus nitroglycerin) for arterial lumen restoration post-cerebral
vasospasm after aneurysmal SAH (Intra-arterial Vasospasm Trial at https://clinicaltrials.gov/ct2/show/
record/NCT01996436).

Table 14. Typical Agents for Intra-arterial Use for Cerebral Vasospasm
Agent Typical Dose Adverse Effects
2–10 mg (usually 1–2 mg), Systemic hypotension
Nicardipine
up to 5 mg/vessel Increased ICP
Systemic hypotension
Verapamil 1–8 mg (usually 1–2 mg) Bradycardia
Increased ICP
Milrinone 8 mg Systemic hypotension
Increased ICP
Systemic hypotension
Papaverine 150–600 mg
Neurologic deterioration
Rebound vasospasm
ICP = intracranial pressure.

C. Intra-arterial Thrombolysis
1. Most often used in patients with ischemic stroke
2. Limited evidence to support combining with intravenous alteplase as a standard of care
3. Current roles
a. Combination with mechanical thrombectomy
b. Rescue therapy in patients having received intravenous alteplase
c. Large hemispheric infarction
i. Dose is not well defined.
ii. Typically applied until thrombus has resolved
iii. Alteplase less than 20 mg

ACCP Updates in Therapeutics® 2022: Critical Care Pharmacy Preparatory Review and Recertification Course

114
Neurocritical Care

Patient Case

Questions 5 and 6 pertain to the following case.


A 49-year-old woman presents to an urgent treatment center with the “worst headache of her life.” She is
sent to your emergency department, where a head CT scan reveals a diffuse SAH. The patient takes no home
medications and has an insignificant medical history other than a 20 pack-year history of smoking.

5. Which therapy is most appropriate to prevent ischemic complications from SAH?


A. Nimodipine for 21 days.
B. Euvolemia and permissive hypertension for 14 days.
C. Simvastatin for 14 days.
D. Aminocaproic acid infusion for 48 hours.

6. On hospital day 5, the patient has reduced alertness, and her GCS score decreases by 2 points. The digital
subtraction angiography suggests cerebral vasospasm. Which treatment modality is best to initiate first?
A. Norepinephrine 0.05 mcg/kg/minute to increase MAP to 90 mm Hg.
B. One unit of packed RBCs to increase Hgb to 10 g/dL.
C. 3% sodium chloride boluses to increase central venous pressure to 14 mm Hg.
D. Milrinone 0.375-mcg/kg/minute infusion to increase cardiac index to 5 L/minute/m2.

D. Stent Deployment and Antiplatelet Agents


1. Intracranial stents are often deployed in place of coils or to support coils for complex aneurysms.
2. Intracranial circulation is different from coronary circulation.
a. Blood vessels are generally smaller and more tortuous.
b. Flow rate is lower.
c. Epithelialization of stent takes longer.
3. Dual antiplatelet therapy is typically used around the time of stent placement.
a. Clopidogrel (a prodrug) plus aspirin
b. Current evidence suggests therapy for up to 3 months (not 4 weeks like after percutaneous coronary
intervention), followed by aspirin monotherapy thereafter.
c. Platelet testing may be necessary in some individuals to evaluate their pharmacogenetic response
to clopidogrel.
d. No gold standard for platelet reactivity testing in this setting. VerifyNow is commonly used and
measures platelet reactivity units (PRU) for aspirin and P2Y12 inhibitors.
e. Some patients may not respond to clopidogrel (up to 30%), and additional loading doses and
increased maintenance doses are not effective.
f. Ticagrelor may play a role in patients with a variable response to clopidogrel (must be used in
combination with aspirin less than 100 mg).
g. Prasugrel (a prodrug) is not recommended (unless absolutely necessary) in patients with high
on-treatment platelet reactivity (e.g. clopidogrel non-responders) because of warnings about use
in patients with a history of stroke – Increased bleeding. Reduced doses (2.5–5 mg) have been
reported with some success in preventing thrombosis without excessive bleeding.

ACCP Updates in Therapeutics® 2022: Critical Care Pharmacy Preparatory Review and Recertification Course

115
Neurocritical Care

Patient Case

Questions 7 and 8 pertain to the following case.


A 52-year-old woman is admitted to your ICU after a single-vehicle crash. She has many orthopedic injuries to
her lower extremities and a subdural hematoma, which will require an emergency craniotomy for evacuation.
Her home medications include metoprolol, rivaroxaban 10 mg (last dose 5 hours ago), lisinopril, and atorvas-
tatin. Her medical history is significant for atrial fibrillation. Laboratory values on admission were notable for
Hct 31.2% and Plt 577,000/mm3.

7. Which laboratory test would best evaluate the extent of anticoagulation from rivaroxaban?
A. INR.
B. Anti-Xa activity concentration.
C. Activated partial thromboplastin time.
D. VerifyNow PRU test measurement.

8. Which therapy would be most appropriate to reverse rivaroxaban before this patient’s emergency craniotomy?
A. 4F-PCC 50 units/kg intravenously × 1.
B. Andexanet 200 mg intravenously x 1.
C. Fresh frozen plasma 15 mL/kg intravenously × 1.
D. rFVIIa 90 mcg/kg intravenously × 1.

XI.  ACUTE SPINAL CORD INJURY (NEUROSURGERY 2013;60(SUPPL 1):82-91)

A. Epidemiology – Annual incidence of 15–40 cases per 1 million people in the United States

B. Diagnosis/Pathogenesis
1. Diagnostic tests
2. Neurologic examination

C. Imaging (Neurosurgery 2013;60(suppl 1):82-91)


1. CT scan of spine
2. Many views of spine radiography are necessary when a CT scan is unavailable.

D. Causes
1. 40%–50% are caused by motor vehicle collisions.
2. Falls (20%), violence (14%), recreational and work activities

E. Clinical Impact
1. Mortality
a. 50%–75% at the time of injury
b. Hospital mortality 4.4%–16%
2. Morbidity
a. Paralysis and loss of sensation
b. Spasticity

ACCP Updates in Therapeutics® 2022: Critical Care Pharmacy Preparatory Review and Recertification Course

116
Neurocritical Care

c. Orthostatic hypotension
d. Autonomic dysreflexia
e. VTE
f. Decubitus ulcers
g. Respiratory insufficiency
h. Bowel and bladder dysfunction
i. Sexual dysfunction
j. Treatment considerations
k. Neurogenic shock
i. Hypotension often occurs after injury (50%–90% of cervical spine injuries).
ii. May be associated with malperfusion of the spinal cord and worsened outcomes
iii. Etiology of shock is decreased sympathetic nervous system outflow. Continues to be
counterbalanced by parasympathetic outflow, which is not affected by SCI
iv. Results in hypotension and bradycardia

F. Blood Pressure Management


1. Typical recommendations after an acute SCI are to maintain MAP 85–90 mm Hg for 7 days to ensure
adequate spinal perfusion.
2. Little high-quality evidence supports this recommendation, but the recommendation is included in the
SCI guidelines as a treatment option.
3. Often requires judicious fluid resuscitation and vasopressor support
4. Persistent hypotension may be treated with fludrocortisone or midodrine.
5. Persistent bradycardia may be treated with pseudoephedrine or low-dose theophylline.
6. Droxidopa, an enteral analog of norepinephrine, may be useful for blood pressure augmentation in this
setting.

G. VTE Prophylaxis
1. Occurs in 80%–100% of patients without pharmacologic prophylaxis
2. Low-molecular-weight heparins are the drugs of choice for prophylaxis and should be initiated within
the first 36 hours post-injury.
3. Duration of prophylaxis is typically about 8 weeks.

H. Role of High-Dose Methylprednisolone (Neurosurgery 2013;60(suppl 1)82-91; J Neurosurg 1998;89:699-


706; JAMA 1997;277:1597-604; J Neurosurg 1992;76:23-31; N Engl J Med 1990;322:1405-11)
1. Controversial topic related to NASCIS-II and NASCIS-III trials
2. Methylprednisolone 30 mg/kg intravenously × 1, followed by 5.4 mg/kg/hour within 8 hours of injury
for 24 or 48 hours
3. Both trials suggested a modest benefit in the first 6 weeks or 6 months (which often did not persist at 1
year) and a modest risk (primarily related to infection). Current guidelines do not support administering
high-dose methylprednisolone.
4. NASCIS-II split enrolled population in half (those who received the drug before the median time to
administration [8 hours] and those who did not).
a. Subgroup analysis may not have been powered to show benefit.
b. Reported motor and sensory scores from one side of the body, not both. The investigators later said
there was no difference but have not allowed others to examine the raw data.
5. Consistently showed risk (GI bleeding, infection) and inconsistently showed benefit
6. Potential treatment effects may have been caused by early surgery or additional benefit of high-dose
methylprednisolone therapy in combination with early surgery.

ACCP Updates in Therapeutics® 2022: Critical Care Pharmacy Preparatory Review and Recertification Course

117
Neurocritical Care

7. NASCIS-III used a functional independence measure (FIM) score to show how improvement in muscle
strength might translate to improved outcomes. Failed to show a difference in FIM score
8. If a practitioner does choose to use high-dose steroids in SCI:
a. Must use methylprednisolone; no other steroids
b. Must use NASCIS-II or NASCIS-III dosing
c. Must give within 8 hours of injury

XII.  BRAIN TUMORS

A. Epidemiology
1. Primary brain tumors (from brain cells such as meninges and neural tissues)
2. 17,000–20,000 cases per year in the United States
3. Glioblastoma
4. Meningioma
5. Pituitary adenoma
6. Astrocytoma
7. Metastases – Common neoplasms that result in spread to brain
a. Lung (40%–50%)
b. Breast (15%–20%)
c. Melanoma (5%–10%)
d. Colon (4%–6%)
e. Renal cell carcinoma
f. CNS lymphoma

B. Diagnosis/Pathogenesis – Diagnostic tests


1. Contrast-enhanced MRI is the most common test.
2. Biopsy is often necessary to reveal the specific histology.

C. Clinical Impact – Mortality is often high, depending on the type and grade of the tumor.

D. Treatment Considerations
1. Corticosteroids (and adverse drug reactions) for brain edema
a. Dexamethasone commonly used for vasogenic edema associated with tumor
i. Reduces peritumoral edema and symptoms associated with increased ICP. Temporarily
reduces symptoms (neurologic dysfunction, seizures, headache)
ii. Dose is commonly 4 mg intravenously every 6 hours.
b. May use other corticosteroids at comparable doses
c. Use of acid-suppressive agents may help with concomitant steroid use to reduce the risk of GI
complications (Wien Med Wochenschr 1988;138:97-101).
d. Consideration for glycemic control and gastric protection with prolonged use
e. Induction of phenytoin metabolism (because of increased metabolic rate)
f. Metabolism induced by phenytoin (because of increased cytochrome P450 [CYP] activity)
2. VTE prophylaxis and treatment
a. High risk of VTE
b. Consider using combination pharmacologic/mechanical prophylaxis.
c. Enoxaparin is superior to warfarin for the treatment of VTE in oncology patients.

ACCP Updates in Therapeutics® 2022: Critical Care Pharmacy Preparatory Review and Recertification Course

118
Neurocritical Care

3. Seizure prophylaxis
a. Not typically indicated
b. Around 50% of patients with primary brain tumor present with seizure, which must be treated with
anticonvulsant medications.
c. Phenytoin, carbamazepine, and levetiracetam are often recommended; however, caution should
be used with agents that have been associated with Stevens-Johnson syndrome/toxic epidermal
necrolysis (e.g., phenytoin, carbamazepine) in patients who also require radiation therapy.
d. Hepatic CYP enzyme-inducing agents, including phenytoin, should be used with caution because
of possible drug interactions with chemotherapy.

XIII.  CRITICAL ILLNESS POLYNEUROPATHY

A. Epidemiology
1. Exact incidence is unknown because of inconsistent monitoring and diagnosis.
2. May be as high as 60% in patients with acute respiratory distress syndrome, 77% in long ICU stay
(greater than 7 days), 80% in patients with multiorgan failure

B. Diagnosis/Pathogenesis – Diagnostic tests


1. Typically suspected when patients do not wean well from the ventilator or if their limbs are weak/flaccid
2. Electrophysiologic studies or muscle biopsy may provide a more precise diagnosis. Differential diagnosis
includes evaluation for critical illness myopathy, Guillain-Barré syndrome, electrolyte abnormalities

C. Causes
1. The cause of critical illness polyneuropathy is unknown, but several hypotheses exist.
a. Mitochondrial dysfunction in critical illness may cause energy stress in vulnerable neurons.
b. Microcirculatory ischemia
c. Protein catabolism in severe critical illness/immobility may cause muscle wasting.
2. Often associated with:
a. Sepsis
b. Multiorgan dysfunction
c. Hyperglycemia
d. Renal failure
e. Neuromuscular blockade
f. Duration of vasopressor or corticosteroid therapy
g. Duration of ICU stay

D. Clinical Impact
1. Limb and diaphragm weakness may persist for weeks to months.
2. About 33% of patients with critical illness polyneuropathy ultimately cannot independently ambulate
or breathe.

E. Treatment Considerations
1. No specific treatments have been shown effective.
2. Intravenous immunoglobulin may play a role (Lancet Neurol 2008;7:136-44).
3. Intensive glycemic control may reduce critical illness neuropathy.
4. Passive mobilization/early physical therapy in the ICU
5. Daily awakening/less time on the ventilator
6. Limiting risk factors as much as possible

ACCP Updates in Therapeutics® 2022: Critical Care Pharmacy Preparatory Review and Recertification Course

119
Neurocritical Care

XIV.  GUILLAIN-BARRÉ SYNDROME

A. Epidemiology
1. 1.11 cases per 100,000 person-years
2. Men > women (almost 2:1)

B. Diagnosis/Pathogenesis – Diagnostic tests


1. Bilateral symmetric progressive weakness of limbs
2. Generalized hyporeflexia or areflexia
3. Nerve conduction studies may provide a more precise diagnosis.

C. Causes
1. Typically associated with Campylobacter jejuni infection. Also associated with Epstein-Barr virus,
varicella-zoster, and Mycoplasma pneumoniae infections
2. Swine flu vaccine in 1976 caused increased risk of Guillain-Barré syndrome.

D. Clinical Impact
1. Progressive weakness over 3–4 weeks
2. 20% of patients remain severely disabled.
3. Mortality rate around 5%
4. Respiratory failure
5. Autonomic dysfunction resulting in arrhythmia, hypertension, hypotension
6. Neuropathic pain

E. Treatment Considerations (N Engl J Med 1992;326:1123-9)


1. Intravenous immunoglobulin versus plasma exchange
2. Therapies are essentially equivalent.
3. Plasma exchange: Five treatments over 2 weeks
4. Intravenous immunoglobulin: 0.4 g/kg intravenously daily × 5 days
5. Combination of therapies no better than single therapy alone
6. Benefit of repeated treatments is unclear.
7. Steroids are not particularly effective.

F. Supportive Care
1. VTE prophylaxis is imperative.
2. Careful ventilation strategies to minimize barotrauma and prevent pneumonia
3. Dysphagia is common, so enteral feeding access is necessary in most cases.
4. Neuropathic pain is common; use opiates with caution to avoid respiratory depression.
5. Euvolemia to minimize autonomic instability

XV.  MYASTHENIA CRISIS

A. Epidemiology
1. Annual incidence of myasthenia gravis is 1 or 2 per 100,000.
2. 15%–20% of patients with myasthenia gravis will develop myasthenia crisis within the first year of
illness.

ACCP Updates in Therapeutics® 2022: Critical Care Pharmacy Preparatory Review and Recertification Course

120
Neurocritical Care

B. Diagnosis/Pathogenesis
1. Patients with myasthenia crisis typically present with respiratory failure caused by muscle weakness.
2. Autoimmune disease targeting acetylcholine receptors at the neuromuscular junction
3. Myasthenia crisis is usually preceded by a predisposing factor.
a. Respiratory infection
b. Emotional stress
c. Aspiration
d. Changes in myasthenia gravis medication regimen
e. Addition of medications that may oppose acetylcholine effects at the neuromuscular junction and
exacerbate myasthenia gravis symptoms (Box 3)
f. Other physiologic stress (trauma, surgery)

Box 3. Pharmacologic Agents Associated with Myasthenia Crisis


Aminoglycosides Neuromuscular blockers
Fluoroquinolones Magnesium
Tetracyclines β-Blockers
Quinidine Verapamil
Procainamide

C. Treatment Considerations
1. Intravenous immunoglobulin versus plasmapheresis
2. Intravenous immunoglobulin 0.4 g/kg/day × 3–5 days
3. Plasmapheresis 20–25 mL/kg plasma × 5 exchanges every other day × 10 days
4. Similarly effective; can choose according to patient risk factors, etc.
5. Corticosteroids moderately effective (e.g., prednisone 60–100 mg/d for 2–4 weeks, followed by slow
taper)

D. Supportive Care – Consider discontinuing cholinergic therapies while the patient is acutely ill. May increase
pulmonary secretions and complicate ventilator/ICU management

XVI.  SEROTONIN SYNDROME (NEUROCRIT CARE 2014;21:108-13)

A. Presentation
1. Autonomic hyperactivity (hypertension, tachycardia)
2. Mental status changes
3. Hyperthermia – Diaphoresis
4. Seizures
5. Neuromuscular abnormalities
6. Rigidity
7. Hyperreflexia and clonus
8. Shivering
9. Diarrhea

ACCP Updates in Therapeutics® 2022: Critical Care Pharmacy Preparatory Review and Recertification Course

121
Neurocritical Care

Table 15. Medications Associated with Serotonin Syndrome


SSRIs (sertraline, fluoxetine, citalopram, paroxetine)
SNRIs (duloxetine, venlafaxine, desvenlafaxine)
Trazodone
Antidepressants
Nefazodone
Buspirone
MAOIs (phenelzine)
Anticonvulsants Valproic acid
Migraine agents Sumatriptan
Antibiotics Linezolid
Cough suppressants Dextromethorphan
St. John’s wort
Herbal products Tryptophan
Ginseng
Ondansetron
Antiemetics
Metoclopramide
Meperidine
Analgesics Fentanyl
Tramadol
MAOI = monoamine oxidase inhibitor; SNRI = serotonin-norepinephrine reuptake inhibitor; SSRI = selective serotonin reuptake inhibitor.

B. Treatment Considerations
1. Removal of precipitating drugs/factors – Mild symptoms may resolve in 24–72 hours; if caused by
antidepressants, may take weeks to resolve
2. Control of agitation, seizures, and rigidity – Benzodiazepines
3. Control of autonomic hyperactivity – Hypotension treatment with direct-acting sympathomimetics
4. Control of hyperthermia
a. Cooling blanket
b. Sedation, neuromuscular paralysis, intubation
c. Avoid succinylcholine.
5. Serotonin-2A antagonist blocks serotonin receptors implicated with serotonin syndrome.
a. Cyproheptadine 12–32 mg/24 hours by mouth or feeding tube. A 12-mg loading dose; then 2 mg
every 2 hours as symptoms continue
b. Chlorpromazine 50–100 mg intramuscularly

XVII.  NEUROLOGIC MONITORING DEVICES

A. ICP Monitors
1. Goal ICP is typically less than 22 mm Hg.
2. Catheters are typically inserted under sterile conditions at the bedside.
3. Antibiotic prophylaxis is not useful and should not be used routinely (Neurocrit Care 2016;24:61-81).
4. Antibiotic-coated catheters are available for patients at high risk of infection.

ACCP Updates in Therapeutics® 2022: Critical Care Pharmacy Preparatory Review and Recertification Course

122
Neurocritical Care

5. Temporary catheters
a. Used primarily in the ICU
b. Ventriculostomy (a.k.a. external ventricular drain) – Diagnostic and therapeutic
i. Catheter inserted into frontal horn of lateral ventricle
ii. Transduces ICP; should be calibrated to zero routinely
iii. Higher infection rate than with intraparenchymal catheter; insertion is more difficult
(particularly with brain swelling)
iv. Permits drainage of CSF and intraventricular hemorrhage
v. Permits intraventricular drug administration
c. Intraparenchymal catheter
i. Wire that sits in brain tissue
ii. Transduces ICP
iii. Low infection rate, fewer complications with insertion
iv. Cannot recalibrate the catheter, experience “drift” in ICP readings after prolonged use. May
not be entirely accurate for duration of use
d. Brain tissue oxygen monitor (Licox)
i. Intraparenchymal catheter
ii. Optimal location of placement is not well defined (injured vs. non-injured tissue).
iii. Transduces partial pressure of brain tissue oxygen
(a) Goal partial pressure of brain tissue oxygen is usually greater than 15%.
(b) Concept similar to Svo2 (mixed venous oxygen saturation) values systemically
iv. Low values reflect increased ICP or reduced oxygen delivery.
v. Typically, will be used in combination with other monitoring modalities
e. Subarachnoid bolt
i. Single-lumen screw inserted through a burr hole into the subarachnoid space
ii. Transduces ICP
iii. Associated with increased CNS infection
6. EEG
a. Scalp electrodes are placed externally.
b. Permits evaluation of cortical electrical activity
c. Standard of care for seizure monitoring
d. Newer, less cumbersome devices are currently available that may improve accessibility.
7. Transcranial Doppler
a. Ultrasound of intracranial vessels
b. Used in monitoring for cerebral blood flow velocity or vasospasm
c. Threshold values
i. 125 cm/second may suggest vasospasm.
ii. 200 cm/second typically suggests severe vasospasm.
iii. Lindegaard ratio: Ratio of target blood vessel (usually middle cerebral artery) to carotid
(internal carotid artery) blood velocity transcranial Doppler values – value of 3 or more
suggests vasospasm

B. Bispectral Index (BIS) Monitor


1. Scalp electrodes are placed externally.
2. Uses EEG information to derive a number; should not be substituted for an EEG in patients with seizures
3. BIS 0–100 (100 being completely wakeful)
4. Little correlation with BIS values and ICP control or extent of pharmacologic coma

ACCP Updates in Therapeutics® 2022: Critical Care Pharmacy Preparatory Review and Recertification Course

123
Neurocritical Care

REFERENCES

1. Achinger SG, Ayus JC. Treatment of hyponatremic 12. Bourdeaux C, Brown J. Sodium bicarbonate low-
encephalopathy in the critically ill. Crit Care Med ers intracranial pressure after traumatic brain
2017;45:1762-71. injury. Neurocrit Care 2010;13:24-8.
2. Adrogue HJ, Madias N. Hyponatremia. N Engl J 13. Bracken M, Shepard M, Collins W, et al. A ran-
Med 2000;342:1581-9. domized, controlled trial of methylprednisolone
3. Albers GW, Marks MP, Kemp S, et al. or naloxone in the treatment of acute spinal cord
Thrombectomy for stroke at 6 to 16 hours with injury: results of the Second National Acute Spinal
selection by perfusion imaging (DEFUSE-3). N Cord Injury Study (NASCIS-2). N Engl J Med
Engl J Med 2018;378:708-18. 1990;322:1405-11.
4. Alldredge BK, Gelb AM, Isaacs SM, et al. A com- 14. Bracken M, Shepard M, Collins W, et al.
parison of lorazepam, diazepam, and placebo for Methylprednisolone or naloxone treatment after
the treatment of out-of-hospital status epilepticus. acute spinal cord injury: 1-year follow up data –
N Engl J Med 2001;345:631-7. results of the Second National Spinal Cord Injury
5. Anadani M, Orabi M, Alawieh A, et al. Blood pres- Study. J Neurosurg 1992;76:23-31.
sure and outcome after mechanical thrombectomy 15. Bracken M, Shepard M, Holford T, et al.
with successful revascularization: a multicenter Administration of methylprednisolone for 24 or
study. Stroke 2019;50:2448-54. 48 hours or tirilazad mesylate for 48 hours in the
6. Anderson CS, Heeley E, Huang Y, et al. Rapid treatment of acute spinal cord injury: results of the
blood-pressure lowering in patients with acute Third National Acute Spinal Cord Injury Study.
intracerebral hemorrhage. N Engl J Med JAMA 1997;277:1597-604.
2013;368:2355-65. 16. Bracken M, Shepard M, Holford T, et al.
7. Baguley IJ, Perkes IE, Fernandez-Ortega JF, et Methylprednisolone or tirilazad mesylate admin-
al. Paroxysmal sympathetic hyperactivity after istration after acute spinal cord injury: 1-year
acquired brain injury: consensus on conceptual follow-up – results of the Third National Spinal
definition, nomenclature, and diagnostic criteria. J Cord Injury Randomized Controlled Trial. J
Neurotrauma 2014;31:1515-20. Neurosurg 1998;89:699-706.
8. Baharoglu MI, Cordonnier C, Salman RA, et al. 17. Brain Trauma Foundation (BTF). Management
Platelet transfusion versus standard care after of severe traumatic brain injury. J Neurotrauma
acute stroke due to spontaneous cerebral hem- 2007;24(suppl 1):S1-S95.
orrhage associated with antiplatelet therapy 18. Brophy GM, Bell R, Claassen J, et al. Guidelines
(PATCH): a randomized, open-label, phase 3 trial. for the evaluation and management of status epi-
Lancet 2016;387:2605-13. lepticus. Neurocrit Care 2012;17:3-23.
9. Barletta JF, Mangram AJ, Byrne M, et al. The 19. Campbell BC, Mitchell PJ, Kleinig TJ, et al.
importance of empiric antibiotic dosing in criti- Endovascular therapy for ischemic stroke with
cally ill trauma patients: are we under-dosing perfusion-imaging selection. N Engl J Med
based on augmented renal clearance and inaccu- 2015;372:1009-18.
rate renal clearance estimates? J Trauma Acute 20. Carney N, Totten AM, O’Reilly C, et al. Guidelines
Care Surg 2016;81:1115-21. for the Management of Severe Traumatic Brain
10. Barr J, Fraser GL, Puntillo K, et al. Clinical practice Injury, Fourth Edition. Neurosurgery 2017;80:6-15.
guidelines for the management of pain, agitation, 21. Chung PW, Kim JT, Sanossian N, et al.; FAST-
and delirium in adult patients in the intensive care MAG Investigators and Coordinators. Association
unit. Crit Care Med 2013;41:263-306. between hyperacute stage blood pressure vari-
11. Bederson JB, Connolly ES Jr, Batjer HH, et al. ability and outcome in patients with spontaneous
Guidelines for the management of aneurysmal sub- intracerebral hemorrhage. Stroke 2018;49:348-54.
arachnoid hemorrhage. Stroke 2009;40:994-1025. 22. Connolly ES Jr, Rabinstein AA, Carhuapoma JR,
et al. Guidelines for the management of aneurysmal
subarachnoid hemorrhage. Stroke 2012;43:1711-37.

ACCP Updates in Therapeutics® 2022: Critical Care Pharmacy Preparatory Review and Recertification Course

124
Neurocritical Care

23. Cook AM, Hatton-Kolpek J. Augmented renal treatment of ischemic stroke. N Engl J Med
clearance. Pharmacotherapy 2019;39:346-54. 2015;372:1019-30.
24. Cook AM, Mieure KD, Owen RD, et al. 36. Hacke W, Kaste M, Bluhmki E, et al. Thrombolysis
Intracerebroventricular administration of medica- with alteplase 3 to 4.5 hours after acute ischemic
tions. Pharmacotherapy 2009;29:832-45. stroke. N Engl J Med 2008;359:1317-29.
25. Cook AM, Morgan Jones G, Hawryluk GWJ, et 37. Hemphill JC III, Greenberg SM, Anderson CS, et
al. Guidelines for the acute treatment of cerebral al.; American Heart Association Stroke Council;
edema in neurocritical care patients. Neurocrit Council on Cardiovascular and Stroke Nursing;
Care 2020;32:647-66. Council on Clinical Cardiology. Guidelines for
26. Cook AM, Peppard A, Magnuson B. Nutrition the Management of Spontaneous Intracerebral
considerations in traumatic brain injury. Nutr Clin Hemorrhage: a guideline for healthcare profession-
Pract 2008;23:608-20. als from the American Heart Association/American
27. de Gans J, van de Beek D, et al. Dexamethasone Stroke Association. Stroke 2015;46:2032-60.
in adults with bacterial meningitis. N Engl J Med 38. Hirschl M. [Report of experience with stomach-
2002;347:1549-56. protective therapy in high-dosage corticosteroid
28. Demaerschalk BM, Kleindorfer DO, Adeoye treatment of patients with brain tumors]. Wien
OM, et al. Scientific rationale for the inclusion Med Wochenschr 1988;138:97-101.
and exclusion criteria for intravenous alteplase 39. Holbrook A, Schulman S, Witt DM, et al.
in acute ischemic stroke. A statement for health- Evidence-based management of anticoagulant
care professionals from the American Heart therapy: Antithrombotic Therapy and Prevention
Association/American Stroke Association. Stroke of Thrombosis, 9th ed: American College of Chest
2016;47:581-641. Physicians Evidence-Based Clinical Practice
29. Edwards P, Arango M, Balica L, et al. Final results Guidelines. Chest 2012;141(2 suppl):e152S-184S.
of MRC CRASH, a randomised placebo-con- 40. Hughes RAC, Donofrio P, Bril V, et al. Intravenous
trolled trial of intravenous corticosteroid in adults immune globulin (10% caprylate-chromatography
with head injury – outcomes at 6 months. Lancet purified) for the treatment of chronic inflamma-
2005;365:1957-9. tory demyelinating polyradiculoneuropathy (ICE
30. Ellison DH, Berl T. Clinical practice. The syn- study): a randomised placebo-controlled trial.
drome of inappropriate antidiuresis. N Engl J Med Lancet Neurol 2008;7:136-44.
2007;356:2064-72. 41. Human T, Cook AM, Anger B, et al. Treatment of
31. Farmakidis C, Pasnoor M, Dimachkie MM, et hyponatremia in patients with acute neurological
al. Treatment of myasthenia gravis. Neurol Clin injury. Neurocrit Care 2017;27:242-8.
2018;36:311-37. 42. Hurlbert RJ, Hadley MN, Walters BC, et al.
32. Fried HI, Nathan BR, Rowe AS, et al. The insertion Pharmacological therapy for acute spinal cord
and management of external ventricular drains: an injury. Neurosurgery 2013;72(suppl 2):93-105.
evidence-based consensus statement: a statement 43. Kelly DF, Goodale DB, Williams J, et al. Propofol
for healthcare professionals from the Neurocritical in the treatment of moderate and severe head
Care Society. Neurocrit Care 2016;24:61-81. injury: a randomized, prospective double-blinded
33. Friederich JA, Butterworth JF. Sodium nitroprus- pilot trial. J Neurosurg 1999;90:1042-52.
side: twenty years and counting. Anesth Analg 44. Kirkpatrick PJ, Turner CL, Smith C, et al.
1995;81:152-62. Simvastatin in aneurysmal subarachnoid haemor-
34. Glauser T, Shinnar S, Gloss D, et al. Evidence- rhage (STASH): a multicentre randomised phase 3
based guideline: treatment of convulsive status trial. Lancet Neurol 2014;13:666-75.
epilepticus in children and adults: report of the 45. Kleindorfer DO, Towfighi A, Chaturvedi S, et
Guideline Committee of the American Epilepsy al. 2021 Guideline for the prevention of stroke in
Society. Epilepsy Curr 2016;16:48-61. patients with stroke and transient ischemic attack:
35. Goyal M, Demchuk AM, Menon BK, et al. a guideline from the American Heart Association/
Randomized assessment of rapid endovascular American Stroke Association. Stroke 2021;52:e364-
467 [erratum: Stroke 2021;52:e483-4].

ACCP Updates in Therapeutics® 2022: Critical Care Pharmacy Preparatory Review and Recertification Course

125
Neurocritical Care

46. Lee P, Jones GR, Center JR. Successful treatment and outcome after subarachnoid haemorrhage:
of adult cerebral salt wasting with fludrocortisone. British aneurysm nimodipine trial. Br Med J
Arch Intern Med 2008;168:325-6. 1989;298:636-42.
47. Li X, Sun Z, Zhao W, et al. Effect of acetylsalicylic 58. Powers WJ, Rabinstein AA, Ackerson T, et al.
acid usage and platelet transfusion on postopera- 2018 guidelines for the early management of
tive hemorrhage and activities of daily living in patients with acute ischemic stroke: a guideline for
patients with acute intracerebral hemorrhage. J healthcare professionals from the American Heart
Neurosurgery 2013;118:94-103. Association/American Stroke Association. Stroke
48. Liu-DeRyke X, Rhoney DH. Cerebral vasospasm 2018;49:e46-e99.
after aneurysmal subarachnoid hemorrhage: 59. Powers WJ, Rabinstein AA, Ackerson T, et al.
an overview of pharmacologic management. Guidelines for the early management of patients
Pharmacotherapy 2006;26:182-203. with acute ischemic stroke: 2019 update to the
49. May CC, Arora S, Parli SE, et al. Augmented renal 2018 guidelines for the early management of
clearance in patients with subarachnoid hemor- acute ischemic stroke: a guideline for health-
rhage. Neurocrit Care 2015;23:374-9. care professionals from the American Heart
50. Morgenstern LB, Hemphill JC III, Anderson C, Association/American Stroke Association. Stroke
et al. Guidelines for the management of sponta- 2019;50:e344-e418.
neous intracerebral hemorrhage: a guideline for 60. Qureshi AI, Palesch YY, Barsan WG, et al.
healthcare professionals from the American Heart Intensive blood-pressure lowering in patients
Association/American Stroke Association. Stroke with acute cerebral hemorrhage. N Engl J Med
2010;41:2108-29. 2016;375:1033-43.
51. Mueller SW, Kiser TH, Anderson TA, et al. 61. Qureshi AI, Suarez JI, Bhardwaj A, et al. Use
Intraventricular daptomycin and intrave- of hypertonic (3%) saline/acetate infusion in the
nous linezolid for the treatment of external treatment of cerebral edema: effect on intracranial
ventricular-drain-associated ventriculitis due to pressure and lateral displacement of the brain. Crit
vancomycin-resistant Enterococcus faecium. Ann Care Med 1999;26:440-6.
Pharmacother 2012;46:e35. 62. Rabinstein AA, Benarroch EE. Treatment of par-
52. Nelson NR, Morbitzer KA, Jordan JD, et al. The oxysmal sympathetic hyperactivity. Curr Treat
impact of capping creatinine clearance on achiev- Options Neurol 2008;10:151-7.
ing therapeutic vancomycin concentrations in 63. Rhoney DH, Parker D Jr. Considerations in fluids
neurocritically ill patients with traumatic brain and electrolytes after traumatic brain injury. Nutr
injury. Neurocrit Care 2019;30:126-131 Clin Pract 2006;21:462-78.
53. Nogueira RG, Jadhav AP, Haussen DC, et al. 64. Roberts I, Yates D, Sandercock P, et al. Effect of
Thrombectomy 6 to 24 hours after stroke with intravenous corticosteroids on death within 14
mismatch between deficit and infarct (DAWN). N days in 10008 adults with clinically significant
Engl J Med 2018;347:11-21. head injury (MRC CRASH trial): randomised
54. Oyler DR, Stump SE, Cook AM. Accuracy placebo-controlled trial. Lancet 2004;364:1321-8.
of nimodipine gel extraction. Neurocrit Care 65. Robertson CS, Valadka AB, Hannay J, et al.
2015;22:89-92. Prevention of secondary ischemic insults after
55. Panos NG, Tesoro EP, Kim KS, et al. Outcomes severe head injury. Crit Care Med 1999;27:2086-95.
associated with transdermal nicotine replacement 66. Sandow N, Diesing D, Sarrafzadeh A, et al.
therapy in a neurosurgery intensive care unit. Am Nimodipine dose reductions in the treatment of
J Health Syst Pharm 2010;67:1357-61. patients with aneurysmal subarachnoid hemor-
56. Pedavally S, Fugate JE, Rabinstein AA. Serotonin rhage. Neurocrit Care 2016;25:29-39.
syndrome in the intensive care unit: clinical pre- 67. Sarode R, Milling TJ Jr, Refaai MA, et al. Efficacy
sentations and precipitating medications. Neurocrit and safety of a 4-factor prothrombin complex
Care 2014;21:108-13. concentrate in patients on vitamin K antagonists
57. Pickard JD, Murray GD, Illingworth R, et al. presenting with major bleeding: a randomized,
Effect of oral nimodipine on cerebral infarction

ACCP Updates in Therapeutics® 2022: Critical Care Pharmacy Preparatory Review and Recertification Course

126
Neurocritical Care

plasma-controlled, phase IIIb study. Circulation of neurointensive care. Neurosurgery 2001;48:249-


2013;128:1234-43. 61; discussion 61-2.
68. Saver JL, Goyal M, Bonafe A, et al. Stent-retriever 79. Treiman DM, Meyers PD, Walton NY, et al. A
thrombectomy after intravenous t-PA vs. t-PA comparison of four treatments for generalized con-
alone in stroke. N Engl J Med 2015;372:2285-95. vulsive status epilepticus. Veterans Affairs Status
69. Schwartz ML, Tator CH, Rowed DW, et al. The Epilepticus Cooperative Study Group. N Engl J
University of Toronto Head Injury Treatment Med 1998;339:792-8.
Study: a prospective, randomized comparison 80. Tunkel AR, Hartman BJ, Kaplan SL, et al. Practice
of pentobarbital and mannitol. Can J Neurol Sci guidelines for the management of bacterial menin-
1984;11:434-40. gitis. Clin Infect Dis 2004;39:1267-84.
70. Seder DB, Schmidt JM, Badjatia N, et al. 81. Udy AA, Roberts JA, Boots RJ, et al. Augmented
Transdermal nicotine replacement therapy in renal clearance: implications for antibacterial
cigarette smokers with acute subarachnoid hemor- dosing in the critically ill. Clin Pharmacokinet
rhage. Neurocrit Care 2011;14:77-83. 2010;49:1-16.
71. Silbergleit R, Durkalski V, Lowenstein D, et al. 82. van der Meche FG, Schmitz PI. A randomized
Intramuscular versus intravenous therapy for trial comparing intravenous immune globulin and
prehospital status epilepticus. N Engl J Med plasma exchange in Guillain-Barre syndrome.
2013;366:591-600. Dutch Guillain-Barre Study Group. N Engl J Med
72. Spasovski G, Vanholder R, Allolio B, et al. 1992;326:1123-9.
Clinical practice guideline on diagnosis and 83. Walters BC, Hadley MN, Hurlbert RJ, et al.
treatment of hyponatraemia. Intensive Care Med Guidelines for the management of acute cervi-
2014;40:320-31. cal spine and spinal cord injuries: 2013 update.
73. Taylor SJ, Fettes SB, Jewkes C, et al. Prospective, Neurosurgery 2013;60(suppl 1):82-91.
randomized, controlled trial to determine the effect 84. Weant KA, Ramsey CN III, Cook AM. Role of
of early enhanced enteral nutrition on clinical out- intra-arterial therapy for cerebral vasospasm sec-
come in mechanically ventilated patients suffering ondary to aneurysmal subarachnoid hemorrhage.
head injury. Crit Care Med 1999;27:2525-31. Pharmacotherapy 2010;30:405-17.
74. Temkin NR, Dikmen SS, Anderson GD, et al. 85. Whelton PK, Carey RM, Aronow WS, et al. 2017
Valproate therapy for prevention of posttrau- ACC/AHA/AAPA/ABC/ACPM/AGS/APhA/ASH/
matic seizures: a randomized trial. J Neurosurg ASPC/NMA/PCNA guideline for the prevention,
1999;91:593-600. detection, evaluation, and management of high
75. Temkin NR, Dikmen SS, Wilensky AJ, et al. A blood pressure in adults: a report of the American
randomized, double-blind study of phenytoin for College of Cardiology/American Heart Association
the prevention of post-traumatic seizures. N Engl J Task Force on Clinical Practice Guidelines. JACC
Med 1990;323:497-502. 2017. DOI: 10.1016/j.jacc.2017.11.006.
76. Thomalla G, Simonsen CZ, Boutitie F, et al. MRI-
guided thrombolysis for stroke with unknown time
of onset. N Engl J Med 2018;347:1-12.
77. Thomas A, Greenwald BD. Paroxysmal sympa-
thetic hyperactivity and clinical considerations for
patients with acquired brain injuries: a narrative
review. Am J Phys Med Rehabil 2019;98:65-72
78. Treggiari-Venzi MM, Suter PM, Romand JA.
Review of medical prevention of vasospasm after
aneurysmal subarachnoid hemorrhage: a problem

ACCP Updates in Therapeutics® 2022: Critical Care Pharmacy Preparatory Review and Recertification Course

127
Neurocritical Care

ANSWERS AND EXPLANATIONS TO PATIENT CASES

1. Answer: C 5. Answer: A
Answer C is correct because midazolam (together with Answer A is correct because nimodipine is the only agent
lorazepam) is recommended by the status epilepticus with an FDA indication for preventing ischemic compli-
guidelines. Answers A, B, and D are incorrect because cations related to SAH. Answer B is incorrect because
phenytoin is less effective than lorazepam as the initial prophylactic Triple-H therapy or variants thereof do not
agent. Although valproic acid and levetiracetam have prevent ischemic complications; rather, hyperperfusion
not been formally compared with lorazepam as the ini- therapies are used when vasospasm develops. Answer
tial agent for status epilepticus, their use is supported by C is incorrect because in clinical trials, the efficacy of
less clinically rigorous evidence. statins for preventing vasospasm has failed. Answer
D is incorrect because aminocaproic acid may in fact
2. Answer: B increase the risk of stroke in patients with SAH.
Answer B is correct because clinical evidence supports
the safety and efficacy of osmotherapy as a first-line 6. Answer: A
therapy in this situation. Hypertonic saline would be Answer A is correct because induction of hypertension
more appropriate than mannitol because of the patient’s with a vasopressor such as norepinephrine appears to
relatively low serum sodium concentration and elevated improve cerebral perfusion. Titrating the infusion to
SCr (mannitol is cleared renally and is thus not optimal MAP values that result in improved neurologic symp-
for patients with renal dysfunction, making Answer A toms is often necessary. Answer B is incorrect because
incorrect). Answers C and D (pentobarbital and mid- data analyses are limited to support transfusing blood
azolam) are not ideal for this patient because of the to a high hemoglobin (in fact, blood transfusion appears
likelihood of hypotension. to be a risk factor for vasospasm). In addition, fluid
resuscitation to hypervolemic levels is not beneficial.
3. Answer: B Answer C is incorrect because when hypervolemia is
Answer B is correct because warfarin is well reversed by compared with euvolemia, neurologic outcomes are no
4F-PCC products in a much more timely and complete different, but patients receiving hypervolemia develop
manner than vitamin K in the acute setting. Answer A is more pulmonary edema. Answer D is incorrect because
incorrect; although blood pressure control is important milrinone is not first-line therapy for vasospasm.
for this patient, amlodipine is unlikely to have timely
effects immediately after ICH. Answer C is incorrect 7. Answer: B
because platelets are minimally effective for reversing Answer B is correct because the anti-Xa activity concen-
ibuprofen. Answer D is incorrect; rFVIIa is not recom- tration is the laboratory value that best correlates with
mended for reversal of warfarin because of thrombosis rivaroxaban activity. Answers A and C are incorrect
risks. because neither INR nor activated partial thrombo-
plastin time is typically affected by rivaroxaban alone.
4. Answer: B Answer D is incorrect because the VerifyNow PRU test
Answer B is correct because nicardipine is recommended measurement is more specific to antiplatelet agents such
for reducing blood pressure after ICH, and the threshold as aspirin or clopidogrel.
for treatment is correct according to the INTERACT-2
and ATACH-2 studies. Answer A is incorrect; although 8. Answer: A
clevidipine may be considered in this case, the typical Answer A is correct because the most consistent reversal
goal blood pressure target after ICH is a SBP 140-150 effects, although with low-quality evidence, occur with
mm Hg. Answers C and D are incorrect; although labet- 4F-PCCs. Answer B is incorrect because the appropriate
alol and esmolol also reduce blood pressure, the optimal bolus dose of andexanet would be 400 mg followed by
SBP goal after ICH 140-150 mm Hg. a 2-hour continuous infusion, given the dose and time
of the last dose. In addition, available data are currently
limited regarding the periprocedural use of andexanet.

ACCP Updates in Therapeutics® 2022: Critical Care Pharmacy Preparatory Review and Recertification Course

128
Neurocritical Care

Answer C is incorrect because fresh frozen plasma does


not reverse factor Xa inhibitors reliably. Answer D is
incorrect; although factor VII may have some useful-
ness, reversal is incomplete, and factor VII is associated
with an increased risk of thrombosis.

ACCP Updates in Therapeutics® 2022: Critical Care Pharmacy Preparatory Review and Recertification Course

129
Neurocritical Care

ANSWERS AND EXPLANATIONS TO SELF-ASSESSMENT QUESTIONS

1. Answer: B (SBP less than 185 mm Hg, DBP less than 110 mm Hg)
Answer B is correct because sodium supplementation before alteplase administration or immediately after
is effective for treating hyponatremia. Although hyper- stroke in general (so-called permissive hypertension
volemia is no longer advocated, ensuring euvolemia to ensure adequate cerebral perfusion). Nicardipine is
is important. Answers A and C are incorrect because not necessary for this patient at this point. Answer D is
water restriction or tolvaptan is undesirable in a patient incorrect because reversal of warfarin with vitamin K is
who is 4 days after ictus for SAH because of the impor- not recommended for an acute thrombosis in the brain.
tance of maintaining adequate cerebral perfusion.
Hyponatremia may be harmful in a patient with a new 5. Answer: C
stroke such as SAH caused by cerebral edema, making Answer C is correct; the current guidelines do not recom-
Answer D incorrect. mend high-dose methylprednisolone therapy because of
the inconsistency in beneficial effects and the relatively
2. Answer: A consistent risk of adverse effects (GI bleeding, infection)
Answer A is correct because vancomycin effectively shown in clinical trials. Answer B is incorrect because
covers MRSE, and the intraventricular dose of 10 mg high-dose methylprednisolone does not augment spinal
daily is appropriate. Answer B is incorrect because gen- perfusion. Answers A and D are incorrect; although the
tamicin is less likely to be effective alone for MRSE and NASCIS-III study showed some potential benefit for
is more associated with seizures than other aminoglyco- patients who received a bolus and a 47-hour infusion
sides. Answer C is incorrect because ampicillin (or other when high-dose methylprednisolone therapy was initi-
penicillins) should not be given by the intraventricular ated 3–8 hours after injury, particularly for incomplete
route, given the risk of seizures. Answer D is incorrect injuries, the therapy is no longer recommended.
because this patient has a treatment-refractory, device-
related CNS infection. In this instance, intraventricular 6. Answer: A
antimicrobials may be considered. Answer A is correct; of the agents listed, buspirone is
the only one that increases CNS serotonin concentra-
3. Answer: B tions. Answers B–D would not be expected to increase
Answer B is correct; although the optimal CPP varies CNS serotonin concentrations. Cyproheptadine is a
with each individual, the typical recommended target potential therapeutic agent for patients with serotonin
range is 60–70 mm Hg. Patients with a TBI have gas- syndrome.
tric intolerance but benefit greatly from early enteral
nutrition, making Answer A incorrect. Answer C is 7. Answer: B
incorrect because dextrose-containing fluids may Answer B is correct; given this patient’s change in neu-
increase cerebral edema in TBI and lower the serum rologic status, vital signs, and monitoring values, she is
sodium concentration. Answer D is incorrect because likely having a cerebral vasospasm with increased ICP.
high-dose methylprednisolone therapy increases mor- Therapy should be targeted at optimizing cerebral per-
tality in patients with a TBI. fusion (fluid bolus or increase in MAP) and reducing
ICP. Hypertonic saline will address both issues, causing
4. Answer: B an increase in intravascular volume to improve perfu-
Answer B is correct because this patient meets the sion and possibly an increase in blood pressure while
criteria for receiving alteplase and has no obvious lowering ICP through osmotic effects. Answer A is
contraindications. The more timely the alteplase admin- incorrect because verapamil must be given superselec-
istration, the more likely the patient will benefit (and the tively in the angiography suite. One possible adverse
less risk). Answer A is incorrect because aspirin should effect of verapamil is cerebral vasodilation, which could
be initiated within the first 24–48 hours after stroke, lead to increased ICP, and ICP would be undesirable in
but not necessarily immediately. Answer C is incor- this patient right now. Answer C is incorrect; although
rect because the blood pressure may be slightly elevated mannitol may decrease ICP, it also causes diuresis,

ACCP Updates in Therapeutics® 2022: Critical Care Pharmacy Preparatory Review and Recertification Course

130
Neurocritical Care

which is undesirable in a patient with ongoing cerebral


vasospasm. Answer D is incorrect; although a blood
transfusion might theoretically increase oxygen-carry-
ing capacity to the brain (enhancing perfusion), a blood
transfusion may in fact cause a risk of cerebral vaso-
spasm. In addition, it is unlikely to affect ICP.

8. Answer: D
Answer D is correct because propofol is an almost-
ideal agent for sedation with its quick onset and offset
of activity and relative lack of dependence on organ
function for clearance; however, boluses and large dose
titrations should be avoided because of the risk of hypo-
tension. Answers A and C are incorrect; long-acting
sedating agents such as lorazepam and morphine are
undesirable in patients with a neurologic injury because
of their propensity to obscure the neurologic exami-
nation for prolonged periods. Answer B is incorrect;
midazolam (and other benzodiazepines) is associated
with increased delirium. In addition, midazolam may
accumulate when used for a prolonged period because
of its long context-sensitive half-life.

ACCP Updates in Therapeutics® 2022: Critical Care Pharmacy Preparatory Review and Recertification Course

131
Pharmacokinetics/
Pharmacodynamics
Joseph M. Swanson, Pharm.D., FCCP, FCCM
University of Tennessee College of Pharmacy
Memphis, Tennessee
Pharmacokinetics/Pharmacodynamics

Pharmacokinetics/
Pharmacodynamics
Joseph M. Swanson, Pharm.D., FCCP, FCCM
University of Tennessee College of Pharmacy
Memphis, Tennessee

ACCP Updates in Therapeutics® 2022: Critical Care Pharmacy Preparatory Review and Recertification Course

135
Pharmacokinetics/Pharmacodynamics

Learning Objectives Self-Assessment Questions


Answers and explanations to these questions can be
1. Describe the changes in critically ill patients that found at the end of this chapter.
alter drug absorption.
2. Explain how critical illness affects drug distribution. 1. J.H. is a 30-year-old man admitted to the inten-
3. Depict the effects of changing hepatic blood flow, sive care unit (ICU) for septic shock. He initially
intrinsic activity, and protein binding on drug received 30 mL/kg of normal saline for intravenous
metabolism. fluid resuscitation. He required further fluid admin-
4. Differentiate between different critically ill patient istration to improve his pulse pressure variation.
populations and the expected pharmacokinetic (PK) Despite prophylaxis with enoxaparin 30 mg subcu-
changes. taneously every 12 hours, J.H. has a proximal deep
5. Incorporate the PK changes in a critically ill patient venous thrombosis. Which pharmacokinetic (PK)
into the design and evaluation of an appropriate alteration most likely contributed to this therapeutic
drug regimen. failure?
6. Identify the desired pharmacodynamic variables A. Decreased anti-factor Xa (anti-Xa) activity
associated with efficacy in select drugs. secondary to decreased volume of distribution
(Vd).
B. Decreased anti-Xa activity secondary to
Abbreviations in This Chapter decreased absorption.
C. Increased anti-Xa activity secondary to
AAG α1-Acid glycoprotein decreased hepatic metabolism.
AKI Acute kidney injury D. Increased anti-Xa activity secondary to
ARC Augmented renal clearance decreased renal elimination.
AUC Area under the curve
AUC/MIC Ratio of area under the curve to the Questions 2–4 pertain to the following case.
minimum inhibitory concentration for E.W. is a 48-year-old man (height 70 inches, weight 85
the bacterial pathogen kg) admitted to the trauma ICU after a motorcycle col-
AUC0-24/MIC Ratio of area under the curve for 24 lision. E.W. presents with a traumatic brain injury (TBI;
hours to the minimum inhibitory head computed tomography [CT] reveals a depressed
concentration for the bacterial pathogen skull fracture, frontal subarachnoid hemorrhage, and
CKD Chronic kidney disease right intraparenchymal hemorrhage), right acetabulum
Cmax/MIC Ratio between the maximum drug fracture, bilateral rib fractures, and abdominal trauma.
concentration and the minimum According to his abdominal CT, E.W. must go to the
inhibitory concentration for the bacterial operating room for an exploratory laparotomy for repair
pathogen of several serosal tears. After surgery, E.W. requires sig-
fT>MIC Free drug concentration time above the nificant resuscitation in his first 24 hours of admission
minimum inhibitory concentration for (5 L of normal saline). He is made NPO to allow bowel
the bacterial pathogen rest.
ECMO Extracorporeal membrane oxygenation E.W.’s laboratory values are as follows: serum cre-
GFR Glomerular filtration rate atinine (SCr) 1.1 mg/dL, blood urea nitrogen (BUN) 17
ICU Intensive care unit mg/dL, and white blood cell count (WBC) 19 × 103 cells/
MIC Minimum inhibitory concentration mm3. The patient’s pulmonary artery catheterization val-
PD Pharmacodynamic(s) ues are cardiac index 4.2 L/minute/m2 (normal 2.8–3.6
PK Pharmacokinetic(s) L/minute/m2) and pulmonary artery wedge pressure
TBI Traumatic brain injury 16 mm Hg. His medication therapy includes a fentanyl
TDM Therapeutic drug monitoring continuous infusion of 75 mcg/hour, a propofol con-
Vd Volume of distribution tinuous infusion of 15 mcg/kg/minute, pantoprazole 40
mg intravenously every 24 hours, enoxaparin 30 mg

ACCP Updates in Therapeutics® 2022: Critical Care Pharmacy Preparatory Review and Recertification Course

136
Pharmacokinetics/Pharmacodynamics

subcutaneously every 12 hours, and phenytoin 150 mg 5. According to S.H.’s known PK changes, which
intravenously every 8 hours. would be the most appropriate intravenous loading
dose of vancomycin?
2. Which most accurately assesses the risk factors for A. 1500 mg.
the decreased absorption of enterally administered B. 2000 mg.
drugs? C. 2500 mg.
A. Intestinal atrophy, pantoprazole therapy, D. 3000 mg.
abdominal surgery.
B. TBI, fentanyl therapy, cardiac output. 6. S.H. is given a diagnosis of methicillin-resistant
C. Abdominal surgery, pantoprazole therapy, TBI. Staphylococcus aureus hospital-acquired pneu-
D. Intestinal atrophy, cardiac output, fentanyl monia. On day 10 of vancomycin therapy, his
therapy. vancomycin area under the curve (AUC) is 700 mg
x h/L. His previous vancomycin AUC was 425 mg x
3. Before E.W.’s ICU admission, his albumin concen- h/L on the same dosing regimen. Which most likely
tration was 3.8 g/dL, but after surgery, it decreased explains what transpired?
to 2.1 g/dL. Given this change in albumin, which A. Augmented renal excretion returned to normal.
change in the total and unbound concentration of B. Vd increased to larger than normal.
propofol would be most likely? C. Tissue penetration decreased to below normal.
A. Increased total concentration, decreased D. Liver blood flow returned to normal.
unbound concentration.
B. No change in total concentration, increased 7. B.B. is 40-year-old woman with a surgical site
unbound concentration. infection caused by Pseudomonas aeruginosa.
C. Increased total concentration, no change in She is initiated on a piperacillin/tazobactam 3.375
unbound concentration. g intravenous infusion over 4 hours every 8 hours.
D. Decreased total concentration, increased Which is the most likely benefit of this approach
unbound concentration. with piperacillin/tazobactam?
A. Decreased mortality supported by prospective
4. On postoperative day 3, E.W.’s SCr increased to 3 controlled studies.
mg/dL. On postoperative day 4, his SCr is 3.2 mg/ B. Decreased neurotoxicity supported by prospec-
dL. Which variable for assessing kidney function tive controlled studies.
would be most important for determining E.W.’s C. Decreased mortality supported by retrospective
dosing adjustments? reviews.
A. BUN/SCr ratio. D. Decreased neurotoxicity supported by retro-
B. Total daily urinary output. spective reviews.
C. Estimation of glomerular filtration rate (GFR).
D. History of chronic kidney disease (CKD). 8. C.W. is a 52-year-old man admitted to the ICU for
acute respiratory failure. He is given scheduled
Questions 5 and 6 pertain to the following case. oral morphine for pain control. Which PK variable
S.H. is a 35-year-old man (height 70 inches, weight would most likely affect the hepatic metabolism of
85 kg) admitted to the medical ICU because of sepsis morphine?
caused by hospital-acquired pneumonia. He is empiri- A. Increased α1-acid glycoprotein (AAG).
cally treated with intermittent vancomycin, piperacillin/ B. Decreased albumin.
tazobactam, and ciprofloxacin. His laboratory values C. Increased hepatic blood flow.
are as follows: SCr 1 mg/dL, BUN 12 mg/dL, and WBC D. Increased intrinsic clearance.
18 × 103 cells/mm3.

ACCP Updates in Therapeutics® 2022: Critical Care Pharmacy Preparatory Review and Recertification Course

137
Pharmacokinetics/Pharmacodynamics

BPS Critical Care Pharmacy Specialist Examination Content Outline

This chapter covers the following sections of the Critical Care Pharmacy Specialist Examination Content Outline:
1. Domain 1: Clinical Knowledge and Application
a. Task 1: 3
b. Task 2: 4
c. Task 3: 1–6
d. Task 4: 1–5
e. Task 6: 1–4
f. Task 7: 1–7
3. Domain 3: Evidence-Based Medicine, Scholarship, Education, and Professional Development
a. Task 1: 3

ACCP Updates in Therapeutics® 2022: Critical Care Pharmacy Preparatory Review and Recertification Course

138
Pharmacokinetics/Pharmacodynamics

I.  INTRODUCTION
Pharmacokinetics refers to the movement of a drug through the body, particularly absorption, distribution, metabo-
lism, and excretion (ADME) of a drug, whereas pharmacodynamics (PD) addresses the biochemical and physiologic
effects of a drug on the body. Physiologic changes in critically ill patients cause alterations that affect the PK and
PD of drugs. Although few studies evaluate the effect of these changes, clinicians must consider the general prin-
ciples when making drug dosing decisions in critically ill patients. The most important consideration in critically
ill patients is that changes can occur rapidly. A patient may have an altered PK variable on one day, only to experi-
ence changes that alter that variable in a completely different way on the following day. An example is a critically
ill patient with augmented renal clearance (ARC) who then has an acute kidney injury (AKI). The patient may have
increased renal elimination of a specific drug, followed by decreased elimination when AKI occurs. Therefore, the
critical care pharmacist should know how the principles can be altered and continually anticipate changes during a
patient’s stay in the ICU.

II.  ROUTES OF ADMINISTRATION

A. Intravenous
1. The intravenous route is the most widely used method of drug administration in the critically ill
population. The bioavailability of an intravenously administered drug is 100%, thus ensuring the entire
dose reaches the systemic circulation.
2. Although intravenous drug administration is the most popular method used in the ICU, it still poses
several potential problems. The intravenous route does not guarantee penetration of the drug into sites
outside the circulatory system. Examples of this include poor penetration of drugs into various tissues
such as the meninges, pulmonary tissue, and bone. In conditions such as septic shock, drug penetration
into muscle and subcutaneous tissue is lower than expected. Finally, there are reports documenting
the severe adverse effects of inadvertent extravascular administration of a drug. There are several
reports of drug (vessicants or non-vessicants, which can be cytotoxic or noncytotoxic) extravasation,
which highlights this potential complication of intravenous administration. For example, extravasation
of norepinephrine creates local vasoconstriction and tissue necrosis, which is sometimes severe enough
to cause loss of a limb.

B. Enteral/Oral – Using the enteral or oral route of administration in critically ill patients results in variable drug
bioavailability. The predominant concern for this route of administration in critically ill patients pertains
to alterations in drug absorption. The issues pertaining to altered drug absorption are discussed in the next
section. Of note, not all drugs show reduced absorption when administered enterally/orally to critically ill
patients. One example comes from a study investigating the PK of atorvastatin. Compared with healthy
volunteers, patients in the ICU had a significantly higher area under the curve (AUC) (110.5 ng/mL vs. 5.9
ng/mL, p<0.01) after a single dose of atorvastatin 20 mg. The increased AUC could only partly be explained
by altered hepatic metabolism (Intensive Care Med 2009;35:717-21). A complicating factor with enteral
administration is first-pass metabolism, which can significantly affect the bioavailability of hepatically
metabolized drugs (discussed later in the chapter). As such, critical care clinicians often question the use of the
enteral/oral route because of both increased and decreased bioavailability with different enteral medications.

ACCP Updates in Therapeutics® 2022: Critical Care Pharmacy Preparatory Review and Recertification Course

139
Pharmacokinetics/Pharmacodynamics

C. Subcutaneous/Intramuscular/Sublingual – Subcutaneous and intramuscular drug administration avoids


first-pass metabolism by the liver and can increase the bioavailability of a drug. However, these routes still
require the drug to be absorbed into the blood. Therefore, these routes are potentially affected by changes in
absorption. Unlike with the enteral or oral route, clinicians do not routinely abandon the use of subcutaneously
or intramuscularly administered medications. Examples include the continued use of low-molecular-weight
heparins, insulin, and the antipsychotic haloperidol in patients with obesity for whom the absorption may
be altered. In addition, clinicians should be aware of similar concerns for altered absorption in critically
ill patients with sepsis or shock states because of changes in perfusion. Use of the sublingual route of
administration in critically ill patients has not been studied. However, one could expect that alterations
in blood flow and absorption would cause similar concerns as for subcutaneous and intramuscular routes.
Critical care clinicians should understand that the subcutaneous route may be used; however, little to no data
are available in critically ill patients to determine if any pharmacokinetic changes will occur.

D. Inhalation – Administration of drugs directly into the lungs of critically ill patients is usually chosen to reduce
systemic exposure and/or achieve a high concentration in the pulmonary tissue. The high local concentration
is intended to maximize the therapeutic effect while reducing any adverse or unwanted effects. For example,
the use of inhaled bronchodilators reduces unwanted systemic effects such as tachycardia. Antibiotics such
as colistin and aminoglycosides are administered to improve the antibiotic concentrations in the lungs and
reduce exposure to the kidneys. However, the efficacy of this route of antibiotic administration is not well
established. Drug particles of 1–5 micrometers have the best opportunity to be delivered to all areas of
the lungs. Smaller particles will be exhaled without being deposited in the lower airways, whereas larger
particles will be deposited in the large bronchi or the oropharynx. Several models of nebulizers are on the
market that use different methods to achieve the desired particle sizes.

E. Intrathecal/Intraventricular – The intraventricular route is used with the same goal as the inhalation route.
Increased local concentrations and reduced systemic concentrations are the desired effect. Data evaluating
the efficacy of this route of administration are lacking in the general population and mainly limited to case
series. Despite the lack of data, clinicians use this route when treating multidrug-resistant meningitis with
antibiotics, or for the administration of analgesics in patients with chronic pain.

Patient Case

1. M.J. is a 70-year-old man admitted to the neurosurgical ICU for an aneurysmal subarachnoid hemorrhage.
His initial treatment included placement of an external ventricular drain. Subsequently, he had a maximum
temperature of 101.5°F, a WBC of 15 × 103 cells/mm3, and a cerebrospinal fluid culture positive for methi-
cillin-resistant S. aureus. Intraventricular vancomycin 20 mg is used for therapy. Which is the best rationale
for this approach?
A. Demonstrated superiority to intravenous antibiotics.
B. Maximizing localized antibiotic concentrations.
C. Safer administration method.
D. Reducing the ototoxicity of vancomycin.

ACCP Updates in Therapeutics® 2022: Critical Care Pharmacy Preparatory Review and Recertification Course

140
Pharmacokinetics/Pharmacodynamics

III.  ABSORPTION

A. Bioavailability (F) refers to the percentage of an administered dose of drug that reaches the systemic
circulation. Bioavailability from subcutaneous and intramuscular or enteral administration is affected by
absorption and first-pass metabolism (enterally administered drugs). Few studies directly assess the enteral
absorption of drugs in critically ill patients, and the results vary. In addition, studies of enterally administered
drugs do not differentiate whether plasma concentrations are altered because of changes in absorption or
first-pass metabolism. Although data on absorption in critically ill patients are limited, clinicians must
consider several factors if a route of administration other than intravenous is desired.

B. Gastrointestinal (GI) Perfusion – Hypotension and/or shock are known to cause the shunting of blood toward
the vital organs (brain, heart, lungs) and away from the less vital organs (muscles, skin, splanchnic organs).
1. GI absorption: No studies clearly show the effect of hypotension or shock on the oral or enteral absorption
of drugs. Clinicians extrapolate changes in splanchnic blood flow to the likelihood that GI absorption
is altered. Redistribution of blood away from the splanchnic circulation is thought to decrease drug
absorption from the GI tract. The hyperdynamic phase of sepsis or septic shock can increase cardiac
output, and studies have shown an increase in hepatosplanchnic (portal vein and hepatic artery) blood
flow. In late-stage (decompensated) sepsis, it is thought that splanchnic blood flow is decreased, but
no studies have verified this. This uncertainty in splanchnic blood flow and GI absorption leads many
clinicians to forgo the enteral route for drug administration.
2. Transdermal, subcutaneous, and intramuscular absorption: No studies have evaluated the effect of
hypotension or shock on transdermal, subcutaneous, or intramuscular absorption. Similar to splanchnic
circulation, the shunting of blood to vital organs reduces blood flow to the skin and muscles, which
is thought to reduce absorption from these sites. This assumption is supported by the observation
that critically injured trauma patients with edema have significantly lower anti-Xa and antithrombin
activity after treatment with subcutaneous enoxaparin (J Trauma 2005;59:1336-43). Altered absorption
is believed to be a contributing factor to these results. However, this may not be generalized to all
critically ill patients because anti-Xa activity was not significantly different in edematous compared with
nonedematous medical-surgical ICU patients after dalteparin administration (Crit Care 2006;10:R93).
Furthermore, the direct effects of critical illness on coagulation may alter the pharmacodynamics of
low molecular weight heparins and obscure the degree to which absorption impacts their effect.
3. Vasopressor effect: Vasopressors may contribute to regional hypoperfusion, which could result in
decreased drug absorption. Vasopressin reduces splanchnic blood flow in patients with distributive
shock. In septic shock, epinephrine results in reduced splanchnic blood flow. Dopamine is not as
effective as norepinephrine in maintaining splanchnic blood flow in patients with stable distributive
shock. Conversely, when gut perfusion is compared between cardiac surgery patients with and
without vasodilatory shock, norepinephrine use results in higher intestinal perfusion. However, this is
countered by a worse splanchnic oxygen demand versus supply. The variable effect of vasopressors on
splanchnic perfusion creates enough concern that most clinicians abandon the use of orally or enterally
administered drugs when vasopressors are being used. In addition, absorption from other sites could be
impaired. One study investigated the anti-Xa activity of the low-molecular-weight heparin certoparin in
critically ill patients, of whom 40.3% were receiving vasopressors. Less than 50% of patients receiving
standard doses of certoparin had anti-Xa activity in the antithrombotic range (0.1–0.3 IU/mL) (Crit Care
2005;9:R541-8).

ACCP Updates in Therapeutics® 2022: Critical Care Pharmacy Preparatory Review and Recertification Course

141
Pharmacokinetics/Pharmacodynamics

C. Intestinal Atrophy – After 3–5 days of fasting, gut mucosal crypt depth and villus height can be decreased.
This correlates with an abnormal lactulose-mannitol test, indicating increased gut permeability. Splanchnic
hypoperfusion can further worsen gut hypoxia, exacerbating gut permeability. However, the effect of
intestinal atrophy on drug absorption has not been systematically evaluated. Atrophy and the corresponding
loss of integrity of the tight junctions could lead to an increased absorption of drugs that are absorbed
through passive diffusion. Conversely, cellular dysfunction caused by atrophy might decrease the absorption
of drugs that require active transport for absorption. Currently, no studies can clarify this quandary.

D. GI dysmotility – GI dysmotility has clearly been established in critically ill patients, with an incidence as high
as 60%. Box 1 shows the conditions in critically ill patients that are associated with delayed gastric emptying
caused by dysmotility. Acetaminophen kinetics show that GI dysmotility causes a delay in absorption and
a reduced peak concentration in most studies. Concern regarding PK changes in the presence of delayed
gastric emptying is a major factor contributing to the avoidance of orally or enterally administered drugs in
critically ill patients. GI dysmotility is usually treated using prokinetic agents such as metoclopramide or
erythromycin. No data exist regarding the effect of prokinetics on drug absorption in critically ill patients
with dysmotility. However, one study found that, in healthy individuals, coadministration of erythromycin
and the controlled-release formulation of pregabalin resulted in a 17% decrease in AUC and a 13% decrease
in peak plasma concentrations of pregabalin (Clin Drug Investig 2015;35:299-305). This suggests that
prokinetics can affect the absorption of other medications. In the critically ill patient, the effect of prokinetics
on the PK of orally or enterally administered remains relatively unclear.

Box 1. Reasons for Delayed Gastric Emptying


Burns Mechanical ventilation Shock
Electrolyte abnormalities Opioid analgesics Surgery
Hyperglycemia Postoperative ileus Trauma
Ileus Sepsis Traumatic brain injury

E. Intestinal Drug Transporters – Transmembrane proteins such as P-glycoprotein (P-gp) and cytochrome
P450 (CYP) enzymes play an integral role in drug absorption. In general, these transporters reduce the
absorption of drug substrates. Therefore, decreased activity of these enzymes will theoretically increase the
absorption of drugs that are substrates. Conversely, several intestinal transporters facilitate drug absorption
and may thus decrease drug absorption. However, no PK studies evaluate changes in drug absorption
caused by changes in intestinal transporters that are specifically related to critical illness or conditions
often present in these patients. Increased proinflammatory cytokines in patients with conditions such as
systemic inflammatory response syndrome and sepsis affect P-gp expression and activity. Therefore, enteral
drug absorption can be altered in these states. However, no studies have directly investigated the effects of
inflammatory states, or systemic inflammatory response syndrome, on drug absorption changes mediated
by changes in P-gp activity.

F. Physical Incompatibilities – Drugs administered through enteral feeding tubes come in contact with gastric
secretions, intestinal secretions, and enteral nutrition formulas, all of which pose a problem for drug
absorption.
1. Drug enteral nutrition binding: Some drugs potentially interact with enteral nutrition. The degree of
interaction and clinical significance varies.
a. Ciprofloxacin bioavailability is reduced when it is administered with enteral nutrition, but most
studies suggest that serum concentrations remain above the minimum inhibitory concentration
(MIC) for most bacterial pathogens.

ACCP Updates in Therapeutics® 2022: Critical Care Pharmacy Preparatory Review and Recertification Course

142
Pharmacokinetics/Pharmacodynamics

b. Enteral nutrition has been reported to significantly reduce the absorption of levothyroxine,
phenytoin, and warfarin. One case report showed a reduction in voriconazole serum concentrations
when enteral nutrition was initiated (J Oncol Pharm Pract 2012;18:128-31).
c. A suggested solution to this interaction is to hold the enteral nutrition 1–2 hours before and after
drug administration. However, this poses two problems. First, interruption of enteral nutrition may
contribute to inadequate nutrition support. Second, this increases the difficulty of administering the
medications appropriately. Increasing the workload of nursing can lead to administration delays or,
even worse, errors. This is very important, because failure to withhold the enteral nutrition could
result in suboptimal dosing and effects of the interacting drug.
2. pH changes: The state of ionization of a drug usually affects the lipophilicity and potentially the
absorption. Examples from non-critically ill patients include increased gastric pH caused by histamine-2
receptor antagonists or proton pump inhibitors (both commonly used in the critically ill population for
GI prophylaxis), resulting in decreased absorption of ketoconazole, itraconazole, atazanavir, indinavir,
dasatinib, mycophenolate mofetil, cefpodoxime, and dipyridamole. Acid-suppressive drugs increased
nifedipine and digoxin absorption, and alendronate had a 2-fold increase in bioavailability in the
presence of these agents (Aliment Pharmacol Ther 2009;29:1219-29). Additionally, drug coating or
extended release matrices may be altered by pH changes and could potentially impact oral absorption.
For example, coadministration of a proton pump inhibitor with enteric coated ketoprofen altered the
maximum concentration and time to maximum concentration.

G. Important Considerations for Absorption


1. The overall uncertainty of a patient’s ability to absorb drugs from the GI tract often results in the
clinician’s avoidance of enterally administered drugs. The decision to use the GI route of administration
is arbitrary. Many clinicians will anecdotally use tolerance of enteral feedings as a surrogate marker
for normal drug absorption. The definition of tolerance is debatable and assessment of tolerance is
highly variable, so it is not an ideal maker for drug absorption. If a drug known to interact with enteral
nutrition requires enteral administration, nutrition should temporarily be interrupted during drug
administration.
2. Subcutaneous and intramuscular routes of administration present similar problems with absorption, but
clinical practice has not abandoned these routes of administration. Some clinicians advocate using larger
doses of drugs being administered subcutaneously, but no studies have verified the safety or efficacy
of this practice. Small studies investigating enoxaparin dosing in medically ill and trauma patients
with obesity found improved anti-Xa serum concentrations when using a weight-based dosing method
(0.5 mg/kg every 12 hours) compared with standard dosing. This suggests that larger subcutaneous fat
distribution can alter the pharmacokinetics and PD of some medications (Thromb Res 2010;125:220-3;
Am J Surg 2013;206:847-51).

Patient Case

2. I.L. is a 32-year-old man receiving stress ulcer prophylaxis with esomeprazole 40 mg intravenously every
day. Which drug will most likely have an increased absorption secondary to the increased gastric pH?
A. Carvedilol.
B. Ciprofloxacin.
C. Diazepam.
D. Digoxin.

ACCP Updates in Therapeutics® 2022: Critical Care Pharmacy Preparatory Review and Recertification Course

143
Pharmacokinetics/Pharmacodynamics

IV.  DISTRIBUTION

A. The Vd of a drug is a PK variable that relates the dose with the resultant serum concentration of said drug.
A simple mathematical representation of this relationship is the following equation:
dose
C =
Vd
where C is the initial serum concentration of an intravenously administered drug and Vd is the volume of
distribution. However, the distribution of most drugs is more complex and is affected by several factors such
as perfusion, degree of protein binding, tissue permeability, drug lipid solubility, drug pKa, and pH of the
environment. Critically ill patients may be subjected to one or more changes in the previously stated factors
that could result in an altered Vd for some drugs.

B. Tissue Perfusion – As noted in the previous section, shock states and vasopressor use can cause the
redistribution of blood flow. This results in decreased perfusion of the muscle, skin, and splanchnic organs.
Hydrophilic drugs with a smaller Vd (ones that remain in the plasma water volume) may have decreased
distribution to parts of the body with decreased blood flow. This is highlighted by animal studies of septic
shock showing lower gentamicin concentrations in the microcirculation than in the central vessels.

C. Fluid Shifts and Tissue Membrane Permeability – Critically ill patients can receive significant volumes of
intravenous fluid for resuscitation. This often results in increased volumes of total body water and interstitial
fluid. In addition to fluid administration, diseases such as sepsis, thermal injury, acute respiratory distress
syndrome, AKI, heart failure, and cirrhosis can increase interstitial fluid volumes. Moreover, surgery
increases extracellular volume postoperatively. In this setting, the Vd for hydrophilic drugs is increased,
whereas the Vd for lipophilic drugs is often unchanged. The increased interstitial water provides a larger
compartment for hydrophilic drugs to distribute, thus decreasing the serum concentrations. In addition,
because distribution is into a larger interstitial space (as the result of increased interstitial water), the drug
concentration can be decreased in this space. This has been shown in microdialysis studies evaluating
subcutaneous tissue concentrations for intravenously administered piperacillin. Compared with healthy
volunteers, patients with septic shock had reduced piperacillin tissue concentrations. However, the increased
Vd of drugs is not universally noted with increased interstitial fluid volumes in some critically ill patients.
Although one study found increases in aminoglycoside Vd, another study was unable to correlate fluid shifts
with changes in the aminoglycoside Vd in critically ill surgical patients (Crit Care Med 1988;16:327-30).

D. Protein Binding – Drugs can bind to plasma proteins such as albumin, AAG, lipoproteins, and cortisol-
binding protein. Albumin and AAG are important in critically ill patients. Albumin usually binds to
acidic drugs (e.g., diazepam, phenytoin), whereas AAG binds to basic drugs (e.g., lidocaine, diltiazem). Of
importance, their concentrations change during various states of critical illness. Albumin concentrations
usually decrease (negative acute phase reactant) under stress, whereas AAC concentrations increase
(positive acute phase reactant). The following equation represents the calculation of Vd:

Vd = ( )
fu
f ut Vt + Vp
where f u is the fraction unbound in the plasma, f ut is the fraction unbound in the tissues, Vt is the volume
of tissue, and Vp is the volume of plasma. When the plasma concentration of albumin decreases, the f u of a
drug increases. This increase results in an increased Vd. Conversely, increases in AAG plasma concentra-
tions decrease the f u of a drug bound to AAG due to increased protein binding, ultimately decreasing the
Vd for that drug.

ACCP Updates in Therapeutics® 2022: Critical Care Pharmacy Preparatory Review and Recertification Course

144
Pharmacokinetics/Pharmacodynamics

1. The clinical relevance of this was noted when a decrease in the Vd of lidocaine correlated with an
increase in AAG in post-cardiac surgery patients. It was suspected that arrhythmias were caused by
these PK changes (Clin Pharmacol Ther 1984;35:617-26).
2. Table 1 provides examples of drugs used in critically ill patients that bind to albumin and AAG.

Table 1. Extraction Ratio (ER) and Protein Binding of Select Drugs Used in Critically Ill Patients
Protein Binding High ER Drugsa Intermediate ER Drugsa Low ER Drugsa
Carbamazepine
Ceftriaxone
Aspirin Dexamethasone
Morphine
Carvedilol Diazepam
Propofol
Albumin Midazolam Diltiazem
Propranolol
Omeprazole Itraconazole
Verapamil
Phenytoin
Valproic acid
Warfarin
Fentanyl
Lidocaine Carbamazepine
AAG Midazolam
Propranolol Diltiazem
Verapamil
a
ER is addressed in section V. Metabolism.
AAG = α1-acid glycoprotein.

E. pH – Acid-base disorders are common among the critically ill population. Although these disorders are
treatable, they create plasma pH changes that could affect drug distribution. Most drugs are either weak acids
or bases and exist in either the ionized or the nonionized state, depending on the surrounding environment.
Nonionized drugs penetrate cell membranes more easily than do ionized drugs. Therefore, a drug in the
ionized state would be expected to have a smaller Vd than when in the nonionized state. Theoretically, a
drug that is a weak acid in a patient experiencing acidemia would be expected to have a larger Vd, whereas
a basic drug would have a smaller Vd in the same patient. Although the potential exists to correlate plasma
pH changes with changes in drug Vd, evidence in humans is lacking.

F. Extracorporeal Membrane Oxygenation: Use of extracorporeal membrane oxygenation (ECMO) in critically


ill patients has increased over the years. Depending on the institution and clinician expertise, ECMO can
play a significant role in the care of critically ill patients with respiratory or circulatory failure. With
implementation of this therapy, changes in the PK of medications have been described. The most common
alteration reported is an increase in Vd. Use of ECMO therapy consists of a large volume of blood being
extracted from the patient through tubing (generally polyvinyl chloride), circulated through an oxygenator
and a heat exchanger before the newly oxygenated blood is returned to the patient. Depending on the type of
circuit and/or pump used for ECMO, the volume required to prime the circuit can increase the Vd of some
commonly used medications, especially those with a small Vd. Important factors to consider when evaluating
the PK changes in medications when ECMO is implemented include (1) data evaluating changes are limited
and (2) most data analyses available are in neonates and pediatric patients; thus, the published data will not
always translate to the same effects in adults. Underdeveloped kidney and liver function in neonates translate
to altered PK that may not be present in critically ill adults. Adult data analyses are limited to mostly case
reports and retrospective data. Prospective studies include evaluations of oseltamivir, vancomycin, and

ACCP Updates in Therapeutics® 2022: Critical Care Pharmacy Preparatory Review and Recertification Course

145
Pharmacokinetics/Pharmacodynamics

meropenem (AACN Adv Crit Care 2018;29:246-58). According to the prospective data, the degree of change in
PK created by ECMO may be less than that found in retrospective studies, at least for the studied medications.
The generally anticipated pharmacokinetic changes created by ECMO are the result of three factors: (1) the
ECMO circuit tubing and membrane oxygenator may bind medications, causing drug sequestration, and the
resulting PK change expected is an increased Vd; (2) circuit priming fluid type, fluid pH, and volume potentially
increase in medication Vd; and (3) as the ECMO circuit ages, medication binding becomes saturated, creating
a scenario in which patient medication requirements may return back to pre-ECMO dosing. Several ex vivo
studies have demonstrated loss of drug in the ECMO circuit, including fentanyl and midazolam. This effect
should prompt clinicians to monitor patients closely for proper analgesia and sedation during ECMO treatment
(Crit Care 2015;20:40., Intensive Care Med 2007;33:1018-1024.) Other PK changes expected with ECMO relate
to the critically ill state of the patient and are discussed throughout this chapter. Of note, these concepts are
generalizations, and data for specific drugs may differ. For example, an ex vivo study investigating drug binding
to ECMO circuits found that ciprofloxacin recovery rates were 96%, even though the drug was lipophilic and
expected to bind to circuit tubing (Crit Care 2015;19:164). A prospective, observational, pilot study evaluated
β-lactam and aminoglycoside PK parameters in patients receiving ECMO. Study results showed variable
achievement of PK objectives, which led the authors to recommend therapeutic drug monitoring for patients
receiving ECMO concurrently with β-lactam or aminoglycoside antibiotics (Anaesth Crit Care Pain Med
2019;38:493-97).

Patient Cases

3. R.H. is 20-year-old man who presents to the emergency department with nausea and vomiting. His vital
signs are significant for heart rate 130 beats/minute, blood pressure 98/62 mm Hg, and respiratory rate 28
breaths/minute. Laboratory tests reveal an arterial blood gas significant for a pH of 7.11, Pco2 of 18 mm Hg,
and sodium bicarbonate of 5.2 mEq/L. His basic metabolic panel is significant for potassium concentration
5 mEq/L, BUN 22 mg/dL, SCr 1.4 mg/dL, and blood glucose 400 mg/dL. Which changes would be most
likely to occur in the Vd of a weak acid like ciprofloxacin in this patient?
A. Increased because of decreased ionization.
B. Decreased because of increased ionization.
C. No change because of no change in ionization.
D. Decreased because of decreased ionization.

4. B.B. is a 62-year-old woman admitted to the ICU for septic shock. She has received 6 L of crystalloids in
the past 24 hours, including her initial resuscitation and maintenance fluids. Which antibiotic would be most
likely to have a Vd similar to that in normal individuals?
A. Tobramycin.
B. Daptomycin.
C. Levofloxacin.
D. Cefepime.

ACCP Updates in Therapeutics® 2022: Critical Care Pharmacy Preparatory Review and Recertification Course

146
Pharmacokinetics/Pharmacodynamics

V.  METABOLISM

A. Introduction – The predominant location for drug metabolism is the liver, but it can include tissues such as
the GI tract, kidneys, lung, and brain. The greatest extent of knowledge regarding drug metabolism and,
more importantly, changes in critically ill patients relates to hepatic metabolism. Therefore, this section
will focus largely on changes in the hepatic metabolism of drugs, specifically high and low extraction ratio
drugs. However, the next section briefly discusses renal metabolism because the clinical ramifications of
drug metabolism in the kidney are a potential area for future research.

B. Renal Metabolism – There is evidence that the kidneys express the CYP isoenzymes 2B6 and 3A5.
Data suggest that CYP 2C8, 2C9, and 3A4 are also expressed in the kidneys. In addition, UGT (UDP-
glucuronosyltransferase) enzymes 1A9 and 2B7 are abundantly expressed in the kidneys and play a role
in the glucuronidation of drugs. However, no data describe how changes in critically ill patients affect
drug metabolism in the kidneys by these enzymes. As evidenced by increased hypoglycemic events and
lower insulin requirements in critically ill patients with AKI, clinically relevant changes in renal insulin
metabolism occur, but the exact mechanisms for these are not well characterized (Nutrition 2011;27:766-72).

C. Hepatic Metabolism – Hepatic clearance refers to the volume of blood that is completely cleared of drug by
the liver per unit of time. The ability of the liver to metabolize drugs depends on three physiologic variables:
hepatic blood flow, drug protein binding, and the intrinsic activity of hepatic enzymes. When evaluating an
intravenously administered drug (bioavailability of 1), clearance by the liver can be simply represented by
the following equation:
CLH = Q × E
where CLH is the hepatic clearance, Q is the hepatic blood flow, and E is the hepatic extraction ratio. The
hepatic extraction ratio can further be described by the following equation:
f u × CLint
E=
Q + f u × CLint
where f u is the fraction unbound in the plasma, CLint is the intrinsic hepatic clearance, and Q is hepatic blood
flow. The hepatic extraction ratio is classified by the fraction of drug removed during one pass through the
liver and can range from 0 to 1. It can be separated into high (greater than 0.7), intermediate (0.3–0.7), and
low (less than 0.3) categories. The extraction ratio would be zero when the liver does not metabolize a drug
and 1 when CLH depends entirely on hepatic blood flow. The effect of changes in critical illness depends
on the extraction ratio of the drug. Table 1 lists select high extraction ratio and low extraction ratio drugs.

D. High Extraction Ratio Drugs


1. Drugs with a high hepatic extraction ratio are metabolized by hepatic enzymes and are thus cleared by
the liver. In drugs with high extraction ratios, clearance does not vary with changes in hepatic enzymatic
activity and is primarily limited by hepatic blood flow. Mathematically, this can be represented by:
f u × CLint >>> Q
Given the previously stated relationship, CLH can be simplified to:
CLH = Q

ACCP Updates in Therapeutics® 2022: Critical Care Pharmacy Preparatory Review and Recertification Course

147
Pharmacokinetics/Pharmacodynamics

As previously noted, clearance of a drug pertains to removal of the drug from the blood. Therefore,
the effect on plasma drug concentration will affect the efficacy of the drug. Because only the free
drug is available to produce a clinical effect, the unbound steady-state concentration (Cssu) is extremely
important. The total concentration, otherwise called the steady-state concentration (Css) of hepatically
metabolized drugs, can be represented by the following equation:
dose
Css =
CLH
where Css is the steady-state concentration (for both protein bound and unbound drugs) and dose is the
rate of drug input. Because CLH = Q for high extraction ratio drugs, the equation can be modified to:
dose
Css =
Q
The unbound steady-state concentration for a high extraction ratio is represented by the following
equation:
f u × dose
Cssu =
Q
Figure 1 shows how a change in each variable of CLH affects the Css and Cssu. For high extraction ratio
drugs, altered hepatic blood flow affects both the Css and the Cssu, whereas changes in f u affect only the
Cssu.

↓  Css
↑  Q
↓  Cssu
CLH = Q
↑  Css
↓  Q
↑  Cssu

↔  Css
↑  CLint
Css = Dose ↔  Cssu

Q ↓  CLint
↔  Css
↔  Cssu

↔  Css
↑  f u
Cssu = fu × Dose ↑  Cssu

Q ↓  f u
↔  Css
↓  Cssu

Figure 1. Effect of variable changes on steady-state and unbound steady-state concentrations of a high extraction
ratio drug.

2. Effect of increased hepatic blood flow: Animal models have shown a clear increase in splanchnic
perfusion during the hyperdynamic phase of sepsis. Critically ill patients in the hyperdynamic phase
of sepsis or septic shock have an increased cardiac output and increased hepatosplanchnic blood flow.
However, no correlation directly relating cardiac output (or an increase in cardiac output) with an
increase in splanchnic blood flow could be established in some studies. Therefore, quantification of
the increase in blood flow and the resultant increase in hepatic metabolism cannot be established. The
clinician is left to assume the potential for increased metabolism of high extraction ratio drugs and the
expected decrease in unbound steady-state concentration and possibly a reduced clinical efficacy.

ACCP Updates in Therapeutics® 2022: Critical Care Pharmacy Preparatory Review and Recertification Course

148
Pharmacokinetics/Pharmacodynamics

a. Adding an inotrope would likely improve hepatic blood flow.


b. Although data analyses in humans are lacking, an animal model of endotoxemia showed improved
hepatic blood flow after dobutamine administration.
3. Effect of decreased hepatic blood flow
a. Conditions with a low cardiac output such as hypovolemic or hemorrhagic shock, decompensated
sepsis, myocardial infarction with or without cardiogenic shock, and acute heart failure exacerbation
would be expected to cause a decrease in hepatic blood flow. Human studies to verify this assertion
are lacking. Animal models of decompensated sepsis and cardiogenic shock show reduced hepatic
blood flow, which would be expected to increase the Cssu and potentially increase the effect of, or
produce toxicity for, drugs with high extraction ratios.
b. Mechanical ventilation produces an increased intrathoracic pressure. This pressure causes a
decrease in venous return to the heart, compresses the ventricles, and reduces ventricular filling.
The result is a decrease in cardiac output (N Engl J Med 1981;304:387-92) and hepatic blood flow
(Crit Care Med 1982;10:703-5).

E. Low Extraction Ratio Drugs – Drugs with a low hepatic extraction ratio undergo a lower degree of hepatic
enzyme metabolism; thus, they are not extracted from hepatic blood flow as high extraction ratio drugs.
In drugs with low extraction ratios, clearance is limited by hepatic enzymatic activity, and clearance is
independent of hepatic blood flow. Mathematically, this can be represented by:
f u × CLint <<< Q
According to this relationship, CLH can be simplified to:

CLH = f u × CLint

Again, the Css for hepatically metabolized drugs can be represented by the following equation:
dose
Css =
CLH

where Css is the steady-state concentration and dose is the rate of drug input. Because CLH = f u × CLint for
low extraction ratio drugs, the equation can be modified to:
dose
Css =
f u × CLint
The unbound steady-state concentration for a low extraction ratio drug is represented by the following
equation:
dose
Cssu =
CLint

ACCP Updates in Therapeutics® 2022: Critical Care Pharmacy Preparatory Review and Recertification Course

149
Pharmacokinetics/Pharmacodynamics

Figure 2 shows how a change in each variable of CLH affects the Css and the Cssu. For low extraction ratio
drugs, altered CLint affects both the Css and the Cssu, whereas changes in the f u affect only the Css.

↔  Css
↑  Q
↔  Cssu
CLH = fu × CLint
↔  Css
↓  Q
↔  Cssu

↓  Css
↑  CLint
Dose ↓  Cssu
Css =
fu × CLint ↓  CLint
↑  Css
↑  Cssu

↓  Css
↑  f u
Dose ↔  Cssu
Cssu =
CLint ↓  f u
↑  Css
↔  Cssu
Figure 2. Effect of variable changes on steady-state and unbound steady-state concentrations of a low extraction
ratio drug.

F. Effect of Changes in Intrinsic Clearance


1. Drug interactions – A major mechanism for altered intrinsic clearance is not caused by changes in
critically ill patients, but it still poses a significant threat to altered metabolism of drugs. The CYP
enzymes play an important role in phase I metabolism. Many drugs used in critically ill patients are
substrates, inducers, inhibitors, or combinations of these. Critically ill patients often have complex
pharmacotherapeutic regimens that create the potential for drug interactions through the CYP system.
As in other patient populations, drug concentrations are increased when substrates are coadministered
with inhibitors of the same CYP and decreased when substrates are coadministered with inducers.
2. Inflammation – Inflammatory states play an important role in altering CYP activity. The inflammatory
cytokines interleukin (IL)-1, IL-6, and tumor necrosis factor alpha decrease the expression and activity
of CYP enzymes. Similarly, patients in early sepsis would have increased inflammatory cytokines
with the resultant depressed CYP activity. This is supported by studies showing that endotoxin
administration results in decreased CYP-mediated drug metabolism in healthy volunteers. However,
studies have not characterized a time course for the cytokine-mediated changes. Clinicians are left to
use patient response and monitoring for toxicity to determine whether drug metabolism is altered or
has returned to normal.
3. Hypothermia – Animal models have shown that hypothermia affects drugs metabolized through
the CYP system. Drugs studied in animal models include fentanyl, pentobarbital, propranolol, and
morphine. Human studies have investigated the effect of hypothermia on low extraction ratio drug
kinetics. One example showed changes in phenytoin PK during mild hypothermia. Specifically,
increased concentrations and reduced metabolism, but no changes in protein binding, were noted
during hypothermia (Ther Drug Monit 2001;23:192-7). Other drugs noted to have decreased hepatic
clearance during hypothermia are midazolam, fentanyl, remifentanil, phenobarbital, and vecuronium.

ACCP Updates in Therapeutics® 2022: Critical Care Pharmacy Preparatory Review and Recertification Course

150
Pharmacokinetics/Pharmacodynamics

4. AKI – One study investigated the effects of AKI on the hepatic metabolism of midazolam. Patients with
worsening AKI, as determined using the RIFLE (risk, injury, failure, loss, end-stage kidney disease)
criteria, had increasing midazolam concentrations. The authors hypothesized that the increased
concentrations were caused by impaired CYP3A activity (Intensive Care Med 2012;38:76-84).

G. Intermediate Extraction Ratio Drugs – Metabolism of an intermediate extraction ratio drug depends on
hepatic blood flow, intrinsic clearance, and fraction of unbound drug. Essentially, intermediate extraction
ratio drugs depend on the same variables as both the low extraction ratio and high extraction ratio drugs. As
such, they are the most complex drugs for determining how hepatic clearance will be affected in critically
ill patients. This is important because critically ill patients usually have more than one change occurring at
the same time. For example, patients with septic shock may have increased hepatic blood flow secondary
to increased cardiac output while having a decreased intrinsic clearance secondary to increased circulating
inflammatory cytokines. Quantifying the overall effect is difficult in the ever-changing critically ill patient.
The clinician is often left to monitor for the expected therapeutic outcome while being aware of the potential
toxicities.

H. Other Factors
1. TBI increases the hepatic clearance of some drugs.
a. One study found that patients with TBI had increased hepatic clearance of phenytoin during the
first 7–14 days. The authors alluded to the possibility that the increased clearance was associated
with changes in protein binding, induction of metabolism, or stress on hepatic metabolic capacity
(Clin Pharmacol Ther 1988;44:675-83).
b. Another study noted a correlation between nutritional protein intake and increased phenytoin
metabolism in patients with TBI.
c. Phase II enzymatic activity may also be enhanced in patients with TBI, as evidenced by increased
lorazepam clearance. Similar data for lorazepam were noted in thermally injured patients. These
data suggest that phase II metabolism can be affected by critical illness.
2. Hepatic failure – Hepatic failure will significantly affect drug dosing in the critically ill patient. See
the Hepatic Failure/GI/Endocrine Emergencies chapter for more information regarding drug dosing in
hepatic failure.

Patient Cases

5. A.P. is a 35-year-old woman admitted to the ICU for an acute asthma exacerbation. She was intubated and
required mechanical ventilation. She is prescribed morphine for pain control. Which best describes the
effect of mechanical ventilation on morphine unbound concentrations?
A. Increases oxygenation delivery to the liver and increases the unbound concentration.
B. Decreases hepatic blood flow and increases the unbound concentration.
C. Increases cytokine production and decreases the unbound concentration.
D. Cannot affect the unbound concentration.

ACCP Updates in Therapeutics® 2022: Critical Care Pharmacy Preparatory Review and Recertification Course

151
Pharmacokinetics/Pharmacodynamics

Patient Cases (continued)

6. J.M. is a 65-year-old woman in the ICU who develops atrial fibrillation. Her rate is controlled using a diltiazem
infusion of 10 mg/hour. J.M. develops a fever and leukocytosis. She is empirically initiated on vancomycin
1 g intravenously every 12 hours, piperacillin/tazobactam 2.25 g intravenously every 6 hours, and flucon-
azole 400 mg intravenously every 24 hours. Which PD response would you most expect in J.M.?
A. Increased heart rate caused by an increased intrinsic clearance of diltiazem.
B. Decreased heart rate caused by a decreased intrinsic clearance of diltiazem.
C. Increased heart rate caused by a decreased unbound fraction of diltiazem.
D. Decreased heart rate caused by an increased unbound fraction of diltiazem.

7. P.M. is receiving phenytoin for the treatment of posttraumatic seizures. You measure a total phenytoin
concentration, which is 8 mcg/mL. You calculate the adjusted concentration according to P.M.’s
hypoalbuminemia (albumin of 2.5 g/dL) and determine the concentration to be around 13 mcg/mL. Which
best depicts why this adjustment was needed?
A. The unbound phenytoin concentration increased because of an increased unbound fraction of phenytoin.
B. The total phenytoin concentration increased because of an increased unbound fraction of phenytoin.
C. The unbound phenytoin concentration decreased because of an increased unbound fraction of phenytoin.
D. The total phenytoin concentration decreased because of an increased unbound fraction of phenytoin.

8. C.P. is a 50-year-old man admitted to the medical ICU for diabetic ketoacidosis. His medical history is sig-
nificant for hypertension, type 1 diabetes, and a myocardial infarction 2 years ago. He quit smoking last year
and drinks alcohol only occasionally. His vital signs are significant for heart rate 125 beats/minute, blood
pressure 95/65 mm Hg, and respiratory rate 22 breaths/minute. His significant hypovolemia contributed to
the development of AKI. His current SCr is 2.8 mg/dL. His blood glucose is significantly elevated at 350 mg/
dL. He will be initiated on a continuous intravenous infusion of insulin to correct his blood glucose. Which
factor is most important to consider when dosing insulin in C.P.?
A. Decreased renal metabolism of insulin.
B. Increased Vd of insulin.
C. Increased hepatic metabolism of insulin.
D. Decreased receptor binding of insulin.

VI.  EXCRETION

A. Renal Excretion
1. For most drugs, the kidneys are the primary site for excretion of the parent drug, metabolites, or both.
Urinary excretion of a drug depends on filtration, secretion, and reabsorption. Patients in the ICU may
have increased, decreased, or normal renal excretion of drugs. The state of renal excretion depends
on many variables and can change rapidly. This is especially true in ICU patients, for whom a clinical
condition can contribute to both increased and decreased excretion, depending on how that condition
progresses.

ACCP Updates in Therapeutics® 2022: Critical Care Pharmacy Preparatory Review and Recertification Course

152
Pharmacokinetics/Pharmacodynamics

2. Filtration
a. Glomerular filtration rate (GFR) is the variable most widely used to describe kidney function. The
National Kidney Foundation defines normal kidney function as 140 ± 30 mL/minute/1.73 m 2 for
healthy young men and 126 ± 22 mL/minute/1.73 m2 for healthy young women. Although there is
no standard definition for increased GFR (ARC), an increase of 10% above the upper end of normal
(greater than 160 mL/minute/1.73 m2 in men and greater than 150 mL/minute/1.73 m2 in women)
has been proposed (Crit Care 2013;17:R35).
b. ARC – Conditions such as surgery, trauma, burns, and sepsis have been associated with increased
renal blood flow. This is usually believed to be caused by an increased cardiac output coupled
with vasodilation. The resulting ARC is believed to be a response to an inflammatory insult (e.g.,
systemic inflammatory response syndrome).
i. One study found glomerular hyperfiltration in 17.9% of patients admitted to the ICU. Most of
these patients were younger and admitted for multi-trauma or surgery (Anaesth Intensive Care
2008;36:674-80). These data are supported by a study showing young, postoperative trauma
patients with peak creatinine clearance (CrCl) values as high as 190 mL/minute/1.73 m 2. A
more recent study found significant risk factors for the ARC to be age 50 or younger, trauma,
and a modified sequential organ failure score of 4 or less. Finally, a study of 133 trauma
patients found age 56 or younger, SCr less than 0.7 mg/dL, and male sex to be independent risk
factors for ARC. From this, study investigators developed a model to predict ARC in patients,
the ARC in trauma intensive care (ARCTIC) score. A score of 6 or higher had a sensitivity
of 0.843, a specificity of 0.682, a positive predictive value of 0.843, and a negative predictive
value of 0.682.
ii. A study of burn patients found an increase in iohexol clearance with a median value of 155
mL/minute/1.73 m2 on day 1 of admission. In this small study, clearance had returned to the
expected baseline of 122 mL/minute/1.73 m 2 by day 7 (Burns 2010;36:1271-6). In addition,
several studies have shown increased excretion of renally eliminated drugs in burn patients,
likely related to the second phase (hyperdynamic) of burn injury, which generally begins
48 hours after the burn injury. Examples include vancomycin, ciprofloxacin, imipenem,
fluconazole, and aminoglycosides.
iii. Fluid administration would be expected to improve cardiac output and thus renal blood flow.
Animal studies have confirmed that the administration of crystalloids can transiently increase
CrCl. Sheep administered normal saline and 3% hypertonic saline have a significantly higher
calculated CrCl than controls. However, no human studies have verified ARC in critically ill
patients after fluid administration.
iv. Vasoactive drugs would be expected to improve cardiac output and thus renal blood flow.
However, studies of humans did not corroborate this expectation and, instead, were only
able to establish an improvement in CrCl in a subset of patients with impaired CrCl before
norepinephrine administration. Although studies were unable to show development of ARC,
patients receiving vasopressors for shock states might be expected to have normal renal blood
flow and CrCl, assuming they are not experiencing AKI. The duration of ARC is not well
established, but the peak CrCl appears to occur at about days 4–5 in most studies, with CrCl
returning to normal by day 7 in one study.
c. Impaired renal clearance
i. Decreased renal excretion of drugs during AKI is the most widely applicable change occurring
in critically ill patients. Depending on the patient population and definition used, the incidence
of AKI in ICU patients can be as high as 78%. AKI significantly affects the excretion of
renally eliminated drugs, and dosing modifications must be made in these situations.

ACCP Updates in Therapeutics® 2022: Critical Care Pharmacy Preparatory Review and Recertification Course

153
Pharmacokinetics/Pharmacodynamics

ii. The Kidney Disease: Improving Global Outcomes (KDIGO) clinical practice guidelines for
AKI recommend that staging of AKI be done using the KDIGO AKI criteria. However, these
guidelines have no specific recommendations regarding drug dosing.
iii. A clinical update to the 2010 KDIGO guidelines does recommend how to approach drug dosing
in critically ill patients with AKI. Because of the complicated picture of AKI in critically ill
patients, however, these recommendations are not as precise as the recommendations for drug
dosing in CKD. In fact, the authors note that most renal dose adjustment recommendations in
the literature and from the FDA (U.S. Food and Drug Administration) are based on data from
patients with CKD (Kidney Int 2011;80:1122-37).
iv. The update recommends a stepwise approach to adjusting drug-dosing regimens in patients
with AKI (Box 2).

Box 2. Recommended Steps for Assessing and Adjusting Drug Regimens in Patients with AKI
Step 1 – Assess the following
Demographic information
Medical history (including history of renal disease)
Current clinical information
Current laboratory information
DNA polymorphisms
Step 2 – Estimate GFR (use the best equation according to patient factors)
Age
Body size
Ethnicity
Concomitant diseases
Step 3 – Review current medications
Identify drugs needing individualized dosing
Step 4 – Calculate individualized treatment regimen
Determine treatment goals (PK or PD values)
Calculate dosage regimen (according to drug PK and changes noted in the patient)
Step 5 – Monitor regimen
Drug response
Signs or symptoms of toxicity
Drug concentrations (if available)
Step 6 – Revise regimen
Adjust regimen according to patient response
Adjust regimen according to changes in patient status
AKI = acute kidney injury; GFR = glomerular filtration rate; PD = pharmacodynamic(s); PK = pharmacokinetic(s).

v. The recommendations include using the estimated GFR (eGFR) or CrCl to assess renal function
for drug dosing. The update provides several equations that can be used to estimate kidney
function, including the Cockcroft-Gault, Modification of Diet in Renal Disease (MDRD), and
Chronic Kidney Disease Epidemiology Collaboration (CKD-EPI) equations. The MDRD
equation is described as follows:

GFR = 175.6 × SCr -1.154 × age -0.212 × 1.212 (if black) × 0.742 (if female)

ACCP Updates in Therapeutics® 2022: Critical Care Pharmacy Preparatory Review and Recertification Course

154
Pharmacokinetics/Pharmacodynamics

where GFR is in milliliters per minute/1.73 m2, SCr is measured in the laboratory using isotope
dilution mass spectroscopy (IDMS), and age is in years. If the laboratory measuring the SCr
does not use IDMS, the following equation should be used:

GFR = 186.3 × SCr -1.154 × age-0.203 × 1.212 (if black) × 0.742 (if female)
vi. The Cockcroft-Gault equation is used to estimate the CrCl as follows:
(140 – age) weight
CrCl = × 0.85 (if female)
SCr × 72
where CrCl is in milliliters per minute, weight is in kilograms, and SCr is in milligrams per
deciliter.
vii. The CKD-EPI equation can be calculated as follows:

GFR∞ = 141 × min(SCr/κ,1)α × max(SCr/κ, 1)-1.209 × 0.993Age × 1.159 [if black] × 1.018 [if female]
where SCr is serum creatinine (mg/dL), κ is 0.7 for females and 0.9 for males, α is -0.329 for
females and -0.411 for males, min indicates the minimum of SCr/κ or 1, and max indicates the
maximum of SCr/κ or 1.
viii. The update also notes that the most important factor when determining kidney function is
having at least one GFR estimate for all patients.
3. Secretion and reabsorption: However, it is difficult to study changes in drug secretion and reabsorption
in patients. Therefore, data are not available to describe the clinically important changes in these two
variables in critically ill patients.
4. Renal replacement therapies: See the Acute Kidney Injury chapter for more information about renal
replacement therapies and drug dosing.
a. Patients receiving renal replacement therapy with a diagnosis of AKI may require hemodialysis
or hemofiltration. The choice of dialytic technique depends on the institution, clinician expertise,
patient hemodynamic stability, and access to various types of renal replacement machines. Drug
removal by dialysis depends on the method of dialysis used.
b. Acute intermittent hemodialysis: Intermittent hemodialysis can significantly contribute to the
excretion of drugs, whereas other drugs are not appreciably removed by hemodialysis. Removal of
drugs during hemodialysis depends on the size of the molecule, Vd, protein binding, and type of
dialysis filter (specifically the membrane size).
c. Continuous renal replacement therapies (CRRT): CRRT refers to several methods of renal
replacement. Many studies have investigated the effect of CRRT on drug removal. Considerable
variability exists in the type of CRRT used. The 2010 clinical update to the KDIGO guidelines
suggests the following equation as one option to determine the appropriate drug dose in CRRT:

dose = dosen ( CLnonrenal + (Qeff × SC)


CLnorm )
where dose is the desired dose for CRRT, dose n is the normal dose of a drug, CLnonrenal is the
nonrenal clearance of a drug, Qeff is the effluent rate, SC is the sieving coefficient, and CLnorm is
the normal clearance of the drug. As the effluent rate (in milliliters per minute) increases, so does
solute clearance. Therefore, it is the most important CRRT parameter pertaining to drug removal.

B. Hepatic Excretion – Hepatic excretion of drugs is less important for most drugs than renal excretion.
However, excretion of drug in the bile can potentially be affected by critical illness. This is evidenced by
changes in the clearance of some neuromuscular blocking agents.

ACCP Updates in Therapeutics® 2022: Critical Care Pharmacy Preparatory Review and Recertification Course

155
Pharmacokinetics/Pharmacodynamics

1. A study of nine patients undergoing surgery for total biliary obstruction showed a significant increase
in pancuronium half-life compared with normal patients (270 minutes vs. 132 minutes, p<0.001). The
urinary excretion of pancuronium and its metabolites did not change. This suggests that the increased
half-life was caused by the decreased hepatic excretion of pancuronium (Br J Anaesth 1977;49:1103-8).
2. Results were similar for vecuronium in patients with cholestasis, in which the mean half-life was 98
minutes in patients with cholestasis and 58 minutes in normal patients (Br J Anaesth 1986;58:983-7).

C. Pulmonary Excretion – Pulmonary excretion is important for volatile gases such as anesthetics. It can be
hypothesized that impaired gas exchange (e.g., acute respiratory distress syndrome) affects the body’s
ability to remove volatile gases. However, data are lacking regarding changes in critically ill patients that
affect their ability to excrete anesthetics.

VII.  PHARMACODYNAMICS

A. Pharmacodynamics refers to the biochemical and physiologic effects of a drug, specifically those related to
the mechanism of action.

B. This term also pertains to drug/receptor binding and clinical effect. Most clinicians use the term to refer to
the physically identifiable effect of a drug in a patient. For example, the PD effect of an opioid is the relief of
pain reported by the patient. However, the PD effect of some drugs is not readily observable. For example,
the PD effect of proton pump inhibitors is an increase in gastric pH. Few PD studies of critically ill patients
are reported in the medical literature, and most pertain to antibiotic therapy.
1. Antibiotics general information: PD studies of antibiotics use models to estimate the combined effects
of the patient population PK of specific antibiotics and the MIC for select bacteria. These techniques
usually allow a calculation of the desired PD outcome. Antibiotics generally fall into three PD categories,
which correlate with efficacy: (1) time-dependent killing (T>MIC), (2) concentration-dependent killing
(Cmax/MIC), and (3) a combination of time- and concentration-dependent killing (ratio of area under the
curve to the minimum inhibitory concentration for the bacterial pathogen [AUC/MIC]).
2. β-Lactam antibiotics: For β-lactam antibiotics, the PD parameter of the free drug concentration time
above the MIC (fT>MIC) is used to predict treatment success. This is reported as a percentage of time
the free drug concentration remains above the MIC. The ideal f T>MIC is 100%. However, PD studies
of β-lactam antibiotic use in critically ill patients have found that a low percentage of patients will
achieve the desired PD targets (Crit Care 2011;15:R206). These failures are often attributed to clinically
important changes that can occur rapidly in critically ill patients (e.g., ARC). As such, epidemiologic
studies have tried to determine a breakpoint at which clinical success is achieved. Studies vary, and
the suggested breakpoint is 50%–100% (fT>MIC) (Br J Clin Pharmacol 2012;73:27-36), with others
suggesting cutoffs of about 40% for carbapenems, 50% for penicillins, and 50%–75% for cephalosporins
(Clin Infect Dis 2008;26:1-10). Modeling usually suggests improved PD of β-lactams when using
prolonged or continuous infusions. Many institutions have adopted the practice of prolonged infusions.
This is supported by quasi-experimental and retrospective studies showing improved outcomes such as
improved clinical cure, improved microbiological cure, and reduced morbidity and mortality. However,
prospective controlled clinical trials have had mixed results (J Crit Care 2014;29:1089-95; Am J Respir
Crit Care Med 2015;192:1298-305). There are many reasons why there appears to be a discrepancy
between PD modeling studies and controlled clinical trials; patient variability, dosing variability, and
disease severity seem to be important factors (Ann Intensive Care 2012;2:37).
3. Aminoglycosides: Aminoglycoside bacterial killing is based on the ratio between the maximum
drug concentration and the MIC for the bacterial pathogen (Cmax/MIC), or concentration-dependent
killing. Efficacy was noted when patients were pooled from four controlled clinical trials and peak

ACCP Updates in Therapeutics® 2022: Critical Care Pharmacy Preparatory Review and Recertification Course

156
Pharmacokinetics/Pharmacodynamics

concentrations, MIC values, and clinical response were evaluated. Peak-to-MIC ratios of 8–10 resulted in
around a 90% clinical response (J Infect Dis 1987;155:93-9). In patients with gram-negative bacteremia,
early therapeutic peak concentrations were a significant discriminating factor for mortality. According
to these and other data, once-daily aminoglycoside dosing has been used. Taking advantage of high
peak concentrations maximizes the PD of aminoglycosides. Variability in the Vd of aminoglycosides in
critically ill patients, together with concern for ARC in this population, raises issues about appropriately
dosing these agents, especially in critically injured trauma patients, whose drug concentrations can be
undetectable for more than 12 hours (J Trauma 2000;49:869-87).
4. Vancomycin: The PD parameter that best describes vancomycin efficacy is the AUC/MIC. Several
studies have evaluated the free 24-hour AUC/MIC (fAUC0-24/MIC), or the AUC × 50% protein binding/
MIC. Current guidelines use the available literature to recommend an AUC/MIC of 400 or greater for
effectiveness and less than 600 for safety. (Am J Health-Syst Pharm 2020;77:835-63). The guidelines
recommending continuous infusion regimens are an acceptable alternative when conventional dosing is
unable to achieve the target AUC. They also note the intravenous access issues presented for critically ill
patients when continuous infusion regimens are employed (Clin Infect Dis 2020:71;1361-4). Specifically,
vancomycin has been demonstrated to be Y-site incompatible with multiple β-lactams, moxifloxacin,
propofol, phenytoin, methylprednisolone, and furosemide—all medications that are commonly used
in the ICU setting. As such, clinicians should closely monitor vancomycin coadministration with
other medications. Of interest is a retrospective study of vancomycin-associated nephrotoxicity in
critically ill patients. In this study, intermittent dosing was associated with a significantly higher risk
of nephrotoxicity than continuous infusion (odds ratio 8.2; p<0.001) (Crit Care Med 2014;42:2527-36).
Of note, more aggressive dosing may be required in critically ill patients. Doses as high as 20 mg/kg
administered as often as every 6 hours were needed to optimize PK variables in critically injured trauma
patients being treated for ventilator-associated pneumonia (J Trauma Acute Care Surg 2012;72:1478-83).
5. Fluoroquinolones: Similar to the efficacy of aminoglycosides, the efficacy of the fluoroquinolones is
based on a Cmax/MIC (10 or greater), and the fluoroquinolones have a post-antibiotic effect against
gram-negative and gram-positive bacteria. PD studies have shown that the fAUC0-24/MIC is associated
with bacterial eradication. In one study of lower respiratory tract infections treated with ciprofloxacin,
an AUC0-24/MIC of 125 was associated with the percent probability of 80% for clinical cure (Antimicrob
Agents Chemother 1993;37:1073-81). However, it is difficult to incorporate these PD variables into
fluoroquinolone dosing in individual critically ill patients. Measuring fluoroquinolone serum
concentrations is not routine; therefore, it is difficult to determine whether the PD targets have been met
in an individual patient.
6. Nonantibiotic drugs: PD studies of other drugs in critically ill patients are sparse.
a. The PD parameter for continuous infusions of many anticoagulants is change in the activated
partial thromboplastin time. Unfractionated heparin infusions are usually predictable in most
patient populations. However, in critically ill patients, just less than one-half of patients (44%)
did not reach a therapeutic activated partial thromboplastin time within 24 hours of initiating
a heparin continuous infusion (Neth J Med 2013;71:466-71). Concern for a variable response in
critically ill patients has led to the development of dosing nomograms/protocols. Researchers have
found a shortened time to therapeutic activated partial thromboplastin times in critically ill patients
receiving unfractionated heparin and direct thrombin inhibitors (argatroban and bivalirudin).
b. As with antibiotic PD studies, most PD studies of other drugs have shown a decreased response in
critically ill patients. For example, critically ill patients in septic shock had a reduced response to
dobutamine compared with critically ill patients without septic shock and with normal volunteers
(Crit Care Med 1993;21:31-9). Trauma patients with edema have lower AUCs for anti-Xa activity
than do non-edematous patients (J Trauma 2005;59:1336-43). Mechanically ventilated patients
with chronic obstructive pulmonary disease were studied for covariates affecting acetazolamide
therapy. Mixed-effects modeling found the Simplified Acute Physiology Score II, serum chloride
concentrations, and concomitant corticosteroids to be the main covariates interacting with
acetazolamide PD.

ACCP Updates in Therapeutics® 2022: Critical Care Pharmacy Preparatory Review and Recertification Course

157
Pharmacokinetics/Pharmacodynamics

VIII.  THERAPEUTIC DRUG MONITORING

A. Therapeutic drug monitoring (TDM) refers to the measurement of medication concentrations in the blood.
The focus of TDM is on drugs with a narrow therapeutic index and aims to achieve two goals: (1) maximize
efficacy and (2) reduce toxicity. Use of TDM in critically ill patients is extremely important because changes
in the PK variables previously described can result in less-than-desirable drug concentrations. Table 2
highlights commonly used medications in the ICU and their therapeutic ranges. One of the main limitations
of TDM is the lack of clinically available assays. In addition, assays for some drugs may not be sufficiently
cost-effective to routinely conduct in certain institutions. In addition, assays for some drugs may not be
sufficiently cost-effective or timely, thus limiting their routine use in some institutions. These issues usually
result in TDM for a very limited spectrum of drugs.
1. Monitoring of blood concentrations depends on the intended use and interpretation of those
concentrations. Most TDM occurs as a method to confirm a therapeutic concentration in a patient
with signs and/or symptoms of toxicity or decreased efficacy. In this case, a concentration is measured
during the appropriate time interval (Table 2), and a clinician interprets the concentration. If needed,
the clinician modifies the drug dosing according to clinical experience. This method may produce
variable results. Critically ill patients require important considerations. For example, if extended-
interval dosing is being used, the likelihood of an increased Vd must be considered. Patients with ARC
have the potential to have a prolonged drug-free period. Finally, the status of a critically ill patient can
change rapidly. Monitoring for decreased kidney function is essential to avoid accumulation.

Table 2. TDM Ranges for Select Drugs Used in Critically Ill Patients
Drug Timing of Blood Sample Therapeutic Rangea Unbound Therapeutic Range
Trough < 8 mg/L
Amikacin —
Peak (traditional) 20–30 mg/L
Carbamazepine Trough 4–12 mg/L 0.5–4 mcg/mL
Cyclosporine Trough 50–500 mcg/L —
Digoxin Trough (8–24 hr postdose) 0.6–2 mcg/L 0.4–0.9 ng/mL
Trough (traditional) < 2 mg/L
Gentamicin Trough (extended interval) Undetectable —
Peak 5–10 mg/L
Lidocaine Peak 1.5–5 mg/L —
Phenobarbital Trough 10–40 mg/L —
Phenytoin Trough 10–20 mg/L 1–2.5 mcg/mL
Trough (traditional) < 2 mg/L
Tobramycin Trough (extended interval) Undetectable —
Peak 5–10 mg/L
Peak and Trough to calculate AUC 400–600 mg ×
Vancomycin —
AUC hour/L
Therapeutic ranges for antibiotics may be modified on the basis of pharmacodynamic target attainments (e.g., peak to MIC ratio).
a

TDM = therapeutic drug monitoring.

ACCP Updates in Therapeutics® 2022: Critical Care Pharmacy Preparatory Review and Recertification Course

158
Pharmacokinetics/Pharmacodynamics

2. For certain intravenously administered drugs (e.g., aminoglycosides, vancomycin), several


concentrations can be measured when the drug is at steady state. The patient’s specific PK can be
determined and used to tailor the drug-dosing regimen. The following PK equations can be used to
calculate the various kinetic variables.

Determination of the elimination rate constant:


C1
ln
C2
ke =
Δ in time
where ke is the elimination rate constant, C1 is the measured peak concentration, C2 is the measured
trough concentration, and Δ in time is the elapsed time from C1 to C2.

Determination of the drug half-life:


0.693
t1/2 =
ke
where t1/2 is the calculated half-life.

Determination of the calculated peak concentration:

Cmax = C1 (eke (t))

where Cmax is the calculated peak concentration and t is the time between the C1 and the end of the
intravenous infusion.

Determination of the calculated trough concentration:

Cmin = C2 (e-ke (t))

where Cmin is the calculated trough concentration and t is the time between C2 and the beginning of the
next dose.

Determination of the drug Vd:


dose (1 – e-ke (t ))inf

Vd = ×
tinf × ke Cmax – Cmin × e-ke (t inf)

where tinf is the duration of the drug infusion.

Determination of the new dosing interval:

τ=
ln ( Cmax,desired
Cmin,desired ) + tinf
ke
where τ (tau) is the new dosing interval, Cmax,desired is the desired peak concentration for the new dosing
regimen, and Cmin,desired is the desired trough concentration for the new dosing regimen.

ACCP Updates in Therapeutics® 2022: Critical Care Pharmacy Preparatory Review and Recertification Course

159
Pharmacokinetics/Pharmacodynamics

Determination of the new dose:


(l – e-ke(τ))
dose = Cmax,desired × ke × Vd × tinf ×
(l – e-ke (t ))
inf

3. One drawback of this method is that it assumes that the PK variables obtained (e.g., vancomycin trough)
correlate with PD variables (AUC/MIC). Although this may be true in many cases, some advocate
incorporating PD into individualized drug dosing. Alternative methods use PK variables from previous
patients (population PK) to estimate the PK in an individual patient. These methods usually require
complicated mathematical calculations, many of which are not practical for clinical use. However,
with the development of PK software, clinicians can carry out complex calculations. Many software
programs are available for clinician use in patient care. More advanced modeling using the Bayesian
method has also been proposed to address the issues posed when using population PK in patient groups
that may not be well represented in the population. Using software with population PK variables from
non-critically ill patients to interpret PK in critically ill patients could result in errors in designing the
appropriate drug-dosing regimen.
4. A recent position paper provides a more detailed review of the PK, PD, and TDM of commonly used
antimicrobials in critically ill patients and notes recommendations that certain antimicrobials should
undergo TDM (Intensive Care Med 2020;46:1127-53). The aminoglycosides and vancomycin are included
on this list because they have routinely been monitored in critically ill patients. The recommendations
also include the β-lactam class of antibiotics, linezolid, and the antifungal voriconazole. Finally, the
document provides PK and PD target variables for these drugs. The biggest issue, as previously stated,
is the ability for local laboratories to measure concentrations and report them in a time frame that is
clinically reasonable. The reader is referred to the position paper for a more in-depth review.

B. Drug Nomograms/Protocols – A nomogram is a diagram representing the relationship between three or


more variables using scales arranged such that one variable can be determined if the other variables are
known. A classic drug-dosing example is the once-daily aminoglycoside nomogram (Antimicrob Agents
Chemother 1995;39:650-5). The nomogram itself allows the user to determine only one variable (usually,
the dosing interval) using other variables (time from infusion to drug concentration and the measured drug
concentration). Many institutions incorporate nomograms into drug-dosing protocols or pathways but still
call them drug-dosing nomograms. Of note, many of these nomograms (e.g., Hartford) were not developed
in critically ill patients and likely do not consider the potential for alterations in PK. One of the most
commonly used drug-dosing nomograms was developed for continuous infusions of unfractionated heparin.
As previously noted, a heparin nomogram can improve time to therapeutic activated partial thromboplastin
time when developed specifically for critically ill patients. However, the patient population used to develop a
nomogram should be considered because the variables may not apply to all patient populations. An example
of this occurs with protocols for insulin infusions in critically ill patients. As previously noted, patients with
AKI are at a greater risk of hypoglycemia if treated with an insulin infusion protocol developed in critically
ill patients without kidney impairment.

IX.  CONCLUSION
There are marked differences in the ways in which critically ill patients respond to drugs. Research in this
area has noted significant changes in the PK and PD of certain medications in select critically ill populations.
Although these studies have highlighted important issues, considerable work is still needed to better define these
changes in different critically ill populations. As research continues to advance, together with our knowledge of
how patients respond to drugs differently, critical care clinicians must stay abreast of new information and the
ways in which it will affect the care of their patients.

ACCP Updates in Therapeutics® 2022: Critical Care Pharmacy Preparatory Review and Recertification Course

160
Pharmacokinetics/Pharmacodynamics

REFERENCES

1. Aarab R, van Es J, de Pont AC, et al. Monitoring 12. Bonnet F, Richard C, Glaser P, et al. Changes in
of unfractionated heparin in critically ill patients. hepatic flow induced by continuous positive pres-
Neth J Med 2013;71:466-71. sure ventilation in critically ill patients. Crit Care
2. Abdul-Aziz MH, Alffenaar JC, Bassetti M, et al. Med 1982;10:703-5.
Antimicrobial therapeutic drug monitoring in crit- 13. Boucher BA, King SR, Wandschneider HL, et al.
ically ill adult patients: a position paper. Intensive Fluconazole pharmacokinetics in burn patients.
Care Med 2020;46:1127-53. Antimicrob Agents Chemother 1998;42:930-3.
3. Abdul-Aziz MH, Dulhunty JM, Bellomo R, et al. 14. Boucher BA, Rodman JH, Jaresko GS, et al.
Continuous beta-lactam infusion in critically ill Phenytoin pharmacokinetics in critically ill trauma
patients: the clinical evidence. Ann Intensive Care patients. Clin Pharmacol Ther 1988;44:675-83.
2012;2:37. 15. Bouglé A, Dujardin O, Lepère V, et al.
4. Andus T, Geiger T, Hirano T, et al. Action of PHARMECMO: Therapeutic drug monitoring
recombinant human interleukin 6, interleukin 1 and adequacy of current dosing regimens of anti-
beta and tumor necrosis factor alpha on the mRNA biotics in patients on extracorporeal life support.
induction of acute-phase proteins. Eur J Immunol Anaesth Crit Care Pain Med 2019;38:493-97.
1988;18:739-46. 16. Brandstrup B, Svensen C, Engquist A. Hemorrhage
5. Ariano RE, Sitar DS, Zelenitsky SA, et al. Enteric and operation cause a contraction of the extra-
absorption and pharmacokinetics of oseltamivir in cellular space needing replacement—evidence
critically ill patients with pandemic (H1N1) influ- and implications? A systematic review. Surgery
enza. CMAJ 2010;182:357-63. 2006;139:419-32.
6. Barletta JF, Johnson SB, Nix DE, et al. Population 17. Brown G, Dodek P. An evaluation of empiric vs.
pharmacokinetics of aminoglycosides in criti- nomogram-based dosing of heparin in an intensive
cally ill trauma patients on once-daily regimens. J care unit. Crit Care Med 1997;25:1534-8.
Trauma 2000;49:869-72. 18. Case J, Khan S, Khalid R, et al. Epidemiology of
7. Barletta JF, Mangram AJ, Byrne M, et al. acute kidney injury in the intensive care unit. Crit
Identifying augmented renal clearance in trauma Care Res Pract 2013;2013:479730.
patients: validation of the augmented renal clear- 19. Chew JL, Plotka A, Alvey CW, et al. Effect of the
ance in trauma intensive care (ARTIC) scoring gastrointestinal prokinetic agent erythromycin on
system. J Trauma Acute Care Surg 2017;82:665-71. pharmacokinetics of pregabalin controlled-release
8. Barquist ES, Gomez-Fein E, Block EF, et al. in healthy individuals: a phase I, randomized cross-
Bioavailability of oral fluconazole in critically over trial. Clin Drug Investig 2015;35:299-305.
ill abdominal trauma patients with and without 20. Craig WA. Pharmacokinetic/pharmacodynamics
abdominal wall closure: a randomized crossover parameters: rationale for antibacterial dosing of
clinical trial. J Trauma 2007;63:159-63. mice and men. Clin Infect Dis 1008;26:1-10.
9. Bellomo R, Kellum JA, Wisniewski SR, et al. 21. Cummins CL, Jacobsen W, Benet LZ. Unmasking
Effects of norepinephrine on the renal vasculature the dynamic interplay between intestinal
in normal and endotoxemic dogs. Am J Respir Crit P-glycoprotein and CYP3A4. J Pharmacol Exp
Care Med 1999;159:1186-92. Ther 2002;300:1036-45.
10. Bersten AD, Hersch M, Cheung H, et al. The effect 22. Dahn MS, Lange P, Lobdell K, et al. Splanchnic and
of various sympathomimetics on the regional total body oxygen consumption differences in sep-
circulations in hyperdynamic sepsis. Surgery tic and injured patients. Surgery 1987;101:69-80.
1992;112:549-61. 23. Dailly E, Kergueris MF, Pannier M, et al.
11. Bickford A, Majercik S, Bledsoe J, et al. Population pharmacokinetics of imipenem in burn
Weight-based enoxaparin dosing for venous patients. Fundam Clin Pharmacol 2003;17:645-50.
thromboembolism prophylaxis in the obese trauma 24. Dasta JF, Armstrong DK. Variability in aminogly-
patient. Am J Surg 2013;206:847-52. coside pharmacokinetics in critically ill surgical
patients. Crit Care Med 1988;16:327-30.

ACCP Updates in Therapeutics® 2022: Critical Care Pharmacy Preparatory Review and Recertification Course

161
Pharmacokinetics/Pharmacodynamics

25. De Paepe P, Belpaire FM, Buylaert WA. 37. Fuster-Lluch O, Geronimo-Pardo M, Peyro-Garcia
Pharmacokinetic and pharmacodynamic consider- R, et al. Glomerular hyperfiltration and albumin-
ations when treating patients with sepsis and septic uria in critically ill patients. Anaesth Intensive
shock. Clin Pharmacokinet 2002;41:1135-51. Care 2008;36:674-80.
26. Desjars P, Pinaud M, Bugnon D, et al. 38. Garrelts JC, Jost G, Kowalsky SF, et al.
Norepinephrine therapy has no deleterious renal Ciprofloxacin pharmacokinetics in burn patients.
effects in human septic shock. Crit Care Med Antimicrob Agents Chemother 1996;40:1153-6.
1989;17:426-9. 39. Goncalves-Pereira J, Povoa P. Antibiotics in
27. Dickerson RN, Hamilton LA, Connor KA, et al. critically ill patients: a systematic review of the
Increased hypoglycemia associated with renal pharmacokinetics of beta-lactams. Crit Care
failure during continuous intravenous insulin infu- 2011;15:R206.
sion and specialized nutritional support. Nutrition 40. Gump FE, Price JB Jr, Kinney JM. Whole body
2011;27:766-72. and splanchnic blood flow and oxygen consump-
28. Dickerson RN, Lynch AM, Maish GO III, et al. tion measurements in patients with intraperitoneal
Improved safety with intravenous insulin ther- infection. Ann Surg 1970;171:321-8.
apy for critically ill patients with renal failure. 41. Haas CE, Nelsen JL, Raghavendran K, et al.
Nutrition 2014;30:557-62. Pharmacokinetics and pharmacodynamics of
29. Dickerson RN, Maish GO III, Minard G, et al. enoxaparin in multiple trauma patients. J Trauma
Clinical relevancy of the levothyroxine-continu- 2005;59:1336-43; discussion 1343-4.
ous enteral nutrition interaction. Nutr Clin Pract 42. Hamilton LA, Christopher Wood G, Magnotti
2010;25:646-52. LJ, et al. Treatment of methicillin-resistant
30. Di Giantomasso D, May CN, Bellomo R. Staphylococcus aureus ventilator-associated pneu-
Norepinephrine and vital organ blood flow. monia with high-dose vancomycin or linezolid. J
Intensive Care Med 2002;28:1804-9. Trauma Acute Care Surg 2012;72:1478-83.
31. Di Giantomasso D, May CN, Bellomo R. 43. Hanrahan TP, Harlow G, Hutchinson J, et al.
Norepinephrine and vital organ blood flow during Vancomycin-associated nephrotoxicity in the crit-
experimental hyperdynamic sepsis. Intensive Care ically ill: a retrospective multivariate regression
Med 2003;29:1774-81. analysis. Crit Care Med 2014;42:2527-36.
32. Di Giantomasso D, Morimatsu H, May CN, et 44. Heming N, Faisy C, Urien S. Population phar-
al. Intrarenal blood flow distribution in hyperdy- macodynamic model of bicarbonate response to
namic septic shock: effect of norepinephrine. Crit acetazolamide in mechanically ventilated chronic
Care Med 2003;31:2509-13. obstructive pulmonary disease patients. Crit Care
33. Dulhunty JM, Roberts JA, Davis JS, et al. A mul- 2011;15:R213.
ticenter randomized trial of continuous versus 45. Hernandez G, Velasco N, Wainstein C, et al. Gut
intermittent ß-lactam infusion in severe sepsis. mucosal atrophy after a short enteral fasting period
Am J Respir Crit Care Med 2015;192:1298-305. in critically ill patients. J Crit Care 1999;14:73-7.
34. Fernandez C, Buyse M, German-Fattal M, et al. 46. Hinshaw LB, Beller BK, Chang AC, et al.
Influence of the pro-inflammatory cytokines on Evaluation of naloxone for therapy of Escherichia
P-glycoprotein expression and functionality. J coli shock. Species differences. Arch Surg
Pharm Pharm Sci 2004;7:359-71. 1984;119:1410-8.
35. Forrest A, Nix DE, Ballow CH, et al. 47. Holley FO, Ponganis KV, Stanski DR. Effects of
Pharmacodynamics of intravenous ciprofloxa- cardiac surgery with cardiopulmonary bypass
cin in seriously ill patients. Antimicrob Agents on lidocaine disposition. Clin Pharmacol Ther
Chemother 1993;37:1073-81. 1984;35:617-26.
36. Fuchs A, Csajka C, Thoma Y, et al. Benchmarking 48. Iida Y, Nishi S, Asada A. Effect of mild therapeu-
therapeutic drug monitoring software: a review tic hypothermia on phenytoin pharmacokinetics.
of available computer tools. Clin Pharmacokinet Ther Drug Monit 2001;23:192-7.
2013;52:9-22.

ACCP Updates in Therapeutics® 2022: Critical Care Pharmacy Preparatory Review and Recertification Course

162
Pharmacokinetics/Pharmacodynamics

49. Jardin F, Farcot JC, Boisante L, et al. Influence of 61. Lebrault C, Duvaldestin P, Henzel D, et al.
positive end-expiratory pressure on left ventricu- Pharmacokinetics and pharmacodynamics of
lar performance. N Engl J Med 1981;304:387-92. vecuronium in patients with cholestasis. Br J
50. Jochberger S, Mayr V, Luckner G, et al. Antifactor Anaesth 1986;58:983-7.
Xa activity in critically ill patients receiving anti- 62. Lemaitre, F., Hasni, N., Leprince, P, et al.
thrombotic prophylaxis with standard dosages of Propofol, midazolam, vancomycin and cyclospo-
certoparin: a prospective, clinical study. Crit Care rine therapeutic drug monitoring in extracorporeal
2005;9:R541-8. membrane oxygenation circuits primed with whole
51. Johnston JD, Harvey CJ, Menzies IS, et al. human blood. Crit Care 2015;19:40. https://doi.
Gastrointestinal permeability and absorptive org/10.1186/s13054-015-0772-5
capacity in sepsis. Crit Care Med 1996;24:1144-9. 63. Marin C, Eon B, Saux P, et al. Renal effects of
52. Joukhadar C, Frossard M, Mayer BX, et al. norepinephrine used to treat septic shock patients.
Impaired target site penetration of beta-lactams Crit Care Med 1990;18:282-5.
may account for therapeutic failure in patients 64. Matzke GR, Aronoff GR, Atkinson AJ Jr, et al.
with septic shock. Crit Care Med 2001;29:385-91. Drug dosing consideration in patients with acute
53. Khwaja A. KDIGO clinical practice guidelines and chronic kidney disease—a clinical update
for acute kidney injury. Nephron Clin Pract from Kidney Disease: Improving Global Outcomes
2012;120:179-84. (KDIGO). Kidney Int 2011;80:1122-37.
54. Kirwan CJ, MacPhee IA, Lee T, et al. Acute kid- 65. McKindley DS, Boucher BA, Hess MM, et al.
ney injury reduces the hepatic metabolism of Effect of acute phase response on phenytoin metab-
midazolam in critically ill patients. Intensive Care olism in neurotrauma patients. J Clin Pharmacol
Med 2012;38:76-84. 1997;37:129-39.
55. Kiser TH, Mann AM, Trujillo TC, et al. Evaluation 66. Mehta NM, Halwick DR, Dodson BL, et al.
of empiric versus nomogram-based direct throm- Potential drug sequestration during extracorporeal
bin inhibitor management in patients with membrane oxygenation: results from an ex vivo
suspected heparin-induced thrombocytopenia. experiment. Intensive Care Med 2007;33:1018-24.
Am J Hematol 2011;86:267-72. 67. Moore RD, Lietman PS, Smith CR. Clinical
56. Klem C, Dasta JF, Reilley TE, et al. Variability response to aminoglycoside therapy: importance of
in dobutamine pharmacokinetics in unstable the ratio of peak concentration to minimal inhibi-
critically ill surgical patients. Crit Care Med tory concentration. J Infect Dis 1987;155:93-9.
1994;22:1926-32. 68. National Kidney Foundation (NKF). K/DOQI
57. Knights KM, Rowland A, Miners JO. Renal drug clinical practice guidelines for chronic kidney dis-
metabolism in humans: the potential for drug ease: evaluation, classification, and stratification.
endobiotic interactions involving cytochrome Am J Kidney Dis 2002;39:S1-266.
P450 (CYP) and UDP-glucuronosyltransferase 69. Nguyen NQ, Ng MP, Chapman M, et al. The
(UGT). Br J Clin Pharmacol 2013;76:587-602. impact of admission diagnosis on gastric empty-
58. Kruger PS, Freir NM, Venkatesh B, et al. A prelim- ing in critically ill patients. Crit Care 2007;11:R16.
inary study of atorvastatin plasma concentrations 70. Nicolau DP, Freeman CD, Belliveau PP, et al.
in critically ill patients with sepsis. Intensive Care Experience with a once-daily aminoglycoside
Med 2009;35:717-21. program administered to 2,184 adult patients.
59. Lahner E, Annibale B, Delle Fave G. Systematic Antimicrob Agents Chemother 1995;39:650-5.
review: impaired drug absorption related to the 71. Pai MP, Neely M, Rodvold KA, et al. Innovative
coadministration of antisecretory therapy. Aliment approaches to optimizing the delivery of vanco-
Pharmacol Ther 2009;29:1219-29. mycin in individual patients. Adv Drug Deliv Rev
60. Lauzier F, Levy B, Lamarre P, et al. Vasopressin 2014;77C:50-7.
or norepinephrine in early hyperdynamic septic 72. Pea F, Pavan F, Furlanut M. Clinical relevance of
shock: a randomized clinical trial. Intensive Care pharmacokinetics and pharmacodynamics in car-
Med 2006;32:1782-9. diac critical care patients. Clin Pharmacokinet
2008;47:449-62.

ACCP Updates in Therapeutics® 2022: Critical Care Pharmacy Preparatory Review and Recertification Course

163
Pharmacokinetics/Pharmacodynamics

73. Rabkin R, Ryan MP, Duckworth WC. The renal 85. Rybak MJ, Le J, Lodise TP, et al. Therapeutic
metabolism of insulin. Diabetologia 1984;27:351-7. monitoring of vancomycin for serious methicil-
74. Redl-Wenzl EM, Armbruster C, Edelmann G, et lin-resistant Staphylococcus aureus infections:
al. The effects of norepinephrine on hemodynam- a revised consensus guideline and review by the
ics and renal function in severe septic shock states. American Society of Health-System Pharmacists,
Intensive Care Med 1993;19:151-4. the Infectious Diseases Society of America, The
75. Ritz MA, Fraser R, Edwards N, et al. Delayed gas- Pediatric Infectious Diseases Society, and the
tric emptying in ventilated critically ill patients: Society of Infectious Diseases Pharmacists. Am J
measurement by 13 C-octanoic acid breath test. Health-Syst Pharm 2020:77;835-64.
Crit Care Med 2001;29:1744-9. 86. Sawchuk RJ, Zaske DE, Cipolle RJ, et al. Kinetic
76. Rivers E, Nguyen B, Havstad S, et al. Early goal model for gentamicin dosing with the use of indi-
directed therapy in the treatment of severe sepsis vidual patient parameters. Clin Pharmacol Ther
and septic shock. N Engl J Med 2001;345:1368-77. 1977;21:362-9.
77. Roberts JA, Norris R, Paterson DL, et al. 87. Secchi A, Ortanderl JM, Schmidt W, et al. Effects
Therapeutic drug monitoring of antimicrobials. Br of dobutamine and dopexamine on hepatic micro
J Clin Pharmacol 2012;73:27-36. and macrocirculation during experimental endo-
78. Roberts JA, Roberts MS, Robertson TA, et al. toxemia: an intravital microscopic study in the rat.
Piperacillin penetration into tissue of critically Crit Care Med 2001;29:597-600.
ill patients with sepsis—bolus versus continuous 88. Shedlofsky SI, Israel BC, McClain CJ, et al.
administration? Crit Care Med 2009;37:926-33. Endotoxin administration to humans inhibits
79. Rommers MK, Van der Lely N, Egberts TC, et al. hepatic cytochrome P450-mediated drug metabo-
Anti-Xa activity after subcutaneous administra- lism. J Clin Invest 1994;94:2209-14.
tion of dalteparin in ICU patients with and without 89. Shedlofsky SI, Israel BC, Tosheva R, et al.
subcutaneous oedema: a pilot study. Crit Care Endotoxin depresses hepatic cytochrome
2006;10:R93. P450mediated drug metabolism in women. Br J
80. Rondina MT, Wheeler M, Rodger GM, et al. Clin Pharmacol 1997;43:627-32.
Weight-based dosing of enoxaparin for VTE pro- 90. Shekar K, Roberts JA, McDonald CI, et al.
phylaxis in morbidly obese, medically ill patients. Protein-bound drugs are prone to sequestration
Thromb Res 2010;125:220-3. in the extracorporeal membrane oxygenation
81. Ruokonen E, Takala J, Kari A, et al. Regional circuit: results from an ex vivo study. Crit Care
blood flow and oxygen transport in septic shock. 2015;19:164.
Crit Care Med 1993;21:1296-303. 91. Silverman HJ, Penaranda R, Orens JB, et al.
82. Ruokonen E, Takala J, Kari A. Regional blood Impaired beta-adrenergic receptor stimulation
flow and oxygen transport in patients with the low of cyclic adenosine monophosphate in human
cardiac output syndrome after cardiac surgery. septic shock: association with myocardial hypo-
Crit Care Med 1993;21:1304-11. responsiveness to catecholamines. Crit Care Med
83. Rybak MJ, Albrecht LM, Berman JR, et al. 1993;21:31-9.
Vancomycin pharmacokinetics in burn patients 92. Somogyi AA, Shanks CA, Triggs EJ. Disposition
and intravenous drug abusers. Antimicrob Agents kinetics of pancuronium bromide in patients
Chemother 1990;34:792-5. with total biliary obstruction. Br J Anaesth
84. Rybak MJ, Le J, Lodise TP, et al. Methicillin- 1977;49:1103-8.
resistant Staphylococcus aureus Infections: a 93. Tarling MM, Toner CC, Withington PS, et al. A
revised consensus guideline and review by the model of gastric emptying using paracetamol
American Society of Health-System Pharmacists, absorption in intensive care patients. Intensive
the Infectious Diseases Society of America, the Care Med 1997;23:256-60.
Pediatric Infectious Diseases Society, and the 94. Tortorici MA, Kochanek PM, Poloyac SM. Effects
Society of Infectious Diseases Pharmacists. Clin of hypothermia on drug disposition, metabolism,
Infect Dis 2020:71;1361-4. and response: a focus of hypothermia-mediated

ACCP Updates in Therapeutics® 2022: Critical Care Pharmacy Preparatory Review and Recertification Course

164
Pharmacokinetics/Pharmacodynamics

alterations on the cytochrome P450 enzyme sys-


tem. Crit Care Med 2007;35:2196-204.
95. Tukas M. Pharmacokinetics and extracorporeal
membrane oxygenation in adults: a literature
review. AACN Adv Crit Care 2018;29:246-58.
96. Udy AA, Roberts JA, Shorr AF, et al. Augmented
renal clearance in septic and traumatized patients
with normal plasma creatinine concentrations:
identifying at-risk patients. Crit Care 2013;17:R35.
97. van den Broek MP, Groenendaal F, Egberts AC,
et al. Effects of hypothermia on pharmacoki-
netics and pharmacodynamics: a systematic
review of preclinical and clinical studies. Clin
Pharmacokinet 2010;49:277-94.
98. Wan L, Bellomo R, May CN. The effects of normal
and hypertonic saline on regional blood flow and
oxygen delivery. Anesth Analg 2007;105:141-7.
99. Wang P, Ba ZF, Chaudry IH. Hepatic extraction
of indocyanine green is depressed early in sepsis
despite increased hepatic blood flow and cardiac
output. Arch Surg 1991;126:219-24.
100. Wang P, Zhou M, Rana MW, et al. Differential
alterations in microvascular perfusion in various
organs during early and late sepsis. Am J Physiol
1992;263:G38-43.
101. Williams D. The effect of enteral nutrition sup-
plements on serum voriconazole levels. J Oncol
Pharm Pract 2012;18:128-31.
102. Yusuf E, Spapen H, Pierard D. Prolonged vs inter-
mittent infusion of piperacillin/tazobactam in
critically ill patients: a narrative and systematic
review. J Crit Care 2014;29:1089-95.
103. Zdolsek HJ, Kagedal B, Lisander B, et al.
Glomerular filtration rate is increased in burn
patients. Burns 2010;36:1271-6.
104. Zhou SF. Structure, function and regulation of
P-glycoprotein and its clinical relevance in drug
disposition. Xenobiotica 2008;38:802-32.

ACCP Updates in Therapeutics® 2022: Critical Care Pharmacy Preparatory Review and Recertification Course

165
Pharmacokinetics/Pharmacodynamics

ANSWERS AND EXPLANATIONS TO PATIENT CASES

1. Answer: B 5. Answer: B
Increasing the concentration of an antibiotic at an Morphine is a high extraction ratio drug. Mechanical
infection site is most important (Answer B is correct). ventilation can decrease cardiac output and thus
Although case reports describe the use of intraven- decrease liver blood flow. The decrease in liver blood
tricular antibiotics in the treatment of meningitis, they flow is inversely proportional to the unbound steady-
have not shown superiority (Answer A is incorrect). state concentration. Therefore, it both decreases the
Although there is the potential to reduce vancomycin- hepatic blood flow and increases the unbound concen-
induced nephrotoxicity, no studies have compared tration (Answer B is correct). The effects of mechanical
the nephrotoxicity of intraventricular antibiotics with ventilation on cardiac output would likely decrease
that of intravenous antibiotics, likely because this is oxygen delivery (Answer A is incorrect). Cytokines
not the rationale for their use (Answer C is incorrect). affect intrinsic clearance but would not affect the con-
Ototoxicity could also be reduced, but this was not the centration of a high extraction ratio drug (Answer C is
intent of the locally instilled antibiotics (Answer D is incorrect). Mechanical ventilation can indirectly affect
incorrect). the unbound concentration (Answer D is incorrect).

2. Answer: D 6. Answer: B
Three studies have documented an increase in digoxin This patient has sepsis, which is associated with an
absorption when the gastric pH is increased, with increased production of inflammatory cytokines. These
two studies noting the cause of increased gastric pH cytokines can decrease the activity of the CYP enzymes
from a proton pump inhibitor (Answer D is correct). and decrease intrinsic clearance. In addition, fluco-
However, no studies have shown a decreased absorp- nazole inhibits CYP3A4 activity. Diltiazem is a low
tion of carvedilol (Answer A is incorrect), ciprofloxacin extraction ratio drug. The hepatic clearance of diltiazem
(Answer B is incorrect), or diazepam (Answer C is is affected by changes in intrinsic clearance (including
incorrect). CYP3A4 activity). The unbound steady-state concen-
tration would be increased, with a decrease in intrinsic
3. Answer: A clearance caused by the inflammatory cytokines. The
A decrease in the ionization of a drug allows the drug increased unbound steady-state concentrations would
to pass more easily through membranes. Increasing the cause a decrease in the heart rate (Answer B is correct).
ionization would decrease the Vd by decreasing its abil- There is no cause for an increase in intrinsic clearance
ity to pass through membranes. A weak acid would be and a corresponding increase in heart rate (Answer A is
less ionized in a more acidic environment. The patient incorrect). There is no reason for this patient to have a
is likely in diabetic ketoacidosis with a definite acido- decrease in the unbound fraction of diltiazem (Answer
sis. Aspirin (a weak acid) is less likely to be ionized C is incorrect). A decrease in albumin could occur with
and would have an increased Vd (Answer A is correct; sepsis and would result in an increase in the unbound
Answers B–D are incorrect). fraction of diltiazem. The increase in unbound fraction
would not affect the unbound steady-state concentra-
4. Answer: C tion, nor would it affect the heart rate (Answer D is
Levofloxacin has a large Vd. The increase in interstitial incorrect).
fluid volume caused by 6 L of crystalloids would increase
the Vd of hydrophilic drugs but would not appreciably 7. Answer: D
affect the Vd of a drug like levofloxacin, which already Under normal conditions, the unbound steady-state con-
has a large Vd (Answer C is correct). Tobramycin, dap- centration for phenytoin is expected to be 10% of the total
tomycin, and cefepime have relatively small Vd values concentration. Thus, a total concentration of 10-20 mg/L
and are increased in patients with increased interstitial is equivalent to an unbound steady-state concentration
volumes (Answers A, B, and D are incorrect). of 1-2 mg/L. When the unbound fraction changes, this
ratio changes. The unbound steady-state concentration
does not change, but the total concentration decreases

ACCP Updates in Therapeutics® 2022: Critical Care Pharmacy Preparatory Review and Recertification Course

166
Pharmacokinetics/Pharmacodynamics

(causing the change in the ratio). Using the equations


for low extraction ratio drugs (like phenytoin) to explain
what has occurred, the unbound fraction is inversely
proportional to the total concentration but has no effect
on the unbound steady-state concentration (Answer D is
correct; Answers A and C are incorrect).

8. Answer: A
The kidney plays a role in the excretion and metabo-
lism of insulin. Acute kidney injury will decrease the
ability of the kidney to metabolize insulin, which will
increase circulating insulin and contribute to hypogly-
cemia (Answer A is correct). The Vd of insulin in AKI
is not well studied and has not been linked with epi-
sodes of hypoglycemia (Answer B is incorrect). Hepatic
impairment may increase the risk of hypoglycemia
with insulin, but an increase in hepatic metabolism
is unlikely to affect the risk of hypoglycemia in AKI
(Answer C is incorrect). Insulin resistance has been
reported in patients with AKI, but this would not con-
tribute to hypoglycemia (Answer D is incorrect).

ACCP Updates in Therapeutics® 2022: Critical Care Pharmacy Preparatory Review and Recertification Course

167
Pharmacokinetics/Pharmacodynamics

ANSWERS AND EXPLANATIONS TO SELF-ASSESSMENT QUESTIONS

1. Answer: B the total concentration is not affected by changes in the


Decreased anti-Xa activity occurs in critically ill patients free fraction, but unbound concentrations are increased
receiving several different low-molecular-weight hepa- when the free fraction increases (Answer B is correct;
rins. This patient received the standard fluid bolus and Answers A, C, and D are incorrect).
then required more fluid to raise his central venous
pressure. This suggests that the patient has redistributed 4. Answer: C
the fluids to the extravascular space. The data show- The update to the KDIGO guidelines notes that the
ing that edematous patients have lower anti-Xa activity most important factor in determining kidney function
than non-edematous patients supports the connection is having at least one estimate of GFR. The update
between anti-Xa activity and absorption (Answer B is recommends that the GFR or the CrCl be estimated to
correct). Given the patient’s fluid distribution, the Vd determine this (Answer C is correct). The BUN value can
of enoxaparin would likely be increased, not decreased help identify the BUN/SCr ratio indicating intravascu-
(Answer A is incorrect). Enoxaparin is not hepatically lar volume contraction and, potentially, a prerenal cause
metabolized (Answer C is incorrect). Although a patient to the patient’s AKI, but it does not help in drug dos-
could have an increased anti-Xa activity secondary to ing (Answer A is incorrect). Total daily urinary output
decreased renal elimination, this would require a CrCl is helpful in diagnosing AKI (oliguric vs. non-oliguric
of less than 30 mL/minute/1.73 m2. That the patient AKI) and in staging AKI (using a urinary output of less
had a deep venous thrombosis does not correlate with than 0.5 mL/kg/hour) but not in drug dosing (Answer
increased activity (Answer D is incorrect). B is incorrect). A history of CKD can help determine a
baseline kidney function, but the GFR is still needed for
2. Answer: C changes in drug dosing (Answer D is incorrect).
Abdominal surgery has been identified as a risk fac-
tor for ileus. Antisecretory agents such as pantoprazole 5. Answer: B
alter drug absorption. Traumatic brain injury is signifi- The ASHP (American Society of Health-System
cantly associated with intolerance of enteral nutrition, Pharmacists) vancomycin dosing guidelines recom-
as indicated by increased gastric residuals. This indi- mend a 20- to 25-mg/kg loading dose of vancomycin
cates delayed gastric emptying and the risk of altered for serious infections. Given the patient’s sepsis, he is
absorption. Therefore, the combination of abdomi- likely to have more interstitial fluid, which will increase
nal surgery, pantoprazole therapy, and TBI contains the Vd of hydrophilic drugs like vancomycin. The
the three variables identified in the literature to alter 2000-mg dose is near the top end of the recommended
absorption (Answer C is correct). Theoretically, intes- 20- to 25-mg/kg loading dose (Answer B is correct).
tinal atrophy could cause changes in absorption, but The other doses are outside the recommended 20- to
no data are available to confirm this theory (Answers 25-mg/kg loading dose range (Answers A, C, and D are
A and D are incorrect). Changes in cardiac output have incorrect).
been correlated with changes in hepatosplanchnic blood
flow. These changes in blood flow are thought to affect 6. Answer: A
absorption, but again, no data have correlated increased Critically ill patients younger than 50 are more likely
cardiac output with increased drug absorption (Answer to have augmented renal excretion. Vancomycin is
B is incorrect). excreted unchanged in the urine. The initial dosing
would result in a therapeutic trough concentration,
3. Answer: B given the increased (augmented) renal excretion of van-
Propofol is a high extraction ratio drug that is bound comycin. Augmented renal excretion usually returns to
to albumin. When the albumin concentration decreases, normal around day 7. An increased vancomycin trough
there is an expected increase in the free fraction of after a previously therapeutic trough is most likely
propofol. Using the equations describing the total and associated with decreased renal excretion at day 10 of
unbound concentrations of a high extraction ratio drug, therapy (Answer A is correct). There is no indication

ACCP Updates in Therapeutics® 2022: Critical Care Pharmacy Preparatory Review and Recertification Course

168
Pharmacokinetics/Pharmacodynamics

ANSWERS AND EXPLANATIONS TO SELF-ASSESSMENT QUESTIONS

that the Vd has changed, which would likely result in


lower or unchanged concentrations (Answer B is incor-
rect). Vancomycin tissue penetration may be better if
inflammation is present, but no documented studies
correlate decreased inflammation with increased serum
concentrations (Answer C is incorrect). The liver does
not appreciably metabolize vancomycin, and liver blood
flow would not be a factor in the vancomycin concentra-
tion (Answer D is incorrect).

7. Answer: C
Prospective controlled studies of prolonged piperacil-
lin/tazobactam infusions have shown no improvement
in mortality (Answer A is incorrect). No studies have
reported neurotoxicity as an outcome (Answers B and
D are incorrect). Only retrospective studies have shown
improvements in mortality with the use of prolonged
or continuous infusions of piperacillin/tazobactam
(Answer C is correct).

8. Answer: C
Morphine is a high extraction ratio drug. As such, its
hepatic metabolism or hepatic clearance depends only
on hepatic blood flow. Hepatic clearance equals hepatic
blood flow (Answer C is correct). Morphine does not
bind to AAG (Answer A is incorrect). Because hepatic
blood flow is the main determinant of hepatic metab-
olism of morphine, changes in protein binding do not
affect metabolism (Answer B is incorrect). Changes
in intrinsic clearance do not affect the metabolism of
morphine as significantly as does hepatic blood flow
(Answer D is incorrect).

ACCP Updates in Therapeutics® 2022: Critical Care Pharmacy Preparatory Review and Recertification Course

169
Pulmonary Disorders I
Mitchell S. Buckley, Pharm.,D., FCCP, FASHP, FCCM, BCCCP
Banner - University Medical Center Phoenix
Phoenix, Arizona
Pulmonary Disorders I

Pulmonary Disorders I

Mitchell S. Buckley, Pharm.,D., FCCP, FASHP, FCCM, BCCCP


Banner - University Medical Center Phoenix
Phoenix, Arizona

ACCP Updates in Therapeutics® 2022: Critical Care Pharmacy Preparatory Review and Recertification Course

173
Pulmonary Disorders I

Learning Objectives Self-Assessment Questions


Answers and explanations to these questions may be
1. Evaluate the role of pharmacologic management found at the end of this chapter.
options in acute respiratory distress syndrome
(ARDS). 1. A 65-year-old man presents to the emergency
2. Review clinical practice guidelines pertaining to department with severe shortness of breath, tachy-
ARDS. pnea, altered mental status, and diaphoresis. A chest
3. Recommend an evidence-based approach for non- radiograph reveals diffuse, bilateral opacities. His
pharmacologic therapy in managing critically ill vital signs are as follows: blood pressure 94/54 mm
patients with ARDS. Hg, respiratory rate 26 breaths/minute, heart rate
4. Evaluate key variables and commonly used modes 120 beats/minute, pain score 2/10, and temperature
for treatment with mechanical ventilation. 105.8°F (41.0°C). The patient’s wife states that his
5. Assess appropriateness of drug therapy for endotra- symptoms began about 2 days ago and gradually
cheal intubation, including agents for premedication, worsened over the past day. The patient is trans-
induction, and neuromuscular blockade. ferred to the intensive care unit (ICU), where he is
intubated. His arterial blood gas values are pH 7.30,
partial pressure of arterial carbon dioxide (Paco2) 50
Abbreviations in This Chapter mm Hg, partial pressure of arterial oxygen (Pao2)
50 mm Hg, and oxygen saturation (Sao2) 85% while
AC/VC Assist control/volume control receiving fraction of inspired oxygen (Fio2) 100%.
ARDS Acute respiratory distress syndrome Which is the best therapy plan for the next 24 hours?
COVID-19 Coronavirus disease 2019 A. Empiric antibiotic therapy, intravenous fluid
CVP Central venous pressure resuscitation, and vasopressors for shock; low
ECMO Extracorporeal membrane oxygenation tidal volume (4–8 mL/kg) ventilation strategy;
ICU Intensive care unit deep sedation to achieve a Richmond Agitation-
MAP Mean arterial pressure Sedation Scale (RASS) score of -4; and prone
MV Mechanical ventilation positioning.
NMBA Neuromuscular blocking agent B. Empiric antibiotic therapy, intravenous fluid
PEEP Positive end-expiratory pressure resuscitation, and vasopressors for shock; low
PS Pressure support tidal volume (4–8 mL/kg) ventilation strategy;
RSI Rapid sequence intubation deep sedation to achieve a RASS score of -5
SIMV Synchronized intermittent mandatory and cisatracurium administration to limit pla-
ventilation teau pressures; and prone positioning.
C. Empiric antibiotic therapy, aggressive diuresis
(goal central venous pressure [CVP] less than 4
mm Hg), and vasopressors for shock; low tidal
volume (4–8 mL/kg) ventilation strategy; deep
sedation to achieve a RASS score of -4; and
prone positioning.
D. Empiric antibiotic therapy, intravenous fluid
resuscitation, and vasopressors for shock; low
tidal volume (4–8 mL/kg) ventilation strategy;
deep sedation to achieve a RASS score of -4;
and supine positioning.

ACCP Updates in Therapeutics® 2022: Critical Care Pharmacy Preparatory Review and Recertification Course

174
Pulmonary Disorders I

2. Which best describes the category of acute respi-


ratory distress syndrome (ARDS) that most
benefits from prone positioning and cisatracurium
administration?
A. Acute lung injury.
B. Moderate to severe.
C. Mild to moderate.
D. Mild.

3. Which order of medication administration would be


most appropriate for a 34-year-old woman with no
significant medical history receiving rapid sequence
intubation (RSI)?
A. Rocuronium, etomidate, midazolam.
B. Fentanyl, succinylcholine, propofol.
C. Atropine, rocuronium, etomidate.
D. Fentanyl, etomidate, succinylcholine.

4. A 78-year-old man (weight 70 kg) presents to


the ICU after being intubated for acute respira-
tory failure. His ventilator settings are as follows:
assist control/volume control (AC/VC) mode, tidal
volume 700 mL (10 mL/kg), respiratory rate 20
breaths/minute, Fio2 50%, positive end-expiratory
pressure (PEEP) 5 cm H2O, and pressure support
(PS) 10 cm H2O. The first arterial blood gas values
are pH 7.25, Paco2 65 mm Hg, bicarbonate (HCO3)
26 mEq/L, Pao2 65 mm Hg, and Sao2 90%. His
mean arterial pressure (MAP) is 72 mm Hg without
requiring vasoactive support with a serum creatinine
of 1.2 mg/dL and urinary output of 350 mL over the
past 6 hours. Which approach would be best for this
patient’s initial treatment?
A. High tidal volume mechanical ventilation (MV)
and conservative fluid management.
B. Low tidal volume MV and conservative fluid
management.
C. High tidal volume MV and liberal fluid
management.
D. Low tidal volume MV and liberal fluid
management.

ACCP Updates in Therapeutics® 2022: Critical Care Pharmacy Preparatory Review and Recertification Course

175
Pulmonary Disorders I

BPS Critical Care Pharmacy Specialist Examination Content Outline

This chapter covers the following sections of the Critical Care Pharmacy Specialist Examination Content Outline:
1. Domain 1: Clinical Knowledge and Application
a. Task 1: 1, 3
b. Task 2: 3-4, 6
c. Task 3: 1-7
d. Task 4: 1-8
e. Task 5: 1, 2, 5-7
f. Task 6: 1-4
g. Task 7: 1-7
2. Domain 2: Practice Management, Policy, and Quality Improvement
a. Task 1: 2, 3
b. Task 4: 2, 3
b. Task 5: 4
3. Domain 3: Evidence-Based Medicine, Scholarship, Education, and Professional Development
a. Task 1: 1-3

ACCP Updates in Therapeutics® 2022: Critical Care Pharmacy Preparatory Review and Recertification Course

176
Pulmonary Disorders I

I.  ACUTE RESPIRATORY DISTRESS SYNDROME

A. Definition and Epidemiology


1. First described in 1967; several definitions have evolved over the years
2. Berlin ARDS definition was established in 2012 to address issues with previous definitions (Table 1)
(JAMA 2012;307:2526-33).

Table 1. Berlin ARDS Definition


Variable Criteria
Timing Onset within 1 wk of a known clinical insult or new or worsening respiratory symptoms
Chest imaging Bilateral opacities not fully explained by effusions, lobar/lung collapse, or nodules
Respiratory failure not fully explained by cardiac failure or fluid overload
Origin of edema
Need objective assessment (ECHO) to exclude hydrostatic edema if no risk factors present
Oxygenation
Mild 200 mm Hg < Pao2 /Fio2 ≤ 300 mm Hg with PEEP or CPAP ≥ 5 cm H2O
Moderate 100 mm Hg < Pao2 /Fio2 ≤ 200 mm Hg with PEEP ≥ 5 cm H2O
Severe Pao2/Fio2 ≤ 100 mm Hg with PEEP ≥ 5 cm H2O
CPAP = continuous positive airway pressure; ECHO = echocardiogram.

3. ARDS represents 10.4% of all ICU admissions and 23.4% of patients requiring mechanical ventilation
(MV) (JAMA 2016;315:788-800).
a. Mild ARDS accounts for 30%, moderate for 46.6%, and severe for 23.4%.
b. Hospital mortality is 34.9% for mild ARDS, 40.3% for moderate ARDS, and 46.1% for severe
ARDS.
c. Multisystem organ failure is the leading cause of death in patients with ARDS, with the number of
extrapulmonary organ failures correlating with an incremental increase in mortality (Intensive Care
Med 2011;37:1932-41).

B. Cause and Pathophysiology


1. Direct and indirect causes of lung injury
a. Direct: Pneumonia, aspiration, trauma
b. Indirect: Sepsis, transfusion injury, pancreatitis, burn injury, trauma
2. Pathogenesis
a. The hallmark clinical manifestation of ARDS is hypoxemia from alveolar collapse and edema.
b. The exudative phase is characterized by increased permeability from the alveolar epithelium and
capillary endothelial complex damage, leading to diffuse alveolar edema with fluid and cellular
debris (e.g., neutrophils, cytokines, platelets). In addition, type II cells responsible for surfactant
production are damaged. The peak incidence of this phase typically occurs within the first week
after the initial insult.
c. The fibroproliferative phase marks either recovery or progression of ARDS. Patients may partly
or fully recover pulmonary function from drainage of the alveolar fluid, type II cellular repair,
and improvement in the integrity of the endothelium-epithelium complex. However, patients whose
conditions are progressively worsening may develop significant alveolar, interstitial, and capillary
fibrosis. This phase typically manifests later in the course of ARDS (i.e., more than 7 days after the
initial insult).

ACCP Updates in Therapeutics® 2022: Critical Care Pharmacy Preparatory Review and Recertification Course

177
Pulmonary Disorders I

C. Management Strategies
1. MV
a. Lung-protective strategies using low tidal volume ventilation are considered the standard of care
and cornerstone in managing ARDS.
b. The landmark multicenter trial by the Acute Respiratory Distress Syndrome Network (ARDSNet)
showed a survival benefit over conventional ventilation using higher tidal volumes (12 mL/kg) (N
Engl J Med 2000;342:1301-8). Clinical practice guidelines recommend limiting tidal volumes (4–8
mL/kg of predicted body weight) and inspiratory pressures (plateau pressure less than 30 cm H2O)
while optimizing higher PEEP values (Am J Respir Crit Care Med 2017;195:1253-63).
c. High-frequency oscillatory ventilation is not currently recommended in ARDS, given a lack of
benefit and the potential for harm (Am J Respir Crit Care Med 2017;195:1253-63; Ann Intensive
Care 2019;9:69).
2. Prone positioning
a. Prone over supine positioning improves gas exchange and may reduce ventilator-induced lung injury.
Compression atelectasis attributed to the weight of the heart, ventral lungs, and abdominal viscera
may be exacerbated in the supine position in patients with ARDS. The gravitational effects by
placing a patient in the prone position improve ventilation/perfusion matching and end-expiratory
lung volume by more homogeneous tidal volume delivery (Chest 2017;151:215-24; Am J Respir Crit
Care Med 2017;195:1253-63).
b. The PROSEVA study group represents the most recently published randomized clinical trial
comparing prone and supine positioning in ARDS. In this trial, prone positioning for at least 16
hours/day compared with supine positioning in early ARDS decreased 28-day (adjusted hazard
ratio [HR] 0.42; 95% confidence interval [CI], 0.26–0.66) and 90-day (adjusted HR 0.48; 95% CI,
0.32–0.72) mortality (N Engl J Med 2013;368:2159-68). Significantly more ventilator-free days at
days 28 and 90 were observed with proning. In addition, 90-day extubation rates were significantly
higher in patients in the prone position.
c. Several meta-analyses have suggested an overall mortality benefit associated with prone positioning
over supine in ARDS, though this finding was inconsistent (Intensive Care Med 2014;40:332-41;
Crit Care 2014;18:R109; Crit Care Med 2014;42:1252-62; Ann Am Thorac Soc 2017;14(suppl
4):S280-S8; J Thorac Dis 2015;7:356-67; CMAJ 2014;186:E381-90). However, most meta-analyses
have shown a survival benefit with proning in patients with moderate to severe ARDS (i.e., Pao2/
Fio2 less than 150 mm Hg) compared with mild hypoxemia (i.e., Pao2/Fio2 greater than 200 mm
Hg) (Crit Care 2014;18:R109; Crit Care Med 2014;42:1252-62; Ann Am Thorac Soc 2017;14(suppl
4):S280-S288; CMAJ 2014;186:E381-90). In addition, mortality was reduced in patients with ARDS
with prone positioning and concurrent lung-protective MV strategies compared with pronation and
high tidal volume ventilation (Intensive Care Med 2014;40:332-41; Crit Care Med 2014;42:1252-
62; J Thorac Dis 2015;7:356-67; CMAJ 2014;186:E381-90).
d. Implementation of prone positioning requires robust planning and resources (J Pharm Pract
2019;32:347-60). Contraindicated criteria of use should be established to mitigate any adverse
events (e.g., spinal cord injury, elevated intracranial pressure [ICP]). Health care providers should
be trained on appropriate movement of patients between supine and prone positioning to ensure
securing of central lines as well as the endotracheal tube. Institutions should also consider the need
for manual maneuvering of patients compared with specialized rotating beds. One important patient
safety consideration is rapid health care staff access to patients in specialized beds during medical
emergencies (e.g., cardiopulmonary arrest, self-extubation).

ACCP Updates in Therapeutics® 2022: Critical Care Pharmacy Preparatory Review and Recertification Course

178
Pulmonary Disorders I

e. Recently published clinical practice guidelines strongly recommend prone positioning in moderate
to severe ARDS (Table 2) (Am J Respir Crit Care Med 2017;195:1253-63; BMJ Open Respir Res
2019;6:e000420; Ann Intensive Care 2019;9:69). Despite overwhelming support for this therapy
in ARDS, several important questions remain, including the optimal duration and timing of use
pertaining to ARDS onset. Patients with ARDS should remain in the prone position for more than
12 hours each day, with consideration for longer daily durations (i.e., at least 16 hours per day).
Prone positioning therapy should be considered early in the course of ICU admission in patients
with moderate to severe ARDS.

Table 2. Clinical Practice Guideline Recommendations for Prone Positioning in ARDS


Source Summary GRADE Recommendation
Am J Respir
Strong recommendation; moderate-
Crit Care Med Suggest prone positioning > 12 hr/day
high level of evidence
2017;195:1253-63
BMJ Open • NOT recommended for all levels of ARDS severity
Respir Res • Recommended for at least 12 hr/day in moderate to Strongly in favor
2019;6:e000420 severe ARDS (i.e., Pao2/Fio2 < 150 mm Hg)
Ann Intensive Should be used in moderate to severe ARDS (Pao2 /
High level of evidence
Care 2019;9:69 Fio2 < 150 mm Hg) for at least 16 consecutive hr

3. Extracorporeal membrane oxygenation (ECMO)


a. The CESAR trial showed that treatment at an ECMO referral center improved 6-month survival
(Lancet 2009;374:1351-63).
i. ARDS with a Murray score (i.e., acute lung injury score on the basis of chest radiography,
hypoxemia score, peak end-expiratory pressure, and static compliance) of 3 or greater, or
hypercapnia and a pH less than 7.20
ii. Not all patients randomized to the ECMO referral center received ECMO (22 of 90 patients in
the experimental group did not receive ECMO).
iii. Potential for confounding because of experiential bias – Clinicians at an ECMO treatment
center may have more experience in treating ARDS.
b. The Respiratory ECMO Survival Prediction (RESP) score was developed from a registry of patients
receiving ECMO (Am J Respir Crit Care Med 2014;189:1374-82).
i. The RESP score predicts patient survival after ECMO initiation.
ii. Does not help quantify the decision of whether to initiate ECMO or determine whether ECMO
treatment will improve survival in a specific patient
c. In the EOLIA trial, early use of ECMO compared with conventional management in severe ARDS
(e.g., Pao2 /Fio2 less than 80 mm Hg) was not associated with reduced 60-day mortality rates (relative
risk [RR] 0.76; 95% CI, 0.55–1.04, p=0.09) (N Engl J Med 2018;378:1965-75).
i. This trial was prematurely discontinued because of predefined futility criteria, which resulted
in not achieving power.
ii. The high crossover rate of 28% of controls into the ECMO group may have introduced bias.
d. The 2019 clinical practice guidelines state that ECMO should be considered in patients with ARDS
with Pao2/Fio2 ratios less than 80 mm Hg and/or in patients with elevated plateau pressures despite
optimizing PEEP, neuromuscular blocking agents (NMBAs), and prone positioning (BMJ Open
Respir Res 2019;6:e000420; Ann Intensive Care 2019;9:69).

ACCP Updates in Therapeutics® 2022: Critical Care Pharmacy Preparatory Review and Recertification Course

179
Pulmonary Disorders I

4. Fluid management
a. The FACTT trial compared optimal fluid management strategies – conservative (CVP less than
4 mm Hg) and liberal (CVP 10–14 mm Hg) – in patients with ARDS and hemodynamic stability
(not requiring vasopressors or MAP greater than 60 mm Hg) (N Engl J Med 2006;354:2564-75).
Diuretics were withheld in patients with shock but were administered according to study protocol
once patients had established hemodynamic stability (discontinuation of vasopressors or MAP
greater than 60 mm Hg). Although 60-day mortality did not differ (p=0.30), the conservative
compared with the liberal strategy was associated with increased ventilator-free days (14.6 ± 0.5 vs.
12.1 ± 0.5, p<0.001) and ICU-free days (28 days) (13.4 ± 0.4 vs. 11.2 ± 0.4, p<0.001).
b. Fluid strategies were compared (conservative, liberal, and simplified conservative) in a retrospective
comparison among protocols pertaining to ARDS (Crit Care Med 2015;43:288-295). The FACTT
Lite protocol provides fluid management recommendations pertaining to the administration of
furosemide or fluids as well as monitoring without intervention-based CVP, MAP, urinary output,
and pulmonary artery occlusion pressure (optional) (Table 3). No significant differences were found
between the FACTT Lite and the FACTT conservative strategies for ventilator-free days (14.9 vs.
14.6, respectively; p=0.61), ICU-free days (14.4 vs. 13.4, respectively; p=0.054), or death at 60 days
(22% vs. 25%, respectively; p=0.15). Compared with FACTT liberal, the FACTT Lite approach had
improved outcomes (ventilator-free and ICU days and 60-day mortality).
c. Net neutral fluid balance may be optimal in patients with ARDS without shock syndromes, given
these findings, with worse clinical outcomes associated with volume overload.

Table 3. FACTT Lite Simplified Conservative Fluid Management Approacha


MAP ≥ 60 mm Hg AND No Vasoactive Support ≥ 12 Hr
CVP Pulmonary Artery Occlu-
Urinary Output < 0.5 mL/ Urinary Output ≥ 0.5 mL/
(Recommended) sion Pressure (optional)
kg/hr kg/hr
Administer furosemide and Administer furosemide and
>8 > 12
reevaluate in 1 hr reevaluate in 4 hr
Administer fluid bolus and Administer furosemide and
4–8 8–12
reevaluate in 1 hr reevaluate in 4 hr
Administer fluid bolus and Monitor and reevaluate in 4
<4 <8
reevaluate in 1 hr hr
Initial recommended furosemide dosing = 20-mg bolus or 3-mg/hr infusion. Recommended 2-fold incremental increase in subsequent furosemide doses up to a
a

maximum of 160-mg bolus/24-mg/hr infusion rate or until goal CVP/pulmonary artery occlusion pressure achieved. See clinical trial publication for further details.
CVP = central venous pressure; MAP = mean arterial pressure.

5. NMBAs
a. Early administration (within 48 hours of ARDS onset) of cisatracurium over 48 hours has
consistently improved oxygenation without increasing the risk of neuromuscular weakness.
b. The ACURASYS trial showed decreased 90-day adjusted mortality with cisatracurium compared
with placebo (HR 0.68; 95% CI, 0.48–0.98; p=0.04). Subgroup analysis suggested the greatest
mortality benefit in patients with ARDS having a Pao2 /Fio2 less than 120 mm Hg. Cisatracurium
was also associated with significantly more ventilator-free days (up to 28 and 90 days) and organ
failure–free days (up to 28 days). Cisatracurium did not significantly increase the risk of ICU-
acquired neuromuscular weakness (N Engl J Med 2010;363:1107-16; Crit Care Med 2017;45:446-
53).

ACCP Updates in Therapeutics® 2022: Critical Care Pharmacy Preparatory Review and Recertification Course

180
Pulmonary Disorders I

c. The ROSE trial showed no significant differences in 90-day mortality between cisatracurium
(42.5%) and placebo (42.8%) (between-group difference -0.3 percentage points; 95% CI, -6.4 to
5.9; p=0.93). No differences between study groups were found for secondary end points at day 28,
including in-hospital mortality, days free of MV, and days not in the ICU or hospital (N Engl J Med
2019;380:1997-2008).
d. The optimal strategy for NMBA dosing and monitoring in ARDS remains debatable. The
ACURASYS and ROSE trials had similar dosing strategies of a cisatracurium 15-mg bolus followed
by continuous infusion at 37.5 mg/hour over 48 hours without titration or train-of-four monitoring.
Additional boluses were allowed in patients with elevated plateau pressures (i.e., greater than 30 cm
H2O), given concerns for increased risk of ventilator-induced lung injury.
e. The 2019 clinical guidelines recommend that early and short-course NMBAs be considered in
patients with moderate-severe ARDS (Pao2 /Fio2 less than 150 mm Hg) (BMJ Open Respir Res
2019;6:e000420; Ann Intensive Care 2019;9:69). However, these guidelines were published before
the results of the ROSE trial.
6. Corticosteroids
a. Proposed rationale for corticosteroid use in ARDS was to prevent fibroproliferation with subsequent
alveolar fibrosis from a proinflammatory response.
b. Steroids may improve oxygenation, but randomized trials found no mortality benefit. Although the
optimal timing of use remains unknown, consideration may be given in patients with early (less
than 7 days) and late (7 days or more), whereas initiation beyond 14 days after onset was associated
with higher death rates (N Engl J Med 2006;354:1671-84; JAMA 1998;280:159-65; Crit Care Med
2017;45:2078-88).
c. Published guidelines provide minimal direction for clinical practice decision-making regarding
the best approach for corticosteroid use in ARDS (BMJ Open Respir Res 2019;6:e000420; Ann
Intensive Care 2019;9:69; Crit Care Med 2017;45:2078-88). The 2017 guidelines suggest use in
selected patients with ARDS, whereas neither of the 2019 guidelines provides any recommendations
because of the paucity of quality data (Table 4).
Table 4. Clinical Practice Guideline Recommendations for Corticosteroids in ARDS
Source Summary GRADE Recommendation
Suggest use in early moderate to severe ARDS
Crit Care Med Conditional recommendation;
(Pao2 /Fio2 < 200 mm Hg AND within 14 days
2017;45:2078-88 moderate quality of evidence
of onset)
Limited quality data, resulting in inability to
BMJ Open Respir Res
make a recommendation; rather, states that Research recommendation
2019;6:e000420
further research is warranted
Ann Intensive Care 2019;9:69 Not reported Not reported

d. The DEXA-ARDS clinical trial was the first randomized, placebo-controlled clinical trial
investigating dexamethasone for patients with early moderate to severe ARDS (Lancet Respir Med
2020;8:267-76). The corticosteroid group consisted of dexamethasone 20 mg intravenously daily
over the first 5 days, followed by 10 mg intravenously daily for up to 10 days or upon extubation
(if occurring prior to day 10). Dexamethasone was associated with increased mean ventilator-
free days over placebo at 1 month (12.3 ± 9.9 and 7.5 ± 9.9 days, respectively; p<0.0001). Other
secondary outcomes, including mortality (all-cause, ICU, in-hospital) and MV duration (among all
ICU survivors at day 60), significantly favored the dexamethasone group. Of note, hyperglycemia,
secondary infections, and barotrauma were similar between study groups. The novelty of this trial
compared with previously published corticosteroid studies was the concurrent use of contemporary
lung-protective MV strategies. Of importance, this clinical trial was released after publication of the
clinical practice guidelines mentioned earlier.

ACCP Updates in Therapeutics® 2022: Critical Care Pharmacy Preparatory Review and Recertification Course

181
Pulmonary Disorders I

7. Corticosteroids in COVID-19
a. Currently, clinical practice guidelines strongly recommend the use of dexamethasone in ICU
patients with COVID-19 (Table 5).
Table 5. Clinical Practice Guideline Recommendations for Corticosteroids in Critically Ill Patients with COVID-19
Source Summary GRADE Recommendation
www.covid19treatmentguidelines.nih.gov Recommends dexamethasone for
Strong recommendation
Updated July 8, 2021 most patients
www.idsociety.org/COVID19guidelines Recommends dexamethasone rather Strong recommendation,
Updated June 25, 2021 than no dexamethasone moderate level of evidence
• Recommends using a short
course of systemic corticosteroids • Strong recommendation,
over not using corticosteroids moderate level of evidence
Crit Care Med 2021;49:e219-234
• Suggests specifically using • Weak recommendation,
dexamethasone over other very low level of evidence
corticosteroid derivatives

b. The landmark RECOVERY clinical trial compared dexamethasone 6 mg daily up to 10 days total
administered either orally or intravenously compared to usual care alone in 6425 hospitalized
patients with COVID-19 (N Engl J Med 2021;384:693-704). Among 1007 patients in the prespecified
subgroup analysis requiring invasive MV, the 28-day morality rate was significantly lower in the
dexamethasone group (29.3%) than in the usual care group (41.4%) (RR 0.64; 95% CI, 0.51–0.81).
In addition, the probability of hospital discharge within 28 days among those receiving invasive
MV significantly improved in patients receiving dexamethasone over usual care (RR 1.45; 95%
CI, 1.13–1.85). Moreover, MV cessation was more likely with dexamethasone than with usual care
(RR 1.47; 95% CI, 1.20–1.78). Of note, use of dexamethasone lowered overall mortality among
all hospitalized patients compared with usual care. However, a survival benefit was only observed
among patients with COVID-19 requiring oxygen support (both invasive and noninvasive) over
those without any respiratory support.
c. The CoDEX clinical trial randomized 299 adult patients with COVID-19 with moderate to severe
ARDS to either dexamethasone (20 mg intravenously daily over 5 days followed by 10 mg intravenously
daily for another 5 days or until ICU discharge) or usual care (JAMA 2020;6;324:1307-16). The
primary end point was ventilator-free days during the initial 28 days consisting of the number of
days alive and liberation from MV for 48 hours or more. The mean number of ventilator-free days
in the dexamethasone and usual care groups was 6.6 days and 4.0 days (adjusted difference of 2.26
days; 95% CI, 0.2–4.38; p=0.04), respectively. No differences were observed between study groups
for secondary aims, including 28-day mortality, ICU-free days, overall MV days, or risk of adverse
events. A limitation of this trial is that it was terminated early because of the RECOVERY trial
findings publication. This may have introduced bias despite showing a positive benefit associated
with dexamethasone.

ACCP Updates in Therapeutics® 2022: Critical Care Pharmacy Preparatory Review and Recertification Course

182
Pulmonary Disorders I

d. The REACT working group conducted a meta-analysis of seven randomized clinical trials
involving 1703 critically ill patients with COVID-19. This trial evaluated the impact of systemic
corticosteroids, including dexamethasone, methylprednisolone, or hydrocortisone, on 28-day
mortality (JAMA 2020;324:1330-41). Overall pooled data among all trials showed that systemic
corticosteroids significantly reduced the risk of all-cause mortality (summary OR 0.66; 95% CI,
0.53–0.82; p<0.001). Subgroup analysis showed the survival benefit favored dexamethasone over
no steroids according to pooled data from three clinical trials. However, of note, this benefit with
dexamethasone was largely driven by the RECOVERY trial consisting of 57% of the overall pooled
data. The remaining clinical trials consisting of hydrocortisone (n=3) and methylprednisolone
(n=1) showed that these agents did not affect survival according to subgroup analysis.
e. The optimal dexamethasone dosing strategy remains unknown, though most clinicians have
adopted the 6-mg/day approach used in the RECOVERY trial. Of note, higher dexamethasone
dosing strategies (20 mg/day) have not shown additional beneficial effects compared with lower
doses (6 mg daily) (JAMA 2020;324:1330-41).
8. Inhaled pulmonary vasodilators
a. The most commonly used agents in the ICU are inhaled nitric oxide and inhaled epoprostenol. Both
provide selective pulmonary vasodilation, resulting in decreased pulmonary vascular resistance
as well as improved ventilation/perfusion mismatching and arterial oxygenation (J Pharm Pract
2019;32:347-60).
b. Inhaled nitric oxide and inhaled epoprostenol have improved gas exchange, despite their lack of
effect on clinical outcomes (length of stay and mortality). Limited data suggest these agents are
equally efficacious. A meta-analysis of randomized clinical trials comparing inhaled nitric oxide
and placebo showed no effect on mortality in patients with ARDS (RR 1.10; 95% CI, 0.94–1.29),
patients with a baseline Pao2 / Fio2 of 100 mm Hg or less (RR 1.01; 95% CI, 0.78–1.32), or patients
with a baseline Pao2 /Fio2 greater than 100 mm Hg (RR 0.89; 95% CI, 0.89–1.42) (Crit Care Med
2014;42:404-12)
c. Institutions have used inhaled epoprostenol as a more cost-effective option over inhaled nitric
oxide. Direct comparisons between these studies suggest safety and efficacy were similar. However,
inhaled epoprostenol has been associated with significant cost savings over inhaled nitric oxide.
Major limitations of widespread inhaled nitric oxide use for ARDS include high costs and dedicated
equipment for drug delivery. Given that cost is the primary differentiating factor between these
two agents, many institutions have transitioned from inhaled nitric oxide to inhaled epoprostenol.
Implementation of inhaled epoprostenol delivery systems and processes requires continuous
education and pharmacovigilance to mitigate the risk of medication errors and preventable adverse
drug events.
d. The 2019 clinical guidelines did not evaluate inhaled epoprostenol; thus, they do not provide
any recommendations on its use in ARDS. However, one of the 2019 guidelines states not to
use inhaled epoprostenol in ARDS because of weak quality of evidence (BMJ Open Respir Res
2019;6:e000420), whereas the other provides an expert opinion statement for consideration in
severe ARDS for patients currently receiving lung-protective ventilation and proning, but before
ECMO (Ann Intensive Care 2019;9:69).

ACCP Updates in Therapeutics® 2022: Critical Care Pharmacy Preparatory Review and Recertification Course

183
Pulmonary Disorders I

e. Despite the paucity of data for inhaled epoprostenol, it is commonly used in the ICU for ARDS as a
bridge to more invasive management strategies (i.e., ECMO) and/or as salvage therapy. Although the
optimal dosing regimen remains unknown, the most commonly used doses include a weight-based
strategy (typically 10–50 ng/kg/minute titrated to effect) (J Pharm Pract 2019;32:347-60). A small
prospective study evaluated a dose-response relationship of weight-based inhaled epoprostenol
in patients with ARDS, titrating from 0 to 50 ng/kg/minute by increments of 10 ng/kg/minute
every 30 minutes (Chest 2000;117:819-27). A significant increase in the absolute median Pao2 /
Fio2 was observed with the 50 ng/kg/minute weight-based dose compared with baseline (202.2
vs. 187.2 mm Hg, respectively; p<0.008). However, no significant difference was observed with
the median Pao2 /Fio2 (183.2 mm Hg) associated with the 10 ng/kg/minute regimen compared with
baseline. Furthermore, no statistically significant difference was found between the 10 and 50 ng/
kg/minute weight-based regimens regarding oxygenation indices. A major caveat of this study was
small sample size (n=9), and the patient population was not reflective of contemporary clinical use
of inhaled epoprostenol in moderate to severe ARDS, given that only three patients had a baseline
Pao2 /Fio2 less than 150 mm Hg. Recently, one study observed a similar effect on oxygenation using
fixed-dose inhaled epoprostenol without titration compared with inhaled nitric oxide (J Intensive
Care Med 2021;36:466-76).
9. High-dose vitamin C
a. The CITRIS-ALI trial compared high-dose vitamin C (50 mg/kg every 6 hours over 96 hours) and
placebo in adult patients with ARDS and sepsis (JAMA 2019;322:1261-70). Primary outcomes
were organ dysfunction (modified SOFA [sequential organ failure assessment] score) and plasma
inflammatory biomarker concentrations (C-reactive protein and thrombomodulin) at 96 hours from
baseline. Vitamin C showed no significant difference for any of the primary outcomes. However,
a significant reduction in 28-day mortality was associated with vitamin C compared with placebo
(29.8% vs. 46.3%, p=0.03). Vitamin C also significantly increased ICU- and hospital-free days to
day 60. Although these results appear to affect clinical outcomes positively, they must be interpreted
with caution. The study was not aimed or powered to detect a difference in outcomes, including
mortality. Thus, although intriguing, the secondary outcomes should be considered hypothesis-
generating until corroborated from further research.

ACCP Updates in Therapeutics® 2022: Critical Care Pharmacy Preparatory Review and Recertification Course

184
Pulmonary Disorders I

Patient Cases

1. A 56-year-old man is admitted to the ICU with ARDS after experiencing increasing dyspnea during the past
24 hours. His medical history is significant for alcoholism and hypertension. Results of the initial arterial
blood gas are as follows: pH 7.24, Paco2 58 mm Hg, HCO3 24 mEq/L, Pao2 50 mm Hg, and Sao2 84% while
receiving MV AC mode with Fio2 100%. Chest radiography reveals diffuse bilateral infiltrates. The patient
has blood pressure 120/40 mm Hg (MAP 67 mm Hg), heart rate 142 beats/minute, and CVP 8 mm Hg while
receiving a norepinephrine (10 mcg/minute) infusion after intravenous fluid resuscitation. Norepinephrine
has now been weaned off while maintaining a MAP of 67 mm Hg. Ceftriaxone 1 g intravenously every 24
hours and levofloxacin 750 mg intravenously every 24 hours have been initiated for community-acquired
pneumonia. Which is the best therapeutic plan for the patient’s ARDS?
A. Continue fluid resuscitation to maintain a CVP of 10–14 mm Hg; low tidal volume strategy of 4–8 mL/
kg of ideal body weight; prone positioning; and sedative administration to target deep sedation and
cisatracurium infusion.
B. Begin diuresis to target a CVP less than 4 mm Hg; low tidal volume strategy of 4–8 mL/kg of ideal
body weight; supine positioning; and sedative administration to target deep sedation and cisatracurium
infusion.
C. Begin diuresis to target a CVP less than 4 mm Hg; low tidal volume strategy of 4–8 mL/kg of ideal body
weight; supine positioning; and sedative administration to target deep sedation.
D. Discontinue fluid resuscitation and begin diuresis to target a CVP less than 4 mm Hg while maintaining
a MAP greater than 65 mm Hg; low tidal volume strategy of 4–8 mL/kg of ideal body weight; prone
positioning; and sedative administration to target deep sedation and cisatracurium infusion.

2. A 70-year-old woman (height 63 inches, weight 65 kg) is transferred to your ICU from an outside hospital
after outside hospital admission for hypoxic respiratory failure. She had been treated for ARDS at the outside
hospital for 3 days before her son requested hospital transfer. On admission, the patient is receiving MV with
the following settings: SIMV mode, tidal volume 600 mL (12 mL/kg), respiratory rate 12 breaths/minute, PS
10 cm H2O, and PEEP 10 cm H2O. Which is the best therapy for her ARDS?
A. AC/VC mode, tidal volume 300 mL (6 mL/kg), respiratory rate 20 breaths/minute, PS 10 cm H2O, PEEP
5 cm H2O; supine positioning.
B. AC/VC mode, tidal volume 300 mL (6 mL/kg), respiratory rate 20 breaths/minute, PS 10 cm H2O, PEEP
5 cm H2O; prone positioning; cisatracurium administration.
C. SIMV mode, tidal volume 300 mL (6 mL/kg), respiratory rate 20 breaths/minute, PS 10 cm H2O, PEEP
5 cm H2O; supine positioning; cisatracurium administration.
D. AC/VC mode, tidal volume 300 mL (6 mL/kg), respiratory rate 20 breaths/minute, PS 10 cm H2O, PEEP
5 cm H2O; prone positioning.

II.  INTUBATION

A. Endotracheal Intubation
1. Provides access for suctioning of tracheobronchial secretions, maintains a patent airway, and allows
administration of medications
2. Indications include airway protection, facilitation of ventilation and oxygenation, assurance of airway
patency, and anesthesia and surgery.

ACCP Updates in Therapeutics® 2022: Critical Care Pharmacy Preparatory Review and Recertification Course

185
Pulmonary Disorders I

3. Orotracheal intubation is preferred for elective and emergency cases.


4. Nasotracheal intubation is beneficial for patients undergoing maxillofacial surgery or dental procedures
and for patients with limited mouth opening. May be associated with increased risk of bleeding and
sinusitis and should be avoided in patients with severe nasal or midface trauma
5. Complications include insertion trauma, gastric aspiration, hypoxemia, laryngospasm, esophageal
intubation, right main bronchus intubation, cardiac arrhythmias, and hemodynamic impairment

B. Rapid Sequence Intubation (RSI)


1. Rapid and sequential administration of a rapid-acting induction agent and a NMBA to facilitate intubation
and decrease the risk of aspiration
2. Series of seven distinct steps: preparation, preoxygenation, pretreatment, induction either prior or
with paralysis, protection and positioning, placement of the tube in the trachea, and management after
intubation

C. Pretreatment
1. Occurs before an induction agent or NMBA is administered
2. Pretreatment attenuates the sympathetic and parasympathetic responses (catecholamine release,
hypertension, tachycardia, potentially increased ICP in patients with impaired cerebral autoregulation)
to laryngoscopy.
3. Fentanyl or lidocaine can be used as a pretreatment medication (Table 6).
4. Atropine and defasciculating doses of nondepolarizing NMBAs are not recommended for routine use in
RSI for adult patients.

Table 6. Pretreatment Agents


Agent Dose Onset Duration Advantages Disadvantages
• Blunts hypertensive response
• Chest wall rigidity
from intubation
(doses > 100 mcg/kg)
Fentanyl IV: 1–3 • Recommended over other
< 30 s 0.5–1 hr • Hypotension,
(Sublimaze) mcg/kg opioids because of its rapid
bradycardia, and
onset and short duration of
respiratory depression
action
• May prevent increase in ICP
through blunting of cough • Contraindicated in
reflex patients with an amide
Lidocaine IV: 1.5
45–90 s 10–20 min • May reduce bronchospasm in anesthetic allergy,
(Xylocaine) mg/kg
patients with reactive airway bradycardia, or severe
disease heart block

ICP = intracranial pressure; IV = intravenous(ly).

D. Induction Agents
1. Given as rapid intravenous push immediately before the paralyzing agent to help achieve optimal
conditions for intubation
2. Agents should provide rapid loss of consciousness, analgesia, amnesia, and stable hemodynamics.
3. Agents used for induction during RSI include barbiturates, benzodiazepines (midazolam), etomidate,
ketamine, and propofol (Table 7).
4. Barbiturates
a. Thiopental is no longer available in the United States.

ACCP Updates in Therapeutics® 2022: Critical Care Pharmacy Preparatory Review and Recertification Course

186
Pulmonary Disorders I

b. Methohexital is rarely used because of its adverse effect profile, including respiratory depression,
hypotension, and histamine release.
5. Etomidate
a. A nonbarbiturate, imidazole derivative with a rapid onset of action and a very short elimination
half-life
b. Enhances the effects of γ-aminobutyric acid, thereby blocking neuroexcitation and inducing
unconsciousness (does not provide analgesia)
c. Transiently inhibits the conversion of cholesterol to cortisol by inhibiting 11ß-hydroxylase, leading
to transient adrenal suppression
i. No convincing or consistent evidence suggests that etomidate is associated with an increased
risk of death (Intensive Care Med 2011;37:901-10; Chest 2015;147:335-46).
ii. Large randomized, prospective, adequately powered studies are needed to clarify the clinical
significance of etomidate’s effects in patients at risk of adrenal insufficiency.

Table 7. Induction Agents


Agent Dose Onset Duration Advantages Disadvantages

• Minimal cardiovascular • Myoclonic jerks


IV:
Etomidate effects • Transient decrease in
0.2–0.6 10–20 s 4–10 min
(Amidate) • Decreased ICP with minimal cortisol production
mg/kg
effects on cerebral perfusion

• Catecholamine reuptake • Negative inotropic/


inhibition (transient increase chronotropic effects (in
in blood pressure and heart catecholamine-depleted
IV: 1–2 IV: 5–15
IV: 10–15 s rate) patients)
mg/kg min
Ketamine • Respiration and airway • Emergence delirium,
(Ketalar) IM: 3–4 reflexes maintained nightmares, and
IM: 4–10 IM: 12–25
min • Does not increase ICP hallucinations
mg/kg min
• Relieves bronchospasm (premedication with a
• Has both amnestic and benzodiazepine does not
analgesic effects reduce the incidence)
• Compared with other
• Recommended over other induction agents, slower
IV or IM:
Midazolam benzodiazepines because of onset and longer duration
0.2–0.3 60–90 s 1–4 hr
(Versed) its relatively faster onset of • Dose-dependent
mg/kg
action respiratory depression
and hypotension
• Decreases ICP; however,
IV: • Hypotension and
Propofol may also decrease CPP
1.5–2.5 15–45 s 3–10 min bradycardia
(Diprivan) • Mild bronchodilating effects
mg/kg • Negative inotropic effects
• Drug of choice in pregnancy
CPP = cerebral perfusion pressure; IM = intramuscular(ly).

ACCP Updates in Therapeutics® 2022: Critical Care Pharmacy Preparatory Review and Recertification Course

187
Pulmonary Disorders I

E. NMBAs
1. Block impulse transmission at the neuromuscular junction, resulting in skeletal muscle paralysis
2. Used immediately after induction to help achieve optimal conditions for intubation
3. Have no sedative, analgesic, or amnestic properties
4. Problematic in patients with a difficult or failed airway
5. Depolarizing NMBAs (Table 8)
a. Succinylcholine: Noncompetitively binds to acetylcholine receptors, leading to sustained
depolarization of the neuromuscular junction and prevention of muscle contraction
b. Preferred agent for RSI
6. Nondepolarizing NMBAs (Table 8)
a. Rocuronium and vecuronium: Competitive antagonists of acetylcholine at the neuromuscular
junction, leading to the prevention of muscle contraction
b. Intermediate-acting nondepolarizing agents are alternatives when succinylcholine is contraindicated.
c. Usually have a slower onset of action and longer duration of action
7. Reversal of nondepolarizing NMBAs with a failed airway
a. An acetylcholinesterase inhibitor (neostigmine or pyridostigmine) in combination with an
anticholinergic (atropine or glycopyrrolate)
b. Sugammadex (Bridion) binds the aminosteroid class of nondepolarizing NMBAs (vecuronium and
rocuronium).

Table 8. Common Neuromuscular Blocking Agents


Agent Dose Onset Duration Cautions
• Prolonged effects in pseudocholinesterase
deficiency
IV: 1–2 mg/kg • Hyperkalemia or patients at risk of
IV: 3–5 min
IV: 1 min hyperkalemia (prolonged immobilization,
Succinylcholine
IM: 3–4 mg/ crush injuries, myopathies, burns, muscular
(Anectine) IM: 10–30
kg (max 150 IM: 2–3 min dystrophy, stroke, and spinal cord injuries)
min
mg) • Malignant hyperthermia
• Bradycardia/hypotension with repeated doses
• Mild increase in ICP
• Moderate increase in duration with liver
Rocuronium IV: 0.6–1.2
1–2 min 30–60 min dysfunction, minimal increase in duration
(Zemuron) mg/kg
with renal dysfunction
Vecuronium IV: 0.08–0.1 • Prolonged duration in renal and liver
2–3 min 20–60 min
(Norcuron) mg/kg dysfunction

F. Management After Intubation


1. Provide continued sedation/analgesia as needed if an intermediate-acting NMBA was used and assists
in adequate oxygenation and ventilation
2. Minimize long-term use of analgesics and sedatives
3. Maintain head-of-bed elevation at 30–45 degrees
4. Mouth and eye care
5. Bowel regimen
6. Stress ulcer and deep venous thrombosis prophylaxis

ACCP Updates in Therapeutics® 2022: Critical Care Pharmacy Preparatory Review and Recertification Course

188
Pulmonary Disorders I

Patient Cases

3. A 55-year-old man (weight 75 kg) is admitted to the burn ICU after a 65% total body surface area burn to the
abdomen, back, and lower extremities from a house fire. He is unconscious and unable to protect his airway.
His medical history is significant for hypertension and hyperlipidemia. He is currently receiving high-dose
norepinephrine and vasopressin to maintain a MAP of 65 mm Hg. His current laboratory test results show
the following: sodium (Na) 130 mEq/L, potassium (K) 5.9 mEq/L, chloride (Cl) 122 mEq/L, carbon dioxide
(CO2) 15 mg/dL, blood urea nitrogen (BUN) 10 mg/dL, and serum creatinine (SCr) 1.3 mg/dL. Which group
of medications would be most appropriate for RSI?
A. Fentanyl, propofol, rocuronium.
B. Fentanyl, ketamine, succinylcholine.
C. Fentanyl, etomidate, rocuronium.
D. Fentanyl, propofol, succinylcholine.

4. A 39-year-old homeless man (weight 70 kg) was admitted to the neurosciences ICU with a traumatic head
injury after falling off a 3-ft ladder while intoxicated. Imaging reveals a subdural hematoma. The team
decides to intubate this patient. His current laboratory values are as follows: Na 133 mEq/L, K 4.5 mEq/L,
Cl 97 mEq/L, CO2 28 mg/dL, BUN 13 mg/dL, SCr 0.7 mg/dL, and glucose 140 mg/dL. Which induction
medication would be most appropriate to use for RSI?
A. Propofol 90 mg intravenous push.
B. Ketamine 100 mg intravenous push.
C. Midazolam 15 mg intravenous push.
D. Etomidate 150 mg intravenous push.

III.  MECHANICAL VENTILATION

A. Critical to Understanding How MV Works: A fundamental knowledge of acid-base disorders and normal
respiratory physiology

B. Two Essential Categories of Respiratory Failure: Hypercapnic and hypoxemic. Derangements in Pao2 or Paco2
will help determine the cause of respiratory failure. (Table 9 provides the context for normal oxygenation and
ventilation values.)

C. Modes
1. Assist control (AC) ventilation
a. Volume control (VC)
i. The patient receives a predetermined respiratory rate and tidal volume, with additional patient-
initiated breaths provided at the preset tidal volume. Patient-initiated respiration generates a
negative pressure within the ventilator circuit, which is sensed by the ventilator, and a full tidal
volume breath is provided.
ii. Potential for ventilator dyssynchrony, “double-stacking,” and respiratory alkalosis
iii. Mode used in the ARDSNet trial of tidal volume strategy to limit spontaneous tidal volumes
(N Engl J Med 2000;342:1301-8)

ACCP Updates in Therapeutics® 2022: Critical Care Pharmacy Preparatory Review and Recertification Course

189
Pulmonary Disorders I

b. Pressure control
i. The patient will receive a breath at a fixed rate until a predetermined peak pressure limit is
reached. The tidal volume is variable and limited by the peak pressure limit.
ii. Not ideal for patients with low minute ventilation and may lead to hypoventilation and further
hypoxia
2. Synchronized intermittent mandatory ventilation (SIMV): Patients will receive a predetermined
respiratory rate and tidal volume plus additional spontaneous, self-generated breaths at whatever tidal
volume they can generate. Not ideal for the treatment of ARDS, given patients’ ability to exceed the
present tidal volume for spontaneous breaths in excess of 6 mL/kg
3. PS ventilation
a. Usually used as a weaning mode of MV from a more intensive mode of MV (i.e., AC ventilation)
b. The patient initiates each breath with assistance from the ventilator in the form of a preset pressure
value. The ventilator is set to provide a correct amount of pressure to assist with each inspiratory
effort. The tidal volume and respiratory rate depend on the patient.

D. Ventilator Variables
1. Fio2
a. Amount of oxygen that is delivered with each breath, from 21% to 100% at sea level
b. Fio2 is generally tapered to provide the minimal oxygenation needed to meet patient needs. Concerns
for oxygen toxicity with high Fio2 requirements over prolonged period
2. Tidal volume
a. Volume of air inspired in a breath (delivered by MV or spontaneously)
b. Set according to oxygenation and ventilation requirements. Patients with ARDS are treated with a
low tidal volume strategy.
3. Respiratory rate
a. Set to provide a minimal number of breaths from the ventilator at the set tidal volume
b. Titrated by minute ventilation, Paco2, and pH. Minute ventilation (liters per minute) = tidal volume
(liters) × respiratory rate (breaths/minute).
4. Flow rate
a. Velocity of air delivered
b. Velocity is greatest initially upon inspiration and decelerates toward the end of the inspiratory
effort.
5. PEEP
a. Positive pressure in the alveoli during expiration
b. Pressure at the end of expiration promotes alveoli recruitment while minimizing the risk of collapse
in ARDS.
c. Titrated with Fio2 to provide the minimum necessary support to meet the patient’s oxygenation
requirements

Table 9. Normal Respiratory Physiology Values


Value Normal Range
Tidal volume (mL/kg) 5–10
Respiratory rate (breaths/min) 12–20
Minute ventilation (L/min) 5–10
Paco2 (mm Hg) 35–45
Pao2 (mm Hg) 80–100
Sao2 (%) 95–100

ACCP Updates in Therapeutics® 2022: Critical Care Pharmacy Preparatory Review and Recertification Course

190
Pulmonary Disorders I

Patient Case

5. A 74-year-old woman (height 63 inches, weight 65 kg) is transferred to your ICU from an outside hospital
after admission for hypoxic respiratory failure. She had been treated at the outside hospital for ARDS for 3
days before her son requested transfer. When the patient arrives in your ICU, she is receiving MV with the
following settings: SIMV mode, tidal volume 600 mL (12 mL/kg), respiratory rate 12 breaths/minute, PS 10
cm H2O, and PEEP 10 cm H2O. Which is the best ventilator plan for treating her ARDS?
A. AC/VC mode, tidal volume 300 mL (6 mL/kg), respiratory rate 20 breaths/minute, PS 10 cm H2O,
PEEP 5 cm H2O.
B. SIMV mode, tidal volume 300 mL (6 mL/kg), respiratory rate 20 breaths/minute, PS 10 cm H2O, PEEP
5 cm H2O.
C. PS mode, PS 10 cm H2O, PEEP 5 cm H2O.
D. SIMV mode, tidal volume 300 mL (6 mL/kg), respiratory rate 10 breaths/minute, PS 10 cm H2O, PEEP
5 cm H2O.

ACCP Updates in Therapeutics® 2022: Critical Care Pharmacy Preparatory Review and Recertification Course

191
Pulmonary Disorders I

REFERENCES

Acute Respiratory Distress Syndrome COVID-19. Available at https://www.idsociety.org/


1. Adhikari NK, Dellinger RP, Lundin S, et al. COVID19guidelines. Accessed August 16, 2021.
Inhaled nitric oxide does not reduce mortality in 12. Buckley MS, Agarwal SK, Garcia-Orr R, et al.
patients with acute respiratory distress syndrome Comparison of fixed-dose inhaled epoprostenol
regardless of severity: systematic review and meta- and inhaled nitric oxide for acute respiratory dis-
analysis. Crit Care Med 2014;42:404-12. tress syndrome in critically ill adults. J Intensive
2. Alhazzani W, Evans L, Alshamsi F, et al. Surviving Care Med 2020 2021;36:466-76.
Sepsis Campaign guidelines on the management 13. Buckley MS, Dzierba AL, Muir J, et al. Moderate
of adults with coronavirus disease 2019 (COVID- to severe acute respiratory distress syndrome man-
19) in the ICU: first update. Crit Care Med agement strategies: a narrative review. J Pharm
2021;49:e219-e234. Pract 2019;32:347-60.
3. Annane D, Pastores SM, Rochwerg B, et al. 14. Combes A, Brodie D, Bartlett R, et al. International
Guidelines to the diagnosis and management of ECMO Network (ECMONet). Position paper for
critical illness-related corticosteroid insufficiency the extracorporeal membrane oxygenation pro-
(CIRCI) in critically ill patients (part 1): Society grams for acute respiratory failure in adult patients.
of Critical Care Medicine (SCCM) and European Am J Respir Crit Care Med 2014;190:488-96.
Society of Intensive Care Medicine (ESICM) 2017. 15. Combes A, Hajage D, Capellier G, et al.
Crit Care Med 2017;45:2078-88. Extracorporeal membrane oxygenation for severe
4. ARDS Definition Task Force. Acute respiratory acute respiratory distress syndrome. N Engl J Med
distress syndrome: the Berlin definition. JAMA 2018;378:1965-75.
2012;307:2526-33. 16. COVID-19 Treatment Guidelines Panel.
5. Acute Respiratory Distress Syndrome Clinical Coronavirus Disease 2019 (COVID-19) Treatment
Trials Network (ARDSNet). Comparison of two Guidelines. National Institutes of Health. Available
fluid-management strategies in acute lung injury. N at https://www.covid19treatmentguidelines.nih.
Engl J Med 2006;354:2564-75. gov/. Accessed August 16, 2021.
6. Acute Respiratory Distress Syndrome Network 17. Dinglas VD, Aronson Friedman L, Colantuoni E,
(ARDSNet). Ventilation with lower tidal volumes et al. Muscle weakness and 5-year survival in acute
as compared with traditional tidal volumes for respiratory distress syndrome survivors. Crit Care
acute lung injury and the acute respiratory distress Med 2017;45:446-53.
syndrome. N Engl J Med 2000;342:1301-8. 18. Fan E, Del Sorbo L, Goligher EC, et al. An official
7. Ashbaugh DG, Bigelow DB, Petty TL, et al. Acute American Thoracic Society/European Society of
respiratory distress in adults. Lancet 1967;2:319-23. Intensive Care Medicine/Society of Critical Care
8. Beitler JR, Shaefi S, Montesi SB, et al. Prone posi- Medicine clinical practice guideline: mechanical
tioning reduces mortality from acute respiratory ventilation in adult patients with acute respiratory
distress syndrome in the low tidal volume era: a distress syndrome. Am J Respir Crit Care Med
meta-analysis. Intensive Care Med 2014;40:332-41. 2017;195:1253-63.
9. Bellani G, Laffey JG, Pham T, et al. Epidemiology, 19. Fowler AA III, Truwit JD, Hite RD, et al. Effect of
patterns of care, and mortality for patients with vitamin C Infusion on organ failure and biomark-
acute respiratory distress syndrome in intensive care ers of inflammation and vascular injury in patients
units in 50 countries. JAMA 2016;315:788-800. with sepsis and severe acute respiratory failure:
10. Bernard GR, Luce JM, Sprung CL, et al. High-dose the CITRIS-ALI randomized clinical trial. JAMA
corticosteroids in patients with acute respiratory 2019;322:1261-70.
distress syndrome. N Engl J Med 1987;317:1565-70. 20. Frat JP, Thille AW, Mercat A, et al.; FLORALI
11. Bhimraj A, Morgan RL, Shumaker AH, et al. Study Group. High-flow oxygen through nasal can-
Infectious Diseases Society of America Guidelines nula in acute hypoxemic respiratory failure. N Engl
on the Treatment and Management of Patients with J Med 2015;372:2185-96.

ACCP Updates in Therapeutics® 2022: Critical Care Pharmacy Preparatory Review and Recertification Course

192
Pulmonary Disorders I

21. Griffiths MJ, McAuley DF, Perkins GD, et respiratory distress syndrome. BMJ Open Respir
al. Guidelines on the management of acute Res 2019;6:e000420.
22. Grissom CK, Hirshberg EL, Dickerson JB, et al. 33. Papazian L, Forel JM, Gacouin A, et al.
Fluid management with a simplified conservative Neuromuscular blockers in early acute respiratory
protocol for the acute respiratory distress syn- distress syndrome. N Engl J Med 2010;363:1107-16.
drome. Crit Care Med 2015;43:288-95. 34. Park SY, Kim HJ, Yoo KH, et al. The efficacy and
23. Guerin C, Reignier J, Richard JC, et al. Prone safety of prone positioning in adults patients with
positioning in severe acute respiratory distress syn- acute respiratory distress syndrome: a meta-anal-
drome. N Engl J Med 2013;368:2159-68. ysis of randomized controlled trials. J Thorac Dis
24. Hu SL, He HL, Pan C, et al. The effect of prone 2015;7:356-67.
positioning on mortality in patients with acute 35. Peek GJ, Mugford M, Tiruvoipati R, et al. Efficacy
respiratory distress syndrome: a meta-analy- and economic assessment of conventional venti-
sis of randomized controlled trials. Crit Care. latory support versus extracorporeal membrane
2014;18:R109. oxygenation for severe adult respiratory failure
25. Lee JM, Bae W, Lee YJ, et al. The efficacy and (CESAR): a multicentre randomised controlled
safety of prone positional ventilation in acute trial. Lancet 2009;374:1351-63.
respiratory distress syndrome: updated study-level 36. RECOVERY Collaborative Group. Dexamethasone
meta-analysis of 11 randomized controlled trials. in hospitalized patients with Covid-19. N Engl J
Crit Care Med 2014;42:1252-62. Med 2021;384:693-704.
26. McAuley DF, Laffey JG, O’Kane CM, et al.; Irish 37. Schmidt M, Bailey M, Sheldrake J, et al.
Critical Care Trials Group. Simvastatin in the Predicting survival after extracorporeal mem-
acute respiratory distress syndrome. N Engl J Med brane oxygenation for severe acute respiratory
2014;371:1695-703. failure: the Respiratory Extracorporeal Membrane
27. Meade MO, Cook DJ, Guyatt GH, et al. Ventilation Oxygenation Survival Prediction (RESP) score.
strategy using low tidal volumes, recruitment Am J Respir Crit Care Med 2014;189:1374-82.
maneuvers, and high positive end-expiratory pres- 38. Scholten EL, Beitler JR, Prisk K, et al. Treatment
sure for acute lung injury and acute respiratory of ARDS with prone positioning. Chest
distress syndrome. JAMA 2008;299:637-45. 2017;151:215-24.
28. Meduri GU, Headley AS, Golden E, et al. 39. Steinberg KP, Hudson LD, Goodman RB, et al.
Effect of prolonged methylprednisolone ther- Efficacy and safety of corticosteroids for persistent
apy in unresolving acute respiratory distress acute respiratory distress syndrome. N Engl J Med
syndrome: a randomized controlled trial. JAMA 2006;354:1671-84.
1998;280:159-65. 40. Sud S, Friedrich JO, Adhikari NK, et al. Effect of
29. Mercat A, Richard JC, Vielle B, et al. Positive end prone positioning during mechanical ventilation
expiratory pressure setting in adults with acute on mortality among patients with acute respiratory
lung injury and acute respiratory distress syn- distress syndrome: a systematic review and meta-
drome. JAMA 2008;299:646-55. analysis. CMAJ 2014;186:E381-90.
30. Munshi L, Del Sorbo L, Adhikari NKJ, et al. Prone 41. Tomazini BM, Maia IS, Cavalcanti AB, et al.
position for acute respiratory distress syndrome. Effect of dexamethasone on days alive and ven-
A systematic review and meta-analysis. Ann Am tilator-free in patients with moderate or severe
Thorac Soc 2017;14(suppl 4):S280-S8. acute respiratory distress syndrome and COVID-
31. National Heart, Lung, and Blood Institute PETAL 19: the CoDEX randomized clinical trial. JAMA
Clinical Trials Network. Early neuromuscular 2020;324:1307-16.
blockade in the acute respiratory distress syn- 42. Truwit JD, Bernard GR, Steingrub J, et al.; The
drome. N Engl J Med 2019;380:1997-2008. National Heart, Lung, and Blood Institute ARDS
32. Papazian L, Aubron C, Brochard L, et al. Formal Clinical Trials Network. Rosuvastatin for sepsis-
guidelines: management of acute respiratory dis- associated acute respiratory distress syndrome. N
tress syndrome. Ann Intensive Care 2019;9:69. Engl J Med 2014;370:2191-200.

ACCP Updates in Therapeutics® 2022: Critical Care Pharmacy Preparatory Review and Recertification Course

193
Pulmonary Disorders I

43. van Heerden PV, Barden A, Michalopoulos N, 3. Klompas M. Potential strategies to prevent ventila-
et al. Dose-response to inhaled aerosolized pros- tor-associated events. Am J Respir Crit Care Med
tacyclin for hypoxemia due to ARDS. Chest 2015;192:1420-30.
2000;117:819-27. 4. Tobin MJ. Medical progress: advances in mechani-
44. Villar J, Ferrando C, Martínez D, et al. cal ventilation. N Engl J Med 2001;344:1986-96.
Dexamethasone treatment for the acute respiratory
distress syndrome: a multicentre, randomised con-
trolled trial. Lancet Respir Med 2020;8:267-76.
45. Weigelt JA, Norcross JF, Borman KR, et al. Early
steroid therapy for respiratory failure. Arch Surg
1985;120:536-40.
46. WHO Rapid Evidence Appraisal for COVID-19
Therapies (REACT) Working Group. Association
between administration of systemic corticoste-
roids and mortality among critically ill patients
with COVID-19: a meta-analysis. JAMA
2020;324:1330-41.

Intubation
1. Albert SG, Ariyan S, Rather A. The effect of
etomidate on adrenal function in critical ill-
ness: a systematic review. Intensive Care Med
2011;37:901-10.
2. Gu WJ, Wang F, Tang L, et al. Single-dose etomi-
date does not increase mortality in patients with
sepsis: a systematic review and meta-analysis of
randomized controlled trials and observational
studies. Chest 2015;147:335-46.
3. Hampton JP. Rapid-sequence intubation and the
role of the emergency department pharmacist. Am
J Health Syst Pharm 2011;68:1320-30.
4. Reynolds SF, Heffner J. Airway management of
the critically ill patient: rapid-sequence intubation.
Chest 2005;127:1397-412.
5. Stollings JL, Diedrich DA, Oyen LJ, et al. Rapid-
sequence intubation: a review of the process and
considerations when choosing medications. Ann
Pharmacother 2014;48:62-76.

Mechanical Ventilation
1. Acute Respiratory Distress Syndrome Network
(ARDSNet). Ventilation with lower tidal volumes
as compared with traditional tidal volumes for
acute lung injury and the acute respiratory distress
syndrome. N Engl J Med 2000;342:1301-8.
2. Cawley MJ. Mechanical ventilation: a tutorial for
pharmacists. Pharmacotherapy 2007;27:250-66.

ACCP Updates in Therapeutics® 2022: Critical Care Pharmacy Preparatory Review and Recertification Course

194
Pulmonary Disorders I

ANSWERS AND EXPLANATIONS TO PATIENT CASES

1. Answer: D 4. Answer: B
The patient has early severe ARDS (for less than Although propofol may promptly lower ICP, it can also
48 hours and Pao2 /Fio2 less than 150 mm Hg) and induce hypotension and thus decrease cerebral perfusion
hemodynamic stability (post-resuscitation MAP greater pressure (Answer A is incorrect). Midazolam could be
than 65 mm Hg). According to the findings of an used as an induction agent; however, it is not the best
ARDSNet-sponsored multicenter trial, he would qualify agent for RSI because of its delayed onset of action
for conservative fluid management (CVP less than 4 mm (Answer C is incorrect). Etomidate may be considered
Hg). In addition, vasopressors should be discontinued in the setting of increased ICP; however, the dose in this
and diuresis initiated to achieve a target CVP less than case is too high (Answer D is incorrect). Ketamine not
4 mm Hg (Answer A is incorrect). Given the timing and only decreases ICP but also prevents fluctuations in ICP
the severity of the patient’s ARDS, he should be placed (Answer B is correct).
in the prone position (Answers B and C are incorrect). In
addition, this patient would qualify for a cisatracurium 5. Answer: A
infusion. Given the timing and severity of his ARDS, he After recognizing that the patient has ARDS, a lung-
qualifies to receive a lung-protective ventilation strategy protective ventilation strategy should be implemented
(tidal volume 4–6 mL/kg of ideal body weight) and (tidal volume 4–6 mL/kg). Choosing a PS or SIMV
diuresis to a CVP less than 4 mm Hg (hemodynamically mode would allow the patient to initiate spontaneous
stable if weaned off vasopressors) while placed in the breaths in excess of the goal tidal volume (Answers B–D
prone position and administered a cisatracurium infusion are incorrect). In the ARDSNet study of tidal volume
(Answer D is correct). strategy, the AC mode was most commonly used to
promote the application of low tidal volumes (Answer
2. Answer: A A is correct).
The patient presents to your ICU after 3 days of care.
Therefore, she does not currently meet the criteria for
being administered cisatracurium or placed in the
prone position (Answers B–D are incorrect). Currently,
the applicable therapy to apply is a lung-protective
ventilation strategy (tidal volume 4–6 mL/kg of ideal
body weight) (Answer A is correct).

3. Answer: C
Propofol, ketamine, and etomidate can all be used
for induction. Propofol may worsen hypotension;
therefore, because this patient is already receiving
vasoactive medications to maintain his blood pressure,
propofol would not be ideal (Answer A is incorrect).
Succinylcholine causes the up-regulation of acetylcholine
receptors, predisposing the muscle fibers to release
excess potassium because they are depolarized, which
leads to significant dysrhythmias or cardiac arrest
(Answers B and D are incorrect). Appropriate induction
and neuromuscular blockade in this patient would be to
administer etomidate (indicated for hemodynamically
unstable patients), fentanyl (providing adequate
analgesia), and rocuronium (does not increase serum
potassium) (Answer C is correct).

ACCP Updates in Therapeutics® 2022: Critical Care Pharmacy Preparatory Review and Recertification Course

195
Pulmonary Disorders I

ANSWERS AND EXPLANATIONS TO SELF-ASSESSMENT QUESTIONS

1. Answer: B 3. Answer: D
This patient has both ARDS and septic shock. In addi- Neuromuscular blocking agents should always be
tion, he has likely had ARDS for less than 48 hours; administered after induction agents (Answers A–C are
therefore, the time to initiation of several treatments is incorrect). In addition, atropine is not routinely rec-
essential. The patient is actively in shock (as evidenced ommended (Answer C is incorrect) for RSI in adult
by his blood pressure), thus making a fluid-conservative patients. Atropine should be kept nearby for patients
strategy (CVP less than 4 mm Hg) impossible (Answer who are at an increased risk of bradycardia during RSI
C is incorrect). Because the time to presentation is less (use of β-blockers, calcium channel blockers, digoxin,
than 48 hours and the patient has severe ARDS, he meets or amiodarone). Induction agents (and pretreatment
the criteria for cisatracurium administration and prone medications) should be administered before NMBAs
positioning, and a treatment plan should include these (Answer D is correct).
two therapies (Answers A and D are incorrect). A ther-
apy plan should include shock resuscitation (fluid-liberal 4. Answer: B
strategy, CVP 10–14 mm Hg), lung-protective ventilation The patient has moderate to severe ARDS. The initial
(tidal volume 4–8 mL/kg of ideal body weight), prone MV strategy is using low tidal volumes (Answers A and
positioning, and cisatracurium administration (Answer B C are incorrect). Considering the patient is hemody-
is correct). namically stable without requiring vasoactive support,
conservative fluid management approaches are war-
2. Answer: B ranted (Answer B is correct). Hemodynamically unstable
According to the Berlin ARDS definition, the category patients may be candidates for liberal fluid management
of acute lung injury was removed in favor of categoriz- strategies (Answer D is incorrect).
ing the severity of ARDS (Pao2 /Fio2 less than 200 mm
Hg) (Answer A is incorrect). Because of the relative dif-
ference in mortality rates, mild and moderate ARDS
cases are less likely to benefit from therapeutic inter-
ventions, given the number needed to treat to show an
effective intervention (Answers C and D are incorrect).
In the trials evaluating prone positioning and cisatracu-
rium, patients with severe ARDS were the most likely to
benefit. Although the criteria used for severe ARDS in
these studies differed from those used in the Berlin defi-
nition (both studies were initiated before publication of
the Berlin ARDS definition), a post hoc analysis shows
a survival benefit in favor of the group with the highest
mortality rate (Answer B is correct).

ACCP Updates in Therapeutics® 2022: Critical Care Pharmacy Preparatory Review and Recertification Course

196
Pulmonary Disorders II
Grace E. Benanti, Pharm.D., BCCCP
Loyola University Medical Center
Chicago, Illinois
Pulmonary Disorders II

Pulmonary Disorders II
Grace E. Benanti, Pharm.D., BCCCP
Loyola University Medical Center
Chicago, Illinois

ACCP Updates in Therapeutics® 2022: Critical Care Pharmacy Preparatory Review and Recertification Course

199
Pulmonary Disorders II

Learning Objectives Self-Assessment Questions


Answers and explanations to these questions may be
1. Design a treatment plan for a CF exacerbation. found at the end of this chapter.
2. Compare treatment strategies for pulmonary hyper-
tension (PH) using the clinical classifications of PH. 1. A 21-year-old woman (height 62 inches, weight
3. Identify classifications and risk factors for increased 50 kg) with a medical history significant for cys-
severity of asthma exacerbation. tic fibrosis (CF) is admitted to the intensive care
4. Outline a treatment plan for patients with acute respi- unit (ICU) with acute respiratory failure requiring
ratory failure caused by asthma exacerbation. mechanical ventilation (MV). After intubation, her
5. Recognize evidence-based treatment options for arterial blood gas results are as follows: pH 7.27,
acute exacerbations of chronic obstructive pulmo- Paco2 45 mm Hg, bicarbonate 22 mEq/L, Pao2 55
nary disease (COPD). mm Hg, and Sao2 88%. Her ventilator settings are
as follows: assist control/volume control (AC/VC)
mode, tidal volume 300 mL (6 mL/kg), respiratory
Abbreviations in This Chapter rate 20 breaths/minute, fraction of inspired oxygen
60%, pressure support 5 cm H2O, and positive end-
CF Cystic fibrosis expiratory pressure (PEEP) 5 cm H2O. Her blood
CFTR Cystic fibrosis transmembrane conductance pressure is 110/70 mm Hg and heart rate is 95 beats/
regulator minute. Which is the best holistic therapy plan?
COPD Chronic obstructive pulmonary disease A. Initiate cefepime 2 g intravenously every 8
CTEPH Chronic thromboembolic pulmonary hours and tobramycin 150 mg intravenously
hypertension every 8 hours; intravenous fluid resuscitation to
CVP Central venous pressure maintain a central venous pressure (CVP) goal
ET Endothelin of 10–14 mm Hg; dornase alfa and hypertonic
ERA Endothelin receptor antagonist saline 7% nebulization.
FEV1 Forced expiratory volume over one second B. Initiate cefepime 2 g intravenously every 8
ICU Intensive care unit hours and tobramycin 500 mg intravenously
MDI Metered dose inhaler every 8 hours; intravenous fluid resuscitation to
mPAP Mean pulmonary arterial pressure maintain a CVP goal of 10–14 mm Hg; dornase
MV Mechanical ventilation alfa and hypertonic saline 7% nebulization.
PAH Pulmonary arterial hypertension C. Initiate cefepime 2 g intravenously every 8
PDE5i Phosphodiesterase type 5 inhibitor hours and tobramycin 150 mg intravenously
PEEP Positive end-expiratory pressure every 8 hours; intravenous fluid resuscitation to
PH Pulmonary hypertension maintain a CVP goal of 10–14 mm Hg.
PVR Pulmonary vascular resistance D. Initiate cefepime 2 g intravenously every 8
RV Right ventricular hours and tobramycin 500 mg intravenously
SABA Short-acting β-agonist every 24 hours; diuresis to maintain a CVP goal
of less than 4 mm Hg while mean arterial pres-
sure is greater than 65 mm Hg; dornase alfa and
hypertonic saline 7% nebulization.

ACCP Updates in Therapeutics® 2022: Critical Care Pharmacy Preparatory Review and Recertification Course

200
Pulmonary Disorders II

2. A 55-year-old woman with pulmonary arterial 4. A 66-year-old man presents to the ICU with acute
hypertension (PAH) is admitted to the ICU for severe respiratory failure from a COPD exacerbation. He
respiratory failure. She reports increased work of has had no exacerbations in the past 2 years. His
breathing for the past 5 days and full adherence home medications include albuterol HFA (hydro-
to her PAH medication regimen, which includes fluoroalkane) 2 puffs four times daily as needed and
macitentan 10 mg daily and sildenafil 40 mg three tiotropium 1 puff once daily. The patient denies any
times daily. Her current vital signs are as follows: recent sick contacts or changes in sputum produc-
blood pressure 76/60 mm Hg, heart rate 140 beats/ tion. He has no known drug allergies. He is placed
minute, respiratory rate 30 breaths/minute, and on 3 L of nasal cannula (Sao2 97%) and inhaled alb-
85% Sao2 on 6 L of nasal cannula. Right heart cath- uterol and ipratropium by nebulization. Which other
eterization reveals the following: mean pulmonary therapy would be most appropriate for this patient?
arterial pressure (mPAP) 50 mm Hg, right arterial A. Azithromycin 500 mg intravenously daily.
pressure 25 mm Hg, cardiac index 1.9 L/minute/m2, B. Prednisone 40 mg orally daily.
and pulmonary capillary wedge pressure 16 mm Hg. C. Levofloxacin 500 mg orally daily.
A transthoracic echocardiogram reveals an ejection D. Methylprednisolone 125 mg intravenously
fraction of 60% with severe right ventricular (RV) every 6 hours.
dilatation. Which regimen would be most appropri-
ate for this patient?
A. Dopamine infusion.
B. Epoprostenol infusion.
C. Phenylephrine infusion.
D. Furosemide intravenous push.

3. A 42-year-old man presents to the emergency depart-


ment (ED) with anxiety and shortness of breath.
Auscultation reveals audible wheezing. He has dif-
ficulty speaking in full sentences. He has used his
albuterol metered dose inhaler (MDI) at home for
the past several hours without symptom resolution.
His forced expiratory volume in 1 second (FEV1) is
35% of predicted. Which best classifies this patient’s
asthma exacerbation?
A. Mild.
B. Moderate.
C. Severe.
D. Life threatening.

ACCP Updates in Therapeutics® 2022: Critical Care Pharmacy Preparatory Review and Recertification Course

201
Pulmonary Disorders II

BPS Critical Care Pharmacy Specialist Examination Content Outline

This chapter covers the following sections of the Critical Care Pharmacy Specialist Examination Content Outline:
1. Domain 1: Clinical Knowledge and Application
a. Task 1: 3
b. Task 2: 3–7
c. Task 3: 1–7
d. Task 4: 1–6, 8
e. Task 5: 1, 2, 5–7
f. Task 6: 1–4
g. Task 7: 1–7
2. Domain 2: Practice Management, Policy, and Quality Improvement
a. Task 1: 2, 3
b. Task 4: 2, 3
c. Task 5: 4
3. Domain 3: Evidence-Based Medicine, Scholarship, Education, and Professional Development
a. Task 1: 2, 3

ACCP Updates in Therapeutics® 2022: Critical Care Pharmacy Preparatory Review and Recertification Course

202
Pulmonary Disorders II

I.  CYSTIC FIBROSIS

A. Cystic Fibrosis – Chronic disease process that affects many organs, including the pancreas, liver, and
intestine, but primarily the lung (Chest 2004;125:1-39)
1. CF is a recessive disorder that is caused by a mutation in the cystic fibrosis transmembrane conductance
regulator (CFTR). CFTR controls the movement of chloride and bicarbonate across the cell membranes of
many organs throughout the body (e.g., lungs, gastrointestinal tract, pancreas) (Chest 2018;154:383-93).
2. CFTR dysfunction within the airways also affects the ability of the airway membranes to absorb sodium
and fluid through the epithelial sodium channel (ENaC), which creates a viscous mucus and impairs
ciliary clearance (Chest 2018;154:383-93).
3. Chronic therapy has been developed to target specific CFTR mutations, and such therapies may be seen
in the ICU as home medications. Several of these agents have important drug-drug interactions to be
aware of as either substrates or inducers of CYP3A4 (Chest 2018;154:383-93).
4. Acute exacerbations include symptoms of increased cough, sputum production, shortness of breath,
weight loss, and a decline in lung function.

B. Pulmonary Exacerbations of CF
1. Definitions and significance
a. There is no universally accepted definition for pulmonary exacerbations of CF (Respir Care
2020;65:233-51).
b. Acute exacerbations include symptoms of increased cough, sputum production, shortness of breath,
weight loss, and a decline in lung function.
c. Pulmonary exacerbations of CF can have a significant impact on the patient’s quality of life and
disease progression. Approximately one quarter of patients with exacerbation will not completely
recover lung function at three months (Am J Respir Crit Care Med 2010;182:627-32).
2. Common bacterial organisms (most common to least common)
a. Staphylococcus aureus is the most common organism isolated from birth to teenage years.
b. Methicillin resistant Staphyloccocus aureus (MRSA) becomes more common in the 11–30 years
of age range.
c. Pseudomonas aeruginosa has a prevalence rate of approximately 15% from birth to teenage years,
but rapidly increases in prevalence after 17 years of age.
d. Haemophilus influenzae
e. Stenotrophomonas maltophilia
f. Burkholderia cepacian complex
g. Achromobacter xylosoxidans
h. Nontuberculous mycobacteria
3. Antibiotic therapy for pulmonary exacerbations of CF (Table 1)
a. Antibiotic dosing is challenging because of the altered pharmacokinetics in patients with CF.
Patients with CF can be distinguished by their large volume of distribution and augmented renal
clearance.
b. It is important to note that most pulmonary exacerbations are not due to the acquisition of a new
microorganism, and patients with CF commonly have routine cultures collected throughout their
care. Taking the patient’s previous susceptibility pattern or restarting a previous successful antibiotic
regimen are common practice.
i. For patients with history of or suspected infection with Pseudomonas aeruginosa, initial
treatment with two antipseudomonal agents is recommended, despite limited evidence that this
practice improves outcomes (Am J Respir Crit Care Med 2009;180:802-8; Cochrane Database
Syst Rev 2021;6:CD002007).

ACCP Updates in Therapeutics® 2022: Critical Care Pharmacy Preparatory Review and Recertification Course

203
Pulmonary Disorders II

ii. Two drugs from different classes are recommended rather than double beta-lactams, with
the most data supporting a beta-lactam and aminoglycoside, though toxicity risk and patient-
specific factors should be taken into account.
iii. Aggressive dosing of β-lactam antibiotics is recommended to optimize time above the minimal
inhibitory concentration, primarily based on pharmacokinetic data.
iv. Extended-interval aminoglycoside dosing (e.g., once-daily dosing of tobramycin 10 mg/kg,
targeting a peak concentration of 20–30 mg/L and a trough concentration of less than 1 mg/L) is
as effective as conventional dosing, with possibly less nephrotoxicity (Lancet 2005;365:573-8).
c. Evidence is currently lacking for the simultaneous administration of inhaled and intravenous
antibiotics for acute CF exacerbations (Am J Respir Crit Care Med 2009;180:802-8; Cochrane
Database of Syst Rev 2018;10:CD010257).
4. Duration of therapy
a. A course of 10–14 days of antibiotics has been common practice with a paucity of data to support
it; durations have been primarily driven by patient’s clinical response, either symptomatology or
FEV1 measurements.
b. The STOP trial reflected this precedent with a mean duration of intravenous antibiotics in adults at
16.2 days (J Cyst Fibros 2017;16:600-6).
c. STOP II trial excluded ICU patients, but identified that for patients who improved within 7 days,
10 days of antibiotics was noninferior to 14 days. Patients who did not respond should complete a
14-day course (Am J Respir Crit Care Med 2021;204:1295-305).

Table 1. Chronic Antimicrobial Therapy


Therapy Clinical benefit(s)
• Improves lung function
Inhaled tobramycina • Improves quality of life
• Reduces exacerbations
• Improves lung function
Inhaled aztreonama
• Improves quality of life
• Improves lung function
Oral azithromycina
• Reduces exacerbations
Oral azithromycinb • Reduces exacerbations
Patients with history of Pseudomonas aeruginosa.
a

b
Patients without history of Pseudomonas aeruginosa.

5. Exacerbation treatment outcomes


a. Providers and practice will vary in defining when treatment is complete for pulmonary exacerbation.
The STOP study identified the most common markers of success for providers including improvement
in FEV1 and patient-reported symptoms (J Cyst Fibros 2017;16:600-6).
b. Though intravenous antibiotic therapy is known to improve lung function during an exacerbation,
eradication of organisms in culture is not required and not expected (Expert Opin Drug Metab
Toxicol 2021;17:53-68).

C. Adjunctive therapy for prevention or treatment of pulmonary exacerbations of CF


1. Airway clearance techniques are a cornerstone of management of lung disease in CF.
a. The type of therapy will be tailored to a patient’s specific needs and preferences; and the use of
these interventions will likely be increased during treatment of a pulmonary exacerbation (Respir
Care 2009;54:522-37).

ACCP Updates in Therapeutics® 2022: Critical Care Pharmacy Preparatory Review and Recertification Course

204
Pulmonary Disorders II

b. High frequency chest wall oscillation vest is the most commonly used airway clearance technique
(after infancy).
c. Positive expiratory pressure/oscillating positive expiratory pressure
d. Physical exercise is recommended as a possible airway clearance therapy and is frequently used as
either primary or secondary clearance for patients teenage or older.
2. Nutrition
a. Providing nutrition during acute exacerbations is key to maintaining metabolic function and
promoting optimal outcomes from potential lung transplantation.
b. Administer pancreatic enzymes to treat exocrine dysfunction.
3. Continuation of patient’s current chronic airway clearance therapy
a. It is recommended to continue nebulized airway clearance therapies during an exacerbation (Table
2); there is limited data to describe the impact of this practice.
b. The only airway clearance therapy that has been studied specifically during pulmonary exacerbations
is hypertonic saline. This study of 132 patients, randomized to either 7% hypertonic saline or
control, failed to detect any difference in the primary outcome of length of stay, but did find patients
in the treatment group were more likely to return to baseline FEV1 and have significantly improved
symptoms by discharge (Thorax 2016;71:141-7).

Table 2. Chronic Airway Clearance Therapy


Therapy Mechanism Clinical benefit(s)
• Enzymatic cleavage of neutrophilic DNA • Improves lung function
Nebulized dornase alfaa
• Reduces mucus viscosity • Improves quality of life
Nebulized hypertonic salinea Osmotically active expectorant • Reduces exacerbations
Nebulized mannitolb Osmotically active expectorant • Improves lung function
a
Mogayzel PJ Jr, Naureckas ET, Robinson KA, et al. Cystic fibrosis guidelines: chronic medications for maintenance of lung health. Am J Respir Crit Care Med
2013;187:680-9.
b
Flume PA, Amelina E, Daines CL, et al. Efficacy and safety of inhaled dry-powder mannitol in adults with cystic fibrosis: an international, randomized controlled study.
J Cyst Fibros 2021;20:1003-9.

4. Data remains unclear on the role of continuing patient’s chronic antimicrobial regimens during an
exacerbation and practice will vary.
5. Corticosteroids
a. Currently, insufficient evidence for administering corticosteroids (Am J Respir Crit Care Med
2009;180:802-8)
b. However, current practice appears at odds with the 2009 guidelines. Though data from several small
studies have been mixed; the STOP trial observed 21% of patients were treated with corticosteroids
during a pulmonary exacerbation (J Cyst Fibros 2017;16:600-6).
c. Attempts to clarify the role of corticosteroids are ongoing. The PIPE trial (currently recruiting)
plans to compare 7 days of oral prednisone with placebo in patients with pulmonary exacerbation on
intravenous antibiotics and who have not recovered their FEV1 after 7 days of antibiotics. Expected
study completion in 2023 (NCT03070522).

ACCP Updates in Therapeutics® 2022: Critical Care Pharmacy Preparatory Review and Recertification Course

205
Pulmonary Disorders II

Patient Case

1. A 20-year-old woman is admitted to the ICU for an acute CF exacerbation. Before admission, her condi-
tion had been stable. She had maintained her ideal body weight and had just completed a home regimen of
suppressive antibiotics. The patient requires MV for the management of hypoxic respiratory failure. She is
initiated on AC/VC mode with a lung-protective strategy (4–8 mL/kg tidal volume). Which would be best
for managing her acute CF exacerbation?
Q. Tobramycin nebulization; hypertonic saline 7% nebulization; and tube feedings to target a hypocaloric
goal during her acute illness (15 kcal/kg/day).
R. Ceftriaxone 2 g intravenously every 24 hours; tobramycin nebulization; normal saline 0.9% nebuliza-
tion; and tube feedings to target her goal caloric intake (25 kcal/kg/day).
S. Piperacillin/tazobactam 4.5 g intravenously every 6 hours; tobramycin 10 mg/kg intravenously once
daily; hypertonic saline 7% nebulization; and tube feedings to target her goal caloric intake (25 kcal/
kg/day).
T. Cefepime 2 g intravenously every 8 hours extended infusion; tobramycin 10 mg/kg intravenously every
8 hours; hypertonic saline 7% nebulization; and tube feedings to target her goal caloric intake (25 kcal/
kg/day).

II.  PULMONARY HYPERTENSION

A. Pathophysiology and Definitions


1. Pulmonary hypertension (PH) is a chronic, life-threatening disease characterized by vascular changes,
including vasoconstriction, cellular proliferation, and thrombosis.
a. Thromboxane A2 (potent vasoconstrictor) concentrations are increased.
b. Prostacyclin (potent vasodilator, inhibitor of platelet aggregation, and antiproliferative properties)
concentrations are decreased.
c. Endothelin-1 (ET-1; potent vasoconstrictor, mitogenic properties on pulmonary artery smooth
muscle cells) concentrations are increased.
d. Nitric oxide concentrations (vasodilator, inhibitor of platelet activation, and inhibitor of vascular
smooth muscle cell proliferation) are decreased.
2. Clinical Classification of PH by the World Health Organization (WHO) (Chest 2019;155:565-86; Eur
Heart J 2016;37:67-119):
a. Group 1 (pulmonary arterial hypertension [PAH]): Idiopathic, heritable, or drug/toxin-induced
PAH, PAH associated with various diseases, long-term responders to calcium channel blockers, or
persistent newborn PH
b. Group 2: PH caused by left heart disease
c. Group 3: PH caused by chronic lung disease and/or hypoxia
d. Group 4: Chronic thromboembolic pulmonary hypertension (CTEPH)
e. Group 5: Unclear multifactorial mechanisms
3. PH is defined as a mean pulmonary arterial pressure (mPAP) of 20 mmHg or greater at rest as measured
by right heart catheterization.
a. Previously defined as mPAP of 25 mm Hg or greater at rest measured by right heart catheterization.
This definition was arbitrary and has since been updated by the 6th World Symposium on PH in
2019 (Eur Respir J 2019;53:1801913; Chest 2019;155:565-86).

ACCP Updates in Therapeutics® 2022: Critical Care Pharmacy Preparatory Review and Recertification Course

206
Pulmonary Disorders II

b. Hemodynamic definitions of PH (Eur Respir J 2019;53:1801913)


i. Groups 1, 3, 4, and 5: Isolated precapillary PH defined as mPAP greater than 20 mmHg,
pulmonary capillary wedge pressure less than or equal to 15 mm Hg, and pulmonary vascular
resistance (PVR) greater than or equal to 3 Wood units
ii. Groups 2 and 5: Isolated postcapillary PH defined as mPAP greater than 20 mmHg, pulmonary
capillary wedge pressure greater than 15 mm Hg, and PVR greater than or equal to 3 Wood
units.
iii. Groups 2 and 5: combined pre- and postcapillary PH defined as mPAP greater than 20 mmHg,
pulmonary capillary wedge pressure greater than 15 mm Hg, and PVR greater than or equal
to 3 Wood units.

B. Diagnosis (Eur Heart J 2016;37:67-119)


1. Initial symptoms are nonspecific
a. Common symptoms: Fatigue, shortness of breath with exertion, chest pain, and potentially syncope
b. Less common presentations: Dry cough, nausea, and vomiting
c. Once PH diagnosis is confirmed, patient’s WHO Functional Class will be assessed using these
symptoms (Table 3).
2. Due to the nonspecific pattern of symptoms, multiple tests will likely be needed to rule out other causes
of such symptoms
a. Electrocardiogram
b. Chest radiograph
c. Pulmonary function tests
d. Arterial blood gas
e. Echocardiogram is used to predict likelihood of PH based on specific findings
i. Peak tricuspid regurgitation velocity: Greater than 3.4 m/s or 2.9–3.4 m/s with other findings
on echocardiogram consistent with PH indicates a high probability of PH.
ii. Various measurements in the ventricles, pulmonary artery, inferior vena cava, and right atrium
can contribute to the echocardiogram
iii. Echocardiogram should not be used independently for decision making about treatment for PH.
f. Ventilation/perfusion scan can be used to identify possible group 4 PH
g. Computed tomography (CT) or CT pulmonary angiography
h. Cardiac magnetic resonance imaging can provide noninvasive prognostic assessment of the right
heart initially and at follow up.
i. Right heart catheterization is required to confirm diagnosis of PH and perform vasoreactivity
testing if appropriate

C. General management of PAH (group 1 PH)


1. Determine WHO Functional Class (FC): Defines patient’s baseline functional status and is monitored
throughout the disease to guide medication therapy (Table 3)

ACCP Updates in Therapeutics® 2022: Critical Care Pharmacy Preparatory Review and Recertification Course

207
Pulmonary Disorders II

Table 3. WHO FC Assessmenta


Class Definition
I No symptoms (dyspnea, fatigue, syncope, chest pain) with normal activities
Symptoms with strenuous normal daily activities that slightly limit functional status and activ-
II
ity level
Symptoms of dyspnea, fatigue, syncope, and chest pain with normal daily activities that
III
severely limit functional status and activity level
IV Symptoms at rest; cannot perform normal daily activities without symptoms
a
Galiè N, Humbert M, Vachiery, J-L, et al. 2015 ESC/ERS guidelines for the diagnosis and treatment of pulmonary hypertension: The Joint Task Force for the Diagnosis
and Treatment of Pulmonary Hypertension of the European Respiratory Society (ERS): Endorsed by Association for European Paediatric and Congenital Cardiology
(AEPC), International Society for Heart and Lung Transplantation (ISHLT). Eur Heart J 2016;37:67-119.

2. Treatment goals for PH


a. Achieve and maintain WHO FC I or II.
b. Preserve 6-minute walk distance to greater than 440 m or 380 m in select patient populations.
c. Preserve right ventricular (RV) size and function (right arterial pressure less than 8 mm Hg and
cardiac index greater than 2.5–3.0 L/minute/m2).
d. Normalize B-type natriuretic peptide.
e. Maintain peak oxygen consumption greater than 15 mL/minute/kg and ventilator equivalent for
carbon dioxide less than 45 L/minute during cardiopulmonary exercise testing.
3. Supportive therapy
a. Oxygen: Maintain Sao2 of 90% or greater and Pao2 of 60 mm Hg or greater.
b. Diuretics should be used for the symptomatic management of RV dysfunction and signs of fluid
overload; choice of diuretic is variable.
c. Digoxin increases cardiac output; consider in patients who develop atrial tachyarrhythmias.
d. Anticoagulation should be considered in idiopathic PAH, heritable PAH, and PAH secondary to
anorexigenic agent use.
e. Aggressive treatment of sleep apnea and hypertension

D. Specific treatment options for group 1 PAH


1. Vasodilator therapy with calcium channel blockers (diltiazem, amlodipine, nifedipine)
a. Patients who should undergo vasoreactivity testing include idiopathic PAH, heritable PAH, and
PAH secondary to toxins or drugs.
b. Considered first-line treatment of the above-described patients who have a positive response to
acute vasoreactivity testing (reduced mPAP by 10 mm Hg or more to an mPAP of 40 mm Hg or less
with increased or unchanged cardiac output) (N Engl J Med 1992;327:76-81)
c. Calcium channel blockers improve mPAP, PVR, WHO FC, and survival in patients with PAH and a
positive vasoreactivity test (Circulation 1987;76:135-41; N Engl J Med 1992;327:76-81).
d. Patients should not be considered candidates for calcium channel blocker therapy if they have RV
dysfunction, depressed cardiac output, or WHO FC IV symptoms.
e. If patients do not respond to calcium channel blocker therapy within 3–4 months, then the therapy
should be discontinued unless another relevant indication exists.
2. Treatment-naïve patients with PAH (nonresponders to vasoreactivity testing or calcium channel blocker
treatment)
a. Drug therapy should be evaluated on the basis of the patient-specific factors below (Chest
2019;155:565-86):
i. Tolerance of combination therapy
ii. Ability and willingness to manage parenteral prostacyclins

ACCP Updates in Therapeutics® 2022: Critical Care Pharmacy Preparatory Review and Recertification Course

208
Pulmonary Disorders II

iii. WHO FC of patient with PAH


iv. Initial therapy resulting in inadequate response
v. Clinical stability
b. Limitations of targeted therapies
i. Indicated only for patients with PAH
ii. Limited data describing efficacy and safety in other PH groups
iii. May result in worsening fluid retention, pulmonary edema, and gas exchange in other PH
groups
iv. Small population size, primarily surrogate markers as outcome measures, limited data in
hospitalized and critically ill patients, and sparse long-term data
v. Outcomes from combination therapy remain elusive.
c. Patients with WHO FC I symptoms at initial diagnosis will be monitored for disease progression
and continually re-evaluated for when to start treatment.
d. Initial guideline recommended therapy for WHO FC II and III is combination therapy with
ambrisentan and tadalafil (N Engl J Med 2015;373:834-44; Chest 2019;155:565-86).
e. If patients with WHO FC II and III are unable or unwilling to tolerate combination therapy,
monotherapy options are described in Table 4.

Table 4. Drug monotherapy summary for treatment-naïve patients with PAHa


WHO FC II WHO FC III WHO FC IV
Endothelin receptor antagonists (ERAs) Recommended b
Recommended b
Not indicated
Phosphodiesterase type 5 inhibitors
Recommendedb Recommendedb Not indicated
(PDE5i)
Guanylate cyclase stimulators Recommendedc Recommendedc Not indicated
Prostacyclins (parenteral) Not indicated c
Recommended c
Recommendedc,d
Prostacyclins (inhaled) Not indicatedc Recommendedb Not indicated
Prostacyclins (oral) Insufficient evidence for or against usee
Selective prostacyclin receptor agonist Insufficient evidence for or against usee
a
Table adapted from Galiè N, Humbert M, Vachiery, J-L, et al. 2015 ESC/ERS guidelines for the diagnosis and treatment of pulmonary hypertension: The Joint Task
Force for the Diagnosis and Treatment of Pulmonary Hypertension of the European Respiratory Society (ERS): Endorsed by Association for European Paediatric and
Congenital Cardiology (AEPC), International Society for Heart and Lung Transplantation (ISHLT). Eur Heart J 2016;37:67-119 and Klinger JR, Elliott CG, Levine DJ,
et al. Therapy for pulmonary arterial hypertension in adults. Chest 2019;155:565-86.
b
Varying strengths of recommendation and level of evidence depending on agent chosen.
Consensus-based level of evidence.
c

d
See “Specific treatment options for group 1 PAH” regarding alternative option.
Recommendation changed from 2016 to 2019 guidelines.
e

PAH = pulmonary arterial hypertension; WHO FC = World Health Organization Functional Class.

f. WHO FC III patients with rapid disease progression or concern for poor prognosis should be
considered for first line parenteral prostacyclin therapy.
g. WHO FC IV who are not willing or not able to manage parenteral prostacyclins are recommended
to start combination therapy with inhaled prostacyclin, PDE5i, and ERA.
h. Failed or inadequate response to monotherapy
i. The sequential addition of another class can be considered.
ii. Combinations in Table 5 are limited to agents with recommendations higher than consensus
statements. For additional options, refer to specific drug class sections or guidelines.

ACCP Updates in Therapeutics® 2022: Critical Care Pharmacy Preparatory Review and Recertification Course

209
Pulmonary Disorders II

Table 5. Failed Monotherapy Option Summarya


Failed Add WHO FC II WHO FC III WHO FC IV
ERA Treprostinil inhaled Not indicated Recommended Recommended
Ambrisentan Tadalafil Recommended Recommended Recommended
Treprostinil inhaled Not indicated Recommended Recommended
PDE5i
Riociguat Not recommended due to drug-drug interaction
Table adapted from Klinger JR, Elliott CG, Levine DJ, et al. Therapy for pulmonary arterial hypertension in adults. Chest 2019;155:565-86.
a

E. Drug Information by Class


1. Endothelin receptor antagonists (ERAs) (Tables 6–7)
a. Competitively block the action of ET-1 from acting on two types of ET receptors: ETa and ETb. This
blockade prevents endogenous ET-1 from its normal actions:
i. ETa = vasoconstriction and cell proliferation
ii. ETb = vasodilation and antiproliferation
b. Minimal place in therapy for initiation in critically ill patients

Table 6. Additional Therapeutic Optionsa


Failed monotherapy Add WHO FC III-IV
PDE5i Macitentan Recommended
Prostacyclins (inhaled) Macitentan Recommended
Consensus-based level of evidence
a

Table 7. Endothelin Receptor Antagonists (oral)


Ambrisentan
Bosentan (Tracleer) Macitentan (Opsumit)
(Letairis)
Receptor affinity Blocks ETa and ETb Blocks ETa only Blocks ETa and ETb
Half-life (hr) Tadalafil Recommended 16
Approved dose 62.5–125 mg BID 5–10 mg daily 10 mg daily
WHO FC II: Delay clinical WHO FC II and III: Delay
WHO FC II and III:
Outcomes worsening clinical worseninga
Improve 6MWDb
WHO FC III: Improve 6MWD WHO FC III: Improve WHO FC
Adverse effects Hepatotoxicity, peripheral edema, anemia, pregnancy category X (REMS programs)
Glyburide (contraindicated) and
Caution with cyclospo-
cyclosporine (contraindicated), CYP2C19 and CYP3A4
Drug interactions rine (max ambrisentan
CYP2C8/9 and CYP3A4 inhibi- inhibitors and inducers
dose is 5 mg daily)
tors and inducers
a
Pulido T, Adzerikho I, Channick RN, et al. Macitentan and morbidity and mortality in pulmonary arterial hypertension. N Engl J Med 2013;369:809-18.
b
Galiè N, Barberà JA, Frost AE, et al. Initial use of ambrisentan plus tadalafil in pulmonary arterial hypertension. N Engl J Med 2015;373:834-44.
6MWD = 6-minute walk distance; BID = twice daily; ET = endothelin; LFT = liver function test; REMS = risk evaluation and mitigation strategies.

2. PDE-5 inhibitors (Table 8)


a. Inhibit PDE-5, which prevents the breakdown of cyclic guanosine monophosphate in pulmonary
vascular smooth muscle. Increased cyclic guanosine monophosphate potentiates relaxing pulmonary
vascular smooth muscle and vasodilation of the pulmonary vascular bed.

ACCP Updates in Therapeutics® 2022: Critical Care Pharmacy Preparatory Review and Recertification Course

210
Pulmonary Disorders II

b. Indicated for patients who continue to be symptomatic on intravenous epoprostenol.


c. Intravenous formulation is available for patients who temporarily cannot ingest tablets; however,
this formulation is limited by its hemodynamic effects.

Table 8. PDE5i
Sildenafil (Revatio) Sildenafil (Revatio) Tadalafil (Adcirca)
Half-life (hr) 4 35
40 mg oral daily; dose adjustment
Approved dose 20 mg oral TID 10 mg IV TID
necessary for renal impairment
WHO FC II and III: Improve 6MWDa
Outcomes WHO FC II and III: Improve 6MWDb
WHO FC III: Improve WHO FCa
WHO FC II and III: Delay clinical
WHO FC III: Improve WHO FC II and III:
Outcomes worseninga
WHO FCa Improve 6MWDb
WHO FC III: Improve WHO FC
Adverse effects Headache, epistaxis, flushing, dyspepsia, hypotension, visual alterations
Contraindicated in patients receiving nitrates; avoid with strong CYP3A4 inhibitors and induc-
Drug interactions
ers; contraindicated with riociguat
a
Klinger JR, Elliott CG, Levine DJ, et al. Therapy for pulmonary arterial hypertension in adults. Chest 2019;155:565-86.
b
Galiè N, Barberà JA, Frost AE, et al. Initial use of ambrisentan plus tadalafil in pulmonary arterial hypertension. N Engl J Med 2015;373:834-44.
IV = intravenously; TID = three times daily.

3. Soluble guanylate cyclase stimulators


a. Riociguat (Adempas) exerts its effect through two mechanisms: Sensitizes endogenous soluble
guanylate cyclase by stabilizing nitric oxide–soluble guanylate cyclase bonding and directly
stimulates soluble guanylate cyclase through an auxiliary binding site. Both processes increase
cyclic guanosine monophosphate, which influences vascular tone, proliferation, fibrosis, and
inflammation.

Table 9. Additional Roles in Therapy for Riociguat


Failed monotherapy Add Clinical outcomes WHO FC III-IV
Bosentan Improved 6MWD Recommendeda
Ambrisentan Riociguat Improve WHO FC Recommendeda
Prostacyclins (inhaled) Delay time to clinical worsening Recommendeda
Consensus-based level of evidence (Chest 2019;155:565-86)
a

b. Currently, riociguat is the only medication approved for patients with group 4 PH (CTEPH) having
residual CTEPH after surgical treatment or inoperable CTEPH to improve exercise capacity (N
Engl J Med 2013;369:319-29).
c. Elimination half-life is 12 hours. If cannot be administered for 3 days or more, retitration is required;
however, a PH specialist should always be involved for issues with access and reinitiation.
d. Approved dose is 1–2.5 mg three times daily (higher doses for smokers).
e. Adverse effects include hypotension, hemoptysis, headache, dizziness, dyspepsia, nausea, diarrhea,
vomiting, and anemia.

ACCP Updates in Therapeutics® 2022: Critical Care Pharmacy Preparatory Review and Recertification Course

211
Pulmonary Disorders II

f. Contraindicated in patients receiving nitrates and phosphodiesterase inhibitors because of


hypotension; avoid with strong CYP3A4/2C8 inhibitors and inducers and with P-glycoprotein/
breast cancer resistance protein inhibitors
g. Adempas risk evaluation and mitigation strategies (REMS) program for embryo-fetal toxicity
4. Prostacyclins
a. Parenteral prostacyclins (Table 8–9)
i. Mimic the actions of PGI2 (endogenous prostacyclin): Direct vasodilation of pulmonary and
systemic arterial vascular beds, inhibition of platelet aggregation, and antiproliferative effects
ii. Continuous-infusion epoprostenol is the most thoroughly studied medication approved for
treating PAH and has been shown to prolong survival.
• 100% survival with intravenous epoprostenol compared with 80% survival with
conventional therapies at 12 weeks (N Engl J Med 1996;334:296-301)
• 88% survival with intravenous epoprostenol compared with 80% survival in historical
controls at 1 year (Circulation 2002;106:1477-82)
• 55% survival with intravenous epoprostenol compared with 34% survival in historical
controls at 5 years (J Am Coll Cardiol 2002;40:780-8)
iii. Complications related to delivery include the need for a dedicated intravenous line, local
catheter infections/bloodstream infections, and catheter-related thrombosis.
b. Inhaled prostacyclins (Table 8–9): Advantages over intravenous route: Selective pulmonary
vasodilation with minimal systemic effects
c. Oral treprostinil (Table 9):
i. FDA approved as monotherapy for PAH, improved 6-minute walk distance
ii. Guidelines do not make a recommendation for or against the use of this agent due to children
being included in the primary randomized controlled trial.
iii. Insufficient evidence for combination therapy at this time.

Table 10. Additional Roles in Therapy for Prostacyclins


Failed
Add Clinical outcomes WHO FC IIIa WHO FC III-IV
monotherapy
1–2 classes of Improve WHO FC
IV epoprostenol Recommendedc
oral agents Improve 6MWD
1–2 classes of
IV treprostinil Improve 6MWD Recommendedc
oral agents
IV epoprostenol Nothing, increase dose Improve 6MWD Recommendedc Recommendedc
Inhaled treprostinil Improve 6MWD Recommendedb Recommendedb
ERA Improve WHO FC
Inhaled iloprost Recommendedc Recommendedc
Delay clinical worsening
Inhaled treprostinil Improve 6MWD Recommendeda Recommendeda
PDE5i Improve WHO FC
Inhaled iloprost Recommendedc Recommendedc
Delay clinical worsening
Signs of rapid progression of disease or poor prognosis.
a

b
Klinger JR, Elliott CG, Levine DJ, et al. Therapy for pulmonary arterial hypertension in adults. Chest 2019;155:565-86.
Consensus-based level of evidence (Chest 2019;155:565-86).
c

ACCP Updates in Therapeutics® 2022: Critical Care Pharmacy Preparatory Review and Recertification Course

212
Pulmonary Disorders II

Table 11. Prostacyclins


Epoprostenol Treprostinil Treprostinil Iloprost Treprostinil
Brand name Flolan Veletri Remodulin Tyvaso Ventavis Orenitram
Route of
IV SC and IV Inhalation Inhalation Oral
administration
0.25 mg every 12 hr
1.25 ng/kg/min
or 0.125 mg every
IV or SC; dose 2.5–5 mcg
18 mcg four 8 hr; caution with
Initial dose 1–4 ng/kg/min IV adjustments six to nine
times daily strong CYP2C8
in hepatic times daily
inhibitors and severe
impairment
hepatic dysfunction
Half-life 6 min 4 hr 25 min 4 hr
Protect from light
8 hr at 48 72
room 48 hr at hr at hr at
Stability — —
temp; 24 room room room
hr with temp temp temp
cold packs
Headache, diarrhea,
Flushing, headache, diarrhea nausea/ Cough, throat irritation, nausea, flushing, jaw
Adverse effects
vomiting, jaw pain, thrombocytopenia bronchospasm, hypotension pain, and abdominal
discomfort
Rebound PH with abrupt discontinuation; dosing weight should not be changed without con-
Cautions
sultation with PH specialist
a
Galiè N, Humbert M, Vachiery, J-L, et al. 2015 ESC/ERS guidelines for the diagnosis and treatment of pulmonary hypertension: The Joint Task Force for the Diagnosis
and Treatment of Pulmonary Hypertension of the European Respiratory Society (ERS): Endorsed by Association for European Paediatric and Congenital Cardiology
(AEPC), International Society for Heart and Lung Transplantation (ISHLT).Eur Heart J 2016;37:67-119.
CYP = cytochrome P450; PH = pulmonary hypertension; SC = subcutaneous.

5. Selective prostacyclin receptor (IP receptor) agonist


a. Selexipag (Uptravi) is a nonprostanoid that targets the prostacyclin pathway.
b. The 2019 guideline update changed their recommendation for this agent based on the included
studies not achieving their prespecified definition of clinically significant improvement (33 m in
the 6MWD). The current guidelines do not provide a recommendation for or against the use of this
agent.
c. Elimination half-life is 0.8–3 hours and, for the active metabolite, 6–14 hours. If unable to administer
for 3 days or more, retitration is required; however, a PH specialist should always be involved for
issues with access and reinitiation.
d. Approved dose is 200 mcg twice daily; increase at weekly intervals to the highest tolerated dose
(maximum dose 1600 mcg twice daily); adjustment necessary for moderate hepatic impairment
e. Contraindicated in patients receiving CYP2C8 inhibitors; dose increase necessary with CYP2C8
inducers
f. Adverse effects similar to prostacyclins

F. Managing decompensated PH
1. Decompensated PH generally presents with worsening symptoms of right heart failure such as shortness
of breath, syncope, fluid retention, or even multiorgan failure due to poor perfusion.

ACCP Updates in Therapeutics® 2022: Critical Care Pharmacy Preparatory Review and Recertification Course

213
Pulmonary Disorders II

2. Markers of imminent mortality from PH/severe right sided heart failure include a decline in central
venous oxygen saturation, lactic acidosis, and decreased urine output.
3. Control contributing factors such as infections, anemia, arrhythmias, rebound PH (nonadherence or
ineffective dosing), hypoxemia, acidosis, and metabolic abnormalities.
4. Supportive therapies include optimizing RV preload through fluid optimization (diuretics or dialysis),
maintaining aortic root pressure, improving RV contractility, and reducing RV afterload.
5. Temporary discontinuation of calcium channel blockers and beta blockers if present to eliminate negative
inotropic effects.
6. Hemodynamic support (Table 12)
a. Maintaining aortic root pressure and minimizing RV ischemia can be accomplished using
vasopressors, which increase the systemic vascular resistance and ultimately improve RV perfusion
(Crit Care Med 2007;35:2037-50).
b. Direct effects on the pulmonary circulation from vasopressors may increase the PVR, potentially
leading to further clinical decompensation.
c. Few studies have been published to help guide selection of the optimal vasopressor in patients with
PH; recommendations are extrapolations from other patient populations.
d. Inotropes are used to further augment the cardiac output of the RV and may improve PVR. Because
of systemic vasodilatory properties from inotropes, expect possible systemic hypotension and need
for vasopressors.
7. Unloading the RV with pulmonary vasodilators is essential to controlling decompensated PH and RV
failure.
8. If patients are not responsive to therapy and may be a transplant candidate, extracorporeal life support
can be considered as a bridge to lung transplant. Candidates must be chosen prudently as patient’s RV
failure must be reversible for this strategy to be successful.

Table 12. Agents Used for Hemodynamic Support for Decompensated PH


Site of Action
Drug Effects on PVR Effects on CO Comments
(receptor activity)
Dose-dependent
May not improve RV ejection
Dopamine dopaminergic, α1  or  
fraction; arrhythmias
and β1
Decreased mortality in subgroup of
cardiogenic shock and decreased
Norepinephrinea α1 > β1   or 
rate of arrhythmias compared with
dopamine in a randomized trial
Reflex bradycardia may be harmful
Phenylephrine α1  
in the setting of RV failure
Arrhythmias, hyperglycemia,
Epinephrine α1, β1 > β2  or  
increased lactate concentrations
Vasopressin V1 receptors  or   or  Use low dose (≤ 0.03 units/min)
Combine with peripheral
Dobutamine β1 > β2   vasoconstrictor to attenuate
systemic vasodilation
Combine with peripheral
Milrinone PDE-3 inhibitor   vasoconstrictor to attenuate
systemic vasodilation
De Backer D, Biston P, Devriendt J, et al. Comparison of dopamine and norepinephrine in the treatment of shock. N Engl J Med 2010;362:779-89.
a

CO = cardiac output; PVR = pulmonary vascular resistance; RV = right ventricular.

ACCP Updates in Therapeutics® 2022: Critical Care Pharmacy Preparatory Review and Recertification Course

214
Pulmonary Disorders II

Patient Case

2. A 44-year-old man is transferred to the medical ICU for treatment of his worsening PAH. He currently
receives no PAH treatment. The patient has had increased dyspnea on exertion for the past 6 months with
exercise, but for the past 3 days, he has had severe shortness of breath at rest. His physical examination is
remarkable for blood pressure 105/64 mm Hg and heart rate 85 beats/minute. Lung examination is clear, and
extremities are notable for trace edema. An echocardiogram reveals an elevated pulmonary systolic pressure
and a normal ejection fraction. The patient has an unfavorable response to vasodilator challenge. Pertinent
laboratory data are BUN 10 mg/dL, SCr 0.6 mg/dL, AST 160 U/L, and ALT 100 U/L. Which regimen would
be most appropriate for this patient?
A. Epoprostenol infusion at 2 ng/kg/minute.
B. Diltiazem 180 mg orally daily.
C. Macitentan 10 mg orally daily.
D. Sildenafil 10 mg intravenously three times daily.

III.  ASTHMA EXACERBATION

A. Pathophysiology and Classification of Exacerbations


1. Asthma is characterized by variable and recurring symptoms, airflow obstruction, bronchial
hyperresponsiveness, and underlying inflammation (NAEPP 2007).
2. Asthma exacerbations are characterized by decreases in expiratory airflow that can be quantified by
spirometry or peak expiratory flow.
3. Definitions and classifications of asthma exacerbations vary between experts and guidelines. Both terms
below are considered life threatening exacerbations.
a. Status asthmaticus is an acute severe asthma exacerbation that does not respond to initial intensive
therapy, with the potential for pulmonary compromise and death.
b. Near-fatal asthma is status asthmaticus that progresses to respiratory failure.

Table 13. Classification of Asthma Exacerbations Acute Care Facility (GINA 2021)
Initial PEF (or
Symptoms Clinical Course
FEV1)
• Talks in phrases
• Prefers sitting to laying
down • Usually cared for at home/primary care setting
• Not agitated • Relief from frequent inhaled SABA
Mild/ > 50% of predicted
• RR increased • Consider addition of ipratropium
Moderate or personal best
• Accessory muscles not • Recommended course of oral CS
used • O2 saturation goal 93%–95%
• Pulse 100–120
• O2 saturation 90%–95%

ACCP Updates in Therapeutics® 2022: Critical Care Pharmacy Preparatory Review and Recertification Course

215
Pulmonary Disorders II

Table 13. Classification of Asthma Exacerbations Acute Care Facility (GINA 2021) (continued)
Onset Duration Advantages Disadvantages
• Talks in words • Requires ED visit and likely hospitalization
• Sits hunched forward • Recommended inhaled SABA + ipratropium
• Agitated bromide
< 50% of predicted
Severe • RR > 30 breaths/min • O2 saturation goal 93%–95%
or personal best
• Accessory muscles used • Oral or IV CS
• Pulse > 120 • Consider IV magnesium
• O2 saturation < 90% • Consider high dose ICS
• Requires hospitalization; possible ICU
• Minimal or no relief from frequent inhaled
• Drowsy
Life SABA
• Confused No definition
threatening • IV corticosteroids
• Silent chest
• Adjunctive therapies are helpful
• (see text that follows)
CS = corticosteroid; ED = emergency department; FEV1 = forced expiratory volume in 1 s; ICS = inhaled corticosteroid; ICU = intensive care unit; PEF = peak
expiratory flow; SABA = short-acting β-agonist.
Adapted from: Global Initiative for Asthma (GINA). Global strategy for asthma management and prevention (2021 update). 2021. Available at https://ginasthma.org/
wp-content/uploads/2021/05/GINA-Main-Report-2021-V2-WMS.pdf.

B. Common Causes of Asthma Exacerbations


1. Viral infections
2. Allergen exposure
3. Food allergy
4. Air pollution
5. Seasonal weather changes
6. Poor adherence to inhaled corticosteroids

C. Mortality Risk Factors


1. History of near-fatal asthma (e.g., requiring mechanical ventilation)
2. Hospitalization or emergency department visit for asthma in the past year
3. Active use of oral corticosteroids or completion of recent course for asthma
4. Not currently using inhaled corticosteroids
5. Use of more than one canister of short-acting β-agonists (SABAs) per month
6. Poor adherence to inhaled corticosteroid containing asthma medications and/or poor adherence to
asthma action plan
7. Social history that includes major psychosocial problems or psychiatric illness
8. Food allergies
9. Comorbidities including pneumonia, diabetes, and arrhythmias

D. COVID-19 (SARS-CoV-2) and Asthma (GINA 2021)


1. Patients with well-controlled asthma do not appear to be at increased risk of infection or mortality due
to SARS-CoV-2.
2. However, patients who recently required oral corticosteroids for their asthma were at increased risk of
death from COVID-19.
3. Bloom, et al. identified that patients on inhaled corticosteroids for their asthma had lower mortality from
COVID-19 compared with patients without underlying lung disease (Lancet Respir Med 2021;9:699-
711).

ACCP Updates in Therapeutics® 2022: Critical Care Pharmacy Preparatory Review and Recertification Course

216
Pulmonary Disorders II

4. Both GINA and the Centers for Disease Control and Prevention recommend that patients receive
vaccination against COVID-19.
5. If patients are receiving biologics for their severe asthma, it is not recommended to give the biologic and
COVID-19 vaccine on the same day.

E. Alternative Causes (mimic severe asthma exacerbation)


1. Upper airway: Vocal cord dysfunction, anaphylaxis, laryngeal stenosis
2. Central airway: Tracheomalacia, tracheal stenosis mucus plugging
3. Lower airway: Bronchiolitis, COPD, valvular heart disease, diastolic heart dysfunction

F. Arterial Blood Gas Assessment


1. Patients with acute severe asthma initially experience a respiratory alkalosis.
2. As respiratory status worsens, arterial carbon dioxide increases (patient exhaustion, inadequate alveolar
ventilation and/or an increase in physiologic dead space), leading to respiratory acidosis.
3. Metabolic (lactic) acidosis may coexist. Lactate production presumably stems from the use of high-dose
β-agonists, increased work of breathing resulting in anaerobic metabolism of the ventilatory muscles,
and tissue hypoxia.

G. Oxygen
1. Oxygen therapy is important in managing acute severe asthma.
2. Oxygen by nasal cannula or mask should be administered to patients with severe exacerbations with
hypoxemia to achieve Sao2 values greater than 93%–95% (pregnant women and patients with a cardiac
history may require higher goals) (GINA 2021; Thorax 2011;66:937-41).

H. Noninvasive Ventilation: Data are insufficient for severe exacerbations. If attempted, patients should not be
sedated to tolerate noninvasive ventilation.

I. Mechanical Ventilation
1. Indications
a. Worsening hypoxemia or hypercarbia
b. Drowsiness or altered mental status
c. Hemodynamic instability
d. Increased work of breathing
2. Low minute ventilation (by reduced tidal volume and/or respiratory rate), high inspiratory flow rate,
and minimal PEEP on the ventilator will help minimize dynamic hyperinflation.

J. β-Agonists
1. SABAs stimulate the β2-receptors on smooth muscle cells, leading to relaxation of respiratory smooth
muscle and causing bronchodilation and decreased airway obstruction.
2. SABAs are the cornerstone in managing acute severe asthma.
3. Patients with an asthma exacerbation should receive a SABA repeatedly with either an MDI or
nebulization at presentation, and the SABA should be continued until acute symptoms have resolved
(NAEPP 2007).
4. Patients with acute severe exacerbations may benefit from continuous versus intermittent nebulization
of SABAs (Cochrane Database Syst Rev 2003;4:CD001115).
a. Intermittent dosing of albuterol: 2.5–5 mg every 20 minutes for three doses; then 2.5–10 mg every
1–4 hours as needed
b. Continuous nebulization of albuterol: 10–15 mg/hour

ACCP Updates in Therapeutics® 2022: Critical Care Pharmacy Preparatory Review and Recertification Course

217
Pulmonary Disorders II

5. Systemic β-agonists (intravenous or subcutaneous epinephrine or terbutaline) can be considered if the


patient does not respond to inhaled therapy after several hours; however, they have no proven advantage
over inhaled agents and are not recommended for routine use.
6. Adverse effects include tremor, tachyarrhythmias, hypokalemia, tachyphylaxis, hyperglycemia, and
type B lactic acidosis.

K. Anticholinergic Agents
1. Inhaled anticholinergic agents selectively bind to the muscarinic receptors on smooth muscle cells in
the airways and thereby reduce bronchoconstriction.
2. Short-acting inhaled anticholinergic agents (ipratropium bromide) should be given in combination with
a SABA to promote additional bronchodilation through a different pathway.
3. Adding ipratropium to inhaled albuterol compared with using albuterol alone in patients with severe
asthma improved the response; however, outcomes with this combination in status asthmaticus or
near-fatal asthma remain elusive (Am J Respir Crit Care Med 2000;161:1862-8). Combination therapy
consisting of inhaled short-acting anticholinergics and a SABA compared with a SABA alone has
reduced hospitalization rates in severe asthma exacerbations. However, combination therapy may
increase the risk of adverse drug events, including agitation and palpitations (Cochrane Database Syst
Rev 2017;1:CD001284).
4. Adverse effects include headache, flushed skin, blurred vision, tachycardia, palpitations, and urinary
retention.

L. Corticosteroid Therapy
1. Corticosteroids decrease airway obstruction during an asthma exacerbation by decreasing inflammation,
increasing the number of β2-receptors and increasing their responsiveness to β-agonists, reducing
airway edema, and suppressing certain proinflammatory cytokines (Respir Med 2004;98:275-84).
2. Systemic corticosteroids should be administered to patients who have moderate or severe exacerbations or
to patients who do not respond promptly and completely to SABA treatment (Am J Med 1983;74:845-51).
3. Typically, there is a 6–8-hour delay in the response to corticosteroids in status asthmaticus or life
threatening asthma; therefore, administration should be considered early in the course (within 1 hour
of presentation) (Cochrane Database Syst Rev 2001;1:CD002178).
4. Oral prednisone is considered as effective as parenteral corticosteroids in most patients; however, it
may not be as effective in critically ill patients with impaired gastric absorption.
5. GINA guidelines recommend prednisone 1 mg/kg/day up to a maximum of prednisone 50 mg daily for
5–7 days for treatment of exacerbations.
a. Other guidelines recommend longer courses and higher doses for life threatening asthma
exacerbations based on the same evidence used by GINA.
b. For corticosteroid courses less than 1 week, tapering is not necessary.
c. For longer courses (up to 10 days), there is probably no need to taper, especially if patients are
concurrently using an inhaled corticosteroid.
6. Inhaled corticosteroids can be initiated any time in the treatment of an asthma exacerbation; it is not
recommended to replace systemic steroids with inhaled corticosteroids (Respir Care 2018;63:783-96).

M. Adjunctive Therapies (Respir Care 2018;63:783-96)


1. Ketamine is a phencyclidine derivative that has bronchodilatory properties through reducing airway
resistance and preventing the reuptake of norepinephrine, which may stimulate β2-receptors (Indian J
Crit Care Med 2013).
2. Helium-oxygen (heliox) is a blended gas (mixture of about 70%–80% helium and 20%–30% oxygen)
that decreases airway resistance, which leads to improved airflow and ventilation. Use of heliox in
status asthmaticus may delay the need for intubation by allowing other therapies to work.

ACCP Updates in Therapeutics® 2022: Critical Care Pharmacy Preparatory Review and Recertification Course

218
Pulmonary Disorders II

3. Magnesium sulfate (2 g intravenously administered over 20–30 minutes) can be considered in patients
who have life-threatening exacerbations and are unresponsive to conventional therapies after 1 hour.
a. Magnesium is thought to cause bronchodilation by inhibiting calcium channels on smooth muscle,
leading to relaxation.
b. In addition, magnesium may have anti-inflammatory properties that interfere with the activation
and release of neutrophils in patients with asthma.

N. Treatments Not Recommended


1. Methylxanthines (theophylline and aminophylline) do not improve lung function or other outcomes in
hospitalized adults. Use is associated with severe adverse effects.
2. Antimicrobials are not generally recommended for acute asthma exacerbations; however, they can be
considered if there is evidence of concurrent infection.
3. Mucolytics may worsen cough or airflow obstruction.

Patient Cases

Questions 3 and 4 pertain to the following case.


A 30-year-old woman (weight 115 kg) with status asthmaticus is admitted to the ICU. She has a history of
severe refractory asthma that has required endotracheal intubation three times in the past 6 months. Her medi-
cal history includes hypertension, diabetes, obesity, and bipolar disorder. She has used at least three canisters of
albuterol per month for the past 2 months to manage her symptoms.

3. Which best represents the patient’s risk factors for higher mortality?
E. Three hospitalizations in the past 6 months and bipolar disorder.
F. Use of more than two canisters of SABAs in the past month and obesity.
G. Hospitalization for asthma in the past month and diabetes.
H. Prior episode of near-fatal asthma and hypertension.

4. The patient is endotracheally intubated and placed on MV. Which would be the most appropriate initial
therapy for this patient with life-threatening asthma exacerbation?
I. Inhaled albuterol by nebulization 2.5 mg every 4 hours.
J. Inhaled albuterol by nebulization 2.5 mg every 4 hours and inhaled ipratropium by nebulization 0.5 mg
every 6 hours.
K. Inhaled albuterol by nebulization 2.5 mg every 4 hours, inhaled ipratropium by nebulization 0.5 mg
every 6 hours, and methylprednisolone 40 mg intravenously daily.
L. Inhaled albuterol by nebulization 2.5 mg every 4 hours, inhaled ipratropium by nebulization 0.5 mg
every 6 hours, and methylprednisolone 125 mg intravenously every 6 hours.

ACCP Updates in Therapeutics® 2022: Critical Care Pharmacy Preparatory Review and Recertification Course

219
Pulmonary Disorders II

IV.  ACUTE CHRONIC OBSTRUCTIVE PULMONARY DISEASE EXACERBATION

A. Chronic Obstructive Pulmonary Disease (COPD)


1. Characterized by chronic airflow limitation, resulting from chronic inflammation caused by a
combination of small airway disease (obstructive bronchitis) and parenchymal destruction (emphysema).
Airflow limitation is usually measured by spirometry (Table 14).
2. A COPD exacerbation can be defined as an acute worsening in the patient’s baseline status (increase in
dyspnea, cough, and/or sputum production), necessitating a change in medications.
3. General criteria for diagnosing an acute exacerbation depend on clinical presentation, including an
acute change in symptoms that is beyond normal day-to-day variation.
a. Arterial blood gas should be measured to assess the degree of hypercarbia. Venous blood gas to
assess pH and bicarbonate is an acceptable alternative.
b. Pulse oximetry can be used to determine the need for supplemental oxygen.
c. Spirometry is not accurate during an exacerbation and is not recommended.

Table 14. Classification of COPD Severity


Spirometric GOLD
Grade Characteristics
Classification
GOLD 1:
FEV1 ≥ 80% predicted
Mild 0 or 1 exacerbation per year AND no hospitalizations for
GOLD 2: exacerbation
50% ≤ FEV1 < 80% predicted
Moderate
GOLD 3:
30% ≤ FEV1 < 50% predicted ≥ 2 exacerbations per year OR 1 hospitalization for
Severe
exacerbation (patients in GOLD classifications 3 and 4 are
GOLD 4: at increased risk of hospital admission and death)
FEV1 < 30% predicted
Very severe
COPD = chronic obstructive pulmonary disease; FEV1 = forced expiratory volume.
Adapted from: Global Initiative for Chronic Obstructive Lung Disease (GOLD). Global strategy for the diagnosis, management and prevention of COPD. 2021. Available
at https://goldcopd.org/2021-gold-reports/.

B. Common Causes of Acute COPD Exacerbation


1. Respiratory tract infection (40%–50% of COPD exacerbations)
a. Bacterial
i. Streptococcus pneumoniae, Haemophilus influenzae, and Moraxella catarrhalis are the most
common organisms.
ii. Pseudomonas aeruginosa is common in patients with Global Initiative for Chronic Obstructive
Lung Disease (GOLD) grades 3 and 4.
b. Viral SARS-CoV-2, (influenza, rhinovirus, parainfluenza, respiratory syncytial virus)
2. Medication nonadherence
3. Temperature change
4. Air pollution

C. Oxygen
1. Oxygen therapy is important in managing COPD exacerbations.
2. Oxygen by nasal cannula or mask should be administered to patients with severe exacerbations to
achieve an Sao2 of 88%–92% by pulse oximetry (BMJ 2010;341:c5462).

ACCP Updates in Therapeutics® 2022: Critical Care Pharmacy Preparatory Review and Recertification Course

220
Pulmonary Disorders II

3. Caution is advised with oxygen supplementation in patients with COPD:


a. A slight level of hypoxemia may serve as a trigger for their respiratory drive secondary to chronic
hypercapnia.
b. May increase ventilation/perfusion mismatch and decrease respiratory rate centrally and perpetuate
the Haldane effect

D. High-Flow Oxygen Therapy by Nasal Cannula


1. Alternative to standard oxygen therapy or noninvasive positive pressure ventilation (GOLD 2021)
2. Improves oxygenation and ventilation in patients with COPD and very severe underlying disease
(Thorax 2016;71:759-61)

E. Noninvasive MV
1. Indications for noninvasive MV include at least one of the following (GOLD 2021):
a. Respiratory acidosis (Paco2 45 mm Hg or greater and/or arterial pH 7.35 or less)
b. Severe dyspnea with clinical signs suggestive of respiratory muscle fatigue, increased work of
breathing, or both
c. Persistent hypoxemia despite supplemental oxygen
2. Improves gas exchange, reduces work of breathing and need for mechanical intubation, decreases
hospitalization duration, and improves survival (N Engl J Med 1995;333:817-22)

F. Mechanical Ventilation: Indications for invasive MV (GOLD 2021)


1. Unable to tolerate or failure of noninvasive ventilation
2. Respiratory or cardiac arrest
3. Altered level of consciousness
4. Aspiration or vomiting/inability to clear secretions
5. Hemodynamic instability despite fluid and vasopressors
6. Severe ventricular arrhythmias
7. Life-threatening hypoxemia and inability to tolerate noninvasive ventilation

G. Bronchodilators
1. Inhaled SABAs (nebulized or MDI) with or without a short-acting anticholinergic are preferred for
bronchodilation in COPD exacerbations.
2. GOLD guidelines recommend to continue long-acting β-agonists and anticholinergic agents though no
studies have evaluated this regimen. If patients have not yet started these agents, recommend initiating
long-acting agents as soon as patient is stable and prior to discharge.
3. Currently, evidence is also lacking regarding a mode of delivery when comparing nebulizers with MDIs
during COPD exacerbations (Cochrane Database Syst Rev 2016;8:CD011826).

H. Corticosteroid Therapy
1. The guidelines recommend prednisone 40 mg daily or the equivalent (preferably oral) for 5 days. If
oral administration is not an option, equivalent doses of intravenous, methylprednisolone or nebulized
budesonide can be administered (GOLD 2021).
2. Administration of systemic corticosteroids in non-critically ill patients reduces the risk of early relapse
rate, treatment failure, and hospital length of stay and improves FEV1 (N Engl J Med 1999;340:1941-7;
Chest 2001;119:726-30).

ACCP Updates in Therapeutics® 2022: Critical Care Pharmacy Preparatory Review and Recertification Course

221
Pulmonary Disorders II

3. Reduction in the Use of Corticosteroids in Exacerbated COPD (REDUCE) was a randomized,


noninferiority trial. Patients were randomized to prednisone 40 mg daily for either 5 days or 14
days (inhaled bronchodilators and antimicrobials in both groups). COPD exacerbations occurred in
35.9% of patients in the 5-day group and in 36.8% of patients in the 14-day group (p=0.006) (JAMA
2013;309:2223-31). This study did not include critically ill patients requiring MV.
4. Studies including ICU patients
a. A randomized, double-blind trial including 83 adult patients with a COPD exacerbation requiring
hospitalization and ventilatory support (invasive or noninvasive) compared methylprednisolone 0.5
mg/kg intravenously every 6 hours for 72 hours, 0.5 mg/kg every 12 hours on days 4–6, and 0.5 mg/
kg daily on days 7–10 with placebo. Patients receiving corticosteroids had a shorter duration of MV
(p=0.04), shorter length of ICU stay (p=0.09), fewer noninvasive ventilation failures (p=0.004),
and more hyperglycemia (p=0.04). This study was not powered to detect differences in length of
stay or mortality (Arch Intern Med 2011;171:1939-46).
b. An open-label, randomized trial included 217 critically ill patients 40 years and older with a COPD
exacerbation requiring MV to receive oral prednisone 1 mg/kg daily for a maximum of 10 days
or usual care. No differences in ICU mortality, noninvasive ventilation failure, duration of MV, or
ICU length of stay were observed. Hyperglycemia significantly increased in the steroid group (Eur
Respir J 2014;43:717-24).
c. A cohort study compared high-dose (greater than 240 mg/day) with low-dose (240 mg/day or less)
methylprednisolone in more than 17,000 patients with a COPD exacerbation admitted to an ICU.
Patients in the high-dose group had longer ICU and hospital lengths of stay, higher hospital costs,
longer durations of MV, and more hyperglycemia and fungal infections. Mortality did not differ
between the two groups (Am J Respir Crit Care Med 2014;189:1052-64).
5. Antimicrobials (GOLD 2021; Cochrane Database Syst Rev 2018;10:CD010257)
a. The indication for antimicrobials in COPD exacerbations has become more elusive, and reports
are conflicting on which populations benefit from antimicrobials. Currently, the GOLD guidelines
support the populations in the text that follows to receive antibiotics.
i. Patients with all three cardinal symptoms: increase in dyspnea, increased sputum, and sputum
purulence
ii. Patients with two cardinal symptoms if purulent sputum is one of the two symptoms
iii. Patients requiring invasive or noninvasive MV for their respiratory insufficiency or failure as
the result of a COPD exacerbation
b. A 2018 systematic review comparing antibiotics with placebo for COPD exacerbation found a
treatment failure relative risk 0.72 (n=1191) among studies using only antibiotics currently in
practice. No differences in all-cause mortality or treatment failure were identified (Cochrane
Database Syst Rev 2018;10:CD010257).
i. Only one trial included ICU patients.
(a) This study showed that administration of antibiotics reduced treatment failure, all-cause
mortality, and hospital length of stay.
(b) Study limitations: Performed in 2001; generalizability to current practice may be limited;
study used ofloxacin, which is no longer used
c. Data remain inconclusive regarding treatment failure, mortality, and repeated exacerbations for
patients who are hospitalized outside the ICU with severe COPD and for outpatients.
d. The findings within this systematic review are further limited by the inconsistency among studies
in the description of COPD severity at baseline.

ACCP Updates in Therapeutics® 2022: Critical Care Pharmacy Preparatory Review and Recertification Course

222
Pulmonary Disorders II

e. Optimal antimicrobial therapy is not established; however, it should be based on local resistance
patterns (GOLD 2021).
i. Macrolide-based antimicrobial regimens have improved clinical outcomes.
• In the BACE randomized trial, azithromycin compared with placebo significantly reduced
the percentage of patients requiring treatment intensification within 3 months (47% vs.
60%, respectively; p=0.0272) and step-up in-hospital care or readmissions attributed to
pulmonary causes (13% vs. 28%, respectively; p=0.0024) (Am J Respir Crit Care Med
2019;200:857-68).
• Another propensity score-matched cohort study found that macrolide treatment compared
with non-macrolide treatment was associated with reduced 30-day readmission rates
(7.3% vs. 8.8%, respectively; p<0.01) (Pharmacotherapy 2019;39:242-52).
ii. Patients with frequent exacerbations or exacerbations requiring MV may be at risk of more
resistant organisms.
f. Procalcitonin may help decrease total antimicrobial exposure without affecting clinical outcomes;
however, patients with an acute COPD exacerbation have largely been excluded from these trials.
g. Duration of antimicrobials should be 5–7 days (GOLD 2021).
6. Vitamin D (GOLD 2021)
a. The GOLD guidelines recommend routine identification and supplementation of vitamin D
deficiency in patients hospitalized with COPD exacerbations with vitamin D concentrations less
than 25 nmol/L.
b. Mechanism of effect: Vitamin D metabolites attenuate inflammation and support immune system
responses to pathogens.
c. A systematic review and meta-analysis of three randomized controlled trials (469 patients with
data available) showed that supplementation of vitamin D did not affect the rate of moderate/severe
COPD exacerbations (Thorax 2019;74:337-45).
i. Prespecified subgroup analysis of 87 patients identified that vitamin D supplementation
reduced the rate of moderate to severe COPD exacerbations in patients whose concentrations
were deficient (25-hydroxyvitamin D concentrations less than 25 nmol/L) at baseline.
ii. No effect on exacerbations for patients whose concentrations were not deficient at baseline

I. Treatments Not Recommended: Methylxanthines (theophylline and aminophylline) because of significant


adverse effects

ACCP Updates in Therapeutics® 2022: Critical Care Pharmacy Preparatory Review and Recertification Course

223
Pulmonary Disorders II

Patient Case

5. A 79-year-old woman (weight 70 kg) is admitted to the ICU for the management of hypercapnic respiratory
failure related to a COPD exacerbation. She has had several admissions for COPD exacerbation. The patient
presents with fever, profound dyspnea, increased sputum production (thick and purulent), and confusion. She
has a history of anaphylaxis to penicillin. Her blood pressure is 190/100 mm Hg, heart rate is 110 beats/min-
ute, and respiratory rate is 22 breaths/minute. Her chest is hyperinflated and has poor air entry bilaterally.
Her arterial blood gas values are as follows: pH 7.20, Pco2 85 mm Hg, and Pao2 44 mm Hg on 6 L of nasal
cannula. She is intubated and placed on MV. Which would be the most appropriate group of medications to
treat this patient’s severe COPD exacerbation?
M. Methylprednisolone 1 mg/kg intravenously administered as two divided doses, inhaled albuterol by
nebulization, ampicillin/sulbactam 3 g intravenously every 6 hours for 7 days, and azithromycin 500 mg
intravenously daily for 5 days.
N. Prednisone 40 mg by nasogastric tube (NGT) daily for 5 days, inhaled albuterol and ipratropium by
nebulization, ampicillin/sulbactam 3 g intravenously every 6 hours for 10 days, and azithromycin 500
mg daily intravenously for 5 days.
O. Methylprednisolone 1 mg/kg intravenously administered as two divided doses, inhaled albuterol and
ipratropium by nebulization, and levofloxacin 750 mg by NGT daily for 7 days.
P. Prednisone 40 mg by NGT daily for 5 days, inhaled albuterol and ipratropium by nebulization, and
levofloxacin 750 mg intravenously daily for 5 days.

ACCP Updates in Therapeutics® 2022: Critical Care Pharmacy Preparatory Review and Recertification Course

224
Pulmonary Disorders II

REFERENCES

Cystic Fibrosis 12. Turcios NL. Cystic fibrosis lung disease: an over-
1. Akkerman-Nijland AM, Akkerman OW, view. Respir Care 2020;65:233-51.
Grasmeijer F, et al. The pharmacokinetics of anti- 13. West NE, Beckett VV, Jain R, et al. Standardized
biotics in cystic fibrosis. Expert Opin Drug Metab Treatment of Pulmonary Exacerbations (STOP)
Toxicol 2021;17:53-68. study: physician treatment practices and outcomes
2. Dentice RL, Elkins MR, Middleton PG, et al. A for individuals with cystic fibrosis with pulmonary
randomised trial of hypertonic saline during hospi- exacerbations. J Cyst Fibros 2017;16:600-6.
talisation for exacerbation of cystic fibrosis. Thorax 14. Yankaskas JR, Marshall BC, Sufian B, et al. Cystic
2016;71:141-7. fibrosis adult care consensus conference report.
3. Flume PA, Amelina E, Daines CL, et al. Efficacy Chest 2004;125:1-39.
and safety of inhaled dry-powder mannitol in adults
with cystic fibrosis: An international, randomized Pulmonary Hypertension
controlled study. J Cyst Fibros 2021;20:1003-9. 1. Barst RJ, Rubin LJ, Long WA, et al. A com-
4. Flume PA, Mogayzel PJ, Robinson KA, et al. parison of continuous intravenous epoprostenol
Cystic fibrosis pulmonary guidelines: treatment of (prostacyclin) with conventional therapy for pri-
pulmonary exacerbations. Am J Respir Crit Care mary pulmonary hypertension. N Engl J Med
Med 2009;180:802-8. 1996;334:296-301.
5. Flume PA, Robinson KA, O’Sullivan BP, et al. 2. De Backer D, Biston P, Devriendt J, et al.
Cystic fibrosis pulmonary guidelines: airway clear- Comparison of dopamine and norepinephrine in the
ance therapies. Respir Care 2009;54:522-37. treatment of shock. N Engl J Med 2010;362:779-89.
6. Gentzsch M, Mall M. Ion channel modulators in 3. Galiè N, Barberà JA, Frost AE, et al. Initial use
cystic fibrosis. Chest 2018;154:383-93. of ambrisentan plus tadalafil in pulmonary arterial
7. Heltshe SL, West NE, VanDevanter DR, et al. hypertension. N Engl J Med 2015;373:834-44.
Study design considerations for the Standardized 4. Galiè N, Corris PA, Frost A, et al. Updated treat-
Treatment of Pulmonary Exacerbations 2 (Stop2): ment algorithm of pulmonary arterial hypertension.
s trial to compare intravenous antibiotic treat- J Am Coll Cardiol 2013;62(25 suppl):D60-D72.
ment durations in CF. Contemp Clin Trials 5. Galiè N, Humbert M, Vachiery JL, et al. 2015
2018;64:35-40. ESC/ERS guidelines for the diagnosis and treat-
8. Mogayzel PJ Jr, Naureckas ET, Robinson KA, ment of pulmonary hypertension: the Joint
et al.; Pulmonary Clinical Practice Guidelines Task Force for the Diagnosis and Treatment of
Committee. Cystic fibrosis pulmonary guide- Pulmonary Hypertension of the European Society
lines: chronic medications for maintenance of lung of Cardiology (ESC) and the European Respiratory
health. Am J Respir Crit Care Med 2013;187:680-9. Society (ERS): Endorsed by: Association for
9. Sanders DB, Bittner RCL, Rosenfeld M, et al. European Paediatric and Congenital Cardiology
Failure to recover to baseline pulmonary function (AEPC), International Society for Heart and
after cystic fibrosis pulmonary exacerbation. Am J Lung Transplantation (ISHLT). Eur Heart J
Respir Crit Care Med 2010;182:627-32. 2016;37:67-119.
10. Smith S, Rowbotham NJ, Charbek E. Inhaled antibi- 6. Ghofrani HA, D’Armini AM, Grimminger F, et
otics for pulmonary exacerbations in cystic fibrosis. al. Riociguat for the treatment of chronic throm-
Cochrane Database Syst Rev 2018;10:CD008319. boembolic pulmonary hypertension. N Engl J Med
11. Smyth A, Tan K, Hyman-Taylor P, et al. Once 2013;369:319-29.
versus three-times daily regimens of tobramycin 7. Hoeper MM, Benza RL, Corris P, et al. Intensive
treatment for pulmonary exacerbations of cystic care, right ventricular support and lung transplan-
fibrosis—the TOPIC study: a randomized con- tation in patients with pulmonary hypertension.
trolled trial. Lancet 2005;365:573-8. Eur Respir J 2019;53:1801906.

ACCP Updates in Therapeutics® 2022: Critical Care Pharmacy Preparatory Review and Recertification Course

225
Pulmonary Disorders II

8. Klinger JR, Elliott CG, Levine DJ, et al. Therapy Asthma Exacerbation
for pulmonary arterial hypertension in adults: 1. Bloom CI, Drake TM, Docherty AB, et al. Risk
update of the CHEST guidelines and expert panel of adverse outcomes in patients with underlying
report. Chest 2019;155:565-86. respiratory conditions admitted to hospital with
9. McLaughlin VV, Shillington A, Rich S. COVID-19: a national, multicentre prospective
Survival in primary pulmonary hypertension: cohort study using the ISARIC WHO Clinical
the impact of epoprostenol therapy. Circulation Characterisation Protocol UK. Lancet Respir Med
2002;106:1477-82. 2021;9:699-711.
10. Pulido T, Adzerikho I, Channick RN, et al. 1. Camargo CA Jr, Spooner CH, Rowe BH.
Macitentan and morbidity and mortality in pul- Continuous versus intermittent beta-agonists in the
monary arterial hypertension. N Engl J Med treatment of acute asthma. Cochrane Database Syst
2013;369:809-18. Rev 2003;4:CD001115.
11. Rich S, Brundage BH. High-dose calcium chan- 2. Fanta CH, Rossing TH, McFadden ER Jr.
nel-blocking therapy for primary pulmonary Glucocorticoids in acute asthma: a critical con-
hypertension: evidence for long-term reduction trolled trial. Am J Med 1983;74:845-51.
in pulmonary arterial pressure and regression 3. Giyal S, Agrawal A. Ketamine in status asth-
of right ventricular hypertrophy. Circulation maticus: a review. Indian J Crit Care Med
1987;76:135-41. 2013;17:154-61.
12. Rich S, Kaufmann E, Levy PS. The effect of high 4. Global Strategy for Asthma Management
doses of calcium-channel blockers on survival in and Prevention, Global Initiative for Asthma
primary pulmonary hypertension. N Engl J Med (GINA). 2021. Available at https://ginasthma.org/
1992;327:76-81. wp-content/uploads/2021/05/GINA-Main-Report-
13. Simonneau G, Montani D, Celermajer DS, et al. 2021-V2-WMS.pdf.
Haemodynamic definitions and updated clini- 5. Kirkland SW, Vandenberghe C, Voaklander B,
cal classification of pulmonary hypertension. Eur et al. Combined inhaled beta-agonist and anti-
Respir J. 2019;53:1801913. cholinergic agents for emergency management in
14. Sitbon O, Channick R, Chin KM, et al. Selexipag adults with asthma. Cochrane Database Syst Rev
for the treatment of pulmonary arterial hyperten- 2017;1:CD001284.
sion. N Engl J Med 2015;373:2522-33. 6. Maselli DJ, Peters JI. Medication regimens for man-
15. Sitbon O, Humbert M, Jaïs X, et al. Long-term aging acute asthma. Respir Care 2018;63:783-96.
response to calcium channel blockers in idiopathic 7. National Institutes of Health National Heart, Lung,
pulmonary arterial hypertension. Circulation and Blood Institute (NHLBI). National Asthma
2005;111:3105-11. Education and Prevention Program Guidelines
16. Sitbon O, Humbert M, Nunes H, et al. Long-term (NAEPP). 2007. NAEPP Expert Panel Report
intravenous epoprostenol infusion in primary 3. Available at https://www.nhlbi.nih.gov/sites/
pulmonary hypertension: prognostic factors and default/files/media/docs/EPR-3_Asthma_Full_
survival. J Am Coll Cardiol 2002;40:780-8. Report_2007.pdf. Accessed September 20, 2020.
17. Vachiery JL, Huez S, Gillies H, et al. Safety, tolera- 8. Perrin K, Wijesinghe M, Healy B, et al. Randomised
bility and pharmacokinetics of an intravenous bolus controlled trial of high concentration versus titrated
of sildenafil in patients with pulmonary arterial oxygen therapy in severe exacerbations of asthma.
hypertension. Br J Clin Pharmacol 2011;71:289-92. Thorax 2011;66:937-41.
18. Zamanian RT, Haddad F, Doyle RL, et al. 9. Proceedings of the ATS workshop on
Management strategies for patients with pulmo- refractory asthma: current understanding, recom-
nary hypertension in the intensive care unit. Crit mendations, and unanswered questions. American
Care Med 2007;35:2037-50. Thoracic Society. Am J Respir Crit Care Med
2000;162:2341-51.
10. Rodrigo GJ, Rodrigo C. First-line therapy for adult
patients with acute asthma receiving a multiple-
dose protocol of ipratropium bromide plus albuterol

ACCP Updates in Therapeutics® 2022: Critical Care Pharmacy Preparatory Review and Recertification Course

226
Pulmonary Disorders II

in the emergency department. Am J Respir Crit obstructive pulmonary disease. Am J Respir Crit
Care Med 2000;161:1862-8. Care Med 2014;189:1052-64.
11. Rowe BH, Edmonds ML, Spooner CH, et al. 9. Kiser TH, Reynolds PM, Moss M, et al. Impact
Corticosteroid therapy for acute asthma. Respir of macrolide antibiotics on hospital readmis-
Med 2004;98:275-84. sions and other clinically important outcomes in
12. Rowe BH, Spooner C, Ducharme FM, et al. Early critically ill patients with acute exacerbations of
emergency department treatment of acute asthma chronic obstructive pulmonary disease: a propen-
with systemic corticosteroids. Cochrane Database sity score-matched cohort study. Pharmacotherapy
Syst Rev 2001;1:CD002178. 2019;39:242-52.
10. Leuppi JD, Schuetz P, Bingisser R, et al. Short-
term vs conventional glucocorticoid therapy in
Acute Chronic Obstructive Pulmonary Disease
acute exacerbations of chronic obstructive pulmo-
Exacerbation
nary disease: the REDUCE randomized clinical
1. Abroug F, Ouanes-Besbes L, Fkih-Hassen M, et
trial. JAMA 2013;309:2223-31.
al. Prednisone in COPD exacerbation requiring
11. Niewoehner DE, Erbland ML, Deupree RH, et al.
ventilatory support: an open-label randomized
Effect of systemic glucocorticoids on exacerba-
evaluation. Eur Respir J 2014;43:717-24.
tions of chronic obstructive pulmonary disease.
2. Alia I, de la Cal MA, Esteban A, et al. Efficacy
Department of Veterans Affairs Cooperative Study
of corticosteroid therapy in patients with an acute
Group. N Engl J Med 1999;340:1941-7.
exacerbation of chronic obstructive pulmonary
12. Ram FS, Rodriguez-Roisin R, Granados-Navarrete
disease receiving ventilatory support. Arch Intern
A, et al. Antibiotics for exacerbations of chronic
Med 2011;171:1939-46.
obstructive pulmonary disease. Cochrane Database
3. Austin MA, Wills KE, Blizzard L, et al. Effect
Syst Rev 2006;2:CD004403.
of high flow oxygen on mortality in chronic
13. Sayiner A, Aytemur ZA, Cirit M, et al. Systemic
obstructive pulmonary disease patients in prehos-
glucocorticoids in severe exacerbations of COPD.
pital setting: randomized controlled trial. BMJ
Chest 2001;119:726-30.
2010;341:c5462.
14. Sivapalan P, Ingebrigtsen TS, Rasmussen DB, et
4. Brochard L, Mancebo J, Wysocki M, et al.
al. COPD exacerbations: the impact of long versus
Noninvasive ventilation for acute exacerbations of
short courses of oral corticosteroids on mortal-
chronic obstructive pulmonary disease. N Engl J
ity and pneumonia: nationwide data on 67 000
Med 1995;333:817-22.
patients with COPD followed for 12 months. BMJ
5. Fraser JF, Spooner AJ, Dunster KR, et al. Nasal
Open Respir Res 2019;6:e000407.
high flow oxygen therapy in patients with COPD
15. Van Geffen WH, Douma WR, Slebos DJ, et
reduces respiratory rate and tissue carbon diox-
al. Bronchodilators delivered by nebulizer
ide while increasing tidal and end-expiratory lung
versus pMDI with spacer or DPS for exacerba-
volumes: a randomised crossover trial. Thorax
tions of COPD. Cochrane Database Syst Rev
2016;71:759-61.
2016;8:CD011826.
6. Global Initiative for Chronic Obstructive
16. Vermeersch K, Gabrovska M, Aumann J, et al.
Lung Disease (GOLD). Global strategy for
Azithromycin during acute COPD exacerbations
the diagnosis, management and prevention of
requiring hospitalizations (BACE): a multicen-
COPD. 2021. Available at https://goldcopd.
tre randomized, double-blind, placebo-controlled
org/2021-gold-reports/.
trial. Am J Respir Crit Care Med 2019;200:857-68.
7. Jolliffe DA, Greenberg L, Hooper RL, et al.
17. Vollenweider DJ, Frei A, Steurer-Stey A, et al.
Vitamin D to prevent exacerbations of COPD:
Antibiotics for exacerbations of chronic obstruc-
systematic review and meta-analysis of individual
tive pulmonary disease. Cochrane Database Syst
participant data from randomised controlled trials.
Rev 2018;10:CD010257.
Thorax 2019;74:337-45.
18. Wedzicha JA, Miravitlles M, Hurst JR, et al.
8. Kiser TH, Allen RR, Valuck RJ, et al. Outcomes
Management of COPD exacerbations: a European
associated with corticosteroid dosage in critically
Respiratory Society/American Thoracic Society
ill patients with acute exacerbations of chronic
guideline. Eur Respir J 2017;49:1600791.

ACCP Updates in Therapeutics® 2022: Critical Care Pharmacy Preparatory Review and Recertification Course

227
Pulmonary Disorders II

ANSWERS AND EXPLANATIONS TO PATIENT CASES

1. Answer: C Other answers include one risk factor or none (Answers


The patient presents with a CF exacerbation, probably A, B, and D are incorrect).
caused by an infection. The most likely causative organ-
ism of her infection is P. aeruginosa; therefore, therapy 4. Answer: C
must be directed to P. aeruginosa (Answer B is incor- For life-threatening asthma exacerbations, SABAs and
rect). In addition, antibiotic treatment should include the intravenous corticosteroids are recommended (Answers
empiric selection of two antibacterial agents, ideally a A and B are incorrect). The recommended dose of intra-
β-lactam and an aminoglycoside, dosed to effectively venous corticosteroids is methylprednisolone 40–80 mg
treat the infection (Answers A and D are incorrect). The intravenously per day administered early in the course
ideal regimen should include a β-lactam dosed to treat P. of the exacerbation (Answer C is correct; Answer D is
aeruginosa – in this case, piperacillin/tazobactam at the incorrect).
recommended dose – and an aminoglycoside dosed once
daily (Answer C is correct). 5. Answer: D
The latest GOLD guidelines recommend systemic corti-
2. Answer: A costeroids to shorten recovery time, improve FEV1, and
The patient’s symptoms and physical findings place him improve hypoxemia. The recommended dose is predni-
in WHO FC IV. His unfavorable response to the vaso- sone 40 mg orally once daily (or equivalent) for 5 days.
dilator challenge makes calcium channel blockers an No evidence suggests that higher corticosteroid doses
undesirable class of medications for him (Answer B is are beneficial, and higher doses may in fact be associ-
incorrect). Epoprostenol continuous infusion is indicated ated with more adverse effects (Answers A and C are
for patients with PAH presenting with WHO FC IV to incorrect). Adding inhaled SABAs (nebulized or MDI)
improve symptoms, exercise capacity, and hemodynam- with or without a short-acting anticholinergic is the pre-
ics. In addition, it is the only treatment shown to reduce ferred treatment in a COPD exacerbation. Antibiotic
mortality in PAH (Answer A is correct). Macitentan treatment for 5–7 days is also indicated because the
and sildenafil could be considered in this patient; how- patient has signs of bacterial infection (fever, purulent
ever, his elevated liver enzymes do not make macitentan sputum) and a very severe COPD exacerbation with
an ideal agent (Answer C is incorrect). Intravenous respiratory failure. This patient has a history of anaphy-
sildenafil is also not ideal because of the hypotension lactic reaction to penicillin and repeated exacerbations
associated with the intravenous formulation (Answer D that may predispose her to resistant organisms such as P.
is incorrect). aeruginosa (Answers A and B are incorrect); therefore,
levofloxacin would be the best recommendation (Answer
3. Answer: C D is correct).
Risk factors for increased mortality in patients with
asthma include (1) history of near-fatal asthma (e.g.,
requiring mechanical ventilation); (2) hospitalization
or ED visit for asthma in the past year; (3) active use
of oral corticosteroids or completion of recent course
for asthma; (4) not currently using inhaled corticoste-
roids; (5) use of more than one canister of SABAs per
month; (6) poor adherence to inhaled corticosteroid
containing asthma medications and/or poor adherence
to asthma action plan; (7) social history that includes
major psychosocial problems or psychiatric illness; (8)
food allergies; (9) comorbidities including pneumonia,
diabetes, and arrhythmias. Two of the patient’s listed
characteristics are risk factors (Answer C is correct).

ACCP Updates in Therapeutics® 2022: Critical Care Pharmacy Preparatory Review and Recertification Course

228
Pulmonary Disorders II

ANSWERS AND EXPLANATIONS TO SELF-ASSESSMENT QUESTIONS

1. Answer: D 4. Answer: B
It is imperative to recognize that this patient has acute The recommended corticosteroid dose for a COPD exac-
respiratory distress syndrome (ARDS) caused by a CF erbation is prednisone 40 mg orally once daily (Answer
exacerbation. Therefore, an inclusive therapy plan should D is incorrect; Answer B is correct). Antimicrobial
include appropriate treatments for ARDS and CF. For treatment should be initiated if (1) all three cardinal
ARDS, a lung-protective ventilation strategy (tidal vol- symptoms of a COPD exacerbation (increased dyspnea,
ume 4–6 mL/kg) and a fluid-conservative strategy (CVP increased sputum production, and increased sputum
less than 4 mm Hg, if not in shock) are of utmost impor- purulence) are present; (2) two of the three cardinal signs
tance (Answers A and C are incorrect because of the are present, with increased sputum purulence as one of
CVP goal; Answer D is correct). Appropriate treatment the symptoms; or (3) the patient requires noninvasive or
of the CF exacerbation includes empiric therapy for P. invasive ventilation. This patient has no indication for
aeruginosa in the form of optimal doses of a β-lactam antimicrobials (Answers A and C are incorrect).
and aminoglycoside (Answer B is incorrect because
of the inappropriate tobramycin dose and CVP goal of
10–14 mm Hg).

2. Answer: D
This patient presents with severe right heart failure. The
primary goal is to optimize RV preload by maintaining a
net negative fluid balance using gentle diuresis and blood
pressure monitoring (Answer D is correct). Dopamine
would increase blood pressure; however, it might
worsen the patient’s tachycardia, thereby worsening her
already tenuous clinical status (Answer A is incorrect).
Epoprostenol would help decrease pulmonary pressures;
however, epoprostenol would potentially worsen the
patient’s blood pressure because of its peripheral vaso-
dilating effects (Answer B is incorrect). Phenylephrine
would not be optimal because this vasopressor might
worsen RV function, further elevate pulmonary arterial
pressure by α1-receptors in the pulmonary vasculature,
and potentially induce a reflex bradycardia (Answer C is
incorrect).

3. Answer: C
This patient has shortness of breath at rest that is inter-
fering with his conversational ability, and his FEV1
is less than 50% of predicted; therefore, his asthma
exacerbation would be classified as severe (Answer C
is correct). FEV1 would be greater than 50% in mild/
moderate asthma exacerbation (Answers A and B are
incorrect). In a life-threatening asthma exacerbation, the
patient would have symptoms such as drowsiness, con-
fusion, or silent chest. This patient could progress to life
threatening but currently would not be classified as such
(Answer D is incorrect).

ACCP Updates in Therapeutics® 2022: Critical Care Pharmacy Preparatory Review and Recertification Course

229
Infectious Diseases II
Gabrielle Gibson, Pharm.D., BCCCP, BCPS
Barnes-Jewish Hospital
St. Louis, Missouri

Simon Lam, Pharm.D., FCCM, BCCCP, BCPS


Cleveland Clinic
Cleveland, Ohio
Infectious Diseases II

Infectious Diseases II
Gabrielle Gibson, Pharm.D., BCCCP, BCPS
Barnes-Jewish Hospital
St. Louis, Missouri

Simon Lam, Pharm.D., FCCM, BCCCP, BCPS


Cleveland Clinic
Cleveland, Ohio

ACCP Updates in Therapeutics® 2022: Critical Care Pharmacy Preparatory Review and Recertification Course

233
Infectious Diseases II

Learning Objectives MSSA Methicillin-sensitive Staphylococcus


aureus
1. Compose a plan to incorporate quality metrics (e.g., OI Opportunistic infection
prevention of catheter-associated urinary tract infec- PCR Polymerase chain reaction
tions and catheter-related bloodstream infections) PCT Procalcitonin
into pre- and postsurgical care. PD Pharmacodynamic(s)
2. Identify key members of, common strategies, and PK Pharmacokinetic(s)
tools (including biomarkers and rapid diagnostic PNA FISH Peptide nucleic acid fluorescent in situ
tests) used by an antimicrobial stewardship team. hybridization
3. Provide empiric antibiotic therapy recommendations SCIP Surgical Care Improvement Project
for critically ill patients with community-acquired SOT Solid organ transplantation
or health care–associated meningitis. SSI Surgical site infection
4. Evaluate therapeutic options for the treatment of UTI Urinary tract infection
multidrug-resistant pathogens in the intensive care WHO World Health Organization
unit (ICU).
5. Devise an optimal treatment plan for critically ill
immunocompromised patients with infectious dis- Self-Assessment Questions
eases. Answers and explanations to these questions can be
found at the end of this chapter.

Abbreviations in This Chapter 1. A 56-year-old man (weight 140 kg) is scheduled to


have elective coronary artery bypass surgery with
ACS NSQIP American College of Surgeons National aortic valve bioprosthetic replacement. The patient
Surgical Quality Improvement Program has a medical history of diabetes and hypertension.
AST Antimicrobial susceptibility testing He has no drug allergies and has a calculated CrCl
CAUTI Catheter-associated urinary tract of 90 mL/minute/1.73 m2. It is anticipated that the
infection patient will be admitted to the cardiovascular sur-
CDC Centers for Disease Control and gery ICU after the surgery with two chest tubes
Prevention placed for drainage. Which is the most appropriate
CLSI Clinical & Laboratory Standards antimicrobial prophylactic regimen for the patient?
Institute A. Cefazolin 2 g once within 2 hours before inci-
CMS Centers for Medicare & Medicaid sion and administered every 8 hours until chest
Services tubes are removed.
CNS Central nervous system B. Vancomycin 2 g once within 2 hours before
CoNS Coagulase-negative Staphylococci incision and administered every 12 hours until
CRE Carbapenem-resistant Enterobacterales 48 hours after surgery.
CSF Cerebrospinal fluid C. Cefazolin 3 g once within 1 hour before inci-
ESBL Extended-spectrum β-lactamase sion, re-dosed if surgery lasts more than 4
HAART Highly active antiretroviral therapy hours, and continued for 48 hours after surgery.
IDSA Infectious Diseases Society of America D. Cefazolin 2 g once within 1 hour before surgery,
ICU Intensive care unit re-dosed if surgery lasts more than 4 hours, and
KPC Klebsiella pneumoniae carbapenemase continued for 48 hours after surgery.
LOS Length of stay
MALDI-TOF Matrix-assisted laser desorption- 2. Which statement best describes antimicrobial stew-
ionization/time of flight ardship in the ICU?
MIC Minimum inhibitory concentration
A. Formal antimicrobial stewardship team
MRSA Methicillin-resistant Staphylococcus
membership should be reserved for clinical
aureus
pharmacists with infectious diseases training.

ACCP Updates in Therapeutics® 2022: Critical Care Pharmacy Preparatory Review and Recertification Course

234
Infectious Diseases II

B. Antibiotic cycling is helpful in minimizing 5. An 81-year-old man is admitted to the medical


antimicrobial resistance. ICU with presumed line-associated sepsis. Blood
C. Daily clinical activities of a critical care cultures are obtained that grow pan-sensitive
pharmacist may constitute an antimicrobial Enterobacter cloacae. The patient is being treated
stewardship effort. with intravenous ceftriaxone, and the offending cen-
D. Antimicrobial stewardship should not be insti- tral venous catheter is removed. The patient initially
tuted in critically ill patients because restriction responds, but on day 10 of therapy, he becomes
of antimicrobial choices may worsen outcomes. febrile, and blood cultures are re-sent. At that time,
the patient was maintained on ceftriaxone because
3. A 33-year-old woman with a history of hydrocepha- he was hemodynamically stable. Gram stain for the
lus since early childhood requiring placement of an new blood cultures is positive for lactose-positive
internal cerebrospinal fluid (CSF) shunt, presents gram-negative bacilli. Which is the most appropriate
with a temperature of 102.8°F (39.3°C), an altered action for this patient’s antimicrobial management?
mental status, and a white blood cell count of 19 x A. Change ceftriaxone to ceftazidime.
103 cells/mm3. After a computed tomography scan B. Change ceftriaxone to piperacillin/tazobactam.
reveals no hydrocephalus or midline shift, a lum- C. Change ceftriaxone to meropenem.
bar puncture is performed. Initial cell count shows D. Continue ceftriaxone alone.
elevated CSF white blood cells with a high propor-
tion of neutrophils and low glucose. Neurosurgery 6. An 86-year-old woman with a history of end-
is consulted for a presumed CSF shunt infection. stage renal disease is admitted to the hospital
Which empiric system antibiotic regimen is most with respiratory distress requiring intubation,
appropriate? fluid resuscitation, and hemodynamic monitoring.
A. Ceftriaxone and ampicillin. Bronchoalveolar lavage cultures show methicillin-
B. Ceftriaxone and vancomycin. resistant Staphylococcus aureus (MRSA). The
C. Piperacillin/tazobactam and tobramycin. patient is being treated with intravenous vancomy-
D. Cefepime and vancomycin. cin. On day 4 of therapy, the patient develops fever,
leukocytosis, and erythema around the insertion site
4. A 75-year-old man is admitted to the medical ICU of her tunneled dialysis catheter. Blood cultures are
from the emergency department with septic shock. sent, and the dialysis catheter is removed. Gram
He is fluid resuscitated and administered broad- stain from blood cultures is significant for gram-
spectrum antibiotics with piperacillin/tazobactam positive cocci in pairs and chains. The medical
and vancomycin. On day 2 of therapy, the patient team discontinues vancomycin and approaches you
remains hemodynamically unstable, requiring nor- to inquire about treatment options for this patient.
epinephrine. The patient’s blood culture is positive, Which agent is most appropriate for this patient?
according to the institution’s microbiology labora- A. Daptomycin.
tory, which recently implemented rapid diagnostic B. Linezolid.
tests using nanoparticle microarray technology, and C. Ceftaroline.
the microarray results show the presence of D. Tigecycline.
Escherichia coli with the CTX-M resistance gene.
Which intervention regarding the patient’s antimi- 7. A 35-year-old man is admitted to the medical ICU
crobial therapy is most appropriate? with respiratory distress. He has a 3-week history
A. Change piperacillin/tazobactam to cefepime. of cough and pleuritic chest pain that has wors-
B. Add tobramycin to the current regimen. ened with time. Chest radiography is performed,
C. Add levofloxacin to the current regimen. which shows bilateral infiltrates with ground-glass
D. Change piperacillin/tazobactam to meropenem. opacities. The patient is HIV positive and not cur-
rently receiving antiretroviral therapy because
of nonadherence. The patient requires intubation
and is receiving 40% fraction of inspired oxygen

ACCP Updates in Therapeutics® 2022: Critical Care Pharmacy Preparatory Review and Recertification Course

235
Infectious Diseases II

(Fio2). His relevant laboratory values are as fol-


lows: SCr 1.0 mg/dL, CD4+ count 100/mm3, lactate
dehydrogenase (LDH) 550 IU/L, partial pressure
of arterial oxygen (PaO2) 80 mm Hg, partial pres-
sure of arterial carbon dioxide (PaCO2) 40 mm Hg,
and white blood cell count (WBC) 4 x 103 cells/
mm3. The patient has a sulfa allergy and a glucose-
6-phosphate dehydrogenase deficiency. In addition
to adjunctive corticosteroids, which regimen for
presumed Pneumocystis jiroveci pneumonia is most
appropriate?
A. Trimethoprim/sulfamethoxazole 15–20 mg/kg
intravenously, divided every 6 hours.
B. Pentamidine 4 mg/kg intravenously every 24
hours alone.
C. Primaquine 30 mg orally once daily plus
clindamycin 600 mg intravenously every 8
hours alone.
D. Atovaquone 750 mg orally every 12 hours.

8. A 50-year-old woman with acute myeloid leuke-


mia is admitted for induction therapy. During the
patient’s clinical course, she develops respiratory
distress, for which she is admitted to the medical
ICU. She is found to have neutropenic fever and is
empirically treated with meropenem plus gentami-
cin plus vancomycin. On day 5 of empiric therapy,
blood cultures are growing pan-susceptible E. coli.
The patient’s relevant laboratory values and vital
signs are as follows: blood pressure 115/70 mm
Hg, heart rate 84 beats/minute, temperature 101.2°F
(38.4°C), and absolute neutrophil count less than 50
cells/mm3. Which is the most appropriate antimicro-
bial regimen for the patient?
A. Continue meropenem, gentamicin, and vanco-
mycin for 14 days.
B. Discontinue all empiric antimicrobials, and
initiate ampicillin intravenously to treat until
neutrophils recover.
C. Discontinue vancomycin, and continue merope-
nem and gentamicin until neutrophil recovery.
D. Add voriconazole, and continue all other anti-
microbials for 14 days.

ACCP Updates in Therapeutics® 2022: Critical Care Pharmacy Preparatory Review and Recertification Course

236
Infectious Diseases II

BPS Critical Care Pharmacy Specialist Examination Content Outline

This chapter covers the following sections of the Critical Care Pharmacy Specialist Examination Content Outline:
1. Domain 1: Clinical Knowledge and Application
a. Task 1: 1-3
b. Task 2: 3-7
c. Task 3: 1-7
d. Task 4: 1-7
e. Task 5: 1-6
f. Task 6: 1-4, 6
g. Task 7: 1-7
h. Task 8: 3, 4
2. Domain 2: Practice Management, Policy, and Quality Improvement
a. Task 1: 2
b. Task 2: 4
c. Task 4: 1-4
d. Task 5: 1
e. Task 7: 4
3. Domain 3: Evidence-Based Medicine, Scholarship, Education, and Professional Development
a. Task 1: 1-3
b. Task 3: 1

ACCP Updates in Therapeutics® 2022: Critical Care Pharmacy Preparatory Review and Recertification Course

237
Infectious Diseases II

I.  QUALITY IMPROVEMENTS

A. The American College of Surgeons National Surgical Quality Improvement Program (ACS NSQIP) provides
general best-practice guidelines. ACS NSQIP also provides a surgical risk calculator that, depending on the
type of surgery and the patient’s baseline characteristics, can generate estimated risks of complications,
which include pneumonia, cardiac complications, surgical site infections (SSIs), urinary tract infections
(UTIs), venous thromboembolism, renal failure, discharge to a rehabilitation facility, and death. (This risk
calculator can be found at http://riskcalculator.facs.org/.)

B. As of 2016, all Surgical Care Improvement Project (SCIP) measures have been retired because of consistently
high compliance rates. SCIP measures reporting is no longer mandatory. However, the practices described
within the retired SCIP measures still represent best practices and should be continued. These include
administration of an appropriate prophylactic antimicrobial within 1 hour of incision, discontinuation of the
prophylactic antibiotic within a specified time, control of post–cardiac surgery serum blood glucose, and
removal of the urinary catheter by postoperative day 2.

C. Sepsis Bundle Project (SEP-1): Specifics regarding early broad-spectrum antimicrobial administration in the
management of severe sepsis and septic shock can be found in the Shock chapter.

D. Prevention of SSIs
1. Epidemiology and clinical significance
a. More than 290,000 SSI cases occur each year.
b. Mortality rate is 2–12 times higher in patients who experience SSIs than those who do not.
c. SSIs account for $4–$10 billion in direct costs.
2. Risk factors for SSIs: Advanced age, obesity, hyperglycemia, dyspnea, hypoxia, smoking, alcoholism,
recent radiotherapy, preoperative albumin less than 3.5 mg/dL, total bilirubin greater than 1.0 mg/dL,
trauma/shock, transfusion, hypothermia, inadequate skin preparation, abdominal surgery, contaminated
procedures, cancer, emergency surgery, staphylococcal colonization, and prolonged procedures.
3. The Centers for Medicare & Medicaid Services (CMS) no longer provides reimbursement to providers
for the treatment of SSIs after cardiac, bariatric, or orthopedic surgical procedures.
4. Prevention strategies: Pharmacy specific
a. Preoperative
i. Control serum blood glucose in patients with diabetes – ACS NSQIP measure
ii. Administer prophylactic antibiotics within 1 hour before surgery (vancomycin and
fluoroquinolones should be administered within 2 hours before surgery because of prolonged
infusion times) – ACS NSQIP measure
iii. Select the appropriate prophylaxis – See Table 1.
iv. Adjust dose of antibiotics for obesity – See Table 2 for suggested dosing (ACS NSQIP measure).
v. Patients with known nasal carriage of S. aureus should receive intranasal applications of
mupirocin 2% ointment with or without a combination of chlorhexidine gluconate body wash
(World Health Organization guidelines [WHO]).
vi. Alcohol-based antiseptic solutions with chlorhexidine gluconate for surgical site skin preparation
should be used in patients undergoing surgical procedures (WHO guidelines).
vii. vii. Preoperative bowel preparation
(a) Although unaddressed by the Centers for Disease Control and Prevention (CDC),
preoperative bowel preparation before colorectal surgery was recommended by the ACS/
Surgical Infection Society and WHO guidelines.
(b) A recent meta-analysis demonstrated that combined oral antibiotic prophylaxis combined
with mechanical bowel preparation was associated with reduced SSI rates compared with
mechanical bowel preparation alone. Oral antibiotic prophylaxis was associated with
significant decreases in rates of anastomotic leak and 30-day mortality without increasing
rates of Clostridioides difficile infections.

ACCP Updates in Therapeutics® 2022: Critical Care Pharmacy Preparatory Review and Recertification Course

238
Infectious Diseases II

(c) Combined bowel preparation with oral antibiotic prophylaxis and mechanical bowel
preparation is recommended by the 2019 American Society of Colon and Rectal Surgeons
Clinical Practice Guidelines.
b. Intraoperative
i. Re-dose antibiotics, if necessary – See Table 2 for dosing schedule during surgery (ACS NSQIP
measure).
ii. In general, antibiotics with short half-lives should be re-dosed at a frequency of 2 times the
half-life of the agent. Antibiotics should also be re-dosed if there is significant intraoperative
blood loss.
iii. Goal of re-dosing is to maintain bactericidal concentrations throughout the operation. It may
be prudent to consider re-dosing prophylaxis intraoperatively if large amounts of fluids and/or
transfusions are being administered.
iv. Maintain normothermia. Two small randomized controlled trials showed that maintaining
normothermia reduced the risk of SSIs. The mechanism for this protective effect is currently
unknown; however, a hypothesis is that this effect may result from impaired neutrophil function
or subcutaneous vasoconstriction and subsequent tissue hypoxia.
c. Postoperative
i. Discontinue prophylactic antibiotics within 24 hours after non-cardiac surgery and within 48
hours after cardiac surgery. Drains are not sufficient reason to continue prophylactic antibiotics
– ACS NSQIP measure.
ii. Early recovery after surgery (ERAS) protocols: Aim to reduce the stress of surgery on patients
by maintaining near normal physiology in the preoperative, intraoperative, and postoperative
phases of care.
(a) Early recovery after surgery protocols incorporate SSI prevention guideline
recommendations, such as parenteral antibiotic prophylaxis and strict glycemic control,
as well as interventions with newer evidence, such as oral antibiotic prophylaxis with
mechanical bowel preparation, goal directed fluid therapy, and early enteral feeding.
(b) A meta-analysis of 27 randomized controlled trials assessing 3279 patients undergoing
abdominal/pelvic surgery showed a significant reduction in postoperative SSI for patients
enrolled in early recovery after surgery programs compared with conventional pathways.
5. Management of SSIs: See Infectious Diseases I chapter.

Table 1. Prophylactic Antibiotic Regimens for Surgery


Alternative Prophylaxis in Patients
Type of Surgery Recommended Prophylaxis
with β-Lactam Allergies
Coronary artery bypass grafting
Other cardiac surgery
Vascular surgery Cefazolin Vancomycin
Hip arthroplasty Cefuroxime Clindamycin
Knee arthroplasty
Neurosurgery
Cefazolin Vancomycin
Thoracic surgery
Ampicillin/sulbactam Clindamycin
Clindamycin or vancomycin +
Gastroduodenal surgery Cefazolin aminoglycoside or fluoroquinolone or
aztreonam

ACCP Updates in Therapeutics® 2022: Critical Care Pharmacy Preparatory Review and Recertification Course

239
Infectious Diseases II

Table 1. Prophylactic Antibiotic Regimens for Surgery (continued)


Alternative Prophylaxis in Patients
Type of Surgery Recommended Prophylaxis
with β-Lactam Allergies
Cefotetan or cefoxitin or ampicillin/ Clindamycin + aminoglycoside or
Colorectal surgery sulbactam or ertapenem fluoroquinolone or aztreonam
Metronidazole + cefazolin or Metronidazole + aminoglycoside or
cefuroxime or ceftriaxone fluoroquinolone
Clindamycin or vancomycin +
Abdominal hysterectomy aminoglycoside or fluoroquinolone or
Cefotetan or cefoxitin or ampicillin/
Vaginal hysterectomy aztreonam
sulbactam
Biliary tract surgery Metronidazole + aminoglycoside or
fluoroquinolone

Table 2. Recommended Dose and Dosing Interval for Commonly Used Antibiotics for Surgical Prophylaxis
Antimicrobial Recommended Dose in Adults Dosing Interval (hr)a
Ampicillin/sulbactam 3g 2
Ampicillin 2g 2
Aztreonam 2g 4
Cefazolin 2 g, 3 g for patients weighing ≥ 120 kg 4
Cefuroxime 1.5 g 4
Cefoxitin 2g 2
Cefotetan 2g 6
Ceftriaxone 2g N/Ab
Ciprofloxacin 400 mg N/Ab
Clindamycin 900 mg 6
Ertapenem 1g N/Ab
Gentamicin 5 mg/kg of ideal or adjusted body weightc N/Ab
Levofloxacin 500 mg N/Ab
Metronidazole 500 mg N/Ab
Vancomycin 15 mg/kg N/Ab
a
With presumed normal renal function and dose as needed
b
One dose of the antibiotic should suffice for the duration of most surgical procedures.
c
Use adjusted body weight if actual body weight is > 20% of ideal body weight (IBW). Adjusted body weight = IBW + 0.4 (actual weight − IBW).
N/A = not applicable.

E. Prevention of Catheter-Associated Urinary Tract Infections (CAUTIs) – ACS NSQIP Measures


1. Definition – CDC – See Figure 1.
2. Epidemiology and clinical significance. UTIs represent 30%–40% of all nosocomial infections. Up to
80% of UTIs are caused by urinary catheterization.
a. More than 5% of postoperative patients will experience a UTI.
b. CAUTIs account for $300–$400 million in additional health care costs per year.
c. CMS no longer provides reimbursement to providers for the treatment of CAUTIs.

ACCP Updates in Therapeutics® 2022: Critical Care Pharmacy Preparatory Review and Recertification Course

240
Infectious Diseases II

3. Risk factors: Increased duration of catheterization, female gender, diabetes mellitus, bacterial
colonization of drainage bag, older age, and azotemia
4. Prevention strategies
a. Minimize the use of prolonged urinary catheters. Target removal of urinary catheters by
postoperative day 1.
b. Pharmacists may provide reminders as part of a multidisciplinary effort to minimize the placement
and duration of urinary catheters.
c. Strategies that should NOT be considered (all of these practices have low-quality evidence suggesting
no benefits in preventing CAUTIs):
i. Routine use of silver or antibiotic-impregnated catheters. According to the CDC, silver or
antibiotic-impregnated catheters can be considered if a comprehensive strategy to reduce
CAUTI rates has failed. The ACS NSQIP best-practices guidelines suggest that use of
antimicrobial urinary catheters can be considered for high-risk patients, such as those who may
require prolonged (greater than 7–10 days) catheterization.

Urinary catheter in place > 2 days and either present on calendar day of event No
Not a CAUTI
or removed the day before the event
Yes
Has the urinary catheter been removed?
Yes No

Was catheter removed on day of


event or the day before
Yes No

At least 1 of the following signs and symptoms Not a CAUTI


•  Fever (temp > 38°C)
• Urgencya At least 1 of the following signs and symptoms
• Frequencya • Fever (temp > 38°C)
• Dysuriaa • Suprapubic tendernessa
•  Suprapubic tenderness • Costovertebral angle pain or tendernessa
•  Costovertebral angle pain or tendernessa

No Yes No

Not a • Positive urine culture > 105 Not a CAUTI


No
CAUTI CFU/mL with no more than Not a CAUTI
two species of organisms
Yes
CAUTI

Figure 1. Diagnosis of catheter-associated urinary tract infection according to CDC criteria.


With no other recognized cause
a

CAUTI = catheter-associated urinary tract infection.

ACCP Updates in Therapeutics® 2022: Critical Care Pharmacy Preparatory Review and Recertification Course

241
Infectious Diseases II

ii. Addition of antibiotics to drainage bag


iii. Use of systemic antibiotic prophylaxis solely due to catheter placement
iv. Routine screening or treatment of asymptomatic bacteriuria
v. Bladder irrigation with antibiotics
5. Management of CAUTIs: See Infectious Diseases I chapter.

Patient Case

1. C.L is a 55-year-old man admitted to the surgical ICU after exploratory laparotomy for duodenal perforation
and peritonitis. The patient was transferred from the operating room to the surgical ICU with a urinary cath-
eter in place. On day 4 of the surgical ICU stay, the patient is extubated, and his urinary catheter is removed.
On day 6 of the surgical ICU stay, the patient develops signs and symptoms of UTI with fever, urgency, and
frequency. Urinalysis reveals no nitrites or leukocyte esterase. Urine cultures show 100,000 CFU/mL of E.
coli. You are asked by the quality officer of the surgical ICU to discuss this case and determine whether this
patient will qualify for the definition of CAUTI according to CDC criteria. Which statement regarding the
diagnosis of CAUTI in this patient is most appropriate?
A. Qualifies for definition of CAUTI because the patient is symptomatic and has positive cultures
B. Does not qualify for definition of CAUTI because the patient had a negative urinalysis
C. Qualifies for definition of CAUTI because the patient is symptomatic and had recent urinary catheter
D. Does not qualify for definition of CAUTI because the catheter was removed 2 days before symptoms

F. Prevention of Catheter-Related Bloodstream Infections – ACS NSQIP


1. Definition (CDC): Must satisfy both criteria
a. Laboratory-confirmed bloodstream infection
i. Definition 1: A recognized pathogen (not a common skin contaminant) found in one or more
blood cultures AND the organism is not related to an infection at a separate site
ii. Definition 2: At least one sign or symptom of infection (temperature greater than 100.4°F
[38°C], chills, hypotension) AND a common skin flora is cultured from two or more blood
samples AND organism is not related to an infection at a separate site
b. Central line placed for more than 2 days before the date of event and central line must be present or
removed within 24 hours of blood cultures being obtained.
2. Epidemiology and clinical significance
a. Incidence of catheter-related bloodstream infections: 2.7–7.4 infections per 1000 catheter-days
b. Accounts for 14,000–28,000 deaths per year
c. Increases mean length of stay (LOS) by 7 days
d. CMS no longer provides reimbursement to providers for the treatment of catheter-related bloodstream
infections.
3. Risk factors
a. Patient factors: Male gender, diabetes, hypotension, neutropenia, parenteral nutrition, prolonged
hospitalization before catheter placement
b. Catheter-related factors: Internal jugular or femoral vein placement site (compared with subclavian
vein), heavy microbial colonization at insertion site, use of non-tunneled catheters, inadequate
catheter care, prolonged duration of venous catheterization

ACCP Updates in Therapeutics® 2022: Critical Care Pharmacy Preparatory Review and Recertification Course

242
Infectious Diseases II

4. Prevention strategies
a. Use appropriate techniques for catheter insertion (e.g., skin preparation and maximal sterile barrier
precautions) and management (e.g., daily site care, routine site evaluation for local erythema).
Minimize the use of central venous catheters. Pharmacists may provide reminders as part of a
multidisciplinary effort to minimize the placement and duration of venous catheters.
b. Consider antibiotic lock therapy in patients with limited venous access or a history of catheter-
related bloodstream infections.
c. Do not use systemic antibiotic prophylaxis.
d. Antimicrobial/silver-impregnated catheter should be considered in patients with an anticipated
catheter duration greater than 5 days if rates of catheter-related bloodstream infections have not
decreased despite the implementation of a comprehensive prevention strategy.
5. Management: See Infectious Diseases I chapter.

II.  BACTERIAL MENINGITIS

A. Epidemiology
1. Bacterial meningitis is a neurologic emergency involving mild to severe inflammation of the meningeal
layers encasing the central nervous system (CNS). Cerebrospinal fluid (CSF) is intimate to the meninges,
and serves as both a medium for pathogen growth and a diagnostic fluid.
2. Bacterial meningitis has community-acquired or health care–associated epidemiology; health care–
associated meningitis is usually associated with neurotrauma or neurosurgical procedures.
3. Community-acquired bacterial meningitis has an annual incidence in adults of about two cases per 100,000
people. The incidence has been decreasing in recent years (two cases per 100,000 in 1998 to 1.38 cases
per 100,000 in 2006; 31% decrease), likely because of increased vaccination against common pathogens.
The incidence of nosocomial bacterial meningitis varies depending on the mechanism of neuro-anatomic
disruption and ranges from 1.5% of patients undergoing craniotomy to 25% of post-trauma patients with
basilar skull fracture.
4. Delayed CSF sterilization beyond 24 hours is a risk factor for subsequent neurologic sequelae, including
intracranial hypertension, seizures, and permanent neurologic deficit. Clinical presentations of septic
shock, altered mental status, and seizures are associated with worse outcomes. Additional complications
include respiratory failure and hyponatremia.
5. Crude mortality for community-acquired meningitis is 19%–37%, whereas mortality for health care–
associated meningitis is generally lower, particularly if associated with a reversible procedure or
removable device.

B. Definitions
1. Community-acquired meningitis is an infection unrelated to a neurosurgical procedure, neurotrauma,
or hospitalization.
2. Health care–associated or nosocomial meningitis is an infection related to invasive procedures,
including craniotomy, internal or external ventricular catheters, lumbar puncture, intrathecal medication
administration, and/or spinal anesthesia. Additional causes include complicated cranial trauma,
traumatic brain injury, and hematogenous spread in patients with hospital-acquired bacteremia.

ACCP Updates in Therapeutics® 2022: Critical Care Pharmacy Preparatory Review and Recertification Course

243
Infectious Diseases II

C. Diagnosis
1. The clinical diagnosis of meningitis is nonspecific and difficult to distinguish from that of other
infections. Although headache, fever, neck stiffness, and altered mental status are present in almost 95%
of patients with community-acquired meningitis, fever and a decreased level of consciousness are the
most consistent clinical features in patients with health care–associated meningitis.
2. Lumbar puncture or other method (e.g., from existing drain or shunt) to sample CSF for cell count and
analysis, as well as Gram stain and culture, is necessary for definitive diagnosis. Neuroimaging with
head computed tomography to detect prelumbar brain shift and risk of brain herniation should be done
before lumbar puncture in patients with suspected cranial mass (e.g., immunosuppressed, papilledema,
history of CNS disease, new-onset seizure, and focal neurologic deficit).
a. Opening pressure during lumbar puncture is usually increased in bacterial meningitis.
b. Cell count and fluid analysis
i. Community-acquired bacterial meningitis can be differentiated from other causes of
meningitis (e.g., viral, aseptic). In general, bacterial meningitis is associated with CSF that is
predominantly neutrophilic and has lower glucose concentration. Individual strong predictors
of bacterial meningitis include:
(a) CSF glucose less than 34 mg/dL
(b) Ratio of CSF to blood glucose less than 0.23
(c) CSF protein greater than 220 mg/dL
(d) CSF leukocyte count greater than 2000 cells/mm3
(e) CSF neutrophil count greater than 1180 cells/mm3
ii. The diagnostic utility of CSF cell count and fluid analysis in health care–associated meningitis
is unknown but is likely limited because of concomitant reasons for local inflammation related
to devices or recent procedures. In the presence of CSF drains or recent history of neurosurgery,
new headache, nausea, lethargy, and/or change in mental status may be a sign of new CNS
infection. Neither normal CSF analysis nor negative Gram stain may exclude the presence of
an infection. Elevated CSF lactate concentration may be useful to distinguish meningitis from
other infectious sources.
c. Gram stain and culture: Bacteriologic examination of CSF can provide rapid and reliable
identification of the causative pathogen(s). Although CSF cell count and analysis is the diagnostic
foundation for community-acquired meningitis, CSF culture is the most specific test for health
care–associated meningitis. Of note, immunocompromised patients with meningitis may present
with normal CSF cell counts.

D. Management and Treatment


1. Appropriate empiric antibiotic therapy administered intravenously, targeted against likely pathogens,
and guided by local antibiotic susceptibility patterns should be initiated as soon as possible after bacterial
meningitis is suspected. See Table 3 for common pathogens and recommended empiric therapy.
2. Antibiotic dosing and administration strategies should be chosen to optimize CSF concentrations relative
to bacterial minimum inhibitory concentration (MIC) and antibiotic pharmacodynamic (PD) properties.
Although meningeal inflammation may promote CSF penetration, data are inconsistent regarding its
impact on effective antimicrobial delivery. Furthermore, as meningitis improves, inflammation will
likely improve, and there is currently no way to predict the level of penetration in correspondence to the
severity of the inflammation.
3. Gram stain can be used to broaden bacterial coverage, but final culture should be reserved for antibiotic
de-escalation and definitive antibiotic regimen.

ACCP Updates in Therapeutics® 2022: Critical Care Pharmacy Preparatory Review and Recertification Course

244
Infectious Diseases II

4. Role of corticosteroids
a. Adjunctive corticosteroids may improve outcomes by reducing reactive meningeal inflammation
and neurologic sequelae related to antibiotic-induced bacterial lysis.
b. Conflicting results are published regarding the effects of systemic corticosteroids on neurologic
sequelae and mortality among patients with bacterial meningitis.
c. Studies from high-income countries tend to suggest that systemic corticosteroids decrease or trend
toward a decrease in mortality and neurologic sequelae.
d. The outcome benefit associated with systemic corticosteroids seems most pronounced with patients
with Streptococcus pneumoniae meningitis. However, because corticosteroids must be administered
before the receipt of antimicrobials, it is unlikely that clinicians will know the etiology of the disease
when making the decision for steroids.
e. The Infectious Diseases Society of America (IDSA) guidelines recommend administering
dexamethasone 0.15 mg/kg every 6 hours for up to 96 hours, with the first dose administered 10–20
minutes before, or at least concomitant with, the first dose of antimicrobial therapy. The IDSA
guidelines also recommend for continuation of dexamethasone only if cultures show the presence of
S. pneumoniae, although this recommendation is not supported by strong clinical evidence.
5. Health care–associated meningitis
a. See Table 3 for recommended empiric treatment of health care–associated meningitis.
b. In patients with CSF shunts, complete removal of an infected CSF shunt and replacement with an
external ventricular drain combined with intravenous antimicrobial therapy is recommended.
c. In selected patients with bacterial meningitis after placing a CSF shunt, the IDSA recommends direct
instillation of antimicrobial agents intraventricularly through either an external ventriculostomy
or shunt reservoir. This practice should only be considered in patients with pathogens that are
difficult to eradicate or for those who cannot undergo catheter replacement. See Table 4 for
selected antimicrobial intraventricular dosing. When intraventricular antibiotics are administered,
drains should be clamped for 15-60 minutes to allow antibiotics to equilibrate throughout the
CSF. Antimicrobial agents administered into the CNS should be preservative free. Dosing and
intervals of intraventricular antimicrobial therapy should be adjusted on the basis of the CSF
antimicrobial concentration being 10-20 times the MIC of causative pathogen, ventricular size,
and daily output from ventricular drain. Given the paucity of pharmacokinetic data surrounding
intraventricular antibiotics administration and the lack of head-to-head comparisons with
intravenous antimicrobials, the administration of systemic antimicrobials should be continued in
conjunction with intraventricular antibiotics.
6. Definitive antibiotic therapy can be determined according to finalized cultures and susceptibility results.
In general, definitive therapy should entail choosing the therapy with the most appropriate spectrum of
activity and an adequate penetration into the CSF. Therapy should be continued for at least 7 days in all
patients with meningitis. S. pneumoniae, coagulase-negative staphylococci, S. aureus, gram-negative
bacilli, and Propionibacterium acnes should be treated for 10–14 days and Listeria monocytogenes for
at least 21 days. CSF should be negative for S. aureus for at least 10 days before shunt replacement.

ACCP Updates in Therapeutics® 2022: Critical Care Pharmacy Preparatory Review and Recertification Course

245
Infectious Diseases II

Table 3. Common Pathogens Seen in Different Bacterial Meningitis Populations and the Corresponding Recommended
Empiric Therapy
Patient Group Common Pathogens Recommended Empiric Therapy
Community-Acquired Pathogens
Vancomycin + third-generation
18–50 yr N. meningitidis, S. pneumoniae
cephalosporin (e.g., ceftriaxone)
N. meningitidis, S. pneumoniae, Vancomycin + third-generation
> 50 yr or any age with
L. monocytogenes, H. influenzae, aerobic GNB cephalosporin (e.g., ceftriaxone) +
predisposing conditiona
(e.g., E. coli) ampicillin
Health care–associated or nosocomial
Basilar skull fracture
S. pneumoniae; H. influenzae; Vancomycin + third-generation
(communication with
group A β-hemolytic streptococci cephalosporin (e.g., ceftriaxone)
sinuses or oropharynx)
Patient Group Common Pathogens Recommended Empiric Therapy
Health care–associated or nosocomial
S. aureus, CoNS, aerobic enteric (e.g., E. coli)
Vancomycin + cefepime or
Penetrating trauma and non–lactose-fermenting (e.g., P. aeruginosa)
ceftazidime or meropenem
gram-negative bacilli
Aerobic enteric (e.g., E. coli) and non–lactose-
Vancomycin + cefepime or
Post-neurosurgery fermenting (e.g., P. aeruginosa) gram-negative
ceftazidime or meropenem
bacilli, S. aureus, CoNS
Aerobic enteric (e.g., E. coli) and non–lactose-
Vancomycin + cefepime or
CSF shunt fermenting (e.g., P. aeruginosa) gram-negative
ceftazidime or meropenem
bacilli, S. aureus, Propionibacterium acnes
a
Altered immune status, alcoholism
CoNS = coagulase-negative staphylococci; GNB = gram-negative bacteria.

Table 4. Selected Antibiotic CSF Penetration and Dosages for Meningitis


CSF Penetration,
Agent Dosage for Meningitis
% of Serum Concentration
Conventional: 10–15 mg/kga IV q8–12hr
Amikacin Low Extended interval: 20–25 mg/kga IV once daily
Intraventricular: 30 mg once daily
Ampicillin 13–14 2 g IV q4hr
Cefazolin 8–15 2 g IV q8hr
Cefepime 10–20 2 g IV q8hr
Cefotaxime 10–12 2 g IV q4hr
Ceftazidime 30–40 2 g IV q8hr
Ceftriaxone 8–12 2 g IV q12hr
Ciprofloxacin 20–25 400 mg IV q8hr
Colistin Low Intraventricular: 10 mg once daily

ACCP Updates in Therapeutics® 2022: Critical Care Pharmacy Preparatory Review and Recertification Course

246
Infectious Diseases II

Table 4. Selected Antibiotic CSF Penetration and Dosages for Meningitis (continued)
CSF Penetration,
Agent Dosage for Meningitis
% of Serum Concentration
2 g IV q8hr (extended infusions over 3 hr may be
Meropenem 6–21
considered)
Nafcillin 1–2.5 2 g IV q4hr
Not recommended because of low penetration of
Piperacillin/tazobactam 20–30
tazobactam
Conventional 2–3 mg/kg IV q8–12hr
Tobramycin 10–20 Extended interval: 5–7 mg/kg IV once daily
Intraventricular 10–20 mg once daily
15–20 mg/kg IV q8–12hr
Vancomycin 30
Intraventricular 10–20 mg once daily
a
Use IBW unless actual body weight is > 20% of IBW. If actual body weight is >20% IBW use adjusted body weight. Adjusted body weight = IBW + 0.4 (actual
weight − IBW).
IV = intravenous(ly); q = every

Patient Case

2. H.K. is a 42-year-old woman who presents with her husband to the emergency department from home with
mental status changes, lethargy, and a temperature of 102.9°F (39.4°C). Her husband reports that she had
a severe headache 24 hours earlier. She has no significant medical history. Diagnostic workup results are
highly suggestive of meningitis. A lumbar puncture is performed, during which a high initial pressure is
noted. Which intervention would be best initially for H.K.?
A. Administer dexamethasone 0.15 mg/kg intravenously.
B. Await laboratory and microbiologic analysis.
C. Initiate empiric ceftriaxone and vancomycin.
D. Initiate empiric ampicillin, cefepime, and vancomycin.

III.  ANTIMICROBIAL STEWARDSHIP

A. Goals: To optimize clinical outcomes while minimizing unintended consequences of antimicrobial use,
which include the emergence of antimicrobial resistance and adverse drug reactions. In addition, a responsible
approach to the use of antimicrobial agents should reduce the overall costs associated with treatment.
1. Antibiotics are the second most common class of drugs that cause adverse effects.
2. Antibiotics are the most common class of medication to be associated with prescribing errors.
3. Minimizing antimicrobial resistance by optimizing therapy should lead to measurable benefits at the
patient level because studies have shown the negative effect of antimicrobial resistance on several
clinical outcomes.
4. In 2018, CMS finalized a rule that requires all acute care hospitals participating in Medicaid/Medicare
services to implement antimicrobial stewardship programs.

ACCP Updates in Therapeutics® 2022: Critical Care Pharmacy Preparatory Review and Recertification Course

247
Infectious Diseases II

5. In 2019, the CDC updated the hospital core elements to reflect the new evidence in the field of
antimicrobial stewardship. Major updates include hospital leadership commitment, accountability,
pharmacy expertise, tracking, reporting, and education.

B. Antimicrobial Stewardship Definition: Multidisciplinary and coordinated interventions designed to improve


and measure the appropriate use of antimicrobial agents and to promote the selection of optimal antimicrobial
drug regimen, including dosing, duration, and route of administration.
1. Core members of a multidisciplinary antimicrobial stewardship team should include an infectious
diseases physician, a clinical pharmacist, a clinical microbiologist, an information systems specialist, an
infection control professional, and a hospital epidemiologist.
2. Broad-spectrum antimicrobials are often necessary in the ICU to provide early effective therapy.
However, continued use of broad-spectrum antimicrobials may lead to the selection of pathogenic
organisms (i.e., C. difficile, Candida spp.), nephrotoxicity, and the emergence of resistance.

C. Critical Care Pharmacists – Well positioned to provide guidance on antimicrobial therapy, including expedited
selection of appropriate initial agents, aggressive dosing to optimize PD, interpretation of microbiological
evidence, appropriate de-escalation of antimicrobials, monitoring of response and potential adverse effects,
and determination of the appropriate treatment duration
1. Critical care pharmacists could either have an active role as a member of a multidisciplinary antibiotic
stewardship team or serve in many of the different roles and activities of antibiotic stewardship.
2. Core activities that can be pursued through formal pathways or everyday clinical interventions
a. Education with active intervention – Interpretation of rapid identification tests and susceptibility
testing
b. Guidelines and clinical pathway development. Recent IDSA guidelines recommend facility-specific
clinical practice guidelines or algorithms as an effective way of standardizing prescribing practices.
c. Streamlining or de-escalation of therapy
i. Shortening therapy duration
ii. Discontinuing unnecessary antimicrobials
d. Dose optimization
i. Application of pharmacokinetic (PK)/PD principles. Per IDSA recommendations, PK and dose-
monitoring programs are specifically recommended for vancomycin and aminoglycosides.
ii. Dose adjustments based on organ function
iii. Allergy detection and assessment, in particular promotion of β-lactam skin testing when
appropriate
e. Parenteral to oral conversion
f. Recent IDSA guidelines recommend against the use of didactic education alone for stewardship.
3. Support from microbiology laboratory and electronic medical record system surveillance and decision
support may further enhance stewardship efforts.

D. Antimicrobial Stewardship Strategies (can be used synergistically)


1. Prospective audit with intervention and feedback (“back-end” strategy)
a. Allows flexibility and minimizes delay in administering therapy
b. The most successful strategy involves direct communication with treating physicians and required
documentation for acceptance of recommendation or rationale for denial.
2. Formulary restriction and preauthorization (“front-end” strategy).
a. May be resource-intensive, and prescribers may feel a loss of autonomy
b. Initial choices of antimicrobials may be optimized through consultation with infectious disease
experts.

ACCP Updates in Therapeutics® 2022: Critical Care Pharmacy Preparatory Review and Recertification Course

248
Infectious Diseases II

c. Antimicrobial cycling: An example of formulary restriction in which there is a scheduled removal


and substitution of a specific antimicrobial or an antimicrobial class. It is an effort to minimize
antimicrobial selection pressures. Evidence is insufficient to suggest that antimicrobial cycling
strategies are effective. IDSA stewardship guidelines recommend against the use of antimicrobial
cycling as a stewardship strategy.
3. Regardless of the strategies used or the quality of clinical pathways, programmatic antibiotic stewardship
is not a substitute for clinical judgment.
4. Process indicators (e.g., days of therapy, provider adherence to clinical pathway, time to effective
therapy) and outcome measures (e.g., length of stay, mortality, 30-day readmission) should be used to
gauge program success.

E. Effective Antimicrobial Stewardship Programs Can Be Financially Self-Supporting


1. Antimicrobials can account for up to 30% of hospital pharmacy budgets, with up to 50% of antimicrobial
use being inappropriate, leading to increased cost, increased selection of resistant pathogens, and
increased selection of opportunistic infections (OIs).
2. Several studies have shown significant cost savings associated with an antimicrobial stewardship
program. However, most of these studies focused only on antimicrobial costs, with little focus devoted
to indirect costs and implementation costs.
3. There is currently a proposed CMS rule for incorporating antibiotic prescribing patterns as one of the
quality-reporting metrics. Many CMS quality metrics are tied to reimbursement. If this metric becomes
tied to reimbursement, there will be further incentives to implement antimicrobial stewardship programs.
4. Effective January 2017, the Joint Commission announced a new medication management standard that
addresses antimicrobial stewardship. Many of the standards are applicable and relevant to critical care
pharmacists: continuing education of staff, education of patients and family members, participation
on an antimicrobial stewardship team, and development and execution of antimicrobial management
protocols.

F. Evidence Supporting Antimicrobial Stewardship Efforts in Critically Ill Patients


1. Studies in this area have been limited by poor study design. However, most studies have reported a
decreased use in either antibiotics overall or a targeted class of antibiotics. Some studies have also
reported a decrease in key resistance rates.
2. Antimicrobial stewardship strategies used in the studies have varied widely, including restriction, formal
infectious disease physician consultation, protocols for de-escalation, implementation of computer-
assisted decision support, and formal reassessments of the empiric antibiotics by a stewardship team.
The compensatory overuse of another class of antibiotics has been reported when attempts to restrict a
different class of antibiotics were implemented.
3. Meta-analyses of before-after studies evaluating directed audit and feedback to ICU patients receiving
antibiotics found no increase in mortality, suggesting that this practice could safely be implemented in
critically ill patients.

ACCP Updates in Therapeutics® 2022: Critical Care Pharmacy Preparatory Review and Recertification Course

249
Infectious Diseases II

IV.  RAPID DIAGNOSTIC TESTS

A. Rationale
1. There is an association between delay in administration of appropriate antimicrobials and decreased
survival among patients with septic shock.
2. Broad-spectrum antimicrobials are usually used in severe infections to ensure that all potential pathogens
are covered.
3. Rapid diagnostic tests may assist in de-escalation efforts in an attempt to practice antimicrobial
stewardship. Critical care pharmacists are advocates for the appropriate use of antimicrobials according
to the results of rapid diagnostic tests.
4. In many cases, the implementation of rapid diagnostic tests may be cost neutral, or even constitute a cost
savings, when antimicrobial stewardship efforts leading to decreased consumption of antibiotics occur.
5. Recommended by IDSA antimicrobial stewardship guidelines to be used in combination with stewardship
team to optimize antibiotic therapy and improve clinical outcomes
6. The Society of Infectious Diseases Pharmacists released an official position statement stating that
rapid diagnostic tests can help antimicrobial stewardship programs decrease unnecessary exposure and
optimize patient care.

B. Early Pathogen Identification from Positive Bloodstream Cultures


1. All discussed methods attempt to shorten the time from blood culture positivity to species identification
or susceptibility testing. Traditional pathogen identification and susceptibility testing can take 72–96
hours. Early pathogen identification techniques seek to provide clinically actionable information within
the first 24 hours from the time of culture positivity. See Table 5 for examples of rapid diagnostic tests,
corresponding targets, and respective turnaround times.
2. Most of these techniques, when combined with antibiotic stewardship efforts, lead to significant
reductions in health care costs and improvements in clinical outcomes in relevant studies. Many of
these were single-center studies, which makes external validity questionable. The overall impact of the
implementation of rapid diagnostic tests in a single institution is determined by several factors.
a. Epidemiology of targeted organisms
b. Presence and actions of existing antimicrobial stewardship teams
c. Current clinician prescribing patterns
d. Patient population
3. The inability to detect polymicrobial infections is a common limitation to most of the techniques
described.
4. Newer methods using whole blood (vs. blood culture medium or agar plates) are being developed, but
most are not currently U.S. Food and Drug Administration (FDA) approved (LightCycler SeptiFast,
SepsiTest).
a. A benefit to identification from whole blood is the ability to identify bacteria in patients with recent
or current antibiotic exposure.
5. Some of the technology discussed has been developed for respiratory cultures, but the clinical adaptation
in that arena is considerably less than that for blood cultures.
6. Application of rapid diagnostic tests that detect genetic encoding of resistance mechanisms requires
further education and guidelines to assist clinicians in choosing the proper therapy because traditional
susceptibility results are not available. See Table 6 for a reasonable approach based on the detection of
resistance genes and species.
7. Peptide nucleic acid (PNA) fluorescent in situ hybridization (FISH)
a. Mechanism: Targets species-specific ribosomal RNA from positive blood cultures
b. Sensitivity and specificity: 96%–100%

ACCP Updates in Therapeutics® 2022: Critical Care Pharmacy Preparatory Review and Recertification Course

250
Infectious Diseases II

c. Limitations: Does not provide antimicrobial sensitivity data. Currently available FISH products are
only used for species identification; however, the FDA approved a new product that could detect
the mecA gene for detecting the presence of methicillin resistance. This product is not currently
commercially available.
d. Application
i. Separates S. aureus from possible skin flora contamination of coagulase-negative staphylococci
(CoNS).
ii. Differentiates Enterococcus faecium (which is often resistant to ampicillin and vancomycin)
from Enterococcus faecalis.
iii. Identifies fluconazole-sensitive Candida spp. for patients empirically treated with echinocandins.
iv. Detects Pseudomonas versus non-Pseudomonas gram-negative spp. in patients treated with
combination gram-negative therapy.
e. Studies
i. A retrospective study evaluating the outcome and economic benefit of PNA FISH methods
for the early differentiation of CoNS and S. aureus bacteremia in clinical practice showed
a significant cost savings and a decrease in median LOS. Of note, the PNA FISH results
were combined with the efforts of an antimicrobial therapy team. Similar results have been
shown with PNA FISH implementations for other pathogens, including Enterococcus spp. and
Candida spp. These results are in contrast to those of another study that evaluated the pre-
and post-staphylococci PNA FISH implementation results without the use of an antimicrobial
stewardship team. This study found no significant effects on patient LOS or vancomycin use.
This suggests that rapid identification tests are probably beneficial only when combined with
educational efforts and prospective alerts to notify clinicians of the clinical applicability of the
test results.
ii. In a prospective randomized controlled study in which patients were randomized to early
notification of PNA FISH results for CoNS or S. aureus within 3 hours or usual care, intervention
was associated with decreased mortality, decreased antibiotic use, and decreased LOS. The
most pronounced benefits occurred in critically ill patients.
8. Mass spectroscopy: Matrix-assisted laser desorption-ionization/time of flight (MALDI-TOF)
a. Mechanism: Mass spectroscopy is compared with library standards for identifying pathogen species
and/or resistance mechanisms. Mass spectroscopy is an analytic technique whereby samples
are ionized that produces a mass spectrogram for the sample. Each sample has a unique mass
spectrogram, similar to a fingerprint, which can be matched to a library of reference standards for
identification.
b. Sensitivity and specificity: 98%–100%
c. Limitations
i. Similar to PNA FISH, no antimicrobial susceptibility is reported; however, the technology
could detect genes encoding resistance. MALDI-TOF for resistance genes is currently not
commercially available.
ii. Cannot be used for polymicrobial cultures
iii. No library is available for unusual organisms, though pathogen libraries are consistently being
updated.
d. Application: Potentially wider clinical applicability than PNA FISH (which is limited by the
availability of specific tests for certain pathogens) with early identification of many more pathogens.
Additional library standards for different pathogens are continually being added.

ACCP Updates in Therapeutics® 2022: Critical Care Pharmacy Preparatory Review and Recertification Course

251
Infectious Diseases II

e. Studies: This strategy has been evaluated in several studies. In a before-and-after study of MALDI-
TOF implementation, a shortened time to pathogen identification and a decrease in LOS, recurrent
infections, and mortality were seen. Of note, the implementation of MALDI-TOF included
communicating the results to the treatment clinicians by an antimicrobial stewardship team with
evidence-based antibiotic recommendations.
9. Polymerase chain reaction (PCR)-based detection systems
a. MRSA PCR test
i. Mechanism: Novel multiplex real-time assay for mecA gene
ii. Sensitivity and specificity: 98%–100% for MRSA and methicillin-sensitive S. aureus (MSSA)
identification
iii. Application: Earlier de-escalation of anti-MRSA antimicrobials and earlier appropriate
treatment with antistaphylococcal penicillin (oxacillin, nafcillin, cefazolin) for MSSA
iv. Studies: In a before-and-after study, implementation of the MRSA PCR test resulted in reduced
time to appropriate therapy and duration of unnecessary MRSA coverage. In addition, the mean
hospital costs were decreased, and there was a trend toward decreased LOS. In a very similar
study, the combination of the MRSA PCR test with antimicrobial stewardship efforts resulted
in significant decreases in LOS and cost and a trend toward decreased mortality (18% vs. 26%).
b. FilmArray System
i. Mechanism: Uses multiplex PCR technology to identify bacterial species and resistance genes
ii. Sensitivity and specificity
(a) For species identification: Greater than 90%
(b) For resistance genes: 100% (currently only available for mecA - methicillin resistance; van
A/B - vancomycin resistance; Klebsiella pneumoniae carbapenemase [KPC] - carbapenem
resistance)
iii. Application: Earlier escalation or de-escalation of antimicrobial agents
iv. Studies: No studies regarding the clinical effects of FilmArray implementation are available.
However, studies have shown an earlier time to pathogen identification.
10. Nanoparticles: Verigene blood culture test
a. Mechanism
i. Direct detection from positive blood culture medium using nanoparticle technology
ii. Nucleic acid extraction and array hybridization
b. Most-developed commercially available product with resistance gene detection
c. Sensitivity and specificity: 93%–100%
d. Studies: Currently, most studies are limited to in vitro evaluations, which showed high levels
of accuracy and decreased time to pathogen and resistance mechanism identification. Limited
clinical evidence; however, two small single-center studies have evaluated the use of Nanosphere
technology to augment clinical decisions. These studies showed a decrease in time to appropriate
antimicrobials, in addition to decreased LOS and overall cost.
11. Chromogenic media
a. Mechanism
i. Microbiological media used to identify different microorganisms by color production
ii. Growth media use enzyme substrates that release colored dyes on hydrolysis, with a wide range
of enzymes that can be targeted
iii. Potential advantage of being able to detect polymicrobial growth
b. Sensitivity and specificity: 95%–100%

ACCP Updates in Therapeutics® 2022: Critical Care Pharmacy Preparatory Review and Recertification Course

252
Infectious Diseases II

c. Limitations
i. Many different companies make different chromogenic agar media (Brilliance, chromID,
CHROMagar). Slight differences in sensitivity and specificity were seen in studies; however,
all were within acceptable ranges.
ii. Time to identification is longer than with other rapid diagnostic tests.
iii. Different manufacturers’ chromogenic agar produces different colors for positive identification.
Readers of chromogenic agar should be sufficiently trained and familiar with the product used
by the local institution.
d. Application
i. Isolation of S. aureus from other Staphylococcus spp.
ii. Detection of methicillin resistance among S. aureus
iii. Detection of vancomycin resistance
iv. Detection of specific Enterobacterales: Salmonella, E. coli O157, extended-spectrum
β-lactamase (ESBL) production
v. Differentiation of different Candida spp.
vi. Detection of KPC
e. Studies: Many clinical studies have shown significant advantages over conventional culture media.
With the advent of newer technology and shorter detection times, the clinical applicability of
chromogenic media may be limited. However, few microbiology laboratories have implemented
rapid diagnostic methods because of the considerable upfront costs. Centers where chromogenic
media are being used may continue to rely on this technology.

Table 5. Examples of Rapid Diagnostic Tests of Positive Blood Cultures and Their Characteristics
Detection Time
Assay Manufacturer/ Organisms/Antimicrobial
After Positive Growth
Technology Trade Name Resistance Targets
on Blood Culture
• AdvanDx/PNA 1.5 hr – PNA FISH
S. aureus, CoNS, E. faecalis, E. faecium, E. coli,
FISH, Traffic 20 min – QuickFISH
K. pneumoniae, P. aeruginosa, C. albicans,
PNA FISH Light – Not currently FDA
C. glabrata, C. parapsilosis, C. tropicalis,
• AdvanDx/ approved for Candida
C. krusei
QuickFISH spp.
• BD GeneOhm/
Staph SR
PCR S. aureus, CoNS/mecA (methicillin) 1–2 hr
• Cepheid/Xpert
MRSA
Enterococcus spp., L. monocytogenes
S. aureus, S. agalactiae, S. pneumoniae,
S. pyogenes/mecA (methicillin), vanA/B
(vancomycin)

PCR
• BioFire/ A. baumannii, H. influenzae, N. meningitidis, 1 hr – Direct from
FilmArray P. aeruginosa, E. cloacae, E. coli, K. oxytoca, blood culture media
K. pneumoniae, Proteus spp., S. marcescens/KPC
(carbapenem)
C. albicans, C. glabrata, C. krusei,
C. parapsilosis, C. tropicalis

ACCP Updates in Therapeutics® 2022: Critical Care Pharmacy Preparatory Review and Recertification Course

253
Infectious Diseases II

Table 5. Examples of Rapid Diagnostic Tests of Positive Blood Cultures and Their Characteristicsa (continued)
Detection Time
Assay Manufacturer/ Organisms/Antimicrobial
After Positive Growth
Technology Trade Name Resistance Targets
on Blood Culture
• BioMérieux/
Several pathogens, including bacteria, yeast,
Vitek MS
MALDI-TOF mold, mycobacteria/resistance mechanisms are in 10–30 min
• Bruker/
development
Microflex
S. aureus, S. epidermidis, S. lugdunensis,
S. anginosus, S. agalactiae, S. pneumoniae,
S. pyogenes, E. faecalis, E. faecium,
Staphylococcus spp., Streptococcus spp.,
Micrococcus spp., Listeria spp./mecA (methicillin),
vanA (vancomycin), vanB (vancomycin)
• Nanosphere/ 2 hr – Direct from
Nanoparticles E. coli, K. pneumoniae, K. oxytoca,
Verigene blood culture media
P. aeruginosa, S. marcescens, Acinetobacter spp.,
Citrobacter spp., Enterobacter spp., Proteus spp./
CTX-M (ESBL), IMP (carbapenemase), KPC
(carbapenemase), NDM (carbapenemase), OXA
(carbapenemase), VIM (carbapenemase)
Candida spp. in development
S. aureus, MRSA, Enterococcus spp.,VRE,
S. agalactiae, E. coli (Shiga toxin, EC-O157),
Chromogenic Klebsiella spp., Proteus spp., Pseudomonas
media
• CHROMagar spp., Acinetobacter spp., Yersinia spp.; KPC 24–48 hr
(carbapenem), CTX-M (ESBL)
C. albicans, C. tropicalis, C. krusei
ESBL = extended-spectrum β-lactamase; KPC = Klebsiella pneumoniae carbapenemase; MALDI-TOF = matrix-assisted laser desorption-ionization/time of flight;
MRSA = methicillin-resistant S. aureus; NDM = New Delhi metallo-β-lactamase; PCR = polymerase chain reaction; PNA FISH = peptide nucleic acid fluorescent in
situ hybridization; VRE = vancomycin-resistant enterococci.
Reproduced with permission from: Goff DA, Jankowski C, Tenover F. Using rapid diagnostic tests to optimize antimicrobial selection in antimicrobial stewardship
programs. Pharmacotherapy 2012;32:677-87.

Table 6. Reasonable Empiric Treatment Approach Associated with Detection of Antimicrobial Resistance Encoding
Genes from Rapid Diagnostic Tests for Positive Blood Culturesa
Resistance Genes Species Recommended Therapy Alternative Therapy
Oxacillin
Vancomycin (if intolerance to
mecA-negative S. aureus Nafcillin
β-lactams)
Cefazolin
Daptomycin
mecA-positive S. aureus Vancomycin Linezolid
Ceftaroline
vanA/B-negative E. faecalis Ampicillin Vancomycin (if allergic to β-lactams)
vanA/B-positive E. faecalis Daptomycin Linezolid

ACCP Updates in Therapeutics® 2022: Critical Care Pharmacy Preparatory Review and Recertification Course

254
Infectious Diseases II

Table 6. Reasonable Empiric Treatment Approach Associated with Detection of Antimicrobial Resistance Encoding
Genes from Rapid Diagnostic Tests for Positive Blood Culturesa (continued)
Resistance Genes Species Recommended Therapy Alternative Therapy
Daptomycin (if intolerance to
vanA/B-negative E. faecium Vancomycin
vancomycin)
vanA/B-positive E. faecium Daptomycin Linezolid
Enteric gram-negative Carbapenems (ertapenem,
CTX-M (ESBL) a Ceftazidime/avibactam
pathogens imipenem, meropenem)
K. pneumoniae Colistin
Ceftazidime/avibactam
KPC a
and other enteric gram- Tigecycline
Meropenem/vaborbactam
negative pathogens Imipenem/cilastatin/relebactam
K. pneumoniae
NDM Colistin Cefiderocol
and other enteric gram-
carbapenemasea Tigecycline Aztreonam + Ceftazidime/Avibactam
negative pathogens
VIM or IMP Colistin
P. aeruginosa
carbapenemasea Aztreonam
Colistin
OXA β-lactamasea P. aeruginosa
Carbapenemb
Colistin
Tigecycline
OXA β-lactamasea A. baumannii Cefiderocol
Carbapenemb
Minocycline
a
Resistance mechanisms associated with gram-negative pathogens are more complicated. Often, there is more than one resistance mechanism. Hence, the recommended
therapies represent reasonable empiric approaches before full antimicrobial susceptibility reports. However, therapy may be tailored to either a broader or a narrower
spectrum.
b
Carbapenem may have a higher MIC in the presence of OXA β-lactamase. If using carbapenem, may consider using maximal doses.

Patient Case

3. A 54-year-old man is admitted to the medical ICU with acute necrotizing pancreatitis. He is found to have
an infected pancreatic abscess, which is being treated with meropenem and vancomycin. Two weeks into the
course, the patient develops a fever and leukocytosis. He also becomes hemodynamically unstable and oligu-
ric. He is initiated on caspofungin empirically to cover for possible invasive candidiasis. Blood cultures are
obtained, which become positive with a preliminary result of yeast. This institution is equipped with PNA
FISH technology, which identifies the yeast as Candida parapsilosis on day 2 of therapy. The patient remains
hemodynamically unstable. Which is the most appropriate choice for this patient’s antifungal therapy?
A. Continue caspofungin while fungal susceptibilities are finalized.
B. Change to voriconazole 6 mg/kg every 12 hours.
C. Change to liposomal amphotericin 5 mg/kg/day.
D. Change to fluconazole 400 mg intravenously daily.

ACCP Updates in Therapeutics® 2022: Critical Care Pharmacy Preparatory Review and Recertification Course

255
Infectious Diseases II

C. Early C. difficile Identification: Nucleic acid amplification methods (PCR and loop-mediated isothermal
amplification)
1. Assays are targeting DNA sequence for the toxin A or toxin B gene.
2. Turnaround time: 1–3 hours
3. Sensitivity and specificity: 90%–96%
a. Because of increased sensitivity, the false-positive rates may be increased.
b. As the prevalence of C. difficile decreases, the positive predictive value decreases, which may lead
to unnecessary overtreatment.
c. Educational efforts should be made to discourage the practice of over-ordering C. difficile rapid
nucleic acid amplification tests. In addition, clinicians should be discouraged from ordering serial
tests, which was a common practice when enzyme immunoassays were used. Because the sensitivity
is sufficiently high, serial ordering only furthers the chance of false positivity.

D. Early Identification of Fungal Organisms


1. Up to 50% of patients with histopathologically proven invasive candidiasis have negative blood cultures.
2. Cultures for Aspergillus lack sensitivity and may require significant growth time.
3. Biopsies are often not feasible in critically ill patients.
4. Surrogate markers of fungal infections are needed for early diagnosis and to avoid delays in treatment,
which have been associated with worsened outcomes.
5. β-d-Glucan
a. Found in the cell membrane of most fungal pathogens (except for Mucor and Cryptococcus).
Potentially a screening tool for Candida, Aspergillus, and Pneumocystis.
b. Fungitell assay can detect β-d-glucan in serum and provide results within 2 hours.
c. Variable cutoffs are described in the literature for a positive result. A suggested cutoff for positive
results, as used in a recent study evaluating prophylactic and preemptive antifungal therapy in
critically ill patients, is greater than 80 pg/mL.
d. Considerable differences in sensitivity and specificity values have been seen in different populations.
e. Consistently high negative predictive values and low positive predictive values have been seen for
the diagnosis of invasive fungal infections, which suggests that it is best used as a screening-out tool
when the values are low. Low positive predictive value may occur because many clinical scenarios
could lead to false-positive results.
i. Exposure to β-d-glucan–containing surgical supplies and topical products
ii. Colonization with Candida
iii. Thrush and mucositis
iv. Cellulose membranes from dialysis filters
v. Bacterial infections
vi. Receipt of β-lactam antibiotics, albumin, or immunoglobulins
f. When used as a tool to potentially initiate treatment, optimal results occur when two consecutive
tests are positive.
g. Nonspecific fungal element makes interpretation difficult. May detect elements from Aspergillus,
Candida, and Pneumocystis. Does not detect different species.
h. Has been evaluated in combination with clinical prediction scores for invasive candidiasis in
critically ill patients for initiating empiric antifungal therapy. When using β-d-glucan as a screening
tool, empiric echinocandin therapy does not result in any significant differences in clinical outcomes
compared with placebo.

ACCP Updates in Therapeutics® 2022: Critical Care Pharmacy Preparatory Review and Recertification Course

256
Infectious Diseases II

6. T2 magnetic resonance
a. Nanodiagnostic approach, which detects amplified Candida DNA. Similar to traditional MRI
techniques, but done on a micro scale.
b. Performed on whole blood, where magnetic particles that are coated with agents specific for binding
to Candida DNA.
c. When Candida is present in whole blood, it will bind to the particles and cluster, causing microscopic
disruptions in the magnetic fields.
d. Currently, FDA approved for 5 pathogenic Candida species (C. albicans, C. glabrata, C.
parapsillosis, C. tropicalis, C. kruseii)
e. High sensitivity (91%) and specificity (98%) but lacks clinical study application
7. Galactomannan
a. Cell wall component of Aspergillus spp.
b. Platelia enzyme immunoassay can detect galactomannan in serum or other sterile fluids
(bronchoalveolar lavage) within 4 hours.
c. Variable sensitivity and specificity, with generally better positive and negative predictive values for
the detection of Aspergillus in the patient population with hematologic malignancies than in the
solid organ transplant (SOT) population
d. False positives can occur with the simultaneous administration of certain β-lactam antibiotics
(piperacillin/tazobactam, amoxicillin/clavulanic acid) or the presence of other invasive mycoses
(Penicillium, histoplasmosis, blastomycosis).
8. Other Candida early diagnostic tools such as PCR, mannan, and d-arabitol require additional investigation
before wide clinical adaptation.

E. Procalcitonin (PCT)
1. An inflammatory biomarker that reflects host response to bacterial infections
2. PCT synthesis is up-regulated by bacterial toxins and certain bacterial proinflammatory mediators such
as interleukin (IL)-1b, IL-6, and tumor necrosis factor alpha (TNFα), but it is neutral to cytokines
that are normally released for viral infections such as interferon-γ. Usual concentrations of PCT are
undetectable (less than 0.05 mcg/L). However, on exposure to bacterial toxins, PCT is rapidly released
within 2–4 hours. The plasma half-life of PCT is 24 hours. Concentrations in the literature for infected
patients vary greatly; however, it appears that a higher max concentration of PCT during infection
correlates with a higher incidence of mortality.
3. PCT is used in many roles, including the diagnosis and prognostication for sepsis. IDSA stewardship
guidelines recommend serial measurements be used in conjunction with other stewardship interventions
to decrease antibiotics use. The Surviving Sepsis Campaign provides a weak recommendation (weak
recommendation, low quality of evidence) for the use of low PCT to assist clinicians in the discontinuation
of empiric antibiotics when no evidence of infection is found among patients with sepsis.
4. For clinical decisions regarding antibiotic use and duration, PCT has been evaluated for antibiotic
initiation, antibiotic cessation, and the combination of both strategies. These strategies assume PCT
availability from an institutional laboratory. If the PCT turnaround time is more than 24 hours, the
effects of minimizing antimicrobial treatment days may be limited.
5. According to current evidence, PCT should not routinely be measured in patients without signs and
symptoms of infection. The decision to initiate patients on antibiotics without signs and symptoms of
infection using PCT alone would probably lead to antimicrobial overuse and possible adverse effects
associated with antimicrobial therapy. In a PCT study of critically ill patients, 1200 patients were
randomized to either a PCT alert strategy or a standard of care. For those randomized to intervention, a
PCT concentration greater than 1.0 mcg/L generated an alert that mandated clinical intervention, which
included microbiological cultures, additional radiologic assessment, and/or initiation or expansion of
antimicrobial coverage. Overall, this strategy did not lead to an improvement in mortality or time to
appropriate antibiotics. In contrast, patients experienced a greater need for mechanical ventilation,
prolonged ICU LOS, and prolonged antibiotic use.

ACCP Updates in Therapeutics® 2022: Critical Care Pharmacy Preparatory Review and Recertification Course

257
Infectious Diseases II

6. In critically ill patients with signs and symptoms of infection, a baseline PCT (at the time of the
symptoms) should not be used to determine whether antibiotics should be initiated. The compliance rate
for withholding antibiotics for a low PCT in this scenario has consistently been low. The compliance rate
in clinical practice is likely even lower than that in clinical studies; however, this has not been evaluated.
If a baseline PCT is obtained, it should be used to trend the PCT for the possible early discontinuation
of antibiotics. In a study of patients with signs and symptoms of infections to determine whether a
PCT-guided strategy would limit the initiation of antibiotics, no difference in antibiotic use was seen.
However, this was probably because only 36% of clinicians were compliant with the recommendation to
withhold antimicrobials when the PCT was low. This is in stark contrast with the 86% compliance rate
with the recommendation to initiate antibiotics when the PCT was high.
7. Critically ill patients with signs and symptoms of infection should have a baseline PCT obtained for
trending purposes. A low PCT (or substantial decrease from baseline) during antibiotic treatment should
be used to shorten the duration of antimicrobial therapy. This could be accomplished through either
eliminating unnecessary antibiotics in patients who are not infected or shortening the course of therapy
for patients who are infected. This strategy has been proved safe and effective in a wide spectrum of
critically ill patients. Several studies have evaluated the utility of a PCT-guided strategy for determining
the appropriate time to discontinue and/or de-escalate antibiotics. These studies consistently show that
PCT guidance for discontinuing antimicrobial therapy led to decreases in antibiotic use without an
untoward outcome effect. This has been shown in various ICU populations, in patients with differing
severity of illness, and in those with proven infections. The largest PCT study of critically ill patients
(n=1575) was published in 2016. It demonstrated a significant decrease in the use of antimicrobials
(median 5 days vs. 7 days) and 28-day mortality (20% vs. 25%; p=0.012). A recent meta-analysis
showed that use of PCT to guide discontinuation of antimicrobial therapy may have a short-term
mortality benefit.
8. A recent prospective study showed that the inability to decrease PCT by at least 80% by day 4 was an
independent predictor of mortality.
9. Many different PCT guidance algorithms exist; Figure 2 represents a reasonable approach to using
PCT for antibiotic cessation. In certain scenarios, PCT concentrations may be affected. For example,
some clinicians recommend using different cutoffs (e.g., greater than 0.5 mcg/L) in patients with stage
5 chronic kidney disease or requiring dialysis. Furthermore, in some studies, the PCT cutoff is greater
than 1.0 mcg/L among patients with recent surgery.
10. Procalcitonin levels have been elevated in patients with COVID-19, although interpretation is challenging
as this could represent bacterial co-infection, could be a marker of severity of acute respiratory distress
syndrome, or respiratory failure could cause immune dysregulation that increases the production of
cytokines. Due to the unclear cause of elevated PCT in COVID-19, it is not advisable to use the PCT
level by itself to guide antibiotic therapy.

ACCP Updates in Therapeutics® 2022: Critical Care Pharmacy Preparatory Review and Recertification Course

258
Infectious Diseases II

Patient with signs and symptoms of infection

Initiate appropriate empiric antimicrobial therapy and obtain initial PCT

Day 3 of empiric ABX

Confirmed diagnosis of Overall, clinical suspicion for


Equivocal clinical suspicion
infection through clinical and infections is low, given clinical
of infection
microbiological criteria resolution and negative cultures

Obtain PCT and use results to


Continue appropriate ABX,
guide therapy. If PCT suggests
de-escalate as appropriate, and
continuing empiric ABX, consider Stop empiric ABX
consider obtaining serial PCT
obtaining serial PCT daily to
to determine therapy duration
determine therapy duration

PCT < 0.25 mcg/L Decrease by ≥ 80% Decrease by < 80% Increase in PCT
from peak PCT, or from peak PCT, and compared with peak
PCT ≥ 0.25 and PCT ≥ 0.5 mcg/L concentration and
< 0.5 mcg/L PCT ≥ 0.5 mcg/L

Strongly encourage
Strongly encourage Encourage stopping Encourage continuing continuing or
stopping ABX ABX ABX escalating ABX

Figure 2. Sample procalcitonin-guided antimicrobial management algorithm.


ABX = antibiotics; PCT = procalcitonin.

ACCP Updates in Therapeutics® 2022: Critical Care Pharmacy Preparatory Review and Recertification Course

259
Infectious Diseases II

Patient Case

4. A 64-year-old woman is admitted to the medical ICU with possible community-acquired pneumonia. The
patient is initiated on ceftriaxone and azithromycin. Chest radiography reveals focal infiltrate. On admission,
she is dyspneic with a respiratory rate of 33 breaths/minute. Her vital signs and laboratory values are as
follows: blood pressure 90/50 mm Hg, heart rate 101 beats/minute, WBC 18 x 103 cells/mm3, and lactate 4.2
mmol/L. A PCT is obtained on admission. The results are available 12 hours after antibiotics are initiated.
The PCT result is 0.1 mcg/L. Which is the most appropriate option, given the PCT result?
A. Continue all current antibiotics.
B. Discontinue all antibiotics.
C. Escalate antibiotics to piperacillin/tazobactam.
D. Discontinue ceftriaxone only.

V.  INTERPRETING SUSCEPTIBILITY REPORTS

A. Rationale: Identifying microorganisms and antibiotic susceptibilities is integral to the care of critically
ill patients with infections. An understanding of the microbiology laboratory methods for pathogen
identification and susceptibility testing will further equip critical care pharmacists with the ability to use the
most appropriate antimicrobial regimens for the treatment of critical care–associated infections. Standards
for antimicrobial identification, sensitivity testing, and determination of MIC breakpoints are determined
by the Clinical & Laboratory Standards Institute (CLSI). CLSI is a volunteer-driven standards development
organization that promotes the development and use of laboratory consensus standards and guidelines within
the healthcare community.

B. Methods for Antimicrobial Susceptibility Testing (AST)


1. Disk diffusion (Kirby-Bauer)
a. Test agar plates are swabbed with a standardized concentration of the test organisms; paper disks
containing a defined antibiotic concentration are then placed on the plates. The diameter of the zone
of inhibited growth is inversely proportional to the MIC value.
b. Usually reported qualitatively as sensitive, intermediate, or resistant
2. Dilution methods (i.e., broth dilution, agar dilution)
a. Two-fold serial dilutions of antibiotics are prepared (in either broth or agar), which are then
inoculated with a standardized concentration of test organisms.
b. The MIC is determined.
c. Precision of this method is within one 2-fold concentration in either direction.
d. Many prefabricated broth microdilution plates are commercially available.
3. E-test
a. A calibrated plastic strip with a range of continuous MIC values (with 15 2-fold dilutions) is placed
on an inoculated agar plate. The MIC can be determined by the intersection between the zone of
inhibition and the edge of the calibrated strip.
b. Complement other systems to determine the MIC for specific antibiotics and test for resistance and
for confirmatory testing

ACCP Updates in Therapeutics® 2022: Critical Care Pharmacy Preparatory Review and Recertification Course

260
Infectious Diseases II

c. Studies have shown that the reading of the exact MIC on an E-test strip is subject to the user’s
interpretation and could lead to a one- or two-dilution error in either direction compared with
standardized methods.
4. Automated systems (e.g., Vitek, Microscan, Sensititre, Phoenix)
a. Uses computerized algorithms for interpreting MIC values
b. Can usually perform AST more quickly than traditional methods
c. More than 80% of all clinical microbiology laboratories report using an automated system as their
primary method of susceptibility testing.
d. May be unable to detect certain resistance mechanisms (i.e., inducible enzymes)
e. Standard inoculums of pathogens are used. For infections in which the in vivo inoculum may be
higher, certain antimicrobials may have higher MIC values.
5. Specific confirmatory tests for antimicrobial resistance
a. Macrolide-lincosamide-streptogramin resistance
i. Strains of Staphylococcus spp. can have a transferable resistance mechanism called macrolide-
lincosamide-streptogramin, which is inducible by clindamycin and can lead to treatment failure.
ii. Inducible resistance is not detected by routine AST.
iii. Detected using double-disk diffusion
b. ESBL
i. CLSI procedures exist for K. pneumoniae, Klebsiella oxytoca, E. coli, and Proteus mirabilis.
ii. Initial screening with susceptibility testing for ceftazidime, aztreonam, cefotaxime, or
ceftriaxone
iii. Confirmatory testing with broth microdilution, disk diffusion, or E-test strips. Presence of
ESBL is confirmed if adding clavulanic acid results in three 2-fold dilution decreases in the
paired-cephalosporin MIC.
iv. Beginning in 2010, CLSI adopted lower MIC breakpoints for many cephalosporins for
Enterobacterales (see Table 7). The rationale for this change is that the lower MIC may
more readily detect the presence of ESBL, hence eliminating the need for the labor-intensive
confirmatory tests. Compliance with this new practice is variable among clinical microbiology
laboratories. Clinicians should inquire within their local clinical microbiology laboratory
regarding whether the new standards have been adopted. If a microbiology laboratory
has adopted the new CLSI recommendations for ESBL detection, confirmatory tests are
unnecessary.

Table 7. Changes to CLSI Enterobacterales Breakpoints with 2020 Updates


Agent Pre-2010 CLSI Breakpoints 2020 CLSI Breakpoints
S I R S I R
Cefazolina ≤8 16 ≥ 32 ≤2 4 ≥8
Cefazolinb ≤8 16 ≥ 32 ≤ 16 — ≥ 32
Cefotaxime ≤8 16–32 ≥ 64 ≤1 2 ≥4
Ceftriaxone ≤8 16–32 ≥ 64 ≤1 2 ≥4
Ceftazidime ≤8 16 ≥ 32 ≤4 8 ≥ 16
Aztreonam ≤8 16 ≥ 32 ≤4 8 ≥ 16
Cefazolin used as therapy for infections other than uncomplicated UTIs.
a

Cefazolin used as therapy for uncomplicated UTIs.


CLSI = Clinical & Laboratory Standards Institute; UTI = urinary tract infection.

ACCP Updates in Therapeutics® 2022: Critical Care Pharmacy Preparatory Review and Recertification Course

261
Infectious Diseases II

c. AmpC β-Lactamase
i. Inhibitor-resistant β-lactamase (e.g., clavulanic acid, tazobactam)
ii. Maintains sensitivity with cephamycins (e.g., cefotetan, cefoxitin) and cloxacillin
iii. Confirmatory test with E-test containing cephamycin alone or a combination of cephamycin
and cloxacillin. If the MIC is decreased by more than three 2-fold dilutions, the presence of
AmpC β-lactamase is confirmed.
d. Carbapenemase
i. Metallo-β-lactamase (e.g., New Delhi metallo-β-lactamase [NDM]; Verona integrin-encoded
metallo-β-lactamase [VIM]); IMP-type carbapenemase)
(a) Hydrolyzes both β-lactam and carbapenem antibiotics, but aztreonam maintains
susceptibility. Clinically, about 80% of metallo-β-lactamase–containing pathogens are
resistant to aztreonam because of other resistance mechanisms.
(b) Zinc-mediated metallo-β-lactamase can be repressed by ethylene-diamine tetraacetic acid
(EDTA). Reduction in imipenem MIC by more than three 2-fold dilutions in the presence
of EDTA confirms presence of zinc-mediated metallo-β-lactamase.
ii. KPC
(a) KPC enzyme can produce a slightly higher MIC that may still be in the range considered
sensitive. This is in contrast to other carbapenem-resistant mechanisms in Pseudomonas
and Acinetobacter, which generally produce a fully resistant MIC. Hence, a carbapenem
MIC of 1 mcg/mL or greater in an Enterobacterales should be evaluated through further
confirmatory testing.
(b) Modified Hodge test should be performed for pathogens with an elevated carbapenem
MIC.
(1) Plate standard sensitive E. coli strain on entire plate.
(2) Place meropenem or carbapenem disk in center of test area.
(3) Streak test organism in a straight line from edge of carbapenem disk to edge of plate
(up to four different organisms can be tested).
(4) Positive carbapenemase results from modified Hodge test have a clover leaf–like
indentation of the E. coli growing along the test organism’s streak.
(c) Starting in 2010, CLSI lowered the susceptibility breakpoints for carbapenem (0.5 mcg/
mL and 1.0 mcg/mL or less for ertapenem and other carbapenems, respectively), with the
intent of eliminating the need for confirmatory tests.
(1) Clinicians should inquire about the practices of their local microbiology laboratory
regarding carbapenem susceptibility.
(2) Particularly when confirmatory tests are not being performed, and a laboratory
continues to use the older MIC breakpoints, a higher clinical suspicion for
carbapenemase is warranted.
(3) Ertapenem resistance seen on AST is a sensitive marker for the presence of
carbapenemase. Before the 2010 reclassification of carbapenem susceptibility
breakpoints, there were reports of up to 46% of clinical isolates with genotypic evidence
of KPC-producing enzymes being inadvertently labeled as imipenem sensitive.
(4) Deleterious outcomes associated with the use of carbapenems in carbapenemase-
producing organisms have been reported.

ACCP Updates in Therapeutics® 2022: Critical Care Pharmacy Preparatory Review and Recertification Course

262
Infectious Diseases II

C. MIC Breakpoints: Determining the correct MIC breakpoint by the governing standards is a complicated
process. CLSI promotes the development of voluntary consensus standards within the medical community.
Determining CLSI breakpoints could be based on several conflicting interests, which may include some of
the following:
1. Microbiological: MIC that distinguishes wild-type bacteria from those that have acquired additional
resistance mechanisms
2. PK/PD: Derived from human or animal data and modeled to determine the likelihood that standard
prescribed doses will meet specified PD criteria for suppressing bacterial growth
3. Breakpoints that may decrease the need for confirmatory tests, hence decreasing the microbiology
laboratory workload. Such an approach, although possibly sensitive for determining the presence of
resistance mechanisms, may potentially lead to the compensatory use of broader antimicrobials.
4. The CLSI breakpoints may be inconsistent with the breakpoints established by the FDA. Automated
systems report MIC breakpoints according to FDA approvals. Changes to reporting and labeling require
additional clearance from the FDA. Unfortunately, the FDA labeling may not be up to date with the
current CLSI recommendations. Hence, despite newer CLSI recommendations, many institutions may
still be reporting susceptibility results that are based on their respective FDA labeling breakpoints.
These breakpoints may result in pathogens being labeled as sensitive, even though, according to the
current CLSI standards, they would be considered resistant.
5. The exact MIC breakpoints that are used by a local microbiology laboratory can be variable, depending
on the laboratory’s adaptation of new standards. In addition, there are significant variations in the FDA-
approved breakpoints used by automated AST methods and updated CLSI guidelines. Once again, this
highlights the importance of a clinician’s understanding of the exact methods and breakpoints that are
used in his or her institution.

D. Minimum recommended information that critical care pharmacists should recognize about their local
microbiology laboratory practices
1. Routine methods for AST
2. Availability of rapid diagnostic tests for speciation or resistance gene identification
3. If additional tests are available (E-test, genotyping, etc.) and how to go about requesting additional
information
4. Are confirmatory tests for resistance mechanisms routinely done (e.g., double-disk diffusion for
macrolide-lincosamide-streptogramin–inducible resistance, ESBL and KPC confirmatory tests)?
5. What is the usual procedure for censoring the AST results?
a. Does the AST result try to guide clinical decision (i.e., if pathogen were sensitive to oxacillin, would
vancomycin results be censored)?
b. If certain resistance mechanisms are detected, would the laboratory automatically update
susceptibility results (i.e., if ESBL were detected, would β-lactam/β-lactamase combination
antibiotics automatically be changed to resistant)?
c. Are any antibiotics part of the standard panel but routinely hidden from clinical reports?
d. Most microbiology laboratories would retain all the information, even if parts of the results had
been censored. Critical care pharmacists should consider contacting the microbiology laboratories
to see whether additional information regarding the pathogen is available.
6. When an MIC is reported, is only the MIC breakpoint reported (i.e., MIC 2 mcg/mL or less), or is the
actual MIC reported (i.e., 0.25 mcg/mL)?
7. Are the breakpoints reported consistent with the updated CLSI recommendations?

ACCP Updates in Therapeutics® 2022: Critical Care Pharmacy Preparatory Review and Recertification Course

263
Infectious Diseases II

E. Clinical Application of Interpreting AST


1. The ideal application of AST interpretation and clinical therapeutics may include the following:
a. Knowledge of the individual pathogen and the corresponding MIC breakpoints for each of the
tested antibiotics
b. Potential reasoning behind each of the clinical breakpoint designations (i.e., wild-type selection,
PK/PD breakpoints assessment, screening for resistance)
c. Accurate comprehension of the local AST methods used and potential limitations of the MIC
reported
d. Understanding of the tested antimicrobial’s PD parameters, which conveys the highest likelihood
for eradication of the microorganism
e. Selecting the most appropriate antibiotic according to an assessment of the following:
i. What genotypic resistance mechanisms may be suggested by the phenotypic representation of
the AST?
ii. How may the underlying resistance mechanisms affect the choice of antimicrobial therapy?
iii. The PK characteristics of the medication and patient characteristics that may affect the
antimicrobial’s PK
iv. The likelihood that the considered antimicrobial agent will reach its PD targets for optimal
therapy at the local site of infection
v. Selecting the antimicrobial according to a balance of antimicrobial stewardship principles,
adverse effects profile, allergies, and cost
2. When an antimicrobial must be chosen despite its having a higher-than-ideal MIC, the penetration into
the site of infection is questionable, or the correlation of PK and PD target attainment is not available,
additional interventions may be required. Optimization of therapy should be tried through one or more
of the following: using a higher dose, using administration strategies that may optimize PK/PD (i.e.,
extended- or continuous-infusion β-lactams), or combination therapy.

VI.  MECHANISMS OF ANTIBACTERIAL RESISTANCE AND TREATMENT OF MULTIDRUG-


RESISTANT PATHOGENS

A. Epidemiology and Clinical Significance: The WHO has identified antimicrobial resistance as one of the
three greatest threats to human health.
1. The National Healthcare Safety Network for the CDC reports recent resistance rates among commonly
encountered pathogens. More than 3000 U.S. hospitals participate in this national surveillance program,
with as many as 100,000–300,000 pathogens reported and about 70% of the reporting from critical care
sites. See Table 8 for selected resistance rates in different hospital-acquired infections.
2. Resistance rates continue to be high, with trends toward an increase in certain pathogens compared with
the reports from previous years.
3. Resistant pathogens consistently correlate with worse clinical outcomes, which is partially explained by
the higher likelihood of empiric treatment with a resistant antibiotic.
4. The antibiotic pipeline has slowed down considerably, with consistent decreases in FDA approvals of
antimicrobial agents during the past 3 decades. The combination of prevailing resistance, including the
emergence of pan-resistant pathogens, with the lack of new antimicrobial agents presents a potential
global health problem.
5. An understanding of resistance mechanisms would assist the ICU clinician in effectively treating
current resistant pathogens while incorporating antimicrobial stewardship principles to prevent further
resistance. See Table 9 for common mechanisms of resistance.

ACCP Updates in Therapeutics® 2022: Critical Care Pharmacy Preparatory Review and Recertification Course

264
Infectious Diseases II

Table 8. Recent Resistance Rates in Hospital-Acquired Infection (2015–2017)


CLABSI % CAUTI % VAP % SSI %
Pathogen
Resistance Resistance Resistance Resistance
Gram-negative Bacteria
E. coli n=1129 n=14,390 n=520 n=15,302
Third-generation cephalosporins 29.8 16.0 23.8 18.5
Carbapenem 2.4 0.3 1.0 0.6
Fluoroquinolone 43.2 32.7 32.7 34.7
Multidrug resistanta 14.4 6.9 11.8 7.8
K. pneumoniae/oxytoca n=1708 n=5775 n=936 n=4591
Third-generation cephalosporins 29.6 16.6 14.3 14.3
Carbapenem 11.0 5.1 5.4 3.5
Multidrug resistanta 20.2 9.6 8.7 6.4
P. aeruginosa n=1061 n=5047 n=1192 n=4405
Aminoglycosides 14.9 15.3 13.6 6.8
Ceftazidime or cefepime 26.5 18.6 25.9 12.3
Fluoroquinolones 27.1 24.9 26.7 12.3
Carbapenem 26.3 19.7 26.3 11.2
Piperacillin/tazobactam 20.2 13.8 21.7 9.6
Multidrug resistanta 18.6 13.6 16.8 5.5
Enterobacter spp. n=1078 n=2120 n=781 n=3471
Third-generation cephalosporins 12.2 12.6 6.7 7.3
Carbapenem 7.2 6.5 3.5 3.7
Multidrug resistanta 7.7 6.2 0.8 2.6
A. baumannii n=392 n=197 n=294 n=118
Carbapenem 47.2 52.8 41.3 32.3
Multidrug resistant 46.6 53.1 37.5 33.6
Gram-positive Bacteria
S. aureus n=2497 n=655 n=2673 n=5909
Oxacillin 50.0 40.5 36.9 55.8
E. faecalis n=2117 n=3881 n=7584
NR
Vancomycin 8.5 4.2 3.4
E. faecium n=1981 n=1034 n=3584
NR
Vancomycin 89.5 82.8 53.1
a
Multidrug resistant is defined as being either intermediate or resistant to at least one drug in three of the following five classes: third- and fourth-generation
cephalosporins, fluoroquinolones, aminoglycosides, carbapenems, and piperacillin and/or piperacillin/tazobactam (definition according to CDC National Healthcare
Safety Network).
CAUTI = catheter-associated urinary tract infection; CLABSI = central line–associated bloodstream infection; NR = not reported; SSI = surgical site infection; VAP =
ventilator-associated pneumonia.

ACCP Updates in Therapeutics® 2022: Critical Care Pharmacy Preparatory Review and Recertification Course

265
Infectious Diseases II

Table 9. Common Resistance Mechanisms


Antibacterial Agent Mechanism of Resistance
β-Lactamases (AmpC, ESBL, KPC)
Reduced permeability
β-Lactams
Altered penicillin binding proteins
Increased efflux
Altered DNA gyrase and topoisomerase
Fluoroquinolones Increased efflux
Decreased protein targets
Increased efflux
Macrolides
Methylating enzymes
Aminoglycoside-modifying enzymes
Aminoglycosides Increased efflux
Modification of target proteins
Altered target
Glycopeptides
Decreased permeability
Increased efflux
Tetracyclines
Protection of target proteins
Increased efflux
Trimethoprim
Altered dihydrofolate reductase
Rifamycin Altered RNA polymerase

B. Mechanisms of Resistance (bacteria often possess several mechanisms)


1. Decreased permeability (i.e., porin loss, thickened cell wall)
2. Increased efflux (i.e., macrolide efflux pump)
3. Target modification (i.e., alteration in penicillin-binding proteins)
4. Hydrolysis (i.e., β-lactamases, aminoglycoside-modifying enzymes)

C. Factors Associated with Resistance Acquisition


1. Crowding of patients with high levels of disease acuity and/or antimicrobial use
2. Prolonged hospital LOS
3. Colonization pressure: Proportions of people colonized with resistant bacteria. Combated by strict
compliance with infection control procedures to prevent colonization, adequate nurse staffing ratios,
and hand hygiene.
4. Use of invasive devices (endotracheal tubes, intravascular catheters, and urinary catheters)
5. Previous or prolonged use of antibiotics

D. Clinical Approach to Common Pathogen Resistance Seen in Critically Ill Patients


1. ESBL
a. Confers resistance to third-generation cephalosporins and aztreonam
b. Found primarily in E. coli and K. pneumoniae spp. but can also be seen in other Enterobacterales
Carbapenems should be considered the drug of choice in severe infections.
c. Non–β-lactams could be used if they showed sensitivity on AST; however, because ESBLs are
usually plasmid mediated, there are often other acquired resistance mechanisms. The rates of cross-
resistance to other classes of antibiotics are 55%–100%.

ACCP Updates in Therapeutics® 2022: Critical Care Pharmacy Preparatory Review and Recertification Course

266
Infectious Diseases II

d. β-lactam/β-lactamase inhibitors and cefepime often have in vitro activity, though clinical failures
have been reported.
i. A previous post hoc analysis found that use of β-lactam/β-lactamase inhibitors was not associated
with worse outcomes for ESBL-producing E. coli bacteremia compared with carbapenems.
ii. However, a recent prospective randomized controlled noninferiority study evaluating definitive
treatment of ceftriaxone-resistant E. coli or K. pneumoniae found increased mortality with
piperacillin/tazobactam compared with meropenem (12.3 vs. 3.7%). These results do not
support the use of piperacillin/tazobactam in the treatment of ESBL-producing organisms.
iii. Data are conflicting regarding the use of cefepime for treating ESBL infections. Some studies
show worse outcomes, whereas others show no difference compared with carbapenems. This
may partly be explained by the cefepime MIC distribution. Traditionally (before 2014), the
susceptibility breakpoint for cefepime for Enterobacterales was 8 mcg/mL or less. However,
in 2014, CLSI recommended decreasing the sensitive cefepime MIC breakpoint to 2 mcg/mL,
in addition to a new category for sensitive dose-dependent, where maximal doses of cefepime
are recommended for MICs of 4 and 8 mcg/mL. Hence, for many years, cefepime may have
been used in ESBL infections when the MIC was 4–8 mcg/mL, but doses were not optimized.
iv. Of interest, a recent investigation correlated cefepime MIC to ESBL Enterobacterales with
mortality, where a cefepime MIC of 1 mcg/mL or less was associated with significantly lower
mortality compared with other MIC values. Together, given the conflicting clinical results, it
is difficult to endorse the use of cefepime for the treatment of ESBL infections. However, in a
stable patient with an ESBL infection having a cefepime MIC of 1 mcg/mL or less, cefepime
may be considered for consolidative therapy to minimize carbapenem use.
2. AmpC β-lactamases
a. Confer resistance to penicillins and narrow-spectrum cephalosporins
b. β-Lactamase inhibitor resistant; hence, tazobactam and clavulanic acid would not provide additional
coverage
c. Innate low-level production in many gram-negative bacteria (i.e., Enterobacter, Citrobacter, Serratia,
Morganella, Providencia, and Pseudomonas). This low-level production leads to resistance against
penicillin, ampicillin, and first-generation cephalosporins.
d. Genetic mutation leading to sustained high-level production is possible (derepressed mutants).
i. Theoretically, treatment with third-generation cephalosporins or broad-spectrum cephalosporins
for non-derepressed mutants may select species with the mutation. The actual incidence of
breakthrough infections is unknown with derepressed mutants when bacteria with innate
AmpC β-lactamase production are treated with third-generation cephalosporins or extended-
spectrum penicillins. However, it is believed to be low.
ii. Several epidemiologic studies have linked the use of third-generation cephalosporins with the
increase in pathogenic species of AmpC hyperproducing Enterobacterales
iii. Derepression confers resistance to third-generation cephalosporins and broad-spectrum
penicillins (piperacillin, ticarcillin).
iv. Commonly occurs in E. cloacae and Citrobacter freundii
v. Cefepime generally retains activity against derepressed mutants.
vi. Carbapenem and cefepime should be considered the drugs of choice in severe infections.
vii. Because derepression results from the mutation of a chromosomal-mediated β-lactamase, and
not from transmitted plasmids, many pathogens with this mutation maintain sensitivity to other
nonβ-lactam antimicrobials (e.g., fluoroquinolones, aminoglycosides).

ACCP Updates in Therapeutics® 2022: Critical Care Pharmacy Preparatory Review and Recertification Course

267
Infectious Diseases II

Patient Case

5. A 74-year-old man is admitted to the surgical ICU after elective hip replacement surgery. The patient, who
has a history of chronic pulmonary obstructive disease, cannot be weaned off the ventilator after surgery.
During the patient’s course, he develops signs and symptoms of infection. His vital signs and laboratory val-
ues are as follows: blood pressure 94/55 mm Hg, heart rate 114 beats/minute, temperature 101.9°F (38.8°C),
WBC 18 x 103 cells/mm3, and lactate 3.2 mmol/L. The patient is empirically initiated on piperacillin/tazo-
bactam and vancomycin and given 2 L of crystalloid fluids; pan cultures are sent. Urinalysis reveals pyuria,
positive leukocyte esterases, and nitrites. Blood and sputum cultures are negative, but urine culture shows E.
coli. The patient’s urinary catheter is removed, and vancomycin is discontinued. On day 3 of therapy, anti-
biotic susceptibility results are available. The patient’s E. coli is resistant to third-generation cephalosporins
with laboratory confirmation of the presence of ESBL. The laboratory reports the following antimicro-
bials and corresponding MIC values: piperacillin/tazobactam less than 2 mcg/mL – S; cefepime 4 mcg/
mL – SDD; imipenem 0.5 mcg/mL – S; and ciprofloxacin 0.25 mcg/mL – S. The patient’s vital signs and
laboratory values are as follows: blood pressure 110/70 mm Hg, heart rate 98 beats/minute, respiratory rate
30 breaths/minute, temperature 98.7°F (37.1°C), and WBC 9 x 103 cells/mm3. Which is the most appropriate
antibiotic option?
A. Change piperacillin/tazobactam to imipenem.
B. Continue piperacillin/tazobactam alone.
C. Change piperacillin/tazobactam to cefepime.
D. Add ciprofloxacin to piperacillin/tazobactam.

3. Carbapenemase
a. Seen in Acinetobacter, Pseudomonas, and Enterobacterales
b. The global spread of carbapenem resistance has become an epidemic.
c. The CDC continues to report that carbapenem-resistant Enterobacterales (CRE) is at an urgent
hazard level, where high consequence and probability for widespread public health concerns exist.
d. Confers resistance to most β-lactams, including carbapenems, cephalosporins, monobactam, and
broad-spectrum penicillins
e. Treatment options
i. Tigecycline:
(a) Glycylcycline antibiotic, which is structurally similar to tetracyclines
(b) Mechanism of action: Inhibition of 30s ribosomal subunit
(c) Spectrum of activity:
(1) Gram-positive bacteria: Enterococcus (including vancomycin-resistant enterococci),
Listeria, Staphylococcus (including MRSA/CoNS), Streptococcus
(2) Most gram-negative bacteria, including Acinetobacter, ESBL-producing Enterobacte-
rales, derepressed AmpC Enterobacterales, CRE, and Stenotrophomonas
(3) Anaerobes, including Bacteroides and Clostridium
(4) Atypicals
(5) Does not cover Pseudomonas, Providencia, Proteus, or Morganella
(d) PK
(1) Wide volume of distribution: 7–10 L/kg
(2) The intracellular distribution of tigecycline results in a decreased serum/tissue
concentration ratio. This has led many clinicians to state that tigecycline is not the
ideal drug for bloodstream infections. However, tigecycline has not been evaluated
exclusively for the treatment of bloodstream infections. In a pooled analysis of eight

ACCP Updates in Therapeutics® 2022: Critical Care Pharmacy Preparatory Review and Recertification Course

268
Infectious Diseases II

studies, patients with secondary bacteremia treated with tigecycline were compared
with patients treated with other antibiotics. Overall, no significant differences in
outcomes were seen. Despite these results, clinicians should exert caution with the use
of tigecycline for bacteremia and should reserve tigecycline for pathogens when no
other antibiotics are viable options.
(3) Primarily biliary elimination
(4) Urinary elimination is 8%–11%. The low urinary elimination limits tigecycline’s role
in treating UTIs. The use of tigecycline for treating multidrug-resistant UTIs has been
reported only in case reports, with most reports showing treatment success. However,
a recent evaluation of tigecycline for the treatment of KPC bacteriuria indicated a
correlation with the subsequent development of tigecycline resistance. Without further
data, tigecycline should not routinely be used for UTIs when other treatment options
are available.
(5) Poor penetration into lung epithelial lining fluid, which is in contrast to high penetration
into lung alveolar cells. This characteristic may partly explain the findings of a study
evaluating tigecycline compared with imipenem for the treatment of hospital-acquired
pneumonia. This study showed that tigecycline had a lower treatment success rate
than imipenem. The difference in treatment success was mainly attributable to the
significant differences in patients with ventilator-associated pneumonia. There was
also a trend toward increased mortality in the subgroup of patients with ventilator-
associated pneumonia.
(e) The FDA issued a safety warning in 2010 indicating a possible increased mortality risk
associated with the use of tigecycline compared with other drugs used to treat a variety
of other serious infections. This was compiled using several phase III studies in which
tigecycline had been proven noninferior to other standard treatments. Subsequently,
several other meta-analyses were published with conflicting results regarding tigecycline’s
increased mortality risk.
(f) Given tigecycline’s possible shortcomings, it seems prudent to avoid the routine use of
tigecycline in infections when other treatment options are available. However, tigecycline
may be one of the few remaining antibiotic options for treating carbapenem-resistant
pathogens. When tigecycline must be used because of limited treatment options, clinicians
should consider administering combination therapy with other agents that have in vitro
susceptibility.
(g) Resistance to tigecycline has been reported. Clinicians should seek confirmation from
their microbiology laboratory regarding tigecycline sensitivity. Of note, CLSI currently
has no recommendation for MIC breakpoint for tigecycline against Acinetobacter spp. The
MIC breakpoint for sensitivity against Enterobacterales is 2 mcg/mL or less.
ii. Polymyxins:
(a) Colistin (polymyxin E)
(1) Mechanism of action: A cationic cyclic decapeptide that functions by displacing calcium
and magnesium from the outer cell membrane, hence changing the permeability of the
cell membrane to allow insertion of the molecule into the cell membrane. Once the
molecule is inserted into the cell membrane, it disrupts the cell membrane integrity
and subsequently leads to cell death.
(2) Spectrum of activity: Covers only gram-negative bacteria, including CRE. Does not
cover Proteus, Providencia, Burkholderia, Serratia, or Stenotrophomonas
(3) First used in the United States in the 1960s but fell out of favor because of reports of
nephrotoxicity and neurotoxicity

ACCP Updates in Therapeutics® 2022: Critical Care Pharmacy Preparatory Review and Recertification Course

269
Infectious Diseases II

(4) Around 2000, with the emergence of CRE, colistin use was reconsidered, given the
lack of treatment options.
(5) Dosing challenges
(A) Different products provide different dosing recommendations.
(B) Colomycin injection is the brand primarily used in Europe. Dosed in international
units (IU).
(C) Coly-Mycin is the brand primarily used in the United States. Dosing in colistin-
based activity
(D) 3 million IU of colistin is equal to about 100 mg of colistin-based activity.
(6) PK/PD
(A) All colistin products, regardless of the dosing units, are administered as
colistimethate, which is a prodrug. Colistimethate is hydrolyzed to the active drug
colistin.
(B) Colistimethate is excreted through renal clearance, whereas the active drug
colistin is cleared by nonrenal pathways. Hence, renal dysfunction leads to a
higher portion of colistimethate being present for hydrolysis into colistin, thus
increasing the final active drug concentration.
(C) PD parameter for maximal activity is area under the curve/MIC ratio.
(D) In the 1960s, the dosing recommendations and PK/PD parameters were largely
unknown because the methods for evaluations were drastically different from the
current standards.
(E) Recent PK/PD evaluations show that traditional dosing methods are probably
insufficient to reach adequate serum concentrations to maximize PD target
attainment.
(F) Several studies have reported that a higher colistin dose is associated with
significant improvements in clinical outcomes.
(G) International consensus dosing recommendations
• 300 mg colistin base activity loading dose.
• 300-360 mg colistin base activity divided every 12 hours, with adjustments
made based on renal function / renal replacement therapy.
(H) Several ongoing investigations are evaluating the effects of high-dose colistin
regimens and the utility of using a loading dose.
• One study from South America is recruiting patients to evaluate the utility of a
200-mg loading dose.
• One study reported a 300-mg loading dose regimen, followed by 150 mg every
12 hours. This study was non-comparative, but it reported high rates of clinical
success.
(7) Because the PK/PD of colistin is unclear and little is known regarding the optimal
dosing regimen, colistin should be reserved for infections in which other treatment
options are not available. In such cases, it may be prudent to administer combination
therapy. Furthermore, many in vitro studies have shown synergy between colistin and
other antimicrobials.
(8) Resistance to colistin has been reported. In vitro studies also show that colistin
resistance develops quickly, which may be another rationale for providing combination
therapy. Clinicians should verify colistin susceptibility with their local microbiology
laboratory.
(A) Routine monitoring of colistin concentrations is currently infeasible.
(B) According to PK data, an MIC of 2 mcg/mL or less should be considered sensitive.

ACCP Updates in Therapeutics® 2022: Critical Care Pharmacy Preparatory Review and Recertification Course

270
Infectious Diseases II

(9) Empiric colistin therapy: In an analysis of the association between covering empiric
antibiotics and mortality for infections caused by carbapenem-resistant gram-
negative bacteria, the authors concluded that empiric use of colistin before pathogen
identification, with or without a carbapenem, was not associated with survival.
(b) Adverse reaction: Nephrotoxicity
(1) Nephrotoxicity
(A) 0%–45% reported in recent literature, depending on the definitions used for renal
dysfunction
(B) In general, seems to be dose-dependent
(C) Usually reversible, with few cases leading to a prolonged need for renal
replacement therapy
(2) Neurotoxicity
(A) Ranging from paresthesia to apnea
(B) Incidence of 8%–27% reported in historical studies
(C) Mentioned only in case reports in the current era of colistin use
(c) Polymyxin B:
(1) Mechanism of action, spectrum of activity, and adverse reactions similar to those of
colistin
(2) Available in the United States but not in Europe and Australia
(3) PK/PD
(A) Administered as an active drug
(B) PD targets similar to those of colistin
• Similar unknowns regarding best dosing regimen to achieve PD parameters
• Manufacturer-recommended dosing: 1.5–2.5 mg/kg/day divided every 12
hours. Recommends renal dose adjustment, but recent studies suggest minimal
renal clearance.
(C) International consensus dosing recommendations
• Loading dose of 2–2.5 mg/kg total body weight
• 1.25–1.5 mg/kg total body weight every 12 hours
(d) International consensus guidelines for the optimal use of polymyxins
(1) Use therapeutic drug monitoring, when possible
(2) Recommend hospitals have access to both polymyxins
(A) Polymyxin B preferred for routine systemic use because of superior and more
predictable PK
(B) Colistin preferred agent for UTIs
(C) Recent systematic review suggests that polymyxin B is associated with less acute
kidney injury than colistin.
iii. Ceftazidime/avibactam
(a) Avibactam is a new β-lactamase inhibitor approved by the FDA in 2015.
(b) Spectrum of activity: Broad gram-negative activity, including multidrug-resistant
Pseudomonas and Enterobacterales. Adding avibactam allows coverage against KPC-
producing bacteria, together with coverage against other β-lactamases (OXA, CTX-M,
AmpC). Minimal coverage against metallo-β-lactamase-expressing strains, Acinetobacter
spp., gram-positive, and anaerobes.
(c) FDA approved for the treatment of complicated intra-abdominal infection and complex
UTIs. However, most clinicians will reserve its coverage for difficult-to-treat pathogens
with minimal coverage options, such as KPC-producing Enterobacterales.
(d) An in vitro study that included 120 KPC-producing pathogens showed good activity with
ceftazidime/avibactam.

ACCP Updates in Therapeutics® 2022: Critical Care Pharmacy Preparatory Review and Recertification Course

271
Infectious Diseases II

(e) A recent multicenter-observational, propensity score-weighted study comparing


ceftazidime/avibactam with colistin as the initial treatment for CRE showed significant
improvements in mortality. These findings require confirmation in a randomized controlled
study.
(f) Study evaluating compassionate use of ceftazidime/avibactam as a second-line agent
compared with a matched cohort of patients with CRE bacteremia treated with a different
agent illustrated significant improvements in mortality (36.5% vs. 55.8%, p=0.005).
(g) No major adverse effects were seen with ceftazidime/avibactam in phase II and phase III
studies.
(h) Currently, the average wholesale price for ceftazidime/avibactam is about $1000 per day
with normal dosing. This is similar to other meropenem/vaborbactam but considerably
more expensive than other β-lactams.
iv. Meropenem/vaborbactam
(a) Vaborbactam, which alone had no antimicrobial activity, is a novel cyclic boronic acid-
based β-lactamase inhibitor that potentiates the activity of meropenem.
(b) Spectrum of activity: Broad gram-negative activity, including multi-drug resistant
Enterobacterales, and remains stable in the presence of ESBLs and AmpC β-lactamases.
Vaborbactam is a potent inhibitor of class A carbapenemases, including KPC and class
A and C β-lactamases. It is not found to expand coverage to carbapenem-resistant
Acinetobacter baumannii, Pseudomonas aeruginosa, or Stenotrophomonas maltophilia.
(c) FDA-approved in 2017 for complicated UTIs. However, most clinicians reserve its coverage
for difficult-to-treat pathogens with minimal coverage options, such as KPC-producing
Enterobacterales.
(d) A recent multicenter, randomized, prospective, open-label, comparative trial evaluated
the efficacy of meropenem/vaborbactam to best available therapy (including mono- or
combination therapy with polymyxin B or colistin, carbapenems, aminoglycosides,
tigecycline, or ceftazidime/avibactam) for the management of complicated UTI, healthcare-
associated pneumonia, ventilator-associated pneumonia, bloodstream infections, and
complicated intra-abdominal infections. Meropenem/vaborbactam demonstrated improved
efficacy and decreased adverse events compared to best available therapy. Of note, only one
patient received ceftazidime/avibactam; therefore, no conclusions can be drawn regarding
comparative efficacy between these two agents.
v. Imipenem/cilastatin/relebactam
(a) Cilastatin is a dehydropeptidase-1 inhibitor that prevents inactivation of imipenem by renal
dehydropeptidase-1.
(b) Relebactam is a non-β-lactam β-lactamase inhibitor that is active against class A
β-lactamases. Compared to avibactam, relebactam has less inhibitory activity against
OXA-48, and neither inhibitor is active against metallo-β-lactamases.
(c) FDA-approved in 2019 for complicated UTIs, intra-abdominal infections, and health care-
associated/ventilator-associated pneumonia. However, most clinicians reserve its coverage
for difficult-to-treat pathogens with minimal coverage options, such as KPC-producing
Enterobacterales.
(d) In vitro studies have demonstrated that relebactam, when added to imipenem/cilstatin,
restores activity against imipenem nonsusceptible strains that produce KPC.

ACCP Updates in Therapeutics® 2022: Critical Care Pharmacy Preparatory Review and Recertification Course

272
Infectious Diseases II

(e) A recent multicenter, randomized, double-blind, comparator controlled trial compared


the efficacy and safety of imipenem/relebactam and colistin combined with imipenem/
cilastatin in patients with imipenem-resistant gram-negative bacterial infections (health
care-associated/ventilator-associated pneumonia, complicated intra-abdominal infections,
or complicated UTI). Clinical response rates were higher for imipenem/relebactam
compared to imipenem/colistin.
vi. Cefiderocol
(a) Novel siderophore cephalosporin with a unique mechanism of action that confers activity
against a broad spectrum of resistant gram-negative bacteria, including Enterobacterales,
Pseudomonas, Acinetobacter, and Stenotrophomonas. It has no clinically relevant in vitro
activity against most gram-positive bacteria and anaerobes.
(b) Cefiderocol is able to chelate ferric ions and use bacterial iron transport systems, which has
been called a “Trojan Horse” strategy and translates to higher concentrations of susceptible
gram-negatives in the periplasmic space.
(c) Due to the cephalosporin/catechol structure, it has increased stability against hydrolysis
by clinically relevant β-lactamases, including serine (including KPCs and OXA-48)
and metallo-β-lactamases. Given the use of the iron transport pathway, cefiderocol does
not require classic β-lactam porin channels to penetrate the cells, and therefore, retains
activity in the presence of porin channel mutations and remains unaffected by common
efflux pumps present in Enterobacterales.
(d) FDA-approved in 2019 for complicated UTIs.
(e) In vitro studies have demonstrated excellent activity of cefiderocol against class A, B, and
D CRE.
(f) A multicenter, randomized, open-label study of cefiderocol compared to best-available
therapy for the treatment of severe infections (hospital-acquired pneumonia, ventilator-
associated pneumonia, complicated UTI, bloodstream infection, and sepsis) caused
by carbapenem-resistant gram-negative pathogens. All-cause mortality versus other
antibiotics in critically ill patients was higher in the cefiderocol arm. The cause of higher
mortality was not established.
vii. Eravacycline
(a) Novel fluorocycline of the tetracycline class with a similar structure to tigecycline, escaping
many resistance mechanisms seen in other tetracyclines
(b) FDA-approved in 2018 for complicated intra-abdominal infections. It was investigated for
use with UTIs, but outcomes were not favorable.
(c) Demonstrated in vitro activity against most gram-positive and gram-negative pathogens,
including CRE and Acinetobacter, and anaerobes. It lacks activity against Pseudomonas.
(d) Low oral bioavailability; therefore, only IV formulation is FDA-approved.
(e) Randomized, double-blind, multicenter trial found that eravacycline was noninferior to
ertapenem for the management of complicated intra-abdominal infections.
viii. Combination therapy
(a) Several retrospective studies suggested that the use of combination therapy is warranted
for the treatment of CRE, particularly a regimen involving colistin, tigecycline, and a
carbapenem. Interpreting these studies is difficult. Many were noninterventional, hence
leading to the evaluation of many treatment regimens. The increased ratio of the number
of treatment regimens to the number of patients substantially increases the risk of spurious
findings. If no optimal therapy exists for a patient, combination therapy may be considered.
These therapies include scenarios in which the carbapenem MIC may be slightly elevated
or in which therapies with suboptimal PK/PD (i.e., colistin and tigecycline) are used.

ACCP Updates in Therapeutics® 2022: Critical Care Pharmacy Preparatory Review and Recertification Course

273
Infectious Diseases II

(b) A recent retrospective study that evaluated the role of combination therapy for carbapenem-
resistant gram-negative pathogens showed that combination therapy with several agents,
all of which had in vitro sensitivity, led to improvements in outcomes. In contrast,
combination therapy with several agents, not all of which had in vitro sensitivity, did not
lead to improvements.
(c) Several case reports recommend considering combining ertapenem with another
carbapenem for the treatment of carbapenem-resistant pathogens. This approach takes
advantage of the increased affinity for ertapenem seen in vitro with carbapenemases.
Hence, administering ertapenem as a sacrificial carbapenem may allow a different
carbapenem to exert its effects. Recent systematic review identified 171 patients who were
treated with this combination carbapenem strategy, and found clinical and microbiological
success reported in 70% of patients. However, this practice requires further testing; hence,
it cannot currently be recommended.
(d) If using polymyxin, combination therapy with another agent that has a susceptible MIC is
recommended.
4. Multidrug-resistant Pseudomonas
a. Resistance mechanisms seen with Pseudomonas aeruginosa are unpredictable. The presence
of several mechanisms, including β-lactamases, porin loss, efflux pump, and alteration of target
proteins, complicates treatment options.
b. Clinical approach usually entails empiric coverage with a β-lactam, which has the best local in vitro
activity against Pseudomonas, with or without a second antipseudomonal agent. Therapy could be
de-escalated to a monotherapy with the narrowest spectrum on availability of AST results.
c. Ceftolozane/tazobactam is a novel β-lactam/β-lactamase inhibitor antimicrobial with enhanced
activity against P. aeruginosa.
i. Ceftolozane is a novel third-generation cephalosporin.
ii. Spectrum of activity: Gram-negative organisms, including P. aeruginosa. Activity includes
coverage against ESBL and AmpC β-lactamase–producing organisms. Limited gram-positive
and anaerobic coverage. Among multidrug-resistant and extended drug–resistant Pseudomonas,
ceftolozane/tazobactam retains good activity, with its MIC90 still below the MIC breakpoint for
resistant, as determined by the FDA.
iii. FDA approved for the treatment of complicated intra-abdominal infection, complicated UTI,
and hospital-acquired and ventilator-associated pneumonia. However, clinically, its coverage
will most likely be reserved for resistant pseudomonal infections. A recent study concluded
that high-dose ceftolozane/tazobactam (3 g every 8 hours) was noninferior to meropenem in
the treatment of nosocomial pneumonia. Twenty-eight day mortality, clinical response, and
adverse reactions were similar between groups.
5. MRSA
a. MRSA is a significant cause of both community- and hospital-acquired infections.
b. Skin and soft tissue infections
i. Community-acquired MRSA often presents as a skin and soft tissue infection.
ii. Cutaneous infections and abscesses are best treated with adequate drainage.
iii. Antibiotics are usually not necessary unless the patient does not respond to drainage, has
extensive disease, or has signs and symptoms of systemic infection.
iv. Antibiotic treatment choices for community-acquired MRSA cutaneous and skin infections
include trimethoprim/sulfamethoxazole, clindamycin, and tetracycline.

ACCP Updates in Therapeutics® 2022: Critical Care Pharmacy Preparatory Review and Recertification Course

274
Infectious Diseases II

c. MRSA bacteremia and endocarditis


i. Can be treated with either vancomycin or daptomycin
ii. In a study of patients with right-sided endocarditis, daptomycin was noninferior to vancomycin.
Of interest, treatment response was poor with both therapies, with only about 40% in each
group meeting that outcome. Daptomycin was dosed at 6 mg/kg in this study.
iii. Some experts recommend giving higher daptomycin doses (8–10 mg/kg) to optimize the
PD target attainment for daptomycin. Daptomycin resistance in S. aureus has been reported.
A correlation appears to exist between intermediate sensitivity to vancomycin, thought
to be caused by a thickened cell wall–limiting penetration, and decreased susceptibility to
daptomycin. Currently, the daptomycin breakpoint MIC for MRSA is 1 mcg/mL or less.
iv. Several investigations have also evaluated daptomycin compared with vancomycin for
MRSA bacteremia when the vancomycin MIC was greater than 1 mcg/mL. Although these
investigations showed outcome benefit associated with daptomycin use, the studies have severe
limitations, which may limit their applicability.
(a) Retrospective analyses
(b) Daptomycin was usually used as definitive therapy after an initial course of vancomycin.
(c) Daptomycin use in one study was associated with a significantly higher rate of infectious
diseases consultation, which may have other treatment implications.
(d) Given study limitations, the routine use of daptomycin for MRSA infections when the
vancomycin MIC is greater than 1 mcg/mL cannot be recommended.
(e) The 2011 IDSA guidelines for the treatment of MRSA state that the treatment of isolates
with a vancomycin MIC of 2 mcg/mL or less should be determined by the patient’s response
to vancomycin, independent of the MIC.
d. MRSA pneumonia.
i. Community-acquired MRSA pneumonia can be treated with either vancomycin or linezolid.
Clindamycin can be considered if the strain is susceptible.
ii. Community-acquired MRSA infections may be associated with the presence of Panton-
Valentine leukocidin, which is a two-component staphylococcal membrane toxin that targets
leukocytes. This toxin has been linked with severe infections, necrotizing pneumonia, and
abscess formation. Theoretically, clindamycin and linezolid, being ribosomal subunit and
protein synthesis inhibitors, may attenuate the amount of toxin production.
iii. A randomized controlled study comparing linezolid to vancomycin included patients with
hospital-acquired pneumonia and ventilator-associated pneumonia who had a baseline
respiratory culture positive for MRSA.
(a) Vancomycin was dosed by weight and adjusted locally according to trough levels.
(b) This was a noninferiority study with nested superiority criteria where the primary end
point was clinical response at the end of the study.
(c) Clinical response was defined as resolution of signs of pneumonia with no further need for
other antibiotics.
(d) Study met both the noninferiority and the superiority criteria with respect to clinical
response and microbiological clearance.
(e) Study may have been confounded by less than 50% of patients reaching vancomycin
target concentrations by day 3 and the vancomycin patients having a slightly higher rate of
baseline bacteremia.
(f) No difference in mortality was seen.
(g) Many clinicians interpret the results from this study as suggesting that either vancomycin
or linezolid can be considered for the treatment of MRSA pneumonia.

ACCP Updates in Therapeutics® 2022: Critical Care Pharmacy Preparatory Review and Recertification Course

275
Infectious Diseases II

e. Updates to vancomycin dosing and monitoring


i. Consensus guidelines for therapeutic monitoring of vancomycin were published in 2020.
ii. AUC24 of 400–600 mg/hour/L is recommended target to improve clinical efficacy and patient
safety. For most isolates, an MIC of 1 mg/L can be assumed.
iii. Trough targets are no longer recommended for patients with serious MRSA infections.
iv. Bayesian dosing is the recommended method for dosing adjustments. Bayesian dosing does
not require steady-state vancomycin concentrations for early determination of AUC target
attainment.
f. Novel agents for the treatment of MRSA
i. Tedizolid is FDA-approved for the treatment of bacterial skin and skin structure infection.
(a) Antimicrobial class: Oxazolidinone
(b) Spectrum of activity: Gram-positive pathogens including MRSA, vancomycin-resistant
enterococci, vancomycin-resistant staphylococci, drug-resistant S. pneumoniae, and
linezolid-resistant gram-positive pathogens
(c) PK and dosing
(1) Bioavailability 91% – Can be administered as a parenteral solution or oral tablets
(2) Half-life: 12 hours
(3) Dosing: 200 mg once daily
(d) Studies: Tedizolid once daily for 6 days was compared with linezolid twice daily for 10
days for the treatment of acute bacterial skin and skin structure infection and was found
to be noninferior.
(e) Application: Place in therapy among critically ill patients is limited. Studies are under way
for the use of tedizolid in the treatment of hospital-acquired pneumonia.
ii. Ceftaroline
(a) Antimicrobial class: Fifth-generation cephalosporin
(b) Spectrum of activity: Enterobacterales (similar to third-generation cephalosporins) and
gram-positive pathogens, including MRSA, drug-resistant S. pneumoniae, and vancomycin-
intermediate and vancomycin-resistant staphylococci
(c) PK and dosing
(1) Activity against MRSA mediated through enhanced affinity to penicillin-binding-
protein 2a
(2) Half-life: 2.7 hours
(3) Excretion: 88% urine
(4) Dosing:
(A) 600 mg intravenously every 12 hours is the FDA label-approved dosing. Reports
of using 600 mg intravenously every 8 hours for severe infections
(B) Adjustments necessary for patients with renal impairment
(d) Studies:
(1) Evaluated in a noninferiority trial compared with ceftriaxone, both in conjunction with
clarithromycin, for the treatment of community-acquired bacterial pneumonia. Results
indicated noninferiority, with numerically higher cure rates in those randomized to
ceftaroline. Clinical response by day 4 of therapy was also higher in the ceftaroline
group.
(2) Evaluated in a noninferiority trial compared with vancomycin plus aztreonam for the
treatment of skin and skin structure infection. Ceftaroline cure rates were within the a
priori–determined margin, thus satisfying the criteria for noninferiority.

ACCP Updates in Therapeutics® 2022: Critical Care Pharmacy Preparatory Review and Recertification Course

276
Infectious Diseases II

(e) Application:
(1) FDA approved for acute bacterial skin and skin structure infections and community-
acquired bacterial pneumonia
(2) Despite its FDA approvals, the main role of ceftaroline is the availability of an
additional agent with activity against MRSA. May be an option when vancomycin
therapy is suboptimal and other treatment options may lead to unwanted adverse
effects. May also be a reasonable option for the treatment of community-acquired
pneumonia in institutions where community-acquired MRSA is common and for the
salvage treatment of MRSA bacteremia

Patient Case

6. A 72-year-old woman with a history of end-stage renal disease is admitted to the medical ICU with signs
and symptoms of sepsis. Blood cultures are obtained in which two of two bottles grow gram-positive cocci.
The patient is initiated on vancomycin, and her tunneled dialysis catheter is removed. On day 4 of therapy,
the blood cultures are finalized to be MRSA with the following antibiotic susceptibility results: oxacillin
greater than 4 mcg/mL – R; vancomycin 2 mcg/mL – S; daptomycin 0.5 mcg/mL – S; and linezolid 1 mcg/
mL – S. A transthoracic echocardiogram is obtained, which reveals echodensities on the mitral valve. The
patient will be treated medically with antibiotics for 4–6 weeks. The patient’s repeat blood cultures are cur-
rently no growth. Her vital signs and laboratory values are as follows: blood pressure 150/90 mm Hg, heart
rate 88 beats/minute, temperature 100.2°F, WBC 11 x 103 cells/mm3, and lactate 1.1 mmol/L. Which is the
most appropriate treatment regimen?
A. Change vancomycin to linezolid 600 mg every 12 hours.
B. Add gentamicin and rifampin to vancomycin.
C. Change vancomycin to daptomycin 6 mg/kg every 48 hours.
D. Continue vancomycin, and target a trough level of 15–20 mcg/mL.

VII.  IMMUNOCOMPROMISED PATIENTS

A. Febrile Neutropenia
1. Definition
a. Fever: A single temperature of 101°F (38.3°C) or greater orally or a temperature of 100.4°F (38.0°C)
or greater orally for more than 1 hour
b. Neutropenia: Less than 500 neutrophils/mm3 or less than 500 neutrophils/mm3 during the next 48
hours
2. Patients with neutropenic fever admitted to an ICU should be considered high risk.
3. Initial therapy should include monotherapy with an intravenous antipseudomonal β-lactam (i.e.,
cefepime, piperacillin/tazobactam, meropenem, imipenem). Patients with type 1 hypersensitivity should
be treated with either ciprofloxacin or aztreonam plus vancomycin.
4. Consider dual gram-negative therapy (fluoroquinolone or aminoglycosides) in patients with shock or if
antimicrobial resistance is suspected.
5. Consider adding vancomycin to gram-negative therapy in patients with shock, suspected catheter-related
infection, skin and soft tissue infection, and/or pneumonia. Gram-positive therapy can be discontinued
in 48–72 hours if no evidence of gram-positive infections is discovered.

ACCP Updates in Therapeutics® 2022: Critical Care Pharmacy Preparatory Review and Recertification Course

277
Infectious Diseases II

6. Modifications to initial antibiotic choices should be considered for patients with worsening clinical
status or if patients’ microbiological data warrant change.
7. Unexplained persistent fever in an otherwise clinically stable patient rarely warrants an escalation in
therapy. Persistent fevers for 4–7 days after initiation of antibacterial agents should warrant consideration
for empiric antifungal coverage in those who have persistent neutropenia.
8. Initial antimicrobials should be de-escalated or escalated in documented infections depending on in
vitro susceptibility. Documented infections and unexplained fevers should be treated for a minimum of
14 days and until the absolute neutrophil count is greater than 500 cells/mm3 (whichever is later).
9. Patients with hemodynamic instability should have their initial antibiotic regimen escalated to include
coverage for resistant bacteria and fungi.
10. Hematopoietic growth factors should not be used for the treatment of febrile neutropenia. Prophylactic
use of hematopoietic growth factors should be considered for patients with a high anticipated risk of
febrile neutropenia (20% or greater).

B. Solid Organ Transplantation


1. Epidemiology
a. Hospital-acquired bacterial infections are the most common types of infections in SOT recipients.
b. 50%–75% of SOT recipients will experience an infection within the first year after transplantation.
c. Posttransplant infections may contribute to graft dysfunction and reduce long-term survival, and
they have been associated with prolonged LOS and cost of care.
d. An analysis of 60,000 renal transplant recipients found that infections were the second leading
cause of death.
2. General risk factors for infections
a. Excessive use of antibiotics before transplantation: With the use of prophylactic antimicrobials in
the pretransplant period, many transplant recipients are experiencing resistant pathogens in the
posttransplant period.
b. Infections are most common in the first 6 months after transplantation, with different pathogens
presenting after various durations of immunosuppression. See Figure 3. OIs are rare in the first month
after transplantation because the full effects of immunosuppression are not yet present. Fungal and
viral infections experienced during the first month after transplantation are usually donor derived.
c. Duration of hospitalization after transplantation
d. Renal dysfunction
e. Several acute rejection episodes
3. General clinical approach for infectious diseases issues in critically ill SOT recipients
a. Be cognizant of the patient’s transplant timeline, particularly with respect to possible OIs,
pretransplant risk factors, immune status, intensity of immunosuppressive therapy, prophylactic
regimens, and recent treatments of rejection.
b. Have a high clinical suspicion for OIs.
c. Severe infections sometimes warrant a reduction in maintenance immunosuppression. Consult with
the transplant team regarding plans for maintenance immunosuppression.
d. Be aware of the many drug interactions between immunosuppressants and antimicrobials.

ACCP Updates in Therapeutics® 2022: Critical Care Pharmacy Preparatory Review and Recertification Course

278
Infectious Diseases II

Conventional
Community-acquired
nosocomial Unconventional or opportunistic infections
persistent infections
infections
Viral
HSV
Onset of CMV CMV retinitis or colitis
EBV, VZV, influenza, RSV, adenovirus
Papillomavirus, PTLD
Onset of hepatitis B or hepatitis C

Bacterial
Wound infections, catheter-related infections, pneumonia
Nocardia
Listeria, tuberculosis

Fungal
Pneumocystis
Aspergillus Cryptococcus
Candida Endemic fungi

Parasitic
Strongyloides
Toxoplasma
Leishmania
Trypanosoma cruzi

0 1 2 3 4 5 6
Months After Transplant
Figure 3. Usual sequence of infections after organ transplantation.a
a
Zero indicates the time of transplantation. Solid lines indicate the most common period for the onset of infection; dotted lines indicate periods of continued risk at a
reduced level.
CMV = cytomegalovirus; EBV = Epstein-Barr virus; HSV = herpes simplex virus; PTLD = posttransplant lymphoproliferative disease; RSV = respiratory syncytial
virus; VZV = varicella-zoster virus.
Reproduced with permission from: Baillie GM. Infectious disease concerns in solid organ transplantation. In: Schumock GT, Brundage DM, Dunsworth TS, et al., eds.
Pharmacotherapy Self-Assessment Program, 5th ed. Book 2. Transplantation. Lenexa, KS: American College of Clinical Pharmacy, 2004:165-86.

ACCP Updates in Therapeutics® 2022: Critical Care Pharmacy Preparatory Review and Recertification Course

279
Infectious Diseases II

VIII.  HUMAN IMMUNODEFICIENCY VIRUS/ACQUIRED IMMUNE DEFICIENCY SYNDROME IN


CRITICALLY ILL PATIENTS

A. Diagnosis
1. Isolated CD4 + lymphopenia has occurred in critically ill patients without HIV disease.
a. Differential diagnosis of CD4 + lymphopenia
i. Idiopathic
ii. Common variable immunodeficiency
iii. Corticosteroid administration
iv. Circadian rhythm
v. Hematologic malignancies
vi. Critical illness
2. Patients with traditional HIV risk factors, isolated CD4 + lymphopenia, or AIDS-defining OIs who
have not been given a diagnosis should receive HIV serologic testing. See Table 10 for AIDS-defining
conditions.

B. Presentation for HIV-Associated Acute Illness


1. Changing prescribing patterns in highly active antiretroviral therapy (HAART) and improvements
in care have changed the landscape of ICU admissions associated with HIV. Although rates of ICU
admissions among HIV-positive patients have remained stable over time, the rationale for ICU admission
has shifted from respiratory distress associated with pneumocystic pneumonia to other sepsis etiologies,
neurologic disorders, and other end-organ dysfunctions.
2. See Table 11 for the management and prophylaxis of commonly occurring OIs in critically ill patients.

C. Management of HAART Therapy


1. Treatment-naive patient
a. Initiation of HAART in an ICU may be deterred by factors such as lack of enteral access, inability
to assess the patient’s willingness to commit to therapy, and high probability of interruptions in
therapy because of surgical interventions.
b. Immune reconstitution inflammatory syndrome
i. Unexpected worsening of existing OI or unmasking of previously unrecognized illness
associated with recent HAART initiation
ii. Recovery of immune response leading to a proinflammatory cytokine storm
iii. Risk factors
(a) Presence of disseminated disease or high antigen load
(b) High baseline HIV viral load
(c) Rapid response to antiretroviral therapy
iv. Evidence does not suggest that early administration of HAART (at the onset of an AIDS-
defining OI) is associated with a higher incidence of immune reconstitution inflammatory
syndrome.
v. In general, early and continued administration of HAART associated with improved outcomes,
particularly in patients with OI. For patients with severe symptoms associated with immune
reconstitution inflammatory syndrome (requiring vasopressors or intubation) consider the
adjunctive use of prednisone.
2. Patient receiving HAART regimen
a. Considerations: Availability of drug formulations conducive to ICU administration; food
requirements; dose adjustments associated with renal or hepatic impairment; new drug interactions;
possible interruptions in therapy

ACCP Updates in Therapeutics® 2022: Critical Care Pharmacy Preparatory Review and Recertification Course

280
Infectious Diseases II

b. In general, if one of the antiretroviral therapies has to be discontinued, all of the therapies should
be discontinued to decrease the promotion of resistance caused by the suboptimal suppression of
viral replication. Exceptions: Nonnucleoside reverse transcriptase inhibitors (NNRTIs) have long
half-lives (as long as 3 weeks); therefore, if NNRTIs are discontinued at the same time as other
antiretrovirals with shorter half-lives, there will be a period of functional NNRTI monotherapy. May
consider continuing antiretrovirals with shorter half-lives, if possible, for 1–2 weeks to minimize
selection of NNRTI resistance.
c. Possible ICU drug interactions – See Table 12.
3. HAART-associated adverse drug reactions
a. Newer generations of HAART regimens are generally well tolerated; however, many patients still
receive older HAART therapy.
b. Many of the HAART-associated adverse effects have no specific treatment; therefore, early
recognition and prompt discontinuation of the offending agent is crucial.
c. Lactic acidosis
i. Two nucleoside/nucleotide reverse transcriptase inhibitors (NRTIs) (e.g., lamivudine and
tenofovir) remain the backbone of many HAART regimens.
ii. NRTIs are associated with a variety of mitochondrial toxicities.
iii. Older NRTIs, such as zidovudine, stavudine, and didanosine, have been associated with lactic
acidosis.
iv. Symptoms: Fatigue, malaise, nausea, vomiting, abdominal pain, hepatomegaly
v. Management: Supportive therapy and discontinuing the potential offending agent
d. Abacavir hypersensitivity
i. Symptoms: Fever, rash, gastrointestinal (GI) symptoms
ii. Reaction associated with the presence of the HLA-B*5701 allele, which has an 8% prevalence
among whites in North America. Genetic screening for allele is recommended.
iii. Management: Supportive care and discontinuing agent, with rechallenge contraindicated
because of the possibility of life-threatening hemodynamic compromise
e. Other HAART-associated significant adverse effects:
i. Nevirapine hypersensitivity: Rash (may progress to Stevens-Johnson syndrome), fever,
hepatotoxicity
ii. Raltegravir-associated rhabdomyolysis
iii. Tipranavir-associated hepatotoxicity and intracranial hemorrhage
iv. Protease inhibitor–associated pancreatitis

D. Management and Prophylaxis of HIV-Associated Infections: See Table 10.

ACCP Updates in Therapeutics® 2022: Critical Care Pharmacy Preparatory Review and Recertification Course

281
Infectious Diseases II

Patient Case

7. A 46-year-old man is admitted to the medical ICU for diabetic ketoacidosis. The patient has a history of
insulin-dependent diabetes, is HIV positive, and has cryptococcal meningitis. His current HAART regimen
consists of atazanavir, ritonavir, tenofovir, and emtricitabine, which is continued on admission to the ICU.
The patient’s CD4 + count is 85/mm3. The patient’s diabetic ketoacidosis is well controlled, and he is ready to
be discharged from the ICU. Before discharge, the patient is noted not to be on any prophylaxis against OIs.
Which prophylactic regimen would be most appropriate for the patient?
A. Azithromycin 1200 mg once weekly and trimethoprim/sulfamethoxazole 1 double-strength (DS) tablet
thrice weekly
B. Fluconazole 200 mg daily and trimethoprim/sulfamethoxazole 1 DS tablet daily
C. Azithromycin 1200 mg once weekly and fluconazole 200 mg daily
D. Fluconazole 200 mg daily and trimethoprim/sulfamethoxazole 1 DS tablet thrice weekly

Table 10. AIDS-Defining Conditions

Bacterial infections, multiple/recurrent (< 13 yr) Lymphoid interstitial pneumonia or pulmonary


Candidiasis: Bronchi, trachea, or lungs lymphoid hyperplasia complex (< 13 yr)
Candidiasis: Esophageal Lymphoma: Burkitt (or equivalent term)
Cervical cancer: Invasive Lymphoma: Immunoblastic (or equivalent term)
Coccidioidomycosis: Disseminated or Lymphoma: Primary or brain
extrapulmonary Mycobacterium avium complex or M. kansasii:
Cryptococcosis: Extrapulmonary Disseminated or extrapulmonary
Cryptosporidiosis: Chronic intestinal (for > 1 mo) M. tuberculosis: Any site (pulmonary or
Cytomegalovirus disease extrapulmonary)
(other than the liver, spleen, or nodes) Mycobacterium: Other species or unidentified
Cytomegalovirus retinitis (with loss of vision) species; disseminated or extrapulmonary
Encephalopathy: Related to HIV P. jiroveci pneumonia
Herpes simplex: Chronic ulcer(s) Pneumonia: Recurrent
(for > 1 mo); bronchitis, pneumonitis, or Progressive multifocal leukoencephalopathy
esophagitis Salmonella septicemia (recurrent)
Histoplasmosis: Disseminated or extrapulmonary Toxoplasmosis of brain
Isosporiasis: Chronic intestinal Wasting syndrome caused by HIV

ACCP Updates in Therapeutics® 2022: Critical Care Pharmacy Preparatory Review and Recertification Course

282
Infectious Diseases II

Table 11. AIDS-Associated OIs and Their Respective Prophylactic and Treatment Options
OI/CD4+ Threshold
Prophylaxis Option Treatment Option
for Prophylaxis
Preferred:
Preferred:
TMP/SMXb 15–20 mg/kg PO/IV divided q6–8hr x 21 days
TMP/SMXb 1 DS PO
plus
daily
prednisone 40 mg PO q12hr x 5 days; 40 mg/day x 5 days;
or
20 mg/day x 11 days
TMP/SMXb 1 SS PO daily
(if Pao2< 70 mm Hg or alveolar-arterial O2 gradient > 35)
P. jiroveci: Alternatives (IV methylprednisolone could be administered at 75% of
< 200 cells/mm3 Dapsonea PO 100 mg/day dose)
Pentamidine 300 mg
Alternatives:
inhalation once monthly
Pentamidineb IV 4 mg/kg/day
TMP/SMX 1 DS PO TIW
Clindamycin 600–900 mg PO/IV q6–8hr + primaquinea PO
Atovaquone PO 1500 mg/
30 mg/day (base)
day
Atovaquone 750 mg PO q12hr
Preferred:
Preferred: Pyrimethamineb 200 mg PO x 1; then 50–75 mg/day +
TMP/SMX 1 DS PO daily sulfadiazineb 1000–1500 mg PO q6hr + leucovorin PO
Toxoplasmosis: 10–20 mg/day x 6 wk
< 100 cells/mm3 Alternatives: Alternatives:
and Toxoplasma IgG Dapsonea PO 50 mg/day + Pyrimethamineb 200 mg PO x 1; then 50–75 mg/day +
positive (pyrimethamine 50 mg clindamycin 600–900 mg PO/IV q8hr + leucovorin PO
PO + leucovorin 25 mg 10–20 mg/day
PO) once weekly Pyrimethamineb 200 mg PO x 1; then 50–75 mg/day +
TMP/SMX 1 DS PO TIW atovaquone 1500 mg PO BID + leucovorin PO 10–20 mg/
day
Preferred:
Preferred:
Azithromycin 1200 mg
At least two drugs: Clarithromycin 500 mg PO BID or
once weekly
azithromycin 600 mg/day
or
plus ethambutolb PO 15–25 mg/kg/day
Mycobacterium avium Clarithromycin 500 mg
Additional agents (consider depending on susceptibility
complex: q12hr
testing, severe disease, or low CD4+ count)
< 50 cells/mm3 or
Rifabutinc PO 300 mg/day
Azithromycin 600 mg
Amikacinb IV 10–15 mg/kg/day
twice weekly
fluoroquinolones (moxifloxacin PO/IV 400 mg/day or
Alternatives:
levofloxacinb PO/IV 500 mg/day)
Rifabutinc 300 mg/day

ACCP Updates in Therapeutics® 2022: Critical Care Pharmacy Preparatory Review and Recertification Course

283
Infectious Diseases II

Table 11. AIDS-Associated OIs and Their Respective Prophylactic and Treatment Options (continued)
OI/CD4+ Threshold
Prophylaxis Option Treatment Option
for Prophylaxis
Latent infection treatment
Preferred:
Isoniazid (INH) PO 300 Active infection treatment
Mycobacterium
mg/day + pyridoxine PO
tuberculosis: Primary Preferred:
50 mg/day Rifampinc PO/IV 600 mg/day + INH PO 300 mg/day +
prophylaxis not
indicated Alternatives: pyrazinamideb PO 20–25 mg/kg/day + ethambutolb PO
INH 900 mg PO twice 15–25 mg/kg/day
weekly + pyridoxine
100 mg PO twice weekly
Preferred:
Induction therapy x 2 wk:
Liposomal amphotericin IV 3-4 mg/kg/day + flucytosineb
PO 25 mg/kg q6hr
Alternatives:
Cryptococcosis:
Induction x 2 wk:
Primary prophylaxis
1. Amphotericin B IV 0.7–1 mg/kg/day + flucytosineb PO
not indicated Secondary prophylaxis:
25 mg/kg q6hr
Fluconazole 200–400 mg/
2. Amphotericin B lipid complex 3-4 mg/kg/day +
Secondary day
flucytosineb PO 25 mg/kg q6hr
prophylaxis may be
3. Liposomal amphotericin IV 3–4 mg/kg/day +
considered
fluconazoleb 800 mg PO/IV daily

Preferred:
Consolidation therapy x 8 wk
Fluconazole PO/IV 400 mg/day
Preferred:
Induction:
Cytomegalovirus: Valganciclovirb PO 900 mg BID for 14–21 days
Primary prophylaxis
Secondary prophylaxis: or
not indicated Ganciclovirb 5 mg/kg q12hr for 14–21 days
Valganciclovirb PO 900
Secondary
mg/day Alternatives:
prophylaxis may be
considered 1. Foscarnetb IV 90 mg/kg q12hr or 60 mg/kg q8hr
2. Cidofovir IV 5 mg/kg/wk (with probenecid 2 g 3 hr
before cidofovir; then 1 g 2 hr after cidofovir)
a
Should not be used in patients with a glucose-6-phosphate dehydrogenase deficiency
b
Renal adjustments may be necessary.
c
Monitor for interactions with HAART.
BID = twice daily; DS = double strength; hr = hour(s); INH = isoniazid; IV = intravenous(ly); OI = opportunistic infection; PO = orally; SS = single strength; TIW =
thrice weekly; TMP/SMX = trimethoprim/sulfamethoxazole.
Reproduced with permission from: Smith CL. HIV/infectious disease. In: ACCP Updates in Therapeutics® 2014: Pharmacotherapy Preparatory Review and
Recertification Course. Lenexa, KS: American College of Clinical Pharmacy, 2014:580-621.

ACCP Updates in Therapeutics® 2022: Critical Care Pharmacy Preparatory Review and Recertification Course

284
Infectious Diseases II

Table 12. Common Interactions Between Antiretrovirals and Commonly Used Medications in Critically Ill Patients
Agent Antiretroviral Interactions
Rilpivirine: Contraindicated with proton
pump inhibitors
Stress ulcer prophylaxis
(proton pump inhibitors, Atazanavir: Relatively contraindicated
with proton pump inhibitors (administer Decrease in antiretroviral concentrations
histamine-2 receptor
antagonists) no more than the equivalent of
omeprazole 20 mg daily, and separate
administration by at least 12 hr)
Triazole antifungals:
Protease inhibitors:
Voriconazole, Increase in antiretroviral concentration
Nonnucleoside reverse transcriptase
posaconazole, itraconazole, Decrease in antifungal concentrations
inhibitors
isavuconazole
Antibacterials:
Rifampin Protease inhibitors: Decrease in antiretroviral concentration
Nonnucleoside reverse transcriptase
inhibitors
Clarithromycin Protease inhibitors Increase in clarithromycin concentration
Nonnucleoside reverse transcriptase
inhibitors
Metronidazole Fosamprenavir, lopinavir, ritonavir Disulfiram reaction
Antiarrhythmics:
Amiodarone Indinavir, ritonavir, tipranavir
Flecainide, Lopinavir, ritonavir, tipranavir Increased antiarrhythmic concentrations
propafenone, quinidine
Diltiazem Atazanavir, fosamprenavir
Increase concentrations of statins – in
Protease inhibitors
decreasing order of interaction potential
Statins Nonnucleoside reverse transcriptase
(lovastatin, simvastatin, rosuvastatin,
inhibitors
atorvastatin, pravastatin)
Anticonvulsants: Protease inhibitors Decrease in antiretroviral concentrations
Carbamazepine, Nonnucleoside reverse transcriptase Increase or decrease in anticonvulsant
phenobarbital, phenytoin inhibitors concentrations
Protease inhibitors
Nonnucleoside reverse transcriptase
Midazolam Increase in midazolam concentration
inhibitors
Cobicistat/elvitegravir
Nonnucleoside reverse transcriptase
inhibitors; fosamprenavir, ritonavir, Decrease in methadone concentration
Methadone
lopinavir, nelfinavir, didanosine, (potentially leading to opioid withdrawal)
saquinavir
Sildenafil Protease inhibitors; delavirdine Increase in sildenafil concentration

ACCP Updates in Therapeutics® 2022: Critical Care Pharmacy Preparatory Review and Recertification Course

285
Infectious Diseases II

Table 12. Common Interactions Between Antiretrovirals and Commonly Used Medications in Critically Ill Patients
(continued)
Agent Antiretroviral Interactions
Increase in warfarin concentration
Delavirdine, efavirenz Increases in apixaban concentrations: Do
Anticoagulants:
Protease inhibitor/cobicistat, protease not administer in patients who require 2.5
Warfarin
inhibitor/ritonavir mg BID; in patients requiring 10 mg or 5
Apixaban
Protease inhibitor/cobicistat, protease mg twice daily, reduce apixaban by 50%
Rivaroxaban
inhibitor/ritonavir Increases in rivaroxaban expected: Do
not coadminister

IX.  ANTIFUNGAL THERAPY

A. Amphotericin B
1. Mechanism of action: Binds to ergosterol in the fungal cell membrane, which alters the membrane
permeability, leading to ion leakage and fungal cell death
2. Spectrum of activity
a. Candida spp. (except for C. lusitaniae), Blastomycetes, coccidioidomycosis, Cryptococcus,
Paracoccidioides, histoplasmosis, aspergillosis, mucormycosis
b. Wide spectrum of activity allows its clinical use in many different systemic fungal infections.
3. Dose
a. Conventional amphotericin: 0.5–1.0 mg/kg/day
b. Lipid amphotericin: 3–5 mg/kg/day (higher doses have not been associated with improved outcomes)
4. Adverse effects
a. Renal toxicity
i. Decrease in glomerular filtration rate, which is associated with cumulative doses greater than
4–5 g
ii. Clinical manifestation
(a) Renal tubular acidosis
(b) Oliguria
(c) Azotemia
(d) Potassium, magnesium, phosphate wasting
iii. Prevention
(a) Avoid concomitant nephrotoxins.
(b) Avoid dehydration.
(c) Salt loading: 500 mL of normal saline before and after infusion of amphotericin
b. Infusion-related reactions
i. Mediated by cytokine release and prostaglandin synthesis
ii. Presentation: Fever, chills, nausea, vomiting, flushing, rigors
iii. Prevention
(a) Premedications – Administered 30–60 minutes before infusion
(1) Hydrocortisone 25–50 mg
(2) Ibuprofen 600 mg
(3) Acetaminophen 650 mg with diphenhydramine 50 mg

ACCP Updates in Therapeutics® 2022: Critical Care Pharmacy Preparatory Review and Recertification Course

286
Infectious Diseases II

(4) Meperidine 50 mg if patient previously experienced rigors


(b) Use lower infusion rate on dose initiation.
c. Lipid amphotericin (liposomal amphotericin and amphotericin B lipid complex)
i. Amphotericin dissociates from lipid over time, which may limit its renal toxicity and infusion-
related reactions.
ii. Premedication may still be necessary for amphotericin B lipid complex.

B. Triazole Antifungals
1. Mechanism of action: Inhibits the synthesis of ergosterol through blocking the CYP enzyme 14-α-sterol-
demethylase. Inhibition of this enzyme leads to the accumulation of 14-α-methyl sterols on the fungal
surface, which in turn leads to fungal cell death.
2. Fluconazole
a. Spectrum of activity:
i. Candida spp.
(a) Candida krusei is intrinsically resistant.
(b) Variable sensitivity with Candida glabrata. Should not be used for C. glabrata unless
antifungal susceptibilities are available. If C. glabrata is sensitive dose-dependent, higher
doses may be necessary (12 mg/kg/day).
(c) Good activity against all other pathogenic Candida spp.
ii. Cryptococcus
b. Dose
i. 6–12 mg/kg/day (400–800 mg/day)
ii. Available in intravenous and oral formulations
iii. Well absorbed orally with high bioavailability
iv. Primarily excreted by the kidneys – Renal dose adjustments are necessary.
c. Adverse effects: Minimal adverse effects and lowest propensity for drug interactions among triazoles
3. Itraconazole
a. Spectrum of activity
i. Candida spp. – Has coverage similar to fluconazole with the addition of activity against C.
krusei
ii. Aspergillus, Blastomyces, Histoplasma
b. Dose
i. 200–400 mg once daily
ii. Intravenous formulation no longer available
iii. Poor and erratic oral absorption. Improved with oral liquid formulation, maintaining high
stomach acidity, avoidance of acid-suppressive therapy, and coadministration with acidic
beverage
iv. Therapeutic drug monitoring may be necessary. Target: Itraconazole plus hydroxy-itraconazole
concentration greater than 1 mcg/mL
v. Primarily used for fungal prophylaxis in immunocompromised patients and treatment of
endemic fungi (histoplasmosis, blastomycosis)
c. Adverse effects
i. GI upset, increase in liver function tests
ii. Drug interactions: CYP 3A4 and 2C9 inhibitor

ACCP Updates in Therapeutics® 2022: Critical Care Pharmacy Preparatory Review and Recertification Course

287
Infectious Diseases II

4. Voriconazole
a. Spectrum of activity
i. Candida spp. – Has coverage similar to fluconazole with the addition of activity against C.
krusei
ii. Aspergillus, Fusarium, Scedosporium: Resistance against voriconazole has occurred with
these pathogens.
b. Dose
i. Drug of choice for invasive aspergillosis
ii. 6 mg/kg every 12 hours x 2 doses as the loading dose, followed by 4 mg/kg every 12 hours
iii. Intravenous and oral formulations are available.
iv. Intravenously formulated in sulfobutyl-ether-β-cyclodextrin, which accumulates in renal
dysfunction, although the clinical significance of this is unknown
v. Extensively metabolized by the liver, with 50% dose reductions recommended for patients with
moderate to severe cirrhosis
vi. Genetic variations in CYP metabolism and high propensity for drug interactions lead to wide
interpatient variability in concentrations.
vii. Therapeutic drug monitoring may be necessary. Target trough concentration: 1–5.5 mcg/mL.
c. Adverse reactions
i. Increase in liver function tests
ii. Visual hallucinations
iii. Rash
iv. Nausea
v. CYP 3A4 and 2C9 inhibitor:
(a) Contraindicated with the use of rifampin, rifabutin, sirolimus, barbiturates, carbamazepine,
and quinidine
(b) Significant dose reductions for cyclosporine and tacrolimus when coadministered with
voriconazole
vi. QTc prolongation
5. Posaconazole
a. Spectrum of activity: Wide spectrum of activity, which includes Candida (similar to voriconazole),
Aspergillus, Zygomycetes, and Fusarium
b. Dose
i. Oral suspension (immediate release): 200 mg every 6 hours. Oral suspension has extremely
erratic absorption that is enhanced by coadministration with a high-fat meal and acidic food. In
critically ill patients in whom coadministration with fatty meals is not possible and avoidance
of acid-suppressive stress ulcer prophylaxis cannot be avoided, would recommend considering
an alternative therapy or administration method (e.g., oral tablets or intravenous)
ii. Oral tablets (delayed release): 300 mg every 12 hours x 2 doses, followed by 300 mg once
daily. Oral tablet is in an extended-release formulation, which cannot be crushed. Oral tablet
absorption not as dependent on gastric pH and meal lipid content.
iii. Intravenous: 300 mg every 12 hours x 2 doses, followed by 300 mg once daily (intravenously
formulated in sulfobutyl-ether-β-cyclodextrin, which accumulates in renal dysfunction,
although the clinical significance of this is unknown)
iv. Therapeutic drug monitoring may be prudent, particularly when oral suspension is used. Target
concentrations: Trough greater than 1.25 mcg/mL for treatment of invasive fungal infection
v. Mainly used for fungal prophylaxis in immunocompromised patients and treatment when
patient is not responding to other therapies

ACCP Updates in Therapeutics® 2022: Critical Care Pharmacy Preparatory Review and Recertification Course

288
Infectious Diseases II

c. Adverse reactions
i. Increase in liver function tests
ii. Nausea/vomiting
iii. Drug interaction: CYP3A4 inhibitor
iv. QTc prolongation
6. Isavuconazole
a. Spectrum of activity: Wide spectrum of activity, including Candida (similar to voriconazole),
Aspergillus (may retain activity for some species that are resistant to other azoles), Zygomycetes,
and dimorphic fungi. Limited activity against Fusarium.
b. Dose
i. Intravenous or oral: 372 mg of isavuconazonium sulfate (equivalent to 200 mg of isavuconazole)
every 8 hours, after a 1116-mg (600 mg of isavuconazole) loading dose
ii. Isavuconazonium sulfate solution is readily water soluble, unlike posaconazole and voriconazole,
and does not require stabilization with cyclodextrin.
c. Mainly PK advantages and safety benefits over voriconazole and posaconazole
i. Linear kinetics
ii. Oral bioavailability: 98%, not affected by food or acidity
iii. Fewer drug-drug interactions
iv. No QTc prolongation; in fact, associated with dose-dependent QTc shortening

C. Echinocandins (caspofungin, micafungin, and anidulafungin)


1. Mechanism of action: Inhibition of glucan synthase, which is an enzyme responsible for the formation of
1,3-β-d-glucan. Inhibition of this enzyme leads to cessation of fungal cell wall formation.
2. Spectrum of activity: All Candida spp. and Aspergillus
a. In vitro, C. parapsilosis has a much higher MIC than other Candida spp. The clinical significance
of this is unknown. Cases of breakthrough infections with C. parapsilosis during treatment with
echinocandins have been reported. Several retrospective studies did not find worsened outcomes
associated with echinocandin treatment of C. parapsilosis. The current IDSA guidelines on invasive
candidiasis recommend using fluconazole for the treatment of C. parapsilosis.
b. Primarily used for invasive candidiasis, neutropenic fever, and invasive Aspergillus if patient cannot
tolerate other therapies. Recent IDSA candidiasis guidelines recommend echinocandin as preferred
initial therapy for both proven and empiric therapy.
3. Dose (available only as an intravenous formulation)
a. Caspofungin: 70 mg once, followed by 50 mg once daily
b. Micafungin: 100 mg once daily
c. Anidulafungin: 200 mg once, followed by 100 mg once daily
4. Adverse reactions
a. Well tolerated with minimal GI adverse effects
b. Minimal drug interactions
i. Avoid caspofungin coadministration with cyclosporine and tacrolimus.
ii. Avoid micafungin coadministration with nifedipine and sirolimus.
iii. Avoid administering anidulafungin with metronidazole (disulfiram reaction).
5. Empiric treatment of critically ill patients: A recent large, multicenter study evaluated the use of empiric
micafungin in immunocompromised critically ill patients with sepsis, multiple Candida colonization,
multiorgan failure, and exposure to broad-spectrum antibiotics. The study showed no improvement
in fungal infection-free survival compared with placebo. Until further evidence is available, empiric
echinocandins should not be administered routinely.

ACCP Updates in Therapeutics® 2022: Critical Care Pharmacy Preparatory Review and Recertification Course

289
Infectious Diseases II

D. Flucytosine
1. Mechanism of action: Converted by fungal enzymes to fluorouracil, which disrupts fungal RNA and
DNA synthesis
2. Spectrum of activity
a. Candida spp.
b. Cryptococcus: Treatment of choice (in conjunction with amphotericin) for Cryptococcus meningitis
3. Dose
a. 25 mg/kg every 6 hours
b. Renal adjustments necessary
c. Well absorbed: Bioavailability 80%–90%
d. Available only as an oral formulation in the United States
e. Therapeutic concentrations: 25–100 mcg/mL
4. Adverse reactions: Bone marrow suppression, particularly with supratherapeutic concentrations

Patient Case

8. A 66-year-old woman (height 66 inches, weight 75 kg) is admitted to the medical ICU for dehydration and
acute kidney injury. The patient recently received an allogeneic bone marrow transplant and has not yet
engrafted. She has been pancytopenic for 12 days. On day 5 of the medical ICU stay, the patient devel-
ops acute respiratory distress requiring intubation. Bronchoalveolar lavage is done, which eventually grows
Aspergillus fumigatus, and the patient is given a diagnosis of invasive pulmonary aspergillosis. The patient’s
current medications include tacrolimus, corticosteroids, and fluconazole fungal prophylaxis. Her current
relevant laboratory values are as follows: WBC 0.2 x 103 cells/mm3, lactate 1.5 mmol/L, and SCr 3.4 mg/dL.
Which antifungal therapy is most appropriate?
A. Amphotericin 50 mg intravenously once daily
B. Isavuconazole load followed by 200 mg intravenously every 8 hours
C. Caspofungin load followed by 50 mg intravenously daily
D. Voriconazole load followed by 300 mg intravenously every 12 hours

ACCP Updates in Therapeutics® 2022: Critical Care Pharmacy Preparatory Review and Recertification Course

290
Infectious Diseases II

REFERENCES

Quality Improvements 11. Rollins KE, Javanmard-Emamghissi H, Acheson


1. Allegranzi B, Bischoff P, de Jonge S, et al. New AG, et al. The role of oral antibiotic preparation
WHO recommendations on preoperative mea- in elective colorectal surgery: a meta-analysis. Ann
sures for surgical site infection prevention: an Surg 2019;270:43-58.
evidence-based global perspective. Lancet Infect 12. Shiloach M, Ingraham AM, Weigelt JA, et al.
Dis 2016;16:e276-e287. Prevention of catheter-related bloodstream infec-
2. Anderson D, Podgorny K, Berríos-Torres SI, et tions. ACS NSQIP Best Practice Guidelines
al. Strategies to prevent surgical site infections in 2009;1-24.
acute care hospitals: 2014. Update Infect Control 13. Specifications Manual for National Hospital
Hosp Epidemiol 2014;35:605-27. Inpatient Quality Measures version 5.0b.
3. Berrios-Torres SI, Umscheid CA, Bratzler DW,
et al. Centers for Disease Control and Prevention Bacterial Meningitis
guideline for the prevention of surgical site infec- 1. De Gans J, Van De Beek D; European
tion, 2017. JAMA Surg 2017;152:784-91. Dexamethasone in Adulthood Bacterial
4. Bratzler DW, Dellinger EP, Olsen KM, et al. Meningitis Study Investigators. Dexamethasone
Clinical practice guidelines for antimicrobial in adults with bacterial meningitis. N Engl J Med
prophylaxis in surgery. Am J Health Syst Pharm 2002;347:1549-56.
2013;70:195-283. 2. Tunkel AR, Hartman BJ, Kaplan SL, et al. Practice
5. CDC Bloodstream Infection Event (Central Line- guidelines for the management of bacterial menin-
Associated Bloodstream Infection and Non-central gitis. Clin Infect Dis 2004;39:1267-84.
Line-Associated Bloodstream Infection). January 3. Tunkel AR, Hasbun R, Bhimraj A, et al. 2017
2020. Available at https://www.cdc.gov/nhsn/ Infectious Diseases Society of America’s clinical
PDFs /pscManual /4PSC_CLABScurrent.pdf. practice guidelines for healthcare-associated ven-
Accessed August 24, 2020. triculitis and meningitis. Clin Infect Dis 2017 Feb
6. CDC Urinary Tract Infection (Catheter-Associated 14. [Epub ahead of print]
Urinary Tract Infection [CAUTI] and Non- 4. Van De Beek D, De Gans J, Tunkell AR, et al.
Catheter-Associated Urinary Tract Infection [UTI]) Community-acquired bacterial meningitis in
and Other Urinary System Infection [USI] Events. adults. N Engl J Med 2006;354:44-53.
January 2020. Available at https://www.cdc.gov/ 5. Van De Beek D, De Gans J, Spanjaard L, et
nhsn/PDFs/pscManual/7pscCAUTIcurrent.pdf. al. Clinical features and prognostic factors in
Accessed August 24, 2020. adults with bacterial meningitis. N Engl J Med
7. Frencher SK, Esnaola NF. Prevention of catheter- 2004;351:1849-59.
associated urinary tract infections. ACS NSQIP 6. Van De Beek D, Drake JM, Tunkell AR.
Best Practice Guidelines 2009;1-28. Nosocomial bacterial meningitis. N Engl J Med
8. Grant MC, Yang D, Wu CL, et al. Impact of 2010;362:146-54.
enhanced recovery after surgery and fast track sur-
gery pathways on healthcare-associated infections. Antimicrobial Stewardship
Ann Surg 2017;265:68-79. 1. Barlam TF, Cosgrove SE, Abbo LM, et al.
9. Ingraham AM, Shiloach M, Dellinger EP, et al. Implementing an Antibiotic Stewardship Program:
Prevention of surgical site infections. ACS NSQIP Guidelines by the Infectious Diseases Society
Best Practice Guidelines 2009;1-24. of America and the Society for Healthcare
10. Migaly J, Bafford AC, Francone TD, et al. The Epidemiology of America. Clin Infect Dis
American Society of Colon and Rectal Surgeons 2016;62:e51-77.
Clinical Practice Guidelines for the use of bowel 2. Dellit TH, Owens RC, McGowan JE Jr, et al.
preparation in elective colon and rectal surgery. Infectious Diseases Society of America and the
Dis Colon Rectum 2019;62:3-8. Society for Healthcare Epidemiology of America
guidelines for developing an institutional program

ACCP Updates in Therapeutics® 2022: Critical Care Pharmacy Preparatory Review and Recertification Course

291
Infectious Diseases II

to enhance antimicrobial stewardship. Clin Infect 6. Forrest GN, Mankes K, Jabra-Rizk MA, et al.
Dis 2007;44:159-77. Peptide nucleic acid fluorescence in situ hybrid-
3. Drew RH, White R, MacDougall C, et al. Insights ization-based identification of Candida albicans
from the Society of Infectious Diseases Pharmacists and its impact on mortality and antifungal therapy
on antimicrobial stewardship guidelines from the costs. J Clin Microbiol 2006;44:3381-3.
Infectious Diseases Society of America and the 7. Forrest GN, Mehta S, Weekes E, et al. Impact of
Society for Healthcare Epidemiology of America. rapid in situ hybridization testing on coagulase-
Pharmacotherapy 2009;29:593-607. negative staphylococci positive blood cultures. J
4. Kaki R, Elligsen M, Walker S, et al. Impact of anti- Antimicrob Chemother 2006;58:154-8.
microbial stewardship in critical care: a systematic 8. Goff DA, Jankowski C, Tenover FC. Using rapid
review. J Antimicrob Chemother 2011;66:1223-30. diagnostic tests to optimize antimicrobial selec-
5. Owens RC Jr. Antimicrobial stewardship: appli- tion in antimicrobial stewardship programs.
cation in the intensive care unit. Infect Dis Clin Pharmacotherapy 2012;32:677-87.
North Am 2009;23:683-702. 9. Hochreiter M, Kohler T, Schweiger AM, et al.
6. Policy statement on antimicrobial stewardship Procalcitonin to guide duration of antibiotic therapy
by the Society for Healthcare Epidemiology of in intensive care patients: a randomized prospec-
America (SHEA), the Infectious Diseases Society tive controlled trial. Crit Care 2009;13:R83.
of America (IDSA), and the Pediatric Infectious 10. Holtzman C, Whitney D, Barlam T, et al.
Diseases Society (PIDS). Infect Control Hosp Assessment of impact of peptide nucleic acid
Epidemiol 2012;33:322-7. fluorescence in situ hybridization for rapid identi-
fication of coagulase-negative staphylococci in the
Rapid Diagnostic Tests absence of antimicrobial stewardship intervention.
1. Beal SG, Ciurca J, Smith G, et al. Evaluation of the J Clin Microbiol 2011;49:1581-2.
Nanosphere Verigene gram-positive blood culture 11. Jensen JU, Hein L, Lundgren B, et al. Procalcitonin-
assay with the VersaTREK blood culture system guided interventions against infections to increase
and assessment of possible impact on selected early appropriate antibiotics and improve survival
patients. J Clin Microbiol 2013;51:3988-92. in the intensive care unit: a randomized trial. Crit
2. Bouadma L, Luyt CE, Tubach F, et al. Use of Care Med 2011;39:2048-58.
procalcitonin to reduce patients’ exposure to anti- 12. Kothari A, Morgan M, Haake DA. Emerging tech-
biotics in intensive care units (PRORATA trial): nologies for rapid identification of bloodstream
a multicentre randomised controlled trial. Lancet pathogens. Clin Infect Dis 2014;59:272-8.
2009;375:463-74. 13. Lam SW, Bauer SR, Fowler R, et al. Systematic
3. Clancy CJ, Nguyen MH. Finding the “missing review and meta-analysis of procalcitonin-guidance
50%” of invasive candidiasis: how nonculture versus usual care for antimicrobial management in
diagnostics will improve understanding of disease critically ill patients: focus on subgroups based on
spectrum and transform patient care. Clin Infect antibiotic initiation, cessation, or mixed strategies.
Dis 2013;56:1284-92. Crit Care Med 2018;46:684-90.
4. Clerc O, Prod’hom G, Vogne C, et al. Impact 14. Ledeboer NA, Lopansri BK, Dhiman N, et al.
of matrix-assisted laser desorption ionization Identification of gram-negative bacteria and genetic
time-of-flight mass spectrometry on the clinical resistance determinants from positive blood cul-
management of patients with Gram-negative bac- ture broths by use of the Verigene gram-negative
teremia: a prospective observational study. Clin blood culture multiplex microarray-based molecu-
Infect Dis 2013;56:1101-7. lar assay. J Clin Microbiol 2015;53:2460-72.
5. de Jong E, van Oers JA, Beishuizen A, et al. 15. Ly T, Gulia J, Pyrgos V, et al. Impact upon clinical
Efficacy and safety of procalcitonin guidance in outcomes of translation of PNA FISH-generated
reducing the duration of antibiotic treatment in laboratory data from the clinical microbiology
critically ill patients: a randomised, controlled, bench to bedside in real time. Ther Clin Risk
open-label trial. Lancet Infect Dis 2016;16:819-27. Manag 2008;4:637-40.

ACCP Updates in Therapeutics® 2022: Critical Care Pharmacy Preparatory Review and Recertification Course

292
Infectious Diseases II

16. Mylonakis E, Clancy CJ, Ostrosky-Zeichner L, et undifferentiated infection or suspected sepsis. A


al. T2 magnetic resonance assay for the rapid diag- randomized controlled trial. Am J Respir Crit Care
nosis of candidemia in whole blood: a clinical trial. Med 2014;190:1102-10.
Clin Infect Dis 2015;60:892-9. 28. Stolz D, Smyrnios N, Eggimann P, et al.
17. Nobre V, Harbarth S, Graf JD, et al. Use of pro- Procalcitonin for reduced antibiotic exposure in
calcitonin to shorten antibiotic treatment duration ventilator-associated pneumonia: a randomised
in septic patients: a randomized trial. Am J Respir study. Eur Respir J 2009;34:1364-75.
Crit Care Med 2008;177:498-505. 29. Timbrook TT, Morton JB, McConeghy KW, et al.
18. Ostrosky-Zeichner L. Invasive mycoses: diagnostic The effect of molecular rapid diagnostic testing on
challenges. Am J Med 2012;125:S14-24. clinical outcomes in bloodstream infections: a sys-
19. Ostrosky-Zeichner L, Shoham S, Vazquez J, et tematic review and meta-analysis. Clin Infect Dis
al. MSG-01: a randomized, double-blind, pla- 2017;64:15-23.
cebo-controlled trial of caspofungin prophylaxis
followed by preemptive therapy for invasive candi- Interpreting Susceptibility Reports
diasis in high-risk adults in the critical care setting. 1. Jorgensen JH, Ferraro MJ. Antimicrobial sus-
Clin Infect Dis 2014;58:1219-26. ceptibility testing: a review of general principles
20. Peman J, Zaragoza R. Current diagnostic and contemporary practices. Clin Infect Dis
approaches to invasive candidiasis in critical care 2009;49:1749-55.
settings. Mycoses 2010;53:424-33. 2. Kuper KM, Boles DM, Mohr JF, et al. Antimicrobial
21. Perry JD, Freydiere AM. The application of chro- susceptibility testing: a primer for clinicians.
mogenic media in clinical microbiology. J Appl Pharmacotherapy 2009;29:1326-43.
Microbiol 2007;103:2046-55. 3. Kuper KM, Coyle EA, Wanger A. Antifungal
22. Rhodes A, Evans LE, Alhazzani W, et al. Surviving susceptibility testing: a primer for clinicians.
Sepsis Campaign: international guidelines for Pharmacotherapy 2012;32:1112-22.
management of sepsis and septic shock: 2016. Crit
Care Med 2017;45:486-552. Mechanisms of Antibacterial Resistance and
23. Sango A, McCarter YS, Johnson D, et al. Treatment of Multidrug-Resistant Pathogens
Stewardship approach for optimizing antimicrobial 1. Akova M, Daikos GL, Tzouvelekis L, et al.
therapy through use of a rapid microarray assay on Interventional strategies and current clini-
blood cultures positive for Enterococcus species. J cal experience with carbapenemase-producing
Clin Microbiol 2013;51:4008-11. gram-negative bacteria. Clin Microbiol Infect
24. Schroeder S, Hochreiter M, Koehler T, et al. 2012;18:439-48.
Procalcitonin (PCT)-guided algorithm reduces 2. Bass SN, Bauer SR, Neuner EA, et al. Impact of
length of antibiotic treatment in surgical inten- combination antimicrobial therapy on mortality
sive care patients with severe sepsis: results of a risk for critically ill patients with carbapenem-resis-
prospective randomized study. Langenbecks Arch tant bacteremia. Antimicrob Agents Chemother
Surg 2009;394:221-6. 2015;59:3748-53.
25. Schuetz P, Birkhahn R, Sherwin R, et al. Serial 3. Boucher HW, Wilcox M, Talbot GH, et al.
procalcitonin predicts mortality in severe sepsis Once-weekly dalbavancin versus daily conven-
patients: results from the Multicenter Procalcitonin tional therapy for skin infection. N Engl J Med
MOnitoring SEpsis (MOSES) study. Crit Care 2014;370:2169-79.
Med 2017;45:781-9. 4. Burgess DS, Hall RG II, Lewis JS II, et al. Clinical
26. Schuetz P, Raad I, Amin DN. Using procalcito- and microbiologic analysis of a hospital’s extended-
nin-guided algorithms to improve antimicrobial spectrum beta-lactamase-producing isolates over a
therapy in ICU patients with respiratory infections 2-year period. Pharmacotherapy 2003;23:1232-7.
and sepsis. Curr Opin Crit Care 2013;19:453-60. 5. Cai Y, Wang R, Liang B, et al. Systematic review
27. Shehabi Y, Sterba M, Garrett PM, et al. and meta-analysis of the effectiveness and safety
Procalcitonin algorithm in critically ill adults with

ACCP Updates in Therapeutics® 2022: Critical Care Pharmacy Preparatory Review and Recertification Course

293
Infectious Diseases II

of tigecycline for treatment of infectious disease. 15. Motsch J, Murta de Oliveira C, Stus V, et al.
Antimicrob Agents Chemother 2011;55:1162-72. RESTORE-IMI 1: a multicenter, randomized,
6. Chopra T, Marchaim D, Veltman J, et al. Impact double-blind trial comparing efficacy and safety
of cefepime therapy on mortality among patients of imipenem-relebactam vs colistin plus imipenem
with bloodstream infections caused by extended- in patients with imipenem-nonsusceptible bacterial
spectrum-beta-lactamase-producing Klebsiella infections. Clin Infect Dis 2020;70:1799-808.
pneumoniae and Escherichia coli. Antimicrob 16. Nation RL, Garonzik SM, Thamlikitkul V, et al.
Agents Chemother 2012;56:3936-42. Dosing guidance for intravenous colistin in criti-
7. Corey GR, Kabler H, Mehra P, et al. Single-dose cally ill patients. Clin Infect Dis 2017;64:565-71.
oritavancin in the treatment of acute bacterial skin 17. Paterson DL, Ko WC, Von Gottberg A, et al.
infections. N Engl J Med 2014;370:2180-90. Antibiotic therapy for Klebsiella pneumoniae
8. Fowler VG Jr, Boucher HW, Corey GR, et al. bacteremia: implications of production of
Daptomycin versus standard therapy for bactere- extended-spectrum beta-lactamases. Clin Infect
mia and endocarditis caused by Staphylococcus Dis 2004;39:31-7.
aureus. N Engl J Med 2006;355:653-65. 18. Prasad P, Sun J, Danner RL, et al. Excess deaths
9. Freire AT, Melnyk V, Kim MJ, et al. Comparison associated with tigecycline after approval
of tigecycline with imipenem/cilastatin for the based on noninferiority trials. Clin Infect Dis
treatment of hospital-acquired pneumonia. Diagn 2012;54:1699-709.
Microbiol Infect Dis 2010;68:140-51. 19. Prokocimer P, De Anda C, Fang E, et al. Tedizolid
10. Harris PNA, Tambyah PA, Lye DC, et al. Effect of phosphate vs linezolid for treatment of acute
piperacillin-tazobactam vs meropenem on 30-day bacterial skin and skin structure infections:
mortality for patients with E coli or Klebsiella the ESTABLISH-1 randomized trial. JAMA
pneumoniae bloodstream infection and ceftriax- 2013;309:559-69.
one resistance: a randomized clinical trial. JAMA 20. Qureshi ZA, Paterson DL, Potoski BA, et al.
2018;320:984-94. Treatment outcome of bacteremia due to KPC-
11. Kang CI, Kim SH, Park WB, et al. Bloodstream producing Klebsiella pneumoniae: superiority of
infections due to extended-spectrum beta-lacta- combination antimicrobial regimens. Antimicrob
mase-producing Escherichia coli and Klebsiella Agents Chemother 2012;56:2108-13.
pneumoniae: risk factors for mortality and 21. Retamar P, Lopez-Cerero L, Muniain MA, et al.
treatment outcome, with special emphasis on anti- Impact of the MIC of piperacillin-tazobactam on
microbial therapy. Antimicrob Agents Chemother the outcome of patients with bacteremia due to
2004;48:4574-81. extended-spectrum-beta-lactamase-producing
12. Lee NY, Lee CC, Huang WH, et al. Cefepime Escherichia coli. Antimicrob Agents Chemother
therapy for monomicrobial bacteremia caused by 2013;57:3402-4.
cefepime-susceptible extended-spectrum beta- 22. Rodriguez-Bano J, Navarro MD, Retamar P, et al.
lactamase-producing Enterobacteriaceae: MIC beta-Lactam/beta-lactam inhibitor combinations
matters. Clin Infect Dis 2013;56:488-95. for the treatment of bacteremia due to extended-
13. Liscio JL, Mahoney MV, Hirsch EB. Ceftolozane/ spectrum beta-lactamase-producing Escherichia
tazobactam and ceftazidime/avibactam: two novel coli: a post hoc analysis of prospective cohorts.
beta-lactam/beta-lactamase inhibitor combination Clin Infect Dis 2012;54:167-74.
agents for the treatment of resistant gram-negative 23. Rybak MJ. Resistance to antimicrobial agents: an
bacterial infections. Int J Antimicrob Agents. June update. Pharmacotherapy 2004;24:203S-15S.
14, 2015. 24. Rybak MJ, Le J, Lodise TP, et al. Therapeutic
14. Liu C, Bayer A, Cosgrove SE, et al. Clinical prac- monitoring of vancomycin for serious methicil-
tice guidelines by the Infectious Diseases Society lin-resistant Staphylococcus aureus infections:
of America for the treatment of methicillin-resis- a revised consensus guideline and review by the
tant Staphylococcus aureus infections in adults American Society of Health-System Pharmacists,
and children: executive summary. Clin Infect Dis the Infectious Diseases Society of America, the
2011;52:285-92. Pediatric Infectious Diseases Society, and the

ACCP Updates in Therapeutics® 2022: Critical Care Pharmacy Preparatory Review and Recertification Course

294
Infectious Diseases II

Society of Infectious Diseases Pharmacists. Am J aureus nosocomial pneumonia: a randomized,


Health-Syst Pharm 2020;77:835-64. controlled study. Clin Infect Dis 2012;54:621-9.
25. Solomkin J, Evans D, Slepavicious A, et al. 35. Yahav D, Lador A, Paul M, et al. Efficacy and safety
Assessing the efficacy and safety of eravacycline of tigecycline: a systematic review and meta-analy-
vs ertapenem in complicated intra-abdominal sis. J Antimicrob Chemother 2010;66:1963-71.
infections in the investigating gram-negative infec-
tions treated with eravacycline (IGNITE 1) Immunocompromised Patients
trial: a randomized clinical trial. JAMA Surg 1. Baden LR, Bensinger W, Angarone M, et al.
2017;152:224-32. NCCN Guidelines, Version 2.2015. Prevention
26. Tamma PD, Girdwood SC, Gopaul R, et al. The and Treatment of Cancer-Related Infections. 2015.
use of cefepime for treating AmpC beta-lactamase- Available at www.nccn.org/professionals/physi-
producing Enterobacteriaceae. Clin Infect Dis cian_gls/f_guidelines.asp. Accessed May 22, 2017.
2013;57:781-8. 2. Fishman JA. Infection in solid-organ transplant
27. Tasina E, Haidich AB, Kokkali S, et al. Efficacy recipients. N Engl J Med 2007;357:2601-14.
and safety of tigecycline for the treatment of infec- 3. Freifeld AG, Bow EJ, Sepkowitz KA, et al.
tious diseases: a meta-analysis. Lancet Infect Dis Clinical practice guideline for the use of antimi-
2011;11:834-44. crobial agents in neutropenic patients with cancer:
28. Tumbarello M, Viale P, Viscoli C, et al. Predictors 2010 update by the Infectious Diseases Society of
of mortality in bloodstream infections caused by America. Clin Infect Dis 2011;52:e56-93.
Klebsiella pneumoniae carbapenemase-producing
K. pneumoniae: importance of combination ther- HIV/AIDS in Critically Ill Patients
apy. Clin Infect Dis 2012;55:943-50. 1. Panel on Antiretroviral Guidelines for Adults and
29. van Duin D, Cober E, Richter SS, et al. Tigecycline Adolescents. Guidelines for the Use of Antiretroviral
therapy for carbapenem-resistant Klebsiella pneu- Agents in HIV-1-Infected Adults and Adolescents.
moniae (CRKP) bacteriuria leads to tigecycline Washington, DC: Department of Health and
resistance. Clin Microbiol Infect 2014;20:O1117-20. Human Services. Available at http://aidsinfo.nih.
30. van Duin D, Lok JJ, Earley M, et al. Colistin versus gov/ContentFiles/AdultandAdolescentGL.pdf.
ceftazidime-avibactam in the treatment of infections Section accessed August 26, 2020
due to carbapenem-resistant Enterobacteriaceae. 2. Panel on Opportunistic Infections in HIV-
Clin Infect Dis 2018;66:163-71. Infected Adults and Adolescents. Guidelines for
31. Weiner LM, Abner S, Edwards JR, et al. the Prevention and Treatment of Opportunistic
Antimicrobial-resistant pathogens associated with Infections in HIV-Infected Adults and Adolescents:
healthcare-associated infections: summary of Recommendations from the Centers for Disease
data reported to the National Healthcare Safety Control and Prevention, the National Institutes of
Network at the Centers for Disease Control and Health, and the HIV Medicine Association of the
Prevention, 2015-2017. Infect Control Hosp Infectious Diseases Society of America. Available
Epidemiol 2020;41:1-18. at http://aidsinfo.nih.gov/contentfiles/lvguidelines/
32. Wu JY, Srinivas P, Pogue J. Cefiderocol: a novel adult_oi.pdf. Accessed May 22, 2017
agent for the management of multidrug-resis- 3. Rosen MJ, Narasimhan M. Critical care of
tant gram-negative organisms. Infect Dis Ther immunocompromised patients: human immuno-
2020;9:17-40. deficiency virus. Crit Care Med 2006;34:S245-50.
33. Wunderink RG, Giamarellos-Bourboulis EJ, 4. Tan DH, Walmsley SL. Management of persons
Rahav G, et al. Meropenem-vaborbactam vs. infected with human immunodeficiency virus
best available therapy for carbapenem-resistant requiring admission to the intensive care unit. Crit
Enterobacteriaceae infections: The TANGO Care Clin 2013;29:603-20.
II randomized clinical trial. Infect Dis Ther
2018;7:439-55. Antifungal Therapy
34. Wunderink RG, Niederman MS, Kollef MH, et al. 1. Limper AH, Knox KS, Sarosi GA, et al. An official
Linezolid in methicillin-resistant Staphylococcus American Thoracic Society statement: treatment

ACCP Updates in Therapeutics® 2022: Critical Care Pharmacy Preparatory Review and Recertification Course

295
Infectious Diseases II

of fungal infections in adult pulmonary and criti-


cal care patients. Am J Respir Crit Care Med
2011;183:96-128.
2. Miceli MH, Kauffman CA. Isavuconazole: A New
Broad-Spectrum Triazole Antifungal Agent. Clin
Infect Dis 2015;61:1558-65.
3. Pappas PG, Kauffman CA, Andes DR, et al.
Clinical Practice Guideline for the Management
of Candidiasis: 2016 Update by the Infectious
Diseases Society of America. Clin Infect Dis
2016;62:e1-50.
4. Timsit JF, Azoulay E, Schwebel C, et al. Empirical
micafungin treatment and survival without inva-
sive fungal infection in adults with ICU-acquired
sepsis, Candida colonization, and multiple organ
failure: the EMPIRICUS randomized clinical trial.
JAMA 2016;316:1555-64.

ACCP Updates in Therapeutics® 2022: Critical Care Pharmacy Preparatory Review and Recertification Course

296
Infectious Diseases II

ANSWERS AND EXPLANATIONS TO PATIENT CASES

1. Answer: D case, and antibiotics should be continued to cover the


The CDC definitions for CAUTIs, which are used by the most likely pathogens (Answer A is correct; Answer
CMS reporting system, were recently updated. Because B is incorrect). Because the patient is admitted with
this patient had his urinary catheter removed for more presumed community-acquired pneumonia, the combi-
than 24 hours before developing UTI symptoms, this nation of ceftriaxone and azithromycin is appropriate
case does not satisfy the definition of CAUTI (Answer D right now (Answers C and D are incorrect).
is correct; Answers A and C are incorrect). The current
CDC CAUTI definitions do not consider the results of a 5. Answer: B
urinalysis (Answer B is incorrect). This patient is being treated for an ESBL E. coli UTI,
in which the source of the infection (urinary catheter)
2. Answer: A is removed. The patient also has signs of response to
This patient presents with community-acquired men- piperacillin/tazobactam with resolution of leukocytosis
ingitis. The high opening pressure is suggestive of a and fever. In this case, given the low inoculum of the
bacterial etiology, of which the most prevalent pathogen infection, initial response, removal of source, and low
for this patient’s risk factors and age is S. pneumoniae. MIC, continuing piperacillin/tazobactam is the best
Administering dexamethasone before initiating anti- choice (Answer B is correct). This choice may only be
biotic therapy decreases mortality in patients with S. the case for UTIs, not other sources of infection (e.g.,
pneumoniae meningitis. (Answer A is correct) Awaiting bacteremias). Traditionally, it was widely believed
laboratory and microbiologic test results analysis may that ESBL infections had to be treated with carbapen-
cause inappropriate delays in therapy (Answer B is incor- ems. However, recent evidence suggests that β-lactam/
rect). In addition, although administering antimicrobials β-lactamase inhibitor combinations are a suitable option.
is a good idea, the beneficial effects associated with In the era of antimicrobial stewardship, the preservation
dexamethasone are diminished if administered after of carbapenem therapy should be regarded with high
antimicrobials have been initiated. At the very least, cor- importance (Answer A is incorrect). This current culture
ticosteroids should be administered concomitantly with has an MIC of 4 mcg/mL to cefepime, which, accord-
antimicrobials (Answer C and D are incorrect). ing to the updated CLSI guidelines, would be considered
resistant (Answer C is incorrect). Because the patient is
3. Answer: D responding to the current therapy, combination therapy
The PNA FISH technology is designed to identify is unnecessary (Answer D is incorrect).
pathogens earlier. It performs well, with good sensitivity
and specificity; hence, identification of C. parapsilosis 6. Answer: D
should not require verification from culture (Answer This patient is being treated for MRSA right-sided
A is incorrect). The drug of choice for the treatment of endocarditis, with the presumed source from her
C. parapsilosis is fluconazole (Answer D is correct). tunneled dialysis catheter. According to the IDSA
Although echinocandin, voriconazole, and amphotericin guidelines for the treatment of MRSA, treatment with
would all cover C. parapsilosis, they all have a broader vancomycin of an isolate with an MIC of 2 mcg/mL or
spectrum than is necessary, making them not the ideal less should be determined by the patient’s response to
choice (Answers A–C are incorrect). therapy, regardless of the actual MIC. This patient has
responded to therapy with the presumed clearance of
4. Answer: A blood cultures and resolution of signs and symptoms of
Although PCT is a promising biomarker for the detec- infection. Hence, continuing vancomycin and targeting a
tion of bacterial infections, it must be interpreted within trough of 15–20 mcg/mL is the most appropriate choice
the context of the patient’s clinical course. In this case, (Answer D is correct). Linezolid is not indicated for the
the patient has clear signs and symptoms of infection, treatment of endocarditis (Answer A is incorrect). Adding
together with signs of hypoperfusion. A low PCT should gentamicin and rifampin is considered in the medical
not be used to guide therapy discontinuation in this treatment of prosthetic valve endocarditis (Answer B is

ACCP Updates in Therapeutics® 2022: Critical Care Pharmacy Preparatory Review and Recertification Course

297
Infectious Diseases II

incorrect). Because the patient is responding to current


vancomycin therapy, an escalation to daptomycin is
inappropriate right now (Answer C is incorrect).

7. Answer: B
This patient is admitted to the ICU for an indication that
is unrelated to the patient’s underlying HIV. The assess-
ment of which prophylactics are necessary against OIs
depends on the patient’s underlying disease, history,
and CD4 + count. In this case, the patient has a CD4 +
count less than 100/mm3 and a history of cryptococcal
meningitis. Hence, prophylaxis should be administered
for toxoplasmosis, P. jiroveci, and Cryptococcus. The
best regimen for this patient is fluconazole 200 mg
daily and trimethoprim/sulfamethoxazole 1 DS tablet
once daily (Answer B is correct). The prophylaxis for
Mycobacterium avium complex is indicated for patients
with a CD4 + count of less than 50 cells/mm3, which is
unnecessary at this point (Answers A and C are incor-
rect). Although trimethoprim/sulfamethoxazole 1 DS
tablet thrice weekly is an option for prophylaxis, it is
not the best choice against toxoplasmosis (Answer D is
incorrect).

8. Answer: B
This patient has invasive pulmonary aspergillosis.
Usually, the treatment of choice is voriconazole.
However, in this case, the patient has acute kidney
injury with a CrCl less than 50 mL/minute/1.73 m2.
According to the package insert, voriconazole is
contraindicated in this case because of the possibility
of accumulation of cyclodextrin, the intravenous drug
carrier for voriconazole. Although the clinical relevance
of this accumulation is controversial, continued use
of a contraindicated therapy is inappropriate when
alternatives may be available (Answer D is incorrect).
Isavuconazole is a new triazole that was found to
be noninferior to voriconazole for the treatment of
aspergillosis and has improved water solubility, which
does not require it to be formulated with cyclodextrin
(Answer B is correct). Conventional amphotericin may
be a reasonable choice, but it will likely worsen the
patient’s acute kidney injury (Answer A is incorrect).
Echinocandins are not the ideal therapies for invasive
aspergillosis and should only be considered if there are
no other treatment options (Answer C is incorrect).

ACCP Updates in Therapeutics® 2022: Critical Care Pharmacy Preparatory Review and Recertification Course

298
Infectious Diseases II

ANSWERS AND EXPLANATIONS TO SELF-ASSESSMENT QUESTIONS

1. Answer: C empiric antibiotic therapy is indicated, including therapy


The prophylactic regimen for a patient without a with agents having empiric activity against suspected
β-lactam allergy who is undergoing cardiac surgery pathogens and the ability to safely achieve relevant CSF
should be cephalosporin administered within 60 min- concentrations. Cefepime and vancomycin are the most
utes of incision time, administered every 4 hours during appropriate options listed with consideration of CNS-
surgery, and continued for no more than 48 hours. In this specific dosing (Answer D is correct). Ceftriaxone does
case, the patient has obesity disorder and weighs more not cover nosocomial-acquired gram negatives such
than 120 kg; hence, the patient requires an initial dose of as Pseudomonas (Answers A and B are incorrect).
cefazolin 3 g (Answer C is correct). Given the patient’s Piperacillin/tazobactam would be reasonable according
weight, administering cefazolin 2 g initially is inappro- to the spectrum of antibacterial activity; however, poor
priate (Answers A and D are incorrect). Vancomycin CNS penetration of tazobactam limits the utility of this
is usually reserved for patients with a β-lactam allergy agent for meningitis and other CNS infections (Answer
(Answer B is incorrect). C is incorrect).

2. Answer: C 4. Answer: D
Although critical care pharmacists may not officially The presence of the CTX-M gene detected on E. coli by
be part of many antimicrobial stewardship teams, many rapid diagnostic testing usually signifies the presence of
of their daily clinical activities constitute antimicrobial ESBL. This may be why the patient has not yet responded
stewardship activities. These may include selecting the to piperacillin/tazobactam. The most appropriate action
most appropriate treatment regimen and advocating the at this point is to broaden the coverage to cover for
early de-escalation of antimicrobials. Even in the pres- potential ESBL-producing E. coli. Carbapenems remain
ence of a formalized antimicrobial stewardship team, the drug of choice for ESBL-producing organisms, par-
these activities are often complementary to the formal- ticularly in a patient who is hemodynamically unstable
ized activities of the team (Answer C is correct). Given (Answer D is correct). Extended-spectrum β-lactamases
the wide variations in clinical practice, it may not be are usually encoded on genes that carry resistance
feasible to include an infectious diseases–trained phar- against other classes of antimicrobials; hence, resistance
macist with every stewardship team. In that case, the to other antimicrobials is common; therefore, adding
activities and involvement of a critical care pharma- either aminoglycosides or fluoroquinolones may not be
cist may be even more crucial (Answer A is incorrect). appropriate (Answers B and C are incorrect). Although
Antimicrobial cycling has not consistently demonstrated at times ESBLs may be covered by cefepime, it must be
beneficial effect on antimicrobial resistance. (Answer determined by final AST (Answer A is incorrect).
B is incorrect). Studies have shown that antimicrobial
stewardship efforts in critically ill patients do not worsen 5. Answer: C
outcomes. Given the aggressive empiric antimicrobial E. cloacae are AmpC β-lactamase–producing
regimens commonly used in critically ill patients, anti- Enterobacterales. The use of ceftriaxone or extended-
microbial stewardship should be instituted to minimize spectrum penicillins (e.g., piperacillin and ticarcillin)
adverse effects and the emergence of resistance (Answer may select out derepressed mutants, which are capable
D is incorrect). of causing the hyperproduction of AmpC β-lactamases.
Derepressed mutants are capable of producing resistance
3. Answer: D against third-generation cephalosporins, monobactams,
This patient presents with a health care–associated and extended-spectrum penicillins. In this case, the
CNS infection, given the post-neurosurgical and device- patient was taking 10 days of ceftriaxone before a new
related etiology of the infection. The most common blood culture was growing lactose-positive gram-negative
pathogens include MRSA and multidrug-resistant bacilli. Because lactose-positive gram-negative bacilli
gram-negative organisms. In addition to neurosurgical are usually Enterobacterales, growing multidrug-resistant
management of the device (e.g., removal or revision), pathogens such as P. aeruginosa and Acinetobacter

ACCP Updates in Therapeutics® 2022: Critical Care Pharmacy Preparatory Review and Recertification Course

299
Infectious Diseases II

baumannii is less likely. Hence, the most likely resistance option (Answer A is incorrect). According to the HIV
mechanism in this patient is either selection of derepressed OI guideline, the second-line agent for the treatment of
mutants or acquisition of a pathogen with ESBL. Both of severe Pneumocystis jiroveci pneumonia is intravenous
these resistance mechanisms are adequately treated by a pentamidine. Because the patient had severe hypoxemia,
carbapenem. Hence, changing to a carbapenem pending adjunctive steroids should be administered (Answer B
final sensitivities is the most reasonable option (Answer is correct). Atovaquone and primaquine/clindamycin
C is correct). Both types of resistance mechanisms are regimens are usually reserved for patients with milder
capable of producing resistance against ceftazidime and Pneumocystis jiroveci pneumonia. Furthermore,
piperacillin/tazobactam (Answers A and B are incorrect). primaquine should not be administered to someone
Because the patient developed a new bacteremia while with a glucose-6-phosphate dehydrogenase deficiency
taking ceftriaxone, it is not reasonable to continue (Answers C and D are incorrect).
ceftriaxone alone (Answer D is incorrect).
8. Answer: B
6. Answer: B This patient has febrile neutropenia with no recovery of
The patient is only on day 4 of therapy from proven neutrophils. According to the IDSA febrile neutropenia
MRSA pneumonia. However, the patient developed guidelines, when a source of infection is identified, the
a bacteremia with gram-positive cocci in pairs and empiric antimicrobial therapy can be de-escalated to a
chains despite receiving systemic vancomycin therapy. more narrow-spectrum regimen according to the antibi-
The most likely culprit is a vancomycin-resistant otic susceptibility report. In this case, because the E. coli
Enterococcus sp. The medical team has already was pan-sensitive, it would be appropriate to de-escalate
discontinued vancomycin; therefore, the new therapy to a narrow-spectrum antimicrobial. The guidelines also
must cover both the MRSA pneumonia and the specify that antimicrobial therapies should be contin-
possibility of a vancomycin-resistant Enterococcus sp. ued for at least 14 days and until neutrophils are greater
Linezolid has good lung penetration and can adequately than 500 cells/mm3 (Answer B is correct). Although
cover vancomycin-resistant enterococci (Answer B is the patient continues to be febrile, an otherwise stable
correct). Daptomycin would provide adequate coverage patient with continued fevers rarely requires additional
for vancomycin-resistant enterococci, but because it is antimicrobial therapy according to the guidelines.
inactivated by lung surfactants, it is not a good option Hence, continuing more broad-spectrum therapy than
for the treatment of MRSA pneumonia (Answer A is necessary or adding other antimicrobials is unwarranted
incorrect). Ceftaroline covers MRSA and has good (Answers A, C, and D are incorrect).
lung penetration; however, it covers only vancomycin-
resistant E. faecalis, not E. faecium. Because, at this
point, the speciation of the gram-positive cocci in pairs
and chains is not available, ceftaroline is not the best
choice (Answer C is incorrect). Tigecycline does cover
MRSA and vancomycin-resistant enterococci; however,
given its large volume of distribution and relatively
low serum concentrations, it is not the ideal choice for
the treatment of bacteremia when other treatments are
available (Answer D is incorrect).

7. Answer: B
The patient’s history and clinical presentation suggest
Pneumocystis jiroveci pneumonia. It is severe enough
to warrant intubation, and the patient has a significant
alveolar-arterial oxygen gradient. The usual drug of choice
for such patients is trimethoprim/sulfamethoxazole, but
because this patient has a sulfa allergy, this is not an

ACCP Updates in Therapeutics® 2022: Critical Care Pharmacy Preparatory Review and Recertification Course

300
Fluids, Electrolytes,
Acid-Base Disorders, and
Nutrition Support
Diana Wells Mulherin, Pharm.D., BCNSP, BCCCP
Vanderbilt University Medical Center
Nashville, Tennessee
Fluids, Electrolytes, Acid-Base Disorders, and Nutrition Support

Fluids, Electrolytes,
Acid-Base Disorders, and
Nutrition Support
Diana Wells Mulherin, Pharm.D., BCNSP, BCCCP
Vanderbilt University Medical Center
Nashville, Tennessee

ACCP Updates in Therapeutics® 2022: Critical Care Pharmacy Preparatory Review and Recertification Course

303
Fluids, Electrolytes, Acid-Base Disorders, and Nutrition Support

Learning Objectives NA Sodium


NB Nitrogen balance
1. Describe normal fluid requirements, and identify NG Nasogastric
common patient conditions that alter fluid needs and NPO Nil per os (nothing by mouth)
homeostasis. NRS Nutrition risk screening (score)
2. Assess common electrolyte disorders in critically ill NUTRIC Nutrition Risk in the Critically Ill (score)
patients, and develop an appropriate treatment plan. Pco2 Partial pressure of carbon dioxide
3. Identify acid-base disorders, differentiate between PCWP Pulmonary capillary wedge pressure
the causative factors, and construct an appropriate PN Parenteral nutrition
treatment algorithm. Po2 Partial pressure of oxygen
4. Specify the appropriate route (parenteral or enteral) REE Resting energy expenditure
of nutrition administration, energy and protein SCR Serum creatinine
needs, and key micronutrients to be provided to a SIAD Syndrome of inappropriate antidiuresis
critically ill patient. SOFA Sequential organ failure assessment (score)
5. Develop an appropriate assessment of the tolerance, TBI Traumatic brain injury
safety, and efficacy of an enteral or parenteral nutri- UUN Urine urea nitrogen
tion regimen. WT Body weight

Abbreviations in This Chapter Self-Assessment Questions


Answers and explanations to these questions can be
ABG Arterial blood gas found at the end of this chapter.
ADH Anti-diuretic hormone
AG Anion gap 1. Which is the most appropriate indication for paren-
AKI Acute kidney injury teral nutrition (PN)?
APACHE Acute physiology and chronic health A. Severe anorexia.
evaluation (score) B. Lack of bowel sounds.
BEE Basal energy expenditure C. Ileus.
BG Blood glucose D. Gastric residual volume (GRV) of 300 mL.
BMI Body mass index
BUN Blood urea nitrogen 2. Which fluid option would most closely match the
CKD Chronic kidney disease composition of nasogastric (NG) fluid losses?
COPD Chronic obstructive pulmonary disease
A. 0.9% sodium chloride and potassium chloride
CRRT Continuous renal replacement therapy
20 mEq/L.
COVID-19 Coronavirus disease 2019
B. 0.45% sodium chloride and potassium chloride
CSWS Cerebral salt wasting syndrome
20 mEq/L.
CVP Central venous pressure
C. 5% dextrose in 0.225% sodium chloride and
DM Diabetes mellitus
potassium chloride 20 mEq/L.
ECF Extracellular fluid
D. Lactated Ringer solution.
EDVI End-diastolic volume index
EN Enteral nutrition
3. A patient with prolonged, intractable diarrhea
GRV Gastric residual volume
would most likely have excess losses of which trace
HCO3 Bicarbonate
mineral?
IBW Ideal body weight
ICF Intracellular fluid A. Zinc.
ICU Intensive care unit B. Copper.
ILE Injectable lipid emulsion C. Iodine.
K Potassium D. Manganese.

ACCP Updates in Therapeutics® 2022: Critical Care Pharmacy Preparatory Review and Recertification Course

304
Fluids, Electrolytes, Acid-Base Disorders, and Nutrition Support

4. A 70-year-old man admitted to the intensive care 7. A 45-year-old man (weight 78 kg) with a history
unit (ICU) for sepsis was recently given a diagnosis significant for hypertension and pancreatitis is
of syndrome of inappropriate antidiuretic hormone admitted to the ICU after operative management of
secretion. His serum sodium fell from 130 mEq/L necrotizing pancreatitis. He is given PN consisting
to 115 mEq/L during the past 3 days, and his course of 650 g of dextrose, 120 g of amino acids, and 55
was subsequently complicated by a seizure. Which g of injectable lipid emulsion (ILE) daily. Blood
would be the most appropriate treatment option? glucose (BG) measurements over the past 24 hours
A. Intravenous 0.9% sodium chloride. have been 180-325 mg/dL. Which change to this
B. Intravenous desmopressin acetate (DDAVP). patient’s nutrition regimen is most appropriate at
C. Intravenous 3% sodium chloride. this time?
D. Intravenous conivaptan. A. Initiate a sliding-scale insulin regimen with
insulin aspart every 4 hours.
5. Other than the absorption/infusion rate, which best B. Temporarily decrease the dextrose dose in the
explains why enteral potassium administration is PN to 200 g and advance to dextrose 650 g once
safer than parenteral potassium administration? blood glucose is less than 180 mg/dL.
A. Bioavailability of potassium is signifi- C. Decrease dextrose to 500 g and make up with
cantly lower with enteral versus parenteral remaining kilocalories by increasing protein to
administration. 160 g.
B. Feed-forward sensing of changes in mesen- D. Decrease dextrose to 315 g and reassess glyce-
teric potassium concentration increases urinary mic control over the next 24 hours.
potassium excretion.
C. Potassium chloride elixir is likely to cause diar- 8. A 48-year-old man is admitted to the trauma ICU
rhea and reduce potassium absorption. after a motorcycle collision. His injuries include
D. Wax matrix tablets sequester potassium release a subarachnoid hemorrhage, right pneumothorax,
throughout the gastrointestinal (GI) tract. multiple rib fractures, grade 5 liver laceration, right
sacral fracture, and transverse process fractures. His
6. A 40-year-old man (weight 60 kg) is admitted to the course is complicated by respiratory failure, acute
trauma ICU after a motor vehicle collision. He is kidney injury (AKI), and hyperglycemia. His labs
noted to have a serum magnesium concentration of today reveal serum sodium 141 mEq/L, chloride
1.2 mg/dL, and his family states that he has a his- 102 mEq/L, carbon dioxide 20 mEq/L, and lactate
tory of alcohol abuse (12–18 beers/day). He is given 2.6 mmol/L. His arterial blood gas values are as
magnesium sulfate 6 g intravenously for 6 hours by follows: pH 7.46, Pco2 31 mmHg, and HCO3 22
the primary service. His repeat serum magnesium mEq/L, Which of the following assessments of his
concentration on the following day is 1.8 mg/dL. acid-base status is correct?
Which would be the most appropriate treatment for A. Primary respiratory alkalosis only.
this patient? B. Primary respiratory alkalosis with underlying
A. No treatment is necessary because his serum metabolic acidosis.
magnesium concentration is normal. C. Primary metabolic alkalosis with underlying
B. If a repeat serum magnesium concentration is respiratory alkalosis.
2 mg/dL or greater, no additional magnesium D. Primary metabolic acidosis only.
therapy is indicated.
C. Supplemental calcium therapy should be given
concurrently with the magnesium therapy.
D. Additional magnesium therapy should be given
daily for the next 4–5 days.

ACCP Updates in Therapeutics® 2022: Critical Care Pharmacy Preparatory Review and Recertification Course

305
Fluids, Electrolytes, Acid-Base Disorders, and Nutrition Support

BPS Critical Care Pharmacy Specialist Examination Content Outline

This chapter covers the following sections of the Critical Care Pharmacy Specialist Examination Content Outline:
1. Domain 1: Clinical Knowledge and Application
a. Task 1: 1-3
b. Task 2: 2-4
c. Task 3: 1-7
d. Task 4: 1-5
e. Task 5: 1-3, 5, 6
f. Task 6: 1-7
g. Task 7: 1-7
h. Task 8: 3
2. Domain 2: Practice Management, Policy, and Quality Improvement
a. Task 1: 2, 3
3. Domain 3: Evidence-Based Medicine, Scholarship, Education, and Professional Development
a. Task 1: 3
b. Task 3: 1

ACCP Updates in Therapeutics® 2022: Critical Care Pharmacy Preparatory Review and Recertification Course

306
Fluids, Electrolytes, Acid-Base Disorders, and Nutrition Support

I.  FLUIDS AND ELECTROLYTES

A. General Overview
1. Body water compartments
a. Total body water (TBW): About 60% of body weight for men; about 50% of body weight for
women; lower percentage for those with obesity and for older adults (0.5 L/kg for men; 0.45 L/kg
for women)
b. About 60% of TBW is intracellular.
c. About 40% of TBW is extracellular water (about 75% is interstitial fluid; about 25% plasma volume).
d. Fluid compartments are separated by membranes that are freely permeable to water. The movement
of fluids through these compartments is due to hydrostatic pressure or osmotic pressure.
2. Estimating daily fluid requirements
a. 30–35 mL/kg (overestimates large person, underestimates small person)
b. 100 mL/kg for the first 10 kg, 50 mL/kg for the next 10 kg, and 20 mL/kg thereafter
c. Increased insensible losses occur with fever (around 10%–15% for every degree Celsius greater
than 37°C).
d. Fluids received from other sources should be considered when estimating a patient’s fluid needs to
avoid fluid overload.

Table 1. Effect of Body Temperature on Insensible Fluid Losses (Surgery 1968;64:154-64)


Rectal Temperature (°C) No. of Patients Mean Fluid Loss (mL/m2/day)
36.7–37.7 205 552
37.8–38.2 160 600
38.3–38.8 48 768
38.9–40 14 840

3. Estimating electrolyte requirements


a. Approximate electrolyte concentrations in the extracellular and intracellular fluids (ECFs and ICFs)
(Fluid, Electrolyte, and Acid-Base Disorders, Vol 1. New York: Churchill Livingstone, 1985:1-38)

Table 2. Electrolyte Concentrations in the ECF and ICF


Extracellular Fluid Intracellular Fluid
Electrolyte
(mEq/L) (mEq/L)
(plasma) (muscle)
Sodium 140 12
Potassium 4.5 160
Chloride 104 2
Bicarbonate 24 10
Calcium 5.0 (10 mg/dL) —
Magnesium 1.5 (1.8 mg/dL) 34
Phosphate 2 (3.5 mg/dL) 54

b. “Normal” daily requirements

ACCP Updates in Therapeutics® 2022: Critical Care Pharmacy Preparatory Review and Recertification Course

307
Fluids, Electrolytes, Acid-Base Disorders, and Nutrition Support

Table 3. “Normal” Daily Requirements (Hosp Pharm 2002;37:1336-42; Medicine (Baltimore) 1981;60:339-54; Crit
Care Med 1995;23:1504-11; Ann Rev Med 1981;32:245-59; Ann Surg 1983:197:1-6)
Sodium 50–150 mEq (1–2 mEq/kg)
Potassium 0.5–1.5 mEq/kg
Phosphorus 10–30 mmol
Magnesium 8–32 mEq
Calcium 10–15 mEq
Chloride a
80–120 mEq
Acetate a
80–120 mEq
a
Depending on the acid-base status of the patient.

c. Approximate electrolyte content of GI secretions (in milliequivalents per liter) (About Surgery:
A Clinical Approach. Philadelphia: Elsevier Health Sciences, 1996:5-17; Acta Chir Scand Suppl
1963:suppl 306:301-65)

Table 4. Electrolyte Content of GI Secretions


Average Daily Sodium Potassium Chloride Bicarbonate Magnesium
Fluid
Volume (mL) (mEq/L) (mEq/L) (mEq/L) (mEq/L) (mEq/L)
Stomach 1000–2000 60–90 10–15 100–130 — 0.9
Duodenum 400–600 140 5–10 90–120 80 —
Jejunum/Ileum 2000–2500 140 5–10 90–120 30–40 6–12
Colon < 300 60 20–30 50 — 6–12
Pancreas 600–800 140 5–10 75 115 0.4
Bile 300–600 140 5–10 100 30 1.1

d. Electrolyte composition of common intravenous solutions (in milliequivalents per liter)

Table 5. Electrolyte Composition of Common Intravenous Solutions


Sodium Potassium Chloride Bicarbonate Calcium Magnesium Osmolarity
Solutions
(mEq/L) (mEq/L) (mEq/L) (mEq/L) (mEq/L) (mEq/L) (mOsm/L)
5% dextrose in water — — — — — — 252
0.9% sodium chloride
154 — 154 — — — 308
(normal saline)
0.45% sodium chloride
77 — 77 — — — 154
(one-half normal saline)
5% dextrose in 0.225%
sodium chloride (5%
34 — 34 — — — 320
dextrose in one-fourth
normal saline)
3% sodium chloride
513 — 513 — — — 1026
(hypertonic saline)
Lactated Ringer solution 130 4 109 28 2.7 — 274
Plasma-Lyte
140 5 98 27 — 3 294–295
A/Normosol-R pH 7.4

ACCP Updates in Therapeutics® 2022: Critical Care Pharmacy Preparatory Review and Recertification Course

308
Fluids, Electrolytes, Acid-Base Disorders, and Nutrition Support

B. Water and Sodium Disorders


1. Dehydration: As evidenced by decreased urine output (unless patient has glycosuria or diuretic therapy),
increased blood urea nitrogen/serum creatinine ratio (BUN/SCr greater than 20), insufficient net fluid
balance (from nursing volume intake and output records), increased serum sodium, poor skin turgor,
dry mucous membranes, orthostatic hypotension, “contraction alkalosis,” increased losses
a. Fever
b. GI fluid losses (e.g., emesis, gastric suction, diarrhea, enterocutaneous fistula, surgical wounds,
drains)
2. Volume excess: As evidenced by the presence of peripheral/sacral/pulmonary edema, anasarca, rapid
weight gain, positive fluid balance
a. Excessive fluid and/or sodium intake
b. Impaired ability to excrete excess water and sodium (e.g., heart failure, cirrhosis with ascites, renal
failure)
3. Hyponatremia (see also the Neurocritical Care chapter for further reading on hyponatremia)
a. Classic evaluation
i. Exclude hyperglycemia, mannitol, and glycine for unmeasured effective osmoles (hypertonic
hyponatremia).
(a) Correct serum sodium for hyperglycemia (once the hyperglycemia is controlled, the
serum sodium will rise).
(b) For every 100-mg/dL increase in BG greater than 100 mg/dL, serum sodium will fall by
about 2.4 mEq/L (or vice-versa).
(c) Corrected serum Na = Measured serum Na + [(BG -100)/100 * 2.4] (Am J Med
1999;106:399-403).
ii. Exclude factitious/pseudo-hyponatremia (isotonic hyponatremia): Arguably still possible
during lipemia (triglycerides greater than 1000 mg/dL or hyperproteinemia (e.g., multiple
myeloma) if the serum is diluted under the assumption that the serum contains 7% solid-phase
particles before the assay (Intensive Care Med 2014;40:320-31).
iii. Evaluate ECF volume (increased, normal, decreased): Evaluate patient for edema, fluid
balance on fluid intake/output records, hemodynamic markers, chest radiography: Pulmonary
infiltrates without pneumonia, enlarged heart or evidence of diseases with decreased urinary
water/sodium excretion (e.g., AKI, congestive heart failure, cirrhosis with ascites).
iv. Consider using urine sodium and osmolality for hypotonic (serum osmolality less than
280 mOsm/kg) hyponatremia in conjunction with volume status (hypovolemic, euvolemic,
hypervolemic):

ACCP Updates in Therapeutics® 2022: Critical Care Pharmacy Preparatory Review and Recertification Course

309
Fluids, Electrolytes, Acid-Base Disorders, and Nutrition Support

Table 6. Comparative Features of Hypotonic Hyponatremia (Intensive Care Med 2014;40:320-31)


ECF volume
Hypervolemic Euvolemic Hypovolemic
status
Edema, large positive
No evidence of edema, fluid Poor skin turgor, dry mucous
fluid balance, pulmonary
Physiologic equilibrium, no evidence of membranes, decreased urine
infiltrates without pneu-
Findings dehydration or fluid overload, output, concentrated urine,
monia, enlarged heart,
normal hemodynamics tachycardia
increased PCWP, EDVI
Urine Osm
> 100 < 100 > 100 > 450
(mOsm/kg)
Urine Na
< 20 >20 < 20 > 20 < 20 >20
(mEq/L)
Diuretics,
CHF, Psychogenic Adrenal
Extra-renal
Cirrhosis polydipsia, SIAD, Cortisol insufficiency,
losses (e.g.,
Potential with Renal Excessive deficiency, Cerebral salt
GI fluid
etiologies ascites, failure hypotonic fluid Hypothyroidism, wasting, thiazide
losses), Third
Nephrotic intake, Drug-induced diuretics,
space losses
syndrome Beer potomania Salt wasting
nephropathy
Common etiologies for SIAD (Am J Kidney Dis 2008;52:144-53; Hosp Pharm 2002;37:1336-42):
Central nervous system (CNS) disorders – Trauma, stroke, infection, brain tumors
Malignancy – Small cell carcinoma of the lung, pancreatic carcinoma, lymphoma, Hodgkin disease, sarcoma
Pulmonary infection, respiratory failure with positive pressure ventilation
Endocrine disorders – Pituitary tumor, hypothyroidism, adrenal insufficiency
Stress response (surgery, trauma, thermal injury, sepsis, pain)
Drugs
CHF = congestive heart failure; EDVI = end-diastolic volume index; GI = gastrointestinal; PCWP = pulmonary capillary wedge pressure; SIAD = syndrome of
inappropriate antidiuresis.

Table 7. Drug-Induced SIAD (Am J Kidney Dis 2008;52:144-53)


Mechanism Examples
Amitriptyline, imipramine
Fluoxetine, sertraline, paroxetine
Thioridazine, trifluoperazine
Haloperidol
Increased hypothalamic Carbamazepine, oxcarbazepine, valproic acid
production of ADH Vincristine, vinblastine, cisplatin, carboplatin, cyclophosphamide,
ifosfamide
Nicotine
Bromocriptine
Monamine oxidase inhibitors
Increased sensitivity to or DDAVP, desmopressin
exogenous administration of Chlorpropamide
ADH Lamotrigine

ACCP Updates in Therapeutics® 2022: Critical Care Pharmacy Preparatory Review and Recertification Course

310
Fluids, Electrolytes, Acid-Base Disorders, and Nutrition Support

Table 7. Drug-Induced SIAD (Am J Kidney Dis 2008;52:144-53) (continued)


Mechanism Examples
Opiates
Barbiturates
Mixed or uncertain
Nonsteroidal anti-inflammatory agents
Angiotensin-converting enzyme inhibitors
ADH = antidiuretic hormone.

v. If the patient has a TBI, it can be difficult to ascertain whether the patient has cerebral salt
wasting syndrome (CSWS) or SIAD.

Table 8. Comparison of Features of Hyponatremia Caused by CSWS vs. SIAD (Hosp Pharm 2002;37:1336-42)
CSWS SIAD
Decreased serum sodium Decreased serum sodium
Decreased ECF Normal or expanded ECF
Negative sodium balance Variable sodium balance
CVP/PCWP/EDVI decreased CVP/PCWP/EDVI normal or increased
Urine osmolality increased Urine osmolality increased
Urine sodium increased Urine sodium increased
CVP = central venous pressure; CSWS = cerebral salt wasting syndrome; ECF = extracellular fluid.

b. 2014 European Society of Endocrinology guidelines (Intensive Care Med 2014;40:320-31)


i. Exclude hyperglycemia and other causes of non-hypotonic hyponatremia.
ii. Evaluate urine sodium and osmolality (the guidelines suggest that these markers should be
assessed before ECF volume because the latter is difficult to determine in the critically ill
patient).
(a) The European guidelines also differ because they recommend a urine sodium concentration
of 30 mEq/L (instead of 20 mEq/L, as previously described) as the point of demarcation
for differentiating the etiology for hyponatremia.
(b) If urine osmolality is less than 100 mOsm/kg, the guidelines recommend accepting
relative excess water intake as a cause of the hypotonic hyponatremia.
iii. Assess ECF and arterial blood volume, diuretics, presence of kidney disease (Am J Med
2010;123:652-7).
c. Treatment of hyponatremia
i. Severe symptoms present (seizures, mental status changes) – Immediate treatment with
hypertonic saline therapy should be adjusted to achieve a serum Na concentration increase
of 5 mEq/L in the first hour, and limit to a total of 10 mEq/L increase during the first 24
hours and additional 8 mEq/L during every 24 hour thereafter until serum sodium reaches
130 mEq/L (Intensive Care Med 2014;40:320-31). More rapid correction could lead to central
pontine myelinolysis. Discontinue hypertonic saline when goal serum sodium change has been
achieved or symptoms have improved (whichever occurs first). Estimate sodium deficit with
the following equation: sodium deficit (mEq) = TBW x (140 − measured serum sodium); TBW
is estimated as 0.5 L/kg for women and 0.6 L/kg for men.
ii. Hypervolemic (ECF expanded) – Fluid and sodium restriction, diuretic therapy as needed.
Consider conivaptan or tolvaptan if other therapies fail.
iii. Hypovolemic (ECF reduced) and low urine sodium – Give sodium and fluids for volume
expansion (treat etiologies if possible); reduce diuretic therapy. Sodium and fluid administration
can be done using 0.9% sodium chloride, lactated Ringer solution, or PlasmaLyte/Normosol
intravenously.

ACCP Updates in Therapeutics® 2022: Critical Care Pharmacy Preparatory Review and Recertification Course

311
Fluids, Electrolytes, Acid-Base Disorders, and Nutrition Support

iv. Euvolemic (ECF normal) – Consider syndrome of inappropriate antidiuresis or secondary


adrenal insufficiency – Fluid restriction first (if clinically appropriate); free water restriction
with use of 0.9% sodium chloride solution with or without diuretic therapy. Consider hypertonic
saline for acute or severe symptoms. Consider salt tablets or demeclocycline to maintain
normal serum sodium concentrations. Consider conivaptan or tolvaptan if other measures are
not effective.
4. Hypernatremia: Reflects a deficit of water compared with total body sodium. Therapy should be
adjusted to achieve a serum sodium concentration decrease of 6–8 mEq/L per day; no faster than 10–12
mEq/L per day to reduce the risk of cerebral edema.
a. Evaluation
i. Evaluate ECF volume status (increased, normal, decreased). Increased ECF volume is
caused by excessive sodium intake (oral salt tablets, hypertonic saline, 0.9% sodium chloride
solution, lactated Ringer solution, PlasmaLyte/Normosol). This is the least common cause of
hypernatremia. Decreased ECF (hypovolemia, dehydration) results from excessive losses of
both water and sodium.
ii. Consider urine sodium together with volume status to determine cause and treatment.
b. Treatment
i. Hypervolemic hypernatremia – Treated with diuretics, sodium restriction, water replacement,
and/or renal replacement therapy
ii. Euvolemic hypernatremia – Treated with free water replacement
• For normal or increased ECF volume, estimate free water deficit with the Adrogue-Madias
equation (N Engl J Med 2000;342:1493-9): 0.6 x Wt (kg) x [serum sodium/140 – 1] (use 0.5 x
Wt (kg) for women). Correct deficit over 2–3 days. The equation often underestimates TBW
deficit and should only be used a gross estimate for water deficit (Am J Clin Nutr 2013;87:79-
85). This is typically accomplished using enteral free water boluses (e.g., 200–300 mL every
4–6 hours) if the patient has a nasogastric feeding or suction tube. Do not give enteral water
boluses if the patient has a post-pyloric tube because it causes cramping and diarrhea. Rare
cases of bowel necrosis post–large water boluses have also occurred when administered
directly into the small bowel (J Parenter Enteral Nutr 2004;28:27-9). If the enteral route is
not possible or the patient is intolerant of enteral water boluses, intravenous dextrose 5% in
water or dextrose 5% in 0.225% sodium chloride can be used.
iii. Hypovolemic hypernatremia – Initially treated with volume expansion to restore hemodynamic
stability because total body sodium and volume is low. If hypernatremia is still present once
the patient is hemodynamically stable, free water replacement can be initiated as stated earlier.
iv. Clinical evaluation during volume repletion is of paramount importance. Serum sodium
should be checked routinely (e.g., every 6–12 hours) to avoid overcorrection. All sources of
excess fluid losses should be monitored and replaced with the appropriate fluid, if indicated.

Table 9. Comparative Features of Hypernatremia (Nutr Clin Pract 2008;23:108-21)


ECF Volume Status Hypervolemic Euvolemic Hypovolemic
Physiologic findings See Table 6
Urine sodium (mEq/L) > 20 Varies < 20 > 20
Excessive sodium Extrarenal losses Extrarenal Renal losses
intake from intravenous (insensible losses), losses (fever, (diuretics,
or oral, mineralocorti- Renal losses diarrhea, glycosuria,
coid excess (diabetes insipidus) respiratory kidney failure/
Potential etiologies losses) injury), diabetic
ketoacidosis,
hyperosmolar
hyperglycemic
syndrome

ACCP Updates in Therapeutics® 2022: Critical Care Pharmacy Preparatory Review and Recertification Course

312
Fluids, Electrolytes, Acid-Base Disorders, and Nutrition Support

Patient Case

Questions 1–3 pertain to the following case.


A 55-year-old woman (70 kg) admitted to the ICU for pneumonia and respiratory failure develops a serum
sodium of 125 mEq/L on her fifth day of hospital admission. Her other laboratory values include a serum
potassium of 4.6 mEq/L, chloride (Cl) 100 mEq/L, total carbon dioxide (CO2) content 24 mEq/L, BUN 20 mg/
dL, serum creatinine (SCr) 1.1 mg/dL, and glucose 167 mg/dL. She currently receives a 1-kcal/mL, 62 g of
protein/L enteral feeding formula at 60 mL/hour and a 5% dextrose in 0.45% sodium chloride infusion at 25
mL/hour. Her fluid balance has ranged from +300 to +600 mL/day during the past 3 days. She has no evidence
of any significant amount of edema. Her measured serum osmolality is 265 mOsm/kg, urine osmolality is 490
mOsm/kg, and urine sodium is 67 mEq/L.

1. Which is the most likely etiology for the patient’s hyponatremia?


A. Factitious hyponatremia
B. Adrenal insufficiency
C. Cerebral salt wasting
D. SIAD

2. Which would be the most appropriate treatment for this woman?


A. Give sodium chloride tablets 1 g three times daily.
B. Limit fluids.
C. Change the intravenous fluid to 0.9% sodium chloride.
D. Provide a short-term intravenous infusion of 3% sodium chloride.

3. Which change in the enteral feeding formula would be best for this patient?
A. Add sodium chloride 100 mEq/L to the current formula.
B. Change the formula to a fish oil–enriched product.
C. Change the formula to a low-carbohydrate, high-fat product.
D. Change the formula to a 2-kcal/mL formula, and decrease the rate.

C. Disorders of Potassium Homeostasis


1. Potassium homeostasis overview
a. 98% intracellular
b. Total body stores: 35–50 mEq/kg in normal healthy adults; 25–30 mEq/kg if significantly
undernourished
c. Normal serum concentration: 3.5–5.2 mEq/L
d. Serum concentration can be influenced by changes in pH (for every 0.1 increase in arterial pH,
serum potassium will decrease by around 0.6 mEq/L [range 0.4–1.3 mEq/L]) (J Clin Invest
1956;35:935-9), and vice versa. This occurs because of potassium exchanging with hydrogen by
the H+/K+-ATPase pump.
e. Average daily requirement: About 0.5–1.5 mEq/kg
f. Kidney is primary route of elimination.
g. Losses can be extensive with severe diarrhea or body fluid drainages (see Table 4).
h. Magnesium status can influence potassium homeostasis (J Am Soc Nephrol 2007;18:2649-52; Crit
Care Med 1996;24:38-45; Arch Intern Med 1992;152:40-5).
i. Magnesium serves as a cofactor for the Na-K-ATPase pump.
ii. Magnesium closes potassium channels in distal nephron.

ACCP Updates in Therapeutics® 2022: Critical Care Pharmacy Preparatory Review and Recertification Course

313
Fluids, Electrolytes, Acid-Base Disorders, and Nutrition Support

2. Hypokalemia
a. Definition: Serum potassium less than 3.5 mEq/L, though most ICUs empirically prefer to keep
patients at 4.0 mEq/L or greater, if possible.
b. Signs and symptoms: Weakness, cramps, cardiac arrhythmias (ST depression, QT prolongation,
flat T wave, U wave). If severe hypokalemia (e.g., serum potassium less than 2 mEq/L): flaccid
paralysis, ileus.
c. Etiologies:
i. Inadequate intake (rare; kidneys can usually adapt)
ii. Increased losses
(a) GI fluid losses (e.g., diarrhea, fistula, drainages)
(b) Hypomagnesemia
(c) Medications (diuretics, amphotericin B, mineralocorticoid excess, cisplatin, extended-
spectrum penicillins such as piperacillin, ticarcillin)
(d) Polyuria (diabetes insipidus)
(e) Renal potassium excretion (type I/distal and type II/proximal renal tubular acidosis)
(f) Diabetic ketoacidosis
iii. Increased requirements (building of new muscle/tissue – refeeding syndrome)
iv. Extracellular to intracellular shift
(a) Medications (β-adrenergic agonists, including albuterol, sodium bicarbonate or other
alkalinizing agents; insulin)
(b) Acute alkalemia
(c) Hypothermia
(d) Pentobarbital
d. Treatment:
i. Treat, alleviate, or reduce the potential etiologies for hypokalemia, if possible.
ii. Ensure that hypokalemia is not at least partly attributable to hypomagnesemia.
iii. The estimated deficit should be replaced during a period of 1–3 days (depending on the extent
of deficit; the larger the deficit, the longer the repletion period) by giving boluses and increasing
the potassium content in intravenous fluids and/or PN formulation.
iv. Enteral or oral potassium replacement is the preferred and safer route of delivery in
asymptomatic patients because of the time of absorption and feed-forward regulation of
potassium homeostasis (Ann Intern Med 2009;150:619-25); administration of potassium
chloride liquid directly into the small bowel (by a jejunal or duodenal feeding tube) should
be avoided because of its osmolality, which can lead to abdominal cramping, distension, and
diarrhea.
v. Intravenous repletion doses of potassium chloride or potassium phosphate should be given by
central vein. Potassium chloride can be given at 20 mEq/hour if the patient has continuous
electrocardiography (ECG) monitoring. Ten mEq/hour is safest if the patient is asymptomatic.
Peripheral intravenous solutions should not contain potassium chloride at more than 40–60
mEq/L in an effort to reduce the pain associated with the infusion of a concentrated potassium
chloride solution and to prevent inappropriate rapid and excessive potassium chloride dosing.
vi. Empiric intravenous potassium dosing. Institution or ICU-specific repletion protocols are
common and may need to be adjusted according to patient body size, renal function, ongoing
losses, and response to previous boluses. The following algorithm is an example of empiric
potassium repletion that is based on serum concentrations.

ACCP Updates in Therapeutics® 2022: Critical Care Pharmacy Preparatory Review and Recertification Course

314
Fluids, Electrolytes, Acid-Base Disorders, and Nutrition Support

Table 10. Empiric Intravenous Potassium Dosing


Serum Potassium Potassium Chloride Dosage
Laboratory Tests
(mEq/L) (mEq)a
3.5–3.9 40 to 60 mEq Obtain BMP, magnesium next AM
Obtain BMP, magnesium next AM; may obtain
3–3.4 80 mEq potassium 1–2 hours after repletion is completed,
especially if losses are thought to be high; reassess
Obtain repeat serum potassium 1–2 hours after
repletion is completed and reassess; may need one
2–2.9 120 mEq
or two additional boluses; repeat; check serum
magnesium next AM; reassess
a
Potassium phosphate may be considered in lieu of potassium chloride if concurrent hypokalemia and hypophosphatemia (very common in critically ill patients receiving
EN/PN). Thirty millimoles of potassium phosphate is equivalent to 44 mEq of potassium.
AM = morning; BMP = basic metabolic panel.

vii. The historical assumption of “a 0.5 to 0.6 mEq/L increase in serum potassium will occur for
every 40 mEq of intravenous potassium administered” (J Clin Pharmacol 1994;34:1077-82;
Arch Intern Med 1990;150:613-7) is potentially inaccurate for many critically ill subpopulations
such as emaciated patients or patients with obesity, those with renal dysfunction, exaggerated
requirements such as trauma or thermally injured patients (J Parenter Enteral Nutr 2017;41:796-
804), those with volume overload, or those receiving diuretic therapy.
viii. Serum potassium concentrations are equilibrated within 1–2 hours after completion of the
intravenous potassium chloride infusion (J Clin Pharmacol 1994;34:1077-82; Crit Care Med
1991;19:694-9), and repeated assessments are recommended for patients with severe and/or
complicated cases of hypokalemia.
3. Hyperkalemia
a. Definition: Serum potassium greater than 5.2 mEq/L, although usually not a significant problem
until serum potassium approaches 6 mEq/L
i. Rule out factitious hyperkalemia (hemolysis of blood sample, white blood cell count greater
than 10 x 103 cells/mm3, platelet count greater than 400,000/mm3). The potential for this
error can be reduced by collecting the blood sample in a heparinized tube. Another source of
factitious hyperkalemia occurs when the blood sample is obtained from the same intravenous
line as a potassium-containing fluid.
ii. Assess arterial blood gas (ABG) (severe acidosis).
iii. Assess for recent administration of blood products (red blood cells contain 7.5–13.5 mEq/L of
potassium).
b. Signs and symptoms: ECG changes (peaked and tented T waves) and arrhythmias (bradyarrhythmias,
ventricular fibrillation, asystole), symptoms similar to those of hypokalemia (weakness, paralysis)
c. Etiologies:
i. Drugs – Potassium-sparing diuretics (spironolactone, amiloride, triamterene), angiotensin-
converting enzyme inhibitors, angiotensin receptor blockers, nonsteroidal anti-inflammatory
drugs, heparin, trimethoprim, octreotide, succinylcholine, digoxin (toxicity)
ii. Excessive intake (usually in combination with compromised renal function) – Be sure to
examine all intravenous fluids, EN and PN regimens, penicillin G (1.7 mEq of potassium per
million units), packed red blood cells.
iii. Renal dysfunction (chronic kidney disease [CKD], AKI)
iv. Hyporeninemic hypoaldosteronism
v. Tissue catabolism (chemotherapy, rhabdomyolysis, tumor lysis syndrome)

ACCP Updates in Therapeutics® 2022: Critical Care Pharmacy Preparatory Review and Recertification Course

315
Fluids, Electrolytes, Acid-Base Disorders, and Nutrition Support

vi. Severe acidemia


vii. Older adult patients are also at risk because of decreased renal function, reduced renal
functional reserve, and loss of body cell mass.
d. Treatment
i. The following regimens are for patients with symptomatic evidence of toxicity from
hyperkalemia such as peaked T waves on ECG:

Table 11. Treatment of Hyperkalemia


Mechanism Treatment Description
Given as slow intravenous push (especially if
Calcium gluconate 10%, 1–2 g IV ECG changes). Repeat dose after 5 minutes
Stabilization of
if ECG changes persist. Should be given in
myocardium
Calcium chloride, 0.5–1 g IV addition to other therapies as calcium does not
affect serum potassium concentration.
Intracellular Usually given with dextrose 50 g IV. Dextrose
Regular human insulin, 10 units IV
shifting of potas- can be omitted if the patient has hyperglycemia.
sium (temporary Sodium bicarbonate, 50–100 mEq IV Especially useful if patient is acidemic
redistribution) Albuterol
Loop diuretics
Intragastric administration preferred as cases of
Sodium polystyrene sulfonate, 25–50 g bowel necrosis have been reported from sodium
Elimination of
polystyrene sulfonate/sorbitol enemas.
potassium
The most effective and predictable method for
Hemodialysis potassium elimination but also the most invasive.
Reserved for life-threatening hyperkalemia.
ECG = electrocardiogram.

ii. Make sure no exogenous sources of potassium (e.g., intravenous fluids, EN, PN); to reduce
intake with EN or PN; use a “renal” (no or low-electrolyte formulation), if necessary.
iii. Patiromer oral suspension – Zirconium citrate nonabsorbable polymer. Delayed onset of action
and designed for patients with chronic hyperkalemia (not for acute hyperkalemia) who receive
angiotensin-converting enzyme inhibitors, angiotensin receptor blockers, spironolactone,
with advanced diabetes (hyporeninemic hypoaldosteronism) or kidney dysfunction. The
manufacturer recommends a starting dose of 8.4 g once daily and dosage adjustments by 8.4
g daily as needed at 1-week intervals. This agent is available in 8.4-, 16.8-, and 25.2-g powder
packets. Adverse effects include hypomagnesemia (7%), constipation (6%), and hypokalemia
(6%).
iv. Sodium zirconium cyclosilicate oral suspension - nonabsorbable polymer. Action similar to
patiromer. The manufacturer recommends a starting dose of 10 g three times daily up to 48
hours, followed by a maintenance dose of 10 g once daily. The dosage may be adjusted by 5 g
at 1-week intervals as needed. This agent is available in 5-g packets.

D. Disorders of Magnesium Homeostasis


1. Magnesium homeostasis overview
a. 99% intracellular (17% of total body content is in the muscle or in the skeleton)
b. Total body stores: Around 2000 mEq
c. Normal serum concentration: 1.8–2.4 mg/dL (about 30% bound to protein)

ACCP Updates in Therapeutics® 2022: Critical Care Pharmacy Preparatory Review and Recertification Course

316
Fluids, Electrolytes, Acid-Base Disorders, and Nutrition Support

d. Average daily requirement: Around 24–32 mEq/day


e. Kidney is primary route of elimination (around 70% reabsorbed in ascending loop of Henle) and is
without any hormonal regulation of renal magnesium reabsorption.
f. Losses can be extensive with severe diarrhea or body fluid drainages (see Table 4).
g. Magnesium depletion can influence potassium and calcium homeostasis.
2. Hypomagnesemia
a. Definition: Although the lower limit of normal for serum magnesium concentrations is 1.8 mg/
dL (1.5 mEq/L), most clinicians define significant hypomagnesemia as 1.5 mg/dL (1.3 mEq/L) or
less (Annu Rev Med 1981;32:245-59; Nutrition 1997;13:303-8). Many ICUs have a target serum
magnesium concentration greater than 2 mg/dL (1.8 mEq/L). Serum concentrations of magnesium
may be slightly falsely lowered in the presence of significant hypoalbuminemia (Annu Rev Med
1981;32:245-59; Nutrition 1997;13:303-8).
b. Signs and symptoms: Muscle weakness, cramping, paresthesias, Chvostek and Trousseau signs,
tetany, QT prolongation, hypokalemia, hypocalcemia
c. Etiologies:
i. GI losses (especially diarrhea) – Average stool loss of about 6 mEq/L; up to 10–12 mEq/L or
greater for secretory diarrheal losses
ii. Alcohol (increased renal excretion; impaired absorption; poor nutritional status of patients
who abuse alcohol)
iii. Sepsis/critical illness (increased urinary excretion – several factors)
iv. Pancreatitis (partly attributable to calcium-magnesium soap formation in peritoneum)
v. Thermal injury/TBI (increased urinary excretion – several factors)
vi. Drugs – Diuretics, amphotericin B, caspofungin, cyclosporin/tacrolimus, foscarnet,
pentamidine, piperacillin/tazobactam, cisplatin/carboplatin/ifosfamide/cetuximab, lactulose/
orlistat, aminoglycosides, and potentially long-term use of digoxin or proton pump inhibitors
vii. Polyuria (osmotic agents, hypercalcemia, ureagenesis)
d. Estimating magnesium deficit: For a serum magnesium concentration of less than 1.5 mg/dL (1.3
mEq/L), a 1- to 2-mEq/kg deficit can be expected.
e. Treatment:
i. Treat the etiology (if possible). Be sure to treat magnesium deficiency at the same time or
before potassium therapy if the patient is also hypokalemic.
ii. Successful treatment of hypomagnesemia usually takes 4–5 days of intravenous therapy.
Intramuscular magnesium therapy for replacement therapy is inadvisable given the limit on
volume per injection site with respect to dosage requirements and tissue irritation.
iii. Intravenous magnesium sulfate 32–48 mEq/day (4–6 g/day) – Suggested to be sufficient to
maintain serum magnesium within 2–2.5 mg/dL for most magnesium-deficient patients (Crit
Care Med 1996;24:38-45; Annu Rev Med 1981;32:245-59)
iv. Empiric intravenous magnesium sulfate dosing. This algorithm was designed primarily for
trauma and thermally injured patients but can likely be universally applied to other critically ill
patient populations (Nutrition 1997;13:303-8). The therapy may need to be adjusted according
to renal function and ongoing severity of losses. An empiric approach to dosing intracellular
electrolytes for patients with significant renal impairment is to give one-half the recommended
dose (see Table 12). However, electrolyte therapy for patients with renal impairment must be
individualized according to patient response.
v. Magnesium concentrations are often elevated for several hours or longer after an infusion
because it takes about 48 hours for the magnesium to fully redistribute to the body tissues
(Nutrition 1997;13:303-8). One-time repletion doses are usually not adequate for large deficits,
and levels should continue to be monitored daily to ensure an appropriate assessment of
magnesium is made after redistribution of previous bolus doses.

ACCP Updates in Therapeutics® 2022: Critical Care Pharmacy Preparatory Review and Recertification Course

317
Fluids, Electrolytes, Acid-Base Disorders, and Nutrition Support

Table 12. Empiric Intravenous Magnesium Sulfate Dosing


Serum Magnesium (mg/dL) Dose (g/kg)
1.6–1.8 0.05
1–1.5 0.1
<1 0.15
For ease of use and preparation, intravenous magnesium sulfate should be ordered in 2-g increments (e.g., 2 g, 4 g, 6 g). The drug should be mixed in 100–250 mL of
normal saline or 5% dextrose and given at a rate no faster than 1 g (8 mEq) per hour (Nutrition 1997;13:303-8). Commercially available sources of magnesium sulfate
are available in 16 mEq/50 mL and 32 mEq/100 mL premixed bags. Maximum dose should be held at a ceiling of 8–10 g per administration. Intravenous magnesium
may be administered by the peripheral or central venous route.

vi. Oral magnesium: It can be difficult to successfully replenish magnesium if given by the
oral route in critically ill patients because of the adverse GI effects of oral magnesium (e.g.,
diarrhea) and the high elemental magnesium doses required to achieve repletion. Although it
has been inferred that certain oral magnesium products are better tolerated than others (e.g.,
gluconate vs. oxide), this tolerability likely pertains to the elemental magnesium content of
the products. The lower the elemental magnesium content, the more tolerable the oral product.
However, the lower the magnesium content, the more difficult it is to achieve magnesium
repletion for a patient with significant magnesium depletion.

Table 13. Common Oral Magnesium Products


Strength Elemental Magnesium Content
Salt Form Usual Dosing
(mg) (mEq)
400 19.8
Oxide 1–2 tablets two or three times daily
140 6.9
Gluconate 500 2.2 1–2 tablets two or three times daily
Chloride 520 2.6 1–2 tablets two or three times daily

Table 14. Oral Magnesium Content


Salt Form % Elemental Magnesium Content
Oxide 60
Carbonate 45
Hydroxide 42
Citrate 16
Lactate 12
Chloride 12
Sulfate 10
Gluconate 5

3. Hypermagnesemia
a. Definition: Serum magnesium concentration greater than 2.4 mg/dL
b. Signs and symptoms: Hypotension, decreased deep tendon reflexes, cardiovascular manifestations
(e.g., bradycardia, somnolence, muscle paralysis, arrhythmias) generally do not occur until serum
concentrations are greater than 4 mg/dL.

ACCP Updates in Therapeutics® 2022: Critical Care Pharmacy Preparatory Review and Recertification Course

318
Fluids, Electrolytes, Acid-Base Disorders, and Nutrition Support

c. Etiologies: Renal failure or impairment, early post-infusion elevation of serum magnesium


concentration, excessive dosing of magnesium/antacids, post-cathartic use (e.g., magnesium
citrate) – To develop hypermagnesemia, these events usually occur together with renal impairment.
d. Treatment:
i. Remove source of magnesium intake.
ii. Intermittent slow bolus doses of calcium gluconate (2 g) for 5–10 minutes until severe symptoms
abate (the effect of calcium is transient, and repeat therapy may be needed as often as every
hour). The effects of magnesium on neuromuscular and cardiac function are antagonized by
calcium.
iii. Ventilate the patient, if necessary.
iv. 0.9% sodium chloride infusion with loop diuretic therapy
v. Hemodialysis

Patient Case

Questions 4 and 5 pertain to the following case.


A 65-year-old man (weight 87 kg) is admitted to the hospital with severe acute pancreatitis. A computed tomog-
raphy (CT) scan of the abdomen reveals a pancreatic pseudocyst. He is to remain NPO (nothing by mouth)
because of marked abdominal pain on a low-fat diet and during a trial of EN; therefore, he is given PN. He is a 2
pack/day tobacco smoker and has a history of frequent alcohol consumption. His serum laboratory values are as
follows: sodium (Na) 139 mEq/L, potassium (K) 3.3 mEq/L, Cl 102 mEq/L, total CO2 content 25 mEq/L, BUN
14 mg/dL, SCr 0.8 mg/dL, calcium 7.6 mg/dL, phosphorus 2.2 mg/dL, magnesium 1.5 mg/dL, and albumin 2.5
g/dL.

4. Which potassium-phosphorus dosing regimen would be most appropriate for this patient?
A. Potassium phosphate 30 mmol intravenously x 1 dose, followed by potassium chloride 40 mEq via NG
tube x 2 doses.
B. Potassium phosphate 30 mmol intravenously x 1 dose, followed by potassium chloride 40 mEq intra-
venously x 1 dose.
C. Potassium phosphate 60 mmol intravenously x 1 dose.
D. Potassium chloride 40 mEq via NG tube x 2 doses, followed by Neutra-Phos 250 mg via NG tube x 2
doses.

5. In addition to potassium and phosphorus supplementation, the patient is given magnesium sulfate 6 g intra-
venously for 6 hours. His repeat serum magnesium the next day is 2.0 mg/dL. Which therapeutic option
would be best for this patient?
A. Give magnesium oxide 500 mg twice daily for the next few days.
B. Give magnesium sulfate 2–4 g intravenously daily for the next few days.
C. Give an additional dose of 8 g of magnesium sulfate intravenously.
D. No additional treatment is necessary.

ACCP Updates in Therapeutics® 2022: Critical Care Pharmacy Preparatory Review and Recertification Course

319
Fluids, Electrolytes, Acid-Base Disorders, and Nutrition Support

E. Disorders of Calcium Homeostasis


1. Calcium homeostasis overview
a. Most prevalent intracellular cation in body; 99% of body’s calcium is in bone; highly protein bound
in plasma
b. Total body stores: About 1–1.2 kg of calcium
c. Normal serum concentration: 8.5–10.5 mg/dL; normal serum ionized concentration 1.12–1.32
mmol/L
d. Serum concentration can be influenced by:
i. Changes in plasma albumin concentration – For every 1 g/dL in serum albumin below 4 g/
dL, serum calcium will decrease by around 0.8 mg/dL (Clin Chim Acta 1971;35:483-9). This
estimation of serum calcium concentrations is inaccurate in critically ill patients and should
not be used (JPEN J Parenter Enteral Nutr 2004;28:133-41). Ionized calcium concentrations
should be used for assessing calcium in critically ill patients. However, most critically ill
patients (85%) with a total serum calcium concentration less than 7 mg/dL are hypocalcemic
(ionized serum calcium of 1.12 mmol/L or less) (Nutr Clin Pract 2007;22:323-8).
ii. Changes in pH (for every 0.1-unit increase in arterial pH, serum ionized calcium will decrease
by about 0.05 mmol/L) (Arch Pathol Lab Med 2002;126:947-50) because of increased protein
binding
e. Average daily requirement: 10–15 mEq/day intravenously with PN (Ann Surg 1983;197:1-6); 1–3
g/day orally
f. Kidney is primary route of elimination.
g. Magnesium status can influence calcium homeostasis (J Parenter Enteral Nutr 2004;28:133-41;
Annu Rev Med 1981;32:245-59).
i. Hypomagnesemia results in end-organ resistance to parathyroid hormone.
ii. Hypomagnesemia may impair parathyroid hormone secretion.
iii. Hypocalcemia will correct within 2 days after hypomagnesemia is corrected.
2. Hypocalcemia
a. Definition: Corrected serum total calcium less than 8.5 mg/dL (non-ICU patients); ionized serum
calcium concentration less than 1.12 mmol/L
b. Signs and symptoms: Tingling, paresthesias, hyperactive deep tendon reflexes, tetany (Chvostek
and Trousseau signs), seizures, prolonged QT interval
c. Etiologies
i. Critical illness, surgery
ii. Continuous renal replacement therapy (CRRT) and citrate anticoagulation
iii. Massive blood transfusion
iv. Hypomagnesemia
v. Hyperphosphatemia
vi. Pancreatitis
vii. Drugs (amphotericin B, cisplatin, cyclosporine, foscarnet, bisphosphonates, loop diuretics,
sodium bicarbonate, cinacalcet, fluoride poisoning)
viii. Malabsorption
ix. Hypoparathyroidism, hypothyroidism
x. Chronic kidney injury/AKI
xi. Vitamin D deficiency
xii. Severe alkalemia
d. Treatment: See Table 15 for an empiric algorithm for intravenous repletion of calcium. (JPEN
J Parenter Enteral Nutr 2007;31:228-33; JPEN J Parenter Enteral Nutr 2005;29:436-4; Nutrition
2007;23:9-15).

ACCP Updates in Therapeutics® 2022: Critical Care Pharmacy Preparatory Review and Recertification Course

320
Fluids, Electrolytes, Acid-Base Disorders, and Nutrition Support

Table 15. Empiric IV Calcium Gluconate Dosing


Ionized Calcium
Intravenous Calcium Gluconate Dosage (g)
(mmol/L)
1–1.12 2 g (9.3 mEq) of calcium gluconate in 100 mL of 0.9% NaCl or D5W for 2 hours
≤ 0.99 4 g (18.6 mEq) of calcium gluconate in 100 or 250 mL of 0.9% NaCl or D5W for 4 hours
For ease of use and preparation, calcium gluconate should be ordered in gram increments. Calcium chloride should be used (preferably) only in code situations, not for
routine replacement therapy, because the chloride salt contains about 2.5 times the amount of elemental calcium and can cause tissue necrosis when given peripherally
in contrast to calcium gluconate. However, during extreme circumstances, such as a national drug shortage of intravenous calcium gluconate, calcium chloride can be
given in 0.67- and 1.3-g doses in lieu of 2- and 4-g calcium gluconate doses. In addition, calcium chloride should never be added to the PN solution unless there is no
phosphate in the PN solution and the commercial amino acids used in the PN solution do not contain phosphorus (some amino acid products do contain phosphate). A
serum ionized calcium concentration determination should be repeated several hours after completing the calcium gluconate infusion to allow equilibration (Nutrition
2007;23:9-15). More aggressive therapy may need to be considered for patients with tetany or life-threatening cardiac arrhythmias caused by hypocalcemia.
Intravenous calcium administration should be used with extreme caution in patients with severe hypokalemia or in those receiving digoxin or other digitalis alkaloids.
Always check the serum phosphorus concentration because hyperphosphatemia can induce hypocalcemia, given the metastatic precipitation of calcium phosphate in the
soft tissues and lungs (usually associated with renal disease).
D5W = 5% dextrose in water; NaCl = sodium chloride.

Patient Case

Questions 6 and 7 pertain to the following case.


A 24-year-old man (weight 90 kg) is admitted to the trauma ICU postoperatively from repair of his duodenal,
jejunal, ileal, and colon injuries; hepatorrhaphy; and splenectomy after several gunshot wounds to the abdomen.
He also received 10 units of packed red blood cells. He has a serum ionized calcium concentration of 0.86
mmol/L, K 4.6 mEq/L, and magnesium 1.8 mg/dL. His SCr concentration is 0.8 mg/dL, and his urine output is
0.5 mL/kg/hour.

6. Which is the most likely etiology of his hypocalcemia?


A. Hypomagnesemia
B. Excessive urinary diuresis
C. Blood transfusion
D. Critical illness

7. Which therapeutic regimen would be best for this patient?


A. Calcium gluconate 2 g intravenously for 2 hours
B. Calcium gluconate 4 g intravenously for 4 hours
C. Calcium chloride 1 g intravenous push for 5–10 minutes
D. No calcium therapy necessary.

3. Hypercalcemia
a. Definition: Corrected serum calcium greater than 10.5 mg/dL or ionized calcium greater than 1.32
mmol/L; signs and symptoms are more evident when total serum calcium of 12 mg/dL or greater
or ionized calcium of 1.5 mmol/L or greater.
b. Signs and symptoms: Mental status changes, polyuria, shortened QT interval, bradycardia,
atrioventricular block
c. Etiologies:
i. Immobilization
ii. Chronic critical illness–associated metabolic bone disease
iii. Excessive calcium or vitamin D intake
iv. Hyperparathyroidism

ACCP Updates in Therapeutics® 2022: Critical Care Pharmacy Preparatory Review and Recertification Course

321
Fluids, Electrolytes, Acid-Base Disorders, and Nutrition Support

v. Granulomatous diseases (tuberculosis, sarcoidosis)


vi. Malignancy
vii. Drugs (thiazide diuretics, vitamin D, lithium, teriparatide)
viii. Dehydration
ix. Thyrotoxicosis
x. Adrenal insufficiency
d. Treatment:
i. Mobilize the patient (if possible); discontinue calcium supplementation
ii. Intravenous fluids with 0.9% sodium chloride (if dehydrated) at 200–300 mL/hour x 48 hours
or until rehydrated with or without furosemide 40–80 mg intravenously every 12 hours
iii. Calcitonin 4 units/kg intramuscularly or subcutaneously every 12 hours; can be increased to
a maximum of 8 units/kg every 8 hours. Calcitonin inhibits bone reabsorption and promotes
the renal excretion of calcium by decreasing tubular reabsorption. The onset of action is rapid
(within hours); however, tachyphylaxis can occur because of down-regulation of calcitonin
receptors within days (J Intensive Care Med 2015;30:235-52) so duration of use should be
limited to 24–72 hours. Calcitonin is most useful in patients with symptomatic hypercalcemia
in combination with rehydration.
iv. Bisphosphonates. Pamidronate 90 mg intravenously once for acute hypercalcemia unrelated
to causes c. i. and c. ii. Pamidronate 30 mg intravenously daily for 3 days may be used if
hypercalcemia in chronic critical illness is associated with metabolic bone disease or
immobilization (Chest 2000;118:761-6). Zoledronic acid may also be an option; however,
evidence regarding its efficacy for chronic critical illness–associated metabolic bone disease
is lacking. Onset of action for bisphosphonates in lowering serum calcium is typically after 48
hours. Bisphosphonates should be used with caution in patients with renal impairment.
v. Parathyroidectomy for patients with primary hyperparathyroidism
vi. Prednisone 40 mg/day and greater for 10 days for patients with granulomatous diseases (e.g.,
sarcoidosis, tuberculosis)
vii. Hemodialysis may be necessary for severe hypercalcemia.

F. Disorders of Phosphorus Homeostasis


1. Phosphorus homeostasis overview
a. 99% intracellular, of which 85% is bound to bone
b. Extracellular pool of phosphorus: Around 600 mg (about 20 mmol), 10% protein bound
c. Normal serum concentration: 2.5–4.5 mg/dL
d. Serum concentration can be influenced by parathyroid hormone (increased parathyroid hormone
leads to increased urinary excretion of phosphorus), and alkalemia can decrease serum phosphorus
concentration.
e. Average daily requirement: Around 20 mg/kg/day
f. Kidney is primary route of elimination.
2. Hypophosphatemia
a. Definition: Serum phosphorus less than 2.5–3 mg/dL; severe hypophosphatemia less than
1–1.5 mg/dL
b. Signs and symptoms: Weakness, paresthesias; severe depletion can lead to congestive
cardiomyopathy, cardiac arrest, seizures, coma, respiratory arrest due to weakness of the
diaphragm, rhabdomyolysis
c. Etiologies:
i. Alcoholism
ii. Malnutrition/refeeding syndrome
iii. Critical illness (especially trauma, TBI, thermal injury)

ACCP Updates in Therapeutics® 2022: Critical Care Pharmacy Preparatory Review and Recertification Course

322
Fluids, Electrolytes, Acid-Base Disorders, and Nutrition Support

iv. Diabetic ketoacidosis


v. Hepatic resection
vi. Drugs – Insulin, catecholamines, antacids, sucralfate, calcium, intravenous iron administration,
diuretics
vii. Alkalemia
viii. Malabsorption – Chronic diarrhea
ix. Hyperparathyroidism
x. Cancer (phosphatonins [e.g., fibroblast growth factor-23])
xi. Renal replacement therapies
xii. Hungry bone syndrome following parathyroidectomy or thyroidectomy
d. Treatment
i. Target serum phosphorus concentration when patient is in the ICU and ventilator dependent:
About 4 mg/dL (Intensive Care Med 1995;21:826-31; N Engl J Med 1985;313:420-4)
ii. Intravenous dosing guidelines

Table 16. Intravenous Phosphorus Dosing Guidelines


High Requirementsa Population
General Medical-Surgical Population
Serum Phosphorus Dosage (mmol/kg)
Dosage (mmol/kg)
(mg/dL) (JPEN J Parenter Enteral Nutr
(Crit Care Med 1995;23:1504-11)
2006;30:209-14)
2.3–3 0.16 0.32
1.6–2.2 0.32 0.64
< 1.6 0.64 1
a
Patients with thermal injury (JPEN J Parenter Enteral Nutr 2001;25:152-9), those with trauma (especially those with a traumatic brain injury) (Nutrition 2010;26:784-
90; JPEN J Parenter Enteral Nutr 2006;30:209-14), those malnourished with evidence of significant complications from refeeding syndrome, or those with hepatic
resections.
The drug should be mixed in 100–250 mL of normal saline or 5% dextrose in water and given at a rate no faster than 7.5 mmol/hour. Phosphorus should always be
ordered in millimoles for ease of use and preparation. For ease of use and preparation in the pharmacy, phosphorus should be ordered in units divisible by 3 mmol (e.g.,
15, 30, 45, 60) whenever possible.
Potassium phosphate salt can be used for patients with a serum potassium less than 4 mEq/L (3 mmol of phosphorus = 4.4 mEq of potassium). Sodium phosphate salt
should be used for patients with a serum potassium of 4 mEq/L or greater (3 mmol of phosphorus = 4 mEq of sodium).

iii. Repletion using oral phosphorus formulations


(a) Difficult to accomplish because of erratic absorption and adverse effects (e.g., diarrhea).
This is especially true for severe hypophosphatemia in which higher doses are required.
(b) Neutra-Phos and Neutra-Phos K (only 8 mmol of phosphorus per tablet/packet)
3. Hyperphosphatemia
a. Definition: Serum phosphorus concentration greater than 5 mg/dL, usually not clinically relevant
until serum phosphorus is greater than 6 mg/dL
b. Signs and symptoms: Hypocalcemia and metastatic calcification (e.g., neuromuscular irritability,
prolonged QT interval, tetany) – Usually does not occur until the serum calcium-phosphorus
product approaches 55 mg/dL or greater (Adv Exp Med Biol 1978;103:195-201).
c. Etiologies:
i. Renal failure
ii. Immobility
iii. Chronic critical illness–associated metabolic bone disease
iv. Excessive phosphorus intake
v. Vitamin D toxicity
vi. Tumor lysis syndrome
vii. Hypoparathyroidism

ACCP Updates in Therapeutics® 2022: Critical Care Pharmacy Preparatory Review and Recertification Course

323
Fluids, Electrolytes, Acid-Base Disorders, and Nutrition Support

d. Treatment:
i. Reduce phosphorus intake (omit from PN solution, with or without reduction of ILE content of
PN solution if a high-fat formulation [controversial as phosphorus in organic form: phospholipids
and not inorganic such as sodium phosphate]; change to “low- or no-electrolyte” renal enteral
formula).
ii. Phosphate binders if consuming oral/enteral nutrition (Am J Health Syst Pharm 2005;62:2355-
61). Dosing should be timed with meals for patients consuming oral diets or given at regular
intervals before and/or during enteral nutrition administration for patients on tube feeds.

Table 17. Phosphate Binders


Phosphorus-Binding Recommended Empiric
Drug Available Strength
Capacity Initial Dose
Calcium carbonatea 500, 750, 1000 mg per tablet Calcium 43 mg/g 1 g QID
Calcium acetate a
667 mg per tablet Calcium 106 mg/g 1334–2001 mg TID
Sevelamer b
800 mg per capsule or packet Sevelamer 80 mg/g 800–1600 mg TID
500, 750, 1000 mg per tablet or
Lanthanumc Data not available 500 mg TID
packet
a
Do not use if the patient is hypercalcemic.
b
Sevelamer carbonate comes in a powder; easier for administering to tube-fed patients; less likely to worsen metabolic acidosis than tablet in patients with renal failure
because tablet is in hydrochloric acid salt form.
750 and 1000 mg available as oral packet; 500, 750, and 1000 mg available as chewable tablet.
c

QID = four times daily; TID = three times daily.

II.  ACID-BASE DISORDERS

A. Normal Homeostasis
1. Normal values

Table 18. Normal Blood Gas Values


Variables Arterial Blood Mixed Venous Blood
pH 7.35–7.45 7.31–7.41
Pco2 35–45 41–51
Po2 80–100 35–40
HCO3 22–26 22–26
Base excess -2 to +2 -2 to +2
O2 saturation > 95% 70%–75%
HCO3 = bicarbonate.

ACCP Updates in Therapeutics® 2022: Critical Care Pharmacy Preparatory Review and Recertification Course

324
Fluids, Electrolytes, Acid-Base Disorders, and Nutrition Support

2. Interpreting ABGs
a. Acidemia (pH less than 7.35) versus alkalemia (pH greater than 7.45)
b. Acidemia and alkalemia refer to an abnormal pH being either low or high, respectively. Acidosis
and alkalosis refer to the metabolic or respiratory processes that led to the abnormal pH. Although
the terms emia and osis are similar, they are different.
c. For simple acid-base disorders, identify pH, Pco2, and HCO3 in that order. Whichever side of 7.40
the pH is on, the respiratory or metabolic processes that coincide with that pH abnormality are the
primary etiology. If the pH is less than 7.40, an elevated Pco2 (respiratory acidosis) or a decreased
HCO3 (metabolic acidosis) is the primary etiology. If the pH is greater than 7.40, a decreased Pco2
(respiratory alkalosis) or an increased HCO3 (metabolic alkalosis) is the primary etiology. An easy
introductory overview to acid-base disorders by Haber is provided in the references (West J Med
1991;155:146-51).
d. However, sometimes more than one primary abnormality is present, or the anticipated compensatory
process (metabolic or respiratory) is inadequate and may be contributing to the acid-base disorder.
As a result, various formulas have been developed to predict what may be considered adequate
compensation. However, many of these mathematical equations have limitations in their clinical
utility and accuracy (J Trauma Acute Care Surg 2012;73:27-32; Clin J Am Soc Nephrol 2007;2:162-
74; Crit Care Med 2007;35:1264-70; Am J Respir Crit Care Med 2000;162:2246-51; Arch Intern
Med 1992;152:1625-9; J Crit Care 2013;28:1103) and can be difficult to memorize (West J Med
1991;155:146-51).
e. In addition, the issue of mixed acid-base disorders is confounded by several factors that can lead
to errors in interpreting acid-base disorders. These include non–steady-state conditions as well as
the inability for the patient to adequately compensate through the respiratory pathway because of
mechanical ventilator restrictions. Some of the more common equations for assessing acid-base
disorders are discussed later in this chapter.

Table 19. Adverse Effects of Severe Acidemia (pH 7.25 or less)


Impaired cardiac output Increased metabolic demands
Peripheral ischemia (centralization of blood) Insulin resistance and hyperglycemia
Increased pulmonary vascular resistance Decreased ATP synthesis
Hypotension Increased protein breakdown
Increased risk of arrhythmias Hyperkalemia
Decreased catecholamine responsiveness Diaphragmatic fatigue/dyspnea
Obtundation/coma Hyperventilation

Table 20. Adverse Effects of Severe Alkalemia (pH of 7.55 or greater)


Arteriolar constriction Hypokalemia
Hypotension Hypophosphatemia
Increased risk of arrhythmias Hypocalcemia/tetany
Decreased coronary blood flow/decreased angina Organic acid production
threshold
Seizures Hypoventilation/hypercapnia/hypoxemia
Lethargy, delirium, stupor

ACCP Updates in Therapeutics® 2022: Critical Care Pharmacy Preparatory Review and Recertification Course

325
Fluids, Electrolytes, Acid-Base Disorders, and Nutrition Support

3. Use of base excess


a. Reflects the amount of base needed in vitro to return the plasma pH to 7.40 at standard conditions
(Pco2 40 mm Hg, 37°C)
b. Base excess reflects the metabolic component to interpreting the ABG. Despite its simplicity,
it has limitations (J Trauma Acute Care Surg 2012;73:27-32). Crystalloid resuscitation (leading
to hyperchloremic acidosis), exogenous bicarbonate (HCO3) administration, ethanol ingestion,
and acetate/HCO3 buffer in hemodialysis or CRRT solutions can lead to erroneous base excess
calculations and errors in interpretation (J Trauma Acute Care Surg 2012;73:27-32).
4. Compensatory response to acid-base disorders

Table 21. Anticipated Compensation to Acid-Base Disorders (Crit Care 2000;4:6-14)


Serum Bicarbonate
Primary Disorder Anticipated Pco2 (mm Hg)
(mEq/L)
Metabolic acidosis ≤ 22 (1.5 x HCO3) + 8 (± 2)
Metabolic alkalosis ≥ 28 (0.7 x HCO3) + 20 ± 5
Primary disorder Pco2 (mm Hg) Anticipated serum bicarbonate (mEq/L)
Respiratory acidosis (acute) a
HCO3 should increase by ~1 mEq/L per 10 mm HG
increase in Pco2 > 40 mm Hg
> 45
Respiratory acidosis (chronic)a HCO3 should increase by ~4 mEq/L per 10-mm Hg
increase in Pco2 > 40 mm Hg
Respiratory alkalosis (acute)a For each 10-mm Hg decrease in Pco2,
Hco3 should decrease by ~2 mEq/L
< 35
Respiratory alkalosis (chronic)a HCO3 should decrease by ~ 4 - 5 mEq/L
for each 10-mm Hg decrease in Pco2
a
Compensation is different for acute versus chronic respiratory disorders because it takes about 2 days for the kidneys to adapt to a persistent change in respiratory status.

B. Respiratory Acidosis
1. Common causes include pulmonary edema, pulmonary embolism, pneumonia, CNS depression, cardiac
arrest, stroke, spinal cord injury, excessive sedation/analgesia, and overfeeding with PN/EN. Make sure
it is not caused by excessive sedation/analgesia or overfeeding with EN/PN.
2. Metabolic compensation – See Table 21.

C. Respiratory Alkalosis
1. Common causes include uncontrolled pain, nicotine and drug withdrawal, agitation, pneumonia,
stimulant drugs, salicylate toxicity (due to direct respiratory stimulation), and head injury. Make sure
the patient is getting adequate sedation/analgesia, fever/pneumonia is being treated; nicotine and drug
withdrawal regimen is/are appropriate.
2. Metabolic compensation – See Table 21.

D. Metabolic Acidosis
1. Use of the serum anion gap (AG)
a. Used to determine the etiology for the metabolic acidosis. AG is the difference between major
cations and anions in blood (trying to detect whether there is an abundance of unmeasured anions).
If an AG is present, then a metabolic acidosis is present, regardless of pH or HCO3.
AG = Na – (Cl + HCO3)

ACCP Updates in Therapeutics® 2022: Critical Care Pharmacy Preparatory Review and Recertification Course

326
Fluids, Electrolytes, Acid-Base Disorders, and Nutrition Support

b. Normal range is about 3–14 mEq/L. Some clinicians will include serum potassium when calculating
cations (and the normal AG will need to be adjusted), but this is uncommon.
c. Adjust AG for serum albumin (Crit Care Med 1998;26:1807-10). The difference in serum albumin
concentration (grams per deciliter) from normal should be multiplied by 2–2.5 and added to the
anions (chloride and bicarbonate).
Albumin adjusted AG = Na - Cl - HCO3 - (2.5 x [4 - serum albumin]).
Some clinicians also adjust for serum phosphorus (Crit Care Med 2007;35:2630-6). Serum phos-
phorus (milligrams per deciliter) can be multiplied by 0.5 and added to anions, and lactate can
also be included but is not common in routine clinical practice. Using this method (and including
serum potassium), the adjusted AG (or sometimes called the strong ion gap when referring to the
physicochemical methodology for interpreting acid-base disorders) should be close to 0 (± 2) if the
patient does not have an AG acidosis.

d. Causes of an AG acidosis: One easy pneumonic to remember (there are others) is A MUD PIE:
A = Aspirin (or other salicylates)
M = Methanol
U = Uremia (including rhabdomyolysis)
D = Diabetes (diabetic ketoacidosis)
P = Paraldehyde, propylene glycol
I = Infection or ischemia (lactic acidosis)
E = Ethylene glycol or ethanol toxicity
e. Types of lactic acidosis (lactate greater than 4 mmol/L and pH less than 7.35)
i. Type A: Hypoperfusion (cardiogenic or septic shock, regional ischemia, severe anemia)
ii. Type B: Metabolic – No tissue hypoxia
(a) B1 = underlying disease (diabetes mellitus, liver disease, leukemia, lymphoma, AIDS)
(b) B2 = drugs/toxins (metformin, didanosine/stavudine/zidovudine, ethanol, linezolid,
propofol, propylene glycol toxicity caused by intravenous lorazepam or pentobarbital),
nitroprusside (cyanide) toxicity
(c) B3 = inborn errors of metabolism (pyruvate dehydrogenase deficiency)
f. Causes of a normal AG acidosis
Another easy pneumonic to remember (there are others) is ACCRUED.
A = Ammonium chloride/acetazolamide (urine bicarbonate loss)
C = Chloride intake (PN, intravenous solutions)
C = Cholestyramine (GI bicarbonate loss)
R = Renal tubular acidosis: Types I, II, and IV
U = Urine diverted into the intestine (e.g., ileal conduit, vesicoenteric fistula)
E = Endocrine disorders (e.g., aldosterone deficiency)
D = Diarrhea or small/large bowel fluid losses (e.g., enterocutaneous fistulas)
2. In the presence of an elevated AG, the delta ratio can be assessed for determining mixed acid-base
disorders
Delta ratio =
ΔAG/ΔHCO3 = (measured AG − normal AG)/(normal HCO3 − measured HCO3) =
(AG−14)/(24 − measured HCO3)

ACCP Updates in Therapeutics® 2022: Critical Care Pharmacy Preparatory Review and Recertification Course

327
Fluids, Electrolytes, Acid-Base Disorders, and Nutrition Support

Table 22. Interpreting Delta Ratioa


Delta Ratio Assessment
< 0.4 Hyperchloremic normal AG acidosis
<1 Mixed high AG acidosis and hyperchloremic normal AG acidosis
1–2 AG acidosis (no hidden process)
High AG acidosis and concurrent metabolic alkalosis OR a preexisting compensated
>2
respiratory alkalosis
aThe ratio should be used cautiously in interpreting mixed acid-base disorders, given that it is associated with poor sensitivity because of several influencing factors (J
Am Soc Nephrol 2007;18:2429-31). This author prefers to look at current clinical state/diagnoses and recent therapeutic interventions for the patient to ascertain whether
a mixed acid-base disorder is potentially present.

3. An alternative method (and perhaps a simpler approach) to the delta ratio is to calculate the “excess gap”
compared with the AG (West J Med 1991;155:146-51).
Excess gap = AG – 14.
4. The excess gap is then added to the measured serum bicarbonate concentration. If the sum is less than
a normal serum bicarbonate concentration (e.g., 28–30 mEq/L), a mixed AG and non-AG acidosis is
present. If the sum is greater than a normal HCO3 concentration, the patient likely has an AG acidosis
and concurrent metabolic alkalosis.
5. Evaluation of respiratory compensation: See Table 21.
6. Treatment
a. Aggressive interventional therapy unnecessary until pH less than 7.20–7.25
b. Treat primary etiology! This should be the focus of treating the acid-base disorder.
c. Intravenous sources of alkali – Done conservatively in conjunction with treating primary disorder
whenever possible. The intent is not to normalize the pH but to improve the pH (definitely avoid
overcorrection).
i. Sodium bicarbonate – Most commonly used
ii. Sodium acetate – Available in PN solutions and compounded intravenous fluids
iii. Sodium citrate – Used orally for patients with chronic kidney injury
d. Total bicarbonate dose (mEq) = 0.5 x Wt (kg) x (24 - HCO3)
i. Give one-third to one-half of the calculated total dose (or 1–2 mEq/kg) for several hours to
achieve a pH of around 7.25 (avoid boluses if possible).
ii. Once the pH is around 7.25 or greater, slower correction without increasing bicarbonate more
than 4–6 mEq/L to avoid exceeding the target pH
iii. Serial ABGs (e.g., every 6 hours); watch rate of decrease in serum potassium and calcium
iv. Use of sodium bicarbonate injection is controversial in patients with lactic acidosis (Curr Opin
Crit Care 2008;14:379-83).
v. The BICAR-ICU study (Lancet 2018;392:31-40) evaluated 389 critically ill patients with
metabolic acidemia in a multicenter, intention-to-treat trial in which patients were randomized
to sodium bicarbonate therapy or placebo. Most patients had an elevated serum lactate at
enrollment. The primary outcome (composite of death by day 28 and presence of at least one
organ failure at day 7) was not statistically significant. However, in the prespecified stratum of
patients with AKI, the primary outcome was decreased in the treatment group (37% vs. 54%,
p=0.0283).
e. Adverse effects of excess sodium bicarbonate:
i. Hypernatremia, hyperosmolality, volume overload
ii. Hypokalemia, hypocalcemia, hypophosphatemia
iii. Paradoxical worsening of the acidosis (if the fractional increase in Pco2 production exceeds the
fractional bicarbonate change)
iv. Over-alkalinization

ACCP Updates in Therapeutics® 2022: Critical Care Pharmacy Preparatory Review and Recertification Course

328
Fluids, Electrolytes, Acid-Base Disorders, and Nutrition Support

Patient Case

Questions 8 and 9 pertain to the following case.


A 60-year-old woman (weight 80 kg) was admitted to the hospital after 1 week of severe diarrhea. She presents
with clinical evidence of dehydration (hypotension, tachycardia, decreased urine output) and is weak. Her serum
laboratory values are as follows: Na 145 mEq/L, K 3.0 mEq/L, Cl 118 mEq/L, total CO2 18 mEq/L, BUN 29
mg/dL, SCr 0.9 mg/dL, glucose 122 mg/dL, calcium 9.1 mg/dL, phosphorus 3.7 mg/dL, magnesium 1.4 mg/L,
albumin 3.9 g/dL, and lactate 1.6 mmol/L. Her ABG is pH 7.29, Po2 93 mm Hg, Pco2 34 mm Hg, HCO3 17 mEq/L,
and base excess -5 mEq/L. She has a 30 pack/year smoking history.

8. Which best describes the patient’s type of acid-base disorder?


A. Hyperchloremic, normal AG acidosis
B. AG acidosis
C. AG acidosis with hyperchloremia
D. Respiratory alkalosis with concurrent metabolic alkalosis

9. Which is the most appropriate initial fluid therapy for this patient?
A. 0.45% sodium chloride with potassium chloride 20 mEq/L
B. 0.9% sodium chloride with potassium chloride 20 mEq/L
C. Lactated Ringer solution
D. 5% dextrose

E. Metabolic Alkalosis: pH greater than 7.45; symptoms are not usually severe until pH is greater than 7.55–7.60.
1. Assessment (to help guide treatment) based on urinary chloride
a. Saline responsive (urinary chloride less than 10 mEq/L)
i. Excessive gastric fluid losses
ii. Diuretic therapy (especially loop diuretics)
iii. Dehydration (contraction alkalosis)
iv. Hypokalemia
v. (Over-) Correction of chronic hypercapnia
b. Saline resistant (urinary chloride greater than 20 mEq/L)
i. Excessive mineralocorticoid activity (e.g., hydrocortisone, fludrocortisone)
ii. Excessive alkali intake
iii. Profound potassium depletion (serum potassium less than 3 mEq/L)
iv. Excess licorice (glycyrrhizic acid, a mineralcorticoid) intake
v. Massive blood transfusion
c. Respiratory compensation (highly variable and may not be possible for ventilator-dependent
patients)
d. Intravascular volume status (important for saline-responsive alkalemia)
2. Treatment – Saline-responsive alkalemia
a. Treat underlying cause (if possible).
b. Decreased intravascular volume: Give intravenous 0.9% sodium chloride, Plasmalyte/Normosol,
or Lactated Ringer’s infusion (with potassium chloride, if necessary).

ACCP Updates in Therapeutics® 2022: Critical Care Pharmacy Preparatory Review and Recertification Course

329
Fluids, Electrolytes, Acid-Base Disorders, and Nutrition Support

c. Increased intravascular volume: Acetazolamide 250–500 mg orally or intravenously once to four


times daily plus potassium chloride if necessary (Intensive Care Med 2010;36:859-63; Crit Care
Med 1999;27:1257-61; Acta Anaesthesiol Scand 1983;27:252-4).
i. Hydrochloric acid therapy if alkalosis is refractory or initial pH greater than 7.6
(a) 0.1 N or 0.2 N of hydrochloric acid (use 0.2 N for patients requiring fluid restriction).
Hydrochloric acid should be given by central venous administration, and the solution must
be in a glass bottle.
ii. Dosage of hydrochloric acid:
(a) Chloride deficit (Arch Surg 1975;110:819-21):
Dose (mEq) = 0.2 L/kg x Wt (kg) x (103 − serum chloride)
(b) Bicarbonate excess (J Am Soc Nephrol 2000;11:369-75):
Dose (mEq) = 0.5 L/kg x Wt (kg) x (serum bicarbonate -24)
(c) A practical approach is to administer one-half of the calculated dose over 12 hours and
repeat the arterial blood gas at 6 and 12 hours with readjustment of infusion rate, if
necessary. Discontinue the infusion when the pH is less than 7.5.
3. Treatment – Saline-unresponsive alkalosis: Treat underlying cause (if possible).
a. Exogenous corticosteroids – Decrease dose or use drug with less mineralocorticoid effect.
b. Excessive alkali intake – Alter regimen.
c. Profound hypokalemia (serum potassium less than 3 mEq/L) – Aggressive potassium
supplementation
d. Rare causes: Endogenous mineralocorticoid excess (Bartter or Gitelman syndrome) –
Spironolactone, amiloride, or triamterene; consider surgery
e. Liddle syndrome: Amiloride or triamterene

III.  NUTRITION SUPPORT

A. Nutritional Assessment
1. Classes of malnutrition
a. American Society for Parenteral and Enteral Nutrition (ASPEN) international consensus
nomenclature (JPEN J Parenter Enteral Nutr 2010;34:156-9)
i. Starvation-related malnutrition (e.g., anorexia nervosa)
ii. Chronic disease–related malnutrition (e.g., Crohn’s disease, organ failure)
iii. Acute disease or injury-related malnutrition (e.g., major infection, burns, trauma)
2. Nutrition Risk Assessment
a. “Determination of Nutrition Risk” – recommended by 2016 SCCM/ASPEN guidelines to guide the
approach to nutrition support therapy in critically ill patients. High nutrition risk identifies those
patients most likely to benefit from early EN therapy. (JPEN J Parenter Enteral Nutr 2016;40:159-
211)

ACCP Updates in Therapeutics® 2022: Critical Care Pharmacy Preparatory Review and Recertification Course

330
Fluids, Electrolytes, Acid-Base Disorders, and Nutrition Support

i. Nutritional Risk Screening (NRS 2002; Clin Nutr 2003;22:321-36)

Table 23. Determination of Nutrition Risk by NRS 2002 score


Nutritional Status Severity of Disease
Score Score
Normal Nutritional Status 0 Normal nutritional requirements 0
Hip fracture, chronic patients with acute
Wt loss > 5% in 3 months or food intake < 50%
1 complications (e.g., cirrhosis, COPD, DM, 1
to 70% of normal in preceding week
oncology)
Wt loss > 5% in 2 months or body mass index
(BMI) 18.5 to 20.5 + impaired general condi- Major abdominal surgery, stroke, pneumonia,
2 2
tion or food intake < 25% to 50% of normal in hematologic malignancy
preceding week
Wt loss > 5% in 1 month (~15% in 3 months) or
Head injury, bone marrow transplantation,
BMI < 18.5 + impaired general condition or food 3 3
critically ill ICU patient
intake 0 to 25% of normal in preceding week
Add the two scores (nutritional status + severity of disease). If age 70 years or greater, add 1 to total score. If age-
corrected score 3 or greater, start nutritional support.

ii. Nutrition Risk in the Critically Ill (NUTRIC) score (Crit Care 2011;15:R268)

Table 24. Determination of Nutrition Risk by the NUTRIC Score


Variable Range Points
< 50 0
Age, yrs 50 - 74 1
> 74 2
< 15 0
15 -19 1
APACHE II
20 - 28 2
> 28 3
<6 0
SOFA 6–9 1
>9 2
0-1 0
Number of comorbidities
>1 1
0-<1 0
Days from hospital to ICU admission
≥1 1
0 - < 400 0
IL-6
≥ 400 1
High Score = 6 to 10 points: Associated with worse clinical outcomes and
are most likely to benefit from aggressive nutrition therapy
Low Score = 0 to 5 points: These patients have a low nutrition risk

iii. Modified NUTRIC Score (Clin Nutr 2016;35:158-62). Because IL-6 is not routinely available
for assessment, a modified NUTRIC (mNUTRIC) score has been validated, which omits IL-6.
An mNUTRIC score of 5 or greater is considered high nutrition risk.

ACCP Updates in Therapeutics® 2022: Critical Care Pharmacy Preparatory Review and Recertification Course

331
Fluids, Electrolytes, Acid-Base Disorders, and Nutrition Support

b. “Classic” definition – not in favor with conventional assessment techniques and tools
i. Marasmus (e.g., decreased fat/muscle protein stores but normal serum proteins)
ii. Kwashiorkor (e.g., normal fat, decreased muscle protein, decreased serum proteins)
iii. Kwashiorkor-Marasmus mix (decreased fat, muscle protein, and serum proteins)
c. Based on weight loss – A 5% unintentional weight loss within a 3-month period or a 10%
unintentional weight loss within a 6-month period is considered significant.
d. Based on body mass index (BMI) = weight (kg)/height2 (m 2)
i. Less than 18.5 kg/m2: Underweight
ii. 18.5–24.9 kg/m2: Normal
iii. 25–29.9 kg/m2: Overweight
iv. 30–34.9 kg/m2: Class I obesity
v. 35–39.9 kg/m2: Class II obesity
vi. Greater than 40 kg/m2: Class III obesity
e. Empiric weight adjustment for amputations

Table 25. Body Compartments’ Contribution to Body Weight (J Am Diet Assoc 1995;95:215-8)
Body Part Amputation Approximate Contribution to Body Weight (%)
Foot 1.5
Calf, foot 5.9
Leg (from hip) 16
Hand 0.7
Hand and forearm 2.3
Arm 5

3. Other laboratory and physical indicators for nutrition assessment: Visceral protein concentrations (e.g.,
albumin, prealbumin, transferrin) are not reliable indicators and should not be used to assess nutrition
status in critically ill patients (JPEN J Parenteral Enteral Nutr 2016;40:159-211).
a. Physical examination: Loss of subcutaneous body fat, muscle atrophy (including temporal wasting),
presence of lower extremity edema and/or ascites
b. Subjective Global Assessment (JPEN J Parenter Enteral Nutr 1987;11:8-13): Incorporates overall
evaluation by incorporating five elements of the patient’s history (presence of weight loss during
the past 6 months; dietary intake change; presence of significant adverse GI symptoms such as
diarrhea, vomiting, nausea, or anorexia that were persistent for more than 2 weeks; physical
functional capacity including difficulty with ambulation/normal activities or bed/chair-ridden,
and metabolic demands of the patient’s disease state); and physical examination (presence of
subcutaneous fat, muscle wasting, edema, ascites). The final Subjective Global Assessment rating
is classified as A = well nourished, B = moderately malnourished, or C = severely malnourished.

B. Energy Requirements
Assessing caloric requirements: Indirect calorimetry – Measured energy expenditure by oxygen consump-
tion and CO2 production – The “gold standard”
1. Respiratory quotient (Vco2/Vo2); 1 for carbohydrate oxidation; 0.7 for fat oxidation; 0.8 for protein
oxidation; greater than 1 usually implies overfeeding (net fat synthesis), less than 0.7 suggests ketosis
or an error in measurement (too much fraction of inspired oxygen [Fio2] variability at higher Fio2
concentrations). Widespread use in clinical practice is limited by availability and cost of indirect
calorimeters. Furthermore, accuracy of measurements is affected by many common factors in the ICU
(presence of chest tubes, use of supplemental oxygen, FiO2 settings, PEEP settings, continuous renal
replacement therapy, anesthesia, movement).

ACCP Updates in Therapeutics® 2022: Critical Care Pharmacy Preparatory Review and Recertification Course

332
Fluids, Electrolytes, Acid-Base Disorders, and Nutrition Support

2. Organization guideline recommendations

Table 26. Guideline Recommendations for Caloric Intake


Organization Recommendation
American College of Chest
25 kcal/kg/day; increase by 10%–20% with SIRS
Physicians (Chest 1997;111:769-78)
25–30 kcal/kg/day or use of indirect calorimetry (lower amounts
may be given to medical ICU patients with ARDS for the first week)
Obesity: Goal EN regimen should not be > 65%–70% target energy
SCCM/ASPEN (JPEN J Parenter
requirements as measured by indirect calorimetry. For BMI 30–50 kg/
Enteral Nutr 2016;40:159-211)
m2, a weight-based equation of 11–14 kcal/kg actual body weight/day is
recommended. For BMI > 50 kg/m2, 22–25 kcal/kg ideal body weight/
day is recommended
ESPEN (Clin Nutr 2019;38:48-79) 20–25 kcal/kg (in the absence of measured REE)
ESPEN PN (Clin Nutr
25 kcal/kg/day (in the absence of measured REE)
2009;28:387-400)
ESPEN EN (Clin Nutr 20–25 kcal/kg/day (during acute phase)
2006;25:210-23) 25–30 kcal/kg/day (during recovery)
ASPEN Obesity Guidelines
< 14 kcal/kg actual weight/day or 50%–70% of estimated requirements
(JPEN J Parenter Enteral Nutr
when given with a high-protein intake
2013;37:714-44)
25–30 kcal/kg/day or 1.2–1.4 x BEE (Harris-Benedict equations)
Eastern Association for Surgery of 30 kcal/kg/day (1.4 x BEE) for patients with TBI
Trauma (J Trauma 2004;57:660-9) 22–25 kcal/kg/day for patients with paraplegia
20–22 kcal/kg for patients with quadriplegia
BEE = basal energy expenditure; ESPEN = European Society for Clinical Nutrition and Metabolism; REE = resting energy expenditure; SCCM = Society of Critical
Care Medicine; SIRS = systemic inflammatory response syndrome.

3. Predictive methods
a. Mifflin-St. Jeor equations (preferred for non–ventilator-dependent patients with obesity, AKI or
CKD, or hepatic encephalopathy when a hypocaloric, high-protein regimen is not possible)
i. Women = (10 x Wt) + (6.25 x Ht) – (5 x Age) − 161*
ii. Men = (10 x Wt) + (6.25 x Ht) − (5 x Age) + 5*
*Age (years); Ht (centimeters); Wt = actual body weight (kilograms).
b. Penn State equation (preferred for ventilator-dependent patients with obesity):
REE = (Mifflin x 0.96) + (Tmax x 167) + (Ve x 31) − 6212*
*REE = resting energy expenditure; Tmax = maximum temperature in degrees Celsius; Ve = min-
ute ventilation, liters per minute.
c. Modified Penn State equation (preferred for ventilator-dependent patients with obesity 60 years or
older
REE = (Mifflin x 0.71) + (Tmax x 85) + (Ve x 64) − 3085*
d. Basal energy expenditure (BEE) – Harris-Benedict equations (preferred for small adults and older
adult patients)
i. Women = 655 + (9.6 x Wt) + (1.7 x Ht) − (4.7 x Age)*
ii. Men = 66 + (13.7 x Wt) + (5 x Ht) − (6.8 x Age)*
*Wt (kilograms), Ht (centimeters), Age (years).

ACCP Updates in Therapeutics® 2022: Critical Care Pharmacy Preparatory Review and Recertification Course

333
Fluids, Electrolytes, Acid-Base Disorders, and Nutrition Support

4. Adverse effects of overfeeding – Do not exceed 5 mg/kg/minute of glucose/carbohydrate in the acute


phase of critical illness (Ann Surg 1979;190:274-85); limit total caloric intake (do not exceed 1.3–1.5 x
measured resting energy expenditure); do not exceed soybean oil (SO)-based ILE intake of 2.5 g/kg/day
(most clinicians limit SO-based ILE to around 1.5 g/kg/day or less).
a. Hypercapnia: It was traditionally thought that excessive glucose intake alone was responsible for
hypercapnia observed during overfeeding. However, studies of acutely ill patients showed that
aggressive feeding resulted in marked increases in CO2 production (Ann Surg 1980;191:40-6;
JAMA 1980;243:1444-7). Substitution of glucose kilocalories with lipid decreases CO2 production
(Anesthesiology 1981;54:373-7) when overfeeding but does not alter CO2 production if not
overfeeding (e.g., 1.3 x BEE) (Chest 1992;102:551-5). Because most institutions lack the ability to
measure energy expenditure, estimates are used. If the patient experiences hypercapnia without a
known cause, the nutrition therapy should be suspected and the caloric intake empirically decreased
(especially if the patient is having difficulty weaning from the ventilator).
b. Hyperglycemia: In a retrospective study of 102 PN-fed patients not predisposed to hyperglycemia,
dextrose intakes in excess of 5 mg/kg/minute resulted in substantial hyperglycemia (blood glucose
[BG] greater than 200 mg/dL) in 18 of 37 patients (Nutr Clin Pract 1996;11:151-6). Patients with
stress-induced hyperglycemia or diabetes are even more susceptible to hyperglycemia with EN or
PN.
c. Fatty infiltration of the liver: May be because of overfeeding with fat or carbohydrate. Usually
presents as a cholestatic liver disease (increased gamma-glutamyl transferase (GGT), alkaline
phosphatase, and ultimately bilirubin) after at least 1 week to 10 days of overfeeding (Arch Surg
1978;113:504-8). May be transient or reversible or can progress to end-stage liver disease. Throughout
a few weeks, patients can appear jaundiced. Patients with critical illness and/or infections tend to
be more susceptible to hepatic steatosis compared with non–critically ill patients (possibly because
of an exaggerated inflammatory process). Although treatment with fish oil appears promising in
infants and children (Nutr Clin Pract 2013;28:30-9; Ann Surg 2009;250:395-402), data for adults
are limited. Usual treatment for adult patients with suspected PN-associated liver disease is first to
ensure that the patient is not being overfed followed by cyclic PN (PN is infused for part of the day).
Cyclic PN is typically not appropriate for critically ill patients due to the high glucose infusion rate
and large volumes of fluid (relative to time) required for administration. Reinstituting EN as soon as
possible (if possible) is of utmost importance.

C. Protein Requirements
1. Guideline recommendations

Table 27. Guideline Recommendations for Protein Intake


Organization Recommendation
1.2–1.5 g/kg/day
American College of Chest
1.5–2 g/kg/day not to exceed 2 g/kg/day with SIRS
Physicians (Chest 1997;111:769-78)
Routine nitrogen balance (NB) determinations recommended
1.2–2 g/kg/day; higher amounts are likely needed for multiple trauma or
burns
SCCM/ASPEN (JPEN J Parenter
For patients with obesity receiving a hypocaloric (low energy, high pro-
Enteral Nutr 2016;40:159-211)
tein) regimen: 2 g/kg/day IBW for BMI 30–40 kg/m 2, up to 2.5 g/kg/day
IBW for BMI ≥ 40 kg/m2
ESPEN PN
1.3–1.5 g/kg IBW/day
(Clin Nutr 2009;28:387-400)

ACCP Updates in Therapeutics® 2022: Critical Care Pharmacy Preparatory Review and Recertification Course

334
Fluids, Electrolytes, Acid-Base Disorders, and Nutrition Support

Table 27. Guideline Recommendations for Protein Intake (continued)


Organization Recommendation
ESPEN EN
Not given
(Clin Nutr 2006;25:210-23)
ASPEN Obesity Guidelines
1.2 g/kg actual weight/day or 2–2.5 g/kg IBW/day when given with a
(JPEN J Parenter Enteral Nutr
hypocaloric regimen
2013;37:714-44)
Eastern Association for Surgery of
1.25–2 g/kg/day; 2 g/kg/day for burns
Trauma (J Trauma 2004;57:660-9)
Minimum of 1.2 g/kg/day with doses up to 2–2.5 g/kg/day
Obese (BMI 30–39.9 kg/m2): 2 g/kg IBW/day
International Protein Summit
Obese (BMI ≥ 40 kg/m2): 2.5 g/kg IBW/day
Consensus recommendations
AKI: 1.2–2 g/kg/day
(Nutr Clin Pract 2017;32(suppl
CRRT: 1.5–2.5 g/kg/day
1):142S-151S) – in reference to ICU
Age > 60 yr: 2–2.5 g/kg/day
patients
Persistent inflammation catabolism syndrome: 1.2–2 g/kg/day
Infants and children: A minimum of 1.5 g/kg/d

Patient Case

10. A 40-kg woman admitted to the trauma ICU receives a PN solution containing 350 g of dextrose, 160 g of
amino acids, and 60 g of lipid daily. She has normal renal and hepatic function. Her most recent ABG from
the morning shows a pH of 7.30, Pco2 55 mm Hg, Po2 96 mm Hg, and HCO3 31 mEq/L. Her fingerstick BG
values from the past 24 hours are 180–200 mg/dL. Which would be best to recommend regarding her PN?
A. Decrease dextrose to 175 g/day, and increase lipid to 120 g/day.
B. Add 20 units of regular human insulin per day to the PN solution.
C. Decrease all the macronutrients by about one-half.
D. Increase the acetate content of the PN solution.

2. Does more protein really make a difference?


a. Weijs et al. (2012) (JPEN J Parenter Enteral Nutr 2012;36:60-8): 28-day mortality improved in
those who received an average of 1.3 g/kg/day versus 1.1 or 0.8 g/kg/day (886 mixed ICU patients).
b. Allingstrup et al. (2012) (Clin Nutr 2012;31:462-8): 28-day mortality improved in those who
received an average of 1.5 g/kg/day versus 1.1 or 0.8 g/kg/day (113 mixed ICU patients).
c. JPEN J Parenter Enteral Nutr 2016;40:45-51: Improved survival with protein intake greater than
80% of goal (1.2 g/kg/day) in 2828 mixed ICU patients. Achieving 80% of caloric goals did not
affect mortality.
d. Crit Care Med 2017;45:156-63: Mortality was decreased with increased protein intake for high-risk
(NUTRIC score greater than 4) mixed ICU patients with a prolonged ICU stay of more than 11
days. Low-risk patients were unaffected.

ACCP Updates in Therapeutics® 2022: Critical Care Pharmacy Preparatory Review and Recertification Course

335
Fluids, Electrolytes, Acid-Base Disorders, and Nutrition Support

Patient Case

Questions 11–12 pertain to the following case.


A 60-year-old man (weight 65 kg; about 90% IBW) is admitted for acute pancreatitis. After 4 days, he still
has significant abdominal pain and distension without resolution. He has a fever, and blood/urine cultures are
obtained. A CT scan of the abdomen reveals an ileus and an intra-abdominal abscess. He is given a goal PN regi-
men containing 300 g of dextrose, 70 g of amino acids, and 40 g of 20% lipids daily.

11. Which best depicts the kilocalories and protein this regimen will provide?
A. 26 kcal/kg/day and 1.1 g/kg/day
B. 26 kcal/kg/day and 1.5 g/kg/day
C. 28 kcal/kg/day and 1.5 g/kg/day
D. 30 kcal/kg/day and 1.5 g/kg/day

12. Which changes would be best for this patient’s PN regimen?


A. Increase dextrose to 400 g/day.
B. Decrease dextrose to 200 g/day.
C. Increase protein to 100 g/day.
D. Increase lipids to 70 g/day.

D. Principles of EN and PN
1. Indications for EN: If the patient is unable to eat adequate amounts to achieve goal nutritional intake.
EN is preferred to PN because EN helps maintain the integrity of the GI tract, has fewer infectious
complications, is associated with decreased ICU length of stay, and is more cost-effective (JPEN J
Parenter Enteral Nutr 2009;33:277-316; Ann Surg 1992;215:503-13; JPEN J Parenter Enteral Nutr
2016;40:159-211). This position has been challenged recently because of newer literature (JAMA
2013;309:2130-8; Lancet 2013;381:385-93; N Engl J Med 2014;371:1673-84; Crit Care 2016;20:117;
Clin Nutr 2017;36:623-50) indicating a reduction in infectious complications with PN as compared
with the past literature (likely because of improvements in catheter care, PN formula management
and compounding practices, and glycemic control standards over the past several decades) with the
difference in infectious complications between EN and PN narrowing.
a. Lack of bowel sounds, flatus, or bowel movement is not a contraindication for EN because these
are nonspecific indicators of GI function (JPEN J Parenter Enteral Nutr 2009;33:277-316; SCCM/
ASPEN 2009).
b. Evidence of ileus (e.g., dilated loops of bowel on abdominal radiography) is, however, a relative
contraindication for EN.
c. High NG output (greater than around 800 mL NG output) in a 24-hour period may indicate delayed
gastric emptying, and the patient may not be ready for EN when fed into the stomach and post-
pyloric feeding is not possible. High NG output may also indicate an ongoing ileus or obstruction,
in which case, PN may be indicated.
d. Refusal to eat/anorexia is not an absolute contraindication for EN. Ensure appropriate dietary
preferences, and add high-calorie/protein liquid supplements to meals and bedtime snack first.

ACCP Updates in Therapeutics® 2022: Critical Care Pharmacy Preparatory Review and Recertification Course

336
Fluids, Electrolytes, Acid-Base Disorders, and Nutrition Support

2. EN formulas

Table 28. EN Formulas


Additional
Product Category Indication Macronutrients Examples
Comments
Standard tube 1–1.2 kcal/mL Polymeric, 300–500 Jevity, Nutren 1.0,
Minimal stress
feeding Protein 40–50 g/L mOsm/kg, fiber Fibersource HN
Congestive heart Nutren 2.0,
2 kcal/mL Polymeric, 700–800
Volume restricted failure, fluid-restricted Resource 2.0,
Protein 60–80 g/L mOsm/kg, fiber
patients TwoCal HN
AKI (predialysis), High calorie, low
renal dysfunction protein, no or low
2 kcal/mL
Renal with increased serum electrolytes, 600 Renalcal, Suplena
Protein 30–40 g/L
potassium, phosphorus, mOsm/kg, volume
magnesium restricted, fiber
High calorie, modest
Renal failure with 1.8- 2 kcal/mL electrolytes, volume Novasource
Renal
hemodialysis Protein 80–90 g/L restricted, 1000 Renal, Nepro
mOsm/kg, fiber
Critically ill patients
Increased protein 1 kcal/mL High-protein content,
(especially trauma, Replete, Promote
needs Protein 62 g/L fiber, isotonic
surgical, burns)
Hyperglycemia, 1.2 kcal/mL Low-carbohydrate Diabetisource
Glucose intolerance
diabetes mellitus Protein 60 g/L content, fiber AC, Glucerna 1.2
Critically ill surgical
1.5 kcal/mL Additional arginine, Impact Peptide
Immune enhancing and trauma patients,
Protein 90–94 g/L glutamine; fish oil 1.5, Pivot
perioperative GI cancer
Patients with obesity
1 kcal/mL High protein, low
Bariatric with good renal Peptamen Intense
Protein 93 g/L calories
function
Low fat/MCT, di/tri-
Malabsorption, fat 1–1.5 kcal/mL Vivonex RTF,
Elemental diet peptides/free AA, no
intolerance Protein 50–68 g/L Vital
fiber, low residue
Pulmonary
(Not recommended Volume restricted,
1.2–1.5 kcal/mL Oxepa, Peptamen
for ARDS/ALI ARDS, ALI fish oil, low omega-6
Protein 63–76 g/L AF
per 2016 SCCM/ fats
ASPEN guidelines)
Hepatic (not
recommended Cirrhosis with hepatic 1.2–1.5 kcal/mL High branched chain, NutriHep,
per 2016 SCCM/ encephalopathy Protein 40–44 g/L low aromatic AA Hepatic-Aid II
ASPEN guidelines)
10–15 g of protein
Protein liquid, gel, or
Protein supplement High protein needs per serving is typical Pro-Stat, ProMod
powder
(products vary)
AA = amino acids; ALI = acute lung injury; ARDS = acute respiratory distress syndrome; MCT = medium-chain triglyceride.

ACCP Updates in Therapeutics® 2022: Critical Care Pharmacy Preparatory Review and Recertification Course

337
Fluids, Electrolytes, Acid-Base Disorders, and Nutrition Support

3. Tube feeding (TF) – Medication interactions: Hold TF 1 hour before and after drug administration.
*Increase TF rate to account for time off TF.
a. Warfarin (Pharmacotherapy 2008;28:308-13)
b. Phenytoin (Nutr Clin Pract 1996;11:28-31)
c. Levothyroxine (J Endocrinol Invest 2014;37:583-7; Nutr Clin Pract 2010;25:646-52) - Liquid
levothyroxine preparation alleviates need to hold TF.
d. Itraconazole (Antimicrob Agents Chemother 1997;41:2714-8)
e. Fluoroquinolones (J Antimicrob Chemother 1996;38:871-6)
*Some clinicians have empirically increased the dosage of these drugs while giving continuous
enteral feeding rather than holding the EN for 1 hour before and after drug administration. This
author discourages this practice, especially for warfarin and phenytoin, because the doses neces-
sary to overcome the effects of drug binding to the continuous EN are potentially toxic when the
EN is held or discontinued without a dose adjustment. Others have increased the ciprofloxacin dose
to 750 mg twice daily during continuous EN to achieve therapeutic plasma concentrations well
above the MIC (minimum inhibitory concentration) for a gram-negative urinary tract infection (J
Antimicrob Chemother 1996;38:871-6).
4. Indications for PN
a. European Society for Clinical Nutrition and Metabolism (ESPEN) ICU guidelines (2019) (Clin
Nutr 2019;38:48-79): When oral nutrition and EN are contraindicated, PN should be initiated within
3–7 days in critically ill patients.
b. SCCM/ASPEN (2016) (JPEN J Parenter Enteral Nutr 2016;40:159-211) It is recommended to initiate
PN as soon as possible after intensive care unit (ICU) admission if patients have contraindications
to EN and are severely malnourished or at “high nutrition risk” as indicated by the NRS 2002 or
NUTRIC score. It is also recommended that supplemental PN be initiated in patients at either low
or high nutrition risk after 7 to 10 days if patients are not able to meet at least 60% of energy and
protein requirements. PN should be avoided in the acute phase of sepsis regardless of the degree of
nutrition risk because studies show longer hospital and ICU stays, longer duration of organ support,
higher incidence of infectious complications, and higher hospital mortality with early supplemental
and/or exclusive PN (JPEN J Parenter Enteral Nutr 2016;40:159-211). See a recent review on the
controversial role of supplemental PN in adult patients (Nutr Clin Pract 2018;33:359-69).
c. Other possible indications for PN: Severe, intractable vomiting or diarrhea, obstruction, impaired
absorption (e.g., short bowel syndrome, high ostomy output), high-output enterocutaneous fistula
(more than 500 mL/day), ischemic bowel, bowel discontinuity (JPEN J Parenter Enteral Nutr
2017;41:324-77)
d. Summary: The approach depends on several factors (e.g., if the patient is malnourished or well
nourished before ICU admission, patient acuity). Early nutrition (defined as within 24–72 hours
according to published studies) appears to be beneficial for those with prolonged ICU stays and
a high level of catabolism, including trauma, TBI, and thermal injury and for some surgical
subpopulations. Impact of early nutrition appears more variable with respect to clinical outcome
for medical ICU patients and is likely related to a shorter duration in ICU stay and a lower level
of catabolism for many patients. Recent literature supports the safe use of PN as a substitute for
EN (when EN is contraindicated or when EN delivery is inadequate) with no difference in the
incidence of infections.
5. PN formulations
a. Peripheral versus central venous administration
i. Osmolality of peripheral administration is limited to less than 900 mOsm/kg.
ii. Because of the osmolality issue, peripheral PN solutions are “diluted,” requiring large volumes
(contraindicated for fluid-restricted patients and difficult for older patients) and typically do
not allow for adequate protein and energy provision.

ACCP Updates in Therapeutics® 2022: Critical Care Pharmacy Preparatory Review and Recertification Course

338
Fluids, Electrolytes, Acid-Base Disorders, and Nutrition Support

iii. Phlebitis is common with peripheral PN and it is difficult to use beyond 2–3 days
iv. A central PN formulation should be delivered into a line with the catheter tip ending in a large
diameter vein, such as the distal superior vena cava, adjacent to the right atrium. Central PN
formulations are generally preferred for critically ill patients who require PN.
b. Safe practice guidelines for prescribing PN solutions – Should be prescribed in total amount per
day (e.g., glucose 200 g/day, amino acids 150 g/day, lipid 30 g/day, fluid volume 2500 mL/day,
sodium chloride 60 mEq/day, potassium acetate 80 mEq/day), NOT by concentrations (e.g., 20%
dextrose in water, 8% amino acids) or by compounding techniques (e.g., 500 mL of 50% dextrose
in water plus 500 mL of 10% amino acids)
c. Glucose requirements
i. Obligatory requirements for CNS, renal medulla, bone marrow, leukocytes, etc.: Around 130
g/day
ii. Surgical wound healing requires about 80–150 g/day (based on atrioventricular differences
and blood flow from a burned limb)
iii. Caloric contribution of glucose: 3.4 kcal/g (as opposed to carbohydrate 4 kcal/g)
iv. Mean glucose oxidation rate in critically ill patients is around 5 mg/kg/day (or about 25 kcal/
kg/day as glucose). In general, most clinicians avoid exceeding this glucose intake in the acute
phase of critical illness.
d. ILE requirements
i. SO-based ILE products have historically been the main source of ILE in the United States –
May be given separately from the PN admixture or as part of the PN solution. When given
separately from the dextrose/amino acid formulation, the maximum allowable hang time
according to the FDA is 12 hours. Guidelines suggest withholding SO-based ILE during the
first week following initiation of PN in critically ill patients (or limiting to 100 g/week during
this time). This is due to the high omega-6 fatty acid (more pro-inflammatory) content of SO-
based ILE and limited evidence suggesting worse outcomes for ICU patients who receive ILE
early in their ICU course. However, this evidence is very limited with several notable design
flaws that may have attributed to study outcomes (J Trauma 1997;43:52-60). Several alternative
ILE products are now commercially available in the United States. SMOFlipid consists of 30%
soybean oil, 30% medium-chain triglycerides, 25% olive oil (OO), and 15% fish oil. It may be
more advantageous than 100% SO ILE because of lower serum triglyceride concentrations
and decreased exposure to SO without compromising caloric intake, and limited data analyses
suggest decreased infections in ICU patients and decreased liver function tests with prolonged
PN use. Daily administration is usually required to meet essential fatty acid requirements (met
only by the SO component). Minimal intravenous drug compatibility data is available with
Smoflipid in adult PN formulations, which may limit its use if a dedicated line is not available
for the PN solution. A 100% fish oil ILE product recently received FDA approval for use in
pediatric patients in the United States, and an OO-based ILE product (80% OO, 20% SO) was
also recently introduced to the U.S. market. While there is no definitive evidence that supports
the use of these alternative ILE products over SO-based ILE, guidelines recommend their
use be considered in ICU patients who require PN. All clinical and practical considerations
listed previously should be evaluated when choosing which ILE product to use for critically
ill patients.
ii. Caloric contribution of intravenous fat emulsion: 10% = 1.1 kcal/mL (11 kcal/g); 20% = 2 kcal/
mL (10 kcal/g); 30% = 3 kcal/mL (10 kcal/g)

ACCP Updates in Therapeutics® 2022: Critical Care Pharmacy Preparatory Review and Recertification Course

339
Fluids, Electrolytes, Acid-Base Disorders, and Nutrition Support

iii. Dosage: About 100–150 g of soybean oil weekly (or 1–1.5 g/kg weekly) is enough to prevent
essential fatty acid deficiency (EFAD). The FDA states a maximum upper limit of 2.5 g/kg/
day in adults, though SO ILE provision in critically ill patients should generally be maintained
at less than 1 g/kg/day. The recommended dosing for SMOFlipid is 1 -2 g/kg/day, and OO-
based ILE formulations are typically dosed at around 1 -1.5 g/kg/day. However, essential fatty
acid requirements can be met with SMOFlipid doses as low as 12.6% and OO-based ILE
formulations as low as 12-25% of total calories. (Nutr Clin Pract 2020;35:769-82).
iv. Biochemical evidence for EFAD (the “classic definition” is an increased triene/tetraene
[eicosatrienoic acid/arachidonic acid] ratio greater than 0.2) occurs in 30%, 66%, 83%, and
100% of patients after 1, 2, 3, and 4 weeks of fat-free “full-calorie, continuous” PN (Surgery
1978;84:271-7). Clinical signs and symptoms of EFAD usually do not occur until about 2
weeks after biochemical evidence in adults. Because the investigators initiated intravenous
lipid emulsion soon after the biochemical appearance of EFAD, only 2 of 32 patients developed
clinical evidence suggestive of EFAD. EFAD can occur much sooner for infants and children.
Patients with obesity receiving hypocaloric high-protein therapy can maintain normal plasma
fatty acid profiles for up to 5 weeks (J Nutr Biochem 1994;5:243-7). Cyclic PN has been
suggested to mobilize lipid from endogenous depots, but conclusive data are lacking.
v. Clinical symptoms (dry, scaly skin; hair loss; poor wound healing) occur about 2 weeks after
biochemical evidence of deficiency in adults. Therefore, in most adults, the earliest appearance
of EFAD is after about 3 weeks of fat-free full-calorie continuous PN.
vi. Serum triglyceride concentration should be monitored at least weekly and more often for those
with proven or suspected impaired triglyceride clearance (consider withholding lipid emulsion
when serum triglyceride approaches or exceeds 400 mg/dL) (Nutr Clin Pract 2020;35:769-82).
vii. Predisposing conditions that may result in impaired clearance of triglycerides:
(a) Excessive lipid intake (often caused by propofol therapy)
(b) Acute pancreatitis
(c) Uncontrolled diabetes
(d) Liver failure
(e) Kidney failure (decreased lipoprotein lipase activity, carnitine deficiency with long-term
hemodialysis patients)
(f) End-stage sepsis (multisystem organ failure)
(g) History of hyperlipidemia
(h) Obesity
(i) HIV (occurred even before current antiretroviral therapy) (Am J Med 1989;86:27-31)
(j) Pregnancy
(k) Small-for-gestational-age neonates (carnitine synthesis is maturational-dependent)
viii. Propofol – A hidden source of lipids (10% soybean emulsion containing 1.1 kcal/mL)
e. Electrolyte requirements (see the Fluids and Electrolytes section)
f. Vitamins (multivitamin infusion or multivitamin complex 10 mL/day; extra vitamins if patient has
any vitamin deficiencies)
g. Trace minerals
i. Zinc 3 mg/day normal requirements; 5 mg/day during critical illness; increased requirements
for patients with diarrhea, intestinal fistulae. An additional 10 mg/day for a total of 13 mg/
day is usually sufficient to meet increased intestinal losses (Gastroenterology 1979;76:458-67).
Deficiency is characterized by loss of hair; erythematous rash, especially in periorbital regions
of face; poor wound healing. Classic zinc deficiency is termed acrodermatitis enteropathica.
ii. Copper 0.3–0.5 mg/day is usually sufficient (Gastroenterology 1981;81:290-7). Copper
deficiency is rare but is becoming more apparent in patients with obesity after obesity
surgical procedures. Prolonged continuous renal replacement therapy may also lead to copper
deficiency in critically ill patients (Nutr Clin Pract 2018;33:439-46). Classic presentation of
copper deficiency includes a microcytic anemia unresponsive to iron therapy or pancytopenia.
Neuropathies can also occur.

ACCP Updates in Therapeutics® 2022: Critical Care Pharmacy Preparatory Review and Recertification Course

340
Fluids, Electrolytes, Acid-Base Disorders, and Nutrition Support

iii. Chromium 10–12 mcg/day normal requirements, up to 20 mcg/day for diarrhea. Commercially
available PN components have significant levels of chromium contamination. One study found
that chromium was present in 65.6% of all PN components (JPEN J Parenter Enteral Nutr
2019;43:970-76). Therefore, many clinicians avoid supplementation with additional chromium
in the PN formula as it is likely unnecessary. Deficiency is rare. Classic presentation of
deficiency is hyperglycemia.
iv. Manganese 150–300 mcg/day is likely sufficient. Some studies state that the amount of
manganese contamination in the compounding of PN may be adequate as opposed to
supplementation. Other literature supports a dose of 55 mcg/day in PN formula to maintain
stable whole blood manganese levels (Am J Clin Nutr 2002;75:112-8). Deficiency is very rare.
Deficiency has been reported to present as a “diaper rash.” Several case reports of manganese
toxicity associated with liver disease and high manganese intake (800 mcg – 1 mg/day)
(Nutrition 2001;17:689-93). Signs and symptoms of toxicity emulate those of Parkinson disease.
v. Selenium 60 mcg/day up to 120 mcg/day for patients with diarrhea or short bowel syndrome.
Deficiency results in extreme muscle weakness and congestive cardiomyopathy. Classic
presentation with cardiomyopathy has been termed Keshan disease (named after a province
in China where the first cases of selenium deficiency with cardiomyopathy were discovered).
vi. It is common clinical practice to withhold copper and manganese in the PN formulation
for patients with hepatobiliary/cholestatic liver disease or a direct (conjugated) bilirubin
concentration greater than 2 mg/dL. MTE additives are omitted from the PN solution. Zinc
and selenium are added separately.
vii. Some clinicians withhold selenium for patients with significant renal disease who do not
receive hemodialysis or CRRT, though data in support of this practice are lacking. This can be
accomplished by providing the desired trace element ingredients individually.
viii. Typical trace element requirements can be provided with 1 mL per day of trace elements
injection 4 (dose per 1 mL: zinc 3 mg, copper 0.3 mg, manganese 55 mcg, selenium 60 mcg).
This is the first FDA-approved multi-trace product, and it replaced MTE-5, which was phased
out of production in 2020. It is important to note that this new four ingredient product is not
equivalent to the previously available multi-trace 4 (MTE-4) product as it contains different
doses of trace elements and does not contain chromium. Its formulation is more consistent
with current recommendations for parenteral trace element requirements than previous multi-
trace element products.
6. Should supplemental PN be given to patients intolerant of EN?
a. ESPEN ICU guidelines (2019) (Clin Nutr 2019;38:48-79): The safety and benefits of PN should
be weighed on a case-by-case basis for patients who do not tolerate EN at goal. When EN is not
feasible for patients at high nutrition risk or those who are severely malnourished, initiation of
low-dose PN should be carefully considered and balanced against the risks of overfeeding and
refeeding.
b. ESPEN EN (2006) (Clin Nutr 2006;25:210-23): For patients intolerant of EN, supplemental PN
should be considered. Overfeeding should be avoided.
c. SCCM/ASPEN (2016) (JPEN J Parenter Enteral Nutr 2016;40:159-211): It is recommended that
supplemental PN be initiated in patients at either low or high nutrition risk after 7 to 10 days if
patients are not able to meet at least 60% of energy and protein requirements. This is a marked
departure from the 2009 guidelines that indicate PN should not be given to [all] patients that are
unable to receive EN within the first 7 days and to reserve supplemental PN for patients unable to
meet 100% of caloric goal after 7 to 10 days.
d. Canadian Practice Guidelines Update (2014) (Nutr Clin Pract 2014;29:29-43): It is strongly
recommended that early supplemental PN or large volumes of hypertonic dextrose solutions not be
used in unselected critically ill patients (i.e., low-risk patients with short stay in ICU). In the patient
who is not tolerating adequate EN, data are insufficient to recommend when PN should be initiated.

ACCP Updates in Therapeutics® 2022: Critical Care Pharmacy Preparatory Review and Recertification Course

341
Fluids, Electrolytes, Acid-Base Disorders, and Nutrition Support

e. Casaer/EPaNIC study (N Engl J Med 2011;365:506-17) mostly surgical patients, randomized


controlled trial: EN only x 7 days; then PN initiated (hypertonic dextrose solutions x 2 days; then
PN) versus supplemental PN in addition to whatever EN patients can tolerate during the first 7 days
i. Worsened survival expressed as discharged alive from the ICU within 8 days (72% vs. 75%),
more infections (26% vs. 23%), ICU length of stay greater than 3 days (51% vs. 48%) with early
supplemental PN
ii. The patient population was limited because malnourished (BMI less than 17 kg/m 2) patients
were excluded. In addition, about 60% of the population were cardiac surgery patients, for
whom the indication for PN is questionable, 50% of patients were extubated by ICU day 2, and
70% of patients had an ICU length of stay of only 3–4 days (which would imply a questionable
severity of critical illness). Finally, only 58% of patients in the early PN group were even given
PN (for 1–2 days), and only 25% patients in the late PN group ever received PN.
f. Heidegger (2013) (Lancet 2013;381:385-93): 307 patients, two medical centers, randomized
controlled trial: Patients who received less than 60% target from EN by day 3 with an anticipated
ICU stay greater than 5 days received supplemental PN or EN alone. Supplemental PN was
discontinued by day 8.
i. Supplemental PN group had decreased infections (27% vs. 38%).
ii. Smaller study than the Casaer study. Not all patients had resting energy expenditure measured
(some were predicted resting energy expenditure). Protein target was only 1.2 g/kg/day. No
difference in ICU/hospital length of stay, mortality.
g. Summary: When to initiate supplemental PN in a critically ill patient remains controversial. The
SCCM/ASPEN guidelines recommend that supplemental PN be considered in all patients unable
to receive at least 60% of target EN by 7–10 days, but supporting evidence has notable limitations.
Many of the patients in the Casaer study should not have been given PN in the first place as it was
not indicated. The majority of those patients were discontinued from ventilator support within 2
or 3 days, able to eat, and discharged from the ICU within 4 to 5 days. This study is a reminder
that PN should not be given indiscriminately to patients as supported by the VA Cooperative
Trial (1991) because PN resulted in increased infectious complications in perioperative GI cancer
patients who were not malnourished as opposed to improved morbidity when given to those who
were malnourished. The Heidegger data showed a benefit from supplemental PN in a sicker patient
population than Casaer et al.’s population. Short-term supplemental PN may be indicated for
patients who are anticipated to have prolonged duration of inability to use GI tract, anticipated
prolonged duration of ICU stay, and who are either malnourished or who have a high level of
catabolism. An evaluation of selected studies that support and question the use of supplemental PN
was recently published (Nutr Clin Pract 2018;33:359-69).

E. Timing of Initiation of Nutrition Support: Early or not for ICU patients?


1. ESPEN EN (2006) (Clin Nutr 2006;25:210-23): Questionable benefit of early versus delayed EN;
however, it was recommended that critically ill patients who are hemodynamically stable and have a
functioning GI tract be fed early (less than 24 hours), if possible.
2. ESPEN ICU guidelines (2019) (Clin Nutr 2019;38:48-79): If oral intake is not possible and there
are no contraindications to EN, EN should be initiated within 48 hours rather than delaying EN or
initiating PN.
3. SCCM/ASPEN (2016) (JPEN J Parenter Enteral Nutr 2016;40:159-211): Early EN should be initiated
within 24 to 48 hours in the critically ill patient who is unable to maintain volitional intake once the
patient is hemodynamically stable. It is suggested that very early EN (within 4 to 6 hours if possible) be
initiated in a patient with burn injury.

ACCP Updates in Therapeutics® 2022: Critical Care Pharmacy Preparatory Review and Recertification Course

342
Fluids, Electrolytes, Acid-Base Disorders, and Nutrition Support

4. Surviving Sepsis Campaign guidelines (2016) (Crit Care Med 2017;45:486-552): EN should be initiated
early rather than a complete fast or only intravenous glucose in critically ill patients with sepsis or septic
shock who can be fed enterally.
5. Summary: The data are confusing because several studies have used different times for the definition
of early nutrition therapy: 24, 36, 48, and 72 hours. Most evidence-based clinicians would suggest that
enteral nutrition therapy be initiated for most patients within 48 hours of ICU admission and no later
than 72 hours. Surgical ICU patients, including those with trauma and thermal injury, have been more
consistently shown to benefit from early EN as opposed to medical ICU patients, for whom results are
more variable.

F. Glycemic Control
1. Definition of the appropriate BG target range
a. Society of Critical Care Medicine (SCCM) guidelines (2012) (Crit Care Med 2012;40:3251-76): A
BG of 150 mg/dL or greater should trigger initiation of insulin therapy to keep BG less than 150
mg/dL for most patients and maintain BG absolutely less than 180 mg/dL.
b. ASPEN guidelines (2013) (JPEN J Parenter Enteral Nutr 2013;37:23-36): A target BG range of
140–180 mg/dL is recommended.
c. Surviving Sepsis Campaign guidelines (2016) (Crit Care Med 2017;45:486-552): An insulin dosing
protocol to keep BG less than 180 mg/dL, rather than an upper target of 110 mg/dL, is recommended
when the patient has two consecutive BG measurements greater than 180 mg/dL.
d. American Diabetes Association (2021) (Diabetes Care 2021;44(suppl 1):S211-220): An insulin
infusion should be used to control hyperglycemia, starting with a threshold of greater than or equal
to 180 mg/dL. BG should be maintained between 140 and 180 mg/dL for most critically ill patients.
More stringent goals such as 110–140 mg/dL may be more appropriate for selected critically ill
patients, as long as this can be accomplished without significant hypoglycemia.
e. Summary: Many evidence-based clinicians use a target BG range of 140–180 mg/dL when caring
for patients in a mixed medical-surgical ICU. A growing amount of evidence from smaller studies
shows that certain subpopulations such as trauma, traumatic brain injury, cardiothoracic surgery,
and thermal injury may benefit from tighter BG (e.g., less than 140–150 mg/dL) control if it can be
done safely without hypoglycemia (Crit Care Med 2012;40:3251-76; Nutr Clin Pract 2014;29:534-41).
2. BG monitoring frequency
a. SCCM guidelines (2012) (Crit Care Med 2012;40:3251-76): BG should be monitored every 1–2
hours for most patients receiving an insulin infusion; monitoring every 4 hours is not recommended
because of the risk of unrecognized hypoglycemia.
b. Surviving Sepsis Campaign guidelines (2016) (Crit Care Med 2017;45:486-552): BG should be
monitored every 1–2 hours during the insulin infusion and then extended to every 4 hours thereafter
once stability in BG control is achieved.
c. American Diabetes Association (2021) (Diabetes Care 2021;44(suppl 1):S211-220): BG should be
monitored every ½–2 hours during the insulin infusion.
3. Hypoglycemia
a. Most guidelines define hypoglycemia as a BG less than 70 mg/dL because increased glucagon,
catecholamine, and growth hormone production occurs when the BG falls below this concentration.
Mild to moderate hypoglycemia is usually defined as a BG concentration of 40–60 mg/dL (because
autonomic symptoms often appear) and severe (life-threatening) hypoglycemia as less than 40
mg/dL.
b. Most common risk factors for hypoglycemia during insulin therapy (Crit Care Med 2007;35:2262-
7; Crit Care Med 2006;34:96-101)

ACCP Updates in Therapeutics® 2022: Critical Care Pharmacy Preparatory Review and Recertification Course

343
Fluids, Electrolytes, Acid-Base Disorders, and Nutrition Support

i. Excessive insulin dose


ii. Abrupt discontinuation of EN or PN without an adjustment in the insulin therapy (purported
to cause 62% of severe hypoglycemic events in the 2006 Van den Berghe trial of medical ICU
patients) (Crit Care Med 2012;40:3251-76)
iii. Decreasing steroid dose while on insulin therapy
iv. Hepatic failure
v. Renal failure (half-life of insulin is prolonged, impaired renal gluconeogenesis in response to
hypoglycemia)
vi. Advanced age
vii. Inotropes, vasopressor agents, octreotide with insulin therapy
viii. Sepsis
4. Transitioning from a continuous intravenous insulin infusion
a. Lack of a transition plan results in loss of glycemic control. Different methods have been described
in the literature, and the best approach depends on whether the patient is transitioning to an oral
diet, bolus EN, or continuous EN (JPEN J Parenter Enteral Nutr 2013;37:506-16; Crit Care Med
2012;40:3251-76; ASPEN Adult Nutrition Support Core Curriculum 2012:580-602).
b. Once stable, ICU patients with type 1 or 2 diabetes receiving insulin infusions at greater than 0.5
unit/hour or those with stress-induced hyperglycemia receiving at rates greater than 1 unit/hour
should be transitioned from an intravenous insulin infusion to a basal-bolus insulin regimen before
the insulin infusion is discontinued. Several criteria should be met before transitioning to a basal-
bolus regimen, including no foreseeable interruptions to nutrition for procedures, resolution of any
peripheral edema, and discontinuation of all vasopressors (Crit Care Med 2012;40:3251-76).
c. Transitioning from a continuous intravenous regular insulin infusion to a subcutaneous basal-bolus
regimen: A long-acting insulin such as glargine administered every 24 hours can be used for basal
coverage with initial dosing at 60%–80% of the total daily dose required from the insulin infusion.
The first dose should be administered 2–4 hours before discontinuing the insulin infusion (Crit
Care Med 2012;40:3251-76). Factors affecting serum blood glucose and patient response to insulin
therapy should be incorporated into clinical decisions related to insulin dosing. Bolus insulin
regimens are designed with rapid-acting, short-acting, or regular insulin doses to be provided as
scheduled doses before meals and/or correction doses based on BG measurements (i.e., every 4–6
hours); correction doses based on BG measurements may be more appropriate for patients receiving
continuous forms of nutrition support such as EN or PN. A protocol for using intermediate-acting
insulin (NPH [neutral protamine Hagedorn]) has also been described for basal insulin coverage
in patients receiving continuous enteral nutrition (JPEN J Parenter Enteral Nutr 2013;37:506-16).

Patient Case

13. A 55-year-old woman (weight 75 kg) without diabetes is given PN after a major GI resection. She has been
weaned from mechanical ventilation and is being transferred from the ICU to the floor. Her current PN
formulation is 200 g of dextrose (1.8 mg/kg/minute), 110 g of amino acids, and 80 g of lipids (1.1 g/kg/day),
which meets her goal requirements at 26 kcal/kg/day and 1.5 g/kg/day of protein. It contains regular human
insulin at 20 units/day. During the past 24 hours, her fingerstick BG measurements have been 170–210 mg/
dL, and her serum glucose concentration is 182 mg/dL. She has received 14 units of sliding-scale regular
human insulin coverage. Which would be best to suggest for optimal glycemic control?
A. Increase regular insulin to 30 units/day.
B. Decrease dextrose to 100 g/day.
C. Increase regular insulin to 50 units/day.
D. Do not change the current regimen.

ACCP Updates in Therapeutics® 2022: Critical Care Pharmacy Preparatory Review and Recertification Course

344
Fluids, Electrolytes, Acid-Base Disorders, and Nutrition Support

G. Nutrition considerations with coronavirus disease 2019 (COVID-19)


a. Several guidance documents have been released with specific recommendations for nutrition for
patients with COVID-19 admitted to the ICU (https://www.nutritioncare.org/COVID19/). General
recommendations from previously published critical care nutrition guidelines (JPEN J Parenter
Enteral Nutr 2016;40:159-211; Clin Nutr 2019;38:48-79) are still applicable with an emphasis
on preserving resources, clustering of care, and modifying clinical practices for prevention
of transmission. Of note, these guidelines are primarily based on expert opinion, and many
recommendations were made with the consideration of minimizing the risk to hospital staff.
Publication of these recommendations also predates vaccine availability, and the guidelines may
represent a risk-benefit perspective that warrants reconsideration as further knowledge is gained
about the care of critically ill patients with COVID-19.
b. Evidence suggests that patients with COVID-19 who are at higher nutrition risk have poorer
outcomes (JPEN J Parenter Enteral Nutr 2021; 45:32-42.). Therefore, close monitoring of their
nutritional adequacy is essential.
c. Enteral nutrition
i. Bedside enteral tube placement is preferred, and some guidance documents recommend that
complicated feeding tube placements (involving radiology or endoscopy) should be avoided.
ii. Continuous EN is preferred over bolus EN regimens.
iii. Supplements to EN (i.e., protein or fiber) should be scheduled once daily instead of more
frequent dosing.
iv. Weight-based estimations for requirements are preferred to minimize healthcare workers’
exposure for indirect calorimetry. However, ongoing research suggests a persistence of
hypermetabolism for up to 3 weeks after intubation in some patients with COVID-19, in
which case weight-based equations may significantly underestimate energy needs (Crit Care
2020;24:581).
v. Prone positioning is not a contraindication to EN. Early EN practices should continue in this
setting, with initiation at trophic doses and advancement as tolerated. The head of the bed
should be elevated to at least 10 to 25 degrees.
d. Parenteral nutrition
i. PN should be considered earlier if practical or clinical barriers to EN are present and prevent
adequate nutrition delivery.
ii. Gastrointestinal involvement of COVID-19 may result in intolerance to EN and warrant
initiation of PN.
iii. PN formulas should be formulated with the minimum volume, especially in the presence of
Acute Respiratory Distress Syndrome or volume overload.
iv. Careful monitoring of serum triglycerides is warranted with provision of ILE due to reports of
severe hypertriglyceridemia in patients with COVID-19.

ACCP Updates in Therapeutics® 2022: Critical Care Pharmacy Preparatory Review and Recertification Course

345
Fluids, Electrolytes, Acid-Base Disorders, and Nutrition Support

REFERENCES

Fluids and Electrolytes critically ill patients receiving specialized nutri-


1. Adrogue HJ. Madias NE. Hypernatremia. N Engl tion support. Nutr Clin Pract 2007;22:323-8.
J Med 2000;342:1493-9. 12. Dickerson RN, Morgan LG, Cauthen AD, et al.
2. Aubier M, Murciano D, Lecocguic Y, et al. Effect Treatment of acute hypocalcemia in critically ill
of hypophosphatemia on diaphragmatic contrac- multiple-trauma patients. JPEN J Parenter Enteral
tility in patients with acute respiratory failure. N Nutr 2005;29:436-41.
Engl J Med 1985;313:420-4. 13. Dickerson RN, Morgan LM, Croce MA, et al.
3. Bongard FS. Fluids, electrolytes, and acid-base Treatment of moderate to severe acute hypocal-
balance. In: Bongard FS, Stamos MJ, Passaro E Jr, cemia in critically ill trauma patients. JPEN J
et al., eds. About Surgery: A Clinical Approach. Parenter Enteral Nutr 2007;31:228-33.
Philadelphia: Elsevier Health Sciences, 1996:5-17. 14. Dickerson RN, Morgan LM, Croce MA, et al.
4. Brown KA, Dickerson RN, Morgan LM, et al. A Dose-dependent characteristics of intravenous
new graduated dosing regimen for phosphorus calcium therapy for hypocalcemic critically ill
replacement in patients receiving nutrition sup- trauma patients receiving specialized nutritional
port. JPEN J Parenter Enteral Nutr 2006;30:209-14. support. Nutrition 2007;23:9-15.
5. Burnell JM, Scribner BH, Uyeno BT, et al. The 15. Fenske W, Maier SKG, Blechschmidt A, et al.
effect in humans of extracellular pH change on Utility and limitations of the traditional diagnos-
the relationship between serum potassium concen- tic approach to hyponatremia: a diagnostic study.
tration and intracellular potassium. J Clin Invest Amer J Med 2010;123:652-7.
1956;35:935-9. 16. Greenlee M, Wingo CS, McDonough AA, et al.
6. Cheuvront SN, Kenefick RW, Sollanek KJ, et al. Narrative review: evolving concepts in potassium
Water-deficit equation: systematic analysis and homeostasis and hypokalemia. Ann Intern Med
improvement. Am J Clin Nutr 2013;97:79-85. 2009;150:619-25.
7. Clark CL, Sacks GS, Dickerson RN, et al. 17. Gump FE, Kinney JM, Long CL, et al.
Treatment of hypophosphatemia in patients receiv- Measurement of water balance—a guide to surgi-
ing specialized nutrition support using a graduated cal care. Surgery 1968;64:154-64.
dosing scheme: results from a prospective clinical 18. Hamill RJ, Robinson LM, Wexler HR, et al.
trial. Crit Care Med 1995;23:1504-11. Efficacy and safety of potassium infusion therapy
8. Dickerson RN. Hyponatremia in neurosurgical in hypokalemic critically ill patients. Crit Care
patients: syndrome of inappropriate antidiuretic Med 1991;19:694-9.
hormone or cerebral salt wasting syndrome? Hosp 19. Hamill-Ruth RJ, McGory R. Magnesium repletion
Pharm 2002;37:1336-42. and its effect on potassium homeostasis in critically
9. Dickerson RN, Alexander KH, Minard G, et al. ill adults: results of a double-blind, randomized,
Accuracy of methods to estimate ionized and “cor- controlled trial. Crit Care Med 1996;24:38-45.
rected” serum calcium concentrations in critically 20. Hillier TA, Abbott RD, Barrett EJ. Hyponatremia:
ill multiple trauma patients receiving specialized evaluating the correction factor for hyperglyce-
nutrition support. JPEN J Parenter Enteral Nutr mia. Am J Med 1999;106:399-403.
2004;28:133-41. 21. Huang CL, Kuo E. Mechanism of hypokalemia
10. Dickerson RN, Gervasio JM, Sherman JJ, et al. in magnesium deficiency. J Am Soc Nephrol
A comparison of renal phosphorus regulation in 2007;18:2649-52.
thermally injured and multiple trauma patients 22. Johnston CT, Maish GO, Minard G, et al.
receiving specialized nutrition support. JPEN J Evaluation of an intravenous potassium dosing
Parenter Enteral Nutr 2001;25:152-9. algorithm for hypokalemic critically ill patients. J
11. Dickerson RN, Henry NY, Miller PL, et al. Low Parenter Enteral Nutr 2017;41:796-804.
serum total calcium concentration as a marker 23. Kruse JA, Carlson RW. Rapid correction of
of low serum ionized calcium concentration in hypokalemia using concentrated intravenous

ACCP Updates in Therapeutics® 2022: Critical Care Pharmacy Preparatory Review and Recertification Course

346
Fluids, Electrolytes, Acid-Base Disorders, and Nutrition Support

potassium chloride infusions. Arch Intern Med treatment of hyponatraemia. Intensive Care Med
1990;150:613-7. 2014;40:320-31.
24. Kruse JA, Clark VL, Carlson RW, et al. 38. Sterns RH, Cox M, Feig PU, et al. Internal
Concentrated potassium chloride infusions in potassium balance and the control of the plasma
critically ill patients with hypokalemia. J Clin potassium concentration. Medicine (Baltimore)
Pharmacol 1994;34:1077-82. 1981;60:339-54.
25. Liamis G, Milonis H, Elisaf M. A review of 39. Thoren L. Magnesium deficiency in gastrointesti-
drug-induced hyponatremia. Am J Kidney Dis nal fluid loss. Acta Chir Scand Suppl 1963:suppl
2008;52:144-53. 306:301-65.
26. Lindsey KA, Brown RO, Maish GO III, et al. 40. Velentzas C, Meindok H, Oreopoulos DG, et al.
Influence of traumatic brain injury on potassium Visceral calcification and the CaXP product. Adv
and phosphorus homeostasis in critically ill mul- Exp Med Biol 1978;103:195-201.
tiple trauma patients. Nutrition 2010;26:784-90. 41. Wang S, McDonnell EH, Sedor FA, et al. pH
27. Maier JD, Levine SN. Hypercalcemia in the effects on measurements of ionized calcium and
intensive care unit: a review of pathophysiology, ionized magnesium in blood. Arch Pathol Lab
diagnosis, and modern therapy. J Intensive Care Med 2002;126:947-50.
Med 2015;30:235-52. 42. Whang R, Whang DD, Ryan MP. Refractory
28. Nierman DM, Mechanick JI. Biochemical response potassium repletion. A consequence of magnesium
to treatment of bone hyperresorption in chroni- deficiency. Arch Intern Med 1992;152:40-5.
cally critically ill patients. Chest 2000;118:761-6. 43. Whitmire SJ. Nutrition-focused evaluation and
29. Oh MS, Carroll HJ. Regulation of extra- and intra- management of dysnatremias. Nutr Clin Pract
cellular fluid composition and content. In: Arieff 2008;23:108-21.
AI, DeFronzo RA, eds. Fluid, Electrolyte, and 44. Zazzo JF, Troche G, Ruel P, et al. High incidence
Acid-Base Disorders, Vol. 1. New York: Churchill of hypophosphatemia in surgical intensive care
Livingstone, 1985:1-38. patients: efficacy of phosphorus therapy on myocar-
30. Orrell DH. Albumin as an aid to the interpretation dial function. Intensive Care Med 1995;21:826-31.
of serum calcium. Clin Chim Acta 1971;35:483-9.
31. Rude RK, Singer FR. Magnesium deficiency and Acid-Base Disorders
excess. Annu Rev Med 1981;32:245-59. 1. Berend K. Bedside rule secondary response in
32. Sacks GS, Brown RO, Dickerson RN, et al. metabolic acid-base disorders in unreliable. J Crit
Mononuclear blood cell magnesium content and Care 2013;28:1103.
serum magnesium concentration in critically ill 2. Berthelsen P. Cardiovascular performance and
hypomagnesemic patients after replacement ther- oxyhemoglobin dissociation after acetazol-
apy. Nutrition 1997;13:303-8. amide in metabolic alkalosis. Intensive Care Med
33. Schloerb PR, Wood JG, Casillan AJ, et al. J 1982;8:269-74.
Parenter Enteral Nutr 2004;28:27-9. 3. Boyd JH, Walley KR. Is there a role for sodium
34. Schucker JJ, Ward KE. Hyperphosphatemia bicarbonate in treating lactic acidosis from shock?
and phosphate binders. Am J Health Syst Pharm Curr Opin Crit Care 2008;14:379-83.
2005;62:2355-61. 4. Brimioulle S, Berre J, Dufaye P, et al. Hydrochloric
35. Segal A. Potassium and the dyskalemias. In: Mount acid infusion for treatment of metabolic alkalosis
DB, Sayegh MH, Singh AK, eds. Core Concepts associated with respiratory acidosis. Crit Care
in the Disorders of Fluid, Electrolytes, and Acid- Med 1989;17:232-6.
Base Balance. New York: Springer, 2013:49-102. 5. Dubin A, Menises MM, Masevius FB, et al.
36. Sloan GM, White DE, Murray MS, et al. Calcium Comparison of three different methods of evalu-
and phosphorus metabolism during total paren- ation of metabolic acid-base disorders. Crit Care
teral nutrition. Ann Surg 1983;197:1-6. Med 2007;35:1264-70.
37. Spasovski G, Vanholder R, Allolio B, et al. 6. Faisy C, Mokline A, Sanchez O, et al. Effectiveness
Clinical practice guideline on diagnosis and of acetazolamide for reversal of metabolic alkalosis

ACCP Updates in Therapeutics® 2022: Critical Care Pharmacy Preparatory Review and Recertification Course

347
Fluids, Electrolytes, Acid-Base Disorders, and Nutrition Support

in weaning COPD patients from mechanical venti- 22. Krintel JJ, Haxholdt OS, Berthelsen P, et al.
lation. Intensive Care Med 2010;36:859-63. Carbon dioxide elimination after acetazolamide in
7. Fencl V, Jabor A, Kazda A, et al. Diagnosis patients with chronic obstructive pulmonary dis-
of metabolic acid-base disturbances in criti- ease and metabolic alkalosis. Acta Anaesthesiol
cally ill patients. Am J Respir Crit Care Med Scand 1983;27:252-4.
2000;162:2246-51. 23. Mazur JE, Devlin JW, Peters MJ, et al. Single
8. Figge J, Jabor A, Kazda A, et al. Anion gap and versus multiple doses of acetazolamide for meta-
hypoalbuminemia. Crit Care Med 1998;26:1807-10. bolic alkalosis in critically ill medical patients: a
9. Galla JH. Metabolic alkalosis. J Am Soc Nephrol randomized, double-blind trial. Crit Care Med
2000;11:369-75. 1999;27:1257-61.
10. Haber RJ. A practical approach to acid-base disor- 24. Rastegar A. Use of the ∆AG/∆HCO3- ratio in the
ders. West J Med 1991;155:146-51. diagnosis of mixed acid-base disorders. J Am Soc
11. Harken AH, Gabel RA, Fencl V, et al. Hydrochloric Nephrol 2007;18:2429-31.
acid in the correction of metabolic alkalosis. Arch 25. Salem MM, Mujais SK. Gaps in the anion gap.
Surg 1975;110:819-21. Arch Intern Med 1992;152:1625-9.
12. Heidegger CP, Berger MM, Graf S, et al. 26. Williams DB, Lyons JH Jr. Treatment of severe
Optimisation of energy provision with supplemen- metabolic alkalosis with intravenous infu-
tal parenteral nutrition in critically ill patients: sion of hydrochloric acid. Surg Gynecol Obstet
a randomised controlled clinical trial. Lancet 1980;150:315-21.
2013;381:385. 27. Zazzo JF, Troche G, Ruel P, et al. High incidence
13. Hoste EA, Colpaert K, Vanholder RC, et al. Sodium of hypophosphatemia in surgical intensive care
bicarbonate versus THAM in ICU patients with patients: efficacy of phosphorus therapy on myocar-
mild metabolic acidosis. J Nephrol 2005;18:303-7. dial function. Intensive Care Med 1995;21:826-31.
14. Jaber S, Paugam C, Futier E, et al. Sodium bicar-
bonate therapy for patients with severe metabolic Nutrition
acidaemia in the intensive care unit (BICAR-ICU): 1. Allingstrup MJ, Esmailzadeh N, Wilkens Knudsen
a multicentre, open-label, randomised controlled, A, et al. Provision of protein and energy in rela-
phase 3 trial. Lancet 2018;392:31-40. tion to measured requirements in intensive care
15. Juern J, Khatri V, Weigelt J. Base excess: a review. patients. Clin Nutr 2012;31:462-8.
J Trauma Acute Care Surg 2012;73:27-32. 2. Alvares TS, Conte-Junior CA, Silva JT, et al. Acute
16. Kaplan JA, Cheung NHT, Maerz L, et al. A physi- L-arginine supplementation does not increase
cochemical approach to acid-base balance in nitric oxide production in healthy subjects. Nutr
critically ill trauma patients minimizes errors and Metab (Lond) 2012;9:54.
reduces inappropriate volume expansion. J Trauma 3. American Diabetes Association (ADA). 15.
2009;66:1045-51. Diabetes care in the hospital: standards of
17. Kaplan JA, Kellum JA. Initial pH, base deficit, lac- medical care in diabetes - 2021. Diabetes Care
tate, anion gap, strong ion difference, and strong 2021;44(suppl 1):S211-220.
ion gap predict outcome from major vacular injury. 4. Arabi YM, Tamim HM, Dhar GS, et al. Permissive
Crit Care Med 2004;32:1120-4. underfeeding and intensive insulin therapy in criti-
18. Kellum JA. Determinants of blood pH in health cally ill patients: a randomized controlled trial.
and disease. Crit Care 2000;4:6-14. Am J Clin Nutr 2011;93:569-77.
19. Kellum JA. Disorders of acid base balance. Crit 5. Arrazcaeta J, Lemon S. Evaluating the signifi-
Care Med 2007;35:2630-6. cance of delaying intravenous lipid therapy during
20. Kellum JA, Kramer DJ, Pinsky MR. Strong ion the first week of hospitalization in the intensive
gap: a methodology for exploring unexplained care unit. Nutr Clin Pract 2014;29:355-9.
anions. J Crit Care 1995;10:51-5. 6. Askanazi J, Carpentier YA, Elwyn DH, et
21. Kraut JA, Madias NE. Serum anion gap: its uses al. Influence of total parenteral nutrition on
and limitations in clinical medicine. Clin J Am fuel utilization in injury and sepsis. Ann Surg
Soc Nephrol 2007;2:162-74. 1980;191:40-6.

ACCP Updates in Therapeutics® 2022: Critical Care Pharmacy Preparatory Review and Recertification Course

348
Fluids, Electrolytes, Acid-Base Disorders, and Nutrition Support

7. Askanazi J, Nordenstrom J, Rosenbaum SH, et al. 19. Dickerson RN. Manganese intoxication and paren-
Nutrition for the patient with respiratory failure: teral nutrition. Nutrition 2001;17:689-93.
glucose vs. fat. Anesthesiology 1981;54:373-7. 20. Dickerson RN. Using nitrogen balance in clinical
8. Askanazi J, Rosenbaum SH, Hyman AI, et al. practice. Hospital Pharmacy 2005;40:1081-7.
Respiratory changes induced by the large glu- 21. Dickerson RN, Garmon WM, Kuhl DA, et al.
cose loads of total parenteral nutrition. JAMA Vitamin K-independent warfarin resistance
1980;243:1444-7. after concurrent administration of warfarin and
9. Barazzoni R, Bischoff SC, Breda J, et al. ESPEN continuous enteral nutrition. Pharmacotherapy
expert statements and practical guidance for 2008;28:308-13.
nutritional management of individuals with 22. Dickerson RN, Gervasio JM, Riley ML, et al.
SARS-CoV-2 infection. Clin Nutr 2020;39:1631-8. Accuracy of predictive methods to estimate resting
10. Battistella FD, Widergren JT, Anderson JT, et al. energy expenditure of thermally-injured patients.
A prospective, randomized trial of intravenous JPEN J Parenter Enteral Nutr 2002;26:17-29.
fat emulsion administration in trauma victims 23. Dickerson RN, Maish GO III, Minard G, et al.
requiring total parenteral nutrition. J Trauma Nutrition support team-led glycemic control pro-
1997;43:52-60. gram for critically ill patients. Nutr Clin Pract
11. Burke JF, Wolfe RR, Mullany CJ, et al. Glucose 2014;29:534-41.
requirements following burn injury. Parameters of 24. Dickerson RN, Maish GO III, Minard G, et al.
optimal glucose infusion and possible hepatic and Clinical relevancy of the levothyroxine-continu-
respiratory abnormalities following excessive glu- ous enteral nutrition interaction. Nutr Clin Pract
cose intake. Ann Surg 1979;190:274-85. 2010;25:646-52.
12. Casaer M, Mesotten D, Hermans G, et al. Early 25. Dickerson RN, Mitchell JN, Morgan LM, et al.
versus late parenteral nutrition in critically ill Disparate response to metoclopramide therapy
adults. N Engl J Med 2011;365:1-17. for gastric feeding intolerance in trauma patients
13. Cerra FB, Benitez MR, Blackburn GL, et al. with and without traumatic brain injury. JPEN J
Applied nutrition in ICU patients. A consen- Parenter Enteral Nutr 2009;33:646-55.
sus statement of the American College of Chest 26. Dickerson RN, Sacks GS. Medication adminis-
Physicians. Chest 1997;111:769-78. tration considerations with specialized nutrition
14. Choban P, Dickerson R, Malone A, et al. A.S.P.E.N. support. In: DiPiro JT, Talbert RL, Yee GC, et
clinical guidelines: nutrition support of hospital- al., eds. Pharmacotherapy: A Pathophysiologic
ized adult patients with obesity. JPEN J Parenter Approach, 8th ed. New York: McGraw-Hill,
Enteral Nutr 2013;37:714-44. 2011:2493-503.
15. Cohn SM, Sawyer MD, Burns GA, et al. Enteric 27. Dickerson RN, Tidwell AC, Minard G, et al.
absorption of ciprofloxacin during tube feed- Predicting total urinary nitrogen excretion from
ing in the critically ill. J Antimicrob Chemother urinary urea nitrogen excretion in multiple-trauma
1996;38:871-6. patients receiving specialized nutritional support.
16. Compher C, Chittams J, Sammarco T, et al. Greater Nutrition 2005;21:332-8.
protein and energy intake may be associated with 28. Dickerson RN, Wilson VC, Maish GO III, et al.
improved mortality in higher risk critically ill Transitional NPH insulin therapy for critically
patients: a multicenter, multinational observa- ill patients receiving continuous enteral nutrition
tional study. Crit Care Med 2017;45:156-163. and intravenous regular human insulin. JPEN J
17. Detsky AS, McLaughlin JR, Baker JP, et al. What Parenter Enteral Nutr 2013;37:506-16.
is subjective global assessment of nutritional sta- 29. Drover JW, Dhaliwal R, Weitzel L, et al.
tus? JPEN J Parenter Enteral Nutr 1987;11:8-13. Perioperative use of arginine-supplemented diets:
18. Dhaliwal R, Cahill N, Lemieux M, et al. The a systematic review of the evidence. J Am Coll
Canadian critical care nutrition guidelines in Surg 2011;212:385-99.
2013: an update on current recommendations 30. Elke G, van Zanten AR, Lemieux M, et al. Enteral
and implementation strategies. Nutr Clin Pract versus parenteral nutrition in critically ill patients:
2014;29:29-43.

ACCP Updates in Therapeutics® 2022: Critical Care Pharmacy Preparatory Review and Recertification Course

349
Fluids, Electrolytes, Acid-Base Disorders, and Nutrition Support

an updated systematic review and meta-anal- 42. Heyland DK, Dhaliwal R, Jiang X, et al. Identifying
ysis of randomized controlled trials. Crit Care critically ill patients who benefit the most from
2016;20:117. nutrition therapy: the development and initial vali-
31. Gadek JE, DeMichele SJ, Karlstad MD, et al. dation of a novel risk assessment tool. Crit Care
Effect of enteral feeding with eicosapentaenoic 2011;15:R268.
acid, gamma-linolenic acid, and antioxidants in 43. Hurt RT, McClave SA, Martindale RG, et al.
patients with acute respiratory distress syndrome. Summary points and consensus recommendations
Enteral Nutrition in ARDS Study Group. Crit Care from the international protein summit. Nutr Clin
Med 1999;27:1409-20. Pract 2017;32(suppl 1):142S-51S.
32. Galban C, Montejo JC, Mesejo A, et al. An 44. Jacobi J, Bircher N, Krinsley J, et al. Guidelines
immune-enhancing enteral diet reduces mortality for the use of an insulin infusion for the manage-
rate and episodes of bacteremia in septic intensive ment of hyperglycemia in critically ill patients.
care unit patients. Crit Care Med 2000;28:643-8. Crit Care Med 2012;40:3251-76.
33. Garrel D, Patenaude J, Nedelec B, et al. Decreased 45. Jacobs DG, Jacobs DO, Kudsk KA, et al. Practice
mortality and infectious morbidity in adult burn management guidelines for nutritional support of
patients given enteral glutamine supplements: a the trauma patient. J Trauma 2004;57:660-9.
prospective, controlled, randomized clinical trial. 46. Jensen GL, Mascioli EA, Seidner DL, et al.
Crit Care Med 2003;31:2444-9. Parenteral infusion of long- and medium-chain
34. Gerlach AT, Thomas S, Murphy CV, et al. Does triglycerides and reticuloendothelial system
delaying early intravenous fat emulsion during function in man. JPEN J Parenter Enteral Nutr
parenteral nutrition reduce infections during criti- 1990;14:467-71.
cal illness? Surg Infect (Larchmt) 2011;12:43-7. 47. Jensen GL, Mirtallo J, Compher C, et al. Adult
35. Gilbert S, Hatton J, Magnuson B. How to mini- starvation and disease-related malnutrition: a pro-
mize interaction between phenytoin and enteral posal for etiology-based diagnosis in the clinical
feedings: two approaches. Nutr Clin Pract practice setting from the International Consensus
1996;11:28-31. Guideline Committee. JPEN J Parenter Enteral
36. Goodgame JT, Lowry SF, Brennan MF. Essential Nutr 2010;34:156-9.
fatty acid deficiency in total parenteral nutrition: 48. Kamel AY, Dave NJ, Zhao VM, et al. Micronutrient
time course of development and suggestions for alterations during continuous renal replacement
therapy. Surgery 1978;84:271-7. therapy in critically ill adults: a retrospective
37. Grunfeld C, Kotler DP, Hamadeh R, et al. study. Nutr Clin Pract 2018;33:439-46.
Hypertriglyceridemia in the acquired immunode- 49. Kanji S, Singh A, Tierney M, et al. Standardization
ficiency syndrome. Am J Med 1989;86:27-31. of intravenous insulin therapy improves the
38. Harvey SE, Parrott F, Harrison DA, et al. Trial of efficiency and safety of blood glucose con-
the route of early nutritional support in critically ill trol in critically ill adults. Intensive Care Med
adults. N Engl J Med 2014;371:1673-84. 2004;30:804-10.
39. Heidegger C, Berger M, Graf S, et al. Optimisation 50. Khalid I, Doshi P, DiGiovine B. Early enteral nutri-
of energy provision with supplemental parenteral tion and outcomes of critically ill patients treated
nutrition in critically ill patients: a randomised with vasopressors and mechanical ventilation. Am
controlled clinical trial. Lancet 2013;381:385-93. J Crit Care 2010;19:261-8.
40. Heyland D, Muscedere J, Wischmeyer PE, et 51. Kondrup J, Rasmussen HH, Hamburg O, et al.
al. A randomized trial of glutamine and anti- Nutritional risk screening (NRS 2002): a new
oxidants in critically ill patients. N Engl J Med method based on an analysis of controlled clinical
2013;368:1489-97. trials. Clin Nutr 2003;22:321-36.
41. Heyland DK, Dhaliwal R. Role of glutamine sup- 52. Kreymann KG, Berger MM, Deutz NE, et al.
plementation in critical illness given the results of ESPEN guidelines on enteral nutrition: intensive
the REDOXS study. JPEN J Parenter Enteral Nutr care. Clin Nutr 2006;25:210-23.
2013;37:442-3.

ACCP Updates in Therapeutics® 2022: Critical Care Pharmacy Preparatory Review and Recertification Course

350
Fluids, Electrolytes, Acid-Base Disorders, and Nutrition Support

53. Krinsley JS, Grover A. Severe hypoglycemia in of adult patients with hyperglycemia. JPEN J
critically ill patients: risk factors and outcomes. Parenter Enteral Nutr 2013;37:23-36.
Crit Care Med 2007;35:2262-7. 63. Metheny NA, Clouse RE, Chang YH, et al.
54. Kudsk KA, Croce MA, Fabian TC, et al. Enteral Tracheobronchial aspiration of gastric contents in
versus parenteral feeding. Effects on septic mor- critically ill tube-fed patients: frequency, outcomes,
bidity after blunt and penetrating abdominal and risk factors. Crit Care Med 2006;34:1007-15.
trauma. Ann Surg 1992;215:503-13. 64. Meynaar IA, van Spreuwel M, Tangkau PL, et
55. Kuppinger DD, Rittler P, Hartl WH, et al. Use of al. Accuracy of AccuChek glucose measure-
gastric residual volume to guide enteral nutrition ment in intensive care patients. Crit Care Med
in critically ill patients: a brief systematic review 2009;37:2691-6.
of clinical studies. Nutrition 2013;29:1075-9. 65. Mirtallo JM, Ayers P, Boullata J, et al. ASPEN
56. Lorente JA, Landin L, De Pablo R, et al. L-arginine lipid injectable emulsion safety recommendations,
pathway in the sepsis syndrome. Crit Care Med Part 1: Background and adult considerations. Nutr
1993;21:1287-95. Clin Pract 2020;35:769-82.
57. Mancl EE, Muzevich KM. Tolerability and safety 66. Mirtallo JM, Forbes A, McClave SA, et al.
of enteral nutrition in critically ill patients receiv- International consensus guidelines for nutrition
ing intravenous vasopressor therapy. JPEN J therapy in pancreatitis. JPEN J Parenter Enteral
Parenter Enteral Nutr 2013;37:641-51. Nutr 2012;36:284-91.
58. Martindale R, Patel JJ, Taylor B, et al. Nutrition 67. Montejo JC, Minambres E, Bordeje L, et al.
therapy in the patient with COVID-19 disease Gastric residual volume during enteral nutrition in
requiring ICU care. 2020. Available at: http:// ICU patients: the REGANE study. Intensive Care
https://www.nutritioncare.org/uploadedFiles/ Med 2010;36:1386-93.
Documents/Guidelines_and_Clinical_Resources/ 68. Newton L, Garvey WT. Nutritional and
COV ID19/ Nut r ition%20T herapy%20in%20 medical management of diabetes mellitus in hos-
t he%20Pat ient %20 wit h%20COV I D -19%20 pitalized patients. In: Mueller CM, Kovacevich
D i s e a s e% 2 0 Re q u i r i n g % 2 0 IC U % 2 0 C a r e _ DS, McClave SA, et al., eds. The ASPEN Adult
Updated%20May%2026.pdf. Accessed August Nutrition Support Core Curriculum, 2nd ed. Silver
30, 2020. Spring, MD: American Society for Parenteral and
59. McClave SA, DeMeo MT, DeLegge MH, et al. Enteral Nutrition, 2012:580-602.
North American Summit on Aspiration in the 69. Nguyen NQ, Chapman M, Fraser RJ, et al.
Critically Ill Patient: consensus statement. JPEN J Prokinetic therapy for feed intolerance in criti-
Parenter Enteral Nutr 2002;26(6 suppl):S80-5. cal illness: one drug or two? Crit Care Med
60. McClave SA, Martindale RG, Vanek VW, et al. 2007;35:2561-7.
Guidelines for the provision and assessment of 70. Nguyen NQ, Chapman MJ, Fraser RJ, et al.
nutrition support therapy in the adult critically ill Erythromycin is more effective than metoclo-
patient: Society of Critical Care Medicine (SCCM) pramide in the treatment of feed intolerance in
and American Society for Parenteral and Enteral critical illness. Crit Care Med 2007;35:483-9.
Nutrition (A.S.P.E.N.). JPEN J Parenter Enteral 71. Nicolo M, Heyland DK, Chittams J, et al. Clinical
Nutr 2009;33:277-316. outcomes related to protein delivery in a criti-
61. McClave SA, Taylor BE, Martindale RG, et al. cally ill population: a multicenter, multinational
Guidelines for the provision and assessment of observation study. JPEN J Parenter Enteral Nutr
nutrition support therapy in the adult critically ill 2016;40:45-51.
patient: Society of Critical Care Medicine (SCCM) 72. Oslon LM, Wieruszewski PM, Jannetto PJ, et al.
and American Society for Parenteral and Enteral Quantitative assessment of trace-element contami-
Nutrition (A.S.P.E.N.) JPEN J Parenter Enteral nation in parenteral nutrition components. JPEN J
Nutr 2016;40:159-211. Parenter Enteral Nutr 2019;43;970-76.
62. McMahon MM, Nystrom E, Braunschweig C, et 73. Osterkamp LK. Current perspective on assessment
al. A.S.P.E.N. clinical guidelines: nutrition support of human body proportions of relevance to ampu-
tees. J Am Diet Assoc 1995;95:215-8.

ACCP Updates in Therapeutics® 2022: Critical Care Pharmacy Preparatory Review and Recertification Course

351
Fluids, Electrolytes, Acid-Base Disorders, and Nutrition Support

74. Parnes HL, Mascioli EA, LaCivita LC, et al. 85. Rosmarin DK, Wardlaw GM, Mirtallo J.
Parenteral nutrition in overweight patients: are Hyperglycemia associated with high, continuous
intravenous lipids necessary to prevent essential infusion rates of total parenteral nutrition dex-
fatty acid deficiency? J Nutr Biochem 1994;5:243-7. trose. Nutr Clin Pract 1996;11:151-6.
75. Patel JJ, Martindale RG, McClave SA. Relevant 86. Russell MK, Wischmeyer PE. Supplemental par-
nutrition therapy in COVID-19 and the contraints enteral nutrition: review of the literature and
on its delivery by a unique disease process. Nutr current nutrition guidelines. Nutr Clin Pract
Clin Pract. 2020;35:792-99. 2018;33:359-69.
76. Pirola I, Daffini L, Gandossi E, et al. Comparison 87. Seidner DL, Mascioli EA, Istfan NW, et al. Effects
between liquid and tablet levothyroxine formula- of long-chain triglyceride emulsions on reticulo-
tions in patients treated through enteral feeding endothelial system function in humans. JPEN J
tube. J Endocrinol Invest 2014;37:583-7. Parenter Enteral Nutr 1989;13:614-9.
77. Pontes-Arruda A, Aragao AM, Albuquerque JD. 88. Sheldon GF, Peterson SR, Sanders R. Hepatic
Effects of enteral feeding with eicosapentaenoic dysfunction during hyperalimentation. Arch Surg
acid, gamma-linolenic acid, and antioxidants in 1978;113:504-8.
mechanically ventilated patients with severe sepsis 89. Shike M, Roulet M, Kurian R, et al. Copper
and septic shock. Crit Care Med 2006;34:2325-33. metabolism and requirements in total parenteral
78. Pontes-Arruda A, Demichele S, Seth A, et al. The nutrition. Gastroenterology 1981;81:290-7.
use of an inflammation-modulating diet in patients 90. Singer P, Blaser AR, Berger MM, et al. ESPEN
with acute lung injury or acute respiratory distress guideline on clinical nutrition in the intensive care
syndrome: a meta-analysis of outcome data. JPEN unit. Clin Nutr 2019;38:48-79.
J Parenter Enteral Nutr 2008;32:596-605. 91. Singer P, Theilla M, Fisher H, et al. Benefit of
79. Puder M, Valim C, Meisel JA, et al. Parenteral an enteral diet enriched with eicosapentaenoic
fish oil improves outcomes in patients with par- acid and gamma-linolenic acid in ventilated
enteral nutrition-associated liver injury. Ann Surg patients with acute lung injury. Crit Care Med
2009;250:395-402. 2006;34:1033-8.
80. Reignier J, Mercier E, Le Gouge A, et al. Effect 92. Takagi Y, Okada A, Sando K, et al. Evaluation
of not monitoring residual gastric volume on of indexes of in vivo manganese status and the
risk of ventilator-associated pneumonia in adults optimal intravenous dose for adult patients under-
receiving mechanical ventilation and early enteral going home parenteral nutrition. Am J Clin Nutr
feeding: a randomized controlled trial. JAMA 2002;75:112-8.
2013;309:249-56. 93. Talpers SS, Romberger DJ, Bunce SB, et al.
81. Rhodes A, Evans LE, Alhazzani W, et al. Nutritionally associated increased carbon dioxide
Surviving Sepsis Campaign: international guide- production. Excess total calories vs high proportion
lines for management of sepsis and septic shock: of carbohydrate calories. Chest 1992;102:551-5.
2016. Crit Care Med 2017;45:486-552. 94. Tenner S, Baillie J, DeWitt J, et al. American
82. Rice TW, Mogan S, Hays MA, et al. Randomized College of Gastroenterology guideline: manage-
trial of initial trophic versus full-energy enteral ment of acute pancreatitis. Am J Gastroenterol
nutrition in mechanically ventilated patients 2013;108:1400-16.
with acute respiratory failure. Crit Care Med 95. Tillman EM. Review and clinical update on paren-
2011;39:967-74. teral nutrition-associated liver disease. Nutr Clin
83. Rice TW, Wheeler AP, Thompson BT, et al. Enteral Pract 2013;28:30-9.
omega-3 fatty acid, gamma-linolenic acid, and 96. Vandewoude K, Vogelaers D, Decruyenaere J, et
antioxidant supplementation in acute lung injury. al. Concentrations in plasma and safety of 7 days
JAMA 2011;306:1574-81. of intravenous itraconazole followed by 2 weeks
84. Rice TW, Wheeler AP, Thompson BT, et al. Initial of oral itraconazole solution in patients in inten-
trophic vs full enteral feeding in patients with sive care units. Antimicrob Agents Chemother
acute lung injury: the EDEN randomized trial. 1997;41:2714-8.
JAMA 2012;307:795-803.

ACCP Updates in Therapeutics® 2022: Critical Care Pharmacy Preparatory Review and Recertification Course

352
Fluids, Electrolytes, Acid-Base Disorders, and Nutrition Support

97. Vanek VW, Matarese LE, Robinson M, et al.


A.S.P.E.N. position paper: parenteral nutrition
glutamine supplementation. Nutr Clin Pract
2011;26:479-94.
98. Vriesendorp TM, van Santen S, DeVries JH, et
al. Predisposing factors for hypoglycemia in the
intensive care unit. Crit Care Med 2006;34:96-101.
99. Weijs PJ, Stapel SN, de Groot SD, et al. Optimal
protein and energy nutrition decreases mortality
in mechanically ventilated, critically ill patients:
a prospective observational cohort study. JPEN J
Parenter Enteral Nutr 2012;36:60-8.
100. Weimann A, Braga M, Carli F, et al. ESPEN
guideline: clinical nutrition in surgery. Clin Nutr
2017;36:623-50.
101. Whittle J, Molinger J, MacLeod D, et al. Persistent
hypermetabolism and longitudinal energy expen-
diture in critically ill patients with COVID-19. Crit
Care 2020;24:581.
102. Wischmeyer PE, Lynch J, Liedel J, et al. Glutamine
administration reduces gram-negative bacteremia
in severely burned patients: a prospective, ran-
domized, double-blind trial versus isonitrogenous
control. Crit Care Med 2001;29:2075-80.
103. Wolman SL, Anderson GH, Marliss EB, et al. Zinc
in total parenteral nutrition: requirements and met-
abolic effects. Gastroenterology 1979;76:458-67.
104. Worthington P, Balint J, Bechtold M, et al. When
is parenteral nutrition approriate? JPEN J Parenter
Enteral Nutr 2017;41:324-77.
105. Zhao X, Li Y, Shi Y, et al. Evaluation of nutri-
tion risk and its association with mortality risk
in severely and critically ill COVID-19 patients.
JPEN J Parenter Enteral Nutr 2021; 45:32-42.

ACCP Updates in Therapeutics® 2022: Critical Care Pharmacy Preparatory Review and Recertification Course

353
Fluids, Electrolytes, Acid-Base Disorders, and Nutrition Support

ANSWERS AND EXPLANATIONS TO PATIENT CASES

1. Answer: D this chapter (unlike the doses given in answers A, C,


Hyperglycemia and other causes of non-hypotonic D). Given that the patient is NPO with intolerance to
hyponatremia have been excluded. Urine osmolality anything by mouth, oral options (Answers A and D) are
is greater than 100 mOsm/kg, which rules out psy- not practical. If the patient had a history of significant
chogenic polydipsia, and a large amount of hypotonic recent weight loss or if he did not respond adequately
fluids were not being given. Urine sodium was greater to these doses, the dose could be increased. Answer
than 30 mEq/L, and the patient did not receive diuretic C is not correct as the dosage of phosphorus is exces-
therapy or have kidney disease. The patient appeared sive. Supplemental potassium and phosphorus would be
to be normovolemic without evidence of significant added to the PN solution, in addition to daily intrave-
edema (expansion of the ECF compartment). Because nous doses of potassium and phosphorus.
the patient also has pneumonia (a common cause of
SIADH), all of these factors indicate that the patient has 5. Answer: B
hyponatremia caused by SIADH (Answer D is correct). Because it takes about 48 hours for serum magnesium to
Answer A is incorrect because the serum glucose con- redistribute, the next day’s serum magnesium concentra-
centration is not high enough to cause hyponatremia; tion is falsely elevated. In general, it will take 4–5 days
nor has the patient received mannitol, glycine: nor is to replete this patient’s magnesium deficiency (presum-
the patient hypertriglyceridemic. Answer B is unlikely ably caused by chronic alcohol ingestion). Thus, answer
because there was no evidence of adrenal insufficiency B would be the best option for this patient. Given that
given in the case. Answer C is impossible because the the patient is NPO, oral dosing options (answer A) are
patient did not have a traumatic brain injury. less desirable. Answer C is incorrect because it is too
aggressive a dosage given the current serum magne-
2. Answer: B sium concentration of 2.0 mg/dL (despite it being falsely
Fluid restriction is the most appropriate treatment of elevated). Supplemental magnesium would be added to
SIADH (Answer B is correct). The “vaptans” may also the PN solution in addition to daily doses of intravenous
be considered; however, this was not a choice. Answer A magnesium sulfate if he remained low or in the low-
is incorrect because giving salt tabs may result in wors- normal range or if he was also hypocalcemic (because
ened fluid overload and edema. Answer C is incorrect hypomagnesemia can elicit hypocalcemia secondary to
for the same reason as Answer A. Answer D is incorrect end-organ resistance to parathyroid hormone). Answer
because the severity of hyponatremia and lack of symp- D is incorrect as the serum magnesium concentration is
toms does not warrant the emergency use of hypertonic falsely elevated to within the normal range but has not
saline. fully redistributed yet. It would be anticipated that the
concentration will decrease the following day since it
3. Answer: D takes multiple days to replenish magnesium stores.
The best way to fluid-restrict an enterally fed patient is
to use the most concentrated formulas, which are the 6. Answer: C
2-kcal/mL formulations that are specifically designed Although critical illness (Answer D) and fluid resuscita-
for patients with congestive heart failure. Unfortunately, tion therapy may have been a factor in the development
protein intake may be inadequate with the use of these of his hypocalcemia, massive blood transfusion is the
formulations in certain populations, and supplemental most profound cause. Citrate, added to the blood as
protein may have to be provided. Answers A, B, and C an anticoagulant, readily binds calcium and can cause
are incorrect because they do not result in the appropri- hypocalcemia. Previous studies have shown that hypo-
ate therapeutic decision to reduce fluid intake. calcemia is common when patients are given more than
5 units of blood at a time (Answer C is correct). Answer
4. Answer: B A is incorrect because a low-normal serum magnesium
These dosages should be selected as the correct answer concentration of 1.8 mg/dL is unlikely to contribute to
because they follow the dosing guidelines given in the pathogenesis of hypocalcemia. A serum magnesium

ACCP Updates in Therapeutics® 2022: Critical Care Pharmacy Preparatory Review and Recertification Course

354
Fluids, Electrolytes, Acid-Base Disorders, and Nutrition Support

concentration of less than or equal to 1.5 mg/dL is more 9. Answer: C


likely to contribute to the pathogenesis of hypocalce- Initial therapy with lactated Ringer solution would
mia. Answer B is incorrect because a urine output of 0.5 be the ideal choice (Answer C is correct). Treatment
mL/kg/hr is not excessive. with 0.9% sodium chloride (Answer B) is incorrect
because it would worsen the hyperchloremic acidosis.
7. Answer: B A 5% dextrose solution (Answer D) would be a poor
A short-term intravenous infusion of 4 g of calcium glu- choice because sodium/isotonic fluids are necessary
conate (1 g = 4.6 mEq) for 4 hours has been shown to to restore intravascular volume. Because the severity
be a safe and effective therapeutic regimen for moder- of the patient’s acidemia is mild (pH 7.29), aggressive
ate to severe hypocalcemia (ionized calcium less than 1 therapy with sodium bicarbonate is not indicated.
mmol/L) (Answer B is correct). Answer A is incorrect After the first day of lactated Ringer solution to restore
because it is an insufficient dosage of elemental calcium. intravascular volume and improve pH, it would be rea-
A bolus/push dose of calcium chloride (1 g = 13.6 mEq), sonable to change to 0.45% sodium chloride to continue
as given in answer C, would be an effective means for to restore volume depletion, depending on the patient’s
treating symptomatic severe hyperkalemia, but it would response to the lactated Ringer solution (e.g., restora-
be unnecessarily aggressive for treating hypocalcemia tion of normal pH, adequate urine output, resolution
in this patient scenario. Answer D is incorrect because of tachycardia). Thus, answer A would be incorrect as
the moderate to severe hypocalcemia should be cor- the question asked for the most appropriate initial treat-
rected in this post-operative trauma patient who is ment. Of course, aggressive potassium and magnesium
anemic and at high risk for bleeding complications. repletion is indicated as well. This could be done with
infusions as previously discussed, and it would be rea-
8. Answer: A sonable to add 20 mEq of potassium chloride per liter to
Examination of the pH indicates that the patient has an the 0.45% sodium chloride upon discontinuation of the
acidemia because it is lower than 7.35. Looking at Pco2 lactated Ringer solution.
and serum bicarbonate would indicate that the primary
etiology is metabolic because both are low. Calculation 10. Answer: C
of the AG (145 − 118 − 18 =9) shows that no AG is present The current PN regimen provides 61 kcal/kg/day total
(Answers B and C are incorrect).A correction for serum (glucose 6.1 mg/kg/minute and lipid emulsion 1.5 g/
albumin concentration is not needed because it is nor- kg/day) and protein 4 g/kg/day. The PN regimen rep-
mal. The serum lactate is near the high end of the normal resents gross overfeeding of this small woman and can
range but still within the normal range. This would indi- explain her hyperglycemia and hypercapnia. Cutting
cate that the patient’s dehydration has not become so all macronutrients by about one-half would result in a
extreme that a significant decrease in tissue perfusion more reasonable regimen for this patient: 30 kcal/kg/
was not evident (yet). Respiratory compensation appears day (glucose 3 mg/kg/minute and lipid emulsion 0.8 g/
to be intact ([1.5 x 17] + 8 = 33 vs. 34 mm Hg) on the kg/day) and protein 2 g/kg/day. Because she is so small
blood gas, and it appears that the patient’s history of (weight 40 kg), it would be important to double-check
smoking did not compromise her ability to mount a rea- the weight-based calculation to see whether this new
sonable respiratory response to the alkalosis. The serum regimen is appropriate to meet her caloric needs without
chloride of 118 mEq/L indicates hyperchloremia. A overfeeding by calculating the BEE using the Harris-
non-AG hyperchloremic metabolic acidosis is common Benedict equation for women (caloric intake should not
for patients with severe diarrhea. The low serum potas- exceed 1.3–1.5 x BEE for a critically ill patient with
sium and magnesium would also strongly suggest that traumatic injuries) (Answer C is correct). Although
the patient has significant diarrhea (Answer A is correct). answer A reduces the respiratory quotient of the nutri-
Answer D is incorrect because the decreased Pco2 is in ent admixture, this will not solve the primary problem
response to the metabolic acidosis and the patient does of overfeeding the patient because excessive calories
not have a metabolic alkalosis but rather an acidosis. are being provided. Answers B and D may help with
the consequences of overfeeding (e.g., hyperglycemia
and respiratory acidosis) but do not address the primary

ACCP Updates in Therapeutics® 2022: Critical Care Pharmacy Preparatory Review and Recertification Course

355
Fluids, Electrolytes, Acid-Base Disorders, and Nutrition Support

problem (overfeeding) and are inappropriate manage- healing) to prevent the use of amino acids for gluconeo-
ment techniques. genesis. Thus, decreasing the glucose intake to 100 g/
day as described in answer B is not desirable, given the
11. Answer: A mild increases in BG concentration. The easiest method
Total kilocalories per day = (300 g x 3.4 kcal/g of dex- to achieve glycemic control and meet caloric needs is to
trose) + (70 g x 4 kcal/g of protein) + (40 x 10 kcal/g of modestly increase the regular human insulin content in
lipid emulsion) = 1020 glucose kcal + 280 protein kcal + the PN solution. Because 14 units of sliding scale insulin
400 lipid kcal = 1700 total kcal/65 kg = 26 kcal/kg/day. still appears insufficient, a modest increase in insulin
Only 20% and 30% lipid emulsions are available for appears prudent. The patient is unlikely to experience
PN compounding. Each solution provides 10 kcal/g of hypoglycemia with the provision of insulin at 30 units/
intravenous lipid (unlike 9 kcal/g with oral fat) because day when given 200 g of intravenous dextrose concur-
glycerol and phospholipids are added to the emulsion. rently (Answer A is correct). As the stress resolves and
Protein intake is 70 g/65 kg = 1.1 g/kg/day (Answer A is glycemic control improves, insulin can be decreased or
correct). Answers B, C, and D do not represent the cor- eliminated from the PN solution. Answer C is incorrect
rect calculations as described previously. because it would likely provide too much insulin, based
on sliding scale coverage and current BG range, and
12. Answer: C increase the patient’s risk for hypoglycemia.
This patient is moderately stressed with a normal BMI.
Appropriate energy intake would be 25–30 kcal/kg/day
(1625–1950 kcal/day), and protein intake would be 1.2–2
g/kg/day (78–130 g). Increasing dextrose to 400 g/day
would provide a total energy of 31 kcal/kg/day. Although
this glucose intake of 4.3 mg/kg/minute does not exceed
5 mg/kg/minute, the total kcal/day from all macronu-
trients would exceed the recommended initial range for
this patient (Answer A is incorrect). Decreasing the dex-
trose dose to 200 g/day would provide a total energy of
21 kcal/kg/day from all macronutrients, which is below
the target energy range for this patient (Answer B is
incorrect). Increasing the lipid dose to 70 g/day would
provide a total energy of 31 kcal/kg/day from all mac-
ronutrients, which is above the target of 25–30 kcal/kg/
day (Answer D is incorrect). A protein dose of 100 g/
day would provide 1.5 g/kg/day of protein. This dose is
appropriate and falls within the recommended 1.2–2 g/
kg/day. Increasing the protein dose to 100 g/day while
maintaining dextrose at 300 g and lipid at 40 g would
provide the patient with 1820 kcal or 28 kcal/kg/day.
This is an appropriate protein and energy intake for this
patient (Answer C is correct).

13. Answer: A
Because the target BG should be within 140–180 mg/dL
for this surgical patient being transferred to the floor, a
modest improvement in glycemic control is indicated.
Thus, answer D (no change) would be incorrect. Ideally,
obligatory glucose requirements should be met (e.g.,
about 130 g/day plus about 80–150 g/day for wound

ACCP Updates in Therapeutics® 2022: Critical Care Pharmacy Preparatory Review and Recertification Course

356
Fluids, Electrolytes, Acid-Base Disorders, and Nutrition Support

ANSWERS AND EXPLANATIONS TO SELF-ASSESSMENT QUESTIONS

1. Answer: C exception of copper (especially with extreme intestinal


An ileus, usually detected on radiologic examination of bypass procedures for obesity).
the lower abdomen, indicates lack of motility and pres-
ence of distention and air within the small bowel. This 4. Answer: C
is usually depicted as “dilated loops of bowel.” Patients Given the severity of the patient’s condition (recent sei-
cannot be fed safely or efficaciously by the enteral zure from severe hyponatremia) and likely diagnosis
route during an ileus (Answer C is correct). Answer A of SIADH, the immediate goal should be to achieve a
is incorrect because a feeding tube can be placed for serum sodium concentration of greater than 120 mEq/L
enteral feeding of the patient with anorexia, and PN is by short-term infusion of 3% sodium chloride (Answer
not indicated. Answer B is incorrect because presence C is correct). Conivaptan (Answer D) could then be
or absence of bowel sounds is not an accurate marker for given to correct the hyponatremia, limiting the increase
assessing bowel function. Answer D is incorrect because in serum sodium concentration to less than 10–12
a high gastric residual volume during enteral feeding, mEq/L/day. Fluid restriction is imperative and is the
combined with abdominal distension, bloating, emesis, primary overall management technique for this patient.
or regurgitation, can often be efficaciously treated with Answer A is incorrect because a more rapid response is
prokinetic pharmacotherapy or advancement of the imperative because of the patient’s seizure and severity
feeding tube into the small bowel with resumption of of the condition. Answer B is incorrect because it would
enteral feeding. In addition, the guidelines recommend be a potentially life-threatening error by providing more
against discontinuing EN for a GRV less than 500 mL in ADH-like substance and because it is used to treat dia-
the absence of other signs of intolerance. betes insipidus (the opposite condition of SIADH).

2. Answer: B 5. Answer: B
Answer B, 0.45% sodium chloride and potassium chlo- Studies show that increases in mesenteric potassium
ride 20 mEq/L, is correct given the average electrolyte concentrations detected by potassium sensors in the
composition of gastric fluid (see Table 4 regarding the splanchnic vascular bed evoke increased renal potas-
electrolyte composition of GI fluids). Answers A, C, sium excretion (feed-forward regulation of potassium
and D are incorrect because they do not as accurately homeostasis), even before regulation by aldosterone
replace the electrolyte content that is lost from gastric (classic feedback regulation) (Answer B is correct).
fluid output. The bioavailability of enteral potassium is 95%–100%
in the absence of aberrations in GI motility, function,
3. Answer: A or anatomy. A primary difference between enteral and
With significant diarrhea, intravenous zinc requirements parenteral potassium is that the rate of absorption is
from GI fluid losses during critical illness increase from slower with enteral potassium (Answer A is incorrect).
the normal requirements of 3–5 mg/day. Data analyses Intravenous potassium administration can inadvertently
show that most patients with increased intestinal losses be infused too quickly (it is acceptable to infuse potas-
can achieve a positive zinc balance on 13 mg of intra- sium at 10 mEq/hour for patients without a cardiac
venous zinc daily (Gastroenterology 1979;76:458-67). monitor and up to 20 mEq/hour for those with a moni-
As a result, most clinicians provide additional zinc sup- tor). Answer C is incorrect because potassium chloride
plementation for patients with short bowel syndrome, elixir or solution is an effective means for providing
intestinal fistulas, or prolonged and sustained diarrhea potassium when given intra-gastrically. It generally only
(Answer A is correct). Answer B is incorrect because causes diarrhea when given in higher doses or when
copper is an extremely rare and unlikely deficiency to administered directly into the small bowel through a
occur during parenteral nutrition therapy. Answers C feeding jejunostomy because it is a hypertonic solution.
and D are incorrect because intractable diarrhea losses Answer D is a nonsensible answer.
are less likely to cause a deficiency, albeit with the

ACCP Updates in Therapeutics® 2022: Critical Care Pharmacy Preparatory Review and Recertification Course

357
Fluids, Electrolytes, Acid-Base Disorders, and Nutrition Support

6. Answer: D anion gap of 19 (141 − 102 − 20 = 19), which indicates


Answer D, additional magnesium therapy should be that a metabolic acidosis is present, regardless of his pH
given daily for the next 4–5 days, is correct because it or HCO3 values (Answer A is incorrect). His delta ratio
takes 48 hours for magnesium to equilibrate after a short- is 1.25 ([19−14] / [24−20]), which indicates that an anion
term infusion, making Answer A incorrect. Although gap metabolic acidosis is present with no other hidden
Answer B is common in clinical practice, failure to pro- metabolic process. Answer B is correct.
vide additional magnesium repletion in the setting of a
normal serum magnesium concentration the day after
a large intravenous dosage usually causes serum con-
centrations to decrease below the normal range again
over the next 1–2 days as magnesium equilibrates. Thus,
Answer B is not ideal management and is incorrect.
Answer C is incorrect because hypocalcemia should
autocorrect with magnesium supplementation within 48
hours of magnesium therapy; however, calcium therapy
can be given concurrently, if necessary (symptomatic or
ionized calcium concentration less than 1 mmol/L).

7. Answer: D
The current PN formula prescribed provides 3240 kcal/
day (42 kcal/kg/day with a glucose infusion rate of 5.8
mg/kg/minute), which is an excessive amount of energy
for this patient, and he has possible signs of overfeed-
ing in the form of hyperglycemia. Answer D is correct
because decreasing the dextrose dose to 315 g would
appropriately bring the patient’s energy down to 2101
kcal/day (27 kcal/kg/day with a glucose infusion rate
of 2.8 mg/kg/minute). An initial energy goal of 25–30
kcal/kg/day is appropriate for this patient, and by sig-
nificantly decreasing the dextrose and total energy load
provided in the PN, his hyperglycemia should improve.
Answer A is incorrect because the additional insulin
will not address the issue of overfeeding, and sliding-
scale insulin is unlikely to provide adequate glycemic
control for this patient. Answer B is incorrect because
temporarily adjusting the dextrose dose will not correct
the issue of overfeeding in the long term. Answer C is
incorrect because the total energy provided with these
changes would still provide 37 kcal/kg/day and exceed
the appropriate initial goal of 25–30 kcal/kg/day.

8. Answer: B
This patient’s pH is high at 7.46 mmHg so his primary
disorder is an alkalosis (Answer D is incorrect). His
serum HCO3 is 20 mEq/L (low) and Pco2 is 31 mmHg
(low). His high pH and low Pco2 indicate that his pri-
mary disorder is a respiratory alkalosis (Answer C is
incorrect). However, his laboratory values reveal an

ACCP Updates in Therapeutics® 2022: Critical Care Pharmacy Preparatory Review and Recertification Course

358
Toxicology
Kyle Weant, PharmD, FCCP, BCPS, BCCCP
University of South Carolina College of Pharmacy
Columbia, South Carolina

Megan Musselman, Pharm.D., M.S., BCPS, BCCCP


North Kansas City Hospital
North Kansas City, Missouri
Toxicology

Toxicology
Kyle Weant, PharmD, FCCP, BCPS, BCCCP
University of South Carolina College of Pharmacy
Columbia, South Carolina

Megan Musselman, Pharm.D., M.S., BCPS, BCCCP


North Kansas City Hospital
North Kansas City, Missouri

ACCP Updates in Therapeutics® 2022: Critical Care Pharmacy Preparatory Review and Recertification Course

361
Toxicology

Learning Objectives Self-Assessment Questions


Answers and explanations to these questions can be
1. Distinguish between the common clinical toxi- found at the end of this chapter.
dromes associated with acute poisonings.
2. Describe the general management of a patient with 1. A 38-year-old woman with type 2 diabetes is admit-
an acute overdose. ted for confusion and altered mental status. She
3. Determine the best options for the management of has an active prescription for glipizide 10 mg by
selected toxins. mouth twice daily, but she cannot respond to fur-
4. Assess a patient with clinical acute overdose, and ther questioning. Her vital signs are as follows:
develop a patient care plan according to current evi- blood pressure (BP) 115/65 mm Hg, heart rate (HR)
dence. 68 beats/minute, respiratory rate (RR) 15 breaths/
5. Identify the adverse effects and monitoring of the minute, and temperature 98.6°F (37°C). Her point-
patient who is poisoned. of-care glucose concentration is 45 mg/dL, and she
is given 50 mL of 50% dextrose in water intrave-
nously twice. Her follow-up point-of-care glucose
Abbreviations in This Chapter concentration is 50 mg/dL after the first dose and
57 mg/dL after the second dose, with no improve-
ABC Airway, breathing, and circulation ment in symptoms. Which is the most appropriate
ALT Alanine aminotransferase intervention at this time?
AST Aspartate aminotransferase A. Dextrose.
BP Blood pressure B. Glucagon.
BUN Blood urea nitrogen C. Octreotide.
cAMP Cyclic adenosine monophosphate D. Sodium bicarbonate.
CB1 Cannabinoid-1
CB2 Cannabinoid-2 2. A 56-year-old man is admitted to the intensive
CIWA Clinical Institute Withdrawal Assessment for care unit (ICU) after a β-blocker overdose. After
Alcohol Scale administering 2 L of 0.9% sodium chloride and 3 g
CK Creatine kinase of calcium gluconate, his vital signs are as follows:
CNS Central nervous system BP 70/40 mm Hg, HR 52 beats/minute, and RR 22
COPD Chronic obstructive pulmonary disease breaths/minute. Which therapy is most appropriate
ECG Electrocardiogram at this time?
ED Emergency department
A. Glucagon 5 mg intravenously.
GABAa γ-aminobutyric acid
B. Atropine 2 mg intravenously.
GI Gastrointestinal
C. Insulin 0.1 unit/kg intravenously.
HIET Hyperinsulinemic euglycemic therapy
D. Dopamine 2 mcg/kg/minute intravenously.
HR Heart rate
ICU Intensive care unit
3. A 76-year-old woman is admitted to the emergency
INR International normalized ratio
department (ED) with the chief concern of decreased
NAPQI N-acetyl-p-benzoquinoneimine
mental status. Her vital signs are as follows: BP
NMDA N-methyl-d-aspartate
118/72 mm Hg, HR 57 beats/minute, and RR 17
RR Respiratory rate
breaths/minute. She is experiencing nausea, but
SBP Systolic blood pressure
her physical examination is otherwise normal. Her
SCr Serum creatinine
husband is concerned that she may not be taking
SSRI Selective serotonin reuptake inhibitor
her medications properly. Given her presentation,
TCA Tricyclic antidepressant
which common toxidrome is the patient most likely
experiencing?

ACCP Updates in Therapeutics® 2022: Critical Care Pharmacy Preparatory Review and Recertification Course

362
Toxicology

A. Anticholinergic. 6. A 57-year-old male patient on the medical floor is


B. Cholinergic. incorrectly administered a dose of methadone 40
C. Opioid. mg by mouth that was written for the patient in the
D. Sympathomimetic. adjoining bed. Two hours later, the nurse finds him
unresponsive with the following vital signs: BP
4. A 38-year-old woman is admitted to the ICU after a 105/67 mm Hg, HR 61 beats/minute, RR 8 breaths/
suspected overdose of risperidone. She was initially minute, and temperature 98.7°F (37.1°C). The nurse
hypotensive, but she was stabilized after the admin- calls for the rapid response team, and, as the team
istration of two 500-mL boluses of lactated Ringer pharmacist, you are asked for a recommendation.
solution. Her BP is now 118/77 mm Hg, HR 75 Which treatment is most appropriate at this time?
beats/minute, and RR 16 breaths/minute. A 12-lead A. Activated charcoal 50 g orally.
electrocardiogram (ECG) shows QT prolongation B. Naloxone 0.04 mg intravenously.
(corrected QT interval [QTc] = 480 milliseconds), C. Whole bowel irrigation orally.
and her chemistry panel is significant for a bicar- D. 1 L of 0.9% sodium chloride intravenously.
bonate of 24 mEq/L, potassium of 3.1 mEq/L, and
magnesium of 1.8 mg/dL. Which intervention is Questions 7 and 8 pertain to the following case.
most appropriate at this time? A 56-year-old female patient is admitted to the ED after
an intentional overdose of 25 amlodipine 10-mg tablets.
A. Potassium chloride 20 mEq every hour for two She is given activated charcoal 50 g, 2 L of 0.9% sodium
doses. chloride, and 3 g of calcium gluconate. Her current vital
B. Activated charcoal 50 g. signs are as follows: BP 90/50 mm Hg, HR 107 beats/
C. Magnesium sulfate 2 g. minute, RR 17 breaths/minute, and temperature 98.7°F
D. Lorazepam 2 mg. (37.1°C). Serum chemistries are as follows: Na 141
mEq/L, K 2.5 mEq/L, Cl 101 mEq/L, HCO3 24 mEq/L,
5. A 48-year-old man is admitted to the medical floor blood urea nitrogen (BUN) 19 mg/dL, serum creatinine
for community-acquired pneumonia. His medical (SCr) 0.9 mg/dL, and glucose 215 mg/dL. The ED phy-
history is significant for hypertension, hyperlipid- sician wants to initiate hyperinsulinemic euglycemic
emia, and chronic obstructive pulmonary disease therapy (HIET).
(COPD), and he reports occasional alcohol use but
no history of alcohol withdrawal. He is initiated 7. Which is most appropriate to initiate first with
on levofloxacin 750 mg intravenously daily and respect to HIET?
nebulizer treatments with albuterol and ipratropium. A. Give insulin 1 unit/kg intravenously.
Twenty-four hours after admission, he is increas- B. Give 50 mL of 50% dextrose in water
ingly more confused and has nausea and vomiting. intravenously.
His vital signs are stable: BP 115/68 mm Hg, HR C. Warn the physician that full effects may take up
122 beats/minute, RR 21 breaths/minute, and tem- to 30 minutes.
perature 99.7°F (37.6°C). The team is concerned D. Give 20 mEq of potassium chloride intrave-
about possible alcohol withdrawal and asks for rec- nously every hour for four doses.
ommendations for initial therapy. Which is the most
appropriate treatment for this patient? 8. The patient is not responding to HIET initiation.
Her current infusion rate is 1 unit/kg/hour. Her BP
A. Lorazepam 2 mg intravenous push every 4
remains low at 70/40 mm Hg, and her HR is now
hours as needed according to the patient’s
58 beats/minute. Her repeat glucose is 186 mg/dL.
Clinical Institute Withdrawal Assessment for
Which is most appropriate to initiate at this time?
Alcohol Scale (CIWA) score.
B. Phenobarbital 65 mg by mouth every 8 hours as A. Continue HIET, and initiate norepinephrine.
needed according to the patient’s CIWA score. B. Continue HIET, and increase the insulin infu-
sion rate.
C. Propofol continuous infusion.
C. Continue HIET, and initiate epinephrine.
D. Clonidine 0.1 mg by mouth every 12 hours.
D. Discontinue HIET, and begin intravenous lipid
therapy.

ACCP Updates in Therapeutics® 2022: Critical Care Pharmacy Preparatory Review and Recertification Course

363
Toxicology

BPS Critical Care Pharmacy Specialist Examination Content Outline

This chapter covers the following sections of the Critical Care Pharmacy Specialist Examination Content Outline:
1. Domain 1: Clinical Knowledge and Application
a. Task 1: 1-3
b. Task 2: 1-7
c. Task 3: 1-7
d. Task 4: 1-8
e. Task 5: 7
f. Task 6: 1-7
g. Task 7: 1-7
2. Domain 2: Practice Management, Policy, and Quality Improvement
a. Task 1: 1-2
b. Task 5: 1, 2, 4
3. Domain 3: Evidence-Based Medicine, Scholarship, Education, and Professional Development
a. Task 1: 1-3
b. Task 3: 1

ACCP Updates in Therapeutics® 2022: Critical Care Pharmacy Preparatory Review and Recertification Course

364
Toxicology

I.  EPIDEMIOLOGY

A. Population based: The American Association of Poison Control Centers releases an annual report based
on all the cases submitted by the 55 regional poison centers to the National Poison Data System to help
clinicians stay abreast of the changing landscape of exposures. (Clin Toxicol 2020;58:1360-541).
1. In 2019, 2,148,141 human exposures were reported. Fatalities were reported in 1411 cases (Table 1).
2. The most common site of exposure was a residence (92.09%), followed by workplace (2.02%) and
school (1.55%).
3. Most of the reported cases (1,236,227) occurred in children, defined in the report as younger than 20
years. To add perspective, 1,054,640 cases were reported in children 12 years and younger.
4. The most common reasons associated with these exposures were unintentional (76.6%), intentional
(18.9%), and adverse reactions (2.6%). Of note, therapeutic errors accounted for 294,317 (13.7%) of
all cases. The scenarios reported for therapeutic errors included inadvertent double dosing (31.8%),
incorrect medication administered or taken (16.1%), incorrect dose (16.0%), doses administered too
close in time (10.8%), and inadvertent exposure to another person’s medication (8.8%).
5. Routes of exposure included ingestion (83.1%), dermal (7.2%), inhalation/nasal (6.8%), and ocular
(4.4%). Of the 1531 exposure-related fatalities, most were by ingestion (73.8%), followed by inhalation/
nasal (7.5%) and parenteral (4.8%).

B. Management based
1. Only 31% of exposures were managed in a health care facility, with 4.5% managed in critical care units.
2. Gastric decontamination was used in about half of all exposures (51.5%); however, antidotes were
given in only 17.3% of the exposures. The most common gastric decontamination strategy was the
use of activated charcoal (1.7% of total exposures), followed by other emetic agents (0.74%), cathartics
(0.13%), whole bowel irrigation (0.07%), and gastric lavage (0.03%).

Table 1. Top 5 Most Common Medication-Related Toxic Exposures in 2019


Most Common Medication-Related Exposures % of Total Cases
All human exposures
Analgesics 11.02
Antidepressants 5.32
Sedatives/hypnotics/antipsychotics 5.21
Cardiovascular drugs 4.57
Antihistamines 4.36
Adult exposures (>19 yr of age)
Analgesics 11.33
Sedatives/hypnotics/antipsychotics 8.65
Antidepressants 7.37
Alcohols 4.72
Anticonvulsants 4.13
Pediatric exposures (< 6 yr of age)
Analgesics 8.97
Dietary/herbal/homeopathic supplements 5.06
Antihistamines 4.58
Topical preparations 4.46
Vitamins 4.08
Information from: Gummin DD, Mowry JB, Beuhler MC, et al. 2019 annual report of the American Association of Poison Control Centers’ National Poison Data System
(NPDS): 37th annual report. Clin Toxicol 2020;58:1360-541.

ACCP Updates in Therapeutics® 2022: Critical Care Pharmacy Preparatory Review and Recertification Course

365
Toxicology

II.  EMERGENCY EVALUATION AND MANAGEMENT

A. The primary treatment strategy for managing a toxic exposure should focus on stabilizing the patient, with
an emphasis on airway, breathing, and circulation (ABC). The most common factor contributing to death
from a poisoning or drug overdose is the loss of the protective reflexes of the airway secondary to flaccid
tongue, aspiration of gastric contents into the lungs, or respiratory compromise including arrest. Patients
should be monitored for vital signs (HR, RR, BP, temperature, and oxygen saturation) and changes in
mental status. After stabilization, the DEFG approach can be considered:
D: Decontamination
E: Enhanced elimination
F: Focused antidote therapy
G: Get help from a poison control center or toxicologist

B. Supportive care should be based on specific patient symptoms and may include the administration of
intravenous fluids, supplemental oxygen, and advanced airway management. Other potential complications
should be assessed, such as presence of rhabdomyolysis, rigidity, or dystonia. Additional tests, such as a
12-lead ECG, chest radiograph, or electroencephalogram may be required. In addition, essential laboratory
tests should be conducted and assessed for the presence of an osmolar gap, anion gap acidosis, hyper/
hypoglycemia, hyper/hyponatremia, hyper/hypokalemia, renal failure, and liver failure.
1. Use of “coma cocktail” preparations is controversial and therefore not routinely recommended because
they should not replace or substitute for a thorough analysis of the patient (JEMS 2002;27:54-60).
Formulations vary, but they typically contain one or more of the following: dextrose, thiamine, and
naloxone. The following text presents an overview of the common additives, a rationale for use, and
potential controversies.
a. Dextrose 50%, 12.5–25 g (25–50 mL) intravenously is administered to treat hypoglycemia; it is
recommended to perform point-of-care blood glucose testing to confirm before administration.
b. Thiamine 100 mg is administered intravenously to prevent Wernicke encephalopathy; often under-
recognized, several doses of high-dose parenteral thiamine (e.g., 500 mg) concurrent with or
immediately following intravenous dextrose before intravenous dextrose are typically required to
effectively treat (J Emerg Med 2012;42:488-94).
c. Naloxone 0.04–2 mg IV is administered in a stepwise titration to reverse respiratory depression
secondary to opiate overdose.

C. Ingestions
1. A thorough physical examination should be performed.
2. A medication history and reconciliation should be done, including all prescription medications, over-
the-counter agents, illicit substance use, and herbal products.
3. The history of the ingestion should be determined, if possible, including the following elements (Ann
Emerg Med 1999;33:735-56):
a. Timing and route of the exposure, the possible agents involved and their strengths and amounts,
and the potential intent of the patient
b. History from the prehospital care providers, family members, or other patient advocates
c. Onset and progression of any symptoms
4. Some providers advocate for the use of toxidromes, which are a collection of symptoms that occur
with particular classes of toxic agents. Toxidromes may help identify the toxic agent and assist in care
by helping providers anticipate additional symptoms. Although they may be very useful in the care of
an acute poisoning, they should be used with caution because some symptoms may overlap with other
classes of toxins or may be absent altogether.

ACCP Updates in Therapeutics® 2022: Critical Care Pharmacy Preparatory Review and Recertification Course

366
Toxicology

5. Common Toxidromes and Presentation (Chest 2011;140:795-806) (Crit Care Clin 2012;28:180-198)
a. Anticholinergic
i. Mechanism of toxicity is through competitive antagonism of the effects of acetylcholine at
peripheral muscarinic receptors and central receptors.
ii. Signs and symptoms include altered mental status, delirium, hallucinations, mumbled
speech, dry mucous membranes, mydriasis, anhidrosis, flushing, hyperthermia, tachycardia,
hypoactive bowel sounds, and urinary retention.
iii. Common drugs that have anticholinergic activity include antihistamines, antipsychotics,
tricyclic antidepressants, and skeletal muscle relaxants.
b. Cholinergic
i. Mechanism of toxicity is inhibition of acetylcholinesterase causing accumulation of
acetylcholine ultimately resulting in overstimulation of muscarinic and nicotinic receptors.
ii. Signs and symptoms include confusion, central nervous system (CNS) depression, miosis,
wet mucous membranes, salivation, lacrimation, diaphoresis, emesis, urination, diarrhea,
muscle weakness/twitching, bronchospasm, bronchorrhea, hypertension, tachycardia, and
bradycardia.
iii. Common drugs that have cholinergic activity include organophosphates, nerve agents,
nicotine, pilocarpine, and physostigmine.
c. Opioid
i. Mechanism of toxicity is stimulation of opioid receptors causing a decrease in autonomic
activity.
ii. Signs and symptoms include sedation, miosis, decreased bowel sounds (ileus), bradypnea,
apnea (respiratory depression), hypotension, and bradycardia.
iii. Common drugs that have opioid activity include heroin, morphine, codeine, synthetic opioids,
loperamide, and dextromethorphan (in large quantities).
d. Sympathomimetic
i. Mechanism of toxicity is through an increase in sympathetic tone through release of
cathecholamines, inhibition of reuptake, by direct receptor stimulation, and alterations in
neurotransmitter metabolism.
ii. Signs and symptoms include agitation, delirium, mydriasis, diaphoresis, myoclonus,
hyperthermia, hypertension, and tachycardia.
iii. Common drugs that have sympathomimetic activity include cocaine, methamphetamine,
pseudoephedrine, and caffeine.
6. Drug screens are used in acute toxic ingestions, the most common of which is the qualitative urine
screen. This method tests for the presence of a substance, but it cannot detect the amount of substance
present. If a toxin is known, a quantitative drug screen may be used to confirm the exact amount present.
Although urine drug screens may vary by institution, they may include amphetamines, barbiturates,
benzodiazepines, cocaine, MDMA (ecstasy), methamphetamines, opiates, THC (marijuana), and tricyclic
antidepressants (TCAs). Urine screens are not considered comprehensive; therefore, the presence of
additional agents should be tested for (e.g., acetaminophen, salicylates). However, comprehensive drug
screens using gas chromatography and liquid chromatography together with mass spectrometry can be
used in patients presenting with severe or unexplained toxicity.
a. A negative screen does not exclude the presence of a toxic substance, especially if the presumed
agent is not present on the screen. Many agents are not identified by their designated screen; this is
especially an issue with standard amphetamines, benzodiazepines, and opiate screens.

ACCP Updates in Therapeutics® 2022: Critical Care Pharmacy Preparatory Review and Recertification Course

367
Toxicology

b. A positive test also does not necessarily confirm the diagnosis because another agent may be present
but at concentrations below a detectable threshold. In addition, a positive test does not indicate that
the patient is intoxicated on the particular substance (e.g., cocaine is positive for 3 days; however,
its effects last only a few hours) or that the agent ingested is the exact agent that is screened (e.g.,
bupropion causes a false-positive amphetamine screen) (Am J Health Syst Pharm 2010;67:1344-50).

Patient Case

1. 1. A 53-year-old man (height 74 inches, weight 97 kg [215 lb]) arrives in the ED confused and disoriented.
He cannot provide any information about his condition or medical history. Vital signs are as follows: BP
85/50 mm Hg, HR 120 beats/minute, RR 28 breaths/minute, and temperature 99.2°F (37.3°C). On physical
examination, an unmarked pill bottle is found in his pocket. Two tablets remain, and a possible drug over-
dose is suspected. Which is most appropriate to do first for this patient?
A. Send a quantitative urine drug screen.
B. Stabilize the patient’s ABC.
C. Order a coma cocktail.
D. Try to identify the tablets in a drug database.

III.  GASTRIC DECONTAMINATION/ENHANCED ELIMINATION

A. Many strategies for gastric decontamination are used to try to remove toxins or prevent further absorption.
No particular strategy is preferred to another; each has certain advantages and disadvantages, and the risks
and benefits must be considered before use. Consensus statements from the American Academy of Clinical
Toxicology and the European Association of Poisons Centres and Clinical Toxicologists recommend against
the routine use of any decontamination strategy but suggest that these strategies play a role in individualized
care after a poison exposure (Clin Toxicol 2013;51:127). Table 2 lists the common dosing strategies for
general decontamination and enhanced elimination.

B. Ipecac
1. Ipecac is no longer manufactured in the United States since 2010 and is no longer recommended because
of concerns for safety and ability to improve outcomes for patients who have been poisoned.
2. Mechanism of action is to induce vomiting through irritation of the gastric mucosa and stimulation of
the chemoreceptor trigger zone in the medulla.
3. Recent guidelines recommend ipecac (if available) to be given only under a specific recommendation
from a poison control center, ED physician, or other qualified medical personnel when all the following
conditions are met (Clin Toxicol 2013;51:134-9; Clin Toxicol 2005;43:1-10):
a. No specific contraindication exists for use.
b. There is a substantial risk of serious toxicity of the toxin to the patient.
c. No alternatives are available or considered effective to reduce the toxin absorption.
d. A delay of more than 1 hour is expected before arrival to a medical facility.
e. Use of ipecac will not adversely affect a more definitive treatment option.

ACCP Updates in Therapeutics® 2022: Critical Care Pharmacy Preparatory Review and Recertification Course

368
Toxicology

C. Gastric Lavage
1. Gastric lavage is performed by inserting a larger-bore orogastric or nasogastric catheter tube (36–40
French for adults and 24–28 French for children) with several holes at the distal end into the stomach.
Aliquots of warmed tap water (200–300 mL) are then instilled until there is clearing of aspirated fluid.
2. The efficacy is highly variable and diminishes over time; therefore, it is optimal to perform within 60
minutes of ingestion.
3. Guidelines emphasize that gastric lavage has not been proven to decrease the severity of illness, improve
recovery times, or improve outcomes (Clin Toxicol 2013;51:140-6).
4. Should be considered only for life-threatening ingestions when it can be safely performed within 30–60
minutes of ingestion.
5. Even in life-threatening overdoses, it may not be beneficial. Gastric lavage should not be performed
routinely, if at all, for treating the patient who is poisoned. In the rare situation when it might be
appropriate, clinicians should consider treatment with activated charcoal or observation and supportive
care in place of gastric lavage (Clin Toxicol 2013;51:140-6).
6. Contraindications for gastric lavage include patients with craniofacial abnormalities, concomitant head
trauma, unprotected airway, increased risk of aspiration, those at risk of GI hemorrhage or perforation
and caustic ingestion such as acids (e.g., boric acid) and alkalis (e.g., dishwasher detergents) because of
the risk of exacerbating any esophageal or gastric injury. Patients with decreased consciousness require
oral or nasal intubation before the procedure.
7. Complications associated with gastric lavage include aspiration, laryngospasm, perforation of
the esophagus or stomach, arrhythmias, fluid imbalance, hyponatremia, and small conjunctival
hemorrhages.

D. Cathartics
1. Used to reduce the transit time of toxins and hence absorption, as well as in combination with activated
charcoal to decrease constipating effects
2. Cathartics have conflicting data regarding decreased transit time in the GI tract and have no data to
support improved patient outcomes (J Toxicol Clin Toxicol 2004;42:243-53).
3. Cathartic use is not recommended; if used, it should be limited to a single dose and not be used as
monotherapy.
4. Contraindications to cathartic use include absence of bowel sounds, recent GI surgery, intestinal
perforation or obstruction, hypotension, electrolyte disturbances, and renal insufficiency (for
magnesium-based cathartics).
5. Complications include nausea, dehydration, hypotension, and magnesium imbalances.

E. Activated Charcoal
1. Activated charcoal is an adsorbent that works by binding the toxin throughout the GI tract to reduce
systemic absorption. Although activated charcoal binds most substances, Table 3 lists the agents for
which activated charcoal is NOT recommended.
a. Acids and alkalis should be avoided because charcoal may cause vomiting, which can be damaging
in these ingestions. The black color and thickness of the activated charcoal may also cause
discoloration of the stomach lining and therefore interfere with endoscopy.
b. Alcohols bind poorly; therefore, large doses are needed, which are difficult to ingest.
c. Cyanide will bind, but not with as much activity as other substances. Because the toxic dose of
cyanide is so small, normal doses of activated charcoal may be ineffective.
d. Hydrocarbons may lead to a significant risk of aspiration.
2. It is optimal to administer activated charcoal within 60 minutes of the toxin ingestion to maximize
efficacy.

ACCP Updates in Therapeutics® 2022: Critical Care Pharmacy Preparatory Review and Recertification Course

369
Toxicology

3. The use of a cathartic (e.g., sorbitol) in combination with activated charcoal is not recommended.
4. If significant nausea occurs, it is recommended to administer an antiemetic. When choosing an
antiemetic, potential drug and symptom interactions should be considered as well.
5. Complications include aspiration, accidental administration into the lung, emesis, constipation, and
gastric obstruction.
6. Contraindications include an unconscious state or an inability to otherwise protect the airway without
endotracheal intubation, ileus or intestinal obstruction, late presentation (more than 2 hours), and recent
GI surgery.
7. Multidose activated charcoal is a method described to enhance the elimination of certain toxins. It is not
more effective in reducing morbidity or mortality than single-dose charcoal, but it may be administered
to enhance elimination in life-threatening ingestions caused by medications that undergo significant
enterohepatic recirculation with active enterohepatic metabolites (J Toxicol Clin Toxicol 1999;37:731-
51).

F. Whole Bowel Irrigation


1. Whole bowel irrigation is a strategy for cleansing the bowel to remove potential toxins by administering
an osmotic polyethylene glycol solution.
2. Not recommended for routine use, but may be useful in potentially life-threatening ingestions of
medications with long half-lives, sustained-release dosage forms, or enteric-coated formulations.
Specifically useful for certain toxic substances not adsorbed by activated charcoal (e.g., lithium and
iron). May also be beneficial for iron overdoses and for packers or stuffers of illicit substances.
3. Concurrent administration of activated charcoal and whole bowel irrigation may decrease the efficacy
of charcoal.
4. Complications of the polyethylene glycol electrolyte solutions include anaphylaxis, angioedema of the
lips, aspiration, Mallory-Weiss tear, and esophageal perforation.
5. Contraindications include bowel obstruction, perforation, ileus, and in patients with recent bowel
surgery. A kidney-ureter-bladder radiograph may be used to rule out these contraindications.

G. Urine Alkalinization
1. Urine alkalinization is a strategy to improve the elimination of toxins by increasing the urine pH to
levels of 7.5 or greater with the administration of sodium bicarbonate or sodium acetate (J Toxicol Clin
Toxicol 2004;42:1-26).
2. Specific substances that may benefit from this strategy include salicylates, phenobarbital, chlorpropamide,
and other weak acids with intrinsic urinary clearance.
3. Contraindications include acute and chronic renal failure and preexisting heart failure owing to the
volume of fluid required for this treatment strategy.
4. Complications include hypokalemia, hypernatremia, hypocalcemia, cerebral vasoconstriction, and
coronary vasoconstriction.
5. To administer urine alkalinization, it is recommended to check baseline blood chemistries, electrolyte
values, and an arterial blood gas and to correct any fluid or electrolyte deficits (especially potassium
because alkalemia will push potassium intracellularly). Hypokalemia will make it impossible to get the
urine alkaline because of the K+-H+ exchange in the kidneys, which will excrete H+ into urine if K+ is
low.
6. Guideline-recommended monitoring includes urine pH every 15–30 minutes (every 30–60 minutes
is more accepted in clinical practice) until the goal pH level of 7.5–8.5 is achieved, followed by every
hour; serum potassium concentrations, central venous pressure, and arterial blood gases should be
measured hourly.

ACCP Updates in Therapeutics® 2022: Critical Care Pharmacy Preparatory Review and Recertification Course

370
Toxicology

Table 2. Common Dosage Strategies for General Decontamination and Enhanced Elimination
Decontamination/
Pediatric Dosing Adult Dosing
Elimination Strategy
10-mL/kg aliquots, followed by return of 200- to 300-mL aliquots, followed by
Gastric lavagea
an equal amount return of an equal amount
Cathartics
Magnesium citrate: 4 mL/kg 240 mL
Sorbitol: 4.3 mL/kg (35% solution) 1–2 mL/kg (70% solution)
Activated charcoal
Single dose Up to 1 yr of age: > 12 years and adults:
0.5–1 g/kg (usually 10–25 g) 25–100 g (doses > 50 g may induce
1–12 years: vomiting)b
0.5–1 g/kg (usually 25–50 g)
Multidose 0.5–1 g/kg (25–50 g), followed by 50 g, followed by 25–50 g every 4 hr
0.25–0.5 g/kg (10–25 g) every 4 hr
9 mo to 6 yr:
> 12 yr and adults: Goal is 2000 mL/hr
500 mL/hr
Whole bowel irrigationc (initiated at 500 mL/hr and doubled
6–12 yr:
every 30 min)
1000 mL/hr
Urine alkalinizationd 25–50 mEq intravenously for 1 hr 250 mEq intravenously for 1 hr
a
Sterile water or 0.9% sodium chloride; may repeat until the return fluid is clear and absent of particulate matter.
b
Upper limit may vary depending on the capacity of the stomach.
c
Polyethylene glycol electrolyte lavage solutions; dose until the rectal effluent is clear or the desired effect has been achieved.
d
Sodium bicarbonate in dextrose 5% in water additional boluses can be given hourly (or begin a continuous infusion at this hourly rate) to maintain a urine pH of 7.5–8.5.

Table 3. Agents for Which Activated Charcoal Is NOT Recommended


Substance Examples
Acids Boric acid, mineral acids
Alcohols Ethanol, ethylene glycol, methanol
Alkalis Bleach, cleaning solutions, dishwasher detergents, lye
Carbamates Insecticides, neostigmine, physostigmine
Cyanide Cyanogen chloride, hydrogen cyanide, potassium cyanide, sodium cyanide
Hydrocarbons Gasoline, kerosene, petroleum oils
Metals Arsenic, iron, lead, lithium, mercury
Organic solvents Acetic acid, acetone, ethylene glycol, glycerin, toluene
Organophosphates Antihelminthic drugs (trichlorfon), insecticides (malathion, parathion), herbicides

IV.  ACETAMINOPHEN

A. Background
1. Acetaminophen is consistently one of the most common toxic drug exposures.
2. Accounted for 53,295 exposures (as a single agent) and resulted in 91 deaths in 2019.
3. In general, acute doses of 150 mg/kg or 7.5 g in adults and 200 mg/kg in children are considered toxic.
It is recommended that doses exceeding this threshold be managed in a health care facility.

ACCP Updates in Therapeutics® 2022: Critical Care Pharmacy Preparatory Review and Recertification Course

371
Toxicology

4. The mechanism of toxicity is caused by the active metabolite N-acetyl-p-benzoquinoneimine (NAPQI),


which can lead to oxidant cell injury, hepatic failure, and death.
5. Around 90% of acetaminophen undergoes phase II conjugation to glucuronide and sulfate conjugates
that are excreted in the urine. An additional 2% is excreted unchanged in the urine. The remaining
amount (8%–10%) is converted by cytochrome P450 (CYP2E1) to NAPQI. NAPQI is normally converted
by glutathione to cysteine conjugates, which are renally excreted. In an overdose, the sulfation and
glucuronidation pathways become saturated, leading to glutathione depletion and a subsequent buildup
of NAPQI (Clin Liver Dis 2013;17:587-607).

B. Clinical Presentation – Four clinical phases are associated with an acetaminophen toxicity (time intervals
are estimated and may vary with individual patients).
1. Phase I occurs within the first 24 hours after ingestion. Patients may present with minimal or no signs
of distress. Potential signs and symptoms include nausea, vomiting, diaphoresis, and anorexia.
2. Phase II occurs 24–48 hours after exposure and is marked by initial damage to the hepatocytes. Patients
may present with right upper quadrant pain, increases in liver transaminases, elevated total bilirubin
concentrations, and prolonged prothrombin time.
3. Phase III occurs 72–96 hours after initial exposure and is the peak of the hepatotoxic effects. Patients
may present with lactic acidosis, acute renal failure, acute pancreatitis, and fulminant hepatic failure, as
evidenced by jaundice, extensive coagulopathies, hypoglycemia, and hepatic encephalopathy.
4. Phase IV occurs about 1 week after exposure and marks the recovery phase if the patient survives phase
III.

C. Treatment
1. The goal of treatment is to prevent the development of hepatic toxicity and reduce mortality.
2. Gastric decontamination with a single dose of activated charcoal can be considered if the patient presents
within the first hour after exposure, is not vomiting, and has no alterations in mental status. However,
with the availability of highly efficacious antidotal therapy, the risks often outweigh the benefits.
3. Antidote therapy is recommended with acetylcysteine. The mechanism of action for acetylcysteine is to
increase the synthesis and bioavailability of glutathione, substituting for glutathione by binding to the
reduced sulfur group of NAPQI, and supplying a substrate for sulfation, thereby increasing nontoxic
metabolism. Additional mechanisms of action have also been proposed, including its facilitation of
reactive oxygen and nitrogen species scavenging.
4. Guidelines suggest that acetylcysteine treatment be administered to patients within the first 8 hours of
exposure if they can be stratified as being at possible or probable risk of hepatotoxicity by the Rumack-
Matthew nomogram (Figure 1). If patients cannot be stratified because of unknown time of ingestion,
they should receive acetylcysteine if any of the following conditions apply: significantly elevated
alanine aminotransferase (ALT) concentration, serum acetaminophen concentrations greater than 20
mcg/mL, or history of chronic ingestions exceeding 4 g/day with an elevated serum ALT concentration
and/or presenting more than 24 hours post-ingestion with evidence of hepatotoxicity (Ann Emerg Med
2007;50:292-313).
a. This includes patients presenting more than 24 hours postingestion with evidence of hepatotoxicity.
b. Limitations to use of the Rumack-Matthew nomogram include the following (Ann Emerg Med
2007;50:292-313):
i. Presentation more than 24 hours postingestion
ii. An unknown or unreliable history of ingestion
iii. Overdoses with extended-release formulations
iv. Chronic or repeated supratherapeutic ingestions
v. Patients with preexisting hepatic disease, chronic alcohol use, or concurrent medications
metabolized by the CYP system

ACCP Updates in Therapeutics® 2022: Critical Care Pharmacy Preparatory Review and Recertification Course

372
Toxicology

Figure 1. Rumack-Matthew nomogram.


Reprinted from: Tylenol for Healthcare Professionals. Guidelines for the Management of Acetaminophen Overdose. McNeil Consumer & Specialty Pharmaceuticals.
Available at www.tylenolprofessional.com/guidelines-and-studies.html. Accessed October 21, 2016.

5. Intravenous acetylcysteine is advantageous because of its decreased overall administration time (21
hours vs. 72 hours for oral) and minimal GI adverse effects. If intravenous acetylcysteine formulation
is not available and cannot be obtained in a timely fashion, poison control centers can be contacted for
instructions on compounding the inhalational acetylcysteine formulation for intravenous use. It is not
recommended to use this strategy except for emergencies.

ACCP Updates in Therapeutics® 2022: Critical Care Pharmacy Preparatory Review and Recertification Course

373
Toxicology

6. Oral acetylcysteine is dosed for 18 total doses; doses may be repeated if emesis occurs within 1 hour
of a dose. Two oral formulations exist. One preparation is an effervescent tablet that must be dissolved
in water before administration, and the other is a liquid preparation that may be diluted in juice or
carbonated beverages to improve palatability. Antiemetics may be administered if significant nausea
or vomiting occurs. Table 4 discusses the dosing strategies for oral and intravenous administration of
acetylcysteine.
a. For exceptionally large ingestions (e.g., over 30 g), some studies have recommended alternative
approaches:
i. An increased dose of acetylcysteine (Med J Aust 2020;212:175-83).
ii. Given that acetaminophen is dialyzable, intermittent hemodialysis is an option (Clin Toxicol
2014;52:856-67). This approach would also require an increase in the acetylcysteine dose (i.e.,
2-fold) due to its concurrent increased clearance (Clin Toxicol 2016;54:519-22).
7. Although treatment guidelines recommend 18 total doses of acetylcysteine administered throughout
72 hours for oral acetylcysteine therapy and 21 hours of the intravenous infusion of acetylcysteine,
many poison control centers recommend early discontinuation (or prolonged therapy) if the following
conditions are met (or if they are not met) (Ann Emerg Med 2007;50:280-1):
a. Serum acetaminophen concentrations are undetectable or less than 10 mcg/mL.
b. ALT concentrations are normal (60 IU/L or less) or improving. Some clinicians also advocate an
international normalized ratio (INR) of 1.3 or less.
c. The patient is clinically improved.
8. Adverse effects (intravenous): Anaphylactoid reactions (rash, urticarial, pruritus), hyponatremia,
hypervolemia, seizures (pediatric patients with unadjusted volume)
9. Adverse effects (oral): Nausea, vomiting, anaphylactoid reactions (rare)
10. Patients experiencing mild anaphylactoid reactions (rash, pruritus, flushing) can be effectively treated
with diphenhydramine, and acetylcysteine therapy can be resumed.
11. Medication errors may occur because of the complex dosing regimens (Ann Pharmacother 2008;42:766-
70). Common errors include delays in therapy, incorrect dosages, and incorrect infusion rates. To
mitigate these errors, some institutions have developed a two-step intravenous regimen (Toxicol
Commun 2018;2:81-4).

Table 4. Acetylcysteine Dosage


Route Dose
Loading dose:
140 mg/kg
Oral
Maintenance doses:
70 mg/kg every 4 hr for a total of 17 doses (72 hr)
Loading dose:
150 mg/kg (max 15 g)a in 200 mL of 5% dextrose in water infused for 60 min
Maintenance dose:
Intravenous 50 mg/kg (max 5 g)a in 500 mL of 5% dextrose in water infused for 4 hr
followed by
100 mg/kg (max 10 g)a in 1000 mL of 5% dextrose in water infused for 16 hr
Patients weighing < 40 kg require reduced volume administration
a
Dose limits are provided by the manufacturer; however, small retrospective studies have shown that acetylcysteine is well tolerated when using actual body weight for
patients > 100 kg and exceeding the recommended max doses (Am J Ther 2016;23:e714-9.)(Am J Ther 2014;21:159-63).

ACCP Updates in Therapeutics® 2022: Critical Care Pharmacy Preparatory Review and Recertification Course

374
Toxicology

D. Monitoring
1. Patients should be monitored for improvement in vital signs and mental status.
2. The following laboratory values should be monitored periodically for improvement as well as for
potential worsening.
a. ALT, aspartate aminotransferase (AST), total bilirubin, INR, and prothrombin time
b. BUN and SCr
c. Serum electrolytes
d. Fulminant hepatic failure: Serum bicarbonate, serum sodium, serum lactate, arterial blood gas,
serum glucose, and ammonia concentrations

Patient Case

Questions 2 and 3 pertain to the following case.


A 42-year-old woman (height 66 inches, weight 79.2 kg [176 lb]) presents to the ED with the chief concern of
flu-like symptoms. Her symptoms include headache, congestion, severe nausea and vomiting, abdominal pain,
and some confusion. She has been taking acetaminophen 500-mg caplets as needed for her symptoms, but she
just ran out of the bottle she purchased yesterday. On presentation, she is alert and oriented. Her vital signs are as
follows: BP 135/90 mm Hg, HR 83 beats/minute, RR 18 breaths/minute, and temperature 101.8°F (38.8°C). An
acetaminophen concentration on admission was 100 mcg/mL, AST 560 IU/L, and ALT 310 IU/L. The physician
wants to begin general management.

2. Which general management strategy is most indicated for this patient?


A. 10% magnesium citrate 240 mL per tube once
B. Continued stabilization of the patient and good supportive care
C. Gastric lavage
D. Charcoal 50 g once

3. Which is the most appropriate treatment for her acetaminophen toxicity?


A. Give acetylcysteine 11,200 mg oral bolus, followed by 5600 mg orally every 4 hours for 17 doses.
B. Give acetylcysteine 11,200 mg intravenous bolus, followed by 5600 mg intravenously every 4 hours
for 12 doses.
C. Give acetylcysteine 12,000 mg intravenously over 1 hour, followed by 4000 mg intravenously over 4
hours; then 8000 mg intravenously over 16 hours.
D. Acetylcysteine therapy is not indicated in this patient.

V.  SALICYLATES

A. Background
1. Salicylates as a single agent (not in combination with other agents) accounted for 8011 overdoses and
15 deaths in 2019. These numbers, which include overdoses of both adult and pediatric formulations of
acetylsalicylic acid, are often underreported because these products are not typically recognized as a
potential cause.
2. However, because of the development of child-resistant containers, aspirin overdose causing accidental
death in children has reduced drastically. (Lancet 1977;2: 289-90)

ACCP Updates in Therapeutics® 2022: Critical Care Pharmacy Preparatory Review and Recertification Course

375
Toxicology

B. Clinical Presentation
1. The mechanism of toxicity for salicylates is through the interference with aerobic metabolism owing
to the uncoupling of mitochondrial oxidative phosphorylation, leading to increases in anaerobic
metabolism, which causes a significant lactic acidosis (Emerg Med Clin North Am 2007;25:333-46).
This also leads to hypoglycemia because of glycogen depletion, gluconeogenesis, and catabolism
of proteins and free fatty acids. Salicylates also directly stimulate the respiratory center, leading to
hyperventilation and respiratory alkalosis. Secondary complications from hyperventilation include
dehydration and compensatory metabolic acidosis.
2. Salicylates are readily absorbed in the stomach and small intestine and are then conjugated with glycine
in the liver to the active component, salicylic acid. In overdoses, the liver cannot metabolize the excess
drug, and most is then excreted unchanged by the kidneys (Postgrad Med 2007;121:162-8).
3. The most common clinical symptoms associated with a salicylate overdose are hyperventilation
(respiratory alkalosis), tinnitus, and GI irritation. Symptoms may vary depending on the serum
salicylate concentration; however, these may be low to normal early in the presentation (Postgrad Med
2007;121:162-8):
a. Serum concentration less than 30 mg/dL: Asymptomatic
b. Serum concentration 15–30 mg/dL: Therapeutic concentrations
c. Serum concentration 30–50 mg/dL: Hyperventilation, nausea, vomiting, tinnitus, dizziness
d. Serum concentration 50–70 mg/dL: Tachypnea, fever, sweating, dehydration, listlessness
e. Serum concentration greater than 70 mg/dL: Coma, seizures, hallucinations, stupor, cerebral
edema, dysrhythmias, hypotension, oliguria, renal failure
4. Acute salicylate toxicity is typically associated more with the GI symptoms; chronic toxicity is more
associated with the CNS-type symptoms.
5. Absorption may be delayed up to 36 hours because of gastric pylorospasm, bezoar formation (precipitate
concretions because of poor solubility), or enteric-coated formulations; therefore, these ranges should be
used with caution because they may not correlate with actual symptoms (Am J Emerg Med 2010;28:383-4).

C. Treatment
1. There is no antidote for salicylate poisoning; the goals of therapy are to limit the additional absorption
of salicylates and to provide supportive care.
2. Maintain a patent airway and assist ventilation, if necessary. Ensure adequate ventilation to prevent
respiratory acidosis; however, do not mechanically ventilate patients unless clinical necessary because
this will interfere with the patient’s ability to appropriately compensate and maintain pH.
3. Gastric decontamination with a single dose of activated charcoal is recommended within 60 minutes of
acute ingestions and if the patient is alert with the absence of vomiting. However, there may be a role for
administration after this time frame, as well as multidose administration based on the characteristics
of the ingestion.
4. Administer intravenous crystalloid fluids at a rate of 10-20 mL/kg/hr for the first 2 hours to maintain
BP and a urine output of 1–1.5 mL/kg/hr.
5. Administer intravenous glucose for hypoglycemia or significant neurologic symptoms.
6. Urine alkalinization is recommended to enhance renal elimination and increase the glomerular filtration
rate.
a. Administration strategies that have been described in the literature:
i. Administer 250 mL of sodium bicarbonate 8.4% over 1 hour; then administer additional 50-
mL boluses as needed to maintain a goal urine pH range of 7.5–8.5.
ii. Administer a continuous infusion of 150 mL of sodium bicarbonate 8.4% in 1 L of 5% dextrose
in water at 2–3 mL/kg/hour to maintain a urine output of 1–2 mL/kg/hour. Do not allow the
serum pH to fall below 7.4.

ACCP Updates in Therapeutics® 2022: Critical Care Pharmacy Preparatory Review and Recertification Course

376
Toxicology

b. Oral bicarbonate is not recommended because it may enhance salicylate absorption by accelerating
tablet dissolution.
c. Discontinue therapy once the serum salicylate concentrations are less than 30 mg/dL or there is a
resolution of clinical symptoms.
7. Alkaline diuresis will increase potassium secretion, and hypokalemia will make it more challenging to
raise the urine pH. Adequate potassium concentrations should be aggressively maintained.
8. Consider hemodialysis for any of the following (Postgrad Med 2007;121:162-8):
a. Acute renal insufficiency
b. End-organ damage (severe pulmonary edema, seizures, rhabdomyolysis)
c. Altered mental status
d. Deterioration of clinical status
e. Severe acid-base disturbances
f. If hemodialysis is not effective, hemoperfusion and continuous renal replacement therapies may be
considered (Ann Emerg Med 2015;66:165-81).

D. Monitoring
1. Patients should be monitored for up to 24 hours because of the possibility of delayed or impaired
absorption.
2. Monitor RR, and support as needed; caution is advised if intubation is required because of a requirement
for an increased minute ventilation (Am J Emerg Med 2010;28:383-4).
3. During urine alkalinization, monitor for signs and symptoms of fluid overload, hypernatremia,
hypokalemia, hypocalcemia, and worsening alkalemia.

Patient Case

4. A 62-year-old man presents to the ED with the chief concern of nausea, tachypnea, and flu-like symptoms.
He is alert and oriented and can communicate that his symptoms have been worsening for the past 2 days.
He has been taking a combination cold product, which he thinks has helped. His medical history is signifi-
cant for a stroke, for which he takes aspirin 325 mg by mouth daily, and hypertension, for which he takes
amlodipine 5 mg by mouth daily. His vital signs are as follows: BP 135/82 mm Hg, HR 78 beats/minute, RR
29 breaths/minute, and temperature 100.2°F (37.9°C). Arterial blood gas results are as follows: pH 7.52, Pco2
25, and HCO3 20 mEq/L. A salicylate concentration is sent, which is 25 mg/dL. Which treatment manage-
ment strategy is most indicated for this patient?
A. Sodium chloride infusion
B. Urgent endotracheal intubation
C. Sodium bicarbonate infusion
D. Hemodialysis

ACCP Updates in Therapeutics® 2022: Critical Care Pharmacy Preparatory Review and Recertification Course

377
Toxicology

VI.  OPIOIDS

A. Background
1. Opioids as a single agent (not in combination with other agents) accounted for 21,211 overdoses and 57
deaths in 2019. The Centers for Disease Control and Prevention reports that almost 15,000 deaths are
caused by prescription opioid painkillers annually.
2. The most common agents associated with a toxicologic event were tramadol, oxycodone, methadone,
morphine, and buprenorphine.
3. The most common agents associated with a toxicologic death were methadone, oxycodone, fentanyl,
morphine.
4. Opioids act at the mu, delta, and kappa opioid receptors, although mu is responsible for most of the
opioids’ clinical effects.

B. Clinical Presentation
1. The most common clinical symptoms associated with opioid overdose are respiratory depression
(defined as fewer than 12 breaths/minute), coma, miosis, and hypoactive bowel sounds.
2. Additional findings may include stupor, hepatotoxicity, acute renal failure, rhabdomyolysis, compartment
syndrome, hypothermia, and seizures (N Engl J Med 2012;367:146-55).
3. Diagnostic workup (Pharmacotherapy: A Pathophysiologic Approach, 9e. New York: McGraw-Hill,
2014):
a. 12-lead ECG to test for QT prolongation – Methadone may cause QT prolongation and potentially
torsades de pointes.
b. Arterial blood gas to monitor for respiratory acidosis
c. Standard chemistry panel for electrolyte and glucose abnormalities – Creatine kinase (CK), BUN,
and SCr for signs of rhabdomyolysis
d. Pulse oximetry

C. Treatment
1. Stabilize the airway, provide supplemental bag-valve mask respirations if needed, and administer
supplemental oxygen. Establishment of an airway, if needed, by endotracheal intubation.
2. Administer intravenous crystalloid fluids to maintain BP.
3. Gastric decontamination with a single dose of activated charcoal may be considered only if the patient
presents within the first hour after exposure and is awake with an intact airway. Whole bowel irrigation
can be considered for extended-release formulations or for packers or stuffers of illicit substances
(including ingestion of fentanyl patches).
4. Antidote therapy (N Engl J Med 2012;367:146-55):
a. Naloxone is a competitive antagonist at the opioid receptor.
b. The intravenous route is preferred, but naloxone is also effective through the endotracheal,
intramuscular, intranasal, inhalational, intraosseous, or intrapulmonary route. However, it is
important to note that the bioavailability and onset of action can vary greatly between different
routes.
c. Onset of action of intravenous naloxone is 1–2 minutes, with a duration of 30–120 minutes.
d. Dosing may be affected by the specific opioid agent and dose, affinity for the mu-receptor, and
patient weight. Goal of care is reversal of ventilatory depression, not necessarily reversal of
neurologic depression, as this is more likely to precipitate acute opioid withdrawal syndrome.

ACCP Updates in Therapeutics® 2022: Critical Care Pharmacy Preparatory Review and Recertification Course

378
Toxicology

e. Initial dose is 0.04 mg in adult patients and 0.1 mg/kg in pediatric patients; if no response, the dose
is increased every 2–3 minutes to 0.5 mg, 2 mg, 4 mg, and 10 mg, followed by 15 mg. Large doses
and frequent repeat administration of naloxone are necessary for patients thought to have had an
overdose of heroin laced with carfentanil.
f. It is recommended that a continuous infusion be initiated at a dose of two-thirds the effective bolus
dose per hour (0.04–4 mg/hour) for patients requiring subsequent naloxone doses to sustain effect
secondary to exposure to long-acting opioids (e.g., methadone) or a sustained-release product (Ann
Emerg Med 1986;15:566-70).
g. Intranasal administration appears to be safe and effective when the intravenous route is not
available. The dose is administered by attaching a 2 mg/2 mL naloxone prefilled syringe to an
atomizer and spraying an equal portion (1 mL) of the contents into each nostril. Commercial
preparations contain up to 4 mg per actuation.
h. Adverse effects are rare and may be more related to a return of sympathetic response to opioid
withdrawal. Nausea and vomiting are common and can lead to aspiration. If the situation
allows, providing assisted manual ventilation prior to administering naloxone can potentiate the
sympathetic response to opioid reversal (Anesth Analg 1988;67:730-6).
i. If no effect is seen at the higher naloxone doses, consider other causes such as secondary or
alternative agents.

D. Monitoring – Observe respiratory status and vital signs for a minimum of 4 hours after the last dose of
naloxone or discontinuation of the continuous infusion. The duration of a naloxone continuous infusion will
vary based on the quantity, pharmacokinetics, and pharmacodynamics of the opioid exposure, in addition
to patient specific factors. Closely monitor for signs and symptoms of opioid withdrawal syndrome, such as
anxiety, piloerection, heightened sensation to pain, abdominal cramps, diarrhea, and insomnia.

VII.  LOPERAMIDE

A. Background
1. A nonprescription antidiarrheal medication with a 91% increase in reported abuse in 2010-2015 resulting
in 15 deaths (Ann Emerg Med 2017;69:73-8). Loperamide as a single agent (not in combination with
other agents) accounted for 902 overdoses and 6 deaths in 2019.
2. Relatively safe at therapeutic doses; however, can be fatal when high doses are ingested for its euphoric
effects
3. Phenylpiperidine opioid that slows intestinal transit time by stimulating mu-opioid receptors in the GI
tract and blocks intestinal calcium channels
4. Toxicity of loperamide involves blockade of sodium channels and potassium channels in the cardiac
tissue, causing QT prolongation and QRS interval widening and leading to life-threatening dysrhythmias
and cardiac death

B. Clinical Presentation
1. Common signs/symptoms include respiratory depression, nausea, vomiting, decreased level of
consciousness, miosis, decreased bowel motility/ileus, palpitations, and syncope.
2. In severe toxicity, ECG findings are abnormal, including widened QRS interval and prolonged QT
interval.

ACCP Updates in Therapeutics® 2022: Critical Care Pharmacy Preparatory Review and Recertification Course

379
Toxicology

3. Diagnostic workup
a. 12-lead ECG to test for QT prolongation and QRS interval widening and development of ventricular
dysrhythmias
b. Arterial blood gas to monitor for respiratory acidosis secondary to respiratory depression
c. Standard chemistry panel for electrolyte and glucose abnormalities – CK, BUN, and SCr for signs
of rhabdomyolysis
d. Pulse oximetry

C. Treatment
1. Stabilize the airway and provide supplemental oxygen, if needed. Establish an airway if patients cannot
protect their airway or have significant respiratory depression.
2. Administer intravenous crystalloid fluids to maintain blood pressure.
3. Gastric decontamination with a single dose of activated charcoal may be considered only if patients
present within 2-4 hours after a large overdose and patients can protect their airway.
4. Antidote therapy (Ann Emerg Med 2017;pii: S0196-0644(17)30424-9.):
a. Naloxone
i. Should be administered in addition to supportive care in patients with respiratory depression
who cannot protect their airway
ii. The lowest effective dose should be used, and repeat doses may be necessary secondary to
loperamide’s slow elimination.
b. Sodium bicarbonate
i. May be beneficial in patients with signs and symptoms of cardiac toxicity and QRS interval
widening secondary to sodium channel blockade by loperamide
ii. Sodium bicarbonate 1-2 mEq/kg intravenously should be administered concomitantly with
magnesium and potassium chloride if electrolyte abnormalities exist.

D. Monitoring
1. Patients should be closely monitored for resolution of clinical symptoms, including respiratory status,
vital signs, and return of baseline mental status.
2. Monitor serum electrolytes, blood glucose, and 12-lead ECG periodically.
3. Closely monitor for signs and symptoms of opioid withdrawal.

VIII.  ALCOHOLS (METHANOL AND ETHYLENE GLYCOL)

A. Background
1. Alcohol poisonings (methanol and ethylene glycol) are not as common as poisonings with other
substances, accounting for <1% of all cases in 2019 (National Poison Data System), but they can be
serious and potentially fatal.
2. Methanol is commonly found in products such as windshield washer fluid, antifreeze, brake and
carburetor fluids, and cooking products.
3. Ethylene glycol is commonly found in products such as antifreeze, de-icing solutions, refrigerants, and
brake fluids.
4. Toxicity of both agents is caused by the breakdown to toxic metabolites by alcohol dehydrogenase and
aldehyde dehydrogenase.

ACCP Updates in Therapeutics® 2022: Critical Care Pharmacy Preparatory Review and Recertification Course

380
Toxicology

a. Methanol is converted to formaldehyde and then to formic acid, which results in an anion gap
acidosis and ocular toxicity.
b. Ethylene glycol is converted to glycoaldehyde and then to glycolic acid, followed by glyoxylic acid,
and, eventually, oxalic acid. Glycolic acid results in an anion gap acidosis and CNS toxicity. Oxalic
acid results in CNS toxicity and renal toxicity because of the formation of calcium oxalate crystals.

B. Clinical Presentation
1. Common symptoms include inebriation, altered mental status, nausea, vomiting, hematemesis,
nystagmus, and depressed reflexes. In rare cases of ethylene glycol toxicity, patients may present with
tetany caused by hypocalcemia. Symptoms typically develop over the first 24 hours; however, they can
be delayed for days in the setting of ethanol coingestion.
2. Early in therapy, an osmolar gap will be present, but this will diminish as the parent compound is
metabolized.
a. As the osmolar gap declines, the anion gap will rise, resulting in a significant metabolic acidosis.
b. Calculations:
i. Osmolar gap (OG):
Measured osmolality - calculated osmolality (normal OG less than 10)
ii. Calculated osmolality:
(sodium x 2) + (glucose/18) + (BUN/2.8)
iii. Calculated osmolality with ethanol ingestion:
(sodium x 2) + (glucose/18) + (BUN/2.8) + (ethanol/4.6)
iv. Calculated osmolality with methanol ingestion:
(sodium x 2) + (glucose/18) + (BUN/2.8) + (methanol/3.2)
v. Calculated osmolality with ethylene glycol ingestion:
(sodium x 2) + (glucose/18) + (BUN/2.8) + (ethylene glycol/6.2)
vi. Anion gap:
Na − (Cl + HCO3)
3. Methanol and ethylene glycol serum concentrations may be monitored to determine severity and to
guide therapy in conjunction with an anion gap metabolic acidosis. Often, the ability to obtain these
serum concentrations is not readily available and may take several hours to perform; therefore, therapy
should not be delayed.

C. Treatment
1. Treatment is focused on blocking the toxic alcohol metabolism and allowing it to be excreted unchanged
in the urine. Treatment is recommended when serum methanol or ethylene glycol concentrations
exceed 20 mg/dL, the patient has a documented history of ingestion, there is a high clinical suspicion of
ingestion combined with an osmolal gap >10 mOsm/kg, or there is a presence of an anion gap metabolic
acidosis of unknown etiology.
2. Gastric decontamination is not recommended.
3. Fomepizole is the preferred antidote because of its predictable response, ease of dosing, and lack of
contraindications to use.
a. Mechanism of action is competitive inhibition of alcohol dehydrogenase.
b. Dosing is a 15-mg/kg intravenous bolus (as a loading dose); then 10 mg/kg every 12 hours for four
doses; then 15 mg/kg every 12 hours until methanol/ethylene glycol concentrations are less than
20 mg/dL.
c. After 48 hours, fomepizole induces its own metabolism, requiring dosage increases.
d. Oral administration is effective and may be considered if intravenous access cannot be established
(Clin Toxicol 2008;46:181-6).

ACCP Updates in Therapeutics® 2022: Critical Care Pharmacy Preparatory Review and Recertification Course

381
Toxicology

e. Therapy is discontinued when methanol/ethylene glycol concentrations are less than 20 mg/dL. If
the patient is still symptomatic with a normal pH, further workup is warranted, and hemodialysis
may be indicated.
f. Hemodialysis increases the clearance of fomepizole; therefore, doses must be administered every 4
hours during hemodialysis. Continuous renal replacement therapy has also been found to increase
the clearance of fomepizole, although guidance regarding dosing adjustments is limited and
requires further investigation.
g. Adverse effects may include headache, nausea, dizziness, abdominal pain, hypotension, and
bradycardia.
4. Ethanol may be administered by diluting 95% alcohol for intravenous, oral, or per-tube administration.
a. Mechanism of action is competitive inhibition of alcohol dehydrogenase.
b. Alcohol dehydrogenase has a higher affinity for ethanol.
c. Intravenous alcohol preparations are no longer commercially available and must be compounded.
d. Initial dosing is 600–700 mg (7.6–8.9 mL/kg) of a 10% solution, followed by an infusion of 66
mg/kg/hour (0.83 mL/kg/hour). The infusion dose may be initiated at 154 mg/kg/hour (1.96 mL/
kg/hour) in chronic drinkers. The goal is to maintain a serum ethanol concentration of 100 mg/dL
(0.1%) until symptoms have diminished and methanol or ethylene glycol serum concentrations are
undetectable. (Ann Emerg Med 2009;53:439-50).
e. Disadvantages include frequent monitoring and ICU admission in some institutions.
f. Adverse effects include CNS depression, nausea, vomiting, abdominal pain, polyuria, and
hypoglycemia (especially in children).
5. Hemodialysis should be considered if the clinical condition deteriorates, as evidenced by:
a. Methanol/ethylene glycol concentration greater than 50 mg/dL
b. Significant metabolic acidosis
c. Development of acute renal failure (ethylene glycol) or visual disturbances (methanol)
d. Development of significant electrolyte abnormalities
6. Additional therapies
a. Pyridoxine and thiamine
i. Serve as cofactors in the metabolism of the toxic metabolites of ethylene glycol to nontoxic
metabolites
ii. Pyridoxine promotes the metabolism of glyoxylate to glycine.
(a) Dosing is generally recommended to be one or more intravenous doses of 50 mg
iii. Thiamine promotes the metabolism of glycolic acid to a nontoxic metabolite; also used to
prevent or treat Wernicke-Korsakoff syndrome.
(a) Dosing is generally recommended to be 100 mg intravenously, although a concomitant
history of chronic alcohol use may warrant use of higher doses (e.g., 500 mg).
b. Folinic acid
i. Serves as a cofactor in the metabolism of the toxic metabolites of methanol to nontoxic
metabolites. May reduce formate accumulation and reduce the development of metabolic
acidosis ingestion (Crit Care Clin 2012;28:661-771).
(a) Dosing is generally recommended to be 1 mg/kg (maximum of 50 mg) intravenously at
4-hour intervals
ii. Folic acid may be used if folinic acid is unavailable.
c. Dextrose
i. Recommended to check a point-of-care level blood glucose concentration before administration
(Crit Care Clin 2012;28: 661-771)
ii. Administer 50 mL of 50% dextrose in water if blood glucose is 70 mg/dL or less or if testing
is unavailable.

ACCP Updates in Therapeutics® 2022: Critical Care Pharmacy Preparatory Review and Recertification Course

382
Toxicology

d. Magnesium – Recommended to administer 1–2 g intravenously for hypomagnesemia (more


common in chronic alcohol use)
e. Antiseizure medications
i. Benzodiazepines are the preferred emergent agents to treat seizures.
ii. Subsequent urgent control therapy can be implemented with intravenous fosphenytoin,
valproate sodium, phenobarbital, or levetiracetam.
(a) Phenytoin would not be recommended for patients with a history of chronic ethanol
ingestion due to enhanced phenytoin metabolism (Gastroenterology 1969;56:412).

D. Monitoring
1. Patient should be closely monitored for resolution of clinical symptoms and return of baseline mental
status.
2. Monitor serum electrolytes and blood glucose periodically.
3. Arterial blood gases with a goal of pH greater than 7.2
4. Methanol/ethylene glycol concentrations with a goal of less than 20 mg/dL
5. Monitor the osmolar and anion gaps to ensure the toxic alcohol and metabolites are being cleared
appropriately.

Patient Cases

5. A 35-year-old man is admitted to the ED appearing inebriated. He is alert but oriented only to person. His
vital signs are BP 122/80 mm Hg, HR 82 beats/minute, and RR 25 breaths/minute. His serum ethanol con-
centration is 20 mg/dL, and his ethylene glycol concentration is 100 mg/dL. Which is the most appropriate
therapy at this time?
A. Fomepizole
B. Ethanol infusion
C. Thiamine
D. Activated charcoal

6. A patient with methanol intoxication is initiated on fomepizole treatment together with hemodialysis. After
the 15-mg/kg bolus dose is given, which would be best for adjusting the maintenance fomepizole doses dur-
ing dialysis?
A. 10 mg/kg every 12 hours
B. 20 mg/kg every 12 hours
C. 10 mg/kg every 4 hours
D. 20 mg/kg every 4 hours

ACCP Updates in Therapeutics® 2022: Critical Care Pharmacy Preparatory Review and Recertification Course

383
Toxicology

IX.  ALCOHOL WITHDRAWAL

A. Background
1. Alcohol withdrawal is a relatively common consequence of hospital admission.
2. The strongest risk factor is a history of alcohol withdrawal.
a. Additional tools have been developed, such as the Prediction of Alcohol Withdrawal Severity Scale,
to help identify patients at risk for alcohol withdrawal before development of severe symptoms
(Alcohol 2014;48:375-90).

B. Clinical Presentation
1. Withdrawal symptoms typically occur within 8 hours after blood alcohol concentrations decrease, peak
at 72 hours, and are markedly reduced at 5–7 days (N Engl J Med 2014;371:2109-13).
2. Common symptoms include tremors, diaphoresis, nausea, vomiting, and abnormal vital signs, including
hypertension, tachycardia, hyperthermia, and tachypnea (Am J Emerg Med 2013;31:734-42).
3. Additional symptoms categorized as moderate to severe withdrawal include:
a. Alcoholic hallucinations – Auditory, visual, or tactile; may last up to 6 days
b. Alcohol withdrawal seizures (tonic-clonic) – Occur within 72 hours; however, it is highest in the
first 24 hours after the last drink.
c. Delirium tremens – Severe and potentially life-threatening symptom that may develop within
72 hours. Includes autonomic hyperactivity, confusion, delirium, psychosis, hallucinations, and
seizures.

C. Treatment
1. The goal of therapy is to keep the patient safe, alleviate and prevent the progression of symptoms,
and treat comorbidities (Crit Care Med 2010;38(suppl):S494-S501). Agents for the treatment of alcohol
withdrawal are listed in Table 5.
2. Benzodiazepines are the primary agents used in treatment. Binds to the γ-aminobutyric acid (GABA A)
receptor, resulting in hyperpolarization and membrane stabilization.
a. Lorazepam and diazepam are preferred because of their more predictable effects.
b. Chlordiazepoxide is not recommended in the acute stabilization setting.
c. Symptom-triggered therapy is preferred because it reduces benzodiazepine use, duration of
mechanical ventilation, and duration of ICU stay.
i. A front-loading regimen (i.e., a single dose of preventative medication) is recommended for
patients at high risk for severe withdrawal (J Addict Med 2020;14:1-72).
d. Scheduled treatment may be necessary if symptoms are severe or difficult to control.
3. Ethanol – Use of ethanol to control alcohol withdrawal is controversial and is not routinely recommended.
4. Phenobarbital
a. Barbiturate with sedative, hypnotic, and antiseizure activity. Increases the binding of GABA to
GABAA receptor and prolonging the chloride channel opening. It may also concomitantly mitigate
neuronal stimulation through NMDA receptor antagonism. Potential advantage over benzodiazepines
in alcohol withdrawal because it does not require GABA to be effective (GABA may be depleted),
and a longer duration of action.
b. Historically considered a second-line agent if benzodiazepines fail to adequately control symptoms.
c. May increase the efficacy of benzodiazepines when used in combination by increasing the binding
to the GABA A receptor.
5. Clonidine: α2-receptor agonist that helps control the catecholamine surge associated with withdrawal
that is responsible for elevations in BP and HR; however, it will not prevent seizures.

ACCP Updates in Therapeutics® 2022: Critical Care Pharmacy Preparatory Review and Recertification Course

384
Toxicology

6. Baclofen: Selective GABAB receptor agonist that reduces the signs and symptoms of alcohol withdrawal;
however, it will not prevent seizures.
7. Gabapentin: Increases GABA syntheses and concentrations; may reduce benzodiazepine requirements;
however, it may not effectively control seizures when used as monotherapy (Ann Pharmacother
2015;49:897-906).
8. Propofol
a. General anesthetic acting through GABA A receptor agonism and N-methyl-D-aspartate (NMDA)
receptor antagonism; chronic alcohol use is associated with an up-regulation of NMDA.
b. Useful for controlling delirium and preventing seizures
9. Dexmedetomidine
a. α2-Receptor agonist, which may help control BP and HR; however, it will not prevent seizures.
b. May reduce overall benzodiazepine requirements; however, it lacks the activity necessary to
prevent seizures.
c. Recommended when clonidine cannot be administered and as adjunct therapy
10. Ketamine: NMDA antagonist; may reduce overall benzodiazepine use and can be considered in patients
refractory to other therapies (Crit Care Med 2018;46:e768-71).
11. Supportive care: Alcohol-dependent patients are often nutritionally deficient and at risk of Wernicke
encephalopathy and hypomagnesemia. High dose intravenous folic acid, thiamine, and magnesium
should be administered to nutrient-deficient patients. Dextrose containing fluids may also be considered
(Crit Care Med 2016;44:1545-52).

D. Monitoring
1. Clinical Institute Withdrawal Assessment for Alcohol Scale (revised version) (CIWA-Ar) or the
Minnesota Detoxification Scale (MINDS) to determine the severity of symptoms and treatment
2. A sedation score, such as the Richmond Agitation-Sedation Scale, should be used in place of the CIWA
in critically ill intubated patients to guide benzodiazepine dosing.
3. Vital signs every 2–4 hours
4. Electroencephalogram for sustained seizure-related activity

Table 5. Agents for Treatment of Alcohol Withdrawal


Agent Suggested Starting Dose Suggested Interval/Infusion Dose Range
Diazepam 5–20 mg PO/IV Every 6–8 hr
Lorazepam 2–4 mg PO/IV Every 4–6 hr
Phenobarbital 65– 260 mg or 10 mg/kg IV Every 15–20 min until symptoms are controlled
Clonidine 0.1–0.3 mg PO Every 8–12 hr
Baclofen 5–10 mg PO Every 8–12 hr
Gabapentin 600–800 mg PO Every 8 hr
Propofol 10–20 mcg/kg/min IV 20–70 mcg/kg/min
Dexmedetomidine 0.2–0.4 mcg/kg/hr IV 0.4–1.5 mcg/kg/hr
Ketamine 0.15–0.2 mg/kg/hr IV 0.2–0.3 mg/kg/hr
Thiamine 100–500 mg IV Every 8–24 hr for 3–5 days
Folic acid 1–5 mg IV Once daily for 3–5 days
Magnesium 1–4 g IV Once daily for 3–5 days
IV = intravenously; PO = orally or per tube.

ACCP Updates in Therapeutics® 2022: Critical Care Pharmacy Preparatory Review and Recertification Course

385
Toxicology

X.  β-BLOCKERS AND CALCIUM CHANNEL BLOCKERS

A. Background
1. Cardiovascular agents accounted for a little more than 100,000 toxic exposures in 2019 and were a
leading cause of death secondary to pharmaceutical exposure.
2. Two of the most common cardiovascular agents involved in single-agent toxic exposures were β-blockers
(11,166 cases and 13 deaths in 2019) and calcium channel blockers (6020 cases and 30 deaths in 2019).

B. Clinical Presentation
1. β-Blocker overdoses are characterized by hypotension, bradycardia, and prolonged atrioventricular
conduction.
2. Calcium channel blocker overdoses by the non-dihydropyridine agents are characterized by
hypotension, prolonged atrioventricular conduction, bradycardia, lethargy, hyperglycemia, and
depressed consciousness. The dihydropyridine agents act peripherally and are primarily associated with
vasodilation, hypotension, and reflex tachycardia. However, this receptor selectivity is lost at high doses
for both categories (Ann Emerg Med 1993;22:196-200).

C. Treatment
1. Consider gastric lavage or activated charcoal if patients present within 1–2 hours of overdose. Whole
bowel irrigation is recommended for delayed presentation or for sustained- or extended-release
formulations.
2. Maintenance of hemodynamic stability
a. Goal of therapy is a mean arterial pressure greater than 65 mm Hg or systolic blood pressure (SBP)
greater than 90 mm Hg.
b. Administer isotonic fluids (0.9% sodium chloride or lactated Ringer solution) at 20–30 mL/kg
(preferred) or colloidal solutions (e.g., albumin 5% 250 mL).
3. Administer intravenous calcium chloride or calcium gluconate. First line therapy for calcium channel
blocker toxicity.
a. Calcium chloride 1–2 g (central line is preferred; however, bolus doses may be administered in a
peripheral line if needed)
b. Calcium gluconate 3–6 g. Repeat dose may be given every 15–30 minutes; may also consider
continuous infusion at 0.3–0.7 mEq/kg/hour.
4. Consider sodium bicarbonate for QRS widening, severe acidosis, or dysrhythmia: 1– 2 mEq/kg bolus;
may consider a continuous infusion if symptoms persist.
5. Treat symptomatic bradycardia:
a. Atropine 0.5–1 mg intravenously; if no response, continue to the following options:
b. Glucagon 5–10 mg (100 mcg/kg) intravenous push for 1 minute (consider for β-blocker toxicity
only). If symptom response is achieved from the bolus, may consider a continuous intravenous
infusion initiated at the same rate (milligrams per hour) as the bolus dose that achieved response.
i. Stimulates adenylate cyclase, which increases intracellular cAMP (cyclic adenosine
monophosphate), leading to increased inotropy, chronotropy, and cardiac conduction
ii. Adverse effects include nausea, vomiting, and hyperglycemia. Consider premedicating with
an antiemetic intravenously before bolus dosing of glucagon to decrease the risk of nausea and
vomiting.
iii. Use with caution with decreased mental status because of possible aspiration or airway
obstruction (also causes lower esophageal sphincter relaxation).
iv. Not recommended as a preferred treatment option for calcium channel blockers because of
limited efficacy and cost

ACCP Updates in Therapeutics® 2022: Critical Care Pharmacy Preparatory Review and Recertification Course

386
Toxicology

c. Norepinephrine continuous infusion initiated at 2–5 mcg/minute (0.1 mcg/kg/minute) and titrated
to target blood pressure in the setting of vasodilatory shock
d. In the presence of cardiogenic shock, epinephrine and dobutamine are the agents of choice.
Epinephrine continuous infusion at 1 mcg/minute (0.01–0.1 mcg/kg/minute) and titrated to target
hemodynamic parameters. Dobutamine continuous infusion at 2–20 mcg/kg/min and titrated to
target hemodynamic parameters. Dopamine is not recommended in this setting.
e. Transcutaneous or transvenous pacing or intra-aortic balloon pumps
6. Hyperinsulinemic euglycemic therapy (HIET)
a. Mechanism of action:
i. Insulin increases the plasma concentrations of ionized calcium, improves the hyperglycemic
acidotic state, improves the myocardial use of carbohydrates, and exerts an independent
inotropic effect (Am J Crit Care Med 2007;16:498-503).
ii. Dextrose prevents the development of hypoglycemia after insulin administration.
iii. Potassium prevents the development of hypokalemia after insulin administration.
iv. Onset of action is as soon as 5 minutes; however, it may take up to 30 minutes for full effects
to be seen.
b. HIET administration (Clin Toxicol 2011;49:277-83; Am J Health Syst Pharm 2006;63:1828-35):
i. If baseline glucose is less than 200 mg/dL, administer 50 mL of 50% dextrose in water before
insulin administration – May consider an infusion of 10%–20% dextrose to maintain a serum
glucose concentration greater than 100 mg/dL
ii. Correct hypokalemia before initiating insulin therapy. Maintain normokalemia with potassium
replacement during insulin infusion and after withdrawal.
iii. Bolus 1 unit/kg of regular insulin intravenously, followed by a continuous intravenous infusion
at 0.5–1 unit/kg/hour; increase rate every 10 minutes to a maximum of 10 units/kg/hour.
(a) HIET should be continued until existing vasopressor therapy is weaned and hemodynamic
stability is obtained. Then, HIET should be tapered over several hours while monitoring
for hemodynamic deterioration. Dextrose therapy likely must be continued after
discontinuation of HIET to prevent hypoglycemia.
c. Adverse effects: Hypoglycemia, hypomagnesemia, and hypokalemia
d. Monitoring:
i. Vital signs every 15–60 minutes with a goal mean arterial pressure greater than 65 mm Hg and
an HR greater than 50 beats/minute
ii. Serum glucose every 15 minutes; then every 30–60 minutes once stable to target serum
concentrations greater than 100 mg/dL
iii. Serum potassium every hour during the insulin infusion; then every 6 hours to maintain
concentrations above 2.8 mEq/L
iv. Monitor for fluid overload and iatrogenic hyponatremia. Utilization of more concentrated
dextrose infusions (e.g., 50% dextrose) and more concentrated insulin infusions (e.g., 16 units/
mL) may be necessary.
7. Intravenous lipid emulsion (J Emerg Med 2015;48:387-97; J Med Toxicol 2017;13:124-5)
a. Mechanism of action is not well known; however, it is thought to be owing to a combination of
binding lipid-soluble agents and the provision of free fatty acids that increase cardiac energy and
intracellular calcium.
b. Improves HR and may reduce mortality as an individual treatment or in combination with other
therapies.
c. Evidence is limited to animal models and human case reports, and its role in therapy is controversial.
d. Administration:
i. Bolus of 1.5 mL/kg of 20% lipid emulsion (Intralipid) over 2-3 minutes (typical dose is usually
100 mL)

ACCP Updates in Therapeutics® 2022: Critical Care Pharmacy Preparatory Review and Recertification Course

387
Toxicology

ii. A repeat bolus may be considered if no response after initial bolus


iii. The bolus may be followed immediately by an infusion of 20% lipid emulsion at a rate of 0.25
mL/kg/minute. After 3 minutes at this infusion rate, the infusion rate may be adjusted to 0.025
mL/kg/minute (i.e., 1/10 the initial rate) if there has been a significant response for a total
duration not to exceed 6 hours.
iv. BP, HR, and other available hemodynamic parameters should be recorded at least every 15
minutes during the infusion.
v. Adverse effects may include pancreatitis, jaundice, coagulopathies, interference with
laboratory results, and fat embolism.
vi. Drug interactions are not well known.

Patient Case

Questions 7 and 8 pertain to the following case.


A 52-year-old man is admitted to the ED with concerns about dizziness and headache. His vital signs are as
follows: temperature 98.9°F (37.2°C), BP 87/50 mm Hg, and HR 58 beats/minute. His wife reports that he has a
history of hypertension and that he was recently given a diagnosis of being in the early stages of Alzheimer dis-
ease. She has brought his medications with her; the 1-month supply was refilled 2 days ago: a bottle of diltiazem
CD 120 mg/day (7 tablets remaining) and a bottle of donepezil 5 mg once daily (28 tablets remaining).

7. Which decontamination strategy would provide the most benefit?


A. Charcoal 25 g every hour until his BP improves
B. Ipecac 30 mL, followed by 240 mL of water
C. Polyethylene glycol-electrolyte solution 1500 mL/hour until the rectal effluent is clear
D. Magnesium citrate 240 mL, followed by 240 mL of water

8. Which antidote would be best to administer first?


A. Calcium chloride 1 g intravenously over 1 minute
B. Glucagon 5 mg intravenously over 1 minute
C. Atropine 2 mg intravenously over 1 minute
D. Epinephrine 1 mg intravenously over 1 minute

XI.  DIGOXIN

A. Background
1. The cardiac glycosides accounted for 1138 single-agent toxic exposures and 27 deaths in 2019.
2. Mechanism of action is inhibition of the sodium-potassium adenosine triphosphatase pump and
suppression of the atrioventricular node.
3. Because of its narrow therapeutic index, toxicity has been reported in as many as 35% of patients
receiving digoxin (Postgrad Med 1993;69:337-9).
a. The normal therapeutic range is 0.8–2.1 ng/mL.
b. Toxicity may be related to an acute ingestion or may be an issue with chronic dosing or renal
dysfunction.

ACCP Updates in Therapeutics® 2022: Critical Care Pharmacy Preparatory Review and Recertification Course

388
Toxicology

4. Risk factors for digoxin toxicity include renal failure, advanced age, ischemic heart disease, left
ventricular dysfunction, electrolyte imbalances (hypokalemia, hypomagnesemia, hypercalcemia), and
hypothyroidism (Postgrad Med 1993;69:337-9).

B. Clinical Presentation
1. Cardiac effects associated with digoxin toxicity include second- and third-degree heart block,
tachyarrhythmias, and bradyarrhythmias. More specific examples include fascicular tachycardia,
ventricular bigeminy, and ventricular tachycardia (Am J Cardiol 1992;69:108G-119G).
2. Noncardiac effects associated with digoxin toxicity include nausea and vomiting, lethargy, headaches,
confusion, and visual disturbances.

C. Treatment
1. Consider decontamination strategies if patients present within 2 hours of overdose.
a. Multidose activated charcoal is beneficial because of the enterohepatic recirculation of digoxin. Load
50–100 g; then 10 g/hour, 10–20 g every 2 hours, or 40 g every 4 hours (Postgrad Med 1993;69:337-
9).
b. Colestipol or cholestyramine is an effective drug-binding alternative to charcoal, but it may not be
useful in acute toxicity (Am J Cardiol 1992;69:108G-119G).
c. Hemodialysis is not considered effective.
2. Correct serum electrolyte abnormalities.
a. Correct serum potassium concentration to a goal of 3.5–4 mEq/L.
b. Correct serum magnesium concentration to a goal of 1.5–2.2 mg/dL.
c. Correct serum calcium concentration to a goal of 8.5–10.5 mg/dL.
3. Treat symptomatic bradyarrhythmias with atropine 0.5 mg intravenously.
4. Digoxin immune antigen-binding fragments (Fab)
a. Antibodies that bind to digoxin molecules that are then renally excreted
b. Indications for use in acute intoxications (Crit Care Clin 2012;28:527-35):
i. Life-threatening arrhythmias: systole, ventricular fibrillation or tachycardia, complete heart
block, symptomatic bradycardia
ii. Evidence of end-organ damage (e.g., renal failure, altered mental status)
iii. Hyperkalemia (greater than 5–5.5 mEq/L)
c. Products:
i. Digibind (Digoxin Immune Fab): 38 mg per vial
ii. DigiFab (Digoxin Immune Fab): 40 mg per vial
d. Dosing:
i. If amount is unknown: 10–20 vials for acute toxicity or 6 vials for chronic toxicity
ii. If the amount of digoxin ingested is known:
Dose (vials) = total body load (0.8 × mg of digoxin ingested)/0.5
iii. If digoxin concentration is known:
Dose (vials) = [serum digoxin concentration (ng/mL) × weight (kg)]/100
iv. May consider lower doses of 1 or 2 vials (40–80 mg) for acute ingestions with repeat doses if
necessary
e. Adverse effects include heart failure exacerbation, atrial fibrillation, orthostatic hypotension,
hypokalemia, and phlebitis.

ACCP Updates in Therapeutics® 2022: Critical Care Pharmacy Preparatory Review and Recertification Course

389
Toxicology

D. Monitoring
1. Monitor vital signs every 30–60 minutes initially. Goal HR of greater than 60 beats/minute and
asymptomatic
2. Monitor serum potassium concentrations hourly for at least the first 6 hours.
3. Additional serum digoxin concentrations are not recommended after the administration of Fab. A
rapid rise in serum concentrations is expected because of the mechanism of the Fab-digoxin complex.
Repeat serum digoxin concentrations may be checked 24 hours after the initial treatment if Fab is not
administered.

XII.  ANTIDEPRESSANTS

A. Background
1. Antidepressants accounted for more than 130,000 toxic exposures and 44 deaths in 2019.
2. The most common agents involved in toxic exposures were the selective serotonin reuptake inhibitors
(SSRIs) and the tricyclic antidepressants (TCAs).
3. SSRIs block the reuptake of serotonin at the presynaptic neuron.
4. Patients with SSRI overdoses are often asymptomatic with self-limiting effects (Emerg Med Clin North
Am 2007;25:477-97). The most common adverse effects may include drowsiness, tremor, altered mental
status, nausea and vomiting, tachycardia, hypotension, seizures, and QRS- or QT-interval prolongation.
5. TCAs exert many effects, including blocking the reuptake of norepinephrine and serotonin at the
presynaptic neuron, blocking muscarinic cholinergic receptors, blocking antihistamine effect, blocking
the sodium channel, and, to a lesser degree, blocking α-adrenergic receptors.
6. Individuals with TCA overdoses may present with the following adverse effects (Emerg Med Clin
North Am 1994;12:533-47):
a. Cardiovascular: Hypo- or hypertension, tachy- or bradycardia, increased QRS or QT interval,
atrioventricular-conduction block, complete heart block
b. Respiratory: Hypoventilation, crackles, hypoxia
c. Neurologic: Delirium, lethargy, seizures, coma
d. Other: Hyperthermia, dry mucous membranes, urinary retention, blurred vision

B. Treatment
1. There are no specific antidotes for antidepressant overdoses; general supportive care is recommended,
with a focus on ABC.
2. Gastric decontamination is not typically recommended; however, single-dose activated charcoal may
be administered within the first hour of exposure (Emerg Med Clin North Am 2000;18:637-54).
3. Administer crystalloid or colloid fluids to maintain BP and HR, with the goal of a mean arterial pressure
>65 mmHg, an SBP greater than 90 mm Hg, and a HR greater than 60 beats/minute.
a. Norepinephrine or epinephrine may be used if fluid resuscitation alone is unsuccessful.
b. Dopamine may not be an effective agent because endogenous norepinephrine stores are depleted
in an overdose.
4. Sodium channel blockade
a. Alkalinization of blood to a pH of 7.45–7.55 is recommended for the TCAs to resolve metabolic
acidosis and improve cardiac symptoms. Requires frequent monitoring of arterial pH (varies by
effect, but as often as every 15–30 minutes). Sodium load may overcome TCA blockade of sodium
channels by increasing the electrochemical gradient.

ACCP Updates in Therapeutics® 2022: Critical Care Pharmacy Preparatory Review and Recertification Course

390
Toxicology

b. Administer sodium bicarbonate


i. Recommended bolus dose of 1 mEq/kg (minimum: 50 mEq)
ii. May repeat bolus every 15 minutes until ECG stabilized or arterial pH goal achieved.
iii. May consider a continuous infusion of hypertonic saline in patients with refractory cardiac
conduction despite optimal serum alkalinization by increasing sodium load.
c. Proposed indications for sodium bicarbonate include the following (Chest 2008;133:1006-13):
i. QRS greater than 100–120 milliseconds
ii. Wide complex tachycardia
iii. Cardiac arrest
iv. Right bundle-branch block
v. Refractory hypotension
d. Replace serum electrolytes, especially magnesium and potassium, if QT prolongation is present.
Magnesium replacement is recommended for membrane stabilization in patients with a QTc greater
than 500 milliseconds and normal serum magnesium concentrations.
e. Seizures should be managed with benzodiazepines. Phenobarbital and propofol may be considered
if the patient is refractory to benzodiazepines and has a stable BP.
i. Phenytoin and lacosamide are not recommended because of their effects on sodium channels
and cardiac adverse effects, respectively.
f. Intravenous fat emulsion
i. Many case reports for use in amitriptyline and SSRI overdose
ii. Administration (use lean body mass):
(a) Bolus of 1.5 mL/kg of 20% lipid emulsion (Intralipid) over 1–5 minutes (typical dose is
usually 100 mL for a 70 kg patient)
(b) May repeat up to two times for persistent cardiovascular collapse
(c) Intravenous infusion of 0.25–0.5 mL/kg/minute for 60 minutes (typical dose is 18 mL/
minute for a 70 kg patient)
(d) Continue for up to 10 minutes after cardiovascular recovery.

C. Monitoring – Patients should be monitored for clinical improvement for at least 6–8 hours and for a minimum
of 24 hours for more severe adverse effects or with citalopram or escitalopram (because of the longer half-
lives of these agents).
a. Monitor for cardiac toxicity with a 12-lead ECG, CK-MB, troponins, BP, and HR.
b. Monitor for signs and symptoms of respiratory depression with RR and pulse oximetry.

D. Serotonin Syndrome
1. Excessive serotonin concentrations lead to overstimulation of serotonin-1A and serotonin-2A receptors
in the central and peripheral nervous systems (Emerg Med Clin North Am 2007;25:477-97).
2. Adverse effects include altered mental status, autonomic instability (hyperthermia, tachycardia,
hypertension, arrhythmias), and neuromuscular changes (hyperreflexia, increased rigidity).
3. Diagnosis is made according to clinical findings; many clinicians support the use of the Hunter Serotonin
Toxicity Criteria (QJM 2003;96:635-42). By this method, patients are likely to have serotonin toxicity if
they have taken a serotonergic agent and one of the following criteria are present:
a. Spontaneous clonus
b. Inducible clonus PLUS agitation or diaphoresis
c. Ocular clonus PLUS agitation or diaphoresis
d. Tremor PLUS hyperreflexia
e. Hypertonia PLUS temperature above 100.4°F (38°C) PLUS ocular clonus or inducible clonus

ACCP Updates in Therapeutics® 2022: Critical Care Pharmacy Preparatory Review and Recertification Course

391
Toxicology

4. Treatment should focus on supportive care with intravenous fluids; symptoms typically resolve within
24–48 hours.
a. Discontinue the offending agent.
b. Benzodiazepines should be administered as first line for agitation and muscle rigidity.
c. Cyproheptadine is a histamine-1 receptor antagonist and nonspecific serotonin receptor antagonist.
A single dose of 8–12 mg by mouth should be administered for agitation and muscle rigidity as an
adjunct to benzodiazepines. A second dose may be administered in 6–8 hours if symptoms persist.
d. If condition worsens, may require intubation with continuous infusion benzodiazepines
e. Although not well studied, case reports have shown the efficacy of dexmedetomidine at doses of
0.05–0.8 mcg/kg/hour (in pediatric patients).

Patient Case

9. A 21-year-old man is admitted to the ED after taking 30 citalopram 20-mg tablets about 2 hours ago. His
vital signs are as follows: BP 125/85 mm Hg, HR 77 beats/minute, RR 15 breaths/minute, and temperature
98.7°F (37.1°C). Which is the best intervention for this patient?
A. Administer lorazepam 2 mg intravenously to prevent seizure activity.
B. Administer cyproheptadine 8 mg by mouth to prevent muscle rigidity.
C. Recommend a cooling blanket to prevent serotonin syndrome–related hyperthermia.
D. Order a 12-lead ECG to monitor for cardiac conduction disturbances.

XIII.  ATYPICAL ANTIPSYCHOTICS

A. Background
1. The atypical antipsychotic agents accounted for about 18,011 toxic exposures and 13 deaths in 2019.
2. These agents are classified primarily as having D2-dopaminergic receptor and serotonin-2A receptor
antagonism. Additional effects include antagonism of the α1- and histamine-1 receptors.
3. Adverse effects associated with the atypical antipsychotics are typically self-limiting.
a. More severe adverse effects may include CNS depression, tachycardia, hypotension, and QT
prolongation.
b. Less severe adverse effects include dizziness, drowsiness, miosis, blurred vision, urinary retention,
and CNS excitation.

B. Treatment
1. There are no specific antidotes for the atypical antipsychotics; general supportive care is recommended,
focusing on ABC.
2. Gastric decontamination is not typically recommended; however, single-dose activated charcoal may be
administered within the first hour of exposure if no contraindications exist (J Emerg Med 2012;43:906-13).
3. Administer crystalloid to maintain BP with a goal of a mean arterial blood pressure >65 mm Hg, an
SBP greater than 90 mm Hg, and an HR greater than 60 beats/minute.
a. Consider vasopressors if fluid resuscitation is inadequate.
b. Because of the α-receptor antagonist activity of these agents, norepinephrine or phenylephrine is
preferred if vasopressors are needed.
4. Administer sodium bicarbonate if QRS prolongation (quetiapine overdose only)

ACCP Updates in Therapeutics® 2022: Critical Care Pharmacy Preparatory Review and Recertification Course

392
Toxicology

5. Replace serum electrolytes, especially magnesium and potassium, if QT prolongation. Magnesium


replacement is recommended for membrane stabilization in patients with a QTc greater than 500
milliseconds and normal serum magnesium concentrations.
6. Seizure activity should be managed with benzodiazepines, barbiturates, or propofol.
7. Anticholinergic symptoms can potentially be treated with IV physostigmine in select patients (e.g.,
normal ECG); however, substantial controversy exists with this approach, particularly in the setting of
a concomitant TCA overdose and its overall functional use considering its short half-life (Ann Emerg
Med 2000;35:374-81). Currently, it is not frequently used and further investigations into its safety and
efficacy are necessary (J Med Toxicol 2015;11:179-84).
8. Lipid emulsion therapy may be effective because of the high lipophilicity of these agents and may be
considered if more traditional treatment means do not improve the cardiovascular complications of
decreased HR and/or BP (J Emerg Med 2012;43:906-13). Administer an intravenous bolus dose of 1.5
mL/kg of 20% intralipid over 2–3 minutes, followed by a continuous intravenous infusion of 0.25–0.5
mL/kg/min for 60 minutes, if necessary.

C. Monitoring: Patients should be monitored for clinical improvement for at least 8–12 hours.
1. Monitor for cardiac toxicity with a 12-lead ECG, CK-MB, and troponins.
2. Monitor for respiratory depression with RR and pulse oximetry.

XIV.  LITHIUM

A. Background
1. Lithium was associated with almost 4000 toxic exposures and four deaths in 2019.
2. Mechanism of action is through an influence on serotonin and norepinephrine reuptake, inhibition of
the phosphatidylinositol cycle, and inhibition of the post-synaptic D2 receptor.
3. Adverse effects associated with lithium include:
a. Acute overdose:
i. GI: Nausea, vomiting, diarrhea
ii. CNS: Confusion, tremor, myoclonus, seizures, coma
iii. Cardiovascular: T-wave inversion, ventricular arrhythmias
b. Chronic adverse effects:
i. Endocrine: Hypothyroidism, myxedema coma
ii. Nephrogenic diabetes insipidus

B. Treatment
1. There are no specific antidotes for lithium; general supportive care is recommended, focusing on ABC.
2. Gastric decontamination is not typically recommended in toxic acute ingestions. Single-dose activated
charcoal is not effective for lithium overdoses; whole bowel irrigation may be beneficial.
3. Administer crystalloid to maintain BP, with a goal of an SBP greater than 90 mm Hg. Consider
vasopressors if fluid resuscitation is not adequate.
4. Replace serum electrolytes, especially magnesium and potassium, if QT prolongation.
5. Seizure activity should be managed with benzodiazepines, barbiturates, or propofol.
6. Lithium overdoses are primarily managed with hemodialysis or continuous renal replacement therapy.
a. Saline infusions may be administered if there are no contraindications to fluid therapy (goal is a
serum sodium concentration of 140–145 mEq/L). Lithium clearance is reduced in hyponatremia.
b. Intermittent hemodialysis may require several sessions to fully remove lithium concentrations
because of the rebound of lithium concentrations that occurs after dialysis sessions. Consider
continuous renal replacement therapy (CRRT) for hemodynamically unstable patients.

ACCP Updates in Therapeutics® 2022: Critical Care Pharmacy Preparatory Review and Recertification Course

393
Toxicology

c. Suggested indications for the use of hemodialysis include (Chest 2008;133:1006-13):


i. Severe toxicity (severe altered mental status or seizures)
ii. Renal failure (cannot eliminate lithium)
iii. Lithium concentrations greater than 2.5 mmol/L in chronic exposures
iv. Lithium concentrations greater than 4 mmol/L in acute exposures

C. Monitoring – Patients should be monitored for clinical improvement for at least 8–12 hours.
1. Monitor for cardiac toxicity with a 12-lead ECG, CK-MB, and troponins.
2. Monitor for respiratory depression with RR and pulse oximetry.
3. Monitor renal function with urine output, BUN, and SCr.
4. Monitor baseline lithium concentrations and then every 6 hours after until concentrations have
decreased to less than 1.5 mmol/L (normal 0.6–1.2 mmol/L).

Patient Case

10. A 24-year-old woman is brought to the ED by her roommate. She has been in a normal state of health, but
the roommate is concerned because she “seems really out of it.” According to the roommate, the patient had
an appointment with the physician today, and she had been given a prescription to refill olanzapine 5 mg by
mouth daily, but the bottle is empty. On physical examination, she is alert and oriented person, place and
time but she dozes off several times. Her vital signs are stable, and a 12-lead ECG shows sinus tachycardia.
Which intervention is most appropriate for this patient?
A. Lactated Ringer solution 500 mL intravenously
B. 8.4% sodium bicarbonate 50 mL intravenously
C. Lorazepam 2 mg intravenously
D. Clinical monitoring for 6 hours

XV.  ORAL HYPOGLYCEMICS

A. Background
1. Oral hypoglycemics accounted for 6173 single-agent exposures and 25 deaths in 2019.
2. The most common oral hypoglycemic involved in toxic exposures was metformin, followed by the
sulfonylureas and the glucagon-like peptide-1 (GLP-1) receptor agonists.
3. The most serious adverse effects were reported with the sulfonylureas; however, most fatalities were
associated with metformin.

B. Clinical Presentation
1. Clinical signs and symptoms include hypoglycemia (not with metformin), nausea, vomiting, dizziness,
tachycardia, and diaphoresis.
2. More severe adverse effects include seizures, palpitations, tachyarrhythmias, electrolyte abnormalities,
and metabolic (lactic) acidosis.

C. Treatment
1. Stabilization of the ABC
2. Identifying the causative agent is important because specific treatment will vary by the agent involved.

ACCP Updates in Therapeutics® 2022: Critical Care Pharmacy Preparatory Review and Recertification Course

394
Toxicology

3. Consider gastric decontamination with single-dose activated charcoal if patients present within 1 hour
of overdose.
4. Observe clinically asymptomatic patients for a minimum of 8 hours (Am J Health Syst Pharm
2006;63:929-38).
5. For symptomatic patients or blood glucose less than 70 mg/dL, treat with glucose:
a. Conscious patients: Administer 8 oz of an oral carbohydrate (such as juice, non-diet sodas, or milk)
or oral glucose tablets or gels.
b. Unconscious patients: Administer intravenous dextrose, 0.5–1 g/kg
c. Repeat doses may be required; consider a continuous infusion of dextrose if needed. Glucose
concentrations should be monitored often (every 15–60 minutes) until stable.
d. Use caution to avoid overcorrection of serum glucose.
6. Octreotide
a. Mechanism of action is a somatostatin analog that inhibits the secretion of insulin.
b. Primarily studied in sulfonylurea overdose, but considered a treatment option for all oral
hypoglycemic toxic exposures
c. Administer 50 mcg subcutaneous or intravenous, followed by 50-mcg doses every 6 hours.
Intravenous dextrose infusion should be slowly tapered off.
d. Adverse effects include headache, dizziness, nausea, abdominal pain, and sinus bradycardia.
7. Sodium bicarbonate
a. Indicated for severe metformin-associated lactic acidosis
b. 1–2 mEq/kg or 50–200 mEq of 8.4% sodium bicarbonate intravenously
8. Glucagon
a. Mechanism of action is stimulation of gluconeogenesis.
b. May trigger additional insulin secretion, leading to a secondary hypoglycemia
c. May provide benefit in prehospital settings when oral or intravenous options are not available, but
is not routinely recommended
d. Not recommended in pediatric patients, in malnourished patients, or for sulfonylurea toxic
exposures
9. Hemodialysis or continuous renal replacement therapy may be necessary to enhance metformin
clearance in severe cases.

D. Monitoring
1. Regular assessment of vital signs and mental status (Emerg Med J 2006;23:565-7)
2. Measure capillary blood glucose at a minimum of every hour for 24 hours with a goal of greater than
70 mg/dL.
3. Measure BP hourly, especially after octreotide administration.

Patient Case

11. As the pharmacist in the ICU satellite, you receive a call from a distressed nurse about a patient in the
cardiac step-down unit. The patient was found unconscious, and, on investigation, it was discovered that
he had received a glyburide 20-mg tablet 1 hour earlier that was meant for another patient. The patient has
stable vital signs, but his point-of-care blood glucose concentration is 37 mg/dL. Which intervention is most
appropriate at this time?
A. 8 oz of milk by mouth
B. 50 mL of 50% dextrose in water intravenously
C. Octreotide 100 mcg subcutaneously
D. Glucagon 1 mg intramuscularly

ACCP Updates in Therapeutics® 2022: Critical Care Pharmacy Preparatory Review and Recertification Course

395
Toxicology

XVI.  DRUGS OF ABUSE

A. Background
1. Miscellaneous stimulants and street drugs accounted for 66,224 toxic exposures and 108 deaths in 2019.
2. The most common drugs of abuse involved in toxic exposures were amphetamines, marijuana (and
derivatives), cocaine, methamphetamines, and heroin.
3. Total numbers are difficult to determine because few of these agents can be detected with current
techniques.

B. Amphetamines/Methamphetamines/MDMA (ecstasy)
1. Mechanism of action/toxicity: stimulation of the CNS, peripheral release of catecholamines, inhibition
of catecholamine reuptake, inhibition of monoamine oxidase (Intensive Care Med 2004;30:1526-36)
2. Clinical presentation (Intensive Care Med 2004;30:1526-36)
a. Common: Confusion, tremor, anxiety, agitation, irritability, mydriasis, tachyarrhythmias
b. Severe: Hepatocellular necrosis, acute hepatitis, myocardial ischemia, hypertension, cerebral
hemorrhage, seizures, hyponatremia
3. Treatment
a. Mostly supportive care with intravenous fluid administration and airway maintenance
b. Gastric lavage or activated charcoal if within 1 hour of ingestion
c. BP control with benzodiazepines, α-blocker, or vasodilator (e.g., nitroglycerin, nitroprusside,
nicardipine). Avoid the use of β-receptor blocking agents because of unopposed α-receptor activity
leading to an increase in BP.
d. Benzodiazepines for agitation, anxiety, psychosis, and/or uncontrolled hypertension titrated to
effect. Haloperidol or phenothiazines may be considered for use (with caution) in patients with
primary psychiatric disorders or dopamine-mediated movement disorders.
e. Cooling therapy if hyperthermia
f. Monitor for serotonin syndrome.

C. Synthetic Cannabinoids: K2/Spice


1. Mechanism of action/toxicity: stimulation of the cannabinoid-1 (CB1) and cannabinoid-2 (CB2)
receptors (Pharmacotherapy 2015;35:189-97)
a. CB1 receptors modulate glutamate and GABA and are found both peripherally and centrally.
b. CB2 receptors are located in immune tissue and the CNS and modulate pain and emesis.
2. Clinical presentation (J Pharm Pract 2015;28:50-65)
a. Predominantly euphoria or excited delirium
b. Common adverse effects include:
i. Psychiatric: Agitation, anxiety, hallucinations, paranoia, catatonia
ii. Neurologic: Cognitive impairment, ataxia, dizziness, headache, seizures
iii. Cardiovascular: Tachycardia, palpitations, hypertension
iv. GI: Nausea, vomiting
v. Renal: Acute kidney injury, rhabdomyolysis
3. Treatment
a. Mostly supportive care with intravenous fluid administration
b. Benzodiazepines for agitation or seizure activity. Antipsychotics for agitation may be considered
for use (with caution) in patients with primary psychiatric disorders or dopamine-mediated
movement disorders.

D. Cocaine
1. Mechanism of action/toxicity (Intensive Care Med 2004;30:1526-36):
a. Potent sympathetic nervous system stimulant

ACCP Updates in Therapeutics® 2022: Critical Care Pharmacy Preparatory Review and Recertification Course

396
Toxicology

b. Inhibits the presynaptic reuptake of epinephrine and norepinephrine; also stimulates norepinephrine
release
c. Acts as a reuptake inhibitor of dopamine, norepinephrine, and serotonin
2. Clinical presentation (Intensive Care Med 2004;30:1526-36)
a. Predominantly euphoria or excited delirium
b. Common/severe adverse effects include:
i. Psychiatric: Agitation, anxiety, psychosis, delirium
ii. Neurologic: Stroke, subarachnoid or intracranial hemorrhage, seizures
iii. Cardiovascular: Tachycardia, hypertension, palpitations, arrhythmias, heart failure, aortic
dissection
iv. GI: Gastric ulcers, gastric perforation, bowel ischemia
v. Respiratory: Status asthmaticus, pulmonary hypertension, pulmonary edema, alveolar
hemorrhage
vi. Renal: Acute kidney injury, rhabdomyolysis
3. Treatment
a. Mostly supportive care with intravenous fluid administration
b. Gastric decontamination is typically not recommended. Activated charcoal may provide benefit if
given within 1 hour of oral ingestion.
c. Benzodiazepines are administered for agitation or seizure activity and titrated to relaxation.
Because of their ability to block the CNS stimulant effect of cocaine, benzodiazepines are also an
effective treatment for hypertension and tachycardia.
d. BP control with benzodiazepines, α-blocker, or vasodilator (e.g., nitroglycerin, nitroprusside,
nicardipine). Avoid the use of β-receptor blocking agents because of unopposed α-receptor activity
leading to an increase in BP.

E. Synthetic Cathinones: Bath Salts


1. Mechanism of action/toxicity: increases presynaptic concentrations of serotonin, dopamine, and
norepinephrine by stimulating release and antagonizing monoamine reuptake (Pharmacotherapy
2014;34:745-57)
2. Clinical presentation (J Pharm Pract 2015;28:50-65):
a. Predominantly euphoria, increased energy, and alertness
b. Common adverse effects include:
i. Psychiatric: Agitation, aggression, anxiety, hallucinations, paranoia
ii. Neurologic: Amnesia, confusion, insomnia, seizures, dizziness, headache
iii. Cardiovascular: Angina, hypertension, tachycardia, palpitations
iv. GI: Abdominal pain, nausea, vomiting, anorexia, hepatic failure
v. Renal: Acute kidney injury, increased SCr, and rhabdomyolysis
3. Treatment
a. Mostly supportive care with intravenous fluid administration
b. Benzodiazepines or antipsychotics for agitation or anxiety
i. Haloperidol is an alternative option, but it should be monitored for worsening hyperthermia.
ii. Consider propofol or dexmedetomidine for severe symptoms.
c. Antiemetics for severe nausea and vomiting
d. Monitor for serotonin syndrome.

F. Piperazines
1. Mechanism of action/toxicity: enhances neurotransmitter release and reuptake inhibition of dopamine,
serotonin, and norepinephrine release (Emerg Med Clin North Am 2014;32:1-28)
2. Clinical presentation (J Pharm Pract 2015;28:50-65):
a. Predominantly euphoria and increased energy

ACCP Updates in Therapeutics® 2022: Critical Care Pharmacy Preparatory Review and Recertification Course

397
Toxicology

b. Common adverse effects include:


i. Psychiatric: Agitation, anxiety, hallucinations, psychosis, depressed mood or mood swings,
paranoia
ii. Neurologic: Confusion, insomnia, tremor, seizures, dizziness, headache
iii. Cardiovascular: Angina, hypertension, tachycardia, palpitations, QT prolongation
iv. GI: Abdominal pain, nausea, vomiting
v. Renal: Urinary retention
3. Treatment
a. Mostly supportive care with intravenous fluid administration
b. Benzodiazepines for agitation or seizures
c. Avoid antipsychotics because of worsening hyperthermia, extrapyramidal effects, and hypotension
or arrhythmias.
d. Treat hypertension with parenteral antihypertensives or clonidine.
e. Hyperthermia treatment if above 104°F (40°C)
f. Monitor for serotonin syndrome.

G. Ketamine
1. Mechanism of action/toxicity (Lancet 2005;365:2137-45):
a. Noncompetitive NMDA receptor antagonist (blocks glutamate and aspartate)
b. Mild to moderate blockade of catecholamine reuptake
2. Clinical presentation:
a. Predominantly hallucinations and vivid dreams
b. Common adverse effects include:
i. Psychiatric: Impaired memory, cognitive dysfunction, severe agitation
ii. Cardiovascular: Hypertension, tachycardia, cardiac arrhythmias
iii. Respiratory: Laryngospasm, apnea, respiratory depression
iv. GI: Anorexia, nausea, vomiting
v. Genitourinary: Cystitis, irritable bladder, urethritis
3. Treatment
a. Mostly supportive care with intravenous fluid administration
i. Monitor for rhabdomyolysis
ii. Aspiration precautions are recommended in comatose patients
iii. Urinalysis and serum chemistries if symptomatic for cystitis
b. Activated charcoal may provide benefit if given within 1 hour of oral ingestion. Additional doses
every 4 hours may be considered.
c. Benzodiazepines are recommended for agitation or seizures.
d. Haloperidol may be used if benzodiazepines are not effective. Monitor closely because of the
potential of lowering the seizure threshold and worsening of dystonia, hypotension, neuroleptic
malignant syndrome, and/or myoglobinuria.

ACCP Updates in Therapeutics® 2022: Critical Care Pharmacy Preparatory Review and Recertification Course

398
Toxicology

REFERENCES

General Decontamination 14. Thanacoody R, Caravati EM, Troutman B, et al.


1. Benson BE, Hoppu K, Troutman WG, et al. Position Position paper update: whole bowel irrigation
paper update: gastric lavage for gastrointestinal for gastrointestinal decontamination of overdose
decontamination. Clin Toxicol 2013;51:140-6. patients. Clin Toxicol 2015;53:5-12.
2. Bledsoe BE. No more coma cocktails. Using sci- 15. Wu AH, Gerona R, Armenian P, et al. Role of
ence to dispel myths & improve patient care. liquid chromatography-high-resolution mass spec-
JEMS 2002;27:54-60. trometry (LC-HR/MS) in clinical toxicology. Clin
3. Brahm NC, Yeager LL, Fox MD, et al. Commonly Toxicol (Phila) 2012;50:733-42.
prescribed medications and potential false-posi-
tive urine drug screens. Am J Health Syst Pharm Acetaminophen
2010;67:1344-50. 1. Bailey B, McGuigan MA. Management of anaphy-
4. Chyka PA, Seger D, Krenzelok EP, et al. Position lactoid reactions to intravenous N-acetylcysteine.
paper: single-dose activated charcoal. Clin Toxicol Ann Emerg Med 1998;31:710-5.
2005;43:61-87. 2. Baum RA, Dugan A, Metts E, et al. Modified two
5. Gummin DD, Mowry JB, Beuhler MC, et al. step N-acetylcysteine dosing regimen for the treat-
2019 annual report of the American Association ment of acetaminophen overdose a safe alternative.
of Poison Control Centers’ National Poison Data Toxicol Commun 2018;2:81-4.
System (BPDS): 37th annual report. Clin Toxicol 3. Bunchorntavakul C, Reddy KR. Acetaminophen-
2020;58:1360-541. related hepatotoxicity. Clin Liver Dis 2013;
6. Höjer J, Troutman WG, Hoppu K, et al. Position 17:587-607.
paper update: ipecac syrup for gastrointestinal 4. Dart RC, Erdman AR, Olson KR, et al.
decontamination. Clin Toxicol 2013;51:134-9. Acetaminophen poisoning: an evidence-based
7. Holstege CP, Borek HA. Toxidromes. Crit Care consensus guideline for out-of-hospital manage-
Clin 2012;28:180-98. ment. Clin Toxicol 2006;44:1-18.
8. Levine M, Brooks DE, Truitt CA, et al. Toxicology 5. Dart RC, Rumack BH. Patient-tailored acet-
in the ICU. Part I: general overview and approach ylcysteine administration. Ann Emerg Med
to treatment. Chest 2011;140:795-806. 2007;50:280-1.
9. Manoguerra AS, Cobaugh DJ. Guideline on the use 6. Hayes BD, Klein-Schwartz W, Doyon S.
of ipecac syrup in the out-of-hospital management Frequency of medication errors with intravenous
of ingested poisons. Clin Toxicol 2005;43:1-10. acetylcysteine for acetaminophen overdose. Ann
10. Pietrzak MP. Clinical policy for the initial approach Pharmacother 2008;42:766-70.
to patients presenting with acute toxic ingestion or 7. Radosevich JJ, Patanwala AE, Erstad BL.
dermal or inhalation exposure. Ann Emerg Med Hepatotoxicity in obese versus nonobese patients
1999;33:735-56. with acetaminophen poisoning who are treated
11. Position paper: cathartics. American Academy of with intravenous N-acetylcysteine. Am J Ther
Clinical Toxicology and European Association 2016;23:e714-9.
of Poisons Centres and Clinical Toxicologists. J 8. Varney SM, Buchanan JA, Kokko J, et al.
Toxicol Clin Toxicol 2004;42:243-53. Acetylcysteine for acetaminophen overdose
12. Position statement and practice guidelines on the in patients who weigh > 100kg. Am J Ther
use of multi-dose activated charcoal in the treat- 2014;21:159-63.
ment of acute poisoning. American Academy of 9. Wolf SJ, Heard K, Sloan EP, Jagoda AS. Clinical
Clinical Toxicology and European Association policy: critical issues in the management of
of Poisons Centres and Clinical Toxicologists. J patients presenting to the emergency department
Toxicol Clin Toxicol 1999;37:731-51. with acetaminophen overdose. Ann Emerg Med
13. Proudfoot AT, Krenzelok EP, Vale JA. Position 2007;50:292-313.
paper on urine alkalinization. J Toxicol Clin
Toxicol 2004;42:1-26.

ACCP Updates in Therapeutics® 2022: Critical Care Pharmacy Preparatory Review and Recertification Course

399
Toxicology

Salicylates 5. Wu PE, Juurlink DN. Clinical review: lop-


1. Bora K, Aaron C. Pitfalls in salicylate toxicity. Am eramide toxicity. Ann Emerg Med 2017; pii:
J Emerg Med 2010;28:383-4. S0196-0644(17)30424-9.
2. Chyka PA, Erdman AR, Christianson G, et al.
Salicylate poisoning: an evidence-based consen- Alcohols
sus guideline for out-of-hospital management. 1. Barceloux DG, Bond GR, Krenzelok EP, et al.
Clin Toxicol 2007;45:95-131. American Academy of Clinical Toxicology prac-
3. O’Malley GF. Emergency department manage- tice guidelines on the treatment of ethylene glycol
ment of the salicylate-poisoned patient. Emerg poisoning. J Toxicol Clin Toxicol 1999;37:537-60.
Med Clin North Am 2007;25:333-46. 2. Barceloux DG, Bond GR, Krenzelok EP, et al.
4. Pearlman BL, Gambhir R. Salicylate intoxication: American Academy of Clinical Toxicology prac-
a clinical review. Postgrad Med 2009;121:162-8. tice guidelines on the treatment of methanol
5. Sibert JR, Craft AW, Jackson, RH. Child-resistant poisoning. J Toxicol Clin Toxicol 2002;40:415-46.
packaging and accidental child poisoning. Lancet 3. Brent J. Fomepizole for ethylene glycol and metha-
1977;2:289-90. nol poisoning. N Engl J Med 2009;360:2216-23.
4. Kruse JA. Methanol and ethylene glycol intoxica-
Opioids tion. Crit Care Clin 2102;28:661-771.
1. Boyer EW. Management of opioid analgesic over- 5. Lepik KJ, Levy AR, Sobolev BG, et al. Adverse
dose. N Engl J Med 2012;367:146-55. drug events associated with the antidotes for
2. Chyka PA. eChapter 10. Clinical toxicology. methanol and ethylene glycol poisoning: a com-
In: DiPiro JT, Talbert RL, Yee GC, et al., eds. parison of ethanol and fomepizole. Ann Emerg
Pharmacotherapy: A Pathophysiologic Approach, Med 2009;53:439-50.
9e. New York: McGraw-Hill, 2014. Available at 6. Marraffa J, Forrest A, Grant W, et al. Oral admin-
http://accesspharmacy.mhmedical.com/content. istration of fomepizole produces similar blood
aspx?bookid=689&sectionid=48811435. Accessed levels as identical intravenous dose. Clin Toxicol
October 30, 2016. 2008;46:181-6.
3. Goldfrank L, Weisman RS, Errick JK, et al. A dos-
ing nomogram for continuous infusion naloxone. Alcohol Withdrawal
Ann Emerg Med 1986;15:566-70. 1. Awissi DK, Lebrun G, Fagnan M, et al. Alcohol,
4. Mills CA, Flacke JW, Miller JD, et al. nicotine, and iatrogenic withdrawals in the ICU.
Cardiovascular effects of fentanyl reversal by nal- Crit Care Med 2013;41:S57-S68.
oxone at varying arterial carbon dioxide tensions 2. Dixit D, Endicott J, Burry L, et al. Management of
in dogs. Anesth Analg 1988;67:730-6. acute alcohol withdrawal syndrome in critically ill
patients. Pharmacotherapy 2016;36:797-822.
Loperamide 3. Flannery AH, Adkins DA, Cook AM. Unpeeling
1. Borron SW, Watts SH, Tull J, et al. Intentional mis- the evidence for the banana bag: evidence-based
use and abuse of loperamide: a new look at a drug recommendations for the management of alcohol-
with “low abuse potential.” J Emerg Med 2017; pii: associated vitamin and electrolyte deficiencies int
S0736-4679(17)30230-5. the ICU. Crit Care Med 2016;44:1545-52.
2. Eggleston W, Clark KH, Marraffa JM. Loperamide 4. Sarff M, Gold JA. Alcohol withdrawal syn-
abuse associated with cardiac dysrhythmia and dromes in the intensive care unit. Crit Care Med
death. Ann Emerg Med 2017;69:83-6. 2010;38(suppl):S494-S501.
3. Lasoff DR, Koh CH, Corbett B, et al. Loperamide 5. Schuckit MA. Recognition and management of
trends in abuse and misuse over 13 years: 2002- withdrawal delirium (delirium tremens). N Engl J
2015. Pharmacotherapy 2017;37:249-53. Med 2014;371:2109-13.
4. Vakkalanka JP, Charlton NP, Holstege CP. 6. Sen S, Grgurich P, Tulolo A, et al. A symptom-trig-
Epidemiologic trends in loperamide abuse and gered benzodiazepine utilizing SAS and CIWA-Ar
misuse. Ann Emerg Med 2017;69:73-8. scoring for the treatment of alcohol withdrawal

ACCP Updates in Therapeutics® 2022: Critical Care Pharmacy Preparatory Review and Recertification Course

400
Toxicology

syndrome in the critically ill. Ann Pharmacother 12. St-Onge M, Dube PA, Gosselin S, et al. Treatment
2017;51:101-10. of calcium channel blocker poisoning: a systematic
7. Stehman CR, Mycyk MB. A rational approach to review. Clin Toxicol 2014;52:926-44.
the treatment of alcohol withdrawal in the ED. Am 13. Turner-Lawrence DE, Kerns W II. Intravenous fat
J Emerg Med 2013;31:734-42. emulsion: a potential novel antidote. J Med Toxicol
2008;4:109-14.
Cardiovascular Agents 14. Wax PM, Erdman AR, Chyka PA, et al. β-Blocker
1. ACMT Position Statement: guidance for the use ingestion: an evidence-based consensus guide-
of intravenous lipid emulsion. J Med Toxicol line for out-of-hospital management. Clin Toxicol
2017;13:124-5. 2005;43:131-46.
2. Cao D, Heard K, Foran M, et al. Intravenous lipid
emulsion in the emergency department: a sys- Digoxin
tematic review of recent literature. J Emerg Med 1. Chan BSH, Buckley NA. Digoxin-specific anti-
2015;48:387-97. body fragments in the treatment of digoxin
3. Engbresten KM, Kaczmarek KM, Morgan J, et toxicity. Clin Toxicol 2014;52:824-36.
al. High-dose insulin therapy in beta-blocker and 2. Kanji S, MacLean RD. Cardiac glycoside toxicity:
calcium-channel blocker poisoning. Clin Toxicol more than 200 years and counting. Crit Care Clin
2011;49:277-83. 2012;28:527-35.
4. Kerns W II. Management of beta-adrenergic 3. Kelly RA, Smith TW. Recognition and man-
blocker and calcium channel antagonist toxicity. agement of digitalis toxicity. Am J Cardiol
Emerg Med Clin North Am 2007;25:309-31. 1992;69:108G-119G.
5. Levine M, Hoffman RS, Lavergne V, et al. 4. Lip GYH, Metcalfe MJ, Dunn FG. Diagnosis and
Systematic review of the effect of intravenous lipid treatment of digoxin toxicity. Postgrad Med J
emulsion therapy for non-local anesthetics toxic- 1993;69:337-9.
ity. Clin Toxicol. 2016;54:194-221.
6. Marraffa JM, Cohen V, Howland MA. Antidotes Antidepressants and Atypical Antipsychotics
for toxicological emergencies: a practical review. 1. Alapat PM, Zimmerman JL. Toxicology in the
Am J Health Syst Pharm 2012;69:199-212. intensive care unit. Chest 2008;133:1006-13.
7. Olson KR, Erdman AR, Woolf AD, et al. Calcium 2. Dunkley EJ, Isbister GK, Sibbritt D, et al. The
channel blocker ingestion: an evidence-based con- Hunter Serotonin Toxicity Criteria: simple and
sensus guideline for out-of-hospital management. accurate diagnostic rules for serotonin toxicity.
Clin Toxicol 2005;43:797-822. QJM 2003;96:635-42.
8. Patel NP, Pugh ME, Goldberg S, et al. 3. Minns AB, Clark RF. Toxicology and over-
Hyperinsulinemic euglycemia therapy for vera- dose of atypical antipsychotics. J Emerg Med
pamil poisoning: a review. Am J Crit Care 2012;43:906-13.
2007;16:498-503. 4. Oruch R, Elderbi MA, Khattab HA, Pryme
9. Ramoska EA, Spiller HA, Winter M, et al. A IF, Lund A. Lithium: A review of pharmacol-
one-year evaluation of calcium channel blocker ogy, clinical uses, and toxicity. Eur J Pharmacol
overdoses: toxicity and treatment. Ann Emerg 2014;740:464-73.
Med 1993;22:196-200. 5. Pimentel L, Trommer L. Cyclic antidepressant
10. Shepherd G. Treatment of poisoning caused by overdoses. Emerg Clin North Am 1994;12:533-47.
β-adrenergic and calcium channel blockers. Am J 6. Reilly TH, Kirk MA. Atypical antipsychotics and
Health Syst Pharm 2006;63:1828-35. newer antidepressants. Emerg Med Clin North Am
11. St-Onge M, Anseeuw K, Cantrell FL, et al. Expert 2007;25:477-97.
consensus recommendations for the management 7. Sarko J. Antidepressants, old and new: a review
of calcium channel blocker poisoning in adults. of their adverse effects and toxicity in overdose.
Crit Care Med 2017;45:e306-15. Emerg Med Clin North Am 2000;18:637-54.
8. Woolf AD, Erdman AR, Nelson LS, et al. Tricyclic
antidepressant poisoning: an evidence-based

ACCP Updates in Therapeutics® 2022: Critical Care Pharmacy Preparatory Review and Recertification Course

401
Toxicology

consensus guideline for out-of-hospital manage-


ment. Clin Toxicol 2007;45:203-33.

Oral Hypoglycemics
1. Fasano CJ, O’Malley G, Dominici P, et al.
Comparison of octreotide and standard therapy
versus standard therapy alone for the treatment of
sulfonylurea-induced hypoglycemia. Ann Emerg
Med 2008;51:400-6.
2. Glatstein M, Scolnik D, Bentur Y. Octreotide
for the treatment of sulfonylurea poisoning. Clin
Toxicol. 2012;50:795-804.
3. Rowden AK, Fasano CJ. Emergency management
of oral hypoglycemic drug toxicity. Emerg Med
Clin North Am 2007;25:347-56.
4. Soderstrom J, Murray L, Daly FF, Little M.
Toxicology case of the month: oral hypoglycaemic
overdose. Emerg Med J 2006;23:565-7.
5. Spiller HA, Sawyer TS. Toxicology of oral
antidiabetic agents. Am J Health Syst Pharm
2006;63:929-38.

Drugs of Abuse
1. Kersten BP, McLaughlin ME. Toxicology and
management of novel psychoactive drugs. J Pharm
Pract 2015;28:50-65.
2. Musselman ME, Hampton JP. “Not for human
consumption”: a review of emerging designer
drugs. Pharmacotherapy. 2014;34:745-57.
3. Mokhlesi B, Garimella PS, Joffe A, et al. Street
drug abuse leading to critical illness. Intensive
Care Med 2004;30:1526-36.
4. Nelson ME, Bryant SM, Aks SE. Emerging drugs
of abuse. Emerg Med Clin North Am 2014;32:1-28.
5. Rech MA, Donahey E, Cappiello Dziedzic JM,
et al. New drugs of abuse. Pharmacotherapy
2015;35:189-97.
6. Ricaurte GA, McCann UD. Recognition and man-
agement of complications of new recreational drug
use. Lancet 2005;365:2137-45.

ACCP Updates in Therapeutics® 2022: Critical Care Pharmacy Preparatory Review and Recertification Course

402
Toxicology

ANSWERS AND EXPLANATION TO PATIENT CASES

1. Answer: B 4. Answer: C
The most important first step in all drug overdose This patient has an acute salicylate overdose. Although
cases is to try to stabilize the patient’s ABC (Answer his serum salicylate concentrations are in the therapeu-
B is correct). This may involve the use of supplemental tic range, he has symptoms consistent with salicylate
oxygen or advanced airway management, establishment toxicity, as evidenced by his nausea, tachycardia, and
of intravenous access, and administration of intrave- respiratory alkalosis. He is currently stable, but his
nous fluids. Once the patient is stable, the process of serum salicylate concentrations may continue to rise;
identifying the suspected toxin can begin (Answer D therefore, enhanced elimination with serum bicarbonate
is incorrect). This may include thoroughly examining is the best option (Answer C is correct). His vital signs,
the patient, speaking with family or first responders, which are stable, should be monitored for changes;
and communicating with the patient’s physicians and however, although he is not experiencing signs of sig-
pharmacies. Blood and urine samples may be sent for nificant dehydration, he would benefit from the fluid
quantitative or qualitative toxicologic assays (Answer A administration of sodium bicarbonate. Sodium chloride
is incorrect). A coma cocktail may provide some benefit, would be more beneficial if his vital signs were more
but a clear cause should be established before consider- unstable (Answer A is incorrect). His RR, which is
ing its use (Answer C is incorrect). elevated, should be monitored; however, he is alert and
able to communicate and therefore does not need intu-
2. Answer: B bation at this time (Answer B is incorrect). He is also
The patient has symptoms of an acute acetaminophen not indicated for hemodialysis because of his moderate
overdose, and stabilizing the patient, together with symptoms, but this could be considered if his condition
providing good supportive care, is indicated until a deter- deteriorates (Answer D is incorrect).
mination for additional therapy can be made (Answer B
is correct). Typical decontamination strategies may pro- 5. Answer: A
vide benefit, but they do not definitively improve patient The most appropriate therapy for an ethylene glycol
outcome. Magnesium citrate (and cathartics as a whole) intoxication is fomepizole (Answer A is correct). An
is not considered an effective decontamination strategy ethanol infusion is a possible treatment option, but it is
(Answer A is incorrect). Gastric lavage is of most benefit not preferred because of the difficulties in dosing and
within the first 60 minutes of exposure, and the potential adverse effects (Answer B is incorrect). Thiamine is
adverse effects outweigh any potential benefit (Answer a cofactor in the metabolism of ethylene glycol, but it
C is incorrect). Similarly, single-dose charcoal requires would not be preferred to administer thiamine before
more rapid administration (Answer D is incorrect). fomepizole (Answer C is incorrect). Activated charcoal
is not an option for gastric decontamination because it is
3. Answer: C not effective for alcohols (Answer D is incorrect).
The patient is considered at high risk of developing
hepatic damage from acetaminophen and requires 6. Answer: C
therapy with intravenous acetylcysteine (Answer D is After the initial bolus of fomepizole, 10 mg/kg should be
incorrect). The dose of intravenous acetylcysteine is as administered every 12 hours. Because of the increased
follows (doses are calculated using actual body weight): clearance of fomepizole during hemodialysis, the fre-
loading dose: 150 mg/kg in 200 mL of 5% dextrose in quency is changed to every 4 hours during dialysis
water for 60 minutes; maintenance dose: 50 mg/kg in (Answer C is correct and Answer A is incorrect). When
500 mL of 5% dextrose in water for 4 hours, followed dialysis is completed, the dose returns to 10 mg/kg
by 100 mg/kg in 1000 mL of 5% dextrose in water for administered every 12 hours; and once the patient has
16 hours (Answer C is correct and Answer B is incor- been on 48 hours of therapy, dosing increases to 15 mg/
rect). Oral dosing of acetylcysteine is not a viable option kg because of self-induction. There is no indication for a
because of the patient’s severe nausea and vomiting dose increase (Answer B and D are incorrect).
(Answer A is incorrect).

ACCP Updates in Therapeutics® 2022: Critical Care Pharmacy Preparatory Review and Recertification Course

403
Toxicology

7. Answer: C applying measures to enhance surface cooling (Answer


The best treatment option for this patient is whole bowel C is incorrect).
irrigation because of the extended-release formulation
of diltiazem (Answer C is correct). Activated char- 10. Answer: D
coal may provide some benefit, but, similar to ipecac, Although the patient appears to have taken an overdose of
the time interval is not known, and diltiazem does olanzapine, she is experiencing only mild symptoms. The
not undergo enterohepatic recirculation (Answer A is best intervention would be to monitor her for 6 hours for
incorrect). Ipecac is not recommended because it may the progression of her symptoms or development of addi-
impede treatment with more effective treatment options tional complications (Answer D is correct). Intravenous
and because it is no longer manufactured in the United fluids would be appropriate if the patient has dehydration
States (Answer B is incorrect). A cathartic would not be or hypotension (Answer A is incorrect). Sodium bicarbon-
useful in this situation; guidelines recommend its use ate is indicated for QRS prolongation and is not warranted
only in combination with other decontamination strate- at this time (Answer B is incorrect). Olanzapine does not
gies, not as a single agent (Answer D is incorrect). cause seizures; therefore, lorazepam would not be indi-
cated (Answer C is incorrect).
8. Answer: A
There are several potential antidotes for a calcium chan- 11. Answer: B
nel blocker overdose. Calcium is the most effective, and The most appropriate intervention at this time is to give
it should be given by bolus, followed by continuous infu- the patient intravenous dextrose (Answer B is correct).
sion if needed (Answer A is correct). Glucagon is not an Oral glucose is a viable option, but it cannot be admin-
effective antidote and is therefore not an option for this istered to an unconscious patient without oral access
patient (Answer B is incorrect). Atropine is effective for (Answer A is incorrect). Octreotide should be reserved
symptomatic bradycardia caused by the calcium chan- for use if the administration of a glucose solution fails
nel blocker, but the dose should be 0.5–1 mg (Answer C to raise the blood glucose above 70 mg/dL for two con-
is incorrect). Epinephrine is an alternative to glucagon, secutive readings (Answer C is incorrect). Glucagon is
but this dose is excessive for a patient not experiencing a potential option for treatment, but because the patient
cardiac arrest (Answer D is incorrect). has intravenous access, the intramuscular route would
not be preferred (Answer D is incorrect).
9. Answer: D
Most of the SSRIs are relatively safe, and many patients
will present as asymptomatic after an overdose.
However, there is a potential for a patient to develop
serious adverse effects, such as serotonin syndrome,
seizures, and cardiac toxicity. Although this patient is
stable and has no specific concerns, it is recommended
to check a 12-lead ECG to measure for QT-interval
prolongation and treat with sodium bicarbonate, if nec-
essary (Answer D is correct). A benzodiazepine should
be administered if muscle rigidity develops, but it
should not be used as a prophylactic measure (Answer
A is incorrect). It is recommended that the patient be
observed for at least 6–8 hours. Cyproheptadine is
only indicated for symptomatic patients (Answer B is
incorrect). Measures should be performed to reduce
hyperthermia if a serotonergic syndrome develops, but
this should be treated with measures to reduce muscle
activity (i.e., sedation or chemical paralysis), not by

ACCP Updates in Therapeutics® 2022: Critical Care Pharmacy Preparatory Review and Recertification Course

404
Toxicology

ANSWERS AND EXPLANATIONS TO SELF-ASSESSMENT QUESTIONS

1. Answer: C 4. Answer: A
The best option for this patient right now is to admin- The patient is experiencing QT prolongation after an
ister octreotide 50–100 mcg subcutaneously (Answer C atypical antipsychotic overdose. It is important to sta-
is correct). The patient has not responded to two doses bilize the patient by administering intravenous sodium
of intravenous dextrose, as evidenced by point-of-care bicarbonate and electrolyte replacement (Answer A is
glucose concentrations less than 70 mg/dL; therefore, correct). Her potassium concentration is low, requiring
additional doses of dextrose are not indicated (Answer replacement. Because the time interval of the overdose
A is incorrect). Although glucagon is also a potential is not known, there is limited benefit for activated char-
option, it is not recommended for sulfonylurea expo- coal (Answer B is incorrect). Although her magnesium
sures (Answer B is incorrect). Sodium bicarbonate concentration is normal, it should be monitored; how-
intravenously may be indicated in this scenario if the ever, her magnesium concentration does not require
patient had metformin-associated lactic acidosis; how- replacement at this time because her QTc is less than
ever, the patient is not on metformin and has no signs of 500 milliseconds (Answer C is incorrect). Lorazepam is
lactic acidosis (Answer D is incorrect). not indicated for prophylaxis of seizure activity (Answer
D is incorrect).
2. Answer: A
Any of the options listed in this question are possible 5. Answer: A
treatments for a patient with a β-blocker overdose who is This patient has the clinical signs and symptoms of
not responding to the administration of intravenous flu- alcohol withdrawal. Management should focus on the
ids and calcium. The optimal choice ultimately involves patient’s safety and controlling his symptoms, and treat-
efficacy and appropriate dosing. Glucagon is an option, ment should be administered using a symptom-triggered
and it should be dosed at 5–10 mg intravenously ini- therapy strategy. The primary agents used to control
tially (Answer A is correct). Atropine is an option for symptoms are the benzodiazepines, and lorazepam is
the patient’s bradycardia, but the initial recommended a good option (Answer A is correct). Barbiturates such
dose is 0.5–1 mg intravenously (Answer B is incorrect). as phenobarbital are typically reserved for patients who
Hyperinsulinemic euglycemic therapy may be preferred do not respond to benzodiazepine therapy because of
in this setting; however, the correct bolus dose is 1 unit/ benzodiazepine’s long elimination half-life and stronger
kg intravenously (Answer C is incorrect). Dopamine sedative effects and oral dosing may be difficult with
is an option for the treatment of hypotension and bra- his level of confusion (Answer B is incorrect). Propofol
dycardia, but the correct dose would be the initiation should be avoided in non-intubated patients (Answer C
of an infusion at 5–10 mcg/kg/minute titrated to effect is incorrect). Clonidine is a potential option, especially
(Answer D is incorrect). because this patient has borderline hypertension, but
oral dosing may be difficult with his level of confusion
3. Answer: B (Answer D is incorrect).
Given the patient’s presentation and the common toxi-
dromes, the most likely scenario is a cholinergic agent 6. Answer: B
(Answer B is correct). The patient is experiencing bra- The patient is experiencing an unintended opioid over-
dycardia with a normal BP and RR, has a decrease in dose, as evidenced by the decreased RR and decreased
mental status, and is experiencing nausea. Although not consciousness. Administration of the antidote, nalox-
an absolute, anticholinergics and sympathomimetics are one, is the best option (Answer B is correct). Because 2
more commonly associated with tachycardia (Answers hours have passed since the methadone dose was given,
A and D are incorrect). Similarly, opioids are typically there is limited usefulness for activated charcoal at this
associated with a decrease in respirations (Answer C is time, and it would not be advisable to administer it to
incorrect). an unconscious patient without an established airway
(Answer A is incorrect). Whole bowel irrigation is also
not useful in this situation because it is too late to prevent

ACCP Updates in Therapeutics® 2022: Critical Care Pharmacy Preparatory Review and Recertification Course

405
Toxicology

drug absorption together with the airway safety concern


(Answer C is incorrect). Administration of intravenous
fluids would be beneficial to improve BP but should not
be administered in this case before naloxone (Answer D
is incorrect).

7. Answer: D
The patient is not responding to the initiation of intrave-
nous fluids and calcium gluconate, so HIET is warranted.
Because of the patient’s low serum potassium concen-
trations, it is critical to replace this before administering
insulin (Answer A is incorrect and Answer D is correct).
The patient’s glucose concentration is greater than 200
mg/dL, so additional glucose need not be given at this
time (Answer B is incorrect). Full effects may take up
to 30 minutes to be seen, but this should not prevent the
initiation of HIET (Answer C is incorrect).

8. Answer: B
The patient is not responding to the initiation of intra-
venous fluids, calcium, and HIET. The most appropriate
option at this time would be to increase the rate of the
insulin infusion (Answer B is correct). The initial infu-
sion rate is 0.5–1 unit/kg/hour and is titrated every
15–20 minutes until hemodynamically stable. The next
option would be to initiate a vasopressor agent (Answers
A and C are incorrect). From the choices listed, the best
first option is norepinephrine initiated at 4 mcg/minute
and titrated to the desired effect. Epinephrine is also
a possible option, but it would be recommended if the
patient were not responding to increasing doses of nor-
epinephrine. Intravenous lipid emulsion is a potential
therapy, but it is typically administered in a patient with
severe decompensation caused by a lipophilic medi-
cation who is not responding to fluids or vasopressors
(Answer D is incorrect).

ACCP Updates in Therapeutics® 2022: Critical Care Pharmacy Preparatory Review and Recertification Course

406
Acute Kidney Injury and
Renal Replacement Therapy
in the Critically Ill Patient
Erin F. Barreto, Pharm.D., M.Sc., FCCM, FASN, BCPS, BCCCP
Mayo Clinic
Rochester, Minnesota
Acute Kidney Injury and Renal Replacement Therapy in the Critically Ill Patient

Acute Kidney Injury and


Renal Replacement Therapy
in the Critically Ill Patient
Erin F. Barreto, Pharm.D., M.Sc., FCCM, FASN, BCPS, BCCCP
Mayo Clinic
Rochester, Minnesota

ACCP Updates in Therapeutics® 2022: Critical Care Pharmacy Preparatory Review and Recertification Course

409
Acute Kidney Injury and Renal Replacement Therapy in the Critically Ill Patient

Learning Objectives Self-Assessment Questions


Answers and explanations to these questions may be
1. Define acute kidney injury (AKI). found at the end of this chapter.
2. Differentiate between common categories of drug-
induced kidney disease. Questions 1–3 pertain to the following case.
3. Discuss key principles of continuous renal replace- E.R. is a 67-year-old man admitted to your intensive care
ment therapy (CRRT), including indications, timing, unit (ICU) several days ago with acute respiratory fail-
and circuit components. ure. He required mechanical ventilation and was placed
4. Apply drug-dosing concepts in CRRT to estimate on empiric antibiotics to cover community-acquired
a sieving coefficient, saturation coefficient, and/or pneumonia. His kidney function has worsened over
drug clearance on the basis of drug characteristics the past 5 days (serum creatinine [SCr] 0.6 mg/dL on
and device settings. admission; today, 2.4 mg/dL), and he now requires renal
replacement therapy (RRT) with continuous venovenous
hemofiltration (CVVH). He is extubated, but still unsta-
Abbreviations in This Chapter ble. His current medications include as-needed fentanyl,
norepinephrine, ceftriaxone, azithromycin, and heparin
AIN Acute interstitial nephritis prophylaxis.
AKI Acute kidney injury
AKIN Acute Kidney Injury Network 1. In E.R., which drug-induced kidney disease pheno-
ARF Acute renal failure type is most associated with his antibiotic program?
ATN Acute tubular necrosis A. AKI.
CKD Chronic kidney disease B. Glomerular disease.
CRRT Continuous renal replacement therapy C. Nephrolithiasis/crystalluria.
CVVH Continuous venovenous hemofiltration D. Tubular dysfunction.
CVVHD Continuous venovenous hemodialysis
CVVHDF Continuous venovenous hemodiafiltration 2. What KDIGO stage best characterizes E.R.’s AKI?
DIKD Drug-induced kidney disease
A. Stage 1.
EDD Extended daily dialysis
B. Stage 2.
GFR Glomerular filtration rate
C. Stage 3.
ICU Intensive care unit
D. Cannot determine because urinary output (UOP)
IHD Intermittent hemodialysis
is not provided.
KDIGO Kidney Disease: Improving Global
Outcomes
3. E.R. continues to worsen. He is now febrile with
MW Molecular weight
an increasing white blood cell count (WBC). You
RIFLE Risk, injury, failure, loss, end-stage renal
are asked to dose his antibiotics while he receives
disease
CVVH. Which would be the best place to begin
RRT Renal replacement therapy
looking for dosing recommendations?
SCr Serum creatinine
SLED Sustained low-efficiency dialysis A. Intermittent hemodialysis (IHD) guidelines.
UOP Urinary output B. Package insert.
Vd Volume of distribution C. Primary literature/dosing summaries.
D. Estimates using an estimated sieving coefficient.

4. Which best describes the method for solute removal


during CVVH?
A. Convection.
B. Diffusion.
C. Both convection and diffusion.
D. Membrane binding.

ACCP Updates in Therapeutics® 2022: Critical Care Pharmacy Preparatory Review and Recertification Course

410
Acute Kidney Injury and Renal Replacement Therapy in the Critically Ill Patient

5. Which drug would most likely be cleared through a


CVVHD circuit?
A. Drug A: 10% protein bound, 140-Da molecular
weight (MW), volume of distribution (Vd) 1.2
L/kg.
B. Drug B: 90% protein bound, 1030-Da MW, Vd
0.8 L/kg.
C. Drug C: 40% protein bound, 250-Da MW, Vd
1.5 L/kg.
D. Drug D: 35% protein bound, 480-Da MW, Vd
4.6 L/kg.

ACCP Updates in Therapeutics® 2022: Critical Care Pharmacy Preparatory Review and Recertification Course

411
Acute Kidney Injury and Renal Replacement Therapy in the Critically Ill Patient

BPS Critical Care Pharmacy Specialist Examination Content Outline

This chapter covers the following sections of the Critical Care Pharmacy Specialist Examination Content Outline:
1. Domain 1: Clinical Knowledge and Application
Task 1: 1, 3
Task 2: 3
Task 3: 1-6
Task 4: 1-6, 8
Task 5: 5
Task 7: 1-4, 6-7
2. Domain 2: Practice Management, Policy, and Quality Improvement
a. Task 2: 2
b. Task 5: 4
3. Domain 3: Evidence-Based Medicine, Scholarship, Education, and Professional Development
a. Task 1: 1-3

ACCP Updates in Therapeutics® 2022: Critical Care Pharmacy Preparatory Review and Recertification Course

412
Acute Kidney Injury and Renal Replacement Therapy in the Critically Ill Patient

I.  ACUTE KIDNEY INJURY

A. Introduction
1. Acute changes to kidney structure or function were previously called acute renal failure (ARF).
2. Acute kidney injury (AKI) is the newer term, which encompasses the full continuum of kidney injury
and functional impairment.
3. AKI is a decrease in kidney function that occurs over hours to days.
4. AKI results in the accumulation of waste products and, as urine volume decreases, metabolic disturbances
and fluid retention.
5. AKI has been associated with increased mortality, development of chronic kidney disease (CKD),
cardiovascular complications, and end-stage renal disease.

B. Epidemiology
1. The incidence of AKI varies and depends on the definition used, its cause, and the patient population.
2. Hospital-acquired AKI occurs infrequently in patients with less severe illness admitted to a general
hospital ward (1.9%–20%). In critically ill patients, the risk is greater. In this group, AKI is estimated
to occur in 20%–67% of patients.
3. Sepsis and shock are common causes of acute tubular necrosis (ATN), which is a leading cause of AKI in
critical illness. Other risk factors for AKI include use of intravenous radiocontrast agents, major surgery
(especially cardiothoracic), nephrotoxic medications, and chronic medical conditions (e.g., history of
CKD, congestive heart failure, and diabetes mellitus). Most patients have more than one risk factor.
4. Mortality rates in patients with AKI are 10%–80%, with the highest in patients with multisystem organ
failure (50%) and those requiring renal replacement therapy (RRT) (up to 80%).

C. Definitions (Table 1)
1. During the past several decades, many definitions have been used for AKI, making it difficult to compare
patient populations across studies. In 2004, the Acute Dialysis Quality Initiative workgroup developed
the RIFLE (risk, injury, failure, loss, end-stage renal disease) definition and staging system.
a. RIFLE categorizes AKI into three grades of increasing severity (risk, injury, and failure) and two
clinical outcomes (loss and end-stage).
b. Staging is based on the degree of SCr increase or a decrease in glomerular filtration rate over 7 days,
or the duration of oliguria or anuria.
2. Because of emerging data suggesting that even small changes in kidney function lead to worse outcomes,
the Acute Kidney Injury Network (AKIN) criteria was developed.
a. Similar to RIFLE, this group defined AKI using a staging system of 1-3, defined by a reduction in
kidney function over no more than 48 hours using measures of SCr, urinary output (UOP), and need
for RRT.
b. The primary difference between the RIFLE and the AKIN criteria is that AKIN includes a small
absolute change in SCr (0.3 mg/dL or more) as part of the diagnostic criteria for AKI and sets a
48-hour time constraint for the criteria to be met. AKIN also omits the outcome classifications (loss
and end-stage) previously proposed in the RIFLE criteria.
c. In patients requiring RRT, AKIN stage 3 is met, regardless of the stage they are in when RRT is
initiated.
d. Several studies have validated these criteria and show that the more severe the RIFLE class or
AKIN stage, the worse the clinical outcome.
3. In 2012, a third consensus definition was introduced, the Kidney Disease: Improving Global Outcomes
(KDIGO) classification system, which is the current criterion standard.
a. Separately, the RIFLE criteria and the AKIN criteria were each shown to be suboptimally sensitive
for detecting AKI.

ACCP Updates in Therapeutics® 2022: Critical Care Pharmacy Preparatory Review and Recertification Course

413
Acute Kidney Injury and Renal Replacement Therapy in the Critically Ill Patient

b. The KDIGO criteria combine the strengths of both RIFLE and AKIN, retain the AKIN criteria of a
rise in SCr of 0.3 mg/dL within 48 hours, and allow 7 days for a 50% increase in SCr from baseline,
as seen in RIFLE.

Table 1. Criteria for Establishing AKI


All Three
RIFLE AKIN KDIGO Classification
Systems
Stage SCr/GFR Criteria Stage SCr Criteria Stage SCr Criteria UOP Criteria
Increase to 1.5- to
Increase to 1.5- to
Increase to ≥ 1.5-fold < 2-fold above base-
2-fold above baseline < 0.5 mL/kg/hr
R or GFR decrease 1 1 line over 7 days or by
or by ≥ 0.3 mg/dL for 6-12 hr
> 25% from baseline ≥ 0.3 mg/dL within
within 48 hr
48 hr
Increase to ≥ 2-fold Increase to ≥ 2- to < 0.5 mL/kg/hr
Increase to ≥ 2- to
I or GFR decrease 2 2 < 3-fold above for ≥ 12 hr per
3-fold above baseline
> 50% from baseline baseline the guideline
Increase to ≥ 3-fold,
Increase > 3-fold
GFR decrease > 75% Increase ≥ 3-fold < 0.3 mL/kg/
above baseline or ≥ 4
from baseline, or SCr above baseline or ≥ 4 hr for ≥ 24 hr
F 3 mg/dL with an acute 3
≥ 4 mg/dL (acute mg/dL or initiation of or anuria for ≥
rise of ≥ 0.5 mg/dL or
increase of at least RRT 12 hr
on RRT
0.5 mg/dL)
Complete loss of
L
function for > 4 wk
Complete loss of
E
function for > 3 mo
AKI = acute kidney injury; AKIN = Acute Kidney Injury Network; GFR = glomerular filtration rate; KDIGO = Kidney Disease: Improving Global Outcomes; RIFLE =
risk (R), injury (I), failure (F), loss (L), end-stage kidney disease (E); RRT = renal replacement therapy; SCr = serum creatinine; UOP = urinary output.

4. Kidney dysfunction/damage is a spectrum.


a. In 2012, KDIGO proposed four categories to encompass the spectrum of kidney function (Figure 1;
Table 2 provides detailed definitions).
i. No known kidney disease (NKD)
ii. AKI: Acute kidney dysfunction over 7 days or less
iii. Acute kidney disease (AKD): For those who meet the AKI definition or with other types of
kidney dysfunction/damage for less than 3 months
iv. Chronic kidney disease (CKD): Kidney dysfunction/damage for more than 3 months
b. The definitions were intended to improve consistency in applied nomenclature in care, research, and
public health and to identify patients with AKD who may need therapy to restore kidney function
and mitigate or reverse kidney damage.
c. Increasingly, patients in the AKD subgroup are targeted for interventions designed to prevent CKD
development, including bundled strategies delivered in AKI follow-up clinics. Interprofessional
survivor clinics for AKI are reported to improve patient education, engagement, follow-up, and
outcomes. Pharmacists can play a central role in such initiatives by providing a careful medication
evaluation (renally eliminated and nephrotoxic drugs), monitoring and supportive care for key
concurrent conditions (e.g., hypertension, lipid management), and patient and caregiver education.

ACCP Updates in Therapeutics® 2022: Critical Care Pharmacy Preparatory Review and Recertification Course

414
Acute Kidney Injury and Renal Replacement Therapy in the Critically Ill Patient

NKD

AKD AKI CKD

Figure 1. Overview of the spectrum of kidney disease.


AKD = acute kidney disease; AKI = acute kidney disease; CKD =chronic kidney disease; NKD = no known kidney disease.

Table 2. Definitions of NKD, AKI, AKD, and CKD According to Function and Structure
Structural Criteria
Functional Criteria (change in SCr, GFR,a or UOP)
(damage or no damage and duration)
b

NKD GFR ≥ 60 mL/min/1.73 m2, stable SCr No


Increase in SCr by 50% within 7 days
Or
AKI Increase in SCr by 0.3 mg/dL within 2 days No criteria
Or
Oliguria
AKI
Or
GFR < 60 mL/min/1.73 m2 for < 3 mo
AKD Kidney damage for < 3 mo
Or
Decrease in GFR by ≥ 35% or increase
in SCr by > 50% for < 3 mo
CKD GFR < 60 mL/min/1.73 m2 for > 3 mo Kidney damage for > 3 mo
a
Assessed from measured or estimated GFR.
b
Kidney damage is assessed by pathology, biomarkers (urine or blood), imaging, and kidney transplantation (for CKD).
AKD = acute kidney disease and disorders; CKD = chronic kidney disease.

D. Diagnostic Workup and Evaluation


1. There is no single specific test for diagnosing AKI.
2. A thorough history and complete examination should be completed, including rate of kidney function
decline, symptoms, and coexisting diseases
3. A comprehensive review of current and recent medications should be conducted. Drug-related injury is
a leading cause of AKI.
4. Chemistries (blood urea nitrogen [BUN], SCr, serum electrolytes, albumin, and a complete blood cell
count) and a urinary analysis (microscopy, sodium, and osmolality) may assist in determining the type
of failure (e.g., prerenal, intrinsic, postrenal).
5. If a bladder catheter is not present, it is reasonable to insert one in most cases. Compelling indications for
catheterization include cases in which ins and outs are difficult to measure accurately such as in patients
with incontinence or those with evident postvoid residuals. If a catheter is present without UOP, it should
be evaluated for patency.
6. Abdominal compartment syndrome should be ruled out if clinically suspected. Acute oliguria and AKI
can be the result of increasing renal outflow pressure and reduced kidney perfusion.
7. Routine use of kidney ultrasonography is of limited usefulness because most ICU-related AKI is
associated with decreased effective arterial blood volume and/or ATN. However, it may be useful in
high-risk patients or after an initial evaluation fails to reveal the cause of AKI.

ACCP Updates in Therapeutics® 2022: Critical Care Pharmacy Preparatory Review and Recertification Course

415
Acute Kidney Injury and Renal Replacement Therapy in the Critically Ill Patient

8. Renal biopsies have limited usefulness but may be necessary. They are most useful in intrinsic renal
failure not associated with ATN or when glomerulopathy is suspected.
9. Biomarkers: Recently, significant advances were made in the field of biomarkers for predicting and
detecting AKI. When the KDIGO guidelines were released in 2012, these data were not available;
thus, they were not included in the criteria for the diagnosis of AKI. Despite emerging literature, the
most recent KDIGO controversies conference on AKI suggests that additional research is needed before
including these biomarkers in the diagnostic criteria for AKI.

Increased Kidney Decreased Kidney Morbidity


NKD
risk for AKI damage GFR failure Mortality

Warning biomarkers Early detection and


(e.g. TIMP2*IGFBP7) diagnostic biomarkers
(e.g. NGAL, KIM-1,
L-FABP, SCr, UOP, Cystatin C)

Figure 2. Evolution of AKI as it corresponds to biomarker release.


Adapted from: Bellomo R, Kellum JA, Ronco C. Acute kidney injury. Lancet 2012;380:756-66.

a. Functional markers: Indicate the kidney’s capacity for clearance and are used to estimate GFR and
functional loss; these can help indicate a need for drug dosing changes
i. Examples: SCr, UOP, serum cystatin C, proenkephalin, beta-trace protein, β2 microglobulin
ii. These biomarkers may rise with or without evidence of structural damage to the kidney (e.g.,
prerenal AKI, which could correspond to a reduced GFR that may not yet have resulted in
kidney damage).
iii. Recent evidence suggests that cystatin C can predict the pharmacokinetics of medications
as well as, if not better than, SCr with certain renally eliminated medications. Drug-specific
dosing models need to be developed and tested.
b. Damage/injury markers: Signal injury to the kidney cells or at least cellular distress; these can help
signal patients who would benefit from limiting exposure to nephrotoxins
i. Warning biomarkers (e.g., cell-cycle arrest markers [TIMP-2•IGFBP7])
(a) TIMP-2•IGFBP7 is a U.S. Food and Drug Administration–approved urinary test; these
biomarkers are released during a pause (an “arrest”) in mitosis when the biomarker
indicates the cell may be injured and should not divide.
(b) In the PrevAKI (Intensive Care Med 2017;43:1551-61) and BigpAK (Ann Surg
2018;267:1013-20) trials, surgical patients with a high risk of AKI on the basis of [TIMP-
2•IGFBP7] greater than 0.3 who were randomized to an AKI prevention bundle had a
lower incidence of AKI than did individuals who received usual care. This is a significant
advancement because it suggests that even in high-risk patients, AKI can indeed be
prevented.
ii. Early detection and diagnostic biomarkers (e.g., NGAL, KIM-1, L-FABP, NAG, urine cystatin
C) are another set of tools that have been studied as early indicators of kidney damage; each
test has different performance characteristics
iii. The cost-effectiveness of using these biomarkers to inform routine care has not been well
studied; however, given the high costs of AKI, particularly when dialysis is required, a
systematic approach to biomarker use could prove economical. Additional research is needed
to explore this further.

ACCP Updates in Therapeutics® 2022: Critical Care Pharmacy Preparatory Review and Recertification Course

416
Acute Kidney Injury and Renal Replacement Therapy in the Critically Ill Patient

c. Although the evidence is promising, novel biomarker use is not yet mainstream. Several factors,
including the evidence base that is still in its infancy, implementation challenges, and cost, have
precluded widespread adoption as an adjunct to SCr and UOP at this time. As use continues,
additional information on the logistics of successfully applying these tests is expected.
10. Kinetic eGFR is another strategy that has been proposed to evaluate rapidly changing kidney function
(J Am Soc Nephrol 2013;24:877-88). Creatinine is a lagging marker of kidney function and may take
48 hours to increase from the onset of kidney damage. The kinetic eGFR equation is thought to provide
a quantitative solution to estimate kidney function in this difficult scenario. Using a simple algebraic
formula, including the initial creatinine content, the volume of distribution, the creatinine production
rate, and the change in SCr over time, inputs can be used to quantitatively interpret creatinine change.
Kinetic eGFR predicted AKI and the need for RRT more accurately than Modification of Diet in Renal
Disease (Clin Kidney J 2017;10:202-8). Published literature on the use of kinetic eGFR to inform
medication dosing remains in infancy.

E. Causes and Mechanisms of AKI


1. Prerenal: AKI attributable to decreased kidney perfusion. Decreased kidney perfusion could be caused by
low effective arterial blood volume, as in hypovolemia or acute decompensated heart failure, peripheral
vasodilation (as with sepsis), or renal vasoconstriction or renal occlusion (caused by hepatorenal
syndrome or arterial thrombosis, respectively). Some have suggested to do away with the nomenclature
“prerenal” because it could be misinterpreted to mean hypovolemia, which only reflects a small subset
of cases with decreased kidney perfusion.
2. Renal (intrinsic): AKI secondary to parenchymal or vascular diseases. Acute tubular necrosis (ATN)
is the most commonly cited example of this category of AKI in the hospital setting. AKI secondary to
vasculitis, glomerulonephritis, interstitial nephritis, infections, connective tissue disease, and crystalluria
are in this category as well.
3. Postrenal: AKI associated with urinary tract obstruction. Common causes include prostatic hypertrophy,
nephrolithiasis, and tumor obstruction. If an obstruction is suspected, placing a urinary catheter may be
a sufficient temporizing measure. If higher in the urinary tract, nephrostomy tubes may be warranted to
alleviate hydronephrosis.

F. Drug-Induced Kidney Disease (DIKD)


1. Drugs are associated with about one-third of AKI cases in the hospital. The concept of “nephrotoxin
stewardship” has been proposed. Nephrotoxin stewardship is a term taken from antimicrobial
stewardship that generally refers to “coordinated interventions to improve and measure the appropriate
use of drugs by promoting the selection of the optimal drug regimen, including dosing, duration of
therapy, and route of administration” (Clin Infect Dis 2016;62:e51-77). Kane-Gill has proposed three
goals for nephrotoxin stewardship: coordinated strategies to enhance medication safety, coordinated
strategies to ensure kidney health, and avoidance of unnecessary costs (Crit Care Clin 2021;37:303-20).
2. Recently, a standardized classification system (Kidney Int 2015;88:226-34) was proposed for DIKD that
classifies kidney dysfunction according to three primary factors:
a. Phenotype of kidney injury
b. Mechanism: Type A (dose-dependent toxicities), type B (unpredictable, not dose-dependent)
c. Time course: Acute (1–7 days), subacute (8–90 days), chronic (more than 90 days)
3. Extrarenal: Drugs that alter renal perfusion or influence intraglomerular hemodynamics can heighten a
patient’s risk of AKI. Examples include the following:
a. Loop diuretics: Intravascular volume depletion leading to loss of effective arterial blood volume
b. Negative inotropes or vasodilatory antihypertensives that reduce renal perfusion
c. Drugs that alter intraglomerular hemodynamics: Nonsteroidal anti-inflammatory drugs (NSAIDs),
angiotensin-converting enzyme inhibitors/angiotensin receptor blockers (ACEIs/ARBs)

ACCP Updates in Therapeutics® 2022: Critical Care Pharmacy Preparatory Review and Recertification Course

417
Acute Kidney Injury and Renal Replacement Therapy in the Critically Ill Patient

i. Normally, glomerular pressure is maintained, in part, by vasodilation of the afferent arterioles


or vasoconstriction of the efferent arterioles.
ii. Vasodilation of the afferent arteriole is partly controlled by the effects of prostaglandins.
Medications that decrease prostaglandin synthesis such as NSAIDs decrease the ability of the
afferent arteriole to vasodilate and thus increase the risk.
iii. Vasoconstriction of the efferent arteriole, which helps maintain transcapillary pressure, is
mediated through angiotensin II. Drugs that block angiotensin II such as ACEIs and ARBs
prevent effective efferent vasoconstriction and thus increase the risk.
iv. The so-called “triple whammy” of NSAIDs/diuretics/ACEIs or ARBs has been described
as having a uniquely high risk of AKI because of the multiplicity of the intraglomerular
hemodynamic impact (BMJ 2013;346:e8525).
v. Acute use of calcineurin inhibitors, particularly in the setting of other risk factors for AKI, is
thought to interfere with renal hemodynamics.
4. Intrarenal toxicity: There are several types of kidney toxicity from xenobiotics. These include direct
tubular toxicity, acute interstitial nephritis (AIN), glomerular diseases, osmotic nephrosis, crystalluria/
nephrolithiasis, and altered electrolyte handling.
a. Direct tubular toxicity: Drugs can be associated with direct tubular damage, cell apoptosis, and
necrosis. ATN may be suggested by the presence of granular casts (“muddy brown casts”) or renal
tubular epithelial cells on a urinalysis.
i. Examples of drugs that contribute to ATN: Intravenous contrast, aminoglycosides, amphotericin
B, vancomycin, colistin, antiviral agents, platinum chemotherapy
ii. Direct tubular toxicity is often a type A injury that responds to dose adjustment or drug
discontinuation and supportive care.
b. AIN: Drug-associated AIN is thought to be a cell-mediated immune response, analogous to a
type 4 hypersensitivity reaction. Microscopic hematuria, sterile pyuria, and nonnephrotic range
proteinuria are features of AIN on urinalysis. Other features of AIN can include urine and/or
peripheral eosinophilia, clinical symptoms of fever, rash, and joint pains. Although commonly
cited in the literature, these may not be universally present; in fact, only 10%–30% of patients
with this diagnosis have the “classic triad” of eosinophilia, fever, and rash. A kidney biopsy can be
considered but, in the acute setting, is rarely performed until other causes are ruled out.
i. Drug causes have been implicated in up to 75% of AIN cases. Other separate or concurrent
contributors to AIN include infections (5%–10%), idiopathic causes (5%–10%), and causes
associated with systemic diseases (10%–15%).
ii. Examples of drugs that contribute to AIN: Penicillins, cephalosporins, sulfonamides,
ciprofloxacin, vancomycin, NSAIDs and cyclooxygenase-2 [COX-2] inhibitors, proton pump
inhibitors, immune checkpoint inhibitors, phenytoin, valproic acid, ranitidine, diuretics, and
cocaine
iii. Management of AIN includes discontinuation of the offending agent. These injuries are
often subacute and take weeks to months to resolve. Steroids may be used, but this remains
controversial.
iv. Recently, the combination of piperacillin/tazobactam and vancomycin received specific
attention for its risk of nephrotoxicity. In ICU patients, observational studies suggest that
empiric courses of less than 72 hours are no more nephrotoxic than other such combinations.
Preclinical models in rats using histopathology and novel urinary kidney biomarkers suggest no
evidence of heightened risk with the combination (J Antimicrob Chemother 2020;75:1228-36).
In observational data in humans, evidence still favors a potential increased risk of AKI with the
combination; thus, the risk-benefit of other combinations should be considered in conjunction
with other potential toxicities or antibiotic stewardship efforts.

ACCP Updates in Therapeutics® 2022: Critical Care Pharmacy Preparatory Review and Recertification Course

418
Acute Kidney Injury and Renal Replacement Therapy in the Critically Ill Patient

c. Glomerular diseases: DIKD rarely manifests as glomerular disease. Glomerular diseases


from medication can include immune-mediated glomerular diseases, including antineutrophil
cytoplasmic antibody (ANCA)-associated vasculitis with necrotizing and crescentic/pauci-immune
glomerulonephritis, drug-induced lupus, and drug-associated membranous nephropathy. In
addition, drugs may cause direct glomerular toxicity, resulting in minimal change disease or focal
segmental glomerulosclerosis. Depending on the type of injury, suggestive diagnostics of drug-
associated glomerular diseases may include proteinuria, hematuria, positive antibody tests, rash,
fever, myalgias, and polyarthritis. A kidney biopsy is often pursued.
i. Examples of drugs that contribute to immune-mediated glomerular diseases include hydralazine
and propylthiouracil. Drugs that contribute to direct glomerular cell toxicity include lithium,
NSAIDs, sirolimus, anabolic steroids, anti-angiogenesis drugs (i.e., bevacizumab, sunitinib,
sorafenib), chemotherapeutics (i.e., mitomycin C, gemcitabine), calcineurin inhibitors,
interferon, thienopyridines (i.e., clopidogrel, prasugrel), and quinine.
ii. Management of drug-induced glomerular disease includes identifying the subtype and
discontinuing the offending agent. These injuries are often type B reactions, subacute or
chronic, and may partly be irreversible. Immunosuppressants and plasmapheresis may be
indicated for treatment.
d. Osmotic nephrosis: Drugs such as mannitol; hydroxyethylstarch; immunoglobulins, especially those
containing sucrose; and dextrans may lead to osmotic nephrosis. In these cases, drug molecules
undergo pinocytosis and cellular accumulation, and the cells become swollen and vacuolated,
leading to tubular lumen obstruction.
e. Crystalluria/nephrolithiasis: Certain medications are insoluble in the urine and can result in
crystalline nephropathy and/or kidney stones. Suggestive diagnostics include crystalluria on urinary
analysis, overt nephrolithiasis, and ultrasound findings indicative of obstruction. Examples of drug
culprits include antibiotics, acyclovir, methotrexate, antiretrovirals, triamterene, and ethylene
glycol–associated calcium oxalate crystal precipitation.
f. Altered electrolyte handling/tubular dysfunction: Drugs may alter how the kidney handles
electrolytes, tubular proteins, and water. Example patterns include the syndrome of inappropriate
antidiuretic hormone, an acquired Fanconi syndrome; renal tubular acidosis; and nephrogenic
diabetes insipidus. Serum and urine electrolyte abnormalities, altered serum and urine osmolality,
and an altered acid-base profile may each indicate this type of DIKD. Examples of drug causes include
antiretrovirals, lithium, selective serotonin reuptake inhibitors, antiepileptics, and vincristine.

G. AKI Survivorship
1. Recently, increased emphasis has been placed on AKI survivor care. AKI survivor care focuses on
patients with an episode of AKI during hospitalization who are being discharged. Individuals have tried
specific AKI survivor clinics to improve comprehensive kidney care follow-up to limit the burden of
CKD.
2. The landmark FUSION randomized controlled trial was recently published (Clin J Am Soc Nephrol
2021;16:1005-14). Patients who survived an episode of stage 2 or 3 AKI were randomized to either
early nephrology follow-up with a bundled care intervention or usual care. Recruitment was slow, and
although process outcomes were improved (more nephrology visits, more kidney function monitoring,
more medication reconciliation and review) clinical outcomes, including the primary outcome, which
was a major acute kidney event at 1 year, were no different. More research is needed to determine
the optimal method for follow-up in AKI survivors that is aligned with best practices and patient-
centeredness.

ACCP Updates in Therapeutics® 2022: Critical Care Pharmacy Preparatory Review and Recertification Course

419
Acute Kidney Injury and Renal Replacement Therapy in the Critically Ill Patient

Patient Case

Questions 1–3 pertain to the following case.


F.B. is a 68-year-old male patient (weight 70 kg) admitted to your ICU with fever, elevated WBC, respiratory
failure requiring mechanical ventilation, and norepinephrine to support his blood pressure. His medical history
is significant for chronic back pain, diabetes, and hypertension. He takes both enalapril and glipizide once daily,
as well as acetaminophen as needed. Before this admission, he had otherwise been healthy. He saw his primary
care provider about 1 week ago. At that time, his blood pressure was 140/80 mm Hg, and his hemoglobin A1C
was 5.2%. His laboratory workup was also unremarkable: WBC 5.0 x 103 cells/mm3, BUN 7 mg/dL, and SCr 0.9
mg/dL. Today, his WBC is 24 x 103 cells/mm3, BUN is 38 mg/dL, and SCr is 3.2 mg/dL, with 325 mL of UOP
since his admission 24 hours ago.

1. Which best describes F.B.’s AKI?


A. RIFLE class R.
B. AKIN stage 1.
C. RIFLE class F or AKIN stage 3.
D. RIFLE class E or AKIN stage 3.

2. Which medication is most likely contributing to his AKI?


A. Enalapril.
B. Glipizide.
C. Acetaminophen.
D. Both enalaparil and glipizide equally.

3. When evaluating F.B.’s potential causes of AKI, which additional information or test would be most impor-
tant to consider or obtain?
A. Rate of loss, symptoms, and coexisting diseases and medications.
B. Renal evaluation using ultrasonography because this can determine the cause of AKI in most patients.
C. Review of blood chemistries because this will likely determine the cause of injury.
D. Renal evaluation using biopsy.

II.  RENAL REPLACEMENT THERAPIES

A. General Approaches to Managing AKI


1. Once injury has occurred, therapy consists of providing supportive care and limiting additional insults,
including nephrotoxins.
2. In patients with shock, adequate fluid resuscitation should be initiated to restore effective circulation
without producing volume overload.
3. No specific pharmacologic therapy is effective in treating or reversing AKI.
4. Metabolic and volume status should be followed closely, and RRT should be initiated when other
approaches have failed.

ACCP Updates in Therapeutics® 2022: Critical Care Pharmacy Preparatory Review and Recertification Course

420
Acute Kidney Injury and Renal Replacement Therapy in the Critically Ill Patient

5. Historically, RRT has been offered when a patient fulfills at least one of the AEIOUs: severe acidosis
(A), electrolyte abnormalities (e.g. hyperkalemia) (E), intoxication (I), refractory volume overload (O),
or symptomatic uremia (U). Recently, the multicenter BICAR-ICU study showed that randomization to
sodium bicarbonate administration for a pH of 7.2 or less and PCO2 of 45 mm Hg or less, rather than no
therapy, failed to improve all-cause mortality or death in ICU patients. However, of note, patients in the
bicarbonate group had a significantly lower rate of new dialysis initiation, suggesting this temporizing
measure has had a secondary benefit and dissuaded dialysis initiation for acidemia or hyperkalemia
while kidneys spontaneously improved.
6. No consensus exists on the optimal timing of RRT. It is unclear whether RRT should be initiated when
a patient reaches KDIGO stage III or earlier to avoid complications associated with metabolic/fluid
derangements. Several recent randomized controlled trials regarding RRT timing have been published
with disparate results. These include the ELAIN and AKIKI trials published in 2016, the IDEAL-ICU
trial published in 2018, and the STARRT-AKI trial published in 2020.
a. STARRT-AKI: Multicenter, multinational trial of 2927 individuals with stage 2 or 3 AKI who
received either “accelerated RRT,” meaning therapy was initiated within 12 hours of meeting
eligibility criteria, or “standard RRT,” meaning therapy was initiated unless the patient fulfilled a
conventional indication (analogous to the AEIOU indications mentioned earlier). Clinicians needed
to attest to equipoise in the decision to initiate RRT before the patient could be enrolled. For the
primary end point of all-cause mortality at 90 days, there was no difference between groups (44%
in each group). Only two-thirds of the patients in the standard RRT group ultimately required
dialysis, whereas 97% of the patients in the accelerated RRT group received dialysis. There were
more adverse events in the accelerated RRT group.
b. At this point, although the findings of these trials somewhat vary, it appears that there is no consistent,
reproducible advantage to beginning RRT early in a patient’s course as opposed to waiting until
stage 3 AKI with an urgent indication.
7. The practice of RRT discontinuation is non-standardized but typically occurs when the precipitant is
improved or resolved (e.g., sepsis controlled) and patients can satisfactorily maintain their fluid and
solute balance without extracorporeal support.

B. Renal Replacement for AKI


1. Modalities are classified according to duration as intermittent (IRRT), prolonged intermittent (PIRRT),
or continuous (CRRT).
2. A defining feature of the type of RRT is the mechanism of solute and water transport through a
semipermeable membrane by convection, diffusion, or both.
a. Convection uses the concept of “solute drag” and can remove both small- (less than 500-1000
Da) and middle- (less than 10,000 Da) molecular-weight solutes. Solute removal occurs when the
transmembrane pressure drives water and solute across a semipermeable membrane. This process
involves adding replacement fluid (pre- and/or post-filter) to replace the excess volume that is being
removed and replenish the desired electrolytes. Pre-filter replacement fluid may improve filter life
and deter circuit clotting. Post-filter replacement fluid may improve solute clearance efficiency
because of the hemoconcentration of blood entering the dialyzer.
b. Diffusion is the movement of solutes from an area of higher solute concentration to an area of
lower concentration. A concentration gradient is produced by running an electrolyte solution (i.e.,
dialysate fluid with a flow rate of 17–40 mL/minute) countercurrent to the flow of blood. Small-
molecular-weight solutes (less than 500 Da) are cleared efficiently.

ACCP Updates in Therapeutics® 2022: Critical Care Pharmacy Preparatory Review and Recertification Course

421
Acute Kidney Injury and Renal Replacement Therapy in the Critically Ill Patient

C. Intermittent Hemodialysis (IHD)


1. IHD is the most common form of RRT used in the acute setting for treating patients with AKI or end-
stage renal disease needing dialysis.
2. Rapid metabolic clearance and/or fluid removal occurs over about 3- to 4-hour sessions.
a. Intradialytic hypotension occurs in up to 30% of patients, which can lead to early dialysis termination
or compromise of residual renal function. Several strategies to improve hemodynamics during IHD
have been suggested, including fluid administration, temperature adjustment, ultrafiltration and
sodium modeling, periprocedural antihypertensive hold, and midodrine and vasopressor therapy.
Selection among these options is patient-specific and should only be done in conjunction with a
careful evaluation of underlying etiologies for hypotension, including infection and cardiac function,
among others.
b. Patients with preexisting hypotension before IHD (e.g., septic shock), may not tolerate these large
fluid and electrolyte shifts and may develop further deterioration of their hemodynamics.
c. In patients with head trauma or hepatic encephalopathy, hypotension can reduce cerebral perfusion.
Changes in solute concentrations can also worsen cerebral edema. Rapid plasma clearance of solutes
acutely decreases their systemic intravascular concentrations while intracranial concentrations
remain unchanged. This relatively high intracranial concentration of solute results in water
transport back across the blood-brain barrier, which contributes to cell swelling, cerebral edema,
and increased intracranial pressure.

D. CRRT
1. CRRT is the most commonly used modality of RRT in hemodynamically unstable ICU patients.
2. CRRT modalities include continuous venovenous hemofiltration (CVVH), continuous venovenous
hemodialysis (CVVHD), and continuous venovenous hemodiafiltration (CVVHDF).
3. SCUF, or slow continuous ultrafiltration, is another type of CRRT that removes fluid without the need
for replacement solutions. This therapy has limited effect on removal of waste products (e.g., BUN) or
electrolytes and cannot correct acid-base abnormalities.
4. Solute clearance during CVVHD occurs by diffusion.
5. Solute clearance during CVVH occurs by convection, and the ultrafiltration rate determines the clearance
rate for most solutes.
6. For CVVHDF, solute removal occurs by both convection and diffusion, with diffusion commonly the
dominant waste removal modality.

E. Prolonged Intermittent RRT (PIRRT)


1. Includes sustained low-efficiency dialysis (SLED), sustained low-efficiency daily diafiltration
(SLEDD-f), extended daily dialysis (EDD), and accelerated venovenous hemofiltration
2. These therapies are a hybrid modality between intermittent and continuous options. PIRRT is typically
delivered using conventional hemodialysis machines with low blood-pump speeds (around 200 mL/
minute) and low dialysate flow rates (around 300 mL/minute) for extended periods: 6–12 hours a day
versus 3–4 hours for IHD or 24 hours for CRRT.
3. Similar to CRRT, they allow for improved hemodynamic stability by producing gradual solute and
volume removal compared with IHD.
4. These therapies have certain advantages over CRRT. They produce high solute clearances, use existing
IHD machines, eliminate the need for external solutions, and allow “time away” when various diagnostic
and therapeutic procedures are needed, liberating health care workers from continuous involvement with
the circuit. Disadvantages include limited data on drug clearance and a period in which solute and fluid
management is paused (when off circuit).

ACCP Updates in Therapeutics® 2022: Critical Care Pharmacy Preparatory Review and Recertification Course

422
Acute Kidney Injury and Renal Replacement Therapy in the Critically Ill Patient

F. Choosing a Mode
1. Data are conflicting regarding the renal replacement mode of choice for critically ill patients.
2. Outcomes such as mortality and renal recovery appear to be no different between IHD and CRRT;
however, most studies are limited by design, patient characteristics, and crossover between different
modalities.

G. CRRT and Drug-Dosing Concepts


1. Drug dosing during CRRT and SLED is often unclear because this information is not included in
package inserts. Manufacturers are not required to study how these therapies alter clearance. General
dosing considerations for CRRT include:
a. For most medications, loading doses require no adjustment.
b. If a drug is normally cleared by the kidneys or is removed by other RRT modalities, CRRT may
significantly affect its removal.
c. When available, drug-specific literature should be used to determine dose and frequency to minimize
the likelihood of dosing errors. Although CRRT is meant to be continuous, it is often interrupted. If
therapy is held for an extended period, dose adjustment may be required.
d. Use caution when extrapolating historical literature to modern practice because current dialyzers
can more readily clear small and middle MW molecules, and flow rates are higher than previously
used. Collectively, these aspects would be expected to increase drug clearance and make historically
recommended doses insufficient when applied to modern practice.
e. With the many variables associated with CRRT delivery (e.g., modality, dialyzers, circuit downtime,
patient instability), therapeutic drug monitoring should be used, when available.
2. RRT modality, dialysis dose (e.g., more frequent IHD, higher dialysis dose/effluent flow rate increase
drug elimination), and drug properties influence medication elimination. Effluent is any fluid leaving
the circuit and not returning to the patient. This is spent fluid and could be dialysate or an ultrafiltrate
from the blood.
3. Drug proprieties that influence removal during CRRT include protein binding, molecular weight (MW),
and volume of distribution (Vd). Drug charge is less important.
a. The ability of a drug to bind to plasma protein (i.e., albumin) greatly influences how it is removed
by CRRT. Removal is inversely proportional to the percent bound (i.e., the higher the percent bound,
the less removed). Protein binding affects removal for both convection and diffusion.
b. Solutes are considered to have a small MW (less than 1000 Da), middle MW (1000-30,000 Da),
or large MW (greater than 30,000 Da). Most unbound drugs have an MW less than 500 Da (e.g.,
cefepime 480 Da). Diffusive clearance (as with IHD and CVVHD) readily removes small MW
drugs to a greater extent than middle MW drugs, whereas convective clearance modalities (as
with CVVH) remove both small and middle MW drugs well. Highly protein bound drugs, as with
albumin binding (64,000 Da), appear to have a high MW because of the association with the plasma
protein. This large size would exceed the typical pore size of a high-flux, high-efficiency dialyzer,
which is standard of care, and lead to reduced drug clearance.
c. Vd is inversely related to extracorporeal removal, wherein a larger Vd decreases elimination through
the circuit because the drug is distributed throughout the body compartments. For CRRT, the slower
rates allow for steady redistribution between body compartments, which can facilitate removal of
drugs with a larger Vd to a greater degree than with IHD.
4. CRRT modalities and their influence during therapy
a. CVVH
i. Solute removal during CVVH is by convection. Convection is influenced by the membrane pore
size; the free fraction of drug, as discussed earlier; and the ultrafiltration rate.

ACCP Updates in Therapeutics® 2022: Critical Care Pharmacy Preparatory Review and Recertification Course

423
Acute Kidney Injury and Renal Replacement Therapy in the Critically Ill Patient

ii. The ability of a substance to pass through a membrane by convection is termed the sieving
coefficient (SC). The SC ranges from 0 to 1. A SC of 1 represents free movement, whereas a SC
of 0 represents no movement across a filter.
iii. SC is best obtained from the primary literature or from patient-specific values. It can be
calculated using a ratio of measured drug or other solute in the ultrafiltrate to its concentration in
the plasma, SC = CUF/Cp, where CUF is concentration in the ultrafiltrate and Cp is concentration
in the plasma.
iv. If a measured SC is not available, it can be estimated using the percent unbound to plasma
protein, SC = 1 − fb, where fb is fraction bound (i.e., percent protein bound).
v. Site of replacement fluid administration can influence solute clearance. Adding replacement
fluids pre-filter will dilute the blood entering the dialyzer. This diluted blood will have
decreased drug concentrations; thus, less drug will be removed than if the replacement solution
is infused after the filter.
If replacement fluids are administered postfilter (i.e., post-dilution), the clearance rate can be
estimated using the following equation:
CVVHpost-dilution = QUF x SC
If pre-dilution (i.e., before the filter) fluids are used, clearance across the membrane is reduced.
Clearance can be estimated using the following equation:
CVVHpre-dilution = QUF x SC x Qb /(Qb + Qrf )
where QUF is ultrafiltration flow rate, Qb is blood flow rate, and Qrf is pre-dilution replacement
fluid flow rate. For pre-dilution fluid replacement to affect overall clearance, the rate must be
high.
b. CVVHD
i. Solute removal during CVVHD occurs by passive diffusion. The flow of dialysate is
countercurrent to that of the blood. Movement of solute across the semipermeable membrane
occurs because of a concentration gradient, with movement from an area of higher concentration
(blood) to an area of lower concentration (dialysate). This process occurs until equilibrium is
established.
ii. Small substances (e.g., urea with an MW of 60 Da) are cleared more rapidly than larger
substances (e.g., drugs with an MW greater than 500 Da).
iii. The ability of a drug to cross the dialysis filter during CVVHD is called the saturation
coefficient (SA). Equations exist for estimating the SA when published data are unavailable. It
can be calculated as SA = CE /Cp, where CE is the concentration in the effluent (spent dialysate)
fluid and Cp is the concentration in plasma. The CVVHD clearance can then be approximated
by Qd x SA, where Qd is the dialysate flow rate.
c. CVVHDF
i. Solute removal during CVVHDF is by diffusion and convection (i.e., both dialysate and
replacement fluids are used).
ii. Clearances of small substances are about equal to the sum of the clearance from CVVH and
CVVHD separately. However, as MW increases, this correlation no longer holds true.
iii. Clearance is estimated as CVVHDF = (QUF + Qd) x SA.
d. PIRRT
i. Limited literature is available to guide drug dosing during PIRRT modalities such as SLED
and EDD.

ACCP Updates in Therapeutics® 2022: Critical Care Pharmacy Preparatory Review and Recertification Course

424
Acute Kidney Injury and Renal Replacement Therapy in the Critically Ill Patient

ii. Extended dialytic therapies of 8-12 hours may alter drug dosing intervals. For example, drugs
with frequent dosing intervals (e.g., time-dependent antibiotics like β-lactams) may need to be
administered more often during the run, given by extended infusion, or repeated/readministered
after the PIRRT treatment.
iii. Solute removal during SLED/EDD is greater than that during CVVHD when estimated over the
same time interval because higher dialysis flow rates are used during SLED/EDD treatments.
iv. General dosing considerations for SLED
(a) The duration of SLED and its flow rates (dialysate, blood) vary between studies and
institutions, making a general approach to dosing problematic.
(b) In addition, little information is available to guide drug dosing.
(c) Like IHD and CRRT, the most important factors determining drug removal are protein
binding, water solubility, MW (less than 500 kDa), and Vd (less than 0.8–1 L/kg); however,
attention should also be given to the timing of drug administration relative to the PIRRT
treatment.

Patient Case

Questions 4–6 pertain to the previous case.


F.B. is a 68-year-old man (weight 70 kg) admitted to your ICU with fever, elevated WBC, respiratory failure requir-
ing mechanical ventilation, and norepinephrine to support his blood pressure. His medical history is significant
for chronic back pain, diabetes, and hypertension. He takes both enalapril and glipizide once daily, as well as
acetaminophen as needed. Before this admission, he had otherwise been healthy, seeing his primary care provider
about 1 week ago. At that time, his blood pressure was 140/80 mm Hg, and his A1C was 5.2%. His laboratory
workup was also unremarkable: WBC 5.0 x 103 cells/mm3, BUN 7 mg/dL, and SCr 0.9 mg/dL. Today, his WBC is
24 x 103 cells/mm3, BUN 38 mg/dL, and SCr 3.2 mg/dL, with 325 mL of UOP since his admission 24 hours ago. It
is determined that F.B. needs RRT to manage his volume and control his metabolic derangements. He is currently
receiving norepinephrine with a mean arterial pressure of 65 mm Hg.

4. Which renal replacement mode will most likely be chosen?


A. IHD.
B. Slow-low EDD.
C. CRRT.
D. Peritoneal dialysis.

5. F.B. will be initiated on a new extended-spectrum cephalosporin with limited information available for its
use during CRRT. Given the following parameters, estimate the drug’s SC: MW 480 Da, positively charged
drug, Vd 1.9 L/kg, protein binding 12%.
A. 0
B. 0.52
C. 1.9
D. 0.88

ACCP Updates in Therapeutics® 2022: Critical Care Pharmacy Preparatory Review and Recertification Course

425
Acute Kidney Injury and Renal Replacement Therapy in the Critically Ill Patient

Patient Case (continued)

6. Given the calculated SC, estimate the new cephalosporin’s clearance during CRRT if the prescription is
CVVH, blood flow 200 mL/minute, ultrafiltration rate 2000 mL/hour, 100% pre-filter replacement fluids,
and use of a high-flux, high-efficiency dialyzer.
A. 0.9 L/hour.
B. 1.5 L/hour.
C. 3.3 L/hour.
D. 5.3 L/hour.

Special acknowledgment to Michael L. Bentley,


Pharm.D., for his contribution to this chapter.

ACCP Updates in Therapeutics® 2022: Critical Care Pharmacy Preparatory Review and Recertification Course

426
Acute Kidney Injury and Renal Replacement Therapy in the Critically Ill Patient

REFERENCES

Acute Kidney Injury 9. Mehta RL, Awdishu L, Davenport A, et al.


1. Barlam TF, Cosgrove SE, Abbo LM, et al. Phenotype standardization for drug-induced kid-
Implementing an antibiotic stewardship pro- ney disease. Kidney Int 2015;88:226-34.
gram: guidelines by the Infectious Diseases 10. Ostermann M, Bellomo R, Burdmann EA, et
Society of America and the Society for Healthcare al. Controversies in acute kidney injury: conclu-
Epidemiology of America. Clin Infect Dis sions from a Kidney Disease: Improving Global
2016;62:e51-77. Outcomes (KDIGO) Conference. Kidney Int
2. Bellomo R, Ronco C, Kellum JA, et al. Acute 2020;98:294-309.
Dialysis Quality Initiative Workgroup. Acute renal 11. O’Sullivan ED, Doyle A. The clinical utility of
failure—definition, outcome measures, animal kinetic glomerular filtration rate. Clin Kidney J
models, fluid therapy and information technol- 2017;10:202-8.
ogy needs: the Second International Consensus 12. Pais GM, Liu J, Avedissian SN, et al. Lack of syn-
Conference of the Acute Dialysis Quality Initiative ergistic nephrotoxicity between vancomycin and
(ADQI) Group. Crit Care 2004;8:R204-12. piperacillin/tazobactam in a rat model and a con-
Original report describing the RIFLE criteria and firmatory cellular model. J Antimicrob Chemother
providing a rationale for their conception. 2020;75:1228-36.
3. Bentley ML, Corwin HL, Dasta J. Drug-induced 13. Schreier DJ, Kashani KB, Sakhuja A, et al.
acute kidney injury in the critically ill adult: rec- Incidence of acute kidney injury among critically
ognition and prevention strategies. Crit Care Med ill patients with brief empiric use of anti-pseudo-
2010;38:S169-74. Provides a good overview of the monal beta-lactams with vancomycin. Clin Infect
mechanisms of drug-induced kidney injury, recog- Dis 2018. [Epub ahead of print]
nition of AKI, and AKI prevention. 14. Silver S, Adhikari NK, Chan C, et al. Nephrologist
4. Chen S. Retooling the creatinine clearance equa- follow-up versus usual care after an acute kidney
tion to estimate kinetic GFR when the plasma injury hospitalization (FUSION). Clin J Am Soc
creatinine is changing acutely. J Am Soc Nephrol Nephrol 2021;16:1005-14.
2013;24:877-88. 15. Uchino S, Kellum JA, Bellomo R, et al. Acute renal
5. Göcze I, Jauch D, Götz M, et al. Biomarker- failure in critically ill patients: a multinational,
guided intervention to prevent acute kidney injury multicenter study. JAMA 2005;294:813-8. A pro-
after major surgery: the prospective randomized spective, observational, multinational study of ICU
BigpAK study. Ann Surg 2018;267:1013-20. patients to determine the prevalence of acute renal
6. Kane-Gill SL. Nephrotoxin stewardship. Crit Care failure; characterize its etiology, illness severity,
Clin 2021;37:303-20. and clinical practice; and investigate the impact of
7. Kidney Disease: Improving Global Outcomes these differences on patient outcomes.
(KDIGO) Acute Kidney Injury Work Group. 16. Zarbock A, Kullmar M, Ostermann M, et al.
KDIGO clinical practice guideline for acute kidney Prevention of cardiac surgery–associated acute
injury. Kidney Int Suppl 2012;2:1-138. Updated kidney injury by implementing the KDIGO guide-
clinical practice guideline for AKI. lines in high-risk patients identified by biomarkers:
8. McCullough PA, Bouchard J, Waikar SS, et al. the PrevAKI-multicenter randomized controlled
Implementation of novel biomarkers in the diag- trial. Anesth Analg 2021;133:292-302
nosis, prognosis, and management of acute kidney
injury: executive summary from the tenth con- Renal Replacement Therapies
sensus conference of the Acute Dialysis Quality 1. Bagshaw SM, Wald R, Adhikari N, et al. Timing
Initiative (ADQI). Contrib Nephrol. 2013;182:5- of initiation of renal-replacement therapy in acute
12. Summarizes the role for novel biomarkers in kidney injury. N Engl J Med 2020;383:40-51.
AKI assessment. 2. Bogard KN, Peterson NT, Plumb TJ, et al.
Antibiotic dosing during sustained low-efficiency
dialysis: special considerations in adult critically ill

ACCP Updates in Therapeutics® 2022: Critical Care Pharmacy Preparatory Review and Recertification Course

427
Acute Kidney Injury and Renal Replacement Therapy in the Critically Ill Patient

patients. Crit Care Med 2011;39:560-70. Addresses


issues of antibiotic dosing during SLED and provides
an experience with it.
3. Heintz BH, Matzke GR, Dager WE. Antimicrobial
dosing concepts and recommendations for criti-
cally ill adult patients receiving continuous renal
replacement therapy or intermittent hemodialysis.
Pharmacotherapy 2009;29:562-77. Provides a critical
review of current literature related to the influence
of CKD and AKI on the pharmacokinetic and phar-
macodynamic properties of antimicrobial agents and
suggests dosing strategies for select antimicrobial
agents.
4. Jaber S, Paugam C, Futier E, et al. Sodium bicar-
bonate therapy for patients with severe metabolic
acidemia in the intensive care unit (BICAR-ICU):
a multicenter, open-label, randomized controlled,
phase 3 trial. Lancet 2018;392:31-40.
5. Tolwani A. Continuous renal-replacement therapy for
acute kidney injury. N Engl J Med 2012;367:2505-14.
A good overview of solute clearance during CRRT;
lists indications and contraindications and clinical
use.

ACCP Updates in Therapeutics® 2022: Critical Care Pharmacy Preparatory Review and Recertification Course

428
Acute Kidney Injury and Renal Replacement Therapy in the Critically Ill Patient

ANSWERS AND EXPLANATIONS TO PATIENT CASES

1. Answer: C or a switch to an alternative therapy. Although PIRRT


Both RIFLE class “F” and AKIN stage 3 are met methods like SLED or EDD are an option, some form of
using similar SCr and UOP criteria. For this case, the CRRT would likely be chosen because of this patient’s
SCr increased by at least 3-fold above baseline, and unfavorable hemodynamics. Continuous renal replace-
the patient’s UOP was less than 0.3 mL/kg/hour for 24 ment therapies such as CVVH, CVVHD, and CVVHDF
hours. Although A and B are true, the worse values in are often used because they allow for slower flow rates
the RIFLE class and AKIN staging systems should be and improved hemodynamics. However, there is no clear
chosen when determining the class and stage of AKI. benefit with one therapy over another.
Answer D is incorrect because his renal indices have not
been present for at least 3 months. Collectively, both the 5. Answer: D
RIFLE and the AKIN criteria have been included in the Protein binding primarily determines drug clearance
KDIGO criteria, which is the current standard definition. during RRT. Other factors that contribute to drug clear-
ance include low MW and low Vd. Drug charge leading
2. Answer: A to dialyzer adhesion has only a limited effect on drug
This patient has severe sepsis that led to decreased renal clearance during CRRT and has been poorly stud-
perfusion, causing AKI. Enalapril likely contributed to ied. When available, SC should be identified from the
the injury by altering renal hemodynamics (Answer A is package insert information or the primary literature or
correct). Neither glipizide nor acetaminophen is likely to calculated from the concentration of the solute in the
cause AKI (Answers B–D are incorrect). ultrafiltrate divided by the concentration of the solute
in the plasma. When unavailable, SC can be estimated
3. Answer: A as SC = 1 − fb. In this case, the SC = 1 – 0.12 = ~0.88
Acute kidney injury cannot currently be diagnosed (Answer D is correct; Answers A–C are incorrect).
with a specific blood test or imaging study (Answer C
is incorrect). The degree of kidney dysfunction and the 6. Answer: B
patient’s symptoms and coexisting diseases may provide The general approach to calculating clearance during
clues regarding the etiology of the patient’s kidney injury CVVH multiplies the SC (the degree to which a sub-
(Answer A is correct). Drug-induced AKI is most com- stance is able to pass through the membrane) by the
mon in the setting of additional insults and may occur ultrafiltration rate. If predilution (i.e., before the fil-
even after the drug is discontinued. Given a patient’s ter) fluids are used, clearance across the membrane is
history and presentation as well as a patient’s physical reduced, which must be accounted for in the formula.
examination, a clinician may pursue additional testing Clearance in that case can be estimated using the follow-
to find the DIKD phenotype. Serum markers (e.g., SCr, ing equation: CVVHpre-dilution = QUF x SC x [Qb /(Qb
UOP) are often nonspecific (Answer C is incorrect); + Qrf )] (where QUF is ultrafiltration flow rate (liters per
kidney biopsy is only warranted for certain DIKD phe- hour), Qb is blood flow rate [converted to liters per hour
notypes (e.g., AIN, glomerular diseases) and is often for consistent units], and Qrf is predilution replacement
not pursued in the acute setting (Answer D is incorrect). fluid flow rate, as stated earlier). In this case, clear-
Renal ultrasonography would be helpful for nephroli- ance = 2 L/hr x 0.88 x [12 L/hr/(12 L/hr + 2 L/hr)] =
thiasis/crystalluria phenotypes or evidence of postrenal 1.5 L/hr (Answer B is correct; Answers A, C, and D are
AKI not alleviated with urinary catheter placement incorrect).
(Answer B is incorrect).

4. Answer: C
Intermittent hemodialysis is often used in critically ill
patients because many physicians are familiar with this
therapy; however, about 20%–30% of patients receiving
IHD become hypotensive and require discontinuation

ACCP Updates in Therapeutics® 2022: Critical Care Pharmacy Preparatory Review and Recertification Course

429
Acute Kidney Injury and Renal Replacement Therapy in the Critically Ill Patient

ANSWERS AND EXPLANATIONS TO SELF-ASSESSMENT QUESTIONS

1. Answer: A is not the primary route of clearance during CVVH


The drug-induced kidney disease phenotype most likely (Answer D is incorrect).
associated with these antibiotics is AIN from ceftriax-
one. Acute interstitial nephritis is considered under the 5. Answer: A
AKI phenotype (Answer A is correct). Azithromycin Answer A is correct. Protein binding is a primary deter-
would unlikely cause drug-induced kidney disease. minant of drug clearance during CRRT. Only unbound
Although AIN is an overall uncommon cause of DIKD, drug can be cleared through the circuit; therefore, an
in 75% of AIN cases, one or more drugs are involved. increase in protein binding while other factors remain
Several medications have been associated with this type stable would lead to reduced drug elimination (Answers
of kidney disease, including penicillins, cephalospo- B and C are incorrect). A high Vd would reduce drug
rins, sulfonamides, ciprofloxacin, vancomycin, NSAIDs clearance, but this is generally more significant in IHD
and COX-2 inhibitors, proton pump inhibitors, immune than in CRRT (Answer D is incorrect) because there is
checkpoint inhibitors, antiepileptic drugs, ranitidine, little time during the 3- to 4-hour IHD run for a drug to
diuretics, and cocaine. This patient may have other con- redistribute out of the peripheral tissues.
tributing factors to AKI, including sepsis and shock.
Glomerular disease, nephrolithiasis, and tubular dys-
function have not been associated with cephalosporins
(Answers B-D are incorrect).

2. Answer: C
Stage 3 AKI is met by both the SCr criteria (3 x baseline)
and the need for RRT (Answer C is correct; Answers
A and B are incorrect). Urinary output is not needed to
stage the kidney dysfunction in this case because the
patient fulfills other criteria (Answer D is incorrect).

3. Answer: C
Answer C is correct. Drug-dosing recommendations for
IHD can be found in many resources. However, dosing
during CRRT and SLED/EDD is less clear. Primary
literature and/or summary tables for CRRT and SLED
should be referenced because these recommendations
are not usually found in other sources (Answers A and
B are incorrect). Use caution to ensure that identical
modes of CRRT are referenced with similar flow rates.
Drug dosing using a sieving coefficient should only be
used when a review of the literature fails to provide spe-
cific drug adjustment recommendations (Answer D is
incorrect).

4. Answer: A
Solute removal during CVVH is by convection, which
is primarily influenced by membrane pore size, free
fraction of drug, and ultrafiltration rate. Diffusion is the
process of clearance during CVVHD making Answers
B and C incorrect. Membrane binding does occur but

ACCP Updates in Therapeutics® 2022: Critical Care Pharmacy Preparatory Review and Recertification Course

430
Management of Pain,
Agitation/Sedation, Delirium,
Immobility, and Sleep Disruption
and Neuromuscular Blockade in
Adult Intensive Care Unit Patients
Joanna L. Stollings, Pharm.D., FCCP, FCCM, BCPS, BCCCP
Vanderbilt University Medical Center
Nashville, Tennessee
Management of PADIS and NMB in Adult Intensive Care Unit Patients

Management of Pain,
Agitation/Sedation, Delirium,
Immobility, and Sleep Disruption
and Neuromuscular Blockade in
Adult Intensive Care Unit Patients
Joanna L. Stollings, Pharm.D., FCCP, FCCM, BCPS, BCCCP
Vanderbilt University Medical Center
Nashville, Tennessee

ACCP Updates in Therapeutics® 2022: Critical Care Pharmacy Preparatory Review and Recertification Course

433
Management of PADIS and NMB in Adult Intensive Care Unit Patients

Learning Objectives Self-Assessment Questions


Answers and explanations to these questions may be
1. Develop a management strategy for the preven- found at the end of this chapter.
tion and treatment of pain, agitation/sedation, and
delirium, immobility, and sleep disruption (PADIS) 1. P.J. has been receiving propofol 50–60 mcg/kg/
in an intensive care unit (ICU) patient with various minute and fentanyl 75–100 mcg/hour for 4 days.
comorbidities. She has no significant medical history. Laboratory
2. Discuss relevant pharmacokinetic and pharmacody- results today show that transaminases have
namic considerations of PADIS medications as they increased to 5 times baseline, lactate is increased to
pertain to disturbances in critical care physiology. 5 mmol/L, and triglyceride concentration is 450 mg/
3. Identify relevant adverse effects, drug interactions, dL. No new medications have been added. Given
and drug withdrawal syndromes in the management these laboratory values, which complication would
of PADIS. be most appropriate to address?
4. Evaluate patients in the ICU for PADIS using a vali- A. Deep venous thrombosis (DVT).
dated screening tool. B. Critical illness-induced polyneuropathy.
5. Construct a plan for the management of delirium. C. Intensive care unit (ICU) delirium.
6. Identify the long-term effects of critical illness in D. Propofol-related infusion syndrome (PRIS).
adult ICU patients.
7. Create a management strategy for PADIS-related 2. T.I. is a 35-year-old man admitted to the ICU for
medications that are continued beyond ICU dis- severe alcohol withdrawal. His medical history is
charge. otherwise unknown. Laboratory values are within
8. Describe a treatment and monitoring plan for criti- normal limits on admission. He has been receiv-
cally ill patients receiving neuromuscular blockade. ing a lorazepam infusion 6–8 mg/hour for active
alcohol withdrawal. On day 4, his blood pressure is
130/75 mm Hg, oxygen saturation is 98% on 2 L
Abbreviations in This Chapter of oxygen, blood urea nitrogen (BUN) is 50 mg/dL,
and serum creatinine (SCr) is 2.0 mg/dL; he has a
ARDS Acute respiratory distress syndrome new anion gap of 20 mEq/L, an osmolar gap of 18
BPS Behavioral Pain Scale mmol/L H2O, and a fractional excretion of sodium
CAM-ICU Confusion assessment method for the of 0.2. Which is the most likely cause for his clinical
intensive care unit presentation?
CPOT Critical-Care Pain Observation Tool
A. Acute respiratory distress syndrome (ARDS).
GABA γ-Aminobutyric acid
B. Propylene glycol toxicity.
ICDSC Intensive Care Delirium Screening
C. Delirium tremens.
Checklist
D. Acute tubular necrosis.
ICP Intracranial pressure
ICU Intensive care unit
3. R.B. is a 25-year-old man admitted to the ICU for
NMBA Neuromuscular blocking agent
acute pancreatitis and sepsis. He is intermittently
PAD Pain, agitation, and delirium
agitated on hydromorphone 2 mg/hour and mid-
PADIS Pain, agitation/sedation, delirium,
azolam 6 mg/hour (Richmond Agitation-Sedation
immobility, and sleep disruption
Scale [RASS] score of –1 to +2) and is not oxygen-
PRIS Propofol-related infusion syndrome
ating adequately after adjustments on the ventilator.
RASS Richmond Agitation Sedation Scale
The physician would like to initiate therapeutic
SAS Sedation-Agitation Scale
paralysis. Which is the next best step in the treat-
SAT Spontaneous awakening trial
ment of this patient?
SBT Spontaneous breathing trial
SCCM Society of Critical Care Medicine
TOF Train of four

ACCP Updates in Therapeutics® 2022: Critical Care Pharmacy Preparatory Review and Recertification Course

434
Management of PADIS and NMB in Adult Intensive Care Unit Patients

A. Spontaneous awakening and breathing trials. 7. H.F., a 65-year-old man admitted to the ICU from
B. Cisatracurium infusion. home for aspiration pneumonia requiring intubation,
C. Intermittent vecuronium. is initiated on levofloxacin and metronidazole. Other
D. Sedate the patient to a “deeply sedated” clini- medications include fentanyl and dexmedetomidine
cal state. infusions as well as amiodarone and quetiapine
given enterally, which are home medications. His
4. P.V. is a 70-year-old woman (weight 50 kg, last RASS was -2), and he has intermittent agitation.
decreased from 60 kg 2 months ago) admitted to the Vital signs and laboratory values are normal, and
ICU in ARDS. She has a history of cirrhosis and corrected QT (QTc) is 500 milliseconds. The team
is currently fluid overloaded (net positive 5 L). She has implemented nonpharmacologic delirium man-
has been on a continuous infusion of fentanyl and agement measures. Which is the most appropriate
propofol for 5 days. Which pharmacologic factor recommendation at this time?
would best be considered with respect to her analge- A. Increase quetiapine for agitation, and monitor
sics or sedatives? QTc.
A. Risk of PRIS in patients with ARDS. B. Change levofloxacin and metronidazole to
B. Unpredictable clearance of fentanyl. piperacillin-tazobactam.
C. Enzymatic induction of fentanyl by propofol. C. Discontinue amiodarone and quetiapine
D. Hypocalcaemia secondary to extended use of because of his prolonged QTc.
propofol. D. Give lorazepam as needed for agitation.

5. L.B. is a 38-year-old woman intubated in the neu- 8. S.V., a 70-year-old woman with a history of hyper-
rosurgery ICU for 72 hours receiving propofol. The tension, is transferred from the floor to the ICU for
nurse is requesting medications for “severe agitation worsening pneumonia and new-onset hypoactive
and hallucinations.” Her heart rate and blood pres- delirium. She has been nil per os (NPO) since admis-
sure have steadily increased since admission, and sion 3 days prior. She remains febrile (temperature
a chart review reveals years of chronic pain while 102°F [38.89°C]) with decreased urine output; other
receiving oxycodone and tramadol at home. Her vital signs and laboratory values are within normal
laboratory values are normal, but she is not toler- limits. Her medications include ceftriaxone, hepa-
ating enteral-route medications. Which is the most rin, and hydrochlorothiazide. Which best represents
appropriate recommendation at this time? the most important set of considerations regarding
A. Quetiapine as needed for agitation. her delirium?
B. Fentanyl infusion. A. Dementia and sleep disorder.
C. Lorazepam as needed for agitation. B. Undetected alcohol withdrawal.
D. Hydromorphone patient-controlled analgesia. C. Adrenal insufficiency.
D. Dehydration and untreated infection.
6. S.P. has just been intubated in the ICU and is in
severe alcohol withdrawal. He has a history of
frequent delirium tremens and alcohol withdrawal
seizures. Which medication is most appropriate to
begin initial management of pain, agitation, and
delirium (PAD) in this patient?
A. Dexmedetomidine.
B. Phenytoin.
C. Fentanyl.
D. Midazolam.

ACCP Updates in Therapeutics® 2022: Critical Care Pharmacy Preparatory Review and Recertification Course

435
Management of PADIS and NMB in Adult Intensive Care Unit Patients

BPS Critical Care Pharmacy Specialist Examination Content Outline

This chapter covers the following sections of the Critical Care Pharmacy Specialist Examination Content Outline:
1. Domain 1: Clinical Knowledge and Application
a. Task 1: 1–3
b. Task 2: 1–7
c. Task 3: 1–7
d. Task 4: 1–6
e. Task 5: 1, 3, 5, 6
f. Task 6: 1–4
g. Task 7: 1–7
h. Task 8: 1, 2
2. Domain 2: Practice Management, Policy, and Quality Improvement
a. Task 1: 1–3
b. Task 5: 1, 2, 4
3. Domain 3: Evidence-Based Medicine, Scholarship, Education, and Professional Development
a. Task 1: 3
b. Task 3: 1, 2
c. Task 4: 1, 2

ACCP Updates in Therapeutics® 2022: Critical Care Pharmacy Preparatory Review and Recertification Course

436
Management of PADIS and NMB in Adult Intensive Care Unit Patients

I.  PAIN, AGITATION/SEDATION, DELIRIUM, IMMOBILITY, AND SLEEP DISRUPTION (PADIS) IN


THE INTENSIVE CARE UNIT

A. Background
1. The Society of Critical Care Medicine (SCCM) published updated guidelines for the management of
PADIS in adult ICU patients in 2018 (Crit Care Med 2018;46:e825-e873). These guidelines, together
with recently published research, help guide ICU clinicians in the challenging task of optimizing
patient comfort and outcomes while avoiding the complications of under- or oversedation. The PADIS
guidelines were written by 32 international experts, four methodologists, and four critical illness
survivors who met virtually monthly and annually at annual SCCM congresses. Rigorous research has
developed our understanding of the assessment tools and medications used for PADIS, the prevention
and treatment methods used for PADIS, and the long-term effects of the ICU environment on patients
and caregivers. Recommendations for specific ICU populations such as burn, neurologic, neurosurgical
(including traumatic brain injury), and cardiac populations may need specialized consideration.
2. The PADIS guidelines add to the 2013 pain, agitation, and delirium (PAD) guidelines by:
a. Adding two critical care topics: rehabilitation/mobilization and sleep disruption
b. Including patients as collaborators and coauthors
c. Inviting an international panel of experts from high-income countries to incorporate more diverse
practices and expertise from the global critical care community
3. Each section of the 2018 PADIS guidelines was represented by content experts, methodologists, and
ICU survivors. Population, Intervention, Comparison, and Outcome and nonactionable descriptive
questions were developed by each section. Members of the guideline group voted on their ranking,
and patients voted on their perceived importance. According to the Grading of Recommendations
Assessment, Development, and Evaluation principles, every Population, Intervention, Comparison, and
Outcomes question was evaluated by members of each section to determine the best level of evidence,
assess quality, and determine recommendations as “strong,” “conditional,” or “good” practice. Evidence
gaps and clinical caveats were explicitly identified.

B. Pharmacy Intervention – Pharmacists provide unique and valuable insight into the management of PADIS
in the ICU. Much of the management for PADIS involves medications with complex pharmacologic profiles,
allowing many opportunities for pharmacy expertise on the critical care team. As the management of
PADIS in the ICU continues to evolve, pharmacists should seek avenues for contributing to the critical care
community through development of hospital protocols and assessing for quality improvement; providing
education for medical, pharmacy, and nursing colleagues; and/or doing research on pertinent questions
surrounding the management of PADIS in the ICU (Crit Care Med. 2020;48:e813-e834).

II.  PAIN IN THE INTENSIVE CARE UNIT

A. Introduction
1. More than half of ICU survivors report severe pain as the most traumatic memory of their ICU stay.
2. Both short- and long-term negative sequelae are related to uncontrolled pain in the ICU. Assessing
pain in the ICU is challenging, particularly in patients who cannot effectively communicate. If patients
cannot adequately communicate their degree of pain but retain motor activity, medications should be
titrated according to validated behavioral pain scales.

ACCP Updates in Therapeutics® 2022: Critical Care Pharmacy Preparatory Review and Recertification Course

437
Management of PADIS and NMB in Adult Intensive Care Unit Patients

B. Incidence and Causes of Pain: Pain may occur in any type of ICU patient, and considerations for pain
management often require an individualized approach to optimize treatment. The interdisciplinary team
should complete a comprehensive review of all variables such as acute and chronic pain, routine nursing
care that may cause discomfort, and procedural-based pain.
1. Common causes of pain in the ICU include, but are not limited to, acute trauma, injury or burns,
postoperative pain, exacerbation of chronic pain, heart disease, ischemia, acute or chronic underlying
disease state pain such as cancer pain, pancreatitis, or other abdominal pathology.
2. Less discernible causes of pain may include those from either routine nursing care or the provision of
life-sustaining measures: presence of an endotracheal tube and endotracheal tube suctioning, wound
care, tube or Foley insertion, immobility, bed repositioning, bathing, medication administration, and
physical and occupational therapy. Other examples of painful invasive procedures include intravenous
line placement, endoscopy and bronchoscopy, chest tube placement or removal, paracentesis, lumbar
puncture, biopsies, and fracture reductions.

C. Short- and Long-term Consequences of Pain in the ICU


1. Acute pain can invoke a stress response, resulting in a hypercatabolic state, decreased tissue perfusion,
and impaired wound healing. Uncontrolled pain decreases a patient’s immune response to infection by
suppressing natural killer cell activity and neutrophil function.
2. Long-term studies (12 months post-ICU stay) report detrimental physiologic and psychological function
in patients who recall significant pain during their hospitalization, particularly in patients admitted with
a traumatic injury (Lancet Respir Med 2014;2:369-79).
a. Health-related quality of life is decreased in up to 20% of patients.
b. Chronic pain is reported in up to 40% of patients.
c. Posttraumatic stress disorder is reported in 5%–20% of patients.

D. Assessment of Pain
1. The gold standard for assessing pain remains the patient’s self-report of pain. Several scenarios in the
ICU make the self-reporting of pain challenging for clinicians (e.g., mechanical ventilation, presence
of sedation and/or delirium). SCCM guidelines currently recommends two validated behavioral pain
scales to be done in a repetitive and routine manner: the Behavioral Pain Scale (BPS) (Table 1) and the
Critical-Care Pain Observation Tool (CPOT) (Table 2).
a. Assessment scales should be used routinely in all ICU patients. Most nursing protocols assess
pain every 4–6 hours while the patient is awake. In addition, it is important to reassess the degree
of pain within about 30 minutes to 1 hour after administering an “as-needed” pain medication to
determine the appropriateness of the pain medication or dose.
b. Pain scores should be documented in the medical chart and then used to help formulate daily
titrations in pain medications.
c. Patients should be treated within 30 minutes of a “significant pain” score. A BPS greater than 5 or
a CPOT score of 3 or greater, or an NRS of 4 or greater is indicative of pain.
2. The use of vital signs alone is not recommended for assessing pain in the ICU patient. Abnormal vital
signs such as tachycardia and hypertension are appropriate for use as a prompt to further investigate
the need for pain control.
3. Further research is needed to determine the effectiveness of a preprocedural pain assessment tool and
the ways in which this assessment will affect analgesic administration. A study by Puntillo et al. in
2014 found the procedures most likely to double the patient’s pain intensity score (from preprocedure to
during-procedure scoring) were chest tube removal, wound drain removal, and arterial line insertion.
This study found that higher-intensity pain and pain distress before the procedure were associated with
a high risk of increased pain during the procedure (Am J Respir Crit Care Med 2014;189:39-47).

ACCP Updates in Therapeutics® 2022: Critical Care Pharmacy Preparatory Review and Recertification Course

438
Management of PADIS and NMB in Adult Intensive Care Unit Patients

4. Kanji et al. found that the CPOT is a valid pain assessment in noncomatose, delirious adult ICU patients
who are not able to reliably self-report the absence or presence of pain (Crit Care Med 2016;44:943-7).

Table 1. Behavioral Pain Scale (BPS)a


Item Description Score
Relaxed 1
Partly tightened (e.g., brow lowering) 2
Facial expression
Fully tightened (e.g., eyelid closing) 3
Grimacing 4
No movement 1
Partly bent 2
Upper limbs
Fully bent with finger flexion 3
Permanently retracted 4
Tolerating movement 1
Coughing but tolerating ventilation
Compliance with 2
most of the time
ventilation
Fighting ventilator 3
Unable to control ventilation 4
a
A BPS score > 5 indicates significant pain.
Adapted with permission from: Lippincott Williams & Wilkins/Wolters Kluwer Health. Payen J, Bru O, Bosson J, et al. Assessing pain in critically ill sedated patients
by using a behavioral pain scale. Crit Care Med 2001;29:2258-63.

Table 2. Critical-Care Pain Observation Tool (CPOT)a


Indicator Description Score
No muscular tension observed Relaxed, neutral 0
Presence of frowning, brow lowering, 
orbit
Tense 1
Facial expression tightening, and levator contraction
All of the above facial movements plus eyelids
Grimacing 2
tightly closed
Does not move at all (does not necessarily mean
Absence of movement 0
absence of pain)
Slow, cautious movements; touching or rubbing the
Protection 1
Body movements pain site; seeking attention through movements
Pulling tube, trying to sit up, moving limbs/
thrashing, not following commands, striking at Restlessness 2
staff, trying to climb out of bed
Muscle tension No resistance to passive movements Relaxed 0
Evaluation by passive Resistance to passive movements Tense, rigid 1
flexion and extension of Strong resistance to passive movements, inability
upper extremities Very tense or rigid 2
to complete them
Tolerating ventilator or
Alarms not activated, easy ventilation 0
Compliance with 
the movement
ventilator 
(intubated Alarms stop spontaneously Coughing but tolerating 1
patients) Asynchrony: Blocking ventilation, alarms often
Fighting ventilator 2
activated

ACCP Updates in Therapeutics® 2022: Critical Care Pharmacy Preparatory Review and Recertification Course

439
Management of PADIS and NMB in Adult Intensive Care Unit Patients

Table 2. Critical-Care Pain Observation Tool (CPOT)a (continued)


Indicator Description Score
Talking in normal tone
OR Talking in normal tone or no sound 0
or no sound
Vocalization 
(extubated
Sighing, moaning Sighing, moaning 1
patients)
Crying out, sobbing Crying out, sobbing 2
Total, range 0–8
a
A CPOT score ≥ 3 indicates significant pain.
Adapted with permission from: Lippincott Williams and Wilkins/Wolters Kluwer Health. Gelinas C, Fillion L, Puntillo K, et al. Validation of the Critical-Care Pain
Observation Tool in adult patients. Am J Crit Care 2006;15:420-7.

E. Prevention and Treatment of Pain in the ICU


1. In a patient whose pain is inadequately controlled in the ICU, intravenous opioids are considered first-
line treatment for nonneuropathic pain. The PADIS guidelines suggest acetaminophen as an adjunct
to an opioid to decrease pain intensity and opioid consumption for pain management in critically ill
patients.
2. Preprocedural pain management should be considered in all ICU patients. One study reported that up
to 60% of patients did not receive preprocedural systemic pain medication for common procedures
and wound care in the ICU, although 89% of patients received a topical anesthetic for central venous
catheter placement (Am J Crit Care 2002;11:415-29).
a. The PADIS guidelines suggest using an opioid at the lowest effective dose for procedural pain
management in critically ill patients undergoing a procedure. The American Society for Pain
Management Nursing (ASPMN) published recommendations for preprocedural pain management
in 2011. The ASPMN recognizes both the psychological and the physical elements of procedural
pain and agrees with combining nonpharmacologic and pharmacologic methods. Examples of
nonpharmacologic options recommended by ASPMN include relaxation and breathing techniques,
imagery, massage, music, thermal measures, and positioning (Pain Manag Nurs 2011;12:95-111).
The PADIS guidelines recommend not using inhaled volatile anesthetics for procedural pain
management in critically ill adults. The PADIS guidelines also suggest using a nonsteroidal anti-
inflammatory drug (NSAID) administered intravenously, orally, or rectally as an alternative to
opioids for pain management during discrete and infrequent procedures in critically ill adults. Use
of an NSAID topical gel for procedural pain management in critically ill adults is not recommended
by the PADIS guidelines.
b. Preemptive analgesia for chest tube removal is recommended together with nonpharmacologic
relaxation techniques. The PADIS guidelines suggest neither local analgesia nor nitrous oxide for
pain management during chest tube removal in critically ill patients.
3. Postoperative thoracic epidural anesthesia/analgesia is recommended for patients undergoing abdominal
aortic aneurysm treatment. Thoracic epidural anesthesia is “suggested” for traumatic rib fractures in
the ICU.
4. Nonpharmacologic therapy for pain
a. The PADIS guidelines suggest offering cold therapy for procedural pain management in critically
ill adults.
b. The PADIS guidelines suggest offering relaxation techniques for procedural pain management
in critically ill adults. Music therapy is suggested for both nonprocedural and procedural pain in
critically ill adults.

ACCP Updates in Therapeutics® 2022: Critical Care Pharmacy Preparatory Review and Recertification Course

440
Management of PADIS and NMB in Adult Intensive Care Unit Patients

5. Pharmacotherapy for pain


a. Intravenous opioids on an as-needed, scheduled, or continuous infusion basis are recommended
to treat pain in the ICU. The pharmacokinetics of different opioids may vary; thus, opioids should
be chosen according to patient comorbidities and individual needs (Table 3). Fentanyl is the most
commonly used intravenous opioids in American adult ICUs.

Table 3. Opiates Commonly Used in the ICU


Metabolic/Drug
Usual CI Starting Drug-Specific
Drug Interaction Drug Accumulation Factors
Dosea Adverse Effectsb
Considerations
12.5–25 mcg/hr; Hepatic failure; high volume of
0.35–0.5 mcg/ distribution; high lipophilicity;
3A4 major
Fentanyl kg loading dose Muscle rigidity unpredictable clearance (long
substrate
follwed by 12.5–25 context-sensitive half-time)
mcg/hr with prolonged infusion
Hepatic failure; active
Hypotension,
metabolite (3- and 6-morphine
Morphine Glucuronidation 1–2 mg/hr bradycardia from
glucuronide) accumulates in
histamine release
renal failure
Overdose
effects 
from
Hydromorphone Glucuronidation 0.25–0.5 mg/hr dosing errors Hepatic failure
(high-potency
opiate)
QTc prolongation, Long half-life; hepatic and
3A4 and 2B6
Methadone N/A serotonin renal failure will delay
major substrates
syndrome clearance
Loading dose: 1.5
Chest wall rigidity;
Blood and tissue mcg/kg
Remifentanil rebound pain on
esterases CI: 0.5–15 mcg/
discontinuation
kg/hr
a
Usual starting dose in the ICU for pain management in an opiate-naive patient.
b
Other common significant adverse effects for all opiates to be considered: Constipation, respiratory depression, bradycardia, hypotension, altered mental status.
CI = continuous infusion.

i. General mechanism of action of opiates: Bind to mu-opioid receptors in the central nervous
system (CNS)
ii. Commonly used intravenous opioids in the ICU: Fentanyl, morphine, hydromorphone,
remifentanil, and methadone
iii. Tolerance: May quickly develop to all opiates, particularly when given as a continuous infusion.
If switching to a different intravenous or oral opiate, equianalgesic dosing may be difficult to
estimate, and low starting doses should be considered.
iv. Significant adverse effects: Decreased respiratory drive: This may be a desired effect in
some ICU scenarios; however, a depressed respiratory drive is a critical negative implication
during the ventilator weaning process; decreased blood pressure and heart rate, constipation,
gastrointestinal (GI) intolerance, altered sensorium.

ACCP Updates in Therapeutics® 2022: Critical Care Pharmacy Preparatory Review and Recertification Course

441
Management of PADIS and NMB in Adult Intensive Care Unit Patients

v. GI intolerance and constipation: A bowel regimen should be initiated on day 1 unless


contraindicated, with assessment for efficacy every 24–48 hours. GI intolerance in the ICU
can result in increased time on mechanical ventilation, delayed time to attaining nutritional
goals, and prolonged ICU stay. Constipation may also contribute to agitation.
vi. Altered mental status: Opiates may induce a sedative effect as well as an altered sensorium in
some patients. Unless contraindicated, clinicians should consider tapering the opiate dose in
an altered patient who has adequate pain control.
vii. Patient-controlled analgesia: In alert and clinically stable patients, use of patient-controlled
analgesia may be considered to titrate to the patient’s perceived level of pain. Patient-controlled
analgesia may also be useful on discontinuation of continuous infusion opiates.
b. Fentanyl
i. Pharmacokinetics: Hepatic metabolism, cytochrome P450 (CYP) 3A4 substrate. Quick onset
and short duration of action; lacks a pharmacologically active metabolite. Highly lipophilic,
high volume of distribution and protein binding; maintains a three-compartment model;
continuous infusion dosing may lead to prolonged and unpredictable clearance (prolonged
context-sensitive half-time).
ii. Many dosage forms: Injectable (intravenous, intramuscular, intrathecal, epidural), transdermal,
transmucosal, nasal spray. Different dosage forms should not be converted on a 1:1 mcg basis;
use specific manufacturer recommendations if converting. Injectable form of fentanyl is most
commonly used in the ICU setting. The fentanyl patch is not generally appropriate for use in
the ICU because of its latent onset (about 12 hours) and erratic/increased absorption in a febrile
patient.
iii. Adverse effects: Respiratory depression, bradycardia, hypotension, CNS depression,
constipation, ileus, risk of serotonin syndrome when used with other serotonergic agents
c. Morphine
i. Pharmacokinetics: Hepatic metabolism by glucuronidation to two major active metabolites,
morphine-3-glucuronide (45%–55%) and morphine-6-glucuronide (10%–15%). The
glucuronide metabolites of morphine are both renally eliminated; accumulation can occur
with the chronic use of morphine or in patients with decreased renal function. Morphine-3-
glucuronide does not have analgesic activity, but adverse effects may include seizure activity
or agitation. Morphine-6-glucuronide does have analgesic activity by the mu-receptor and
may cause additive sedation and respiratory depression if accumulation occurs. Continuous
morphine infusions are rarely used for analgesia in the ICU setting because of the concerns
with the active metabolites.
ii. Dosage forms: Injectable (intravenous, subcutaneous, intrathecal, epidural) and oral.
Intravenous-to-oral conversion is not a 1:1 mg ratio.
iii. Adverse effects: Histamine release may cause significant hypotension; bradycardia, respiratory
depression, CNS depression, constipation, ileus.
d. Hydromorphone
i. Pharmacokinetics: Hepatic metabolism by glucuronidation to an inactive but potentially
neurotoxic metabolite. Low volume of distribution, highly water soluble, and relatively low
protein binding.
ii. Dosage forms: Injectable (intravenous, subcutaneous) and oral; intravenous-to-oral conversion
is not a 1:1 mg ratio.
iii. Adverse effects: CNS alterations (e.g., abnormal dreams, aggressive behavior, altered thinking),
respiratory depression, hypotension, constipation

ACCP Updates in Therapeutics® 2022: Critical Care Pharmacy Preparatory Review and Recertification Course

442
Management of PADIS and NMB in Adult Intensive Care Unit Patients

e. Remifentanil (Ultiva)
i. Pharmacokinetics: Clearance by blood and tissue esterase; clearance not dependent on organ
function. Fast onset and short duration of action with little to no accumulation. High volume
of distribution, high protein binding.
ii. Research primarily done in Europe; limited reported use in U.S. adult ICUs for ongoing
analgesic use.
iii. Dosage form: Injectable only
iv. Adverse effects: Respiratory depression, hypotension, bradycardia, constipation
v. Rebound pain: Quick offset (5–10 minutes) may lead to rebound pain and withdrawal symptoms,
and additional pain medication may be needed if remifentanil is interrupted or discontinued.
vi. Benefit in adult ICUs: Decreased time on mechanical ventilation with short-term use (72 hours
or less)
vii. Cost (AWP): 1 mg = $55.16; 5 mg = $234.74.
f. Methadone
i. Pharmacokinetics: Phase I hepatic metabolism to inactive metabolites. Many drug
interactions: major substrate of CYP 2B6 and 3A4. Moderate inhibitor of CYP2D6, weak
inhibitor of CYP3A4. Longer-acting opiate with variable duration of action (12–48 hours); may
accumulate quickly in patients with hepatic failure or patients receiving hemodialysis. Animal
studies have found that the d-isomer of methadone works as both a partial mu-agonist and an
N-methyl-d-aspartate receptor antagonist (the l-isomer is a full mu-agonist). These properties
of the d-isomer are thought to decrease the tolerance effect to other opioids. Methadone is
currently marketed as the racemic mixture. On initiating oral methadone, steady state and
peak analgesic effect may not be reached for 3–5 days; oversedation and respiratory depression
may occur if titrated too quickly.
ii. Dosage forms: Injectable (intravenous, intramuscular, subcutaneous) and oral. Intravenous-to-
oral conversion is not a 1:1 mg ratio.
iii. Adverse effects: Dose-dependent QTc prolongation, altered mental status, respiratory
depression, confusion, dizziness, arrhythmias, constipation, risk of serotonin syndrome when
used with other serotonergic agents
6. Non-opioid adjunctive pain medications should be considered in combination with opioids to reduce
opioid requirements. Clinically stable patients may tolerate a conversion from opiates to non-opiate
medications.
a. Local and regional anesthetics such as bupivacaine. The PADIS guidelines do not recommend the
use of intravenous lidocaine as an adjunct to opioid therapy for pain management in critically ill
adults.
b. Acetaminophen (Tylenol, Ofirmev): The PADIS guidelines suggest using acetaminophen as an
adjunct to opioids to decrease pain intensity and opioid consumption.
i. Total daily acetaminophen doses should be considered from all acetaminophen combination
products, with a maximum total daily dose of 4 g. Decreased total daily dosing should be
considered in patients with significant liver disease.
ii. Intravenous acetaminophen: Dose reduction recommended if the creatinine clearance
(CrCl) is 30 mL/minute/1.73 m2 or less or with continuous renal replacement therapy (every
8 hours); contraindicated in severe hepatic disease. The cost of the intravenous formulation
of acetaminophen is considerably higher than that of the oral or rectal formulations. The
intravenous formulation can also cause hypotension.
c. Intravenous or oral NSAIDs: Ibuprofen, ketorolac. The PADIS guidelines suggest not routinely
using a COX-1 selective NSAID as an adjunct to opioid therapy for pain management in critically
ill adults. Use with caution in critically ill patients with renal or hepatic dysfunction. May increase
the risk of acute renal failure, bleeding, or GI adverse effects.

ACCP Updates in Therapeutics® 2022: Critical Care Pharmacy Preparatory Review and Recertification Course

443
Management of PADIS and NMB in Adult Intensive Care Unit Patients

d. Ketamine (Ketalar) has been used for analgesia and sedation in the ICU, primarily in the pediatric
population. The PADIS guidelines suggest using low-dose ketamine (1–2 mcg/kg/hour) as an
adjunct to opioid therapy when seeking to reduce opioid consumption in postsurgical adults
admitted to the ICU. Published data for the use of ketamine in adults for analgesia and/or sedation
are limited to case reviews, and long-term cognitive effects of ketamine are not known. Data from
animal studies suggest a significant decline in cognitive function after continued use of ketamine.
i. Called a “dissociative anesthetic,” providing analgesic activity at subanesthetic doses. It is a
schedule III controlled substance and works primarily as an N-methyl-d-aspartate receptor
antagonist. Ketamine is void of the constipation, respiratory depression, and hypotensive
effects that plague the opiate class.
ii. May decrease dose requirements of concurrently administered opioids
iii. Other uses include rapid sequence intubation, refractory pain syndromes, cancer pain,
neuropathic pain, asthma (bronchodilatory effects), refractory seizure activity, and depression.
iv. Dosing range is varied; usual starting dose for analgesia or sedation is 0.1 mg/kg/hour. Reviews
of ketamine use in adult ICUs report a dosing range of 0.1–2.5 mg/kg/hour and a range in
duration of 3 hours to 9 days.
v. Significant adverse effects: Mild to severe emergence reactions (e.g., confusion, excitement,
irrational behavior, hallucinations, delirium) in around 12% of patients, enhanced skeletal
muscle tone, tachycardia, hypertension, hypotension
e. The PADIS guidelines suggest not offering cybertherapy or hypnosis for pain management in
critically ill adults.
f. The PADIS guidelines suggest offering massage for pain management in critically ill adults.
g. The PADIS guidelines suggest offering music therapy to relieve both non-procedural and procedural
pain in critically ill adults.
7. The PADIS guidelines suggest using a neuropathic pain medication (e.g., gabapentin, carbamazepine,
pregabalin) with opioids for neuropathic pain management in critically ill patients. The PADIS guidelines
recommend using neuropathic pain medications with opioids for pain management in ICU adults
after cardiovascular surgery. There is a potential for significant adverse effects and drug interactions,
requiring close monitoring and follow-up. If the patient is discharged home on an anticonvulsant for
neuropathic pain, follow-up should be documented and the primary care provider notified.
a. Gabapentin (Neurontin)
i. Suggested starting dose range: 300–600 mg/day divided two or three times daily; requires
renal adjustment. The target dose is 900–3600 mg/day in three divided doses.
ii. Pharmacokinetics: Renally excreted, dose adjusted for reduced CrCl
iii. Adverse effects: May be severe, including CNS depression, paresthesias, and asthenias
b. Carbamazepine (Tegretol)
i. Suggested starting dose range: 50–100 mg twice daily; use with caution in patients with hepatic
impairment, and adjust for a CrCl less than 10 mL/minute/1.73 m2 or with hemodialysis. The
target dose is 100–200 mg every 4–6 hours; the maximum dose is 1200 mg/day.
ii. Pharmacokinetics: Strong inducer of many CYP enzymes, substrate of CYP3A4. Closely
monitor for drug interactions.
iii. Adverse effects: Somnolence, severe skin reactions (e.g., Stevens-Johnson syndrome, toxic
epidermal necrolysis), pancytopenia, syndrome of inappropriate antidiuretic hormone
c. Pregabalin (Lyrica)
i. 50 mg three times daily; may be increased in 1 week depending on tolerability and effect;
maximum dose: 100 mg three times daily.
ii. Pharmacokinetics: 90% excretion in urine
iii. Adverse effects: Peripheral edema, dizziness, drowsiness, headache, fatigue, weight gain,
xerostomia, visual field loss, and blurred vision

ACCP Updates in Therapeutics® 2022: Critical Care Pharmacy Preparatory Review and Recertification Course

444
Management of PADIS and NMB in Adult Intensive Care Unit Patients

F. Analgosedation Method in the ICU: This method of sedation advocates the use of opiate medications before
prescribing an anxiolytic/hypnotic medication to provide patient comfort in the ICU unless anxiolytics
are otherwise indicated. The PADIS guidelines recommend analgosedation (analgesia is used before a
sedative to reach the sedative goal) or analgesia-based sedation (an opioid is used instead of a sedative to
reach the sedative goal). Providing pain relief early in the ICU stay may decrease the agitation associated
with pain and/or general discomfort while minimizing the use of alternative medications commonly used
for agitation (e.g., benzodiazepines). The guidelines recognize that current data using analgosedation are
primarily limited to open-label trials, using remifentanil as the analgesic, and mostly conducted in Europe,
where critical care staffing and management practices differ from those in the United States. Despite these
limitations, it remains notable that studies using the analgosedation method found a significant decrease in
benzodiazepine dosage requirements when opiates were the primary medications used for discomfort and
agitation. This is a positive step in decreasing the untoward adverse effects of the benzodiazepine class
of sedatives. There is a potential for high cumulative doses of opiates with the analgosedation method,
necessitating daily monitoring of their adverse effects (e.g., respiratory depression, altered mental status,
GI slowing).

Patient Case

Questions 1 and 2 pertain to the following case.


T.O. is a 70-year-old man just admitted to the ICU with multiple fractures after a motor vehicle accident. His
medical history includes hypertension. He is now agitated after intubation. His laboratory values are normal,
and his vital signs include blood pressure 175/95 mm Hg and heart rate 110 beats/minute.

1. Which grouping of initial sedatives is most appropriate at this time?


A. Fentanyl infusion and midazolam infusion
B. Propofol infusion and fentanyl as needed
C. Midazolam as needed and fentanyl as needed
D. Fentanyl infusion and propofol infusion

2. After 2 weeks in the ICU, T.O. is being prepared for chest tube removal. He is currently receiving a fentanyl
drip with adequate pain control. Which is the best pain management regimen for chest tube removal?
A. Give intravenous acetaminophen 15 minutes before chest tube removal.
B. Make no change in pain treatment because his current pain regimen is adequate.
C. Increase his pain medication infusion dose by 50% the morning of his chest tube removal.
D. Give fentanyl 50 mcg injectable 15 minutes before chest tube removal.

ACCP Updates in Therapeutics® 2022: Critical Care Pharmacy Preparatory Review and Recertification Course

445
Management of PADIS and NMB in Adult Intensive Care Unit Patients

III.  AGITATION IN THE INTENSIVE CARE UNIT

A. Agitation in the ICU – Maintaining patient comfort for the duration of an ICU stay can be extremely
challenging, requiring significant resources and daily discipline from the nursing, medical, and pharmacy
team. Ongoing research has improved our understanding of the consequences of either under- or overtreating
agitation in the ICU, and clinicians should continue to apply this knowledge to their daily selection and
titration of medications. Treatment of a patient who presents with agitation must always begin with attempts
to identify and correct the etiology of the agitation. Common causes of agitation in the ICU include pain,
delirium, hypoxia, hypoglycemia, dehydration, and drug or alcohol withdrawal. Close inspection of
significant patient variables will also help determine the appropriate sedative:
1. Pain control
2. Substance abuse and smoking history
3. Neurologic function: Baseline and acute mental status, history of seizure activity, dementia, psychiatric
history
4. Clinical variables: Blood pressure, heart rate, respiratory rate
5. Comorbidities (baseline and acute): Cardiac, renal, hepatic, gastric, pulmonary, pancreatic
6. Home medication use: Any medication from which a patient could withdraw: Benzodiazepines, opioids,
antidepressants, other γ-aminobutyric acid (GABA) receptor agonists

B. Primary Medications for the Treatment of Agitation – Include propofol, dexmedetomidine, and
benzodiazepines (usually lorazepam and midazolam) (Table 4). Benzodiazepines are first-line agents for
status epilepticus, alcohol withdrawal, benzodiazepine dependence or withdrawal, and are useful for deep
sedation or amnesia and with the use of neuromuscular blockade. Other indications for benzodiazepines
may exist, which must be scrutinized throughout the ICU stay.

Table 4. Sedatives for Patients on Mechanical Ventilation in the ICU


CYP Significant
Precautions
Drug Onset and Duration Substrate Usual Dose Adverse
for Use
(major) Effects
Onset: 1 min
Duration: 
short term:
Hypotension, Hypotension,
0.5–1 hr; long term
bradycardia, 5–50 mcg/kg/ respiratory
> 7 days: variable;
Propofol hepatic/ 2B6 min; 0.3–3 mg/ depression,
25–50 hr has been
renal failure, kg/hr bradycardia,
observed (depends
pancreatitis PRIS
on depth and time on
sedation)
Onset: 5–10 min 
 LD: 0.5–1 mcg/ Hypo/hyper-
Hepatic failure;
(with LD)
1–2 hr kg (optional) tension,
Dexmedetomidine symptomatic 2A6
(without LD) MD: 0.2–0.7 bradycardia,
bradycardia
Duration: 1–2 hr mcg/kg/hr pyrexia
Onset: 5–20 min
Intermittent: Oversedation,
Duration: 4–8 hr; Delirium, renal
Lorazepam N/A 1–4 mg IV propylene
prolonged with failure
every 4–6 hr glycol toxicity
continuous infusion
Hepatic failure,
Onset: 3–5 min
end-stage 3A4
Duration: 2–6 hr, 0.02–0.1 mg/
Midazolam renal failure (active Oversedation
prolonged with kg/hr
or dialysis, metabolite)
continuous infusion
delirium
CI = continuous infusion; IV = intravenously; LD = loading dose; MD = maintenance dose; N/A = not applicable; PRIS = propofol-related infusion syndrome.

ACCP Updates in Therapeutics® 2022: Critical Care Pharmacy Preparatory Review and Recertification Course

446
Management of PADIS and NMB in Adult Intensive Care Unit Patients

C. SCCM provides the following statement in the PADIS guidelines regarding sedation in the ICU: “We
suggest using either propofol or dexmedetomidine over benzodiazepines [midazolam or lorazepam] for
sedation in critically ill, mechanically ventilated adults” and those after cardiac surgery. SCCM states
that “benzodiazepine use may be a risk factor for the development of delirium in adult ICU patients.” Two
randomized studies evaluated the differences in clinical outcomes while adult ICU patients were receiving
sedation with either a benzodiazepine or a non-benzodiazepine strategy. Heterogeneity occurred among the
findings of these two studies (see No. 1 and No. 2 below), which may be partly because of differences in study
design.
1. The MENDS study compared the sedative effects of lorazepam and dexmedetomidine in medical and
surgical adult ICU patients (n=103). Dexmedetomidine had more “delirium-free + coma-free” days
than lorazepam (7 vs. 3 days, p=0.01), and the prevalence of “delirium or coma” was lower in the
dexmedetomidine group (87% vs. 98%, p=0.03). The assessment of “delirium-free + coma-free” was
a more appropriate outcome to evaluate than “delirium without coma,” given that delirium cannot
be assessed in patients with coma. More patients were within 1 point of their RASS goal with
dexmedetomidine (67%) than with lorazepam (55%), p=0.008, but there was no difference in mechanical
ventilator–free days, ICU length of stay, or 28-day mortality. This study has been criticized because
both groups received continuous infusions of sedatives without additional bolusing, whereas in clinical
practice, most practitioners would bolus lorazepam before increasing the infusion rate. In addition,
patients were not required to have a spontaneous awakening trial (SAT). (JAMA 2007;298:2644-53).
2. The SEDCOM study compared the sedative effects of midazolam with those of dexmedetomidine
in medical and surgical adult ICU patients (n=366). The prevalence of delirium was lower in the
dexmedetomidine group (54%) than in the midazolam group (76.6%), p<0.001. Median time to
extubation was shorter in the dexmedetomidine group (3.7 days) than in the midazolam group (5.6
days), p<0.01; however, the times in target sedation range, ICU length of stay, and mortality were no
different between the two groups. This study did allow bolus dosing of study drugs, and patients were
required to have an SAT if safety criteria were met (JAMA 2009;301:489-99).

D. Clinical Outcome Differences Among Sedative Agents in Medical and Surgical ICU Patients: A meta-
analysis, “Benzodiazepine versus Nonbenzodiazepine-Based Sedation for Mechanically Ventilated
Critically Ill Adults,” reviewed trials from 1996 to 2013 (Crit Care Med 2013;41:S30-8):
1. Studies from this review contained the following criteria: (1) randomized controlled parallel-group
design; (2) medical and surgical adult ICU patients on mechanical ventilation receiving intravenous
sedation; (3) patients receiving a non-benzodiazepine (propofol 1% or dexmedetomidine) compared
with a benzodiazepine (lorazepam or midazolam); and (4) patients having predefined outcomes.
Excluded cardiac surgery and obstetric patients.
2. Four primary outcomes from six randomized trials were reported in the review (1235 patients):
a. ICU length of stay (all six studies reported): ICU length of stay was longer in a benzodiazepine
strategy than in a non–benzodiazepine-based strategy (mean difference 1.6 days; 95% confidence
interval [CI], 0.72–2.5; p=0.0005).
b. Duration of mechanical ventilation (four studies reported): Longer duration of mechanical
ventilation in a benzodiazepine-based strategy than in a non–benzodiazepine-based strategy
(mean difference 1.9 days; 95% CI, 1.7–2.09; p=0.00001)
c. Delirium prevalence (two studies reported): No difference in delirium prevalence between a
benzodiazepine and a non–benzodiazepine-based strategy (relative risk [RR] 0.98; 95% CI, 0.76–
1.27; p=0.94)
d. Short-term (45 days or less) all-cause mortality (four studies reported): No difference in risk of
death between a benzodiazepine and a non–benzodiazepine-based strategy (RR 0.98; 95% CI,
0.76–1.27, p=0.94)

ACCP Updates in Therapeutics® 2022: Critical Care Pharmacy Preparatory Review and Recertification Course

447
Management of PADIS and NMB in Adult Intensive Care Unit Patients

E. Propofol
1. SCCM suggests using a non-benzodiazepine (propofol or dexmedetomidine) for sedation to improve
clinical outcomes in mechanically ventilated patients. Throughout the past decade, propofol has been
increasingly used worldwide for sedation in the ICU. Propofol’s short duration of action, lack of
accumulation, and relatively clean adverse effect profile at low to moderate doses makes it an appealing
alternative.
2. Mechanism of action: General anesthetic by potentiation of the GABA A receptor; may inhibit N-methyl-
D-aspartate receptor activity at high doses. Propofol decreases cardiac β-adrenergic responsiveness and
attenuates β-adrenergic signal transduction in cardiac myocytes, resulting in direct cardiac depressive
effects.
3. Pharmacokinetics: Hepatic conjugation; clearance may be prolonged (from minutes to hours) in patients
with severe hepatic impairment or cirrhosis or with long-term infusions as it redistributes from fat
and muscle to plasma. Highly lipophilic pharmacokinetics and a large volume of distribution lead to
extensive tissue distribution. Propofol maintains a three-compartment linear model: plasma, rapidly
equilibrating tissues (e.g., major organs), and slowly equilibrating tissues (e.g., fat deposits). Substrate
of CYP 2B6, 2C9, 2C19, and 3A4; pharmacokinetic studies of healthy volunteers show a 25% increase
in propofol plasma concentrations when given with midazolam, a weak CYP 3A4 and 2C9 inhibitor.
4. Lipid formulation considerations: Standard propofol is a 1% (10 mg/mL) lipid emulsion containing 1.1
kcal/mL (0.1 g of fat per 1 mL of propofol); this should be accounted for when calculating nutritional
intake (e.g., propofol at 50 mcg/kg/minute in a 70-kg patient would provide around 500 calories per
day contributed by fat). Propofol contains 0.005% disodium edetate (EDTA) to decrease the rate of
microorganism growth, which is known to chelate trace metals, including zinc. Zinc supplementation
should be considered in patients at high risk of zinc deficiency (sepsis, burns, large-volume diarrhea) if
propofol is used for more than 5 days. Strict aseptic technique must be followed when handling propofol;
manufacturers recommend discarding propofol bottles and changing intravenous tubing every 12 hours
to decrease the risk of contamination.
5. Dosing range for ICU sedation: Usual starting dose 5–10 mcg/kg/minute, titrated every 5–10 minutes
to goal sedative effect. Abrupt discontinuation of propofol is not recommended because of its rapid
clearance (5–10 minutes).
6. Data: NONSEDA is a randomized, clinical parallel-group, multinational superiority study comparing
no sedation (bolus doses of morphine) and light sedation (propofol for 48 hours then midazolam titrated
to a RASS of −2 to −3). There was no difference in 90-day mortality between groups (no sedation 148
[42.4%] vs. light sedation 130 [37%]; 95% CI −2.2–12.2; p=0.65). However, 27% of the no-sedation
group crossed over to receive sedation during the first 24 hours after randomization.
7. Data: In a multicenter European trial (PRODEX), Jakob et al. compared propofol (n=249) with
dexmedetomidine (n=251) for sedation in prolonged mechanical ventilation. Patients in both groups
were treated with daily sedation interruption trials and spontaneous breathing trials (SBTs), and pain
was treated with fentanyl boluses. Proportion of time in target RASS (0, −3) without rescue therapy
was the same in the propofol group (65%) as in the dexmedetomidine group (65%). There was no
difference in median time on mechanical ventilation in propofol (5 days) versus dexmedetomidine
(4 days), p=0.24. Patients’ ability to communicate discomfort was better in the dexmedetomidine
group. Rates of hypotension and bradycardia were similar between the two groups. Critical illness
polyneuropathy was more common in the propofol group (n=11) than in the dexmedetomidine group
(n=2), p<0.02. The composite outcome of agitation, anxiety, and delirium occurred in propofol (29%)
versus dexmedetomidine (18%) (p=0.008) (JAMA 2012;307:1151-60).

ACCP Updates in Therapeutics® 2022: Critical Care Pharmacy Preparatory Review and Recertification Course

448
Management of PADIS and NMB in Adult Intensive Care Unit Patients

8. Adverse effects: Bradycardia and hypotension (may be more common or severe in patients with
cardiac dysfunction, intravascular volume depletion, or low systemic vascular resistance); respiratory
depression, hypertriglyceridemia (most institutions use a threshold of 800–1000 as a cutoff of when to
stop propofol), pancreatitis with or without hypertriglyceridemia, PRIS
9. PRIS: This is a rare but life-threatening complication of propofol, usually occurring at doses greater
than 50 mcg/kg/minute for 48 hours or more. The mechanism of PRIS may include alterations in
hepatic metabolism of the lipid emulsion, leading to an accumulation of ketone bodies and lactate
and/or disruptions in the mitochondrial respiratory chain and inhibition of oxidative phosphorylation.
Patients with urea cycle disorders may experience alterations in propofol metabolism within 24–48
hours of propofol use. Consider avoiding in patients with acute liver failure, or pancreatitis, because
the symptoms of PRIS may be difficult to distinguish from the underlying disease state abnormalities.
PRIS carries a high mortality rate, and propofol should be discontinued immediately if symptoms are
present.
a. Clinical characteristics of PRIS: Metabolic acidosis, acute renal failure, cardiovascular collapse,
cardiac arrhythmias including Brugada-like syndrome, rhabdomyolysis, myoglobinemia,
myoglobinuria, hyperkalemia, hypertriglyceridemia, elevated creatine kinase concentrations
b. Risk factors for PRIS or other adverse effects of propofol: Neurologic injury, sepsis, use of
vasoactive medications, high-dose propofol, acute liver failure

F. Dexmedetomidine
1. SCCM suggests that in adult ICU patients with delirium unrelated to alcohol or benzodiazepine
withdrawal, dexmedetomidine infusions rather than benzodiazepine infusions should be administered
for sedation to reduce the duration of delirium. Although ICU surveys report low use of dexmedetomidine
compared with other sedatives, clinical outcomes research of dexmedetomidine has shown favorable
results.
2. Mechanism of action: Highly selective and dose-dependent α2-adrenoceptor agonist in the CNS.
Dexmedetomidine provides a hypnotic and sedative effect by inhibition of norepinephrine release from
the locus coeruleus; dexmedetomidine also produces a weak antinociceptive effect by way of inhibition
of neuronal transmission through presynaptic C-fibers and release of substance P, and hyperpolarization
of postsynaptic α receptors in the dorsal horn of the spinal column. Dexmedetomidine does not
directly affect respiratory drive; therefore, intubation is not required with use. Dexmedetomidine is
considered a weak sedative and would not be appropriate for use when deep sedation is required (e.g.,
in a patient requiring neuromuscular blockade). Both anterograde amnesia and retrograde amnesia have
been described in some patients (20%–50%) in adult and pediatric studies. A benzodiazepine may be
required if full amnesia is desired because of the clinical scenario.
3. Pharmacokinetics: Hepatic by glucuronidation and renal excretion. Onset with loading dose 15–20
minutes; onset without loading dose greater than 20 minutes to 1 hour; terminal half-life = 3 hours (may
be significantly prolonged in hepatic impairment). Highly protein bound 94%.
4. Clinical effects: Sedation and weak opiate-sparing antinociceptive effects.
5. Dosing for ICU sedation: Optional loading dose 0.5–1 mcg/kg intravenously for 10 minutes, followed
by 0.2–0.7 mcg/kg/hour. The loading dose may initially cause severe tachycardia and hypertension, but
it can then quickly lead to significant bradycardia and/or hypotension secondary to receptor saturation.
Because of these untoward hemodynamic effects, the loading dose is rarely administered in clinical
ICU practice on initiation of dexmedetomidine, and the drip is usually initiated at 0.2–0.4 mcg/kg/
hour. For the maintenance infusion dose, randomized trials have safely used dexmedetomidine at
higher than manufacturer-recommended doses, up to 1.5 mcg/kg/hour. Clinical efficacy with doses
greater than 1.5 mcg/kg/hour remains unclear. Other routes of administration have been described for
dexmedetomidine, including intramuscular, subcutaneous, epidural, and intranasal.

ACCP Updates in Therapeutics® 2022: Critical Care Pharmacy Preparatory Review and Recertification Course

449
Management of PADIS and NMB in Adult Intensive Care Unit Patients

6. Duration of use: Although the package insert recommends a therapy of 24 hours or less, randomized trials
have used dexmedetomidine for up to 5–7 days; thus, ICU clinicians often administer dexmedetomidine
for longer than 24 hours. Safety beyond 7 days of use has not been well established.
7. Data: The Dexmedetomidine to Lessen ICU Agitation (DahLIA) study was a double-blind placebo-
controlled, parallel-group randomized clinical trial in 15 ICUs in Australia and New Zealand in which 39
patients were randomized to dexmedetomidine and 32 patients to placebo. At 7 days, dexmedetomidine
increased ventilator-free hours compared with placebo (median, 144.8 hours vs. 127.5 hours, 95% CI
4 to 33.2 hours, p=0.01). Patients in the dexmedetomidine group had decreased time to extubation
compared with placebo (median 21.9 hours vs. 44.3 hours, 95% CI 5.3 to 3.1 hours, p<0.001). An
accelerated resolution of delirium was found in the dexmedetomidine group compared with placebo
(median, 23 hours vs. 40 hours, 95% CI, 3 to 28 hours, p=0.01). However, propofol use was common
in both groups after randomization (72% in the dexmedetomidine group vs. 88% in the placebo group)
(median cumulative dose 980 mg (IQR 280–3050) vs. 5390 mg (IQR 1880–10,803), p<0.001).
8. Data: The DESIRE trial was a open-label, multicenter randomized clinical trial conducted in eight ICUs
in Japan in which 100 patients with sepsis were randomized to dexmedetomidine and 101 patients with
sepsis were randomized to placebo. Mortality at 28 days was not different between the two groups (23%
in the dexmedetomidine group vs. 31% in the placebo group; HR 0.69; 95% CI, 0.38–1.22; p=0.20).
Ventilator-free days over 28 days were not different between groups (median 20 days (IQR 5–24) in the
dexmedetomidine group vs. 18 days (IQR 0.5–23) in the control group (p=0.20).
9. Data: SPICE III is an open-label, randomized controlled trial comparing dexmedetomidine as primary
sedation with usual care (propofol, midazolam, or other sedation) in patients receiving less than 12 hours
of mechanical ventilation who are expected to be on the ventilator for at least one additional day. The
target RASS goal was -2 to +1. Death at 90 days occurred in 569 of 1956 (29.1%) of the usual care group
and 566 of 1948 (29.1%) of the dexmedetomidine group (adjusted risk difference 0.0 percentage points;
95% CI, –2.9 to 2.8). In the dexmedetomidine group, 64% of patients additionally received propofol, 3%
received midazolam, and 7% received both during the first 2 days after randomization. Sixty percent
of patients received propofol, 12% received midazolam, and 20% received both in the usual care group.
Given the many sedatives administered in both groups, application of the study results is difficult. In
the dexmedetomidine group, bradycardia and hypotension occurred in 5.1% and 2.7% of patients.
10. Data: MENDS 2 is multicenter, double-blind trial comparing dexmedetomidine to propofol. No
difference was found between dexmedetomdine and propofol in the number of days alive and without
delirium of coma (adjusted median 10.7 vs 10.8 days; odds ratio, 0.96; 95% CI 0.74–1.26), ventilator free
days (adjusted median, 23.07 vs. 24 days, odds ratio 0.98; 95% CI, 0.63–1.51), or death at 90 days 38%
vs. 39%; hazard ratio, 1.06; 95% CI 0.74–1.52).
11. Adverse effects: Tachycardia, pyrexia bradycardia, hypertension, hypotension, dry mouth. Should
generally be avoided in patients with acute decompensated heart failure or advanced heart block
12. Other potential uses in the ICU: Procedural sedation, palliative care pain and anxiety control, adjunct
to opiates for sickle cell crisis, adjunct to benzodiazepines or propofol for alcohol withdrawal, bridge to
extubation while tapering off longer-acting sedatives and/or opiates, to provide sedation and anxiolysis
during noninvasive mechanical ventilation

G. Lorazepam (Ativan)
1. Pharmacokinetics: Benzodiazepine that binds to the postsynaptic GABA A receptor, undergoes hepatic
clearance by conjugation to inactive compounds; moderate to high volume of distribution and high
protein binding. Onset of action is 15–30 minutes, slower than more lipophilic benzodiazepines (e.g.,
midazolam). Duration of action of intermittent dosing is 4–8 hours. As a continuous infusion, clearance
of lorazepam decreases in an unpredictable fashion, and prolonged sedation may occur.

ACCP Updates in Therapeutics® 2022: Critical Care Pharmacy Preparatory Review and Recertification Course

450
Management of PADIS and NMB in Adult Intensive Care Unit Patients

2. Effects: Anxiolysis/sedation, anticonvulsant, muscle relaxant. Maintains anterograde amnesia


properties; however, studies report that patients who received benzodiazepines in the ICU may maintain
delusional versus factual memories.
3. Dosing range: 1–4 mg every 4–6 hours intermittent dosing is recommended before using continuous
infusion; accumulation and prolonged awakening times with continuous infusion of lorazepam may
occur because of prolonged duration of action.
4. Data: Carson et al. studied the number of days on mechanical ventilation in medical ICU patients
receiving intermittent lorazepam (n=64) compared with continuous infusion propofol (n=68); each
group underwent daily interruption of sedation if the fraction of inspired oxygen (Fio2) was less than
80%. Median time on mechanical ventilation was 9 days in the lorazepam group versus 4.4 days in the
propofol group, p=0.006; ICU length of stay was 12.7 days in the lorazepam group versus 8.6 days in
the propofol group (p=0.05); no difference in hospital mortality. Delirium was not assessed in this study
(Crit Care Med 2006;34:1326-32).
5. Propylene glycol toxicity: Because of its insolubility, injectable lorazepam is diluted in propylene glycol.
Propylene glycol toxicity can occur with lorazepam infusions for more than 48 hours, particularly
at doses of 6–8 mg/hour or greater, and can manifest as new-onset renal failure, respiratory failure,
metabolic acidosis, and altered mental status. Most hospitals cannot measure quantitative levels of
propylene glycol; therefore, surrogate markers of propylene glycol toxicity such as an elevated osmolar
gap (greater than 10 mOsm/kg) and elevated anion gap with new metabolic acidosis are recommended
for monitoring. If these metabolic abnormalities are present while on a lorazepam infusion, lorazepam
should be discontinued.
6. Other adverse effects: Paradoxical agitation, confusion, prolonged duration of sedative action,
respiratory depression, hypotension, bradycardia

H. Midazolam (Versed)
1. Pharmacokinetics: Benzodiazepine that binds to the postsynaptic GABA A receptor; undergoes phase
I hepatic metabolism to an active glucuronidated metabolite, α1-hydroxymidazolam, which is then
renally excreted. A short- to medium-acting benzodiazepine in patients with normal renal and hepatic
function. CYP3A4 substrate. Midazolam is highly lipophilic, has a large volume of distribution, and is
highly protein bound.
2. Clearance: Clearance of midazolam or its metabolite is significantly altered if either hepatic (primary
drug accumulation) or renal (active metabolite α-hydroxymidazolam accumulation) functions are
significantly impaired. Continuous renal replacement therapy partly clears the active metabolite but does
not effectively clear the parent compound and is therefore not recommended as a method for definitive
midazolam clearance (Am J Kidney Dis 2005;45:360-71). High lipophilicity and a large volume of
distribution may lead to significant drug accumulation and a depot effect in the ICU patient. In general,
clearances of midazolam infusions have wide interpatient variability in the ICU, and emergence times
may be significantly prolonged.
3. Effects: Anxiolysis/sedation, anticonvulsant, muscle relaxant. Maintains anterograde amnesia
properties.
4. Dosing range: 1–4 mg every 2–4 hours intermittently or as needed should be considered before initiating
continuous infusion. Older adult patients may tolerate only 1–2 mg per dose.

ACCP Updates in Therapeutics® 2022: Critical Care Pharmacy Preparatory Review and Recertification Course

451
Management of PADIS and NMB in Adult Intensive Care Unit Patients

5. Data: In a multicenter European trial (MIDEX), Jakob et al. compared midazolam (n=251) with
dexmedetomidine (n=249) for sedation in prolonged mechanical ventilation. Patients in both groups
were treated with daily sedation interruption trials and SBTs, and pain was treated with fentanyl
boluses. There was no difference in the primary outcome: proportion of time in target RASS (0, -3)
without rescue therapy in the midazolam group (56%) versus the dexmedetomidine group (60%).
Median time on mechanical ventilation was lower in the dexmedetomidine group (5 days) than in the
midazolam group (6.8 days), p=0.03. Patients in the dexmedetomidine group were more arousable,
more cooperative, and better able to communicate discomfort or pain to clinical staff than were patients
in the midazolam group. Hypotension occurred more often in the dexmedetomidine group (20.6%) than
in the midazolam group (11.6%; p=0.007); and bradycardia was more common in the dexmedetomidine
group (14.2%) than in the midazolam group (5.2%; p<0.001). The two treatment groups showed no
difference in neurocognitive adverse events after 48 hours of follow-up, including agitation, anxiety,
and delirium (JAMA 2012;307:1151-60).
6. Adverse effects: Paradoxical agitation and prolonged duration of sedative action, respiratory depression,
hypotension, bradycardia

I. Titration of Sedation in the ICU


1. Titration of medications using a sedation protocol to a goal level of sedation is arguably one of the
most salient clinical practice standards within ICU care. This titration practice has consistently been
shown to decrease time on mechanical ventilation, decrease ICU length of stay, and decrease rates of
tracheostomy, which may all result in faster physical and cognitive rehabilitation time.
a. The goal level of sedation should be reassessed daily, documented, and communicated clearly to
the nursing and medical staff because the goal may change throughout a patient’s ICU stay.
b. Level of sedation should be assessed every 2–4 hours throughout the day and evening. Consider
assessing every 4 hours during nighttime sleeping hours to minimize sleep interruption.
c. The two validated sedation scales currently recommended by the PADIS guidelines are the RASS
(Table 5) and the Riker Sedation-Agitation Scale (Table 6).
2. Recommended methods for titration of medications include either (1) titration to a “light” versus “deep”
level of sedation, unless clinically contraindicated, or (2) daily SAT. The PADIS guidelines suggest
using light sedation (vs. deep sedation) in critically ill, mechanically ventilated adults.
a. Patients should receive sedation only if required.
b. Sedatives should be titrated to allow patient responsiveness and awareness, as shown by patients’
ability to purposefully respond to a combination of any three of the following actions on request:
open eyes, maintain eye contact, squeeze hand, stick out tongue, and wiggle toes. A numerical
score on a sedation scale may not fully represent a patient’s ability to follow purposeful commands.
3. Data: In a multicenter, prospective, longitudinal Australian and New Zealand Study, Shehabi et al.
evaluated 251 medical/surgical patients who were ventilated and sedated for 24 hours or more. Within 4
hours of initiating ventilation, deep sedation occurred in 191 patients (76.1%) and in 171 patients (68%)
at 48 hours. Early deep sedation was an independent predictor of time to extubation (HR 0.90; 95% CI,
0.87–0.94; p<0.001), hospital death (HR 1.11, 95% CI, 1.02–1.20; p=0.01), and 180-day mortality (HR
1.08, 95% CI, 1.01–1.16; p=0.026) (JAMA 2012;307:1151-60).
4. In a single-center, nested cohort study within the Awakening and Breathing Controlled (ABC)
randomized trial, Seymour et al. evaluated 140 patients for whom hourly doses of benzodiazepines
and propofol during the daytime (7 a.m. to 11 a.m.) and nighttime (11 p.m. to 7 a.m.) for 5 days were
measured. Greater daytime benzodiazepine doses were independently associated with failed SBT and
extubation and subsequent delirium in adjusted models (p<0.02 for all). A failed SBT (p<0.01) and
delirium (p=0.05) were associated with nighttime increases in benzodiazepine doses (Crit Care Med
2012;40:2788-96).

ACCP Updates in Therapeutics® 2022: Critical Care Pharmacy Preparatory Review and Recertification Course

452
Management of PADIS and NMB in Adult Intensive Care Unit Patients

J. SAT Paired with SBT: The daily coordination of an SAT completed before an SBT is a method of weaning
sedation before attempts at breathing trials in order to maximize a patient’s chances of weaning from
mechanical ventilation. This pairing of an SAT before an SBT is becoming recognized as an important
component to ICU care and management of sedation. Important safety screens are incorporated into the
daily SAT because studies have shown that the SAT is not appropriate for all ICU patients. If a patient does
not pass the safety screen and does not undergo the SAT, this should not preclude the appropriate titration
of sedatives to a goal level of sedation throughout the remainder of the day:
1. SAT safety screen (criteria may vary; published trial protocols have had variations): If any are present,
discontinue the protocol and repeat in 12–24 hours or according to hospital protocol:
a. Current RASS greater than 2; or goal for deeper sedation (e.g., RASS -3 to -5)
b. Active seizures requiring a continuous infusion of a sedative to control
c. Active alcohol withdrawal requiring a continuous infusion of a sedative to control
d. Fio2 of 70% or greater (these criteria are not consistently present among published trial protocols)
e. Neuromuscular blockade
f. Myocardial ischemia in previous 24 hours or ongoing myocardial ischemia
g. Intracranial pressure (ICP) greater than 20 mm Hg or need for control of ICP
2. If pass SAT safety screen, begin SAT: Hold continuous sedative and analgesic infusions. Bolus opioids
are recommended for breakthrough pain. Continuous opioid infusions allowed to continue while
stopping sedatives if presence of active pain. If the patient “passes” the SAT, continue to the SBT safety
screen.
3. SAT failure (if any are present, discontinue the protocol, and repeat in 12–24 hours or according to
hospital protocol):
a. Anxiety/agitation/pain present (e.g., RASS greater than +1 for 5 minutes or more)
b. Respiratory rate greater than 35 breaths/minute for 5 minutes or more
c. Oxygen saturation less than 88% for 5 minutes or more
d. ICP greater than 20 mm Hg
e. Acute cardiac ischemia or arrhythmia
f. Respiratory or cardiac distress (e.g., heart rate increase of 20 beats/minute or greater, heart rate
less than 55 beats/minute, use of accessory muscles, abdominal paradox, diaphoresis, or dyspnea)
4. If SAT fails: Consider giving patient bolus opioids first (up to three doses in 1 hour) before restarting
infusion. Reinitiate sedation infusion, if necessary, at half the previous dose and titrate to goal.
Determine the reasons for SAT failure. Repeat SAT steps in 12–24 hours or according to hospital
protocol.
5. SBT safety screen (if any are present, discontinue the protocol; repeat in 12–24 hours or according to
hospital protocol):
a. Agitation
b. Oxygen saturation less than 88%, Fio2 greater than 50%
c. PEEP (positive end-expiratory pressure) of 7 cm H2O or greater
d. Myocardial ischemia in previous 24 hours
e. Increasing vasopressor requirements
f. Lack of inspiratory efforts
6. SBT: If a patient tolerates the SBT for 30 to 120 minutes, consider extubation.
7. SBT failure:
a. Respiratory rate greater than 35 breaths/minute (for more than 5 minutes) or less than 8 breaths/
minute
b. Oxygen saturation less than 88% for more than 5 minutes
c. ICP greater than 20 mm Hg, mental status change
d. Acute cardiac ischemia or arrhythmia
e. Respiratory distress (use of accessory muscles, abdominal paradox, diaphoresis, and dyspnea)

ACCP Updates in Therapeutics® 2022: Critical Care Pharmacy Preparatory Review and Recertification Course

453
Management of PADIS and NMB in Adult Intensive Care Unit Patients

8. If SBT fails: Place the patient on prior ventilator settings. Repeat bundle in 12–24 hours or according
to hospital protocol.

K. Sedation Protocol Compared with the Paired SAT-SBT Protocol: Studies comparing a standard sedation
protocol with daily pairing of a SAT with SBT have shown decreased days on mechanical ventilation, days
in the ICU, and decreased rates of delirium when the SAT is paired with the SBT.
1. The ABC trial included 336 mechanically ventilated patients from four tertiary care hospitals. Patients
were randomized to patient-targeted sedation protocol plus the SBT (“usual care” control group) or to
daily SAT paired with the SBT (intervention group). Both groups were deeply sedated on enrollment
(RASS -4), and both groups had been admitted for 2.2 days before enrollment. In the intervention group,
patients who passed the safety screen underwent an SAT: sedatives and analgesics used for sedation
were discontinued, and analgesics used for active pain were continued. Patients “passed” their SAT if
they opened their eyes to command or tolerated being off sedation for at least 4 hours without meeting
failure criteria. The mean ventilator-free days was 11.6 days in the usual care control group versus 14.7
days in the SAT plus SBT group (p=0.02). The time to discharge was 12.9 days in the control group
versus 9.1 days in the intervention group (p=0.01). Self-extubations were higher in the intervention
group, but there was no difference in self-extubations requiring reintubation between groups. Rates of
delirium assessed by the confusion assessment method for the intensive care unit (CAM-ICU) were no
different between groups (74% vs. 71%).
2. The first study evaluating the ABCDE (Awakening and Breathing Coordination, Delirium Monitoring/
Management, and Early Mobility) Bundle compared clinical outcomes in patients before (n=146) and
after (n=150) bundle-protocol implementation; 187 patients were on mechanical ventilation. The bundle
protocol consisted of a daily-paired SAT/SBT, delirium screening with the CAM-ICU every 8 hours,
and an early mobility protocol. The “before” bundle patients were enrolled from February to October
2011; the “after” bundle patients were enrolled from October 2011 to April 2012. There were some
differences in patient type on admission, including more elective admissions in the post-bundle group
(39 vs. 30), more cardiothoracic surgery patients in the post-bundle group (20 vs. 6), more surgical
patients in the pre-bundle group (21 vs. 11), and more patients coming from an outside hospital in the
pre-bundle group (9 vs. 1). The post-bundle group had more median ventilator-free days (24 vs. 21
days, p=0.04), less delirium at any time (49 vs. 62%, p=0.03), and shorter percentage of ICU days with
delirium (33.3 vs. 50 %, p=0.003) (Crit Care Med 2014;42:1024-36).
3. The SLEAP investigators from the Canadian Critical Care Trials Group studied the outcomes of
patients receiving a daily sedation protocol alone versus patients receiving a daily sedation protocol
plus a daily sedation interruption (Crit Care Med 2015;43:557-66; Crit Care Med 2015;43:2180-90;
JAMA 2012;308:1985-92). From January 2008 to July 2011, 430 patients were enrolled from 16 tertiary
care medical and surgical ICUs. Only opiate and benzodiazepine infusions were allowed in the study.
According to the sedation-alone protocol, the RASS goal was -3 to 0, and the Sedation-Agitation Scale
(SAS) goal was 3 or 4. Nurses assessed sedation levels on an hourly basis and titrated medications
every 15–30 minutes to achieve sedation goals. If patients were oversedated in either group (SAS 1 or
2; RASS −4 or −5), infusions were discontinued. According to the sedation protocol with daily sedation
interruption, nurses stopped benzodiazepine and opiate infusions once a day and assessed hourly for
wakefulness (e.g., a light SAS or RASS score, plus ability to follow at least three commands).

ACCP Updates in Therapeutics® 2022: Critical Care Pharmacy Preparatory Review and Recertification Course

454
Management of PADIS and NMB in Adult Intensive Care Unit Patients

a. Clinical outcomes (published 2012): There was no difference in the primary outcome of time to
successful extubation between the two groups (7 days in both groups). There was a significant
difference in time to extubation in the prespecified surgical/trauma group between sedation
protocol with daily sedation interruption and sedation protocol alone (6 vs. 13 days; hazard ratio
[HR] 2.55; 95% CI, 1.40–5.44). No difference in time to extubation was detected between groups
among medical ICU patients (9 vs. 8 days; HR 0.92; 95% CI, 0.72–1.18). However, significantly
lower daily doses of both benzodiazepines and opiates boluses and continuous infusions were
used in the sedation protocol–alone group than in the sedation protocol plus interruption group.
Although the sedation protocol suggested to target light sedation, the actual mean RASS/SAS was
not reported for either group making it unclear how deeply sedated either group was.
b. Delirium outcomes (published 2015): Delirium by the Intensive Care Delirium Screening Checklist
(ICDSC) was diagnosed in 53.8% of patients in the study; there was no difference in delirium
in the sedation protocol–alone group versus the protocol plus daily sedation interruption group.
Patients who had delirium had a longer duration of mechanical ventilation, longer ICU and hospital
stay, longer use of restraints, higher rates of tracheostomy, and higher incidence of unintentional
device removal. Patients with delirium received almost the twice the mean dose of midazolam
equivalents/patient/day (104 mg vs. 57 mg), higher fentanyl equivalents/patient/day (1497 mcg vs.
1150 mcg), more frequent use of anticholinergics (18 vs. 8.6%), and more frequent use of trazadone
or zopiclone (17.7 vs. 9.8%) than did patients who were not delirious. Patients who developed
delirium had a higher incidence of alcohol and cigarette use than did patients who did not develop
delirium.
c. Recall in ICU survivors (published 2015): The SLEAP investigator study did patient interviews on
days 3, 28, and 90 post-ICU discharge to determine differences in recall between the sedation-alone
protocol group and the protocol plus daily sedation interruption group. There were no differences in
type of recall between the sedation strategies. Delusional memories were common at day 28 (70%
of patients) but were unrelated to the presence of delirium or the total dose of benzodiazepines or
opiates. Patients with no recall had received lower total doses of benzodiazepines than had patients
with recall. Emotional memories such as panic and confusion declined over time.

Table 5. Richmond Agitation-Sedation Scale (RASS)


Score Term Description
+4 Combative Overtly combative or violent; immediate danger to staff
+3 Very agitated Pulls on or removes tube(s) or has aggressive behavior toward staff
+2 Agitated Frequently nonpurposeful movement or patient-ventilator dyssynchrony
+1 Restless Anxious or apprehensive, but movements not aggressive or vigorous
0 Alert and calm
−1 Drowsy Not fully alert but has sustained (> 10 s) awakening, with eye contact, to voice
−2 Light sedation Briefly (< 10 s) awakens with eye contact to voice
−3 Moderate sedation Any movement (but no eye contact) to voice
−4 Deep sedation No response to voice, but any movement to physical stimulation
−5 Unarousable No response to voice or physical stimulation

ACCP Updates in Therapeutics® 2022: Critical Care Pharmacy Preparatory Review and Recertification Course

455
Management of PADIS and NMB in Adult Intensive Care Unit Patients

Table 5. Richmond Agitation-Sedation Scale (RASS) (continued)


Score Term Description

Procedure

1. Observe patient. Is patient alert and calm (score 0)? Does patient have behavior that is consistent with rest-
lessness or agitation (score +1 to +4 using the criteria listed above, under Description)?
2. If patient is not alert, in a loud speaking voice, state the patient’s name and direct the patient to open eyes
and look at speaker. Repeat once if necessary. Can prompt patient to continue looking at speaker. Patient has
eye opening and eye contact, which is sustained for more than 10 seconds (score −1). Patient has eye opening
and eye contact, but this is not sustained for 10 seconds (score −2). Patient has any movement in response to
voice, excluding eye contact (score −3).
3. If patient does not respond to voice, physically stimulate patient by shaking shoulder and then rubbing ster-
num if there is no response to shaking shoulder. Patient has any movement to physical stimulation (score
−4). Patient has no response to voice or physical stimulation (score −5).
Reprinted with permission from: American Thoracic Society. Copyright © 2014 American Thoracic Society. Sessler CN, Gosnell M, Grap MJ, et al. The Richmond
Agitation-Sedation Scale: validity and reliability in adult intensive care patients. Am J Respir Crit Care Med 2002;166:1338-44. Official Journal of the American
Thoracic Society.

Table 6. Riker Sedation-Agitation Scale


Pulling at ETT, trying to remove catheters, climbing over bedrail, striking at
7 Dangerous agitation
staff, thrashing side to side
Does not calm despite frequent verbal reminding of limits, requires physical
6 Very agitated
restraints, biting ETT
5 Agitated Anxious or mildly agitated, trying to sit up, calms down to verbal instructions
4 Calm and cooperative Calm, awakens easily, follows commands
Difficult to arouse, awakens to verbal stimuli or gentle shaking but drifts off
3 Sedated
again, follows simple commands
Arouses to physical stimuli but does not communicate or follow commands,
2 Very sedated
may move spontaneously
Minimal or no response to noxious stimuli, does not communicate or follow
1 Unarousable
commands
ETT = endotracheal tube.
Adapted with copyright permission from: Lippincott Williams and Wilkins/Wolters Kluwer Health. Simmons LE, Riker RR, Prato BS, et al. Assessing sedation during
intensive care unit mechanical ventilation with the Bispectral Index and the Sedation Agitation Scale. Crit Care Med 1999;27:1499-504.

L. Acute Withdrawal Syndrome of Long-term Analgesia and/or Sedation in the ICU


1. Patients who have been receiving high doses of continuous infusion sedation and/or analgesia in the
ICU for an extended period may be at risk of sedative or analgesia withdrawal as dose tapering begins.
In a retrospective review of adult trauma/surgical ICU patients, 32% of patients experienced either
sedative or opiate withdrawal soon after discontinuing these medications. The patients in this study
had been in the ICU for 20 days or more and were receiving higher mean daily analgesic and sedative
doses than were the non-withdrawal patients (fentanyl 6.4 mg vs. 1.4 mg; lorazepam 38 mg vs. 11 mg).
The withdrawal patients in this study were also more likely to have received an NMBA (Crit Care Med
1998;26:676-84).

ACCP Updates in Therapeutics® 2022: Critical Care Pharmacy Preparatory Review and Recertification Course

456
Management of PADIS and NMB in Adult Intensive Care Unit Patients

2. The risk factors and incidence of sedation or analgesia withdrawal in adult ICU patients have not been
well characterized; however, these are important considerations in the long-term ICU patient receiving
high doses of these medications. Use of longer-acting agents given orally or by feeding tube has been
described to assist in the transition off long-term continuous infusions. The medical indication and
dosing plan for using oral medications to taper off continuous infusions should be clearly documented
in the medical chart on patient discharge from the ICU. Clonidine is a potential consideration in patients
to aid in dexmedetomidine withdrawal.

Patient Cases

3. A 48-year-old man with cirrhosis and now hepatorenal syndrome was intubated for respiratory distress. He
has been receiving midazolam 1 mg/hour and fentanyl 75 mcg/hour for 2 days; his RASS (−4 to −5) and
CPOT has been 1 for 24 hours. Oxygen requirements have decreased, and vital signs are normal. Which is
the most appropriate change in his medications?
A. Discontinue midazolam; give as-needed lorazepam for agitated RASS score.
B. Decrease midazolam; give as-needed fentanyl for agitated RASS score.
C. Discontinue midazolam; initiate propofol drip.
D. Change midazolam to dexmedetomidine drip.

4. T.L. is a 55-year-old woman intubated for respiratory distress for severe pneumonia. She is receiving
fentanyl 50 mcg/hour and dexmedetomidine 1.0 mcg/kg/hour. Her home medications are confirmed to
include esomeprazole 20 mg daily, lorazepam 1 mg three times daily, and citalopram 10 mg daily. The nurse
reports intermittent agitation with tachycardia and a negative pain score. Which is the most appropriate
recommendation?
A. Increase fentanyl drip for agitated RASS score.
B. Reinitiate lorazepam and citalopram.
C. Give fentanyl boluses as needed for agitation.
D. Increase dexmedetomidine.

IV.  DELIRIUM IN THE INTENSIVE CARE UNIT

A. Delirium is an acute and fluctuating disturbance in consciousness resulting in the inability to receive,
process, store, or recall information. In the ICU, delirium may present as hyperactive (agitated and restless),
hypoactive (flat affect, apathy, lethargy, decreased responsiveness), or mixed hyper/hypoactive states.
Most common in the ICU are mixed and hypoactive states of delirium. Two screening tools are currently
recommended by the PAD guidelines: (1) the CAM-ICU and (2) the ICDSC. Both the CAM-ICU and the
ICDSC require a RASS (-2) or a SAS (3) or more alert to be completed.
1. The CAM-ICU assesses four features: (1) acute change or fluctuation in mental status from baseline,
(2) inattention, (3) altered level of consciousness, and (4) disorganized thinking. If features 1 and 2 plus
feature 3 or 4 are present, the patient is considered positive for delirium. Detailed training is available
at www.icudelirium.org.

ACCP Updates in Therapeutics® 2022: Critical Care Pharmacy Preparatory Review and Recertification Course

457
Management of PADIS and NMB in Adult Intensive Care Unit Patients

2. The ICDSC consists of eight items, evaluated during an 8- to 24-hour period. The eight symptoms
are level of consciousness, inattention, disorientation, hallucinations-delusions-psychosis, psychomotor
agitation or retardation, inappropriate speech or mood, sleep-wake cycle disturbances, and symptom
fluctuation. A point is given for any symptom that is present during the previous 24 hours; a score of 4
or higher indicates the presence of delirium.

B. Background – 30%–80% of ICU patients reportedly develop delirium, depending on the severity of illness
and the diagnostic method, yet assessment for delirium is still not routine in most U.S. ICUs. During a
patient’s hospitalization, the presence of delirium is associated with difficulty in weaning mechanical
ventilation and longer duration of mechanical ventilation, increased use of physical and chemical restraints,
longer duration of ICU stay, and additional stress to family and friends who may not understand the course
of delirium. Delirium is also associated with up to a 3-fold increase in mortality, increase in cognitive
decline, delay in cognitive recovery, and increased likelihood of being discharged to a nursing home. Two
studies found that a longer duration of delirium was independently associated with worse activity of daily
living scores and worse cognitive impairment scores at 3 and 12 months post-ICU discharge (Crit Care
Med 2014;42:369-77; N Engl J Med 2013;369:1306-16). A recent retrospective study reported increased
difficulty in the weaning of mechanical ventilation when delirium was detected in patients during the first
spontaneous weaning trial compared with in patients who did not have delirium (Respirology Nov 2015;1-8).
1. The underlying pathophysiology of delirium is not well understood; however, it may involve a complex
set of factors:
a. Cerebral hypoperfusion and alterations in cerebral blood flow
b. Degradation of the blood-brain barrier, causing influx of inflammatory cytokines and microvascular
thrombosis
c. Depletion in central neurotransmitters (e.g., dopamine, norepinephrine, serotonin)
d. Depletion in acetylcholine
e. Medication withdrawal
2. Risk factors for delirium: A systematic review of studies from 2001 to 2013 described 11 variables
identified as risk factors for developing delirium in the ICU, extracting from only a strong or moderate
level of evidence (Crit Care Med 2015;43:40-7):
a. Age
b. Preexisting dementia
c. History of baseline hypertension
d. Sedative-associated coma
e. APACHE II (Acute Physiology and Chronic Health Evaluation II) score
f. Delirium on the previous day
g. Emergency surgery
h. Mechanical ventilation
i. Organ failure
j. (Poly)trauma
k. Metabolic acidosis
3. Other reported risk factors or precipitants:
a. Infection
b. Dehydration or malnutrition
c. Sleep deprivation
d. Centrally acting medications (benzodiazepines, opiates, anticholinergics)
e. Lack of exposure to sunlight
f. Lack of personal interaction
g. Physical restraints or insertion of catheters or tubes

ACCP Updates in Therapeutics® 2022: Critical Care Pharmacy Preparatory Review and Recertification Course

458
Management of PADIS and NMB in Adult Intensive Care Unit Patients

4. Medication-induced altered mental status – Although the development of delirium is considered


multifactorial, any patient who presents with a change in mental status should have his or her
medications and doses immediately scrutinized as part of the initial workup for delirium. Several classes
of medications have long been recognized for their effects on mental status and cognitive function, in
or out of the ICU. These medications have the potential to affect a patient’s level of consciousness
or course of delirium at any point in the patient’s hospital stay. Anticholinergics, benzodiazepines,
opiates, antipsychotics, antispasmodics, anticonvulsants, corticosteroids, and others should be used
with caution in a hospitalized patient, with close monitoring of the patient’s cognitive adverse effects.
Because renal and hepatic function may fluctuate throughout an ICU stay and affect the clearance
of these medications, doses must be thoughtfully titrated. Research on the degree of impact these
medications have on the overall course of sedation and delirium in the ICU is difficult to characterize,
and research is undergoing. Given the multifactorial nature of delirium in the ICU, clinicians should be
leery of solely assigning blame to medications but should remain vigilant when assessing the need and
doses of the aforementioned medications.
a. Benzodiazepines: The PADIS guidelines state “benzodiazepine use may be a risk factor for
the development of delirium in adult ICU patients.” Of interest, research directed at finding an
independent association between the use of benzodiazepines and the development of delirium in
the ICU including a meta-analysis, a randomized trial, and a systematic review has yielded mixed
results (Crit Care Med 2015;43:40-7; Crit Care Med 2015;43:557-66; Crit Care Med 2013;41:S30-
8). Recently, investigators used more appropriate statistical analysis for a fluctuating illness such
as delirium (e.g., Markov monitoring was used to determine the probability of a daily transition
to delirium while assuming this was independent of the patient history beyond the prior day) in
addition to more frequent delirium monitoring to examine the use of benzodiazepines and the
transition from an awake state without delirium to delirium, or from coma to delirium by the
next day (Intensive Care Med 2015;41:2130-7). This study found that a midazolam equivalent
dose of just 5 mg/day increased the odds of developing delirium the next day by 4%, and the
use of benzodiazepine infusions was an independent risk factor for the transition to delirium in
the study population. However, there was a large difference in the daily dose of benzodiazepines
between the continuous infusion (daily median dose 99 mg) compared with the intermittent dosing
group (daily median dose 4.1 mg), bringing into question whether the risk factor for the transition
to delirium was indeed the cumulative dose and not the method of administration. As the data
evolve, the scrutiny of benzodiazepine use should persist for ICU clinicians. Routine strategies
to preferentially use a non-benzodiazepine sedative (e.g., propofol or dexmedetomidine) and to
avoid benzodiazepine infusions unless clinically indicated should be used in addition to diligent
performance of daily SATs to discontinue use as soon as possible.
b. Anticholinergics: These medications are known for their sedating and altering effects on mentation
and should be avoided or used with extreme caution in the ICU setting. One proposed mechanism
for delirium is a decline in acetylcholine concentrations; therefore, any medication that may further
inhibit the activity of acetylcholine could worsen the patient’s mental status. A prospective cohort
study of 1112 critically ill patients was conducted to determine whether anticholinergic exposure
increased the probability of transitioning to delirium. On 6% of ICU days, transition from “awake
and without delirium” to “delirium” occurred. A nonsignificant increase in the probability of
transitioning to delirium the following day resulted from a 1-unit increase in the Anticholinergic
Drug Scale (odds ratio 1.05; 95% CI, 0.99–1.10). The dose of the medication was not evaluated
to determine the effects on the transition to delirium, and it was not evaluated if patients were
already delirious and remained delirious while receiving anticholinergic medications (Crit Care
Med 2015;43:1846-52).

ACCP Updates in Therapeutics® 2022: Critical Care Pharmacy Preparatory Review and Recertification Course

459
Management of PADIS and NMB in Adult Intensive Care Unit Patients

c. Systemic corticosteroids: The neuropsychiatric effects of systemic steroids in various clinical


settings have been described for more than 50 years. Common symptoms include mania,
depression, mood lability, anxiety, insomnia, delirium, and psychosis. The incidence of these
symptoms varies greatly depending on the clinical setting, dose of steroid, and patient’s underlying
medical history. Reported risk factors for neuropsychiatric effects secondary to steroids include a
daily prednisone dose equivalent to 40 mg or greater, hypoalbuminemia, underlying psychiatric
disorder, and blood-brain barrier damage (Curr Opin Organ Transplant 2014;19:201-8). Although
steroids are commonly used in the ICU for various indications and at various doses, significant
research directed at the neuropsychiatric effects of steroids in the critically ill population is
lacking. In a secondary analysis of a multicenter observational study of adult medical and surgical
ICU patients with acute lung injury (n=330), Schreiber et al. found a significant and independent
association between the use of systemic corticosteroids and the transition to delirium from a non-
comatose, non-delirious state within 24 hours of corticosteroid administration (odds ratio [OR]
1.52 [1.05–2.21], p=0.03). Delirium was documented on one or more days in 83% of patients, with
a median duration of 7 days. There was no significant association in prednisone-equivalent dose
and transition to delirium. Schreiber et al. recognize that a direct causal relationship could not be
determined between corticosteroid use and delirium from this observational study; however, they
believe that the study adds valuable data toward our understanding of risk factors for delirium in
the ICU (Crit Care Med 2014;42:1480-6). A second study that investigated steroids and transition
to delirium in a mixed medical and surgical ICU population (n=1112) found no association between
steroid use and a transition to delirium. The median prednisone equivalent dose was 50 mg (Crit
Care Med 2015;43:e585-8).
5. Outcomes of sedation-related versus illness-related delirium: A single-center study using propofol
and fentanyl timed its CAM-ICU assessments before and after a daily sedation interruption protocol.
Rapidly reversible delirium was defined as delirium while patients were receiving sedation that resolved
within 2 hours after performing an SAT. This type of delirium was rare (12% of the 102 patients), but
these patients has a prognosis that was similar to patients who did not have delirium. Most patients
(75%) had persistent delirium, delirium that did not resolve with cessation of sedatives, a higher risk
of death, and longer length of stay (Am J Respir Crit Care Med 2014;189:658-65). Patients can have
both sedation- and illness-related delirium, and additional research in this area is needed to clarify the
differences in short- and long-term outcomes.

C. Monitoring for Delirium: SCCM recommends assessing critically ill patients for delirium with a validated
tool such as either the CAM-ICU or the ICDSC. The PAD guidelines summarized their review of five
delirium assessment scales used for adult ICU patients. The two scales with the highest psychometric (e.g.,
validity and reliability) scores were the CAM-ICU and the ICDSC. Both scales were designed for patients in
the ICU either on or off mechanical ventilation, and both showed high sensitivity and specificity when tested
against the American Psychiatric Association’s criteria for delirium.
1. Delirium should be assessed at least every 8-12 hours and documented in the medical chart; results
should be discussed with the medical team. Because these assessment scales cannot distinguish
between sedation- and disease-related causes of delirium, delirium assessments should ideally be timed
both before and after SATs with appropriate time allowed for drug clearance (www.icudelirium.org,
Am J Respir Crit Care Med 2014;189:658-65). If this timing is not feasible and a patient screens positive
for delirium while receiving ongoing analgesia or sedation, an SAT should be conducted if the patient
passes the safety screen to assist in ruling out a medication-induced cause of delirium.

ACCP Updates in Therapeutics® 2022: Critical Care Pharmacy Preparatory Review and Recertification Course

460
Management of PADIS and NMB in Adult Intensive Care Unit Patients

2. If a patient’s delirium score is positive, the medical team should correct possible etiologies (e.g.,
decrease sedative doses, if safe), decrease ongoing risk factors, address inciting factors (e.g., metabolic
derangements, infection, withdrawal), and try nonpharmacologic treatment and preventive measures
when appropriate.

D. Prevention of Delirium: With a lack of data supporting the use of pharmacologic agents to prevent delirium,
the PADIS guidelines suggest not using haloperidol, an atypical antipsychotic, dexmedetomidine, a statin,
or ketamine to prevent delirium in critically ill patients. Instead, the recommendations are focused on
nonpharmacologic prevention methods when feasible, particularly for patients at high risk of delirium.
Preventive efforts may help avert 30%–40% of new-onset delirium cases, particularly in older adults.
Recommended nonpharmacologic strategies by the PADIS guidelines include:
1. Performing rehabilitation or mobilization in critically ill adults
2. Using a multicomponent intervention to reduce or shorten delirium (e.g., reorientation, cognitive
stimulation, use of clocks); improve sleep (e.g., minimize light and noise); improve wakefulness (i.e.,
reduce sedation); reduce immobility (e.g., early rehabilitation/mobilization); and reduce hearing and/or
visual impairment (e.g., enable use of devices such as hearing aids or eyeglasses).
3. Decreasing the use of benzodiazepines and anticholinergics in patients at risk of delirium; use the
lowest effective doses of any sedating medication (e.g., opiates, antipsychotics).
4. REDUCE was a randomized, double-blind, placebo-controlled study of 1789 critically ill patients
randomized to prophylactic haloperidol 1 mg, haloperidol 2 mg, or placebo. The 1-mg haloperidol group
was prematurely stopped because of futility (JAMA 2018;319:680-90). No difference occurred in the
median days that patients survived in 28 days in the 2-mg haloperidol group compared with 28 days in
the placebo group (95% CI, 0-0; p=0.93) with a hazard ratio of 1.003 (95% CI, 0.78–1.30; p=0.82). None
of the 15 secondary outcomes were statistically different between the three groups. These outcomes
included delirium incidence (mean difference 1.5%; 95% CI, −3.6% to 6.7%), delirium- and coma-free
days (mean difference 0 days; 95% CI, 0–0 days), and duration of mechanical ventilation, ICU, and
hospital length of stay (mean difference 0 days; 95% CI, 0–0 days for all three measures). Adverse
events did not differ between groups.

E. Sleep in the ICU: Uninterrupted sleep (ideally 4 hours or more) is vital for an adequate immune response to
illness, to maintain normal metabolic and hormonal balance, and to help decrease delirium and/or agitation.
Disturbances in the ICU such as multiple alarms and frequent physical interruptions (e.g., examination,
turning, laboratory tests, medication administration) make it challenging for patients to maintain the slow-
wave sleep cycle needed for optimal immune function. Sleep research in the ICU is ongoing, and more
information will be forthcoming regarding its effects in the critically ill patient. The PADIS guidelines
suggest not routinely using physiologic sleep monitoring clinically in critically ill adults. In addition, the
PADIS guidelines suggest not using aromatherapy, acupressure, or music at night to improve sleep in
critically ill adults. However the PADIS guidelines do suggest using noise and light reduction strategies
to improve sleep in critically ill adults. The PADIS guidelines make no recommendation regarding the
use of melatonin to improve sleep or regarding the use of dexmedetomidine at night to improve sleep. The
PADIS guidelines suggest not using propofol to improve sleep in critically ill adults. In addition, the PADIS
guidelines suggest using a sleep-promoting, multicomponent protocol in critically ill adults.
1. To avoid waking patients at night, pharmacists should ensure that medications are scheduled during
the daytime and evening hours, if possible—particularly orally or subcutaneously administered
medications.
2. Sleep protocols should seek to cluster patient care activities (e.g., vital sign checks, radiology tests,
laboratory checks, sedation assessments) around nighttime sleeping hours unless clinically indicated in
a specific patient population.

ACCP Updates in Therapeutics® 2022: Critical Care Pharmacy Preparatory Review and Recertification Course

461
Management of PADIS and NMB in Adult Intensive Care Unit Patients

3. A two-center, double-blind randomized trial randomized 100 critically ill adults without delirium to
nocturnal dexmedetomidine or placebo (Am J Respir Crit Care Med 2018;197:1147-56). During the ICU
stay, nocturnal dexmedetomidine was associated with a greater proportion of patients who remained
delirium free (dexmedetomidine (40 of 50 patients [80%]) than with placebo (27 of 50 patients [54%];
relative risk 0.44; 95% CI, 0.23–0.82; p=0.006). Adverse events did not differ between the two groups.

F. Treatment of Delirium: The cause of delirium may be multifactorial, and identifying and correcting the
underlying etiology is the first step in management. Patients can also progress to alcohol withdrawal or
withdrawal from other chronic medications/substances and present with hyperactive delirium. The PADIS
guidelines suggest not using an atypical antipsychotic, haloperidol, or a statin to treat subsyndromal delirium
or delirium (N Engl J Med 2018;379:2506-16). The Modifying the Impact of the ICU-Associated Neurological
Dysfunction-USA (MIND USA) Study is a multicenter, randomized, placebo-controlled study of 566 patients
showing that haloperidol and ziprasidone did not reduce delirium, time on the ventilator, ICU or hospital
length of stay, or death compared with placebo. Arrhythmias, parkinsonism (extrapyramidal symptoms),
neuroleptic malignant syndrome, study drug discontinuation, and other safety concerns were extremely low
across all three groups. Antipsychotics remain viable for the short-term control of agitation (e.g., alcohol
or drug withdrawal) or severe anxiety with the need to avoid respiratory suppression (e.g., heart failure,
COPD, or asthma). If an antipsychotic is initiated, low starting doses should be considered, and daily review
of drug interactions, adverse effects, dosing titration, and need for the antipsychotic should be completed.
In addition, a strategy for discontinuation or outpatient follow-up should be documented to help avoid
inadvertent continuation beyond the hospital environment (Table 7). Serious adverse effects are associated
with the use of any antipsychotic; effects such as arrhythmias, serotonin syndrome, neuroleptic malignant
syndrome, extrapyramidal symptoms, and oversedation should be closely monitored on a daily basis. Dose
ranges for atypical antipsychotics for ICU delirium are not well described. The American Geriatrics Society
2015 Beers Criteria for medication use in older adults includes the following recommendation: “Avoid
antipsychotics for behavioral problems of dementia or delirium unless nonpharmacological options have
failed or are not possible AND the older adult is threatening substantial harm to self or others.” If the ICU
team decides to use antipsychotics in older adults, lower starting doses should be considered, together with
daily review of drug interactions and adverse effects. The PADIS guidelines suggest using dexmedetomidine
for delirium in mechanically ventilated adults when agitation precludes weaning/extubation. The PADIS
guidelines suggest not using bright light therapy to reduce delirium in critically ill adults.

G. Inadvertent Continuation of Antipsychotics Beyond ICU Discharge: The PADIS guidelines discuss
the use of adjunctive medications for ICU patients (e.g., antipsychotics, gabapentin, carbamazepine).
Although their use in the ICU may be appropriate, there is a potential for inadvertent continuation of these
medications on hospital discharge if a treatment plan is not clear in the medical record. This has been a well-
documented problem with other medications initiated in the ICU (e.g., histamine receptor blockers, proton
pump inhibitors), and recent studies have been published describing the continuation of newly prescribed
antipsychotics from the ICU and hospital, even when an indication for continuation was not documented
(J Crit Care 2015;30:814-6; J Crit Care 2016;33:119-24). Continued use of these medications beyond the
hospital stay could lead to serious adverse effects, drug interactions, and significant drug cost as well as a
presumption of a psychiatric or neuromuscular disorder associated with these drugs. Communication to the
next direct patient care provider is crucial to appropriately direct the next steps in medication reconciliation.

ACCP Updates in Therapeutics® 2022: Critical Care Pharmacy Preparatory Review and Recertification Course

462
Management of PADIS and NMB in Adult Intensive Care Unit Patients

Table 7. Antipsychoticsa
CYP Substrate Usual
Drug Significant Adverse Effectsb Formulations
(major) Starting Dose
Anticholinergic: *
1–2 mg older adults;
3A4, 2D6 Sedation: *
Haloperidol 2–4 mg if history of PO, IM, IV 

EPS: **
psychiatric disorders
NMS: *
Anticholinergic: **
Sedation: **
PO, disintegrating
Olanzapine 1A2 5 mg EPS: *
tablet, IV, IM
NMS: *
Neuromuscular weakness
Anticholinergic: **
Sedation: **
Quetiapine 3A4 12.5–25 mg PO
NMS: *
Orthostatic hypotension: **
Anticholinergic: *
Sedation *
EPS: ** PO, disintegrating
Risperidone 2D6 0.5–1 mg
NMS: * tablet
Orthostatic hypotension: **
Cardiac conduction abnormalities
Anticholinergic: *
1A2 (minor) 20 mg PO; Sedation: * Oral,
Ziprasidone
3A4 (minor) 10 mg IM EPS: * IM
NMS: *
NOTE: * = lower risk; ** = medium-higher risk
a
Not all medications listed are FDA label approved for use in delirium; not all are recommended by SCCM for the treatment of delirium in the ICU.
b
Adverse effects other than QTc prolongation. Documented QTc prolongation incidence: IV haloperidol = ziprasidone > risperidone > olanzapine = quetiapine.
EPS = extrapyramidal symptoms; IM = intramuscular(ly); IV = intravenous(ly); NMS = neuroleptic malignant syndrome; PO = oral(ly)

1. Quetiapine (Seroquel): A randomized, placebo-controlled pilot trial compared the efficacy and safety
of scheduled quetiapine with placebo for the treatment of delirium in ICU patients during a 10-day
study (Crit Care Med 2010;38:419-27). Significant exclusions were as follows: patients with end-stage
liver disease, those with alcohol withdrawal, those with a QTc greater than 500, and those receiving
concomitant QTc-prolonging agents. This small pilot study (n=36), in which the placebo group was
administered as-needed intravenous haloperidol, found that quetiapine was associated with a shorter time
to first resolution of delirium, reduced duration of delirium, and less agitation than placebo. Mortality
and ICU length of stay were not different from placebo.
a. Pharmacokinetics: Hepatically metabolized to one active and two inactive metabolites. Metabolites
renally cleared. Many drug interactions, CYP3A4 (major) and CYP2D6 (minor) substrates. Peak
plasma concentrations for oral about 1½ hours (immediate release).
b. Initial dose range for ICU delirium: 50 mg one to three times daily. Consider lower starting doses
for older adult patients because of sedating effects. The Devlin study initiated 50 mg every 12
hours and titrated to a maximum dose of 200 mg every 12 hours.
c. Adverse effects (early onset): Sedation, orthostatic hypotension, extrapyramidal symptoms, QTc
prolongation

ACCP Updates in Therapeutics® 2022: Critical Care Pharmacy Preparatory Review and Recertification Course

463
Management of PADIS and NMB in Adult Intensive Care Unit Patients

2. Olanzapine (Zyprexa): Available in oral, orally disintegrating, and intramuscular (immediate and
extended release) dosage forms. Intramuscular administration may result in plasma concentrations 5
times those of oral administration. The U.S. Food and Drug Administration (FDA) warns that the use
of intramuscular olanzapine has resulted in unexplained deaths; use of intramuscular olanzapine with
benzodiazepines may result in significant oxygen desaturation.
a. Pharmacokinetics: Metabolized by glucuronidation and CYP 1A2, 2D6 oxidation. Clearance is
significantly increased (around 40%) in smokers and decreased in females (around 30%). Many
drug interactions, CYP1A2 (major) and CYP2D6 (minor) substrates. Weak inhibitor of several
CYP isoenzymes. Peak plasma concentrations for oral: About 6 hours.
b. Suggested starting dose for ICU delirium: 5 mg orally once daily
c. Adverse effects (early onset): Drowsiness, extrapyramidal symptoms, neuromuscular weakness,
serotonin syndrome. High doses may cause cardiac arrhythmias, cardiopulmonary arrest, and
extreme sedation to coma-like states.
3. Risperidone (Risperdal): Available in oral and oral dispersible tablets (M-tabs) and intramuscular
injection dosage forms
a. Pharmacokinetics: Hepatically metabolized to active metabolites, renally cleared. Many drug
interactions, CYP2D6 (major) and CYP3A4 (minor) substrates and P-glycoprotein. Peak plasma
concentrations for oral about 1 hour.
b. Suggested starting dose for ICU delirium: 0.25–0.5 mg once or twice daily
c. Adverse effects (early onset): Cardiac arrhythmias, anticholinergic effects, extrapyramidal
symptoms
4. Ziprasidone (Geodon): Studied in a multicenter, randomized, placebo-controlled pilot trial of
mechanically ventilated patients to test the hypothesis that antipsychotics would improve days alive
without delirium or coma in the ICU (MIND trial). Medical and surgical adult ICU patients (n=101)
from six tertiary-care centers in the United States on mechanical ventilation who had an abnormal level
of consciousness or were receiving analgesia/sedative medications were randomly assigned to receive
haloperidol, ziprasidone, or placebo every 6 hours for up to 14 days during a 21-day study. During the
study, no difference was found in median days alive without delirium or coma between the haloperidol
(14 days), ziprasidone (15 days), and placebo (12.5 days) groups, p=0.66. The study also found no
difference in ventilator-free days, hospital length of stay, or mortality among the three groups (Crit Care
Med 2010;38:428-37).
a. Ziprasidone is available in oral and intramuscular dosage forms.
b. Pharmacokinetics: Hepatic by glutathione and aldehyde oxidase. Minor substrates of CYP 1A2,
3A4. Peak plasma concentrations for oral about 6 hours; intramuscular about 1 hour.
c. Suggested starting dose for ICU delirium: 20 mg twice daily (oral)
d. Adverse effects (early onset): Somnolence, extrapyramidal symptoms, dizziness, orthostatic
hypotension

ACCP Updates in Therapeutics® 2022: Critical Care Pharmacy Preparatory Review and Recertification Course

464
Management of PADIS and NMB in Adult Intensive Care Unit Patients

V.  ABCDEF BUNDLE

A. Incorporating multiple concomitant patient care interventions into one consolidated bundle may be an
effective strategy to improve clinical outcomes in critically ill patients. SCCM recommends implementing the
“ABCDEF” bundle to align and coordinate care using an interprofessional approach (e.g., physician, nursing,
pharmacy, respiratory therapy, physical and occupational therapy). The following practice principles are
applied to the bundle:
1. A: Assess, prevent, and manage pain
2. B: Both SATs and SBTs
3. C: Choice of analgesia and sedation
4. D: Delirium: Assess, prevent, and manage
5. E: Early mobility and exercise
6. F: Family engagement and empowerment.

B. Assess, Prevent, and Manage Pain (“A” of the bundle): Asking the patient to self-report pain or to use the
CPOT or BPS if the patient is nonverbal. Preventing pain by recognizing patients with known sources of pain
(i.e., rib fractures) and scheduling analgesics when indicated. Managing pain by ordering the most appropriate
pharmacologic agent on the basis of the source of pain and renal and liver function.

C. Daily coordination of the SAT with the SBT (“B” of the bundle) versus usual care with the SBT has been
shown to significantly decrease the time on mechanical ventilation and ICU length of stay in randomized
studies. This was reviewed earlier in the chapter in the Agitation section.

D. Choice of Sedation (“C” of the bundle): Use a multidisciplinary approach, including focused pharmacy
input, to choose a sedative according to individual patient needs, hemodynamic stability, and organ function
(e.g., hepatic, renal, cardiac, pulmonary, pancreatic).

E. Delirium Assessment, Prevention, and Management (“D” of the bundle): Regularly assess for delirium
using the CAM-ICU or the ICDSC every 8–12 hours. Use delirium preventive measures in all patients when
safe to do so.

F. Early Mobility (“E” of the bundle): Perform a mobility safety screen, and implement a daily mobility
protocol.

G. Family Engagement and Empowerment (“F” of the bundle): Good communication with the family is critical
at every step of the patient’s clinical course, and empowering the family to be part of the team to ensure best
care is adhered to diligently will improve many aspects of the patient’s experience.

H. In a prospective, multicenter, cohort study from 68 academic, community, and federal ICUs during a
20-month collection period, performance of the complete ABCDEF bundle was associated with a lower
likelihood of death within 7 days (HR 0.32; CI, 0.17–0.62), next-day mechanical ventilation (OR 0.28; CI,
0.22–0.36), coma (OR 0.35; CI, 0.22–0.56), delirium (OR 0.60; CI, 0.49–0.72), physical restraint use (OR
0.37; CI, 0.30–0.46), ICU readmission (OR 0.54; CI, 0.37–0.79), and discharge to a facility other than home
(OR 0.64; CI, 0.51–0.80). There was a dose response in between higher proportional bundle performance and
improvement in each clinical outcome (p<0.002). Pain was more commonly reported as bundle performance
increased (p=0.0001), probably because more patients were awake.

ACCP Updates in Therapeutics® 2022: Critical Care Pharmacy Preparatory Review and Recertification Course

465
Management of PADIS and NMB in Adult Intensive Care Unit Patients

VI.  POST-INTENSIVE CARE SYNDROME

A. Advances in critical care have decreased mortality and resulted in an increased likelihood of surviving
critical illness. Post-intensive care syndrome (PICS) describes new or worsening impairments in physical,
cognitive, or mental health status after critical illness and persisting beyond acute care hospitalization.
Physical impairments include both pulmonary dysfunction and neuromuscular weakness. Impairments
in memory and executive functioning are examples of cognitive dysfunction. Mental health impairments
include depression, posttraumatic stress disorder, and anxiety. Medication management including glucose
control, use of the ABCDEF bundle, and review of medication lists at every transition of care are important
roles of the pharmacist to prevent PICS. Family members can also have PICS, termed PICS-F, in which these
individuals have depression, posttraumatic stress disorder, anxiety, and prolonged grief. SCCM has taken
progressive steps to help clinicians and families recognize the prolonged PICS and PICS-F through THRIVE.
The establishment of an ICU follow-up clinic is one proposed method to manage long-term complications
of patients with PICS and family members of PICS-F through optimization of physical, cognitive, and
mental health; improved coordination of care; and reduction in health care use. The pharmacist should be
considered a key member of the PICS clinic team who performs complete medication management on all
patients seen in the clinic.
1. Physical impairment: A prospective, longitudinal study of 109 survivors with ARDS was conducted.
Median (Interquartile Range) Total Lung Capacity (TVC) was 92% (77-97%), 92% (82–101%), and
95% (81–103%) of predicted value, respectively, at 1, 6, and 12 months after ICU discharge. Forced
expiratory volume in 1 second (FEV1) was 75% (58–92%), 85% (69–98%), and 86% (74–100%) of
predicted value, respectively, at 1, 6, and 12 months after ICU discharge. Six-minute walk test was 49%,
64%, and 66% of predicted value, respectively, at 1, 6, and 12 months after ICU discharge. These 109
survivors of ARDS were further analyzed annually up to 5 years after ICU discharge. TVC was 94%
(84–108%), 93% (78–107%), 92% (79–104%), and 94% (78–105%) of predicted value, respectively, at
2, 3, 4, and 5 years after ICU discharge. FEV1 was 87% (75–99%), 79% (66–97%), 85% (68–98%), and
83% (69–98%) of predicted value, respectively, at 2, 3, 4, and 5 years after ICU discharge. Six-minute
walk test was 68%, 67%, 71%, and 76% of predicted value, respectively, at 2, 3, 4, and 5 years after
ICU discharge.
2. Cognitive impairment: A large, multicenter, prospective observational cohort study of 821 adult medical
ICU and surgical ICU patients (called the BRAIN-ICU study) estimated the prevalence of long-term
cognitive impairment after critical illness secondary to respiratory failure, cardiogenic shock, or septic
shock (N Engl J Med 2013;369:1306-16). Delirium was the strongest independent predictor of cognitive
impairment in the 50% of patients after critical illness. A Repeatable Battery for Neuropsychological
Status (RBANS) score similar to Alzheimer disease (2 standard deviations below the population mean)
was found in 26% of patients, and a score similar to moderate traumatic brain injury (1.5 standard
deviations below the population mean) was found in 40% of patients 3 months after discharge. Both
young and older adults, with and without comorbidities, experienced these deficits, which persisted at
12 months in 24% and 34% of these individuals having RBANS scores similar to Alzheimer disease
and moderate traumatic brain injury, respectively.
3. Mental health impairments: A multicenter, prospective observational cohort study of 821 adult medical
ICU and surgical ICU patients estimate the prevalence of depression, posttraumatic stress disorder
(PTSD), and functional disability after critical illness secondary to respiratory failure, cardiogenic
shock, or septic shock. Depression was found in 149 patients (37%) and 116 patients (33%) at 3 and
6 months after ICU discharge, respectively. Disabilities in basic activities of daily living were found
in 139 patients (32%) and 102 patients (23%) at 3 and 6 months after ICU discharge, respectively.
Disabilities in instrumental activities of daily living were found in 108 patients (26%) and 87 patients
(23%) at 3 and 6 months after ICU discharge, respectively. PTSD was found in 27 patients (7%) at 3 and
6 months after ICU discharge.

ACCP Updates in Therapeutics® 2022: Critical Care Pharmacy Preparatory Review and Recertification Course

466
Management of PADIS and NMB in Adult Intensive Care Unit Patients

4. A prospective, observational feasibility study was conducted at an academic hospital between July 2012
and December 2015 (Crit Care 2018;46:141-8). Patients were identified if at high risk of PICS; if so, post-
ICU care was offered. Sixty-two patients were seen in the clinic. Median time from hospital discharge to
ICU recovery center visit was 29 days. Cognitive impairment was identified in 64% of patients. Anxiety
and depression were identified in 37% and 27% of patients. One-third of the patients were unable to
ambulate independently. The median 6-minute walk distance was 56% of predicted. Only seven of the
previously working patients (15%) had returned to work. Referral services and case management were
provided 142 times. The median number of pharmacy interventions per patient was four.

B. Medication Management
1. Glucose dysregulation: A retrospective study of 74 patients with ARDS found that a blood glucose
value of 153.5 mg/dL resulted in a 2.9 greater chance of developing cognitive impairment. A second
retrospective case-control study of 37 surgical ICU patients with at least one episode of hypoglycemia
found that cognitive impairment was higher in the hypoglycemic group (p<0.01). Intensive insulin
therapy (maintaining blood glucose levels between 80 and 100 mg/dL) in surgical ICU patients
decreased neuropathy from 51.9% to 28.7%, and the prevalence of critical illness polyneuropathy (CIP)
and critical illness myopathy (CIM) from 49% to 25% in surgical ICU patients (p<0.0001) (Neurology
2005;64:1348-53). Intensive insulin therapy also decreased CIP and CIM from 51% to 39% in the medical
ICU (p=0.02) in patients who had an ICU stay of at least 1 week. The percentage of patients needing
mechanical ventilation for at least 2 weeks was reduced from 42% to 32% in the surgical ICU (p=0.04)
and from 47% to 35% in the medical ICU (p=0.01). Subsequently, NICE-SUGAR showed increased
mortality in the intensive insulin group (81 to 108 mg/dL) (27.5%) versus conventional glucose control
(less than 180 mg/dL) (24.5%) (p=0.02). The SCCM guidelines for the use of an insulin infusion for the
management of hyperglycemia in critically ill patients suggests that a blood glucose of 150 mg/dL or
greater initiates interventions to maintain blood glucose less than 180 mgdL and to avoid hypoglycemia
based off the results of NICE-SUGAR.
2. Continuation of inappropriate medications: The frequency of prescribed potentially inappropriate
medications (PIMs) and actually inappropriate medications (AIMs) was evaluated in a single-center
study of 120 older adult ICU survivors. PIMs were defined as potentially harmful on the basis of prior
studies and pharmacologic effects. PIMs could further be classified as AIMs if the benefit of the drug
was considered less than the harm. The 2003 Beers Criteria and medication safety data published since
2003 were used to identify medications (Arch Intern Med. 2011;171:1032-4). Medications were identified
at five points during the hospital stay: admission, ward admission, ICU admission, ICU discharge, and
hospital discharge. The most common categories of PIMs identified at hospital discharge were opioids,
anticholinergic medications, antidepressants, and drugs causing orthostasis. Thirty-six percent of
these PIMs were considered AIMs. The PIM categories at hospital discharge with the highest positive
predictive values for being AIMs were anticholinergics (55%), nonbenzodiazepine hypnotics (67%),
benzodiazepines (67%), atypical antipsychotics (71%), and muscle relaxants (100%). In multivariate
analysis, the number of discharge PIMs was independently predicted by the number of preadmission
PIMs (p<0.001), discharge to somewhere other than home (p=0.03), and discharge from a surgical
service (p<0.001). Almost two-thirds of AIMs were initiated in the ICU.
3. Not restarting home medications: A large population-based Canadian cohort study of 396,380 patients
evaluated records of hospital and outpatient medications prescribed from at least one of five of the
following groups: (1) statins, (2) antiplatelet/anticoagulant agents, (3) levothyroxine, (4) respiratory
inhalers, and (5) gastric acid–suppressing drugs. Patients were divided into three groups: hospitalization
with an ICU admission, hospitalization without ICU admission, and nonhospitalized patients who
served as the control group. Compared with control patients, those admitted to a hospital without an
ICU stay were more likely to have medications discontinued among all five of the medication groups.

ACCP Updates in Therapeutics® 2022: Critical Care Pharmacy Preparatory Review and Recertification Course

467
Management of PADIS and NMB in Adult Intensive Care Unit Patients

Patients admitted to a hospital with an ICU stay were also more likely to have medications discontinued
among all five of the medication groups than were control patients. Except for respiratory inhalers,
there was a higher risk of medication discontinuation in all medication groups in patients hospitalized
with an ICU admission than in patients hospitalized without an ICU admission. The composite outcome
of death, hospitalization, and emergency department visit up to 1 year after hospital discharge in all
study patients was higher in patients in whom a statin or antiplatelet or anticoagulant was discontinued
(JAMA. 2011;306:840-7).
4. A prospective, observational cohort study was conducted of all outpatient appointments of a tertiary
care hospital’s post-ICU clinic between July 2012 and December 2015. The pharmacist completed
medication reconciliation, interview, counseling, and resultant interventions during the post-ICU clinic
appointment. The pharmacist did a full medication review in 56 of the patients (90%). All 56 patients
had at least one pharmacy intervention. Medications were discontinued at the clinic appointment for 22
of the patients (39%). New medications were initiated in 18 of the patients (32%). An adverse drug event
was identified in nine of the patients (18%). Adverse drug event preventive measures were implemented
in 18 patients (32%). Thirteen patients (23%) had an influenza vaccination administered. Two patients
(4%) received a pneumococcal vaccination (Ann Pharmacother 2018;52:713-23).

Patient Cases

5. T.L. (from question 4) was extubated 24 hours ago, is currently receiving dexmedetomidine 0.2 mcg/kg/
hour, and has received two doses of fentanyl 25 mcg over 24 hours for pain. She is alert and calm with
intermittent periods of agitation. Her pain score is now negative, and she is newly positive for delirium by
CAM-ICU. Her laboratory values and vital signs are normal. Which would best be recommended for the
management of delirium?
A. Continue dexmedetomidine, and start quetiapine for delirium.
B. Discontinue dexmedetomidine, and increase maintenance fluids for dehydration.
C. Discontinue dexmedetomidine, and order patient mobility as tolerated.
D. Continue dexmedetomidine, and schedule oxycodone sustained release every 12 hours.

6. P.V. is a 70-year-old woman intubated for severe respiratory failure (Fio2 80%) and refractory shock from
methicillin-resistant Staphylococcus aureus pneumonia, for which she was administered antibiotics, vaso-
pressors, and steroids. She is on day 5 of mechanical ventilation (Fio2 50%) and has been off vasopressors for
48 hours. The nurse describes PAD, but the patient denies pain. Medications include vancomycin 1000 mg
daily, heparin 5000 units subcutaneously every 12 hours, hydrocortisone 50 mg every 6 hours, and fentanyl
75 mcg/hour. Which is the most appropriate recommendation at this time?
A. Increase fentanyl, and add midazolam for agitation.
B. Decrease fentanyl, and discontinue hydrocortisone.
C. Decrease fentanyl, and add haloperidol for delirium.
D. Increase fentanyl, and change vancomycin to linezolid.

ACCP Updates in Therapeutics® 2022: Critical Care Pharmacy Preparatory Review and Recertification Course

468
Management of PADIS and NMB in Adult Intensive Care Unit Patients

VII.  NEUROMUSCULAR BLOCKADE IN THE INTENSIVE CARE UNIT

A. The most recent SCCM guidelines for the sustained use of neuromuscular blockade in the ICU were published
in 2016. Surveys have reported a dramatic decrease in the use of NMBAs during the past 20 years, from
around 80% to 15% in patients on mechanical ventilation (Crit Care Med 2016;44:2079-103). This change in
practice may be secondary to a better understanding of the serious adverse effects of prolonged paralysis,
together with accepted standards of care for modes of mechanical ventilation in patients with ARDS.

B. Clinical Scenarios for the Use of NMBAs in the ICU May Include:
1. Rapid sequence intubation
2. ARDS
3. Status asthmaticus
4. Elevated ICP
5. Elevated intra-abdominal pressure
6. Targeted temperature management after cardiac arrest

C. Acute Respiratory Distress Syndrome


1. Cisatracurium has been the most-studied NMBA for ARDS since 2000, primarily as short-term
treatment and in severe cases of ARDS. In 2010, a randomized placebo-controlled trial (n=340) found
that short-term fixed-dose cisatracurium (48 hours) significantly improved 90-day survival, increased
ventilator-free days, increased organ dysfunction–free days, and decreased barotrauma in patients with
severe ARDS (Pao2/Fio2 less than 120 mm Hg). The investigators found no difference in neuromuscular
weakness compared with placebo. Other cisatracurium studies have shown improvements in oxygenation
and a reduction in inflammatory mediators.
2. In retrospective studies, the use of NMBAs in ARDS was associated with a prolonged duration of
mechanical ventilation, prolonged ICU length of stay, and increased mortality.
3. Protective mechanisms of NMBAs in severe ARDS: Researchers have proposed mechanisms by which
an NMBA may protect the lung against further injury in severe ARDS. These mechanisms are not
completely understood, but they may help explain the beneficial effects of NMBAs in early, severe
ARDS:
a. Provide improved adaptation to the ventilator through increased thoracopulmonary compliance.
b. Increase functional residual capacity, and decrease intrapulmonary shunt.
c. Provide uniform distribution of pulmonary perfusion and pressures, favoring the perfusion of
ventilated areas.
d. Limit over-distention of high-compliance lung regions and recruit areas of smaller compliance.
e. Decrease muscular oxygen consumption by decreasing ventilator asynchrony.
f. Decrease production of proinflammatory cytokines in lungs and blood.
g. Provide protective role against ventilator-induced trauma, including decreased incidence of
pneumothoraces.
4. The 2016 SCCM NMBA guidelines suggest that a NMBA be administered by continuous intravenous
infusion early in the course of ARDS for patients with a Pao2/Fio2 of less than 150 mm Hg (weak
recommendation). Use of NMBAs in ARDS remains controversial. Short-term use of cisatracurium (48
hours or less) when used early may be beneficial for severe ARDS (Pao2/Fio2 less than 120 mm Hg). The
Reevaluation of Systemic Early Neuromuscular Blockade (ROSE) PETAL was a randomized, parallel-
design study comparing cisatracurium with placebo for 48 hours on the effects of 90-day mortality. It
was published after the guidelines and did not support using this early in course of ARDS.

ACCP Updates in Therapeutics® 2022: Critical Care Pharmacy Preparatory Review and Recertification Course

469
Management of PADIS and NMB in Adult Intensive Care Unit Patients

D. Targeted Temperature Management After Cardiac Arrest


1. The 2016 SCCM NMBA guidelines make no recommendation on the routine use of NMBAs for patients
undergoing therapeutic hypothermia after cardiac arrest (insufficient evidence). They recommend
use of a protocol that includes guidance on NMBA administration in patients undergoing therapeutic
hypothermia (good practice statement).
2. NMBAs have been used to prevent or treat shivering during therapeutic hypothermia.
3. The optimal combination and dosing of sedatives and paralytics have not been well established
because the metabolism of these drugs is significantly slowed during hypothermia, and potency may be
decreased. NMBAs have been used in both a bolus and a continuous infusion fashion during therapeutic
hypothermia.

E. Sedation During NMBA: The 2016 SCCM NMBA guidelines recommend that optimal clinical practice
require administering analgesic and sedative drugs before and during neuromuscular blockade with the goal
of achieving deep sedation (good practice statement). They also suggest that patients receiving a continuous
infusion of NMBA should receive a structured regimen of physiotherapy (weak recommendation). It is
critical that patients be in a sedated, non-agitated, and pain-free state before initiating an NMBA. Once
the patient becomes paralyzed from the NMBA, the ability to accurately assess mental status or pain is
ostensibly challenging and often unattainable. The deeper the degree of paralysis, the higher the risk of
drug accumulation because nurses cannot routinely complete sedation interruption or taper to a lighter
level of sedation. Common scenarios that slow the clearance of sedatives (e.g., hepatic and renal failure or a
hypothermic state) can add to the likelihood of increased drug exposure and delayed awakening times once
the paralytic and sedatives are discontinued. This risk of drug accumulation underscores the importance of
a daily assessment for need of paralysis and frequent tapering of NMBA dosing once it is safe for the patient.

F. Two Classes of NMBAs According to Mechanism of Action: Depolarizing and Nondepolarizing:


1. Depolarizing NMBAs: Bind and activate acetylcholine receptors, causing persistent depolarization,
which then renders muscle fibers resistant to further cholinergic stimulation. Succinylcholine is the
only available depolarizing NMBA. Because of its quick onset and short duration, it is commonly the
drug of choice for urgent or emergency intubation.
a. Pharmacokinetics: Hydrolyzed by plasma pseudocholinesterase
b. Usual dose: 0.05–1.5 mg/kg intravenously or intramuscularly
c. Onset intravenously: 30–60 seconds; intramuscularly: 2–3 minutes
d. Duration intravenously: 4–6 minutes; intramuscularly: 10–30 minutes
e. Should not be used in patients with a history of malignant hyperthermia, hyperkalemia, stroke,
paralysis, glaucoma, penetrating eye injuries, or spinal, crush, or burn injuries after 24 hours
f. Adverse effects: Arrhythmias, bradycardia or tachycardia, hyperkalemia, rhabdomyolysis
2. Nondepolarizing NMBAs: Nicotinic receptor antagonists (competitive), blocking the action of
acetylcholine at the neuromuscular junction. Divided into aminosteroid group (pancuronium,
vecuronium, and rocuronium) and benzyl isoquinolinium group (atracurium, cisatracurium,
doxacurium, and mivacurium).
a. Pancuronium: Long-acting aminosteroid; intermittent or scheduled bolus may be preferred to
continuous infusion because of accumulation and variable clearance. Older NMBA, not used much
in the United States.
i. Pharmacokinetics: Hepatically metabolized (30%–50%) and renally cleared as unchanged
drug (50%–70%). Accumulation and prolonged duration of paralysis will occur with varying
degrees of hepatic and/or renal dysfunction. Duration about 60–120 minutes.
ii. Adverse effects: Vagolytic activity, sympathetic stimulation, bradycardia, prolonged effect

ACCP Updates in Therapeutics® 2022: Critical Care Pharmacy Preparatory Review and Recertification Course

470
Management of PADIS and NMB in Adult Intensive Care Unit Patients

b. Vecuronium: Intermediate-acting aminosteroid; often used as a continuous infusion


i. Pharmacokinetics: Hepatically metabolized (30%–50%); cleared renally (20%–30%), with
fecal excretion. Has an active metabolite, around half the activity of parent compound.
Duration 30 minutes after bolus intubation dose.
ii. Adverse effects: Vagolytic activity at higher doses, prolonged weakness
c. Rocuronium: An intermediate-acting aminosteroid; considered a suitable alternative to
succinylcholine for rapid sequence intubation (dose: 0.6–1.2 mg/kg) because of its rapid onset of
action (60–90 seconds). Duration 30–40 minutes.
i. Pharmacokinetics: Primarily hepatically metabolized, minimal renal excretion. No active
metabolite. Prolonged effects have been observed in patients with hepatic or renal failure.
ii. Adverse effects: Vagolytic activity at higher doses, bradycardia
d. Atracurium: Intermediate-acting benzyl isoquinolinium; a mixture of 10 stereoisomers (contains
15% cisatracurium)
i. Pharmacokinetics: Undergoes Hofmann elimination to form the toxic metabolite laudanosine
at high levels. Laudanosine is a cerebral stimulant that may precipitate seizure activity,
clearance dependent on liver and kidney function. Duration of atracurium 20–40 minutes.
ii. Adverse effects: Histamine release may cause cardiovascular adverse effects and bronchospasm;
laudanosine accumulation may cause seizure activity.
e. Cisatracurium: An intermediate-acting benzyl isoquinolinium. Differences compared with
atracurium: It is only one isomer, has a slower onset at normal bolus doses, no histamine release.
i. Pharmacokinetics: Undergoes Hofmann elimination, forms laudanosine but at much lower
levels than atracurium. Renal and hepatic dysfunction do not alter cisatracurium clearance.
Duration 30–60 minutes.
ii. Adverse effects: Prolonged weakness with continued use

G. Drug Interactions with NMBAs: Certain medications may decrease the activity of NMBAs, whereas others
can enhance or prolong the paralytic action.
1. Drugs decreasing the activity of NMBAs:
a. Calcium: Antagonizes the effect of magnesium on neuromuscular blockade
b. Carbamazepine: Competitor of acetylcholine receptor
c. Phenytoin: Depressed postsynaptic response to acetylcholine
d. Ranitidine: Unknown mechanism
e. Theophylline: Unknown mechanism
2. Drugs prolonging the activity of NMBAs:
a. Antibiotics: Aminoglycosides, clindamycin, tetracyclines, vancomycin. Decreases prejunctional
acetylcholine release with decreased postjunctional acetylcholine receptor sensitivity; blocks
acetylcholine receptor.
b. Cardiac medications: β-Blockers, calcium channel blockers, procainamide, quinidine, and
furosemide. Decreases prejunctional acetylcholine release.
c. Immunosuppressants: Steroids (decrease end plate sensitivity to acetylcholine), cyclosporine
(inhibits metabolism of certain NMBAs)

H. Choice of NMBA: Intermediate- to longer-acting agents such as vecuronium may be tried in bolus fashion
initially before continuous infusion, particularly if organ dysfunction is present. The duration of paralysis
for NMBAs cleared by Hofmann degradation may be more reliable when used as a continuous infusion
because their clearance is not dependent on renal or hepatic function.

ACCP Updates in Therapeutics® 2022: Critical Care Pharmacy Preparatory Review and Recertification Course

471
Management of PADIS and NMB in Adult Intensive Care Unit Patients

I. Train-of-Four (TOF) Monitoring and Dose Titration


1. The SCCM NMBA guidelines make no recommendation concerning the use of electroencephalogram-
derived parameters (e.g., Bispectral Index [BIS], E-entropy, Cerebral State Index, and Patient State
Index) as a measure of sedation during continuous NMBA administration (insufficient evidence).
They suggest against the use of peripheral nerve stimulation (PNS) with TOF alone for monitoring the
depth of neuromuscular blockade in patients receiving continuous NMBA infusions (very low quality
evidence). They make no recommendation on the use of PNS to monitor the degree of block in patients
undergoing therapeutic hypothermia (insufficient evidence). Typically, the goal of using an NMBA is
to improve patient-ventilator synchrony and increase oxygenation. This may be achieved with varying
degrees of paralysis and may not necessitate 100% block.
2. Monitoring the depth of neuromuscular blockade by peripheral nerve stimulators (e.g., TOF), together
with measured oxygenation parameters, helps find the “lowest effective paralytic dose” and allows
quicker recovery of spontaneous neuromuscular transmission once the NMBA is discontinued. Some
clinicians do not believe that TOF monitoring is necessary and believe that using the clinical values
alone is sufficient to determine NMBA dosing.
3. TOF delivers four supramaximal electrical impulses every 0.5 seconds to the ulnar, facial, or posterior
tibial nerve. Response to the impulse is then measured by muscle twitches visualized from the associated
innervated muscles (thumb or eye). Goals of paralysis can usually be reached with 2 or 3 of 4 twitches;
0 of 4 twitches indicates complete neuromuscular blockade, usually necessitating a decrease in NMBA
dose. Oxygenation goals may be reached even with 4 of 4 twitches, indicating that the NMBA dose is
effective and an increase is not warranted.
4. A baseline electrical current should be established before initiating an NMBA to determine how
much electrical current is needed to produce a twitch. Usually 10–20 mA (amperage) is sufficient. The
conduction of the electrical impulse may be dampened because of peripheral edema, loss of electrode
adhesion, incorrect electrode placement, and hypothermia, which can lead to inaccurate readings.
These factors should be reassessed with each use of the TOF.

J. Complications of NMBAs
1. Prolonged weakness: Several case reports associate the use of NMBAs and prolonged weakness,
which could include myopathy, polyneuropathy, or neuromyopathy. Other risk factors may include
concomitant use of corticosteroids, persistent hyperglycemia, and type of NMBA used. However,
data are inconsistent and not controlled, and further studies are needed to clarify specific risk factors
for prolonged weakness associated with NMBAs. Following a trend in creatine kinase concentration
every 48–72 hours may help assess the presence of myopathy secondary to paralysis and prolonged
immobilization. A creatine kinase concentration should not be solely relied on for the presence of
myopathy, and daily determination of the need for the NMBA should still be considered, even with a
normal creatine kinase.
2. Corneal abrasions: Paralysis eliminates the ability of the eyes to close and blink, increasing the risk of
corneal ulcerations and infection. Prophylactic eye protection must be used in all patients on NMBAs
(e.g., lubricating eye ointments or eye covers).
3. Thrombosis: Caused partly by immobility, patients receiving an NMBA may be up to 8 times more
likely to have a DVT than those not on an NMBA. Prophylaxis for a DVT must be provided for all
patients on an NMBA.
4. Awareness: Recent case reports document patient awareness during paralysis in the ICU. These patients
report weird dreams, fear, resistance of restraints, thoughts of life and death, and pain. Patients must be
deeply sedated before initiating an NMBA (Ann Emerg Med 2021;77:532-44).
5. Resistance to paralysis and/or potentiation: Certain disease states may produce an up-regulation in
acetylcholine skeletal muscle receptors, leading to higher-than-normal doses of the NMBA (e.g., muscle
trauma, muscle atrophy, burns). Acid-base disorders, electrolyte imbalances, and adrenal insufficiency
may also cause unpredictable alterations in dosing requirements.

ACCP Updates in Therapeutics® 2022: Critical Care Pharmacy Preparatory Review and Recertification Course

472
Management of PADIS and NMB in Adult Intensive Care Unit Patients

6. Anaphylaxis: Allergic reactions can occur after the first dose of an NMBA because the ammonium ions
in NMBAs are commonly found in the household environment and in household products. If an allergic
reaction is suspected, skin prick testing for the NMBA against a control can be done within 6 weeks of
the reaction.

Patient Cases

7. A 55-year-old man intubated for severe ARDS (Pao2/Fio2 ratio less than 100 mm Hg) is receiving fentanyl
200 mcg/hour, midazolam 8 mg/hour, and propofol 40 mcg/kg/minute. He is deeply sedated but remains
hypoxic and dyssynchronous with the ventilator after several changes in mechanical ventilation settings.
Which is the most appropriate consideration at this time?
A. Start scheduled lorazepam every 6 hours.
B. Add quetiapine 50 mg every 8 hours.
C. Change propofol to dexmedetomidine.
D. Start a cisatracurium infusion.

8. A 70-year-old woman who is day 2 in the ICU is receiving a neuromuscular blocking agent (NMBA) and
is sedated for severe ARDS. The TOF over 24 hours is 2 of 4 twitches at an amplitude of 10 mA. Arterial
blood gas is pH 7.38, Pco2 40 mm Hg, Po2 91 mm Hg, and bicarbonate 24 mEq/L on 50% inspired oxygen
and 10 cm H2O PEEP; the patient is synchronous with the ventilator, and other clinical markers are stable.
Which changes in management would be best to recommend?
A. Decrease stimulator amplitude to decrease pain from excessive electrical current.
B. Increase stimulator amplitude to test for more frequent twitches.
C. Decrease the NMBA dose because the patient is clinically stable.
D. Increase the NMBA dose until the TOF induces fewer twitches.

Acknowledgment: ACCP wishes to acknowledge Dr. Kimberly Varney Gill for her contributions to this
chapter.

ACCP Updates in Therapeutics® 2022: Critical Care Pharmacy Preparatory Review and Recertification Course

473
Management of PADIS and NMB in Adult Intensive Care Unit Patients

REFERENCES

Pain 9. Mather LE. Clinical pharmacokinetics of fentanyl


1. Barr J, Fraser GL, Puntillo K, et al. Clinical and its newer derivatives. Clin Pharmacokinet
practice guidelines for the management of pain, 1983;8:422-46.
agitation, and delirium in adult patients in the 10. McClave SA, Martindale RG, Vanek VW, et al.
intensive care unit. Crit Care Med 2013;41:263- JPEN J Parenter Enteral Nutr 2009;33:277-316.
306. The most updated guidelines from SCCM, 11. Mostafa SM, Bhandara S, Ritchie G, et al.
the American College of Critical Care Medicine, Constipation and its implications in the critically
and the American Society of Health-System ill patient. Br J Anaesth 2003;91:815-9.
Pharmacists. This was a multidisciplinary effort 12. Nguyen T, Frenette AJ, Johanson C, et al. Impaired
coordinating all three areas of management (pain, gastrointestinal transit and its associated mor-
agitation, and delirium). bidity in the intensive care unit. J Crit Care
2. Breen D, Karabinis A, Malbrain M, et al. Decreased 2013;28:537e11-7.
duration of mechanical ventilation when compar- 13. Payen J, Bru O, Bosson J, et al. Assessing pain
ing analgesia-based sedation using remifentanil in critically ill sedated patients by using a behav-
with standard hypnotic-based sedation for up to 10 ioral pain scale. Crit Care Med 2001;29:2258-63.
days in intensive care unit patients: a randomized Validation study for the BPS.
trial. Crit Care 2005;9:R200-10. 14. Puntillo K, Ley SJ. Appropriately timed analgesics
3. Czarnecki M, Turner H, Collins PM, et al. control pain due to chest tube removal. Am J Crit
Procedural pain management: a position state- Care 2004;13:292-301.
ment with clinical practice recommendations. Pain 15. Puntillo KA, Max A, Timsit J, et al. Determinants
Manag Nurs 2011;12:95-111. of procedural pain intensity in the intensive care
4. Devlin JW, Skrobik Y, Gélinas C, et al. Clinical unit. The Europain® study. Am J Respir Crit Care
practice guidelines for the prevention and man- Med 2014;189:39-47.
agement of pain, agitation/sedation, delirium, 16. Puntillo KA, Pasero C, Li D, et al. Evaluation of
immobility, and sleep disruption in adult patients pain in ICU patients. Chest 2009;135:1069-74.
in the ICU. Crit Care Med 2018;46:e825-73. Concise review on how to evaluate pain in the ICU
5. Devabhakthuni S, Armahizer MJ, Dasta JF, et patient who is nonverbal. Brief discussion on the
al. Analgosedation: a paradigm shift in intensive validated pain scales for the adult ICU patient.
care unit sedation practice. Ann Pharmacother 17. Puntillo KA, Wild LR, Morris AB, et al. Practices
2012;46:530-40. Review of literature using an and predictors of analgesic interventions for adults
analgesic-based method of sedation. undergoing painful procedures. Am J Crit Care
6. Erstad BL, Puntillo K, Gilbert H, et al. Pain 2002;11:415-29.
management principles in the critically ill. Chest 18. Rotondi AJ, Chelluri L, Sirio C, et al. Patients’ rec-
2009;135:1075-86. Comprehensive review of opi- ollections of stressful experiences while receiving
ate use in the ICU; discusses a spectrum of issues, prolonged mechanical ventilation in an inten-
including acute versus chronic pain, adverse sive care unit. Crit Care Med 2002;30:746-52.
effects, non-opiate medications, nonpharmaco- Questionnaire study describing patients’ recalled
logic methods of treating pain, and the withdrawal memories after being in the ICU and on mechani-
effects of opiates in the ICU. cal ventilation for 48 hours or more.
7. Gelinas C, Fillion L, Puntillo K, et al. Validation 19. Scholz J, Steinfath M, Schulz M. Clinical pharma-
of the critical-care pain observation tool in adult cokinetics of alfentanil, fentanyl, and sufentanil.
patients. Am J Crit Care 2006;15:420-7. An update. Clin Pharmacokinet 1996;31:275-92.
8. Kanji S, MacPhee H, Singh A, et al. Validation of 20. Strøm T, Martinussen T, Toft P. A protocol of no
the critical care pain observation tool in critically sedation in critically ill patients receiving mechan-
ill patients with delirium: a prospective cohort ical ventilation: a randomized controlled trial.
study. Crit Care Med 2016;44:943-7. Lancet 2010;375:475-80.

ACCP Updates in Therapeutics® 2022: Critical Care Pharmacy Preparatory Review and Recertification Course

474
Management of PADIS and NMB in Adult Intensive Care Unit Patients

21. Taylor D. Iatrogenic drug dependence—a problem ventilated critically ill patients. Iran J Pharm Res
in intensive care? Intensive Crit Care Nurs 2013;12:483-8.
1999;15:95-100. A good discussion on the potential 5. Bienvenu OJ, Gellar J, Althouse BM, et al. Post-
for opiate and benzodiazepine dependence in traumatic stress disorder symptoms after acute
patients who require long-term therapy of these lung injury: a 2-year prospective longitudinal
agents in the ICU setting. Pharmacists can assist study. Psychol Med 2013;43:2657-71.
the ICU team in the appropriate dosing of opiates 6. Cammarono WB, Pittet JF, Weitz S, et al. Acute
throughout the ICU stay and monitoring for withdrawal syndrome related to the administra-
symptoms of withdrawal while tapering off. tion of analgesic and sedative medications in
adult intensive care unit patients. Crit Care Med
Agitation 1998;26:676-84.
1. Balas MC, Vaselevskis EE, Olsen KM, et al. 7. Carson SC, Kress JP, Rodgers J, et al. A random-
Effectiveness and safety of the awakening and ized trial of intermittent lorazepam versus propofol
breathing coordination, delirium monitoring/man- with daily interruption in mechanically ventilated
agement, and early exercise/mobility bundle. Crit patients. Crit Care Med 2006;34:1326-32. Nice
Care Med 2014;42:1024-36. Known as the ABCDE study from two large university hospitals, medi-
trial, this was the second study to coordinate the cal ICU population (n=546). Primary outcome was
efforts of SATs and SBTs. The investigators added median days on mechanical ventilation, which
delirium monitoring and management as well as was lower in the propofol daily interruption group
an early mobility protocol to this study; medical or (propofol = 5.8 days; intermittent lorazepam = 8.4
surgical ICU population, n=296. Primary outcome days; p=0.04). Hospital mortality was no different
was median time breathing without mechani- between the two groups.
cal ventilator assistance during the 28-day study 8. Devlin JW, Mallow-Corbett S, Riker RR. Adverse
period. This study used safety screens for the drug events associated with the use of analgesics,
SAT, SBT, and mobility protocols. The interven- sedatives, and antipsychotics in the intensive care
tion group had 3 more days of breathing without unit. Crit Care Med 2010;38:S231-43.
mechanical ventilator assistance than did the stan- 9. de Wit M, Gennings C, Jenvey W, et al.
dard care group (median 24 vs. 21 days; p=0.04). Randomized trial comparing daily interruption of
The intervention group was almost half as likely sedation and nursing-implemented sedation algo-
to develop delirium (OR 0.55; p=0.03) and had rithm in medical intensive care unit patients. Crit
increased odds of getting out of bed at least once Care 2008;12:1-9. Pivotal study terminated early
during the ICU stay (p=0.003) compared with the because of complications in patients randomized
standard care group. to the daily interruption of sedation group. The
2. Barnes BJ, Gerst C, Smith JR, et al. Osmol gap authors concluded that daily interruption is not
as a surrogate marker for serum propylene glycol appropriate in all medical ICU patients. A patient
concentrations in patients receiving lorazepam for safety screen is now recommended for all patients
sedation. Pharmacotherapy 2006;26:23-33. as part of any daily interruption or SAT.
3. Barr J, Fraser GL, Puntillo K, et al. Clinical 10. Fodale V, La Monaca E. Propofol infusion syn-
practice guidelines for the management of pain, drome: an overview of a perplexing disease. Drug
agitation, and delirium in adult patients in the Saf 2008;31:293-303.
intensive care unit. Crit Care Med 2013;41:263- 11. Fraser GL, Devlin J, Worby CP, et al.
306. The most updated guidelines from SCCM, Benzodiazepine versus nonbenzodiazepine-based
the American College of Critical Care Medicine, sedation for mechanically ventilated critically ill
and the American Society of Health-System adults: a systematic review and meta-analysis of
Pharmacists. randomized trials. Crit Care Med 2013;41:S30-
4. Beigmohammadi MT, Hanifeh M, Rouini MR, et 8. Good review of trials comparing the major
al. Pharmacokinetic alterations in midazolam infu- outcomes of drugs used for sedation: days on
sion versus bolus administration in mechanically mechanical ventilation, days in the ICU, incidence
of delirium, and mortality.

ACCP Updates in Therapeutics® 2022: Critical Care Pharmacy Preparatory Review and Recertification Course

475
Management of PADIS and NMB in Adult Intensive Care Unit Patients

12. Galvin M, Jago-Byrne MC, Fitzsimons M, et al. dexmedetomidine. Only fentanyl boluses were
Int J Clin Pharm 2013;35:14-211. Clinical pharma- used for pain management. Primary outcomes
cist’s contribution to medication reconciliation on were time at target sedation level and duration of
admission to a hospital in Ireland. mechanical ventilation. The time at target seda-
13. Girard TD, Kress JP, Fuchs BD, et al. Efficacy and tion level was the same between groups from both
safety of a paired sedation and ventilator wean- studies; the only difference in median time on
ing protocol for mechanically ventilated patients mechanical ventilation was between midazolam
in intensive care (Awakening and Breathing and dexmedetomidine (164 hours vs. 123 hours;
Controlled trial): a randomized, controlled trial. p=0.03).
Lancet 2008;371:126-34. Known as the Wake Up 19. Kane-Gill SL, Jacobi J, Rothschild JM. Adverse
and Breathe, or ABC, trial, this was the first study drug events in the intensive care units: risk factors,
to coordinate SATs with SBTs. The study was from impact, and the role of team care. Crit Care Med
four tertiary care hospitals (n=336), and primary 2010;38:S83-89.
outcome was time breathing without assistance 20. Kawazoe Y, Miyamoto K, Morimoto T, et al.
during a 28-day study period. This study used a Effect of dexmedetomidine on mortality and ven-
safety screen for both the SAT and the SBT pro- tilator-free days in patients requiring mechanical
tocols. The intervention group spent more time ventilation with sepsis: a randomized clinical trial.
breathing without assistance than did the standard JAMA 2017;317:1321-8.
care group (median 14.7 vs. 11.6 days; p=0.02). 21. Kukoyi AT, Coker SA, Lewis LD. Two cases of
14. Herridge MS, Tansey CM, Matte A, et al. Functional acute dexmedetomidine withdrawal syndrome fol-
disability 5 years after acute respiratory distress lowing prolonged infusion in the intensive care
syndrome. N Engl J Med 2011;364:1294-304. unit: report of cases and review of the literature.
15. Honiden S, Siegel M. Analytic reviews: managing Hum Exp Toxicol 2013;32:106-110.
the agitated patient in the ICU: sedation, analge- 22. Mehta S, Burry L, Cook D, et al. Daily seda-
sia, and neuromuscular blockade. J Intensive Care tion interruption in mechanically ventilated
Med 2010;25:187-204. Succinct review of the pri- critically ill patients cared for with a sedation pro-
mary issues in sedation management. This was tocol. JAMA 2012;308:1985-92. This study (for the
published before the 2013 PAD guidelines, but it SLEAP study investigators and Canadian critical
contains a very good discussion. care trials group) compared daily sedation inter-
16. Horinek EL, Kiser TH, Fish DN, et al. Propylene ruption with a standard sedation protocol targeting
glycol accumulation in critically ill patients receiv- lighter sedation scores (e.g., RASS 0 to -3; SAS 3
ing continuous infusion lorazepam infusions. Ann or 4). The study found no difference between the
Pharmacother 2009;43:1964-71. two groups in their end points of time on mechani-
17. Hughes CG, Girard TD, Pandharipande P. Daily cal ventilation or duration of ICU stay.
sedation interruption versus targeted light seda- 23. Pandharipande PP, Pun BT, Herr DL, et al. Effects
tion strategies in ICU patients. Crit Care Med of sedation with dexmedetomidine versus loraz-
2013;41:S39-45. This review finds that it is still epam on acute brain dysfunction in mechanically
unclear whether one sedation strategy is bet- ventilated patients: the MENDS randomized con-
ter than the other, that using the two strategies trolled trial. JAMA 2007;298:2644-53.
together may offer more benefit than either alone, 24. Pandharipande PP, Shintani A, Peterson J, et al.
and that coordinating either strategy with SBT Lorazepam is an independent risk factor for tran-
should be considered. sitioning to delirium in intensive care unit patients.
18. Jakob SM, Ruokonen E, Grounds RM, et al. Anesthesiology 2006;104:21-6. One of the first
Dexmedetomidine versus midazolam or propofol studies specifically addressing the use of loraz-
for sedation during prolonged mechanical ven- epam as a risk of developing delirium.
tilation. JAMA 2012;307:1151-60. This article 25. Riker R, Shahabi Y, Bokesch P, et al.
reported on two trials; the “MIDEX” trial com- Dexmedetomidine versus midazolam for seda-
pared midazolam with dexmedetomidine, and tion of critically ill patients. A randomized trial:
the “PRODEX” trial compared propofol with SEDCOM. JAMA 2009;301:489-99. This study

ACCP Updates in Therapeutics® 2022: Critical Care Pharmacy Preparatory Review and Recertification Course

476
Management of PADIS and NMB in Adult Intensive Care Unit Patients

investigated differences in time at targeted 2. Brummel NE, Jackson JC, Pandharipande PP, et al.
sedation level between dexmedetomidine and Delirium in the intensive care unit and subsequent
midazolam. The investigators found no difference long-term disability among survivors of mechani-
in efficacy of sedation; the prevalence of delirium cal ventilation. Crit Care Med 2014;42:369-77.
was lower in the dexmedetomidine group, and the 3. Brummel NE, Vasilevskis EE, Han JH, et al.
median time to extubation was shorter in the dex- Implementing delirium screening in the ICU:
medetomidine group. secrets to success. Crit Care Med 2013;41:2196-208.
26. Seymour CW, Pandharipande PP, Koestner T, et 4. Devlin JW, Roberts RJ, Fong JJ, et al. Efficacy and
al. Diurnal sedative changes during intensive care: safety of quetiapine in critically ill patients with
impact on liberation from mechanical ventilation delirium: a prospective, multicenter, randomized,
and delirium. Crit Care Med 2012 40:2788-96. double-blind, placebo-controlled pilot study. Crit
27. Shafer A. Complications of sedation with Care Med 2010;38:419-27.
midazolam in the intensive care unit and a com- 5. Devlin JW, Zaal I, Slooter A. Clarifying the confu-
parison with other sedative regimens. Crit Care sion surrounding drug-associated delirium in the
Med 1998;26:947-56. ICU. Crit Care Med 2014;42:1565-66. Discusses
28. Shehabi Y, Bellomo R, Reade MC et al. Early issues with current delirium research and latest
intensive care sedation predicts long-term mortal- findings from the corticosteroid-associated delir-
ity in ventilated critically ill patients. Am J Respir ium study by Schreiber MP, et al. (Crit Care Med
Crit Care Med. 2012;15:724-31. 2014;42:1480-6).
29. Shehabi Y, Howe BD, Bellomo R, et al. Early 6. Girard TD, Exline MC, Carson SS, et al.
sedation with dexmedetomidine in critically ill Haloperidol and ziprasidone for treatment of
patients. N Engl J Med 2019;380:2506-17. delirium in critical illness. N Engl J Med 2018;
30. Swart EL, de Jongh J, Zuideveld KP, et al. 379:2506-16.
Population pharmacokinetics of lorazepam and 7. Girard TM, Pandharipande P, Carson SS, et al.
midazolam and their metabolites in intensive care Feasibility, efficiency, and safety of antipsychot-
patients on continuous venovenous hemofiltration. ics for intensive care unit delirium: the MIND
Am J Kidney Dis 2005;45:360-71. randomized, placebo-controlled trial. Crit Care
31. Uijtendaal EV, van Harssel LL, Hugenholtz GW, Med 2010;38:428-37. Pilot trial showing that treat-
et al. Analysis of potential drug-drug interac- ment with antipsychotics in a 21-day study did not
tions in medical intensive care unit patients. improve the number of days alive without delirium
Pharmacotherapy 2014;34:213-9. or coma.
32. Verbeek RK. Pharmacokinetics and dosage adjust- 8. Girard TM, Pandharipande P, Ely W. Delirium in
ment in patients with hepatic dysfunction. Eur J the intensive care unit. Crit Care 2008;12(suppl
Clin Pharmacol 2008;64:1147-61. 3):1-9. Good review of pathophysiology and risk
33. Vonbach P, Dubied A, Krähenbühl S, et al. factors for delirium. Also has a discussion on how
Prevalence of drug-drug interactions at hospital to monitor, prevent, and treat delirium.
entry and during hospital stay of patients in inter- 9. Jackson P, Khan A. Delirium in critically ill
nal medicine. Eur J Intern Med 2008;19:413-20. patients. Crit Care Clin 2015;31:589-603.
10. Jeon K, Jeong BH, Ko MG, et al. Impact of delir-
Delirium ium on weaning from mechanical ventilation in
1. Adler J, Malone D. Early mobilization in the inten- medical patients. Respirology 2016;21:313-20.
sive care unit: a systematic review. Cardiopulm 11. Kamdar BB, King LM, Collop NA, et al. The effect
Phys Ther J 2012;23:5-13. Good review of cur- of a quality improvement intervention on perceived
rent studies that investigated the effects of early sleep quality and cognition in a medical ICU. Crit
mobilization of patients in the ICU on functional Care Med 2013;41:800-9. This study investigated
outcomes. More research is ongoing in this area, the effects of a sleep protocol before and after
particularly as early mobility pertains to cognitive implementation. Study investigators found that
function and delirium. implementation of a sleep protocol is feasible in an

ACCP Updates in Therapeutics® 2022: Critical Care Pharmacy Preparatory Review and Recertification Course

477
Management of PADIS and NMB in Adult Intensive Care Unit Patients

adult ICU, decreases perceived level of noise, and 21. Schreiber MP, Colantuoni E, Bienvenu O, et
may decrease the incidence of delirium. Very good al. Corticosteroids and transition to delirium in
study that moves forward the discussion regarding patients with acute lung injury. Crit Care Med
the importance of adequate sleep in the ICU and 2014;42:1480-6.
for future research in this area. 22. Skrobik Y, Duprey MS, Hill NS, et al. Low-dose
12. Kees J, Kalisvaart MD, Jos FM, et al. Haloperidol nocturnal dexmedetomidine prevents ICU delir-
prophylaxis for elderly hip surgery patients at risk ium: a randomized, placebo-controlled trial. Am J
for delirium: a randomized placebo controlled Respir Crit Care Med 2018;197:1147-56.
study. J Am Geriatr Soc 2005;53:1658-66. 23. Sosnowski K, Lin F, Mitchell ML. Early reha-
13. Kram BR, Kram SJ, Brooks KR. Implications of bilitation in the intensive care unit: an integrated
atypical antipsychotic prescribing in the intensive literature review. Aust Crit Care 2015;28:216-25.
care unit. J Crit Care 2015;30:814-8. 24. Van den Boogaard M, Slooter AJC, Bruggemann
14. Lichtenbelt BJ, Olofson E, Van Kleef JW, et al. FJM, et al. Effect of haloperidol on survival among
Propofol reduces the distribution and clearance of critically ill adults with a high risk of delirium:
midazolam. Anesth Analg 2010;110:1597-606. the REDUCE randomized clinical trial. JAMA
15. Marshall J, Herzig SJ, Howell MD, et al. 2018;319:680-90.
Antipsychotic utilization in the intensive care 25. Vanderbilt University Medical Center. ABCDEFs
unit and in transitions of care. J Crit Care of Prevention and Safety. Available at www.icu-
2016;33:119-24. delirium.org. Accessed June 12, 2016. Vanderbilt
16. Mehta S, Cook D, Devlin JW, et al. Prevalence, risk website containing extensive educational mate-
factors, and outcomes of delirium in mechanically rials about delirium in the ICU for medical
ventilated adults. Crit Care Med 2015:43:557-66. professionals, patients, and families. This website
17. Pandharipande PP, Girard TD, Jackson JC, et al. uses several modalities for education, including
Long-term cognitive impairment after critical ill- references, training manuals, Vanderbilt’s proto-
ness. N Engl J Med 2013;369:1306-16. This trial is cols, and videos, to assist medical professionals in
from the BRAIN-ICU investigators group, which assessing and managing delirium in their ICU.
studied the risk factors for and incidence of cog- 26. West S, Kenedi C. Strategies to prevent the neuro-
nitive impairment 3 and 12 months from ICU psychiatric side effects of corticosteroids: a case
discharge from a medical or surgical ICU. They report and review of the literature. Curr Opin
found that 74% of patients developed delirium Organ Transplant 2014;19:201-8.
during their ICU stay, and a longer duration of 27. Wilcox ME, Brummel NE, Archer K, et al.
delirium was associated with worse global cog- Cognitive dysfunction on ICU patients: risk fac-
nition and executive function scores at 3 and 12 tors, predictors, and rehabilitation interventions.
months after discharge. Use of sedative and analge- Crit Care Med 2013;41:S81-S98.
sic medications was not associated with cognitive 28. Wolters AE, Veldhuijzen DS, Peelen LM, et
impairment in this study at 3 and 12 months. al. Systemic corticosteroids and transition to
18. Patel SB, Poston JT, Pohlman A, et al. Rapidly delirium in critically ill patients. Crit Care Med
reversible, sedation-related delirium versus per- 2015;43:e585-8.
sistent delirium in the intensive care unit. Am J 29. Wolters AW, Zaal IJ, Vedhuiizen DS, et al.
Respir Crit Care Med 2014;189:658-65. Anticholinergic medication use and transition to
19. Pisani MA, Friese RS, Gehlbach BK, et al. Sleep delirium in critically ill patients: a prospective
in the intensive care unit. Am J Respir Crit Care cohort study. Crit Care Med 2015;42:1846-52.
Med 2015;191:731-8. 30. Zaal IJ, Devlin JW, Haelbag, M, et al.
20. Pun BT, Balas MC, Barnes-Daly MA, et al. Caring Benzodiazepine-associated delirium in critically
for critically ill patients with the ABCDEF bundle: ill adults. Intensive Care Med 2015;41:2120-7.
results of the ICU liberation collaborative in over 31. Zaal IJ, Devlin JW, Peelen LM, et al. A systematic
15,000 adults. Crit Care Med 2018 Oct 18. [Epub review of risk factors for delirium in the ICU. Crit
ahead of print] Care Med 2015;43:40-7. Most updated review of
risk factors for delirium in adult ICU patients.

ACCP Updates in Therapeutics® 2022: Critical Care Pharmacy Preparatory Review and Recertification Course

478
Management of PADIS and NMB in Adult Intensive Care Unit Patients

Post Intensive Care Syndrome pharmacology, TOF monitoring, and complica-


1. Duning T, van den Heuvel I, Dickmann A, et tions of prolonged paralysis.
al. Hypoglycemia aggravates critical illness- 2. Hraiech S, Dizier S, Papazian L. The use of para-
induced neurocognitive dysfunction. Diabetes lytics in patients with acute respiratory distress
Care 2010;33:639-44 syndrome. Clin Chest Med 2014;35:753-63.
2. Hermans G, Wilmer A, Meersseman W, et al. 3. Huang DT, Angus DC, Moss M, et al. Design and
Impact of intensive insulin therapy on neuromus- rationale for the reevaluation of systemic early
cular complications and ventilator dependency in neuromuscular blockade trial for acute respira-
the medical intensive care unit. Am J Respir Crit tory distress syndrome. Ann Am Thorac Soc
Care Med 2007;175:480-9 2017;14:124-33.
3. Herridge MS, Cheung AM, Tansey CM, et al. One- 4. Lagneau F, D’honneur G, Plaud B, et al. A com-
year outcomes in survivors of the acute respiratory parison of two depths of prolonged neuromuscular
distress syndrome. N Engl J Med 2003;348:683-93 blockade induced by cisatracurium in mechani-
4. Herridge MS, Tansey CM, Matte A, et al. Functional cally ventilated critically ill patients. Intensive
disability 5 years after acute respiratory distress Care Med 2002;28:1735-41.
syndrome. N Engl J Med 2011;364:1293-304 5. Murray MJ, DeBlock H, Erstad B, et al. Clinical
5. Hopkins RO, Suchyta MR, Snow GL, et al. Blood practice guidelines of the sustained neuromuscular
glucose dysregulation and cognitive outcome in blockade in the adult critically ill patient. Crit Care
ARDS survivors. Brain Inj 2010;24:1478-84 Med 2016;44:2079-103.
6. Jackson JC, Pandharipande PP, Girard TD, et al. 6. National Heart, Lung, and Blood Institute PETAL
Depression, post-traumatic stress disorder, and Clinical Trials Network. Early neuromuscular
functional disability in survivors of critical illness blockade in the acute respiratory distress syn-
in the BRAIN-ICU study: a longitudinal cohort drome. N Engl J Med 2019;380:1997-2008.
study. Lancet Respir Med. 2014 May;2(5):369-79. 7. Papazian L, Forel JM, Gacouin A, et al.
7. Sevin CM, Bloom SL, Jackson JC, et al. Neuromuscular blockers in early acute respiratory
Comprehensive care of ICU survivors: develop- distress syndrome. N Engl J Med 2010;363:1107-
ment and implementation of an ICU recovery 16. Large multicenter, double-blind trial
center. J Crit Care 2018;46:141-8. completed in France (n=340); studied the effects
8. Stollings JL, Bloom SL, Huggins SL, et al. of cisatracurium in patients with early and severe
Medication management to ameliorate post ARDS. Primary end point was 90-day mortality.
intensive care syndrome. AACN Adv Crit Care Investigators found that a fixed dose of cisatracu-
2016;27:133-40. rium for 48 hours decreased mortality in patients
9. Stollings JL, Bloom SL, Wang L, et al. Critical with severe ARDS (Pao2/Fio2 less than 120 mm
care pharmacists and medication management Hg).
in an ICU recovery center. Ann Pharmacother 8. Tortorici MA, Kochanek PM, Poloyac SM. Effects
2018;52:713-23. of hypothermia on drug disposition, metabolism,
10. Stollings JL, Caylor MM. Post intensive care syn- and response: a focus of hypothermia-mediated
drome and the role of a follow-up clinic. Am J alterations on the cytochrome P450 enzyme sys-
Health Syst Pharm 2015;72:1315-23. tem. Crit Care Med 2007;35:2196-204.
11. Van den BG, Schoonheydt K, Becx P, et al. Insulin
therapy protects the central and peripheral ner-
vous system of intensive care patients. Neurology
2005;64:1348-53

Neuromuscular Blocking Agents


1. Greenburg SB, Vender J. The use of neuromuscu-
lar blocking agents in the ICU: where are we now?
Crit Care Med 2013;41:1332-44. Very good review
and discussion of NMDAs, including indications,

ACCP Updates in Therapeutics® 2022: Critical Care Pharmacy Preparatory Review and Recertification Course

479
Management of PADIS and NMB in Adult Intensive Care Unit Patients

ANSWERS AND EXPLANATIONS TO PATIENT CASES

1. Answer: D days of admission. The onset of withdrawal symptoms


This patient has a clear indication for intravenous pain will vary depending on the half-life of each medication.
medication from his recent trauma and multiple frac- Symptoms may include agitation, anxiety, psychosis,
tures. An “as-needed” opiate would likely not keep up insomnia, hypertension, and tachycardia and can occur
with his pain control needs (Answers B and C are incor- with medications such as opiates, GABA receptor ago-
rect). His age and history of hypertension place him at nists, antiepileptics, antidepressants, and antipsychotics.
risk of delirium; therefore, a benzodiazepine is not the A pharmacist can assist the medical team by obtaining
best initial choice for this patient. A propofol infusion a thorough medication history and assessment of home
would be the most appropriate sedative (Answer A is medication adherence to help identify drug withdrawal
incorrect; Answer D is correct). symptoms. Reinitiating these medications can be con-
sidered, unless contraindicated because of the clinical
2. Answer: D scenario (e.g., drug-drug interactions, drug-disease state
Chest tube removal is specifically cited in the PADIS interactions). The Agitation and Sedation section of the
guidelines as an indication for both preemptive analge- PADIS guidelines discusses identifying and treating
sia and nonpharmacologic relaxation techniques. This the etiology of agitation before adding other medica-
is given a “strong” recommendation, determining that tions; reinitiating the benzodiazepine and antidepressant
the benefits outweigh the risks of preemptive therapy to treat withdrawal symptoms is the most appropri-
(Answer D is correct). Acetaminophen given just before ate answer (Answer B is correct). Neither fentanyl nor
the procedure will most likely not adequately treat pain dexmedetomidine would treat withdrawal from a ben-
associated with chest tube removal (Answer A is incor- zodiazepine or antidepressant (Answers A, C, and D are
rect). Increasing an opiate infusion several hours before a incorrect).
bedside procedure can expose the patient to substantially
higher amounts of drug than needed and cause delayed 5. Answer: C
awakening times or other significant adverse effects The PADIS guidelines stress using nonpharmaco-
from opiates (Answer C is incorrect). Extensive stud- logic means to manage delirium when it is safe for the
ies of appropriate preemptive analgesia for chest tube patient. Strong evidence for using dexmedetomidine
removal have not been completed; however, administer- to treat delirium is still not available and the MIND
ing an opiate appropriately timed before manipulation of USA Study has shown that anti-psychotics are ineffec-
a chest tube is an accepted standard of therapy (Answer tive at treating delirium. This patient’s presentation of
B is incorrect). “alert and calm with intermittent periods of agitation”
is a common scenario, and initial therapy should focus
3. Answer: A on reorienting and getting the patient interactive and
This patient has end-stage liver failure and acute renal mobile (Answers A and D are incorrect; Answer C is
failure; he will therefore not predictably clear mid- correct). Dehydration is a common cause of agitation,
azolam or fentanyl infusions. With a RASS of −4 to and it should be addressed; however, with normal lab-
−5, indicating no meaningful responsiveness to stimuli, oratory values and vital signs, this patient is unlikely
all sedatives should be held if the patient is otherwise dehydrated at this time (Answer B is incorrect).
clinically stable to allow time for clearance of medi-
cations (Answer A is correct). A decrease in sedative 6. Answer: B
dose or changing to a different sedative is not needed In the general population, systemic corticosteroids are
at this time based on the deeply sedated RASS score known to cause many neuropsychiatric events, includ-
and would only further delay awakening time (Answers ing hyperactivity and agitation; in a recent study of adult
B–D are incorrect). ICU patients with acute lung injury, only age and use of
systemic corticosteroids in the preceding 24 hours were
4. Answer: B independently associated with the transition to delir-
Withdrawal from certain home medications may occur ium from a non-delirious state (Answer B is correct).
if these medications are not reinitiated within a few Benzodiazepines have a sedating effect and may calm

ACCP Updates in Therapeutics® 2022: Critical Care Pharmacy Preparatory Review and Recertification Course

480
Management of PADIS and NMB in Adult Intensive Care Unit Patients

an acutely agitated patient; however, they would not be time, and clinicians should try to decrease the NMBA as
recommended in this patient because they could worsen soon as the patient is clinically stable by laboratory val-
her confusion or delirium (Answer A is incorrect). The ues and ventilator management (Answer D is incorrect).
PADIS guidelines state that no evidence supports the A baseline electrical current intensity (amperage) should
use of haloperidol to reduce the duration of delirium be established before the onset of neuromuscular block-
(Answer C is incorrect). Vancomycin is not currently ade and should not be changed during paralysis unless
recognized as a cause of delirium; therefore, changing a new baseline is indicated (Answer A is incorrect). As
to linezolid is not indicated (Answer D is incorrect). the electrical intensity (amperage) is established, an
increase in the amperage is not indicated during infu-
7. Answer: D sion of the NMBA in order to increase the number of
The midazolam dose is high, and the patient is “deeply twitches. A decrease in the dose of NMBA would be
sedated”; therefore, adding another benzodiazepine will indicated if an increase in the number of twitches were
likely not improve this patient’s clinical status. Quetiapine the clinical goal (Answer B is incorrect).
has no indication for general sedation in a critically ill
patient, and it should not be a consideration for sedation
in this patient with severe ARDS. Dexmedetomidine is
considered a weak sedative with no effect on respiratory
drive; therefore, it would likely not improve this patient’s
ventilator dyssynchrony and hypoxia (Answers A–C are
incorrect). At this stage in the patient’s clinical course,
it is reasonable to consider an NMDA. In a 2010 study
of cisatracurium versus placebo for 48 hours in early
ARDS, the cisatracurium group had more days free of
mechanical ventilation and decreased mortality (30%
vs. 44%) at 90 days for the subgroup of patients with
severe ARDS (Pao2/Fio2 ratio less than 120 mm Hg).
The incidence of pneumothorax was lower in the cisa-
tracurium group than in the placebo group (4% vs. 11%).
There were more days free of organ failure (non-lung)
in the cisatracurium group than in the placebo group
(15.8 vs. 12.2 days) in the first 28 days. A recent meta-
analysis concluded that using short-term cisatracurium
in patients with severe ARDS decreases mortality and
time on mechanical ventilation compared with placebo.
The risk of prolonged neuromuscular weakness was not
found in these studies; however, use beyond 48 hours
may increase this risk (Answer D is correct).

8. Answer: C
The TOF method of assessment is primarily used to help
determine the degree of neuromuscular blockade and
should not be used to titrate the dose of the NMBA. The
patient’s clinical status and laboratory values are the
true determinants for dose adjustment of the NMBA.
Patients may be at their clinical goal with a TOF of 2
or 3 twitches of 4. This is the ideal scenario, and it will
predict a faster reversal of neuromuscular blockade
(Answer C is correct). A TOF of 0 or 1 of 4 twitches
predicts a significantly slower neuromuscular recovery

ACCP Updates in Therapeutics® 2022: Critical Care Pharmacy Preparatory Review and Recertification Course

481
Management of PADIS and NMB in Adult Intensive Care Unit Patients

ANSWERS AND EXPLANATIONS TO SELF-ASSESSMENT QUESTIONS

1. Answer: D goal should be to achieve a deeply sedated and/or non-


Propofol infusion syndrome is a well-documented and agitated state before initiating an NMBA in an effort
complex set of adverse events, potentially resulting in to avoid any patient discomfort that may be undetected
multiorgan failure. An elevation in lactate, creatine during paralysis (Answer D is correct). The SAT would
kinase, transaminases, SCr, and triglycerides and the be inappropriate in someone who is rated “agitated” on
presence of a metabolic acidosis are some of the abnor- the sedation scale or in a patient requiring escalating
malities that should concern the critical care provider doses of sedation (Answer A is incorrect).
for the presence of PRIS (Answer D is correct). Both
DVT and critical illness polyneuropathy are serious 4. Answer: B
concerns in the ICU patient; however, the abnormalities The pharmacokinetics/dynamics of prolonged fentanyl
in the case are not representative of these complications infusions have not been well described in the adult ICU
(Answers A and B are incorrect). There are currently population. Most data for fentanyl are derived from
no known abnormal laboratory values to help determine short-term infusions or boluses in healthy volunteers
whether delirium is present in an ICU patient (Answer and in animal models. Fentanyl is hepatically metabo-
C is incorrect). lized primarily by the CYP3A4 enzyme, and decreased
clearance of fentanyl has been described in patients with
2. Answer: B significant liver disease. Other properties of fentanyl
This patient is at risk of propylene glycol toxicity after (e.g., high volume of distribution, high protein binding,
receiving a lorazepam drip for more than 48 hours. and high lipophilicity) may contribute to unpredictable
Lorazepam is dissolved in propylene glycol, an alcohol clearance and a prolonged context-sensitive half-time
that can induce an osmolar gap and metabolic lactic aci- for patients in acute renal failure or in patients who have
dosis, particularly in patients with significant hepatic or inadequate nutritional status (Answer B is correct).
renal failure. Quantitative propylene glycol levels may Propofol is a CYP3A4 inhibitor; therefore, it should not
be unavailable; therefore, surrogate markers such as an induce the metabolism of fentanyl (Answer C is incor-
abnormal osmolar gap (greater than 10 mmol) and met- rect). Propofol is known to chelate trace elements and
abolic acidosis may indicate propylene glycol toxicity increase urinary loss of zinc when used for more than
and a need to discontinue lorazepam. Although loraz- 5 days; propofol has not been shown to cause hypocal-
epam drips are not routinely used for general sedation cemia (Answer D is incorrect). Disease states identified
in adult ICUs, they may be used for other indications as risk factors for PRIS may include sepsis, acute liver
(e.g., severe EtOH [ethyl alcohol] or benzodiazepine failure, and history of pancreatitis; ARDS is not cur-
withdrawal), and clinicians should remain aware of this rently a documented risk factor (Answer A is incorrect).
serious complication (Answer B is correct). With an
oxygen saturation of 98% on 2 L of oxygen, this patient 5. Answer: B
does not meet the predefined criteria of ARDS (Answer Withdrawal from certain home medications may occur
A is incorrect). Encephalopathy will not cause a meta- if these medications are not reinitiated within a few
bolic acidosis; therefore, an ammonia concentration days of admission. The onset of withdrawal symptoms
would not be helpful at this stage (Answer C is incor- will vary depending on the half-life of each medication.
rect). With a low fractional excretion of sodium and a Symptoms may include agitation, anxiety, psychosis,
high BUN/SCr ratio, the patient’s laboratory values are insomnia, hypertension, and tachycardia and can occur
indicative of a pre-renal concern versus acute tubular with medications such as opiates, GABA receptor ago-
necrosis (Answer D is incorrect). nists, antiepileptics, antidepressants, and antipsychotics.
A pharmacist can assist the medical team by obtaining
3. Answer: D a thorough medication history and assessment of home
It is inappropriate to initiate an NMBA in a patient who medication compliance to help identify drug withdrawal
has a sedation score indicating “agitation.” This implies symptoms. Reinitiating these medications can be con-
that the patient may potentially detect pain or discomfort sidered unless contraindicated because of the clinical
while paralyzed (Answers B and C are incorrect). The scenario (e.g., drug-drug interactions, drug-disease

ACCP Updates in Therapeutics® 2022: Critical Care Pharmacy Preparatory Review and Recertification Course

482
Management of PADIS and NMB in Adult Intensive Care Unit Patients

state interactions). The Agitation and Sedation sec- would decrease the risk of increasing the QTc interval
tion of the PADIS guidelines discusses identifying and further (Answer B is correct). This patient is at risk of
treating the etiology of agitation before adding other delirium given the patient’s age and being critically ill
medications; fentanyl should treat this patient’s chronic in the ICU. Avoiding medications that may cause or
pain and treat opiate/tramadol withdrawal (Answer B worsen delirium, such as benzodiazepines, is a strong
is correct). Adding quetiapine or lorazepam for agita- recommendation in the PAD guidelines (Answer D is
tion would not address or treat the underlying etiology incorrect). With a QTc of 500 milliseconds, increasing
of potential opiate/tramadol withdrawal (Answers A the quetiapine dose for agitation could further increase
and C are incorrect). Patient-controlled analgesia with the QTc and put this patient at high risk of cardiac
an opiate would help treat opiate withdrawal; however, arrhythmias (Answer A is incorrect). Amiodarone and
this patient is not alert enough to use it (Answer D is quetiapine are home medications for this patient, and
incorrect). they should be continued, if safe. Alternative methods
for decreasing the risk of QTc prolongation should be
6. Answer: D considered before discontinuing chronic medications
Up to 30%–60% of ICU patients may go through alcohol (Answer C is incorrect).
withdrawal on cessation of alcohol use. The presence of
alcohol withdrawal in ICU patients may prolong their 8. Answer: D
ICU stay, increase hospital costs significantly, and lead Dementia is a progressive and chronic state of cognitive
to other complications during the hospital stay. Early impairment that worsens over weeks to months; this dif-
and aggressive symptom-triggered management with a fers from the acute onset characteristic of ICU delirium
benzodiazepine, particularly in a patient with a history (Answer A is incorrect). Alcohol withdrawal should
of alcohol withdrawal, is a key element in management. always be a consideration for patients who become
There is no otherwise published protocol for alcohol altered or agitated in the ICU. This patient’s vital signs
withdrawal in ICU patients. In this case, although dex- are normal, and she is presenting in a hypoactive state;
medetomidine is useful as an adjunctive agent to help these findings are not typical for acute alcohol with-
decrease sympathetic storm and agitation secondary to drawal, and alternative causes for her decline in mental
alcohol withdrawal, it is not currently recommended for status need to be considered (Answer B is incorrect).
use as a single agent for alcohol withdrawal (Answer Adrenal insufficiency in the ICU usually presents
A is incorrect). Opiates do not treat alcohol withdrawal with abnormal laboratory values and/or hemodynamic
(Answer C is incorrect). Phenytoin is a known anti- instability. This patient’s laboratory values and hemo-
epileptic for use in epilepsy; however, it has not been dynamics are reported as normal; therefore, adrenal
shown to be effective in preventing or treating alcohol insufficiency is unlikely (Answer C is incorrect). Both
withdrawal (Answer B is incorrect). Benzodiazepines iatrogenic and non-iatrogenic causes for delirium are
are the drugs of choice for alcohol withdrawal seizures; well documented in the literature. Dehydration and
therefore, midazolam is the most appropriate drug for infection are common causes of delirium, particularly
this patient, whose medical history is significant for in the older adult population. This patient has several
recurrent alcohol withdrawal seizures (Answer D is reasons for being dehydrated in the hospital: NPO sta-
correct). tus, persistent fevers, and taking hydrochlorothiazide.
Untreated infection or lack of source control is also a
7. Answer: B concern with the presence of persistent fever, even while
Giving several medications that carry a risk of QTc pro- the patient is taking antibiotics (Answer D is correct).
longation is a common scenario in the ICU. Clinicians
should seek to find alternatives to decrease this risk, if
possible, particularly if the QTc is already high (e.g.,
QTc of 500 or greater is considered high risk of cardiac
arrhythmias, including torsades de pointes). Switching
levofloxacin to piperacillin-tazobactam is an accept-
able alternative to treat an aspiration pneumonia and

ACCP Updates in Therapeutics® 2022: Critical Care Pharmacy Preparatory Review and Recertification Course

483
Practice Administration
and Development:
Pharmacoeconomics and
Safe Medication Use
Adrian Wong, Pharm.D., MPH, BCCCP
MCPHS University
Boston, Massachusetts
Practice Administration and Development: Pharmacoeconomics and Safe Medication Use

Practice Administration
and Development:
Pharmacoeconomics and
Safe Medication Use
Adrian Wong, Pharm.D., MPH, BCCCP
MCPHS University
Boston, Massachusetts

ACCP Updates in Therapeutics® 2021: Critical Care Pharmacy Preparatory Review and Recertification Course

487
Practice Administration and Development: Pharmacoeconomics and Safe Medication Use

Learning Objectives C. Indirect.


D. Incremental.
1. Apply the principles of pharmacoeconomics to
patient care. 2. Your institution wants to use pharmacoeconom-
2. Compare and contrast between a medication error, ics data to evaluate the impact of this medication
an adverse drug event (ADE), a preventable ADE, on neurologic disability and cost compared with
and an adverse drug reaction. prothrombin complex concentrate. Which analysis
3. Design an ADE reporting program, including com- would be best to find in the literature?
mittee structure, committee reporting mechanisms, A. Cost-benefit.
and methods of detecting, reporting, and managing B. Cost-effectiveness.
ADEs. C. Cost-minimization.
4. Provide recommendations for improving medication D. Cost-utility.
use safety using the 2017 SCCM Safe Medication
Use Guidelines for the intensive care unit. 3. B.E. is a 60-year-old woman admitted to the surgical
5. Provide safety measures for drug interaction detec- intensive care unit (ICU) with a history of penicillin
tion and prevention. allergy (shortness of breath). She is administered
6. Develop and implement a drug formulary proposal. cefazolin before a surgical procedure and develops
a life-threatening anaphylactic reaction. Which best
describes this patient’s reaction to cefazolin?
Abbreviations in This Chapter
A. Adverse drug reaction (ADR).
ADE Adverse drug event
B. Side effect.
ADR Adverse drug reaction
C. Adverse drug event (ADE).
ASHP American Society of Health-System
D. Preventable ADE.
Pharmacists
FDA U.S. Food and Drug Administration
4. K.S. is a 50-year-old man admitted to the neurology
ICU Intensive care unit
ICU from the medical floor for the management of
PICO Population/problem/patient, intervention/
an apixaban-induced cerebral hemorrhage. Which
indicator, control/comparator/comparison, and
medication would best have been used as an effec-
outcomes
tive tracer medication to help detect this ADE?
P&T Pharmacy and therapeutics (committee)
QALY Quality-adjusted life-year A. Coagulation factor VIIa.
TJC The Joint Commission B. Phytonadione.
WHO World Health Organization C. Protamine.
D. Prothrombin complex concentrate.

Self-Assessment Questions Questions 5 and 6 pertain to the following case.


Answers and explanations to these questions may be As a new pharmacist in the medical ICU at your insti-
found at the end of this chapter. tution, you have been tasked with improving the safety
of medication use in the ICU. You have read the 2017
Questions 1 and 2 pertain to the following case. SCCM Guidelines for Safe Medication Use in the ICU
Your institution wants to evaluate the addition of coagu- and are prepared to provide recommendations to the
lation factor Xa (recombinant), inactivated-zhzo to your interdisciplinary critical care committee regarding your
formulary for reversal of direct oral anticoagulants. suggestions.

1. Which would best describe the type of cost this 5. According to the grading system of the guideline
medication would have? recommendations, which is the best initial recom-
mendation to improve medication use safety, given
A. Fixed medical.
the strength of recommendation?
B. Variable medical.

ACCP Updates in Therapeutics® 2021: Critical Care Pharmacy Preparatory Review and Recertification Course

488
Practice Administration and Development: Pharmacoeconomics and Safe Medication Use

A. Fully integrating the institution’s elec- Which would be the most appropriate restriction to
tronic health record systems to improve its use?
communication. A. Restriction to only intensivist use.
B. Integrating interruptive clinical decision sup- B. Restriction to only patients with hospital-
port into the electronic health record. acquired pneumonia.
C. Creating a policy requiring that intravenous C. Restriction to only infectious diseases clini-
medications only be prepared by pharmacists. cians’ approval.
D. Investing in smart intravenous infusion pumps D. Restriction to only patients with appropriate
to reduce medication errors. cardiac monitoring.

6. After your institution has integrated your recom-


mendation into routine medical care, a former
patient recommends including the patient and other
caregivers in reducing medication errors. According
to the available literature, which best supports this
recommendation?
A. The 2017 SCCM guidelines provide a strong
recommendation for this integration into rou-
tine care.
B. Areas in the hospital outside the ICU provide
data supporting this integration.
C. Data analyses on direct observations of inter-
actions between the medical team and patients
have shown a reduction in medical errors in the
ICU.
D. Data analyses on chart reviews of ICU patients
have shown that these reported outcomes are
not routinely accounted for.

7. E.S. is a 50-year-old woman who was initiated on


meropenem for empiric coverage of pneumonia,
given prior culture data. She was subsequently initi-
ated on valproic acid for the management of agitation;
however, this was not effective for management and
resulted in self-extubation and subsequent reintu-
bation. In investigating this adverse event, which
would best evaluate the likelihood of its occurrence?
A. Drug interaction database.
B. Drug interaction probability scale.
C. Naranjo nomogram.
D. Roussel Uclaf Causality Assessment Method.

8. Your hospital wants to add meropenem/vabor-


bactam, a broad-spectrum antimicrobial targeting
highly drug-resistant organisms that are U.S. Food
and Drug Administration (FDA) approved for com-
plicated urinary tract infections, to the formulary.

ACCP Updates in Therapeutics® 2021: Critical Care Pharmacy Preparatory Review and Recertification Course

489
Practice Administration and Development: Pharmacoeconomics and Safe Medication Use

I.  PHARMACOECONOMICS

A. Overview
1. Outcome research: The economic, clinical, and humanistic outcomes (ECHO) model includes the
variables commonly measured in this type of research.
2. Pharmacoeconomics is defined as the description and analysis of the costs of drugs and pharmaceutical
services and their effects on individuals, health care systems, and society.
3. Economic evaluations: Studies that identify, measure, and compare the costs and consequences of a
pharmaceutical product or service. Outcome-based economic analysis allows one to retain a patient
advocacy role while remaining financially responsible.
a. Full pharmacoeconomic evaluations
i. Cost-minimization analysis
ii. Cost-benefit analysis
iii. Cost-effectiveness analysis
iv. Cost-utility analysis
b. Other cost evaluations
i. Cost analysis
ii. Cost-of-illness
iii. Budget impact analysis
4. Economic studies can convey different results, depending on:
a. Disease and therapy under evaluation
b. Other therapies available to treat the condition
c. Perspective: Providers, payers, patients, and society
d. Primary economic message: The therapy provides good value for the cost. Always requires clinical
judgment
5. Humanistic outcomes or patient-reported outcome measures
a. Quality of life
b. Health-related quality of life
c. Health status
d. Well-being
e. Symptoms and functional status
f. Patient satisfaction

B. Types of Costs
1. Costs versus charges
a. Charges are often reported in the literature and are called costs.
b. However, charges are costs plus profit.
c. Ratio of cost to charge used to estimate costs from charges
2. Total cost of care – Extends beyond the acquisition cost of a drug and includes waste cost, preparation
cost, distribution cost, administration cost, toxicity cost, monitoring cost
3. Direct costs – Resources consumed in the prevention, detection, or treatment of a disease or illness.
These costs can be medical or nonmedical.
a. Medical – Costs associated with medical care
i. Fixed costs – Overhead costs; costs remain constant and do not change. Not typically included
in a pharmacoeconomic analysis
Examples: Cost of electricity, rent, lighting, building maintenance, salaried workers (par-
ticularly administrators)
ii. Variable costs – These costs depend on the volume of use; the more services used, the greater
the expense.
Examples: Medications, hospitalization, laboratory testing, procedures

ACCP Updates in Therapeutics® 2021: Critical Care Pharmacy Preparatory Review and Recertification Course

490
Practice Administration and Development: Pharmacoeconomics and Safe Medication Use

b. Nonmedical – Costs as a result of the illness or disease that do not involve the purchase of medical
services
Examples: Transportation to health care, child care, specialty diets, clothing
4. Indirect costs – Costs as a result of morbidity or mortality. Relate to the change in productivity of
patient/ caregiver as a result of the disease or illness. Sometimes a challenge to assign dollar values
Examples: Income lost because of premature death, inability to work
5. Intangible costs – Costs that represent nonfinancial outcomes of the disease and medical care
Examples: Costs from pain and suffering, grief, other nonfinancial outcomes of disease
6. Incremental costs – Extra costs needed to purchase an additional benefit or effect of the medical care.
Often used in cost-effectiveness analysis
a. Example: Additional medications to control hypertension above standard therapy
b. Costs and charges considered always depend on the perspective (providers, payers, patients, and
society) taken for the analysis.

C. Economic Evaluations (Table 1)


1. Cost-minimization analysis – Outcomes are compared and determined/assumed to be equal; therefore,
results are expressed in monetary form.
a. Arguably the simplest form of pharmacoeconomic evaluation, which compares the cost of two or
more treatment alternatives, treatments, or services and they are determined to be equal in efficacy
b. Costs are compared to determine the least expensive alternative.
c. Should not be used if no evidence supports the efficacy of the treatment alternative
d. General example: Comparing therapeutic agents in the same therapeutic class; comparing different
dosage forms or dosing strategies of the same drug
e. Specific example: Cisatracurium compared with atracurium in patients undergoing general
anesthesia, assuming the same efficacy
f. Challenge: Proving identical efficacy in advance of the evaluation, so use of a cost-minimization
analysis is in question; when this occurs, the focus is redirected to a partial economic evaluation,
the cost analysis
2. Cost-benefit analysis – Compares the outcomes of the intervention in monetary terms with the cost of
the resources consumed for two or more programs or interventions
a. Resources consumed in the program are measured in dollars.
b. Benefits of the intervention or program are translated to dollar values, including direct benefits,
indirect benefits, intangible benefits.
c. Assigning monetary value to health outcomes: Human capital approach, revealed preference studies,
stated preference studies, or willingness to pay
d. Results expressed as either cost-benefit ratio or net cost or net benefit
e. Provides the yield of an investment
Example: May be appropriate to use when justifying and documenting the value of an existing
pharmacy service or the potential worth of an existing service, when determining the value of
pharmacokinetic service compared with a vaccination program
f. Challenge: Putting a dollar value to an outcome, consequence, or benefit and the variability/
imprecision of such methods
3. Cost-effectiveness analysis – Compares two or more treatment alternatives or programs in which
resources used are measured in monetary terms and consequences are measured in units of effectiveness
a. Goals of cost-effectiveness analysis – To determine which alternative can produce the desired effect
or maximization of effect
b. Tries to reveal the optimal alternative, which may not always be the least expensive but provides the
a more desirable outcome than a less expensive option

ACCP Updates in Therapeutics® 2021: Critical Care Pharmacy Preparatory Review and Recertification Course

491
Practice Administration and Development: Pharmacoeconomics and Safe Medication Use

c. Results expressed as the incremental cost-effectiveness ratio (ICER) = the additional cost to the next
most effective intervention producing another unit of output
i. These results can be displayed in a cost-effectiveness plane, which graphs the ICER in four
quadrants.
ii. This shows the incremental effectiveness on the x-axis and the incremental cost on the y-axis.
d. Example: Comparing treatment options for smoking cessation. Unit of effectiveness is a successful
quit attempted. Costs per successful quit attempted on average for each treatment and then
incremental costs per quit attempted between treatment options are reported.
e. Example: Comparing the cost-effectiveness of anticoagulants for suspected heparin-induced
thrombocytopenia. Unit of analysis or effect is an averted adverse event such as thrombosis.
f. Challenge: Determining the total cost of care, identifying the probabilities to complete the decision
tree for a formal analysis
4. Cost-utility analysis – Method to compare two or more treatment alternatives or programs in which costs
are measured in monetary terms and outcome is expressed in terms of patient preferences, quality of life,
or quality-adjusted life-years (QALYs)
a. Form of cost-effectiveness analysis in which values (utilities) are assigned to the outcome
b. Life and death are reference states for the utilities, with perfect health = 1.0 and death = 0.0.
c. To calculate QALYs, determine the life-years gained and then multiply by the utility of that life-year.
For example, if a person has a life expectancy of 20 years but is disabled and the person believes he
or she is functioning at only 70% of his or her capacity, the QALY is 20 years × 0.7 = 14 QALYs.
d. Utilities or a patient’s preference for a disease state can be measured by obtaining information from
the literature, approaching a convenience sample of experts, using decision theory with a direct
approach, or using psychometric methods with indirect approaches. Direct approaches include
rating scales, time trade-off (acute and chronic conditions), and standard gamble (acute and chronic
conditions). Indirect approaches include multi-attribute health state scores (e.g., quality-of-life
assessment tools).
e. Measures the function and overall well-being of patients using quality-of-life assessment tools, such
as those used in the ICU, which are general and/or disease-specific
i. General health-related quality-of-life instruments
(a) Health Utilities Index III (HUI III)
(b) EuroQol 5D Questionnaire (EQ5D)
(c) Medical Outcome Study Short Form 6D (SF6D)
ii. Disease-specific quality-of-life instruments: European Organization for Research and
Treatment of Cancer 8 Dimensions (EORTC 8D)
f. Example: Pharmacist-managed sedation in the critically ill population; the effect of sedation
medications on the critically ill population. The evaluation could measure quality of sedation
provided according to the patient’s perception.
g. Challenge: Determining utilities
5. Cost analysis
a. Compares the cost of two or more treatment alternatives, treatments, or services
b. Does not evaluate efficacy in advance of study evaluation
c. May be more applicable than a cost-minimization analysis when efficacy is unknown
d. Example: Cisatracurium compared with atracurium in patients undergoing general anesthesia but
unsure of efficacy between drugs; thus, focus is on cost
6. Cost-of-illness analysis
a. Estimate of the overall cost of a particular disease in a defined population
b. Considers direct and indirect costs of the disease or illness
c. Examples: Cost of diabetes; cost of kidney disease; burden of drug overdose on critical care units

ACCP Updates in Therapeutics® 2021: Critical Care Pharmacy Preparatory Review and Recertification Course

492
Practice Administration and Development: Pharmacoeconomics and Safe Medication Use

7. Budget impact analysis or budget impact model


a. Estimates the financial impact of a new health care intervention within the health care setting and
is therefore limited in its perspective
b. Useful in budget planning
c. Helps determine the projected impact of a drug formulary addition
d. Example: Budget impact of applying new treatment guidelines for community-acquired pneumonia
e. Example: Budget impact of using liposomal amphotericin and amphotericin B lipid complex to treat
fungal infections

Table 1. Summary of Economic Evaluations (modified from Drummond 2015)


Measurement/Valuation
Economic Cost
Consequences or Outcomes of Consequence or Challenges
Evaluation Measurement
Outcomes

Must be evaluated. Single


Cost-
effect/outcome of interest for Proving
minimization Monetary for
all alternatives considered Identical consequences are
analysis total cost of care
and effect/outcome achieved identical
(CMA)
to a similar/identical degree

Single or multiple effect/


outcome that does not
have to be common to
Cost-benefit Translating
Monetary for both alternatives. Effects
analysis Monetary units consequences to
total cost of care are translated to monetary
(CBA) monetary units
units, so comparison among
various alternatives is
possible
Determining
Must have a single effect/ the probabilities
Cost- Nonmonetary physical
outcome of interest for all of the clinical
effectiveness Monetary for units of effectiveness.
alternatives considered, pathway for the
analysis total cost of care (e.g., life-years gained,
and the effect/outcome is decision tree to
(CEA) surrogate end point)
achieved to a different degree determine the
consequences
Single or multiple effect/
outcome that does not have
Cost-utility to be common to both
Monetary for Measuring
analysis alternatives. Effects are Utility values and QALYs
total cost of care utilities
(CUA) translated to QALYs, so
comparison among various
alternatives is possible
Challenges
Cost analysis
Not considered, so not a full Not considered, so not a limited compared
(partial Monetary
economic evaluation full economic evaluation with CMA, CBA,
evaluation)
CEA, CUA
QALY = quality-adjusted life-year.

ACCP Updates in Therapeutics® 2021: Critical Care Pharmacy Preparatory Review and Recertification Course

493
Practice Administration and Development: Pharmacoeconomics and Safe Medication Use

D. Application of Pharmacoeconomics
1. Formulary management
a. Inclusion of newly marketed or other target drugs
b. Exclusion of newly marketed drugs
c. Inclusion of drugs with criteria or restriction
d. Deletion of medications from the formulary
e. Limiting the use of nonformulary items
f. Affecting physician or provider prescribing patterns
2. Clinical guidelines, policies, or protocols – Help influence prescribing and promote the most cost-
effective and desirable use of drugs in the context of safety and efficacy
3. Drug use policy – Policies implemented to promote the most efficient use of health care products and
services. Drug use policies can influence providers’ prescribing practices to provide high-quality care,
given the resources available.
4. Services or program evaluations – Pharmacoeconomics can help determine the value of an existing
medical or pharmacy service or the potential worth of starting a new service, in addition to guiding the
establishment of key business objectives or metrics of growth, adoption, or process control.
5. Population health analytics
6. Individual patient treatment decisions – Most important application of economic outcomes
7. Publication of pharmacoeconomic evaluations with pharmacist involvement

Patient Case

1. J.R. is a 30-year-old woman admitted to the surgical ICU after a motor vehicle accident. Which would best
exemplify an indirect cost from her perspective?
A. Loss of wages from her occupation as an engineer.
B. Effects of posttraumatic stress disorder from her accident.
C. Fentanyl administered to manage her pain.
D. Cost of the ambulance that transported her to the hospital.

II.  DRUG-RELATED EVENTS: MEDICATION ERRORS, ADES AND ADRS, SIDE EFFECTS,
PREVENTABLE ADES

A. In the United States, 770,000 injuries or deaths are caused by ADRs annually; the cost burden of ADEs is
greater than $5.6 million per hospital.

B. At least FOUR emergency department (ED) visits for ADEs occur per 1000 individuals annually.

C. Up to 60% of hospital readmissions are attributable to either medication access issues or ADEs – This was
before the data explosion with the electronic health record, which may have made it worse if it is used in
discordant fashion.

D. A severe ADE increases the hospital length of stay by 8–12 days at a cost of $16,000–$24,000 per patient.

E. The cost burden of preventable ADEs in the United States is $4.1 million per hospital annually.

ACCP Updates in Therapeutics® 2021: Critical Care Pharmacy Preparatory Review and Recertification Course

494
Practice Administration and Development: Pharmacoeconomics and Safe Medication Use

F. Medical malpractice lawsuits are often related to, or occur because of, ADEs.

G. The risk of ADEs increases when patients are critically ill and taking many high-risk medications –
specifically intravenous medications – with several illnesses and multiorgan failure.

H. The Joint Commission (TJC) medication management standard requires hospitals to respond to actual or
potential ADEs, significant ADRs, and medication errors.

I. At the very least, hospitals need to respond, document and report, and manage ADEs. The pharmacist,
who is the drug expert, should play an integral role in prospective order verification to prevent ADEs by
intervening as needed.

J. Definitions of drug-related events


1. Medication errors
a. Mishaps that occur during reconciling, prescribing/ordering, transcribing, dispensing, administering,
or monitoring a drug. Medication errors that are intercepted and stopped before they occur are
called “near misses.” Some medication errors cause injury (result in an ADE), and some do not.
Medication errors that cause significant harm may be reported through the health system’s patient
incident reporting system.
i. Harm is physical or psychological injury or damage to a patient’s health, including both
temporary and permanent injury.
ii. Medication errors are categorized as follows: no error (category A), error and no harm
(categories B–D), error and harm (categories E–H), and error and death (category I). Per TJC,
a sentinel event includes events such as those that cause severe temporary harm (requiring
intervention), permanent harm, or death (e.g., potentially Category E, Categories F through I).
Specifically,
(a) Category A: No error – Scenarios that can cause error
(b) Category B: An error occurred but did not reach the patient
(c) Category C: An error occurred and reached the patient but did not cause harm
(d) Category D: An error occurred that reached the patient and required monitoring to confirm
that it resulted in no harm to the patient or required intervention to prevent harm
(e) Category E: An error occurred that may have contributed to or resulted in temporary harm
to the patient and required intervention
(f) Category F: An error occurred that may have contributed to or resulted in temporary harm
to the patient and required initial or prolonged hospitalization
(g) Category G: An error occurred that may have contributed to or resulted in permanent
patient harm
(h) Category H: An error occurred that required intervention necessary to sustain life
(i) Category I: An error occurred that may have contributed to or resulted in the patient’s
death
b. May be reported through the Institute for Safe Medication Practices (ISMP Medication Error
Reporting Program)
i. A confidential and voluntary system
ii. Reports are investigated by ISMP staff and then reported to the FDA’s MedWatch program and
product vendors.
iii. A separate reporting method is used for vaccines, specifically the ISMP Vaccine Errors
Reporting Program.
iv. Vaccine medication error reports provide an option to submit the vaccine’s manufacturer name,
dosage, lot number, expiration date, and National Drug Code.

ACCP Updates in Therapeutics® 2021: Critical Care Pharmacy Preparatory Review and Recertification Course

495
Practice Administration and Development: Pharmacoeconomics and Safe Medication Use

v. Consumers may also report medication errors through ISMP.


vi. In addition to describing the error, images may be submitted and uploaded.
vii. Medication errors may be reported directly to the FDA by the MedWatch system. The FDA’s
MedWatch allows ADE and medication error reporting.
2. Adverse drug events
a. Injuries resulting from the use of a drug, which include harm caused by the drug (ADRs and
overdoses) and harm from use of the drug, including dose changes and discontinuations of drug
therapy
b. Overall, ADE is the broader term used to describe any harmful event associated with a medication,
including inappropriate use such as an overdose, whereas ADR is used when an adverse response,
including harm, occurs with normal use of the medication.
3. Adverse drug reactions
a. Defined by the World Health Organization (WHO) as “any response to a drug which is noxious and
unintended, and which occurs at doses normally used in man for prophylaxis, diagnosis, or therapy
of disease, or for the modification of physiological function”
b. According to Karch and Lasagna (JAMA 1975;234:1236-41): “any response to a drug that is noxious
and unintended, and that occurs at doses used in humans for prophylaxis, diagnosis, or therapy,
excluding failure to accomplish the intended purpose”
c. The primary difference between the WHO and the Karch and Lasagna ADR definitions is that,
according to the WHO definition, therapeutic failures are an unintended effect and an ADR,
whereas according to the definition by Karch and Lasagna, they are not. ADRs caused by therapeutic
failures with easily retrievable, detailed data that document the drug and the therapeutic failure
can significantly affect patient care. Also of note, both definitions refer to doses normally used in
humans, which excludes overdoses and medication errors that exceed normal doses.
4. Examples of therapeutic failures that may be documented as ADRs according to the WHO criteria
include:
a. Clopidogrel failure to prevent an ischemic stroke
b. Enoxaparin failure to prevent deep venous thrombosis
c. Famotidine failure to prevent intravenous ketorolac-induced gastropathy
d. Pantoprazole failure to prevent gastrointestinal bleeding from a stress ulcer in a critically ill patient
5. Examples of ADRs
a. Lorazepam when used to treat anxiety may cause sedation – This is an unintended adverse reaction;
conversely, lorazepam for sedation in a critically ill patient is an intended effect, not an ADR.
b. Diphenhydramine causing unintended sedation when used to treat allergic rhinitis – This is an
ADR; conversely, diphenhydramine as a sedative hypnotic is an intended effect, not an ADR.
6. Side effect
a. Controversy often surrounds whether all ADRs should be reported and documented and whether
ADRs are the same as or different from side effects.
b. The American Society of Health-System Pharmacists (ASHP) defines a side effect as an expected
well-known reaction resulting in little or no change in patient treatment, such as antihistamine-
induced drowsiness. The term side effect may minimize or downplay the risk of injury from
medications. It has been suggested that the term side effect be avoided in favor of the term ADR. In
general, side effects and ADRs are terms used synonymously.
c. Although the ASHP definition of a side effect differs from the WHO definition of an ADR, any
reaction to a medication, including a side effect, must be documented in the patient’s medical
record. It is also prudent and good practice to document side effects in the patient’s electronic
medical record.

ACCP Updates in Therapeutics® 2021: Critical Care Pharmacy Preparatory Review and Recertification Course

496
Practice Administration and Development: Pharmacoeconomics and Safe Medication Use

d. Medication side effects are not without consequences and can lead to harmful effects (e.g.,
antihistamine-induced drowsiness may lead to a traumatic fall and hip fracture, resulting in hip
replacement surgery). Documenting the ADR and managing the event with a lower antihistamine
dose or changing a less sedating agent could prevent a side effect from leading to an ADE.
e. ASHP further defines a side effect as an effect with a predictable frequency and an effect whose
intensity and occurrence are related to the size of the dose. Predictable “side effects” should also be
reported as ADRs, such as:
i. Insulin-induced hypoglycemia
ii. Chemotherapy-induced nausea
iii. Opioid-induced pruritus
7. Preventable ADEs
a. A preventable ADE occurs when a breach of standard professional behavior, practice, or technique
was identified or when necessary precautions were not taken, or when the event was preventable by
modifying behavior, practice, technique, or care.
b. Results from any medication error that reaches the patient and causes harm
c. About 30%–50% of all ADEs are preventable.
d. Drug interactions account for 3%–5% of all preventable in-hospital ADRs (drug interactions are
discussed further in section IV, Drug Interaction Surveillance and Prevention).
e. Nonpreventable ADEs are ADEs not associated with a medication error. Unintended reactions
(ADRs) without known mitigation strategies resulting in patient harm (a patient with a bleed despite
appropriate dosing, administration, and monitoring of the anticoagulant)
f. Examples of preventable ADEs (note that all examples have definitive harm):
i. Carbamazepine prescribed for an Asian patient with bipolar disorder without testing for the
HLA-B*1502 allele; in turn, the patient develops carbamazepine-induced toxic epidermal
necrolysis
ii. Heparin administered without the use of weight-based dosing, causing a supratherapeutic
partial thromboplastin time and an intracranial hemorrhage
iii. Intravenous levofloxacin and intravenous ondansetron prescribed to a patient receiving
haloperidol with a known prolonged corrected QT interval on the electrocardiogram (ECG),
causing torsades de pointes
iv. Phenytoin mixed accidentally with dextrose rather than normal saline, causing precipitation
and lack of drug potency and leading to a patient developing withdrawal seizures and status
epilepticus
v. Rivaroxaban prescribed to a patient for nonvalvular atrial fibrillation stroke prevention who is
receiving rifampin; rifampin increases rivaroxaban hepatic metabolism through cytochrome
P450 (CYP) 3A4 and induction of P-glycoprotein, which may cause subtherapeutic rivaroxaban
serum concentrations and lead to decreased efficacy, in turn causing the patient to develop a
stroke
vi. Sodium bicarbonate and calcium chloride coadministered during cardiac resuscitation,
resulting in a physical incompatibility, precipitation, and loss of vascular access
vii. Sulfamethoxazole/trimethoprim intravenously prescribed to a patient receiving warfarin,
causing inhibition of warfarin metabolism, displacement from warfarin’s plasma protein-
binding sites, and hypoprothrombinemia, causing the patient to develop a gastrointestinal bleed

ACCP Updates in Therapeutics® 2021: Critical Care Pharmacy Preparatory Review and Recertification Course

497
Practice Administration and Development: Pharmacoeconomics and Safe Medication Use

K. FDA-Reportable ADEs
1. For reporting an ADE to the FDA, the FDA defines ADEs as serious adverse events related to drugs
or devices in which “the patient outcome is death, life threatening (real risk of dying), hospitalization
(initial or prolonged), disability (significant, persistent, or permanent), congenital anomaly, or required
intervention to prevent permanent impairment or damage.” FDA-reportable serious ADEs may include
the following:
a. Acetaminophen-induced hepatotoxicity
b. Apixaban-induced intracranial hemorrhage
c. Clopidogrel-induced thrombotic thrombocytic purpura with seizures and hepatic failure and the use
of plasmapheresis
d. Gentamicin-induced irreversible ototoxicity
e. Heparin-induced thrombocytopenia with stroke
f. Linezolid-induced thrombocytopenia with genitourinary hemorrhage
g. Olanzapine-induced torsades de pointes
h. Phenytoin-induced toxic epidermal necrolysis
2. The FDA is also interested in serious ADE reports for newly marketed drugs or novel adverse events that
have not been previously reported for new or old drugs.
3. The FDA Adverse Event Reporting System (FAERS) database contains information on adverse
events and medication error reports that have been submitted to the FDA. This database is used as a
postmarketing surveillance system for medications and therapeutic biological products. Limitations of
FAERS include:
a. Lack of certainty of adverse event (i.e., may not be validated)
b. Underreporting
c. Less severe adverse events likely to be underreported
d. Potential lack of detail provided
4. Reporting of ADEs by health care professionals and consumers to the FDA is voluntary and may be
done through the FDA’s MedWatch program (begun in 1993). All of the following may be reported with
respect to an FDA-regulated medication, biologic, tobacco product, dietary supplement, cosmetic, or
medical device:
a. Serious ADE
b. Serious ADR
c. Product quality problem
d. Product use error
e. Therapeutic inequivalence
f. Therapeutic failure
5. Clinicians may report an ADE directly to the drug’s manufacturer. The pharmaceutical manufacturer
has a legal responsibility to follow up with the reporter on all ADEs reported and to report the ADE to
the FDA.
6. ADE reports and medication error reports submitted directly by health care professionals or manufacturers
are entered in the FDA Adverse Event Reporting System database.
7. If the FDA detects a safety concern, regulatory action may be needed to protect the public, such as
updating the product labeling, restricting the drug, communicating the safety concerns to health care
professionals and consumers, or removing the drug from the market.

L. Designing an ADE Reporting Program


1. A comprehensive ADE program should have a policy and procedure, with guidelines for ADE detection,
reporting, management, surveillance, and education (see Appendix 1 for an ADE reporting form).
2. Because of pharmacists’ expertise in drug-induced diseases and their role in preventing and managing
ADEs, pharmacists often serve as chair or co-chair of the ADE committee.

ACCP Updates in Therapeutics® 2021: Critical Care Pharmacy Preparatory Review and Recertification Course

498
Practice Administration and Development: Pharmacoeconomics and Safe Medication Use

3. The ADE committee should be multidisciplinary and composed of the following:


a. Physicians
b. Pharmacists
c. Nurses
d. Risk management personnel
e. Quality assurance and performance improvement personnel
f. Other health care providers
4. In general, the ADE committee is a subcommittee of the pharmacy and therapeutics (P&T) committee
that reports to the P&T committee.
5. The ADE committee should meet monthly, bimonthly, or quarterly, depending on the number of reports
and actionable items that need review.
6. ADE data can be reported by a specialty unit or service, such as:
a. Emergency medicine
b. Geriatrics
c. Intensive care
d. Internal medicine
e. Oncology
f. Operating room
g. Pediatrics
h. Psychiatry
i. Rehabilitation
j. Telemetry
Moreover, this will ensure that appropriate preventive measures are developed for that specialty unit.
7. ADR and ADE data should be reported as mild, moderate, or severe. Many scales are available in the
literature. Definitions for each should be established. An example is provided in Box 1.

Box 1. Definitions for the Degree of ADR or ADE Severitya

1.  Mild ADR or ADE: Results in heightened need for patient monitoring with or without a change in vital signs,
but no ultimate patient harm, or any adverse event that results in the need for increased laboratory monitoring

2.  Moderate ADR or ADE: Results in the need for aggressive intervention with antidotes or increased length of
hospital stay (e.g., severe hypotension [e.g., BP < 90/50 mm Hg], bleeding necessitating transfusions)

3.  Severe ADR or ADE: Results in harm to the patient, prolonged hospitalization, transfer to a higher level of
care, permanent organ damage (e.g., irreversible hepatotoxicity or renal failure), or death with probable ADE
causality nomogram score
Other ADE severity scales are available.
a

ADE = adverse drug event; ADR = adverse drug reaction; BP = blood pressure.

8. The ADE committee should designate which ADEs are preventable and explain why they were
preventable. Examples of preventable ADEs include:
a. Patient receiving vancomycin who develops nephrotoxicity because of an incorrect dosing regimen
and lack of serum concentration monitoring
b. Patient with epilepsy maintained on intravenous valproic acid who develops breakthrough seizures
because of subtherapeutic valproate serum concentrations caused by a drug-drug interaction with
the concomitant use of doripenem

ACCP Updates in Therapeutics® 2021: Critical Care Pharmacy Preparatory Review and Recertification Course

499
Practice Administration and Development: Pharmacoeconomics and Safe Medication Use

9. The ADE committee determines which ADEs will be reported to the FDA or the manufacturer.
10. The committee should report the data regarding who is detecting ADEs and who reports, documents,
and manages the ADEs.
11. The committee should provide trending data on the basis of either drug or drug class and by specialty
units.
12. The committee should benchmark the hospital’s ADEs against itself in previous years and compare
them with the data from other hospitals published in the biomedical literature. Total ADE data can be
reported according to the following:
a. Total number of ADEs
b. ADEs per admission
c. ADEs per patient
d. ADEs per patient-days
e. ADEs per doses dispensed
f. ADEs per doses administered
13. A popular method of reporting ADEs is by the total number of admissions, with an acceptable benchmark
of 2.5%–10% (e.g., a hospital with 10,000 admissions reporting 1000 ADEs would have a 10% ADE
reporting rate).
14. ADE benchmarks are difficult to determine because of the many variables that affect the reporting
methods, such as the ADE definition used by the facility or the definition used by the reporter, the
number of clinical pharmacists available to report, the vigilance and emphasis of the reporting systems,
and the use of technology or reports to increase reporting.

M. Documenting ADRs and ADEs in the Patient’s Medical Record


1. Allergy data are always documented in the patient’s medical record and are required in the patient
profile, which also includes demographics, diagnosis, and pregnancy category. Preferably, the specific
medication that caused the allergy, the type of reaction, and when the allergy occurred should be
documented
2. “Meaningful Use” is the Centers for Medicare & Medicaid Services incentive program meant to aid
in adopting “meaningful use” of electronic health record technology, including advanced data sharing
and a focus on quality of care. “Meaningful Use” in 2018 was changed to “Promoting interoperability”
a. Stage 2 (modified) requires that eligible professionals who transfer their patients to another setting of
care or provider of care supply a summary care record on referral. A summary of care record contains
several items, including a current medication list, and a current allergy list, including a notation of no
current medication allergies, if applicable. For stage 2, this summary of care record must be provided
for more than 10% of the referrals using the reporting period (information directly from Centers
for Medicare & Medicaid Services: https://www.cms.gov/Regulations-and-Guidance/Legislation/
EHRIncentivePrograms/Downloads/TableofContents_EH_Medicare_ModifiedStage2.pdf)
b. Stage 3 requires all eligible professionals, eligible hospitals, or critical access hospitals to provide
a summary of care when transitioning or referring their patients to another care setting, and the
receiving party reviews the summary of care record. For more than 80% of transitions or referrals
received and patient encounters in which the provider has never before encountered the patient, the
eligible professional, eligible hospital, or critical access hospital performs a clinical information
reconciliation. The provider must implement clinical information reconciliation for the following
three clinical information sets (information directly from Centers for Medicare & Medicaid
Services: https://www.cms.gov/Regulations-and-Guidance/Legislation/EHRIncentivePrograms/
Downloads/MedicareEHStage3_Obj5.pdf):
i. Medication: Review of the patient’s medication, including the name, dosage, frequency, and
route of each medication

ACCP Updates in Therapeutics® 2021: Critical Care Pharmacy Preparatory Review and Recertification Course

500
Practice Administration and Development: Pharmacoeconomics and Safe Medication Use

ii. Medication allergy: Review of the patient’s known medication allergies


iii. Current problem list: Review of the patient’s current and active diagnoses
3. A drug-induced allergy can be an ADR or ADE, depending on severity. It is paramount to report and
document ADE data in the patient’s medical record for the same reason as documenting allergy data:
to prevent recurrence with the same drug or a drug from the same or similar drug class and to mitigate
risk when the same drug may need to be used again or to mitigate risk when other medications that can
cause the same adverse event are used.
4. If possible, differentiate between a true allergy and an ADE/ADR within the electronic health record.
5. ADR and ADE data should be recorded in the electronic medical record and maintained in the record
indefinitely.
6. For severe ADRs and ADEs: When the same drug is prescribed, the electronic health record, including
the clinical decision support system, can be programmed to cause a hard stop and prevent the drug from
being prescribed or a hard stop requiring a prompt with an explanation for use. For example, a case of
isoniazid-induced hepatotoxicity may be classified as a severe ADE, and if it is prescribed to the same
patient again, the prescriber will receive a hard stop and will need to provide an explanation for its use
to have the order verified by the pharmacist.
7. For moderate ADRs: When the same drug is prescribed, the electronic health record, including clinical
decision support system, can be programmed to highlight or warn the prescriber or require a text
response with an explanation for its continued use.
8. For mild ADRs: When the same medication is prescribed, the clinical decision support system can be
set to alert the prescriber, or the data can be retrievable for review but without prompting the clinician.
A case of enalapril-induced hypotension may be recorded as a mild ADR and be maintained for
informational purposes but will not prevent the medication from being prescribed to the patient again,
nor will it require the prescriber to provide an explanation.

N. Methods of Medication Error and ADR or ADE Surveillance


1. ADEs can be detected prospectively and retrospectively. Pharmacists may detect ADEs prospectively
while on patient care daily rounds, or from communication with patients while administering medication
histories, transitions of care (e.g., ICU to step-down), or during discharge counseling. Clinical decision
support with automated triggers can be used for prospective surveillance of events (i.e., before an event
has occurred). Pharmacists may also detect ADEs retrospectively through medical record reviews or
during medication histories (i.e., after an event has occurred, so limited opportunity for intervention).
2. Retrospective incident voluntary reporting is an ethical obligation of all health care professionals.
Voluntary reporting is the primary source of event identification for most institutions; however, events
are grossly underreported. Typically, safety pharmacists are responsible for aggregating these data.
These aggregate reports are reviewed by the ADE committee for the institution. Possible prevention
methods are determined by the committee. This is a retrospective evaluation because events are typically
evaluated when there is allotted time, which is often when the patient is discharged from the hospital.
This can be a prospective method if real-time evaluation is an option.
a. ADEs can be detected using a clinical decision support system within an electronic health record
for trigger or tracer drugs – these terms are used synonymously. Trigger or tracer drugs are drugs
that are routinely prescribed to treat ADEs, such as antidotes, or physiologic antagonists or agents
for gastric decontamination. These triggers prompt a targeted medical record review. Examples of
tracer drugs:
i. Dextrose for hypoglycemia
ii. Naloxone for opioid-induced respiratory depression
iii. Protamine for heparin reversal
iv. Activated charcoal for drug toxicity, such as with acetaminophen

ACCP Updates in Therapeutics® 2021: Critical Care Pharmacy Preparatory Review and Recertification Course

501
Practice Administration and Development: Pharmacoeconomics and Safe Medication Use

b. The clinical decision support system can be programmed with a list of tracer drugs so that when
prescribers order a tracer drug, they are asked whether the order is for an ADE and, if so, what type
of ADE occurred. The potential ADE can then be reported and managed by the clinical pharmacist.
One of the benefits of this system is that it captures ADEs that might not have been reported,
especially when a clinical pharmacist may not be present on the unit or on daily patient care rounds,
and it allows a greater number of physician-reported ADEs. Some trigger or tracer drugs are listed
in Box 2.
c. Therapeutic drug concentration and abnormal laboratory value monitoring can be another source
for ADE detection. The pharmacy may design daily reports that contain all the abnormal drug
serum concentrations such as the phenytoin, digoxin, lithium, vancomycin, and aminoglycoside
concentrations. The clinical pharmacist can then review all cases of supratherapeutic concentrations
by performing a targeted medical record review for ADEs such as:
i. Phenytoin-induced confusion, ataxia, or nystagmus
ii. Digoxin-induced confusion, headache, and visual disturbances
d. A daily report can also provide abnormal electrolytes and abnormal serum creatinine and hepatic
function (bilirubin, aspartate aminotransferase, and alanine aminotransferase) to detect things such
as:
i. Rifampin-elevated liver enzymes
ii. Lisinopril-induced hyperkalemia
iii. Esomeprazole-induced hypomagnesemia
e. The trigger drug list can be monitored by clinical pharmacists, allowing them to ensure that ADE
documentation has occurred and appropriate management is taking place.
f. Although these are basic clinical decision support functions that include medications and laboratory
values as the knowledge for the alert, more advanced, “smarter” alerts are being used that include
risk scoring–based approaches that help prioritize the pharmacist’s reviews.
g. Identified events must be confirmed for causality. Many instruments can help link the drug with
the event. Structured instruments create a more reliable and valid assessment. A sample ADE form
with the Naranjo criteria for causality determination is described in Appendix 1. This is the most
commonly used ADR causality instrument, though its reliability and validity in the critically ill
population need improvement. Another instrument specific for drug-induced liver injury is the
Roussel Uclaf Causality Assessment Method (RUCAM).
h. Causality assessment includes temporal sequence, dechallenge (removal of the suspect drug),
rechallenge (reintroduction of the suspect drug), evaluation of other causes, objective evidence that
is available or obtained, and history of a reaction to a similar drug.
3. Prospective
a. Direct observation – One method of medication error detection is through direct observation; this
can be accomplished using the medication pass method, in which the pharmacist observes nurses in
the medication administration process and notes any errors that occur. Direct observation provides
the unique advantage of capturing medication administration errors that are not typically identified
with other detection methods. Other examples of direct observation include nurses observing nurses
or physicians observing physicians during medication administration. This is considered prospective
because observation occurs in real-time but errors are evaluated later.
b. Clinical decision support with alert generation can be used for prospective or real-time surveillance.
i. Prescribing – Physicians and advanced practice providers can be notified of high-risk scenarios
during the ordering process to prevent medication errors, such as prescribing enoxaparin in
a patient with a creatinine clearance of less than 30 mL/minute. Alerts during medication
ordering should be used judiciously and created with greater precision to avoid alert fatigue.

ACCP Updates in Therapeutics® 2021: Critical Care Pharmacy Preparatory Review and Recertification Course

502
Practice Administration and Development: Pharmacoeconomics and Safe Medication Use

ii. Order verification – Pharmacists receive preventive alerts during order verification to avoid
medication errors and the potential for ADEs. Common alerts that pharmacists receive are to
prevent drug-drug interactions or duplicate therapy. These alerts still need to be improved for
specificity to prevent over-alerting or alert fatigue.
iii. High-risk scenarios in real-time – Pharmacist may receive advanced alerts outside order
verification, indicating when a patient is initiated on a drug, but the patient’s clinical scenario
presents a risk. An alert for a patient receiving an epidural and initiated on an anticoagulant
presents an opportunity for intervention to prevent an ADE.

Box 2. Examples of medications Used as Triggers or Tracers to Aid in Reporting ADEs


1. Acetylcysteine (enteral, injection) 12. Flumazenil
2. Activated charcoal suspension 13. Glucagon
3. Atropine (injection) 14. Hydrocortisone (injection, topical)
4. Benztropine 15. Loperamide
5. Dantrolene 16. Naloxone
6. Diphenhydramine 17. Phytonadione
7. Diphenoxylate/atropine 18. Prednisone
8. D50W 19. Protamine
9. Digoxin immune FAB 20. Prothrombin complex concentrate
10. Epinephrine 0.15/0.3 mg auto-injector 21. Sodium polystyrene sulfonate
11. Fidaxomicin 22. Vancomycin (enteral, rectal)
D50W = dextrose 50% in water.

Patient Cases

2. Which best classifies the degree of severity of an ADE in a patient who develops enalapril-induced asymp-
tomatic hyperkalemia with no noted ECG changes (potassium 5.7 mEq/L) managed with one dose of sodium
polystyrene sulfonate?
A. Mild.
B. Moderate.
C. Severe.
D. Life threatening.

3. Which best classifies the degree of severity of an ADE in a patient who is in the geriatric psychiatry unit and
develops intravenous haloperidol-induced torsades de pointes that is successfully treated with intravenous
magnesium and managed with additional monitoring in the cardiac care unit and telemetry?
A. Mild.
B. Moderate.
C. Severe.
D. No classification.

ACCP Updates in Therapeutics® 2021: Critical Care Pharmacy Preparatory Review and Recertification Course

503
Practice Administration and Development: Pharmacoeconomics and Safe Medication Use

Patient Cases (continued)

Questions 4 and 5 pertain to the following case.


M.S., a 77-year-old man residing in a nursing home, has taken lisinopril 10 mg daily for the past 3 months. He is
admitted to the ICU with lisinopril-induced angioedema and presents with severe tongue swelling, stridor, and
shortness of breath that necessitated a tracheotomy. He had no history of allergies and did not miss any doses of
lisinopril.

4. Which best describes this patient’s reaction to lisinopril?


A. ADE.
B. Preventable ADE.
C. Medication error.
D. Preventable ADR.

5. Which best classifies the degree of severity of this patient’s ADE to lisinopril?
A. Mild.
B. Moderate.
C. Severe.
D. No classification.

III.  CLINICAL PRACTICE GUIDELINE: SAFE MEDICATION USE IN THE ICU

A. Structure
1. 34 population, intervention, comparator, and outcomes (PICO) questions; five quality of evidence
statements; one commentary statement
2. Literature search ending in December 2015
3. Three key components:
a. Environment and patient
b. Medication use process
c. Patient safety surveillance systems
4. GRADE system
a. Quality of evidence: A= high; B = moderate; C = low; D = very low
b. Recommendation: 1 = strong; 2 = weak; 0 = no recommendation

B. Highlights of Environment and Patient


1. Adult and pediatric ICU patients have different risk factors for ADEs and medication errors compared
with general care (non-ICU) patients (C).
2. This is a quality of evidence statement with a low quality of evidence.
3. Some data analyses have compared the risk factors for ADEs and medication errors in this population,
but more definitive comparisons are needed.

C. Highlights of Medication Use Process


1. Four stages of process:
a. Prescribing

ACCP Updates in Therapeutics® 2021: Critical Care Pharmacy Preparatory Review and Recertification Course

504
Practice Administration and Development: Pharmacoeconomics and Safe Medication Use

b. Dispensing
c. Administration
d. Monitoring
2. Prescribing
a. Guidelines suggest implementing computerized provider order entry to decrease medication errors
and preventable ADEs (2B).
i. 14 studies were evaluated that met the PICO criteria, with most (n=12) focusing on medication
error assessments and only two including an evaluation of ADEs.
ii. Some data support medication error reduction, but evidence is conflicting, with a decrease in
omission errors but an increase in duplicate order errors.
b. Guidelines suggest using clinical decision support systems to decrease the number of medication
errors and ADEs (2C).
i. 10 studies were evaluated that met the PICO criteria.
ii. Heterogeneous types of clinical decision support systems (i.e., drug allergies, renal dosing)
iii. Trigger-initiated clinical decision support alerts cause a concern for alert fatigue that requires
a balance between sensitivity and specificity.
iv. More patient-centered clinical decision support alerts are needed to appreciate their true benefit.
c. Guidelines make no recommendation regarding the use of medication reconciliation to decrease
medication errors and ADEs in ICU patients. Evidence is not available supporting the PICO criteria
for transitions of care in the ICU.
3. Dispensing
a. We recommend compliance with safe medication concentration practices to reduce the number of
medication errors and potential ADEs (1B).
b. Safe medication concentration practices include the use of premade intravenous preparations,
requirement of pharmacists to prepare all intravenous medications, standardization of concentrations
for the institution, and titration charts.
c. 10 studies were evaluated that met the PICO criteria (seven specific to the ICU).
d. Studies showed a consistent reduction in medication errors.
4. Administration
a. We suggest that smart intravenous infusion pumps be used to reduce the rate of medication errors
and/or ADEs in the ICU (2C).
b. Eight studies were evaluated that met the PICO criteria.
c. Evidence was conflicting on the use of the guard rails and types of errors evaluated.
d. Concern for auditory alarm fatigue exists with smart pumps, especially in the context of other
auditory alarms in the ICU.

D. Patient Safety Surveillance Systems Highlights


1. Nontargeted chart review
a. Perform nontargeted chart reviews for detecting ADEs as part of a surveillance system (2B)
b. Although a nontargeted chart review can identify events, it is a resource-intensive process and only
receives a recommendation of “suggested.”
2. Targeted chart review
a. Use of trigger-initiated target chart review in addition to voluntary reports to increase the quantity
of ADEs reported (2B)
b. A targeted chart review includes evaluating only a specific section of the patient’s medical record
(i.e., ICU discharge notes) or reviewing a medical record for a specific patient according to a trigger
alert.

ACCP Updates in Therapeutics® 2021: Critical Care Pharmacy Preparatory Review and Recertification Course

505
Practice Administration and Development: Pharmacoeconomics and Safe Medication Use

3. Direct observation
a. Include direct observation as a component of an active medication surveillance system to identify
the medication errors (1A)
b. Description in methods of medication error and ADR or ADE surveillance (above N3a)
4. Patient and family reporting
a. Use of a patient/family-reported outcome interviews at or after ICU discharge about possible
medication errors or ADEs that occurred (2C)
b. Formalizes a process for interviewing patients or family members about possible medication errors
or ADEs that occurred while the patient was in the ICU
c. Supporting data are available outside the ICU but have not been applied in the ICU; thus, this
appears as a suggestion in the guideline.

E. Summary: This is the first clinical guideline to address safe medication use in the ICU, and rigorous research
to guide practice is still needed in many areas.

IV.  DRUG INTERACTION SURVEILLANCE AND PREVENTION

A. Definition of Drug Interactions


1. When the effects of one drug can be changed by the presence of another
a. Can be benign and insignificant
b. Can be intentional
c. Can be harmful and life threatening
2. Related toxicity is preventable.
a. Avoid the combination and change to an alternative therapy.
b. Adjust doses to compensate for the interaction.
c. If the combination cannot be avoided, monitor for the efficacy and toxicity of the object drug.

B. A documented drug interaction with known outcomes can be considered an ADR, a medication error, or a
preventable ADE.

C. Specific to the ICU are more concerns for drug infusion compatibility interactions.
1. Expansion of prolonged infusions has increased concerns for drug incompatibilities.
2. Pharmacist vigilance to medication administration practices is needed for prevention.

D. Drug interaction databases include Lexicomp, IBM Micromedex, Epocrates, Clinical Pharmacology,
Hansten and Horn’s Drug Interaction Analysis and Management, Stockley’s Drug Interactions, and PDR
Drug Interactions. Many electronic health records include Medi-Span and First Databank.
1. Most compilation databases have drug interaction software in which the pharmacist can provide a list of
drugs and the database will provide the type of interaction and severity. Of note, the harmony between
databases is for severity of interactions is estimated to be less than 50%.
2. Lexicomp scale for drug interactions
A = No known interaction
B = No action needed
C = Monitor therapy
D = Consider therapy modification
X = Avoid combination

ACCP Updates in Therapeutics® 2021: Critical Care Pharmacy Preparatory Review and Recertification Course

506
Practice Administration and Development: Pharmacoeconomics and Safe Medication Use

3. Micromedex scale for drug interactions


a. Severity scale: Unknown, minor, moderate, major, contraindicated
b. Documentation scale: Excellent, good, fair, unknown
4. Hansten and Horn’s Drug Interaction Analysis and Management
a. Provides summary, risk factors, mechanism, clinical evaluation, related drugs, and references
b. Class of interaction with brief explanation
1 = Avoid combination. Risk always outweighs benefit.
2 = In general, avoid combination. Use combination only under special circumstances.
3 = Minimize risk. Take action as necessary to reduce risk.
4 = No action needed. Risk of adverse outcomes appears small.
5 = No interaction. Evidence suggests no interaction.
5. Stockley’s Drug Interactions: Provides outcome, clinical evidence, mechanism, importance, and
management
6. Clinical judgment should always be applied when assessing drug interactions.

E. Safety Measures to Avoid Drug Interactions


1. Pharmacist review and validation
a. Pharmacists are highly trained in pharmacology and drug interactions and are expected to apply
advanced analysis and clinical qualification/management of drug interactions.
b. Pharmacists’ scope of practice includes a comprehensive review of patients’ medications for drug
interactions and use of appropriate tools.
c. All prescription records are maintained in one profiling system.
d. Fill all medications in the same pharmacy, if possible, to maintain a comprehensive and accurate
medication record.
2. Using electronic health records with clinical decision support alerts that detect drug interactions
a. Passive pop-up warnings (alerts)
b. Active alerts with soft stops with explanation required
c. Directional active hard stops
d. Dynamic ordering in which blatantly wrong options are censored, leaving only viable options
available to make a “right” decision
e. May prompt prescriber or pharmacist during order verification with a high frequency of alerts and
desensitize the clinicians, known as alert fatigue or burden
f. Drug-drug interaction alert fatigue can be managed by reducing the alert number, using severity or
tiering of the interaction as a criterion for alert selection.
3. Education as a method to prevent drug interactions
a. Lectures, grand rounds, daily patient care rounds
b. Pocket drug cards
c. Drug alerts
d. Electronic drug information databases

F. How to Evaluate Drug Interaction Cases


1. Assessment of causation of a drug interaction includes a temporal relationship, consideration of the
pharmacologic properties of the object and precipitant drug, patient factors and disease states, the
possible contribution of other drugs, and, when possible, a positive dechallenge. The temporal sequence
is the key element in these considerations to help determine the cause.
2. ADE nomograms such as the Naranjo nomogram are designed to evaluate ADEs, not drug-drug
interactions; therefore, they should not be used to evaluate drug-drug interaction cases.

ACCP Updates in Therapeutics® 2021: Critical Care Pharmacy Preparatory Review and Recertification Course

507
Practice Administration and Development: Pharmacoeconomics and Safe Medication Use

3. The drug interaction probability scale may be used to determine drug-drug interaction causation,
including the adverse outcomes in a specific patient (see Appendix 2).
4. The drug interaction probability scale is patterned after the Naranjo ADR probability scale. A series of
10 questions related to the drug interaction are assessed with “yes,” “no,” “unknown,” or “not available”
answers and then scored and tabulated. The total score determines the probability of the drug-drug
interaction occurring in the patient and is scaled as follows:
a. Highly probable: More than 8
b. Probable: 5–8
c. Possible: 2–4
d. Doubtful: 2 or fewer
5. When using the drug interaction probability scale, the evaluator must have comprehensive knowledge of
the pharmacologic properties of both the object and the precipitator drug, especially their pharmacokinetic
and pharmacodynamic properties and their mechanisms of drug action and mechanisms of drug
interactions.

V.  FORMULARY PROPOSAL

A. The drug formulary should be comprehensive and include all the medications needed for patient care.

B. TJC medication management standard requires hospitals to develop and approve the criteria for selecting
medications into the drug formulary.

C. At a minimum, the drug selection criteria should include the following:


1. Indications for use (FDA label approved and off-label)
2. Efficacy and effectiveness
3. Drug interactions
4. Adverse effects
5. Sentinel event advisories
6. Cost acquisition and total cost of care – This is a good application of the budget impact analysis. Ideally,
if cost-effectiveness and cost-utility analyses are also available, they should be considered; however,
they are typically not available this early in the drug approval process.

D. The P&T committee is responsible for maintaining the drug formulary and ensuring that new medications
that can improve patient care are reviewed for formulary inclusion.

E. Critical care pharmacists should be cognizant of new medications and new medication dosage forms that
may be used to improve the treatment of critically ill patients. Critical care pharmacists should collaborate
with intensivists to request the addition or deletion of a drug from the formulary.

F. Box 3 lists the elements of a drug evaluation monograph that should be reviewed before a drug is approved
to the hospital formulary.

ACCP Updates in Therapeutics® 2021: Critical Care Pharmacy Preparatory Review and Recertification Course

508
Practice Administration and Development: Pharmacoeconomics and Safe Medication Use

Box 3. Elements of a Drug Evaluation Monograph

Brand name
Generic name
Manufacturer
Therapeutic classification
FDA status: Prescription, nonprescription, or controlled substance
Look-alike sound-alike names with any other FDA label-approved medications
Look-alike sound-alike names with any other FDA label-approved medications on the formulary
Date of FDA label approval
FDA rank (priority or standard)
FDA label-approved indication
Unlabeled indications
Potential unlabeled uses
Similar agents not on the formulary
Similar agents on the formulary
How the drug can be used when applied to available national guidelines
How the drug can be used when applied to hospital guidelines, protocols, or pathways
Dosage form
Dosage strength
Mechanism of action
Absorption
Distribution
Metabolism
Excretion
Common ADRs
Significant or life-threatening ADRs or ADEs
Boxed warnings
Precautions
Contraindications
Drug-drug interactions
Drug-food interactions
Drug-laboratory test interactions
IV incompatibilities
IV compatibilities
Pregnancy category
Use during breastfeeding
Dosage regimen recommendations
Dosage regimen recommendations for special populations such as pediatrics, geriatrics, renal and hepatic
impairment, and dialysis
Routes of administration and/or limitations
Any special administration techniques (prescriber certification, smart-pump limitations)
Preparations available
Storage and stability considerations
Any availability concerns (specialty pharmacy restrictions)
Critical review of pertinent clinical trials with salient critique and conclusions
Critical review of comparison trials with similar or alternative agents

ACCP Updates in Therapeutics® 2021: Critical Care Pharmacy Preparatory Review and Recertification Course

509
Practice Administration and Development: Pharmacoeconomics and Safe Medication Use

Box 3. Elements of a Drug Evaluation Monograph (continued)

Cost analysis including annual projected costs


Economic evaluations
Pharmacoeconomic analyses available
Budget impact analysis specific to the institution
Reimbursement from third-party payers
Are there any severe medication errors or sentinel events with this agent?
Does this medication need to be stored under specific circumstances to avoid medication errors or mix-ups?
Does this medication require tall man lettering labeling or precautionary or high-risk labeling to avoid potential
medication errors or mix-ups?
Is the manufacturer-provided labeling considered clear and safe for dispensing?
Is an abuse potential associated with the use of this medication?
Patient education requirements
Will this agent replace an existing agent, and should a formulary deletion take place?
Reason why or why not this medication should be included in the formulary
Recommendation for addition to the formulary
References

G. Evidence-based decisions should be made when adding drugs to the formulary.

H. A comprehensive literature review should be used to determine a drug’s efficacy and toxicity profile, with
stronger levels of evidence guiding decisions.
1. Prospective double-blind randomized controlled trials should have greater weight than retrospective
cohort and case-control trials. Even a meta-analysis can be in question, depending on the number and
criteria for included studies.
2. Case reports should be used only when no other evidence is available.

I. Pharmacists developing the drug evaluation monograph must be adept in drug literature evaluation and
pharmacoeconomics.

J. Drug evaluations should contain references to evidence, and opinion statements should be so noted.

K. Internal prescribing data may also be used in formulary decisions, such as


1. Quantity of drug used over a specified time
2. Medication use evaluation data
3. ADR data
4. Medication error data

L. Most new drugs are studied in 1500–3000 patients, which may make it difficult to detect less common severe
or life-threatening adverse effects. Some drugs may cause life-threatening toxicity such as hepatic failure
at a rate of 1:20,000 patients, thus requiring more than 100,000 postmarketing patient exposures before
generating a signal of toxicity. It may be prudent to observe the safety profiles of all new drugs for 1–2 years
before admitting them into the formulary, if possible.

M. Drugs may be added to the formulary without any use restrictions, or they may be added with restrictions.

ACCP Updates in Therapeutics® 2021: Critical Care Pharmacy Preparatory Review and Recertification Course

510
Practice Administration and Development: Pharmacoeconomics and Safe Medication Use

N. Drugs can be restricted for many reasons, such as the following:


1. Efficacy
2. Safety
3. Patient-specific populations (because of limited efficacy or safety evidence)
4. Cost

O. Drugs are commonly restricted to a prescriber who is a specialist or a clinical pharmacy specialist, a specialty
unit such as the ICU, or a population such as pediatric patients or postoperative surgical patients.
1. For example, propofol may be restricted to use in intensive care and surgical units or to use by intensivists
or anesthesiologists.
2. Parenteral fosphenytoin may be restricted to use in the intensive care and surgical units and the ED or
units, where appropriate continuous cardiac monitoring will take place, such as in telemetry.
3. Select antimicrobials are often restricted to approval from infectious disease physicians or infectious
disease pharmacy specialists.
4. Dexmedetomidine, indicated for sedation in intubated and mechanically ventilated patients during
treatment in an ICU setting, may be restricted to use by intensivists; however, to curtail costs, additional
criteria may be required of the intensivist before use.

P. Once a drug is admitted for formulary approval, periodic assessments in the form of a medication use
evaluation or reviews of use (including compliance with institutional guidelines), cost, safety, and efficacy
should be made, preferably within 3–6 months and again in 1 year. The goal is to determine the effectiveness
of the drug (different from efficacy). Effectiveness is the use of a drug in the real-world setting outside a
randomized controlled trial.

Q. All drugs on the formulary within a class should be assessed annually or more often when there is an
important change in prescribing information, when a landmark trial or publication affects the drug’s use, or
when new FDA label–approved agents are available within the drug class. Medication assessments typically
prompt updates and modifications to the drug’s current use.

Patient Case

Questions 6–8 pertain to the following case.


Your institution is considering adding angiotensin II (ATII) to the hospital formulary. Angiotensin II is a non-
catecholamine vasopressor that is FDA approved for increasing blood pressure in distributive shock following
publication of the ATHOS-3 trial. The trial, which compared intravenous ATII with placebo once patients received
a certain dose of vasopressors, showed that patients who received ATII reached a hemodynamic goal more often
than placebo, with no differences in mortality or safety outcomes. Your hospital formulary includes the catechol-
amine vasopressors and vasopressin. A particular concern of your institution is the cost of ATII therapy.

6. When creating the drug evaluation monograph for adding ATII, which would be most appropriate to include?
A. Recommendation to remove vasopressin from the hospital formulary to make room for ATII.
B. Recommendation that ATII decreases mortality in patients with distributive shock.
C. Recommendation that both the intravenous and oral formulations of ATII be added to the formulary.
D. Recommendation to restrict use to patients who received the same dose of vasopressors as those included
in the ATHOS-3 trial.

ACCP Updates in Therapeutics® 2021: Critical Care Pharmacy Preparatory Review and Recertification Course

511
Practice Administration and Development: Pharmacoeconomics and Safe Medication Use

7. In considering adding ATII to the formulary, which characteristic of your institution would be most impor-
tant to consider?
A. Data from ATHOS-3 suggesting its clinical superiority to other FDA-approved medications.
B. How often you encounter patients who require several vasopressors for management of distributive
shock.
C. The increased risk of ADEs associated with the use of ATII.
D. Data analyses suggesting the cost-saving benefits of ATII compared with other FDA-approved
medications.

8. Angiotensin II was approved by your institution last December, after presentation to the P&T committee.
Which best depicts the month in which the first medication use evaluation should be completed?
A. March of this year.
B. June of this year.
C. October of this year.
D. December of this year.

ACCP Updates in Therapeutics® 2021: Critical Care Pharmacy Preparatory Review and Recertification Course

512
Practice Administration and Development: Pharmacoeconomics and Safe Medication Use

REFERENCES

Pharmacoeconomics 2. American Society of Hospital Pharmacists. ASHP


1. Agency for Healthcare Research and Quality. guidelines on preventing medication errors in hos-
Quality Indicators Toolkit. Available at www. pitals. Am J Hosp Pharm 1993;50:305-14.
ahrq.gov/professionals/systems/hospital/qitoolkit/ 3. Bates DW, Cullen DJ, Laird N, et al. Incidence
index.html. Accessed August 28, 2018. of adverse drug events and potential adverse
2. Bootman JL, Townsend RJ, McGhan WF. drug events. Implications for prevention. JAMA
Principles of Pharmacoeconomics, 2nd ed. 1995;274:29-34.
Cincinnati: Harvey Whitney, 1996. 4. Centers for Medicare & Medicaid Services.
3. Coons SJ, Rao S, Keininger DL, et al. A compara- Medicaid Promoting Interoperability Program
tive review of generic quality-of-life instruments. Modified Stage 2 Eligible Professionals.
Pharmacoeconomics 2000;17:13-35. Objectives and Measures for 2018. Available at
4. Dasta JF, Kane-Gill S. Pharmacoeconomics of seda- https://www.cms.gov/Regulations-and-Guidance/
tion in the ICU. Anesthesiol Clin 2011;29:707-20. Legislation/EHRIncentivePrograms/Downloads/
5. Drummond MF, Sculpher MJ, Claxton K, et al. MedicaidEPModStage2_2018_Obj5.pdf. Accessed
Methods for the Economic Evaluation of Health October 21, 2018.
Care Programmes, 4th ed. Oxford: Oxford 5. Centers for Medicare & Medicaid Services. Medicaid
University Press, 2015. EHR Incentive Program Health Information
6. Ernst RF, Levy H, Qualy RL. Simplified pharma- Exchange Objective Stage 3. Available at https://
coeconomics of critical care and severe sepsis. J w w w.cm s.gov / Reg u lat ions-a nd- Gu id a nce /
Intensive Care Med 2007;22:283-93. Legislation/EHRIncentivePrograms/Downloads/
7. Grauer DW, Lee J, Odom TD, et al., eds. HealthInformationExchange_Stage3Medicaid.pdf.
Pharmacoeconomics and Outcomes: Applications Accessed October 21, 2018.
for Patient Care, 2nd ed. Kansas City: American 6. Cooper SL, Somani SK. Pharmacy-based adverse
College of Clinical Pharmacy, 2003. drug reaction monitoring program. Consult Pharm
8. Jo C. Cost-of-illness studies: concepts, scopes and 1990;5:659-64.
methods. Clin Mol Hepatol 2014;20:327-37. 7. Danan G, Benichou C. Causality assessment of
9. Lat I, Paciullo C, Daley MJ, et al. Position paper on adverse reactions to drugs. I. A novel method based
critical care pharmacy services: 2020 update. Crit on the conclusions of international consensus meet-
Care Med 2020;48:e813-e834. ings: application to drug-induced liver injuries. J
10. Robinson A, Gyrd-Hansen D, Bacon P, et al.; Clin Epidemiol 1993;46:1323-30.
EuroVaQ Team. Estimating a WTP-based value 8. Flowers P, Dzierba S, Baker O. A continuous
of QALY: the chained approach. Soc Sci Med quality improvement team approach to adverse
2013;92:92-104. drug reaction reporting. Top Hosp Pharm Manag
11. Rubenfeld GD, Angus DC, Pinsky JR, et al. 1992;12:60-7.
Outcomes research in critical care. Am J Respir 9. Hakkarainen KM, Hedna K, Petzold M, et al.
Crit Care Med 1999;160:358-67. Percentage of patients with preventable adverse
12. Sullivan SD, Mauskopf JA, Augustovski F, et al. drug reactions and preventability of adverse
Budget impact analysis – principles of good prac- drug reactions: a meta-analysis. PLoS One
tice: report of the ISPOR 2012 budget impact 2012;7:e33236.
analysis good practice II task force. 2014;17:5-14. 10. Hassan E, Badawi O, Weber RJ, et al. Using technol-
ogy to prevent adverse drug events in the intensive
Medication Errors, Adverse Drug Reactions, care unit. Crit Care Med 2010;38(suppl):S97-S105.
Adverse Drug Events, and Side Effects 11. Howard RL, Avery AJ, Slavenburg S, et al. Which
1. American Society of Health-System Pharmacists. drugs cause preventable admissions to hospi-
ASHP guidelines on adverse drug reaction moni- tal? A systematic review. Br J Clin Pharmacol
toring and reporting. Am J Health Syst Pharm 2007;63:136-47.
1995;52:417-9.

ACCP Updates in Therapeutics® 2021: Critical Care Pharmacy Preparatory Review and Recertification Course

513
Practice Administration and Development: Pharmacoeconomics and Safe Medication Use

12. Kane-Gill SL, Buckley MS. Medication safety and approaches to manage alert quantity with intent
active surveillance. In: Erstad BL, ed. Critical Care to reduce alert fatigue and suggestions for alert
Pharmacotherapy. Lenexa, KS: American College fatigue metrics. Crit Care Med 2017;45:1481-8.
of Clinical Pharmacy, 2016:971-82. 3. Shah PK, Irizarry J, O’Neill S. Strategies for manag-
13. Kane-Gill SL, Dasta JF, Schneider PJ, et al. ing smart pump alarm and alert fatigue: a narrative
Monitoring abnormal laboratory values as review. Pharmacotherapy 2018;38:842-50.
antecedents to drug-induced injury. J Trauma 4. Winters BD, Cvach MM, Bonafide CP, et al.
2005;59:1457-62. Technological Distractions (Part 2): a summary
14. Kane-Gill SL, Jacobi J, Rothschild JM. Adverse of approaches to manage clinical alarms with
drug events in intensive care units: risk factors, intent to reduce alarm fatigue. Crit Care Med
impact and the role of the team. Crit Care Med 2018;46:130-7.
2010;38:S83-9.
15. Karch FE, Lasagna L. Adverse drug reactions. A Drug Interaction Surveillance and Prevention
critical review. J Am Med Assoc 1975;234:1236-41. 1. Horn JR, Hansten PD, Chan LN. Proposal for a
16. Kelly WN. Can the frequency and risks of new tool to evaluate drug interaction cases. Ann
fatal adverse drug events be determined? Pharmacother 2007;41:674-80.
Pharmacotherapy 2001;21:521-7. 2. Smithburger PL, Buckley MS, Bejian S, et al. A
17. Kimelblatt BJ, Young SH, Heywood PM, et al. critical evaluation of clinical decision support for
Improved reporting of adverse drug reactions. Am the detection of drug-drug interactions. Expert
J Hosp Pharm 1988;45:1086-9. Opin Drug Saf 2011;10:871-82.
18. Koppel R, Metlay JP, Cohen A. Role of computer- 3. Uijtedaal EV, van Harssel LLM, Hugenholtz
ized physician order entry systems in facilitating GWK, et al. Analysis of potential drug-drug inter-
medication errors. JAMA 2005;293:1197-203. actions in medical intensive care unit patients.
19. Morimoto T, Gandhi TK, Seger AC, et al. Adverse Pharmacotherapy 2014;34:213-9.
drug events and medication errors: detection and 4. Zheng WY, Richardson LC, Li L, et al. Drug-
classification methods. Qual Saf Health Care drug interactions and their harmful effects in
2004;13:306-14. hospitalized patients: a systematic review and meta-
20. Prosser TR, Kamysz PL. Multidisciplinary adverse analysis. Eur J Clin Pharmacol 2018;74:15-27.
drug reaction surveillance program. Am J Hosp
Pharm 1990;47:1334-9. Formulary
21. Shehab N, Lovegrove MC, Geller AI, et al. US 1. American Society of Health-System Pharmacists.
emergency department visits for outpatient adverse ASHP guidelines on the pharmacy and therapeu-
drug events. JAMA 2016;316:2115-25. tics committee and the formulary system. Am J
22. Stockwell DC, Kane-Gill SL. Developing a patient Health Syst Pharm 2008;65:1272-83.
safety surveillance system to identify adverse 2. American Society of Hospital Pharmacists. ASHP
events in the intensive care unit. Crit Care Med technical assistance bulletin on drug formularies.
2010;38(6 suppl):S117-25. Am J Hosp Pharm 1991;48:791-3.
23. Vaisman A, McCready J, Powis J. Clarifying a 3. American Society of Hospital Pharmacists. ASHP
“penicillin” allergy: a teachable moment. JAMA guidelines on formulary system management. Am
Intern Med 2017;177:269-70. J Hosp Pharm 1992;49:648-52.
4. Corman SL, Skledar SJ, Culley CM. Evaluation of
Clinical Practice Guidelines: Safe Medication Use in conflicting literature and application to formulary
the ICU decisions. Am J Health Syst Pharm 2007;64:182-5.
1. Kane-Gill SL, Dasta JF, Buckley MS, et al. Clinical 5. Leape LL, Bates DW, Cullen DJ, et al. Systems
practice guideline: safe medication use in the ICU. analysis of adverse drug events. ADE Prevention
Crit Care Med 2017;45:e877-e915. Study Group. JAMA 1995;274:35-43.
2. Kane-Gill SL, O’Connor MF, Rothschild JM, et
al. Technologic Distractions (Part 1): summary of

ACCP Updates in Therapeutics® 2021: Critical Care Pharmacy Preparatory Review and Recertification Course

514
Practice Administration and Development: Pharmacoeconomics and Safe Medication Use

6. Naranjo CA, Busto U, Sellers EM, et al. A method


for estimating the probability of adverse drug reac-
tions. Clin Pharmacol Ther 1981;30:239-45.
7. Principles of a sound drug formulary system. In:
Hawkins B, ed. Best Practices for Hospital and
Health System Pharmacy: Positions and Guidance
Documents of ASHP. Bethesda, MD: ASHP,
2006:110-3.
8. Skledar SJ, Corman SL, Smitherman T. Addressing
innovative off-label medication use at an aca-
demic medical center. Am J Health Syst Pharm
2015;72:469-77.

ACCP Updates in Therapeutics® 2021: Critical Care Pharmacy Preparatory Review and Recertification Course

515
Practice Administration and Development: Pharmacoeconomics and Safe Medication Use

ANSWERS AND EXPLANATIONS TO PATIENT CASES

1. Answer: A occurs from normal use of a medication and is therefore


Indirect costs occur from loss of employment or pro- by definition non-preventable (Answer D is incorrect).
ductivity as the result of illness (Answer A is correct). Therefore, this would be considered an ADE (Answer
Effects from stress would be considered an intangible A is correct).
cost (Answer B is incorrect). Costs from fentanyl and
transportation would be considered direct costs (Answers 5. Answer: C
C and D are incorrect). A severe ADE is defined as an ADE that results in patient
harm, prolonged hospitalization, transfer to a higher
2. Answer: A level of care, permanent organ damage, or death, with
A mild ADE is defined as an ADE that resulted in a the probable ADE causality nomogram score. Because
heightened need for patient monitoring with or without a this patient developed life-threatening lisinopril-induced
change in vital signs but no ultimate patient harm, or as angioedema and needed a tracheotomy, resulting in
any adverse event that resulted in the need for increased patient harm, hospitalization, and transfer to a higher
laboratory monitoring. Enalapril-induced hyperkalemia level of care, this case meets the criteria for a “severe”
managed by one dose of sodium polystyrene sulfonate ADE, making Answer C correct and Answers A, B, and
did not require aggressive interventions, nor did it lead D incorrect.
to any patient harm; however, it did require increased
laboratory monitoring, making Answer A correct and 6. Answer: D
Answers B, C, and D incorrect. When considering what to include in a drug evaluation
monograph for formulary addition, it is important to
3. Answer: C consider the data analyses available supporting its use.
A severe ADE results in patient harm, prolonged hospi- According to what was provided in the case, ATII was
talization, transfer to a higher level of care, permanent only compared with placebo (Answer A is incorrect),
organ damage, or death, which did occur in this case did not show any significant differences in mortality
(Answer D is incorrect). Haloperidol-induced torsades (Answer B is incorrect), and was studied in an intrave-
de pointes is a life-threatening dysrhythmia (end-organ nous formulation (Answer C is incorrect). Answer D is
damage) that required aggressive and successful man- correct because considering the inclusion criteria for the
agement with intravenous magnesium and patient study would be helpful for formulary restriction, espe-
transfer to higher level of care (Answer C is correct). cially given the institutional concern of cost.
A moderate ADE is defined as an ADE that resulted in
the need for aggressive intervention with antidotes or 7. Answer: B
an increased length of hospital stay (Answer B is incor- Considerations for formulary addition are important
rect). A mild ADE is defined as an ADE that resulted in before P&T committee proposal. According to the
a heightened need for patient monitoring with or without ATHOS-3 trial, ATII was not compared with other cur-
a change in vital signs but no ultimate patient harm, or as rently available medications (Answer A is incorrect),
any adverse event that resulted in the need for increased there were no significant differences in ADEs (Answer
laboratory monitoring, which did not occur in this case, C is incorrect), and no data analyses are available sug-
making Answer A incorrect. gesting that ATII is cost-saving (Answer D is incorrect).
Considering how often you might require the use of ATII
4. Answer: A would be an important initial consideration (Answer B is
Because this patient developed lisinopril-induced correct).
angioedema, had no history of allergy to angiotensin-
converting enzyme inhibitors, and missed no lisinopril 8. Answer: A
doses, his ADE was not a preventable error (Answer B It is recommended to occur within 3-6 months of for-
is incorrect). This was also not a medication error, given mulary addition. Therefore, March would be most
the information provided (Answer C is incorrect). There appropriate (Answer A is correct). Although June would
is no such thing as a preventable ADR because an ADR fall into the 3- to 6-month range, given the novelty of

ACCP Updates in Therapeutics® 2021: Critical Care Pharmacy Preparatory Review and Recertification Course

516
Practice Administration and Development: Pharmacoeconomics and Safe Medication Use

this agent and cost, it would be important to conduct a


medication use evaluation as soon as possible (Answer B
is incorrect). October and December would be too long
to wait for an evaluation to be performed (Answers C
and D are incorrect).

ACCP Updates in Therapeutics® 2021: Critical Care Pharmacy Preparatory Review and Recertification Course

517
Practice Administration and Development: Pharmacoeconomics and Safe Medication Use

ANSWERS AND EXPLANATIONS TO SELF-ASSESSMENT QUESTIONS

1. Answer: B bleeding event (Answer B is incorrect). Protamine would


The cost of a medication would be considered a variable be an appropriate tracer for a heparin- or enoxaparin-
medical cost (Answer B is correct). The cost of the lights induced bleeding event (Answer C is incorrect).
in the clean room would be considered a fixed medical
cost (Answer A is incorrect). The time resulting in loss 5. Answer: C
of work for a patient requiring this medication would be The guidelines provide several recommendations, with
considered an indirect cost (Answer C is incorrect). The only one having a strong recommendation, which is
additional use of blood products after administration of based on the dispensing of medications and streamlin-
this medication would be considered an incremental cost ing them as only being from the pharmacy (Answer C
(Answer D is incorrect). is correct). The other potential answers are potential
opportunities to improve medication safety but are based
2. Answer: B on weak recommendations (Answers A, B, and D are
Given the need to consider efficacy and cost, a cost- incorrect).
effectiveness analysis would be the best type of analysis
(Answer B is correct). A cost-benefit analysis would be 6. Answer: B
used if we were purely interested in a monetary outcome The guidelines suggest (not recommend) that patient/
(Answer A is incorrect). A cost-minimization analysis caregiver-reported outcomes be integrated into routine
would only evaluate the difference in costs between patient care (Answer A is incorrect). Data analyses on
these medications (Answer C is incorrect). A cost-utility the benefit of these interactions are limited to non-ICU
analysis includes the patient perspective, which was not environments, though the theoretical benefits listed in
a focus of this evaluation (Answer D is incorrect). Answers C and D may be true (Answers C and D are
incorrect; Answer B is correct).
3. Answer: D
A preventable ADE, by definition, is a medication error 7. Answer: B
that occurs and reaches the patient to cause harm because To determine the probability of a drug interaction
of a breach of standard professional behavior or prac- between meropenem and valproic acid, the drug inter-
tice. The patient had a documented history of penicillin action probability scale should be used (Answer B is
allergy with shortness of breath but still received cefazo- correct). A drug interaction database would provide gen-
lin and developed another life-threatening anaphylaxis eral information and is not a patient-specific evaluation
reaction; this is a medication error, and the anaphylactic (Answer A is incorrect). The Naranjo nomogram inves-
reaction is the harm. Allergy cross-reactivity between tigates the likelihood of an ADE but is not specific to a
penicillin and cefazolin is well documented for those drug interaction (Answer C is incorrect). The RUCAM
with a history of shortness of breath; therefore, this is evaluates the likelihood of drug-induced liver injury
a preventable ADE (Answer D is correct). Although (Answer D is incorrect).
this case of cefazolin-induced life-threatening anaphy-
laxis is an ADE, a preventable ADE best describes this 8. Answer: C
case (Answer C is incorrect). In general, ADRs and side Broad-spectrum antimicrobials should be restricted to
effects are synonymous terms, and a cefazolin-induced the appropriate clinicians with expertise in this area
allergy is an ADR; however, a preventable ADE best (Answer C is correct). Restricting use to intensivists
describes this case (Answers A and B are incorrect). would not be appropriate because non-ICU patients may
require this agent (Answer A is incorrect). Although
4. Answer: D FDA approved only for complicated urinary tract
Prothrombin complex concentrate would be the best infections, restriction to pneumonia would not be appro-
tracer for an ADE associated with apixaban (Answer D is priate (Answer B is incorrect). Cardiac monitoring is not
correct). Coagulation factor VIIa would not be effective required for this medication (Answer D is incorrect).
for this indication (Answer A is incorrect). Phytonadione
would be an appropriate tracer for a warfarin-induced

ACCP Updates in Therapeutics® 2021: Critical Care Pharmacy Preparatory Review and Recertification Course

518
Practice Administration and Development: Pharmacoeconomics and Safe Medication Use

Appendix 1. Adverse Drug Event Reporting Form


ADE#:
ADVERSE EVENT INFORMATION
1. NAME 2. PATIENT ID # 3. LOCATION 4. AGE 5. SEX 6. REACTION ONSET DATE

7. DATE OF REPORT

8. DESCRIBE REACTION AND ITS MANAGEMENT. (Continue on the back if necessary. Use Arial Narrow Font Size 10) 9. Check all appropriate
Patient Expired
Reaction Treated with Drug
Resulted in, or prolonged
inpatient hospitalization
None of the Above
10. Did event abate after stopping
the drug?
YES
NO
MAYBE

11. Was patient’s electronic allergy/


ADE profile updated
YES
NO
(If no, please explain on second page)

12. RELEVANT TESTS/LABORATORY DATA

SUSPECTED DRUG(S) INFORMATION


13. SUSPECTED DRUG(S) Give manufacturer & lot number for vaccine/ biologics/ biotechnological 17. DATES OF ADMINSTRATION

14. DOSE AND FREQUENCY 15. ROUTE OF ADMINISTRATION 18. DURATION OF ADMINISTRATION

16. INDICATION(S) FOR USE

CONCOMITANT DRUG HISTORY


19. CONCOMITANT DRUGS AND DATES OF ADMINISTRATION (Exclude those used to treat the reaction)

20. OTHER RELEVANT HISTORY (e.g., diagnoses, past medical history, allergies, pregnancy, etc.)

INITIAL REPORTER (In confidence)


JCAHO  Standard  PI. 2.20 states that all serious adverse drug reactions are intensely analyzed.  NAME AND ADDRESS OF REPORTER (Including zip code)
Standard MM. 6.20 maintains that the responsible individual complies with internal and external reporting
requirements for adverse drug reactions. (2006 Comprehensive Accreditation Manual for Hospitals)
Please take the time to complete this form for each suspected adverse drug reaction, and forward it to TELEPHONE NO. (Include area code)
the Department of Pharmacy for reporting at the next Adverse Drug Reaction Subcommittee meeting.
Date Time MD notified about possible ADR Pharmacist’s Signature HAVE YOU ALSO REPORTED THIS REACTION TO THE
MANUFACTURER? YES
Submission of a report does not necessarily constitute an admission that the drug caused the reaction NO

ACCP Updates in Therapeutics® 2021: Critical Care Pharmacy Preparatory Review and Recertification Course

519
Practice Administration and Development: Pharmacoeconomics and Safe Medication Use

Appendix 1. Adverse Drug Event Reporting Form (continued)

KJMC ADVERSE DRUG EVENT REPORTING FORM


Modified Naranjo Criteria for Causality (also called ADR Probability Scale)

DON’T KNOW
SCORING
ASSESSMENT YES NO (not conducted,
SYSTEM
not reported)
1.  Are there previous reports (in the literature)
of this reaction? (If no, please provide docu- +1 0 0 Based on the total
mentation of search strategy) score, circle the
term that best
2.  Did the ADR appear after the suspected drug
defines this ADR:
was administered, confirming a temporal +2 -1 0
relationship? (If no, please explain)
≥ 9 Definite
3.  Did the ADR improve when the drug was
discontinued or a specific antagonist was +1 0 0 5–8 Probable
administered? (If no, please explain)
4.  Did the ADR reappear when the drug was 1–4 Possible
+2 -1 0
readministered?
≤ 0 Doubtful
5.  Are there alternative causes (other than the
drug) that could have caused the ADR? (If -1 +2 0
yes, please explain)
6.  Did the ADR reappear when a placebo was
-1 +1 0
given?
7.  Was the drug detected in the blood or other
+1 0 0
fluids in toxic concentrations?
8.  Evaluate the dose-response relationship; was
the ADR more severe when the dose was
+1 0 0
increased or less severe when the dose was
decreased?
9.  Did the patients have a similar reaction to the
same or similar drugs (i.e., same drug class) +1 0 0
in a previous exposure?
10.  Consider signs, symptoms, and laboratory
values other than drug concentrations. Was +1 0 0
the ADR confirmed by objective evidence?

TOTAL SCORE
Source: Naranjo CA, Busto U, Sellers EM, et al. A method for estimating the probability of adverse drug reactions. Clin Pharmacol Ther 1981;30:239-45.

ACCP Updates in Therapeutics® 2021: Critical Care Pharmacy Preparatory Review and Recertification Course

520
Practice Administration and Development: Pharmacoeconomics and Safe Medication Use

Appendix 2. Drug Interaction Probability Scale

The drug interaction probability scale (DIPS) is designed to assess the probability of a causal relationship between
a potential drug interaction and an event. It is patterned after the Naranjo ADR probability scale (Naranjo CA,
Busto U, Sellers EM, et al. A method for estimating the probability of adverse drug reactions. Clin Pharmacol Ther
1981;30:239-45).

Directions
o Circle the appropriate answer for each question and add up the total score.
o Object drug = Drug affected by the interaction.
o Precipitant drug = Drug that causes the interaction.
o Use the Unknown (Unk) or Not Applicable (NA) category if (a) you do not have the information or (b) the question
is not applicable (e.g., no dechallenge; dose not changed).

Questions Yes No NA/Unk


1. Are there previous credible reports of this interaction in humans reported in
the literature? +1 –1 0
2. Is the observed interaction consistent with the known interactive properties
of the precipitant drug? +1 –1 0
3. Is the observed interaction consistent with the known interactive properties
of object drug? +1 –1 0
4. Is the event consistent with the known or reasonable time course of the
interaction (onset or offset)? +1 –1 0
5. Did the interaction remit upon dechallenge of the precipitant drug with no change
in the object drug? +1 –2 0
(if no dechallenge, use Unknown or NA and skip question 6)
6. Did the interaction reappear when the precipitant drug was readministered in the
presence of continued use of object drug? +2 –1 0
7. Are there reasonable alternative causes for the event? a
–1 +1 0
8. Was the object drug detected in the blood or other fluids in concentrations
consistent with the proposed interaction? +1 0 0
9. Was the drug interaction confirmed by any objective evidence consistent with the
effects on the object drug (other than drug concentrations from question 8)? +1 0 0
10. Was the interaction greater when the precipitant drug dose was increased or less
when the precipitant drug dose was decreased? +1 –1 0

a
Consider clinical conditions, other interacting drugs, lack of adherence, risk factors (e.g., age, inappropriate doses of object drug). A NO answer presumes that enough
information was presented so that one would expect any alternative causes to be mentioned. When in doubt, use Unknown or NA designation.

Total Score _____ Highly Probable: > 8


Probable: 5–8
Possible: 2–4
Doubtful: < 2

ACCP Updates in Therapeutics® 2021: Critical Care Pharmacy Preparatory Review and Recertification Course

521
Cardiovascular Critical Care I
Stephanie Dwyer Kaluzna, Pharm.D., BCCP
University of Illinois at Chicago College of Pharmacy
University of Illinois Hospital and Health Sciences System
Chicago, Illinois
Cardiovascular Critical Care I

Cardiovascular Critical Care I


Stephanie Dwyer Kaluzna, Pharm.D., BCCP
University of Illinois at Chicago College of Pharmacy
University of Illinois Hospital and Health Sciences System
Chicago, Illinois

ACCP Updates in Therapeutics® 2022: Critical Care Pharmacy Preparatory Review and Recertification Course

525
Cardiovascular Critical Care I

Learning Objectives Self-Assessment Questions


Answers and explanations to these questions may be
1. Evaluate a patient’s hemodynamic status to develop found at the end of this chapter.
an appropriate pharmacotherapeutic regimen.
2. Design a treatment plan for patients with cardio- Questions 1–8 pertain to the following case.
genic shock. A 58-year-old man (height 71 inches, weight 106 kg)
3. Develop treatment plans for critically ill patients was transferred to the intensive care unit (ICU) from an
with cardiovascular diseases, including, but not lim- outlying hospital after 24 hours of progressively worsen-
ited to, coronary artery disease, arrhythmias, heart ing chest pain and shortness of breath. He arrives on 6
failure (HF), and disease. L of oxygen by high-flow nasal cannula, with a heparin
4. Recognize the utility and application of mechani- infusion at 16 units/kg/hour and dopamine infusion at
cal circulatory support and heart transplantation for 15 mcg/kg/minute. Notes show that he was bradycardic
patients with advanced HF and cardiogenic shock. (heart rate of 50–58 beats/minute) and hypotensive
(78/49–86/55 mm Hg) on presentation to the outlying
hospital. His 12-lead electrocardiogram (ECG) at the
Abbreviations in This Chapter outlying hospital showed ST-segment elevation in leads
II, III, and aVF. Given his chest pain and ECG findings,
ABG Arterial blood gas aspirin 324 mg (chewed and swallowed), clopidogrel 600
ACE Angiotensin-converting enzyme mg, and morphine 4 mg intravenously once were admin-
ACS Acute coronary syndrome(s) istered before transfer. Notes also show that β-blockers
AF Atrial fibrillation and nitroglycerin were held because of bradycardia and
ARB Angiotensin receptor blocker hypotension.
AV Atrioventricular His past medical history is significant for nonadher-
BNP Brain natriuretic peptide ence, hypertension, diabetes mellitus, dyslipidemia, and
CABG Coronary artery bypass grafting heart failure with preserved ejection fraction (HFpEF),
CAD Coronary artery disease with a last-reported left ventricular ejection fraction
CVP Central venous pressure (LVEF) of 65% 1 year ago. The patient reports that he
ECG Electrocardiography/electrocardiogram currently takes no medications at home.
ECHO Echocardiography/echocardiogram • Vital signs on transfer: blood pressure 87/52 mm
ECMO Extracorporeal membrane oxygenation Hg, heart rate 110 beats/minute, respiratory rate 19
HF Heart failure breaths/minute, temperature 99.7°F (37.6°C)
HOCM Hypertrophic obstructive cardiomyopathy • The team’s physical assessment indicates ongoing
ICU Intensive care unit distress, radiating chest pain 7/10, evidence
LV Left ventricle/ventricular of rales, absence of any cardiac murmurs, and
LVAD Left ventricular assist device presence of a right radial arterial line.
LVEF Left ventricular ejection fraction • A preexisting urinary catheter with 20 mL of urine
MAP Mean arterial pressure in the reservoir is also present (the patient reports
MCS Mechanical circulatory support that his most recent void was yesterday morning).
MI Myocardial infarction According to outlying hospital records, only 30 mL
NSTEMI Non–ST-segment elevation myocardial of urine was reported before time of transfer.
infarction • His chest radiography reveals evidence of diffuse
PCI Percutaneous coronary intervention patchy opacities; however, the report indicates that
PVR Pulmonary vascular resistance an infiltrate cannot be ruled out.
RV Right ventricle/ventricular • His serum chemistry panel results are as follows:
SA Sinoatrial sodium 132 mEq/L, potassium 4.2 mEq/L, chloride
STEMI ST-segment elevation myocardial infarction 102 mEq/L, carbon dioxide 22 mEq/L, blood urea
SVR Systemic vascular resistance nitrogen (BUN) 34 mg/dL, serum creatinine (SCr)
VAD Ventricular assist device 1.9 mg/dL, and glucose 163 mg/dL.
VT Ventricular tachycardia

ACCP Updates in Therapeutics® 2022: Critical Care Pharmacy Preparatory Review and Recertification Course

526
Cardiovascular Critical Care I

• Results of the complete blood cell count (CBC) are 4. The patient has been taken to the cardiac cath-
as follows: white blood cell count (WBC) 11.3 × eterization laboratory, and a “code blue” is called
103 cells/mm3, hemoglobin 10.9 g/dL, hematocrit overhead for immediate emergency response to this
31.1%, and platelet count 213,000/mm3. patient’s procedural area. On arrival, chest com-
• Additional laboratory values include troponin-T pressions have just been paused for defibrillation,
3.9 ng/mL, aspartate aminotransferase (AST) 14 and a single dose of epinephrine has been admin-
IU/L, alanine aminotransferase (ALT) 46 IU/L, istered. The interventional team indicates that,
hemoglobin A1C (A1C) 8.3%, and brain natriuretic when attempting visualization of the right coronary
peptide (BNP) of 1423 pg/mL. artery, the patient went into ventricular tachycardia
(VT). The patient’s telemetry monitor now shows
1. Which is the most likely cause of this patient’s sinus tachycardia with noted ectopy—heart rate
admission and transfer? 113 beats/minute and blood pressure 84/52 mm Hg.
A. Septic shock caused by suspected pneumonia The cardiologist asks for recommendations for an
with subsequent myocardial depression and antiarrhythmic because he is concerned about a VT
demand ischemia. recurrence, given the bigeminy on telemetry. Which
B. Cardiogenic shock caused by acute on chronic agent would be best to recommend?
decompensated systolic heart failure (HF). A. Lidocaine 100 mg intravenous push for 2–3
C. Cardiogenic shock caused by a suspected minutes, followed by an infusion at 1 mg/
inferior ST-segment elevation myocardial minute.
infarction (STEMI). B. Amiodarone 300 mg intravenous push for less
D. Cardiogenic shock caused by a suspected non– than 1 minute.
ST-segment elevation myocardial infarction C. Metoprolol 10 mg intravenous push for 1–2
(NSTEMI) affecting the lateral wall. minutes.
D. Diltiazem 20 mg intravenous push for 2 min-
2. Given this patient’s presentation, which coronary utes, followed by a continuous infusion at 5
artery is most likely to be the culprit lesion? mg/hour and titrated to maintain a heart rate
A. Left main coronary artery. less than 110 beats/minute.
B. Left anterior descending artery.
C. Left circumflex coronary artery. 5. The patient returns to the ICU after his left heart
D. Right coronary artery. catheterization, which was performed through the
femoral artery. In addition to his acute decompen-
3. The interventional cardiologist who is evaluating sation, which major procedural complication is of
the patient for potential revascularization asks the greatest concern during the next 12 hours?
ICU team to place a central venous catheter. Which A. Bleeding (particularly retroperitoneal bleeding).
changes/interventions regarding this patient’s hemo- B. Dissection/rupture.
dynamic support would be best to recommend? C. Stent thrombosis.
A. Increase dopamine to achieve a mean arterial D. Papillary muscle rupture.
pressure (MAP) greater than 65 mm Hg.
B. Convert the patient to norepinephrine, and
titrate the dose to achieve a MAP greater than
65 mm Hg while weaning off dopamine.
C. Initiate milrinone at 0.375 mcg/kg/minute, and
continue dopamine at 15 mcg/kg/minute.
D. Administer 1000 mL of normal saline as a bolus
because of low urine output.

ACCP Updates in Therapeutics® 2022: Critical Care Pharmacy Preparatory Review and Recertification Course

527
Cardiovascular Critical Care I

6. The patient returns to the ICU with a pulmonary 8. Which medication-related quality metric would not
artery catheter in place and is currently receiving require documentation of contraindications based
dopamine at 12 mcg/kg/minute and norepinephrine on this patient’s clinical presentation (acute MI with
at 0.08 mcg/kg/minute with heart rate 108 beats/ preserved LVEF)?
minute, blood pressure 82/51 mm Hg, cardiac A. Aspirin contraindication.
index 2.0, central venous pressure (CVP) 26 mm B. Statin contraindication.
Hg, and pulmonary artery pressure 49/21 mm Hg. C. β-Blocker contraindication.
The physician is concerned about right ventricular D. Angiotensin-converting enzyme (ACE)
dysfunction in the setting of shock and approaches inhibitor/angiotensin receptor blocker (ARB)
you for a recommendation to increase blood pres- contraindication.
sure (BP). Ideally, the physician would prefer to
wean dopamine and minimize further increases in
pulmonary vascular resistance (PVR) because of the
presence of pulmonary hypertension. Which strat-
egy would be best to recommend?
A. Keep the current infusions, and reevaluate later.
B. Initiate phenylephrine at 1 mcg/kg/minute, and
wean the dopamine off if MAP is greater than
65 mm Hg.
C. Initiate vasopressin at 0.04 unit/minute, and
wean the dopamine off if MAP is greater than
65 mm Hg.
D. Administer a 1-L bolus of normal saline, and
wean the dopamine off if MAP is greater than
65 mm Hg.

7. Hours later, this patient goes into atrial fibrillation


(AF) with a heart rate of 126 beats/minute; how-
ever, the patient’s blood pressure remains 86/56 mm
Hg according to the regimen selected in the previ-
ous question. Which agent would you most likely
administer to manage the patient’s AF?
A. Amiodarone 150 mg intravenous push, fol-
lowed by a continuous infusion at 1 mg/minute.
B. Amiodarone 150 mg intravenous infusion for
10 minutes, followed by a continuous infusion
at 1 mg/minute.
C. Metoprolol 5 mg intravenous push once,
followed by 5 mg intravenous push every
6 hours.
D. Diltiazem 20 mg intravenous push, followed by
a continuous infusion at 5 mg/hour and titrated
to maintain a heart rate of less than 110 beats/
minute.

ACCP Updates in Therapeutics® 2022: Critical Care Pharmacy Preparatory Review and Recertification Course

528
Cardiovascular Critical Care I

BPS Critical Care Pharmacy Specialist Examination Content Outline

This chapter covers the following sections of the Critical Care Pharmacy Specialist Examination Content Outline:
1. Domain 1: Clinical Knowledge and Application
a. Task 1: 1–3
b. Task 2: 1, 3–5, 7
c. Task 3: 1–6
d. Task 4: 1–4, 5, 7, 8
e. Task 5: 1, 6, 7
f. Task 6: 1–6
g. Task 7: 1–7
h. Task 8: 1
2. Domain 2: Practice Management, Policy, and Quality Improvement
a. Task 1: 2
b. Task 3: 1–3
c. Task 5: 1
d. Task 6: 2–4
3. Domain 3: Evidence-Based Medicine, Scholarship, Education, and Professional Development
a. Task 1: 3
b. Task 3: 1

ACCP Updates in Therapeutics® 2022: Critical Care Pharmacy Preparatory Review and Recertification Course

529
Cardiovascular Critical Care I

I.  CARDIOVASCULAR FUNDAMENTALS OVERVIEW

A. The myocardium has five functions and distinctive properties:


1. Chronotropy: Ability to generate an electrical impulse at an intrinsic rate
2. Dromotropy: The speed and ability to facilitate electrical impulse conduction
3. Inotropy: Ability to contract in relation to a given preload, afterload, and heart rate
4. Bathmotropy: Demonstration of an intrinsic excitatory threshold
5. Lusitropy: Relaxation of the myocardium, independent from termination of contraction
6. Other related terms:
a. Preload: Volume of blood (represented in most cases by a pressure) in a ventricular cavity at the end
of diastole imparting stretch on a resting myocardial sarcomere
b. Afterload: The pressure that a ventricle must overcome to generate cardiac output. The greater the
afterload (vascular resistance or impedance), the greater the amount of energy and force required
to enable ejection of blood from a ventricle.

B. Coronary Artery Circulation


1. Myocardial perfusion occurs through the coronary arteries during diastole.
2. Coronary artery anatomy and perfusion are not the same in everyone.
3. ECG abnormalities, hemodynamic assessment, and patient symptoms may assist with coronary artery
disease (CAD) localization.
4. Circulatory dominance:
a. Right dominant: Posterior descending artery and atrioventricular (AV) nodal artery arise from the
right coronary artery (85% of the population).
b. Left dominant: Posterior descending artery arises from the circumflex artery (8% of the population).
c. Codominant: Posterior descending artery arises from branches of the circumflex and the right
coronary artery (7% of the population).
d. Other notable variations: The sinoatrial (SA) node may have variation in the vessels that supply it;
it is most commonly perfused by the right coronary artery (about 70%), circumflex (about 25%), or
right coronary artery and circumflex (about 5%).

Developed by Erik Abel, PharmD, BCPS

Figure 1. Coronary artery circulation.

ACCP Updates in Therapeutics® 2022: Critical Care Pharmacy Preparatory Review and Recertification Course

530
Cardiovascular Critical Care I

C. Anatomy in Relation to the ECG


1. A single lead of an ECG tracing is a summative representation of the action potentials occurring from
a single cell, facilitating myocardial conduction, contraction, and relaxation.
2. A 12-lead ECG can provide a geographic representation of conduction within the myocardial tissue.
Conduction abnormalities within specified leads may indicate perfusion defects in some clinical
scenarios (i.e., acute coronary syndromes [ACS]).

Phase Electrolyte movement Conduction change


0 Na+ influx into cell Depolarization
1 K+ and Cl- out of cell Repolarization
2 Ca2+ into cell and K+ out of cell Plateau
3 K+ out of cell Repolarization
4 Resting Membrane Potential

ECG point Representation


P Sinoatrial (SA) node impulse generation/atrial depolarization
Duration for conduction to occur from the SA node through the AV
PR interval
node, bundle of His, bundle branches and Purkinje fibers
QRS Concurrent atrial repolarization and ventricular depolarization
ST segment Ventricular refractory period
T Ventricular repolarization
QT interval Duration for ventricular repolarization
RR interval Duration between ventricular depolarizations (represents heart rate)
Further ventricular depolarizations not listed or commonly
U visualized; may be more prominent in hypothermia, bradycardia, or
severe electrolyte depletions

12 — Lead Electrocardiogram (ECG)


Limb leads Precordial Leads
I Anterolateral (Circumflex) aVR V1 Anteroseptal (LAD) V4 Anteroapical (Distal LAD)
Anteroseptal (LAD)
II Inferior (RCA) aVL Anterolateral (Circumflex) V2 V5 Anterolateral (Circumflex)
Posterior (RCA)
Anteroapical (Distal LAD)
III Inferior (RCA) aVF Inferior (RCA) V3 V6 Anterolateral (Circumflex)
Posterior (RCA)
adapted from ACLS instructor book
Figure 2. Anatomy in relation to the ECG.

II.  HEMODYNAMIC MANAGEMENT AND THE HEART

A. Hemodynamic Assessment Relies on the Clinician’s Ability to:


1. Understand cardiovascular circulation (Figure 3) and pathophysiology contributing to the hemodynamic
variables in isolation and in the context of other changes and conditions.
2. Account for hemodynamic trends in view of end-organ function and use surrogates of oxygen delivery.
3. Understand the roles of hemodynamic tools, limitations in use, and interpretation of the devices/
technology.
4. Identify appropriate therapeutic targets (Figure 4), and apply the pharmacologic/pharmacodynamic
principles (Table 1) to initiate, modify, or discontinue therapy depending on clinical response.

ACCP Updates in Therapeutics® 2022: Critical Care Pharmacy Preparatory Review and Recertification Course

531
Cardiovascular Critical Care I

B. Advantages and Disadvantages of Invasive Hemodynamic Monitoring for Assessing Cardiac Output and/
or Volume Status
1. Hemodynamic monitoring techniques facilitate diagnosis
2. Therapy guided by clinical assessment and a pulmonary artery (PA) catheter compared with
clinical assessment alone for heart failure (HF) did not result in differences in overall mortality or
hospitalizations. However, more anticipated adverse events (i.e., infection, bleeding) were seen with PA
catheter-guided care (JAMA 2005;294:1625-33).
3. Diligent use and skilled interpretation of these technologies, together with patient assessment, can be
used to guide therapeutic interventions.
4. See comparisons of invasive and minimally invasive hemodynamic monitoring devices (Table 2 in the
Shock Syndromes I chapter).

C. Pharmacologic Support in Cardiovascular Critical Illness (see Table 1)

RA = Right Atrium Left Atrium = LA


RV = Right Ventricle Left Ventricle = LV
TV = Tricuspid Valve Mitral Valve = MV
PV = Pulmonic Valve Aortic Valve = AV
Developed by Erik Abel, PharmD, BCPS

Figure 3. Cardiovascular systemic circulation.

ACCP Updates in Therapeutics® 2022: Critical Care Pharmacy Preparatory Review and Recertification Course

532
Cardiovascular Critical Care I

Based Upon
Metabolic need of
disease state
Hemoglobin
Oxygen
Consumption
VO2

Afterload

α1 b2
b1
b1

Developed by Erik Abel, PharmD, BCPS

Figure 4. Integrated model of the hemodynamic variables and therapeutic targets.

Table 1. Pharmacologic Support in Cardiovascular Critical Illness


Other
Vasopressors Dopa α1 β1 β2 HR CVP CO SVR PVR
Mechanism
Dopaminea +++++ +++ ++++ ++ ↑ ↔ ­↑ ↔ or ­↑ ↔ or ­↑
Epinephrine ++++ ++++ +++ ↑­ ↔ or ­­↑ ­↑ ­↑ ­↑
Norepinephrine +++++ +++ ++ ↑­ ­↑ ↑­ or ↓ ­↑ ↑
Phenylephrine +++++ ↔ or ↓ ­↑ ↔ or ↓ ­↑ ­↑
N/A V1 and V2
Vasopressin ↔ or ↓ ­↑ ↔ or ↓ ↑­ ↔ or ↓
“α1-like effects” agonism
Angiotensin
N/A II agonism
Angiotensin II ↔ or ↓ ­↑ ↔ or ↓ ↑­ ↔ or ↓
“α1-like effects” of the AT1
receptor
Inotropes
Dobutamine + ++++ ++ ­↑ ↔ or ↓ ­↑ ↔ or ↓ ↔ or ↓
Isoproterenol +++++ +++++ ­↑ ↔ or ↓ ­↑ ↔ or ↓ ↔ or ↓
N/A PDE3
Milrinoneb ↑­ ↓ ↑­ ↓ ↓
“Β1 and β2-like effects” inhibition

ACCP Updates in Therapeutics® 2022: Critical Care Pharmacy Preparatory Review and Recertification Course

533
Cardiovascular Critical Care I

Table 1. Pharmacologic Support in Cardiovascular Critical Illness (continued)


Other
Vasopressors Dopa α1 β1 β2 HR CVP CO SVR PVR
Mechanism
Vasodilators
Nitroglycerin cGMP ↔ or ­↑ ↓ ↔ or ­↑ ↔ or ↓ ↔ or ↓
Nitroprusside cGMP ↔ or ­↑ ↓ ↔ or ­↑ ↓ ↓
Nitric oxide
cGMP N/A ↔ ↔ or ­↑ ↔ ↓
(inhaled)
Epoprostenol
cAMP N/A ↔ ↔ or ­↑ ↔ ↓
(inhaled)
a
High doses associated with increasing α1 activity.
b
Normal half-life is 2.5 hours, but drug is eliminated renally. Loading dose rarely used in routine management because of hypotension.
AMP = cyclic adenosine monophosphate; AT1 = angiotensin II type 1; cGMP = cyclic guanosine monophosphate; CVP = central venous pressure; N/A = not applicable;
PDE3 = phosphodiesterase type 3; PVR = pulmonary vascular resistance; SVR = systemic vascular resistance. Information from: Br J Pharmacol 2012;165:2015-33; J
Am Coll Cardiol 2014;63:2069-78; Pathophysiol Heart Dis 2011:xiv; Circulation 2008;118:1047-56; Crit Care Med 2014:xix; J Pharm Pract 2011;24:44-60.

III.  CARDIOGENIC SHOCK

A. Characterized by three hallmarks:


1. Sustained hypotension unresponsive to fluid administration alone (systolic blood pressure [SBP] less
than 90 mm Hg for at least 30 minutes) (Cardiol Clin 2013;4:567-80)
2. Evidence of myocardial dysfunction with reduced cardiac index (less than 2.2 L/minute/m2)
3. Signs and symptoms of malperfusion in the setting of volume overload and elevated cardiac filling
pressures (e.g., pulmonary capillary wedge pressure [PCWP] greater than 18 mm Hg)

B. Society for Cardiovascular Angiography and Interventions (SCAI) Classifications for Cardiogenic Shock
(Catheter Cardiovasc Interv 2019;94:29-37):
1. Stage A – at risk. These patients are not in shock but are at risk of future development.
2. Stage B – beginning. These patients have clinical evidence of relative hypotension or tachycardia without
hypoperfusion.
3. Stage C – classic. These patients have evidence of hypotension and hypoperfusion requiring intervention,
including vasoactive therapies and mechanical circulatory support.
4. Stage D – deteriorating. These patients are those in stage C who are getting worse despite attempts to
restore optimal hemodynamics.
5. Stage E – extremis. These patients are experiencing cardiac arrest with ongoing cardiopulmonary
resuscitation or extracorporeal membrane oxygenation (ECMO).

C. Signs and Symptoms: See Heart Failure section.

D. Epidemiology: Without appropriate diagnosis and management, in-hospital mortality rates as high as 60%
have been described (Semin Respir Crit Care Med 2011;32:598-606).

ACCP Updates in Therapeutics® 2022: Critical Care Pharmacy Preparatory Review and Recertification Course

534
Cardiovascular Critical Care I

100 - Post-operative cardiac


stunning (n=41)
Congestive HF (n=56)
80 -
Pulmonary oedema (n=27)
Cardiogenic shock (n=62)

Survival (%)
60 -

40 -

20 -

0 - ı ı ı ı ı ı
30 60 90 120 150 180
Time (days)

Figure 5. Survival rates of ICU patients over time presenting with different acute heart failure syndromes (Crit Care
2010;14:201).

E. Etiology
1. Usually caused by left ventricular (LV) failure secondary to an acute MI, but a list of other potential
causes can be found in Box 1
2. May be multifactorial, including one or more of the causative etiologies; however, can also coexist with
other types of shock syndromes

ACCP Updates in Therapeutics® 2022: Critical Care Pharmacy Preparatory Review and Recertification Course

535
Cardiovascular Critical Care I

Box 1. Potential Causes of Cardiogenic Shock (Critical Care Medicine 2014:xix; Cardiol Clin 2013;31:567-80, viii;
Cardiol Clin 2013;31:519-31, vii-viii; Semin Respir Crit Care Med 2011;32:598-606)

LV Failure
• Large MI
• Small MI with preexisting systolic HF
• Reinfarction
• Septic shock with severe myocardial depression
RV Failure
• RV infarction
• End-stage pulmonary hypertension
Acute Mechanical Dysfunction
• Papillary muscle rupture or chordal rupture with subsequent severe mitral regurgitation
• Free-wall rupture
• Ventricular septal rupture
• Cardiac tamponade
Cardiomyopathy
• End-stage HF
• Myocarditis
• Peripartum cardiomyopathy
• Left ventricular outflow tract obstruction
• Stress-induced cardiomyopathy (i.e., Takotsubo)
Valvular Disease
• Acute aortic regurgitation
• Ischemic mitral regurgitation
• Aortic or mitral stenosis with tachyarrhythmia or other condition causing decompensation
• Infectious endocarditis
• Prosthetic valve dysfunction or thrombosis
Arrhythmias
Other Conditions
• Prolonged cardiopulmonary bypass and/or coronary air embolus
• Cardiac trauma (blunt or penetrating)
• Heart transplant rejection
• Pulmonary embolism
• Medical nonadherence
HF = heart failure; LV = left ventricular; MI = myocardial infarction; RV = right ventricular.

F. Resuscitation/Treatment
1. Treatment largely depends on managing underlying chronic or acute cardiovascular disease(s) outlined
in Box 1, with consideration given to chronicity of clinical changes before onset of shock.
2. Means of management will be primarily discussed under the Major Contributing Etiologies headings
in sections IV–VII.
3. Hemodynamic management and pharmacotherapeutic considerations
a. Because cardiogenic shock (CS) may have different underlying, contributing etiologies,
hemodynamic management requires careful interpretation of clinical values. Treatment strategies
could be devised according to the algorithm in Figure 6.
b. DOREMI trial (N Engl J Med 2021;385:516-25)

ACCP Updates in Therapeutics® 2022: Critical Care Pharmacy Preparatory Review and Recertification Course

536
Cardiovascular Critical Care I

i. First prospective trial studying milrinone and dobutamine for CS management (n=192) during
the index hospitalization
(a) Median ejection fraction was 25%, and around two-thirds of patients had ischemic
cardiomyopathy. Most patients (90%) fell into SCAI CS classes C (classic) and D
(deteriorating).
(b) Milrinone or dobutamine was initiated at a dose determined by a standard dosing scale
ranging from 1 to 5, corresponding to 2.5, 5, 7.5, 10, and greater than 10 mcg/kg/minute
for dobutamine and 0.125, 0.25, 0.5, and greater than 0.5 mcg/kg/minute for milrinone.
Inotropes were adjusted in a blinded fashion.
ii. Composite primary outcome (in-hospital mortality from any cause, resuscitated cardiac arrest,
receipt of cardiac transplantation, or mechanical circulatory support, nonfatal MI, transient
ischemic attack or stroke, or initiation of renal replacement therapy) did not differ between the
two groups. Of importance, this study highlights the persisting high mortality rates in patients
with CS (around 40%).
iii. Both dobutamine and milrinone had high rates of arrhythmias leading to intervention, and there
was no difference in sustained hypotension or vasopressor requirements for either medication.
c. Sepsis Occurrence in Acutely Ill Patients (SOAP) II landmark trial (N Engl J Med 2010;362:779-89)
i. International multicenter trial (n=1679) that compared the 28-day mortality of norepinephrine
with that of dopamine in the management of shock
(a) Included if signs of malperfusion and MAP less than 70 mm Hg or SBP less than 100
mm Hg, despite adequate fluid challenge (at least 1000 mL of crystalloids or 500 mL of
colloids unless the CVP was greater than 12 mm Hg or the pulmonary artery occlusion
pressure was greater than 14 mm Hg)
(b) Allowed titration to maximums of norepinephrine 0.19 mcg/kg/minute versus dopamine
20 mcg/kg/minute, after which open-label norepinephrine was allowed (open-label
norepinephrine doses did not exceed 1.1 mcg/kg/minute during the study period)
(c) Epinephrine and vasopressin were permitted as rescue agents (similar use in both groups).
(d) Dobutamine use was greater in the norepinephrine group (19.4% vs. 14.8%).
ii. Showed that dopamine was associated with a significantly higher mortality rate than was
norepinephrine with or without dobutamine in patients with CS in subgroup analysis
iii. Dopamine was associated with more adverse events than norepinephrine in the management
of all shock subtypes, predominantly driven by the incidence of arrhythmias (24.1% vs. 12.1%,
p<0.001). Tachyarrhythmias with dopamine had the greatest incidence within the first 36
hours after randomization.
d. Study Comparing the Efficacy and Tolerability of Epinephrine and Norepinephrine in CS
[OptimaCC] (J Am Coll Cardiol 2018;72:173-82)
i. Prospective, double-blind, multicenter, randomized study (n=57) that compared cardiac index
evolution and refractory CS between epinephrine and norepinephrine in patients with CS post-
MI
ii. Trial showed that cardiac index evolution did not differ between groups, though the rate of
refractory CS was significantly higher in the epinephrine arm than in the norepinephrine arms
(37% vs. 7%).
iii. Tachycardia and lactic acidosis were also significantly increased in the epinephrine group.

ACCP Updates in Therapeutics® 2022: Critical Care Pharmacy Preparatory Review and Recertification Course

537
Cardiovascular Critical Care I

Preload Afterload Contractility

↓ CVP or PCWP ↑ CVP


↑ PAP +/- ↑ HR ↓ BP/↑ HR ↓ CO/CI
• Milrinone • Phenylepherine Normotensive/Hypertensive
• Nitroglycerin • Vasopressin • Milrinone
• 500 mL of ↑ PCWP • Nitroprusside • Dobutamine
crystalloid/ • Inhaled ↓ BP
colloid vasodilators • Norepinephrine ↓ BP/unresponsive
• Blood products • Dopamine • Epinephrine
• Passive leg
↑ BP +/- ↑ HR ↓ HR (symptomatic +/- ↓ BP)
raising
• Nitroprusside • Epinephrine
• Nitroglycerin • Dobutamine
• Loop +/- thiazide • Other IV • Isoproterenol
diuretic antihypertensive • Dopamine
• Other means of
fluid removal
Adapted from: Blais, DM. PSAP- IV: Critical and urgent care. 2010

Figure 6. Shock management and treatment considerations based on hemodynamic indices.

Patient Case

Questions 1–8 pertain to the following case.


J.M. is a 68-year-old man with a history of CAD including STEMI 6 months ago with placement of two drug-
eluting stents, type 2 diabetes, hypertension, dyslipidemia, gastroesophageal reflux disease, obstructive sleep
apnea, frequent epistaxis, ischemic cardiomyopathy (LVEF 30%–35%), and moderate to severe mitral regurgi-
tation. He is admitted to the cardiac intensive care unit for severe shortness of breath and altered mental status,
and he is currently on continuous positive airway pressure (CPAP). He has gained 13 kg during the past 2 weeks
(now weighs 121 kg) and has had decreased urine output, despite having had his diuretic dose increased. Home
medications include the following: aspirin 81 mg once daily, ticagrelor 90 mg every 12 hours, pantoprazole 40
mg once daily, atorvastatin 40 mg once daily, insulin glargine 10 units every night, metformin 500 mg every 12
hours, furosemide 40 mg twice daily, and potassium chloride 20 mEq twice daily. His wife states that he was
taking clopidogrel until 1 month ago, at which time he was given ticagrelor samples from his primary care phy-
sician because of cost concerns. A physical examination reveals rales throughout his lung fields. He is afebrile,
anxious, and alert.

1. Given J.M.’s comorbidities, some drugs or drug classes have proven mortality benefits. Which group of
added or modified medications not on the patient’s home medication profile might best help further slow the
progression of his disease and confer mortality benefits?
A. Amlodipine, clopidogrel, and sitagliptin
B. Spironolactone, lisinopril, and carvedilol
C. Pravastatin, amlodipine, and aspirin 325 mg
D. Prasugrel, sildenafil, and atenolol

2. Given the patient’s presentation, which group of diagnostic tests would be most helpful to guide your recom-
mendations to the team for J.M.’s current management? (Assume that a basic metabolic panel [Chem 7], a
pulse oximetry, and a capillary blood glucose have already been completed.)
A. Urine culture, respiratory culture, lactate, and procalcitonin
B. Chest radiography, arterial blood gas (ABG), liver function tests, and serial troponins
C. Chest radiography, echocardiogram (ECHO), lactate, and BNP
D. Chest radiography, arterial line, ABG, and lactate

ACCP Updates in Therapeutics® 2022: Critical Care Pharmacy Preparatory Review and Recertification Course

538
Cardiovascular Critical Care I

Patient Case (contined)

3. All tests previously mentioned have been ordered, and the following results are available. J.M.’s blood
pressure is 90/56 mm Hg (MAP 67 mm Hg), and his heart rate is 56 beats/minute. A 12-lead ECG showed
normal sinus rhythm without evidence of acute ST-T changes.
• Chest radiography reveals diffuse patchy opacities; however, infiltrate cannot be ruled out; lines are all in
appropriate positions.
• Serum chemistry panel results are as follows: sodium 126 mEq/L, potassium 4.8 mEq/L, chloride 102
mEq/L, carbon dioxide 21 mEq/L, BUN 32 mg/dL, SCr 1.6 mg/dL, and glucose 134 mg/dL.
• Results of the CBC are as follows: WBC 9.8 × 103 cells/mm3, hemoglobin 11.1 g/dL, hematocrit 32.6%,
and platelet count 173,000/mm3.
• Additional laboratory values include the following: troponin 0.9 ng/mL, AST 114 IU/L, ALT 102 IU/L,
and BNP 1936 pg/mL.
• Invasive hemodynamic variables include CVP 28 mm Hg, pulmonary artery pressures 46/22 mm Hg,
cardiac index 1.8 L/minute/m2, and central venous oxygen saturation (Scvo2) 53%; pulmonary artery
occlusion pressure is not yet available.
• ABG results are as follows: pH 7.36, partial pressure of oxygen (Po2) 93.7, partial pressure of carbon
dioxide (Pco2) 43.2, bicarbonate 23.9, oxygen (O2) saturation 89%, and lactate 6.9.
• ECHO results are pending.

The patient’s physical examination reveals that his extremities are cold to the touch, and his capillary refill
is poor. The team has ordered furosemide 80 mg intravenously once and would like to initiate a vasopressor
or inotrope for this patient. Which would be best to recommend at this time?
A. Norepinephrine 0.08 mcg/kg/minute
B. Epinephrine 0.08 mcg/kg/minute
C. Milrinone 0.25 mcg/kg/minute
D. Dobutamine 5 mcg/kg/minute

IV.  ACUTE CORONARY SYNDROMES

A. Pathophysiology
1. Manifestation of prolonged cessation of oxygenated blood supply to a portion of the myocardium that is
most commonly caused by an acute thrombus at the site of coronary atherosclerotic stenosis leading to
local or regional myocardial ischemia and necrosis
2. Other disease states leading to an elevated myocardial oxygen demand with a concurrent inability to
meet such demands may result in a scenario where “demand ischemia” is considered versus a diagnosis
of ACS.

B. Presentation and Diagnosis (Circulation 2014;130:e344-426; Circulation 2013;127:e362-425) – Although


other cardiac enzymes assays are available for clinical use, cardiac troponins are usually used as sensitive
markers indicative of myocardial necrosis, with a negative troponin conferring a greater than 95% negative
predictive value for MI. Troponin is eliminated renally, so patients with end-stage renal disease often have
elevations in cardiac troponin without other evidence of myocardial ischemia. Therefore, other clinical signs
and symptoms of acute coronary syndromes as well as the rate of troponin increase should be assessed.

ACCP Updates in Therapeutics® 2022: Critical Care Pharmacy Preparatory Review and Recertification Course

539
Cardiovascular Critical Care I

Clinical suspicion of ACS based upon signs and symptoms


• Non-traumatic origin of chest pain/discomfort radiating to neck, jaws
or shoulders; or anginal equivalents of persistent shortness of breath,
nausea/vomiting, indigestion, or new weakness/malaise
• Higher suspicion should be given to those with a history of CAD, MI,
CABG or PCI
• Women, elderly, and those with diabetes tend to present with atypical
symptoms

12-Lead ECG (Negative) 12-Lead ECG (Positive)


• Minimal change up to ischemic changes that may include • ST-segment elevation in two or more
T-wave inversion or ST-segment depression contiguous leads
• New Left Bundle Branch Block

Cardiac Enzymes Cardiac Enzymes Cardiac Enzymes


Negative Positive Positive

Unstable Angina NSTEMI STEMI

Acute Myocardial Infarctions


Figure 7. Presentation and diagnosis.

Box 2. Causes of Troponin Elevation (Heart 2006;92:987-93)


Acute decompensated heart failure Early post-cardiac surgery
Acute MI Heart transplantation
Acute pulmonary embolism Myocarditis
Aortic stenosis Pericarditis
Cardiac amyloidosis Post-PCI
Cardiotoxic chemotherapy Rhabdomyolysis
Chest compressions Sepsis
Chest wall trauma or compressions Severe strenuous exercise
Chronic heart failure Tachyarrhythmia
Direct current cardioversion/defibrillation Type A aortic dissection
PCI = percutaneous coronary intervention.

ACCP Updates in Therapeutics® 2022: Critical Care Pharmacy Preparatory Review and Recertification Course

540
Cardiovascular Critical Care I

C. Acute Management of MI

Table 2. Acute Management (Circulation 2014;130:e344-426; Circulation 2013;127:e362-425)


STEMI NSTEMI/ Unstable Angina
Goals of care Reperfusion therapy as soon as possible Prevent total occlusion of the vessel
• Primary PCI preferred if it can be performed within 90 • Decision and need for
min of medical contact revascularization (PCI or surgery)
• If primary PCI is unavailable within 90 min of medical vs. medical management should
contact, fibrinolytics should be administered within 30 be made on the basis of risk
min of presentation unless contraindications exist stratification, symptom resolution,
• Surgical revascularization may be indicated, depending and indicators of ongoing
on severity of CAD, complexity of anatomy, or myocardial damage/ischemia
development of other complications
CAD = coronary artery disease.

Table 3. Predominant Interventions on Presentation/Onset for Stabilization – Any ACS (Circulation 2014;130:e344-
426; Circulation 2013;127:e362-425)
Predominant interventions on presentation/onset for stabilization – Any ACS
• Provides analgesia, and decreases pain-induced sympathetic/adrenergic tone
• Morphine commonly used because it may also induce vasodilation and mediate some
degree of preload reduction
• Morphine is also an independent predictor of high residual platelet reactivity in ticagrelor/
Morphine or other prasugrel use (may impair absorption because of delayed gastric emptying) (Am Heart J
narcotic analgesic 2014;6:909-14)
• Morphine now carries a class IIb-B recommendation and may be not be favored more than
other narcotic analgesics, given that at least two large trials have identified an association
between morphine administration and risk of death (N Engl J Med 2014;371:1016-27; Am
Heart J 2005;149:1043-9; J Am Coll Cardio. 2014;64:e139-228)
• Can help attenuate anginal pain secondary to tissue hypoxia
• Current evidence shows no benefit for supplemental oxygen (and potential harm) in patients
Oxygen with normal Sao2 (N Engl J Med 2017;13:1240-9; Circulation 2015;24:2143-50)
• Supplemental oxygen considered if Sao2 < 90%, respiratory distress, or other high-risk
features for hypoxemia
• Can facilitate coronary vasodilation and may also be helpful in scenarios of severe
cardiogenic pulmonary edema caused by increased venous capacitance
• Nitroglycerin (NTG) 0.4 mg sublingually every 5 minutes for total of THREE doses;
afterward, assess need for IV NTG
• IV NTG is indicated in the first 48 hr after onset of ischemia, HF, or hypertension but
should not preclude therapy such as β-blockers or ACE inhibitors when indicated
Nitrates
Should NOT be administered:
• If SBP < 90 mm Hg OR if SBP is > 30 mm Hg below baseline
• If severe bradycardia (including heart block) with HR ≤ 50 beats/min
• If tachycardia (HR ≥ 100 beats/min) in the absence of symptomatic HF
• RV infarction, as evidenced by ischemic changes in inferior leads on ECG
• If patient has received an oral phosphodiesterase inhibitor within the past 24–48 hr
• Inhibits platelet activation
Aspirin • Four aspirin 81 mg each (324 mg total) or one 325 mg tablet (non–enteric coated) should be
chewed and swallowed immediately

ACCP Updates in Therapeutics® 2022: Critical Care Pharmacy Preparatory Review and Recertification Course

541
Cardiovascular Critical Care I

Table 3. Predominant Interventions on Presentation/Onset for Stabilization – Any ACS (Circulation 2014;130:e344-
426; Circulation 2013;127:e362-425) (continued)
Predominant interventions on presentation/onset for stabilization – Any ACS
• Decreases risk of ventricular arrhythmias and sudden cardiac death in early post-MI period
• Decreases HR and myocardial oxygen demand and increases diastolic filling time of
ventricles, thereby improving oxygen flow through the coronary arteries
β-Blockade should be initiated within the first 24 hr of an ACS unless:
• There are signs of HF
• Active evidence of other shock states
β-Blockade • If at increased risk of cardiogenic shock (SBP < 120 mm Hg, HR > 110 beats/min or < 60
beats/min, age > 70, and increased time since onset of symptoms)
Relative contraindications to β-blockade include:
• PR interval > 0.24 s, second- or third-degree heart block, and active asthma/reactive
airway disease
*Note: Intravenous β-blockers may be particularly harmful in patients with risk factors for
shock.
• Morphine
Helpful • Oxygen
acronym • Nitrates (unless contraindicated)
MONA +/- BB • Aspirin
• β-Blocker (unless contraindicated)
• ACE inhibitors/ARBs should be used with caution within the first 24 hr of ACS because of
the risk of hypotension and potential contribution to contrast-induced nephrotoxicity
Other • Any NSAID other than aspirin should be avoided and/or discontinued for reasons beyond
precautions GI bleeding and nephrotoxicity, which may include reinfarction, hypertension, HF
exacerbation, myocardial rupture, and overall increased risk of mortality associated with
their use
ACS = acute coronary syndrome(s); GI = gastrointestinal; HF = heart failure; IV = intravenous; LMNOP = lasix-morphine-nitro-oxygen-position/positive pressure
ventilation; NSAID = nonsteroidal anti-inflammatory drug; RV = right ventricular; Sao2 = arterial oxygen saturation.

D. Revascularization
1. Nonsurgical – Details of interventional cardiology procedures are too broad to be discussed in great
detail in this chapter; however, the following are some considerations of the intervention that may play
a role in post-procedural management during a left heart catheterization and percutaneous coronary
intervention (PCI):
a. Access site
i. Radial artery (and, rarely, brachial artery) (Catheter Cardiovasc Interv 2011;78:840-6)
(a) Easily accessible
(b) Increased risk of vasospasm during procedure
(c) Easily compressible vessel when hemostasis is needed post-procedure
(d) Does not prevent patient mobility post-procedure
ii. Femoral artery
(a) Easily accessible
(b) More difficult to compress vessel when hemostasis is needed post-procedure and also
associated with increased bleeding complications
(c) Limits patient mobility post-procedure for at least 12–24 hours (bleeding risk after sheath
removal)

ACCP Updates in Therapeutics® 2022: Critical Care Pharmacy Preparatory Review and Recertification Course

542
Cardiovascular Critical Care I

b. Common interventions performed:


i. Stent placement
(a) Important to note the number of stents placed, types of stents, and locations of placement
(b) Bare metal stent
(1) Requires aspirin for life and a P2Y12 antagonist for at least 1 month (12 months
preferred) to allow adequate time for endothelialization of the stent(s)
(2) Longer therapies may be considered, depending on the number of stents and location(s)
of the stent(s).
(3) Higher risk of in-stent stenosis over time (because of neointimal cell proliferation)
(4) Typically preferred for patients unable to adhere to (i.e., cost or compliance) or not
appropriate for (i.e., bleed risk) longer-term dual antiplatelet therapy
(c) Drug-eluting stent
(1) Requires aspirin for life and a P2Y12 antagonist for at least 6 months (12 months
preferred). Longer or shorter therapy durations may be considered, depending on
the number and/or location(s) of the stent(s) as well as patient bleeding risk. Ongoing
investigations are evaluating the optimal duration of dual antiplatelet therapy, but an
individualized approach is also needed (J Am Coll Cardiol 2015;66:832-47; Circulation
2016;133:2094-8; Eur Heart J 2021;42:1289-367).
(2) The benefit of drug-eluting stents (i.e., everolimus, zotarolimus, biolimus) is the
mitigation of in-stent restenosis, more common with bare metal stenting. However, the
rate of stent endothelialization is also impaired such that the risk of stent thrombosis
persists for a longer period. Consequently, drug-eluting stents mandate longer-term
dual antiplatelet therapy than bare metal stents.
ii. Thrombectomy: Thrombus aspiration generally followed by stent placement at site of lesion
iii. Percutaneous transluminal coronary angioplasty (PTCA), also called “plain old balloon
angioplasty” (POBA): Balloon expansion and at least temporary displacement of occlusion at
site of lesion
2. Surgical – Details of coronary artery bypass grafting (CABG) procedures, including conduit type and
use of cardiopulmonary bypass (or performing off pump), are too broad to be discussed in greater
detail in this chapter. According to the Society of Thoracic Surgeons/American College of Cardiology/
American Heart Association (STS/ACC/AHA) CABG guidelines, emergency CABG is recommended
in patients with an acute MI in the following scenarios (J Am Coll Cardiol 2011;58:e123-210):
a. Primary PCI has failed or cannot be performed
b. Coronary anatomy is more suitable for CABG
c. Persistent ischemia of a significant area of myocardium at rest and/or hemodynamic instability
refractory to nonsurgical therapy is present
d. Requirement of surgical repair of a postinfarction mechanical complication of MI (i.e., ventricular
septal rupture, mitral valve insufficiency caused by papillary muscle infarction and/or rupture, or
free wall rupture)
e. Patients with cardiogenic shock who are suitable for CABG, irrespective of the time interval from
MI to onset of shock and time from MI to CABG
f. Patients with life-threatening ischemic ventricular arrhythmias in the presence of a left main
stenosis of 50% or more and/or three-vessel CAD

ACCP Updates in Therapeutics® 2022: Critical Care Pharmacy Preparatory Review and Recertification Course

543
Cardiovascular Critical Care I

g. CABG use is reasonable as a revascularization strategy in patients with multivessel CAD with
recurrent angina or MI within the first 48 hours of STEMI presentation as an alternative to a more
delayed strategy.
h. Early revascularization with PCI or CABG is reasonable for select patients older than 75 years with
ST-segment elevation or left bundle branch block who are suitable for revascularization irrespective
of the time interval from MI to onset of shock.
3. Medical management – No revascularization:
a. Less invasive strategies may be opted for rather than revascularization in some patients.
Considerations reinforce a patient-centered approach driven by evidence of ongoing/recurrent
ischemia and feasibility of revascularization, including myocardial viability, patient frailty, and
comorbid disease states.
b. Ongoing care outlined in section E remains the focus of optimizing aggressive medical management.

E. Antithrombotics in MI
1. The roles and combinations of antithrombotics continue to be refined in select populations (those with
NSTEMI/ACS, STEMI, and PCI). For the most current guidelines and landmark trials, please see www.
acc.org/guidelines.
2. Oral antiplatelet therapy
a. Platelets can be activated by several different mechanisms, only some of which can be inhibited
by medications.
b. Evaluation of antiplatelet therapy can be performed with platelet function testing and/or genotyping.
However, neither are currently performed routinely in the clinical setting. Clinical outcomes with
the use of platelet function testing to modify antiplatelet therapy (i.e., high-dose vs. standard-dose
clopidogrel) in PCI patients with high on-treatment platelet reactivity have been negative to date
(JAMA 2011;305:1097-1105). Outcomes with prospective genotype-guided antiplatelet therapy
(CYP2C19) from large cohorts of PCI patients have had lower rates of major adverse cardiovascular
events and lower rates of bleeding (JACC Cardiovasc Interv 2018;2:181-91; J Am Coll Cardiol
2018;71:1869-77; N Engl J Med 2019;381:1621-31).
c. The response to antiplatelets in certain critical care patient scenarios, such as acute hepatic
and kidney injury, have not been well documented. In other situations, such as when enteral
administration is not an option, parenteral antiplatelets may be considered (Table 5). However,
these options, which include glycoprotein IIb/IIIa (GP IIb/IIIa) inhibitors and cangrelor, have
primarily been studied in the setting of PCI.

ACCP Updates in Therapeutics® 2022: Critical Care Pharmacy Preparatory Review and Recertification Course

544
Cardiovascular Critical Care I

Table 4. Oral Antiplatelet Therapy (Circulation 2013;127:e362-425; Circulation 2014;130:e344-426; J Am Coll


Cardiol 2011;58:e123-210; J Am Coll Cardiol 2011;58:e44-122; Br J Clin Pharmacol 2011;72:647-57)
Medication Aspirin Clopidogrel Prasugrel Ticagrelor
Inhibits
Inhibits ADP- Inhibits ADP-
thromboxane Inhibits ADP-mediated
Mechanism mediated platelet mediated platelet
A2–mediated platelet activation at
of action activation at activation at
platelet P2Y12 receptor
P2Y12 receptor P2Y12 receptor
activation
Loading dose 162–325 mg 300-600 mg 60 mg 180 mg
Maintenance
81 mg daily 75 mg daily 5–10 mg dailya 90 mg BID
dose
Route Oral Oral Oral Oral
Prodrug No Yes Yes No
Reversible
No No No Yes
platelet binding
Onset 30 min 2–6 hr 30 min 30 min
% Platelet
~ 10–20 30–40 60–70 60–70
inhibition
Recommended
holding duration Do not hold 5 days 7 days 5 days
before CABGb
• Adenosine-induced
dyspnea and
Contraindication in
bradyarrhythmias
patients with any
• Older adult patients and
Pharmacogenomic history of stroke
patients with moderate or
Other notable variability or TIA because of
severe hepatic impairment
adverse effect(s) (CYP2C19) in bleeding risk, and
may be at increased risk of
or clinical pearls response is well warning for use in
bleeding
documented patients ≥ 75 years
• Concomitant maintenance
old or weight < 60
doses of aspirin > 100 mg
kg
should be avoided because
of lack of efficacy
a
Patients with lean body weight (< 60 kg) should receive a 5 mg daily maintenance dose of prasugrel
b
Decisions to hold these agents before other invasive procedures must consider indications for use, risk of thrombosis (i.e., stent type, locations, and number), and risk
of bleeding associated with intended procedure/surgery (JAMA 2013;310:1451-2; Catheter Cardiovasc Interv 2013;82:1113-4; JAMA 2013;310:189-98; N Engl J Med
2013;368:2113-24; J Am Coll Cardiol 2011;58:e44-122; Reg Anesth Pain Med 2010;35:64-101; J Am Coll Cardiol 2007;49:734-9).
ADP = adenosine diphosphate; BID = twice daily; TIA = transient ischemic attack.

3. Parenteral antithrombotics
a. Use of these agents is most concentrated in the procedural setting, although use may continue for
a finite period post-procedurally.
b. The selection and use among the agents in Table 5 may depend on presentation, timing/dose of pre-
procedural antiplatelet medication administration, clot burden during procedure, and estimated
bleeding risk of the procedure.

ACCP Updates in Therapeutics® 2022: Critical Care Pharmacy Preparatory Review and Recertification Course

545
Table 5. Parenteral Antithrombotics (Circulation 2014;130:e344-426; Circulation 2013;127:e362-425;Circulation 2015;131:1123-49; J Am Coll Cardiol
2011;58:e123-210; J Am Coll Cardiol 2011;58:e44-122; Br J Clin Pharmacol 2011;72:647-57)
Medication Heparin (UFH)a Bivalirudin Abciximab Eptifibatide Tirofiban Cangrelorb
Binds to GP IIb/ Binds to GP IIb/ Binds to GP IIb/
Inhibits ADP-
Indirect thrombin IIIa platelet receptor, IIIa platelet receptor, IIIa platelet receptor,
Mechanism of mediated platelet
inhibition (binds to Direct thrombin inhibitor inhibiting final common inhibiting final inhibiting final common
action activation at P2Y12
antithrombin III) pathway for platelet common pathway for pathway for platelet
receptor
aggregation platelet aggregation aggregation
50–100 units/kg
based on target 180 mcg/kg x 2, 10
Bolus dose 0.75 mg/kg 0.25 mg/kg 25 mcg/kg 30 mcg/kg
activated clotting min apart
time (ACT)
Continuous Not typically given 0.125 mcg/kg/min
infusion 1.75 mg/kg/hr 2 mcg/kg/min 0.15 mcg/kg/min 4 mcg/kg/min
during PCI (max 10 mcg/min)
Restoration of
Indirectly inhibits Irreversibly inhibits for Restoration of platelet Restoration of platelet
Indirectly inhibits thrombin- platelet function
Platelet inhibition thrombin-mediated the life of the platelet function within 6–8 hr function within 6–8 hr
mediated platelet activation within 1 hr of
platelet activation (7–10 days) of discontinuation of discontinuation
discontinuation
Hepatic and reticulo- 80% plasma proteolysis Reticulo-endothelial Plasma
Elimination Renal (50%) Renal (65%)
endothelial system 20% renal system dephosphorylation

546
Removed by
No Partial No Yes Yes N/A
dialysis

Oral prasugrel and


clopidogrel must be
Cardiovascular Critical Care I

given immediately
Requires renal dose adjustment
after discontinuation
Useful in patients with of cangrelor infusion
Target ACT of about suspected or confirmed HIT Murine monoclonal to maintain platelet
200–300 s during Requires renal dose Requires renal dose inhibition. Ticagrelor
Other notable May be preferred in patients antibody – may produce
procedure based on adjustment adjustment should be given as
adverse effect(s) with higher bleeding risk antigenicity
presence of GP IIb/ Acute, profound Acute, profound soon as possible,
or clinical pearls (particularly compared with Acute, profound
IIIa inhibitor and thrombocytopenia thrombocytopenia and administration
UFH with GP IIb/IIIa inhibitor thrombocytopenia
device used may overlap with
and among patients who
cangrelor infusion
undergo a transfemoral PCI
to maintain
approach)
platelet inhibition
after cessation of
cangrelor

ACCP Updates in Therapeutics® 2022: Critical Care Pharmacy Preparatory Review and Recertification Course
a
Enoxaparin may be used as an alternative to UFH.
b
U.S. Food and Drug Administration (FDA) approved as an adjunct to PCI for reducing the risk of periprocedural myocardial infarction (MI), repeat coronary revascularization, and stent thrombosis (ST) in patients who have
not been treated with a P2Y12 platelet inhibitor and are not receiving a glycoprotein IIb/IIIa inhibitor.
ACT = activated clotting time; GP = glycoprotein; HIT = heparin-induced thrombocytopenia; N/A = not applicable; UFH = unfractionated heparin.
Cardiovascular Critical Care I

F. Post-Intervention Complications
1. Bleeding (particularly retroperitoneal bleeding)
a. Several antithrombotic agents are used during PCI to inhibit both the platelets and the clotting
cascade, causing potential coagulopathies.
b. In addition to antithrombotic use, the catheterization access site has been identified as a major
contributor to post-PCI bleeding complications. Note that use of the trans-radial approach has been
demonstrated to be a bleeding avoidance strategy compared with the trans-femoral approach.
2. Dissection/rupture of free wall, coronary artery, or aorta: Although this may be spontaneous, it may
also be caused by vessel trauma from the catheter itself.
3. Stent thrombosis
a. When antiplatelet therapy is discontinued early (aspirin, P2Y12 inhibitor, or both), stent thrombosis
may occur in up to 25% of coronary artery stents, irrespective of type of stent (drug-eluting stent
or bare metal stent).
b. Almost 1 in 7 patients may discontinue P2Y12 inhibitors within 30 days post-PCI, thus increasing
mortality risk (adjusted hazard ratio [HR] 9.0; 95% confidence interval [CI], 1.3–60.6) (JAMA
2013;310:189-98).
c. Mortality rates associated with stent thrombosis can be as high as 45%.
d. Despite bleeding risks in critically ill patients, careful consideration should be given to correlating
these risks with the risk of stent thrombosis.
4. Papillary muscle rupture and mitral regurgitation (mechanical complications)
5. Arrhythmias (particularly after reperfusion)
6. Contrast-induced nephropathy

G. Ongoing Care – Quality measures for NSTEMI or STEMI independent of revascularization or medical
management
1. Medications that should be initiated before discharge or contraindications should be documented in the
medical record:
a. Aspirin
b. Statin
c. P2Y12 inhibitor
d. β-Blocker
e. If LVEF less than or equal to 40%
i. ACE inhibitor or ARB
ii. Aldosterone antagonist (if also evidence of HF and/or diabetes mellitus)
2. Interventions and/or referrals
a. LV function assessment (by imaging or during catheterization)
b. Cardiac rehabilitation
c. Smoking cessation counseling
d. Measurement of a lipid profile, including the low-density lipoprotein (LDL) cholesterol, should
preferably be obtained within 24 hours of admission. Any lipid profile measured between 6 months
before first medical contact and hospital discharge qualifies for this quality measure.
3. For more information on cardiology-related quality measures and registries, see www.ncdr.com.

ACCP Updates in Therapeutics® 2022: Critical Care Pharmacy Preparatory Review and Recertification Course

547
Cardiovascular Critical Care I

Patient Case (Continued)

4. J.M.’s ECG results reveal no acute evidence of ST segment changes. However, the resident is still consider-
ing a diagnosis of ACS, given the patient’s shortness of breath and mild troponin elevation. Which statement
would be most accurate regarding the potential for other potential diagnoses?
A. No, this is most likely an NSTEMI.
B. Yes, it is likely undiagnosed chronic obstructive pulmonary disease.
C. Yes, it is likely early sepsis.
D. Yes, it is likely decompensated HF.

5. J.M. has been experiencing intermittent bradycardia on telemetry. The team has consulted the cardiac elec-
trophysiology team. In the meantime, which statement most accurately reflects whether any other underlying
correctable/contributing causes can be addressed?
A. Ticagrelor could be discontinued and switched back to clopidogrel.
B. Ticagrelor should be discontinued altogether.
C. No treatment intervention is needed; this is likely the result of J.M.’s HF progression.
D. J.M.’s hyperkalemia should be treated.

V.  ARRHYTHMIAS AND ANTIARRHYTHMICS

A. Pathophysiology: Arrhythmias are generally caused by automaticity and/or re-entrant conduction


abnormalities.

B. Bradyarrhythmias (beyond sinus) and Types of Heart Block (Critical Care Medicine 2014:xix;
Pathophysiology of Heart Disease 2011:xiv; BMJ 2002;324:662-5; BMJ 2002;324:415-8)
1. Etiologies of heart block (Box 3)

Box 3. Causes of Bradyarrhythmias

• CAD
• Degenerative conduction disease
• Drug toxicity
• Electrolyte disturbances (particularly hyperkalemia)
• Endocarditis
• Myocarditis
• Surgery (particularly cardiac surgery)
• Tumors
• Vagus nerve–mediated heart block
CAD = coronary artery disease.

2. When evaluating ECGs in the presence of heart block, QRS complex evaluation can guide some
differential diagnoses to the source of conduction problems (see Table 6).
a. Narrow QRS complexes commonly indicate AV nodal dysfunction.
b. Wide QRS complexes may indicate dysfunction in either the AV node or the His-Purkinje system.

ACCP Updates in Therapeutics® 2022: Critical Care Pharmacy Preparatory Review and Recertification Course

548
Cardiovascular Critical Care I

Table 6. Bradyarrhythmias and Types of Heart Block (Critical Care Medicine 2014:xix; Pathophysiology of Heart
Disease 2011:xiv; BMJ 2002;324:662-5; BMJ 2002;324:415-8)
Type ECG Example Description
Delayed conduction from the sinoatrial
(SA) node to the atrioventricular (AV)
node characterized by a P-R interval
> 0.2 s
First degree
Relatively benign; however, underlying
contributors should be evaluated and
minimized (i.e., β-blockers and other
agents)
Consistent P-P interval with progressive
prolongation of the P-R (indicating
impaired SA to AV node conduction)
eventually resulting in absence of a QRS
complex because of the lack of AV node
Second-degree conduction of atrial impulse
Mobitz type 1
Of most concern in older adult patients
(Wenckebach)
in whom this may be indicative of
progressive conduction disease; may be
more benign in younger patients
“Longer, longer, longer, drop … must be
Wenckebach”
Consistent P-P interval and consistent
P-R interval duration with spontaneous
absence of a QRS complex because of
Second degree the lack of AV node conduction of atrial
Mobitz type 2 impulse
Usually indicative of more significant
conduction disease and associated with
syncope, HF, and increased mortality rates
Characterized by consistent P-P intervals,
consistent R-R intervals, and variable/
random P-R interval representing
Third degree
independent, uncoordinated atrial and
(Complete heart
ventricular conduction (A-V dissociation).
block)
The arrows on this ECG strip identify P
waves, showing regular P-P interval with
varying P-R interval

Manifested when sinus node dysfunction


Junctional allows the AV node to take over as the
rhythm active cardiac pacemaker, resulting in
retrograde conduction through the atria

ACCP Updates in Therapeutics® 2022: Critical Care Pharmacy Preparatory Review and Recertification Course

549
Cardiovascular Critical Care I

N
Assess appropriateness for clinical condition
Heart rate typically < 50 bpm if bradyarrhythmia

Identify and treat underlying cause


• Maintain patent airway; assist breathing as necessary
• If patient is short of breath, administer oxygen
• Oxygen as indicated
• Cardiac monitor to identify rhythm; monitor blood pressure and oximetry
• IV access
• 12-lead ECG if available, don’t delay therapy

Persistent bradyarrhythmia causing:


• Hypotension?
Monitor and • Acutely altered mental status?
observe N
• Signs of shock?
• Ischemic chest discomfort?
• Acute heart failure?

Atropine IV Dose:
First dose: 1 mg
Repeat every 3-5 minutes
Max: 3 mg

If atropine ineffective:
• Transcutaneous pacing
OR
• Dopamine IV infusion: 5-20 mcg/kg/min
OR
• Epinephrine IV infusion: 2-10 mcg/min

Consider:
• Expert consultation
• Transvenous pacing

Figure 8. Advanced cardiac life support bradycardia algorithm adapted from the 2020 American
Heart Association guidelines for Cardiopulmonary Resuscitation and Emergency Cardiovascular
Care (Circulation 2021;142(14_suppl_2):S366-468).

ACCP Updates in Therapeutics® 2022: Critical Care Pharmacy Preparatory Review and Recertification Course

550
Cardiovascular Critical Care I

3. Management of bradyarrhythmias includes three principal strategies (see also Figure 8). Stabilize the
patient, if symptomatic.
a. Consider atropine for temporary correction to decrease vagal tone. Note that the recommended
dose of atropine was increased from 0.5 mg to 1 mg, which can be given every 3–5 minutes for a
maximum of 3 mg.
b. Consider pacing strategies (temporary transvenous pacer, transcutaneous pacer, pacing pulmonary
artery catheter). Interrogate permanent pacemaker for malfunction/optimization.
c. Consider chronotropic β-agonist infusion.
i. Dopamine
ii. Epinephrine
iii. Isoproterenol
iv. Dobutamine
d. Identify and treat underlying causes/toxidromes.

C. Tachyarrhythmias (beyond sinus) – Etiologies and hemodynamic consequences


1. Etiologies of tachyarrhythmias
a. Usually related to enhanced automaticity, reentry, or triggered activity
b. A history that includes ischemic heart disease or congestive cardiac failure is 90% predictive of
VT.
2. In evaluating ECGs for tachyarrhythmias, some fundamental considerations include:
a. Evaluate for the presence of P waves.
b. Evaluate the width of the QRS complex.

Table 7. Tachyarrhythmias (beyond sinus)a (Circulation 2014;130:2071-104; BMJ 2002;324:594-7776-9; BMJ


2002;324:1264-7; BMJ 2002;324:1201-4; Circulation 2012;125:381-9; Heart Fail Rev 2014;19:285-93; J Am Coll
Cardiol 2018;72:1677-749)
Supraventricular Tachyarrhythmias
P-wave Atrial Rate
Type Rhythm Description
Attributes (beats/min)
Generally benign but may be more evident with
Premature atrial increased sympathetic tone, stress, and pericardi-
complexes Irregular N/A N/A tis or with sympathomimetic use
(PACs) In some cases, can lead to an AV block or initiate a
reentrant supraventricular tachycardia (SVT) or AF
Usually sudden onset/offset with narrow QRS
complexes
Often caused by reentry within the atrium or AV
Supraventricular Hidden
node or by an accessory conduction pathway
tachycardia Regular or can be 140–250
Can be subcategorized as:
(SVT) retrograde
AV nodal reentrant tachycardia (AVNRT)
AV reentrant tachycardia (AVRT)
Sinus node reentry tachycardia
Generally conducts through the ventricles in a 2:1
fashion, resulting in ventricular rates of 100–150
beats/min
Atrial flutter Saw-tooth In some scenarios, slowing the atrial rate may
Regular 180–350
(AFl) appearance increase the number of conducted beats, leading
to rapid ventricular rates and potential hemody-
namic compromise
Associated with increased risk of stroke

ACCP Updates in Therapeutics® 2022: Critical Care Pharmacy Preparatory Review and Recertification Course

551
Cardiovascular Critical Care I

Table 7. Tachyarrhythmias (beyond sinus)a (Circulation 2014;130:2071-104; BMJ 2002;324:594-7776-9; BMJ


2002;324:1264-7; BMJ 2002;324:1201-4; Circulation 2012;125:381-9; Heart Fail Rev 2014;19:285-93; J Am Coll
Cardiol 2018;72:1677-749) (continued)
Supraventricular Tachyarrhythmias
P-wave Atrial Rate
Type Rhythm Description
Attributes (beats/min)
Most common arrhythmia, characterized by irregu-
lar ECG appearance because of multiple reentry
circuits and ectopic foci (“irregularly irregular”)
Atrial
No distinct P Unable to Often associated with structural heart disease and
fibrillation Irregular
wave visible determine potentiated by increased left atrial pressures
(AF)
among other influencing contributors such as age,
inflammation, and sympathetic tone
Associated with increased risk of stroke
Can be misdiagnosed as AF
> 3 different
Multifocal atrial Commonly associated with respiratory disease, HF,
types of
tachycardias Irregular 100–130 critical illness
distinct P
(MATs) May be exacerbated by electrolyte abnormalities or
waves
toxicity with digoxin or theophylline
Ventricular Arrhythmias
Type Description
Results from an ectopic ventricular focus conduction that can be identified by the
lack of a preceding P wave. Commonly benign or asymptomatic but may be of
concern if present in patterns or in patients with advanced heart disease
Premature ventricular
Bigeminy: Every other beat is a PVC
complexes (PVCs)
Trigeminy: Every third beat is a PVC
Couplets: Patterns of two consecutive PVCs
Triplets: Patterns of three consecutive PVCs
Potentially lethal wide QRS complex tachycardia characterized according to
morphology and duration; can degenerate into ventricular fibrillation (VF) or
asystole
Monomorphic VT: When every QRS complex appears the same and the rate is
regular (100–200 beats/min); commonly caused by reentry circuit related to
Ventricular myocardial scar or fibrosis
tachycardia (VT) Polymorphic: When the QRS complexes continually vary in shape and rate; may
be related to multiple ectopic foci, myocardial ischemia, or Torsades de pointes
Non-sustained VT: Self-terminating episodes lasting for < 30 s
Sustained VT: If VT persists for more than 30 s, produces severe symptoms,
including syncope, or requires termination by administration of an
antiarrhythmic drug or direct cardioversion/defibrillation
Life-threatening arrhythmia with a chaotic ECG with no discernible QRS,
Ventricular fibrillation
representing rapid disorganized conduction with no resultant coordinated
(VF)
ventricular contractions

ACCP Updates in Therapeutics® 2022: Critical Care Pharmacy Preparatory Review and Recertification Course

552
Cardiovascular Critical Care I

a
Other arrhythmias beyond the scope of this review include sick sinus syndrome (tachy-brady syndrome) and Wolff-Parkinson-White.

3. Management of tachyarrhythmias with a pulse includes the following strategies:


a. Stabilization of patient: See Figure 9
i. Atrial tachyarrhythmias
(a) Heart rate control typically with class II agents, class IV agents, and/or digoxin (see
Appendix A)
(b) Heart rhythm control
(1) Antiarrhythmic therapy typically with class Ic or class III agents (see Appendix A)
(2) Catheter-based ablation
(3) Surgical ablation
ii. Ventricular tachyarrhythmias
(a) Catheter ablation
(b) Antiarrhythmic therapy
(c) Evaluation for implantable cardioverter-defibrillator
b. Overall considerations for antiarrhythmics (see Appendix A) (Circulation 2014;130:2071-104; Heart
Fail Rev 2014;19:285-93; Circulation 2012;125:381-9; BMJ 2002;324:594-7; BMJ 2002;324:776-9;
BMJ 2002;324:1264-7; BMJ 2002;324:1201-4)
i. Abrupt discontinuation of chronic antiarrhythmics (i.e., chronic medication before ICU
admission) should be done with caution and awareness of antiarrhythmic indication as well as
risks-benefits of continuation/cessation.
ii. Appropriate monitoring and potential adjustment may be warranted with many of these agents
in the critically ill patient.
(a) QT/QTc prolongation increases the risk of Torsades de pointes. This risk can be influenced
not only by the absolute duration of the QT/QTc but also by the rate at which the QT/QTc
is changed (i.e., faster change may also increase risk).
(b) The class III antiarrhythmics sotalol and dofetilide can be used for a variety of arrhythmias
(most notably AF/AFl) and, because of some notable characteristics, should be watched
closely when used in the critically ill patient.
(1) Both are renally eliminated.
(2) Both have notable interactions with several other medications (especially dofetilide
on the basis of CYP3A4 and renal transport) in addition to their effect on the QT/QTc
interval.
(3) ECG monitoring should be performed for safety and tolerability 2–3 hours after the
first five doses when initiating, reinitiating, or introducing another interacting or QT-
prolonging agent.
(4) Electrolyte abnormalities may place a patient at increased risk of Torsades de pointes
(particularly magnesium and potassium).
(5) Dofetilide is particularly useful among patients with structural heart disease
(including HF) because of a neutral effect on mortality. In contrast, sotalol has been
associated with increased mortality in patients with HF and is not recommended in
this patient population for atrial arrhythmias.

ACCP Updates in Therapeutics® 2022: Critical Care Pharmacy Preparatory Review and Recertification Course

553
Cardiovascular Critical Care I

N Assess appropriateness for clinical condition


Heart rate typically > 150 bpm if tachyarrhythmia

Identify and treat underlying cause


• Maintain patent airway; assist breathing as necessary
• Oxygen as indicated
• Cardiac monitor to identify rhythm; monitor blood pressure and oximetry

Persistent tachyarrhythmia causing:


• Hypotension? Synchronized Cardioversion*
• Acutely altered mental status? • Consider sedation
• Signs of shock? Y • If regular narrow complex,
• Ischemic chest discomfort? consider adenosine
• Acute heart failure?
*Initial Recommendations:
N Narrow regular: 50-100 J
Narrow irregular: 120-200 J biphasic or
200 J monophasic
Wide regular: 100 J
Wide irregular: Defibrillation dose

Wide QRS?
0.12 seconds Y • IV access and 12-lead ECG if available
• Consider adenosine only if regular and
N monomorphic
• Consider antiarrhythmic infusion
• Consider expert consultation

• IV access and 12-lead ECG if available


• Vagal maneuvers
• Adenosine (if regular)
• Beta-blocker or calcium channel blocker
• Consider expert consultation

Medication Doses/Details

Adenosine IV Dose Sotalol IV Dose


• First dose = 6 mg rapid IVP, follow with NS flush • 100 mg (1.5 mg/kg) over 5 minutes
• Second dose = 12 mg if required • Avoid if prolonged QT

Amiodarone IV Dose Procainamide IV Infusion Dose (for stable wide-QRS)


• First dose: 150 mg over 10 minutes • 20-50 mg/min until arrhythmia suppressed,
• Repeat as needed if VT recurs hypotension ensues, QRS duration increases > 50%,
• Maintenance infusion: 1 mg/min for 6 hours or maximum dose of 17 mg/kg given
• Maintenance infusion: 1-4 mg/min
• Avoid if prolonged QT or HF

Figure 9. Advanced cardiac life support tachycardia algorithm adapted from the 2020 American Heart
Association guidelines for Cardiopulmonary Resuscitation and Emergency Cardiovascular Care (Circulation
2021;142(16_suppl_2):S366-468).

ACCP Updates in Therapeutics® 2022: Critical Care Pharmacy Preparatory Review and Recertification Course

554
Cardiovascular Critical Care I

iii. Common agents in the ICU used to treat tachyarrhythmias


(a) β-Blockers
(1) May be of limited use in patients receiving vasopressors and/or inotropes, but can be
used for both atrial and ventricular tachyarrhythmias
(2) Esmolol has the shortest half-life (cleared by plasma esterases), and administration
rates commonly coincide with high volumes of fluid.
(3) Agents with combined α1-antagonism and nonselective β-antagonism will have
greater effects on blood pressure than their β1-specific antagonist counterparts.
• β1-specific antagonist examples:
• Esmolol (intravenous), metoprolol tartrate (oral/intravenous)
• Agents with combined α1- and nonselective β-antagonism effects include:
• Carvedilol (oral) – about 25:1 ratio of β:α1 receptor activity
• Labetalol (intravenous) – about 7:1 ratio of β:α1 receptor activity
• Labetalol (oral) – about 3:1 ratio of β:α1 receptor activity
(b) Non-dihydropyridine calcium channel blockers (diltiazem, verapamil)
(1) Not recommended in patients with HF with reduced ejection fraction
(HFrEF) (systolic HF) because of potent negative inotropic effects (class III C).
(J Am Coll Cardiol 2013;62:e147-239)
(2) May be of limited use in patients on vasopressors and/or inotropes, but can be used
for atrial tachyarrhythmias
(c) Amiodarone
(1) Commonly used for both atrial and ventricular tachyarrhythmias
(2) Demonstrated safety in patients with structural heart disease (CHF) because of
neutral effect on mortality
(3) Affects all phases of the action potential (sodium [Na], calcium [Ca], potassium [K]
channel and provides some α- and β-antagonism)
(4) Severe hypotension can occur with intravenous amiodarone because of the solvent
polysorbate 80 (Tween 80) used in some formulations. This transient hypotension
in patients with a pulse can be associated with rapid infusions, particularly of
undiluted drugs, and is best avoided by dilution and slower administration. However,
in pulseless patients, rapid intravenous administration of undiluted amiodarone is
commonly indicated.
(5) Among common agents used in the ICU, amiodarone has one of the largest volumes
of distribution (about 60 L/kg) and prolonged half-lives (about 35–110 days).
(6) In adults, common total intravenous or oral loading doses throughout several days are
about 8–10 g before switching to a maintenance dose.
Loading dose = concentration × volume of distribution × weight 7.2–12 g = (1.5–2.5
mg/L) × (60 L/kg) × (80 kg)
(7) Extensive metabolism by CYP3A4 and CYP2C8 and enzyme inhibitor of CYP3A4,
CYP1A2, CYP2C9, and CYP2D6, resulting in several drug-drug interactions ((e.g.,
warfarin, statins)
(8) If patients are to be continued on amiodarone for a prolonged duration, thyroid and
liver function tests, pulmonary function tests, chest radiographs, and ophthalmic
examinations should be evaluated periodically (Heart Rhythm 2007;4:1250-9).
(d) Lidocaine
(1) Indicated only for treatment of ventricular tachyarrhythmias and is particularly useful
if the tachyarrhythmia is caused by active ischemic myocardial tissue

ACCP Updates in Therapeutics® 2022: Critical Care Pharmacy Preparatory Review and Recertification Course

555
Cardiovascular Critical Care I

(2) Efficacy and toxicity are both concentration dependent.


(3) Metabolism is largely dependent on hepatic blood flow; the primary metabolites are
monoethylglycinexylidide (MEGX) and glycine xylidide (GX) and are mediated by
CYP1A2. Both lidocaine and MEGX contribute to therapeutic effect and toxicity,
whereas GX has predominantly toxic adverse effects. Lidocaine’s metabolites are
eliminated renally.
(4) Concentration-dependent protein binding includes about 25% bound to albumin and
about 50% bound to α1-acid glycoprotein (AAG).
(5) As AAG increases and decreases as an acute-phase reactant within the first 12–72
hours after certain stresses (e.g., acute MI, HF exacerbation, trauma), an unsuspected
variation can occur in free lidocaine concentrations. Because of the toxicity profile
and concentration-dependent efficacy, routine concentrations should be monitored in
most patients if lidocaine is to be continued beyond 24 hours (particularly in patients
with advanced age, HF, liver disease, and renal dysfunction).

Table 8. Lidocaine Concentrations and Toxicity Symptoms


Total Serum
Toxicity
Concentration (mg/L)
< 1.5–5 Typical therapeutic goal concentration
Increasing risk of light-headedness, confusion, dizziness, tinnitus, twitching,
> 5–8
tremor, blurred or double vision, hypotension, and bradycardia
High risk of seizures, respiratory depression, hypotension, bradycardia, AV nodal
>8
blockade, decreased cardiac output
AV = atrioventricular.

(e) Digoxin
(1) Digoxin may be considered to slow a rapid ventricular response in patients with ACS
and AF associated with severe LV dysfunction and HF or hemodynamic instability
(Circulation 2019;140:e125-51).
(2) Use for AF should be done with caution. Two published retrospective trials have
shown considerable evidence for increased hospitalization rates and risk of death.
• 71% increased risk of death (HR 1.71; 95% CI, 1.52–1.93) and 63% increased risk
of hospitalization (HR 1.63, 95% CI, 1.56–1.71) (Circ Arrhythm Electrophysiol
2015;8:49-58)
• Digoxin-treated patients had higher mortality rates (95 vs. 67 per 1000 person-
years; p<0.001), and use was independently associated with mortality, despite
multivariate adjustment (HR 1.26; 95% CI, 1.23–1.29, p<0.001) (J Am Coll
Cardiol 2014;64:660-8).
• Eliminated renally; thus, may pose risks of digoxin toxicity in patients with acute
or chronic renal failure
(3) Although the volume of distribution is relatively large (7–10 L/kg in healthy adults),
only a small percentage of total body stores are present in the serum.
(4) Digoxin has a smaller volume of distribution in patients with renal failure (around 4.5
L/kg); thus, loading doses may be lower in these patients.

ACCP Updates in Therapeutics® 2022: Critical Care Pharmacy Preparatory Review and Recertification Course

556
Cardiovascular Critical Care I

(5) Digoxin efficacy and toxicity do not correlate well with drug concentrations.
• Common therapeutic targets for heart rate control are 0.8–1.5 ng/mL, although
many clinical laboratories report therapeutic concentrations within 0.5–2.0 ng/
mL.
• Toxicity, which can present at any serum concentration, should be evaluated
according to clinical manifestations, including: nausea, vomiting, anorexia,
mental status changes, visual disturbances, ventricular arrhythmias, bradycardia,
and hyperkalemia.
• Efficacy is largely based on clinical control of the heart rate; steady-state
concentrations (more than 5–7 days after initiation) are typically used only to
validate that concentrations are not supratherapeutic.
4. Anticoagulation for AF or atrial flutter
a. According to the 2019 AHA/ACC/Heart Rhythm Society (HRS) AF guidelines, all patients with
atrial flutter or paroxysmal, persistent, or permanent AF should be evaluated for anticoagulation,
preferably using the CHA2DS2-VASc score to approximate stroke risk (Circulation 2019;140:e125-51).
i. For a CHA2DS2-VASC score of 2 or greater in men and 3 or greater in women, an oral
anticoagulant is recommended (J Am Coll Cardiol 2019;12:104-32).
ii. It is reasonable to omit anticoagulation in patients with a CHA2DS2-VASc score of 0 in men
or 1 in women.
iii. For patients with AF and a mechanical prosthetic heart valve or valves, bridging with
unfractionated heparin or low-molecular-weight heparin should be performed in the context
of bleeding versus stroke risk. Warfarin anticoagulation and international normalized ratio
(INR) goals should be consistent with the type and location of the prosthetic valve.
iv. In patients with AF at intermediate risk for thromboembolism (average CHADS2 score of
2.3), the use of bridging with low-molecular-weight heparin was non-inferior to no bridging
for elective procedures. However, there was a significant excess of major bleeding in the
low-molecular-weight heparin group. Note that patients with a mechanical valve or stroke/
transient ischemic attack/systemic embolism within the preceding 12 weeks were not eligible
for inclusion in the study (N Engl J Med 2015;373:823-33).
v. Although the CHADS2 and CHA2DS2-VASc scores are commonly used to estimate annual
stroke risk, these scoring systems were not founded in the context of critically ill patients.

Table 9A. CHADS2 Stroke Risk Score in AF


Total Patient Adjusted Annual
Risk Assessment Score
Score Stroke Rate, %
CHADS2
Congestive heart failure 1 0 1.9
Hypertension 1 1 2.8
Age ≥ 75 1 2 4.0
Diabetes 1 3 5.9
Stroke/TIA/thromboembolism 2 4 8.5
Maximum score 6 5 12.5
6 18.2

ACCP Updates in Therapeutics® 2022: Critical Care Pharmacy Preparatory Review and Recertification Course

557
Cardiovascular Critical Care I

Table 9B. CHA2DS2-VASc Stroke Risk Score in AF


Total Patient Adjusted Annual
Risk Assessment Score
Score Stroke Rate, %
CHA2DS2-VASc
Congestive heart failure 1 0 0
Hypertension 1 1 1.3
Age ≥ 75 2 2 2.2
Diabetes 1 3 3.2
Stroke/TIA/thromboembolism 2 4 4.0
Vascular disease 1 5 6.7
Age 65–74 1 6 9.8
Sex category (female) 1 7 9.6
Maximum score 9 8 6.7
9 15.2
AF = atrial fibrillation; TIA = transient ischemic attack.
Information from: January CT, Wann LS, Alpert JS, et al. 2014 AHA/ACC/HRS guideline for the management of patients with atrial fibrillation: a report of the
American College of Cardiology/American Heart Association Task Force on Practice Guidelines and the Heart Rhythm Society. Circulation 2014;23:2071-104.

b. Anticoagulation decisions should balance the risk of stroke versus the risks of bleeding in the context
of duration of bridging and/or lack of anticoagulation. Scoring systems have been described to help
assess bleeding risk in anticoagulation decisions for patients with AF on warfarin anticoagulation
(Table 10). Not unlike stroke risk scoring systems, these scoring systems were not founded in the
context of critically ill patients.
c. Direct-acting oral anticoagulants (DOACs) are recommended over warfarin for patients with AF
(exclusions: moderate to severe mitral stenosis, mechanical heart valve) (Chest 2018;154:1121-201;
J Am Coll Cardiol 2019;12:104-32).
d. For patients with AF who have a CHA2DS2-VASc score of 2 or greater (men) and 3 or greater
(women) and who have end-stage chronic kidney disease (creatinine clearance [CrCl] less than
15 mL/minute/1.73 m 2) or are receiving dialysis, it may be reasonable to prescribe warfarin (INR
2.0–3.0) or apixaban (J Am Coll Cardiol 2019;12:104-32).

Table 10. HAS-BLED or HEMOR 2RHAGES Bleeding Risk Scores in AF


Total Patient Bleeds/100 Patient-Yr
Risk Factor Assessment Score
Score of Warfarin
HAS-BLED
Hypertension 1 0 1.13
Abnormal renal or liver function (1 point each) 1 or 2 1 1.02
Stroke 1 2 1.88
Bleeding 1 3 3.74
Labile INRs 1 4 8.70
Elderly (> 65 yr) 1 5 12.5
Drugs or alcohol (1 point each) 1 or 2 6 0
Any score 1.56

ACCP Updates in Therapeutics® 2022: Critical Care Pharmacy Preparatory Review and Recertification Course

558
Cardiovascular Critical Care I

Table 10. HAS-BLED or HEMOR 2RHAGES Bleeding Risk Scores in AF (continued)


Total Patient Bleeds/100 Patient-Yr
Risk Factor Assessment Score
Score of Warfarin
HEMOR2RHAGES
Hepatic or renal disease 1 0 1.9
Ethanol abuse 1 1 2.5
Malignancy 1 2 5.3
Older age 1 3 8.4
Reduced platelet count or function 1 4 10.4
Rebleeding risk 2 ≥5 12.3
Hypertension (uncontrolled) 1 Any score 4.9
Anemia 1
Genetic factors 1
Excessive fall risk 1
Stroke 1
AF = atrial fibrillation.

e. In the critically ill patient, parenteral anticoagulation with unfractionated heparin or low-molecular-
weight heparin may be more favorable if the benefit of anticoagulation exceeds the risk of bleeding.
Unfractionated heparin infusions would be favored more than low-molecular-weight heparin in
renal failure (CrCl less than 30 mL/minute/1.73 m 2).
f. For anticoagulation of most critically ill patients, oral anticoagulation may be less favorable or even
detrimental compared with parenteral anticoagulation.
i. Elimination of DOACs (apixaban, dabigatran, edoxaban, and rivaroxaban) is significantly
affected in renal compromise. An additional consideration in the critical care setting is the
feasibility of reversal in the event of an acute bleed or need for an invasive procedure. Novel
antidotes continue to be studied and approved (idarucizumab, andexanet); however, clinical
experience with reversibility in these settings is evolving. Finally, the aforementioned DOACs
are not devoid of relevant drug-drug interactions, particularly when given in combination with
strong CYP and/or P-glycoprotein inhibitors/inducers.
ii. Warfarin is challenging due to issues with malnutrition, drug interactions, and unpredictable
dose response in the critically ill patient. Complications with this agent are more predictably
managed than are complications with other oral anticoagulants if a patient requires an invasive
procedure or develops an acute bleed, but warfarin use is still not without risk.
g. For patients with AF/flutter for 48 hours or more (or if undetermined) without therapeutic
anticoagulation, absence of thrombus on the left side of the heart should be confirmed by
transesophageal ECHO before pharmacologic or direct current cardioversion.
h. If AF/flutter for more than 48 hours or an unknown duration that requires direct current or
pharmacologic cardioversion for hemodynamic instability, anticoagulation should be initiated as
soon as possible and continued for at least 4 weeks after cardioversion unless contraindicated

ACCP Updates in Therapeutics® 2022: Critical Care Pharmacy Preparatory Review and Recertification Course

559
Cardiovascular Critical Care I

Patient Case (Continued)

The appropriate intervention has been made from question 5, and the patient has not required any need for pac-
ing. For the next 48 hours, J.M. continues on inotrope therapy and is being diuresed (his net fluid balance has
been 1250 and 900 mL negative each day for the past 2 days). The transthoracic ECHO results showed J.M.’s
ejection fraction to be 15%–20%, with a dilated, hypokinetic LV, severe mitral regurgitation, and dilated atria.
No evidence of intracardiac thrombus was seen, but this could not be ruled out.

6. J.M. has had increasing premature atrial complexes on telemetry, and his heart rate has consistently been
83–96 beats/minute. He is receiving dobutamine at 5 mcg/kg/minute and a furosemide infusion at 10 mg/
hour. The team is notified that J.M. has gone into AF with rapid ventricular response and a heart rate of 132
beats/minute (he has been in AF for about 30 minutes). His blood pressure is now 83/52 mm Hg. Which
treatment plan would be most preferred for this patient?
A. Synchronized cardioversion
B. Metoprolol 5 mg intravenous push
C. Adenosine 6 mg rapid intravenous push
D. Amiodarone 300-mg intravenous push, followed by a continuous infusion at 1 mg/minute

7. J.M.’s BP and HR improved after the previous intervention; however, he remains in AF. The team is now
concerned about evaluating the patient for anticoagulation. You are asked to provide input about the appro-
priateness of anticoagulation, given the patient’s clinical course and past medical history. His calculated
CHA2DS2-VASc score is 5 and HAS-BLED score is 5. The physician is considering anticoagulation with a
heparin infusion while the patient is in the ICU and asks for a recommendation. Which is the most appro-
priate response?
A. According to these scores, the patient’s risk of bleeding is the same as his stroke risk; therefore, it would
be reasonable to either initiate or withhold anticoagulation.
B. According to these scores, the patient’s risk of stroke exceeds his risk of bleeding; therefore, it would
be reasonable to initiate heparin anticoagulation.
C. The HAS-BLED score is less relevant because it is derived from chronic warfarin use. However, anti-
coagulation is still likely warranted eventually, given the patient’s CHA2DS2-VASc score.
D. The HAS-BLED and CHA2DS2-VASc scores are irrelevant to this patient’s current care, and antico-
agulation is not warranted.

ACCP Updates in Therapeutics® 2022: Critical Care Pharmacy Preparatory Review and Recertification Course

560
Cardiovascular Critical Care I

VI.  HEART FAILURE

A. Clinical Syndrome – Manifested because of congenital or acquired structural or functional myocardial


dysfunction that impairs filling and/or emptying of the heart
1. Mortality rate of 50% within 5 years of diagnosis
2. Predominantly descriptive of LV and its pump performance and the ejection fraction
3. Prognosis and treatment of HF largely depend on its etiology and staging.

Table 11. Diastolic Dysfunction Versus Systolic Dysfunction in HF


Heart Failure with Preserved EF Heart Failure with Reduced EF
Type
(HFpEF) (HFrEF)
• Systolic – blood emptying from the heart
Dysfunction • Diastolic – blood filling the heart
• May also coexist with impaired diastolic filling
• LVEF ≥ 50%
• Impaired relaxation and filling of the • LVEF ≤ 40%
ventricle before contraction • Impaired contraction and emptying of the
Characteristics
• Commonly described as a “stiffened” ventricular cavity
ventricle • Commonly described as a “dilated” ventricle
• Contractility remains “normal”
• Preload (volume optimization) is • Preload optimization essential – at high risk of
essential volume overload
• As diastolic dysfunction worsens • As systolic dysfunction worsens, cardiac output
– ventricular preload and diastolic becomes increasingly afterload sensitive
Clinical-
filling time must be maintained • Conditions acutely decreasing or increasing
management
• Conditions acutely decreasing preload preload or increasing afterload may lead to
pearls
may lead to decompensation (e.g., decompensation
tachyarrhythmias) • Volume overload and increasing myocardial
• Keep heart rate “slow” and ventricles dilation may contribute to decreased valvular
“full” coaptation and regurgitant flow
• NSAIDs, with the exception of aspirin
Common • Sympathomimetics
• NSAIDs, with the exception of aspirin
medications • Most antiarrhythmics except for amiodarone
• Sympathomimetics
to avoid and dofetilide
• Non-dihydropyridine calcium channel blockers
EF = ejection fraction; LVEF = left ventricular ejection fraction.

B. HF Etiologies
1. Ischemic cardiomyopathy
a. Accounts for about two-thirds of HF cases
b. Caused by myocardial damage/death owing to CAD
2. Non-ischemic cardiomyopathy
a. Accounts for about one-third of HF cases
b. May be attributed to other causes (Box 4)

ACCP Updates in Therapeutics® 2022: Critical Care Pharmacy Preparatory Review and Recertification Course

561
Cardiovascular Critical Care I

Box 4. Non-ischemic Cardiomyopathies


Non-ischemic Cardiomyopathies
• Amyloidosis • Familial
• Sarcoidosis • Hypertension
• Drug induced • Hyper/hypothyroidism
• Alcohol • Hypertrophic obstructive (HOCM)
• Anabolic steroids • Idiopathic
• Chemotherapy • Myocarditis
§ Anthracyclines • Autoimmune
§ Cyclophosphamide • Eosinophilic
§ Fluorouracil • Giant cell
§ Bevacizumab • Viral
§ Trastuzumab • Other infectious cause
§ Tyrosine kinase inhibitors • Non-compaction
• Cocaine • Peripartum
• Methamphetamine • Obesity
• Tricyclic antidepressants • Stress induced (Takotsubo)
• Anorexigens

C. RV Dysfunction and/or Failure (Exp Clin Cardiol 2013;18:27-30; J Am Coll Cardiol 2010;56:1435-46; Circ
Heart Fail 2010;3:340-6; Anesth Analg 2009;108:422-33; Anesth Analg 2009;108:407-21)
1. Less well characterized than LV HF
2. No guidelines to direct management of acute RV failure
3. RV physiology
a. The RV normally has only about one-sixth the myocardial mass of the LV. Primary means of
compensation is heart rate increase.
b. Normal RV ejection fraction is 40%–45%.
c. Conduit to a low pressure system in the pulmonary vasculature (normally)
d. Physiologic deficits:
i. Preload dependent
ii. Interdependent on LV and septal contribution to contraction
iii. Highly sensitive to acute increases in afterload. PVR elevations may be:
(a) Drug induced (e.g., α1 agents, protamine)
(b) Caused by ventilator settings (high positive end-expiratory pressure [PEEP], high tidal
volumes)
(c) Caused by hypoxia
(d) Caused by hypercarbia
(e) Caused by pulmonary embolism
iv. Poor elastic response to acute preload increases compared with LV
v. Atypical geometric form contributes to:
(a) Difficulty in objective assessment of RV function by ECHO
(b) Increases susceptibility to further inefficiency/dysfunction when severely dilated

ACCP Updates in Therapeutics® 2022: Critical Care Pharmacy Preparatory Review and Recertification Course

562
Cardiovascular Critical Care I

4. Etiologies of RV failure

Box 5. Etiologies of RV Failure (Exp Clin Cardiol 2013;18:27-30; J Am Coll Cardiol 2010;56:1435-46)

Arrhythmias
Cardiac tamponade
Congenital heart disease
Heart transplantation (particularly if prolonged ischemic time)
Hypovolemia
Hypoxia
LV dysfunction
Mitral valve disease
Post-cardiac surgery
Pulmonary embolism
Pulmonary hypertension
Pulmonary regurgitation
Right coronary artery infarction/ischemia
RV overload
Sepsis
Tricuspid regurgitation/stenosis

D. Diagnostic Tests for New or Worsening HF – In addition to routine chemistry and CBC tests, additional
testing may include:
1. Liver function tests (may be indicative of congestive hepatopathy, if elevated)
2. 12-lead ECG
3. Troponin
4. Left heart catheterization if suspected new ischemic contribution
5. BNP or N-terminal pro-brain natriuretic peptide (NT-proBNP) (elevations may help in the diagnosis of
acutely decompensated HF in scenarios of uncertainty)
6. Transthoracic or transesophageal ECHO
7. Invasive hemodynamic monitoring (to guide volume optimization and dosing response to inotropes or
vasopressors)
8. For less common cardiomyopathies, noninvasive imaging (i.e., cardiac magnetic resonance imaging
[MRI]) and/or myocardial biopsy may be required.

E. General Management Considerations


1. Assess volume status.
a. In selected cases of hypotension where HF is not known to be the exclusive culprit, small volume
fluid challenges (250–500 mL intravenous fluid bolus) or passive leg raising (PLR) maneuvers may
help show whether a patient is volume responsive.
b. PLR is a reliable and alternative way of evaluating volume status, without the need to administer a
fluid challenge (Curr Opin Crit Care 2017;23:237-43).
c. Physical examination will generally guide fluid status decision.
2. Once other causes of symptomatic congestion and/or low perfusion at rest have been ruled out and
a diagnosis of decompensated HF is deemed likely, the following should be considered: If volume
overloaded, provide intravenous diuretics equal to the patient’s home oral dose (or alternatively up to
2–2.5 times the patient’s home dose may be used).
a. Monitor for clinical improvement in symptoms and adequate urine output.

ACCP Updates in Therapeutics® 2022: Critical Care Pharmacy Preparatory Review and Recertification Course

563
Cardiovascular Critical Care I

b. Proactive evaluation of urine output adequacy should be used to facilitate optimization/escalation


of diuretic regimen.
c. Increase in loop diuretic dose and/or addition of sequential nephron blockade (i.e., metolazone) may
be considered as needed to achieve the desired urine output and fluid balance.
d. For patients unresponsive to diuretic escalation as above or those with end-stage renal disease,
hemodialysis and isolated ultrafiltration can be used for fluid removal.
3. Evaluate for use of vasodilator therapy.
a. If normotensive or hypertensive (e.g., SBP greater than 100 mm Hg) in the presence of acute
pulmonary edema (despite diuretic therapy):
i. Continuous infusion intravenous vasodilators (e.g., nitroglycerin or nitroprusside) should be
considered.
ii. If the presence of high afterload is confirmed (systemic vascular resistance), then nitroprusside
may be warranted more than other vasodilators. However, careful attention to patient selection
is important because nitroprusside elimination is dependent on both hepatic metabolism
and renal clearance. Note that nitroprusside is converted to nitric oxide and cyanide, with
subsequent conversion to thiocyanate. Signs of potential toxicity include metabolic acidosis
and mental status changes. In addition, patients receiving nitroprusside should have an arterial
line for close blood pressure monitoring and typically a Swan-Ganz catheter in place.
iii. Pulmonary vasodilators may be required in patients with evidence of pulmonary arterial
hypertension: inhaled nitric oxide, epoprostenol, or alternatively phosphodiesterase type 5
inhibitors (sildenafil). Inhaled nitric oxide and epoprostenol may also be considered as short-
term adjuncts in patients with right ventricular dysfunction and pulmonary hypertension in the
perioperative setting. The delivery system and high cost associated with inhaled nitric oxide is
a limitation relative to other options.
b. If hypotensive (e.g., SBP of 100 mm Hg or less): Can consider vasodilators, but with caution
4. Monitor for clinical improvement of symptoms and adequate urine output.
a. Proactive evaluation of urine output adequacy should be used to facilitate optimization/escalation
of diuretic regimen. An optimized diuretic dose should produce 100–150 mL/hour of urine output
at 6 hours or 50–70 mEq/L of urine sodium at 2 hours.
b. An increase in loop diuretic dose and/or the addition of sequential nephron blockade (i.e.,
metolazone) may be considered as needed to achieve the desired urine output and fluid balance.
Administration of loop diuretics as a continuous infusion may also be considered.
5. Evaluate for use of inotrope therapy
a. If inadequate response on escalation of diuretics (and vasodilators, if appropriate), inotrope therapy
should be considered, accounting for any concurrent physiologic considerations (e.g., right HF,
pulmonary hypertension, ischemia, or valvular disease)
b. Dobutamine and milrinone are most often used for suspected or confirmed low cardiac output
states. Milrinone may be preferred more than dobutamine in the presence of recent administration
of β-blockade or in the setting of concomitant pulmonary hypertension (because of its post-β
receptor effects and potent vasodilatory properties, respectively). In terms of pharmacokinetic
differences, dobutamine has a faster onset and shorter half-life (2 min) compared with milrinone
(half-life of 3 hrs with renal-dependent elimination). However, no differences in clinical efficacy
or safety have been observed between agents. Both agents should be closely monitored for the
development of proarrhythmias and worsening ischemia.

ACCP Updates in Therapeutics® 2022: Critical Care Pharmacy Preparatory Review and Recertification Course

564
Cardiovascular Critical Care I

VII.  VALVULAR HEART DISEASE

A. Valvular heart disease is an important comorbid condition that must be considered in hemodynamic
management of the critically ill patient. Valvular heart disease can also independently lead to the presenting
critical illness.

B. Among the four cardiac valves, several etiologiesa contribute to the manifested pathology; however, the
depth and breadth of these etiologies are beyond the scope of this chapter. Nevertheless, conditions that may
require repair/replacement include those listed in Table 12.

Table 12. Conditions That May Require Valvular Repair/Replacement


Narrowing at the opening of the valve(s)
Stenosis
Can lead to concurrent regurgitation
Regurgitation “Leaky” valve(s) resulting in less blood pumping forward through the heart
Prolapse “Floppy” valve(s) with part of valve not working
Endocarditis Infection of one or more valves
Malformation Often occurs at birth when the valve (or valves) is defective
Valvular disease secondary to rheumatic heart disease is a rare but possible contributor.
a

Table 13. Valvular Disease Characteristics and Management Considerationsa,b (Circulation 2017; Available at http://
circ.ahajournals.org/content/early/2017/03/14/CIR.0000000000000503)
Valvular
Management Considerations
Disease Type
• One of the most common and serious valvular diseases seen in the ICU
• As stenosis and disease progresses, severe/critical AS limits the heart to a fixed stroke
volume; this inability to increase stroke volume occurs despite intrinsic or extrinsic attempts
to compensate (i.e., increased chronotropy or inotropy) and often only increases myocardial
oxygen demand without improving delivery
• Must be extremely cautious in approaching and reacting to invasive hemodynamic variables
(particularly cardiac output and cardiac index) – adding agents with inotropic/chronotropic
effects may expedite demand ischemia and an acute MI because of the physiologic inability
to increase cardiac output with a fixed partial LV outflow tract obstruction
• For reasons similar to those previously listed, treating hypertension with afterload-reducing
agents must be done judiciously because as afterload (SVR) decreases, cardiac output cannot
Aortic stenosis
increase
(AS)
• Can coexist with concurrent aortic insufficiency/regurgitation because the stenotic or
calcified valve leaflets may no longer move and come together (coapt) well
• These patients may be at risk of developing mitral regurgitation and subsequent increases in
left atrial pressures (increasing risk of AF) because of increased LV filling pressures
• Must be cautious in decreasing HR in sinus tachycardia because this can be a primary means
of compensation
• Patients need adequate preload; however, they can become symptomatic even with slight
volume overload. As an example, atrial fibrillation can be very detrimental simply because it
decreases preload to the ventricle
• Patients with severe aortic stenosis and systolic HF (described as low output – low gradient)
have a poorer prognosis

ACCP Updates in Therapeutics® 2022: Critical Care Pharmacy Preparatory Review and Recertification Course

565
Cardiovascular Critical Care I

Table 13. Valvular Disease Characteristics and Management Considerationsa,b (Circulation 2017; Available at http://
circ.ahajournals.org/content/early/2017/03/14/CIR.0000000000000503) (continued)
Valvular
Management Considerations
Disease Type
• Also known as aortic insufficiency (AI)
Aortic • Must be cautious in decreasing HR in sinus tachycardia because this can be a primary means
regurgitation of compensation to maintain adequate cardiac output
(AR) • Patients need adequate preload, but the predominant target is to maintain decreased afterload
(SVR) to facilitate forward blood flow and cardiac output
• Contributes to decreased LV filling and increased left atrial pressures; can increase risk of
AF and secondary pulmonary hypertension
• Increasing diastolic filling time and avoiding tachycardias can facilitate stabilization until
valve is corrected
Mitral stenosis
• Use of selective pulmonary vasodilators (i.e., inhaled nitric oxide or inhaled epoprostenol)
(MS)
may be detrimental because these agents can facilitate pulmonary congestion, given the
preexisting pulmonary venous hypertension and elevated left atrial pressures
• Can coexist with concurrent mitral regurgitation because the stenotic or calcified valve
leaflets may no longer move and coapt well
• Primary clinical target is to decrease LV afterload (SVR) to minimize augmentation of MR
and to facilitate forward blood flow. If SVR is too high, blood will travel in the path of least
Mitral resistance until an adequate LV pressure is generated to open the aortic valve (must exceed
regurgitation the systemic diastolic blood pressure)
(MR) • Use of selective pulmonary vasodilators (i.e., inhaled nitric oxide or inhaled epoprostenol)
may be detrimental because these agents can facilitate pulmonary congestion, given the
preexisting pulmonary venous hypertension and elevated left atrial pressures
• Likely to influence pulmonary artery catheter assessments of cardiac output by way of
Tricuspid thermodilution technique
regurgitation • Can be influenced by infectious causes and presence of indwelling transvenous catheters
(TR) or leads; but moderate or severe TR is more commonly a marker of RV overload and
dysfunction
a
Infective endocarditis can cause progressive valve disease, leading to regurgitant flow and impaired valve leaflet coaptation; however, it can also lead to near obstruction
in some cases.
b
Degree of valvular disease is graded as mild, moderate, or severe, as defined by objective ECHO or catheter-based assessments.

ACCP Updates in Therapeutics® 2022: Critical Care Pharmacy Preparatory Review and Recertification Course

566
Cardiovascular Critical Care I

C. Procedural or Surgical Correction

Table 14. Procedural or Surgical Correction (Circulation 2017; Available at http://circ.ahajournals.org/content/


early/2017/03/14/CIR.0000000000000503)
Balloon
• Performed by percutaneous intervention as a temporizing intervention
valvuloplasty
• May entail direct surgical repair of a damaged valve leaflet or implantation of a ring at the
Valve repair
valve annulus to facilitate improved coaptation of a regurgitant valve
Tissue • Made of animal or human tissue
(bioprosthetic) • Usually does not require long-term anticoagulation (see Table 15)
valve • Does not last as long as a mechanical valve (may last 10–15 yr)
• Made of synthetic materials (newer valves use ceramic or carbon)
• They are durable and generally unlikely to need replacement
Mechanical valve • Require lifelong anticoagulation
• Warfarin is currently the only anticoagulant approved for use by the FDA in patients with
mechanical heart valves (see Table 15)
• Made of animal tissue (bioprosthetic) and attached to a wire frame stent and placed using
Transcatheter
catheter inside the old aortic valve
aortic valve
• This may be considered in patients who are at higher perioperative risk for surgical aortic
replacement
valve replacement or when surgery is not an option
(TAVR)
• Anticoagulation and/or antiplatelet agents are required for at least a short time after TAVR
• Minimally invasive technique for treatment of symptomatic chronic moderate-severe or
severe primary (degenerative) mitral regurgitation in patients at prohibitive surgical risk
Transcatheter • A leaflet repair device (MitraClip) is currently the only FDA approved device for this
mitral valve repair indication
(TMVR) • This device uses a cobalt chromium clip to suture the regurgitant orifices of the mitral
valve leaflets together, thereby increasing coaptation
• Anticoagulation and/or antiplatelet agents are required for at least a short time after TMVR

D. Anticoagulation
1. Three guidelines exist regarding valve anticoagulation, with varying agreement in the recommendations.
2. Prosthetic mitral valves have increased risk of thrombosis (blood flow across the valve is passive and
occurs during diastole) versus aortic valves, where blood flow across the valve is active occurring
during systole.

ACCP Updates in Therapeutics® 2022: Critical Care Pharmacy Preparatory Review and Recertification Course

567
Cardiovascular Critical Care I

Table 15. Summary of Anticoagulation Recommendations for Patients with Prosthetic Valves (Circulation 2017;
Available at http://circ.ahajournals.org/content/early/2017/03/14/CIR.0000000000000503)
Class of Class of
Warfarin, Antiplatelet
Procedure Recommendation/ Recommendation/
Target INR Range Therapy
Level of Evidencea Level of Evidencea
2–3 (bileaflet or current-
generation single-tilting
IB
disc and no risk factors
for thromboembolismb) Aspirin 81 mg daily
2.5–3.5 (older-generation may be considered
[i.e., ball-in-cage valve, if indicated for
Mechanical AVR IIbB
or with additional IB antiplatelet therapy
risk factors for and bleeding risk
thromboembolismb]) is low
1.5–2 (On-X® valve
and no thromboembolic IIbB
risk factorsc)
Aspirin 81 mg daily
may be considered
2–3 for ≥ 3 mo (and up to
Bioprosthetic AVR IIaB if indicated for IIbB
6 mo if low bleed risk)
antiplatelet and
bleeding risk low
Alternative
to warfarin:
2–3 for ≥ 3 mo if low
Transcatheter AVR IIbB Clopidogrel 75 mg IIbB
bleed risk
daily for 6 mo and
aspirin 81 mg daily
Aspirin 81 mg daily
may be considered
if indicated for
Mechanical MVR 2.5–3.5 IB IIbB
antiplatelet therapy
and bleeding risk
is low
Bioprosthetic 2–3 for ≥ 3 mo (and up to
IIaB Aspirin 81 mg daily IIaB
MVR 6 mo if low bleed risk)
a
Atrial fibrillation, previous thromboembolism, left ventricular dysfunction, or hypercoagulable conditions.
b
Class of Recommendations: I = evidence and/or general agreement that a given treatment or procedure is beneficial, useful, effective; II = conflicting evidence and/
or a divergence of opinion about the usefulness/efficacy of the given treatment or procedure; IIa = weight of evidence/opinion is in favor of usefulness/efficacy; IIb =
usefulness/efficacy is less well established by evidence/opinion; III = evidence of general agreement that the given treatment or procedure is not useful/effective and, in
some cases, may be harmful; Levels of Evidence: A = data derived from several randomized clinical trials or meta-analyses; B = data derived from a single randomized
trial or large nonrandomized studies; C = consensus of opinion of the experts and/or small studies, retrospective studies, registries.
c
Target INR 2-3 for the first 3 mo.

E. Valve Thrombosis
1. Highest risk within the first year after replacement
2. Most predominant risk is in mechanical prosthetic valves, but can also occur with bioprosthetic valves
3. Treatment involves systemic fibrinolytic treatment or surgical reoperation with varying success rates –
Both carry significant risks of morbidity and mortality, including:

ACCP Updates in Therapeutics® 2022: Critical Care Pharmacy Preparatory Review and Recertification Course

568
Cardiovascular Critical Care I

a. Thrombolysis (predominantly recommended for right-sided valve thrombosis)


i. Cardiac tamponade
ii. Stroke/transient ischemic attack
iii. Systemic embolization
iv. Major bleeding
v. Death
b. Surgery (predominantly recommended for left-sided valve thrombosis)
i. Cardiac tamponade
ii. Stroke/transient ischemic attack
iii. Renal failure
iv. Heart block
v. Systemic embolization
vi. Prolonged ventilation
vii. Major bleeding
viii. Death

F. Other Cardiac Functional Defects


1. Left ventricular outflow tract (LVOT) obstruction (Circulation 2020;142:e558-e631; Circulation
2008;117:429-39; J Am Coll Cardiol 2000;36:1344-54)
a. Hypertrophic obstructive cardiomyopathy (HOCM)
i. Genetic disease leading to hypertrophy of the LV (particularly the ventricular septum) with or
without the presence of LV outflow tract obstruction
ii. LVOT obstruction is of greatest concern when HOCM exists with systolic anterior motion
(SAM) (see text that follows); however, in HOCM, LVOT obstruction can exist in the absence of
SAM because of septal hypertrophy; nonetheless, acute clinical management is predominantly
the same as outlined in 1.a.i–iii and in 1.b.i.
iii. Treatment of HOCM without SAM relies heavily on the management of contributors to
myocardial hypertrophy and diastolic HF.
(a) Non-vasodilating β-blockers are first-line agents in the treatment of patients with HOCM
and are commonly titrated to heart rate goals of 60–65 beats/minute.
(b) Dihydropyridine calcium channel blockers (particularly verapamil) are recommended
in patients with contraindications to β-blockade or in those without advanced HF or
bradycardia.
(c) For recommendations specific to HOCM (patients also with SAM), see “b” in the text that
follows.
(d) Patients with HOCM may be at increased risk of sudden cardiac death. Evaluation should
consider the patient’s candidacy for implantable cardioverter-defibrillator placement by
ambulatory ECG (Holter) monitoring at least biannually.
iv. Treatment may be indicated by surgical septal myectomy or catheter-directed alcohol ablation.
b. SAM of the mitral valve
i. SAM and LVOT obstructions result in a systolic outflow tract obstruction and are commonly
associated with HOCM but can also occur in other clinical scenarios, particularly after cardiac
surgery.
ii. Can lead to severe cardiogenic shock
iii. SAM is more of a dynamic obstruction in which the degree of obstruction and flow gradient is
dependent on heart rate, cardiac contractility, and ventricular preload volume.

ACCP Updates in Therapeutics® 2022: Critical Care Pharmacy Preparatory Review and Recertification Course

569
Cardiovascular Critical Care I

(a) In an underfilled LV, there is physically less distance between the mitral valve and septum,
thus generating an increased risk of obstruction because the LVOT is generally narrower
at the onset of systole, particularly if the mitral valve leaflet is affected.
(b) Increasing cardiac contractility and heart rate increases LVOT obstruction and gradient
by inducing a stronger contraction, increasing the contact between the septum and mitral
leaflets, and increasing the rate of systolic attempts.
iv. For patients with SAM who have a potential for obstructive physiology, management involves
maintaining normal or increased LV preload and low heart rates.
(a) Acute hypotension is best managed with phenylephrine or vasopressin (pure
vasoconstrictors) to selectively increase SVR without increasing contractility or heart
rate. Concomitant β-blocker use may also be considered to improve cardiac filling.
(b) Inotropes and vasopressors that mediate increases in heart rate or contractility should be
avoided, if possible, because they may be harmful and worsen the LVOT.
(c) Afterload-reducing agents (e.g., ACE inhibitors, ARBs, non-dihydropyridines) should be
used with caution (if at all).
2. Septal defects (atrial or ventricular)
a. Septal defects can be acquired (i.e., postinfarction ventricular septal defect) or can be congenital.
b. Diagnosed predominantly by ECHO using a bubble study. If the patient presents in a seemingly low
cardiac output state, a left-to-right intracardiac shunt should be suspected if mixed venous oxygen
saturation (Svo2) saturations are greater than Scvo2 saturations.
c. Important principles
i. Goals include minimizing the degree of intracardiac shunt while maintaining adequate cardiac
output. It is generally favored to accept a right-to-left intracardiac shunt while recognizing that
partial pressure of arterial oxygen (Pao2) saturations will be somewhat decreased and reflective
of venous and arterial blood mixing in the LV before ejection.
ii. Decreasing right-sided cardiac filling pressures can augment left-to-right intracardiac shunting
of blood. Administration of venodilators (i.e., nitrates) or aggressive diuresis could augment
left-to-right intracardiac shunts and lead to clinical deterioration.
iii. Intravenous medications should preferably be filtered to minimize the risk of air/particulate
embolus traveling through to the left side of the heart, being ejected, and causing a potential
stroke.
d. Treatment may include surgical correction or percutaneous catheter placement of a closure device.

Patient Case (Continued)

8. J.M. is no longer in AF but remains in cardiogenic shock. The cardiac intensive care unit team has consulted
the cardiothoracic surgeons for evaluation of his mitral valve disease and has considered advanced HF
therapies. In the patient’s decompensated state, he would likely need additional optimization if he were to
undergo surgery. Which temporary means of mechanical circulatory support (MCS) might be most favorable
to help stabilize this patient’s cardiogenic shock in the setting of moderate to severe mitral regurgitation?
A. Venoarterial extracorporeal membrane oxygenation (ECMO)
B. Venovenous ECMO
C. Intra-aortic balloon counterpulsation
D. None; the patient likely requires urgent surgery.

ACCP Updates in Therapeutics® 2022: Critical Care Pharmacy Preparatory Review and Recertification Course

570
Cardiovascular Critical Care I

VIII.  ADVANCED THERAPIES FOR HEART FAILURE AND CARDIOGENIC SHOCK

A. The goals behind advanced therapies can be thought of as dynamic, depending on patient progression and
resolution or presentation of comorbid conditions.

B. Dynamic Progression of HF Advanced Therapies

Figure 10. Dynamic progression of heart failure advanced therapies.

C. Mechanical Circulatory Support (MCS) (J Heart Lung Transplant 2013;32:157-87)


1. Intent, duration, and type of MCS support depend on several factors, including patient acuity,
comorbidities, and prognosis.
2. Common terms
a. Destination therapy – Formal designation for patients who meet the criteria for long-term mechanical
support but who are not a transplant candidate because of relative or absolute contraindications
b. Bridge to transplantation – Formal designation for patients eligible to be listed as candidates for
heart transplantation
c. Bridge to candidacy and bridge to recovery – Used in concept and are not formally recognized
abbreviations but describe the approach to temporary MCS when short-term left ventricular assist
devices (LVADs) are used to support a patient until a long-term prognosis can be determined,
which may include explantation with recovery, implantation of long-term durable LVAD support,
heart transplantation, or palliative care

ACCP Updates in Therapeutics® 2022: Critical Care Pharmacy Preparatory Review and Recertification Course

571
Cardiovascular Critical Care I

Table 16. Mechanical Interventions for Shock


Short to Intermediate Term
• Placed by femoral arterial catheter or percutaneously through the axillary artery and
advanced up to the aorta
• Inflation enables diastolic augmentation of systemic blood pressure to improve vital
organ and coronary perfusion pressures
• Deflation facilitates selective afterload during systole to ease cardiac output (does not
technically increase cardiac output)
• Can be set to trigger from ECG, pacer, or arterial line pressure, or can be manually set
Intra-aortic balloon
• Tachyarrhythmias and aortic regurgitation/insufficiency are not well supported with this
pump (IABP)
means of MCS
counterpulsation
• Level of support coincides with timing of inflation/deflation per related heartbeat; for
example:
1:1 = one inflation/deflation per every heartbeat (maximal support)
1:4 = one inflation/deflation for every fourth heartbeat (less support)
• When setting duration in deflated state increases (providing less support),
the thrombosis risk associated with the IABP increases, commonly requiring
anticoagulation
• One common example is Impella: intraluminal axial support that provides varying
degrees of LV output support (2.5, CP, and 5.0) and RV output support (RP)
• Another example is TandemHeart: left atrium-to-femoral artery bypass using
transseptal cannulation
• May be considered in cardiogenic shock or as temporary support during high-risk PCI
Percutaneous VAD
• Anticoagulation regimen is a common topic of debate and medication safety discussion.
In general, UFH is accepted as the standard anticoagulant for percutaneous VADs, and
bivalirudin and argatroban can be used in patients with a history of or concern for HIT
• Complications may include hemolysis/bleeding, arrhythmias, and migration and/or
malposition of the catheter/cannula
Extracorporeal or
Examples include CentriMag, BVS 5000, and AB5000 ventricle
paracorporeal VAD
• Similar to cardiopulmonary bypass in which large-bore cannulas drain venous blood
that is pumped through an oxygenator, where it is oxygenated and/or cleared of carbon
dioxide and then actively pumped back into the body
• Modality of support depends on the means of vascular cannulation
Extracorporeal life Venoarterial: Removal of venous blood from the vena cava with circulation through the
support (ECLS) ECMO circuit and delivery in retrograde fashion up the aorta; potentially indicated
or extracorporeal for primary cardiogenic shock, cardiopulmonary failure, and post-cardiopulmonary
membrane circulatory shock
oxygenation (ECMO) Venovenous: Removal of venous blood from the vena cava with circulation through
the ECMO circuit and delivery back to the right atrium; potentially indicated for
hypoxic respiratory failure owing to any cause, hypercarbic respiratory failure with
bronchospastic disease or other cause of carbon dioxide (CO2) retention, or severe air
leak syndromes
Long term
Implantable LVADs Examples include the Heartmate II, HeartWare, Heartmate III
Total artificial heart Example includes Syncardia TAH (biventricular support)
LVAD = left ventricular assist device; VAD = ventricular assist device.

ACCP Updates in Therapeutics® 2022: Critical Care Pharmacy Preparatory Review and Recertification Course

572
Cardiovascular Critical Care I

d. Anticoagulation considerations
i. Anticoagulation strategies are specific to the proprietary device, and many institutions have
standardized protocols.
ii. Safety and efficacy of DOACs in patients with ventricular assist devices (VADs) have not been
well established.
3. Complications of MCS (other than device failure)
a. Bleeding
i. Common sources
(a) Nasal/upper airway
(b) Gastrointestinal (GI)
(c) Arteriovenous malformations in one of the previously stated locations
(d) Hemolysis
ii. Workup and/or acute treatment options
(a) Laboratory workup
(1) Prothrombin time/INR/aPTT
(2) Increase frequency of hemoglobin/hematocrit evaluation.
(3) Multimeric von Willebrand testing for acquired von Willebrand factor deficiency
(some clinicians believe that all patients with prolonged continuous flow MCS develop
acquired von Willebrand disease)
(4) If no overt sign of bleeding – Consider hemolysis workup.
(b) If suspected/confirmed bleeding, hold anticoagulation and consider reversal with
caution. Consider any history of bleeding/clotting-related problems and indications for
antithrombotic therapy in addition to MCS.
(c) Obtain appropriate consults, and consider common interventions.
(d) Ear, nose, and throat:
• Evaluate for source control and/or cauterization.
• Mupirocin 2% (Bactroban) every 12 hours to each nostril to maintain moist nasal
passages
• Oxymetazoline 0.05% (Afrin) to each nostril every 12 hours as needed for epistaxis
(e) Gastroenterology
(1) Proton pump inhibitor continuous infusion until location of GI source identified or for
up to 72 hours. Intermittent intravenous dosing may also be considered.
(2) Enteroscopy with or without colonoscopy; may consider one or more of the following:
• Angiography and cautery
• Balloon-assisted enteroscopy
• Video-capsule enteroscopy
• Surgery
iii. Long-term management
(a) Consider decreasing anticoagulation/antiplatelet therapy intensity.
(b) Refractory bleeding despite previously stated interventions
(1) Consider role of oral antifibrinolytics (i.e., aminocaproic acid).
(2) Consider role of desmopressin or von Willebrand factor replacement if confirmed
acquired von Willebrand disease and if LVAD speed cannot be further decreased.
(3) If GI arteriovenous malformations are present that are not amenable to intervention,
octreotide 50 mcg subcutaneously every 8 hours may be considered.
b. Hemolysis and/or thrombosis
i. Hemolysis may be the presenting symptom of an underlying process, including infection,
pump thrombosis, or other mechanical or physiologic dysfunction.

ACCP Updates in Therapeutics® 2022: Critical Care Pharmacy Preparatory Review and Recertification Course

573
Cardiovascular Critical Care I

ii. Common presentation includes:


(a) Nonhemorrhagic anemia
(b) Urine color changes with appearance of hematuria; in severe cases, can be brown or black
(c) Hyperkalemia
(d) LVAD pump alarms
iii. Workup for contributing factors:
(a) Evaluate LVAD and cardiac function for contributing factors, including documentation
and alarm history for suction events, power spikes, speed changes, volume status, RV
function, arrhythmias
(1) Evaluate cannula(e) position and evaluate for obstruction/thrombus by ECHO or
computed tomography (CT).
(2) Evaluate RV function by ECHO.
(3) Evaluate for intra-device thrombosis by ECHO ramp testing (Echocardiography
2014;31:E5-9; J Am Coll Cardiol 2013;62:2149-50; J Am Coll Cardiol 2012;60:1764-75).
(b) Ensure adherence to anticoagulation regimen according to patient’s established goals.
(c) Evaluate laboratory values to establish the presence and degree of hemolysis and to
identify any other potential contributors.
(1) Lactate dehydrogenase (LDH); normal values are 300–600 IU/L for most forms of
MCS.
(2) Haptoglobin
(3) Plasma-free hemoglobin
(4) Blood cultures (an association has been identified with bacteremia and hemolysis in
patients with LVADs – in some cases, presenting as thrombosis).
(5) Urine assessment – Send baseline urinalysis, urine culture, and appearance of
changes daily to assess for improvement. Likely hemolysis, as evidenced by the
presence of casts and color changes (darkened tea, red, or black are highly suggestive
of hemolysis).
(d) Other laboratory values/considerations
(1) Hypercoagulable states (i.e., heparin-induced thrombocytopenia [HIT])
(2) CBC with differential
(3) Reticulocyte count
iv. Treatment
(a) Address any evidence of mechanical dysfunction by adjusting speed/flow rates, if possible.
(b) Consider medical optimization of RV function.
(c) If hypovolemic, give volume challenge. Consider alkalinization of urine and optimize
fluid status (sodium bicarbonate 150 mEq/1000 mL of sterile water) at 0.5–1 mL/kg/hour;
treat to a goal urine pH of greater than 7.5 to avoid additional hemolysis/hemoglobinuria-
related acute kidney injury.
(d) Evaluate and optimize anticoagulation strategy.
(1) Ensure therapeutic anticoagulation with heparin or warfarin.
(2) Antithrombotic therapy that is more aggressive may be appropriate. Optimal acute
antithrombotic strategies, although not yet defined, may include heparin infusion,
glycoprotein (GP) IIb/IIIa infusions, parenteral direct thrombin inhibitors, or
thrombolytics.
(3) Severe hemolysis can potentiate platelet activation – Can consider GP IIb/IIIa
antagonist therapy (see Table 5), depending on bleeding risks and potential surgical
plan.

ACCP Updates in Therapeutics® 2022: Critical Care Pharmacy Preparatory Review and Recertification Course

574
Cardiovascular Critical Care I

(e) Reassess long-term antithrombotic strategy.


(1) Consider augmentation of antiplatelet therapy, or increase the INR therapeutic goal
range.
(2) If thought to be related to a concurrent infection, can consider acutely increasing
anticoagulation goals until infection control is gained
c. Infection
i. LVAD infections are often complex and have been characterized by the International Society for
Heart & Lung Transplantation in the following manner (J Heart Lung Transplant 2011;30:375-
84):
(a) VAD-specific infections
(1) Pump and/or cannula infections
(2) Pocket infections
(3) Percutaneous driveline infections
• Superficial infection
• Deep infection
(b) VAD-related infections
(1) Infective endocarditis
(2) Bloodstream infections that may be VAD related or non–VAD related
(3) Mediastinitis
• Sternal wound infection: Surgical site infection-organ space
• VAD pocket infection (continuous with mediastinum or already situated in the
mediastinum, depending on the device used)
• Other causes of mediastinitis, perforation of the esophagus
(4) Non-VAD infections
• Lower respiratory tract infection
• Cholecystitis
• Clostridium difficile infection
• Urinary tract infection
ii. Antibiotic treatment
(a) Antibiotic coverage should account for site of suspected infection, previous pathogens
and susceptibilities, proximity to driveline site or VAD pocket, and any other potential
exposure or bacteremia secondary to the procedure. Prophylactic antibiotic coverage
for VAD-related or non–VAD-related surgical procedures should account for site of
procedure, previous infections, proximity to driveline site or VAD pocket, and any other
potential exposure or bacteremia secondary to the procedure. In some circumstances,
this requires broader prophylactic antibiotic coverage (Interact Cardiovasc Thorac Surg
2012;14:209-14).
(b) Treatment duration depends largely on the type of infection. However, if the infection is
VAD related or VAD specific, prolonged antimicrobial therapy (more than 4 weeks) is
commonly used.
(c) Because LVAD exchange is not without considerable risk, long-term oral antibiotic
suppression therapy may be considered for some infections.

D. Heart Transplantation (J Heart Lung Transplant 2010;29:914-56)


1. Transplantation remains the gold-standard treatment for end-stage HF.
2. Many variables limit the utility of heart transplantation, foremost of which is donor availability and
donor-recipient tissue compatibility. These limitations (and potentially others) may make long-term
MCS a more viable option until heart transplantation becomes an option.

ACCP Updates in Therapeutics® 2022: Critical Care Pharmacy Preparatory Review and Recertification Course

575
Cardiovascular Critical Care I

3. Other limitations may include recipient characteristics of:


a. Mental health
b. Social support
c. Adherence to medication and appointments
d. Severe pulmonary hypertension
e. Cancer
f. Infection
g. Tobacco or ethanol use
h. Illicit drug use history
4. Additional considerations associated with thoracic transplantation in the ICU, such as immunosuppression,
rejection, and other complications, are beyond the scope of this review.

Acknowledgment: ACCP wishes to acknowledge Dr. Erik E. Abel and Dr. James C. Coons for their contributions
to this chapter.

ACCP Updates in Therapeutics® 2022: Critical Care Pharmacy Preparatory Review and Recertification Course

576
Cardiovascular Critical Care I

REFERENCES

1. Acharya MN, Som R, Tsui S. What is the opti- 11. Collet JP, Thiele H, Barbato E, et al. 2020 ESC
mum antibiotic prophylaxis in patients undergoing guidelines for the management of acute coro-
implantation of a left ventricular assist device? nary syndromes in patients presenting without
Interact Cardiovasc Thorac Surg 2012;2:209-14. persistent ST-segment elevation. Eur Heart J
2. Al-Khatib SM, Stevenson WG, Ackerman MJ, et 2021;42:1289-367.
al. 2017 AHA/ACC/HRS guideline for manage- 12. Cooper HA, Panza JA. Cardiogenic shock. Cardiol
ment of patients with ventricular arrhythmias and Clin 2013;4:567-80, viii.
the prevention of sudden cardiac death: execu- 13. Costanzo MR, Dipchand A, Starling R, et al.
tive summary: a report of the American College The International Society of Heart and Lung
of Cardiology/American Heart Association Transplantation guidelines for the care of heart
Task Force on Clinical Practice Guidelines and transplant recipients. J Heart Lung Transplant
the Heart Rhythm Society. J Am Coll Cardiol 2010;8:914-56.
2018;72:1677-749. 14. De Backer D, Biston P, Devriendt J, et al.
3. Amsterdam EA, Wenger NK, Brindis RG, et al. Comparison of dopamine and norepinephrine in the
2014 AHA/ACC guideline for the management treatment of shock. N Engl J Med 2010;9:779-89.
of patients with non–ST-elevation acute coronary 15. Dickinson O, Chen LY, Francis GS. Atrial
syndromes: a report of the American College fibrillation and heart failure: intersecting popula-
of Cardiology/American Heart Association tions, morbidities, and mortality. Heart Fail Rev
Task Force on Practice Guidelines. Circulation 2014;3:285-93.
2014;25:e344-426. 16. Edhouse J, Morris F. ABC of clinical electrocardi-
4. Bangash MN, Kong ML, Pearse RM. Use of ography: broad complex tachycardia-part II. BMJ
inotropes and vasopressor agents in critically ill 2002;7340:776-9.
patients. Br J Pharmacol 2012;7:2015-33. 17. Edhouse J, Thakur RK, Khalil JM. ABC of clini-
5. Baran DA, Grines CL, Bailey S, et al. SCAI clini- cal electrocardiography. Conditions affecting the
cal expert consensus statement on the classification left side of the heart. BMJ 2002;7348:1264-7.
of cardiogenic shock: this document was endorsed 18. Esberger D, Jones S, Morris F. ABC of clinical
by the American College of Cardiology (ACC), the electrocardiography. Junctional tachycardias. BMJ
American Heart Association (AHA), the Society 2002;7338:662-5.
of Critical Care Medicine (SCCM), and the 19. Feldman D, Pamboukian SV, Teuteberg JJ, et al.
Society of Thoracic Surgeons (STS) in April 2019. The 2013 International Society for Heart and Lung
Catheter Cardiovasc Interv 2019;94:29-37. Transplantation Guidelines for mechanical circu-
6. Baron TH, Kamath PS, McBane RD. Management latory support: executive summary. J Heart Lung
of antithrombotic therapy in patients under- Transplant 2013;2:157-87.
going invasive procedures. N Engl J Med 20. Fifer MA, Vlahakes GJ. Management of symp-
2013;22:2113-24. toms in hypertrophic cardiomyopathy. Circulation
7. Brilakis ES, Banerjee S. Patient with coro- 2008;3:429-39.
nary stents needs surgery: what to do? JAMA 21. Francis GS, Bartos JA, Adatya S. Inotropes. J Am
2013;14:1451-2. Coll Cardiol 2014;20:2069-78.
8. Brilakis ES, Banerjee S. Perioperative management 22. Freeman JV, Reynolds K, Fang M, et al. Digoxin
of drug-eluting stents: the Achilles heel of bridg- and risk of death in adults with atrial fibrilla-
ing. Catheter Cardiovasc Interv 2013;7:1113-4. tion: the ATRIA-CVRN study. Circ Arrhythm
9. Brilakis ES, Patel VG, Banerjee S. Medical man- Electrophysiol 2015;1:49-58.
agement after coronary stent implantation: a 23. Goldschlager N, Epstein AE, Naccarelli GV,
review. JAMA 2013;2:189-98. et al. A practical guide for clinicians who treat
10. Claassens DMF, Vos GJA, Bergmeijer TO, et al. A patients with amiodarone: 2007. Heart Rhythm
genotype-guided strategy for oral P2Y12 inhibitors 2007;9:1250-9.
in primary PCI. N Engl J Med 2019;381:1621-31.

ACCP Updates in Therapeutics® 2022: Critical Care Pharmacy Preparatory Review and Recertification Course

577
Cardiovascular Critical Care I

24. Goodacre S, Irons R. ABC of clinical electrocardi- 33. January CT, Wann LS, Alpert JS, et al. 2014
ography: atrial arrhythmias. BMJ 2002;7337:594-7. AHA/ACC/HRS guideline for the management
25. Grines CL, Bonow RO, Casey DE Jr, et al. of patients with atrial fibrillation: a report of the
Prevention of premature discontinuation of dual American College of Cardiology/American Heart
antiplatelet therapy in patients with coronary artery Association Task Force on Practice Guidelines
stents: a science advisory from the American Heart and the Heart Rhythm Society. Circulation
Association, American College of Cardiology, 2014;23:2071-104.
Society for Cardiovascular Angiography and 34. January CT, Wann LS, Calkins H, et al. 2019
Interventions, American College of Surgeons, and focused update of the 2014 AHA/ACC/HRS
American Dental Association, with representation guideline for the management of patients with
from the American College of Physicians. J Am atrial fibrillation: a report of the American College
Coll Cardiol 2007;6:734-9. of Cardiology/American Heart Association Task
26. Haddad F, Couture P, Tousignant C, et al. The right Force on Clinical Practice Guidelines and the
ventricle in cardiac surgery, a perioperative per- Heart
spective: II. Pathophysiology, clinical importance, 35. Rhythm Society in collaboration with the Society of
and management. Anesth Analg 2009;2:422-33. Thoracic Surgeons. Circulation 2019;140:e125-51.
27. Haddad F, Couture P, Tousignant C, et al. The right 36. Kanei Y, Kwan T, Nakra NC, et al. Transradial
ventricle in cardiac surgery, a perioperative per- cardiac catheterization: a review of access site
spective: I. Anatomy, physiology, and assessment. complications. Catheter Cardiovasc Interv
Anesth Analg 2009;2:407-21. 2011;6:840-6.
28. Hannan MM, Husain S, Mattner F, et al. Working 37. Kato TS, Colombo PC, Nahumi N, et al. Value of
formulation for the standardization of definitions serial echo-guided ramp studies in a patient with
of infections in patients using ventricular assist suspicion of device thrombosis after left ventricu-
devices. J Heart Lung Transplant 2011;4:375-84. lar assist device implantation. Echocardiography
29. Harrigan RA, Jones K. ABC of clinical electrocar- 2014;1:E5-9.
diography. Conditions affecting the right side of 38. Korff S, Katus HA, Giannitsis E. Differential diag-
the heart. BMJ 2002;7347:1201-4. nosis of elevated troponins. Heart 2006;7:987-93.
30. Hillis LD, Smith PK, Anderson JL, et al. 2011 39. Kutty RS, Jones N, Moorjani N. Mechanical com-
ACCF/AHA Guideline for Coronary Artery plications of acute myocardial infarction. Cardiol
Bypass Graft Surgery. A report of the American Clin 2013;4:519-31, vii-viii.
College of Cardiology Foundation/American 40. Lahm T, McCaslin CA, Wozniak TC, et al. Medical
Heart Association Task Force on Practice and surgical treatment of acute right ventricular
Guidelines. Developed in collaboration with the failure. J Am Coll Cardiol 2010;18:1435-46.
American Association for Thoracic Surgery, 41. Levine GN, Bates ER, Blankenship JC, et al. 2011
Society of Cardiovascular Anesthesiologists, and ACCF/AHA/SCAI Guideline for Percutaneous
Society of Thoracic Surgeons. J Am Coll Cardiol Coronary Intervention. A report of the American
2011;24:e123-210. College of Cardiology Foundation/American Heart
31. Holmes DR Jr, Mack MJ, Kaul S, et al. 2012 ACCF/ Association Task Force on Practice Guidelines and
AATS/SCAI/STS expert consensus document on the Society for Cardiovascular Angiography and
transcatheter aortic valve replacement. J Am Coll Interventions. J Am Coll Cardiol 2011;24:e44-122.
Cardiol 2012;13:1200-54. 42. Levy B, Clere-jehl R, Legras A, et al. Epinephrine
32. Horlocker TT, Wedel DJ, Rowlingson JC, et al. versus norepinephrine for cardiogenic shock after
Regional anesthesia in the patient receiving anti- acute myocardial infarction. J Am Coll Cardiol
thrombotic or thrombolytic therapy: American 2018;72:173-82.
Society of Regional Anesthesia and Pain Medicine 43. Lilly LS, Harvard Medical School. Pathophysiology
Evidence-Based Guidelines (Third Edition). Reg of Heart Disease: A Collaborative Project of
Anesth Pain Med 2010;1:64-101. Medical Students and Faculty. Baltimore: Wolters
Kluwer/Lippincott Williams & Wilkins, 2011:xiv.

ACCP Updates in Therapeutics® 2022: Critical Care Pharmacy Preparatory Review and Recertification Course

578
Cardiovascular Critical Care I

44. Lip GYH, Banerjee A, Boriani G, et al. patients with valvular heart disease: a report of the
Antithrombotic therapy for atrial fibrillation. American College of Cardiology/American Heart
Chest 2018;154:1121-201. Association Joint Committee on Clinical Practice
45. Mathew R, Di Santo P, Jung RG, et al. Milrinone Guidelines. Circulation 2021;143:e72-e227.
as compared with dobutamine in the treatment of 56. Overgaard CB, Dzavik V. Inotropes and vaso-
cardiogenic shock. N Engl J Med 2021;385:516-25. pressors: review of physiology and clinical
46. Mebazaa A, Pitsis AA, Rudiger A, et al. Clinical use in cardiovascular disease. Circulation
review: practical recommendations on the man- 2008;10:1047-56.
agement of perioperative heart failure in cardiac 57. Panchal AR, Bartos JA, Cabañas JG, et al.
surgery. Crit Care 2010;2:201. Part 3: adult basic and advanced life sup-
47. Meek S, Morris F. ABC of clinical electrocardi- port: 2020 American Heart Association
ography. Introduction. I-Leads, rate, rhythm, and guidelines for cardiopulmonary resuscitation
cardiac axis. BMJ 2002;7334:415-8. and emergency cardiovascular care. Circulation
48. Meine TJ, Roe MT, Chen AY, et al. Association of 2020;142(16_suppl_2):S366-468.
intravenous morphine use and outcomes in acute 58. Parrillo JE, Dellinger RP. Critical Care Medicine:
coronary syndromes: results from the CRUSADE Principles of Diagnosis and Management in the
Quality Improvement Initiative. Am Heart J Adult. Philadelphia: Mosby Elsevier, 2014:xix.
2005;6:1043-9. 59. Patel AK, Hollenberg SM. Cardiovascular failure
49. Montalescot G, van’t Hof AW, Lapostolle F, et and cardiogenic shock. Semin Respir Crit Care
al. Prehospital ticagrelor in ST-segment eleva- Med 2011;32:598-606.
tion myocardial infarction. N Engl J Med 60. Sherrid MV, Gunsburg DZ, Moldenhauer S, et
2014;11:1016-27. al. Systolic anterior motion begins at low left
50. Moranville MP, Mieure KD, Santayana EM. ventricular outflow tract velocity in obstructive
Evaluation and management of shock States: hypertrophic cardiomyopathy. J Am Coll Cardiol
hypovolemic, distributive, and cardiogenic shock. 2000;4:1344-54.
J Pharm Pract 2011;1:44-60. 61. Turakhia MP, Santangeli P, Winkelmayer WC, et
51. Nahumi N, Jorde U, Uriel N. Slope calcula- al. Increased mortality associated with digoxin
tion for the LVAD ramp test. J Am Coll Cardiol in contemporary patients with atrial fibrillation:
2013;22:2149-50. findings from the TREAT-AF study. J Am Coll
52. O’Gara PT, Kushner FG, Ascheim DD, et al. 2013 Cardiol 2014;7:660-8.
ACCF/AHA guideline for the management of 62. Uriel N, Morrison KA, Garan AR, et al.
ST-elevation myocardial infarction: a report of Development of a novel echocardiography ramp
the American College of Cardiology Foundation/ test for speed optimization and diagnosis of device
American Heart Association Task Force on thrombosis in continuous-flow left ventricular
Practice Guidelines. Circulation 2013;4:e362-425. assist devices: the Columbia ramp study. J Am
53. Ommen SR, Mital S, Burke MA, et al. 2020 AHA/ Coll Cardiol 2012;18:1764-75.
ACC guideline for the diagnosis and treatment 63. Vahanian A, Iung B. The new ESC/EACTS guide-
of patients with hypertrophic cardiomyopathy: lines on the management of valvular heart disease.
a report of the American College of Cardiology/ Arch Cardiovasc Dis 2012;10:465-7.
American Heart Association Joint Committee 64. van Diepen S, Katz JN, Albert NM, et al.
on Clinical Practice Guidelines. Circulation Contemporary management of cardiogenic shock:
2020;142:e558-e631. a scientific statement from the American Heart
54. Ondrus T, Kanovsky J, Novotny T, et al. Right Association. Circulation 2017;136:e232-e268.
ventricular myocardial infarction: From patho- 65. Verhaert D, Mullens W, Borowski A, et al. Right
physiology to prognosis. Exp Clin Cardiol ventricular response to intensive medical therapy
2013;1:27-30. in advanced decompensated heart failure. Circ
55. Otto CM, Nishimura RA, Bonow RO, et al. 2020 Heart Fail 2010;3:340-6.
ACC/AHA guideline for the management of

ACCP Updates in Therapeutics® 2022: Critical Care Pharmacy Preparatory Review and Recertification Course

579
Cardiovascular Critical Care I

66. Whitlock RP, Sun JC, Fremes SE, et al.; American 67. Wijeyeratne YD, Heptinstall S. Anti-platelet
College of Chest P. Antithrombotic and thrombo- therapy: ADP receptor antagonists. Br J Clin
lytic therapy for valvular disease: Antithrombotic Pharmacol 2011;4:647-57.
Therapy and Prevention of Thrombosis, 9th ed: 68. Yancy CW, Jessup M, Bozkurt B, et al. 2017 ACC/
American College of Chest Physicians Evidence- AHA/HFSA focused update of the 2013 ACCF/
Based Clinical Practice Guidelines. Chest AHA guideline for the management of heart fail-
2012;2(suppl):e576S-600S. ure. J Am Coll Cardiol 2017;136:e137-e161.
69. Zimetbaum P. Antiarrhythmic drug therapy for
atrial fibrillation. Circulation 2012;2:381-9.

ACCP Updates in Therapeutics® 2022: Critical Care Pharmacy Preparatory Review and Recertification Course

580
Cardiovascular Critical Care I

ANSWERS AND EXPLANATIONS TO PATIENT CASES

1. Answer: B HF. Answer D (dobutamine 5 mcg/kg/min) is correct,


Given this patient’s CAD, type 2 diabetes, dyslipidemia, and Answers A (norepinephrine 0.08 mcg/kg/min), B
gastroesophageal reflux disease, hypertension, obstruc- (epinephrine 0.08 mcg/kg/min), and C (Milrinone 0.25
tive sleep apnea, and ischemic cardiomyopathy, only mcg/kg/min) are incorrect.
Answer B consists of the three agents that reduce morbid-
ity and mortality among these comorbidities: carvedilol 4. Answer: D
(HF and potentially diabetes), spironolactone (HF pro- The patient’s troponin elevation is most likely a result of
gression post-MI), and lisinopril (HF and diabetes), the patient’s decompensated HF (especially considering
making Answer B correct and Answers A (Amlodipine, clinical evidence of acute HF, such as high BNP, CVP,
clopidogrel, and sitagliptin.), C (Pravastatin, amlodip- and low cardiac output) superimposed on chronic CAD
ine, and aspirin 325 mg), and D (Prasugrel, sildenafil, (Answer D is correct). Given the patient’s chemistry and
and atenolol) incorrect. ABG results, chronic obstructive pulmonary disease
(Answer B) is highly unlikely. Although this presenta-
2. Answer: C tion could be attributed to NSTEMI (Answer A), it is
Although many of these tests might be helpful in this less likely, given the lack of new ST-T changes on ECG
patient’s differential diagnosis, the most likely consid- and the accompanying evidence of an HF exacerbation.
eration would be respiratory decompensation or an HF Sepsis (Answer C) is also a potential contributor to tro-
decompensation, given the patient’s presentation; thus, ponin elevation; by comparison, an infectious etiology
a chest radiograph, ECHO, BNP, and lactate would be is less likely in this patient’s case.
most appropriate, making Answer C correct. Answer A
is incorrect; although an infectious etiology cannot be 5. Answer: A
ruled out, it is less likely than noninfectious cardiore- Ticagrelor has been associated with adenosine-mediated
spiratory differentials. Answer B is not the best answer; dyspnea and bradycardia; therefore, this medication
although serial troponins and an ABG could be argued should be evaluated as a contributing cause. According
for, liver function tests are unlikely to provide addi- to the current ACC/AHA guidelines, this patient should
tional insight into this patient’s differential. Answer D continue P2Y12 inhibitor therapy because his stent was
also is not the best answer; although it provides addi- placed less than 12 months ago, and he should con-
tional information for hemodynamic assessment, it is tinue aspirin therapy indefinitely (Answer A is correct).
lacking in an assessment of cardiac function, which is a Conversely, it would not be appropriate to stop ticagre-
likely contributor to the patient’s presentation. lor without switching to an alternative P2Y12 inhibitor
(Answer B is incorrect). A likely cause has been iden-
3. Answer: D tified (Answer C is incorrect), and the patient has no
Dobutamine would be the ideal choice, given the evidence of hyperkalemia (Answer D is incorrect).
patient’s acutely worsening cardiac index and malperfu-
sion and its rapid onset; this would facilitate β2 arterial 6. Answer: A
vasodilation (both SVR and PVR reduction) as well as Because of this patient’s hypotension and ongoing car-
increase chronotropy and inotropy. Milrinone, although diogenic shock, synchronized cardioversion would be
likely beneficial for both cardiac index and PVR reduc- preferred (Answer A is correct). Metoprolol (Answer
tion, would be less favorable, given that its time to peak B) would be unfavorable because of the patient’s con-
effect will exceed 6 hours (the time required to achieve current dobutamine use and hypotension; similarly,
greater than 87.5% of steady state, even in normal renal amiodarone 300 mg intravenous push (Answer D) could
function). Neither norepinephrine nor epinephrine contribute to further hypotension. The primary of role
would be favored (particularly at these doses) because of adenosine (Answer C) is to slow AV nodal conduction
the patient’s MAP is currently elevated, and these when patients are tachycardiac with a regular rhythm
agents would contribute to increased α1-mediated after- (i.e., supraventricular tachycardia) to terminate the
load increases, causing increased myocardial workload arrhythmia or to help differentiate atrial from ventricu-
in a patient who already has decompensated systolic lar arrhythmias.

ACCP Updates in Therapeutics® 2022: Critical Care Pharmacy Preparatory Review and Recertification Course

581
Cardiovascular Critical Care I

7. Answer: C
The physician is considering heparin anticoagulation
while the patient is in the ICU. Although valuable to
consider for long-term anticoagulation, the HAS-BLED
score is not validated for bleeding risk specific to hepa-
rin anticoagulation. Nonetheless, given the patient’s
CHA 2DS2-VASc score (annual stroke risk of 6.7%),
anticoagulation should be considered, making Answer
C correct and Answer D (“anticoagulation is not war-
ranted”) incorrect. Answer A, “the patient’s risk of
bleeding is the same as his stroke risk,” and Answer B,
“the patient’s risk of stroke exceeds his risk of bleed-
ing,” are incorrect statements because the foundation of
these scores does not include risk of thromboembolism
during ICU stay, nor do the scores assess risk of bleed-
ing on heparin infusions.

8. Answer: C
Given his ongoing cardiogenic shock, this patient is
unlikely to remain stable without intervention; thus,
Answer D is incorrect. To help stabilize him, intra-
aortic balloon counterpulsation (Answer C) would best
facilitate selective afterload reduction during systole
(increasing cardiac output in a patient with a low ejection
fraction and severe mitral regurgitation) while provid-
ing augmented diastolic pressures. Venovenous ECMO
(Answer B) is unlikely to help because this means of
MCS depends on a functional LV and RV to provide for-
ward blood flow. Venoarterial ECMO (Answer A) may
stabilize the patient; however, it would increase after-
load on the aortic valve, which could worsen his mitral
regurgitation. Furthermore, this degree of MCS may be
unwarranted at this time unless the patient develops pro-
gressive hypoxic cardiopulmonary failure.

ACCP Updates in Therapeutics® 2022: Critical Care Pharmacy Preparatory Review and Recertification Course

582
Cardiovascular Critical Care I

ANSWERS AND EXPLANATIONS TO SELF-ASSESSMENT QUESTIONS

1. Answer: C has a pulse and blood pressure and is currently not in


A diagnosis of cardiogenic shock is most likely, given VT. Furthermore, rapid administration of amiodarone
the patient’s history, presentation, ongoing vasopres- may lead to worsening hypotension.
sor requirement, elevated BNP, and lack of infectious
symptoms (Answer A is incorrect). Furthermore, this 5. Answer: A
patient has positive troponins and ST-segment elevation The patient’s heart catheterization was performed
in leads II, III, and aVF (inferior leads), making STEMI through the femoral artery, a vessel that is much more
the primary diagnosis and reason for cardiogenic shock difficult to compress to facilitate hemostasis. Hematomas
(Answer C is correct; Answer D is incorrect). The pres- can occur at the access site; however, the most serious
ence of hypotension and bradycardia are more likely bleeding complication associated with this access site is
consistent with complications of an inferior MI with a retroperitoneal bleed (Answer A is correct). Answer B
possible RV involvement, as opposed to an acute exac- (dissection/rupture), Answer C (stent thrombosis), and
erbation of HF. The patient also has a preserved LVEF Answer D (papillary muscle rupture) are incorrect.
and not systolic HF (Answer B is incorrect).
6. Answer: C
2. Answer: D Vasopressin would be favored because, when admin-
ST-segment elevation in leads II, III, and aVF (infe- istered at normal physiologic doses, it mediates
rior) are most consistent with the right coronary artery predominant increases in SVR while minimally affect-
(Answer D is correct). Answers A (left main coronary ing the PVR (Answer C is correct). Phenylephrine
artery), B (left anterior descending artery), and C (left (Answer B), however, will increase both PVR and SVR
circumflex coronary artery) are incorrect. by the α1-receptors. Given the patient’s ongoing hypo-
tension, low cardiac index, and rising CVP, inaction
3. Answer: B (Answer A) would be inappropriate, and additional vol-
According to the SOAP II investigation, dopamine is ume administration (Answer D) would be detrimental in
associated with increased rates of adverse events in the setting of volume overload and RV failure.
patients with cardiogenic shock compared with norepi-
nephrine (Answer B is correct; Answer A is incorrect). 7. Answer: B
Milrinone (Answer C), although helpful in some patients Amiodarone boluses would be safest if administered
with HF, would not be favored, given the patient’s cur- slowly for 10 minutes to avoid additional hypotension,
rent hypotension and acute renal failure. Normal saline followed by a continuous infusion (Answer B is cor-
administration (Answer D) would likely be detrimen- rect; Answer A is incorrect). Metoprolol (Answer C)
tal because of the patient’s signs of fluid overload, and diltiazem (Answer D) would be contraindicated
including BNP elevation, hyponatremia, and rales on because of ongoing cardiogenic shock and vasopressor
examination. requirements.

4. Answer: A 8. Answer: D
Lidocaine would be most favorable in patients with isch- Because of the diagnosis of acute MI, current quality
emia-mediated ventricular arrhythmias, and although measures would require initiation of or documenta-
the patient is not currently in VT, he is having persistent tion to contraindications for each item except for ACE
premature ventricular contractions (bigeminy), further inhibitors/ARBs (Answer D) because the patient still
increasing concern for ongoing ischemia and myocardial has an LVEF greater than 40%. Answer A (aspirin con-
irritability (Answer A is correct). Metoprolol (Answer traindication), Answer B (statin contraindication), and C
C) and diltiazem (Answer D) would be contraindicated (β-blocker contraindication) are incorrect.
because of ongoing cardiogenic shock and vasopressor
requirements. Amiodarone 300 mg intravenous push
(Answer B) is no longer appropriate because the patient

ACCP Updates in Therapeutics® 2022: Critical Care Pharmacy Preparatory Review and Recertification Course

583
APPENDIX A – OVERVIEW OF ANTI-ARRHYTHMICS
ECG Effects
Defibril-
Drug TdP risk
Class MOA Management Considerations and Pearls Notable Drug Interactions lation
(available dosage forms) (%)
Threshold

PR
QRS
QT/QTc
Different formulations available (no exact conversions)
α-Blockade may contribute to hypotension CYP2D6 (inhibitor)
Quinidine (PO and IV) Prolonged half-life in CHF and hepatic dysfunction CYP3A4 (substrate and inhibitor)
↑↓ ↑ ↑ ­↑ 2–8
May require dose adjustments in renal and/or hepatic dysfunction QT-prolonging drugs
Enhanced AV node conduction usually requires combination with
AV node-blocking agent
Sodium channel
May contribute to hypotension
IA blockade (inter-
Metabolized by hepatic acetylation
mediate potency) Procainamide
↑ ↑ ↑ Active metabolite (NAPA) renally eliminated and has increased QT-prolonging drugs ↑↓ 1–10
(IV)
class III properties
Dialyzable
May require dose adjustments in renal and/or hepatic dysfunction
Disopyramide CYP3A4 (substrate)
↑↓ ↑ ↑ Potent anticholinergic adverse effects ↑ 1–3
(PO) QT-prolonging drugs
May be used to treat hypertrophic cardiomyopathy
Ventricular arrhythmias only
Increased risk of toxicities in hepatic dysfunction, CHF, and elderly

584
patients
Lidocaine Enhanced efficacy in ischemic tissue
CYP3A4 (substrate) ↑
Sodium channel (IV) N/A Adverse effects include CNS depression, somnolence, tremors,
IB blockade (low N/A N/A or seizures
potency) Lidocaine concentrations may be checked (total: 1.5-5 mcg/mL;
↓ free: 0.5-2 mcg/mL)
Cardiovascular Critical Care I

Ventricular arrhythmias only


Mexiletine CYP2D6 (substrate)
Prolonged half-life in CHF and hepatic dysfunction ↑
(PO) CYP1A2
Oral derivative of lidocaine
Useful for AF/flutter in patients without structural heart disease
Increased risk of mortality post-MI (Cardiac Arrhythmia
Flecainide
Suppression Trial [CAST]) CYP2D6 ↑
(PO)
Avoid in CHF because of its potent negative inotropic effects
Sodium channel N/A
Combine with AV node blocker to prevent rapid atrial flutter
IC blockade (high ↑ ↑ or
potency) Useful for AF/flutter in patients without structural heart disease
­↑ Increased risk of mortality post-MI (CAST trial) First-pass metabolism
Propafenone
Avoid in CHF because of its potent negative inotropic effects CYP2D6, CYP3A4, CYP1A2 ↑↓
(PO)
Combine with AV node blocker to prevent rapid atrial flutter substrate
SR and IR formulations not equivalent

ACCP Updates in Therapeutics® 2022: Critical Care Pharmacy Preparatory Review and Recertification Course
APPENDIX A – OVERVIEW OF ANTI-ARRHYTHMICS (continued)
Ex. Sinus bradycardia
Carvedilol (PO) N/A N/A AV block
II β-Blockade Labetalol (PO/IV) or N/A or Hypotension more likely with nonselective agents Predominantly CYP2D6 N/A
Metoprolol (PO/IV) ­↑ ↓ Continuous infusions of esmolol or labetalol may contribute to
Esmolol (IV) large amounts of fluid
CYP3A4
Requires ECG monitoring for initiation, dose titration, reinitiation, Trimethoprim, verapamil,
Dofetilide
or introduction of new interacting agents HCTZ, and many others ↓ 1–8
(PO)
Requires renal dosing adjustments contraindicated
QT-prolonging drugs
Ibutilide Only indicated for cardioversion of AF or enhancement of
QT-prolonging drugs ↓ 1–8
(IV) electrical cardioversion
Sotalol Requires renal dosing adjustments
N/A N/A QT-prolonging drugs ↓ 1–6
(PO/IV) Bradycardia
Potassium channel or or
III blockade ↑ Multi–channel-blocking properties
­↑ ­↑ Inhibits CYP3A4, CYP2D6,
Monitor liver, thyroid, and pulmonary function tests
Amiodarone CYP2C9
Average half-life 53 days; highly lipophilic; active metabolite ­­↑ <1
(PO/IV) Adjust warfarin and digoxin by
Bradycardia and hypotension
50%
High protein binding
Multi–channel-blocking properties
Only approved for AF/flutter; less efficacious vs. amiodarone
Dronedarone Inhibits CYP3A4, CYP2D6,
Less potential for organ-system toxicity vs. amiodarone <1
(PO) CYP2C9

585
Avoid in CHF, particularly NYHA class III/IV
Bradycardia

Hepatic dosing
Verapamil
Sinus bradycardia CYP3A4 (substrate and inhibitor) N/A
(PO/IV)
Negative inotrope – avoid in patients with systolic heart failure
Calcium channel
↑ N/A N/A
Cardiovascular Critical Care I

IV blockade Decreased dosing in elderly patients and patients with hepatic


Diltiazem dysfunction
CYP3A4 (substrate and inhibitor) N/A
(PO/IV) Bradycardia
Negative inotrope - avoid in patients with systolic heart failure
Alternative for rate control of AF/flutter; also used for systolic heart
failure
Decreased dosing in elderly and patients with kidney dysfunction
Digoxin
↑ N/A N/A Loading dose may be given based on ideal body weight Many interactions
(PO/IV)
Not dialyzable
Digoxin concentrations may be checked (AF/flutter: 0.8-2 ng/mL;
Misc. Others heart failure: 0.5-0.9 ng/mL)
Half-life < 10 s
Adenosine
↑ Used for acute treatment of AV node reentrant tachycardias
(IV)
Can help to distinguish atrial from ventricular arrhythmias
Magnesium Used to treat Torsades and arrhythmias associated with
(IV) ↑ hypomagnesemia

ACCP Updates in Therapeutics® 2022: Critical Care Pharmacy Preparatory Review and Recertification Course
AF = atrial fibrillation; AV = atrioventricular; CHF = congestive heart failure; CNS = central nervous system; ECG = electrocardiogram; HCTZ = hydrochlorothiazide; IR = immediate release; IV = intravenous(ly); MI =
myocardial infarction; MOA = mechanism of action; NAPA = N-acetylprocainamide; N/A = not applicable; NYHA = New York Heart Association; PO = oral(ly); SR = sustained release; TdP = Torsades de pointes.
Research Design, Biostatistics,
and Literature Evaluation
Edward T. Van Matre, Pharm.D., M.S., BCCCP
University of Tennessee Health Science Center
Memphis, Tennessee

Brittany D. Bissell, Pharm.D., PhD, BCCCP


University of Kentucky
Lexington, Kentucky

Ishaq Lat, Pharm.D., FCCP, FCCM, BCPS, BCCCP


Shirley Ryan Abilitylab
Chicago, Illinois
Research Design, Biostatistics, and Literature Evaluation

Research Design, Biostatistics,


and Literature Evaluation
Edward T. Van Matre, Pharm.D., M.S., BCCCP
University of Tennessee Health Science Center
Memphis, Tennessee

Brittany D. Bissell, Pharm.D., PhD, BCCCP


University of Kentucky
Lexington, Kentucky

Ishaq Lat, Pharm.D., FCCP, FCCM, BCPS, BCCCP


Shirley Ryan Abilitylab
Chicago, Illinois

ACCP Updates in Therapeutics® 2022: Critical Care Pharmacy Preparatory Review and Recertification Course

589
Research Design, Biostatistics, and Literature Evaluation

Learning Objectives drug compound for septic shock. The novel drug
compound is a recombinant protein that mediates
1. Identify factors influencing the conduct of essential the inflammatory cascade of sepsis and has had
critical care research. impressive results for all etiologies of septic shock
2. Evaluate the appropriateness of various statistical in preclinical animal studies. Which best describes
tests for a set of data. the rationale for selecting a study population or pri-
3. Apply concepts of research design and analysis to mary end point?
clinical care. A. The study should limit study inclusion to
patients without a high level of comorbid con-
ditions at baseline to limit confounders.
Abbreviations in This Chapter B. The study should expand the target population
to include patients with sepsis, not just septic
ARDS Acute respiratory distress syndrome shock, to show a 28-day mortality benefit.
CI Confidence interval C. The study should expand the total sample size
ICU Intensive care unit to increase likelihood of obtaining power.
IRB Institutional review board D. The study should limit study inclusion to
NMBA Neuromuscular blocking agent patients with septic shock caused by pneumo-
OR Odds ratio nia to test a relevant study population.
QI Quality improvement
3. A quality improvement (QI) initiative is imple-
mented to improve antimicrobial dosing for patients
Self-Assessment Questions with septic shock presenting to the emergency
Answers and explanations to these questions may be department. As the pharmacy representative, you
found at the end of this chapter. have worked with the pharmacy operations team to
ensure that an appropriate selection of antibacterial
1. A clinical trial is being planned to determine the agents and doses is available in the automated drug-
optimal resuscitation fluid for trauma patients. This dispensing machines. Which factor will be most
study will seek to determine whether administering important in showing the effectiveness of this QI
crystalloid fluid, lactated Ringer solution, or a initiative?
transfusion strategy – packed red blood cells – in
A. Obtaining informed consent for participation in
the trauma field improves survival to hospital
the QI initiative.
discharge. The investigator team has consulted with
B. Identifying patients with septic shock in triage.
various ethics scholars to identify relevant issues to
C. Creating a community advertising campaign to
be addressed in the trial design. Which issue is most
bring awareness to the initiative.
relevant to the ethical conduct of this study?
D. Determining the social value of the QI initiative.
A. Treatment blinding.
B. Uninformative study population. 4. An epidemiologic study seeks to determine the
C. Consent obtained from injured subjects. impact of adverse drug events in the intensive care
D. Design as a noninferiority trial. unit (ICU) on patient outcomes, which have an esti-
mated incidence of 52%. Which would be the best
2. In recent studies of septic shock, 28-day mortality approach to conducting this study?
has been reported as around 20% in the standard
A. Randomized controlled trial with a test for con-
treatment arm. Earlier estimates of septic shock
tinuous variables to determine the difference in
mortality were 35%–40%, according to the results
outcomes.
of previous trials and epidemiologic studies from
B. Retrospective case-control study with a test
the past 10 years. The investigators of a new study
for proportions to determine the difference in
are seeking to identify the optimal end point or
outcomes.
study population to test the effectiveness of a novel

ACCP Updates in Therapeutics® 2022: Critical Care Pharmacy Preparatory Review and Recertification Course

590
Research Design, Biostatistics, and Literature Evaluation

C. Prospective observational study with survival 7. A critical care pharmacist is faced with an acute
analysis to determine the difference between drug shortage in which no furosemide is available
cohorts. for immediate use in patient care. During patient
D. Retrospective case-cohort study with a test care rounds in the ICU, the decision is made to
for proportions to determine the difference in implement a fluid-conservative strategy for a patient
outcomes. with acute respiratory distress syndrome (ARDS)
(central venous pressure goal less than 4 mm Hg).
5. A case-control study is conducted to determine The critical care pharmacist procures an allotment
whether proton pump inhibitor (PPI) use is asso- of bumetanide. Which statement best describes the
ciated with an increased risk of developing course of action for this patient?
Clostridium difficile infection (CDI). The final anal- A. The pharmacist uses her understanding of the
ysis shows the odds ratio (OR) for CDI with PPI medical literature and experiential knowledge
exposure to be 1.3 (95% confidence interval [CI], to develop a titration scheme using bumetanide
0.8–1.5). Which best describes the results? to achieve a central venous pressure of less than
A. PPI exposure increases the risk of CDI by 4 mm Hg.
130%. B. The pharmacist uses her understanding of
B. PPI exposure reduces the risk of CDI by 20%. research trial design and experiential knowl-
C. PPI exposure increases the risk of CDI by 30%. edge to develop a titration scheme using
D. PPI exposure is not associated with an increased bumetanide to achieve a central venous pres-
risk of CDI. sure of less than 4 mm Hg.
C. The pharmacist uses her friendly rapport to con-
6. A systematic review evaluated the effect of albu- vince the nephrologist to treat this patient with
min for fluid resuscitation. A meta-analysis that hemodialysis to achieve a goal central venous
evaluated the effect of albumin use compared pressure of less than 4 mm Hg.
with normal saline on 28-day mortality reported a D. The pharmacist uses her understanding of
combined OR of 0.45 (95% CI, 0.3–0.75) for the research ethics to obtain informed consent
treatment of hypovolemic shock caused by trauma. from the patient’s surrogate for treatment with
For the treatment of septic shock, albumin com- bumetanide.
pared with normal saline resulted in a combined OR
of 1.1 (95% CI, 0.98–1.21) when evaluating 28-day 8. In a study of ARDS, patients are treated with a neu-
mortality. Which best represents the findings of the romuscular blocking agent (NMBA) or placebo to
review? determine whether administering a NMBA within
A. Albumin increased mortality in trauma but did the first 48 hours of presentation improves 28- and
not affect survival in the treatment of septic 90-day survival. The study enrolled an unequal
shock. distribution of patients, with more patients with
B. Albumin increased survival in trauma but did severe ARDS than patients with moderate and mild
not affect survival in the treatment of septic ARDS. The post hoc analysis of the results showed
shock. a survival benefit to administering a NMBA to the
C. Albumin did not affect survival in the treat- patients with severe ARDS. Which rationale best
ment of hypovolemic shock caused by trauma describes why NMBAs should not be administered
but improved survival in the treatment of septic to patients with mild to moderate ARDS?
shock. A. Patients with mild ARDS have a lower mortal-
D. Albumin did not affect survival in the treatment ity rate and are therefore less likely to benefit
of hypovolemic shock caused by trauma or sep- from the test treatment.
tic shock. B. Patients with mild and moderate ARDS
are inherently different from patients with
severe ARDS because of their etiology and
presentation.

ACCP Updates in Therapeutics® 2022: Critical Care Pharmacy Preparatory Review and Recertification Course

591
Research Design, Biostatistics, and Literature Evaluation

C. NMBAs are periodically on shortage from the


manufacturer and need to be prioritized for nec-
essary medical indications.
D. The end points selected do not carry sufficient
social value to warrant treatment.

ACCP Updates in Therapeutics® 2022: Critical Care Pharmacy Preparatory Review and Recertification Course

592
Research Design, Biostatistics, and Literature Evaluation

BPS Critical Care Pharmacy Specialist Examination Content Outline

This chapter covers the following sections of the Critical Care Pharmacy Specialist Examination Content Outline:
1. Domain 3: Evidence-Based Medicine, Scholarship, Education, and Professional Development
a. Task 1: 1–3
b. Task 2: 1–3

ACCP Updates in Therapeutics® 2022: Critical Care Pharmacy Preparatory Review and Recertification Course

593
Research Design, Biostatistics, and Literature Evaluation

I.  INTRODUCTION

A. Epidemiology of Critical Care in the United States – why continued research is essential to improving the
delivery of care to patients
1. 10.1%–28.5% of all hospital inpatients receive care in an intensive care unit (ICU), approximating 5.7
million adults. Estimated mortality for ICU care is 20%–40% for common critical care syndromes (Crit
Care Med 2012;40:1072-9).
2. 16.9%–38.4% of total hospital costs are spent on critical care services, approximating $121–$263 billion.
a. 5.2%–11.2% of national health care expenditures
b. 1% of gross domestic product

B. Why Pharmacists Need to Understand the Fundamentals of Research Practice, Trial Design, and Literature
Evaluation – High rates of morbidity and mortality necessitate efficient and efficacious decision-making on
the part of caregivers.

C. Necessity of Clinical Research in Critical Care – To optimize patient outcomes and minimize potential
patient harm while providing efficient stewardship of finite resources

D. Synthesizing Medical Knowledge with Experiential Knowledge and Pathophysiologic Reasoning – Essential
to creating patient-specific therapy care plans

II.  BIOETHICS

A. The Belmont Report – Originally created as a result of the National Research Act of 1974; outlines the
fundamental ethical principles for the conduct of clinical research
1. Respect for individuals dictates that each research participant be treated with respect for his or her
dignity and autonomy. As such, informed consent shall be obtained from research participants or their
surrogates.
2. The principle of justice requires that investigators recruit research subjects in a manner that allows
equal access to participation for all populations that may potentially benefit from the research endeavor.
3. Beneficence requires research investigators to ensure that risks are minimized and benefits maximized
for research participants.

B. A framework for the ethical conduct of clinical research includes seven requirements: (1) social value, (2)
scientific validity, (3) fair selection of research participants, (4) favorable risk-benefit ratio, (5) independent
review, (6) informed consent, and (7) respect for enrolled participants (JAMA 2000;283:2701-11).

C. Equipoise – Must be present for the conduct of a clinical trial. Equipoise is defined as the state of uncertainty
between treatments A and B for a given population of subjects with a predefined disease and/or syndrome.
Once the balance of uncertainty between treatments A and B is disturbed such that one treatment is believed
to be superior, the risk-benefit ratio is altered such that treatment may not be beneficial to the individual
research subject. Example: a proposal for clinical research ethics in critically ill patients during pandemics
(Crit Care Med 2010;38:e138-142)

ACCP Updates in Therapeutics® 2022: Critical Care Pharmacy Preparatory Review and Recertification Course

594
Research Design, Biostatistics, and Literature Evaluation

D. The Common Rule – Created in 1991 as uniform ethical standards for research after a series of exposures
of unethical research misconduct, such as the Tuskegee syphilis study. This rule defined human participant
research, developed the process of informed consent, and established the requirement of institutional review
boards (IRBs) (JAMA 2017;317:1521-2).
1. The Common Rule has undergone major changes, with most implementation occurring in 2019 (Chest
2019;155:272-8). These included the following:
a. New categories and definitions of exempt or excluded research studies
b. Allowance of a single IRB for multi-institutional studies
c. Exclusion of public health surveillance as research
d. Requirement of concise, focused consent forms and online posting of consent as applicable
e. Allowance of broad consent for identifiable biospecimens in future unspecified research studies

III.  PRACTICAL CHALLENGES TO CRITICAL CARE RESEARCH

A. Research Subject Recruitment – To maximize external validity, research subjects recruited for participation
in a clinical trial should be representative of the general population of patients with the disease or syndrome.
1. Critical care is exemplified by the provision of supportive care for the treatment of diseases and
syndromes.
a. Diseases are characterized by the specific test used to identify the pathophysiologic process.
b. Syndromes are often identified by the presence of a constellation of signs and symptoms that
suggest the presence of a disease that exists across a spectrum of severity.
2. Recognizing the attendant syndrome is critical for the timely provision of therapy and, in research,
subject recruitment.
a. Heterogeneity in syndromes challenges the ability to identify subjects with similar severity or
probability for similar clinical outcomes for participation in clinical research.
b. In addition, heterogeneity challenges the ability to interpret the results from dissimilar populations,
even though they may have a single syndrome in common.
c. Discrepancies between results from basic and early clinical studies versus adequately powered
controlled trials show that surrogate outcome measures (e.g., organ function) may not reliably
predict clinical benefit (Am J Respir Crit Care Med 2015;191:1367-73).
3. Unique to critical care clinical research is the need to enroll subjects similar in acuity or at a similar stage
in the process of their syndrome to allow meaningful comparisons (e.g., N Engl J Med 2010;363:1107-
16).
4. Use of non-standardized definitions and lack of power complicate data synthesis and meta-analysis
(Am J Respir Crit Care Med 2014;189:1469-78).
5. Selection of end points should be based, in part, on the ethical values outlined previously. End points
should provide social value and have scientific validity. A growing area of emphasis in research is on
the need to design patient-centered studies (JAMA 2012;307:1583-4). (Example: Positive vs. negative
neurological outcome, rather than mean change in modified Rankin Score.)

B. Informed Consent
1. Because of the acute nature of critical illness, it may be singularly difficult to obtain informed consent
within a short time.

ACCP Updates in Therapeutics® 2022: Critical Care Pharmacy Preparatory Review and Recertification Course

595
Research Design, Biostatistics, and Literature Evaluation

2. Obtaining informed consent is particularly challenging for critically ill patients, primarily because
many critically ill patients lack decisional capacity because of the acute nature of their illness and/or
the effect of medications.
3. Research that involves subjects with impaired consent within the ICU should include plans for
assessment of capacity. The University of California, San Diego Brief Assessment of Capacity to
Consent (UBACC) is a 10-item scale that assesses capacity for enrollment into clinical research (Arch
Gen Psychiatry 2007;64:966-74).
4. Informed consent is usually not required in studies that are deemed quality initiatives, because these
studies are meant to improve the delivery of care. However, application of this principle is variable and
may vary between institutional review boards (IRBs).
a. Keystone ICU project was a joint collaboration between investigators at Johns Hopkins University
and the state of Michigan to reduce central line–associated bloodstream infections in ICU patients
using a checklist (N Engl J Med 2006;355:2725-32).
i. Study was classified as exempt from IRB review and informed consent because of the study’s
QI nature.
ii. After a complaint, the Office for Human Research Protections of the Department of Health
and Human Services originally faulted the investigators for not obtaining informed consent.
Subsequently, the Office for Human Research Protections reversed course and classified the
project as a “non-research” activity.
b. Inconsistent interpretation by local IRBs, in the absence of clear guidance from the Office for
Human Research Protections, contributes to confusion regarding the necessity of informed consent
in QI initiatives (Jt Comm J Qual Patient Saf 2008;34:349-53).
5. Surrogates and family members are recognized as having authority to provide consent on the behalf of
patients, often termed “legally authorized representative,” despite the ambiguous legal standing on this
issue in several states.
a. It is important to realize that underlying motives for proxy consent may include the belief that
participating in the research protocol will lead to improved care.
b. Important to acknowledge that patients and surrogates entrust clinical researchers to act in their
best interests
c. Research supports that surrogate consent and patient preferences agree most of the time, but
discrepancies may occur in studies considered higher risk (Chest 2001;119:603-12; Crit Care Med
2012;40:2590-4).
6. One area of growing utilization is digital consent. Digitalized consent has the ability to increase
comprehension, particularly in those with poor literacy or a lower socioeconomic status (JAMA Cardiol
2020;5:845-7; JAMA 2015;313:463-4).

C. Waiver of Informed Consent


1. In select circumstances, informed consent may be waived by the IRB.
2. Of note, differences exist in the guidance between the Department of Health and Human Services and
the U.S. Food and Drug Administration (FDA) for waiver of informed consent. In many circumstances,
local IRBs will follow the Common Rule as the minimum standard. Further, restrictions regarding the
use of a waiver of informed consent vary by country and culture (Theor Med Bioeth 2016;37:441-6;
Bioethics 1999;13:249-55).
3. Waiver of informed consent is typically granted for any of the following circumstances:
a. Research that is deemed of minimal risk to the participant, does not adversely affect the welfare of
the subject, and could not otherwise be practicably carried out
b. Research that is carried out to evaluate public benefits or service programs
c. Research in emergencies when consent would be impractical to obtain.
i. An example is a study testing the hypothesis of whether drug A is noninferior to drug B for
status epilepticus (N Engl J Med 2012;366:591-600).

ACCP Updates in Therapeutics® 2022: Critical Care Pharmacy Preparatory Review and Recertification Course

596
Research Design, Biostatistics, and Literature Evaluation

ii. A trial conducted in France that investigated the timing of initiation of renal replacement
therapy did not require written informed consent because the standard of care was included in
both treatment arms (N Engl J Med 2016;375:122-33).
4. Controversy exists regarding waiver of consent for QI research. Currently, QI research is subject to the
interpretation of local IRBs, as mentioned previously (N Engl J Med 2015;372:855-62).

D. Community Consent
1. Occasionally, critical care research will need to be conducted in the general community. Obtaining
consent in this scenario would be impossible, given the medical condition of the research subject.
Example: intramuscular therapy compared with intravenous therapy for prehospital status epilepticus
(N Engl J Med 2012;366:591-600). In this study of prehospital emergency care, informed consent would
be impossible to obtain in the field. Therefore, IRBs engaged local community consultation, according
to regulations, and provided approval.
2. Guidance exists for investigators to inform the community and community leaders before undertaking
the research endeavor.
3. Approval for this type of research is required from local IRBs.

IV.  STUDY DESIGN (FIGURES 1 AND 2)

Meta-analysis
RCT and observational trials
RCT demonstrate consistency

Cohort Meta-analysis:
RCT or observational studies
Case-control
RCT or observational studies
Cross-sectional
Case reports/series
Case reports/series
Opinions and reviews
Opinions and reviews

Figure 1. Hierarchy of clinical evidence. Figure 2. Proposed revision to hierarchy of clinical


RCT = randomized controlled trial. evidence.
Sevransky JE, Checkley W, Martin GS. Critical care trial design and Gershon AS, Jafarzadeh S, Wilson KC, et al. Clinical knowledge from
interpretation: a primer. Crit Care Med 2010;38:1882-9. observational studies. Am J Respir Crit Care Med 2018;198:859-67.

A. Randomized Controlled Trial


1. Hallmark of clinical research. Study design seeks to minimize bias through randomization of subjects,
blinding of participants, and analytic approach. This should leave at least two groups that differ only
in study treatments.
2. Experimental design to test the effects of an intervention compared with either placebo or the established
standard of care (treatment or process of care); allows for description and causality
3. Preliminary research should exist to suggest that the intervention is based on pathophysiology and
repeated outcomes consistent with the existing scientific understanding sufficient to warrant the
proposed testing on patients.

ACCP Updates in Therapeutics® 2022: Critical Care Pharmacy Preparatory Review and Recertification Course

597
Research Design, Biostatistics, and Literature Evaluation

4. While historically considered the gold standard for research, many limitations exist to application and
interpretation of randomized controlled trials in the ICU (Front Immunol. 2018;9:1502).
a. Heterogeneous population: The assumption that all patients would benefit from a treatment is
unlikely.
b. Gaps in understanding: Prognostic markers and definitions are often limited, raising concern for
inappropriate inclusion and/or exclusion.
c. Therapeutic optimization often unknown: Preclinical data is often highly different from the
population studied (healthy humans or rodent models) limiting confidence in extrapolation.
5. Trial enrichment is a mechanism in which a specific subgroup of a population who have higher likelihood
of a response to an intervention are selected over those who may not (Nat Rev Nephrol 2020;16:20-31).
a. Prognostic enrichment selects for patients with a higher likelihood of disease-related interest (i.e.,
mortality)
b. Predictive enrichment selects for those patients who are more likely to respond to a treatment based
on underlying biologic mechanisms or physiology (i.e., inflammatory barkers in ARDS)

B. Observational
1. Observation of clinical practice; no intervention is tested
2. Describes associations between phenomena
3. Hypothesis-generating: Case-control study
a. Retrospective design
b. Provides cost-effective means to determine the association between the risk factor and the outcome
of interest
c. Two groups (with and without the outcome) are compared to identify the differences and risk
factors for developing the outcome of interest.
d. Potential for selection bias and confounding
e. Cases and controls are representative of the population with the disease and are chosen to minimize
selection bias.
f. Missing data are frequently a problem within observational research and may be classified as
missing completely at random, missing at random, or missing not at random (Am J Epidemiol
2012;175:210-7).
i. Missing completely at random data are truly random and case-dependent and are at decreased
risk for introduction of bias
ii. Missing at random indicates data that are absent and the absence is related to other patient data
(i.e., correlation with age), and therefore may increase risk of bias.
iii. Missing not at random indicates data that are absent but the absence is not related to one of
the above.
g. There are multiple ways to handle missing data, but the method should be defined a priori given
that significant amounts of missing data may introduce bias.
h. Confounding variables must be handled in a manner that can be controlled during analysis.
4. Cohort study – Prospective or retrospective design
a. Observational study of a given population over a given time to determine the association between
risk factors and the outcome of interest. Identifies the relationship between exposure and outcome
b. Describes the natural progression of a disease or syndrome
c. Example: Delirium, defined a positive Confusion Assessment Method for the ICU examination, is
an independent predictor of mortality in mechanically ventilated patients (JAMA 2004;291:1753-
62).
i. Cohort of mechanically ventilated patients observed during ICU stay for presence of delirium
ii. Determination of outcome: ICU mortality, 6-month mortality

ACCP Updates in Therapeutics® 2022: Critical Care Pharmacy Preparatory Review and Recertification Course

598
Research Design, Biostatistics, and Literature Evaluation

iii. Regression models to determine whether delirium is predictive of 6-month mortality, controlled
for covariates
5. Incidence versus prevalence
a. Incidence – Measures the occurrence of a disease (or event) over a period
b. Prevalence – Measures the occurrence of a disease (or event) at a fixed point in time
6. Case reports/case series: Describes the experience in treating a single patient or the cumulative
experience in treating a series of patients
7. Validity
a. Internal
i. Does the study design adequately test the hypothesis?
ii. Are the study methods sound?
b. External
i. Is the study population representative of the clinical setting?
ii. Are the study findings generalizable outside the study setting?
iii. Can the study be replicated in clinical practice?
8. Bias: Systematic error leading to an estimate of association in the study population that varies from the
source population (Am J Health Syst Pharm 2008;65:2159-68)
a. Selection bias: Systematic selection of subjects that leads to an imbalance, or an advantage, in favor
of one cohort over the other
b. Observation bias: Observers (research team) are aware of the research purpose and allow this
knowledge to influence interpretation of results.
c. Recall bias: Methodological error that is introduced in survey research when the participant is
asked to provide recall of a past event
d. Misclassification bias: Inappropriately categorizing a group of patients with, or without, the
disease/syndrome
e. Immortal time bias: Occurs when populations studied include an exposed and unexposed group
without a predefined time-zero. This results in a delay before a subject is considered treated. Given
that the subjects must survive up until the time of first exposure, the time before exposure is
considered immortal time (Am J Respir Crit Care Med 2003;168:49-53).
f. Measurement bias: When information collected for use as a study variable is inaccurate.
g. Confounding variables: Extraneous variables that influence both the dependent and the independent
variable, affecting how the overall result can be interpreted
i. Residual confounding occurs when unmeasured variables may effect overall outcome results.
This can only be avoided with randomization (Int J Public Health 2010;55:701-3).
ii. Confounding by indication occurs when a contributor to the outcome is present in those at
a high risk and is an indication for intervention. This results in care differences between the
exposed and nonexposed groups that may be based on differences in indication for exposure
(Pharmacoepidemiol Drug Saf 2003;12:551-8).
h. Bias can be accounted for with sufficient planning for design, data collection, and analysis and can
potentially be minimized (i.e., “effect modification”).
9. Sensitivity analysis: These may be used to determine robustness of study findings in different analysis
methods or presence of unmeasured confounding. They may also be used to detect misclassification
(Am J Respir Crit Care Med 2018;198:859-67).
10. Approaches to control for bias:
a. Study design: Randomization, matching
b. Analysis: Multivariable analysis, matching potential confounding variables with a controlled cohort

ACCP Updates in Therapeutics® 2022: Critical Care Pharmacy Preparatory Review and Recertification Course

599
Research Design, Biostatistics, and Literature Evaluation

C. Adaptive trials: Increasing attention has been given to the use of adaptive trial designs. Such trials seek to
increase flexibility in clinical trials by allowing for trial modification. Prespecified rules dictate modifications
upon scheduled interim evaluations of the data as the trial is ongoing (BMC Med 2018;16:29).
1. Examples of adaptive designs include: Continual reassessment method; group-sequential method;
sample size re-estimation; multi-arm, multi-stage population enrichment; biomarker adaptive; adaptive
randomization; adaptive dose-ranging; and seamless phase I/II or II/II trials.
2. Adaptive trials seek to increase efficiency of any type of prospective trial and to minimize potential
harm to patients.
3. Challenges to adaptive trials include the complexity of statistical interpretation, lack of knowledge of
the scientific community about these designs, and difficulty in communication of the results.

D. Cluster randomized trials


1. Randomize groups (clusters) of subjects to either control or intervention groups
2. Require a large sample size given the intracluster correlation coefficient and resulting design effect
3. Useful in system or group-level intervention (i.e., medication diluent change for entire ICU), when
individual randomization is not possible, and to prevent contamination, or the phenomenon which occurs
when providers or subjects learn about the intervention during the study period and start adopting it as
standard of care, whether within the treatment group or not
4. May result in recruitment bias, more baseline imbalance between groups, loss of entire clusters, or
inappropriate analyses (Arztebl Int. 2018;115:163-8).

E. Pharmacokinetic and Pharmacodynamic trials


1. Studies designed to evaluate effects of unique disease states on clinically relevant pharmacokinetic and
pharmacodynamic parameters
2. Trial designs can be prospective based on predefined sampling strategies or retrospective based in
routine clinical care laboratory samples (Clin Pharmacokinet 2015;7:783-95)
3. Large study size not needed to define properties in unique patient populations
4. Population based pharmacokinetic and pharamacodynamic studies can help define novel clinical dosing
strategies in unique patient populations

V.  STATISTICAL ANALYSIS

A. Statistical analyses can be categorized into one of two main classifications: Bayesian or central tendency
(frequentist)
1. Bayesian analysis allows for incorporation of prior knowledge into current study and calculates the
probability of benefit.
2. Frequentist statistics data are presented using descriptive statistics such as the mean, variance, standard
deviation, median, and interquartile range.
3. Interpretation of studies may be different based on whether frequentist or Bayesian analyses are selected
(JAMA 2019;321:654-64; Am J Respir Crit Care Med 2020;201:423-9).

B. Hypothesis Testing – To determine whether the observation was caused by chance


1. Null hypothesis (H0): No difference exists between groups. Accepting the null hypothesis means that
no difference exists between groups (or is unlikely to exist).
2. Alternative hypothesis (H1): There is a difference between predefined group A and group B.

ACCP Updates in Therapeutics® 2022: Critical Care Pharmacy Preparatory Review and Recertification Course

600
Research Design, Biostatistics, and Literature Evaluation

a. Type I error (alpha [α] error): To reject the H0 when, in fact, it is true. Decisional threshold to reject/
not reject the H0 is conventionally set at α = 0.05. The α value represents the likelihood that a type
I error will be made (i.e., rejecting the H0 when the H0 should not be). An α set at 0.05 means that
the H1 will erroneously be accepted 1 in 20 times.
i. Two-tailed tests distribute half of the α value at the far left hand and far right hand ends of the
distribution
ii. One-tailed tests distribute all of the α value at the far left hand or far right hand ends of the
distribution
b. Type II error (beta [β] error): Fail to reject the H0 when, in fact, there is a difference between
groups (i.e., the H1 is true). Decisional threshold to set β at 0.20. What does “power” really mean?
The ability to detect a difference if one truly exists. Contingent on sample size. However, this is an
estimate and may be inaccurate (usually based on previous literature).
i. Sample size is the number of subjects or samples collected or assessed within a study and is
influenced by the following:
(a) Difference within the predicted outcomes that is considered to be clinically significant
(i.e., difference in control vs. treatment group mortality rate)
(b) The variability in the predicted outcomes (i.e., the variability in baseline mortality)
(c) What is deemed to be an acceptable rate of error (type I, or alpha)
(d) Type II error rate (beta)
ii. Typically the greater the estimated difference and the lower the variability, the smaller the
sample size required
iii. Decreasing the rate of type I error (alpha) and increasing the power (decreasing beta) increases
the necessary sample size

C. Types of Data (Table 1)


1. Continuous
a. Counting, chronological order (e.g., 7.0, 7.1, 7.2, 7.3)
b. Sample tests: t-test, Wilcoxon rank sum/Mann-Whitney U
2. Categorical
a. a. Nominal
i. Categories or groups to which data are designated to (e.g., race, sex, hypertension, heart failure)
ii. Sample tests: Chi-square, Fisher exact test
b. Ordinal
i. Categorical data that include an order (e.g., Likert scale)
ii. Sample tests: Wilcoxon signed-rank test, Wilcoxon rank-sum test, Kruskal-Wallis 1-way test
3. Descriptive statistics
a. Measures of central tendency
b. Mean: Used for continuous and normally distributed data (parametric), sensitive to outliers
c. Median: 50th percentile; used for ordinal data or nonparametric, continuous data
d. Mode: Value occurring most frequently in a distribution
e. Standard deviation
i. Used for continuous, parametric data
ii. Describes variability around the mean
iii. Approximately 95% of the data fall within two standard deviations of the mean
f. Range: Describes spread of data, minimum to maximum values
g. Percentiles: The value where the percentage of values falls below. Example: The 75th percentile is
where 75% of values are smaller.
i. Interquartile range: Describes the values between the 25th and 75th percentiles
ii. Usually presented in conjunction with medians

ACCP Updates in Therapeutics® 2022: Critical Care Pharmacy Preparatory Review and Recertification Course

601
Research Design, Biostatistics, and Literature Evaluation

4. Paired data
a. There are circumstances in which matching will occur naturally between data points
b. Duplicate measurements on one sample
c. Sequential measurements, such as a pre-post test
d. Cross-over trials
e. Matched samples
Table 1. Simple Statistical Test Selection
Variable Two Samples >Two Samples
Independent Related Independent Related

Nominal χ2 or Fisher exact test McNemar test χ2 Cochran Q

Wilcoxon rank sum Wilcoxon signed rank Kruskal-Wallis


Ordinal Friedman ANOVA
Mann-Whitney U test Sign test (MCP)
Continuous with Equal variance t test 1-way ANOVA Repeated measures
Paired t test
no factors Unequal variance t test (MCP) ANOVA
2-way repeated 2-way repeated
Continuous with 2-way ANOVA
ANCOVA measures measures
one factor (MCP)
ANOVA ANOVA
ANCOVA = analysis of covariance; ANOVA = analysis of variance; MCP = multiple comparison procedures.

D. Parametric Continuous Data


1. Student t-test: Comparison of means between two independent groups
2. Analysis of variance: Comparison of means between three or more groups

E. Nonparametric Continuous Data


1. Two groups: Wilcoxon rank sum or Mann-Whitney U test
2. Three or more groups: Kruskal-Wallis

F. Nominal Data
1. Categories (e.g., sex, race, treatment groups, confusion assessment method for the ICU)
2. Chi-square test
3. Fisher exact test: More accurate for smaller data sets (fewer than five observations per group)

G. Ordinal Data
1. Groups represented by scale (e.g., Richmond Agitation-Sedation Scale score, Sequential Organ Failure
Assessment score)
2. Central tendency expressed as median and quartiles
3. Statistical tests: Wilcoxon rank sum, Mann-Whitney U test, Kruskal-Wallis

H. Confidence Interval
1. A method to describe variability around a point estimate within the study population
2. CIs offer a more descriptive interpretation of the data.
a. Magnitude of difference between groups
b. Range of values (possible spread of point estimates)
c. A 95% CI has a 0.95 probability of containing the true mean

ACCP Updates in Therapeutics® 2022: Critical Care Pharmacy Preparatory Review and Recertification Course

602
Research Design, Biostatistics, and Literature Evaluation

I. Correlation: Describes the association between two variables


1. Correlation value (r) contains a range of values from -1 to +1. An r value of -1 or +1 indicates a perfect
negative or positive relationship, respectively. Relative to 0, the closer the values are to 1, the stronger
the relationship between the two variables.
2. Pearson: Parametric continuous data
3. Spearman rank: Nonparametric continuous data or ordinal data

J. Regression Analysis: Describes whether an independent variable can predict an outcome. Can be used to
describe the strength of the association between a predictor variable and a dependent variable
1. Linear regression
a. Used when the dependent variable is continuous (e.g., length of stay)
b. A single predictor variable (continuous or categorical) can be tested in a simple linear regression
model.
c. More than one predictor variable (continuous or categorical) can be tested at a time in a multivariable
linear regression model.
2. Logistic regression
a. Used when the dependent variable is a categorical variable (e.g., mortality)
b. A single predictor variable (continuous or categorical) can be tested in a simple logistic regression
model.
c. More than one predictor variable (continuous or categorical) can be tested at a time in a multivariable
logistic regression model.

K. Odds Ratio
1. Describes the odds of patients being exposed to a risk factor and the occurrence of the outcome of
interest compared with those who are not exposed to the risk factor
2. The OR is interpreted in relation to a reference point (1.0). If the 95% CI includes 1, the odds of the event
occurring are equally likely in either group.

L. Survival Analysis (time-to-event analysis) (Table 2)


1. Censoring: Adjusts data so that patients are not included (i.e., “censored”) in the analysis if they did not
experience, or were not observed for, the event
2. Kaplan-Meier method
a. Describes the impact of a single predictor variable on the time-to-event between cohorts
b. Compares the survival times between two cohorts while controlling for a singular predictor variable
c. Survival curves are typically analyzed using the log-rank test.
d. Results typically reported as OR with 95% CI
3. Cox proportional hazards model
a. Describes the impact of several predictor variables on the time-to-event
b. Compares the survival times between two cohorts while controlling for several predictor variables
c. Results typically reported as hazard ratio with 95% CI

Table 2. Statistical Tests According to Type of Data


Dependent Variables Independent Variables Test(s)
Continuous, parametric Categorical t-test
Continuous, nonparametric Categorical Mann-Whitney U test
Categorical Categorical Chi-square test, Fisher exact
Continuous Continuous Linear regression
Continuous Categorical Linear regression
Categorical Continuous Logistic regression
Categorical Categorical Logistic regression

ACCP Updates in Therapeutics® 2022: Critical Care Pharmacy Preparatory Review and Recertification Course

603
Research Design, Biostatistics, and Literature Evaluation

VI.  NONINFERIORITY TRIAL DESIGN

A. Traditional approach to randomized controlled trials seeks to establish novel treatment as superior to
established standard. Noninferiority trials seek to answer whether a competing treatment is no worse than
the established standard therapy (JAMA 2015;313:2371-2). H1 = treatment A is no worse than treatment B;
H0 = treatment B is better than treatment A.

B. Seeks to establish utility by showing similar effectiveness while improving safety or reducing treatment
burden (costs, inconvenience, labor, etc.) (JAMA 2012;308:2605-11). Clinically important difference,
severity of disease, toxicity, costs, regulatory standards, etc.

C. Analytic Approach to a Noninferiority Trial


1. What is an acceptable threshold for “noninferiority”?
a. First step is to determine the marginal difference.
b. What is the maximum allowable negative outcome events acceptable when comparing the treatment
with the standard therapy? (The maximum increase in risk that you are willing to accept for a
tradeoff in reducing treatment burden)
i. Typically, some fraction of the standard treatment effect to be preserved
ii. The FDA provides guidance for noninferiority thresholds: (1) Establish the smallest plausible
benefit of the existing standard therapy. (2) Insist on some preservation of the treatment effect
of the standard therapy. The FDA recommends that 50% of the standard treatment effect be
preserved when evaluating mortality for thrombolytic trials.
c. Some examples of threats to meaningful comparisons to establishing noninferiority:
i. Effect of standard treatment was not preserved. Suboptimal standard treatment administered.
Example: Heparin infusion does not achieve therapeutic anticoagulation
ii. Intention-to-treat analysis – Suboptimal administration of the standard treatment results in a
large proportion of undertreated patients for comparison
d. Superiority may also be tested; however, a different threshold for measuring differences between
groups must be used.
2. Interpreting the results of a noninferiority trial
a. Looking for an acceptable difference between groups that can plausibly be described as “noninferior”
b. Event rate can be established by existing literature and clinical practice. Example: A trial to
compare alternative approaches to vasopressor selection for patients in septic shock while reducing
the incidence of adverse events
i. Vasopressor A (novel treatment) achieved a goal mean arterial pressure greater than 65 mm
Hg at 6 hours in 50% of patients with septic shock compared with 53% of patients treated with
vasopressor B (standard treatment). The a priori noninferiority margin is set at 5% between
groups. Simultaneously, the incidence of new-onset atrial fibrillation is 7% for vasopressor A
compared with 15% for vasopressor B (p=0.02).
ii. Vasopressor A is noninferior with respect to the outcome of goal mean arterial pressure at
hour 6 while reducing the harm associated with the incidence of new-onset atrial fibrillation.

ACCP Updates in Therapeutics® 2022: Critical Care Pharmacy Preparatory Review and Recertification Course

604
Research Design, Biostatistics, and Literature Evaluation

VII.  APPLICATION OF KNOWLEDGE TO PATIENT CARE

Table 3. Assessment of Primary Literature for Clinical Application


Assessment
• Were the hypotheses and study purpose clearly stated?
• Is the study sample representative of the population with the disease/syndrome?
• Are the inclusion/exclusion criteria too restrictive?
• Did the study meet power? Was a sample size calculation described?
Study design
• How were blinding and randomization conducted?
• Is the study design translatable to clinical practice?
• How were the primary and secondary end points defined? Have those definitions been
validated in critically ill patients?
• Is the primary outcome scientifically valid and meaningful?
Outcomes • Are the secondary outcomes clearly described?
• How were adverse effects defined and analyzed?
• Were the statistical tests appropriate?
• How were the data analyzed (intention-to-treat, per-protocol, as-treated)?
• How large was the treatment effect?
Analysis
• Will the effect size be duplicated in clinical practice?
• Did the author(s) provide an interpretation of the study findings and describe them in the
context of the available knowledge?

A. Integration of Various Types of Knowledge (Table 3)


1. Medical literature
a. Remaining current with evolving literature is a necessary skill for the critical care clinician.
b. Knowledge gained from the primary literature can be objective, can limit bias, and may be
translatable compared with experiential knowledge.
c. The findings of clinical research can be limited to the conditions of the study and may not easily
confer the same benefit in clinical practice.
i. Number needed to treat (NNT) – Quantifies the anticipated effect of a treatment in a patient
population on the basis of study results. A low number signifies an effective treatment; a high
number indicates a less effective therapy. NNT = 1/absolute risk reduction
ii. Number needed to harm (NNH) – Quantifies the associated harm after exposure to a treatment.
A low number signifies a harmful treatment; a high number indicates a safer treatment. NNH
= 1/attributable risk
d. The study results limit the findings attributable to chance but may not fully exclude chance.
e. Fragility index: The number of non-events in the overall sample that need to occur to render a
significant result nonsignificant. In the CRICS-TRIGGERSEP trial, an additional three events for
the outcome of 90-day mortality would have rendered the trial findings nonsignificant (shifting to
a p=0.053), indicating the fragility of the final results (N Engl J Med 2018;378:809-18).
2. Experiential knowledge
a. Knowledge accumulated through clinical experience and for a sustained period is valuable.
b. Valuable when determining how an individual patient differs from a study population in a clinical
trial
c. When possible, this type of knowledge should be reinforced with scientific evidence. Example: N
Engl J Med 2009;361:1925-34

ACCP Updates in Therapeutics® 2022: Critical Care Pharmacy Preparatory Review and Recertification Course

605
Research Design, Biostatistics, and Literature Evaluation

3. Pathophysiologic reasoning
a. Application of physiologic concepts to drive therapeutic choices
b. Can aid in determining short-term goals. Example: Selecting an initial dose of loop diuretic
according to the home medication dose and current therapeutic goal of urinary output greater than
1 L in 24 hours
c. Primary literature on which to base treatment decisions for all clinical scenarios may not exist;
therefore, use of pathophysiologic reasoning is key to the sound provision of pharmaceutical care
for critically ill patients.

B. Negotiating Between Various Types of Medical Knowledge for Direct Application to Patient Care
1. Applying the knowledge gained from the primary literature that is based exclusively on the hierarchy
of study design may not be practical for all patient interactions.
2. Integrating the various types of knowledge can lead to systematic problem solving rather than arbitrary
judgments. This becomes necessary when making systems-based treatment decisions (i.e., treatment
protocols) and understanding when to modify existing protocols to fit the needs of individual patients.

C. Synthesizing the Available Evidence to Develop Treatment Protocols


1. Protocols are intended to synthesize the best available evidence and standardize a series of treatment
options to reduce variability and error while maximizing treatment effect. Examples: Sterile technique
for central line placement, heparin infusion, and aPTT (activated partial thromboplastin time)
monitoring, the mnemonic FASTHUG, sepsis resuscitation protocols)
2. Determining which literature to incorporate and its applicability to the local practice environment
is key.
3. Determine the homogeneity of various studies on the same topic (e.g., early goal-directed therapy
for septic shock) and confounding variables that influence the interpretation of results. Study design
affecting the screening and time to treatment for subjects compared with actual practice, improvements
in the processes of usual care over time, etc.

D. Unique Factors That Promote/Impede the Application of Treatment Protocols


1. Resource use
a. Personnel (e.g., Lancet 2010;375:475-80)
b. Drug shortage
c. Fiscal
2. Ease of protocols
a. Complexity
b. Familiarity
3. Lack of consensus (e.g., sepsis 1-hour bundle)

ACCP Updates in Therapeutics® 2022: Critical Care Pharmacy Preparatory Review and Recertification Course

606
Research Design, Biostatistics, and Literature Evaluation

REFERENCES

Introduction ill patients in the United States and Canada. Am J


1. Angus DC, Barnato AE, Linde-Zwirble WT, et al. Respir Crit Care Med 2004;170:1375-84.
Use of intensive care at the end of life in the United 2. Clark D 3rd, Woods J, Patki D, et al. Digital
States: an epidemiological study. Crit Care Med informed consent in a rural and low-income popu-
2004;32:638-43. lation. JAMA Cardiol 2020;5:845-7.
2. Coopersmith CM, Wunsch H, Fink MP, et al. A 3. Coppolino M, Ackerson L. Do surrogate decision
comparison of critical care research funding and makers provide accurate consent for intensive care
the financial burden of critical illness in the United research? Chest 2001;119:603-12.
States. Crit Care Med 2012;40:1072-9. 4. Dickenson DL. Cross-cultural issues in European
bioethics. Bioethics 1999;13:249-55.
Bioethics 5. Gaille M, Horn R. Solidarity and autonomy: two
1. DeRenzo EG, Moss J, Singer EA. Implications of conflicting values in English and French health
the revised common rule for human participant care and bioethics debates? Theor Med Bioeth
research. Chest 2019;155:272-8. 2016;37:441-6.
2. Emanuel EJ, Wendler D, Grady C. What makes 6. Gaudry S, Hajage D, Schortgen F, et al. Initiation
clinical research ethical? JAMA 2000;283:2701-11. strategies for renal-replacement therapy in the
3. Freedman B. Equipoise and the ethics of clinical intensive care unit. N Engl J Med 2016;375:122-33.
research. N Engl J Med 1987;317:141-5. 7. Grady C. Enduring and emerging challenges of
4. Hodge JG Jr, Gostin LO. Revamping the US fed- informed consent. N Engl J Med 2015;372:855-62.
eral common rule: modernizing human participant 8. Jeste DV, Palmer BW, Appelbaum PS, et al. A new
research regulations. JAMA 2017;317:1521-2. brief instrument for assessing decisional capac-
5. Goligher EC, Douflé G, Fan E. Update in mechani- ity for clinical research. Arch Gen Psychiatry
cal ventilation, sedation, and outcomes 2014. Am J 2007;64:966-74.
Respir Crit Care Med 2015;191:1367-73. 9. Kass N, Pronovost P, Sugarman J, et al. Controversy
6. Harhay MO, Wagner J, Ratcliffe SJ, et al. and quality improvement: lingering questions
Outcomes and statistical power in adult critical about ethics, oversight, and patient safety research.
care randomized trials. Am J Respir Crit Care Med Jt Comm J Qual Patient Saf 2008;34:349-53.
2014;189:1469-78. 10. Newman JT, Smart A, Reese TR, et al. Surrogate
7. Koski G. Ethics, science, and oversight of criti- and patient discrepancy regarding consent for crit-
cal care research: the Office of Human Research ical care research. Crit Care Med 2012;40:2590-4.
Protections. Am J Respir Crit Care Med 11. Papazian L, Forel JM, Gacouin A, et al.
2004;169:982-6. Neuromuscular blockade in early acute respiratory
8. National Commission for the Protection of Human distress syndrome. N Engl J Med 2010;363:1107-16.
Subjects of Biomedical and Behavioral Research. 12. Pronovost P, Needham D, Berenholtz S, et al.
The Belmont Report: Ethical Principles and An intervention to reduce catheter-related blood-
Guidelines for the Protection of Human Subjects stream infections in the ICU. N Engl J Med
in Research. Washington, DC: U.S. Government 2006;355:2725-32.
Printing Office, 1979. 13. Selby JV, Beal AC, Frank L. The Patient-Centered
9. Russell JA. Vasopressin in septic shock: clinical Outcomes Research Institute (PCORI) national
equipoise mandates a time for restraint. Crit Care priorities for research and initial research agenda.
Med 2003;31:2707-9. JAMA 2012;307:1583-4.
14. Silbergleit R, Durkalski V, Lowenstein D, et al.
Practical Challenges to Critical Care Research Intramuscular versus intravenous therapy for
1. American Thoracic Society (ATS). The ethical prehospital status epilepticus. N Engl J Med
conduct of clinical research involving critically 2012;366:591-600.

ACCP Updates in Therapeutics® 2022: Critical Care Pharmacy Preparatory Review and Recertification Course

607
Research Design, Biostatistics, and Literature Evaluation

15. Silverman HJ, Luce JM, Schwartz J. Protecting 11. Sevransky JE, Checkley W, Martin GS. Critical
subjects with decisional impairment in research: care trial design and interpretation: a primer. Crit
the need for a multifaceted approach. Am J Respir Care Med 2010;38:1882-9.
Crit Care Med 2004;169:10-4. 12. Stanski NL, Wong HR. Prognostic and pre-
16. Tait AR, Voepel-Lewis T. Digital multimedia: dictive enrichment in sepsis. Nat Rev Nephrol
a new approach for informed consent? JAMA 2020;16:20-31.
2015;313:463-4. 13. Suissa S. Effectiveness of inhaled corticosteroids
17. World Health Organization (WHO). The Process of in chronic obstructive pulmonary disease: immor-
Obtaining Informed Consent. Available at https:// tal time bias in observational studies. Am J Respir
www.who.int/docs/default-source/ethics/process- Crit Care Med 2003;168:49-53.
seeking-if-printing.pdf?sfvrsn=3fac5edb_4. 14. Talisa VB, Yende S, Seymour CW, et al. Arguing
for adaptive clinical trials in sepsis. Front Immunol
Research Design 2018;9:1502.
1. Ely EW, Shintani A, Truman B, et al. Delirium
as a predictor of mortality in mechanically ven- Statistical Analysis
tilated patients in the intensive care unit. JAMA 1. Gaddis ML, Gaddis GM. Introduction to bio-
2004;291:1753-62. statistics. I. Basic concepts. Ann Emerg Med
2. Gerhard T. Bias: considerations for research prac- 1990;19:86-9.
tice. Am J Health Syst Pharm 2008;65:2159-68. 2. Gaddis GM, Gaddis ML. Introduction to biosta-
3. Gershon AS, Jafarzadeh S, Wilson KC, et al. tistics. II. Descriptive statistics. Ann Emerg Med
Clinical knowledge from observational studies. 1990;19:309-15.
Am J Respir Crit Care Med 2018;198:859-67. 3. Gaddis GM, Gaddis ML. Introduction to bio-
4. Groenwold RHH, Donders ART, Roes KCB, et al. statistics. III. Sensitivity, specificity, predictive
Dealing with missing outcome data in randomized value and hypothesis testing. Ann Emerg Med
trials and observational studies. Am J Epidemiol 1990;19:591-7.
2012;175:210-7. 4. Gaddis GM, Gaddis ML. Introduction to bio-
5. Krag M, Marker S, Perner A, et al. Pantoprazole in statistics. IV. Statistical inference techniques in
patients at risk for gastrointestinal bleeding in the hypothesis testing. Ann Emerg Med 1990;19:820-5.
ICU. N Engl J Med 2018;379:2199-208. 5. Gaddis GM, Gaddis ML. Introduction to bio-
6. McMahon AD. Approaches to combat with con- statistics. V. Statistical inference technique for
founding by indication in observational studies of hypothesis testing with nonparametric data. Ann
intended drug effects. Pharmacoepidemiol Drug Emerg Med 1990;19:1054-9.
Saf 2003;12:551-8. 6. Gaddis ML, Gaddis GM. Introduction to biosta-
7. Mehta S, Burry L, Cook D, et al. Daily sedation tistics. VI. Correlation and regression. Ann Emerg
interruption in mechanically ventilated critically Med 1990;19:1462-8.
ill patients cared for with a sedation protocol. 7. Hernández G, Ospina-Tascón GA, Damiani LP,
JAMA 2012;308:1985-92. et al.; The ANDROMEDA SHOCK Investigators
8. Moodie EE, Stephens DA. Using directed acyclic and the Latin America Intensive Care Network
graphs to detect limitations of traditional regres- (LIVEN). Effect of a resuscitation strategy targeting
sion in longitudinal studies. Int J Public Health peripheral perfusion status vs serum lactate levels
2010;55:701-3. on 28-day mortality among patients with septic
9. Pallmann P, Bedding AW, Choodari-Oskooei B, shock: the ANDROMEDA-SHOCK Randomized
et al. Adaptive designs in clinical trials: why use Clinical Trial. JAMA 201919;321:654-64.
them, and how to run and report them. BMC Med 8. Kier KL. Biostatistical applications in epidemiol-
2018;16:29. ogy. Pharmacotherapy 2011;31:9-22.
10. Scales DC, Dainty K, Hales B, et al. A multifaceted 9. Lorenz E, Köpke S, Pfaff H, et al. Cluster-
intervention for quality improvement in a network randomized studies. Dtsch Arztebl Int
of intensive care units. JAMA 2011;305:363-72. 2018;115:163-8.

ACCP Updates in Therapeutics® 2022: Critical Care Pharmacy Preparatory Review and Recertification Course

608
Research Design, Biostatistics, and Literature Evaluation

10. Yarnell CJ, Granton JT, Tomlinson G. Bayesian research results in the practice of critical care med-
analysis in critical care medicine. Am J Respir Crit icine. Am J Respir Crit Care Med 2012;185:1117-24.
Care Med 2020;201:396-8. 7. Vincent J. Evidence-based medicine in the
11. Zampieri FG, Damiani LP, Bakker J, et al. Effects ICU: important advances and limitations. Chest
of a resuscitation strategy targeting peripheral per- 2004;126:592-600.
fusion status versus serum lactate levels among 8. Weiß KT, Zeman F, Schreml S. A randomized trial
patients with septic shock. A Bayesian reanalysis of early endovenous ablation in venous ulceration:
of the ANDROMEDA-SHOCK Trial. Am J Respir a critical appraisal: Original Article: Gohel MS,
Crit Care Med 2020;201:423-9. Heatly F, Liu X et al. A randomized trial of early
endovenous ablation in venous ulceration. N Engl
Noninferiority Trials J Med 2018; 378:2105-114. Br J Dermatol 2019;
1. Kaji AH, Lewis RJ. Noninferiority trials: is a new 180:51-5.
treatment almost as effective as another. JAMA
2015;313:2371-2.
2. Kaul S, Diamond GA. Good enough: a primer on
the analysis and interpretation of noninferiority
trials. Ann Intern Med 2006;145:62-9.
3. Le Henanff A, Giraudeau B, Baron G, et al. Quality
of reporting of noninferiority and equivalence of
randomized trials. JAMA 2006;295:1147-51.
4. Mulla SM, Scott IA, Jackevicius CA, et al. How
to use a noninferiority trial: Users’ guides to the
medical literature. JAMA 2012;308:2605-11.
5. Temple R, O’Neill R. Guidance for Industry
Noninferiority Clinical Trials. Rockville, MD:
FDA, Department of Health and Human Services,
2010.

Application of Knowledge to Patient Care


1. ANZIC Influenza Investigators, Webb SA, Pettilä
V, Seppelt I, et al. Critical care services and 2009
H1N1 influenza in Australia and New Zealand. N
Engl J Med 2009;361:1925-34.
2. Mohad D, Angus DC. Thought outside the box:
intensive care unit freakonomics and decision
making in the intensive care unit. Crit Care Med
2010;38:S637-41.
3. Murad MH, Montori VM. Synthesizing evidence:
shifting the focus from individual studies to the
body of evidence. JAMA 2013;309:2217-18.
4. Strøm T, Martinussen T, Toft P. A protocol of
no sedation for critically ill patients receiving
mechanical ventilation: a randomised trial. Lancet
2010;375:475-80.
5. Tonelli MR. Integrating evidence into clini-
cal practice: an alternative to evidence-based
approaches. J Eval Clin Pract 2006;12:248-56.
6. Tonelli MR, Curtis JR, Guntupalli KK, et al. An
official multisocietal statement: the role of clinical

ACCP Updates in Therapeutics® 2022: Critical Care Pharmacy Preparatory Review and Recertification Course

609
Research Design, Biostatistics, and Literature Evaluation

ANSWERS AND EXPLANATIONS TO SELF-ASSESSMENT QUESTIONS

1. Answer: C 3. Answer: B
This study seeks to compare two viable treatments of A QI study would likely show the effectiveness using a
hypovolemic shock in trauma patients. Because subject pre/postintervention cohort design. In this type of study,
identification would be trauma patients from the com- informed consent is generally not required because the
munity and treatment would be initiated in the field, treatment is provided to all patients as a standard of care
potential harm is associated with each treatment, and (Answer A is incorrect). A community advertising cam-
it is expected that many potential patients would lack paign might improve the delivery of care and improve
decisional capacity at the time of informed consent. patient adherence, but it is unnecessary to measure the
Although treatment blinding is an essential component effectiveness of the QI initiative (Answer C is incorrect).
to trial design to limit investigator and clinician bias, it The QI initiative is believed necessary and has therefore
is not an ethical consideration (Answer A is incorrect). been deemed to have intrinsic social value (Answer D is
Given the incidence of trauma in the general popula- incorrect). To show the effectiveness of the intervention,
tion and its burden on society, treatments to improve the the syndrome must be recognized before initiating treat-
outcomes of trauma patients are necessary and informa- ment (Answer B is correct).
tive (Answer B is incorrect). Acknowledging the limited
supply of blood products, a superiority trial is essential 4. Answer: C
to help steward the use of a finite resource (Answer D is To effectively determine the incidence and clinical
incorrect). The issue of informed consent in this study impact of adverse drug events on clinical outcomes in
is challenging, but the issue needs to be addressed for the ICU, it would be unethical to randomize patients
the ethical conduct of this study (Answer C is correct). to experience the event (Answer A is incorrect). A ret-
There is precedent for this trial to receive an exception rospective design would not be ideal because of the
for informed consent requirements under the FDA code limitations in data extraction, assignment of events,
of regulations. and interpretation of causality (Answers B and D are
incorrect). A prospective observational design would
2. Answer: C allow the investigator team to identify the incidence
The study seeks to establish the effectiveness of a novel of adverse events, sequential outcomes, and determine
drug compound for septic shock. This is challeng- causality (Answer C is correct).
ing because of the dramatic improvements in 28-day
mortality during the past decade. To show the effec- 5. Answer: D
tiveness of a novel drug compound, the drug should be A correct interpretation of the results is recognizing that
tested in a representative population of patients with even though the OR suggests an associated increase of
the disease. Excluding patients because of baseline 30% in the risk of being exposed to CDI, the 95% CI
comorbidity would limit external validity (Answer A is crosses 1, meaning that the odds of exposure to CDI
incorrect). Including patients with sepsis, which has a are as likely to increase the risk as to decrease that risk
lower 28-day mortality rate than septic shock, would not (Answer D is correct; Answers A–C are incorrect).
address the issue of an appropriate end point (Answer
B is incorrect). Limiting study inclusion to patients 6. Answer: B
with a pneumonia etiology would be inappropriate Use of albumin reduced the odds of mortality (and con-
unless the pharmacology of the novel drug specifically versely, increased the odds of survival) with an OR
targeted pneumonia pathophysiology (Answer D is of 0.45, and the 95% CIs were all less than 1 in treat-
incorrect). Answer C is correct as the anticipated inci- ing hypovolemic shock in trauma. In addition, the OR
dence of mortality has changed overtime as well as the was 1.1 for septic shock, whereas the 95% CI crossed
likely anticipated minimum detectable effect. With all 1, showing that the odds were as likely that albumin
other assumptions remaining equal, the authors will increased mortality as that it reduced it (Answer B is
have to increase enrollment to find the effect relative to correct). Because the OR and the 95% CI for treating
previous trials. hypovolemic shock are both less than 1, mortality was

ACCP Updates in Therapeutics® 2022: Critical Care Pharmacy Preparatory Review and Recertification Course

610
Research Design, Biostatistics, and Literature Evaluation

decreased with albumin use (Answers A, C, and D are


incorrect).

7. Answer: A
This patient case provides a practical example of a criti-
cal care pharmacist’s integration of various types of
knowledge to optimize patient care. The FACT trial
showed that a fluid-conservative strategy improves
ventilator-free days for patients with acute lung injury
and ARDS. Furosemide was used in the study to show
the outcome benefit. However, the study was testing
a treatment strategy, not specifically a drug strategy.
Therefore, it can be reasoned that similar treatment
outcomes can be shown with similar drugs if the study
drug is unavailable – in this case, because of drug short-
age. Knowledge of trial design is helpful but not critical
to optimizing this patient’s therapy with bumetanide
(Answer B is incorrect). Hemodialysis is invasive,
requires finite resource use, and has an associated mor-
bidity risk (Answer C is incorrect). Because this patient
case does not include a broader hypothesis test in a
systematic design, this is not a research activity, and
informed consent is not required (Answer D is incor-
rect). A critical care pharmacist, using her knowledge
of the FACT trial (medical knowledge) together with an
understanding and experience with bumetanide therapy
(experiential knowledge), can develop a treatment plan
(Answer A is correct).

8. Answer: A
Acute respiratory distress syndrome is a clinical
syndrome with an associated mortality with each pro-
gressing phase (mild, moderate, and severe). Although
ARDS is also a constellation of findings and pathologic
observations, patients with this disease present with
a similar finding of severe, refractory hypoxia from a
common etiology (Answer B is incorrect). Given the
associated mortality of about 45% for severe ARDS, a
finite resource should be prioritized for it (in this case,
a NMBA) (Answer C is incorrect). Mortality/survival
is readily identified as an end point of social value for
research design (Answer D is incorrect). Given the rela-
tively low mortality associated with mild ARDS (around
20%) compared with severe ARDS (around 45%), admin-
istering NMBAs would less likely improve survival and
more likely increase harm if systematically administered
to patients with mild ARDS (Answer A is correct).

ACCP Updates in Therapeutics® 2022: Critical Care Pharmacy Preparatory Review and Recertification Course

611
Shock Syndromes I:
Introduction, Vasodilatory,
and Sepsis
Gretchen L. Sacha, Pharm.D., BCCCP
Cleveland Clinic
Cleveland, Ohio
Shock Syndromes I: Introduction, Vasodilatory, and Sepsis

Shock Syndromes I:
Introduction, Vasodilatory,
and Sepsis
Gretchen L. Sacha, Pharm.D., BCCCP
Cleveland Clinic
Cleveland, Ohio

ACCP Updates in Therapeutics® 2022: Critical Care Pharmacy Preparatory Review and Recertification Course

615
Shock Syndromes I: Introduction, Vasodilatory, and Sepsis

Learning Objectives SVR Systemic vascular resistance


SVV Stroke volume variation
1. Distinguish between the various shock syndromes Vo2 Oxygen consumption
on the basis of a patient’s clinical and hemodynamic
parameters.
2. Interpret hemodynamic data from monitoring Self-Assessment Questions
devices and markers of perfusion. Answers and explanations to these questions may be
3. Devise a treatment strategy for when to use intra- found at the end of this chapter.
venous fluids and/or vasopressors in a patient with
shock. Questions 1 and 2 pertain to the following case.
4. Develop a treatment pathway for the care of patients An 80-year-old woman presents to the intensive care
with sepsis or septic shock that incorporates current unit (ICU) with septic shock caused by an Escherichia
evidence and the Surviving Sepsis Campaign guide- coli urinary tract infection. Pertinent vital signs on
line recommendations. admission are as follows: blood pressure 80/40 mm
Hg, heart rate 155 beats/minute with a rhythm of atrial
fibrillation, respiratory rate 26 breaths/minute, and tem-
Abbreviations in This Chapter perature 105.8°F (41°C). On physical examination, the
AVP Arginine vasopressin patient is weak, lethargic, and confused. Pertinent labo-
Cao2 Arterial oxygen content ratory values are as follows: sodium (Na) 155 mEq/L,
CO Cardiac output potassium (K) 3.6 mEq/L, serum creatinine (SCr) 1.8
CVP Central venous pressure mg/dL, and lactate 4.2 mmol/L.
Do2 Oxygen delivery
Hgb Hemoglobin 1. Which clinical symptoms and physiologic variables
ICU Intensive care unit most likely indicate that this patient has a shock
IVC Inferior vena cava syndrome?
LV Left ventricular A. Fever, lethargy, and tachycardia.
LVOT VTI Left ventricular outflow tract velocity B. Hypotension, fever, and tachypnea.
time integral C. Hypotension, confusion, and hyperlactatemia.
MAP Mean arterial pressure D. Tachypnea, fever, and hyperlactatemia.
O2ER Oxygen extraction ratio
PAC Pulmonary artery catheter 2. Which variable is most likely contributing to
Pco2 Partial pressure of carbon dioxide impaired oxygen delivery (Do2) to the patient’s end
PCWP Pulmonary capillary wedge pressure organs?
PH Pulmonary hypertension
A. Serum lactate.
PLR Passive leg raising (test)
B. Atrial fibrillation.
PPV Pulse pressure variation
C. Acute kidney injury.
PRBC Packed red blood cell
D. Fever.
PVR Pulmonary vascular resistance
RV Right ventricular
3. A 62-year-old woman (weight 119 kg) develops
SBP Systolic blood pressure
ventilator-associated pneumonia in the setting of
SCr Serum creatinine
prolonged intubation after aortic valve replacement
Scvo2 Central venous oxygen saturation
surgery. Her pneumonia is complicated by septic
SOFA Sequential Organ Failure Assessment
shock, and she is given 2 L of 0.9% sodium chloride
SSC Surviving Sepsis Campaign
and 1 L of 5% albumin for resuscitation and initi-
Sto2 Tissue oxygen saturation
ated on norepinephrine. Her laboratory values are as
SV Stroke volume
follows: Na 144 mEq/L, chloride (Cl) 110 mEq/L,
Svo2 Venous oxygen saturation
K 3.8 mEq/L, bicarbonate 18 mEq/L, SCr 1.8 mg/
dL, arterial pH 7.28, and albumin 3.2 g/dL. Having

ACCP Updates in Therapeutics® 2022: Critical Care Pharmacy Preparatory Review and Recertification Course

616
Shock Syndromes I: Introduction, Vasodilatory, and Sepsis

determined that the patient is still fluid responsive, pressure variation (PPV) 18%. The patient has a cen-
you would like to give another fluid bolus. Which tral venous catheter in his right femoral vein. Which
fluid is best for the fluid bolus? next step is best?
A. 0.9% sodium chloride. A. Give 1 L of lactated Ringer’s solution.
B. 5% albumin. B. Do a passive leg raising (PLR) test.
C. 6% hydroxyethyl starch. C. Measure CVP.
D. Lactated Ringer’s solution. D. Send a blood gas for venous oxygen saturation.

4. A 48-year-old man (weight 82 kg) presents to the 6. A 19-year-old man is admitted to the medical ICU
medical ICU for septic shock secondary to a urinary for hypotension after being stung by a bee. He was
tract infection. His medical history is significant only given intramuscular epinephrine by emergency
for hypertension. The patient has an initial mean medical services and transferred to the emergency
arterial pressure (MAP) of 58 mm Hg and a lactate department (ED). On arrival at the ED, his blood
concentration of 4.8 mmol/L. The patient is initi- pressure was 78/42 mm Hg; he was given 1 L of
ated on broad-spectrum antimicrobials, has a central 0.9% sodium chloride, diphenhydramine, famoti-
venous catheter placed in his right subclavian vein, dine, and methylprednisolone. The patient remained
and is resuscitated with quantitative resuscitation. He hypotensive but responded to an additional 2 L of
receives 2 L of 0.9% sodium chloride total and 2 L 0.9% sodium chloride. He was transferred to the
of lactated Ringer’s solution, and he is initiated on medical ICU for further treatment. On arrival in the
norepinephrine. Three hours after presentation, his medical ICU, his MAP is 62 mm Hg. A right inter-
current pertinent vital signs, hemodynamic param- nal jugular central venous catheter is placed, which
eters, and laboratory values are as follows: MAP 68 shows CVP 3 mm Hg, Scvo2 61%, venous lactate
mm Hg on norepinephrine 8 mcg/minute, central concentration 4.4 mmol/L, and hemoglobin (Hgb)
venous pressure (CVP) 10 mm Hg, central venous 9.6 g/dL. Together with further fluid resuscitation,
oxygen saturation (Scvo2) 72%, urinary output 0.2 which agent would be best to initiate or administer?
mL/kg/hour, and lactate 4.6 mmol/L. Which is the A. Packed red blood cells (PRBCs).
next best step for the patient’s hemodynamic therapy? B. Dobutamine.
A. Continue current therapy. C. Milrinone.
B. Increase the norepinephrine dose. D. Norepinephrine.
C. Give 1 L of 0.9% sodium chloride.
D. Initiate dobutamine. 7. A 56-year-old man (weight 66 kg) presents to the
ED with presumed community-acquired pneumo-
5. An 82-year-old man is admitted to the surgical nia. Blood cultures are obtained, and the patient is
ICU after an exploratory laparotomy and small given ceftriaxone 1 g and levofloxacin 750 mg. His
bowel resection for a small bowel obstruction that initial blood pressure is 83/47 mm Hg with a lactate
was complicated by fecal peritonitis and hypoten- concentration of 6.2 mmol/L, and he is given 1.5 L
sion. The patient received 2 L of lactated Ringer’s of 0.9% sodium chloride over 1 hour. Subsequently,
solution, 500 mL of 5% albumin, and 500 mL of his blood pressure is 92/54 mm Hg with a lactate
6% hydroxyethyl starch in the operating room, concentration of 4.6 mmol/L and urinary output of
but vasopressors were never initiated. He remains 30 mL/hour. A central venous catheter placed in his
intubated and mechanically ventilated, requiring right internal jugular vein shows a CVP of 6 mm
a 90% fraction of inspired oxygen (Fio2), with the Hg, and a venous blood gas reading obtained from
following vital signs: heart rate 131 beats/minute in the central venous catheter shows an Scvo2 of 63%,
atrial fibrillation, MAP 62 mm Hg (by arterial blood Hgb 9.2 g/dL, and hematocrit (Hct) 28%. Which
pressure catheter), respiratory rate 22 breaths/min- would be the best therapy for this patient right now?
ute, and temperature 100.8°F (38.2°C). An arterial A. 0.9% sodium chloride.
blood gas show lactate 5.2 mmol/L. An arterial pulse B. 5% albumin.
pressure waveform analysis monitor shows pulse C. Phenylephrine.
D. PRBCs.

ACCP Updates in Therapeutics® 2022: Critical Care Pharmacy Preparatory Review and Recertification Course

617
Shock Syndromes I: Introduction, Vasodilatory, and Sepsis

8. A 68-year-old woman (weight 88 kg) presents to the


ED with a urinary tract infection. Her medical his-
tory is significant for paroxysmal atrial fibrillation.
The patient’s vital signs in the ED are as follows:
blood pressure 83/47 mm Hg, heart rate 118 beats/
minute in atrial fibrillation, respiratory rate 24
breaths/minute, and temperature 102°F (38.9°C).
Her laboratory values of interest in the ED include
white blood cell count (WBC) 19.6 × 103 cells/mm3,
Hgb 9.3 g/dL, albumin 2.4 g/dL, lactate 4.9 mmol/L,
and SCr 1.2 mg/dL. Blood and urinary cultures are
obtained, and she receives levofloxacin 750 mg and
3 L of 0.9% sodium chloride. Thirty minutes after
completing the 0.9% sodium chloride infusion, her
blood pressure is 92/49 mm Hg with a lactate con-
centration of 4.6 mmol/L and urinary output of 30
mL/hour. Which would be the best therapy for this
patient right now?
A. Norepinephrine.
B. Vasopressin.
C. Phenylephrine.
D. Dopamine.

9. A 34-year-old man is admitted to the surgical


ICU with septic shock associated with necrotizing
soft tissue infection of the right leg. He received
100% oxygen by high-flow mask; adequate broad-
spectrum antibiotics with vancomycin, piperacillin/
tazobactam, and clindamycin; and quantitative
resuscitation. The patient has been resuscitated
for the past 4 hours with 5 L of normal saline
and currently is hemodynamically unstable on
norepinephrine 14 mcg/minute with a corresponding
blood pressure of 92/45 mm Hg and heart rate of 132
beats/minute in sinus rhythm. His CVP is 12 mm
Hg, Scvo2 72%, urinary output 0.3 mL/kg/hour, and
lactate 7.4 mmol/L. Which would be best to initiate
for this patient?
A. 0.9% sodium chloride.
B. Vasopressin.
C. Phenylephrine.
D. Epinephrine.

ACCP Updates in Therapeutics® 2022: Critical Care Pharmacy Preparatory Review and Recertification Course

618
Shock Syndromes I: Introduction, Vasodilatory, and Sepsis

BPS Critical Care Pharmacy Specialist Examination Content Outline

This chapter covers the following sections of the Critical Care Pharmacy Specialist Examination Content Outline:
1. Domain 1: Clinical Knowledge and Application
a. Task 1: 1-3
b. Task 3: 1-6
c. Task 4: 1-5, 8
d. Task 5: 1, 7
e. Task 6: 1-4
f. Task 7: 1-7
2. Domain 2: Practice Management, Policy, and Quality Improvement
a. Task 1: 2
b. Task 3: 1, 3
c. Task 4: 2, 3
3. Domain 3: Evidence-Based Medicine, Scholarship, Education, and Professional Development
a. Task 1: 3

ACCP Updates in Therapeutics® 2022: Critical Care Pharmacy Preparatory Review and Recertification Course

619
Shock Syndromes I: Introduction, Vasodilatory, and Sepsis

I.  INTRODUCTION

A. Shock
1. Shock is a heterogeneous group of syndromes best defined as “acute circulatory failure.” This arises
when the tissues receive an insufficient supply of oxygen to be able to perform vital metabolic functions.
2. Shock is often categorized into four distinct etiology mechanisms: (1) hypovolemic, (2) obstructive, (3)
distributive and vasodilatory, and (4) cardiogenic. In some clinical scenarios, multiple shock syndromes
can occur simultaneously.
3. The diagnosis of shock typically includes the interpretation of three variables: hemodynamic assessment,
clinical presentation, and biochemical signs.
4. In many cases, shock is first identified by the presence of hypotension. However, the blood pressure
limits used to define shock are arbitrary and may not be patient-specific (e.g., a patient with hypertension
before critical illness).
a. In shock states, the value typically used to describe hypotension is a systolic blood pressure (SBP)
less than 90 mm Hg or a mean arterial pressure (MAP) less than 70 mm Hg.
b. These values may vary within a range to permit autoregulation, allowing acceptable perfusion in
the setting of acute hypotension.
5. Clinical presentation of shock may be subtle and can manifest in many different ways. Usually, shock
is identified through an assessment of mentation, skin, and kidney function.
a. Assessment of mentation should include a careful examination for signs of confusion and
obtundation. These signs should be compared with those in the patient’s preexisting status. This
may be challenging in a patient with a poor medical history or a diminished baseline status.
b. Evidence of an existing shock syndrome can manifest with decreased capillary refill and cold,
clammy skin.
c. Altered kidney function in the setting of shock primarily presents with reduced urinary output
(e.g., less than 0.5 mL/kg/hour). Laboratory values such as serum creatinine (SCr) often lag behind
the immediate observation of urine volume and quality.
6. Biochemical assessment of patients with shock typically reveals hyperlactatemia (greater than 2
mmol/L) or reduced venous oxygen saturation (Svo2) (less than 70%), indicating abnormal cellular
oxygen metabolism.

B. Physiology
1. Hemodynamic parameters can be either directly measured from a monitoring device or calculated
according to direct measurements (see Table 1).

Table 1. Hemodynamic and Oxygen Transport Values


Value Equation (as applicable) Normal Value
Systolic blood pressure (SBP) 90–140 mm Hg
Diastolic blood pressure (DBP) 60–90 mm Hg
Mean arterial blood pressure (MAP) a
[SBP + (2 × DBP)]/3 70–100 mm Hg
Heart rate (HR) 60–80 beats/min
Cardiac output (CO) b
HR•SV 4–7 L/min
Cardiac index (CI) CO/BSA 2.5–4.2 L/min/m2
Stroke volume (SV) CO/HR 60–130 mL/beat
Pulmonary artery systolic pressure (PASP) 20–30 mm Hg

ACCP Updates in Therapeutics® 2022: Critical Care Pharmacy Preparatory Review and Recertification Course

620
Shock Syndromes I: Introduction, Vasodilatory, and Sepsis

Table 1. Hemodynamic and Oxygen Transport Values (continued)


Value Equation (as applicable) Normal Value
Pulmonary artery diastolic pressure
8–12 mm Hg
(PADP)
Mean pulmonary artery pressure (mPAP) [PASP + (2 × PADP)]/3 12–15 mm Hg
Pulmonary capillary wedge pressure
(PCWP) or pulmonary arterial occlusion 5–12 mm Hg
pressure (PAOP)
Central venous pressure (CVP) or right
2–6 mm Hg
atrial pressure (RAP)
80 × [(mPAP − PCWP)/CO] 20–120 dynes•s•cm-5
Pulmonary vascular resistance (PVR)c
(divide by 80 for Wood units) (< 2 Wood units)
Systemic vascular resistance (SVR)c 80 × [(MAP − CVP)/CO] 800–1200 dynes•s•cm-5
Oxygen delivery (Do2)d 10 × CO (L/min) × Cao2 520–570 mL/min/m2
(1.34 × Hgb × Sao2) +
Arterial oxygen content (Cao2)d 20 mL/dL
(0.003 × Pao2)
(1.34 × Hgb × Svo2) +
Venous oxygen content (Cvo2)d 15 mL/dL
(0.003 × Pvo2)
Resting oxygen consumption (Vo2)d 10 × CO (L/min) × (Cao2 − Cvo2) 110–160 mL/min/m 2
Oxygen extraction ratio (O2ER) Vo2/Do2 × 100 20%–30%
a
May be directly measured or calculated.
b
May be measured using several mechanisms, including thermodilution with a pulmonary artery catheter. May also be calculated using the Fick equation (see later in
the chapter).
c
May also be expressed as an indexed value calculated by dividing the value by body surface area. BSA = body surface area in square meters.
d
All are components of the Fick principle for cardiac output measurement.
BSA = body surface area; Cao2 = arterial oxygen content; CI = cardiac index; CO = cardiac output; Cvo2 = venous oxygen content; CVP = central venous pressure;
DBP = diastolic blood pressure; Do2 = oxygen delivery; Hgb = hemoglobin; HR = heart rate; MAP = mean arterial pressure; mPAP = mean pulmonary artery
pressure; O2ER = oxygen extraction ratio; PADP = pulmonary artery diastolic pressure; Pao2 = arterial partial pressure of oxygen or arterial oxygen tension;
PAOP = pulmonary arterial occlusion pressure; PASP = pulmonary arterial systolic pressure; PCWP = pulmonary capillary wedge pressure; P vo2 = venous partial
pressure of oxygen or venous oxygen tension; PVR = pulmonary vascular resistance; RAP = right atrial pressure; Sao2 = arterial oxygen saturation; SBP = systolic
blood pressure; SV = stroke volume; Svo2 = venous oxygen saturation; Vo2 = resting oxygen consumption.
Modified with permission from: Wittbrodt ET, Tietz KJ. Shock syndromes. In: Carter BL, Lake KD, Raebel MA, et al., eds. Pharmacotherapy Self-Assessment Program,
3rd ed. Module 2: Critical Care. Kansas City, MO: American College of Clinical Pharmacy, 1998:87-127.

2. MAP is the driving pressure for peripheral blood flow (and end-organ perfusion). Sufficient arterial
pressure allows redistribution of cardiac output (CO) to vital organs.
3. Blood pressure is the product of CO and systemic vascular resistance (SVR).
4. CO is the product of heart rate and stroke volume (SV).
5. SV is determined by many factors, but predominantly preload, intrinsic contractility, and afterload.
a. Preload refers to ventricular end-diastolic volume and is proportionally related to SV (i.e., when
preload increases, the SV increases) by the Frank-Starling mechanism (though the magnitude of
this relationship is reduced beyond a point of ventricular increasing end-diastolic volume).
b. Intrinsic contractility is the ability of the myocardium to contract and may be reduced by several
factors, including myocardial ischemia, cardiomyopathy, and sepsis.
c. Afterload is the force the ventricle must overcome to eject its volume and is inversely related to SV
(i.e., when afterload increases, the SV decreases). Left ventricular (LV) afterload is predominantly
influenced by aortic pressure, whereas right ventricular (RV) afterload is predominantly influenced
by pulmonary artery pressure.

ACCP Updates in Therapeutics® 2022: Critical Care Pharmacy Preparatory Review and Recertification Course

621
Shock Syndromes I: Introduction, Vasodilatory, and Sepsis

6. SVR (also termed total peripheral resistance) is the resistance to flow that must be overcome by the left
ventricle.
a. SVR is the major determinant of LV afterload.
b. Systemic vasoconstriction increases SVR, whereas vasodilation decreases SVR.
c. Skin temperature may be used as an approximation (surrogate) of SVR, in which warm skin
temperature suggests decreased SVR (vasodilation) and cold skin temperature suggests increased
SVR (vasoconstriction).
7. The right ventricle better tolerates increases in ventricular volume (preload) than increases in afterload.
Contrarily, the left ventricle better tolerates increases in afterload than increases in ventricular volume.
8. Coronary artery perfusion occurs primarily in diastole. Aortic diastolic pressure must be sufficient to
ensure perfusion of the coronary arteries.

C. Do2 and Oxygen Consumption (Vo2)


1. The circulatory system delivers oxygen and vital nutrients to the tissue beds for homeostasis and end-
organ function.
2. Oxygen is inspired and delivered to the alveoli, where it binds reversibly to hemoglobin.
3. Oxygen bound to hemoglobin is then transported by CO to the tissues. The rate of Do2 is the product
of the CO and the arterial oxygen content (Cao2), as described in Table 1. Metabolic function of tissue
beds requires consistent Do2.
4. At the tissues, oxygen dissociates from hemoglobin and is taken up by the mitochondria through
systemic capillaries for aerobic metabolism. Oxygen uptake or Vo2 is the rate at which oxygen transfers
from systemic capillaries into the tissues and is the byproduct of CO, Cao2, and venous oxygen content
(Cvo2).
5. The Fick equation states that CO = Vo2/(Cao2 − Cvo2).
6. The oxygen extraction ratio (O2ER), or the ratio of Vo2/Do2, is 20%–30% at resting state, meaning that
about 25% of the oxygen delivered to the capillaries is taken up by the tissues. The O2ER is relatively
stable and can accommodate temporary fluctuations in Do2 or Vo2. Sustained Do2/Vo2 mismatches
contribute to tissue hypoxia and deranged metabolic function.
7. Treatment of shock syndromes should be rapid to minimize permanent tissue and organ damage.
8. In the early stages of a shock state, blood pressure is preserved through stimulation of the sympathetic
system, release of endogenous vasopressin, and vasoconstriction through the formation of angiotensin
II. The synergy of these actions preserves blood flow and Do2 to vital organs.
9. Blood flow is prioritized to maximize Do2 to the heart and brain. Consequently, blood flow to extravital
organs (e.g., skin, gut, kidneys) is redirected.
10. When the endogenous response to shock is insufficient either in magnitude or duration, blood pressure
decreases, and overt shock develops.

II.  MONITORING TECHNIQUES

A. Hemodynamic Monitoring Devices


1. Hemodynamic variables may be obtained through noninvasive or invasive monitoring devices
(Table 2).
2. In patients with shock in whom the clinical examination does not lead to a clear diagnosis, further
hemodynamic assessment is recommended.

ACCP Updates in Therapeutics® 2022: Critical Care Pharmacy Preparatory Review and Recertification Course

622
Shock Syndromes I: Introduction, Vasodilatory, and Sepsis

3. In general, less invasive devices are desired, but they often have limited accuracy in estimating
hemodynamic parameters compared with invasive devices.
4. When further hemodynamic assessment is indicated, echocardiography is the preferred modality to
initially evaluate the type of shock as opposed to more invasive approaches.
5. Some ICU monitors can display additional hemodynamic parameters (e.g., automated PPV from an
arterial catheter) without additional equipment or devices.
6. Bioimpedance uses electrodes on the skin or an endotracheal tube to estimate the CO through
mathematical modeling of the impedance by the body to electrical current flow induced by cyclic changes
in blood flow during the cardiac cycle. Bioreactance improves on bioimpedance models by analyzing
the variations in the whole frequency spectra of the oscillating current instead of only variations in
amplitude changes. This increases the signal-to-noise ratio and improves device performance.

Table 2. Hemodynamic Monitoring Devices


Device or Obtainable
Advantages Limitations
Category Parameters
• Limited accuracy in shock
• Noninvasive • Does not provide continuous
Noninvasive BP
SBP, DBP, MAP • Bedside practitioner monitoring
monitoringa
familiarity • Less sensitive in predicting end-organ
dysfunction
• Invasive
• More accurate BP
• Inaccurate damping influences SBP
measurement in shock
and DBP measurements (MAP still
than noninvasive
Arterial BP accurate)
SBP, DBP, MAP methods
catheter • Catheter-related infection
• Ready access for arterial
• Brachial site lacks collateral
blood gas sampling
circulation (may result in decreased
• Continuous monitoring
distal arterial perfusion)
• Easier and safer to insert
than a PAC • CVP/RAP not a true estimate of LV
• Scvo2 may be available end-diastolic pressure
Central venous as a continuous • CVP/RAP does not accurately predict
CVP/RAP, Scvo2
catheter (CVC) measurement fluid responsiveness
• Access for administra- • Scvo2 not equivalent to Svo2 (see later
tion of highly osmotic in the chapter)
and caustic agents

ACCP Updates in Therapeutics® 2022: Critical Care Pharmacy Preparatory Review and Recertification Course

623
Shock Syndromes I: Introduction, Vasodilatory, and Sepsis

Table 2. Hemodynamic Monitoring Devices (continued)


Device or Obtainable
Advantages Limitations
Category Parameters
• Outcomes data supporting superiority
to CVC lacking
Measured: PASP, • May cause arrhythmias
PADP, mPAP, • Assumes right heart function
CVP/RAP, PCWP/ • Only method available approximates left heart function
PAOP, CO, and CI to directly measure (usually, but not always, true)
by thermodilution pulmonary artery • Fick CO calculation typically uses an
Pulmonary artery or continuous pressures estimated value for Vo2, which may
catheter (PAC) measurement (copper • Direct measurement of be falsely low in a patient with septic
filament adapted CO and Svo2 (may be shock and underestimate CO
catheter), Svo2 available as continuous • Valvular abnormalities may make
Calculated: PVR, variables) values inaccurate (particularly mitral
SVR, CO, and CI by stenosis, mitral regurgitation, tricuspid
Fick equation, SV regurgitation, or aortic regurgitation)
• Correct catheter tip location (lung
zone 3) needed for accurate readings
Cardiac chamber
size and function, • Noninvasive
• Subjectivity of user assessment
pericardial (transthoracic)
• Not done continuously; therefore,
appearance (and • Visualization of
cannot detect acute changes or must
presence of fluid), ventricular function
be repeated when the patient’s status
Echocardiography IVC collapsibility/ instead of presumed
changes
distensibility, ejection function based on CO
• May have limited visibility/windows
fraction, RVSP (an • IVC collapsibility
depending on patient body habitus or
estimate of PASP), can predict fluid
positioning
LVOT VTI (to responsiveness
calculate CO/CI)
• Assumptions used by the device
may not be valid in the setting of
Esophageal • Ease of use hemodynamic instability (fixed
Doppler (ODM • May be used to partition of blood flow to cephalic
CO, CI, SV, flow time
II, CardioQ, determine fluid vessels and descending aorta, constant
HemoSonic 100) responsiveness aortic cross-sectional area)
• Accuracy depends on position (need
for frequent repositioning)

ACCP Updates in Therapeutics® 2022: Critical Care Pharmacy Preparatory Review and Recertification Course

624
Shock Syndromes I: Introduction, Vasodilatory, and Sepsis

Table 2. Hemodynamic Monitoring Devices (continued)


Device or Obtainable
Advantages Limitations
Category Parameters
• Accuracy relies on optimal arterial
waveform from arterial catheter
• Inaccurate in patients with mitral or
aortic valve disease or when used
concomitantly with an intra-aortic
balloon pump
• Continuous
• Arrhythmias reduce the accuracy of
Arterial pulse measurement of values
reported CO and CI (though this may
pressure wave- • Allows for assessment
be accounted for by internal software
form analysis of SVV and PPV, which
with some devices)
(FloTrac/Vigileo, CO and CI, SV, SVR are dynamic markers of
• Accuracy may be limited during rapid
PiCCOplus, (calculated), SVV, fluid responsiveness in
changes in vascular resistance
PulsioFlex, PPV mechanically ventilated
• Some devices require a CVC in
LiDCO plus, patients (see later in the
addition to an arterial pressure catheter
PRAM-MostCare, chapter)
• Interpretation of SVV and PPV is
Nexfin) • Minimally invasive
limited by the need for positive
(Nexfin is noninvasive)
pressure ventilation, no spontaneous
breaths being triggered by the patient,
and relatively large tidal volumes
• SVV and PPV are not accurate
predictors of fluid responsiveness in
the setting of arrhythmias
• Noninvasive
• Conflicting validation results with
Bioimpedance/ • NICOM CO correlates
BioZ and ECOM, particularly in
bioreactance Continuous CO and well with CO values
patients with septic shock
(NICOM, BioZ, CI, SV, SVR, SVV from thermodilution
• ECOM requires endotracheal
ECOM) and pulse pressure
intubation
waveform analysis
a
Includes manual sphygmomanometry and automated oscillometric (cuff) techniques.
BP = blood pressure; CVC = central venous catheter; IVC = inferior vena cava; LV = left ventricular; LVOT VTI = left ventricular outflow tract velocity time integral;
PAC = pulmonary artery catheter; PPV = pulse pressure variation; RVSP = right ventricular systolic pressure; Scvo2 = central venous oxygen saturation; SVR = systemic
vascular resistance; SVV = stroke volume variation.
Information from: Alhashemi JA, Cecconi M, Hofer CK. Cardiac output monitoring: an integrative perspective. Crit Care 2011;15:214.

B. Markers of Perfusion
1. Global perfusion
a. End-organ function (altered mental status, low urinary output, and mottled skin, as noted earlier)
b. Elevated blood lactate concentration (above 2 mmol/L)
i. Lactate is produced from pyruvate by lactate dehydrogenase as an end product of glycolysis
under anaerobic conditions.
ii. Most lactate is cleared by the liver by conversion back to pyruvate in the Cori cycle, with a
small amount cleared by the kidneys. Severe liver dysfunction may impair lactate clearance
and accentuate lactate concentration elevations in shock.

ACCP Updates in Therapeutics® 2022: Critical Care Pharmacy Preparatory Review and Recertification Course

625
Shock Syndromes I: Introduction, Vasodilatory, and Sepsis

iii. Elevated lactate concentrations may be the result of increased production, decreased clearance,
or both.
(a) Type A lactic acidosis occurs in the setting of Do2/Vo2 mismatch (oxygen demand exceeds
supply).
(b) Type B lactic acidosis is not related to tissue hypoxia and typically occurs in the setting
of impaired lactate clearance or medication-related causes (e.g., metformin, epinephrine,
linezolid, or toxic alcohols).
iv. Arterial and venous lactate concentrations are slightly different in value but may be used
interchangeably. When trending lactate values over time, consistent methods of collection
should be used..
c. Venous oximetry (Scvo2 and Svo2)
i. Central venous oxygen saturation (Scvo2) and mixed venous oxygen saturation (Svo2) are the
oxyhemoglobin saturations of venous blood obtained from a central vein and the pulmonary
artery, respectively, and are expressed as a percentage.
ii. Scvo2 is obtained from a central venous catheter (i.e., subclavian or internal jugular access)
where the catheter tip terminates in the superior vena cava. As such, Scvo2 is more reflective
of oxygen extraction in the brain and upper body than of systemic oxygen extraction. Of
importance, the oxyhemoglobin saturation of blood obtained from a central venous catheter
terminating in the IVC (i.e., femoral access) cannot be used interchangeably with the Scvo2
obtained from a central venous catheter terminating in the superior vena cava and should not
be used as a marker of perfusion.
iii. Svo2 better represents systemic oxygen extraction because it represents the mixing of venous
blood from the superior vena cava, inferior vena cava (IVC), and coronary sinus.
iv. In normal physiology, Scvo2 is about 2%–3% lower than Svo2 because the upper body and the
brain extract more oxygen than the lower body.
v. In the setting of shock, Scvo2 exceeds Svo2 by about 5%–8% because of increased mesenteric
and renal oxygen extraction with a similar cerebral extraction ratio.
vi. The difference between Scvo2 and Svo2 decreases in low CO states.
vii. Although Scvo2 and Svo2 are not equivalent, they have a good (though not perfect) correlation,
and Scvo2 may be a reasonable approximation of Svo2, given its requirement for a PAC for
accurate measurement.
viii. A decreased Scvo2 or Svo2 is a sign that tissue oxygen demands are not completely met by Do2
(more discussion on this topic later in this chapter).
ix. Rearrangement of the Fick equation shows that a decrease in Svo2 indicates a decrease in CO,
whereas an increase in Svo2 indicates an increase in CO.
x. In general, Svo2 values above 70% are considered adequate, whereas Svo2 values less than 40%
are considered critically low and approach the critical O2ER where anaerobic metabolism will
occur and lactate concentrations will increase. Svo2 values of 50%–70% by themselves do
not lead to firm conclusions about the O2ER and must be interpreted in the context of other
markers of tissue perfusion (e.g., lactate concentrations).
xi. Svo2 values above 80% likely indicate poor tissue oxygen extraction capacity.
(a) This may occur because of the heterogeneity of microvascular and macrovascular blood
flow (i.e., microcirculatory dysfunction), peripheral shunting of oxygen past the tissues, or
impaired mitochondrial oxygen use.
(b) Patients with septic shock and venous hyperoxia (Scvo2 greater than 89%) within the first
6 hours of their treatment had a higher mortality than those with normoxia (Scvo2 71%–
89%).

ACCP Updates in Therapeutics® 2022: Critical Care Pharmacy Preparatory Review and Recertification Course

626
Shock Syndromes I: Introduction, Vasodilatory, and Sepsis

2. Regional tissue perfusion


a. Microcirculatory blood flow
i. The microcirculation consists of arterioles, capillaries, and venules and is where oxygen
release to the tissues occurs.
ii. Traditional resuscitation strategies have focused on hemodynamic and Do2 end points (the
“macrocirculation”), but the microcirculation plays a key role in tissue oxygenation in shock
(particularly in septic shock) and has historically been overlooked.
iii. Of importance, microcirculatory blood flow and Do2 cannot be predicted by global
(macrocirculatory) hemodynamics.
iv. Microcirculatory blood flow can be visualized with orthogonal polarization spectral imaging
or sidestream darkfield imaging. These devices use green light to illuminate tissue, which
is absorbed by the hemoglobin of red blood cells. This allows the microcirculation to be
visualized because of its red blood cell content.
v. The sublingual microcirculation has been studied most often because of its accessibility.
vi. Studies have shown that the microcirculation is often altered in patients with sepsis, persistent
microvascular alterations are associated with multisystem organ failure and death, alterations
are more severe in non-survivors than in survivors, and improvements in microcirculatory
blood flow correspond with improved patient outcomes.
vii. Decreased vascular density, decreased capillary perfusion, and a decreased percentage of
perfused small vessels are the most common microcirculatory alterations. The proportion of
perfused small vessels seems to be the strongest microcirculatory blood flow predictor of
patient outcomes. In one study of patients with sepsis and septic shock, this was a stronger
predictor of mortality than global hemodynamic markers.
viii. Heterogeneity has been observed in microcirculatory blood flow in the same tissue bed (with
as little as a few millimeters between observations) and between different tissue beds.
ix. Evaluation of the microcirculation is not commonly used in clinical practice because it requires
extensive user experience to obtain proper measurements and time to analyze the results.
However, this is an attractive marker of tissue perfusion that, with technical advances, may be
used more often in the future.
b. Elevated lactate concentrations may also indicate regional tissue hypoperfusion (e.g., mesenteric
ischemia or critical limb ischemia).
c. Other measures of regional tissue perfusion include gastric tonometry to indirectly assess gastric
mucosal perfusion and near-infrared spectroscopy, a noninvasive method of measuring tissue
oxygen saturation (Sto2) in a skeletal muscle.

ACCP Updates in Therapeutics® 2022: Critical Care Pharmacy Preparatory Review and Recertification Course

627
Shock Syndromes I: Introduction, Vasodilatory, and Sepsis

Patient Case

1. A 77-year-old man presents to the ED with light-headedness and fatigue. He reports increasing melena
during the past 24 hours. His medical history is significant for hypertension, asthma, and gastroesophageal
reflux disease. Vital signs in the ED are as follows: blood pressure 88/54 mm Hg, heart rate 124 beats/min-
ute, respiratory rate 18 breaths/minute, and temperature 102.2°F (39°C). While interviewing the patient,
you note that he appears lethargic and confused. His serum chemistry panel shows the following: Na 138
mEq/L, K 3.8 mEq/L, Cl 105 mEq/L, carbon dioxide 22 mEq/L, blood urea nitrogen (BUN) 25 mg/dL,
SCr 1.1 mg/dL, and glucose 78 mg/dL. Results of the complete blood cell count (CBC) are as follows: WBC
10.2 × 103 cells/mm3, Hgb 6.6 g/dL, Hct 19.2%, and platelet count (Plt) 180,000/mm3. Which is most likely
contributing to this patient’s compromised Do2 to the end organs?
A. Medical history of hypertension.
B. Reduced Hgb.
C. Tachycardia.
D. Leukocytosis.

III.  DIFFERENTIATION OF SHOCK STATES

A. Determining which shock state is present is typically based on assessments of preload (CVP or pulmonary
capillary wedge pressure [PCWP]), CO (Scvo2 or Svo2 may serve as a surrogate), and afterload (SVR) (see
Table 3).

B. Values to describe this hemodynamic profile have historically been obtained from a pulmonary artery
catheter (PAC), but bedside echocardiography is now recommended as the preferred modality to initially
evaluate the type of shock.

Table 3. Hemodynamic Profiles of Shock States


Shock State CVP PCWP CO SVR
Hypovolemic ↓a ↓a ↓ ↑
Cardiogenic ↑ ↑ ↓a ↑
Obstructive
Impaired diastolic filling (e.g., cardiac tamponade) ↑ ↑ ↓a ↑
Impaired systolic contraction (e.g., massive PE) ↑ ↓ or ↔ ↓a

Vasodilatory/distributive
Pre-resuscitation ↓ ↓ ↓ ↓a
Post-resuscitation ↑ ↑ ↑ ↓a
a
Pathophysiologic hallmark of shock state.
PE = pulmonary embolism; SVR = systemic vascular resistance.
Information from: Vincent JL, De Backer D. Circulatory shock. N Engl J Med 2013;369:1726-34; Dellinger RP. Cardiovascular management of septic shock. Crit Care
Med 2003;31:946-55; Weil MH, Shubin H. Proposed reclassification of shock states with special reference to distributive defects. Adv Exp Med Biol 1971;23:13-23.

C. The hemodynamic profiles in Table 3 occur when the stated shock state occurs independently, which often
does not occur in because patients often have features of combined shock states.

ACCP Updates in Therapeutics® 2022: Critical Care Pharmacy Preparatory Review and Recertification Course

628
Shock Syndromes I: Introduction, Vasodilatory, and Sepsis

D. A patient’s history of present illness can help differentiate between shock states. For example, a patient
presenting after motor vehicle collision likely has hypovolemic shock, whereas a patient presenting after bee
sting likely has vasodilatory/distributive shock (immune-mediated [anaphylactic] subtype).

IV.  RESUSCITATION PARAMETERS AND END POINTS

A. The approach to treating a patient with circulatory shock can be divided into four phases, each having
different (but sometimes overlapping) treatment goals and therapeutic strategies.
1. The first phase focuses on salvage, in which efforts should be directed to achieving the minimum
perfusion pressure and CO needed to maintain the patient’s survival. Treating the underlying cause
of the patient’s shock, which consists of lifesaving measures, should be done at this time. Examples of
these measures include antimicrobials for sepsis, revascularization for acute myocardial infarction, and
surgical hemostasis for trauma.
2. Optimization is the second phase with the goal to ensure adequate Do2.
3. In the third phase, patient stabilization is targeted with the goal of preventing or minimizing end-organ
dysfunction.
4. The fourth phase is de-escalation, in which the goals of therapy include vasoactive medications weaning
(or cessation), fluid elimination (e.g., diuresis or ultrafiltration), and antimicrobial de-escalation as
dictated by microbiology cultures, local antibiogram, and patient clinical picture.
5. Although the rest of this chapter will focus on the first two phases, understanding the phase of a patient’s
circulatory shock is essential for establishing treatment goals and subsequent therapeutic approaches.

B. Blood Pressure
1. As noted earlier, blood pressure is the driving pressure for peripheral blood flow. As such, an adequate
blood pressure is vital to ensure end-organ perfusion.
2. MAP is the true driving pressure for peripheral blood flow and end-organ perfusion and is preferred to
SBP or diastolic blood pressure as a therapeutic target.
3. The perfusion pressure of any organ can be calculated by subtracting the pressure within the organ or
anatomic space from the MAP (e.g., cerebral perfusion pressure = MAP − intracranial pressure).
4. The target blood pressure for a patient in shock is usually a MAP greater than 65 mm Hg or an SBP
greater than 90 mm Hg, but this must be individualized according to other clinical/biochemical markers
of perfusion.
5. MAP is an insensitive resuscitation parameter (e.g., blood pressure may be at goal when CO is inadequate),
because of this, additional resuscitation parameters should be used to ensure the optimization of all
hemodynamic components that may influence end-organ perfusion and Do2.
6. These additional resuscitation goals typically include ensuring (1) adequate end-organ perfusion, (2)
lack of fluid responsiveness, and (3) adequate Do2.

C. Adequate End-Organ Perfusion


1. Each organ has a critical perfusion pressure that must be exceeded to maintain adequate perfusion. This
critical perfusion pressure is organ- and patient-specific because of adaptation for chronic conditions.
2. As the MAP decreases, the perfusion pressure of the organ decreases, and subsequently, organ function
decreases.
3. Adequate organ perfusion is best assessed clinically on a per-patient basis.

ACCP Updates in Therapeutics® 2022: Critical Care Pharmacy Preparatory Review and Recertification Course

629
Shock Syndromes I: Introduction, Vasodilatory, and Sepsis

4. General goals of therapy include resolution of altered mental status and adequate urinary output (above
0.5 mL/kg of body weight per hour). However, these goals may be challenging to assess in patients who
are given medications that mask the ability to assess the organ function (e.g., sedatives) or in patients
with chronic organ dysfunction (e.g., end-stage renal disease).

D. Lack of Fluid Responsiveness


1. Intravenous fluids are given to increase preload and subsequently increase SV, increase CO, and
increase Do2.
2. Fluids should be given only if there is inadequate effective organ perfusion caused by inadequate CO
(presumably because of inadequate SV) and if the patient is fluid responsive.
a. Fluid responsiveness is defined as at least a 10%–15% increase in CO after fluid administration.
b. This is best assessed by giving a fluid challenge and evaluating the response.
c. A change in blood pressure is not a reliable indicator of CO response to a fluid challenge; an
assessment of CO (or SV) change should be used instead.
d. In one systematic review, only 57% of hemodynamically unstable patients were fluid responsive.
3. Patients given additional fluid when they are no longer fluid responsive will not experience the
beneficial effects of fluid (increased CO)—only the harmful effects (e.g., pulmonary edema). Although
an argument could be made to determine fluid responsiveness in each patient, it is most critical in
patients for whom harmful fluid effects cannot be tolerated (e.g., those with refractory hypoxemia in
the setting of shock).
4. Fluid responsiveness may be predicted by static or dynamic markers.
a. Static markers of fluid responsiveness include the cardiac filling pressures CVP and PCWP.
b. Dynamic markers of fluid responsiveness include stroke volume variation (SVV), systolic pressure
variation, pulse pressure variation (PPV), and IVC variation.
5. Although CVP and PCWP may help differentiate shock states and may help to provide an understanding
of static intravascular volume status, they are not reliable predictors of fluid responsiveness.
a. In one study of patients with sepsis and septic shock, a CVP less than 8 mm Hg and a PCWP less
than 12 mm Hg had fluid responsiveness positive predictive values of 47% and 54%, respectively.
b. Systematic reviews and meta-analyses suggest that CVP should not be used as a resuscitation
parameter.
6. Fluid responsiveness is better predicted by dynamic markers of fluid responsiveness than by static
markers of volume status or fluid responsiveness.
a. The area under the receiver operating characteristic curve (AUC ROC), with the optimal area
of 1, for predicting fluid responsiveness is as follows: PPV 0.94 (95% confidence interval [CI],
0.93–0.95), systolic pressure variation 0.86 (0.82–0.90), and SVV 0.84 (0.78–0.88). In contrast, the
AUC ROC for predicting fluid responsiveness for CVP is 0.55 (0.48–0.62).
b. Because they can be obtained from monitoring devices (Table 2), PPV and SVV are more commonly
used in practice than is systolic pressure variation.
7. The dynamic markers, PPV, SVV, and IVC collapsibility, are based on heart-lung interactions in
mechanically ventilated patients.
a. In mechanically ventilated patients, a controlled positive pressure breath increases pleural pressure.
This increase in intrathoracic pressure leads to a decrease in venous return, decreased RV preload,
and decreased RV SV. LV preload is subsequently decreased, which may lead to a decrease in LV SV.
b. Patients who are preload (fluid) responsive (on the steep rather than the flat portion of the Frank-
Starling curve) will have relatively large changes in LV SV with positive pressure breaths. This
leads to variation in the LV SV between periods with and without positive pressure breaths.
c. Some ICU monitors can display automated PPV from an arterial catheter without additional
monitoring equipment or devices.

ACCP Updates in Therapeutics® 2022: Critical Care Pharmacy Preparatory Review and Recertification Course

630
Shock Syndromes I: Introduction, Vasodilatory, and Sepsis

d. The specific values of PPV and SVV used to predict fluid responsiveness vary by study, specific
conditions (e.g., use of vasopressors), and assessment method or device. In a systematic review,
thresholds to predict fluid responsiveness were PPV greater than 12.5% and SVV greater than
11.6%.
e. IVC variation (also termed IVC collapsibility or IVC distensibility) uses echocardiography to
visualize the diameter of the IVC during positive pressure ventilation. With a positive pressure
breath from mechanical ventilation, venous return is impaired, and the diameter of the IVC
increases (IVC distensibility). The opposite change in IVC diameter occurs in patients breathing
spontaneously (IVC collapsibility). The change in IVC diameter during inspiration is higher in
patients who are fluid responsive than in those who are not fluid responsive.
i. In one study, the AUC ROC for predicting fluid responsiveness for IVC distensibility was 0.84
(95% CI, 0.63–1.0), and the best cutoff was 16% (sensitivity 67%, specificity 100%).
ii. In a larger, multicenter study of mechanically ventilated patients with varied causes of
circulatory shock, IVC distensibility had an AUC ROC for predicting fluid responsiveness
of 0.64 for a threshold of 8% (sensitivity 55%, specificity 70%). In the patient subset with
all assessed dynamic markers of fluid responsiveness available, the AUC ROC for predicting
fluid responsiveness was significantly higher for respiratory variations in superior vena cava
diameter (0.74, threshold of 21% or greater) than PPV (0.66, threshold of 11% or greater,
p=0.01) and IVC distensibility (0.65, threshold of 13% or greater, p=0.02). Of note, superior
vena cava diameter changes must be assessed by transesophageal echocardiography, which is
not commonly performed for this purpose in the United States. IVC variation may be assessed
by transesophageal or transthoracic echocardiography.
8. The passive leg raise (PLR) test measures the hemodynamic effects of a positional change in the patient’s
legs. Changing the position of the patient’s bed such that the patient’s legs are lifted to a 45-degree angle
with the patient’s head placed horizontally leads to a transfer of blood from the abdominal compartment
and lower extremities to the intrathoracic compartment.
a. This increase in venous return may subsequently increase SV and CO if the patient is preload
responsive.
b. An increase in CO by 10%–15% after PLR is considered a positive test.
c. The benefit of the PLR test is that it can be used in spontaneously breathing or nonintubated patients.
In addition, it does not require the administration of fluid (which may be harmful if the patient is
not fluid responsive) and can easily be reversed by returning patients to their previous position.
d. A caveat to use of the PLR test is that a method of evaluating CO is required to determine response
(or lack thereof). A change in blood pressure is not an adequate surrogate marker for CO, as noted
earlier. The CO measurement may be obtained from an arterial pulse pressure waveform analysis
monitor (minimally invasive approach) or from a bioimpedance device, bioreactance device, or
echocardiographic LVOT VTI (noninvasive approach).
9. Several caveats exist for using dynamic markers of fluid responsiveness.
a. PPV and SVV assume the following: sinus cardiac rhythm, the absence of significant valvular
dysfunction, intubation and mechanical ventilation without spontaneous breaths, and tidal volume
of 8 mL/kg or more of predicted body weight. Values for PPV and SVV above the noted thresholds
do not reliably predict fluid responsiveness in the setting of arrhythmias (e.g., atrial fibrillation). If
these assumptions are not fulfilled, PPV and SVV are not reliable in predicting fluid responsiveness.
b. IVC distensibility also requires intubation and mechanical ventilation without spontaneous breaths
and is not conducive to continuous monitoring.
c. The real-time response of CO (or lack thereof) with PLR must be assessed using a CO monitoring
device. In addition, intra-abdominal hypertension reduces the ability of PLR to detect fluid
responsiveness.

ACCP Updates in Therapeutics® 2022: Critical Care Pharmacy Preparatory Review and Recertification Course

631
Shock Syndromes I: Introduction, Vasodilatory, and Sepsis

10. The best dynamic marker of fluid responsiveness to use in practice is unclear.
a. In the previously mentioned study, the AUC ROC for predicting fluid responsiveness was higher
for PPV than for SVV.
b. In one study of postoperative patients, IVC distensibility was not noninferior to PPV (noninferiority
p=0.28), considering a noninferiority margin of 15%.
c. However, obtaining PPV from echocardiography leads to a lower AUC ROC for predicting fluid
responsiveness (0.66–0.68) than does obtaining PPV from other methods (AUC ROC 0.94).
d. In patients in whom transesophageal echocardiography is performed, superior vena cava diameter
changes appear to be superior to IVC distensibility and PPV.
11. Despite the superiority of dynamic markers to static markers in predicting fluid responsiveness,
incorporating dynamic markers into a resuscitation strategy that improves patient outcomes in the ICU
is still lacking.
a. In a randomized controlled trial of patients with septic shock and/or acute respiratory distress
syndrome that randomized patients to treatment on the basis of pulse index continuous cardiac
output (PiCCO)-derived parameters (one of the arterial pulse pressure waveform analysis devices
described in Table 2) or control, 28-day mortality did not differ between groups.
b. A pilot study using protocol-guided assessments of fluid responsiveness after initial resuscitation
in patients with septic shock requiring vasopressors found this approach feasible and safe, paving
the way for larger trials using this approach.

Patient Case

Questions 2 and 3 pertain to the following case.


A 59-year-old man with a medical history of cirrhosis complicated by ascites was transferred from the ward to
the medical ICU for gross hematemesis, with an Hgb decrease from 9.2 g/dL to 7.3 g/dL, blood pressure 82/36
mm Hg, and new-onset confusion. After 2 L of lactated Ringer’s solution and 2 units of PRBCs, the patient’s
Hgb increased to 9.1 g/dL, but he remained hypotensive. The medical team placed a PAC and an arterial blood
pressure catheter, which showed the following: CVP 8 mm Hg, PCWP 14 mm Hg, CO 7.4 L/minute, and MAP
58 mm Hg.

2. With which shock state are the patient’s post-resuscitation hemodynamic parameters most consistent?
A. Hypovolemic.
B. Obstructive.
C. Vasodilatory.
D. Cardiogenic.

3. After further resuscitation, the patient developed hypoxemia requiring intubation and mechanical ven-
tilation. A post-intubation radiograph revealed diffuse bilateral alveolar opacities. The patient remained
hypoxemic with an Fio2 of 90% and was subsequently deeply sedated and given atracurium. The patient also
remained hypotensive with low urinary output. Which best predicts that the patient will respond favorably
to a fluid bolus?
A. CVP 7 mm Hg.
B. PCWP 11 mm Hg.
C. SVV 16%.
D. MAP 62 mm Hg.

ACCP Updates in Therapeutics® 2022: Critical Care Pharmacy Preparatory Review and Recertification Course

632
Shock Syndromes I: Introduction, Vasodilatory, and Sepsis

E. Adequate Do2
1. CO, Scvo2, and Svo2
a. Historically, CO was monitored and used as a therapeutic target in most patients with a PAC.
i. Because treatment of general ICU patients with a PAC has not been shown to improve patient
outcomes, routine use of a PAC has decreased substantially.
ii. Use of venous oximetry (Scvo2 and Svo2) and echocardiographic findings has largely replaced
use of a PAC.
b. Scvo2 may be used as a component of an early resuscitation strategy (though it is not a mandatory
component).
i. A decreased Scvo2 or Svo2 is a sign that tissue oxygen demands are not completely met by Do2.
ii. Strategies to increase Do2 (and subsequently increase Scvo2 or Svo2) include fluids to optimize
preload, red blood cell transfusion to increase Cao2, and inotropes to increase CO. Because
Pao2 does not contribute significantly to Cao2, it should not be used as a therapeutic target.
c. If Scvo2 or Svo2 is used as a resuscitation goal, it may be more important to use predefined targets
as well as trends in the early resuscitation period (first 6 hours after presentation) than in the later
resuscitation periods.
d. Caution must be used with using Scvo2 or Svo2 in isolation as a resuscitation goal for the following
reasons:
i. The assumption that decreased Scvo2 or Svo2 is synonymous with Do2 and oxygen demand
mismatch is not true because, by definition, tissue oxygen demand exceeds Vo2 in shock.
ii. During resuscitation, Vo2 depends on Do2, and increasing Do2 will increase the Vo2 without
substantially changing Scvo2 or Svo2 until the critical Do2 threshold is reached.
iii. Svo2 and CO are not directly proportional and are better described by a hyperbolic relationship.
As such, in hyperdynamic states when the CO is already high, the Svo2 will not increase
substantially with increases in CO.
iv. Venous hyperoxia may indicate mitochondrial dysfunction and impaired tissue oxygen use or
shunting of blood from peripheral circulation; therefore, achieving a high Scvo2 or Svo2 is not
always best.
e. CO, Scvo2, and Svo2 are likely best interpreted as either adequate or inadequate (not high or low).
i. Adequacy is best determined by assessing end-organ perfusion and lactate concentrations.
ii. If CO, Scvo2, or Svo2 is inadequate, Do2 should be raised.
f. A strategy of systematically increasing CO to predefined “supranormal” values was not associated
with a mortality benefit; hence, it is not recommended. The decision to augment CO must be
individualized on the basis of organ perfusion.
2. Lactate clearance and normalization
a. Lactate clearance (a decrease in lactate concentration from the initial value) suggests improvement
in global tissue perfusion and is associated with a decreased mortality rate.
b. Significant discordance between lactate clearance and Scvo2 may occur. In one study, 79% of
patients with a lactate clearance of less than 10% had a concomitant Scvo2 of 70% or greater.
c. A protocol-based approach to resuscitating patients with sepsis or septic shock targeting a lactate
clearance of at least 10% was noninferior to an approach targeting an Scvo2 above 70%.
d. Lactate normalization (to a concentration below 2 mmol/L) is a strong independent predictor of
survival and may be an even better predictor of outcomes than lactate clearance.
e. Targeting lactate clearance or normalization is an attractive end point because it does not require
invasive hemodynamic monitoring.
f. Use of lactate clearance together with Scvo2 as a resuscitation goal may be best because this
improves outcomes compared with use of Scvo2 alone.
g. Further discussion of lactate clearance as a resuscitation target in patients with sepsis or septic
shock is included later in the chapter.

ACCP Updates in Therapeutics® 2022: Critical Care Pharmacy Preparatory Review and Recertification Course

633
Shock Syndromes I: Introduction, Vasodilatory, and Sepsis

V.  AGENTS USED TO TREAT SHOCK – FLUIDS AND VASOACTIVE AGENTS

A. See the Fluids, Electrolytes, and Nutrition chapter for a further discussion of fluid components.

B. The below description of fluids and vasoactive agents is specific to shock states in general. Description of
agents used to treat septic shock will be discussed in section VII, Sepsis.

C. Pharmacology of Vasoactive Agents


1. Vasoactive agents can be broadly differentiated to (1) vasopressors, (2) inotropes, or (3) vasodilators.
Vasoactive agents may have several of these properties.
2. Vasopressor agents are indicated if hypotension is refractory to fluid administration (the patient is no
longer fluid responsive) or in the setting of severe hypotension while fluids are being administered. In
addition, if a patient is severely hypotensive, vasopressors may be initiated together with additional
fluid administration, even if a patient is still fluid responsive, to ensure adequate end-organ perfusion.
Because there are no definitive criteria for when vasopressors should be initiated in relationship to fluid
administration, bedside clinicians must often make a patient-specific assessment and decision.
3. Vasopressor agents primarily exert pharmacologic benefit by augmenting SVR. Some vasopressors
may also increase CO. Table 4 highlights the receptor pharmacology of the various agents.
a. Once the decision to start a vasoactive agent is made, a vasoactive agent or inotrope is largely
selected on the basis of the agent that best achieves the desired pharmacodynamic effect(s) (e.g.,
increase in SVR or increase in CO). In most shock syndromes, limited literature exists to guide
optimal vasoactive agent selection.
b. Vasopressor agents should be administered through central venous access to minimize the risk of
extravasation and tissue necrosis.
c. Catecholamine vasopressors primarily increase blood pressure by increasing SVR through their
actions on α1-receptors. Differing α1, β1, and β2 activity leads to differing pharmacodynamic effects
between agents, as outlined in Table 4.
d. Vasopressin increases SVR through V1R activity in smooth muscles resulting in vasoconstriction.
Activity at V2R receptors lead to antidiuretic hormone effects in the kidney. Selective sparing
of V1R in some vascular beds or V2R-mediated vasodilation leads to decreased vasoconstrictive
effects of vasopressin in the coronary, cerebral, and pulmonary circulation.
e. Angiotensin II also primarily increases SVR by activating AT-R1 receptors, resulting in
vascular smooth muscle contraction. Activation of AT-R1 receptors also stimulates the release of
norepinephrine, vasopressin, adrenocorticotropic hormone (ACTH), and aldosterone. Activation of
AT-R2 receptors can counterbalance the vasoconstrictive effects of AT-R1 in some vascular beds.
f. Some clinicians use an initial intravenous push/bolus of vasopressors (“push dose pressors”) in
patients with hypotension that is either severe or expected to be short in duration (e.g., medication-
associated hypotension in the setting of endotracheal intubation). Common doses include
phenylephrine 100–200 mcg and norepinephrine 1–2 mcg, each given every 2–3 minutes. The
efficacy and safety is unclear because few studies have evaluated this approach.
4. Inotropes exert a pharmacodynamic effect that increases CO after adequate fluid administration.
a. Dobutamine is a β-agonist that improves cardiac function by improving SV and CO. Because of
its β1 activity, dobutamine may induce tachyarrhythmias and, given its β2 activity, hypotension.
b. Isoproterenol is also a β-agonist, but because it has stronger β2 activity than dobutamine, it causes
more hypotension. For this reason, isoproterenol is not commonly used in the ICU as an inotrope.

ACCP Updates in Therapeutics® 2022: Critical Care Pharmacy Preparatory Review and Recertification Course

634
Shock Syndromes I: Introduction, Vasodilatory, and Sepsis

Table 4. Vasoactive Pharmacology and Pharmacodynamic Effectsa


Other
DA α1 β1 β2 HR CVP CO SVR PVR
Mechanism
Vasopressors
Dopamine b
— or — or
++++ 0 0 0 — or ↑ ↔ ↔
< 5 mcg/kg/min ↑ ↑
Dopamineb — or — or
0 + ++++ + ↑ ↑ — or ↑
5–15 mcg/kg/min ↑ ↑
Dopamineb — or
0 ++++ ++ ++ — or ↑ ↑ ↑ ↑
< 5 mcg/kg/min ↑
— or
Epinephrine 0 ++++ ++++ + ↑ ↑ ↑ ↑

↓ or — — or — or
Norepinephrine 0 ++++ +++ + ↑ ↑
or ↑ ↑ ↑
— or — or
Phenylephrine 0 +++ + 0 — or ↓ ↑ ↑
↑ ↓
V1R and V2R — or — or
Vasopressin N/A — or ↓ ↔ ↑
agonism ↑ ↓
AT-R1 and — or
Angiotensin II N/A ↑ ↔ ↑ ?
AT-R2 agonism ↑
Inotropes
— or — or — or
Dobutamine 0 + ++++ ++ ↑ ↑
↓ ↓ ↓
— or — or — or
Isoproterenol 0 0 ++++ ++++ ↑ ↑
↓ ↓ ↓
Ca2+
— or
Levosimendan N/A sensitization and ↔ ↑ ↓ ↓

KATP activation
PDE3 — or
Milrinonec N/A — or ↑ ↑ ↓ ↓
inhibition ↓
a
Receptor activity ranges from no activity (0) to maximal (++++) activity.
b
Dopamine has dose-dependent effects. At lower doses (< 5 mcg/kg/min), activity at dopamine receptors predominates. At moderate doses (5–10 mcg/kg/min), β1 effects
predominate, and at doses > 10 mcg/kg/min, α1 effects predominate. However, there is overlap in these effects, and these dose ranges are not exact.
c
Normal half-life is 2.5 hr, but milrinone is eliminated renally. Loading dose rarely used in routine management.
AT-R1 = angiotensin receptor type 1; AT-R2 = angiotensin receptor type 2; DA = dopaminergic; KATP = adenosine triphosphate-sensitive potassium channels; N/A =
not applicable; PDE3 = phosphodiesterase type 3; V1R = vasopressin receptor type 1; V2R = vasopressin receptor type 2; ? = unclear.
Information from: Bangash MN, Kong ML, Pearse RM. Use of inotropes and vasopressor agents in critically ill patients. Br J Pharmacol 2012;165:2015-33; Hollenberg
SM. Vasoactive drugs in circulatory shock. Am J Respir Crit Care Med 2011;183:847-55; Antonucci E, Gleeson PJ, Annoni F, et al. Angiotensin II in refractory septic
shock. Shock 2017;47:560-6; Chawla LS, Busse L, Brasha-Mitchell E, et al. Intravenous angiotensin II for the treatment of high-output shock (ATHOS trial): a pilot
study. Crit Care 2014;18:534; and Khanna A, English SW, Wang XS, et al. Angiotensin II for the treatment of vasodilatory shock. N Engl J Med 2017;377:419-30.

ACCP Updates in Therapeutics® 2022: Critical Care Pharmacy Preparatory Review and Recertification Course

635
Shock Syndromes I: Introduction, Vasodilatory, and Sepsis

c. Milrinone is a phosphodiesterase type 3 (PDE3) inhibitor. PDE3 inhibition potentiates cyclic


adenosine monophosphate, leading to increased ventricular contractility and vasodilation.
i. Milrinone may be desirable for patients receiving β-antagonists before critical illness or for
patients having tachyarrhythmias while receiving dobutamine.
ii. Milrinone may also be favored in patients with pulmonary hypertension (PH) because it
decreases cardiac filling pressures and pulmonary vascular resistance (PVR).
iii. Milrinone may increase or decrease blood pressure, depending on the individual patient’s
SVR and CO when it is administered. In general, a decrease in blood pressure is more common
with milrinone than with dobutamine.
iv. The half-life of milrinone is significantly longer (2.3–2.4 hours) than that of dobutamine (2
minutes) and isoproterenol (2.5–5 minutes) and thus limits its utilization in emergent scenarios.
d. Levosimendan is a calcium channel sensitizer that also activates adenosine triphosphate (ATP)-
sensitive potassium channels.
i. Levosimendan’s pharmacodynamic effects are similar to those of milrinone.
ii. Like milrinone, levosimendan may be preferred in patients receiving β-antagonists before
critical illness or in those with PH.
iii. Levosimendan is not available for use in the United States.

D. Outcomes Studies
1. Studies of therapies in patients with sepsis and septic shock will be discussed in section VII, Sepsis.
2. Fluid and resuscitation studies in ICU patients
a. Resuscitation fluids are commonly given for patients in the ICU, with 25%–37% of patients
receiving this therapy in a 24-hour period in cross-sectional studies.
b. Crystalloids vs. colloids
i. The Saline versus Albumin Fluid Evaluation (SAFE) study, which enrolled almost 7000 patients
with varied shock types requiring fluid resuscitation, with 90% power, found no difference in
28-day mortality between treatment with 0.9% sodium chloride and 4% albumin (20.9% vs.
21.1%, p=0.87). However, this was not a study of strictly initial fluid resuscitation because the
allocated study fluid was used for all fluid resuscitation in the ICU until death, discharge, or 28
days after randomization. Because of this study, crystalloids are usually preferred to albumin
for the initial resuscitation of patients with shock because of their lower cost.
ii. A pragmatic, open-label, randomized study of crystalloids compared with colloids for
resuscitation found no difference between groups in 28-day mortality (27.0% vs. 25.4%,
p=0.26) but did find a difference in 90-day mortality favoring the colloid group (34.2% vs.
30.2%, p=0.03). However, because 90-day mortality was a secondary (not primary) outcome,
the results must be interpreted with caution. In addition, the study’s open-label nature (which
may bias toward finding a difference between groups) and use of many different resuscitation
fluids within each study group make this study challenging to implement into practice.
c. Balanced (chloride-poor) vs. unbalanced (chloride-rich) crystalloids
i. Administration of chloride-rich fluids may lead to afferent renal arteriole vasoconstriction,
leading to a decrease in renal perfusion and kidney injury, and may cause a metabolic acidosis
by lowering the strong ion difference. As such, crystalloids that better approximate the
electrolyte composition of plasma (“chloride-poor,” “balanced salt,” or “balanced crystalloid”
solutions) have been evaluated.

ACCP Updates in Therapeutics® 2022: Critical Care Pharmacy Preparatory Review and Recertification Course

636
Shock Syndromes I: Introduction, Vasodilatory, and Sepsis

Table 5. Sodium and Chloride Content of Commonly Used Resuscitation Fluids


Fluid Sodium (mmol/L) Chloride (mmol/L)
“Chloride-rich”a
0.9% sodium chloride 154 154
5% albumin 130–160b 0–128b
Hydroxyethyl starch 6% (130/0.4) 154 154
“Chloride-poor”a
25% albumin 130–160b 0–19b
Lactated Ringer’s solution 130 109
Plasma-Lyte A and Plasma-Lyte 148 140 98
Normosol-R 140 98
a
Distinction between “chloride rich” and “chloride poor” is based on chloride content > or < 120 mmol/L.
b
Differs according to manufacturer because of differences in buffer type (e.g., sodium bicarbonate or sodium chloride) and amount used. Reported chloride content of
4% Albumex (CSL Bioplasma) is 128 mmol/L, and that of 20% Albumex (CSL Bioplasma) is 19 mmol/L (products used in Australia/New Zealand), which led to the
distinction of “chloride rich” and “chloride poor” for 4%–5% albumin and 20%–25% albumin, respectively. However, neither 5% Flexbumin (Baxter, Westlake Village,
CA) nor 25% Flexbumin (Baxter; products available in the United States) contains chloride.
Information from: Guidet B, Soni N, Della Roca G, et al. A balanced view of balanced solutions. Crit Care 2010;14:325; Frazee EN, Leedahl DD, Kashani KB. Key
controversies in colloid and crystalloid fluid utilization. Hosp Pharm 2015;50:446-53; and Yunos NM, Bellomo R, Hegarty C, et al. Association between a chloride-
liberal vs chloride-restrictive intravenous fluid administration strategy and kidney injury in critically ill adults. JAMA 2012;308:1566-72.

ii. An open-label sequential period study evaluated outcomes between a control period in which
chloride-rich fluids were routinely administered (n=760) and an intervention period in which
chloride-rich fluids were restricted to attending physician approval and chloride-poor fluids
were routinely used (n=773). In the intervention period, the incidence of acute kidney injury
(8.4% vs. 14%, p<0.001) and use of renal replacement therapy were significantly lower (6.3%
vs. 10%, p=0.005). Because the study was not blinded or randomized, the results should be
considered hypothesis generating.
iii. A large, double-blind, cluster randomized, double-crossover trial, the SPLIT trial, allocated
four ICUs in New Zealand to either 0.9% sodium chloride (n=1025) or a balanced crystalloid
solution (Plasma-Lyte; n=1067) for all patients requiring crystalloid fluid therapy. Two
crossovers occurred with 7-week intervals such that each ICU used each fluid twice during
the 28-week study. A total of 2278 patients were enrolled, with 2092 patients having data
available for analysis of the primary outcome of the incidence of acute kidney injury within
90 days. The balanced crystalloid solution group had no lower risk of acute kidney injury than
did the 0.9% sodium chloride group (9.6% vs. 9.2%, relative risk [RR] 1.04; 95% CI, 0.80–
1.36; p=0.77). There was also no difference between groups in the use of renal replacement
therapy (RR 0.96; 95% CI, 0.62–1.50; p=0.91) or hospital mortality (RR 0.88; 95% CI, 0.67–
1.17; p=0.40). Critiques of this study include the use of the study fluid for all crystalloid fluid
therapy needs (not just as a resuscitation fluid for circulatory shock), receipt of over 1 L of
balanced crystalloid in the majority of patients at baseline, relatively low volumes of fluid
administered during the study period (2 L in each group), significant heterogeneity (p=0.05)
of the treatment effect on acute kidney injury by specific ICU study site (suggesting center-
related treatment differences [though the authors attributed this heterogeneity to chance]),
and debatable widespread external validity in light of the predominantly surgical population
(72% of patients were admitted to the ICU postoperatively, with 49% overall admitted after
cardiovascular surgery) and the relatively low severity of patient illness (mean APACHE II
[Acute Physiology and Chronic Health Evaluation II] scores of 14.1).

ACCP Updates in Therapeutics® 2022: Critical Care Pharmacy Preparatory Review and Recertification Course

637
Shock Syndromes I: Introduction, Vasodilatory, and Sepsis

iv. The Isotonic Solutions and Major Adverse Renal Events Trial (SMART) was a large, cluster-
randomized, multiple-crossover, single-center trial, in which balanced crystalloids (lactated
Ringer’s solution or Plasma-Lyte A; n=7942) were compared with 0.9% sodium chloride
(n=7860). The primary outcome was a major adverse kidney event within 30 days, which was
a composite outcome comprising death from any cause, new renal replacement therapy, or
persistent renal dysfunction (defined as a creatinine elevation to 200% or more of baseline). The
median volume of fluid received in the balanced crystalloid group was 1000 mL (interquartile
range [IQR] 0–3210) and 1020 mL in the 0.9% sodium chloride group (IQR 0–3500). Patients in
the balanced crystalloids group had a significantly lower incidence of a major adverse kidney
event (14.3% vs. 15.4%; OR 0.90; 95% CI, 0.82–0.99; p=0.04). In-hospital mortality was not
significantly different between groups (10.3% vs. 11.1%, p=0.06). Of note, in a prespecified
subgroup analysis, there was a greater difference in the primary outcome favoring patients
who received smaller volumes of intravenous fluids; thus, the differences between fluid types
are likely magnified and more significant in patients who receive larger volumes of intravenous
fluids. In addition to being a single-center study, this study is limited by its unblinded design,
which may have influenced the clinical decision to initiate renal replacement therapy (a
component of the primary outcome) and the statistical fragility of the results, indicating that
the statistical significance depends on only a few events.
v. The BaSICS study was a double-blind, randomized controlled trial that compared Plasma-Lyte
148 with 0.9% sodium chloride in ICU patients requiring fluid expansion. The intervention
included all fluids over 100 mL administered for fluid challenges, maintenance fluids, and
drug infusions. Over 10,000 patients were included in this study. There was no difference in
the primary outcome of 90-day survival (26.4% balanced solution vs. 27.2% saline; HR 0.97;
95% CI, 0.90–1.05). Of note, this study did not evaluate fluids specifically for resuscitation;
rather, it evaluated any fluid volume over 100 mL.
vi. Balanced crystalloid solutions may lead to hyponatremia (with lactated Ringer’s solution) or
cardiotoxicity (with acetate-containing solutions) when administered in large volumes. Caution
should be used with these solutions in patients with brain injury because of the risk of cerebral
edema and in those with hyperkalemia because they contain potassium (lactated Ringer
solution contains 4 mmol/L of potassium, Plasma-Lyte A contains 5 mmol/L of potassium).
d. Hydroxyethyl starch solutions should not be used for fluid resuscitation in the ICU.
i. A study of 7000 critically ill patients requiring fluid resuscitation compared a low-molecular-
weight, low-molar-substitution (130/0.4) hydroxyethyl starch solution with 0.9% sodium
chloride. Ninety-day mortality between the hydroxyethyl starch and 0.9% sodium chloride
groups did not differ (18.0% vs. 17.0%, p=0.26), but patients allocated to hydroxyethyl starch
had a greater need for renal replacement therapy (7.0% vs. 5.8%, p=0.04) and a higher incidence
of adverse events (5.3% vs. 2.8%, p<0.001).
ii. A systematic review and meta-analysis that analyzed only unbiased trials found an association
between hydroxyethyl starch use and increased patient mortality (RR 1.09; 95% CI, 1.02–1.17;
p=0.02) and need for renal replacement therapy (RR 1.32; 95% CI, 1.15–1.50; p<0.001).
3. Vasopressors and inotropes in shock
a. A multicenter randomized trial, the SOAP II trial, included patients requiring vasopressors for shock
of any type and excluded those requiring vasopressors for more than 4 hours before enrollment.
Enrolled patients were allocated to either blinded norepinephrine or dopamine. Twenty-eight–day
mortality between patients receiving dopamine and those receiving norepinephrine did not differ
(52.5% vs. 48.5%, p=0.10), but patients receiving dopamine more often developed an arrhythmia
(24.1% vs. 12.4%, p<0.001), required open-label norepinephrine (26% vs. 20%, p<0.001), and had
fewer open-label vasopressor-free days (12.6 days vs. 14.2 days, p=0.007).

ACCP Updates in Therapeutics® 2022: Critical Care Pharmacy Preparatory Review and Recertification Course

638
Shock Syndromes I: Introduction, Vasodilatory, and Sepsis

i. A predefined subgroup analysis evaluated the influence of shock type on the outcome. Patients
with cardiogenic shock allocated to dopamine had a higher mortality rate than those allocated
to norepinephrine (log-rank p=0.03). However, the overall effect of treatment did not differ
among the shock subgroups (interaction p=0.87), suggesting that the reported differences in
mortality according to subgroup are spurious.
ii. These data suggest that although norepinephrine does not improve mortality compared with
dopamine, it is safer and more effective at increasing a patient’s blood pressure. Given these
data, a case could be made for norepinephrine as the first-line vasoactive medication of choice
in all shock types.
b. A multicenter randomized trial comparing norepinephrine with epinephrine for patients with
undifferentiated shock found no difference between agents in the time to achieving a goal MAP
(median 40 hours vs. 35.1 hours, p=0.26) or median number of vasopressor-free days at day 28
(25.4 days vs. 26.0 days, p=0.31). However, patients allocated to epinephrine had higher heart rates
and lactic acid concentrations on the first study day (but not on subsequent days) and were more
often withdrawn from the study by the treating clinician (12.9% vs. 2.8%, p=0.002). These data
suggest that epinephrine has no efficacy benefits over norepinephrine and is associated with an
increased incidence of adverse effects.
c. In a systematic review and meta-analysis of vasopressors for patients with circulatory shock of
all types, all-cause mortality did not differ in any comparison of different vasopressor agents or
combinations.
i. Single vasopressors evaluated included norepinephrine (reference group), dopamine,
epinephrine, terlipressin, vasopressin, and phenylephrine. Vasopressor combinations included
norepinephrine plus dobutamine and norepinephrine plus dopexamine (which were compared
with epinephrine).
ii. Participants treated had more arrhythmias with dopamine than with norepinephrine. The
authors concluded that major changes to clinical practice are not needed but that selection of
vasopressors could be better individualized and could be based on clinical variables reflecting
hypoperfusion. This systematic review is limited by the small number of patients enrolled in
randomized studies of some agents (e.g., phenylephrine) and few studies with combination
therapy.
d. Angiotensin II is a novel vasopressor agent being evaluated for patients with vasodilatory shock.
i. In a phase III study of patients with shock (the Angiotensin II for the Treatment of High-Output
Shock [ATHOS-3] study) (n=321) without evidence of a low CO (about 90% with sepsis), use
of angiotensin II, compared with placebo, more often led to a MAP response (MAP of 75 mm
Hg or greater or MAP increase of 10 mm Hg or greater, without an increase in open-label
vasopressor dose) at hour 3 (69.9% vs. 23.4%, p<0.001). Angiotensin II significantly reduced
the cardiovascular Sequential Organ Failure Assessment (SOFA) subscore at hour 48, but not
the total SOFA score. Patients in the angiotensin II group more often developed a thrombotic
event (12.9% vs. 5.1%, p=0.02), a new infection (30.1% vs. 19.0%, p=0.029), and delirium
(5.5% vs. 0.6%, p=0.036).
ii. A post hoc analysis of patients from the ATHOS-3 study with acute kidney injury requiring
renal replacement therapy found that patients treated with angiotensin II had longer survival
(adjusted hazard ratio [HR] 0.44; 95% CI, 0.24–0.80) and were more likely to discontinue
renal replacement therapy within 7 days (adjusted HR 2.90; 95% CI, 1.29–6.52) than those
who received placebo.
iii. One retrospective study evaluating angiotensin II use found that of the 270 included patients,
67% had a positive hemodynamic response 3 hours after its initiation. Of note, 92% of patients
were also receiving vasopressin at the time of angiotensin II initiation.

ACCP Updates in Therapeutics® 2022: Critical Care Pharmacy Preparatory Review and Recertification Course

639
Shock Syndromes I: Introduction, Vasodilatory, and Sepsis

iv. These evaluations suggest that angiotensin II is an effective vasopressor for patients with
vasodilatory shock, particularly in those with acute kidney injury, but outcomes-based studies
are needed before this agent is implemented into routine clinical practice.
v. Because of the increased risk of thrombosis associated with angiotensin II, concurrent venous
thromboembolism prophylaxis is recommended.
e. Levosimendan
i. In a study of patients with sepsis, adding blinded levosimendan for 24 hours compared with
placebo did not lead to a significant difference in the mean SOFA score (6.68 ± 3.89 vs. 6.06 ±
3.89, respectively, p=0.053). Mortality at 28 days also did not differ between the levosimendan
and placebo groups (34.5% vs. 30.9%, p=0.43). Patients in the levosimendan group more often
developed a supraventricular tachyarrhythmia (3.1% vs. 0.4%, p=0.04).
ii. A study of levosimendan compared with placebo for 48 hours in patients with circulatory
shock after cardiac surgery was terminated early for futility. Thirty-day mortality did not
differ between the levosimendan and placebo groups (12.9% vs. 12.8%, p=0.97), nor did the
groups differ in duration of mechanical ventilation or incidence of cardiac arrhythmias.
iii. These data suggest that levosimendan does not improve outcomes in patients with sepsis or
circulatory shock after cardiac surgery and should not be used in these patients.

Patient Case

Questions 4 and 5 pertain to the following case.


J.B. is a 28-year-old man (weight 92 kg) who presented to the surgical ICU with shock after an appendectomy
was complicated by appendiceal perforation. In the operating room, the patient received 2 L of lactated Ringer’s
solution and 500 mL of 5% albumin and was initiated on norepinephrine. The patient is still receiving norepi-
nephrine 12 mcg/minute (0.13 mcg/kg/minute) with a lactate of 6.8 mmol/L and had a CO increase of 18% with
a PLR.

4. Which would best meet J.B.’s fluid needs?


A. 5% albumin.
B. 6% hydroxyethyl starch.
C. 0.9% sodium chloride.
D. No fluids necessary.

5. After 12 hours in the surgical ICU, J.B. remains hypotensive with a lactate concentration of 5.2 mmol/L. He
currently requires norepinephrine 14 mcg/minute (0.15 mcg/kg/minute); his MAP is 64 mm Hg and Scvo2
is 61%. A bedside echocardiogram done by the ICU team reveals large ventricles with poor contractility.
Which action is best?
A. Start phenylephrine.
B. Start vasopressin.
C. Increase norepinephrine.
D. Start epinephrine.

ACCP Updates in Therapeutics® 2022: Critical Care Pharmacy Preparatory Review and Recertification Course

640
Shock Syndromes I: Introduction, Vasodilatory, and Sepsis

VI.  VASODILATORY AND DISTRIBUTIVE SHOCK

A. Etiology and Epidemiology


1. Vasodilatory shock broadly describes tissue hypoperfusion secondary to a decrease in SVR or
hypoperfusion despite a normal or elevated CO, whereas distributive shock is technically a subset
of vasodilatory shock that describes maldistribution of blood flow at the level of microcirculation
(shunting) or at the organ level. However, this differentiation is likely trivial because distributive shock
usually exists in vasodilatory shock, and the terms are often used interchangeably.
2. Vasodilatory shock is the most common type of shock, with about 66% of the patients requiring
vasoactive medications having this shock type.
3. Septic shock is the most common cause of vasodilatory shock, but this shock type may also occur
in several other conditions, including immune-mediated (“anaphylactic”) and nonimmunologic
(“anaphylactoid”) reactions, neurogenic shock (classically secondary to spinal cord injury), intoxication,
peridural or epidural infusion, adrenal insufficiency (Addisonian crisis), and thyroid insufficiency
(myxedema coma) or as a component of ischemia-reperfusion injury (e.g., after cardiac arrest or
cardiopulmonary bypass). Vasodilatory shock also occurs because of prolonged severe hypotension
from any initial shock type (vasodilation is a final common pathway).
4. The three most common causes of vasodilatory shock are septic shock (which will be covered in detail
in section VII, Sepsis), immune-mediated (anaphylactic) shock, and neurogenic shock.

B. Pathophysiology
1. Vasodilatory shock occurs because of a failure of the vascular smooth muscle cells to constrict, whether
from a failure of vasoconstriction methods or the inappropriate activation of vasodilatory mechanisms.
In most cases (except for neurogenic shock), this failure occurs despite high plasma concentrations of
endogenous vasoconstrictors (e.g., norepinephrine, epinephrine, and angiotensin II).
2. Potential mechanisms of vasodilation
a. Activation of cellular ATP-dependent potassium channels leads to hyperpolarization of the
vascular smooth muscle cell through potassium efflux, which prevents extracellular calcium influx
by voltage-gated calcium channels. As a result, cellular depolarization is prevented, high cytosolic
calcium concentrations needed for vasoconstriction are not achieved, and vasodilation occurs.
b. Increased expression of inducible nitric oxide synthase leads to increased intracellular nitric
oxide concentrations and resultant vasodilation by a cyclic guanosine monophosphate–mediated
mechanism. Nitric oxide may also induce vasodilation by activating potassium channels in the
plasma membrane, leading to cellular hyperpolarization, as described earlier.
c. Inappropriately low plasma vasopressin concentrations despite the level of shock (“relative
vasopressin deficiency”) may contribute to the inability of the vascular smooth muscle cell to
contract. Although initial plasma vasopressin concentrations may be high in the initial setting of
shock, vasopressin concentrations may decrease to physiologic concentrations as quickly as 1 hour
after the onset of hypotension.
3. The pathogenesis of vasodilation depends on the underlying cause.
a. Septic shock involves complex interactions between an infecting pathogen and the host
inflammatory, immune, and coagulation response. The pattern-recognition (e.g., toll-like) receptors
on innate immune system cells recognize specific molecules present in microorganisms and signal
the release of nuclear factor B, which leads to the transcription of both proinflammatory cytokines
(e.g., interleukin-1, interleukin-6, tumor necrosis factor alpha) and anti-inflammatory cytokines
(i.e., interleukin-10). These proinflammatory cytokines activate neutrophils and endothelial cells,
leading to an increased expression of inducible nitric oxide synthase and subsequent vasodilation.

ACCP Updates in Therapeutics® 2022: Critical Care Pharmacy Preparatory Review and Recertification Course

641
Shock Syndromes I: Introduction, Vasodilatory, and Sepsis

b. Neurogenic shock involves a decrease in sympathetic outflow from the central nervous system with
unopposed parasympathetic activity. As such, vascular tone is lost, resulting in a decrease in SVR
and venous pooling of blood with a subsequent decrease in preload. Concomitant bradycardia is
common, and decreased CO (even after fluid administration) may occur because of the interruption
of cardiac sympathetic innervation, further contributing to hypotension. This shock type classically
occurs as a complication of an acute spinal cord injury at the level of the thoracic or cervical
vertebra, most commonly when the injury is above the fifth cervical vertebra.
c. Immune-mediated (anaphylactic) shock occurs because of reexposure to a sensitizing foreign
pathogen that stimulates immunoglobulin E–mediated mast cell or basophil degranulation and
resultant cytokine (e.g., histamine and tryptase) release. The mechanism of vasodilation is complex
and multifaceted; however, for example, the binding of histamine to the histamine-1 receptor can
activate nitric oxide synthase with resultant increases in nitric oxide and vasodilation.
4. Profound vasodilation leads to ineffective circulating plasma volume (either from venodilation [“venous
pooling”] or from fluid shifts because of increased vascular permeability) and resultant decreases in
cardiac preload and CO.

C. Resuscitation and Treatment


1. Resuscitation and treatment of patients specifically with sepsis and septic shock is covered later in the
chapter.
2. The underlying cause of the shock state must quickly be addressed when resuscitation is initiated.
a. Septic shock requires rapid (within 1 hour of disease recognition) administration of antimicrobials
with activity against all likely pathogens.
b. The potential offending agent should be discontinued (for medication-related reactions) and
potentially removed (in the setting of envenomation) for patients with immune-mediated
(anaphylactic) shock.
3. Treatment goals and end points of resuscitation are usually similar to those listed in section IV,
Resuscitation Parameters and End Points. In the setting of acute spinal cord injury, a MAP goal of
at least 85 mm Hg has been associated with improved outcomes in uncontrolled studies. As such, the
guidelines recommend an initial MAP goal of 85–90 mm Hg in patients with acute spinal cord injury
for 1 week after injury (level III recommendation).
4. Initial resuscitation
a. The treatment of choice is intravenous fluids, which restore effective circulating volume.
i. Crystalloids (e.g., lactated Ringer solution or normal saline) are typically the initial fluid of
choice compared with colloids (e.g., albumin or other blood products).
ii. Fluid should be administered until the patient is no longer fluid responsive.
b. Vasopressors should be initiated for hypotension unresponsive to fluid administration.
i. Norepinephrine is usually considered the first-line vasopressor because of its ability to increase
SVR without decreasing CO.

ACCP Updates in Therapeutics® 2022: Critical Care Pharmacy Preparatory Review and Recertification Course

642
Shock Syndromes I: Introduction, Vasodilatory, and Sepsis

ii. Epinephrine is typically used for patients with immune-mediated (anaphylactic) shock to
provide mast cell stabilization, histamine suppression, bronchodilation, and vasoconstriction.
During initial treatment, intramuscular epinephrine (0.01 mg/kg; maximum 0.5 mg) into
the anterolateral thigh is preferred for speed and fewer adverse events, including overdoses,
compared with intravenous epinephrine. Intramuscular epinephrine may be repeated at 5-
to 15-minute intervals if the response is inadequate. Beyond initial management, in patients
who continue to be hypotensive despite intramuscular epinephrine, epinephrine infusions
(1–10 mcg/minute) are often used for ongoing shock, largely because of convention rather than
evidence. In patients with actual or impending cardiac arrest, slow, intravenous administration
of epinephrine 50 mcg may be necessary. Norepinephrine, vasopressin, and other vasopressor
agents have been used with refractory hypotension associated with anaphylaxis, though
it is unclear whether this strategy is superior to epinephrine alone. Dosing and route of
administration of epinephrine for anaphalaxis should be monitored closely and verified;
accidental overdoses involving decimal errors or confusing “mcg” with “mg” have led to
adverse events such as cardiac arrest.
iii. In neurogenic shock, agents with combined vasoconstrictive and inotropic properties (e.g.,
norepinephrine, dopamine, epinephrine) are preferred. Phenylephrine may also be used, but a
concomitant inotropic agent (e.g., dobutamine) is often administered.
(a) Atropine may also be given for symptomatic bradycardia.
(b) Adjunctive oral pseudoephedrine or midodrine may also be used to maintain a MAP
greater than 85 mm Hg for 1 week.
5. Additional therapies
a. Immune-mediated (anaphylactic) shock is also conventionally treated with concomitant histamine-1
and histamine-2 receptor antagonists and corticosteroids, though no strong data support the
use of these agents. Corticosteroids have a slow onset of action (4–6 hours) and are therefore
ineffective for acute management; however, they may decrease the risk of late symptoms. If used,
methylprednisolone 1–2 mg/kg or the equivalent is recommended.
b. The most recent guidelines regarding the treatment of patients with acute cervical spine and spinal
cord injuries recommend against administering methylprednisolone (level I recommendation).
c. Patients with vasodilatory shock secondary to adrenal insufficiency (Addisonian crisis) should
receive intravenous corticosteroids (typically hydrocortisone 50 mg every 6 hours).
d. Management of vasodilatory shock secondary to thyroid insufficiency (myxedema coma) is
outlined in the Hepatic Failure/GI/Endocrine Emergencies chapter.

ACCP Updates in Therapeutics® 2022: Critical Care Pharmacy Preparatory Review and Recertification Course

643
Shock Syndromes I: Introduction, Vasodilatory, and Sepsis

VII.  SEPSIS

A. Introduction
1. Sepsis is caused by a dysregulated host response to infection.
2. While continuing to evolve, the diagnosis of sepsis traditionally has required a known or suspected
source of infection with two or more criteria of the systemic inflammatory response syndrome. Sepsis
was further classified as sepsis, severe sepsis (sepsis with organ dysfunction), or septic shock (sepsis
with arterial hypotension unresponsive to fluid administration).

B. Definitions
1. The Surviving Sepsis Campaign (SSC), a joint collaboration between the Society of Critical Care
Medicine and the European Society of Intensive Care Medicine, has published five iterations of
international guidelines for the treatment of patients with sepsis and septic shock. The 2021 SSC
guidelines were sponsored or endorsed by 24 international organizations and inform many of the
recommendations in this section.
2. Sepsis is defined as life-threatening organ dysfunction caused by a dysregulated host response to
infection.
3. Organ dysfunction can be identified as an acute change in the total SOFA score of 2 points or higher
consequent to the infection.
4. A prompt identification tool (“qSOFA”) for bedside use outside the ICU was also proposed in 2016,
which includes altered mental status, SBP of 100 mm Hg or less, and respiratory rate of 22 breaths/
minute or greater. Fulfillment of two of these three criteria had predictive validity for mortality similar
to the full SOFA score outside the ICU (technically, a Glasgow Coma Scale score of 13 or less was used
in the regression model but was simplified to any alteration in mental status for the qSOFA).
a. Of importance, qSOFA does not define sepsis, but two or more qSOFA criteria are a predictor of
both increased mortality and ICU stays of more than 3 days in non-ICU patients. These criteria
should also be used to investigate further for infection.
b. The full SOFA score is superior to the qSOFA for predicting mortality in patients in the ICU.
c. Emerging data analyses suggest that in patients outside the ICU, the qSOFA score has a lower
discriminant ability for predicting ICU admission or death than other early warning scores (NEWS
or MEWS) and should be further investigated before implementation as a risk-stratification tool for
patients with suspected infection.
d. Because data are lacking to support its use as a screening tool, the 2021 SSC guidelines recommend
against using qSOFA as a single screening tool for sepsis or septic shock.
5. Septic shock is defined as a subset of sepsis in which underlying circulatory and cellular/metabolic
abnormalities are sufficient to increase mortality substantially.
6. Patients with septic shock can be identified as those with sepsis with persistent hypotension requiring
vasopressors to maintain a MAP of 65 mm Hg and greater and having a serum lactate concentration
greater than 2 mmol/L despite adequate fluid resuscitation.
7. With this new definition, the systemic inflammatory response syndrome criteria are no longer used, and
severe sepsis no longer exists as a clinical entity.
8. The new sepsis definition better identifies patients at risk of in-hospital mortality secondary to sepsis
(about 10% mortality) than the previous definition.
9. These definitions will likely shift patient identification and terminology in clinical practice, particularly
for patient inclusion in studies.
10. The SSC bundles are no longer included in the SSC guidelines, but they are published on the SSC
website (www.survivingsepsis.org). This website also contains the most up-to-date recommendations,
tools for implementation, and official statements from the campaign regarding ongoing events (e.g., the
new sepsis definitions).

ACCP Updates in Therapeutics® 2022: Critical Care Pharmacy Preparatory Review and Recertification Course

644
Shock Syndromes I: Introduction, Vasodilatory, and Sepsis

C. Epidemiology
1. Sepsis is common, with an annual incidence in the United States above 890,000, rising by about 13%
per year, and an associated cost exceeding $24 billion.
2. In a European epidemiologic study, 37% of all ICU patients had sepsis.
3. Septic shock is the most common shock type, accounting for 62% of all cases of shock syndromes
requiring vasopressors.
4. In-hospital mortality associated with severe sepsis and septic shock (using previous definitions) is
around 18%–25%, whereas mortality at 2 years is around 45%.
5. In-hospital mortality of patients with septic shock defined according to the new definition was 35%–
54% in large retrospective data sets.

D. Pathophysiology
1. Sepsis and septic shock involve complex interactions between an infecting pathogen and the host
inflammatory, immune, and coagulation response.
2. Hypotension develops through the inappropriate activation of vasodilatory mechanisms (increased nitric
oxide synthesis) and the failure of vasoconstrictive pathways (activation of ATP-dependent potassium
channels in vascular smooth muscle cells and vasopressin deficiency), resulting in a vasodilatory
shock. In addition, blood flow is inappropriately dispersed at the organ level or in the microcirculation
(shunting), leading to distributive shock.
3. Vascular endothelial cell injury leads to capillary fluid leak and a resultant decrease in preload. Venous
dilation further exacerbates the decrease in cardiac preload.
4. Tissue Do2 may further be impaired by a decrease in CO, inadequate Cao2 (low hemoglobin concentration
or saturation), or impaired oxygen unloading from hemoglobin. These multifactorial hemodynamic
abnormalities lead to decreased effective tissue perfusion, in which Vo2 exceeds Do2 and cellular injury
results. This further compounds the proinflammatory and procoagulant state, precipitating multiple
organ dysfunction and possibly death.

E. Diagnosis
1. There is no specific diagnostic test for sepsis or septic shock, and the diagnosis is typically based on the
definitions noted earlier.
2. Recognizing that pneumonia is the most common site of infection (in about 45% of cases), followed
by intra-abdominal infection (about 30%) and urinary tract infection (about 11%), microbial cultures
should be sent from all suspected infectious sites as soon as possible.
3. All microbial cultures are negative in more than 25% of cases, and only 30% of patients have positive
blood cultures. The most commonly isolated pathogens in ICU patients are gram-negative bacteria
(62%), gram-positive bacteria (47%), and fungi (19%).
4. Cultures should be obtained before antimicrobial therapy is initiated unless doing so would significantly
delay (greater than 45 minutes) therapy.

F. Treatment of Sepsis and Septic Shock


1. The pharmacologic-related 2021 SSC recommendations can be found in Box 1.

ACCP Updates in Therapeutics® 2022: Critical Care Pharmacy Preparatory Review and Recertification Course

645
Shock Syndromes I: Introduction, Vasodilatory, and Sepsis

Box 1. Pharmacologic-Related Surviving Sepsis Campaign Recommendations for Patients with Sepsis and Septic
Shocka
Initial Resuscitation
1. Sepsis and septic shock are medical emergencies, and treatment and resuscitation are recommended to begin
immediately (BPS)
2. For patients with sepsis-induced hypoperfusion or septic shock, at least 30 mL/kg of IV crystalloid fluid is
suggested to be given within the first 3 hr of resuscitation (weak recommendation, low quality of evidence)
3. Dynamic measures to guide fluid resuscitation are suggested over physical examination or static parameters
alone (weak recommendation, very low quality of evidence)
4. Guiding resuscitation to decrease serum lactate in patients with elevated lactate level is suggested (weak
recommendation, low quality of evidence)
5. Capillary refill time to guide resuscitation as an adjunct to other measures of perfusion is suggested (weak
recommendation, low quality of evidence)
6. An initial target MAP of 65 mm Hg is recommended in patients with septic shock requiring vasopressors
(strong recommendation, moderate quality of evidence)
Antimicrobial Therapy
1. In patients with possible septic shock or a high likelihood of sepsis, administering antimicrobials immediately,
ideally within 1 hr of recognition, is recommended (strong recommendation, low quality of evidence for septic
shock; very low quality of evidence for sepsis without shock)
2. In patients with possible sepsis without shock, rapid assessment of the likelihood of infectious vs. noninfectious
causes of acute illness is recommended (BPS)
3. A time-limited course of rapid investigation and, if concern for infection persists, the administration of
antimicrobials within 3 hr from the time when sepsis was first recognized is suggested for patients with
possible sepsis without shock (weak recommendation, very low quality of evidence)
4. In patients with a low likelihood of infection and without shock, deferring antimicrobials while continuing to
closely monitor the patient is suggested (weak recommendation, very low quality of evidence)
5. Optimizing dosing strategies of antimicrobials based on accepted pharmacokinetic/pharmacodynamic
principles and specific drug properties is recommended (BPS)
6. Empiric antimicrobials with MRSA coverage are recommended in patients with sepsis or septic shock at high
risk of MRSA (BPS)
7. Two antimicrobials with gram-negative coverage are suggested for empiric treatment in patients with sepsis
or septic shock and at high risk for MDR organisms (weak recommendation, very low quality of evidence)
8. Double gram-negative coverage is not suggested once the causative pathogen and susceptibilities are known
(weak recommendation, very low quality of evidence)
9. Empiric antifungal therapy is suggested in patients with sepsis or septic shock at high risk of fungal infection
(weak recommendation, low quality of evidence)
10. Daily assessment for de-escalation of antimicrobials is suggested over using fixed durations of therapy (weak
recommendation, very low quality of evidence)
11. In patients with an initial diagnosis of sepsis or septic shock and adequate source control, shorter over longer
durations of antimicrobial therapy are suggested (weak recommendation, very low quality of evidence)
12. Procalcitonin in combination with clinical evaluation is suggested to decide when to discontinue antimicrobials
in patients with an initial diagnosis of sepsis or septic shock and adequate source control where optimal
duration of therapy is unclear (weak recommendation, low quality of evidence)

ACCP Updates in Therapeutics® 2022: Critical Care Pharmacy Preparatory Review and Recertification Course

646
Shock Syndromes I: Introduction, Vasodilatory, and Sepsis

Box 1. Pharmacologic-Related Surviving Sepsis Campaign Recommendations for Patients with Sepsis and Septic
Shocka (continued)
Fluid Therapy
1. Crystalloids are recommended as first-line fluid for resuscitation (strong recommendation, moderate quality
of evidence)
2. Balanced crystalloids instead of normal saline for resuscitation are suggested (weak recommendation, low
quality of evidence)
3. Albumin is suggested in patients who received large volumes of crystalloids (weak recommendation, moderate
quality of evidence)
4. Starches are not recommended for resuscitation (strong recommendation, high quality of evidence)
5. Gelatin is not suggested for resuscitation (weak recommendation, moderate quality of evidence)
Vasoactive Medications
1. Norepinephrine is recommended as the first-line vasopressor (strong recommendation, high quality of evidence
over dopamine, moderate quality of evidence over vasopressin, low quality of evidence over epinephrine)
2. Vasopressin is suggested in patients with septic shock on norepinephrine with inadequate MAP levels, instead
of escalating the dose of norepinephrine (weak recommendation, moderate quality of evidence)
3. Epinephrine is suggested in patients with septic shock and inadequate MAP levels despite norepinephrine and
vasopressin (weak recommendation, low quality of evidence)
4. For patients with septic shock and cardiac dysfunction with persistent hypoperfusion despite adequate volume
status and arterial blood pressure, either adding dobutamine to norepinephrine or using epinephrine alone is
suggested (weak recommendation, low quality of evidence)
5. Adding dobutamine to norepinephrine or using epinephrine alone is suggested in patients with septic shock
and cardiac dysfunction with persistent hypoperfusion despite adequate volume status and arterial blood
pressure (weak recommendation, low quality of evidence)
6. Starting vasopressors peripherally to restore MAP rather than delaying initiation until a central venous access
is secured is suggested (weak recommendation, very low quality of evidence)
Corticosteroids
1. Corticosteroids are suggested for patients with septic shock and an ongoing requirement for vasopressor
therapy (weak recommendation, moderate quality of evidence)
a
Strength and evidence level of recommendations listed in parentheses are according to the Grading of Recommendations Assessment, Development and Evaluation
(GRADE) system. The system classifies strength of recommendations as strong or weak and quality of evidence as high, moderate, low, or very low. BPSs represent
ungraded strong recommendations developed using strict criteria.
BPS = best practice statement; IV = intravenous(ly); MAP = mean arterial pressure; MDR = multidrug resistant; MRSA = methicillin-resistant Staphylococcus aureus.
Information from: Evans L, Rhodes A, Alhazzani W, et al. Surviving Sepsis Campaign: international guidelines for management of sepsis and septic shock 2021. Crit
Care Med 2021;49:e1063-e1143.

ACCP Updates in Therapeutics® 2022: Critical Care Pharmacy Preparatory Review and Recertification Course

647
Shock Syndromes I: Introduction, Vasodilatory, and Sepsis

2. Early recognition and treatment of patients with sepsis, particularly those with sepsis-induced end-
organ hypoperfusion, is of utmost importance.
3. Initial resuscitation
a. Initial goals of therapy are to restore effective tissue perfusion (by administering intravenous
fluids and vasoactive medications) while treating the underlying cause of the syndrome (through
antimicrobial administration and infectious source control, as applicable).
b. Patients should first be administered a fluid challenge of 30 mL/kg of crystalloid solution as quickly
as possible.
c. Quantitative resuscitation is no longer recommended for routine use in patients with sepsis or
septic shock.
i. This strategy includes intensive monitoring (e.g., placement of a central venous [superior
vena cava] catheter or echocardiography), setting goals for hemodynamic support, and using
therapies to achieve those goals (e.g., use and optimization of fluids, vasopressors, and Do2
methods).
ii. Instead, further resuscitation should be guided by reassessment of hemodynamic parameters.
d. Quantitative resuscitation strategies have been evaluated in several large studies.
i. “Early goal-directed therapy”
(a) The landmark study of early goal-directed therapy evaluated 263 patients with sepsis and
septic shock treated in the ED for the first 6 hours after presentation.
(b) Treatment of patients in the standard therapy group was at the clinician’s discretion,
whereas treatment of those in the intervention arm uniformly incorporated a protocol to
achieve the previously mentioned goals and incorporated Scvo2 as a treatment goal (70%
or greater).
(c) Early, aggressive, goal-directed resuscitation was associated with a 16% absolute risk
reduction in hospital mortality compared with standard therapy (30.5% vs. 46.5%,
p<0.009).
(d) Critiques of this study include a higher-than-expected mortality rate in the standard
therapy arm, the incorporation of CVP as a resuscitation goal, and the study’s single-
centered nature.
(e) Many centers faced logistical and financial barriers to implementing the early goal-
directed therapy protocol. As such, alternative approaches to quantitative resuscitation
were developed and studied.
ii. Protocolized Care for Early Septic Shock (ProCESS) study
(a) Randomized 1341 ED patients with septic shock in U.S. academic medical centers to three
treatment arms:
(1) Early goal-directed therapy (with the same protocol as noted previously)
(2) Protocol-based standard care that did not require use of a central venous catheter
but that used clinician judgment for fluid administration and hypoperfusion, together
with a shock index (heart rate/SBP) of less than 0.8 and an SBP greater than 100 mm
Hg as resuscitation targets
(3) Usual care (treatment according to the bedside physician)
(b) 60-day mortality did not differ between the treatment arms (early goal-directed therapy
21.0% vs. protocol-based standard care 18.2% vs. usual care 18.9%, p=0.55, for the three-
group comparison).
(c) Patients in the early goal-directed therapy arm were more often admitted to the ICU
(91.3% vs. protocol-based standard care 85.4% vs. usual care 86.2%, p=0.01).

ACCP Updates in Therapeutics® 2022: Critical Care Pharmacy Preparatory Review and Recertification Course

648
Shock Syndromes I: Introduction, Vasodilatory, and Sepsis

(d) This study has several critiques, primarily a likely shift of usual care toward therapy that
resembles quantitative resuscitation (both of which may have contributed to a type II
error).
iii. Australasian Resuscitation in Sepsis Evaluation (ARISE) study
(a) Randomized ED patients in both academic and nonacademic centers mainly in Australia
and New Zealand to either early goal-directed therapy (with the same protocol noted
previously) or usual care (treatment according to the bedside physician)
(b) 90-day mortality did not differ between the treatment arms (early goal-directed therapy
18.6% vs. usual care 18.8%, p=0.90).
(c) Because the enrolled patients had lower severity-of-illness scores and lower mortality at
90 days, they may have been less acutely ill than the patients enrolled in the ProCESS trial.
However, about 70% of the patients in the ARISE study had septic shock at randomization
(only 54% of the patients in the ProCESS trial met this criterion), suggesting the patients
in ARISE were critically ill and the intended population was studied.
iv. Protocolised Management in Sepsis (ProMISe) study
(a) Randomized 1260 ED patients in both academic and nonacademic centers in England to
either early goal-directed therapy or usual care (identical to the ARISE study in design)
(b) 90-day mortality did not differ between those treated with early goal-directed therapy and
those treated with usual care (29.5% vs. 29.2%, p=0.90).
(c) Patients in the early goal-directed therapy arm had higher SOFA scores at 6 hours (mean
6.4 ± 3.8 vs. 5.6 ± 3.8, p<0.001), more often received advanced cardiovascular support
(37.0% vs. 30.9%, p=0.026), and had a longer ICU length of stay (median [IQR] 2.6 [1.0–
5.8] days vs. 2.2 [0.0–5.3] days, p=0.005).
v. Discussion of ProCESS, ARISE, and ProMISe studies
(a) In a systematic review and meta-analysis that included the ProCESS, ARISE, and ProMISe
studies, early goal-directed therapy was not associated with a difference in mortality
compared with control (OR 1.01; 95% CI, 0.88–1.16; p=0.9), but it was associated with
increased vasopressor use (OR 1.25; 95% CI, 1.10–1.41; p<0.001) and more frequent ICU
admission (OR 2.19; 95% CI, 1.82–2.65; p<0.001).
(b) Patients in these studies received about 30 mL/kg of crystalloid solution before study
enrollment. This is significantly different from the patients in the landmark early goal-
directed therapy study, in which patients were enrolled before resuscitation.
(c) The ARISE and ProMISe studies required antimicrobial administration before enrollment.
In ProCESS, 76% of patients had antimicrobials administered before enrollment and 97%
within 6 hours of enrollment. In the landmark early goal-directed therapy study, only 86%
of patients had antimicrobials administered within 6 hours of enrollment.
(d) These studies highlight the benefits of timely administration of antibiotics and intravenous
fluids, which should be a focus of the early care of patients with sepsis and septic shock.
(e) 51%–62% of patients in the usual care arms of these studies had a central venous catheter
inserted, even though it was not required in the study protocol.
(f) The consistent findings of lower mortality rates in the contemporary studies suggest that
care of patients with septic shock has evolved since the landmark early goal-directed
therapy study.
(g) In centers with ubiquitous early recognition and aggressive resuscitation strategies,
protocolized care may no longer be mandatory.

ACCP Updates in Therapeutics® 2022: Critical Care Pharmacy Preparatory Review and Recertification Course

649
Shock Syndromes I: Introduction, Vasodilatory, and Sepsis

vi. Lactate clearance


(a) In one study of patients with sepsis and septic shock, failure to achieve a lactate clearance
of at least 10% was associated with a higher mortality rate than was failure to achieve an
Scvo2 greater than 70%.
(b) A large, multicenter, noninferiority study randomized patients to a quantitative resuscitation
protocol that incorporated Scvo2 as a treatment goal or to a quantitative resuscitation
protocol that incorporated a lactate clearance of 10% or greater as a treatment goal.
(1) The study aimed to address whether using Scvo2 as a treatment goal was necessary in
a quantitative resuscitation protocol.
(2) The lactate clearance strategy was noninferior to the Scvo2 strategy (in-hospital
mortality 17% vs. 23%, respectively; 95% CI for mortality difference -3% to 15%
[above the -10% predefined noninferiority threshold]).
(3) Only 10% of the patients in each arm received a therapy specifically directed to
improve either lactate clearance or Scvo2, which biased the study toward finding no
difference between treatment arms (and incorrect rejection of a true null hypothesis
of the existence of a difference between groups in this noninferiority study; a type I
error).
(4) Equally pertinent is that most patients with septic shock in this study (90%) achieved
adequate Do2 with only fluids and vasopressors.
(c) Another multicenter, open-label study randomized ICU patients with a lactate
concentration of 3 mmol/L or greater to the addition of lactate clearance of 20% or more
(evaluated every 2 hours for the first 8 hours of therapy) or standard therapy (in which the
treatment team was blinded for lactate concentrations).
(1) Both treatment arms had resuscitation targets of CVP 8–12 mm Hg, MAP 60 mm Hg
or greater, and urinary output above 0.5 mL/kg/hour. Scvo2 monitoring was optional
in the standard therapy group but was mandated as part of the lactate clearance
group. In the lactate clearance group, if the Scvo2 was 70% or greater but the lactate
concentrations did not decrease by 20%, vasodilators (e.g., nitroglycerin) were
initiated to improve microvascular perfusion.
(2) Adding lactate clearance as a treatment goal was not associated with a difference in
hospital mortality on bivariate analysis (33.9% vs. 43.5%, p=0.067). After adjustment
for baseline differences between groups with multivariable analysis, a significant
difference favored the lactate clearance group (HR 0.61; 95% CI, 0.43– 0.87; p= 0.006).
(d) Given these findings, in a low-level recommendation, the SSC suggested targeting
resuscitation to decrease lactate in patients with elevated lactate concentrations as a
marker of tissue hypoperfusion.
(e) A recent multicenter trial, the ANDROMEDA-SHOCK trial, randomized patients with
septic shock to either peripheral perfusion-targeted resuscitation (goal capillary refill time
3 seconds) or lactate level-targeted resuscitation (goal lactate normalization or a >20%
reduction in lactate every 2 hours). This study found no association between resuscitation
strategy and 28 day all cause mortality (HR 0.75; 95% CI, 0.55–1.02). This trial indicates
that in resource-poor settings, or when lactate cannot be readily checked, capillary refill
time may be a reasonable alternative to ensure adequate perfusion in patients with septic
shock.

ACCP Updates in Therapeutics® 2022: Critical Care Pharmacy Preparatory Review and Recertification Course

650
Shock Syndromes I: Introduction, Vasodilatory, and Sepsis

4. Blood pressure (MAP) goal


a. As discussed previously, MAP is the true driving pressure for peripheral blood flow and end-organ
perfusion and is preferred to SBP as a therapeutic target.
b. A multicenter, open-label study randomized patients with septic shock to resuscitation with a MAP
goal of either 65–70 mm Hg (low-target group) or 80–85 mm Hg (high-target group). The higher
MAP target was achieved through vasopressor administration; patients in the high-target group
had a significantly higher infusion rate and duration of vasopressors than did those in the low-target
group, but the groups did not differ in total volume of fluid administration. The treatment arms did
not differ in 28-day mortality (34.0% in the low-target group vs. 36.6% in the high-target group,
p=0.57). However, the incidence of atrial fibrillation was significantly higher in the high-target
group (6.7% vs. 2.8%, p=0.02). In an a priori–defined subgroup analysis of patients with chronic
hypertension (with randomization stratified according to this covariate), those randomized to the
high-target group had a lower incidence of a doubling of the SCr concentration (38.9% vs. 52.0%,
p=0.02, stratum interaction p=0.009) and the need for renal replacement therapy (31.7% vs. 42.2%,
p=0.046, stratum interaction p=0.04).
5. Summary and recommendations for initial resuscitation
a. After an initial fluid challenge, fluid therapy should be continued, using a fluid challenge technique,
until the patient is no longer fluid responsive.
b. Vasopressors should be applied to initially target a MAP of 65–70 mm Hg, but the MAP goal may
subsequently be adjusted if adequate organ perfusion is not attained (particularly in patients with
chronic hypertension).
c. Adequate tissue Do2 should be ensured. If a central venous catheter is not inserted, lactate clearance
is a reasonable target. If a central venous catheter is inserted, a combination of markers can be used
(e.g., lactate clearance and Scvo2 of 70% or greater).
6. Sepsis and septic shock care bundle
a. The SSC, in collaboration with the Institute for Healthcare Improvement, has developed a core set
of process steps and treatment goals grouped into a care bundle for patients with sepsis and septic
shock.
b. The goal of the care bundle is to improve early recognition and treatment of patients with sepsis
and septic shock.
c. This SSC care bundle was updated in 2015 in response to new evidence from the three previously
noted quantitative resuscitation studies (Box 2).
d. The updated bundle acknowledges the findings of the three studies and recommends using
techniques in addition to CVP and Scvo2 to reassess fluid responsiveness and tissue perfusion.
These techniques include either use of a repeat focused examination by a licensed practitioner to
evaluate for vital signs, cardiopulmonary findings, capillary refill, pulse, and skin findings or use
of at least two of the following: CVP, Scvo2, bedside ultrasonography, or dynamic markers of fluid
responsiveness (PLR or fluid challenge).
e. This care bundle has been adopted by the Centers for Medicare & Medicaid Services as a quality
(core) measure (SEP-1).
f. The slight differences between the SSC bundle and the SEP-1 quality measure are noted in Box 2.
g. In July 2016, a letter to the editor from the SEP-1 measure stewards and a representative from the
Centers for Medicare & Medicaid Services indicated that the SEP-1 measure would not be updated
to correspond with the new sepsis definitions until further data are available (JAMA 2016;316:457-
8).
h. Adjustment of the SSC bundle and implementation of sepsis as a quality measure have led to a shift
in resuscitation approaches for many clinicians.

ACCP Updates in Therapeutics® 2022: Critical Care Pharmacy Preparatory Review and Recertification Course

651
Shock Syndromes I: Introduction, Vasodilatory, and Sepsis

i. Practitioners should systematically evaluate their institutional compliance and implement broad
process steps to ensure compliance with the quality measure. These steps may include, but are not
limited to, patient identification by clinical decision support tools in the electronic medical record
and implementation of care paths and order sets for treatment.
j. In 2018, the SSC released an update recommending that all bundle elements be completed within
1 hour to ensure that aggressive resuscitation begins immediately in patients with septic shock.
At this time, this recommendation has not been adopted by the Centers for Medicare & Medicaid
Services.

Box 2. Sepsis and Septic Shock Management Bundlea

Accomplished within 3 hr of presentationb


1. Measure lactate concentration
2. Obtain blood cultures before administering antibiotics
3. Administer broad-spectrum antibioticsc
4. Administer crystalloid 30 mL/kg for hypotension or lactate ≥ 4 mmol/Ld

If septic shocke is present, additional measures to be accomplished within 6 hr of presentationb


5. Apply vasopressors (for hypotension that does not respond to initial fluid resuscitation) to maintain MAP
≥ 65 mm Hg
6. If persistent arterial hypotension after initial fluid administration (MAP < 65 mm Hg) or if initial lactate
was ≥ 4 mmol/L, reassess volume status and tissue perfusion, and document findingsf
7. Re-measure lactate if the initial lactate concentration was elevatedg
a
Applies to all patients presenting with severe sepsis and septic shock (of note, the previous [Sepsis-2] definitions, which include the term severe sepsis, continue to
be used for this bundle). Patients are excluded from the Centers for Medicare & Medicaid Services quality measure (SEP-1) for many reasons, including (1) they are
transferred from another care facility (including an ED), (2) they have advanced directives for comfort care, (3) they have clinical conditions that preclude total measure
completion (i.e., mortality within the first 6 hr of presentation), (4) they have a length of stay > 120 days, or (5) they were administered intravenous antibiotics within 24
hr of presentation. All components outlined must be fulfilled to satisfy the SEP-1 quality measure (it is an “all-or-none” measure).
b
Time of presentation is defined as the time of triage in the ED or, if the patient is located in another care venue, from the earliest chart annotation consistent with all
elements of severe sepsis or septic shock as ascertained through chart review.
c
Before July 1, 2021, specific antibiotics were outlined in the SEP-1 quality measure to qualify as “broad spectrum.” As of July 1, 2021, any IV antibiotic satisfies
the measure if given within the appropriate time.
d
Starting January 1, 2022, the SEP-1 quality measure allows for resuscitation volumes < 30 mL/kg if appropriate documentation with rationale is recorded in the
medical record as outlined in the measure documents.
Septic shock defined as hypotension (to SBP < 90 mm Hg, MAP < 70 mm Hg, or SBP decrease > 40 mm Hg from known baseline) or a lactate concentration ≥ 4
e

mmol/L.
f
To meet the requirements, one of the following must be documented: (1) a focused examination by a licensed independent practitioner including vital signs,
cardiopulmonary, capillary refill, pulse, and skin findings; or (2) any two of the following: measure CVP, measure Scvo2, bedside cardiovascular ultrasonography, or
dynamic assessment of fluid responsiveness with PLR or fluid challenge.
g
In the SEP-1 quality measure, an initial lactate concentration ≥ 2 mmol/L is considered elevated.
Information from: Surviving Sepsis Campaign (SSC). Updated Bundles in Response to New Evidence [homepage on the Internet]. Available at www.survivingsepsis.
org/SiteCollectionDocuments/SSC_Bundle.pdf. Accessed May 24, 2017; and The Joint Commission. Specifications Manual for National Hospital Inpatient Quality
Measures. 2016. Available at https://www.jointcommission.org/specifications_manual_for_national_hospital_inpatient_quality_measures.aspx. Accessed May 24, 2017.

ACCP Updates in Therapeutics® 2022: Critical Care Pharmacy Preparatory Review and Recertification Course

652
Shock Syndromes I: Introduction, Vasodilatory, and Sepsis

Patient Case

6. A 56-year-old woman with a medical history of hypertension presents to the ED with shortness of breath
and cough productive of sputum. Her vital signs on admission are as follows: blood pressure 92/68 mm Hg,
heart rate 104 beats/minute, respiratory rate 26 breaths/minute, and temperature 101.6°F (38.7°C). A chest
radiograph reveals an opacity in the left lower lobe, but the radiograph is otherwise unremarkable. Her labo-
ratory values of interest include Hgb 12.7 g/dL, WBC 16.4 × 103 cells/mm3, Plt 80,000/mm3, albumin 2.0
g/dL, lactate 3.2 mmol/L, and SCr 1.3 mg/dL. Her Glasgow Coma Scale score is 13. Which best describes
the patient’s condition?
A. Systemic inflammatory response syndrome.
B. Sepsis.
C. Severe sepsis.
D. Septic shock.

7. Antimicrobials
a. Timing of initiation
i. Adequate empiric antibiotics should be initiated within 1 hour after recognizing sepsis or
septic shock.
ii. A multicenter, retrospective study of patients with septic shock found that within the first 6 hours
after the onset of hypotension, each hour of delay beyond the first hour in the administration of
appropriate antibiotics was associated with a 7.6% decrease in hospital survival.
iii. Other studies have further shown the importance of empiric antimicrobials used together with
initial resuscitation, associating antimicrobial administration either before shock or within the
initial hour of shock with improved survival.
b. Initial empiric broad-spectrum therapy should include one or more drugs with activity against all
likely pathogens (bacterial and/or fungal and/or viral).
i. In an observational study of more than 5700 patients with septic shock, those who received
initial appropriate antimicrobials had a significantly higher hospital survival rate than did
those who received initial inappropriate antimicrobials (52.0% vs. 10.3%, p<0.0001).
ii. Combination antibacterial therapy (at least two different classes of antibiotics) is indicated for
patients with septic shock, but not for those with sepsis without shock.
(a) A randomized controlled trial of patients with sepsis (termed severe sepsis in the study) that
allocated patients to meropenem monotherapy or combination therapy with meropenem
and moxifloxacin found no difference between groups in mean SOFA scores over 14
days (7.9 points vs. 8.3 points, p=0.36) or mortality rates at 28 or 90 days. Important
caveats to this study are that the patient population studied was at a low risk of resistant
pathogens (half of the patients had a community-acquired infection) and that moxifloxacin
inadequately covers pathogens with a high likelihood of multidrug resistance (e.g.,
Pseudomonas aeruginosa and Acinetobacter spp.).
(b) A meta-analysis that included 50 studies and more than 8500 patients with sepsis
detected no overall mortality benefit of combination antibacterial therapy compared
with monotherapy (pooled OR of death 0.86; 95% CI, 0.71–1.03, p=0.09); however, a
stratified analysis showed significantly lower mortality with combination therapy in more
severely ill patients (monotherapy risk of death greater than 25%, pooled OR of death
with combination therapy 0.54; 95% CI, 0.45–0.66, p<0.001). The benefit of combination
therapy was confined to patients with septic shock (with no benefit of combination therapy
in patients without shock). In addition, a meta-regression analysis, which tried to elucidate

ACCP Updates in Therapeutics® 2022: Critical Care Pharmacy Preparatory Review and Recertification Course

653
Shock Syndromes I: Introduction, Vasodilatory, and Sepsis

the differences in the benefit with combination therapy according to baseline mortality
risk, showed a significant benefit of combination therapy in those with a mortality risk
greater than 25%. These data suggest that severely ill patients benefit from combination
antibacterial therapy.
iii. Combination therapy increases the likelihood that at least one drug is effective against the
pathogen, particularly for known or suspected multidrug-resistant organisms such as P.
aeruginosa.
iv. The 2021 SSC guidelines suggest combination therapy for patients at high risk of multidrug-
resistant pathogens.
v. Empiric combination therapy should be de-escalated within the first few days of therapy if the
patient has clinical improvement.
c. As noted in the Pharmacokinetics/Pharmacodynamics chapter, sepsis and septic shock can
significantly affect the probability of attaining the antimicrobial pharmacokinetic/pharmacodynamic
target. Dosing strategies for patients with sepsis should be optimized according to pharmacokinetic/
pharmacodynamic principles.
i. Most notably, the volume of distribution of hydrophilic antibiotics (e.g., β-lactams,
aminoglycosides, and vancomycin) will be increased. Clearance may either be increased (in the
setting of augmented renal clearance) or decreased (in the presence of end-organ dysfunction).
ii. A pharmacokinetic/pharmacodynamic study of the first dose of β-lactams in patients with
sepsis and septic shock suggested that the pharmacokinetic/pharmacodynamic target was
attained in less than 50% of the patients given ceftazidime 2 g (28% target attainment),
cefepime 2 g (16%), and piperacillin/tazobactam 4 g/0.5 g (44%). The pharmacokinetic/
pharmacodynamic target was attained in 75% of patients receiving meropenem 1 g. For each
antibiotic, the volume of distribution was higher and the clearance was lower than the values
reported in healthy volunteers.
iii. A multicenter cross-sectional study of β-lactam concentrations in critically ill patients found
that the minimum pharmacokinetic/pharmacodynamic target of at least 50% of free drug time
above the minimal inhibitory concentration (MIC) (50% fT>MIC) was not achieved in 16%
of patients with an infection. Of importance, achievement of 50% fT>MIC or 100% fT>MIC
was independently associated with a higher likelihood of a positive outcome on multivariable
analysis (OR 1.02; 95% CI, 1.01–1.04 and OR 1.56; 95% CI, 1.15–2.13, respectively).
iv. These data suggest that a loading dose approach for these antibiotics is necessary for patients
with sepsis and septic shock. In addition, the impact of methods to improve the time that the
free drug concentration is above the MIC should be studied further.
d. A recent retrospective analysis revealed that over 30% of patients had a delay in the second dose of
antibiotics. Having a major delay in the second antibiotic dose was independently associated with
increased odds of hospital mortality (OR 1.61; 95% CI, 1.01–2.57) and mechanical ventilation (OR
2.44; 95% CI, 1.24–4.69).
e. Antimicrobial therapy should be evaluated on a daily basis to determine whether opportunities for
de-escalation or discontinuation exist.
i. Continued use of broad-spectrum antimicrobial therapy may cause untoward adverse effects
and promote the development of resistance.
ii. De-escalation may be clear-cut in infections in which a contributive pathogen has been
identified; in such cases, antimicrobial therapy should be reduced to the narrowest-spectrum
agent with adequate activity. However, de-escalation may be more challenging in culture-
negative sepsis.
iii. Antimicrobials are typically continued for 7–10 days, though longer courses may be
indicated in patients with a poor clinical response, those with bacteremia, or those who are
immunocompromised.

ACCP Updates in Therapeutics® 2022: Critical Care Pharmacy Preparatory Review and Recertification Course

654
Shock Syndromes I: Introduction, Vasodilatory, and Sepsis

iv. Procalcitonin and other biomarkers may be used to limit the duration of antimicrobial therapy
and are discussed in detail in the Infectious Diseases II chapter.
8. Source control measures (e.g., drainage of an abscess, debridement of infected necrotic tissue, or
removal of a potentially infected device [including intravascular access devices]), as applicable, should
be done as soon as possible (within 12 hours after the diagnosis is made).
9. Fluid therapy
a. A fluid challenge technique should be used in which fluids are administered as long as the patient
has improved clinical factors (particularly end-organ perfusion).
b. After initial resuscitation, additional fluids should be guided by frequent reassessment of
hemodynamic status. Dynamic markers (instead of static markers) of fluid responsiveness should
be used, when available.
c. Crystalloids (over colloids) are the recommended initial fluid type for an initial fluid challenge and
subsequent intravascular replacement in patients with septic shock.
i. Crystalloid solutions that approximate the electrolyte composition of plasma (“balanced”
solutions) are an attractive alternative to 0.9% sodium chloride, and either may be used for
fluid resuscitation (see section V, Agents Used to Treat Shock, for further discussion). The 2021
SSC guidelines suggest using balanced crystalloids instead of normal saline for resuscitation
in patients with sepsis and septic shock.
(a) No prospective randomized study has directly compared 0.9% sodium chloride with
balanced salt solutions for fluid resuscitation when enrollment was restricted to patients
with sepsis.
(b) A propensity-matched retrospective cohort study of medical ICU patients with sepsis
found that receipt of a balanced solution compared with 0.9% sodium chloride was
associated with a lower incidence of in-hospital mortality (19.6% vs. 22.8%, p=0.001).
Contrary to analyses of general critical care patients, this study found no difference in the
incidence of acute renal failure between groups, which leads to questions regarding the
mechanism of the detected mortality difference between groups.
(c) A systematic review and network meta-analysis of patients with sepsis suggested a lower
mortality rate in patients resuscitated with balanced solutions than in patients resuscitated
with 0.9% sodium chloride (OR 0.78, credibility interval 0.58–1.05; low confidence in
estimate of effect), but this difference was not statistically significant.
(d) A recent meta-analysis of all trials comparing balanced crystalloids with 0.9% sodium
chloride (including the aforementioned SMART study) in the patient cohort with sepsis
found that mortality was not affected by fluid choice (OR 0.98; 95% CI, 0.80–1.20).
However, major adverse kidney events (defined as the composite of death, new renal
replacement therapy, or persistent renal dysfunction in the first 30 days) were reduced
with balanced solutions (OR 0.78; 95% CI, 0.66–0.91).
d. Although not recommended for initial resuscitation, iso-oncotic (4%–5%) albumin may be
considered in some patients.
i. A prospectively defined subgroup analysis of the SAFE study evaluated the effect of albumin
4% compared with 0.9% sodium chloride in patients with sepsis and septic shock. The
unadjusted RR of death with albumin was 0.87 (95% CI, 0.74–1.02) in patients with sepsis
and 1.05 (95% CI, 0.94–1.17) in patients without sepsis (p=0.06 for heterogeneity of treatment
effect by subgroup). In a multivariable analysis that accounted for baseline factors, albumin
administration was associated with a lower mortality risk (OR 0.71; 95% CI, 0.52–0.97;
p=0.03).

ACCP Updates in Therapeutics® 2022: Critical Care Pharmacy Preparatory Review and Recertification Course

655
Shock Syndromes I: Introduction, Vasodilatory, and Sepsis

ii. A systematic review and fixed-effect meta-analysis of albumin compared with alternative fluids
for resuscitation in patients with sepsis found an association between albumin use and lower
mortality (OR 0.82; 95% CI, 0.67–1.0; p=0.047). The benefit of albumin was retained when the
analysis was restricted to crystalloids as the comparator (OR 0.78; 95% CI, 0.62–0.99; p=0.04).
These data should be interpreted with caution, however, because many of the included studies
had poor methodological quality, and when a random-effects model was used, the results for
the overall analysis were not statistically significant (OR 0.84; 95% CI, 0.69–1.02, p=0.08).
iii. These data have renewed interest in 4%–5% albumin as a resuscitation fluid for patients with
sepsis and septic shock. As such, the 2016 SSC guidelines suggest albumin as a component
of the fluid resuscitation regimen when patients require a substantial amount of crystalloids.
e. Hydroxyethyl starch solutions should not be used for fluid resuscitation.
i. Compared with patients with sepsis allocated to lactated Ringer’s solution, those allocated to
pentastarch (a hydroxyethyl starch formulation) had a significantly higher incidence of acute
renal failure (34.9% vs. 22.8%, p=0.002) and need for renal replacement therapy (31.0% vs.
18.8%, p=0.001), with no difference in 28-day mortality (26.7% vs. 24.1%, p=0.48).
ii. Compared with patients with sepsis randomized to receive Ringer’s acetate solution, patients
with sepsis randomized to receive hydroxyethyl starch had a significantly higher 90-day
mortality (51% vs. 43%, p=0.03) and need for renal replacement therapy (22% vs. 16%, p=0.04).
iii. The increased need for renal replacement therapy and lack of mortality benefit in these studies
led to the strong recommendation against the use of hydroxyethyl starch for fluid resuscitation
in patients with sepsis and septic shock in the 2016 SSC guidelines.
f. Hyperoncotic (20%–25%) albumin replacement may be beneficial in patients with septic shock.
An open-label study compared the replacement of albumin (with 20% albumin) to a goal serum
albumin concentration of 3 g/dL plus crystalloid solution administration with the administration
of crystalloid solution alone in patients with sepsis or septic shock. The albumin and crystalloid
groups did not differ in the incidence of 28-day mortality (31.8% vs. 32.0%, p=0.94), but patients
allocated to albumin had a shorter time to cessation of vasoactive agents (median 3 vs. 4 days,
p=0.007). A post hoc subgroup analysis of patients with septic shock at enrollment showed that
those randomized to albumin had a lower 90-day mortality rate (RR 0.87; 95% CI, 0.77–0.99); 90-
day mortality did not differ in patients without septic shock (RR 1.13; 95% CI, 0.92–1.39; p=0.03
for heterogeneity). Of note, this post hoc subgroup analysis was performed on the secondary end
point of 90-day mortality, and an analysis of the primary outcome of 28-day mortality was not
reported. These data suggest that albumin replacement does not improve outcomes in patients with
sepsis but that it has hemodynamic (and potentially mortality) advantages in patients with septic
shock. Hence, the role of albumin replacement in patients with septic shock warrants further study.
10. Vasoactive agents and inotropes
a. Norepinephrine is the recommended first-line vasoactive medication. A meta-analysis of
randomized trials that compared norepinephrine with dopamine for the treatment of septic shock
found a higher risk of short-term mortality (RR 1.12; 95% CI, 1.01–1.20; p=0.035) and arrhythmias
(RR 2.34; 95% CI, 1.46–3.77; p=0.001) in patients allocated to dopamine.
b. The optimal approach to using vasoactive agents and inotropes beyond the choice of initial
vasopressor is unclear, but the choice should be based on patient-specific clinical factors.
i. Potential interventions could include using norepinephrine monotherapy (with appropriate
dose escalation) or initiating an additional therapy (i.e., a second catecholamine vasopressor,
arginine vasopressin (AVP), corticosteroids, an inotrope, or a combination of these therapies).
ii. The optimal time or norepinephrine dose at which to consider additional therapies is unknown.
A dose that constitutes the failure of norepinephrine is not well defined in the literature, and
the maximal doses used by clinicians (or institutions) are variable and often subjective.

ACCP Updates in Therapeutics® 2022: Critical Care Pharmacy Preparatory Review and Recertification Course

656
Shock Syndromes I: Introduction, Vasodilatory, and Sepsis

c. Low-dose AVP (up to 0.03 units/minute) can be added to norepinephrine as the second-line
adjunctive agent rather than escalating the dose of norepinephrine in patients not at goal MAP.
i. In patients with septic shock, a relative vasopressin deficiency may exist, and patients may be
sensitive to the vasoconstrictive effects of AVP.
ii. Because of these effects, AVP has been used in patients with septic shock as both an endocrine
replacement therapy (with a fixed-dose infusion) and a vasopressor (titrated to MAP).
iii. The Vasopressin and Septic Shock Trial (VASST) compared the effects of adding AVP (0.01–
0.03 units/minute) to norepinephrine with using norepinephrine monotherapy. Patients were
initiated on the study drug an average of 12 hours after meeting the trial’s inclusion criteria
and when norepinephrine was at a mean dose of 20.7 mcg/minute. Overall, the study found
no difference in 28-day mortality between treatment arms (35.4% vs. 39.3%, p=0.26), but
norepinephrine requirements were significantly lower during the first 4 study days in the patients
allocated to AVP (p<0.001). Adverse effects did not differ between groups. In an a priori–
defined subgroup analysis of patients on the basis of shock severity (less severe shock defined
as baseline norepinephrine dose 5–14 mcg/minute) with randomization stratified according to
this covariate, 28-day mortality in the less severe shock stratum (defined as norepinephrine-
equivalent doses less than 15 mcg/minute) was lower in patients randomized to AVP plus
norepinephrine (26.5% vs. 35.7%, p=0.05), with no difference between groups in the more
severe shock stratum (44.0% vs. 42.5%, p=0.76, stratum interaction p=0.10).
iv. In the VANISH trial, which compared first-line vasopressin (up to 0.06 units/minute) with
norepinephrine, there was no difference between groups in the incidence of kidney failure in
survivors (absolute difference -2.3%; 95% CI, -13.0 to 8.5) or the number of kidney failure–free
days in nonsurvivors (absolute difference -4 days; 95% CI, -11 to 5). Patients were initiated on
the study drug within an average of 3.5 hours from shock onset and at a mean norepinephrine
dose of 0.16 mcg/kg/minute. Patients were also randomized to hydrocortisone or placebo (in
a 2 × 2 factorial design), but there was no significant interaction between vasopressin and
hydrocortisone (interaction p=0.98 for 28-day mortality). Although the study intended to
evaluate vasopressin as a first-line agent, 85% of patients were receiving open-label vasopressors
before randomization.
v. One consistent finding with the use of AVP in clinical trials is the reduction in norepinephrine
dose requirements after its initiation, making it an attractive norepinephrine- and catecholamine-
sparing agent.
vi. Findings regarding the effects of AVP (and its analogs) on mortality have been conflicting
in meta-analyses. One meta-analysis found a decreased mortality risk in patients with septic
shock allocated to AVP (RR 0.87; 95% CI, 0.75–1.0; p=0.05), whereas another study found no
significant benefit with AVP (RR 0.91; 95% CI, 0.79–1.05; p=0.21). Both meta-analyses found
a significant benefit of AVP in decreasing norepinephrine doses. In an updated meta-analysis
performed for the 2016 SSC guidelines, mortality did not differ between norepinephrine and
vasopressin (RR 0.89; 95% CI, 0.79–1.00). A recent meta-analysis (which included the VANISH
study) corroborated this and found no significant effect on mortality with vasopressin use (RR
0.98; 95% CI, 0.86–1.12).
vii. The 2021 SSC guidelines note that the threshold for adding vasopressin is unclear, but it is
reasonable to consider initiation when the dose of norepinephrine is 0.25–0.5 mcg/kg/minute.
viii. High-dose vasopressin (doses above 0.03–0.04 units/minute) should be reserved for salvage
therapy and, if used, should be used cautiously.

ACCP Updates in Therapeutics® 2022: Critical Care Pharmacy Preparatory Review and Recertification Course

657
Shock Syndromes I: Introduction, Vasodilatory, and Sepsis

(a) A study of patients with vasodilatory shock (about 50% with septic shock) requiring
high-dose norepinephrine (greater than 0.6 mcg/kg/minute) randomized patients to AVP
0.033 units/minute versus AVP 0.067 units/minute. The study was designed to evaluate
hemodynamic changes, not mortality. Patients randomized to the higher dose had a greater
reduction in norepinephrine requirements than did those allocated to the lower dose
(p=0.006). High-dose AVP did not lead to a significantly lower cardiac index than low-
dose AVP (as might be expected with this pure vasoconstrictor), but this finding is difficult
to interpret because most patients were receiving concomitant inotropes, and the study was
likely underpowered to assess this outcome.
(b) High-dose vasopressin is best reserved for patients with septic shock requiring high
norepinephrine doses (greater than 0.6 mcg/kg/minute) with a high CO.
ix. Despite the unclear benefits of AVP on mortality, it offers an alternative mechanism to
catecholamines for vasoconstriction, and low-dose AVP is commonly used in practice (often as
the second vasoactive medication).
d. The SSC guidelines suggest adding epinephrine in patients with inadequate MAP levels despite
norepinephrine and vasopressin.
i. A randomized trial of patients with septic shock compared epinephrine with norepinephrine
with or without dobutamine. The groups did not differ in 28-day mortality (40% vs. 34%,
p=0.31), but patients allocated to epinephrine had significantly higher lactate concentrations on
day 1 (p=0.003) and lower arterial pH values on each of the first 4 study days. Caution should
be used in concluding that mortality does not differ between epinephrine and norepinephrine
because the study was powered to detect a 20% absolute difference in mortality rates, and a
smaller difference between agents cannot be ruled out.
ii. The benefit of adding epinephrine to norepinephrine (whether this approach has a
norepinephrine-sparing effect or whether it is best used in norepinephrine failure) is unclear.
iii. Because norepinephrine may cause tachycardia or tachyarrhythmias (often the impetus to
limit doses), alternative vasopressors may be added. If catecholamine vasopressors with β1-
adrenergic properties (e.g., epinephrine) are added to augment MAP in patients receiving
norepinephrine, they are unlikely to prevent tachyarrhythmias and will likely increase the risk
of this adverse effect.
iv. In addition, epinephrine may preclude the use of lactate clearance as an initial resuscitation
goal because it increases lactate concentrations through increased production by aerobic
glycolysis (by stimulating skeletal muscle β2-adrenergic receptors), an effect that likely wanes
with continued epinephrine administration.
v. It seems most prudent to use epinephrine in patients receiving norepinephrine with a low MAP
who require CO augmentation.
e. Phenylephrine is no longer recommended by the SSC guidelines for use in patients with septic
shock because of limited clinical trial data.
i. Because of its afterload augmentation effects without β1-adrenergic properties, phenylephrine
may theoretically decrease SV and CO. As such, it is not recommended as a first-line vasopressor
in patients with septic shock who have decreased CO because of inadequate preload.
ii. A small study (n=32) that randomized patients with septic shock to phenylephrine or
norepinephrine as first-line therapy found no difference in hemodynamic measures (including
CO) between agents in the first 12 hours of therapy. This finding is likely because of the
inotropic effect of myocardial α1-adrenergic receptor augmentation by phenylephrine. These
data are in contrast to the theoretical concerns with phenylephrine and warrant further study.

ACCP Updates in Therapeutics® 2022: Critical Care Pharmacy Preparatory Review and Recertification Course

658
Shock Syndromes I: Introduction, Vasodilatory, and Sepsis

iii. During a shortage of norepinephrine in 2011, phenylephrine was used as an alternative for
patients with septic shock. Compared with normal use, in-hospital mortality was higher during
the norepinephrine shortage (absolute risk increase of 3.7%; 95% CI, 1.5%–6.0%; p=0.03).
Because phenylephrine was the most commonly used vasoactive agent during the shortage
period, phenylephrine should be used cautiously in patients with circulatory shock.
iv. Because phenylephrine is a pure α1-adrenergic agent, it is attractive for patients who develop
a tachyarrhythmia while receiving norepinephrine. Although phenylephrine has been used in
clinical practice for many years for this indication, no clinical trial data support this practice.
f. Dopamine is best used in a select patient population, including those with a low risk of
tachyarrhythmias (which is difficult to predict) or those with bradycardia-induced hypotension.
Low-dose (“renal dose” 2 mcg/kg/minute) dopamine should not be used to improve renal blood
flow and urinary output (renal protection).
g. Preliminary data suggest that midodrine helps decrease vasopressor duration in patients in the
convalescent phase of septic shock.
i. MIDAS was a multicenter, randomized, double-blind, placebo-controlled trial of patients with
hypotension admitted to an ICU requiring a single-agent vasopressor who could not be liberated
from vasopressors for at least 24 hours. Patients were randomized to receive midodrine 20 mg
every 8 hours or placebo. There was no difference in time to vasopressor discontinuation in the
136 included patients (23.5 hours of midodrine vs. 22.5 hours of placebo). Of note, this study
was not specific for patients with septic shock.
ii. Further, larger studies are needed before midodrine is used in routine practice.
iii. When midodrine is used for this indication, care should be taken to ensure appropriate
discontinuation after resolution of shock. Reports have noted inappropriate continuation of
midodrine that may have been found in the outpatient setting.
h. Dobutamine may be indicated in patients with evidence or suggestion of myocardial dysfunction.
i. A low CO is common in early septic shock but is often corrected with fluid resuscitation alone.
ii. Myocardial dysfunction may persist despite fluid resuscitation, with 39% of patients having
left ventricular hypokinesia on ICU admission and an additional 21% of patients developing
hypokinesia 24–48 hours after ICU admission in one series.
iii. If myocardial dysfunction leads to significantly impaired Do2, an inotrope such as dobutamine
(up to 20 mcg/kg/minute) may be indicated to improve CO.
iv. Dobutamine should not be used to achieve a predetermined supranormal cardiac index or
Scvo2.
i. As noted previously, levosimendan, compared with placebo, did not lead to a significant difference
in the mean SOFA score in patients with sepsis. In a meta-analysis performed by the SSC,
levosimendan and dobutamine did not differ with respect to mortality (RR 0.83; 95% CI, 0.66–
1.05). Given these data, dobutamine is recommended as the preferred inotrope in patients with
septic shock.
j. The 2021 SSC guidelines do not recommend angiotensin II as a first-line agent because of the low
quality of evidence and lack of clinical experience in sepsis; however, they note that it may play a
role as an adjunctive vasopressor therapy.

ACCP Updates in Therapeutics® 2022: Critical Care Pharmacy Preparatory Review and Recertification Course

659
Shock Syndromes I: Introduction, Vasodilatory, and Sepsis

Patient Case

7. A 55-year-old man presents to the medical ICU with presumed urosepsis. His medical history is significant
for congestive heart failure with a baseline ejection fraction of 20%. In the medical ICU, his vital signs are
as follows: blood pressure 69/45 mm Hg, heart rate 84 beats/minute, respiratory rate 32 breaths/minute,
and temperature 100.6°F (38.1°C). Blood cultures are obtained, and piperacillin/tazobactam is initiated. A
central venous catheter shows CVP 18 mm Hg and Scvo2 70%. Which is the most appropriate initial vaso-
pressor for this patient?
A. Norepinephrine.
B. Vasopressin.
C. Dobutamine.
D. Epinephrine.

11. Corticosteroids should only be used in patients with septic shock who do not achieve resuscitation goals
despite fluid administration and vasopressors.
a. Corticosteroids are an attractive option for patients with septic shock because of their anti-
inflammatory effects (through inhibition of nuclear factor κB) and ability to improve blood pressure
response to catecholamines (through up-regulation of adrenergic receptors and potentiation of
vasoconstrictor actions).
b. Use of corticosteroids for patients with septic shock has been a source of controversy for years.
c. In studies of short courses of high-dose corticosteroids (typically at doses of 30 mg/kg of
methylprednisolone or greater) in patients with sepsis and septic shock, corticosteroids did not
improve patient outcomes.
d. Four large studies have evaluated the effect of corticosteroids in patients with septic shock, with
conflicting findings on mortality.
i. In a French trial of patients with septic shock and vasopressor-unresponsive shock (i.e.,
inability to increase SBP above 90 mm Hg for 1 hour despite fluids and vasopressors), patients
randomized to low-dose hydrocortisone and fludrocortisone had improved survival in a time-
to-event analysis (HR 0.71; 95% CI, 0.53–0.97). The mortality benefit with corticosteroids was
limited to patients unable to increase their cortisol concentration by more than 9 mcg/dL in
response to ACTH administration (nonresponders).
ii. In a larger, multicenter Corticosteroid Therapy of Septic Shock (CORTICUS) study, which
had less-stringent inclusion criteria than the earlier-noted French trial, hydrocortisone
administration was not associated with improved survival in ACTH nonresponders (28-day
mortality 39.2% vs. 36.1%, p=0.69).
iii. The Adjunctive Corticosteroid Treatment in Critically Ill Patients with Septic Shock
(ADRENAL) trial randomized 3800 patients to hydrocortisone or placebo. 90-day mortality
did not differ between the hydrocortisone or placebo groups (27.9% vs. 28.8%, OR 0.95; 95%
CI, 0.82–1.10; p=0.50). However, patients randomized to hydrocortisone had quicker time
to shock resolution (3 vs. 4 days; p<0.001). A subgroup analysis of patients who received
hydrocortisone on the basis of time from shock onset to randomization showed improvement
in death at 90 days in patients who received hydrocortisone within 6–12 hours (OR 0.71; 95%
CI, 0.54–0.94).

ACCP Updates in Therapeutics® 2022: Critical Care Pharmacy Preparatory Review and Recertification Course

660
Shock Syndromes I: Introduction, Vasodilatory, and Sepsis

iv. The Activated Protein C and Corticosteroids for Human Septic Shock (APROCCHSS) trial
randomized 1241 patients to hydrocortisone and fludrocortisone or placebo within 24 hours of
shock onset. Patients were also randomized to drotrecogin alfa (activated) in a 2 x 2 factorial
design, but after the drug was withdrawn from the market, the study was continued with only
the corticosteroid randomization. Patients in the hydrocortisone and fludrocortisone group
had lower 90-day mortality than those receiving placebo (43.0% vs. 49.1%; RR 0.88; 95% CI,
0.78–0.99; p=0.03). In addition, patients randomized to the corticosteroid arm had increased
vasopressor-free days (16 vs. 11 days; p<0.001) and organ failure–free days (14 vs. 13 days;
p=0.003).
v. These studies differed in the patient populations evaluated, with patients more severely ill
demonstrating a mortality benefit from corticosteroids. When this question was specifically
evaluated in the ADRENAL trial, there was no heterogeneity of effect by subgroup, suggesting
severity of illness did not influence the outcome.
vi. Other notable differences between studies that may account for the disparate results are the
drug regimens used and the timing of therapy initiation.
e. Two meta-analyses that included these four studies found that corticosteroids had no mortality
benefit (RR 0.96; 95% CI, 0.91–1.02 in one meta-analysis and RR 0.93; 95% CI 0.84–1.03 in another
meta-analysis).
f. In meta-analyses, corticosteroid administration has consistently been associated with improvements
in shock reversal and a higher incidence of adverse effects (particularly hypernatremia and
hyperglycemia).
g. Guidelines from the Society of Critical Care Medicine and the European Society of Intensive
Care Medicine suggest using corticosteroids in patients with septic shock that is not responsive
to fluids and moderate- to high-dose vasopressor therapy. These guidelines further state that if
hydrocortisone is indicated, a dose of less than 400 mg/day for 3 days or more at full dose is
suggested.
h. The SSC guidelines suggest using hydrocortisone in patients with septic shock and an ongoing
requirement for vasopressor therapy at a dose of 200 mg/day given as either 50 mg intravenously
every 6 hours or a continuous infusion. The guidelines note the suggestion that hydrocortisone be
initiated when norepinephrine or epinephrine doses exceed 0.25 mcg/kg/minute at least 4 hours
after initiation.
i. Although two of the trials mentioned earlier used fludrocortisone, the SSC guidelines recommend
hydrocortisone alone. This is likely because of the COIITSS study, a 2 x 2 factorial trial that
compared hydrocortisone in combination with fludrocortisone with hydrocortisone alone. No
difference in the primary outcome of in-hospital death (42.9% hydrocortisone and fludrocortisone
vs. 45.8% hydrocortisone alone) or other clinical outcomes was detected between treatment arms.
j. Because of immunoassay imprecision and the inconsistent benefit of identifying patient response,
the ACTH test should not be used to identify patients for corticosteroid administration.

ACCP Updates in Therapeutics® 2022: Critical Care Pharmacy Preparatory Review and Recertification Course

661
Shock Syndromes I: Introduction, Vasodilatory, and Sepsis

Patient Case

8. A 62-year-old woman was admitted to the surgical ICU with presumed aspiration pneumonia. The patient
has a history of insulin-dependent diabetes and hyperthyroidism. On admission, she was intubated and
required aggressive resuscitation and vasopressor therapy. Currently, the patient is receiving norepineph-
rine 8 mcg/minute and has the following vital signs and laboratory values: blood pressure 100/60 mm Hg,
heart rate 90 beats/minute, and CVP 14 mm Hg; her Scvo2 is 55%. Which is best regarding the patient’s
steroid therapy?
A. Administer hydrocortisone 50 mg every 6 hours.
B. Administer hydrocortisone 8.3 mg/hour as a continuous infusion.
C. Perform the ACTH stimulation test.
D. No steroids are necessary right now.

12. Supportive therapies


a. Vitamin C, hydrocortisone, and thiamine
i. A retrospective before-after study evaluated the effect of the combination of intravenous
vitamin C (1.5 g every 6 hours for 4 days or until ICU discharge), hydrocortisone (50 mg every
6 hours for 7 days or until ICU discharge, followed by a taper over 3 days), and intravenous
thiamine (200 mg every 12 hours for 4 days or until ICU discharge) for patients with sepsis.
Patients receiving this medication combination had a lower propensity-adjusted odds of
mortality (OR 0.13; 95% CI, 0.04–0.48; p=0.002). This study is limited by the before-after
design and a control group mortality rate (8.5%) lower than in contemporary sepsis studies
(leading to questions about the study population).
ii. Several subsequent randomized controlled trials have evaluated this clinical question,
including the VITAMINS, VICTAS, and ACTS trials, which have all shown no benefit with
the use of vitamin C, hydrocortisone, and thiamine.
iii. The 2021 SSC guidelines suggest against the use of intravenous vitamin C in patients with
sepsis and septic shock. They do not discuss the role of thiamine or the combination of vitamin
C, hydrocortisone, and thiamine.
b. Intravenous immunoglobulins should not be used in patients with sepsis and septic shock. In a
randomized study of patients with sepsis, 28-day mortality was no different between patients
allocated to intravenous immunoglobulin G and placebo (39.3% vs. 37.3%, p=0.67).
c. A transfusion threshold of 7 g/dL or less is appropriate for patients with septic shock unless
extenuating circumstances exist (i.e., myocardial ischemia, severe hypoxemia, or acute hemorrhage).
i. A multicenter study compared two different transfusion thresholds in patients with septic
shock. The study enrolled patients with septic shock and a hemoglobin concentration below 9
g/dL to receive PRBC transfusion if their hemoglobin was 7 g/dL or less (lower threshold) or
9 g/dL or less during their ICU stay.
ii. As expected, patients allocated to the lower-threshold group less commonly received PRBC
transfusion and had lower hemoglobin concentrations.
iii. Treatment arms did not differ in 90-day mortality (43.0% in the lower threshold group vs.
36.6% in the higher-threshold group, p=0.44). Patients in the lower-threshold group had no
higher incidence of ischemic events in the ICU (7.2% vs. 8.0%, p=0.64), and outcomes did not
differ between groups in the subgroup of patients with chronic cardiovascular disease (RR
1.08 [95% CI, 0.75–1.40]; p=0.25 for heterogeneity of treatment effect by subgroup; only 14%
of the overall population had chronic cardiovascular disease).
iv. Although the study did not specifically evaluate the practice of transfusing blood in patients
with evidence of hypoperfusion (low Scvo2 or elevated lactate concentrations) associated with
a low hemoglobin concentration, the median Scvo2 at baseline was below 70% and lactate
concentrations were above 2 mmol/L in both groups, suggesting that most patients had
evidence of hypoperfusion.

ACCP Updates in Therapeutics® 2022: Critical Care Pharmacy Preparatory Review and Recertification Course

662
Shock Syndromes I: Introduction, Vasodilatory, and Sepsis

v. These data suggest that use of a hemoglobin transfusion threshold of 7 g/dL or lower is safe for
most patients, including those with sepsis and septic shock.
13. Continued care of the patient with septic shock (resuscitation end points beyond first 6 hours): See
section IV, Resuscitation Parameters and End Points, for an additional discussion of this topic.
a. Uncertainty persists regarding hemodynamic targets beyond the first 6 hours of presentation.
i. Therapy should be directed toward maintaining adequate end-organ perfusion and
normalization of lactate concentrations, but the specific method(s) to achieve these goals will
be patient-specific.
ii. A strategy of systematically increasing CO to predefined “supranormal” values was not
associated with a mortality benefit; hence, it is not recommended.
b. Care should be used to avoid giving excessive fluids.
i. In a retrospective analysis of data from a randomized controlled trial of patients with septic
shock, patients in the highest quartile of fluid balance had a significantly higher mortality rate
than patients in the lowest two quartiles. This association was present when fluid balance was
evaluated at both 12 hours and 4 days after study enrollment. In the same analysis, a CVP
greater than 12 mm Hg conferred a higher risk of mortality than a lower CVP.
ii. Fluid administration can be limited by only giving fluid in patients who are proven or predicted
to respond to fluid. This is best accomplished using dynamic markers of fluid responsiveness.
c. Resuscitation targeted to improving microcirculatory perfusion is a potential new therapeutic
frontier.
i. Studies have shown that the microcirculation is often altered in patients with sepsis, persistent
microvascular alterations are associated with multisystem organ failure and death, alterations
are more severe in non-survivors than in survivors, and improvements in microcirculatory
blood flow correspond with improved patient outcomes.
ii. Impaired microcirculatory perfusion may at least partly explain why patients have elevated
lactate concentrations despite achievement of (macrovascular) hemodynamic goals.
iii. Several strategies to improve microcirculatory perfusion have been investigated.
(a) In a nonrandomized trial, fluid resuscitation improved microvascular perfusion in early,
but not late, sepsis. Another study found that microcirculatory perfusion improved with
PLR or fluid administration in patients who were fluid responsive.
(b) Use of norepinephrine to increase the MAP above 65 mm Hg has not been associated with
correction of impaired microcirculatory perfusion.
(c) In a randomized trial of patients who underwent quantitative resuscitation, adding
nitroglycerin was not associated with improved microcirculatory blood flow. In-hospital
mortality was numerically higher in patients allocated to nitroglycerin than in placebo,
but this difference was not statistically significant (34.3% vs. 14.2%, p=0.09).
(d) A prospective open-label study evaluated the effects of dobutamine 5 mcg/kg/minute
initiated after quantitative resuscitation (but within the first 48 hours of presentation).
Dobutamine significantly improved microcirculatory perfusion compared with baseline.
Of interest, the beneficial effects of dobutamine were unrelated to changes in cardiac
index or blood pressure.
(e) Initiation of inhaled nitric oxide after quantitative resuscitation did not improve
microcirculatory flow or lactate clearance.
(f) In summary, fluid resuscitation and dobutamine initiation appear to improve
microcirculatory perfusion. It is still unknown whether using these therapies to improve
microcirculatory perfusion can improve patient outcomes such as mortality.

ACCP Updates in Therapeutics® 2022: Critical Care Pharmacy Preparatory Review and Recertification Course

663
Shock Syndromes I: Introduction, Vasodilatory, and Sepsis

REFERENCES

Introduction of Intensive Care Medicine. Intensive Care Med


1. Huang YC. Monitoring oxygen delivery in the crit- 2014;40:1795-815.
ically ill. Chest 2005;128:554S-60S. 5. De Backer D, Creteur J, Dubois MJ, et al. The
2. Mascha EJ, Yang D, Weiss S, et al. Intraoperative effects of dobutamine on microcirculatory
mean arterial pressure variability and 30-day alterations in patients with septic shock are inde-
mortality in patients having noncardiac surgery. pendent of its systemic effects. Crit Care Med
Anesthesiology 2015;123:79-91. 2006;34:403-8.
3. Ronco JJ, Fenwick JC, Tweeddale MG, et al. 6. De Backer D, Donadello K, Sakr Y, et al.
Identification of the critical oxygen delivery for Microcirculatory alterations in patients with severe
anaerobic metabolism in critically ill septic and sepsis: impact of time of assessment and relation-
nonseptic humans. JAMA 1993;270:1724-30. ship with outcome. Crit Care Med 2013;41:791-9.
4. Salmasi V, Maheshwari K, Yang D, et al. 7. De Backer D, Hollenberg S, Boerma C, et al. How
Relationship between intraoperative hypotension, to evaluate the microcirculation: report of a round
defined by either reduction from baseline or abso- table conference. Crit Care 2007;11:R101.
lute thresholds, and acute kidney and myocardial 8. Gutierrez G, Palizas F, Doglio G, et al. Gastric
injury after noncardiac surgery: a retrospective intramucosal pH as a therapeutic index of tis-
cohort analysis. Anesthesiology 2017;126:47-5. sue oxygenation in critically ill patients. Lancet
5. Sato Y, Weil MH, Tang W. Tissue hypercarbic 1992;339:195-9.
acidosis as a marker of acute circulatory failure 9. Hollenberg SM. Think locally: evaluation of the
(shock). Chest 1998;114:263-74. microcirculation in sepsis. Intensive Care Med
6. Vincent JL. Understanding cardiac output. Crit 2010;36:1807-9.
Care 2008;12:174. 10. Pope JV, Jones AE, Gaieski DF, et al. Multicenter
7. Vincent JL, De Backer D. Circulatory shock. N study of central venous oxygen saturation
Engl J Med 2013;369:1726-34. (ScvO(2)) as a predictor of mortality in patients
8. Walsh M, Devereaux PJ, Garg AX, et al. with sepsis. Ann Emerg Med 2010;55:40-46.e1.
Relationship between intraoperative mean arterial 11. Sakr Y, Dubois MJ, De Backer D, et al. Persistent
pressure and clinical outcomes after noncardiac microcirculatory alterations are associated with
surgery: toward an empirical definition of hypo- organ failure and death in patients with septic
tension. Anesthesiology 2013;119:507-15. shock. Crit Care Med 2004;32:1825-31.
9. Weil MH, Shubin H. Proposed reclassification of
shock states with special reference to distributive Differentiation of Shock States
defects. Adv Exp Med Biol 1971;23:13-23. 1. Cecconi M, De Backer D, Antonelli M, et al.
Consensus on circulatory shock and hemodynamic
Monitoring Techniques monitoring. Task force of the European Society
1. Alhashemi JA, Cecconi M, Hofer CK. Cardiac of Intensive Care Medicine. Intensive Care Med
output monitoring: an integrative perspective. Crit 2014;40:1795-815.
Care 2011;15:214. 2. Dellinger RP. Cardiovascular management of sep-
2. Bloos F, Reinhart K. Venous oximetry. Intensive tic shock. Crit Care Med 2003;31:946-55.
Care Med 2005;31:911-3. 3. Vincent JL, De Backer D. Circulatory shock. N
3. Boerma EC, van der Voort PH, Spronk PE, et Engl J Med 2013;369:1726-34.
al. Relationship between sublingual and intesti- 4. Weil MH, Shubin H. Proposed reclassification of
nal microcirculatory perfusion in patients with shock states with special reference to distributive
abdominal sepsis. Crit Care Med 2007;35:1055-60. defects. Adv Exp Med Biol 1971;23:13-23.
4. Cecconi M, De Backer D, Antonelli M, et al.
Consensus on circulatory shock and hemodynamic
monitoring. Task force of the European Society

ACCP Updates in Therapeutics® 2022: Critical Care Pharmacy Preparatory Review and Recertification Course

664
Shock Syndromes I: Introduction, Vasodilatory, and Sepsis

Resuscitation Parameters and End Points 12. Marik PE, Cavallazzi R, Vasu T, et al. Dynamic
1. Chen C, Kollef MH. Targeted fluid minimization changes in arterial waveform derived variables
following initial resuscitation in septic shock: a and fluid responsiveness in mechanically venti-
pilot study. Chest 2015;148:1462-9. lated patients: a systematic review of the literature.
2. Cherpanath TG, Hirsch A, Geerts BF, et al. Crit Care Med 2009;37:2642-7.
Predicting fluid responsiveness by passive leg rais- 13. Marik PE, Monnet X, Teboul JL. Hemodynamic
ing: a systematic review and meta-analysis of 23 parameters to guide fluid therapy. Ann Intensive
clinical trials. Crit Care Med 2016;44:981-91. Care 2011;1:1.
3. De Backer D, Ospina-Tascon G, Salgado D, et al. 14. Monnet X, Teboul JL. Passive leg raising. Intensive
Monitoring the microcirculation in the critically Care Med 2008;3:659-63.
ill patient: current methods and future approaches. 15. Osman D, Ridel C, Ray P, et al. Cardiac filling
Intensive Care Med 2010;36:1813-25. pressures are not appropriate to predict hemody-
4. Dellinger RP, Levy MM, Rhodes A, et al. namic response to volume challenge. Crit Care
Surviving Sepsis Campaign: international guide- Med 2007;35:64-8.
lines for management of severe sepsis and septic 16. Puskarich MA, Trzeciak S, Shapiro NI, et al.
shock: 2012. Crit Care Med 2013;41:580-637. Prognostic value and agreement of achieving
5. de Oliveira OH, Freitas FG, Ladeira RT. lactate clearance or central venous oxygen satura-
Comparison between respiratory changes in the tion goals during early sepsis resuscitation. Acad
inferior vena cava diameter and pulse pressure Emerg Med 2012;19:252-8.
variation to predict fluid responsiveness in postop- 17. Teboul JL, Hamzaoui O, Monnet X. Svo2 to monitor
erative patients. J Crit Care 2016;34:46-9. resuscitation of septic patients: let’s just under-
6. Gattinoni L, Brazzi L, Pelosi P, et al. A trial of stand the basic physiology. Crit Care 2011;15:1005.
goal-oriented hemodynamic therapy in critically 18. Vignon P, Repesse X, Begot E, et al. Comparison
ill patients. Svo2 Collaborative Group. N Engl J of echocardiographic indices used to predict fluid
Med 1995;333:1025-32. responsiveness in ventilated patients. Am J Respir
7. Jansen TC, van Bommel J, Schoonderbeek FJ, et Crit Care Med 2017;195:1022-32.
al. Early lactate-guided therapy in intensive care 19. Vincent JL, Weil MH. Fluid challenge revisited.
unit patients: a multicenter, open-label, random- Crit Care Med 2006;34:1333-7.
ized controlled trial. Am J Respir Crit Care Med 20. Walley KR. Use of central venous oxygen satura-
2010;182:752-61. tion to guide therapy. Am J Respir Crit Care Med
8. Jones AE, Shapiro NI, Trzeciak S, et al. Lactate 2011;184:514-20.
clearance vs central venous oxygen saturation as 21. Zhang Z, Ni H, Qian Z. Effectiveness of treatment
goals of early sepsis therapy: a randomized clinical based on PiCCO parameters in patients with septic
trial. JAMA 2010;303:739-46. shock and/or acute respiratory distress syndrome:
9. Mahjoub Y, Touzeau J, Airapetian N, et al. The a randomized controlled trial. Intensive Care Med
passive leg-raising maneuver cannot accurately 2015;41:444-51.
predict fluid responsiveness in patients with
intra-abdominal hypertension. Crit Care Med Agents Used to Treat Shock – Fluids and Vasoactive
2010;38:1824-9. Agents
10. Marik PE, Baram M, Vahid B. Does central venous 1. Annane D, Siami S, Jaber S, et al. Effects of fluid
pressure predict fluid responsiveness? A system- resuscitation with colloids vs crystalloids on
atic review of the literature and the tale of seven mortality in critically ill patients presenting with
mares. Chest 2008;134:172-8. hypovolemic shock: the CRISTAL randomized
11. Marik PE, Cavallazzi R. Does the central venous trial. JAMA 2013;310:1809-17.
pressure predict fluid responsiveness? An updated 2. Antonucci E, Gleeson PJ, Annoni F, et al.
meta-analysis and a plea for some common sense. Angiotensin II in refractory septic shock. Shock
Crit Care Med 2013;41:1774-81. 2017;47:560-6.

ACCP Updates in Therapeutics® 2022: Critical Care Pharmacy Preparatory Review and Recertification Course

665
Shock Syndromes I: Introduction, Vasodilatory, and Sepsis

3. Bangash MN, Kong ML, Pearse RM. Use of 19. Vincent JL, Weil MH. Fluid challenge revisited.
inotropes and vasopressor agents in critically ill Crit Care Med 2006;34:1333-7.
patients. Br J Pharmacol 2012;165:2015-33. 20. Wieruszerski PM, Witter ED, Kashani KB, et
4. Chawla LS, Busse L, Brasha-Mitchell E, et al. al. Angiotensin II infusion for shock: a mul-
Intravenous angiotensin II for the treatment of ticenter study of postmarketing use. Chest
high-output shock (ATHOS trial): a pilot study. 2021;159:596-605.
Crit Care 2014;18:534. 21. Young P, Bailey M, Beasley R, et al. Effect of a
5. De Backer D, Biston P, Devriendt J, et al. buffered crystalloid solution vs saline on acute
Comparison of dopamine and norepinephrine in the kidney injury among patients in the intensive care
treatment of shock. N Engl J Med 2010;362:779-89. unit: the SPLIT randomized clinical trial. JAMA
6. Finfer S, Bellomo R, Boyce N, et al. A comparison 2015;314:1701-10.
of albumin and saline for fluid resuscitation in the 22. Yunos NM, Bellomo R, Hegarty C, et al.
intensive care unit. N Engl J Med 2004;350:2247-56. Association between a chloride-liberal vs chlo-
7. Gamper G, Havel C, Arrich J, et al. Vasopressors ride restrictive intravenous fluid administration
for hypotensive shock. Cochrane Database Syst strategy and kidney injury in critically ill adults.
Rev 2016;2:CD003709. JAMA 2012;308:1566-72.
8. Gordon AC, Perkins GD, Singer M, et al. 23. Yunos NM, Bellomo R, Story D, et al. Bench-to-
Levosimendan for the prevention of acute bedside review: chloride in critical illness. Crit
organ dysfunction in sepsis. N Engl J Med Care 2010;14:226.
2016;375:1638-48. 24. Zampieri FG, Machado FR, Biondi RS, et al.
9. Guidet B, Soni N, Della Roca G, et al. A balanced Effect of intravenous fluid treatment with a bal-
view of balanced solutions. Crit Care 2010;14:325. anced solution vs 0.9% saline solution on mortality
10. Hammond NE, Taylor C, Saxena M, et al. in critically ill patients: the BaSICS randomized
Resuscitation fluid use in Australian and New clinical trial. JAMA 2021;326:1-12.
Zealand intensive care units between 2007 and 25. Zampieri FG, Ranzani OT, Azevedo LC, et al.
2013. Intensive Care Med 2015;41:1611-9. Lactated Ringer is associated with reduced mor-
11. Hollenberg SM. Vasoactive drugs in circulatory tality and less acute kidney injury in critically ill
shock. Am J Respir Crit Care Med 2011;183:847-55. patients: a retrospective cohort analysis. Crit Care
12. Khanna A, English SW, Wang XS, et al. Med 2016;44:2163-70.
Angiotensin II for the treatment of vasodilatory 26. Zarychanski R, Abou-Setta AM, Turgeon AF,
shock. N Engl J Med 2017;377:419-30. et al. Association of hydroxyethyl starch admin-
13. Landoni G, Lominvorotov VV, Alvaro G, et al. istration with mortality and acute kidney injury
Levosimendan for hemodynamic support after in critically ill patients requiring volume resus-
cardiac surgery. N Engl J Med 2017;376:2021-31. citation: a systematic review and meta-analysis.
14. Myburgh JA, Finfer S, Bellomo R, et al. JAMA 2013;309:678-88.
Hydroxyethyl starch or saline for fluid resuscitation
in intensive care. N Engl J Med 2012;367:1901-11. Vasodilatory and Distributive Shock
15. Myburgh JA, Higgins A, Jovanovska A, et al. 1. Angus DC, van der Poll T. Severe sepsis and septic
A comparison of epinephrine and norepineph- shock. N Engl J Med 2013;369:840-51.
rine in critically ill patients. Intensive Care Med 2. Campbell RL, Li JTC, Nicklas RA et al. Emergency
2008;34:2226-34. department diagnosis and treatment of anaphy-
16. Myburgh JA, Mythen MG. Resuscitation fluids. N laxis: a practice parameter. Ann Allergy Asthma
Engl J Med 2013;369:1243-51. Immunol 2014;113:599-608.
17. Semler MH, Self WH, Wanderer JP, et al. Balanced 3. Landry DW, Oliver JA. The pathogenesis of vaso-
crystalloids versus saline in critically ill adults. N dilatory shock. N Engl J Med 2001;345:588-95.
Engl J Med 2018;378:829-39. 4. Longo DL. Acute spinal cord compression. N Engl
18. Vincent JL, De Backer D. Circulatory shock. N J Med 2017;376:1358-69.
Engl J Med 2013;369:1726-34. 5. Sampson HA, Munoz-Furlong A, Bock SA, et al.
Symposium on the definition and management

ACCP Updates in Therapeutics® 2022: Critical Care Pharmacy Preparatory Review and Recertification Course

666
Shock Syndromes I: Introduction, Vasodilatory, and Sepsis

of anaphylaxis: summary report. J Allergy Clin 9. Bellomo R, Chapman M, Finfer S, et al. Low-dose
Immunol 2005;115:584-91. dopamine in patients with early renal dysfunction:
6. Simons FER, Ebisawa M, Sanchez-Borges M, a placebo-controlled randomised trial. Australian
et al. 2015 update of the evidence base: World and New Zealand Intensive Care Society (ANZICS)
Allergy Organization anaphylaxis guidelines. Clinical Trials Group. Lancet 2000;356:2139-43.
World Allergy Organization J 2015;8:32. 10. Boonen E, Vervenne H, Meersseman P, et al.
7. Walters BC, Hadley MN, Hurlbert RJ, et al. Reduced cortisol metabolism during critical ill-
Guidelines for the management of acute cervi- ness. N Engl J Med 2013;368:1477-88.
cal spine and spinal cord injuries: 2013 update. 11. Boyd JH, Forbes J, Nakada TA, et al. Fluid resus-
Neurosurgery 2013;60(suppl 1):82-91. citation in septic shock: a positive fluid balance
8. Wood GC, Boucher AB, Johnson JL, et al. and elevated central venous pressure are asso-
Effectiveness of pseudoephedrine as adjunctive ciated with increased mortality. Crit Care Med
therapy for neurogenic shock after acute spi- 2011;39:259-65.
nal cord injury: a case series. Pharmacotherapy 12. Brunkhorst FM, Engel C, Bloos F, et al. Intensive
2014;34:89-93. insulin therapy and pentastarch resuscitation in
severe sepsis. N Engl J Med 2008;358:125-39.
Sepsis 13. Brunkhorst FM, Oppert M, Marx G, et al. Effect
1. Angus DC, Barnato AE, Bell D, et al. A systematic of empirical treatment with moxifloxacin and
review and meta-analysis of early goal-directed meropenem vs meropenem on sepsis-related organ
therapy for septic shock: the ARISE, ProCESS dysfunction in patients with severe sepsis: a ran-
and ProMISe Investigators. Intensive Care Med domized trial. JAMA 2012;307:2390-9.
2015;41:1549-60. 14. Caironi P, Tognoni G, Masson S, et al. Albumin
2. Angus DC, van der Poll T. Severe sepsis and septic replacement in patients with severe sepsis or septic
shock. N Engl J Med 2013;369:840-51. shock. N Engl J Med 2014;370:1412-21.
3. Angus DC, Yealy DM, Kellum JA. Protocol- 15. Churpek MM, Snyder A, Han X, et al. Quick
based care for early septic shock. N Engl J Med sepsis-related organ failure assessment, systemic
2014;371:386. inflammatory response syndrome, and early warn-
4. Annane D, Pastores SM, Rochwerg B, et al. ing scores for detection of clinical deterioration in
Guidelines for the Diagnosis and Management infected patients outside the intensive care unit.
of Critical Illness-Related Corticosteroid Am J Respir Crit Care Med 2017;195:906-11.
Insufficiency (CIRCI) in Critically Ill Patients 16. De Backer D, Biston P, Devriendt J, et al.
(Part I): Society of Critical Care Medicine (SCCM) Comparison of dopamine and norepinephrine in the
and European Society of Intensive Care Medicine treatment of shock. N Engl J Med 2010;362:779-89.
(ESICM) 2017. Crit Care Med 2017;45:2078-88. 17. De Backer D, Creteur J, Dubois MJ, et al. The
5. Annane D, Renault A, Brun-Buisson C, et al. effects of dobutamine on microcirculatory
Hydrocortisone plus fludrocortisone for adults alterations in patients with septic shock are inde-
with septic shock. N Engl J Med 2018;378:809-18. pendent of its systemic effects. Crit Care Med
6. Annane D, Sebille V, Charpentier C, et al. Effect 2006;34:403-8.
of treatment with low doses of hydrocortisone and 18. Delaney AP, Dan A, McCaffrey J, et al. The role
fludrocortisone on mortality in patients with septic of albumin as a resuscitation fluid for patients with
shock. JAMA 2002;288:862-71. sepsis: a systematic review and meta-analysis. Crit
7. Annane D, Vignon P, Renault A, et al. Care Med 2011;39:386-91.
Norepinephrine plus dobutamine versus epi- 19. Evans L, Rhodes A, Alhazzani W, et al. Surviving
nephrine alone for management of septic shock: a Sepsis Campaign: international guidelines for
randomised trial. Lancet 2007;370:676-84. management of sepsis and septic shock 2021. Crit
8. Asfar P, Meziani F, Hamel JF, et al. High versus Care Med 2021;49:e1063-e1143.
low blood-pressure target in patients with septic 20. Finfer S, Bellomo R, Boyce N, et al. A comparison
shock. N Engl J Med 2014;370:1583-93. of albumin and saline for fluid resuscitation in the
intensive care unit. N Engl J Med 2004;350:2247-56.

ACCP Updates in Therapeutics® 2022: Critical Care Pharmacy Preparatory Review and Recertification Course

667
Shock Syndromes I: Introduction, Vasodilatory, and Sepsis

21. Finfer S, McEvoy S, Bellomo R, et al. Impact of 32. Jones AE, Shapiro NI, Trzeciak S, et al. Lactate
albumin compared to saline on organ function and clearance vs central venous oxygen saturation as
mortality of patients with severe sepsis. Intensive goals of early sepsis therapy: a randomized clinical
Care Med 2011;37:86-96. trial. JAMA 2010;303:739-46.
22. Frazee EN, Leedahl DD, Kashani KB. Key contro- 33. Kumar A, Ellis P, Arabi Y, et al. Initiation of inap-
versies in colloid and crystalloid fluid utilization. propriate antimicrobial therapy results in a fivefold
Hosp Pharm 2015;50:446-53. reduction of survival in human septic shock. Chest
23. Fujii T, Luethi N, Young PJ, et al. Effect of vitamin 2009;136:1237-48.
C, hydrocortisone, and thiamine vs hydrocortisone 34. Kumar A, Roberts D, Wood KE, et al. Duration
alone on time alive and free of vasopressor support of hypotension before initiation of effective anti-
among patients with septic shock: the VITAMINS microbial therapy is the critical determinant of
randomized clinical trial. JAMA 2020;323:423-31. survival in human septic shock. Crit Care Med
24. Gaieski DF, Edwards JM, Kallan MJ, et al. 2006;34:1589-96.
Benchmarking the incidence and mortality of 35. Kumar A, Safdar N, Kethireddy S, et al. A sur-
severe sepsis in the United States. Crit Care Med vival benefit of combination antibiotic therapy for
2013;41:1167-74. serious infections associated with sepsis and sep-
25. Gattinoni L, Brazzi L, Pelosi P, et al. A trial of tic shock is contingent only on the risk of death:
goal-oriented hemodynamic therapy in critically a meta-analytic/meta-regression study. Crit Care
ill patients. Svo2 Collaborative Group. N Engl J Med 2010;38:1651-64.
Med 1995;333:1025-32. 36. Landry DW, Oliver JA. The pathogenesis of vaso-
26. Gordon AC, Mason AJ, Thirunavukkarasu N, et dilatory shock. N Engl J Med 2001;345:588-95.
al. Effect of early vasopressin vs norepinephrine 37. Leisman D, Huang V, Quiping Z, et al. Delayed
on kidney failure in patients with septic shock: second cose antibiotics for patients admitted from
the VANISH randomized clinical trial. JAMA the emergency department with sepsis: preva-
2016;316:509-18. lence, risk factors, and outcomes. Crit Care Med
27. Hammond DA, Lam SW, Rech MA, et al. 2017;45:956-65.
Balanced crystalloids versus saline in critically 38. Levy MM, Evans LE, Rhodes A. The Surviving
ill adults: a systematic review and meta-analysis. Sepsis Campaign Bundle: 2018 update. Crit Care
Ann Pharmacother 2019. [Epub ahead of print] Med 2018;46:997-1000.
28. Hernandez G, Ospina-Tascon GA, Damiani LP, 39. Marik PE. Early management of severe sepsis: con-
et al. Effect of a resuscitation strategy targeting cepts and controversies. Chest 2014;145:1407-18.
peripheral perfusion status vs serum lactate lev- 40. Marik PE, Khangoora V, Rivera R, et al.
els on 28-day mortality among patients with septic Hydrocortisone, vitamin C, and thiamine
shock: the ANDROMEDA-SHOCK randomized for the treatment of severe sepsis and septic
clinical trial. JAMA 2019;321:654-64. shock: a retrospective before-after study. Chest
29. Holst LB, Haase N, Wetterslev J, et al. Lower ver- 2017;151:1229-38.
sus higher hemoglobin threshold for transfusion in 41. Morelli A, Ertmer C, Rehberg S, et al.
septic shock. N Engl J Med 2014;371:1381-91. Phenylephrine versus norepinephrine for initial
30. Jansen TC, van Bommel J, Schoonderbeek FJ, et hemodynamic support of patients with septic
al. Early lactate-guided therapy in intensive care shock: a randomized, controlled trial. Crit Care
unit patients: a multicenter, open-label, random- 2008;12:R143.
ized controlled trial. Am J Respir Crit Care Med 42. Moskowitz A, Huang DT, Hou PC, et al. Effect
2010;182:752-61. of ascorbic acid, corticosteroids, and thiamine on
31. Joint Commission. Specifications Manual for organ injury in septic shock: the ACTS random-
National Hospital Inpatient Quality Measures. 2016. ized clinical trial. JAMA 2020;324:642-50.
Available at https://www.jointcommission.org/ 43. Mouncey PR, Osborn TM, Power GS, et al. Trial of
specifications_manual_for_national_hospital_ early, goal-directed resuscitation for septic shock.
inpatient_quality_measures.aspx. Accessed June N Engl J Med 2015;372:1301-11.
14, 2016.

ACCP Updates in Therapeutics® 2022: Critical Care Pharmacy Preparatory Review and Recertification Course

668
Shock Syndromes I: Introduction, Vasodilatory, and Sepsis

44. Nagendran M, Russell JA, Walley KR, et al. and trial sequential analysis. Intensive Care Med
Vasopressin in septic shock: an individual patient 2108;44:1003-16.
data meta-analysis of randomised controlled trials. 57. Santer P, Anstey MH, Patrocinio MD, et al.
Intensive Care Med 2019;45:844-55. Effect of midodrine versus placebo on time to
45. Ospina-Tascon G, Neves AP, Occhipinti G, et al. vasopressor discontinuation in patients with
Effects of fluids on microvascular perfusion in persistent hypotension in the intensive care unit
patients with severe sepsis. Intensive Care Med (MIDAS): an international randomised clinical
2010;36:949-55. trial. Intensive Care Med 2020;46:1884-93.
46. Perner A, Haase N, Guttormsen AB, et al. 58. Serpa Neto A, Nassar APJ, Cardoso SO, et al.
Hydroxyethyl starch 130/0.42 versus Ringer’s ace- Vasopressin and terlipressin in adult vasodila-
tate in severe sepsis. N Engl J Med 2012;367:124-34. tory shock: a systematic review and meta-analysis
47. Polito A, Parisini E, Ricci Z, et al. Vasopressin for of nine randomized controlled trials. Crit Care
treatment of vasodilatory shock: an ESICM sys- 2012;16:R154.
tematic review and meta-analysis. Intensive Care 59. Sevransky JE, Rothman RE, Hager DN, et al.
Med 2012;38:9-19. Effect of vitamin C, thiamine, and hydrocortisone
48. Raghunathan K, Shaw A, Nathanson B, et al. on ventilator- and vasopressor-free days in patients
Association between the choice of IV crystal- with sepsis: the VICTAS randomized clinical trial.
loid and in-hospital mortality among critically ill JAMA 2021;325:742-50.
adults with sepsis. Crit Care Med 2014;42:1585-91. 60. Seymour CW, Liu VX, Iwashyna TJ, et al.
49. Rhodes A, Evans LE, Alhazzani W, et al. Assessment of clinical criteria for sepsis: for
Surviving Sepsis Campaign: international guide- the Third International Consensus Definitions
lines for management of sepsis and septic shock: for Sepsis and Septic Shock (Sepsis-3). JAMA
2016. Crit Care Med 2017;45:486-552. 2016;315:762-74.
50. Rivers E, Nguyen B, Havstad S, et al. Early goal- 61. Singer M, Deutschman CS, Seymour CW, et al.
directed therapy in the treatment of severe sepsis The Third International Consensus Definitions
and septic shock. N Engl J Med 2001;345:1368-77. for Sepsis and Septic Shock (Sepsis-3). JAMA
51. Roberts JA, Lipman J. Pharmacokinetic issues for 2016;315:801-10.
antibiotics in the critically ill patient. Crit Care 62. Sprung CL, Annane D, Keh D, et al. Hydrocortisone
Med 2009;37:840-51. therapy for patients with septic shock. N Engl J
52. Roberts JA, Paul SK, Akova M, et al. DALI: Med 2008;358:111-24.
defining antibiotic levels in intensive care unit 63. Surviving Sepsis Campaign (SSC). Updated
patients: are current beta-lactam antibiotic doses Bundles in Response to New Evidence.
sufficient for critically ill patients? Clin Infect Dis 2015. Available at www.survivingsepsis.org/
2014;58:1072-83. SiteCollectionDocuments/SSC_Bundle.pdf.
53. Rochwerg B, Alhazzani W, Sindi A, et al. Fluid Accessed September 14, 2018.
resuscitation in sepsis: a systematic review 64. Taccone FS, Laterre PF, Dugernier T, et al.
and network meta-analysis. Ann Intern Med Insufficient beta-lactam concentrations in the
2014;161:347-55. early phase of severe sepsis and septic shock. Crit
54. Rochwerg B, Oczkowski SJ, Siemieniuk RA, et Care 2010;14:R126.
al. Corticosteroids in sepsis: an updated system- 65. Torgersen C, Dunser MW, Wenzel V, et al.
atic review and meta-analysis. Crit Care Med Comparing two different arginine vasopressin
2018;46:1411-20. doses in advanced vasodilatory shock: a random-
55. Russell JA, Walley KR, Singer J, et al. Vasopressin ized, controlled, open-label trial. Intensive Care
versus norepinephrine infusion in patients with Med 2010;36:57-65.
septic shock. N Engl J Med 2008;358:877-87. 66. Townsend SR, Rivers E, Tefera L. Definitions for
56. Rygard SL, Butler E, Granholm A, et al. Low- sepsis and septic shock. JAMA 2016;316:457-8.
dose corticosteroids for adult patients with septic 67. Vail E, Gershengorn HB, Hua M, et al. Association
shock: a systematic review with meta-analysis between US norepinephrine shortage and

ACCP Updates in Therapeutics® 2022: Critical Care Pharmacy Preparatory Review and Recertification Course

669
Shock Syndromes I: Introduction, Vasodilatory, and Sepsis

mortality among patients with septic shock. JAMA


2017;317:1433-42.
68. Venkatesh B, Finfer S, Cohen J, et al. Adjunctive
glucocorticoid therapy in patients with septic
shock. N Engl J Med 2018;378:797-808.
69. Vieillard-Baron A, Caille V, Charron C, et al.
Actual incidence of global left ventricular hypo-
kinesia in adult septic shock. Crit Care Med
2008;36:1701-6.
70. Vincent JL, Rello J, Marshall J, et al. International
study of the prevalence and outcomes of infection
in intensive care units. JAMA 2009;302:2323-9.
71. Werdan K, Pilz G, Bujdoso O, et al. Score-based
immunoglobulin G therapy of patients with sepsis:
the SBITS study. Crit Care Med 2007;35:2693-701.
72. Whitson MR, Mo E, Nabi T, et al. Feasibility, util-
ity, and safety of midodrine during recovery phase
from septic shock. Chest 2016;149:1380-3.
73. Yealy DM, Kellum JA, Huang DT, et al. A ran-
domized trial of protocol-based care for early
septic shock. N Engl J Med 2014;370:1683-93.

ACCP Updates in Therapeutics® 2022: Critical Care Pharmacy Preparatory Review and Recertification Course

670
Shock Syndromes I: Introduction, Vasodilatory, and Sepsis

ANSWERS AND EXPLANATIONS TO PATIENT CASES

1. Answer: B 4. Answer: C
The patient has hypovolemic shock caused by his upper The patient has shock with hypoperfusion and a posi-
GI hemorrhage and has symptoms of compromised end- tive response to a PLR test, which suggests he is still
organ perfusion (i.e., confusion). Although his history fluid responsive (Answer D is incorrect). Data from a
of hypertension is relevant in relation to determining a large randomized trial of patients with heterogeneous
resuscitation goal, it does not acutely affect Do2 (Answer shock types showed no difference in efficacy and safety
A is incorrect). Tachycardia is a symptom in response to between albumin and 0.9% sodium chloride, but the
his hypovolemia, and, in the absence of complicating cost of albumin is substantially higher. These data sug-
factors (e.g., atrial fibrillation or LV diastolic dysfunc- gest that crystalloids such as 0.9% sodium chloride are
tion), tachycardia will increase (not decrease) Do2 preferred for fluid resuscitation in the ICU. In addition,
(Answer C is incorrect). Leukocytosis does not impede the patient has not received a substantial volume of
Do2 until it reaches an exorbitant threshold (greater than fluid (he has received less than 30 mL/kg; Answer C
75 L/mm3) (Answer D is incorrect). When examining the is correct; Answer A is incorrect). Hydroxyethyl starch,
Fick equation for Do2, acute anemia is the determinant which has been associated with an increased need for
that is adversely affecting Do2 (Answer B is correct). renal replacement therapy without a mortality benefit,
should be avoided for fluid resuscitation in the ICU
2. Answer: C (Answer B is incorrect).
The patient has evidence of vasodilatory shock post-
resuscitation, with his relatively high CVP and PCWP 5. Answer: D
values and high CO (Answer C is correct). This is further The patient has evidence of ventricular dysfunction
confirmed by a calculated SVR of 541 dynes x second x with a low Scvo2 and poor ventricular contractility on
cm-5. The patient could be thought to have spontaneous echocardiogram. A vasoactive agent with strong ino-
bacterial peritonitis in the setting of cirrhosis and asci- tropic properties is indicated. Epinephrine has strong
tes complicated by upper GI hemorrhage. Even though β1-adrenergic properties and is the best selection in this
his presentation suggests hypovolemic shock (from GI case (Answer D is correct). Both phenylephrine and
hemorrhage), his MAP did not respond to fluid and vasopressin are essentially pure vasoconstrictors that
blood product administration. Furthermore, he does not do not increase CO and could theoretically decrease
have low preload or low CO (Answer A is incorrect). If CO. In addition, use of phenylephrine is no longer rec-
the patient had a low CO together with a high SVR, car- ommended by the guidelines (Answers A and B are
diogenic shock or obstructive shock might be possible incorrect). Although norepinephrine has β1-adrenergic
(with one differentiation depending on CVP/right atrial properties, it primarily increases blood pressure
pressure and PCWP), but this is untrue for the patient through vasoconstriction secondary to its α1-adrenergic
(Answers B and D are incorrect). properties, with only minimal effects on CO. Increasing
norepinephrine in this case would probably not improve
3. Answer: C the patient’s CO sufficiently to improve tissue perfusion
The patient’s clinical scenario of refractory hypoxemia (Answer C is incorrect).
and hypotension with hypoperfusion suggests that an
accurate prediction of fluid responsiveness is needed. 6. Answer: B
Dynamic markers of fluid responsiveness (e.g., SVV) The patient has life-threatening organ dysfunction
are superior to static markers of fluid responsiveness secondary to infection. Although not all the criteria to
(e.g., CVP and PCWP) (Answer C is correct; Answers calculate the score are available, her SOFA score is at
A and B are incorrect). A low MAP may be from either least 4 points (2 points for Plt, 1 point for Glasgow Coma
a low CO or a low SVR. Furthermore, a low preload is Scale score, and 1 point for SCr), indicating organ dys-
only one of many components that may contribute to a function. In addition, the patient fulfills all three qSOFA
low CO. As such, a low MAP is not a good predictor of criteria (altered mental status, SBP of 100 mm Hg or less,
fluid responsiveness (Answer D is incorrect). and respiratory rate of 22 breaths/minute or greater).

ACCP Updates in Therapeutics® 2022: Critical Care Pharmacy Preparatory Review and Recertification Course

671
Shock Syndromes I: Introduction, Vasodilatory, and Sepsis

Therefore, she meets the criteria for sepsis, according infusion could be considered because this might lead to
to the new definition (Answer B is correct). Although fewer variations in serum glucose. The ACTH stimula-
she has an elevated lactate concentration, she does not tion test to determine which patients with sepsis should
require vasopressors to maintain a MAP above 65 mm receive hydrocortisone is no longer recommended
Hg and therefore does not meet the criteria for septic because the CORTICUS study showed that the ACTH
shock (Answer C is incorrect). In the new sepsis defini- stimulation test did not predict response to hydrocorti-
tion, severe sepsis and systemic inflammatory response sone therapy (Answer C is incorrect).
syndrome are no longer clinical entities in the spectrum
of sepsis severity and should not be used (Answers A
and D are incorrect).

7. Answer: A
The patient currently has a MAP less than 65 mm Hg
and signs of a global lack of perfusion with an increased
lactate concentration. Vasopressor therapy is indicated
to sustain perfusion. This patient has underlying severe
congestive heart failure with an ejection fraction of 20%;
however, the Scvo2 shows that the patient’s Do2 is suffi-
cient, which suggests he is in distributive septic shock as
opposed to cardiogenic shock. The SSC guidelines rec-
ommend initiating vasopressor therapy to target a MAP
of 65 mm Hg and norepinephrine as the first-choice
vasopressor (Answer A is correct). Vasopressin is cur-
rently recommended only as a secondary vasopressor,
as an addition to catecholamine therapy (Answer B is
incorrect). In septic shock, dobutamine is recommended
only when the Scvo2 is less than 70% and the patient has
an adequate Hgb concentration (Answer C is incorrect).
Epinephrine is currently recommended as an alternative
to norepinephrine. In a study that compared epinephrine
with norepinephrine plus dobutamine, the two arms did
not differ in mortality outcomes; however, epinephrine
was associated with lower pH values and higher lactate
concentrations on day 1 (Answer D is incorrect).

8. Answer: D
According to the SSC guidelines, corticosteroids may
be considered if patients have a poor response to fluid
resuscitation and vasopressor therapy. In addition,
guidelines from the Society of Critical Care Medicine
and the European Society of Intensive Care Medicine
suggest corticosteroids only in patients requiring mod-
erate- to high-dose vasopressors. In this case, the patient
is receiving relatively low doses of vasopressors with
an adequate MAP; hence, no steroids are currently
necessary (Answer D is correct; Answers A and B are
incorrect). Hydrocortisone is recommended at a dose
of 200–400 mg/day. Administering it as a continuous

ACCP Updates in Therapeutics® 2022: Critical Care Pharmacy Preparatory Review and Recertification Course

672
Shock Syndromes I: Introduction, Vasodilatory, and Sepsis

ANSWERS AND EXPLANATIONS TO SELF-ASSESSMENT QUESTIONS

1. Answer: C population, with no mortality benefit. Hydroxyethyl


This older adult patient presents with septic shock. starch solutions should be avoided for fluid resuscitation
Shock is a syndrome of impaired Do2, leading to tissue in the ICU (Answer C is incorrect).
injury and end-organ failure. In this case, it is important
to realize that fever is a symptom of an inflammatory 4. Answer: B
syndrome, not a shock syndrome (Answers A, B, and The patient has evidence of end-organ hypoperfusion
D are incorrect). This patient’s presentation with hypo- (urinary output less than 0.5 mL/kg/hour and elevated
tension, confusion, and hyperlactatemia suggests the lactate concentration without significant clearance),
presence of a shock syndrome in the setting of impaired despite a MAP greater than 65 mm Hg and quantita-
Do2 (Answer C is correct). tive resuscitation. The patient probably needs a higher
perfusion pressure because of a right-shifted zone of
2. Answer: B autoregulation secondary to hypertension. The norepi-
The Fick equation for Do2 best shows Do2. According nephrine dose should be increased to target a higher
to this equation, Do2 depends on CO and Cao2. Cardiac MAP; the exact target will depend on the patient’s
output depends on heart rate and SV. Typically, an ele- response and should be selected as the threshold that
vated heart rate will increase CO and Do2. However, improves end-organ perfusion (Answer B is correct).
if a patient has atrial fibrillation, ventricular filling is The patient has no evidence of impaired CO (his Scvo2
impaired and SV is decreased, causing a decrease in CO is not low); therefore, fluids (to improve SV) and ino-
and Do2 (Answer B is correct). Lactate does not impede tropes (e.g., dobutamine) are not indicated (Answers C
Do2 but is a byproduct of impaired Do2 (Answer A is and D are incorrect). Because the patient has continued
incorrect). Similarly, Do2 is not impaired by acute kid- evidence of hypoperfusion, action should be taken, and
ney injury (Answer C is incorrect). Finally, fever is an the current therapy should be modified (Answer A is
inflammatory response that increases Vo2 but does not incorrect).
impair Do2 (Answer D is incorrect).
5. Answer: B
3. Answer: D The patient has hypotension and signs of hypoperfusion
The patient’s laboratory values and arterial pH are with an elevated lactate concentration; possible inter-
consistent with hyperchloremic metabolic acidosis, ventions such as fluid administration should be explored
and chloride-rich fluids should be avoided. Lactated further. Because the patient requires a high Fio2, a reli-
Ringer’s solution is considered a relatively chloride- able predictor of fluid responsiveness should be used to
poor solution (chloride content 111 mEq/L) and is best guide fluid therapy instead of administering fluid with-
in this case (Answer D is correct). With its chloride out respect to predicting responsiveness. In the setting
content of 154 mEq/L, 0.9% sodium chloride is consid- of atrial fibrillation, an elevated PPV is not a reliable
ered a chloride-rich fluid. Liberal use of chloride-rich predictor of fluid responsiveness, and further evaluation
fluids such as 0.9% sodium chloride has been associated should be done before fluids are administered (Answer
with an increased need for renal replacement therapy A is incorrect). A PLR test can be used in both spontane-
(Answer A is incorrect). Although albumin 5% is con- ously breathing patients and those receiving mechanical
sidered a chloride-poor solution, data from randomized ventilation to predict fluid responsiveness, and the patient
controlled trials have not supported a mortality ben- has a method to assess the presence (or absence) of a CO
efit with albumin over crystalloids, even in the setting response (Answer B is correct). Although the accuracy
of hypoalbuminemia. Moreover, the patient has not of the CO value from arterial pulse pressure waveform
received a “substantial volume” of crystalloids (she has analysis may be somewhat limited by atrial fibrillation,
received less than 30 mL/kg of fluid) that would coin- this may be accounted for with the internal software of
cide with recommendations to give albumin (Answer most devices and can be used to gauge a response to
B is incorrect). Hydroxyethyl starch solutions may also the PLR test. The patient has a femoral central venous
have a high chloride content (depending on the formula- catheter, which cannot be used to assess hemodynamic
tion) and have been associated with an increased need markers such as CVP or Scvo2 (Answers C and D are
for renal replacement therapy in the general critical care

ACCP Updates in Therapeutics® 2022: Critical Care Pharmacy Preparatory Review and Recertification Course

673
Shock Syndromes I: Introduction, Vasodilatory, and Sepsis

incorrect). Furthermore, CVP is an inadequate predictor by the SSC as the first-line vasopressor (Answer A is
of fluid responsiveness. correct). Vasopressin is not recommended as the single
initial vasopressor, but it may be added to norepineph-
6. Answer: D rine (Answer B is incorrect). Phenylephrine is no longer
The patient has features of vasodilatory shock second- recommended in the SSC guidelines. Although this
ary to an immune-mediated (“anaphylactic”) reaction patient has a history of atrial fibrillation and a high heart
(low preload, a low Scvo2 [suggestive of poor Do2], and rate, norepinephrine should still be tried and the patient
an elevated lactate concentration). The patient should observed for signs of worsening tachyarrhythmias
receive aggressive fluid resuscitation and be initiated (Answer C is incorrect). Dopamine is recommended as
on a vasopressor such as norepinephrine with the pri- an alternative vasopressor to norepinephrine in select
mary effects of augmenting afterload (Answer D is patients, such as those with bradycardia. In a meta-
correct). Although the patient has features of poor Do2, analysis of patients with septic shock, dopamine was
this is likely because of inadequate preload, which will associated with a higher mortality rate and more fre-
be augmented by fluid administration. Agents targeted quent tachyarrhythmias (Answer D is incorrect).
toward improving Cao2 (PRBCs) and CO (dobutamine
and milrinone) should not be initiated unless the patient 9. Answer: B
has inadequate Do2 and is not fluid responsive (Answers Vasopressin can be added to norepinephrine to either
A–C are incorrect). increase MAP or lower norepinephrine requirements. In
the VASST (Vasopressin and Septic Shock Trial) study
7. Answer: A comparing norepinephrine monotherapy with nor-
The patient has received an initial fluid challenge of epinephrine plus vasopressin, mortality did not differ
only 23 mL/kg of crystalloids and still has evidence of between the two groups, but norepinephrine require-
end-organ hypoperfusion (elevated lactate concentra- ments were significantly lower in the patients allocated
tion and urinary output less than 0.5 mL/kg/hour). An to receive AVP. According to the SSC guidelines,
additional bolus of at least 500 mL of 0.9% sodium chlo- vasopressin can be added to norepinephrine to either
ride is indicated to ensure an initial fluid challenge of at raise the MAP or decrease the norepinephrine dosage
least 30 mL/kg of crystalloids and to improve end-organ (Answer B is correct). Although a dynamic marker of
perfusion (Answer A is correct). Because the patient has fluid responsiveness is not presented, the patient’s CO
not received a complete initial fluid challenge (or even and cardiac preload are likely adequate, given that
a substantial amount of crystalloids), albumin is not his Scvo2 is 72%. Additional fluid loading is not indi-
indicated (Answer B is incorrect). Vasopressors are not cated, given the information presented (Answer A is
indicated right now because the patient’s MAP is above incorrect). Phenylephrine is no longer recommended
65 mm Hg (the patient’s MAP is 67 mm Hg). In addition, as a treatment in the SSC guidelines; however, it has
if an initial vasopressor were to be selected, norepi- theoretical benefit as a second-line vasopressor when a
nephrine would be preferred (Answer C is incorrect). malignant tachyarrhythmia is associated with norepi-
The patient’s low Scvo2 is likely caused by inadequate nephrine. In this case, phenylephrine is not indicated
preload (resulting in inadequate SV and CO). Adequate because sinus tachycardia is not considered a malignant
preload should be ensured before giving PRBCs as part tachyarrhythmia (Answer C is incorrect). Epinephrine
of improving Do2 (Answer D is incorrect). is recommended when an additional agent is needed to
raise the MAP to the target. Adding epinephrine to nor-
8. Answer: A epinephrine would only add inotropic support, but this
Despite an initial fluid challenge of greater than 30 mL/ patient has no signs of low CO with an Scvo2 of 72%,
kg of crystalloids, the patient has continued evidence and the patient’s inadequate blood pressure is the most
of hypotension (MAP 63 mm Hg) and hypoperfusion likely reason for his hypoperfusion (elevated lactate and
(an elevated lactate and a urinary output less than 0.5 low urinary output). Adding epinephrine would also
mL/kg/hour). A vasopressor should be initiated to likely increase the patient’s heart rate and potential for a
improve blood pressure (MAP greater than 65 mm Hg) tachyarrhythmia. As such, epinephrine is not indicated
and organ perfusion. Norepinephrine is recommended (Answer D is incorrect).

ACCP Updates in Therapeutics® 2022: Critical Care Pharmacy Preparatory Review and Recertification Course

674
Shock Syndromes II:
Hypovolemic, Critical Bleeding,
and Obstructive
Mitchell J. Daley, Pharm.D., FCCM, BCCCP
Dell Seton Medical Center at the University of Texas
Austin, Texas

Mahmoud A. Ammar, Pharm.D., FCCM, BCCCP, BCPS


Yale New Haven Hospital
New Haven, Connecticut
Shock Syndromes II: Hypovolemic, Critical Bleeding, and Obstructive

Shock Syndromes II:


Hypovolemic, Critical Bleeding,
and Obstructive
Mitchell J. Daley, Pharm.D., FCCM, BCCCP
Dell Seton Medical Center at the University of Texas
Austin, Texas

Mahmoud A. Ammar, Pharm.D., FCCM, BCCCP, BCPS


Yale New Haven Hospital
New Haven, Connecticut

ACCP Updates in Therapeutics® 2022: Critical Care Pharmacy Preparatory Review and Recertification Course

677
Shock Syndromes II: Hypovolemic, Critical Bleeding, and Obstructive

Learning Objectives Self-Assessment Questions


Answers and explanations to these questions may be
1. Identify critical determinants affecting oxygen found at the end of this chapter.
delivery and the physiologic response to hypovole-
mic and obstructive shock. 1. A 41-year-old man presents to the emergency
2. Evaluate resuscitation strategies and end points in department (ED) after a motorcycle accident. While
the management of hypovolemic, hemorrhagic, and the patient is being evaluated, it is apparent that
obstructive shock. he has broken ribs, a broken pelvis, and bilateral
3. Devise a treatment strategy for pharmacotherapy broken femurs. He is confused, and his vital signs
adjuncts in the management of bleeding and acute are as follows: blood pressure 82/48 mm Hg, heart
coagulopathy when treating patients with hemor- rate 125 beats/minute, respiratory rate 34 breaths/
rhagic shock. minute, and temperature 95°F (35°C). Which most
4. Develop a treatment pathway for the care of patients accurately reflects this patient’s class of hemorrhage
receiving anticoagulants and antiplatelet agents for from trauma?
a life-threatening hemorrhage or critical bleeding A. I.
that incorporates current evidence and guideline B. II.
recommendations. C. III.
5. Apply risk stratification to guide the effective and D. IV.
safe use of thrombolytic agents in the management
of acute pulmonary embolism. 2. A 62-year-old man with a history of alcoholic cirrho-
sis and portal hypertension presents to the ED with
weakness and hematemesis. Pertinent vital signs on
Abbreviations in This Chapter admission are as follows: blood pressure 76/42 mm
4PCC 4-factor prothrombin complex concentrate Hg, heart rate 134 beats/minute with a rhythm of
ABC Assessment of Blood Consumption sinus tachycardia, and respiratory rate 24 breaths/
ACC American College of Cardiology minute. Frank red blood is noted on nasogastric
aPCC Activated prothrombin complex concentrate lavage. On physical examination, the patient is con-
aPTT Activated partial thromboplastin time fused, with cold and clammy extremities. Pertinent
CDT Catheter-directed thrombolysis laboratory values are as follows: hemoglobin (Hgb)
CO Cardiac output 5.5 g/dL, blood urea nitrogen (BUN) 56 mg/dL,
DOAC Direct oral anticoagulant serum creatinine (SCr) 1.8 mg/dL, international nor-
Do2 Oxygen delivery malized ratio (INR) 2.6, total bilirubin 7.5 mg/dL,
FAST Focused Assessment with Sonography central venous oxygen saturation (Scvo2) 58%, and
Fio2 Fraction of inspired oxygen lactate 2.7 mmol/L. In this patient, which variable
LV Left ventricular would best explain reduced oxygen delivery (Do2)
MTP Massive transfusion protocol and which associated clinical or biochemical sign
PCC Prothrombin complex concentrate would best indicate a shock syndrome?
PE Pulmonary embolism
A. Elevated INR and hematemesis.
PH Pulmonary hypertension
B. Sinus tachycardia and elevated serum
PRBC Packed red blood cell
creatinine.
PT Prothrombin time
C. Low hemoglobin and cold and clammy
rFVIIa Recombinant activated factor VII
extremities.
RV Right ventricular
D. Low Scvo2 and hyperlactatemia.
TBSA Total body surface area
TEG Thromboelastogram

ACCP Updates in Therapeutics® 2022: Critical Care Pharmacy Preparatory Review and Recertification Course

678
Shock Syndromes II: Hypovolemic, Critical Bleeding, and Obstructive

3. A 44-year-old man (weight 82 kg) presents to the C. Fresh frozen plasma 15-mL/kg infusion once.
ED after a motor vehicle collision at highway D. Phytonadione 10-mg intravenous push.
speeds secondary to his windshield being shattered
by a rock thrown from an overpass. On primary 5. A 66-year-old man with a medical history of non–
survey, his King airway device is replaced for an small cell lung cancer presents to the ED with
endotracheal tube; his respiratory rate is 37 breaths/ new-onset shortness of breath. A chest computed
minute, systolic blood pressure (SBP) is 77 mm Hg, tomography (CT) scan reveals a PE at the bifurca-
heart rate is 146 beats/minute, and Glasgow Coma tion of the right and left pulmonary arteries. The
Scale score is 3. In addition, he is noted to have a patient is initiated on parenteral anticoagulation and
positive seatbelt sign. The Focused Assessment transferred to the medical intensive care unit (ICU).
with Sonography (FAST) examination is positive. On ICU admission, he develops pulseless electrical
He is taken to the operating room for an emergency activity. He is intubated and mechanically venti-
exploratory laparotomy. Which best represents the lated, with recovery of spontaneous circulation after
most appropriate initial transfusion, resuscitation, one round of chest compressions and epinephrine 1
and hemostasis strategy? mg. Which is the next best step to evaluate and/or
A. Give 2 L of warmed lactated Ringer solution, treat this patient’s PE?
followed by a 2-unit transfusion of erythrocytes. A. Administer alteplase 100 mg infused over 2
B. Administer 30 mL/kg of warmed lactated hours.
Ringer solution and a tranexamic acid 1-g B. Check a troponin T concentration.
bolus, followed by 1 g infused over 8 hours. C. Check a brain natriuretic peptide concentration.
C. Obtain and send an INR, activated partial D. Do a transthoracic echocardiogram.
thromboplastin time (aPTT), fibrinogen, and
complete blood cell count to guide initial
resuscitation. Questions 6–8 pertain to the following case.
D. Initiate massive transfusion, giving erythrocytes R.M is a 78-year-old man with a history of alcoholic
and plasma in a 1:1 ratio plus a tranexamic acid cirrhosis and portal hypertension who is in an urban,
1-g bolus and 1 g infused over 8 hours. academic ICU after treatment of a variceal hemorrhage
(now stable) and respiratory failure. Five days after
4. H.S. is a 76-year-old man (weight 142 kg) with a admission, he develops hypoxemia (partial pressure
history of several pulmonary embolisms (PEs) and oxygen saturation [Pao2] 78%), requiring an increased
associated chronic thromboembolic pulmonary fraction of inspired oxygen (Fio2) on the ventilator. A
hypertension (PH) on warfarin who is admitted to chest computed tomography angiography (CTA) reveals
the ED with weakness and hematemesis. Pertinent several filling defects at the bifurcation of the main pul-
vital signs on admission are as follows: blood pres- monary artery, suggesting a “saddle” PE. His heart rate
sure 82/46 mm Hg, heart rate 121 beats/minute with is 142 beats/minute with a rhythm of sinus tachycardia.
a rhythm of sinus tachycardia, and respiratory rate His blood pressure is 97/62 mm Hg and current weight
22 breaths/minute. Frank red blood is noted on naso- is 87 kg. R.M. has a transthoracic echocardiogram that
gastric lavage. On physical examination, the patient reveals right ventricle (RV) hypokinesis and tricuspid
is confused, with clammy extremities. Pertinent regurgitation. His cardiac troponin I (0.6 ng/mL) and
laboratory values are as follows: Hgb 5.2 g/dL, troponin T (0.2 ng/mL) are positive.
BUN 52 mg/dL, SCr 2 mg/dL, INR 9.2, and platelet
count (Plt) 120,000/mm3. Which is most effective to 6. Which would best categorize R.M.’s PE?
immediately reverse his apparent warfarin toxicity? A. High risk.
A. Phytonadione 10 mg orally once. B. Intermediate-high risk.
B. 4-factor prothrombin complex concentrate C. Intermediate-low risk.
(4PCC) 50 units/kg intravenously infused over D. Low risk.
30 minutes.

ACCP Updates in Therapeutics® 2022: Critical Care Pharmacy Preparatory Review and Recertification Course

679
Shock Syndromes II: Hypovolemic, Critical Bleeding, and Obstructive

7. After initiating therapeutic heparin and monitoring,


R.M.’s blood pressure is now maintained at 90/50
mm Hg, but his oxygenation continues to worsen,
now requiring 80% Fio2. The medical team has
decided that the patient’s potential benefit from
thrombolytic therapy outweighs the potential risks.
Which is the best approach for this patient’s throm-
bolytic administration?
A. Alteplase 100 mg infused over 2 hours.
B. Catheter-directed thrombolysis (CDT).
C. Alteplase 50 mg infused over 2 hours.
D. Alteplase 0.5 mg/hour by a pulmonary artery
catheter.

8. As you prepare to begin administering your strategy


chosen previously, R.M. goes into a cardiac arrest
with pulseless electrical activity. Which is the best
intervention for managing R.M.’s PE while still in
the ICU?
A. Give alteplase 50 mg intravenous push once.
B. Proceed to surgical embolectomy.
C. Give alteplase 100 mg over 2 hours.
D. Abort cardiopulmonary resuscitation.

ACCP Updates in Therapeutics® 2022: Critical Care Pharmacy Preparatory Review and Recertification Course

680
Shock Syndromes II: Hypovolemic, Critical Bleeding, and Obstructive

BPS Critical Care Pharmacy Specialist Examination Content Outline

This chapter covers the following sections of the Critical Care Pharmacy Specialist Examination Content Outline:
1. Domain 1: Clinical Knowledge and Application
a. Task 1: 1, 3
b. Task 2: 1-7
c. Task 3: 1-5
d. Task 4: 1, 2, 4, 5, 8
e. Task 5: 1, 7
f. Task 6: 1-6
g. Task 7: 1-7
h. Task 8: 1, 2, 4
Systems and patient care problems:
– Hypovolemic shock
– Acute traumatic coagulopathy
– Burn resuscitation
– Antithrombotic reversal
– Obstructive shock
2. Domain 3: Evidence-Based Medicine, Scholarship, Education, and Professional Development
a. Task 1: 1-3

ACCP Updates in Therapeutics® 2022: Critical Care Pharmacy Preparatory Review and Recertification Course

681
Shock Syndromes II: Hypovolemic, Critical Bleeding, and Obstructive

I.  HYPOVOLEMIC SHOCK

A. Etiology and Epidemiology


1. Patients with hypovolemic shock constitute about 16% of the cases of shock requiring vasoactive
medications.
2. In the United States, the most common form of shock secondary to trauma is hypovolemic shock, and
exsanguination is estimated to be the direct cause of 50,000 deaths from trauma in the United States
(Table 1).
a. Responsible for up to 30%–40% of trauma-related mortality
b. Leading cause of death in patients younger than 45; thus, burden to society is even larger
3. Although commonly associated with trauma, hypovolemic shock can also occur in other clinical
scenarios (e.g., acute gastrointestinal [GI] bleeding, surgical, obstetrics, pharmacologic toxicity, burns,
pancreatitis, loss of extracellular fluid).

Table 1. Estimated Hemorrhage-Related Deaths/yr and Years of Life Lost in the United States and Worldwide,
According to the Cause of Hemorrhage
Death from U.S. Cases Global Cases
Cause of Hemorrhage Hemorrhage No. of Years of No. of Years of
(%)a Deaths/yr Life Lost Deaths/yr Life Lost
Trauma 30 49,400 1,931,786 1,481,700 74,568,000
Abdominal aortic
100 9988 65,273 191,700 2,881,760
aneurysm
Peptic ulcer disease 60 1860 38,597 141,000 3,903,600
Maternal disorder 23 138 7572 69,690 4,298,240
a
Estimates of deaths from hemorrhage as a percentage of all deaths from the given diagnosis.
Modified from: Cannon JW. Hemorrhagic shock. N Engl J Med 2018;378:370-9.

B. Pathophysiology
1. Reduced intravascular volume leads to reduced tissue perfusion. This can be categorized as:
a. Whole blood loss: Whole blood loss from an open wound or into a body compartment (e.g., limb,
retroperitoneal space)
b. Plasma loss: Loss of extracellular fluid (e.g., burns, pancreatitis, peritonitis, vomiting, diarrhea)
2. Hemorrhagic shock occurs when intravascular volume loss impairs Do2. Estimated blood volume for
a patient weighing 70 kg is 5 L. Shock occurs when intravascular volume loss impairs Do2, which is
generally greater than 30% loss of total blood volume.
3. Clinical features of hypovolemic shock include hypotension, tachycardia, diaphoresis, altered
mentation, and decreased urinary output. If hypovolemic shock is secondary to blood loss from trauma,
physiologic variables may be used to estimate the extent of blood loss (Table 2).

ACCP Updates in Therapeutics® 2022: Critical Care Pharmacy Preparatory Review and Recertification Course

682
Shock Syndromes II: Hypovolemic, Critical Bleeding, and Obstructive

Table 2. Classification of Trauma Hemorrhagea


Class I Class II Class III Class IV
< 750 750–1500 1500–2000 > 2000
Blood loss (mL)/%
< 15% 15%–30% 30%–40% > 40%
Heart rate (beats/min) < 100 > 100 > 120 > 140
Respiratory rate (breaths/min) 14–20 20–30 30–40 > 35
Urinary output (mL/hr) > 30 20–30 5–15 <5
CNS symptoms Normal Anxious Confused Lethargic
a
Estimated blood loss is based on a man weighing 70 kg. Classification system is only intended as a guide to initial therapy because the physiologic response to
hemorrhage represents a continuum. Confounding factors that influence the physiologic response to hemorrhage include patient age, severity of injury, time from injury,
prehospital interventions, and medications for chronic conditions. Therefore, it is not intended to wait for a patient to fit each precise physiologic classification before
initiating volume resuscitation.
CNS = central nervous system.
Modified from: Committee on Trauma: Advanced Trauma Life Support Manual. Chicago: American College of Surgeons, 2012:62-78.

4. Physiologic response
a. Compensatory responses try to restore the volume deficit.
b. Neural response is immediate, occurring within minutes.
i. Sympathetic response: Activation of the low-pressure receptors within the right and left atria
and high-pressure receptors within the aortic arch and carotid sinus lead to increased secretion
of epinephrine and norepinephrine, resulting in increased heart rate, myocardial contractility,
and arteriolar/venous tone. Blood flow is preserved to critical organs.
ii. Parasympathetic response: Reduced vagal tone leads to increased heart rate. Often, tachycardia
is the earliest sign of circulatory shock from acute blood loss.
c. Intrinsic response compensates for acute blood loss within hours.
i. Reduced capillary pressure leads to fluid redistribution from the interstitial space to the
vascular compartment as albumin shifts into the plasma.
ii. The transcapillary refill can recruit up to 1 L into the intravascular compartment.
d. Humoral response is delayed, developing over hours to several days. After decreased renal
perfusion, secretion of antidiuretic hormone, aldosterone, and renin increases sodium and volume
retention to restore the interstitial deficit from the transcapillary refill.

C. Acute Traumatic Coagulopathy (ATC):


1. Defined as an increase in standard plasma-based coagulation tests above normal limits not associated
with antiplatelet or anticoagulation therapy (aPTT, prothrombin time [PT], and INR, fibrinogen). In a
multicenter, observational study, an INR-based definition of acute coagulopathy (INR greater than 1.5)
was significantly associated with death (odds ratio [OR] 1.88; p<0.001), venous thromboembolism (OR
1.73; p<0.001), and multiorgan failure (OR 1.38; p=0.02).
2. Present in 25%–35% of injured civilian trauma patients on arrival at the ED and associated with a 40%–
70% mortality rate, which likely plays a significant role in preventable deaths caused by hemorrhage
3. Pathophysiology of ATC is multifactorial:
a. Inadequate tissue perfusion because of the resulting hypovolemia
b. Subsequent cell hypoxia, anaerobic respiration, and metabolic acidosis
c. Thrombin–thrombomodulin-complex generation caused by the tissue injury and the activation of
anticoagulant and fibrinolytic pathways
d. Hyperfibrinolysis plays a central role in the initial coagulopathy.
e. Severe systemic hypothermia is associated with decreases in the enzymatic activity of clotting
factors.

ACCP Updates in Therapeutics® 2022: Critical Care Pharmacy Preparatory Review and Recertification Course

683
Shock Syndromes II: Hypovolemic, Critical Bleeding, and Obstructive

f. Coagulopathy, acidosis, and hypothermia—known as the “lethal triad”—exacerbate one another,


rapidly leading to death if not reversed.
4. Characterization of ATC
a. Standard coagulation laboratory or viscoelastic tests are recommended as monitoring techniques to
characterize ATC (European Trauma Guidelines, grade 1A and 1C recommendation, respectively).
b. Viscoelastic tests play an emerging role in screening for coagulopathy in hemorrhagic shock.
i. Whole blood coagulation tests that represent global clot function, including clot initiation,
formation, stabilization, and fibrinolysis, correlate well with standard plasma coagulation
studies and may be available as point-of-care tests with a faster turnaround time.
ii. Available tests include thromboelastogram (TEG) and rotational thromboelastometry, which
are more commonly used in the United States and Europe, respectively.
iii. TEG outputs include a graphic representation of clot kinetics (Figure 1 and Figure 2), including:
(a) Reaction (R) time, normal range 3–9 minutes, which represents factor initiation and
function
(b) Alpha angle (α), normal range 55–78 degrees, which represents clot strengthening,
predominantly fibrinogen
(c) Maximum amplitude (MA), normal range 51–69 mm, which represents clot strength,
predominantly platelets but also fibrinogen
(d) Estimated percent lysis (LY30) or lysis 30 minutes, normal range 0%–8%, which
represents fibrinolysis
(e) Kinetics (K) represents time from the end of R (factor initiation and start of clot formation)
until clot reaches 20 mm of clot strength, normal range 1–3 minutes, which represents
platelet activity, factor levels, and fibrinogen.

Figure 1. Thromboelastogram visible tracing.


α = angle; K = kinetics; LY30 = lysis 30 minutes; MA = maximum amplitude; R = reaction (time).

ACCP Updates in Therapeutics® 2022: Critical Care Pharmacy Preparatory Review and Recertification Course

684
Shock Syndromes II: Hypovolemic, Critical Bleeding, and Obstructive

TEG Tracing Interpretation Treatment

Enoxaparin or heparin prophylaxis


Hypercoagulable
when possible

Hyperfibrinolysis Tranexamic acid

Cryoprecipitate -or-
Fibrinogen deficiency
Concentrated fibrinogen

Plasma
Factor deficiency -or-
prothrombin complex concentrate

Platelet transfusions
Platelet dysfunction -or-
desmopressin

Figure 2. Examples of TEG tracings (dark) relative to the control (light) to characterize and describe treatments.

D. Coagulopathy of Chronic Liver Disease


1. Patients with chronic liver disease often have episodes of clinically meaningful bleeding events
confounded by decreased concentrations of most endogenous procoagulant factors, except for factor
VIII and von Willebrand factor.
2. Historically, the basic laboratory tests of coagulation (e.g., PT, aPTT) have been used to determine the
risk of bleeding, despite their poor correlation with onset and duration of bleeding after liver biopsy or
predicting the occurrence of GI hemorrhage.
3. This is likely explained by the parallel reduction in endogenous anticoagulants (e.g., antithrombin,
protein C) in chronic liver disease, leading to a “balanced state of coagulation.”

ACCP Updates in Therapeutics® 2022: Critical Care Pharmacy Preparatory Review and Recertification Course

685
Shock Syndromes II: Hypovolemic, Critical Bleeding, and Obstructive

4. More recently, randomized controlled trials evaluating recombinant activated factor VII (rFVIIa) to
reverse the coagulopathy of chronic liver disease showed dramatically improved PT, but rFVIIa failed
to alter bleeding complications during liver transplantation or control bleeding in variceal hemorrhage.
5. The American Association for the Study of Liver Diseases recently published a 2017 updated guideline
for managing portal hypertensive bleeding in cirrhosis. This guideline recommends against the
use of fresh frozen plasma or rFVIIa to correct an INR because INR is not a reliable indicator of
coagulation in cirrhosis, and evidence is lacking that this improves outcomes. In addition, the American
Gastroenterological Association 2019 guidelines on coagulation in cirrhosis recommend that although
4PCC may be desirable for factor deficiency correction on the basis of minimal volume required, dosing
based on the INR is problematic in patients with cirrhosis, and this approach warrants further study.
6. Global clotting tests (e.g., the thrombin generation test or TEG) have an increasing potential to
characterize the coagulopathy that occurs in chronic liver disease, yet they may not be routinely
available, and optimal transfusion thresholds remain undefined.

E. Resuscitation
1. Rapid identification and correction of the source of bleeding is the definitive treatment (e.g., surgical
exploration, angiographic embolization, stabilization of the pelvic ring, damage control surgery).
2. Fluids
a. Indications: Diminished mental status or absent radial pulse (SBP less than 90 mm Hg)
b. Benefit: Fluids restore intravascular volume, reverse tissue hypoperfusion, and correct oxygen
debt.
c. Risk: Fluids do not increase oxygen-carrying capacity, can precipitate dilutional coagulopathy, and
lead to tissue edema, including the development of acute respiratory distress syndrome.
d. Crystalloids: Balanced crystalloids (e.g., lactated Ringer solution) and isotonic saline are both
recommended as resuscitation fluids. Isotonic saline is most commonly used, but it can cause
hyperchloremic metabolic acidosis and resultant acute kidney injury. Balanced crystalloids are
associated with less hyperchloremia in trauma patients, but it is unknown whether this improves
morbidity and mortality. Hypertonic saline resuscitation has failed to improve outcomes.
e. Colloids: Confer no incremental benefit over crystalloids, were associated with increased mortality
in a subgroup analysis of patients with severe brain injury in the SAFE trial, likely secondary to
hypo-osmolarity. Synthetic colloids (e.g., hydroxyethyl starches) contribute to coagulopathy and
the risk of acute kidney injury and should therefore be avoided.
f. Recommendations: Isotonic crystalloids are indicated in hemorrhagic shock. In trauma, restricted
volume replacement, usually less than 1.5 L of a balanced crystalloid, should be used initially, and
hypotonic solutions (e.g., lactated Ringer with a sodium content of 131 mmol/L) should be avoided
in patients with head trauma to minimize fluid shifts into the cerebral tissue. The most recent
European guidelines recommend isotonic balanced crystalloids over saline.
3. Packed red blood cells (PRBCs) and blood products
a. Indicated when the estimated blood loss is greater than 30% of the total blood volume
b. Amount of blood products to transfuse is based on clinical examination, given that the initial
hemoglobin or hematocrit reading may not reflect blood loss because of compensatory mechanisms.
c. Although there are no randomized controlled trials evaluating transfusion thresholds for trauma, the
European guidelines recommend maintaining a hemoglobin of 7–9 g/dL after initial resuscitation.
d. In acute upper GI bleeding, a restrictive transfusion threshold (Hgb less than 7 g/dL) compared with
a liberal transfusion threshold (Hgb less than 9 g/dL) was associated with a higher 6-week survival
rate (95% vs. 91%, hazard ratio [HR] 0.55 [95% confidence interval [CI], 0.33–0.92; p=0.02]) and
lower rates of further bleeding (10% vs. 16%, p=0.01) and adverse effects (40% vs. 48%, p=0.02).

ACCP Updates in Therapeutics® 2022: Critical Care Pharmacy Preparatory Review and Recertification Course

686
Shock Syndromes II: Hypovolemic, Critical Bleeding, and Obstructive

i. Notable limitations of the study include its single-center design and exclusion of patients with
cardiovascular disease.
ii. Despite these limitations, a transfusion threshold of 7 g/dL was recently endorsed by the 2017
updated variceal hemorrhage guidelines, with recommendations to maintain hemoglobin at
7–9 g/dL.
e. A randomized controlled trial of patients undergoing cardiac surgery compared a restrictive
transfusion threshold (less than 7.5 g/dL) with a liberal transfusion threshold (less than 9 g/dL).
i. The groups did not differ with respect to the composite primary end point of serious infection
or ischemic event (35.1% vs. 33.0%; OR 1.11; 95% CI, 0.91–1.34; p=0.30).
ii. However, more deaths occurred in the restrictive transfusion group (4.2% vs. 2.6%; HR 1.64;
95% CI, 1.00–2.67; p=0.045).
iii. Therefore, a restrictive transfusion strategy after cardiac surgery cannot be recommended.
4. Vasopressors
a. Attractive adjunct in hemorrhagic shock to minimize the amount of fluid required to reverse tissue
hypoperfusion, but can increase cardiac afterload and are independently associated with increased
mortality in trauma
b. Similar to other shock states, a relative arginine vasopressin deficiency develops in hemorrhagic
shock that is associated with catecholamine resistance, vasoplegia, and increased venous capacitance.
Supplemental vasopressin may be an adjunct to blood product resuscitation in hemorrhagic shock.
i. The AVERT-Shock trial was a single-center, randomized controlled trial investigating the
impact of low dose vasopressin (up to 0.04 units/min), compared to placebo, in adult trauma
patients who required at least 6 units of any blood product within 12 hours of injury. They
found that patients who received vasopressin required significantly less cumulative volume of
all blood products by around 1 L, and had a decreased incidence of deep vein thrombosis. No
difference was noted in other complications, such as acute respiratory distress syndrome, or
outcomes, including mortality.
ii. Although this study challenges the traditional practice of avoiding vasopressors in a bleeding
trauma patient, confirmatory research is needed to determine if vasopressin significantly
improves morbidity and mortality.
c. In life-threatening hypotension, vasopressors may be recommended only after hypovolemia has
been corrected or when cardiac arrest is imminent.
5. End points of resuscitation
a. General recommendations include permissive hypotension, defined as SBP 80–90 mm Hg for
most patients, and urinary output greater than 30 mL/hour. Up to 85% of patients may be under-
resuscitated using SBP and urinary output as end points.
b. In patients with traumatic brain injury (TBI), permissive hypotension should be avoided, given
that the 2016 Brain Trauma Foundation guidelines recommend maintaining SBP greater than 110
mm Hg for patients age 15–49 or older than 70 and greater than 100 mm Hg in patients age 50–69.

ACCP Updates in Therapeutics® 2022: Critical Care Pharmacy Preparatory Review and Recertification Course

687
Shock Syndromes II: Hypovolemic, Critical Bleeding, and Obstructive

Patient Case

1. A 67-year-old man is accidentally shot in the buttocks while deer hunting with his friends. He is brought
to the ED immediately. While in transfer, the patient receives 500 mL of lactated Ringer solution. His vital
signs on admission to the ED are as follows: blood pressure 104/58 mm Hg, heart rate 108 beats/minute,
respiratory rate 22 breaths/minute, and temperature 95°F (35°C). On examination, he has 8/10 pain and
appears anxious. Which is best for resuscitation strategies?
A. Administer lactated Ringer solution at 1000 mL/hour to maintain a urinary output greater than 30 mL/
hour and an SBP greater than 100 mm Hg.
B. Transfuse 2 units of PRBCs and administer a 1-L bolus of lactated Ringer solution to maintain a urinary
output greater than 1 mL/kg/hour.
C. Transfuse 2 units of PRBCs, 2 units of fresh frozen plasma, and a 1-L bolus of lactated Ringer solution
to normal mentation.
D. Administer lactated Ringer solution at 1000 mL/hour to maintain a urinary output greater than 30 mL/
hour, an SBP greater than 80–90 mm Hg, and normal mentation.

F. Burn Resuscitation
1. Acute thermal injury triggers an inflammatory state that ultimately leads to third spacing of intravascular
fluid.
2. Fluid resuscitation is initiated to maintain perfusion to tissue beds and end-organ function. A
concomitant concern is to avoid over-administration of fluids, leading to abdominal or extremity
compartment syndrome, acute respiratory distress syndrome, and further third spacing.
3. American Burn Association practice guidelines promote fluid resuscitation to target a urinary output
of 0.5–1 mL/kg/hour in adults, with the initial rate determined by the total body surface area (TBSA)
burned and body weight. Given a lack of strong evidence, the American Burn Association recommends
against a preload-driven resuscitation target from an invasive monitor unless special circumstances
exist (e.g., older patients or inadequate response to treatment).
4. Many formulas exist to estimate crystalloid need, but the most common strategy is administering
crystalloid fluid using lactated Ringer solution by the Parkland formula: 4 mL x kg x % TBSA, with
half administered over the first 8 hours and the remaining half over the next 16 hours.
5. In practice, significantly more fluid is often given to burn patients than is predicted by traditional
formulas.
a. While maintaining a urinary output more than 0.5 mL/kg/hour, the average patient with a severe
burn receives around 6 mL/kg/% body surface area within the first 24 hours, leading to the potential
for “fluid creep” and associated complications of over-resuscitation.
b. Therefore, some formulas now advocate less initial fluid, such as the “modified” Brooke formula
(2 mL/kg/% body surface area).
c. The rule of 10 (% TBSA burn x 10 = initial rate) is a simplified equation that determines the
initial starting rate for fluid resuscitation, and subsequent rate changes are guided by maintaining
a urinary output greater than 0.5 mL/kg/hour.
6. Burn resuscitation formulas serve as an initial starting dose, and ongoing resuscitation should be
individualized.
7. Difficult to resuscitate: When a patient’s fluid requirement exceeds what is expected according to the
traditional formulas, albumin may be added. When added, the current fluid rate is maintained, but
albumin replaces 33%–66% of the total rate. High dose ascorbic acid (e.g., 66 mg/kg/hr) has been
shown to decrease total fluid requirements, but guidelines state more evidence is needed before this
therapy can be recommended routinely.

ACCP Updates in Therapeutics® 2022: Critical Care Pharmacy Preparatory Review and Recertification Course

688
Shock Syndromes II: Hypovolemic, Critical Bleeding, and Obstructive

Patient Case

2. A 29-year-old man (height 72 inches, weight 85 kg) is admitted to the burn unit with a 40% TBSA burn to
the lower extremities and buttocks after a fall into a molten slag at work in a steel mill. He received 500 mL
of normal saline during transfer to the hospital. On his presentation to the burn unit, the surgical resident
asks for your help in calculating the patient’s fluid resuscitation needs. Which is best for resuscitation?
A. 15 L of lactated Ringer solution over 24 hours; initiate at 1000 mL/hour for the first 12 hours, followed
by 250 mL/hour, titrating to goal urinary output of 1 mL/kg/hour.
B. 13 L of lactated Ringer solution over 24 hours; initiate at 813 mL/hour for the first 8 hours, followed by
406 mL/hour, titrating to goal urinary output of 0.5 mL/kg/hour.
C. 12 L of lactated Ringer solution over 12 hours; initiate at 1000 mL/hour, titrating to goal urinary output
of 0.5 mL/kg/hour.
D. 24 L of lactated Ringer solution over the first 24 hours; initiate at 1000 mL/hour, titrating to goal uri-
nary output of 0.5 mL/kg/hour.

G. Blood Product Resuscitation


1. Lethal triad: Hypothermia, acidosis, and coagulopathy
a. Hypothermia, severe acidemia (pH less than 7.20), and hypocalcemia inhibit the procoagulant
enzyme function.
b. Therefore, treatment of the acutely bleeding patient should prioritize warming the patient to a
temperature greater than 93.2°F (34°C), correcting acidosis to a pH greater than 7.20, and
administering calcium to an ionized calcium greater than 4.4 mg/dL (1.1 mmol/L).
2. Initial bleeding and coagulopathy should be managed with blood component therapy (e.g., plasma,
platelets, cryoprecipitate) to build whole blood, termed damage control resuscitation.
a. Standard massive transfusion protocols include at least a 1:1:1 strategy of PRBCs/plasma/platelets.
b. A recent multicenter, randomized trial (PROPPR) compared a balanced 1:1:1 damage control
resuscitation transfusion strategy of plasma/platelets/PRBCs with a ratio 1:1:2 in adult trauma
patients.
i. 1:1:1 group received 6 units of plasma, one dose of platelets (6 units of platelets), and 6 units of
PRBCs. Transfusion order was platelets first, followed by alternating plasma and PRBC units.
ii. 1:1:2 group received an initial container (and all subsequent odd-numbered containers) with 3
units of plasma, no doses of platelets, and 6 units of PRBCs. The second (and all subsequent
even-numbered containers) had 3 units of plasma, one dose of platelets (6 units of platelets),
and 6 units of PRBCs. Transfusion order was platelets first, followed by alternating 2 units of
PRBCs and 1 unit of plasma.
iii. 24-hour mortality (1:1:1 group of 12.7% vs. 1:1:2 group of 17.0%; OR 0.75; 95% CI, 0.52–1.08;
p=0.12) and 30-day mortality (1:1:1 group of 22.4% vs. 1:1:2 group of 26.1%; OR 0.86; 95% CI,
0.65–1.12; p=0.26) did not differ significantly.
iv. However, in the 1:1:1 arm, hemostasis was achieved in more patients (86.1% vs. 78.1%,
p=0.006), possibly highlighting the importance of early platelet transfusion, and death from
exsanguination at 24 hours was less (9.2% vs. 14.6%; p=0.03).
3. More recently, application of damage-control resuscitation in the prehospital setting has been investigated
to intervene as close to the time of injury as possible to prevent the development of coagulopathy,
irreversible shock, and the ensuing inflammatory response.

ACCP Updates in Therapeutics® 2022: Critical Care Pharmacy Preparatory Review and Recertification Course

689
Shock Syndromes II: Hypovolemic, Critical Bleeding, and Obstructive

a. The Prehospital Air Medical Plasma (PAMPer) trial was designed to determine the efficacy and
safety of prehospital administration of 2 units of plasma resuscitation with or without red blood
cells compared with standard of care (crystalloids with or without red blood cells) in severely
injured patients at risk of hemorrhagic shock.
i. Patients were eligible if they had at least one episode of hypotension (SBP less than 90 mm
Hg), tachycardia (heart rate greater than 108 beats/minute), or severe hypotension (SBP less
than 70 mm Hg) in the prehospital setting.
ii. 501 patients were enrolled, with an average prehospital transport time of around 40–42 minutes
in both treatment groups.
iii. After arrival at the trauma center, the PT ratio was lower in the plasma group (1.2 vs. 1.3;
p<0.01). Mortality rates at 24 hours (13.9% vs. 22.1%; p=0.02) and 30 days (23.2% vs. 33%;
p=0.03) were lower in the plasma group, with early separation in the Kaplan-Meier survival
curve at 3 hours. There was no difference in inflammation-mediated complications, including
multiorgan failure, acute respiratory distress syndrome, or nosocomial infections, nor did
transfusion-related reactions differ, suggesting prehospital plasma is safe.
b. In contrast, the benefits of prehospital plasma may not be as apparent in an urban setting with short
prehospital transport times. In a single-center study at Denver Health Medical Center, prehospital
plasma failed to improve mortality at 28 days (15% plasma vs. 10% control; p=0.37) and was
terminated early for futility after 144 enrollments.
c. Although logistical challenges are related to the storage and shelf life of plasma in a variety of
preparations, trauma centers may consider implementing prehospital plasma for patients at risk of
hemorrhagic shock, particularly in rural settings with longer transport times.

H. Massive Transfusion Protocol (MTP)


1. Defined as requiring greater than 10 units of PRBCs in a 24-hour period
2. Clinical scores have been developed to assist in early identification of patients who would benefit from
activation of the MTP. The Assessment of Blood Consumption (ABC) Score was recently utilized
in the PROPPR trial for study enrollment. Two or more of the following criteria are associated with
high sensitivity and specificity for requiring a MTP, including: penetrating mechanisms of injury, ED
systolic blood pressure less than 90 mm Hg, ED heart rate more than 120 bpm or a positive Focused
Assessment with Sonography (FAST) exam.
3. Current recommendations for the initial management of an expected massive hemorrhage include
PRBCs and plasma in a 2:1 or 1:1 ratio (European Trauma Guidelines; grade 1B).
4. Further resuscitation, including platelet transfusions, should be guided in a goal-directed fashion, using
either standard laboratory coagulation assays or viscoelastic tests (grade 1C). An example of goal-
directed resuscitation can be found in Table 3.
5. In a prospective study of 111 patients, TEG-guided resuscitation (e.g., Table 3) was superior to standard
laboratory coagulation-guided resuscitation with improved 6-hour mortality (7.1% vs. 21.8%; p=0.049)
and total mortality (19.6% vs. 36.4%; p=0.03) and required less plasma and platelets in the first 2 hours
of resuscitation. Notable limitations include that it was a single-center study and that TEG was available
as point of care.

ACCP Updates in Therapeutics® 2022: Critical Care Pharmacy Preparatory Review and Recertification Course

690
Shock Syndromes II: Hypovolemic, Critical Bleeding, and Obstructive

Table 3. Example of Goal-Directed Resuscitation


Conventional Coagulation Assay (CAA) Thromboelastography (TEG)
Initial activated clotting time > 140 s: 2 units of FFP, 10
INR ≥ 1.5: 2 units of FFP pack of cryoprecipitate, 1 apheresis platelet
Fibrinogen < 150 mg/dL: 10 pack of cryoprecipitate Activated clotting time > 110a s: 2 units of FFP
Plt < 100,000/mm3: 1 apheresis platelet Alpha angle < 63: 10 pack of cryoprecipitate
D-dimer > 0.5 mcg/mL: 1 g of tranexamic acid MA < 55 mm: 1 apheresis platelet
LY30b > 3%: 1 g of tranexamic acid
a
Equivalent to R-time > 10
b
LY30 is a reflection of fibrinolysis, similar to estimated percent lysis.
FFP = fresh frozen plasma; LY30 = percent decrease in amplitude after 30 minutes; MA = maximum amplitude; Plt = platelets.
Information from: Gonzalez E, Moore EE, Moore H, et al. Goal directed resuscitation of trauma induced coagulopathy: a pragmatic randomized clinical trial comparing
a viscoelastic assay to conventional coagulation assay. Ann Surg 2016;263:1051-9.

6. Some metabolic complications are associated with massive transfusions that are notable for
pharmacotherapy considerations.
a. Potassium abnormalities may include hypokalemia or hyperkalemia.
i. Hyperkalemia:
(a) ATPase pumps become deactivated in stored blood, leading to elevations in potassium
concentrations from 7 mEq/L to 77 mEq/L.
(b) After transfusion, the ATPase pump is restored, and potassium is shifted intracellularly.
(c) Although this generally leads to minimal sequelae, hyperkalemia may develop in those
with renal insufficiency or severe tissue injury. Rapid transfusion through a central venous
catheter has been associated with hyperkalemia-induced cardiac arrest.
(d) For patients at risk, the recommended management is to slow the infusion rate to less than
100–150 mL/minute.
ii. Hypokalemia: Secondary to restoration of the ATPase pump on transfusion, shifting potassium
intracellularly, metabolic alkalosis from citrate administration, release of hormones (e.g.,
aldosterone, antidiuretic hormone), and co-infusion of potassium-poor solutions (e.g., normal
saline)
iii. Potassium should be closely monitoring and managed in a massive transfusion.
b. Complications secondary to citrate:
i. Each unit of stored blood is anticoagulated with around 3 g of citrate. The metabolic capacity
of the liver for citrate metabolism is around 3 g every 5 minutes, which is diminished with
underlying liver dysfunction or in shock states.
ii. Accumulation of citrate after several transfusions may bind endogenous calcium, inducing
severe hypocalcemia, prolonged QT intervals, circulatory depression, hypotension, tremors,
and pulseless electrical activity.
iii. Therefore, it is critically important to monitor ionized calcium concentrations and to administer
calcium chloride or calcium gluconate to maintain normal concentrations.
iv. Evidence is currently lacking regarding optimal calcium dosing in a massive transfusion.
Calcium chloride 1–3 g is often the preferred dose and preparation in the setting of MTP due
to the higher potency of elemental calcium relative to calcium gluconate. The total elemental
calcium concentration in calcium chloride 10% and calcium gluconate 10% is 270 mg/10 mL
and 90 mg/10 mL, respectively.
v. Magnesium may also be bound by citrate, necessitating appropriate monitoring and
management.

ACCP Updates in Therapeutics® 2022: Critical Care Pharmacy Preparatory Review and Recertification Course

691
Shock Syndromes II: Hypovolemic, Critical Bleeding, and Obstructive

vi. Although citrate is metabolized to bicarbonate, which may induce metabolic alkalosis,
metabolic acidosis is more common in patients requiring a massive transfusion, given
hypoperfusion and anaerobic metabolism in a shock state.

I. Antifibrinolytics
1. Tranexamic acid
a. An antifibrinolytic agent that binds plasminogen to prevent the dissolution of a fibrin clot.
Tranexamic acid is a competitive inhibitor of plasmin and plasminogen.
b. CRASH-2 trial:
i. Tranexamic acid (1 g over 10 minutes of bolus, followed by a 1-g infusion over 8 hours) reduced
28-day mortality for all-cause trauma compared with placebo (relative risk [RR] 0.91; 95% CI,
0.85–0.97; p=0.0035).
ii. The number needed to treat is 67, and no difference between groups was noted in the risk of
vascular occlusive events.
iii. The greatest benefit for tranexamic acid occurred with administration within 3 hours of injury
(RR 0.79; 95% CI, 0.64–0.97; p=0.03), with an increased risk of death from bleeding after 3
hours from injury (RR 1.44; 95% CI, 1.12–1.8; p=0.004).
iv. Major limitations of this trial were that few patients were enrolled from regions with modern
trauma systems (e.g., blood product availability, massive transfusion protocol), and the baseline
mortality was higher than expected in North America or Europe.
c. CRASH-3 trial:
i. Evaluated the effects of tranexamic acid (same dosing regimen as used in CRASH-2) in
patients with TBI. Patients were enrolled within less than 3 hours of injury with either Glasgow
Coma Scale (GCS) score less than 13 or intracranial hemorrhage on CT scan and no major
extracranial bleeding.
ii. There was no difference in head injury–related death within 28 days with tranexamic acid
(18.5%) compared with placebo (19.8%) (RR 0.94; 95% CI, 0.86–1.02).
iii. Tranexamic acid reduced death in patients with mild to moderate TBI, GCS 9–15 (RR 0.78;
95% CI, 0.64–0·95), but not in patients with a severe head injury, GCS 3–8 (RR 0.99; 95% CI,
0.91–1.07) who already had extensive intracranial hemorrhage before treatment.
iv. Patients in CRASH-3 were sicker than those in CRASH-2, with head injury–related death 12%
in CRASH-2 versus 19% in CRASH-3.
d. In the MATTERs (Military Application of Tranexamic Acid in Trauma Emergency Resuscitation)
study, tranexamic acid reduced the absolute mortality by 6.5% in all patients and by 13.7% in those
requiring a massive transfusion, suggesting a greater benefit in those with a higher severity of
injury.
e. In the MATTERs II trial, adding fibrinogen (cryoprecipitate) to tranexamic acid further improved
in-hospital mortality compared with tranexamic acid alone (11.6% vs. 18.2%; p=0.03).
f. In trauma, tranexamic acid has not been shown to decrease the need for transfusion.
g. Recommendations related to tranexamic acid use in trauma:
i. When given, should be administered as early as possible to a trauma patient with bleeding or
within 3 hours of injury to a patient at risk of significant hemorrhage
ii. Some hospital systems have begun giving the tranexamic acid loading dose prehospital to
prevent full activation of fibrinolysis, with improved outcomes when the loading dose was
given within 1 hour of injury. This strategy requires validation in a clinical trial.
iii. More recently, additional safety concerns have been identified related to tranexamic acid in
trauma patients.
(a) Although not shown in initial trials, tranexamic acid was subsequently associated with
thrombotic events, including PE and venous thromboembolism, in a large military trial.

ACCP Updates in Therapeutics® 2022: Critical Care Pharmacy Preparatory Review and Recertification Course

692
Shock Syndromes II: Hypovolemic, Critical Bleeding, and Obstructive

(b) In addition, tranexamic acid may be associated with fibrinolysis shutdown, a state of
suppressed fibrinolysis associated with high mortality from multiorgan failure.
iv. Because of criticisms of previous trials and potential safety concerns, some experts advocate
the use of tranexamic acid only in select patients having a diagnosis of hyperfibrinolysis, but
no study on selective use of tranexamic acid has shown improved outcomes.
v. In the United States, the Eastern Association for the Surgery of Trauma (EAST) guidelines for
damage control resuscitation conditionally recommended tranexamic acid in severely injured
trauma patients.
h. Tranexamic acid in postpartum hemorrhage:
i. For postpartum hemorrhage, a tranexamic acid 1-g bolus (which may be repeated at 30
minutes), investigated in the WOMAN trial, did not affect all-cause mortality but did reduce
mortality from bleeding if given within 3 hours (1.2% vs. 1.7%; p=0.045).
ii. The World Health Organization strongly recommends tranexamic acid within 3 hours for
postpartum hemorrhage.
i. The importance of early tranexamic acid was evaluated in an integrated analysis of both the
WOMAN trial and the CRASH2 trial. For every 15-minute delay to tranexamic acid, the overall
benefit was decreased by around 10%, highlighting the importance of early administration.
j. In cardiac surgery, perioperative tranexamic acid reduced blood product transfusions and rates
of major hemorrhage without a higher risk of death from thrombotic complications, but with an
increased risk of seizures (0.7% vs. 0.1%; p<0.01).
i. Of note, the tranexamic acid doses of 50–100 mg/kg used in this study are much higher than
those used in trauma.
ii. The increased risk of seizures may be unique to cardiac surgery because the risk of stroke and
seizures has been described after cardiac surgery, independent of tranexamic acid use.
2. Aminocaproic acid
a. A lysine analog that is 10-fold weaker than tranexamic acid
b. Recommended dose is a 150-mg/kg bolus, followed by a 15-mg/kg/hour infusion.
c. Compared with tranexamic acid, aminocaproic acid may cause more adverse drug reactions,
including hypotension and bradycardia, with rapid administration and rhabdomyolysis.
d. Lower quality of evidence exists for aminocaproic acid in the treatment of the acutely bleeding
patient.

J. Recombinant Activated Factor VIIa (rFVIIa)


1. Activates hemostasis through the extrinsic pathway of the coagulation cascade by interacting with
tissue factor to activate factor X to factor Xa and factor IX to factor IXa. Adequate concentrations of
platelets and fibrinogen are needed to support activity.
2. Case series and case reports suggest that rFVIIa improves clinical outcomes.
3. In trauma, a randomized controlled trial (CONTROL trial) was terminated early because of the futility
for the primary end point of mortality, though investigators did find a reduction in PRBC transfusion
with rFVIIa (200 mcg/kg followed by 100 mcg/kg at 1 hour and 3 hours) in patients with blunt trauma.
4. There is no consensus on rFVIIa use for acute bleeding. Institutional protocols vary with respect to
appropriate indication, timing, dosing, and readministration of rFVIIa.
5. Database reviews document an increased incidence of venous and arterial thromboembolic events with
off-label rFVIIa use for non-hemophilia indications.
6. In trauma, rFVIIa should only be considered if major bleeding and traumatic coagulopathy persist
despite all other attempts to control bleeding and optimize the physiologic environment, including
correction in acidosis, hypothermia, and hypocalcemia.

ACCP Updates in Therapeutics® 2022: Critical Care Pharmacy Preparatory Review and Recertification Course

693
Shock Syndromes II: Hypovolemic, Critical Bleeding, and Obstructive

K. Prothrombin Complex Concentrates (PCCs)


1. A combination of concentrated clotting factors II, VII, IX, and X and proteins C and S
a. Four different forms of PCC are available in the United States. Factor content of the PCCs varies,
particularly for factor VII (Table 4).
b. FEIBA (factor eight inhibitor bypassing activity) is the only PCC composed of activated clotting
factors, making it an activated prothrombin complex concentrate (aPCC). Although theoretically
more efficacious, this may also lead to increased thrombotic events.
2. Kcentra, a nonactivated 4PCC, has U.S. Food and Drug Administration (FDA)-approved labeling for
use in the reversal of warfarin-related acute bleeding disorders (further discussed under Reversal of
Oral Anticoagulant Agents) in life-threatening hemorrhage or for urgent surgery or invasive procedures.
a. Not indicated for the treatment of hemophilia-related bleeding because other PCC products are
approved.
b. The package insert warns of an increased thromboembolic risk with Kcentra administration.
Kcentra is contraindicated in patients with known heparin-induced thrombocytopenia because the
product contains heparin.
c. Few data support the off-label use of PCCs as a hemostatic agent in a bleeding patient who was not
previously receiving an anticoagulant agent; however, interest in this use and evidence is growing.
i. Recently, the RETIC trial, which used PCC and concentrated fibrinogen compared with plasma
in trauma patients with bleeding, was terminated early for safety concerns because of the high
need of rescue therapy and massive transfusion in the plasma group.
ii. Although use of coagulation factor concentrates (CFCs) for hemostatic resuscitation is less
common in the United States, current European guidelines recommend CFCs as an alternative
to plasma.

Table 4. Factor Content of PCCs Available in the United States


Vial Dose
Factor Factor Factor Factor
Additivesb (estimated
IIa (IU) VIIa (IU) IXa (IU)a Xa (IU)
factor IX IU)
3-factor PCCc
Bebulin VH 100 <5 100 100 Trace 200–1200
Profilnine 150 35 100 100 heparin N/A 500, 1000, 1500
Protein C, S, Z,
4PCCd
antithrombin III,
Kcentra 106.9 55.1 100 141.4 500, 1000
heparin
aPCCe
FEIBA 91.7–125 68-135 100 80–93.3 Protein C 500, 1000, 2500
a
Expressed as international units (IU) per 100 IU of factor IX; the individual factor contents vary depending on vial size and can be determined by multiplying the
individual factor content of interest by vial dose and dividing by 100 IU of factor IX equivalent.
b
Anticoagulant factors in some PCC products are added to attenuate excessive thrombogenicity.
c
Considered a 3-factor PCC because of limited amounts of factor VII.
d
Considered a 4-factor PCC (4PCC) because of a significant concentration of factor VII.
e
FEIBA is an aPCC that contains mainly nonactivated factors II, IX, and X; factor VII is mainly in the activated form.
FEIBA = factor eight inhibitor bypassing activity; N/A = not applicable.
Information from: Frontera JA, Lewin JJ, Rabinstein AA, et al. Guideline for reversal of antithrombotics in intracranial hemorrhage. Neurocrit Care 2016;24:6-46.

ACCP Updates in Therapeutics® 2022: Critical Care Pharmacy Preparatory Review and Recertification Course

694
Shock Syndromes II: Hypovolemic, Critical Bleeding, and Obstructive

Patient Case

Questions 3 and 4 pertain to the following case.


D.R. is a 37-year-old man with an unknown medical history who presents after a helmeted motorcycle collision
into a deer. On primary survey, his endotracheal tube is secured, respiratory rate is 37 breaths/minute, SBP is
72 mm Hg, heart rate is 141 beats/minute, and Glasgow Coma Scale score is 5T. His FAST examination is posi-
tive. A massive transfusion protocol is initiated, and the patient is taken to the operating room for an emergency
exploratory laparotomy.

3. Which is the best initial resuscitation strategy to manage his hypotension?


A. Initiate a sodium chloride (0.9%) 3000-mL bolus to maintain a MAP over 65 mm Hg.
B. Obtain an Hgb concentration to guide erythrocyte transfusions, with goal Hgb 7–9 g/dL.
C. Initiate a norepinephrine infusion to maintain an SBP goal of 80–90 mm Hg.
D. Initiate PBRCs, plasma, and platelets in a 1:1:1 ratio on the basis of clinical examination.

4. In the operating room, the patient is found to have grade 5 hepatic laceration and grade IV spleen injury
status after perihepatic packing and splenectomy. His abdomen is left open with a wound vacuum-assisted
closure, and he is taken to the ICU. On arrival at the ICU (5 hours after injury), an arterial blood gas reveals
a pH 7.18, partial pressure of carbon dioxide 22 mm Hg, lactate 12.7 g/dL, and Pao2 of 86% on 40% Fio2
with assist control ventilation. He has received 14 units of erythrocytes, 12 units of plasma, and 12 units of
platelets. His laboratory values include aPTT 37 seconds, INR 1.3, Hgb 7.5 g/dL, and Plt 154,000/mm3. His
temperature is 99°F (37.2°C) and ionized calcium is 0.7 mmol/L. Which is the best pharmacologic treatment
strategy to pursue next for managing his ongoing hemorrhagic shock in a goal-directed fashion?
A. Infuse 150 mEq of sodium bicarbonate in 1 L of sterile water at 200 mL/hour.
B. Give 1 g of calcium chloride once infused over 1 hour.
C. Give rFVIIa 90 mcg/kg once intravenous push.
D. Give a tranexamic acid 1000-mg bolus and 1000 mg infused over 8 hours.

L. Reversal of Oral Anticoagulant Agents (Table 5)


1. Patient selection
a. Assessment of bleed severity in patients treated with oral anticoagulants is critical to guide
decisions related to reversal.
b. The American College of Cardiology (ACC) 2017 expert consensus decision pathway suggests
consideration for anticoagulation reversal in a major bleed, generally defined as bleeding into a
critical site, hemodynamic instability, or clinically overt bleeding.
i. Critical bleed site:
(a) Bleeding in a critical site can compromise the organ function, may cause severe disability,
and can potentially require surgical intervention.
(b) These sites include: intracranial, other central nervous system (spine, ocular), thoracic
(e.g., pericardial tamponade, hemothorax), intra-abdominal, retroperitoneal, airway
bleeding associated with respiratory distress, or extremity bleeding (e.g., intraarticular,
intramuscular) causing compartment syndrome
(c) Intraluminal GI bleeding is not considered a critical site; however, GI bleeding may be
considered a major bleed on the basis of hemodynamic or overt bleeding criteria.

ACCP Updates in Therapeutics® 2022: Critical Care Pharmacy Preparatory Review and Recertification Course

695
Shock Syndromes II: Hypovolemic, Critical Bleeding, and Obstructive

ii. Hemodynamic instability is defined as a SBP less than 90 mm Hg, SBP decrease by at least
40 mm Hg, or orthostatic blood pressure changes (SBP at least 20 mm Hg, diastolic blood
pressure [DBP] at least 10 mm Hg upon standing).
iii. Overt bleeding, with a hemoglobin decrease of at least 2 g/dL or requiring the administration
of at least 2 units of PRBCs, is associated with increased mortality, particularly in patients
with chronic cardiovascular disease.
iv. Recommendations for managing a major bleed:
(a) Discontinue the anticoagulant, provide manual compression, provide supportive care,
assess and manage comorbidities that could contribute to bleeding (e.g., thrombocytopenia,
uremia, liver disease, antiplatelets), and consider surgical or procedural management of
the bleeding site.
(b) Specific anticoagulation reversal is indicated for bleeds at a critical site or those causing
hemodynamic instability. For overt bleeding, initial bleeding control procedures are
recommended initially, and reversal is indicated if the earlier measures fail to control the
bleed.
2. Warfarin
a. Warfarin has a long physiologic half-life; thus, the reversal decision is usually based on the presence
of an elevated INR in a life-threatening hemorrhage or the need for urgent surgery or an invasive
procedure within 24 hours.
b. Phytonadione administration is the definitive reversal of warfarin, promoting hepatic production of
vitamin K–dependent clotting factors depleted by warfarin.
i. In life-threatening hemorrhage, injectable phytonadione at a dose of 5–10 mg given
intravenously is preferred.
ii. Previous reports of anaphylaxis are related to solubilizing agents no longer included in the
formulation.
iii. Phytonadione has a delayed onset (6–12 hours); therefore, clotting factors are given concurrently
to expedite reversal.
c. Clotting factors can be given as either plasma or PCC.
i. Plasma has traditionally been used, but it requires large volumes, leads to incomplete INR
correction, requires compatibility testing, and requires an extended time to achieve hemostasis.
ii. Evidence to support 4PCC over plasma:
(a) Compared with plasma in life-threatening hemorrhage, 4PCC is superior for laboratory
reversal, is noninferior for clinical hemostasis, and has a similar rate of thromboembolic
events.
(b) In an urgent surgical or invasive procedure, 4PCC is superior for rapid INR reversal and
achievement of hemostasis.
(c) Integrated analyses from both life-threatening hemorrhage and urgent surgical trials
suggest similar rates of thromboembolic events (4PCC 7.3% vs. fresh frozen plasma 7.1%),
with fluid overload more common in the plasma group (4PCC 4.7% vs. 12.7%). Although
nonactivated 4PCC has a black box warning for an increased risk of arterial and venous
thromboembolic events, this risk appears to be similar to that for plasma.
(d) Although 4PCC is associated with a high direct acquisition cost, pharmacoeconomic
analyses are investigating strategies for warfarin reversal to fully understand the cost
implications.
iii. rFVIIa is no longer recommended for warfarin reversal because of incomplete correction of
factor concentrations inhibited by warfarin.
iv. Recommendations in a life-threatening hemorrhage:
(a) Phytonadione 10 mg intravenously and a 4PCC (dose is determined by patient weight and
pretreatment INR; see Table 6) are preferred for most patients.

ACCP Updates in Therapeutics® 2022: Critical Care Pharmacy Preparatory Review and Recertification Course

696
Shock Syndromes II: Hypovolemic, Critical Bleeding, and Obstructive

(b) Fixed dose 4PCC (e.g., 1000–1500 units) has been evaluated for warfarin reversal showing
correction of the INR, but the lower dose may fail to correct the INR to goal, necessitating
additional treatment. The ACC guidelines now consider low fixed dose as an alternative
to the package insert dosing, but this is off-label and the supporting evidence is of lower
quality.
(c) In patients with a contraindication to 4PCC (e.g., a history of heparin-induced
thrombocytopenia), fresh frozen plasma 10–15 mL/kg or non-heparin containing PCC
may replace 4PCC for reversal.
(d) Although the goal INR is controversial, Neurocritical Care Society guidelines recommend
trending an INR in patients with intracranial hemorrhage with repeat reversal strategies
until the INR is less than 1.4. Randomized controlled trials of 4PCC included only patients
with an INR of 2 or greater.
3. Direct oral anticoagulants (DOACs) – Dabigatran/apixaban/rivaroxaban/edoxaban:
a. Coagulation assays that have direct, linear relationships to the DOAC anticoagulation activity are
not available for clinical use (Table 5).
b. Therefore, the initial approach to reversal should be guided by clinical bleeding and the knowledge
of preexisting DOAC use rather than coagulation assays.
c. Given the relatively short DOAC half-life in patients with normal end-organ function and lack
of drug-drug interactions, establishing the time from the last dose is critical in determining the
likelihood that the drug is contributing to bleeding. In general, if it has been more than 3–5 half-
lives since the last dose, reversal is probably not indicated (Table 5).

Table 5. Summary of Oral Anticoagulants


Warfarin Dabigatran Apixaban Rivaroxaban Edoxaban
Action Vitamin K Direct factor IIa Direct factor Xa Direct factor Xa Direct factor Xa
antagonist inhibitor inhibitor inhibitor inhibitor
Peak action 4–5 days ~2 hr ~2 hr ~2 hr 1–2 hr
Half-life (hr)
CrCl: > 80 12–14 5–9 8–15 8.5
> 48 hr
CrCl: 50–79 17 9 15 9
CrCl: 30–49 19 9 18 9
CrCl < 30 28 10 17 9.5
Protime
aPTT and TT:
Rivaroxaban, edoxaban: Qualitative, if validated
Qualitative
Clinical coagulation Protime/INR: Apixaban: Insensitive
ECT and
monitoring Quantitative Chromogenic anti-Xa: Quantitative
dilute TT:
Heparin or low-molecular-weight heparin assay:
Quantitative
Qualitative
anti-Xa = anti-factor Xa; CrCl = creatinine clearance reported as mL/min/1.73 m2; ECT = ecarin clotting time; TT = thrombin time.
Information from: Dager WE. Developing a management plan for oral anticoagulation reversal. Am J Health Syst Pharm 2013;70:S21-31.

d. Activated charcoal may be considered if the last dose was administered less than 2 hours previously.
Given significant enterohepatic recirculation, charcoal can be given up to 6 hours after the last
apixaban dose.
e. Dabigatran reversal: Idarucizumab was recently studied and FDA approved for dabigatran reversal.
i. Idarucizumab is a monoclonal antibody that directly binds both bound and unbound dabigatran
with greater than 350 times the affinity of thrombin.

ACCP Updates in Therapeutics® 2022: Critical Care Pharmacy Preparatory Review and Recertification Course

697
Shock Syndromes II: Hypovolemic, Critical Bleeding, and Obstructive

ii. In the full analysis of 503 patients with life-threatening hemorrhage or need for urgent
procedure, all who received idarucizumab 5 g intravenously had maximum dilute TT reversal
by 4 hours after idarucizumab administration.
iii. However, given that dabigatran has a volume of distribution of greater than 50 L, about 23% of
patients develop redistribution of unbound dabigatran and associated coagulopathy at 12–24
hours. The efficacy of repeat doses in this setting has not been established. If there is ongoing
bleeding and suspicion for dabigatran activity, the package insert recommends obtaining an
aPTT to rule out the presence of dabigatran before a repeat dose is given.
iv. The thrombotic event rate was 6.3%–7.4%, where around two-thirds of patients had not
resumed an antithrombotic agent before the event.
v. Given the availability of idarucizumab, adjunctive reversal options such as dialysis should
only be considered if there are concurrent indications on the basis of renal function or if
idarucizumab is ineffective.
f. Factor Xa inhibitors: Andexanet alfa has also been studied for the reversal of factor Xa inhibitors and
was recently FDA approved for rivaroxaban and apixaban reversal in life-threatening hemorrhage.
i. Andexanet is a modified factor Xa decoy protein such that it directly binds to the factor Xa
inhibitors in a 1:1 ratio to inactivate their anticoagulant response.
(a) Andexanet’s structural modifications render it incapable of having prothrombotic or
antithrombotic effects.
(b) Mutation of the active serine site ensures that andexanet lacks the catalytic activity needed
to convert prothrombin to thrombin.
(c) Deletion of the GLA fragment makes andexanet unable to bind to phospholipids and
interact with factor V and calcium to form a prothrombinase complex.
ii. Andexanet reverses the effects of both direct (DOAC) and indirect (low-molecular-weight
heparin) factor Xa inhibitors.
iii. Promising results have been published of the ANNEXA-4 trial, which evaluated the effects
of andexanet alfa in patients with acute major bleeding and recent ingestion of factor Xa
inhibitors, predominantly apixaban and rivaroxaban.
(a) After an initial bolus of andexanet, results show 90%–92% correction of the anti-factor
Xa (anti-Xa) activity from baseline, which is sustained through the 2-hour infusion of the
study drug, allowing for a hemostatic plug to form.
(b) Of note, only 72% of patients met the predefined threshold for anticoagulation activity
(baseline anti-Xa concentrations greater than 75 ng/mL for DOACs or 0.5 IU/mL for
enoxaparin) to be included in this efficacy analysis.
(c) An important limitation of this study was the exclusion of patients expected to require
surgery within 12 hours, a population commonly encountered when treating bleeding
patients.
iv. Andexanet has a short half-life (1 hour); thus, redistribution and associated anticoagulant
activity are also common after discontinuing andexanet.
(a) In theory, this 2-hour time interval allows for a hemostatic plug to form and bleeding to
stop.
(b) In the ANNEXA-4 trial, 2 hours after discontinuing the infusion, anti-Xa activity
rebounded, decreasing by only 32%–42% relative to baseline.
(c) However, at 12 hours, 82% of patients had good or excellent clinical hemostasis, a co-
primary end point, which questions the clinical significance of the rebound effect.
(d) In patients with ongoing bleeding, patients with critical site bleeds, or surgical populations,
the significance of this rebound has not yet been determined.

ACCP Updates in Therapeutics® 2022: Critical Care Pharmacy Preparatory Review and Recertification Course

698
Shock Syndromes II: Hypovolemic, Critical Bleeding, and Obstructive

v. Anticipated limitations when using andexanet in clinical practice:


(a) The largest limitation of the ANNEXA-4 trial is the lack of a control group, with little
ability to determine how andexanet affects patient-centered outcomes.
(b) Given that only 72% of patients were eligible for the efficacy analysis, these results may
have an exaggerated effect relative to the general population.
(c) One of the greatest concerns is the relative short duration of laboratory reversal, given the
short pharmacodynamic half-life of andexanet alfa and its reversible bond with anti-Xa
inhibitors.
(d) This will raise several questions in clinical practice for patients who continue to have
clinical evidence of bleeding beyond the andexanet infusion and the need to repeat doses
or extend infusions.
(e) In addition, a notable exclusion from this trial were patients with severe brain injuries,
because the average GCS score was 14, or patients requiring surgery. Implications of
short-term reversal may be even greater for these high-acuity patient populations.
vi. Safety of andexanet alfa:
(a) 10% of the population had a thrombotic event, with most occurring before the initiation
of therapeutic anticoagulation.
(b) In an updated analysis presented at the ACC, the thrombotic event rate was 11%. Andexanet
does bind to tissue factor pathway inhibitor, a natural, endogenous factor Xa inhibitor.
(c) Although these updated data suggest an improved thrombotic rate, 11% is generally higher
than what has been shown for other reversal agents in bleeding patients, including PCCs
for warfarin reversal (7%–8%) and idarucizumab for dabigatran reversal (6.35%–7.4%).
(d) When interpreting this, it is important to recognize differences in population risks that
confound cross-study comparisons, given that the updated ANNEXA-4 analysis included
at least double the population with intracranial hemorrhage compared with other reversal
studies. In this setting, reintroduction of anticoagulation may be delayed.
vii. Andexanet dosing is complex (Table 6) and individualized according to the underlying
anticoagulant and the time from the last dose. For example, rivaroxaban achieves higher serum
concentrations, so andexanet doses are higher for rivaroxaban with recent administration than
for apixaban.
viii. Although andexanet may work for all direct and indirect anti-Xa inhibitors, it is only FDA
approved for apixaban and rivaroxaban reversal in life-threatening or uncontrolled bleeding.
(a) Use of other anticoagulants would be off-label, with overall lower supporting evidence.
(b) If used for edoxaban or enoxaparin, the dose currently being investigated is high dose.
g. The novel antidotes for DOAC reversal were FDA reviewed under the Fast Track designation.
i. Because it is unethical to randomize a patient to placebo during antidote trials, the studies
establishing efficacy and safety are single-arm trials.
ii. The FDA has requested an andexanet arm compared with a usual care arm for anti-Xa reversal
in patients with intracranial hemorrhage, but results are not expected for many years.
iii. Therefore, it remains unknown whether the novel antidotes will affect morbidity or mortality
associated with anticoagulants in life-threatening hemorrhage.
h. PCCs for DOAC reversal
i. PCCs and aPCCs have traditionally been used off-label as a nonspecific reversal agent for
DOAC reversal before the development of novel antidotes.
ii. Supporting data are primarily based on laboratory reversal of factor Xa inhibitors from in
vitro animal models, ex vivo human samples, and healthy volunteer subjects. These trials have
important limitations and inconsistent results.

ACCP Updates in Therapeutics® 2022: Critical Care Pharmacy Preparatory Review and Recertification Course

699
Shock Syndromes II: Hypovolemic, Critical Bleeding, and Obstructive

iii. More recently, PCC was an effective reversal for anti-Xa inhibitors in bleeding patients, as
evaluated by hemostatic response at defined time intervals, but these trials are limited by
single-arm observational design and lack of analysis of laboratory reversal.
iv. PCC has not consistently been shown to reverse laboratory values associated with dabigatran.
v. Several guidelines recommended 4PCC or aPCC administration at 25–50 units/kg for anti-Xa
reversal before the availability of andexanet. It is not yet known how guideline recommendations
will evolve with the availability of andexanet, especially considering the duration of reversal
is not sustained, nor has it been studied in patients requiring emergency surgical procedures

Table 6. Anticoagulant Reversal Agent Dosing in Life-Threatening Hemorrhage


Which Anticoagulant Reversal Agent Dose
INR 2–4: 25 units/kg (max 2500 units)
Warfarin with elevated INR 4PCC INR 4–6: 35 units/kg (max 3500 units)
INR > 6: 50 units/kg (max 5000 units)
Dabigatran within 3–5 expected half-lives 5 g IV push once
Idarucizumab
Limited data to repeat 5 g
Factor Xa inhibitor within 3–5 expected half-lives
Agent Last Dose
≤ 10 mg – Low dose Low dose: 400-mg bolus, 4 mg/min for
Rivaroxaban > 10 mg within 8 hr: High dose 120 min
> 10 mg after 8 hr: Low dose
High dose: 800-mg bolus and 8 mg/min
≤ 5 mg – Low dose Andexanet
for 120 min
Apixaban > 5 mg within 8 hr: High dose
> 5 mg after 8 hr: low dose If time from last dose not known, use
Edoxaban High dose at any time appropriate dose if consumed within 8 hr
Enoxaparin Off-label
4PCC
Direct and indirect anti-Xa inhibitors aPCC 25–50 units/kg
Off-label
IV = intravenous(ly).
Information from: Sarode R, Milling TJ, Refaai MA, et al. Efficacy and safety of 4PCC in patients on vitamin K antagonists presenting with major hemorrhage.
Circulation 2013;128:1234-43; Pollack CV, Reilly PA, Eikelboom J, et al. Idarucizumab for dabigatran reversal. N Engl J Med 2015;373:511-20; and Connolly SJ,
Milling TJ, Eikelboom JW, et al. Andexanet alfa for acute major bleeding associated with factor Xa inhibitors. N Engl J Med 2016;365:1131-41.

M. Reversal of Antiplatelet Agents (Table 7)


1. Clinical trials are controversial regarding the impact of antiplatelet agents on traumatic bleeding.
2. Recently, the first randomized controlled trial was published evaluating the impact of platelet
transfusions for spontaneous intracranial hemorrhage in patients receiving antiplatelet therapy.
a. In this study, the odds of death were higher in the platelet transfusion group at 3 months than in
placebo (OR 2.05; 1.18–3.56), suggesting platelet transfusions are inferior to standard of care in
non-operative patients.
b. Study limitations include the lack of platelet function tests, inability to ensure medication adherence
before the event, exclusion of surgical patients, and predominant use (greater than 90%) of aspirin
as the antiplatelet agent.
c. How this study translates to traumatic hemorrhage is unclear, but randomized controlled trials are
clearly needed.

ACCP Updates in Therapeutics® 2022: Critical Care Pharmacy Preparatory Review and Recertification Course

700
Shock Syndromes II: Hypovolemic, Critical Bleeding, and Obstructive

3. The European guidelines for managing traumatic bleeding recommend platelet transfusions if clinically
bleeding and platelet function assays show inhibition.
4. Desmopressin enhances platelet adherence and aggregation.
a. Although its clinical efficacy is controversial, desmopressin 0.3–0.4 mcg/kg may be considered to
reverse platelet-inhibiting drugs but may not be routinely used in trauma patients with bleeding.
b. There is a low risk of serious adverse effects, including hypervolemia, hyponatremia, facial
flushing, and rare thrombosis.

ACCP Updates in Therapeutics® 2022: Critical Care Pharmacy Preparatory Review and Recertification Course

701
Shock Syndromes II: Hypovolemic, Critical Bleeding, and Obstructive

Table 7. Antiplatelet Reversal in Life-Threatening Hemorrhage


Generic Drug Usual Duration
Coagulation Assay
(trade name): Half-Life of Platelet Recommendations
Effects
Mechanism of Action (hr) Inhibition
Aspirin Use caution with antiplatelet
Inhibits platelet activation Lifetime of reversal in patients with
Irreversible inhibition 3 platelets active or recent (within 3
of COX-1 and COX-2 5–7 days mo) coronary artery disease
enzymes Reversal strategies:
Optimal platelet
function assay Platelets*
Dipyridamole ± aspirin 2–3 days
(Aggrenox, Persantine) and thresholds (dipyridamole) Neurosurgical hemorrhage:
Inhibits platelet activation controversial 10–12 Lifetime of 1 apheresis unit (moderate
Increases endogenous platelets 5–7 days quality of evidence)
adenosine Light transmission for aspirin
platelet aggregation Neurosurgery is not
is gold standard of indicated: Platelet
platelet function transfusions controversial
Clopidogrel (Plavix)
testing but is not and not recommended by
Inhibits platelet activation Lifetime of
routinely available 6 the Neurocritical Care
Active metabolite platelets 5–7 days
Society (low quality of
irreversibly blocks ADP
TEG with evidence)
platelet mapping, -and/or-
Prasugrel (Effient) VerifyNow, platelet
function analyzer, Desmopressin (DDAVP)
Inhibits platelet activation Lifetime of
and bleeding time 2–15 0.4 mcg/kg IV push once
Active metabolite platelets 5–7 days
may be used (low quality of evidence)
irreversibly blocks ADP
*Transfused platelets with
ticagrelor may become
Ticagrelor (Brilinta)
inhibited because of
Inhibits platelet activation 7–9 3–5 days
reversible activity at the
Reversibly blocks ADP
P2Y12 receptor
AA = arachidonic acid; ADP = adenosine diphosphate; COX = cyclooxygenase.
Information from: Levi M, Eerenberg E, Kamphuisen PW, et al. Bleeding risk and reversal strategies for old and new anticoagulants and antiplatelet agents. J
Thromb Haemost 2011;9:1705-12; Mega J, Simon T. Pharmacology of antithrombotic drugs: an assessment of oral antiplatelet and anticoagulant treatments. Lancet
2015;386:281-91; Rossant R, Bouillon B, Cerny V, et al. The European guideline on management of major bleeding and coagulopathy following trauma, 4th ed. Crit
Care 2016;20:R100; Frontera JA, Lewin JJ, Rabinstein AA, et al. Guideline for reversal of antithrombotics in intracranial hemorrhage. Neurocrit Care 2016;24:6-46.

ACCP Updates in Therapeutics® 2022: Critical Care Pharmacy Preparatory Review and Recertification Course

702
Shock Syndromes II: Hypovolemic, Critical Bleeding, and Obstructive

Patient Cases

5. A 54-year-old man with an unknown medical history presents after a rollover automobile accident. On pri-
mary survey, his respiratory rate is 34 breaths/minute, SBP is 79 mm Hg, and Glasgow Coma Scale score
is 3. The FAST examination is positive, a massive transfusion protocol is initiated, and the patient is taken
to the operating room for an emergency exploratory laparotomy. In the operating room, he is found to have
grade 5 splenic injury, so the spleen is removed. Four hours after admission, he has received 14 units of
PRBCs, 12 units of fresh frozen plasma, and 10 units of platelets. His laboratory values include aPTT 666
seconds, INR 1.7, fibrinogen 176 mg/dL, Plt 160,000/mm3, and ionized calcium 1.1 mmol/L. The TEG is
normal except for the “lysis” or percent decrease in amplitude after 30 minutes of 0% (normal 0.8%–8%).
The patient’s wife arrives at the hospital and clarifies that he was out for a Sunday afternoon ride. She also
identifies that the patient has a medical history of atrial fibrillation and takes dabigatran (last taken this
morning). Which is the best pharmacologic treatment of his ongoing hemorrhagic shock?
A. Administer 4PCC 50 units/kg.
B. Administer a tranexamic acid 1-g bolus with 1-g infusion over 8 hours.
C. Administer an idarucizumab 5-g intravenous push.
D. Administer an idarucizumab 5-g intravenous push and a tranexamic acid 1-g bolus, followed by 1 g
infused over 8 hours.

6. B.P is a 76-year-old man (120 kg) with atrial fibrillation on warfarin admitted to the ED with weakness and
hematemesis. Pertinent vital signs on admission are as follows: blood pressure 79/52 mm Hg, heart rate 136
beats/minute with a rhythm of sinus tachycardia, and respiratory rate 26 breaths/minute. Frank red blood
is noted on nasogastric lavage. On physical examination, the patient is confused, with clammy extremities.
Pertinent laboratory values are as follows: Hgb 6.3 g/dL, INR 9.2, and Plt 300,000/mm3. In addition to intra-
venous phytonadione, which is best for managing warfarin reversal in this patient?
A. 4PCC 1000 units.
B. 4PCC 5000 units.
C. rFVIIa 1-mg intravenous push.
D. 4PCC 6000 units.

ACCP Updates in Therapeutics® 2022: Critical Care Pharmacy Preparatory Review and Recertification Course

703
Shock Syndromes II: Hypovolemic, Critical Bleeding, and Obstructive

II.  OBSTRUCTIVE SHOCK

A. Etiology and Epidemiology


1. Obstructive shock occurs because of extracardiac obstruction to flow in the cardiovascular system.
2. The source of extracardiac obstruction may be either impaired diastolic filling (e.g., cardiac tamponade,
tension pneumothorax, or constrictive pericarditis) or impaired systolic contraction (e.g., massive PE,
acute or chronic PH or aortic dissection).
3. Obstructive shock is relatively rare, and only about 2% of patients requiring vasoactive medications
have this type of shock.

B. Pathophysiology
1. The pathophysiologic hemodynamic hallmark of obstructive shock is decreased cardiac output (CO).
2. The specific pathophysiologic derangements depend on the underlying cause of the extracardiac
obstruction but are generalized as follows.
a. In impaired diastolic filling, RV preload is significantly decreased because of the inhibition of
venous return.
i. In cardiac tamponade, an accumulation of fluid in the pericardium leads to an increase in
pericardial pressure.
ii. This increases and equalizes diastolic pressures between the left and right heart (equalization of
central venous pressure [CVP], pulmonary artery diastolic pressure, and pulmonary capillary
wedge pressure [PCWP] and impaired ventricular filling). CVP and PCWP elevations should
not be mistaken as representing an increase in ventricular volume (preload).
b. In impaired systolic function, ventricular afterload is acutely increased, leading to ventricular
failure.
i. This typically occurs in an acute RV afterload (pulmonary vascular resistance) increase caused
by a massive PE or in acute PH.
ii. An acute increase in LV afterload does not typically lead to shock.
iii. An acute rise in RV afterload leads to reduced RV CO and a subsequent decrease in LV
CO. Subsequently, systemic hypotension develops, leading to reduced RV tissue perfusion
(decreased right coronary artery perfusion), RV free wall ischemia, reduced RV free wall
contractility, and further impairment of RV CO (a vicious cycle).
iv. In addition, acute RV pressure overload leads to a shift of the intraventricular septum toward
the LV, impairing LV diastolic filling (because of intraventricular dependence) and further
decreasing in LV CO.

C. Resuscitation and Treatment


1. Fluid administration and vasoactive medications may be used as a temporizing measure to increase
tissue perfusion.
a. Intravenous fluids (typically crystalloids) are usually recommended, but they may not improve CO.
i. Cardiac tamponade: Patients with preexisting hypovolemia may respond to fluids, but in
general, hemodynamics may not improve with fluid administration. Despite this finding, fluid
administration is usually recommended in cardiac tamponade.
ii. Massive PE: Initial fluid administration improves CO, but care should be taken because
excessive fluid administration can lead to further RV dilation and impaired LV CO from
worsened septal shifting and decreased LV filling (because of intraventricular dependence).
iii. Acute or chronic PH: Optimization of fluids in patients with acute or chronic PH is challenging.
Some patients, such as those with signs of intravascular volume depletion, may require fluids,
whereas others may require diuretics to reduce RV dilation and improve LV filling, even with
vasoactive medication administration.

ACCP Updates in Therapeutics® 2022: Critical Care Pharmacy Preparatory Review and Recertification Course

704
Shock Syndromes II: Hypovolemic, Critical Bleeding, and Obstructive

b. Vasopressors should be initiated to increase mean arterial pressure (MAP) and maintain an
adequate perfusion pressure.
i. This is particularly important in a massive PE because adequate right coronary artery perfusion
is of greatest importance to prevent/reduce RV free wall ischemia.
ii. Caution must be used, though, because catecholamine vasopressors may increase pulmonary
vascular resistance, which may worsen RV dysfunction.
c. Inotropes may increase RV CO in a massive PE or acute or chronic PH but are likely ineffective in
tamponade.
d. Inhaled nitric oxide or aerosolized prostacyclin therapy may decrease RV afterload in acute or
chronic PH, but neither therapy is likely effective for massive PE or cardiac tamponade.
2. Definitive treatment of the extracardiac obstruction is paramount.
a. Impaired diastolic filling
i. Cardiac tamponade: Pericardiocentesis or surgical evacuation and potential drain placement
ii. Tension pneumothorax: Needle decompression and potential chest tube thoracostomy
b. Impaired systolic function (massive PE): Embolism dissolution (thrombolytic therapy) or removal
(surgical or catheter thrombectomy)
i. Thrombolytic agents (Table 8) bind to the plasminogen/plasmin complex, which may be either
circulating or bound to the clot surface.
ii. This binding hydrolyzes a peptide bond to form free plasmin. Circulating plasmin is quickly
neutralized by α-antiplasmin, but fibrin-bound plasmin subsequently hydrolyzes bonds within
the clot matrix, leading to clot lysis.
iii. As such, thrombolytic agents can lead to rapid PE dissolution and a subsequent decrease in RV
afterload (pulmonary vascular resistance), but they may also cause bleeding.

Table 8. Thrombolytic Agents for Acute PE


Dose
Drug Pearl
Initial Infusion
10 mg over 10 min 90 mg over 2 hr Fibrin selective half-life: 5 min
Alteplasea Dose in cardiac arrest may be
100 mg infused over 2 hr
50 mg
Fibrin selective half-life: 13
Reteplaseb Two doses of 10 units IV push given 30 min apart
min
< 60 kg: 30-mg push
Fibrin selective (14-fold) half-
61–70 kg: 35-mg push
life: 90–130 min; only need
Tenecteplaseb 71–80 kg: 40-mg push N/A
bolus dose
81–90 kg: 45-mg push
Less expensive than alteplase
> 91 kg: 50-mg push
Nonselective
Streptokinasea 250,000 units over 30 min 100,000 units/hr over 24 hr
Antigenic
Urokinasea 4400 units/kg over 10 min 4400 units/kg/hr over 12 hr Nonselective
a
Indicates FDA approved for the treatment of PE.
b
Identifies drug is not FDA approved for the treatment of PE.
PE = pulmonary embolism.
Information modified from: Kearon C, Akl EA, Comerota AJ, et al. Antithrombotic therapy for VTE disease: Antithrombotic Therapy and Prevention of Thrombosis,
9th ed: American College of Chest Physicians Evidence-Based Clinical Practice Guidelines. Chest 2012;141:e419S-94S; and Daley MJ, Lat I. Clinical controversies in
thrombolytic therapy for the management of acute pulmonary embolism. Pharmacotherapy 2012;32:158-72.

ACCP Updates in Therapeutics® 2022: Critical Care Pharmacy Preparatory Review and Recertification Course

705
Shock Syndromes II: Hypovolemic, Critical Bleeding, and Obstructive

iv. Thrombolytic agents do not decrease mortality when evaluated among all patients with a PE
compared with heparin alone (6.7% vs. 9.6%; OR 0.67 [95% CI, 0.40–1.12]), but they may
improve outcomes in patients with an increased risk of death. As such, early PE-related
mortality risk stratification is necessary to guide thrombolytic therapy administration.
(a) High-risk or massive PE. Defined not by clot burden, but by acute PE causing hemodynamic
changes: Hypotension (SBP less than 90 mm Hg for at least 15 minutes or requiring
vasoactive support not from another cause), pulselessness, or bradycardia (heart rate less
than 40 beats/minute with signs of shock)
(b) Intermediate-risk or submassive PE. Acute PE without systemic hypotension but with
either (1) RV dysfunction (RV dilation or systolic dysfunction on echocardiogram, RV
dilation on chest CT, brain natriuretic peptide greater than 90 pg/mL, N-terminal pro-
brain natriuretic peptide greater than 500 pg/mL, or electrocardiographic changes [new
complete or incomplete right bundle-branch block, anteroseptal ST-segment elevation
or depression, or anteroseptal T-wave inversion]) or (2) myocardial necrosis (troponin I
greater than 0.4 ng/mL or troponin T greater than 0.1 ng/mL)
(c) Intermediate-risk PE may further be classified by the presence of both RV dysfunction and
myocardial necrosis, termed intermediate-high risk. If only RV dysfunction or myocardial
necrosis is present, it is classified as intermediate-low risk.
(d) Low-risk PE: Acute PE not meeting the definition of massive or submassive PE
v. Thrombolytics in cardiac arrest:
(a) Guidelines recommend either an alteplase bolus of 50 mg (repeated if needed to complete
a 100-mg intravenous total dose) or weight-based tenecteplase if PE is confirmed as the
contributing cause of the arrest.
(b) If PE is suspected but not confirmed, the guidelines state that evidence is insufficient to
either support or refute empiric systemic thrombolytics.
(c) These recommendations are based on possible improved outcomes with thrombolytics in
peri-arrest settings, but prospective evaluations are conflicting.
vi. Thrombolytics in massive PE:
(a) The CHEST guidelines and the European Society of Cardiology recommend systemic
thrombolytics for patients with a massive PE and an acceptable risk of bleeding
complications.
(b) These recommendations are supported by a meta-analysis, which found that in patients
with a massive PE, thrombolytics were associated with a lower rate of recurrent PE or
death than heparin alone (9.4% vs. 19.0%; OR 0.45 [95% CI, 0.22–0.92]).
(c) In addition, a recent analysis of a National Inpatient Sample containing over 70,000
patients with an unstable PE (defined as the presence of shock or a requirement for
mechanical ventilation) suggested reduced mortality attributable to PE with the use of
systemic thrombolytics (8.4% vs. 42%; p<0.01).
(d) Systemic thrombolytics for a massive PE are likely underused because about 70% of
patients in this database were not treated.
vii. Use of thrombolytics in submassive PE is controversial and should be based on the risk-benefit
profile:

ACCP Updates in Therapeutics® 2022: Critical Care Pharmacy Preparatory Review and Recertification Course

706
Shock Syndromes II: Hypovolemic, Critical Bleeding, and Obstructive

(a) In a study of patients with a submassive PE, adding alteplase 100 mg infused over 2 hours
to heparin compared with heparin plus placebo (heparin alone) was associated with a
lower rate of death or clinical deterioration requiring an escalation in treatment (11.0%
vs. 24.6%, p=0.006). This result was driven by more patients in the heparin-alone arm
who received secondary thrombolytics (23.2% vs. 7.6%, p=0.001), which may have been
influenced by the investigators’ ability to break the blinding of treatment allocation in the
study. Bleeding did not differ between study arms.
(b) A recently published study, known as PEITHO, evaluated patients with a submassive PE
(fulfilling both RV dysfunction and myocardial necrosis criteria) randomized to weight-
based tenecteplase plus heparin or placebo plus heparin (heparin alone).
(1) Between randomization and day 7, patients allocated to tenecteplase less commonly
died or had hemodynamic decompensation (2.6% vs. 5.6%, p=0.02; number needed to
treat 33 patients [95% CI, 18–162 patients]) but more commonly had major extracranial
bleeding (6.3% vs. 1.2%, p<0.001; number needed to harm 20 patients [95% CI, 13–35
patients]) and stroke (2.4% vs. 0.2%, p=0.003; number needed to harm 47 patients
[95% CI, 26–107 patients]).
(2) The difference in stroke incidence was driven by a higher incidence of hemorrhagic
stroke in the tenecteplase arm (2.0% vs. 0.2%, p=0.01; number needed to harm 57
patients [95% CI, 30–164 patients]).
(3) The overlapping 95% CIs for number needed to treat and number needed to harm for
clinically important outcomes call into question the risk-benefit profile of tenecteplase
for a submassive PE.
(c) Around 70% of patients from the PEITHO trial had a long-term follow-up for at least 24
months (average 37.8 months).
(1) Overall and cause-specific mortality at 30 days or long term did not differ in those
who received tenecteplase compared with placebo.
(2) Of the 290 patients who had echocardiograms with long-term follow-up, chronic
thromboembolic PH was confirmed in 2.1% of those who received tenecteplase
compared with 3.2% who did not (p=0.79).
(3) In addition, functional status, symptoms, RV dysfunction, and survival did not differ.
(d) Since publication of the trial of tenecteplase in intermediate-risk PE, the CHEST guidelines
and the European Society of Cardiology do not recommend routine systemic thrombolytics
for most patients. However, in select patients with signs or risk of deterioration and a low
bleeding risk, systemic thrombolytics may be considered.
viii. The risk-benefit of thrombolytics is best determined on a case-by-case basis by the bedside
clinician (Table 9).
(a) In the PEITHO trial, a subgroup analysis showed that age older than 75 may be associated
with a risk of major extracranial bleed (OR 20.38; 95% CI, 2.69–154.53; p=0.09).
(b) Commonly used bleeding scores such as the HAS-BLED moderately predict the risk of an
intracranial hemorrhage after thrombolytic agents for PE but lack precision.
ix. In extreme circumstances (e.g., massive PE with impending cardiac arrest), even the presence
of strong risk factors for bleeding may not preclude some clinicians from administering
thrombolytics because of the potential for benefit from the therapy. In these settings, even the
“contraindications” noted in the product labeling, other than active internal bleeding, may be
considered “relative contraindications” by some clinicians for thrombolytic administration.

ACCP Updates in Therapeutics® 2022: Critical Care Pharmacy Preparatory Review and Recertification Course

707
Shock Syndromes II: Hypovolemic, Critical Bleeding, and Obstructive

Table 9. Risk Factors for Bleeding After Thrombolytic Therapy


Alteplase package insert contraindications
Recent (within 3 mo) intracranial or intraspinal surgery or
Active internal bleeding
serious head trauma
Current cerebrovascular accident (intracranial
Current severe uncontrolled hypertensiona
hemorrhage or ischemic stroke)
Malignant intracranial neoplasm, arteriovenous
Known bleeding diathesisb
malformation, or aneurysm
Alteplase package insert warnings
Significant closed-head or facial trauma with radiographic
Recent major surgery
evidence of bony fracture or brain injury within 3 mo
Cerebrovascular disease Recent GI or genitourinary bleeding
Hemostatic defects (including those secondary to
Hypertension (SBP ≥ 175 mm Hg and/or DBP ≥ 110 mm Hg)
severe hepatic or renal disease)
Acute pericarditis Subacute bacterial endocarditis
Recent trauma Significant hepatic dysfunction
Age > 75 Needle puncture at noncompressible site
Left heart thrombus Pregnancy
Septic thrombophlebitis or occluded Diabetic hemorrhagic retinopathy or other hemorrhagic
arteriovenous cannula at seriously infected site ophthalmic conditions
Recent receipt of oral anticoagulants
Bleeding risk factors reported in published literature
Recent internal bleeding Suspected aortic dissection
Poorly controlled baseline hypertensiona Acute pancreatitis
Acute myocardial infarction Cardiopulmonary resuscitation > 10 min
Stool occult blood positive Bilirubin concentration > 3 mg/dL
Presence of an intra-aortic balloon pump Dementia
African American race Coagulopathyb
a
Undefined for PE, but noted as > 185 mm Hg systolic or > 110 mm Hg diastolic for acute ischemic stroke.
b
Undefined for PE, but noted as INR > 1.7 or Plt < 100,000/mm3 for acute ischemic stroke.
Adapted with permission from: Curtis GC, Lam SW, Reddy AJ, et al. Risk factors associated with bleeding after alteplase administration for pulmonary embolism:
a case-control study. Pharmacotherapy 2014;34:818-25; and Daley MJ, Lat I. Clinical controversies in thrombolytic therapy for the management of acute pulmonary
embolism. Pharmacotherapy 2012;32:158-72.

x. Catheter-directed thrombolysis (CDT) is an emerging treatment that enables the administration


of lower-dose thrombolytic agents directly into the thrombus.
(a) CDT has the advantage of concurrent mechanical disruption, including ultrasound-
assisted fragmentation, aspiration, or embolectomy.
(b) Applying CDT, several clinical trials have shown improved hemodynamics and strain on
the RV with minimal risk of a major bleed (less than 1%).
(c) However, prospective comparative studies evaluating long-term outcomes with CDT are
currently lacking, and the site experiences and resources required limit widespread use.
(d) The current CHEST guidelines recommend systemic thrombolytic therapy over CDT for
most patients; however, CDT may be considered in the presence of high bleeding risk,
failed systemic therapy, or requirement for time-sensitive intervention.

ACCP Updates in Therapeutics® 2022: Critical Care Pharmacy Preparatory Review and Recertification Course

708
Shock Syndromes II: Hypovolemic, Critical Bleeding, and Obstructive

xi. Reduced-dose thrombolytic therapy has been investigated as a strategy to minimize bleeding
risk.
(a) Several trials have evaluated “half-dose” alteplase compared with full dose, showing
improved hemodynamic parameters and potentially less bleeding, but all trials had severe
methodologic limitations that limit generalizability.
(b) More recently, a retrospective database of 3768 patients showed that patients who
received half-dose alteplase were more likely to require treatment escalation (53.8% vs.
41.4%; p<0.01) driven by secondary thrombolysis without notable difference in bleeding
complications.
(c) Given the available data, experts do not recommend consideration for half-dose in PE
management. For patients who need reperfusion treatment because of hemodynamic
decompensation but who have absolute or relative contraindications to systemic
fibrinolysis, CDT is recommended.
xii. Patients with a massive or submassive PE should be considered for surgical embolectomy or
catheter thrombectomy if they (1) have an unacceptably high risk of bleeding if administered
thrombolytics, (2) remain unstable despite thrombolytic administration, or (3) have shock
likely to cause death within hours (before the onset of systemic thrombolytics).
xiii. Unless contraindicated, all patients should also receive a parenteral anticoagulant.
(a) Intravenous unfractionated heparin is recommended over alternative agents for patients in
whom thrombolytic therapy is being considered or planned.
(b) The alteplase package insert recommends holding heparin during the alteplase infusion
and reinstituting it when the aPTT returns to less than 2 times the upper limit of normal.
(c) However, in some clinical trials, heparin was continued during thrombolytic administration.
(d) Therefore, concurrent heparin administration in practice varies, and this decision should
be individualized on the basis of risk assessment (e.g., bleeding vs. hemodynamics).
xiv. If anticoagulation is contraindicated, an inferior vena cava filter should be placed.

ACCP Updates in Therapeutics® 2022: Critical Care Pharmacy Preparatory Review and Recertification Course

709
Shock Syndromes II: Hypovolemic, Critical Bleeding, and Obstructive

Patient Cases

7. A 48-year-old woman (weight 75 kg) presents to the ED with shortness of breath. The patient’s hypoxemia
does not improve with supplemental oxygen, and her chest radiograph is not significant for any lung abnor-
malities. A contrasted chest CT scan reveals a PE in the subsegmental branch of the right pulmonary artery
and no RV dilation. The patient’s vital signs and significant laboratory values are as follows: heart rate 118
beats/minute, blood pressure 98/62 mm Hg, urinary output 1 mL/kg/hour, troponin T 0.06 ng/mL, brain
natriuretic peptide 60 pg/mL, lactate 0.9 mmol/L, and SCr 1.1 mg/dL. In addition to initiating a parenteral
anticoagulant, which is best for the patient?
A. Tenecteplase 40-mg bolus.
B. Alteplase 100-mg infusion over 2 hours.
C. Alteplase 50-mg bolus.
D. No thrombolytic therapy.

8. A 56-year-old man (weight 140 kg) with a history of smoking and chronic obstructive pulmonary disease
was admitted to the medical ICU with sudden-onset dyspnea, chest pain, and hypoxemia (Pao2 78%). A
chest CTA reveals a subsegmental PE. His blood pressure is 87/56 mm Hg, now requiring norepinephrine.
Which is the next best step to evaluate and/or treat his PE?
A. Administer tenecteplase 40-mg intravenous push, followed by a therapeutic heparin infusion.
B. Obtain an echocardiogram and cardiac enzymes to guide further risk stratification.
C. Administer catheter-directed alteplase 1 mg/hour for 12 hours by EKOS EndoWave system catheter.
D. Administer alteplase 100 mg infused over 2 hours, followed by a therapeutic heparin infusion.

ACCP Updates in Therapeutics® 2022: Critical Care Pharmacy Preparatory Review and Recertification Course

710
Shock Syndromes II: Hypovolemic, Critical Bleeding, and Obstructive

REFERENCES

Hypovolemic Shock a randomised, placebo-controlled trial. Lancet


1. Alvarado R, Chung KK, Cancio LC, et al. Burn 2019;394:1713-23.
resuscitation. Burns 2009;35:4-14. 13. Dager WE. Developing management plan for oral
2. Baharoglu MI, Cordonnier C, Salman RA, et al. anticoagulation reversal. Am J Health Syst Pharm
Platelet transfusion versus standard of care after 2013;70:S21-31.
acute stroke due to spontaneous cerebral hemor- 14. De Backer D, Biston P, Devriendt J, et al.
rhage associated with antiplatelet therapy. Lancet Comparison of dopamine and norepinephrine in the
2016;387:2605-13. treatment of shock. N Engl J Med 2010;362:779-89.
3. Boffard KD, Riou B, Warren B, et al. Recombinant 15. Delano MJ, Rizoli SB, Rhind SG, et al. Prehospital
factor VIIa as adjunctive therapy for bleeding resuscitation of traumatic hemorrhagic shock with
control in severely injured trauma patients: two hypertonic solutions worsens hypocoagulation
parallel randomized, placebo-controlled, double- and hyperfibrinolysis. Shock 2015;44:25-31.
blind clinical trials. J Trauma 2005;59:8-18. 16. Eriksson BI, Quinlan DJ, Weitz JI. Comparative
4. Bourke DL, Smith TC. Estimating allowable pharmacodynamics and pharmacokinetics of oral
hemodilution. Anesthesiology 1974;41:609-12. direct thrombin and factor Xa inhibitors in devel-
5. Brohi K, Singh J, Heron M, et al. Acute traumatic opment. Clin Pharmacokinet 2009;48:1-22.
coagulopathy. J Trauma Injury Infect Crit Care 17. Franchini M, Lippi G. Prothrombin complex con-
2003;54:1127-30. centrates: an update. Blood Transfus 2010;8:149-54.
6. Cannon JW. Hemorrhagic shock. N Engl J Med 18. Frontera JA, Lewin JJ, Rabinstein AA, et al.
2018;378:370-9. Guidelines for reversal of antithrombotics in intra-
7. Cannon JW, Khan MA, Raja AS, et al. Damage cranial hemorrhage. Neurocrit Care 2016;24:6-46.
control resuscitation in patients with severe 19. Garcia-Tsao G, Abraldes JG, Berzigotti A, et al.
traumatic hemorrhage: a practice management Portal hypertensive bleeding in cirrhosis: risk
guideline from the Eastern Association for the stratification, diagnosis, and management: 2016
Surgery of Trauma. J Trauma Acute Care Surg practice guidelines by the American Association
2017;82:605-17. for the Study of Liver Disease. Hepatology
8. Carney N, Totten AM, O’Reilly C, et al. 2017;65:310-35.
Guidelines for the management of severe trau- 20. Gayet-Ageron A, Prieto-Merino D, Ker K, et al.
matic brain injury, fourth edition. Neurosurgery Effect of treatment delay on the effectiveness and
2017;80:164-71. safety of antifibrinolytics in acute severe hem-
9. Committee on Trauma: Advanced Trauma Life orrhage: a meta-analysis of individual patient
Support Manual. Chicago: American College of level data from 40,138 bleeding patients. Lancet
Surgeons, 1997:103-12. 2018;391:125-32.
10. Connolly SJ, Crowther M, Eikelboom JW, et al. 21. Godier A, Samama CM, Susen S. Plasma/plate-
Full study report of andexanet alfa for bleeding lets/red blood cell ratio in the management of the
associated with factor Xa inhibitors. N Engl J Med bleeding traumatized patient: does it matter? Curr
2019;380:1326-35. Opin Anesth 2012;25:242-7.
11. CRASH-2 Trial Collaborators. Effects of 22. Goldstein JN, Refaai MA, Milling TJ, et al. 4PCC
tranexamic acid on death, vascular occlusive vs. plasma for rapid vitamin k antagonist reversal
events, and blood transfusion in trauma patients in patients needed urgent surgery or invasive inter-
with significant haemorrhage (CRASH-2): a ventions. Lancet 2015;385:2077-87.
randomised, placebo-controlled trial. Lancet 23. Gonzalez E, Moore EE, Moore H, et al. Goal
2010;376:23-32. directed resuscitation of trauma induced coagu-
12. CRASH-3 Trial Collaborators. Effects of lopathy: a pragmatic randomized clinical trial
tranexamic acid on death, disability, vascular comparing a viscoelastic assay to conventional
occlusive events and other morbidities in patients coagulation assay. Ann Surg 2016;263;1051-9.
with acute traumatic brain injury (CRASH-3):

ACCP Updates in Therapeutics® 2022: Critical Care Pharmacy Preparatory Review and Recertification Course

711
Shock Syndromes II: Hypovolemic, Critical Bleeding, and Obstructive

24. Gruen RL, Brohi LK, Schreiber M, et al. 36. Milling TJ, Refaai MA, Sarode R, et al. Safety a
Haemorrhage control in severely injured patients. four-factor prothrombin complex concentrate ver-
Lancet 2012;380:1099-108. sus plasma for vitamin K antagonist reversal. Acad
25. Gutierrez G, Reines HD, Wulf-Gutierrez ME. Emerg Med 2016;23:466-75.
Clinical review: hemorrhagic shock. Crit Care 37. Moore HB, Moore EE, Chapman MP, et al. Plasma
2004;8:373-81. first resuscitation to treat hemorrhagic shock dur-
26. Harrois A, Hamada SR, Duranteau J. Fluid resusci- ing emergency ground transportation in an urban
tation and vasopressors in severe trauma patients. area: a randomized trial. Lancet 2018;392:281-91.
Curr Opin Crit Care 2015;20:632-7. 38. Morrison JJ, Ross JD, Dubose JJ, et al. Association
27. Hauser CJ, Boffard K, Dutton R, et al. Results of of cryoprecipitate and tranexamic acid with
the CONTROL trial: efficacy and safety of recom- improved survival following wartime injury: find-
binant activated factor VII in the management ings from the MATTERs II study. JAMA Surg
of refractory traumatic hemorrhage. J Trauma 2013;148:218-25.
2010;69:489-500. 39. Murphy GJ, Pike K, Rogers CA, et al. Liberal or
28. Holcomb JB, Tilley BC, Baraniuk S, et al. restrictive transfusion after cardiac surgery. N
Transfusion of plasma, platelets, and red blood Engl J Med 2015;372:997-1008.
cells in a 1:1:1 vs. a 1:1:2 ratio and mortality in 40. Myles PS, Smith JA, Forbes A, et al. Tranexamic
patients with severe trauma: the PROPPR random- acid in patients undergoing coronary-artery sur-
ized clinical trial. JAMA 2015;313:471-82. gery. N Engl J Med 2016;376:136-48.
29. Innerhofer P, Fries D, Mittermayr M, et al. Reversal 41. Neal MD, Hoffman MK, Cuschieri J, et al.
of trauma-induced coagulopathy using first-line Crystalloid to packed red blood cell transfusion
coagulation factor concentrates of fresh frozen ratio in the massively transfused patient: when a
plasma (RETIC): a single-centre, parallel-group, little goes a long way. J Trauma Acute Care Surg
open-label, randomized trial. Lancet Haematol 2012;72:892-8.
2017;4:258-71. 42. Nutescu EA, Dager WE, Kalus JS, et al.
30. Kaatz S, Kouides PA, Garcia DA, et al. Guidance Management of bleeding and reversal strategies
on the emergent reversal of oral thrombin and fac- for oral anticoagulants: clinical practice consid-
tor Xa inhibitors. Am J Hematol 2012;87(suppl erations. Am J Health Syst Pharm 2013;70:e82-97.
1):S141-5. 43. O’Leary JG, Greenberg CS, Patton HM, et al.
31. Kauvar DS, Lefering R, Wade CE. Impact of AGA clinical practice update: coagulation in cir-
hemorrhage on trauma outcome: an overview of rhosis. Gastroenterology 2019;157:34-43.
epidemiology, clinical presentations, and thera- 44. Panteli M, Pountos I, Giannoudis PV.
peutic considerations. J Trauma 2006;60:S3-11. Pharmacologic adjuncts to stop bleeding. Eur J
32. Kcentra (prothrombin complex concentrate, Trauma Emerg Surg 2016;42:303-10.
human) [prescribing information]. Kankakee, IL: 45. Peltan ID, Vande Vusse LK, Maier RV, et al. An
CSL Behring, April 2013. international normalized ratio-based definition of
33. Lacroix J, Hebert PC, Fergusson DA, et al. Age of acute traumatic coagulopathy is associated with
transfused blood in critically ill patients. N Engl J mortality, venous thromboembolism, and mul-
Med 2015;372:1410-8. tiple organ failure after injury. Crit Care Med
34. McKinley BA, Kozar RA, Cocacnour CS, et al. 2015;43:1429-38.
Normal versus supranormal oxygen delivery goals 46. Pham TN, Cancio LC, Gibran NS. American Burn
in shock resuscitation: the response is the dame. J Association practice guidelines burn shock resus-
Trauma 2002;53:825-32. citation. J Burn Care Res 2008;29:259-66.
35. Meizoso JP, Dudaryk R, Mulder MB, et al. 47. Pollack CV, Reilly PA, Ryn J, et al. Idarucizumab
Increased risk of fibrinolysis shutdown among for dabigatran reversal – full cohort analysis. N
severely injured patients receiving tranexamic Engl J Med 2017;377:431-41.
acid. J Trauma Acute Care Surg 2018;84:426-32.

ACCP Updates in Therapeutics® 2022: Critical Care Pharmacy Preparatory Review and Recertification Course

712
Shock Syndromes II: Hypovolemic, Critical Bleeding, and Obstructive

48. Porter JM, Ivatury RR. In search of the optimal Obstructive Shock
end points of resuscitation in trauma patients: a 1. Angle MR, Molloy DW, Penner B, et al. The car-
review. J Trauma 1998;44:908-14. diopulmonary and renal hemodynamic effects of
49. Rossant R, Bouillon B, Cerny V, et al. Management norepinephrine in canine pulmonary embolism.
of bleeding following major trauma: an updated Chest 1989;95:1333-7.
European guideline. Crit Care 2010;14:R52. 2. Chatterjee S, Weinberg I, Yeh RW, et al. Risk
50. Sarode R, Milling TJ, Refaai MA, et al. Efficacy factors for intracranial haemorrhage in patients
and safety of 4PCC in patients on vitamin K with pulmonary embolism treated with thrombo-
antagonists presenting with major hemorrhage. lytic therapy. Development of the PE-CH score.
Circulation 2013;128:1234-43. Thromb Haemost 2017;117:246-51.
51. Siegal DM, Curnutte JT, Connolly SJ, et al. 3. Curtis GC, Lam SW, Reddy AJ, Bauer SR. Risk
Andexanet alfa for the reversal of factor Xa inhibi- factors associated with bleeding after alteplase
tor activity. N Engl J Med 2015;373:2413-24. administration for pulmonary embolism: a case-
52. Sihler KC, Napolitano LM. Complications of mas- control study. Pharmacotherapy 2014;34:818-25.
sive transfusion. Chest 2010;137:209-20. 4. Daley MJ, Lat I. Clinical controversies in
53. Sims CA, Holena D, Kim P, et al. Effect of low- thrombolytic therapy for the management of
dose supplementation of arginine vasopressin on acute pulmonary embolism. Pharmacotherapy
need for blood product transfusions in patients 2012;32:158-72.
with trauma and hemorrhagic shock. A random- 5. Greyson CR. Pathophysiology of right ventricular
ized clinical trial. JAMA Surg 2019;154:994-1003. failure. Crit Care Med 2008;36:S57-65.
54. Spahn DR, Bouillon B, Cerny V, et al. The 6. Hoeper MM, Granton J. Intensive care unit
European guideline on management of major management of patients with severe pulmonary
bleeding and coagulopathy following trauma: fifth hypertension and right heart failure. Am J Respir
edition. Crit Care 2019;23:98. Crit Care Med 2011;184:1114-24.
55. Sperry JL, Guyette FX, Brown JB, et al. 7. Jaff MR, McMurtry MS, Archer SL, et al.
Prehospital plasma during air medical transport in Management of massive and submassive pulmo-
trauma patients at risk for hemorrhagic shock. N nary embolism, iliofemoral deep vein thrombosis,
Engl J Med 2018;26:315-26. and chronic thromboembolic pulmonary hyper-
56. Tomaselli GF, Mahaffey KW, Cuker A, et al. tension: a scientific statement from the American
2017 ACC expert consensus decision pathway on Heart Association. Circulation 2011;123:1788-830.
management of bleeding in patients on oral antico- 8. Kearon C, Akl EA, Comerota AJ, et al.
agulants. J Am Coll Cardiol 2017;70:3042-67. Antithrombotic therapy for VTE disease:
57. Tripodi A, Mannucci PM. The coagulopa- Antithrombotic Therapy and Prevention of
thy of chronic liver disease. N Engl J Med Thrombosis, 9th ed: American College of Chest
2011;365:147-56. Physicians Evidence-Based Clinical Practice
58. Villaneuva C, Colomo A, Bosch A, et al. Guidelines. Chest 2012;141:e419S-94S.
Transfusion strategies for acute gastrointestinal 9. Kearon C, Akl EA, Ornelas J, et al. Antithrombotic
bleeding. N Engl J Med 2013;368:11-21. therapy for VTE disease: CHEST guideline and
59. WOMAN Trial Collaborators. Effect of early expert panel report. Chest 2016;149:315-52.
tranexamic acid administration on mortality, 10. Kerbaul F, Rondelet B, Motte S, et al. Effects of
hysterectomy, and other morbidities in women norepinephrine and dobutamine on pressure load-
with post-partum haemorrhage (WOMAN): an induced right ventricular failure. Crit Care Med
international, randomized, double-blind, placebo- 2004;32:1035-40.
controlled trial. Lancet 2017;389;2105-16. 11. Kerber RE, Gascho JA, Litchfield R, et al.
60. Yank V, Tuohy CV, Logan AC, et al. Systematic Hemodynamic effects of volume expansion and
review: benefits and harms of in-hospital use of nitroprusside compared with pericardiocentesis in
recombinant factor VIIa for off-label indications. patients with acute cardiac tamponade. N Engl J
Ann Intern Med 2011;154:529-40. Med 1982;307:929-31.

ACCP Updates in Therapeutics® 2022: Critical Care Pharmacy Preparatory Review and Recertification Course

713
Shock Syndromes II: Hypovolemic, Critical Bleeding, and Obstructive

12. Konstantinides S. Clinical practice. Acute pulmo-


nary embolism. N Engl J Med 2008;359:2804-13.
13. Konstantinides SV, Barco S, Lankeit M, et al.
Management of pulmonary embolism: an update.
J Am Coll Cardiol 2016;67:976-90.
14. Konstantinides SV, Meyer G, Becattini C, et al.
2019 ESC guidelines for the diagnosis and man-
agement of acute pulmonary embolism developed
in collaboration with the European Respiratory
Society (ERS). Eur Heart J. 2020;41:543-603.
15. Konstantinides SV, Vicaut E, Danays T, et al.
Impact of thrombolytic therapy on the long term
outcome of intermediate risk pulmonary embo-
lism. J Am Coll Cardiol 2017;69:1536-44.
16. Kucher N, Goldhaber SZ. Management of massive
pulmonary embolism. Circulation 2005;112:e28-32.
17. Lavonas EJ, Drennan IR, Gabrielli A, et al. 2015
American Heart Association guidelines update
for cardiopulmonary resuscitation and emergency
cardiovascular care. Circulation 2015;132:S501-18.
18. Maisch B, Seferovic PM, Ristic AD, et al.
Guidelines on the diagnosis and management
of pericardial diseases executive summary; the
task force on the diagnosis and management of
pericardial diseases of the European Society of
Cardiology. Eur Heart J 2004;25:587-610.
19. Meyer G, Vicaut E, Danays T, et al. Fibrinolysis
for patients with intermediate-risk pulmonary
embolism. N Engl J Med 2014;370:1402-11.
20. Mostafa A, Briasoulis A, Telila T, et al. Treatment
of massive or submassive acute pulmonary embo-
lism with catheter directed thrombolysis. Am J
Cardiol 2016;117:1014-20.
21. Stein PD, Matta F. Thrombolytic therapy in unsta-
ble patients with acute pulmonary embolism: saves
lives but underused. Am J Med 2012;125:465-70.
22. Todd JL, Tapson VF. Thrombolytic therapy for
acute pulmonary embolism: a critical appraisal.
Chest 2009;135:1321-9.
23. Wan S, Quinlan DJ, Agnelli G, et al. Thrombolysis
compared with heparin for the initial treat-
ment of pulmonary embolism: a meta-analysis
of the randomized controlled trials. Circulation
2004;110:744-9.

ACCP Updates in Therapeutics® 2022: Critical Care Pharmacy Preparatory Review and Recertification Course

714
Shock Syndromes II: Hypovolemic, Critical Bleeding, and Obstructive

ANSWERS AND EXPLANATIONS TO PATIENT CASES

1. Answer: D a normal hemoglobin likely will not reflect the extent of


This patient has class II hypovolemic shock after blood blood loss secondary to hemoconcentration until cap-
loss caused by penetrating trauma (heart rate greater illary recruitment is complete or resuscitation begins.
than 100 beats/minute, respiratory rate 20–30 breaths/ In addition, resuscitation efforts in a hemodynamically
minute, and anxious on examination). The appropri- unstable patients should not be delayed while awaiting
ate resuscitation strategy should focus on selecting the laboratory results (Answer B is incorrect). Although
fluid, volume, and resuscitation goal. Recommended vasopressors may be considered an adjunct to maintain
end points for resuscitation include SBP greater than 90 tissue perfusion while minimizing fluid volume, this
mm Hg, urinary output greater than 30 mL/hour, and strategy should be reserved for those with life-threaten-
normal mentation. Administering fluid to target an SBP ing hypotension despite resuscitation efforts (Answer C
greater than 90 mm Hg would be appropriate (Answer is incorrect).
A is incorrect). Blood products for transfusion are indi-
cated when the patient’s estimated blood loss is greater 4. Answer: B
than 30% (Answers B and C are incorrect because Acute hypocalcaemia is a common complication in a
the patient has class II hypovolemic shock). Given his massive transfusion secondary to citrate added to stored
hemorrhagic shock class, the patient may be initially blood. Calcium is essential for stabilization of fibrin,
managed with crystalloid boluses targeting a urinary and low ionized calcium concentrations in traumatic
output greater than 30 mL/hour, an SBP greater than 90 hemorrhage are associated with increased mortality.
mm Hg, and normal mentation (Answer D is correct). Therefore, ionized calcium concentrations should be
maintained within the normal range (Answer B is cor-
2. Answer: B rect). Evidence is insufficient to recommend sodium
Using the Parkland formula for burn resuscitation, the bicarbonate therapy in patients with acute trauma and
24-hour fluid requirement would be 13.6 L, to be seg- has the potential for increased mortality. The patient’s
mented into the first 8 hours of resuscitation and the acidosis should be corrected by addressing the under-
remaining 16 hours. Calculating fluid requirements lying cause – in this case, hemorrhage (Answer A is
according to weight and TBSA to target a urinary out- incorrect). Use of rFVIIa should only be considered
put of greater than 0.5 mL/kg/hour would be correct if major bleeding and persistent traumatic coagulopa-
(Answers A, C, and D are incorrect). Answer B is cor- thy persist despite best practice, inconsistent with the
rect because the total volume requirement accounting patient’s current condition. In addition, a pH less than
for prehospital fluid, titration of fluid rates, and target 7.2 is a predictor of a poor response to rFVIIa (Answer
urinary output are appropriate. C is incorrect). Unfortunately, this patient’s injury was 5
hours ago, and he has no laboratory evidence of hyper-
3. Answer: D fibrinolysis. In this setting, tranexamic acid does not
Given this patient’s hemodynamic parameters in a posi- improve mortality after 3 hours from injury and has
tive FAST examination, he will likely require a massive been associated with an increased risk of death from
transfusion, as assessed by the ABC (Assessments of bleeding (Answer D is incorrect).
Blood Consumption) score for a massive transfusion
protocol, which should be initiated promptly. A fixed 5. Answer: C
ratio of 1:1:1 PRBC, plasma, and platelets is more likely At this point, ongoing resuscitation should be goal
to achieve hemostasis and lower mortality from exsan- directed. The TEG reveals hypofibrinolysis or “shut-
guination at 24 hours than is 2:1:1 (Answer D is correct). down,” not hyperfibrinolysis for which tranexamic acid
Although fluid resuscitation is an adjunct to increase is expected to help. In addition, if there was no TEG to
intravascular volume and tissue perfusion, 0.9% sodium evaluate for hypofibrinolysis, tranexamic acid should be
chloride does not increase oxygen-carrying capacity, administered within the first 3 hours from injury, and
and total volume should be minimized to less than 1.5 this patient’s injuries occurred more than 4 hours ear-
L (Answer A is incorrect). In a traumatic hemorrhage, lier (Answers B and D are incorrect). With the updated

ACCP Updates in Therapeutics® 2022: Critical Care Pharmacy Preparatory Review and Recertification Course

715
Shock Syndromes II: Hypovolemic, Critical Bleeding, and Obstructive

history that the patient was taking dabigatran preinjury, 8. Answer: D


dabigatran is likely contributing to the patient’s ongoing The patient has a massive or high-risk PE, as evidenced
coagulopathy and hemorrhage. A normal TEG and ele- by signs of hemodynamic complications requiring nor-
vated aPTT may not rule out the presence of dabigatran. epinephrine, likely confounded by poor physiologic
In addition, it seems the patient took the last dabigatran reserve with chronic obstructive pulmonary disease.
dose within the previous 12 hours. Therefore, dabigatran The current guidelines recommend systemic throm-
reversal is indicated. Idarucizumab is FDA approved for bolysis in patients with a massive PE and an acceptable
dabigatran reversal and recommended in various guide- risk of bleeding. This patient does not appear to have
lines, whereas PCCs have weak supporting evidence any obvious risk of bleeding, including age; therefore,
and are recommended only if idarucizumab is unavail- systemic thrombolytic agents are indicated in addition
able (Answer C is correct; Answer A is incorrect). to a therapeutic heparin infusion. The most common
options include alteplase as a fixed dose of 100 mg
6. Answer: B infused over 2 hours or tenecteplase adjusted according
This patient has an acute GI hemorrhage complicated by to patient weight. Because this patient weighs 140 kg,
supratherapeutic warfarin and hemodynamic instability the recommended tenecteplase dose would be a 50-mg
necessitating warfarin reversal with 4PCC. Given his intravenous push once (Answer A is incorrect); there-
weight and degree of INR elevation, the package insert fore, alteplase 100 mg infused over 2 hours is the most
dose is 50 units/kg not to exceed 5000 units (Answer B appropriate answer (Answer D is correct). The 2016
is correct; Answer D is incorrect). Although fixed, low- CHEST guidelines state that if thrombolytic agents are
dose PCC has been evaluated for warfarin reversal, this indicated, systemic thrombolysis is preferred to cathe-
dose is off-label and supported by a lower quality of evi- ter-directed thrombolytic administration (Answer C is
dence. In addition, the 1000-unit dose has shown lower incorrect). Echocardiogram and cardiac enzymes are
achievement of a goal INR, necessitating rescue doses, not considered necessary to guide therapy in patients
and a low dose would probably not achieve adequate with hypotension/shock and a CTA-confirmed PE
laboratory reversal, given his weight and excessively (Answer B is incorrect).
high INR (Answer A is incorrect). Finally, although
rFVIIa lowers INR, it is no longer recommended for the
reversal of vitamin K antagonists, given the incomplete
correction of factors II, IX, and X and the possibility
that the INR does not reflect the patient’s underlying
coagulopathy (Answer C is incorrect).

7. Answer: D
The patient has evidence of a PE, but she lacks features
of an increased risk of early mortality from it. She does
not have shock or evidence of end-organ hypoperfusion
and thus does not have a massive PE. In addition, she
has no evidence of RV dysfunction (no RV dilation on
chest CT, brain natriuretic peptide less than 90 pg/mL)
or myocardial necrosis (troponin less than 0.1 mg/mL)
and thus does not have a submassive PE. The patient is
best classified as having a low-risk PE. A meta-analysis
suggested that thrombolytics do not decrease mortality
in unselected (and low risk) patients and may increase
bleeding risk (Answer D is correct; Answers A–C are
incorrect).

ACCP Updates in Therapeutics® 2022: Critical Care Pharmacy Preparatory Review and Recertification Course

716
Shock Syndromes II: Hypovolemic, Critical Bleeding, and Obstructive

ANSWERS AND EXPLANATIONS TO SELF-ASSESSMENT QUESTIONS

1. Answer: C would increase oxygen-carrying capacity but not cor-


This patient presents with likely hemorrhagic shock rect the likely underlying coagulopathy (Answer A is
after blunt trauma. The extent of injuries in highly vas- incorrect). Although warmed lactated Ringer solution
cularized areas suggests a hemorrhagic source. Given may be indicated in a trauma patient with bleeding,
the extent of confusion, oliguria, tachycardia, and tachy- excessive fluid resuscitation (above 1.5 L) should ini-
pnea, this patient has class III hemorrhage (Answer C is tially be avoided (Answer B is incorrect). Given the
correct; Answers A, B, and D are incorrect). patient’s blood pressure, heart rate, and positive FAST,
he is at high risk of requiring massive transfusion,
2. Answer: C which should be initiated promptly with an empiric
The Fick equation describes oxygen delivery (Do2 (mL/ ratio. In addition, given the presence of hemorrhagic
minute) = 10 x CO (L/minute) x Cao2). A diagnosis of shock requiring erythrocyte transfusions, tranexamic
shock is typically based on hemodynamic, clinical, and acid is indicated early to improve mortality (Answer D
biochemical assessment of impaired Do2. Although is correct). Although goal-directed resuscitation is rec-
an elevated INR may represent a coagulopathy that ommended for continued resuscitation, empiric blood
can contribute to bleeding, it is not directly related to product transfusions should not be delayed while await-
impaired Do2. In addition, hematemesis is a clinical sign ing laboratory results unless point-of-care assays are
of bleeding but is not necessarily specific to a shock syn- available (Answer C is incorrect).
drome (Answer A is incorrect). Although heart rate is a
determinant of CO, CO is not generally impaired unless 4. Answer: B
the patient has acute supraventricular or ventricular This patient has apparent warfarin toxicity contribut-
tachycardia. In addition, an elevated serum creatinine ing to an acute GI hemorrhage. Because he meets the
may be a biochemical sign of impaired tissue oxygen- criteria for a major bleed, particularly associated with
ation, but this generally presents in a delayed fashion and hemodynamic instability, many guidelines recommend
may represent chronic renal dysfunction in this specific anticoagulation reversal. Although phytonadione is the
patient (Answer B is incorrect). A low Scvo2 represents specific reversal for warfarin to promote hepatic produc-
the balance between oxygen consumption and delivery tion of factors II, VII, IX, and X, it does not immediately
(not solely delivery) and does not explain reduced Do2. correct coagulopathy, and factor replacement is recom-
Although an elevated lactate concentration may indi- mended (Answer D is incorrect). In addition, when used
cate impaired Do2 in shock syndrome, lactate may be for life-threatening hemorrhage, intravenous admin-
elevated secondary to altered hepatic metabolism and is istration is recommended because of its quicker onset
not a specific marker for circulatory shock (Answer D is of action of 4–6 hours compared with 18–24 hours for
incorrect). Therefore, according to the Fick equation, a oral administration (Answer A is incorrect). Although
low hemoglobin reduces the Cao2, decreasing Do2. Cold plasma has traditionally been used for immediate factor
and clammy extremities is a common clinical marker of replacement for warfarin reversal, the time to labora-
a shock syndrome, indicating a low-flow state (Answer tory reversal is inferior to PCC. In addition, the volume
C is correct). of plasma needed to reverse his INR would likely not
be well tolerated by this patient with chronic thrombo-
3. Answer: D embolic PH (Answer C is incorrect). Therefore, 4PCC,
This patient presents with likely hemorrhagic shock after which contains nonactivated highly concentrated fac-
blunt trauma. The patient has signs consistent with intra- tors II, VII, FIX, and X, is recommended for warfarin
abdominal bleeding with the positive FAST examination reversal in life-threatening hemorrhage because of its
in hemodynamic instability; thus, urgent intervention superiority for laboratory reversal and noninferiority for
is required. According to the extent of altered menta- clinical hemostasis. Rates of thrombotic events appear
tion, tachycardia, and tachypnea, this patient has class similar when comparing 4PCC with plasma for warfa-
IV hemorrhage, consistent with greater than 40% blood rin reversal in life-threatening hemorrhage and surgical
loss, indicating the requirement for erythrocyte trans- populations, but rates of fluid overload complications
fusions. However, giving only erythrocyte transfusions are greater with plasma (Answer B is correct).

ACCP Updates in Therapeutics® 2022: Critical Care Pharmacy Preparatory Review and Recertification Course

717
Shock Syndromes II: Hypovolemic, Critical Bleeding, and Obstructive

1. Answer: A (e.g., ultrasound-assisted thrombolysis) and specific


The patient has a massive PE, as evidenced by pulseless- catheters (not pulmonary artery catheters) (Answer B is
ness. Massive PE should be treated with thrombolytic correct; Answer C is incorrect).
therapy unless absolute contraindications to thrombo-
lytics are present. Prolonged chest compressions may be 4. Answer: A
considered a relative contraindication to thrombolytics, This patient now has a cardiac arrest secondary to his
but this patient had a short duration of chest compres- PE. Because he has a confirmed PE as the precipitant of
sions (Answer A is correct). Troponin T and brain cardiac arrest, systemic thrombolytic is recommended
natriuretic concentrations may help classify a patient as an emergency treatment. Given the mortality rates
as having a submassive PE, but the patient in this case from PE in a cardiac arrest, standard contraindications
has already fulfilled the criteria for a massive PE, and to thrombolysis may be suspended in favor of a lifesav-
these laboratory values will not change the patient’s ing intervention. Systemic thrombolytics in this setting
treatment (Answers B and C are incorrect). Although may be associated with return of spontaneous circulation
a transthoracic echocardiogram can reveal the patient’s and possibly survival benefits (Answer D is incorrect).
RV function, it is unlikely to change his treatment, The recommended alteplase dose is a 50-mg intrave-
and therapy with thrombolytic therapy should not be nous bolus, repeated in 15 minutes if needed (Answer
delayed while an echocardiogram is done (Answer D is A is correct; Answer C is incorrect). Although surgi-
incorrect). cal embolectomy would be reasonable in the operating
room, systemic thrombolytics would be preferred, given
2. Answer: B the patient’s current location in the ICU (Answer B is
This patient’s PE is causing stress on the RV, as evi- incorrect).
denced by his echocardiograph and positive cardiac
enzymes. Therefore, this is not a low-risk PE (Answer
D is incorrect). However, the patient is hemodynami-
cally stable at this point, so it is also not a high-risk PE
(Answer A is incorrect). Given that the patient has both
evidence for RV straining and myocardial ischemia as
represented by both the echocardiogram and the cardiac
enzymes, this meets the criteria for an intermediate-high
risk PE (Answer B is correct; Answer C is incorrect).

3. Answer: B
This patient appears to be clinically worsening with
blood pressures that are approaching hemodynamic
instability and worsening oxygenation requiring high
levels of Fio2. Consideration for reperfusion therapy is
warranted. Given his history of a recent GI bleed requir-
ing ICU admission, systemic, full-dose thrombolytics
would be contraindicated (Answer A is incorrect). A
thrombolytic strategy to minimize the risk of bleeding
may be appropriate. Although half-dose alteplase has
been studied, it is not recommended because of insuf-
ficient evidence (Answer C is incorrect). Therefore, in
intermediate-high risk PE with clinical deterioration, the
guidelines recommend CDT when systemic thrombo-
lytic contraindications exist and resources are available,
such as in an urban, academic ICU. Catheter-directed
thrombolysis is administered with mechanical methods

ACCP Updates in Therapeutics® 2022: Critical Care Pharmacy Preparatory Review and Recertification Course

718
Cardiovascular Critical Care II
Scott T. Benken, Pharm.D., FCCM, BCPS-AQ Cardiology
University of Illinois at Chicago, College of Pharmacy
University of Illinois Health
Chicago, Illinois
Cardiovascular Critical Care II

Cardiovascular Critical Care II


Scott T. Benken, Pharm.D., FCCM, BCPS-AQ Cardiology
University of Illinois at Chicago, College of Pharmacy
University of Illinois Health
Chicago, Illinois

ACCP Updates in Therapeutics® 2022: Critical Care Pharmacy Preparatory Review and Recertification Course

721
Cardiovascular Critical Care II

Learning Objectives fraction (most recent ejection fraction was 45% on


echocardiogram in 2013), and gastroesophageal reflux
1. Manage cardiac arrest from the initiation of basic disease. T.B. is sent to the catheterization laboratory
life support to the use of post–cardiac arrest care. for suspected acute myocardial infarction. Laboratory
2. List the indications and contraindications for medi- values for T.B. are as follows: international normalized
cation administration during cardiac arrest. ratio (INR) 1, platelet count 200,000/mm3, hemoglobin
3. Categorize the patient groups that should receive 12 g/dL, serum creatinine (SCr) 1.7 mg/dL (baseline 1.5
targeted temperature management. mg/dL), white blood cell count (WBC) 17 x 103 cells/
4. Predict the common complications of targeted tem- mm3, and aspartate aminotransferase (AST) 100 IU/L.
perature management and explain how to ameliorate He is admitted to the coronary care unit for observation
them. after catheterization when he suddenly loses conscious-
5. Analyze the therapeutic goals and clinical indi- ness and becomes pulseless. The coronary care unit
cations for the medications used in hypertensive team of which you are part is called to the bedside. Of
emergency. note, T.B. has peripheral intravenous access, and before
this event, he was on room air by nasal cannula.

Abbreviations in This Chapter 1. Which is the most appropriate first step in T.B.’s
resuscitation?
ACLS Advanced cardiac life support A. Promptly intubate because this is likely a
AED Automated external defibrillator hypoxic pulmonary arrest.
BLS Basic life support B. Place central line for vasopressor administration.
BPV Blood pressure variability C. Initiate cardiopulmonary resuscitation (CPR),
CPR Cardiopulmonary resuscitation beginning with chest compressions.
DBP Diastolic blood pressure D. Initiate CPR, beginning with 2 breaths by bag-
ED Emergency department mask ventilator.
ICP Intracranial pressure
ICU Intensive care unit 2. The monitor reveals that T.B. is in ventricular fibril-
IO Intraosseous lation (VF), and T.B. remains pulseless. Which is the
MAP Mean arterial pressure most appropriate management of T.B.’s VF arrest?
MICU Medical intensive care unit
A. Provide an unsynchronized shock/defibrillation
PEA Pulseless electrical activity
at 120 J from a biphasic defibrillator.
pVT Pulseless ventricular tachycardia
B. Give intravenous amiodarone at a dose of 300
ROSC Return of spontaneous circulation
mg.
SCA Sudden cardiac arrest
C. Give intravenous atropine at a dose of 0.4 mg.
SBP Systolic blood pressure
D. Provide emergency transcutaneous pacing.
TTM Targeted temperature management
VF Ventricular fibrillation
3. T.B.’s rhythm changes from VF to pulseless electri-
VT Ventricular tachycardia
cal activity (PEA) on the monitor. Which is the most
appropriate management of T.B.’s PEA arrest?
Self-Assessment Questions A. Provide an unsynchronized shock/defibrillation
Answers and explanations to these questions may be at 120 J from a biphasic defibrillator.
found at the end of this chapter. B. Begin high-quality chest compressions, and
consider treatable causes of cardiac arrest.
Questions 1–5 pertain to the following case. C. Give intravenous lidocaine 1 mg/kg x 1, fol-
T.B. is a 72-year-old man with a history of atrial fibril- lowed by an infusion at 1 mg/minute.
lation, coronary artery disease with drug-eluting stent D. Give intravenous atropine at a dose of 0.4 mg.
placement in 2009, heart failure with reduced ejection

ACCP Updates in Therapeutics® 2022: Critical Care Pharmacy Preparatory Review and Recertification Course

722
Cardiovascular Critical Care II

4. T.B. has return of spontaneous circulation (ROSC) 6. Which laboratory abnormality or presenting symp-
after 15 minutes of total resuscitation. The team is tom best qualifies J.H. for hypertensive emergency?
deciding whether targeted temperature management A. None; she is having a hypertensive urgency.
(therapeutic hypothermia) would be appropriate for B. Her SCr.
T.B. Which is the most accurate statement regarding C. Her AST/ALT.
targeted temperature management for T.B.? D. Her D-dimer.
A. He should not be considered because his initial
presentation was VF arrest. 7. Which is the most appropriate initial treatment strat-
B. He should not be considered because he has egy for J.H.’s BP?
transaminitis. A. Phentolamine 1 mg intravenously every 30
C. He should be considered with SCr monitoring. minutes.
D. He should be considered, but thrombolysis B. Metoprolol 25 mg orally every 12 hours.
should be initiated concurrently because of the C. Nitroprusside 0.25 mcg/kg/minute by continu-
associated risk of clotting. ous intravenous infusion.
D. Enalaprilat 10 mg intravenously every 6 hours.
5. The medical team wants further information about
the literature regarding therapeutic hypothermia. 8. Which is the most appropriate goal for J.H.’s BP
Which is the most appropriate information regard- reduction?
ing the data to support therapeutic hypothermia?
A. Goal mean arterial pressure (MAP) reduction
A. Improves survival in PEA cardiac arrests. of 25% during the first 60 minutes.
B. Shown to improve neurologic outcomes. B. Goal MAP reduction of 50% during the first 60
C. Superiority found when targeting a core tem- minutes.
perature of 36°C compared with 33°C. C. Goal MAP reduction of 25% during the first 24
D. Most studied in patients with asystole. hours.
D. Goal MAP reduction of 50% during the first 24
Questions 6–8 pertain to the following case. hours.
J.H. is a 48-year-old woman with no known medical his-
tory who presents to the emergency department (ED) for
acute onset of shortness of breath, right-side pain, and
some blurry vision. She denies any illicit drug or ciga-
rette use, but she confirms social alcohol intake (about
3 drinks per week). Urine toxicology is negative. Initial
vital signs are as follows: blood pressure (BP) 202/140
mm Hg, heart rate (HR) 88 beats/minute, respiratory rate
(RR) 22 breaths/minute, and pain 4/10 (chest and right-
side pain). Initial laboratory values are as follows: SCr
0.8 mg/dL, AST 608 U/L, ALT 458 U/L, lipase 20 U/L,
total bilirubin (Tbil) 1 mg/dL, direct bilirubin (Dbil) 0.4
mg/dL, WBC 6 x 103 cells/mm3, hemoglobin 11 mg/dL,
troponin T less than 0.01 ng/mL, and D-dimer less than
0.5 mcg/mL. Chest radiography shows moderate bilat-
eral pleural effusions and no focal consolidations. Chest
computed tomography (CT) is negative for pulmonary
embolism. Of note, J.H. is taking no prescription or over-
the-counter medications.

ACCP Updates in Therapeutics® 2022: Critical Care Pharmacy Preparatory Review and Recertification Course

723
Cardiovascular Critical Care II

BPS Critical Care Pharmacy Specialist Examination Content Outline

This chapter covers the following sections of the Critical Care Pharmacy Specialist Examination Content Outline:
1. Domain 1: Clinical Knowledge and Application
a. Task 1: 3
b. Task 2: 2-5
c. Task 3: 1-7
d. Task 4: 1-5, 8
e. Task 5: 1, 3-7
f. Task 6: 1, 2, 4, 6, 7
g. Task 7: 1-3, 7
h. Task 8: 2
2. Domain 2: Practice Management, Policy, and Quality Improvement
a. Task 1: 2
b. Task 4: 4
c. Task 6: 3, 4
3. Domain 3: Evidence-Based Medicine, Scholarship, Education, and Professional Development
a. Task 1: 2, 3
b. Task 3: 1
c. Task 5: 1

ACCP Updates in Therapeutics® 2022: Critical Care Pharmacy Preparatory Review and Recertification Course

724
Cardiovascular Critical Care II

I.  ADVANCED ADULT CARDIAC LIFE SUPPORT

A. Background
1. Foundation of advanced cardiac life support (ACLS) is effective and timely basic life support (BLS).
2. Sudden cardiac arrest (SCA) continues to be a leading cause of death in many parts of the world.
3. SCA can vary in etiology (noncardiac vs. cardiac), circumstances (unwitnessed vs. witnessed), and
setting (in vs. out of hospital).
4. Because of heterogeneity, action links that are denoted the “chain of survival” were developed for
guidance (Circulation 2016;133:447-54). There is an increased recognition in the differences between
arrests that occur in the hospital and those that do not, which has led to two distinct chains of survival:
a. In-hospital cardiac arrest (IHCA) chain of survival
i. Appropriate surveillance and early warning systems (prevention)
ii. Activation of multidisciplinary response team
iii. Early CPR that emphasizes chest compressions
iv. Rapid defibrillation, if indicated
v. Advanced life support and post-cardiac arrest care
vi. Recovery
b. Out-of-hospital cardiac (OHCA) arrest chain of survival
i. Immediate recognition of SCA and activation of emergency response system
ii. Early CPR that emphasizes chest compressions
iii. Rapid defibrillation, if indicated
iv. Basic and advanced emergency medical services
v. Advanced life support and post-cardiac arrest care
vi. Recovery
5. Following the chain of survival effectively can improve survival (e.g., with out-of-hospital, witnessed
VF arrest, survival rates can approach 50%) (Circulation 2006;114:2760-5). Because BLS and ACLS
are often experienced as a team approach in the hospital setting, it is imperative that hospital providers
be familiar with all aspects of BLS and ACLS in order to fulfill any role during the arrest situation.

B. Basic and advanced emergency medical services


1. Immediate recognition of SCA and activation of emergency response system (OHCA - chain of survival):
Unresponsive patient or witnessed sudden collapse with absent or gasping abnormal breathing (Circulation
2016;133:447-54; Acad Emerg Med 2007;14:877-83).
a. Ensure that the scene is safe.
b. Check for response by tapping on shoulder and shouting at patient; simultaneously check for normal
breathing.
c. Activate emergency response system (e.g., facility emergency response team), and follow instructions
from trained dispatchers/responders (Ann Emerg Med 1993;22:354-65).
2. Early CPR. Follow the sequence of C-A-B (compressions-airway-breathing) (IHCA and OHCA - chain
of survival).
a. C - Chest compressions are an essential component of CPR.
i. Not often provided by nonmedical individuals until professional emergency responders arrive
(Circ Cardiovasc Qual Outcomes 2010;3:63-81).
ii. Both an increase in intrathoracic pressure and a direct compression of the heart lead to perfusion
and oxygen delivery to the brain and myocardium.
iii. All patients with SCA should receive chest compressions (Acta Anaesthesiol Scand 2008;
52:914-9).

ACCP Updates in Therapeutics® 2022: Critical Care Pharmacy Preparatory Review and Recertification Course

725
Cardiovascular Critical Care II

iv. Place patient on a hard surface, use backboard (unless it will cause interruptions in chest
compressions, delay in initiation of CPR, or dislodgment of lines/tubing), and/or deflate air-
filled mattresses (J Intensive Care Med 2009;24:195-9; Acta Anaesthesiol Scand 2007;51:747-
50; Resuscitation 2004;61:55-61).
v. Perform high-quality chest compressions at a rate of 100–120 compressions per minute at
a depth of 2–2.4 inches, and allow chest recoil after each compression. High-quality chest
compressions optimize coronary perfusion, cardiac index, myocardial blood flow, and cerebral
perfusion (Crit Care Med 2010;38:1141-6; Resuscitation 2006;71:137-45; Resuscitation
2006;71:341-51; Circulation 2015;132(suppl 2):S315-S367).
vi. Without the use of a compression feedback device, it may be difficult to judge compression
rate and depth. In a randomized study, compression feedback devices were shown to increase
adherence to CPR guidelines, increase CPR quality, increase rates of ROSC, and decrease
injury (rib fractures) seen with CPR (Crit Care 2016;20:147:1-8).
vii. Actual number of chest compressions given per minute is a function of the compression rate and
proportion of time without interruption. Goal is to minimize interruptions to chest compressions.
(a) Increasing the number of compressions given per minute can increase the rate of ROSC,
increase survival rates, and increase rates of neurologically-intact survival (JAMA
2008;299:1158-64; Circulation 2009;120:1241-7; Circulation 2005;111:428-34).
(b) Rescuer fatigue is common and may lead to inadequate compression quality (Resuscitation
2009;80:918-4). It is recommended to change compressors every 2 minutes (or after five
cycles of compressions at a rate of 30:2 compressions/ventilations) with no more than 10
seconds between changes (Resuscitation 2009;80:1015-8).
(c) Providers have difficulty and take too long to check for a pulse (Resuscitation 2000;44:195-
201). Pulse checks (including initial) should last no more than 10 seconds.
(d) Duration of the single longest interruption to chest compressions (regardless of reason) is
negatively associated with survival (Circulation 2015;132:1030-7), reemphasizing the need
to minimize interruptions to chest compressions.
(e) Compression-first (or “compression only” or “hands only”) CPR decreases time until first
compression (Resuscitation 2004;62:283-9) and for suspected OHCA is acceptable for
nonmedical rescuers (Circulation 2015;132(suppl 2):S315-S367).
(f) A recent Cochrane review found that bystander-administered chest compression-only
CPR (supported by telephone instruction) increases the number of people who survive to
hospital discharge (Cochrane Database Syst Rev 2017;3:CD010134). It is unclear how well
neurologic function is preserved in this population.
viii. Mechanical chest compression devices have not been shown to be superior to conventional
CPR. They can be considered when prolonged CPR is used, being mindful of interruptions
in CPR during device deployment and removal (Circulation 2015;132(suppl 2):S315-S367).
A recent Cochrane review confirmed that mechanical chest compressions are not superior to
manual chest compressions (Cochrane Database Syst Rev 2018;8:CD007260). Furthermore,
in shockable rhythms, mechanical chest compression devices may lead to longer times to
first defibrillation and greater intervals between additional defibrillations (Resuscitation
2017;115:155-62).
ix. Emerging data suggest that patients can have ROSC with meaningful neurological recovery
even after prolonged (>25 min) pre-hospital resuscitation efforts (Circulation 2016;133:1386-
96; Resuscitation 2016;105:45-51).

ACCP Updates in Therapeutics® 2022: Critical Care Pharmacy Preparatory Review and Recertification Course

726
Cardiovascular Critical Care II

x. The use of extracorporeal CPR is emerging as a therapy for those who fail conventional CPR.
The current data seems promising in benefiting neurologic outcomes but is limited by bias and
quality (Circulation 2019;140:e881-e894). Current recommendation is that extracorporeal CPR
may be considered for selected patients as rescue therapy when conventional CPR efforts are
failing in settings in which it can be successfully implemented by skilled providers.
xi. Partial pressure of end-tidal CO2 (PETCO2), coronary perfusion pressure (aortic diastolic
pressure - right atrial diastolic pressure), arterial relaxation pressure, regional cerebral
oxygenation and central venous oxygen saturation (Scvo2) correlate with cardiac output and
myocardial blood flow during CPR. Threshold values have been reported at which ROSC is
rarely achieved, and an abrupt increase in any of these variables is a sensitive indicator of
ROSC (Table 1 - Circulation 2015;132(suppl 2):S315-S367; Ann Emerg Med 1992;21:1094-
101; JAMA 1990;263:1106-13; Am J Emerg Med 1985;3:11-4; J Emerg Med 2010;38:614-21;
Crit Care 2008;12:R115; Crit Care Med 2016;44:1663-74).

Table 1. Useful Physiological Variables During CPR


Variable ROSC Rarely Achieved
PETCO2 < 10 mm Hg
Coronary perfusion pressure < 15–20 mm Hg
Arterial relaxation (diastolic) pressure < 20–40 mm Hg
Regional cerebral oxygenation < 25%
Scvo2 < 30%

Patient Case

1. A.C., a 50-year-old man with a history of gastroesophageal reflux disorder and chronic obstructive pulmonary
disease, was admitted for shortness of breath, palpitations, and presumed exacerbation of his lung disease.
On hospital admission day 4, A.C. has a witnessed cardiac arrest on the family medicine unit. The emergency
response team of which you are part is called, and when you arrive, the bedside nurse has already begun
chest compressions. Which insight would be best shared regarding chest compressions for A.C.?
A. Because the nurse has already begun chest compressions, she should continue chest compressions for
the duration of CPR.
B. Compressions increase intrathoracic pressure and directly compress the heart, which can generate car-
diac output and deliver oxygen.
C. Because increasing the intrathoracic pressure is vital to oxygen delivery, chest recoil is unnecessary and
should be avoided.
D. Number of chest compressions per minute does not affect outcomes, so pulse check and line and airway
placements can interrupt chest compressions.

b. A - Airway
i. To place an artificial airway in patients without an advanced airway, use the head-tilt, chin-
lift technique if patients have no evidence of head or neck trauma and the jaw thrust alone if
cervical spine injury is suspected (JACEP 1976;5:588-90; JAMA 1960;172:812-5) (see “Rescue
Breaths” following).

ACCP Updates in Therapeutics® 2022: Critical Care Pharmacy Preparatory Review and Recertification Course

727
Cardiovascular Critical Care II

ii. Cricoid pressure is the technique of applying pressure to the patient’s cricoid cartilage to push
the trachea posteriorly and compress the esophagus with the goal of preventing aspiration.
(a) May help in visualizing vocal cords during tracheal intubation.
(b) Recommend against use for adult cardiac arrest because of possible delay or prevention of
advanced airway, lack of protection from aspiration, and lack of mastery from “expert” and
nonexpert rescuers (Emerg Med Australas 2005;17:376-81; Br J Anaesth 1994;72:47-51).
iii. If a foreign body airway obstruction (FBAO) occurs:
(a) Do not act if the patient is coughing forcefully because this is a mild FBAO.
(b) Signs of severe FBAO include a silent cough, stridor, or increasing respiratory difficulty.
If these occur, ask the patient, “Are you choking?” If patients clutch their neck (universal
sign of choking) or nod without answering verbally, consider severe FBAO:
(1) Activate the emergency response system.
(2) Administer abdominal thrusts to nonobese adults.
(3) In adults with obesity or women in the late stage of pregnancy, administer chest thrusts.
(c) If the patient becomes unresponsive:
(1) Place on ground and begin CPR as chest compressions have been shown to
generate higher airway pressure than abdominal thrusts (Resuscitation 2000;
44:105-8).
(2) Each time the airway is opened during CPR to provide a rescue breath, look for an
object in the patient’s mouth and, if found, remove it. If not found, continue giving the
rescue breaths (two total breaths), followed by 30 chest compressions.
(3) No studies have evaluated the routine use of the finger sweep to clear an airway in
the absence of visible airway obstruction. Case reports have shown some efficacy, but
harm has also been demonstrated in patients and rescuers. A finger sweep should not
be used in the absence of visible airway obstruction.
iv. Advanced airways
(a) Supraglottic airway devices such as the laryngeal mask airway, the esophageal-tracheal
Combitube, and the King Airway device are considered within the scope of BLS in some
districts.
(b) See the ACLS section.
v. Naloxone for opioid-related life-threatening emergencies. Now with an independent algorithm
in the most recent guidelines (Circulation 2020;142(16_suppl_2):S366-468).
(a) Should be considered when a pulse is present but the patient has abnormal breathing or
gasping (e.g. respiratory arrest).
(b) Can consider (in addition to BLS) administration of intramuscular or intranasal naloxone.
(c) If patients lose their pulse, provision of CPR should commence with continued consideration
of naloxone if opioid intoxication is suspected.
(d) Administer 0.4 mg intramuscularly or 2 mg intranasally diluted in 3 mL of normal saline
– may repeat every 4 minutes.
c. B - Rescue breaths
i. Primary purpose is to assist in maintaining oxygenation, with secondary purpose of eliminating
carbon dioxide (CO2).
ii. Compressions should always be initiated first as the arterial oxygen content of blood remains
unchanged until CPR is initiated.
iii. Optimal compression/ventilation ratio, inspired oxygen concentration, tidal volume, and RR
yet to be determined.

ACCP Updates in Therapeutics® 2022: Critical Care Pharmacy Preparatory Review and Recertification Course

728
Cardiovascular Critical Care II

iv. No rescue breaths are recommended for untrained nonmedical rescuers for OHCA.
Compression-only CPR is recommended (see earlier text for more detail). Trained nonmedical
rescuers are encouraged to perform rescue breaths at a ratio of 30 compressions to 2 breaths.
When the patient has an advanced airway in place during CPR, rescuers no longer deliver
cycles of 30 compressions and 2 breaths (i.e., they no longer interrupt compressions to deliver
2 breaths). Instead, the provider may reasonably deliver 1 breath every 6 seconds (10 breaths/
minute) while continuous chest compressions are being performed (Circulation 2015;132(suppl
2):S414-35).
(a) Low minute ventilation (low tidal volume and/or low RR) can maintain oxygenation and
ventilation during CPR as there is low oxygen uptake at the tissues and low CO2 production
(Circulation 1997; 95:1635-41).
(b) Give sufficient tidal volume to produce visible chest rise (Resuscitation 1996;31:231-4).
Excessive ventilation can increase intrathoracic pressure and decrease venous return as
well as cause gastric inflation, which can lead to aspiration, regurgitation, and decreased
survival (Circulation 2004;109:1960-5; Resuscitation 1998;36:71-3; JAMA 1987;
257:512-5).
(c) Continuous chest compressions with asynchronous ventilation compared with chest
compressions with interruptions for synchronous ventilation have not improved outcomes
when delivered by emergency medical services (N Engl J Med 2015;23:2203-14).
v. Deliver each rescue breath over 1 second. Mouth-to-mouth, mouth-to-barrier, mouth-to-stoma,
and mouth-to-nose variations in initial rescue breathing are all acceptable and can produce
oxygenation and ventilation (Chest 1994;106:1806-10; Br J Anaesth 1964;36:542-9).
vi. Positive-pressure ventilation
(a) Bag-mask ventilation
(1) Components include a nonjam inlet valve, either no pressure relief valve or a pressure
relief valve that can be bypassed, standard 15-mm/22-mm fittings, an oxygen reservoir
to allow for high oxygen delivery, and a non-rebreathing outlet valve (Respir Care
1992;37:673-90; discussion 690-4).
(2) Should not be used by a single rescuer.
(3) Should use an adult bag (1 or 2 L) and deliver (two-thirds or one-third of bag
volume, respectively) about 600 mL of tidal volume, which should produce chest
rise, oxygenation, and normocarbia (Resuscitation 2005;64:321-5; Resuscitation
2000;43:195-9).
(b) Supraglottic airway devices (e.g., King Airway device) are considered an acceptable
alternative to bag-mask ventilation during cardiac arrest (assuming proper training is
supplied to rescuer) (Circ J 2009;73:490-6; Prehosp Emerg Care 1997;1:1-10).

ACCP Updates in Therapeutics® 2022: Critical Care Pharmacy Preparatory Review and Recertification Course

729
Cardiovascular Critical Care II

Patient Case

2. L.S. is a 66-year-old woman visiting her husband at the hospital on the hospice unit. She is buying lunch
in the cafeteria, and while in line to check out, she collapses. The emergency response team of which you
are part is summoned. L.S. does not respond to voice or tapping of the shoulder, and a brief look at her
chest shows no chest movement. Chest compressions are initiated while the crash cart and defibrillator are
retrieved. Of note, a bag-mask ventilator is available at the scene because it is carried with the emergency
response team. Which is most accurate about L.S.’s airway and breathing management?
A. A compression/ventilation ratio of 15:2 should be used throughout resuscitation efforts.
B. Because it is a multiple-rescuer scene, bag-mask ventilation should not be used because it is recom-
mended only in single-rescuer resuscitations.
C. Rescue breaths should be given at a ratio of 30 compressions to 2 breaths, avoiding excessive ventilation
given as synchronous ventilations.
D. Bag-mask ventilation should not be used in any patient because advanced airways are preferred to supply
oxygen and eliminate CO2.

3. Rapid defibrillation with a manual or automated external defibrillator (AED) (IHCA and OHCA - chain
of survival)
a. Defibrillation shock = unsynchronized shock.
b. Successful defibrillation is defined as 5 seconds or greater of termination of arrhythmia after a
shock is delivered.
c. Early defibrillation of VF is crucial because it is the most common rhythm in witnessed OHCA,
survival diminishes rapidly over time, and VF often progresses to asystole (Resuscitation 2000;44:7-
17; Circulation 1997;96:3308-13).
d. Three key actions must occur within moments of VF SCA to increase the likelihood of survival: (1)
activation of the emergency medical services system (e.g., emergency response team), (2) provision
of CPR, and (3) shock delivery (Ann Emerg Med 1993;22:1652-8).
e. Performing chest compressions while a defibrillator is obtained significantly improves the
probability of survival (Circulation 2009;120:1241-7). When VF is present for more than a few
minutes, the myocardium becomes depleted of oxygen and energy substrates (e.g., adenosine
triphosphate [ATP]).
i. CPR can provide the oxygen and ATP needed until the shock is delivered.
ii. Increased likelihood of termination of VF from shock delivery and ROSC if CPR given first
(Circulation 2004;110:10-5).
iii. If CPR is initiated immediately, survival can double or triple at most time intervals until
defibrillation occurs (Resuscitation 2000;44:7-17; Ann Emerg Med 1995;25:780-4; Ann Emerg
Med 1993;22:1652-8).
f. Early defibrillation is a powerful predicator of ROSC after VF.
i. Survival rates are highest for VF when CPR and defibrillation occur within 3–5 minutes of the
event (Circ Cardiovasc Qual Outcomes 2010;3:63-81; Resuscitation 2009;80:1253-8). In the
IHCA setting, prompt defibrillation (less than 2 minutes from the VF event) was associated
with higher rates of long-term survival (Circulation 2018;137:2041-51).
(a) For every minute that passes after collapse, survival from VF decreases 7%–10% (Ann
Emerg Med 1993;22:1652-8).
(b) CPR prolongs VF and delays the progression to asystole (Resuscitation 2000;47:59-70;
Am J Emerg Med 1985;3:114-9).
ii. CPR should be initiated immediately, with shock delivery as soon as possible.

ACCP Updates in Therapeutics® 2022: Critical Care Pharmacy Preparatory Review and Recertification Course

730
Cardiovascular Critical Care II

iii. One-shock biphasic (bidirectional) shock protocols are better than or equivalent to three-shock
monophasic (one-directional) stacked protocols in terminating VF.
(a) Almost all AEDs manufactured currently are biphasic.
(b) Polyphasic waveform defibrillators are currently under investigation.
(c) Polyphasic waveform defibrillators are currently under investigation. Double-sequential
defibrillation is not supported (Circulation 2020;142:S366-S468).
iv. With any shock delivery, chest compressions should resume immediately, and pulse check
should be delayed until the end of the next cycle of CPR because this can increase successful
defibrillation with ROSC (Circulation 2004;110:10-5; Circulation 2002;105:2270-3).
v. The optimal shock energy for biphasic first shock has yet to be determined.
(a) Low-energy dosages (200 J or less) are safe and have equivalent or higher efficacy of
termination of VF than monophasic waveform shocks at the same or higher energy
(Circulation 2007;115:1511-7; Prehosp Emerg Care 2000;4:305-13).
(b) The energy dosage used should be according to the manufacturer’s recommendation (e.g.,
120 or 200 J).
vi. If additional shocks are needed, it is recommended that at least equivalent and potentially
higher energy be used. Double-sequential defibrillation – shock delivered by two defibrillators
almost simultaneously – is a new approach, but a systematic review has failed to establish its
usefulness in resuscitation (Circulation 2020;142(suppl 1):S92-S139).
vii. Electrode placement
(a) Pad positioning is equally effective in terminating ventricular arrhythmias in four positions:
anterolateral, anteroposterior, anterior-left infrascapular, and anterior-right infrascapular
(Medicina (Kaunas) 2006;42:994-8; Physiol Meas 2006;27:1009-22).
(b) Lateral pads should be placed under breast tissue, and hirsute men should be shaved before
the placement of pads.
(c) In patients with an implantable cardioverter-defibrillator or a pacemaker, it may be
beneficial to avoid placing the pads or paddles over the device.
(d) Do not place the pads on top of a medication patch because this can cause current
impedance or burning of the skin (Am J Emerg Med 1992;10:128-9).
viii. In-hospital AED use should be considered in ambulatory and unmonitored areas.
g. Pulseless VT (pVT) is treated like VF.
h. Pacing is not recommended for unstable VF or pVT (Circulation 2010;122:S685-705).

ACCP Updates in Therapeutics® 2022: Critical Care Pharmacy Preparatory Review and Recertification Course

731
Cardiovascular Critical Care II

Patient Case

3. T.V. is a 72-year-old man with a history of chronic liver disease, hypoglycemia, and atrial fibrillation. He
was admitted to the medical intensive care unit (MICU) 2 days ago for sepsis requiring aggressive fluid
resuscitation and intravenous antibiotics. T.V. did not require vasopressors to treat his sepsis. On ICU day 3,
T.V. develops VF on telemetry, loses consciousness, and becomes pulseless; the MICU team is summoned
for a presumed VF cardiac arrest. Pads are placed on T.V. by the time the team arrives, and the rhythm is
confirmed to be VF. Which is the most accurate statement regarding defibrillation for T.V.?
A. Three vital actions with VF can lead to increased survival if they occur rapidly: activate emergency
response system, provide CPR, and deliver shock.
B. T.V. should not be defibrillated but should be paced out of the VF, if possible, because pacing is more
effective for pulseless VF.
C. Chest compressions should be delayed until the defibrillator is charged because defibrillation is the
definitive treatment of VF.
D. Alternative treatments such as antiarrhythmics, vasopressors, and magnesium should be tried first.

C. ACLS Interventions
1. Airway control and ventilation
a. Background
i. During CPR, oxygen delivery to the heart and brain becomes more flow-dependent than arterial
oxygen saturation–dependent (Ann Emerg Med 1990;19:1104-6).
ii. Placement of an advanced airway in cardiac arrest should not delay CPR or defibrillation.
iii. No studies address the optimal timing of advanced airway placement. The guiding general
concept is to place the advanced airway while minimizing interruptions to chest compressions.
iv. Conflicting evidence exists for the urgent placement of an advanced airway.
(a) Study of IHCA has shown an increased 24-hour survival in patients with an advanced
airway placed within 5 minutes. No difference was found regarding ROSC (Resuscitation
2010;81:182-6); however, another study has shown a worse overall survival rate in cardiac
arrest patients who required intubation (Arch Intern Med 2001;161:1751-8).
(b) OHCA studies have shown that intubation in the rural and urban setting and, more
specifically intubation within 13 minutes, is associated with better survival (Med J Aust
2006;185:135-9; Prehosp Emerg Care 2004;8:394-9).
(c) One large randomized controlled trial comparing endotracheal intubation to bag-mask
ventilation in OHCA found no difference in 28-day survival or 28-day survival with
favorable neurological function (JAMA 2018;319:779-87).
b. Oxygen during CPR
i. Unclear what the optimal concentration of inspired oxygen content should be during CPR, but
it is currently recommended that maximum (e.g., 100%) inspired oxygen be used to optimize
arterial oxyhemoglobin content and, subsequently, oxygen delivery (Circulation 2015;132(suppl
2):S315-S367).
ii. Patients with a higher Pao2 were associated with higher survival to hospital admission but
not neurologically intact survival, which is likely a function of underlying pathophysiology
(Resuscitation 2013;84:770-5).
iii. Extended exposure to high oxygen concentrations carries the risk of toxicity, but this toxicity
has not been shown in the short-term setting of adult CPR (Resuscitation 1999;42:221-9).

ACCP Updates in Therapeutics® 2022: Critical Care Pharmacy Preparatory Review and Recertification Course

732
Cardiovascular Critical Care II

iv. During chest compressions, air is forcefully expelled from the chest, and oxygen is drawn into
the chest by passive recoil. Because the ventilation requirements are lower than normal, passive
oxygen delivery is theorized to be sufficient for several minutes of initial CPR (Circulation
1994;90:3070-5), but recommendations to remove ventilation cannot be made.
c. Bag-mask ventilation: Viable option for oxygenation and ventilation during CPR but should be
provided only when there is more than one rescuer and/or trained personnel (for more details, see
earlier discussion) (Circulation 2010;122:S729-767).
d. Airway adjuncts
i. Cricoid pressure should be used only in special circumstances to help visualize the vocal
cords and should be relaxed, released, or adjusted if it impedes ventilation or advanced airway
placement.
ii. Oropharyngeal airways can be considered to help facilitate bag-mask ventilation in the
unresponsive patient with no cough or gag reflex.
iii. Nasopharyngeal airways can be considered in patients with airway obstruction and clenched
jaw but should be used cautiously in craniofacial injury and avoided in known coagulopathy
because of an increased risk of bleeding (J Trauma 2000;49:967-8; Anaesthesia 1993;
48:575-80).
e. Advanced airways
i. Endotracheal intubation
(a) Attempted placement by unskilled providers leads to unacceptably large periods of chest
compression interruption and hypoxemia.
(b) Benefits include keeping the airway patent, allowing for suctioning of airway, ensuring high
oxygen concentration delivery, providing a third line medication administration option,
allowing for specific tidal volume delivery, and providing protection from aspiration.
ii. Supraglottic airways
(a) Do not require visualization of glottis, which allow for continuous chest compressions.
(b) Types studied during cardiac arrest include laryngeal mask airway, esophageal-tracheal
tube (Combitube), laryngeal tube (laryngeal tube or King Airway LT), among others. No
difference in successful prehospital ventilation, ROSC, or 1-month neurologic outcome
between laryngeal mask airway and laryngeal tube for OHCA (Am J Emerg Med
2015;33:1360-3).
(c) When used by trained providers, they allow as effective oxygenation and ventilation as
bag-mask ventilation and endotracheal intubation.
iii. Recommendations regarding type of advanced airway are equivocal (Circulation
2019;140:e881-e894).
(a) Supraglottic airways can be used for adults with OHCA in settings with low tracheal
intubation success rate or minimal training opportunities for ETT placement.
(b) Supraglottic airways or ETT can be used for adults with OHCA in settings with high
tracheal intubation success rates or optimal training opportunities for ETT placement.
(c) If an advanced airway is used in the in-hospital setting by expert providers trained in these
procedures, either the supraglottic airway or ETT can be used.
(d) Frequent experience/retraining is recommended for providers who perform endotracheal
intubation.
(e) Emergency medical services systems that perform prehospital intubation should provide a
program of ongoing quality improvement.
iv. After advanced airways are secured, proper placement should be confirmed with clinical
assessment and objective measures without interruptions to chest compressions.

ACCP Updates in Therapeutics® 2022: Critical Care Pharmacy Preparatory Review and Recertification Course

733
Cardiovascular Critical Care II

(a) Physical assessments include visually inspecting chest rise bilaterally and listening to
the epigastrium (breath sounds should be absent) and lung fields (should be equal and
adequate).
(b) Exhaled CO2 or esophageal detector devices are a reasonable and objective means of
confirmation if continuous waveform capnography is not readily available.
(c) Continuous waveform capnography is the most reliable and objective way to ensure,
confirm, and monitor correct endotracheal tube placement. Although not specifically
studied with supraglottic airways, readings should be similar to endotracheal readings.
(d) False-positive CO2 detection (CO2 detected not from ventilation) is rare, whereas false-
negative CO2 detection (no CO2 detection when ventilation is occurring) is more common.
Most common cause of false-negative CO2 detection is a reduction in blood flow or CO2
delivery to lungs (e.g., lack of quality chest compressions, pulmonary embolism, severe
airway obstruction). Partial pressure of end-tidal CO2 (PETCO2) less than 10 mm Hg
during CPR suggests ROSC is unlikely, and maneuvers such as improving quality of chest
compressions, adding vasopressor therapy, and others, should be considered.
v. Post-intubation airway management
(a) Airway should be marked (from front of teeth/gums) and secured (with tape or commercial
device), avoiding compression around the neck, which could impair venous return from
brain.
(b) Chest radiography is suggested for confirmation of location of end of endotracheal tube in
relation to the carina.
(c) Slower ventilator rates (6–12 breaths/minute) have been shown to improve hemodynamic
values and short-term survival in animal models of cardiac arrest (Crit Care Med
2006;34:1444-9; Circulation 2004;109:1960-5; Resuscitation 2004;61:75-82).
vi. After placement, continuous chest compressions should be given at a rate of 100-120
compressions per minute. A breath should be delivered every 6–8 seconds (8–10 breaths/
minute), making sure to avoid over-ventilation, which could decrease venous return and cardiac
output.

Patient Case

4. F.V. is a 63-year-old woman with a history of diabetes, heart failure with preserved ejection fraction, hyper-
tension, and obstructive sleep apnea who presents to the ED with chest tightness and “feeling funny.” In
the ED, F.V. loses consciousness and develops pVT. Chest compressions are initiated immediately, pads
are placed, and bag-mask ventilation is given at a compression/ventilation ratio of 30:2 for two cycles. The
monitor confirms the rhythm of pVT. The defibrillator is charged, the patient is cleared, and the first shock is
delivered. Chest compressions resume, and during the next pulse check, the patient is intubated. F.V. still has
no pulse, and chest compressions are continued. Which is the most accurate statement about F.V.’s resuscita-
tion after advanced airway placement?
A. The compression/ventilation ratio should remain 30:2 to avoid excessive ventilation.
B. The advanced airway should have been placed before defibrillation and CPR for pVT.
C. The patient should be placed on room air (21% Fio2 [fraction of inspired oxygen]).
D. Proper airway placement should be confirmed with clinical assessment and objective measures.

2. Management of cardiac arrest

ACCP Updates in Therapeutics® 2022: Critical Care Pharmacy Preparatory Review and Recertification Course

734
Cardiovascular Critical Care II

a. Background
i. Cardiac arrest can be caused by four primary “rhythms”: VF, pVT, PEA, and asystole.
ii. These rhythms can also be classified as shockable (VF and pVT) and non-shockable (PEA and
asystole).
(a) Ventricular fibrillation
(1) Wide complex
(2) Polymorphic
(3) Disorganized
(4) Coarse or fine
(5) No/minimal forward flow

(b) Pulseless VT
(1) Wide complex
(2) Monomorphic or polymorphic
(3) Generally organized
(4) No/minimal forward flow

(c) Pulseless electrical activity


(1) Not a rhythm itself but defined as an organized rhythm that would be expected to
produce mechanical activity but does not
(2) Absent or insufficient mechanical ventricular activity

(d) Asystole (ventricular asystole)


(1) Absence of detectable ventricular electrical activity
(2) Accompanied by absence of ventricular mechanical activity

ACCP Updates in Therapeutics® 2022: Critical Care Pharmacy Preparatory Review and Recertification Course

735
Cardiovascular Critical Care II

iii. Survival requires both BLS and ACLS. Foundation of ACLS is high-quality BLS. In addition
to CPR, the only proven rhythm-specific therapy that increases survival at hospital discharge
is defibrillation of VF/pVT.
iv. Medications and advanced airways have not consistently been shown to increase survival of
cardiac arrest but have been shown to increase the rate of ROSC or survival to hospital admission
(Resuscitation 2010;81:182-6; Med J Aust 2006;185:135-9; Resuscitation 2000;45:161-6; N
Engl J Med 1999;341:871-8).
(a) Vascular access and medication delivery should never interrupt CPR and/or defibrillation.
All other therapies are “considerations” and should never compromise chest compressions.
(b) If interruptions in chest compressions are necessary (e.g., rhythm assessment, delivery of
defibrillation shocking in VF/VT), recommendation is to minimize duration (less than 10
seconds).
v. During cardiac arrest treatment, it is imperative to evaluate, treat, and/or reverse any treatable
causes of cardiac arrest (Table 2).
vi. Post–cardiac care should begin immediately after ROSC is obtained to avoid re-arrest.

Table 2. Treatable Causes of Cardiac Arrest


H’s T’s
Hypoxia Toxins
Hypovolemia Tamponade (cardiac)
Hydrogen ion (acidosis) Thrombosis
• Pulmonary embolism
Hypoglycemia • Coronary thrombosis
Hypo/hyperkalemia
Tension pneumothorax
Hypothermia

Patient Case

5. V.B., a 62-year-old man with an unknown medical history, comes to your ED altered and incoherent. He is
admitted to the ED for observation, where he suddenly becomes unconscious and pulseless. The ED staff
immediately initiates CPR for V.B., who is found to be in PEA. Which statement best describes appropriate
cardiac arrest treatment?
A. Survival from PEA is solely dependent on medications and advanced airways, so airway intubation and
line placement take priority over CPR.
B. The treatable causes of PEA cardiac arrest should be reviewed and addressed while CPR and laboratory
tests are sent.
C. Because the patient is in PEA, pads should be placed, the patient cleared, and shock delivered immediately.
D. No strategies exist for post-arrest care to avoid re-arrest or improve outcomes from the PEA cardiac arrest.

ACCP Updates in Therapeutics® 2022: Critical Care Pharmacy Preparatory Review and Recertification Course

736
Cardiovascular Critical Care II

b. Medication background (rhythm independent)


i. Goal: Increase myocardial blood flow during CPR and help achieve ROSC
ii. Drug delivery
(a) Central intravenous administration is recommended, if available. Central line placement
should not interrupt CPR. The advantage of central administration is higher peak
concentration and shorter drug circulation times than with peripheral routes (Crit Care
Med 1988;16:1138-41; Ann Emerg Med 1981;10:73-8; Ann Emerg Med 1981;10:417-9).
(b) If peripheral administration of medications is necessitated, a bolus injection should be
followed by a 20-mL bolus of an intravenous fluid (e.g., 0.9% sodium chloride) to facilitate
drug flow from the extremity to the central circulation (Am J Emerg Med 1990;8:190-3).
(c) If proximal humerus intraosseous (IO) access is used, administration and drug delivery
are similar to central venous access. If proximal or distal tibial intraosseous access is
used, administration and drug delivery are similar to peripheral venous access and would
necessitate a 20-mL bolus or pressure bag to facilitate drug administration. In a randomized
cadaver study, tibial IO access has been shown to be easier to obtain, which allows for
faster medication administration (Medicine 2016; 95: e3724). Recent data analyses suggest
that intraosseous access is associated with poorer OHCA survival and fewer favorable
neurologic outcomes, even after controlling for many other variables (Ann Emerg Med
2018;71:588-96). Intraosseous access may have initially been selected by first responders
because of perceived complexity and influential patient characteristics. However, additional
analysis has found that tibial intraosseous access compared with intravenous access is
associated with a lower likelihood of ROSC and survival to hospitalization (Resuscitation
2017;117:91-6). The most recent guidelines now support the use of intravenous access
preferentially unless it cannot be obtained (Circulation 2020;142:S366-S468). In these
circumstances, intraosseous access is reasonable.
(d) Endotracheal (ET) delivery
(1) NAVEL acronym summarizes medications that have been studied and shown to have
tracheal absorption by ET tube delivery. N – naloxone, A – atropine, V – vasopressin,
E – epinephrine, L – lidocaine.
(2) Serum concentrations of medications are lower when given by ET delivery.
(3) Optimal ET doses unknown; typically, they are 2–2.5 times intravenous doses but may
be higher (e.g., up to 3–10 times higher for epinephrine) (Crit Care Med 1987;15:1037-
9).
(4) Should be diluted with either 5–10 mL of 0.9% sodium chloride or sterile water and
injected directly into the ET tube, ideally during pulse check to avoid medication
expulsion back out the endotracheal tube (Crit Care Med 1994;22:1174-80)
(e) Peak effect of intravenous or intraosseous medication delayed 1–2 minutes during CPR.
(f) Theoretical concern that giving high-dose bolus vasopressors after ROSC following
defibrillation may precipitate cardiac instability and re-arrest. May be avoided by using
physiological monitoring such as quantitative waveform capnography, intra-arterial
pressure monitoring, or continuous central venous oxygen saturation monitoring and
avoiding administration of vasopressors if ROSC occurs (J Emerg Med 2009;38:614-21;
Ann Emerg Med 1992;21:1094-101; Ann Emerg Med 1990;19:1104-6).

ACCP Updates in Therapeutics® 2022: Critical Care Pharmacy Preparatory Review and Recertification Course

737
Cardiovascular Critical Care II

Patient Case

6. M.G., a 58-year-old woman with a history of chronic osteoarthritis and peptic ulcer disease, is admitted to
the MICU with hypovolemic shock caused by a suspected bleeding gastric ulcer. Endoscopy is performed,
confirming the gastric ulceration. The ulcer is cauterized, and M.G. is stabilized. On ICU day 2, M.G.
becomes lethargic, hypoxic, and subsequently pulseless. The MICU team is summoned, and the monitor
reveals VF. Resuscitation efforts begin, and the ACLS algorithm is followed. The patient will receive amio-
darone. Which general principles are most accurate regarding amiodarone administration for M.G.’s VF
arrest?
A. Endotracheal delivery is preferred because all cardiac arrest medications can be delivered by the endo-
tracheal route.
B. Peripheral administration is preferred because the peak concentrations are higher and the circulation
time is shorter than with other routes.
C. Intraosseous administration is preferred because administration and dosing are similar to that for the
endotracheal route.
D. Central intravenous administration is preferred because the peak concentration and circulation time is
shorter than with other routes.

c. Management of VF/pVT (Figure 1): Defibrillation (summary, details available previously in the
IHCA and OHCA chain of survival: Defibrillation section)
i. When VF/pVT detected, CPR should continue until defibrillator (either manual or automatic)
charging period is over.
ii. It is strongly recommended that CPR be performed while the defibrillator is readied because
chest compressions can deliver oxygen and potentially unload the ventricles, increasing the
likelihood that a perfusing rhythm will return after shock is delivered.
iii. Because intentionally delaying defibrillation for CPR has mixed results, it cannot currently be
recommended.
iv. Once defibrillator is charged, patient is “cleared” (ensuring all members of the resuscitation
team are no longer in contact with the patient).
v. Shock is delivered, and CPR is immediately resumed, beginning with chest compressions.
(a) Pulse check is delayed until 2 minutes of CPR is given.
(b) Pause for rhythm and pulse check (less than 10 seconds); continue CPR, if necessary.
vi. If biphasic defibrillator is available:
(a) Provider should use manufacturer’s recommended energy dose (e.g., 120–200 J).
(b) If information unavailable, the maximum dosing can be considered.
(c) Second and subsequent defibrillator energy dosages should be equivalent, and consideration
should be made for escalating energy doses, if possible.
vii. If monophasic defibrillator is available:
(a) Initial energy dose should be 360 J.
(b) Second and subsequent defibrillator energy dosages should be 360 J.
viii. If VF/pVT is terminated by defibrillation and reoccurs, resulting in an arrest, the successful
energy dosage used previously should be considered.
ix. Change of multimodal defibrillator from automatic to manual mode may result in fewer
interruptions of CPR but also an increased frequency of inappropriate shocks (Resuscitation
2007;73:131-6; Resuscitation 2007;73:212-20).
d. Medication therapy for VF/pVT

ACCP Updates in Therapeutics® 2022: Critical Care Pharmacy Preparatory Review and Recertification Course

738
Cardiovascular Critical Care II

i. Previous guidelines suggested medication consideration after one shock and 2 minutes of CPR
(one cycle). Recent data from witnessed OHCA suggest that early administration of epinephrine
is associated with improved survival in shockable rhythms (Resuscitation 2015;96:180-5) and
that every minute beyond 5 minutes significantly increases the odds of death. With IHCA
for shockable rhythms, patients receiving epinephrine within 2 minutes of initial shock was
associated with a decreased odds of survival, ROSC, and good functional outcome (BMJ
2016;353:i1577). Guideline recommendation supports the use of epinephrine for shockable
rhythms after initial defibrillation attempts have failed (Circulation 2019;140:e881-e894).
ii. Vasopressors:
(a) First-line medication is epinephrine. A recent randomized, double-blind, placebo-
controlled trial showed that use of epinephrine in OHCA (around 20% shockable rhythms)
was associated with greater 30-day survival, but there was no difference in neurologically
favorable outcomes because of the severe neurologic impairment of survivors (N Engl J
Med 2018;379:711-21).
(1) Recent meta-analysis of randomized controlled trials found that epinephrine (1 mg
every 3–5 minutes) when compared to placebo for OHCA increased rate of ROSC,
survival to hospital admission and survival to hospital discharge but did not change
discharge with favorable neurological function (Resuscitation 2019;139:106-21).
(2) The traditional dose of epinephrine is considered 1 mg. Recent data analyses suggest
that lower doses (e.g., 0.5 mg) are not associated with a change in survival to hospital
discharge or favorable neurologic outcomes (Resuscitation 2018;124:43-8).
(3) High-dose epinephrine (0.1–0.2 mg/kg) has recently been investigated and although
it may increase the rate of ROSC, it has not been found to increase rate of survival
to hospital admission, survival to hospital discharge, or survival with favorable
neurological function (Circulation 2019;140:e881-e894).
(b) Vasopressin has previously been removed from guidelines because of equivalence of
effect with epinephrine and a drive to simplify treatment (Figure 1; Table 3). Recent meta-
analysis affirms the equivalence of vasopressin with epinephrine (either in isolation or in
combination), supporting the recommendation that vasopressin offers no advantage over
epinephrine in cardiac arrest (Resuscitation 2019;139:106-21).
(c) No other vasopressors (e.g., phenylephrine or norepinephrine) have shown any benefit
compared with epinephrine (JAMA 1992;268:2667-72; Acta Anaesthesiol Scand
1985;29:610-3).
(d) Adding methylprednisolone and vasopressin to epinephrine during ACLS plus stress dose
hydrocortisone for post-ROSC shock compared with placebo may aid in ROSC during
cardiac arrest and improve discharge neurologic function in patients who survive. Role
of steroids may be debated, given the addition of vasopressin in the study arm (JAMA
2013;310:270-9). A recent network meta-analysis suggests that the combination of
vasopressin-steroids-epinephrine was the only vasopressor regimen associated with ROSC
and, in IHCA, also survival (Crit Care Med 2018;46:e443-e451). Guidelines suggest this
treatment bundle can be considered, but further confirmatory data are needed (Circulation
2015;132(suppl 2):S315-S367).
iii. Antiarrhythmics:
(a) No evidence that, in cardiac arrest, any antiarrhythmics increase survival to discharge.
(b) Conflicting evidence on benefit of amiodarone and lidocaine (Table 3) in the prehospital
arrest situation:
(1) Amiodarone or lidocaine may be considered for VF/pVT that is unresponsive to
defibrillation (Circulation 2019;140:e881-e894).

ACCP Updates in Therapeutics® 2022: Critical Care Pharmacy Preparatory Review and Recertification Course

739
Cardiovascular Critical Care II

(2) Investigation has found that amiodarone (300 mg) increased rates of survival to hospital
admission compared to placebo in shock-refractory VF/pVT in those who received at
least 3 shocks and epinephrine (N Engl J Med 1999;341:871-8). Additionally, 5 mg/
kg amiodarone improved survival to hospital admission compared with 1.5 mg/kg
lidocaine (N Engl J Med 2002;346:884-90). These 2 studies did not find evidence
of improved discharge survival. Additionally, a comparison in patients with VF/pVT
considered refractory after at least 1 shock, which found that ROSC was higher in
patients receiving lidocaine compared with those receiving placebo, but not for those
receiving amiodarone. Survival to hospital admission was higher with amiodarone or
lidocaine compared to placebo, but no difference in survival with good neurological
outcome or survival to hospital discharge in any group (N Engl J Med 2016;374:1711-
22). These drugs may be particularly useful for patients with witnessed arrest, for whom
time to drug administration may be shorter (Circulation 2019;140:e881-e894).
(3) Amiodarone should be administered as undiluted intravenous/intraosseous push if
pulseless. Dosing should be 300 mg with repeat dosing of 150mg (Table 3).
(4) If pulse is obtained, the suggested dose is 150 mg, which must be given as slow
intravenous piggyback over at least 15 minutes and is typically followed by a continuous
intravenous infusion.
(5) Hemodynamic effects of bradycardia and hypotension may be partly related to
vasoactive solvents (polysorbate 80) (Am J Cardiol 2002;90:853-9).
(c) Magnesium sulfate (Table 3) is effective for cardiac arrest caused by torsades de pointes.
(i.e., caused by early afterdepolarizations during phase 2 of the action potential).
(1) Not effective when VF/pVT is not associated with torsades de pointes.
(2) May consider for emergency magnesium replacement in patients who sustain cardiac
arrest and are hypomagnesemic.
(3) Optimal dosing has not been established.
(d) In patients with non-shockable rhythms that become shockable in OHCA, investigation has
shown some numeric benefit of antiarrhythmics, perhaps warranting further investigation
in this patient group (Circulation 2017;136:2119-31).

ACCP Updates in Therapeutics® 2022: Critical Care Pharmacy Preparatory Review and Recertification Course

740
Cardiovascular Critical Care II

Figure 1. Adult cardiac arrest algorithm. CPR = cardiopulmonary resuscitation; IO = intraosseous(ly);


IV = intravenous(ly); PEA = pulseless electrical activity; PETCO2 = partial pressure of end-tidal CO2; pVT = pulse-
less ventricular tachycardia; VF = ventricular fibrillation.

ACCP Updates in Therapeutics® 2022: Critical Care Pharmacy Preparatory Review and Recertification Course

741
Cardiovascular Critical Care II

Table 3. Medications Used During Sudden Cardiac Arrest


Primary Mechanism
Dosage, Route,
Medication of Action in Cardiac Clinical Benefits
Frequency
Arrest

Epinephrine 1 mg IV/IO q3–5 min Increases coronary and cerebral


(N Engl J Med 2–2.5 mg ET q3–5 perfusion pressure during CPR
α-Adrenergic agonist
1992;327:1045-50; min (diluted with
effects leading to Increases ROSC
Circulation 1984;69: either 5–10 mL of
vasoconstriction
822-35; Crit Care Med 0.9% sodium chloride Increases survival to hospital
1979;7:293-6) or sterile water) admission and discharge in OHCA

Amiodarone
(N Engl J Med Na+/K+/Ca2+ channel First dose: 300 mg or
and β-receptor 5 mg/kg IV/IO x 1
2002;346:884-90; Conflicting data on clinical outcomes
antagonist
N Engl J Med Second dose: 150 mg compared with lidocaine or placebo for
1999;341:871-8; (Class III IV/IO x 1 OHCA
N Engl J Med antiarrhythmic) Max: 2.2 g/day
2016;374:1711-22)
First dose: 1–1.5 mg/
kg IV/IO x 1 Conflicting data on clinical outcomes
Lidocaine Na+ channel for OHCA; no improvement in overall
(Resuscitation antagonist Subsequent dosing: or discharge survival
1997;33:199-205; 0.5–0.75 mg/kg q5–10
N Engl J Med (Class IB min Insufficient evidence to recommend
2016;374:1711-22) antiarrhythmic) for refractory VF/pVT unless
Max 3-mg/kg amiodarone unavailable
cumulative dose
Magnesium
(Clin Cardiol
Stops EAD in
1993;16:768-74; New 1–2 g diluted in 10
torsades de pointes Can aid in stopping torsades de pointes
Trends Arrhythmias mL of 5% dextrose or
by inhibiting Ca2+ in patients with prolonged QT interval
1991;7:437-42; sterile water IV/IO × 1
channel influx
Circulation
1988;77:392-7)
EAD = early afterdepolarization; ET = endotracheal; IO = intraosseously; IV = intravenously; pVT = pulseless ventricular tachycardia; q = every.

e. Management of PEA/asystole
i. CPR and treatment of reversible causes are vital to treatment of PEA/asystole.
ii. Medication therapy
(a) Epinephrine can be given as soon as feasible (Table 3).
(1) Stepwise increased mortality is seen if first epinephrine dose withheld for more than 4
minutes after non-shockable IHCA (BMJ 2014;348:1-9). A recent randomized, double-
blind, placebo-controlled trial found that use of epinephrine in OHCA (around 80%
non-shockable rhythms) was associated with greater 30-day survival, but there was
no difference in neurologically favorable outcomes because of the severe neurologic
impairment of survivors (N Engl J Med 2018;379:711-21).

ACCP Updates in Therapeutics® 2022: Critical Care Pharmacy Preparatory Review and Recertification Course

742
Cardiovascular Critical Care II

(2) Recommended for all non-shockable rhythms to administer epinephrine as soon as


feasible (Circulation 2015;132(suppl 2):S315-S367). Recent data analyses suggest that
prompt (less than 5 minutes from non-shockable rhythms arrest) administration of
epinephrine was associated with greater 1-year survival (Circulation 2018;137:2041-
51). These results have been found at both the patient level and the hospital system
level (Circulation 2016;134:2105-14). Recent guideline update supports the early usage
of epinephrine as soon as feasible in the nonshockable rhythm context (Circulation
2019;140:e881-e894).
(b) Atropine has been removed from the algorithm because of its lack of therapeutic benefit.
f. Role in treating reversible causes
i. Echocardiography may be helpful, if available, in the management of PEA to help differentiate
the following (Am J Cardiol 1992;70:1056-60):
(a) Intravascular volume status (ventricular volume).
(b) Cardiac tamponade.
(c) Massive pulmonary embolism (right ventricular size, function).
(d) Mass lesions (tumor, clot).
(e) Coronary thrombosis (right and left ventricular function, regional wall motion
abnormalities).
ii. Because hypoxia is often a cause of PEA arrest, more focused attention may be given to
placement of airway and oxygen delivery.
iii. Please see the specific chapters for pulmonary disorders (massive pulmonary embolism and
tension pneumothorax), cardiology (acute myocardial infarction and cardiac tamponade), shock
(hypovolemic shock and oxygen delivery), acid-base disorders (acidemia), endocrinologic
disorders (hypoglycemia), and electrolytes (hypo/hyperkalemia).
g. Controversial interventions in cardiac arrest
i. Sodium bicarbonate
(a) Tissue acidosis and acidemia result during cardiac arrest for several reasons, including
inadequate or absent blood flow, arterial hypoxia, or underlying pathophysiology.
(b) Mainstays of restoring acid-base status include high-quality chest compressions and
appropriate ventilation/oxygenation.
(c) Conflicting evidence exists for the use of sodium bicarbonate, with most data showing
no benefit or poor outcome with use (Ann Emerg Med 1998;32:544-53; Resuscitation
1995;29:89-95; Am J Emerg Med 1992;10:4-7; Chest 1990;97:413-9; Resuscitation
1989;17(suppl):S161-172; discussion S199-206).
(d) Recent data in patients with prolonged CPR efforts (> 20 mins) primarily from VF (~80%)
demonstrated an association with increased rate of ROSC (Am J Emerg Med 2016; 34:225-
9) after complex retrospective analysis. Caution should be used extrapolating results
because significant limitations exist in the study design and all patients included likely
were in the metabolic phase of VF (JAMA 2002;288:3035–8) and had pH values < 7.1.
Recent prospective, observational, propensity-matched data suggest that OHCA use of
prehospital sodium bicarbonate was associated with a decreased probability of favorable
neurologic outcomes and survival (Resuscitation 2017;119;63-9).
(e) Detrimental effects may be associated with sodium bicarbonate in cardiac arrest, including:
(1) Compromised coronary perfusion pressure by reducing systemic vascular resistance
(JAMA 1991;266:2121-6).
(2) Shifting the oxyhemoglobin dissociation curve to the left by creating an extracellular
alkalosis and decreased release of oxygen.
(3) Causing hypernatremia and subsequent hyperosmolarity leading to hyperviscosity,
potentially impairing blood flow.

ACCP Updates in Therapeutics® 2022: Critical Care Pharmacy Preparatory Review and Recertification Course

743
Cardiovascular Critical Care II

(4) Producing excess CO2 through rapid dissociation, which can freely diffuse
intracellularly (e.g., myocardial and cerebral cells) and cause intracellular acidosis
(Science 1985;227:754-6).
(5) Inactivation of concurrently administered catecholamines (e.g., epinephrine) (Hosp
Pharm 1969;4:14-22).
(f) Certain circumstances may warrant sodium bicarbonate use such as tricyclic antidepressant
overdose, bicarbonate (HCO3-)-wasting causes of metabolic acidosis, and hyperkalemia.
In addition, it should be noted that endogenous catecholamines have a blunted response
during acidosis which may improve if the acidosis is corrected. Initial dosage should usually
be 1 mEq/kg intravenous push with monitoring of clinical status, HCO3- concentration,
laboratory values, and blood gas analysis.
ii. Calcium
(a) No trial has established any impact on survival in either IHCA or OHCA (Ann Emerg Med
1985;14:626-9; Ann Emerg Med 1985;14:630-2).
(b) Consider in patients with preexisting hypocalcemia and signs and symptoms of acute
hypocalcemia (e.g., severe tetany or seizures).
iii. Atropine
(a) No prospective studies have evaluated atropine for bradycardic PEA or asystolic cardiac
arrest.
(b) Conflicting results exist from retrospective analyses and case reports (Acta Anaesthesiol
Scand 2000;44:48-52; Ann Emerg Med 1984;13:815-7; Ann Emerg Med 1981;10:462-7).
(c) Atropine has not been associated with harm in treating bradycardic PEA or asystolic
cardiac arrest, but because of the lack of convincing evidence of benefit, it is no longer
recommended for cardiac arrest but is reserved for symptomatic/life-threatening
bradycardia.
iv. Intravenous fluids
(a) Normothermic, hypertonic, and chilled fluids have been evaluated in animal models and
small human studies, with no survival benefit.
(b) If hypovolemic shock is the suspected cause of the cardiac arrest, fluid resuscitation should
be initiated immediately.
v. For indications of fibrinolysis for cardiac arrest, please see the Pulmonary chapter for treatment
of pulmonary embolism and the Cardiology chapter for treatment of acute myocardial infarction.
vi. Pacing
(a) Transcutaneous, transvenous, and transmyocardial pacing is not beneficial in cardiac arrest
and does not improve ROSC or survival.
(b) Not recommended for routine use in cardiac arrest.
vii. Dextrose
(a) Animal data has demonstrated that dextrose administration before, during, or after cardiac
arrest leads to higher rates of mortality and worse neurological outcomes (J Crit Care.
1987;2:4–14; Surgery 1990;72:1005–11; Acta Anaesthesiol 1986;100:505–11).
(b) Human retrospective data from IHCA demonstrated that dextrose administration during
resuscitation was associated with a significantly decreased chance of survival to discharge
and good neurological outcome (Crit Care 2015;19:160[1-8]).
viii. Cyclosporine - A randomized, single-blind trial found no impact on target organ damage,
survival, or favorable neurologic function, contrary to experimental evidence (JAMA Cardiol
2016;1:557-65).
h. For information on acute symptomatic arrhythmias (bradycardias and tachycardias), see the
Cardiovascular Critical Care I chapter.

ACCP Updates in Therapeutics® 2022: Critical Care Pharmacy Preparatory Review and Recertification Course

744
Cardiovascular Critical Care II

D. Post–Cardiac Arrest Care and Recovery (IHCA and OHCA - chain of survival): Objectives of post–
cardiac arrest care can be divided into initial and subsequent (Circulation 2010;122:S768-786; Circulation
2008;118:2452-83; Circulation 2020;142:S366-S468).
1. Initial
a. Optimize hemodynamics: Target MAP 65–100 mm Hg, central venous pressure 8–12 mm Hg,
central venous oxygen saturation greater than 70%, urine output greater than 1 mL/kg/hour, and
normal serum lactate.
i. Consider the patient’s normal BP, cause of arrest, and severity of myocardial dysfunction for
all values above.
ii. Use intravenous crystalloids and colloids, continuous vasopressors and inotropes, transfusions,
and renal replacement as needed to meet target goals.
iii. In a randomized study of vasopressor dependent shock post-cardiac arrest, the provision of
corticosteroids (hydrocortisone 100 mg intravenously every 8 hours x 7 days or until shock
reversal) did not improve time to shock reversal, rate of shock reversal, or clinical outcomes
(Crit Care 2016;20:821-8]) and thus should be avoided unless otherwise indicated.
b. Transfer patient (OHCA) to a system or unit (IHCA) that can provide advanced post–cardiac
arrest care, including continuous electrocardiographic (ECG) monitoring with immediate 12-lead
ECG, central intravenous access if possible, coronary reperfusion, and/or targeted temperature
management.
c. Try to identify and treat the reversible causes of cardiac arrest (Table 2). Laboratory and diagnostic
tests should be performed to aid in identifying a potential underlying cause.
2. Continued management and additional emergency activities
a. Consider body temperature regulation (“targeted temperature management” [TTM], also known
as therapeutic hypothermia). Should be considered for any patient with ROSC who does not follow
commands (i.e., comatose) after cardiac arrest.
i. Early evidence suggested significant benefit for primarily shockable (VF/pVT) OHCA (N Engl
J Med 2002;346:549-56; N Engl J Med 2002;346:557-63; Circulation 2010;122:S768-786).
Recent meta-analysis demonstrated a similar nonsignificant reduction in mortality and poor
neurological outcome (Am J Med 2016;129:522-527) when including all randomized studies.
After exclusion of one trial that allowed for TTM in the control arm, there was a significant
reduction in poor neurological outcome when implementing TTM. A recent Cochrane analysis
found that conventional TTM compared with no TTM conferred the benefit of favorable
neurological outcome and survival with only a slight increase in the incidence of pneumonia
and hypokalemia (Cochrane Database Syst Rev 2016;2:CD004128). The Targeted Temperature
Management 2 (TTM2) study, published in 2021 comparing goal temperature of 33°C with
normothermia (N Engl J Med 2021;384:2283-94), found similar outcomes surrounding
neurologic function and overall survival. This has led some to advocate for normothermia
(i.e., avoidance of fevers) versus hypothermia. In addition, in the TTM2 study, patients who
underwent therapeutic hypothermia had more adverse events.
ii. Evidence for nonshockable rhythms IHCA and OHCA has conferred results similar to those in
early studies of shockable rhythms demonstrating increased survival and better neurological
outcomes (Circulation 2015;132:2146-51).
iii. Optimal targets, timing, duration, and other variables still unclear:

ACCP Updates in Therapeutics® 2022: Critical Care Pharmacy Preparatory Review and Recertification Course

745
Cardiovascular Critical Care II

(a) Goal target body temperature 32°C–36°C given data showing that targeting 36°C instead
of 33°C may be equivocal (Circulation 2015;132(suppl 2):S315-S367; N Engl J Med
2013;369:2197-206; N Engl J Med 2002;346:549-56; N Engl J Med 2002;346:557-63).
Additional investigations have found that targeting a warmer temperature (36°C) may lead
to reduced times in goal temperature range but with increased fever rates (Resuscitation
2017;113:39-43). Similar results have been found in other studies (Crit Care Med
2020;48:362-9; Resuscitation 2019;143:142-7). The TTM2 study, as mentioned earlier,
has led some to consider strict avoidance of hyperthermia, using active maintenance
of normothermia instead of targeting hypothermic temperature goals (N Engl J Med
2021;384:2283-94).
(b) Duration of at least 12 hours; optimally, a minimum of 24 hours. A 48-hour duration
did not seem to confer any benefit compared with a 24-hour duration in a multicenter
randomized study (JAMA 2017;318:341-50). Multiple studies are ongoing investigating the
optimal duration of TTM.
(c) Initiate therapeutic hypothermia as soon as possible (within 2 hours, if possible), with goal
temperature attainment within 6–8 hours; however, several retrospective studies have not
confirmed the timing of initiation or the timing of temperature attainment as predictors
of neurologic outcome (Acta Anaesthesiol Scand 2009;53:962-34; Int J Cardiol 2009;
133:223-8). Intra-arrest TTM is currently being investigated but human data is limited.
Prehospital infusion of cold intravenous fluids for OHCA in several randomized controlled
studies conferred no benefit and may increase the number of complications; thus, it is not
recommended (Circulation 2015;132(suppl 2):S315-S367; Circulation 2016;134:797-805).
(d) Modality for cooling includes feedback-controlled endovascular systems, surface-cooling
devices, ice packs/bags, cooling blankets, and/or iced isotonic fluids.
(e) Axillary or oral temperature monitoring is inadequate for therapeutic hypothermia (Acta
Anaesthesiol Scand 1998;42:1222-6; J Cardiothorac Vasc Anesth 1996;10:336-41) and
requires central/core temperatures by esophageal, bladder (avoid in anuric patients), or
pulmonary artery temperature monitoring. Ideally, the monitoring modality chosen will
be used for other indications as well.
iv. Major complications of therapeutic hypothermia include those related to the cooling and
rewarming process and those found in patients undergoing targeted temperature management
(Table 4). The adverse events of sepsis, myoclonus, seizures, and hypoglycemia within 72 hours
of targeted temperature management have been associated with poor neurologic outcome (Crit
Care 2015;19:283-96).

ACCP Updates in Therapeutics® 2022: Critical Care Pharmacy Preparatory Review and Recertification Course

746
Cardiovascular Critical Care II

Table 4. Major Organ-Specific Complications of Therapeutic Hypothermia


Musculoskeletal Shivering:
(Crit Care Med 2015;43:2228-38; Body’s natural response to hypothermia, preceded by arteriovenous
N Engl J Med 2013;369:2197-206; vasoconstriction; most common complication of hypothermia; can increase
Anesthesiology 2009;111:110-5; Br metabolic heat production by 600%, thereby slowing the induction of
J Anaesth 2005;94:756-62; N Engl hypothermia. Typically slows or stops at core temperatures < 33.5°C
J Med 2002;346:549-56; N Engl 1.  Agents that decrease the shivering threshold (i.e., decreases the
J Med 2002;346:557-63; JAMA temperature at which shivering will occur):
1997;277:1127-34; Am J Physiol a)  Scheduled acetaminophen 650 mg q4–6 hr or buspirone 30 mg
1960;198:481-6) q12 hr reduces the shivering threshold by 0.2ºC–0.4ºC
b)  Magnesium sulfate reduces the shivering threshold by ~0.3°C
c)  Meperidine decreases the shivering threshold by up to 2°C but
should be avoided because of decreased effective glomerular
filtration rate (GFR) during hypothermia and increased risk of
seizures when meperidine is used in patients with decreased GFR
d)  Dexmedetomidine and clonidine decrease the shivering threshold
by ~0.8°C, but extreme caution should be used because of the
hypotensive and bradycardic effects of both agents
e)  Propofol decreases the shivering threshold by ~0.6°C and has a
linear relationship between serum concentrations and reduction in
body temperature; caution should be used, given the hypotensive
and bradycardic effects
2.  Options to stop shivering include:
a)  Continuous or as-needed paralytics can be used for prevention
and treatment of shivering (see the chapter on management of
paralytics for appropriate selection and dosing of agent[s])
—Hypothermia decreases clearance and prolongs the duration of
neuromuscular blockade (see Table 5 for examples)
—Train of four is not a reliable method of monitoring
during hypothermia; clinical monitoring or continuous EEG
(electroencephalography) may be warranted
b)  Surface warming if using internal cooling devices
c)  Chilled fluids to promote faster core temperature reduction

ACCP Updates in Therapeutics® 2022: Critical Care Pharmacy Preparatory Review and Recertification Course

747
Cardiovascular Critical Care II

Table 4. Major Organ-Specific Complications of Therapeutic Hypothermia (continued)


Neurologic Sedation and Analgesia:
(Circulation 2010;122:S729-767; Adequate pain control and sedation must be used
Circulation 2008;118:2452-83; Target Richmond Agitation and Sedation Scale score of -3 to -5 (see chapter
Pharmacotherapy 2008;28:102-11; on sedation for agent selection and dosing) during hypothermia
Ther Hypothermia Temp Manag Accumulation of parent and active metabolites can be expected for each
2016;6:189-93) analgesic and sedative, which lead to prolonged sedation and poten-
tially untoward adverse effects (see Table 5 for examples)
Seizures:
Possible complication of cardiac arrest and therapeutic hypothermia
Consider benzodiazepines as first line to abort seizure
Phenytoin, barbiturates, valproic acid, and propofol can all be used, with
vigilant monitoring of adverse effects because of decreased clearance
(see Table 5 for examples; for dosing and monitoring values, see the
Status Epilepticus chapter)
Intracranial Pressure:
Increases post-cardiac arrest likely because of ischemic injury and
cytotoxic edema and is partially attenuated by TTM
Major increase appears to occur during rewarming phase of TTM
Uncertain if targeted therapy would impact outcomes
Cardiac Arrhythmias:
(Heart Lung 2001;30:161-3; Int Include VT, VF, and atrial fibrillation
Anesthesiol Clin 1964;2:803-27) If life threatening, should consider discontinuing therapeutic hypothermia
and active rewarming
Sinus bradycardia is common and, in isolation, should not be treated unless it
leads to hemodynamic instability (e.g., hypotension or organ dysfunction)
ECG observations:
Prolonged PR, QRS, and QT intervals
Caution should be exercised when using medications that prolong the QT
interval
Hemodynamics:
Decreased cardiac output
Fluid shifts away from central compartment
Hepatobiliary Elevated transaminases
(Pharmacotherapy 2008;28:102-11; Reduced activity of non-cytochrome and cytochrome P450–mediated
Ther Drug Monit 2001;23:192-7; metabolism
Anesthesiology 2000;92:84-93; See Table 5 for examples of metabolic changes in selected medications
Clin Pharmacol Ther 1979;25:1-7) during hypothermia
Endocrinologic Hyperglycemia caused by decreased insulin production and effect in
(N Engl J Med 2002;346:557-63; periphery, increased gluconeogenesis, and glycogenolysis
Endocrinology 1970;87:750-5) Continuous insulin infusions may be necessary for glucose control. Goal
should be < 180 mg/dL without inducing hypoglycemia

ACCP Updates in Therapeutics® 2022: Critical Care Pharmacy Preparatory Review and Recertification Course

748
Cardiovascular Critical Care II

Table 4. Major Organ-Specific Complications of Therapeutic Hypothermia (continued)


Renal Decreased effective glomerular filtration (urine output may increase
(Resuscitation 2004;60:253-61; J because of cold diuresis, but not effective clearance)
Neurosurg 2001;94:697-705) Electrolyte shifts (K+, PO43-, Na+, Ca2+) during cooling phase; reverse
upon rewarming, thus caution should be exerted with replacement
Hematologic
Coagulopathy caused by thrombocytopenia, impaired activation and
(Pharmacotherapy 2008;28:102-11;
activity of clotting factors, impaired platelet function
Br J Haematol 1999;104:64-8;
Actively bleeding patients should not be cooled
Crit Care Med 1992;20:1402-5)

Table 5. Pharmacokinetic and Pharmacodynamic Changes of Selected Medications During Hypothermia


Fentanyl Plasma concentration increases by 25% with a 3.7-fold decrease in clearance
Morphine Receptor affinity (µ) decreases as temperature decreases
Propofol Plasma concentration increases by 28%; decreased clearance
Midazolam Clearance decreases by about 11% per degree below 36.5°C
Rocuronium Clearance decreases by 50%, increases duration of action 2-fold
Vecuronium Clearance decreases by 11% per degree Celsius; increases duration of action 2-fold
Eliminated by Hofmann elimination, which is a temperature-dependent enzymatic process;
Cisatracurium
anticipate prolonged activity
AUC (area under the concentration-time curve) increases by 180%; clearance and elimination
Phenytoin
rate constant decrease by 50%
Information from: Pharmacotherapy 2008;28:102-11; Ther Drug Monit 2001;233:192-7; Anesthesiology 2000;92:84-93; Br J Haematol 1999;104:68-8; Eur J
Anaesthesiol Suppl 1995;1:95-106; Clin Pharmacol Ther 1979;25:1-7)

v. Rewarming should be a passive process over 8 hours (around 0.33°C–0.5°C per hour)
(Acta Anaesthesiol Scand 2009;53:926-34; N Engl J Med 2002;346:557-63; N Engl J Med
2002;346:549-56).
vi. Though the exact temperature goals and duration of TTM is still being investigated, avoidance
of fever and hyperthermia is clear and should be maintained during the first 48 hours after
cardiac arrest (Arch Intern Med 2001;161:2007-12; Resuscitation 2013;84:1062-7; J Crit Care
2017;38:78-83).
b. Identify and treat acute coronary syndromes.
i. Cardiovascular disease and acute coronary ischemia are the most common causes of cardiac
arrest (Am Heart J 2009;157:312-8; N Engl J Med 1997;336:1629-33).
ii. Consideration of treatment of acute coronary syndromes should not be deferred in patients who
are comatose or when therapeutic hypothermia is used.
iii. Increased emphasis on early angiography, particularly in OHCA because of the high prevalence
of acute coronary occlusion as the cause for the cardiac arrest (Circulation 2015;132(suppl
2):S315-S367). Recent analysis suggests that risk stratification and admission prognosis can
help determine which patients will benefit from angiography (JACC Cardiovasc Interv
2018;11:249-56).
iv. Effects of antiplatelet and antithrombotic drugs augmented by derangements in normal platelet
activation/coagulation pathways caused by hypothermia.
v. See the Cardiology chapter for the workup and treatment of acute coronary syndromes.
c. Optimize mechanical ventilation.
i. Goal arterial oxygen saturation is 94% or greater.

ACCP Updates in Therapeutics® 2022: Critical Care Pharmacy Preparatory Review and Recertification Course

749
Cardiovascular Critical Care II

ii. Avoid hyperventilation or over-bagging to avoid increase in intrathoracic pressure and decrease
in cardiac output.
iii. Goal Paco2 is 40–45 mm Hg or Petco2 35–40 mm Hg.
d. Support organ systems
i. Vasopressor/inotropes to support end-organ perfusion (see Table 6)
(a) Adrenergic medications should not be administered with alkaline solutions or sodium
bicarbonate because they are inactivated (Hosp Pharm 1969;4:14-22).
(b) Central line is recommended because any agent with α-agonist properties can lead to
extravasation and tissue necrosis.

Table 6. Common Vasoactive Agents Used After Cardiac Arrest


Typical Dosing
Medication Range Clinical Pearls
(mcg/kg/min)
Mixed α and β (1 > 2) activity
Used to treat severe hypotension (e.g., SBP < 70 mm Hg)
Epinephrine 0.03–0.3 Used for symptomatic bradycardia
Used for hemodynamically unstable anaphylactic reactions
Higher doses associated with increased α1 activity
α > β (1 > 2) receptor activity
Used to treat severe hypotension (e.g., SBP < 70 mm Hg)
Norepinephrine 0.03–0.3 Should be used in volume-resuscitated patients
First line for septic shock
Higher doses associated with increased α1 activity
Pure α-agonist
Used to treat severe hypotension (e.g., SBP < 70 mm Hg)
Phenylephrine 0.3–3
Should be used in volume-resuscitated patients
Avoid in patients with low cardiac output
In general, dose-related receptor activity:
2–5 mcg/kg/min dopamine receptor,
5–10 mcg/kg/min β1-receptor,
> 10 mcg/kg/min α1-receptor
Dopamine 2–20
Does not provide exclusive receptor activity across dosing ranges and, thus,
can be arrhythmogenic at any dose
Use cautiously in patients with a history of heart disease or arrhythmias
Useful for patients with bradycardia and hypotension
Predominance of inotropic properties but with activity on β1 > β2 > α1-receptor
Used to treat low cardiac output
α1-agonist and β2-agonist counterbalance, leading to little change in systemic
vascular resistance
Dobutamine 2–20
Can lead to vasodilation in select patients
Less systemic or pulmonary vasodilation than milrinone
More tachycardia than milrinone but similar risk of ventricular arrhythmias
Use cautiously in patients with a history of arrhythmias

ACCP Updates in Therapeutics® 2022: Critical Care Pharmacy Preparatory Review and Recertification Course

750
Cardiovascular Critical Care II

Table 6. Common Vasoactive Agents Used After Cardiac Arrest (continued)


Typical Dosing
Medication Range Clinical Pearls
(mcg/kg/min)
Phosphodiesterase type 3 inhibitor leading to increased intracellular cAMP
leading to influx of calcium and subsequently inotropy and chronotropy
Used to treat low cardiac output
Loading dose rarely used because of significant systemic hypotension
Milrinone 0.25–0.75 Longer duration of activity than dobutamine
Accumulates in renal dysfunction
More systemic and pulmonary vasodilation than dobutamine
Less tachycardia than dobutamine but similar risk of ventricular arrhythmias
Use cautiously in patients with a history of arrhythmias
See chapter on shock for a detailed discussion regarding selection of agent, dosing, pharmacology, and clinical considerations.
a

SBP = systolic blood pressure

ii. Glucose management (Circulation 2010;122:S768-786; Circulation 2008;118:2452-83)


(a) Avoidance of severe hypoglycemia (40 mg/dL or less)
(b) Target moderate glucose control: 144–180 mg/dL
(c) May require continuous insulin infusion to maintain goals
iii. Seizure control/prevention (Neurology 1988;38:401-5; JAMA 1985;253:1420-6): Seizures,
myoclonus, or both occur in 5%–15% of adult patients who achieve ROSC and are more
common in those who remain comatose.
(a) Clonazepam, valproic acid, and levetiracetam are all effective for myoclonus, but
clonazepam should be considered first line.
(b) Benzodiazepines, phenytoin, valproic acid, propofol, and barbiturates are all effective for
post–cardiac arrest seizures.
(c) For dosing and monitoring guidelines, see the Status Epilepticus chapter.
iv. Renal dysfunction: The indications for initiating renal replacement therapy in cardiac arrest
survivors are the same as in critically ill patients in general (Lancet 2005;365:417-30).
v. Infectious disease: Association has been shown between the use of prophylactic antibiotics and
decreased incidence of pneumonia and sepsis (Resuscitation 2015;92:154-9) in patients who
underwent TTM to goal 32-34°C. Impact on length of stay or neurological outcome was not
demonstrated and no information is available regarding impact of higher temperature goals on
above outcomes.
e. Assess prognosis.
i. Brain injury and cardiovascular instability are the main determinants of survival after cardiac
arrest (Intensive Care Med 2004;30:2126-8).
ii. If therapeutic hypothermia is considered, a delay of 72 hours after rewarming should be
implemented for prognostication.
iii. If therapeutic hypothermia is not considered:
(a) Prognostication should wait until 72 hours after ROSC.
(b) No clinical neurologic sign has shown to be predictive of neurologic prognosis less than 24
hours after arrest (Neurology 2006;66:62-8; Crit Care Med 1987;15:820-5).
iv. Prudent to perform any prognostication after removal of opioids, sedatives, paralytics, and so
forth.
v. Using several modalities of testing including clinical examination, neurophysiological testing,
and imaging is recommended.
f. Recovery – Care and support during recovery (Circulation 2020;142(16_suppl_2):S366-468)

ACCP Updates in Therapeutics® 2022: Critical Care Pharmacy Preparatory Review and Recertification Course

751
Cardiovascular Critical Care II

i. Assist survivors with rehabilitation needs. Multimodal rehabilitation assessment and treatment
of physical, neurologic, cardiopulmonary, and cognitive impairments are recommended before
hospital discharge.
ii. Cardiac arrest survivors and their caregivers are recommended to receive comprehensive,
multidisciplinary discharge planning, including medical and rehabilitative treatment
recommendations, and to return to activity/work expectations.
iii. Structured assessment for anxiety, depression, posttraumatic stress, and fatigue is recommended
for cardiac arrest survivors and their caregivers.
iv. New recommendations also call for provider debriefing and referral for emotional support,
when needed, for those responding to and participating in resuscitation efforts.

Patient Case

7. K.G., a 71-year-old woman with a history of atrial fibrillation, coronary artery disease status post three-
vessel coronary bypass artery grafting 6 years ago, diabetes, and osteoarthritis, is being admitted to your
MICU for therapeutic hypothermia after PEA arrest and subsequent ROSC. K.G., who remained comatose
after the ROSC, was intubated and is now hemodynamically stable (BP 94/72 mm Hg and HR 86 beats/
minute). Which is the most accurate statement regarding targeted temperature management (therapeutic
hypothermia)?
A. Hypoglycemia is a common complication of hypothermia and may require continuous dextrose infusions.
B. Dose modifications or avoidance of cytochrome P450 (CYP)–metabolized medications should occur
during hypothermia.
C. The optimal duration of therapeutic hypothermia should be at least 72 hours to affect survival.
D. Temperature targets should be 28°C–30°C to improve the neurologic recovery.

II.  HYPERTENSIVE CRISIS

A. Definitions (Cardiol Clin 2012;30:533-43; Chest 2007;131:1949-62; Cardiol Clin 2006;24:135-46):


1. Hypertensive urgency: A systolic blood pressure (SBP) of 180 mm Hg or greater and/or a diastolic blood
pressure (DBP) of 110 mm Hg or greater without evidence of target organ damage,
2. Hypertensive emergency: Presence of an abrupt significantly elevated BP (often defined as SBP greater
than 200 mm Hg and/or DBP greater than 120 mm Hg) with concurrent target organ dysfunction (e.g.,
acute kidney injury/failure, heart failure exacerbation, obtundation). Table 7 lists example conditions
that, when accompanied by high BP, define hypertensive emergency.
3. MAP: Average pressure in the arteries experienced during one cardiac cycle. Calculated by MAP = 1/3
SBP + 2/3 DBP.

ACCP Updates in Therapeutics® 2022: Critical Care Pharmacy Preparatory Review and Recertification Course

752
Cardiovascular Critical Care II

Table 7. Examples of Acute Target Organ Damage and Clinical Presentations


Eclampsia, preeclampsia Hypertensive encephalopathy
Acute shortness of breath, flash pulmonary edema, or acute left
Acute kidney injury/failure
ventricular dysfunction
Acute aortic dissection (type A or B) Acute intracranial bleeding (nontraumatic)
Seizures Acute myocardial ischemia/infarction
Retinopathy Cerebral infarction

B. Estimated that 1-3% of patients with hypertension will experience a hypertensive crisis (Acta Med Scand
Suppl 1981;650:1-62; Am J Cardiol 2011;108:1277-82),

C. 10-year survival approaches 70% (Q J Med 1993;96:485-93), with 1-year survival greater than 90%.

D. Common Causes (Cardiol Clin 2012;30:533-43; Cardiol Clin 2006;24:135-46):


1. Intoxications – Cocaine, amphetamines, phencyclidine hydrochloride, stimulant diet pills
2. Nonadherence to antihypertensive regimen
3. Withdrawal syndromes (e.g., clonidine or β-antagonists)
4. Drug-drug/drug-food interactions (e.g., monoamine oxidase inhibitors and tricyclic antidepressants,
antihistamines, or tyramine)
5. Spinal cord disorders
6. Pheochromocytoma
7. Pregnancy

E. Management:
1. Hypertensive urgency: Lower BP slowly during the first 24–48 hours using oral medications (often
home regimen reinitiation). Does not require an ICU admission for treatment.
2. Hypertensive emergency: Requires ICU monitoring and intravenous medications. See goals listed in
Table 8.

Table 8. Time Interval for BP Lowering with Hypertensive Emergencya


Goal Time BP Target
< 60 min ↓ DBP by 10%–15% or MAP by 25% with goal DBP ≥ 100 mm Hg
2–6 hr SBP 160 mm Hg and/or DBP 100–110 mm Hg
6–24 hr Keep above BP goals (hours 2–6) during first 24 hr
24–48 hr Outpatient BP targets
a
See compelling conditions in the text that follows.

a. A 25% reduction in MAP during the first hour is targeted to maintain cerebral perfusion (blood
flow autoregulation) and to not precipitate ischemia, which has been found with >50% reductions
(Stroke 1984;15:413-6).
b. If neurologic function deteriorates during the initial 25% decrease (or during subsequent lowering),
therapy should be discontinued or a lesser percent decrease targeted (N Engl J Med 1990;323:1177-
83).

ACCP Updates in Therapeutics® 2022: Critical Care Pharmacy Preparatory Review and Recertification Course

753
Cardiovascular Critical Care II

c. Compelling conditions leading to unique treatment timing and goals:


i. Acute aortic dissection
(a) Propagation of acute aortic dissection is dependent on arterial BP and shear stress (force of
left ventricular contraction as a function of time).
(b) HR and contractility control can minimize shear stress and, together with BP, become a
target of management.
(c) Goal HR less than 60 beats/minute and SBP less than 100 mm Hg as soon as possible
(within 5–10 minutes).
ii. Acute ischemic stroke
(a) Hypertension with ischemic stroke is an adaptive response to maintain cerebral perfusion
pressure to the brain.
(b) Cerebral perfusion pressure (CPP) equals mean arterial pressure minus intracranial
pressure (ICP): CPP = MAP − ICP.
(c) Treatment should occur only if (1) thrombolytic therapy is required (goal SBP less than 185
mm Hg and DBP less than 110 mm Hg before thrombolysis initiation - has been shown to
decrease risk of bleeding), (2) other target organ damage occurs, or (3) SBP is greater than
220 mm Hg and/or DBP is greater than 120 mm Hg (Stroke 2013; 44: 870-947).
(d) If treatment in indicated (outside of thrombolytic goal stated above): goal 10%–20% MAP
reduction over 24 hours (Cardiol Clin 2012;30:533-43; CNS Drugs 2015; 29: 17-28).
iii. Intracerebral hemorrhage
(a) BP reduction goals will be based on individual factors, including medical history; ICP,
if known; demographics such as age; presumed cause of hemorrhage (e.g., arteriovenous
malformation); and interval since onset.
(b) High BP is associated with worse outcomes, including hematoma expansion, neurological
deterioration, death, and inability to perform activities of daily living after intracranial
hemorrhage (Eur J Neurol 2013;20:1277-83; Stroke 2013;44:1846-51; J Hypertens
2008;26:1446-52).
(c) BP reductions in patients without ICP elevations to goal SBP targets of less than 140
mm Hg and/or less than 160 mm Hg have been shown to be safe and may confer benefit
regarding functional recovery (Stroke 2013;44:1846-51; Stroke 2012;43:2236-8; Arch
Neurol 2010;67:570-6; Hypertension 2010;56:852-8; Lancet Neurol 2008;7:391-9; N Engl
J Med 2013; 368: 2355-2365).
(d) It is unclear whether aggressive targets are safe in patients with extreme elevations in
BP (i.e., SBP greater than 220 mm Hg), patients with large hematomas, or those with
elevations in ICP. Aggressive therapy can be considered, though a more modest reduction
to an SBP less than 180 mm Hg or a MAP less than 130 mm Hg over the first 24 hours is
recommended (Stroke 2015;46:2032-60).
3. Agents for BP management of hypertensive emergency
a. The drug of choice for hypertensive emergency is intravenous nitroprusside.
i. Intravenous nitroprusside works rapidly and is safe in the presence of renal and/or hepatic
impairment for short-term use (24 hours or less).
ii. Continuous BP monitoring (e.g., arterial line) is recommended with use because rapid changes
can occur.
iii. Nitroprusside can increase ICP and may result in coronary steal; caution or avoidance should
be considered in patients with elevated ICP and acute myocardial ischemia/infarction.
b. Table 9 summarizes available agents, dosing, onset, duration, and hemodynamic considerations.
Table 10 summarizes possible indications and special considerations.

ACCP Updates in Therapeutics® 2022: Critical Care Pharmacy Preparatory Review and Recertification Course

754
Cardiovascular Critical Care II

Table 9. Medications Used in Hypertensive Emergencies


Cardiac
Medication Usual Dosing Range Onset Duration Preload Afterload
Output
IV 0.25–10 mcg/kg/min
Nitroprusside Titrate by 0.1–0.2 mcg/kg/min q5 Seconds 1–2 min ↓ ↓↓ ↑­
min
IV bolus: 10–20 mg
IV: 10 min
IV: 1–4 hr
Hydralazine IM: 10–40 mg IM: 20 ↔ ↓ ­↑
IM: 2–6 hr
min
q30 min PRN
IV 5–15 mg/hr
Nicardipine Titrate by 2.5 mg/hr 5–10 min 2–6 hr ↔ ↓ ­↑
q5–10 min
IV 1–6 mg/hr
Clevidipine Titrate by 1–2 mg/hr q90s. Max 32 1–4 min 5–15 min ↔ ↓ ­↑
mg/hr
IV 5–200 mcg/min
Nitroglycerin 2–5 min 5–10 min ↓↓ ↓↔ ­
↔↑
Titrate by 5–25 mcg/min q5–10 min
IV 25–300 mcg/kg/min
(bolus of 500 mcg/kg, not often
Esmolol required given short onset) 1–2 min 10–20 min ↔ ↔ ↓
Titrate by 25 mcg/kg/min
q3–5 min
Metoprolol IV bolus: 5–15 mg q5–15 min PRN 5–20 min 2–6 hr ↔ ↔ ↓
IV bolus: 20 mg, may repeat
escalating doses of 20–80 mg q5–10
min PRN 2–5 min,
Labetalol peak 5–15 2–6 hr ↔ ↓ ↓
IV 1–2 mg/min
min
Titrate by 1–2 mg/min q2 hr given
longer half-life
IV bolus: 1.25 mg q6 hr
Enalaprilat Titrate no more than q12–24 hr; max 15–30 min 12–24 hr ↓ ↓ ­↑
dose 5 mg q6 hr
IV bolus: 1–5 mg PRN;
Phentolamine Seconds 15 min ↔ ↓ ­↑
max 15 mg
IV 0.03–1.6 mcg/kg/min
Fenoldopam 10–15 min 10–15 min ↔↓ ↓ ↑­
Titrate by 0.05–1 mcg/kg/min q15 min
IM = intramuscular; IV = intravenous; PRN = as needed.

ACCP Updates in Therapeutics® 2022: Critical Care Pharmacy Preparatory Review and Recertification Course

755
Cardiovascular Critical Care II

Table 10. Indications and Special Considerations for Medications Used for Hypertensive Emergencies (continued)
Medication Indication Special Consideration
Liver failure – cyanide accumulation
Renal failure – thiocyanate accumulation
Most indications
Can draw serum cyanide and thiocyanate concentrations to monitor
(exclusions: ICP
Nitroprusside Toxicity associated with prolonged infusions (> 72 hr) or high doses
elevation, coronary
(> 3 mcg/kg/min)
infarction/ischemia)
May result in coronary steal
Increases ICP
Can result in prolonged hypotension (less predictable dose response)
Hydralazine Pregnancy Risk of reflex tachycardia
Headaches, lupus-like syndrome (with long-term use)
Acute ischemic or Risk of reflex tachycardia
Nicardipine
hemorrhagic stroke Infusion can lead to large fluid volumes administered
Acute ischemic or Formulated in oil-in-water formulation providing 2 kcal/mL of lipid calories
Clevidipine
hemorrhagic stroke Caution for patients allergic to soy or eggs
Coronary ischemia/
infarction Tachyphylaxis occurs rapidly, requiring dose titrations
Adverse effects: Flushing, headache, erythema; often dose-limiting adverse
Nitroglycerin Acute left
effects
ventricular failure
Veno > arterial vasodilator
Pulmonary edema
Contraindicated in acute decompensated heart failure
Should be used in conjunction with an arterial vasodilator for BP management
Aortic dissection
in aortic dissection (initiate esmolol first because of the delayed onset
Esmolol Coronary ischemia/
relative to vasodilators such as nitroprusside)
infarction
Metabolism is organ-independent (hydrolyzed by esterases in blood)
Useful in tachyarrhythmias
Contraindicated in acute decompensated heart failure
Aortic dissection Should be used in conjunction with an arterial vasodilator for BP manage-
Metoprolol Coronary ischemia/ ment in aortic dissection (initiate metoprolol first because of the delayed
infarction onset relative to vasodilators such as nitroprusside)
Useful in tachyarrhythmias
Acute ischemic or
hemorrhagic stroke
May be used as monotherapy in acute aortic dissection
Aortic dissection Contraindicated in acute decompensated heart failure
Labetalol
Coronary ischemia/ Prolonged hypotension and/or bradycardia may be experienced with over-
infarction treatment; dose cautiously

Pregnancy
Acute left Contraindicated in pregnancy
Enalaprilat
ventricular failure Caution in dose adjustments given prolonged duration of action
Catecholamine
Phentolamine excess (e.g., Use in catecholamine-induced hypertensive emergency
pheochromocytoma)

ACCP Updates in Therapeutics® 2022: Critical Care Pharmacy Preparatory Review and Recertification Course

756
Cardiovascular Critical Care II

Table 10. Indications and Special Considerations for Medications Used for Hypertensive Emergencies (continued)
Medication Indication Special Consideration
Risk of reflex tachycardia
Caution with glaucoma
Fenoldopam Most indications
Can cause hypokalemia, flushing
May increase ICP
ICP = intracranial pressure.

c. Certain populations require specific medication therapy approaches:


i. Pregnancy
(a) Severe preeclampsia can only be managed by delivery of the baby.
(b) Magnesium can be considered as an adjunctive therapy to decrease seizure risk or if
seizures develop.
(c) Intravenous medications should only be considered for (1) severely elevated BPs which,
in this population, are considered an SBP greater than 160 mm Hg and/or a DBP greater
than 110 mm Hg, (2) preeclampsia/eclampsia, and/or (3) hypertensive emergency (signs of
target organ damage) in a pregnant patient.
(d) Hydralazine and labetalol are feasible first-line options, and labetalol may have fewer
adverse effects (Am J Health Syst Pharm 2009;66:337-44).
(e) BP target goal for severely elevated BPs, preeclampsia/eclampsia, and hypertensive
emergency in a pregnant patient is 160/110 mm Hg or less with avoidance of abrupt
decreases in BP, which can lead to potentially harmful fetal effects. Because of this
caution, the MAP should be decreased by ~20%-25% over the first few minutes to hours,
and the BP should then be further decreased to the target of 160/110 mm Hg or less over
the subsequent hours.
ii. Catecholamine-induced hypertensive emergency
(a) Phentolamine is the drug of choice because it competitively inhibits α-adrenergic receptors.
(b) β-Selective antagonists are contraindicated unless the patient is fully α-blocked.
iii. Cocaine-induced hypertensive emergency (Ann Emerg Med 2008;52:S18-20; Chest
2007;131:1949-62)
(a) Benzodiazepines are used to target the central effects of cocaine as first-line therapy and
often will result in control of tachycardia and hypertension. Can consider diazepam 5–10
mg intravenously or lorazepam 2–4 mg intravenously titrated to effect.
(b) If central control of cocaine-induced hypertension fails, consider direct α-antagonism with
phentolamine.
(1) Phentolamine 1 mg intravenously; repeat every 5 minutes as needed.
(2) If direct α-antagonism does not gain control or is unavailable, consider additional
antihypertensives:
(A) Nitroglycerin, nicardipine, nitroprusside, or fenoldopam titrated to effect are
viable options (see Tables 9 and 10 for dosing and considerations).
(B) Verapamil and diltiazem decrease coronary vasospasm associated with acute
cocaine intoxication (Am J Cardiol 1994;73:510-3).
(C) Controversy exists regarding the use of β-blockers.
• Labetalol has shown conflicting results regarding ability to control MAP but
does not alleviate cocaine-induced coronary vasoconstriction.
• Consensus opinion recommends β-selective antagonists only if full
α-antagonism is used first.

ACCP Updates in Therapeutics® 2022: Critical Care Pharmacy Preparatory Review and Recertification Course

757
Cardiovascular Critical Care II

iv. Blood pressure variability (BPV)


(a) Concept of BPV emerging as a key therapeutic target in various populations (Stroke
2014;45:2275-9; Eur J Neurol 2013;20:1277-83; J Cardiothorac Vasc Anesth 2014;28:579-
85).
(b) BPV is often expressed as the standard deviation of SBP, MAP, DBP, or the area under the
curve of time spent outside of blood pressure target.
(c) In the intracerebral hemorrhage population, decreased BPV has been correlated with
improved early neurological function (Eur J Neurol 2013;20:1277-83), favorable outcome
(Stroke 2014;45:2275-9), and death or major disability (Lancet Neurol 2014;13:364–73).
(d) Agent selection can influence BPV.
(1) Compared with labetalol, nicardipine has demonstrated superior results regarding
BPV (Neurocrit Care 2013;19:41-47; Neurocrit Care 2008;9:167-176), but impact on
clinical outcomes has not been demonstrated.
(2) In the cardiac surgery population, clevidipine demonstrated better perioperative
BPV compared with nitroglycerin or sodium nitroprusside but not compared with
nicardipine (Anesth Analg 2008;107:1110-1121) and was associated with decreased
time to extubation and postoperative length-of-stay (J Cardiothorac Vasc Anesth
2014;28:579-85).
(e) Unclear what the future clinical impact will be on other populations, how agent selection
will influence BPV, but should be an area of future research.
d. All intravenous medications should be transitioned to oral medications as soon as possible.
i. Oral antihypertensives should be initiated within 24 hours.
ii. Medication history and reconciliation can assist in resuming home regimens.
iii. Additional or new agents should be selected according to disease-specific indications.

Patient Case

8. B.B. is a 44-year-old man with no significant medical history who presents to the ED with a ripping sensa-
tion in his chest. His social history includes cigarette smoking, 1.5 packs/day for the past 20 years. Chest
radiography in the ED reveals mediastinal widening. Cardiac enzymes are within normal limits. Laboratory
values include sodium 135 mEq/L, potassium 4.3 mEq/L, HCO3- 24 mEq/L, SCr 0.55 mg/dL, glucose 110
mg/dL, DBil 0.2 mg/dL, and AST 39 U/L. B.B. is rushed for a chest CT and angiography, which reveal an
acute type A and B aortic dissection. His vital signs include BP 208/140 mm Hg and HR 120 beats/minute.
Which is the most appropriate management for B.B.?
A. Use esmolol to achieve goal of 25% reduction in MAP during the first 60 minutes.
B. Use lisinopril and hydrochlorothiazide to achieve BP reduction goal to 160/100 mm Hg during the first
24 hours.
C. Use labetalol to achieve 25% reduction in MAP and HR less than 60 beats/minute in the first 60 minutes.
D. Use esmolol with or without nitroprusside to achieve SBP less than 100 mm Hg and HR less than 60
beats/minute as soon as possible.

ACCP Updates in Therapeutics® 2022: Critical Care Pharmacy Preparatory Review and Recertification Course

758
Cardiovascular Critical Care II

REFERENCES

1. Abazi L, Awad A, Nordberg P, et al. Long-term in hematoma growth after acute intracerebral hem-
survival in out-of-hospital cardiac arrest patients orrhage. Hypertension 2010;56:852-8.
treated with targeted temperature control at 33 °C 12. Arima H, Huang Y, Wang JG, et al. Earlier blood
or 36 °C: a national registry study. Resuscitation pressure-lowering and greater attenuation of hema-
2019;143:142–7. toma growth in acute intracerebral hemorrhage:
2. Abella BS, Sandbo N, Vassilatos P, et al. Chest INTERACT pilot phase. Stroke 2012;43:2236-8.
compression rates during cardiopulmonary resus- 13. Aronson, S, Dyke CM, Stierer KA, et al. The
citation are suboptimal: a prospective study ECLIPSE trials: comparative studies of clevi-
during in-hospital cardiac arrest. Circulation dipine to nitroglycerin, sodium nitroprusside,
2005;111:428-34. and nicardipine for acute hypertension treat-
3. Aggarwal DA, Hess EP, Atkinson EJ, et al. ment in cardiac surgery patients. Anesth Analg
Ventricular fibrillation in Rochester, Minnesota: 2008;107:1110-1121.
experience over 18 years. Resuscitation 14. Aronson S, Levy J, Lumb PD, et al. Impact of
2009;80:1253-8. perioperative blood pressure variability on health
4. Aggarwal M, Khan IA. Hypertensive crisis: hyper- resource utilization after cardiac surgery: an anal-
tensive emergencies and urgencies. Cardiol Clin ysis of the ECLIPSE trials. J Cardiothorac Vasc
2006;24:135-46. Anesth 2014;28:579-85..
5. Andersen LO, Isbye DL, Rasmussen LS. Increasing 15. Arpino PA, Greer DM. Practical pharmacologic
compression depth during manikin CPR using aspects of therapeutic hypothermia after cardiac
a simple backboard. Acta Anaesthesiol Scand arrest. Pharmacotherapy 2008;28:102-11.
2007;51:747-50. 16. Arrich J, Holzer M, Havel C, et al. Hypothermia
6. Andersen LW, Kurth T, Chase M, et al. Early for neuroprotection in adults after cardiopulmo-
administration of epinephrine in patients with nary resuscitation. Cochrane Database Syst Rev
cardiac arrest with initial shockable rhythm in 2016;2:CD004128.
hospital: propensity score matched analysis. BMJ 17. Asai T, Barclay K, Power I, et al. Cricoid pressure
2016;353:i1577 impedes placement of the laryngeal mask airway
7. Anderson CS, Heeley E, Huang Y, et al. Rapid and subsequent tracheal intubation through the
blood-pressure lowering in patients with acute mask. Br J Anaesth 1994;72:47-51.
intracerebral hemorrhage. N Engl J Med 2013; 18. Aufderheide TP, Martin DR, Olson DW, et al.
368: 2355-2365. Prehospital bicarbonate use in cardiac arrest: a
8. Anderson CS, Huang Y, Wang JG, et al. Intensive 3-year experience. Am J Emerg Med 1992;10:4-7.
blood pressure reduction in acute cerebral haemor- 19. Aufderheide TP, Pirrallo RG, Yannopoulos D, et
rhage trial (INTERACT): a randomised pilot trial. al. Incomplete chest wall decompression: a clini-
Lancet Neurol 2008;7:391-9. cal evaluation of CPR performance by trained
9. Anyfantakis ZA, Baron G, Aubry P, et al. Acute laypersons and an assessment of alternative man-
coronary angiographic findings in survivors ual chest compression-decompression techniques.
of out-of-hospital cardiac arrest. Am Heart J Resuscitation 2006;71:341-51.
2009;157:312-8. 20. Aufderheide TP, Sigurdsson G, Pirrallo RG, et
10. Argaud L, Cour M, Dubien PY, et al. Effect of CSA al. Hyperventilation-induced hypotension during
in non-shockable OHCA: the CYRUS RCT. JAMA CPR. Circulation 2004;109:1960-5.
Cardiol 2016;1:557-65. 21. Aung K, Htay T. Vasopressin for cardiac arrest: a
11. Arima H, Anderson CS, Wang JG, et al. Intensive systematic review and meta-analysis. Arch Intern
Blood Pressure Reduction in Acute Cerebral Med 2005;165:17-24.
Haemorrhage Trial Investigators. Lower treatment 22. Barnes TA. Emergency ventilation techniques and
blood pressure is associated with greatest reduction related equipment. Respir Care 1992;37:673-90;
discussion 690-4.

ACCP Updates in Therapeutics® 2022: Critical Care Pharmacy Preparatory Review and Recertification Course

759
Cardiovascular Critical Care II

23. Barsan WG, Levy RC, Weir H. Lidocaine levels medical services for out-of-hospital cardiac arrest.
during CPR: differences after peripheral venous, JAMA 2008;299:1158-65.
central venous, and intracardiac injections. Ann 35. Bougouin W, Dumas F, Karam N, et al. Should
Emerg Med 1981;10:73-8. we perform an immediate coronary angiogram
24. Baskett P, Nolan J, Parr M. Tidal volumes which in all patients after cardiac arrest? Insights from
are perceived to be adequate for resuscitation. a large French registry. JACC Cardiovasc Interv
Resuscitation 1996;31:231-4. 2018;11:249-56.
25. Beaufort AM, Wierda JM, Belopavlovic M, et al. 36. Brazdzionyte J, Babarskiene RM, Stanaitiene G.
The influence of hypothermia (surface cooling) on Anterior-posterior versus anterior-lateral electrode
the time-course of action and on the pharmacoki- position for biphasic cardioversion of atrial fibrilla-
netics of rocuronium in humans. Eur J Anaesthesiol tion. Medicina (Kaunas) 2006;42:994-8.
Suppl 1995;11:95-106. 37. Brewin EG. Physiology of hypothermia. Int
26. Becker LB, Pepe PE. Ensuring the effectiveness Anesthesiol Clin 1964;2:803-27.
of community-wide emergency cardiac care. Ann 38. Brouwer TF, Walker RG, Chapman FW, et al.
Emerg Med 1993;22:354-65. Association between chest compression inter-
27. Belletti A, Benedetto U, Putzu A, et al. Vasopressors ruptions and clinical outcomes of ventricular
during CPR. A network meta-analysis of random- fibrillation out-of-hospital cardiac arrest.
ized trials. Crit Care Med 2018;46:e443-e451. Circulation 2015;132:1030-7.
28. Berg KA, Kern KB, Hilwig RW, et al. Assisted 39. Caldwell JE, Heier T, Wright PM, et al.
ventilation does not improve outcome in a porcine Temperature-dependent pharmacokinetics and
model of single-rescuer bystander cardiopulmo- pharmacodynamics of vecuronium. Anesthesiology
nary resuscitation. Circulation 1997;95:1635-41. 2000;92:84-93.
29. Berg KM, Soar J, Andersen LW, et al.; on behalf 40. Calhoun DA, Oparil S. Treatment of hypertensive
of the Adult Advanced Life Support Collaborators. crisis. N Engl J Med 1990;323:1177-83.
Adult advanced life support: 2020 International 41. Callaham M, Madsen CD, Barton CW, et al. A ran-
Consensus on Cardiopulmonary Resuscitation domized clinical trial of high-dose epinephrine and
and Emergency Cardiovascular Care Science norepinephrine vs standard-dose epinephrine in pre-
with Treatment Recommendations. Circulation hospital cardiac arrest. JAMA 1992;268:2667-72.
2020;142(suppl 1):S92-S139. 42. Chandra NC, Gruben KG, Tsitlik JE, et al.
30. Berg MD, Idris AH, Berg RA. Severe ventilatory Observations of ventilation during resuscitation in a
compromise due to gastric distention during pedi- canine model. Circulation 1994;90:3070-5.
atric cardiopulmonary resuscitation. Resuscitation 43. Christenson J, Andrusiek D, Everson-Stewart
1998;36:71-3. S, et al. Resuscitation Outcomes Consortium I:
31. Berg RA, Hemphill R, Abella BS, et al. Part 5: chest compression fraction determines survival in
adult basic life support: 2010 American Heart patients with out-of-hospital ventricular fibrillation.
Association guidelines for cardiopulmonary Circulation 2009;120:1241-7.
resuscitation and emergency cardiovascular care. 44. Clark RK, Trethewy CE. Assessment of cricoid
Circulation 2010;122:S685-705. pressure application by emergency department staff.
32. Bernard SA, Gray TW, Buist MD, et al. Treatment Emerg Med Australas 2005;17:376-81.
of comatose survivors of out-of-hospital cardiac 45. Coon GA, Clinton JE, Ruiz E. Use of atropine for
arrest with induced hypothermia. N Engl J Med brady-asystolic prehospital cardiac arrest. Ann
2002;346:557-63. Emerg Med 1981;10:462-7.
33. Bernard SA, Smith K, Finn J, et al. Induction of 46. Cummins RO, Eisenberg MS, Hallstrom AP, et al.
therapeutic hypothermia during OHCA using a Survival of out-of-hospital cardiac arrest with early
rapid infusion of cold saline: the RINSE trial. initiation of cardiopulmonary resuscitation. Am J
Circulation 2016;134:797-805. Emerg Med 1985;3:114-9.
34. Bobrow BJ, Clark LL, Ewy GA, et al. Minimally 47. Curry DL, Curry KP. Hypothermia and insulin
interrupted cardiac resuscitation by emergency secretion. Endocrinology 1970;87:750-5.

ACCP Updates in Therapeutics® 2022: Critical Care Pharmacy Preparatory Review and Recertification Course

760
Cardiovascular Critical Care II

48. D’Alecy LG, Lundy EF, Barton KJ, Zelenock GB. 61. Eftestol T, Sunde K, Steen PA. Effects of
Dextrose containing intravenous fluid impairs out- interrupting precordial compressions on the
come and increases death after eight minutes of calculated probability of defibrillation success
cardiac arrest and resuscitation in dogs. Surgery. during out-of-hospital cardiac arrest. Circulation
1986;100:505–11. 2002;105:2270-3.
49. Dankiewicz J, Cronberg T, Lilja G, et al. Hypothermia 62. Eftestol T, Wik L, Sunde K, et al. Effects of car-
versus normothermia after out-of-hospital cardiac diopulmonary resuscitation on predictors of
arrest. N Engl J Med 2021;384:2283-94. ventricular fibrillation defibrillation success dur-
50. Delooz HH, Lewi PJ. Are inter-center differences in ing out-of-hospital cardiac arrest. Circulation
ems-management and sodium-bicarbonate admin- 2004;110:10-5.
istration important for the outcome of CPR? The 63. Elam JO, Greene DG, Schneider MA, et al.
Cerebral Resuscitation Study Group. Resuscitation Head-tilt method of oral resuscitation. JAMA
1989;17(suppl):S161-172; discussion S199-206. 1960;172:812-5.
51. Delvaux AB, Trombley MT, Rivet CJ, et al. Design 64. Emerman CL, Pinchak AC, Hancock D, et al.
and development of a CPR mattress. J Intensive Effect of injection site on circulation times during
Care Med 2009;24:195-9. cardiac arrest. Crit Care Med 1988;16:1138-41.
52. Deshmukh A, Kumar G, Kumar N, et al. Effect of 65. Emerman CL, Pinchak AC, Hancock D, et al. The
Joint National Committee VII reports on hospital- effect of bolus injection on circulation times during
izations for hypertensive emergencies in the United cardiac arrest. Am J Emerg Med 1990;8:190-3.
States. Am J Cardiol 2011;108:1277-82. 66. Ewy GA, Bobrow BJ, Chikani V, et al. The time
53. Donnino MW, Andersen LW, Berg KM, et al. dependent association of adrenaline administration
Corticosteroid therapy in refractory shock follow- and survival from out-of-hospital cardiac arrest.
ing cardiac arrest: a randomized, double-blind, Resuscitation 2015;86:180-5.
placebo-controlled trial. Crit Care 2016;20:82 (1-8). 67. Feinstein BA, Stubbs BA, Rea T, et al. Intraosseous
54. Donnino MW, Salciccioli JD, Howell MD, et al. compared to intravenous drug resuscitation in
Time to administration of epinephrine and outcome OHCA. Resuscitation 2017;117:91-6.
after in-hospital cardiac arrest with non-shockable 68. Field JM, Kudenchuk PJ, O’Connor R, et al., eds.
rhythms: retrospective analysis of large in-hospital The Textbook of Emergency Cardiovascular Care
data registry. BMJ 2014;348:1-9. and CPR. Philadelphia: Lippincott Williams &
55. Dorges V, Ocker H, Hagelberg S, et al. Optimisation Wilkins, 2008.
of tidal volumes given with self-inflatable bags with- 69. Figueroa SA, Zhao W, and Aiyagari V. Emergency
out additional oxygen. Resuscitation 2000;43:195-9. and Critical Care Management of Acute Ishcemic
56. Dorian P, Cass D, Schwartz B, et al. Amiodarone as Stroke. CNS Drugs 2015;29:17-28.
compared with lidocaine for shock-resistant ventric- 70. Fazekas T, Scherlag BJ, Vos M, et al. Magnesium
ular fibrillation. N Engl J Med 2002;346:884-90. and the heart: antiarrhythmic therapy with magne-
57. Dumot JA, Burval DJ, Sprung J, et al. Outcome of sium. Clin Cardiol 1993;16:768-74.
adult cardiopulmonary resuscitations at a tertiary 71. Fisk CA, Olsufka M, Yin L, et al. Lower-dose
referral center including results of “limited” resus- epinephrine administration and OHCA outcomes.
citations. Arch Intern Med 2001;161:1751-8. Resuscitation 2018;124:43-8.
58. Dybvik T, Strand T, Steen PA. Buffer therapy dur- 72. Frank SM, Fleisher LA, Breslow MJ, et al.
ing out-of-hospital cardiopulmonary resuscitation. Perioperative maintenance of normothermia
Resuscitation 1995;29:89-95. reduces the incidence of morbid cardiac events. A
59. Edelson DP, Abella BS, Kramer-Johansen J, et al. randomized clinical trial. JAMA 1997;277:1127-34.
Effects of compression depth and pre-shock pauses 73. Garnett AR, Ornato JP, Gonzalez ER, et al.
predict defibrillation failure during cardiac arrest. End-tidal carbon dioxide monitoring during cardio-
Resuscitation 2006;71:137-45. pulmonary resuscitation. JAMA 1987;257:512-5.
60. Edgren E, Hedstrand U, Nordin M, et al. Prediction 74. Gebhardt K, Guyette FX, Doshi AA, et al.
of outcome after cardiac arrest. Crit Care Med Prevalence and effect of fever on outcome following
1987;15:820-5.

ACCP Updates in Therapeutics® 2022: Critical Care Pharmacy Preparatory Review and Recertification Course

761
Cardiovascular Critical Care II

resuscitation from cardiac arrest. Resuscitation 88. Holmberg M, Holmberg S, Herlitz J. Effect of
2013;84:1062-7. bystander cardiopulmonary resuscitation in out-
75. Graf H, Leach W, Arieff AI. Evidence for a det- of-hospital cardiac arrest patients in Sweden.
rimental effect of bicarbonate therapy in hypoxic Resuscitation 2000;47:59-70.
lactic acidosis. Science 1985;227:754-6. 89. Holmberg MJ, Issa MS, Moskowitz A, et al.;
76. Grossestreuer AV, Gaieski DF, Donnino MW, et al. International Liaison Committee on Resuscitation
Magnitude of temperature elevation is associated Advanced Life Support Task Force Collaborators.
with neurologic and survival outcomes in resusci- Vasopressors during adult cardiac arrest: a sys-
tated cardiac arrest patients with postrewarming tematic review and meta-analysis. Resuscitation
pyrexia. J Crit Care 2017;38:78-83. 2019;139:106-21.
77. Gudbrandsson T. Malignant hypertension: a clini- 90. Holzer M, Cerchiari E, Martens P, et al. Mild
cal follow-up study with special reference to renal therapeutic hypothermia to improve the neuro-
and cardiovascular function and immunogenic fac- logic outcome after cardiac arrest. N Engl J Med
tors. Acta Med Scand Suppl 1981;650:1-62. 2002;346:549-56.
78. Guildner CW. A comparative study of techniques 91. Hornchen U, Schuttler J, Stoeckel H, et al.
for opening an airway obstructed by the tongue. Endobronchial instillation of epinephrine during
JACEP 1976;5:588-90. cardiopulmonary resuscitation. Crit Care Med
79. Hammel HH, Hardy JD, Fusco JD. 1987;15:1037-9.
Thermoregulatory responses to hypothalamic 92. Gagnon DJ, Nielsen N, Fraser GL, et al. Prophylactic
cooling in unanesthetized dogs. Am J Physiol antibiotics are associated with a lower incidence of
1960;198:481-6. pneumonia in cardiac arrest survivors treated with
80. Handley AJ, Handley JA. Performing chest com- targeted temperature management. Resuscitation
pressions in a confined space. Resuscitation 2015;92:154-9.
2004;61:55-61. 93. Iida Y, Nishi S, Asada A. Effect of mild therapeutic
81. Hardig BM, Lindgren E, Ostlund O, et al. hypothermia on phenytoin pharmacokinetics. Ther
Outcome among VF/VT patients in the LINC Drug Monit 2001;23:192-7.
trial - a randomised, controlled trial. Resuscitation 94. Imamura M, Matsukawa T, Ozaki M, et al. The
2017;115:155-62. accuracy and precision of four infrared aural
82. Hempfill JC, Greenberg SM, Anderson CS, et al. canal thermometers during cardiac surgery. Acta
Guidelines for the management of spontaneous Anaesthesiol Scand 1998;42:1222-6.
intracerebral hemorrhage. Stroke 2015;46:2032-60. 95. Jabre P, Penaloza A, Pinero D, et al. Effect of
83. Heidenreich JW, Higdon TA, Kern KB, et al. bag-mask ventilation vs endotracheal intuba-
Single-rescuer cardiopulmonary resuscitation: tion during cardiopulmonary resuscitation on
“two quick breaths”—an oxymoron. Resuscitation neurological outcome after out-of-hospital car-
2004;62:283-9. diorespiratory arrest: a randomized clinical trial.
84. Herlitz J, Ekstrom L, Wennerblom B, et al. JAMA 2018;319:779-87.
Lidocaine in out-of-hospital ventricular fibril- 96. Jasani MS, Nadkarni VM, Finkelstein MS, et al.
lation. Does it improve survival? Resuscitation Effects of different techniques of endotracheal
1997;33:199-205. epinephrine administration in pediatric porcine
85. Higgins SL, Herre JM, Epstein AE, et al. A hypoxic-hypercarbic cardiopulmonary arrest. Crit
comparison of biphasic and monophasic shocks Care Med 1994;22:1174-80.
for external defibrillation. Prehosp Emerg Care 97. Jauch EC, Saver JL, Adams HP, et al. Guidelines
2000;4:305-13. for the early management of patients with acute
86. Hollander JE. Cocaine intoxication and hyperten- ischemic stroke. Stroke 2013;44:870-947.
sion. Ann Emerg Med 2008;51:S18-20. 98. Jennings PA, Cameron P, Walker T, et al. Out-of-
87. Holmberg M, Holmberg S, Herlitz J. Incidence, hospital cardiac arrest in Victoria: rural and urban
duration and survival of ventricular fibrillation in outcomes. Med J Aust 2006;185:135-9.
out-of-hospital cardiac arrest patients in Sweden. 99. Johnson NJ, Danielson KR, Counts CR, et al.
Resuscitation 2000;44:7-17. Targeted temperature management at 33 versus

ACCP Updates in Therapeutics® 2022: Critical Care Pharmacy Preparatory Review and Recertification Course

762
Cardiovascular Critical Care II

36 degrees: a retrospective cohort study. Crit Care 110. Kolar M, Krizmaric M, Klemen P, et al. Partial
Med 2020;48:362-9. pressure of end-tidal carbon dioxide success-
100. Johnson W, Nguyen ML, Patel R. Hypertension ful predicts cardiopulmonary resuscitation in the
crisis in the emergency department. Cardiol Clin field: a prospective observational study. Crit Care
2012;30:533-43. 2008;12:R115.
101. Kawano T, Grunau B, Scheuermeyer FX, et al. 111. Kramer-Johansen J, Edelson DP, Abella BS,
Prehospital sodium bicarbonate use could worsen et al. Pauses in chest compression and inap-
long-term survival with favorable neurological propriate shocks: a comparison of manual
recovery among patients with OHCA. Resuscitation and semi-automatic defibrillation attempts.
2017;119;63-9. Resuscitation 2007;73:212-20.
102. Kawano T, Grunau B, Scheuermeyer FX, et 112. Krasteva V, Matveev M, Mudrov N, et al.
al. Intraosseous vascular access is associated Transthoracic impedance study with large self-
with lower survival and neurologic recovery adhesive electrodes in two conventional positions
among patients with OHCA. Ann Emerg Med for defibrillation. Physiol Meas 2006;27:1009-22.
2018;71:588-96. 113. Krumholz A, Stern BJ, Weiss HD. Outcome
103. Kette F, Weil MH, Gazmuri RJ. Buffer solu- from coma after cardiopulmonary resuscitation:
tions may compromise cardiac resuscitation by relation to seizures and myoclonus. Neurology
reducing coronary perfusion pressure. JAMA 1988;38:401-5.
1991;266:2121-6. 114. Kudenchuk PJ, Brown MD, Nichol G, et al.
104. Khera R, Chan PS, Donnino M, et al. Hospital Amiodarone, lidocaine, or placebo in out-of-hospi-
variation in time to epinephrine for nonshockable tal cardiac arrest. N Engl J Med 2016; 374:1711-22.
IHCA. Circulation 2016;134:2105-14. 115. Kudenchuk PJ, Cobb LA, Copass MK, et al.
105. Kim J, Kim K, Park J, et al. Sodium bicarbon- Amiodarone for resuscitation after out-of-hospital
ate administration during ongoing resuscitation is cardiac arrest due to ventricular fibrillation. N Engl
associated with increased return of spontaneous J Med 1999;341:871-8.
circulation. Am J Emerg Med 2016;34:225-9. 116. Kudenchuk PJ, Leroux BG, Daya M, et al.
106. Kim YM, Youn CS, Kim SH, et al. Adverse events Antiarrhythmic drugs for nonshockable-turned-
associated with poor neurological outcome during shockable OHCA: the ALPS study. Circulation
targeted temperature management and advanced 2017;136:2119-31.
critical care after out-of-hospital cardiac arrest. 117. Kuhn GJ, White BC, Swetnam RE, et al. Peripheral
Crit Care 2015;19:283-96. vs central circulation times during CPR: a pilot
107. Kirkegaard H, Rasmussen BS, de Haas I, et al. study. Ann Emerg Med 1981;10:417-9.
Time-differentiated target temperature man- 118. Lameire N, Van Biesen W, Vanholder R. Acute
agement after out-of-hospital cardiac arrest: a renal failure. Lancet 2005;365:417-30.
multicenter randomized, parallel-group, assessor- 119. Langhelle A, Sunde K, Wik L, et al. Airway pres-
blinded clinical trial (the TTH48 trial): study sure with chest compressions versus Heimlich
protocol for a randomized controlled trial. Trials manoeuvre in recently dead adults with complete
2016;17:228 (1-9). airway obstruction. Resuscitation 2000;44:105-8.
108. Kirkegaard H, Soreide E, de Haas I, et al. TTM 120. Larsen MP, Eisenberg MS, Cummins RO, et
for 48 vs. 24 hours and neurologic outcome after al. Predicting survival from out-of-hospital car-
OHCA: a randomized clinical trial. A duration diac arrest: a graphic model. Ann Emerg Med
of 48 hours does not seem to confer any benefit 1993;22:1652-8.
compared to a 24 hour duration in multicenter, ran- 121. Laver S, Farrow C, Turner D, et al. Mode of death
domized study. JAMA 2017;318:341-50. after admission to an intensive care unit following
109. Kleinman ME, Brennan EE, Goldberger cardiac arrest. Intensive Care Med 2004;30:2126-8.
ZD, et al. Part 5: Adult Basic Life Support 122. Lenhardt R, Orhan-Sungur M, Komatsu R, et al.
and Cardiopulmonary Resuscitation Quality. Suppression of shivering during hypothermia using
Circulation 2015;132:S414-35. a novel drug combination in healthy volunteers.
Anesthesiology 2009;111:110-5.

ACCP Updates in Therapeutics® 2022: Critical Care Pharmacy Preparatory Review and Recertification Course

763
Cardiovascular Critical Care II

123. Levy DE, Caronna JJ, Singer BH, et al. Predicting 136. McCoy S, Baldwin K. Pharmacotherapeutic
outcome from hypoxic-ischemic coma. JAMA options for the treatment of preeclampsia. Am J
1985;253:1420-6. Health Syst Pharm 2009;66:337-44.
124. Lipinski CA, Hicks SD, Callaway CW. Normoxic 137. Mentzelopoulos SD, Malachias S, Chamos C, et
ventilation during resuscitation and outcome from al. Vasopressin, steroids, and epinephrine and
asphyxial cardiac arrest in rats. Resuscitation neurologically favorable survival after in-hospital
1999;42:221-9. cardiac arrest: a randomized clinical trial. JAMA
125. Liu-DeRyke X, Janisse J, Coplin WM, et al. A 2013;310:270-9.
comparison of nicardipine and labetalol for acute 138. Michael JR, Guerci AD, Koehler RC, et al.
hypertension management following stroke. Mechanisms by which epinephrine augments
Neurocrit Care 2008;9:167-176 cerebral and myocardial perfusion during car-
126. Liu-DeRyke X, Levy PD, Parker D, et al. A pro- diopulmonary resuscitation in dogs. Circulation
spective evaluation of labetalol versus nicardipine 1984;69:822-35.
for blood pressure management in patients with 139. Michelson AD, Barnard MR, Khuri SF, et al. The
acute stroke. Neurocrit Care 2013;19:41-47. effects of aspirin and hypothermia on platelet func-
127. Lloyd-Jones D, Adams RJ, Brown TM, et al. tion in vivo. Br J Haematol 1999;104:64-8.
Executive summary: heart disease and stroke sta- 140. Moule P. Checking the carotid pulse: diagnostic
tistics—2010 update: a report from the American accuracy in students of the healthcare professions.
Heart Association. Circulation 2010;121:948-54. Resuscitation 2000;44:195-201.
128. Lovstad RZ, Granhus G, Hetland S. Bradycardia 141. Nagao K, Nonogi H, Yonemoto N, et al. Duration
and asystolic cardiac arrest during spinal anaesthe- of Prehospital Resuscitation Efforts After
sia: a report of five cases. Acta Anaesthesiol Scand Out-of-Hospital Cardiac Arrest. Circulation
2000;44:48-52. 2016;133:1386-96.
129. Lundy EF, Kuhn JE, Kwon JM, et al. Infusion of 142. Naito H, Isotani E, Callaway CW, et al. Intracranial
five percent dextrose increases mortality and mor- pressure increases during rewarming period after
bidity following six minutes of cardiac arrest in mild therapeutic hypothermia in postcardiac
resuscitated dogs. J Crit Care 1987;2:4–14. arrest patients. Ther Hypothermia Temp Manag
130. Mahmoud A, Elgendy IY, Bavry AA. Use of 2016;6:189-93.
targeted temperature management after out-of-hos- 143. Nakakimura K, Fleischer JE, Drummond JC, et al.
pital cardiac arrest: a meta-analysis of randomized Glucose administration before cardiac arrest wors-
controlled trials. Am J Med 2016;129:522-527. ens neurologic outcome in cats. Anesthesiology.
131. Manders S, Geijsel FE. Alternating providers 1990;72:1005–11.
during continuous chest compressions for car- 144. Negus BH, Willard JE, Hillis LD, et al. Alleviation
diac arrest: every minute or every two minutes? of cocaine-induced coronary vasoconstric-
Resuscitation 2009;80:1015-8. tion with intravenous verapamil. Am J Cardiol
132. Manning L, Hirakawa Y, Arima H, et al. Blood 1994;73:510-3.
pressure variability and outcome after acute 145. Neumar RW, Nolan JP, Adrie C, et al. Post-cardiac
intracerebral hemorrhage: a post-hoc analysis arrest syndrome: epidemiology, pathophysiology,
of INTERACT2, a randomized controlled trial. treatment, and prognostication. A consensus state-
Lancet Neurol 2014;13:364-73. ment from the International Liaison Committee
133. Manz M, Pfeiffer D, Jung W, et al. Intravenous on Resuscitation (American Heart Association,
treatment with magnesium in recurrent persistent Australian and New Zealand Council on
ventricular tachycardia. New Trends Arrhythmias Resuscitation, European Resuscitation Council,
1991;7:437-42. Heart and Stroke Foundation of Canada, Inter-
134. Marik PE, Varon J. Hypertensive crises: challenges American Heart Foundation, Resuscitation Council
and management. Chest 2007;131:1949-62. of Asia, and the Resuscitation Council of Southern
135. McAllister RG Jr, Bourne DW, et al. Effects Africa); the American Heart Association Emergency
of hypothermia on propranolol kinetics. Clin Cardiovascular Care Committee; the Council on
Pharmacol Ther 1979;25:1-7. Cardiovascular Surgery and Anesthesia; the Council

ACCP Updates in Therapeutics® 2022: Critical Care Pharmacy Preparatory Review and Recertification Course

764
Cardiovascular Critical Care II

on Cardiopulmonary, Perioperative, and Critical resuscitation and emergency cardiovascular care.


Care; the Council on Clinical Cardiology; and the Circulation 2019;140:e881-e894.
Stroke Council. Circulation 2008;118:2452-83. 157. Paradis NA, Martin GB, Rivers EP. Coronary
146. Neumar RW, Shuster M, Callaway CW, et al. perfusion pressure and the return of spontaneous
Part 1: executive summary: 2015 American Heart circulation in human cardiopulmonary resuscita-
Association guidelines update for cardiopulmo- tion. JAMA 1990;263:1106-13.
nary resuscitation and emergency cardiovascular 158. Parker EA. Parenteral incompatibilities. Hosp
care. Circulation 2015;132(suppl 2):S315-S367. Pharm 1969;4:14-22.
147. Neumar RW, Otto CW, Link MS, et al. Part 159. Parnia S, Yang J, Nguyen R, et al. Cerebral oxim-
8: adult advanced cardiovascular life support: etry during cardiac arrest: a multicenter study of
2010 American Heart Association guidelines for neurologic outcomes and survival. Crit Care Med
cardiopulmonary resuscitation and emergency car- 2016;44:1663-74.
diovascular care. Circulation 2010;122:S729-767. 160. Patel KK, Spertus JA, Khariton Y, et al. Association
148. Nichol G, Leroux B, Wang H, et al. Trial of between prompt defibrillation and epinephrine
Continuous or Interrupted Chest Compressions treatment with long-term survival after IHCA.
during CPR. N Engl J Med 2015; 23: 2203-2214. Circulation 2018;137:2041-51.
149. Nielsen N, Hovdenes J, Nilsson F, et al. Outcome, 161. Peberdy MA, Callaway CW, Neumar RW, et al.
timing and adverse events in therapeutic hypo- 2010 American Heart Association guidelines for
thermia after out-of-hospital cardiac arrest. Acta cardiopulmonary resuscitation and emergency car-
Anaesthesiol Scand 2009;53:926-34. diovascular care. Part 9: post-cardiac arrest care.
150. Nielsen N, Wetterslev J, Cronberg T, et al. Targeted Circulation 2010;122:S768-786.
temperature management at 33°C versus 36°C after 162. Peng TJ, Andersen LW, Saindon BZ, et al. The
cardiac arrest. N Engl J Med 2013;369:2197-206. administration of dextrose during in-hospital car-
151. Olasveenegen TM, Wik L, Steen PA. Standard diac arrest is associated with increased mortality
basic life support vs. continuous chest compres- and neurologic morbidity. Crit Care 2015;19:160
sions only in out-of-hospital cardiac arrest. Acta (1-8).
Anaesthesiol Scand 2008;52:914-9. 163. Perkins GD, Ji C, Deakin CD, et al. A random-
152. Ornato JP, Garnett AR, Glauser FL. Relationship ized trial of epinephrine in OHCA. N Engl J Med
between cardiac output and the end-tidal carbon 2018;379:711-21.
dioxide tension. Ann Emerg Med 1990;19:1104-6. 164. Perman S, Grossestreuer AV, Wiebe DJ, et al. The
153. Ono Y, Hayakawa M, Maekawa K, et al. Should utility of therapeutic hypothermia for post-cardiac
laryngeal tubes or masks be used for out-of-hos- arrest syndrome patients with an initial nonshock-
pital cardiac arrest patients? Am J Emerg Med able rhythm. Circulation 2015;132:2146-51.
2015;33:1360-3. 165. Pokorna M, Necas E, Kratochvil J, et al. A sudden
154. Panacek EA, Munger MA, Rutherford WF, et al. increase in partial pressure end-tidal carbon dioxide
Report of nitropatch explosions complicating defi- (P(ET)CO(2)) at the moment of return of sponta-
brillation. Am J Emerg Med 1992;10:128-9. neous circulation. J Emerg Med 2009;38:614-21.
155. Panchal AR, Bartos JA, Cabañas JG, et al. Part 3: 166. Polderman KH, Peerdeman SM, Girbes AR.
adult basic and advanced life support: 2020 American Hypophosphatemia and hypomagnesemia induced
Heart Association guidelines for cardiopulmonary by cooling in patients with severe head injury. J
resuscitation and emergency cardiovascular care. Neurosurg 2001;94:697-705.
Circulation 2020;142(16_suppl_2):S366-468. 167. Porter TR, Ornato JP, Guard CS, et al.
156. Panchal AR, Berg KM, Hirsch KG, et al. 2019 Transesophageal echocardiography to assess mitral
American Heart Association focused update valve function and flow during cardiopulmonary
on advanced cardiovascular life support: use of resuscitation. Am J Cardiol 1992;70:1056-60.
advanced airways, vasopressors, and extracor- 168. Pujol A, Fusciardi J, Ingrand P, et al. Afterdrop
poreal cardiopulmonary resuscitation during after hypothermic cardiopulmonary bypass: the
cardiac arrest: an update to the American Heart value of tympanic membrane temperature monitor-
Association guidelines for cardiopulmonary ing. J Cardiothorac Vasc Anesth 1996;10:336-41.

ACCP Updates in Therapeutics® 2022: Critical Care Pharmacy Preparatory Review and Recertification Course

765
Cardiovascular Critical Care II

169. Pytte M, Pedersen TE, Ottem J, et al. Comparison prehospital comparative study of ventilatory device
of hands-off time during CPR with manual and effectiveness and cost-effectiveness in 470 cases
semi-automatic defibrillation in a manikin model. of cardiorespiratory arrest. Prehosp Emerg Care
Resuscitation 2007;73:131-6. 1997;1:1-10.
170. Qureshi AI, Bliwise DL, Bliwise NG, et al. Rate of 181. Sakamoto Y, Koga M, Yamagami H, et al. Systolic
24-hour blood pressure decline and mortality after blood pressure after intravenous antihypertensive
spontaneous intracerebral hemorrhage: a retro- treatment and clinical outcomes in hyperacute
spective analysis with a random effects regression intracerebral hemorrhage: the Stroke Acute
model. Crit Care Med 1999;27:480-5. Management With Urgent Risk- Factor Assessment
171. Qureshi AI, Palesch YY, Martin R, et al. and Improvement-Intracerebral Hemorrhage Study.
Antihypertensive Treatment of Acute Cerebral Stroke 2013;44:1846-51.
Hemorrhage Study Investigators. Effect of systolic 182. Sanders AB, Ogle M, Ewy GA. Coronary perfusion
blood pressure reduction on hematoma expan- pressure during cardiopulmonary resuscitation.
sion, perihematomal edema, and 3-month outcome Am J Emerg Med 1985;3:11-4.
among patients with intracerebral hemorrhage: 183. Sasson C, Rogers MA, Dahl J, et al. Predictors of
results from the Antihypertensive Treatment of survival from out-of-hospital cardiac arrest: a sys-
Acute Cerebral Hemorrhage Study. Arch Neurol tematic review and meta-analysis. Circ Cardiovasc
2010;67:570-6. Qual Outcomes 2010;3:63-81.
172. Rajan S, Folke F, Kragholm K, et al. Prolonged 184. Schade K, Borzotta A, Michaels A. Intracranial
cardiopulmonary resuscitation and outcomes malposition of nasopharyngeal airway. J Trauma
after out-of-hospital cardiac arrest. Resuscitation 2000;49:967-8.
2016;105:45-51. 185. Shadwan A. Electrocardiographic changes in
173. Rea TD, Helbock M, Perry S, et al. Increasing hypothermia. Heart Lung 2001;30:161-3.
use of cardiopulmonary resuscitation during out- 186. Shy BD, Rea TD, Becker LJ, et al. Time to intu-
of-hospital ventricular fibrillation arrest: survival bation and survival in prehospital cardiac arrest.
implications of guideline changes. Circulation Prehosp Emerg Care 2004;8:394-9.
2006;114:2760-5. 187. Silfvast T, Saarnivaara L, Kinnunen A, et al.
174. Rivers EP, Martin GB, Smithline H, et al. The Comparison of adrenaline and phenylephrine in
clinical implications of continuous central venous out-of-hospital cardiopulmonary resuscitation.
oxygen saturation during human CPR. Ann Emerg A double-blind study. Acta Anaesthesiol Scand
Med 1992;21:1094-101. 1985;29:610-3.
175. Roberts D, Landolfo K, Light RB, et al. Early predic- 188. Somberg JC, Bailin SJ, Haffajee CI, et al.
tors of mortality for hospitalized patients suffering Intravenous lidocaine versus intravenous ami-
cardiopulmonary arrest. Chest 1990;97:413-9. odarone (in a new aqueous formulation) for
176. Rodriguez-Luna D, Piñeiro S, Rubiera M, et al. incessant ventricular tachycardia. Am J Cardiol
Impact of blood pressure changes and course on 2002;90:853-9.
hematoma growth in acute intracerebral hemor- 189. SOS-KANTO study group. Comparison of arte-
rhage. Eur J Neurol 2013;20:1277-83. rial blood gases of laryngeal mask airway and bag
177. Rohrer MJ, Natale AM. Effect of hypother- valve-mask ventilation in out-of-hospital cardiac
mia on the coagulation cascade. Crit Care Med arrests. Circ J 2009;73:490-6.
1992;20:1402-5. 190. Spaulding CM, Luc-Marie J, Rosenberg A, et al.
178. Rosow CE. Pharmacokinetic and Pharmacodynamic Immediate coronary angiography in survivors
Effects of Cardiopulmonary Bypass. Baltimore: of out-of-hospital cardiac arrest. N Engl J Med
Williams & Wilkins, 2000. 1997;336:1629-33.
179. Ruben H. The immediate treatment of respiratory 191. Spindelboeck W, Schindler O, Moser A. et al.
failure. Br J Anaesth 1964;36:542-9. Increasing arterial oxygen partial pressure during
180. Rumball CJ, MacDonald D. The PTL, Combitube, cardiopulmonary resuscitation is associated with
laryngeal mask, and oral airway: a randomized

ACCP Updates in Therapeutics® 2022: Critical Care Pharmacy Preparatory Review and Recertification Course

766
Cardiovascular Critical Care II

improved rates of hospital admission. Resuscitation 204. Tanaka E, Koga M, Kobayashi J, et al. Blood pres-
2013;84:770-5. sure variability on antihypertensive therapy in acute
192. Stiell IG, Hebert PC, Weitzman BN, et al. High- intracerebral hemorrhage. Stroke 2014;45:2275-9.
dose epinephrine in adult cardiac arrest. N Engl J 205. Tzivoni D, Banai S, Schuger C, et al. Treatment
Med 1992;327:1045-50. of torsade de pointes with magnesium sulfate.
193. Stiell IG, Hebert PC, Wells GA, et al. Vasopressin Circulation 1988;77:392-7.
versus epinephrine for in-hospital cardiac arrest: a 206. Vahedian-Azimi A, Hajiesmaeili M,
randomised controlled trial. Lancet 2001;358:105-9. Amirsavadkouhi A, et al. Effect of the Cardio First
194. Stiell IG, Walker RG, Nesbitt LP, et al. Biphasic Angel device on CPR indices: a randomized con-
trial: a randomized comparison of fixed lower trolled clinical trial. Crit Care 2016;20:147-154.
versus escalating higher energy levels for defibril- 207. Vaillancourt C, Verma A, Trickett J, et al.
lation in out-of-hospital cardiac arrest. Circulation Evaluating the effectiveness of dispatch-assisted
2007;115:1511-7. cardiopulmonary resuscitation instructions. Acad
195. Stoneham MD. The nasopharyngeal airway. Emerg Med 2007;14:877-83.
Assessment of position by fibre-optic laryngos- 208. Valenzuela TD, Roe DJ, Cretin S, et al. Estimating
copy. Anaesthesia 1993;48:575-80. effectiveness of cardiac arrest interventions: a
196. Strandgaard S, Paulson OB. Cerebral autoregula- logistic regression survival model. Circulation
tion. Stroke 1984;15:413-6. 1997;96:3308-13.
197. Stueven HA, Thompson B, Aprahamian C, et al. 209. van Walraven C, Stiell IG, Wells GA, et al. Do
Lack of effectiveness of calcium chloride in refrac- advanced cardiac life support drugs increase
tory asystole. Ann Emerg Med 1985;14:630-2. resuscitation rates from in-hospital cardiac
198. Stueven HA, Thompson B, Aprahamian C, et al. arrest? The OTAC study group. Ann Emerg Med
The effectiveness of calcium chloride in refractory 1998;32:544-53.
electromechanical dissociation. Ann Emerg Med 210. Vandycke C, Martens P. High dose versus standard
1985;14:626-9. dose epinephrine in cardiac arrest—a meta-analy-
199. Stueven HA, Tonsfeldt DJ, Thompson BM, et al. sis. Resuscitation 2000;45:161-6.
Atropine in asystole: human studies. Ann Emerg 211. Von Goedecke A, Bowden K, Wenzel V, et al.
Med 1984;13:815-8. Effects of decreasing inspiratory times during sim-
200. Sugerman NT, Edelson DP, Leary M, et al. Rescuer ulated bag-valve-mask ventilation. Resuscitation
fatigue during actual in-hospital cardiopulmo- 2005;64:321-5.
nary resuscitation with audiovisual feedback: 212. Wadhwa A, Sengupta P, Durrani J, et al. Magnesium
a prospective multicenter study. Resuscitation sulphate only slightly reduces the shivering thresh-
2009;80:981-4. old in humans. Br J Anaesth 2005;94:756-62.
201. Sunjic KM, Webb AC, Sunjic I, et al. 213. Wang PL, Brooks SC. Mechanical versus manual
Pharmacokinetic and other considerations for drug chest compressions for cardiac arrest. Cochrane
therapy during targeted temperature management. Database Syst Rev 2018;8:CD007260.
Crit Care Med 2015;43:2228-38. 214. Webster J, Petrie JC, Jeffers TA, et al. Accelerated
202. Swor RA, Jackson RE, Cynar M, et al. Bystander hypertension patterns of mortality and clinical
CPR, ventricular fibrillation, and survival in wit- factors affecting outcomes in treated patients. Q J
nessed, unmonitored out-of-hospital cardiac arrest. Med 1993;96:485-93.
Ann Emerg Med 1995;25:780-4. 215. Wenzel V, Idris AH, Banner MJ, et al. The compo-
203. Szarpak L, Truszewski Z, Smereka J, et al. A sition of gas given by mouth-to-mouth ventilation
Randomized Cadaver Study Comparing First- during CPR. Chest 1994;106:1806-10.
Attempt Success Between Tibial and Humeral 216. Wenzel V, Krismer AC, Arntz HR, et al.
Intraosseous Insertions Using NIO Device by European Resuscitation Council Vasopressor dur-
Paramedics. Medicine 2016;95:e3724 (1-6). ing Cardiopulmonary Resuscitation Study G: a
comparison of vasopressin and epinephrine for

ACCP Updates in Therapeutics® 2022: Critical Care Pharmacy Preparatory Review and Recertification Course

767
Cardiovascular Critical Care II

out-of-hospital cardiopulmonary resuscitation. N 228. Zuercher M, Hilwig RW, Ranger-Moore J, et


Engl J Med 2004;350:105-13. al. Leaning during chest compressions impairs
217. Wolff B, Machill K, Schumacher D, et al. Early cardiac output and left ventricular myocardial
achievement of mild therapeutic hypothermia and blood flow in piglet cardiac arrest. Crit Care Med
the neurologic outcome after cardiac arrest. Int J 2010;38:1141-6.
Cardiol 2009;133:223-8.
218. Wong ML, Carey S, Mader TJ, et al. American Heart
Association National Registry of Cardiopulmonary
Resuscitation I: Time to invasive airway placement
and resuscitation outcomes after in-hospital car-
diopulmonary arrest. Resuscitation 2010;81:182-6.
219. Yakaitis RW, Otto CW, Blitt CD. Relative impor-
tance of alpha and beta adrenergic receptors during
resuscitation. Crit Care Med 1979;7:293-6.
220. Yannopoulus D, Aufderheide TP, Gabrielli A,
et al. Clinical and hemodynamic comparison of
15:2 and 30:2 compression-to-ventilation ratios
for cardiopulmonary resuscitation. Crit Care Med
2006;34:1444-9.
221. Yannopoulus D, McNite S, Aufderheide TP, et al.
Effects of incomplete chest wall decompression
during cardiopulmonary resuscitation on coronary
and cerebral perfusion pressures in a porcine model
of cardiac arrest. Resuscitation 2005;64:363-72.
222. Yannopoulus D, Sigurdsson G, McNite S, et
al. Reducing ventilation frequency combined
with an inspiratory impedance device improves
CPR efficiency in swine model of cardiac arrest.
Resuscitation 2004;61:75-82.
223. Zanbergen EG, Hijdra A, Koelman JH, et al.
Prediction of poor outcome within the first 3 days
of postanoxic coma. Neurology 2006;66:62-8.
224. Zeiner A, Holzer M, Sterz F, et al. Hyperthermia
after cardiac arrest is associated with an unfa-
vorable neurologic outcome. Arch Intern Med
2001;161:2007-12.
225. Zeiner A, Sunder-Plassmann G, Sterz F, et al. The
effect of mild therapeutic hypothermia on renal
function after cardiopulmonary resuscitation in
men. Resuscitation 2004;60:253-61.
226. Zhan L, Yang LJ, Huang Y, et al. Continuous chest
compressions versus interrupted chest compres-
sions for CPR of non-asphyxial OHCA. Cochrane
Database Syst Rev 2017;3:CD010134.
227. Zhang Y, Reilly KH, Tong W, et al. Blood pres-
sure and clinical outcome among patients with
acute stroke in Inner Mongolia, China. J Hypertens
2008;26:1446-52.

ACCP Updates in Therapeutics® 2022: Critical Care Pharmacy Preparatory Review and Recertification Course

768
Cardiovascular Critical Care II

ANSWERS AND EXPLANATIONS TO PATIENT CASES

1. Answer: B 3. Answer: A
Because rescuer fatigue is common and may lead to Three vital actions with VF aid in survival: call emer-
inadequate compression quality, it is recommended to gency response team (already accomplished in case),
change rescuers every 2 minutes, with no more than begin CPR (must be initiated in case), and deliver shock
5 seconds between changes (Answer A is incorrect). (must occur in case) (Answer A is correct). Pacing can
Compressions are vital because they increase intratho- be effective in overriding stable VT but should not be
racic pressure and directly compress the heart, leading used in the cardiac arrest or hemodynamically unstable
to oxygen delivery to the vital organs (Answer B is cor- patient (Answer B is incorrect). It is currently unclear
rect). Specific aspects of chest compression quality are whether postponing defibrillation for the provision of
necessary. These include a rate of 100–120 compres- chest compressions first is of benefit, but it is clear that
sions per minute at a depth of 2–2.4 inches in adults, chest compressions should be initiated until the defi-
allowing for recoil after each compression; placement brillator is ready, charged, and set to deliver the shock
of the patient on a hard surface (e.g., backboard); and because this increases the likelihood of success with
minimization of interruptions (Answer C is incorrect). defibrillation (Answer C is incorrect). Because time in
Outcomes, including neurologically intact survival, VF predicts survival, and the longer patients are in VF
ROSC, and possibly overall survival, are linked to mini- the more difficult it is to terminate the arrhythmia, alter-
mizing interruptions in chest compressions. Because of native treatments such as medications should not impede
this, it is recommended that interruptions (e.g., pulse the provision of defibrillation (Answer D is incorrect).
checks and intubation) be less than 10 seconds and that
chest compressions be resumed immediately (Answer D 4. Answer: D
is incorrect). After the advanced airway is in place, it is crucial to
confirm placement in order to provide the intended oxy-
2. Answer: C genation/ventilation. The confirmation should occur
Cardiac arrest patients have minimal blood flow, and with both clinical and objective measurements (Answer
oxygenation/ventilation requirements are lower; the new D is correct). These include a physical assessment of the
recommendation is to provide 1 breath every 5–6 sec- chest and epigastrium, end-tidal CO2 monitoring, and/
onds (10–12 breaths/minute) until an advanced airway or continuous waveform capnography. In most cardiac
is in place (Answer C is correct). Although the optimal arrests (particularly in this patient’s pVT), airway man-
ratio is unclear and chest compressions appear to be agement should not impede the provision of CPR and/or
more vital to resuscitation, other ratios cannot currently defibrillation (when defibrillation is indicated) (Answer
be recommended (Answer A is incorrect). It is clear, B is incorrect). After the advanced airway is in place,
however, that excessive ventilation can lead to decreased 100% oxygen should be delivered to optimize the arterial
venous return and gastric inflation, which can lead to oxygen saturation (Answer C is incorrect). In the car-
aspiration, regurgitation, and impacts on outcomes. In diac arrest population, this has not been shown to carry
this case, a bag-mask ventilator is available, and more the same toxicity as in other populations. Furthermore,
than one rescuers are involved, so the bag-mask venti- after advanced airway is placed, compressions should be
lator should be used (Answer B is incorrect). In single administered at a rate of 100–120 compressions per min-
health care provider rescuer situations, the bag-mask ute continuously, with breaths every 6 seconds (Answer
ventilator should never be used, and mouth-to-mouth A is incorrect).
or mouth-to-barrier resuscitation is recommended. In
a single nonmedical rescuer situation, hands-only CPR 5. Answer: B
is recommended. Advanced airways can be considered In all cardiac arrests, the treatable causes (i.e., H’s
but should be placed only by experienced and trained and T’s) should be reviewed and addressed, if possible
personnel. Bag-mask ventilation can provide adequate (Answer B is correct). In patients for whom the labo-
oxygenation/ventilation until an airway can be secured ratory and diagnostic data are known, the information
(Answer D is incorrect). should be reviewed while CPR is being provided. In
patients for whom the information is unknown, clinical

ACCP Updates in Therapeutics® 2022: Critical Care Pharmacy Preparatory Review and Recertification Course

769
Cardiovascular Critical Care II

evaluation and attainment of information should occur, temperature is 32°C–36°C. Newer data suggest there is
when possible. This retrieval of information, together no benefit of 33°C versus 36°C in improvement of sur-
with the administration of medications and advanced vival or neurologic outcomes (Answer D is incorrect).
airway placement, should never impede on the provision If targeted temperature management will be used, close
of CPR or defibrillation, if indicated, because defibrilla- monitoring of complications should occur. These com-
tion (for VF and pVT)) and CPR are the only strategies plications include hyperglycemia caused by decreased
that have been shown to affect survival from cardiac insulin production and peripheral activity (Answer A is
arrest (Answer A is incorrect). Pulseless electrical incorrect), bradycardias, enzymatic slowing (including
activity is not a wide complex dysrhythmia, and defibril- CYP system), increased incidence of sepsis and infec-
lation would not be indicated (Answer C is incorrect). tions, coagulopathies, decreased glomerular filtration,
Post-cardiac arrest care is crucial in the prevention of re- and shivering (Answer B is correct).
arrest and, in therapeutic hypothermia, can significantly
affect neurologic outcomes (Answer D is incorrect). 8. Answer: D
This patient is experiencing a hypertensive emergency
6. Answer: D with his target organ damage being an acute aortic
In general, it is important to remember that medication dissection. Aortic dissection is one of the individual
administration benefits only myocardial blood flow, hypertensive emergencies that has a specific mechanism
ROSC, and possibly survival to hospital admission in of worsening (propagation) from BP and shear stress,
cardiac arrest. Medication administration should never which require both rapid BP and HR control. Given the
impede the provision of CPR and/or defibrillation. gravity of propagation, goals for aortic dissection are HR
Central administration, if already available, is pre- less than 60 beats/minute and SBP less than 100 mm Hg
ferred for several reasons. These include higher peak within minutes, if possible (Answer A is incorrect). This
concentrations, shorter circulation time, more standard can be accomplished with a single agent like labetalol,
dosing, and the lack of additional administration tech- which will control HR with its β-antagonist properties
niques needed (Answer D is correct). Endotracheal and decrease BP (afterload) with its α-antagonist prop-
administration is an option, but only NAVEL (nalox- erties (Answers B and C are incorrect). Esmolol can also
one, atropine, vasopressin, epinephrine, and lidocaine) be used as first line but will likely require an additional
medications can be administered, the optimal doses afterload-reducing agent such as nitroprusside (Answer
are unknown, and medications must be diluted before D is correct).
administered (Answer A is incorrect). Given that this
patient is in VF arrest, amiodarone, for example, could
not be administered by endotracheal administration if
it were indicated. Peripheral administration can be used
and is the most common route of administration given
that most hospitalized patients have it already, but it
requires an additional bolus of fluid afterward and has a
longer circulating time than does central administration
(Answer B is incorrect). Intraosseous can also be used,
with the caveat that tibial intraosseous administration is
similar to peripheral administration and requires train-
ing to master the technique for placement (Answer C is
incorrect).

7. Answer: B
Targeted temperature management (therapeutic hypo-
thermia) improves neurologic recovery when initiated,
optimally within 2 hours but in up to 6–8 hours after
VF cardiac arrest (application to all forms), and used
for 12–24 hours. Answer C is incorrect. The goal

ACCP Updates in Therapeutics® 2022: Critical Care Pharmacy Preparatory Review and Recertification Course

770
Cardiovascular Critical Care II

ANSWERS AND EXPLANATIONS TO SELF-ASSESSMENT QUESTIONS

1. Answer: C 4. Answer: C
The largest portion of adult cardiac arrests are caused Targeted temperature management is a consideration,
by cardiovascular, not respiratory, events (Answer A is according to the international guidelines for all patients
incorrect). In addition, this patient was on room air and with ROSC who remain comatose after a cardiac arrest
nasal cannula immediately before the event, which could (Answer A is incorrect). Although most well-designed
suggest his respiratory status was stable and unlikely to and executed studies primarily enrolled patients with
lead to cardiac arrest. Advanced airways and medica- VF cardiac arrest, application is recommended for all
tions have only been shown to facilitate ROSC in cardiac patients with a cardiac arrest independent of rhythm.
arrest in contrast to chest compressions and defibrilla- Although worsening transaminitis and hepatic enzy-
tion (if indicated), which can improve survival. Because matic function slowing is likely to occur during
of this, CPR should begin immediately for this patient hypothermia, neither of these principles would be con-
before line placement to facilitate medication delivery sidered a contraindication for hypothermia (Answer B
(Answer B is incorrect), starting with chest compres- is incorrect). Furthermore, renal function with respect
sions in accordance with the BLS guidelines with pads to glomerular filtration worsens, which requires vigilant
placed simultaneously to facilitate rapid defibrillation if monitoring of renal function and close attention to renal
the patient’s rhythm reveals a shockable rhythm (Answer dose modifications and serum concentration monitoring
C is correct; Answer D is incorrect). of medications when applicable (Answer C is correct).
An additional complication of therapeutic hypothermia
2. Answer: A is an induced coagulopathy, which leads patients to be at
The cornerstone of therapy for VF cardiac arrest is rapid risk of bleeding. Thrombolytics may carry specific indi-
defibrillation (Answer A is correct). The recommended cations for cardiac arrest (e.g., pulmonary embolism or
dosage of voltage for biphasic defibrillators is 200 J or acute coronary syndromes), but they would not be rec-
the manufacturer’s recommendation (often the same dos- ommended empirically (Answer D is incorrect).
age). Although amiodarone is in the treatment algorithm
for VF cardiac arrest, it is reserved and recommended 5. Answer: B
for refractory VF cardiac arrest, which is defined as defi- Therapeutic hypothermia improves neurologic recovery
brillation refractory (Answer B is incorrect). Therefore, in patients after a cardiac arrest (Answer B is correct).
defibrillation should occur first. Atropine has been Most patients included in randomized clinical trials
removed from the cardiac arrest algorithms for PEA and have VF as the causative rhythm (Answers A and D are
asystole because of a lack of benefit on outcomes and incorrect). Because of its impact on neurologic recovery,
should not be considered for VF cardiac arrest (Answer guideline recommendations have applied this literature
C is incorrect). Pacing has not shown benefit in cardiac to cardiac arrests of all rhythms, and many institutions
arrest and should not be used (Answer D is incorrect). have adopted this same recommendation. Recent stud-
ies targeting mild hypothermia (36°C vs. 33°C) have
3. Answer: B shown no difference regarding outcomes between the
Because the rhythm detected is PEA and no longer VF, two modalities, and because of this, some question the
the cornerstone of therapy changes from giving defibril- utility of hypothermia at all in an era of potentially more
lation (Answer A is incorrect) to giving high-quality advanced cardiac arrest care (Answer C is incorrect).
chest compressions and addressing the treatable causes
of cardiac arrest (H’s and T’s) (Answer B is correct). 6. Answer: C
Lidocaine is reserved for refractory VF/pVT) (defined By definition, this patient is having a hypertensive emer-
as defibrillator refractory) when amiodarone is unavail- gency because she has an abrupt, severe increase in BP
able (Answer C is incorrect). Atropine was previously with target organ damage—in this case, potentially
recommended for PEA/asystole, but in the 2010 ACLS shock liver and vision changes. (Answer A is incorrect).
guidelines it was removed because of a lack of data sup- The patient’s SCr is not elevated from baseline and is
porting any beneficial outcomes (Answer D is incorrect). thus not presently showing target organ damage (Answer

ACCP Updates in Therapeutics® 2022: Critical Care Pharmacy Preparatory Review and Recertification Course

771
Cardiovascular Critical Care II

B is incorrect). This patient’s D-dimer is normal as well,


but even if it were elevated, this is not a specific marker
of target organ damage (Answer D is incorrect). The
patient has significant elevations in her transaminases,
indicating injury to the liver and thus qualifying her for
hypertensive emergency (Answer C is correct).

7. Answer: C
The patient should be initiated on nitroprusside for BP
management (Answer C is correct). Although she has
transaminitis, nitroprusside can be used safely in the
first 24 hours in these patients. Cyanide accumulation
would be a concern with extended use, but for the first
24 hours, the patient will be at minimal risk, and the
benefit of rapid BP control outweighs any potential risk.
Phentolamine would be reserved for hypertensive crisis
that presents from a catecholamine crisis (Answer A
is incorrect). Oral metoprolol might be an appropriate
option to transition to after the emergency is resolved,
but because of the target organ damage, more rapid
reduction is needed using an intravenous agent such as
nitroprusside (Answer B is incorrect). Enalaprilat is an
option, but 10 mg every 6 hours would not be an appro-
priate starting dose because it would put the patient at
risk of overshooting and, given the long duration of
activity, could lead to unwanted consequences (Answer
D is incorrect).

8. Answer: A
The initial goal reduction for this patient’s BP, given that
she is experiencing a hypertensive emergency, is a 25%
reduction in MAP within the first 60 minutes (Answer
A is correct). More rapid BP reductions may result in a
lack of cerebral perfusion; therefore, they are not rec-
ommended (Answer B is incorrect). The patient is not
experiencing any of the specific hypertensive emergen-
cies (e.g., aortic dissection) that would call for a more
rapid BP reduction. In addition, she is not experiencing a
hypertensive emergency that would require a slower BP
reduction (e.g., stroke) (Answers C and D are incorrect).

ACCP Updates in Therapeutics® 2022: Critical Care Pharmacy Preparatory Review and Recertification Course

772
Infectious Diseases I
Eric W. Mueller, Pharm.D., FCCP, FCCM
University of Cincinnati Medical Center
Cincinnati, Ohio
Infectious Diseases I

Infectious Diseases I
Eric W. Mueller, Pharm.D., FCCP, FCCM
University of Cincinnati Medical Center
Cincinnati, Ohio

ACCP Updates in Therapeutics® 2022: Critical Care Pharmacy Preparatory Review and Recertification Course

775
Infectious Diseases I

Learning Objectives IDSA Infectious Diseases Society of


America
1. Develop risk factor–based empiric antibiotic regi- IVIG Intravenous immunoglobulin
mens for patients with suspected ventilator-associated MDRO Multidrug-resistant organism
pneumonia. MICU Medical intensive care unit
2. Identify a definitive management strategy for central MRSA Methicillin-resistant Staphylococcus
line–associated bloodstream infections. aureus
3. Describe definitive and supportive care pharmaco- MRSE Methicillin-resistant Staphylococcus
therapeutic interventions for patients with severe epidermatitis
influenza. MSSA Methicillin-sensitive Staphylococcus
4. Develop empiric and definitive antimicrobial therapy aureus
plans for patients with catheter-related urinary tract MV Mechanical ventilation
infection. NHSN National Healthcare Safety Network
5. Differentiate between location of intra-abdominal SARS-CoV-2 Novel severe acute respiratory
infection and respective empiric antimicrobial syndrome coronavirus 2
therapy. RT-PCR Reverse transcription polymerase
6. Describe the role of antibiotic therapy in patients chain reaction
with acute pancreatitis. SICU Surgical intensive care unit
7. Develop a definitive management strategy for SIRS Systemic inflammatory response
critically ill patients with severe Clostridioides syndrome
difficile infection. SJS Stevens-Johnson syndrome
8. Recommend empiric and definitive antibiotic ther- TBSA Total body surface area
apy for patients with severe postoperative wound TEN Toxic epidermal necrolysis
infection. UTI Urinary tract infection
9. Describe the role of pharmacotherapy in the man- VAP Ventilator-associated pneumonia
agement of severe cutaneous reactions.
10. Identify a disease-specific and supportive care man-
agement plan for critically ill patients with novel Self-Assessment Questions
severe acute respiratory syndrome coronavirus 2. Answers and explanations to these questions may be
found at the end of this chapter.

Abbreviations in This Chapter Questions 1 and 2 pertain to the following case.


K.P., a 38-year-old otherwise healthy woman, was
ARDS Acute respiratory distress syndrome involved in a motor vehicle collision, sustaining a trau-
BAL Bronchoalveolar lavage matic brain injury and severe chest injuries requiring
CAUTI Catheter-associated urinary tract endotracheal intubation and mechanical ventilation.
infection After being in the trauma intensive care unit (ICU) for
CDI Clostridioides difficile infection 96 hours, a new infiltrate is noted on her chest radio-
CLABSI Central line–associated bloodstream graph, as well as a temperature of 101.9°F (38.8°C), a
infection white blood cell count (WBC) of 15 x 103 cells/mm3, and
COVID-19 Coronavirus disease 2019 macroscopically purulent sputum. K.P. is hemodynami-
CVC Central venous catheter cally stable. Accordingly, your team decides to obtain
DRESS Drug rash with eosinophilia and a bronchoscopic bronchoalveolar lavage (BAL) of the
systemic symptoms affected lung field to assess for ventilator-associated
ECMO Extracorporeal membrane pneumonia (VAP).
oxygenation
ED Emergency department 1. Which is the most likely causative pathogen of
ICU Intensive care unit K.P.’s suspected VAP?

ACCP Updates in Therapeutics® 2022: Critical Care Pharmacy Preparatory Review and Recertification Course

776
Infectious Diseases I

A. Acinetobacter spp. 5. T.S. is a 79-year-old woman admitted to the MICU


B. Methicillin-resistant Staphylococcus aureus for respiratory failure and severe community-
(MRSA) acquired pneumonia. T.S. has had a urethral catheter
C. Multidrug-resistant Pseudomonas aeruginosa in place for 6 days while mechanically ventilated
D. Streptococcus pneumoniae on fentanyl infusion and intermittent haloperidol
as needed for pain and delirium, respectively. This
2. Based on the 2016 IDSA guidelines, which empiric morning, T.S. had a temperature of 101.6°F (38.7°C)
antibiotic regimen is best for the likely causative and an elevation in WBC to 16 x 103 cells/mm3; she
pathogen(s) of K.P.’s suspected VAP? is hemodynamically stable. Blood and urine cul-
A. Azithromycin plus moxifloxacin tures are sent. Urinalysis reveals significant pyuria.
B. Cefepime Which pathogen is most likely to cause a catheter-
C. Ceftriaxone associated urinary tract infection (CAUTI) in T.S.?
D. Ceftriaxone plus vancomycin A. Enterococcus faecalis
B. Escherichia coli
3. T.D. is a 57-year-old man admitted to the medical C. P. aeruginosa
intensive care unit (MICU) with severe hyperosmo- D. S. aureus
lar hyperglycemic syndrome. A subclavian central
venous catheter (CVC) is placed on his arrival at the 6. K.D. is a 59-year-old man admitted to the surgi-
MICU. On ICU day 4, T.D. has a temperature of cal intensive care unit (SICU) after an emergency
102.7°F (39.2°C) and a WBC of 17 x 103 cells/mm3. operation and partial bowel resection with primary
The nurse notes new-onset erythema at the catheter anastomosis for mid-small bowel necrosis and perfo-
site. After an appropriate diagnostic workup, it is ration likely secondary to severe peripheral vascular
decided to initiate empiric antibiotic therapy for disease. During the operation, significant peritoneal
a suspected central line–associated bloodstream contamination with evidence of gross peritonitis
infection (CLABSI). Which agent is best for empiric was noted together with persistent hypotension and
therapy? need for vasopressors. K.D. received perioperative
A. Cefazolin cefazolin and metronidazole. Which empiric antibi-
B. Linezolid otic regimen would be most appropriate for K.D.?
C. Piperacillin/tazobactam A. Ceftriaxone and vancomycin
D. Vancomycin B. Ciprofloxacin and metronidazole
C. Ertapenem
4. D.G., a 31-year-old woman, presents to the MICU D. Piperacillin/tazobactam
with severe respiratory failure and acute respiratory
distress syndrome (ARDS) requiring intubation after 7. T.M. is a 42-year-old man with chronic alcoholism
48 hours of malaise, fever, and myalgias. The nasal who presents to the ED with severe epigastric pain
washing sent by the emergency department (ED) for and serum lipase greater than 10 times the upper
rapid diagnostic testing is positive for influenza A. limit of normal. The resident orders a computed
The local prevalence of subtype 2009 H1N1 is high. tomography (CT) scan, which reveals necrosis
Which agent would be most appropriate for initial affecting almost 40% of the pancreas but no abnor-
treatment of D.G.’s severe influenza? mal fluid collections or evidence of abscess. T.M. is
A. Amantadine febrile and tachycardic, and his WBC is elevated.
B. Inhaled zanamivir T.M.’s urine output is less than 0.5 mL/kg/hour, sug-
C. Intravenous zanamivir gestive of hypovolemia. Which best describes the
D. Oseltamivir role of antibiotic therapy for T.M. at this time?
A. Antibiotic therapy is not indicated.
B. Initiate empiric antibiotic therapy for presumed
sepsis and likely pancreatic infection.

ACCP Updates in Therapeutics® 2022: Critical Care Pharmacy Preparatory Review and Recertification Course

777
Infectious Diseases I

C. Initiate perioperative antibiotic therapy in prep- 10. A.C. is a 27-year-old man with human immunode-
aration for pancreatic debridement. ficiency virus (HIV) on active antiretroviral therapy
D. Initiate prophylactic antibiotic therapy to pre- who presents to the burn ICU from an outside
vent infection of necrotic tissue. hospital with 30% total body surface area (TBSA)
epidermolysis of his back and upper arms sugges-
8. J.E. is a 67-year-old woman admitted to the MICU tive of toxic epidermal necrolysis (TEN), likely
for severe metabolic acidosis secondary to uninten- from sulfamethoxazole/trimethoprim. Reportedly,
tional metformin overdosage. Her ICU stay has been A.C. arrived at the outside hospital about 6 hours
complicated by a femoral vein CLABSI and related ago with less than 10% TBSA involvement. Vital
severe sepsis caused by pan-sensitive E. coli. J.E. signs include heart rate 142 beats/minute, respira-
received a 3-day course of empiric piperacillin/tazo- tory rate 30 breaths/minute, and blood pressure
bactam, removal of her central line, and 11 days of 100/50 mm Hg. Which would be the best initial
ceftriaxone as definitive therapy with resolution of pharmacotherapeutic intervention?
sepsis. Starting yesterday, it was noted that J.E. had A. Administer high-dose corticosteroids to halt the
nine loose bowel movements and new leukocytosis progression of TEN.
of 14,500 cells/mm3, which suggests Clostridioides B. Initiate crystalloid resuscitation for
difficile infection (CDI). J.E. continues to tolerate hypovolemia.
enteral nutrition. Per the 2021 IDSA focused guide- C. Place a nasogastric tube to administer antiretro-
line update, which is the most appropriate regimen viral medications.
for J.E.’s suspected CDI? D. Initiate broad-spectrum antibiotic prophylaxis
A. Fidaxomicin 200 mg per feeding tube every 12 for wound infection.
hours
B. Metronidazole 500 mg per feeding tube every 11. T.F. is a 68-year-old man with diabetes and hyper-
8 hours tension who presents to the ED with altered mental
C. Metronidazole 500 mg intravenously every 8 status and acute kidney injury. T.F. was seen at an
hours outside hospital 5 days ago, where he tested positive
D. Vancomycin 125 mg per feeding tube every 6 for novel severe acute respiratory syndrome corona-
hours virus 2 (SARS-CoV-2). On initial assessment, T.F.
is hemodynamically stable with an oxygen satura-
9. J.S. is a 42-year-old man admitted to the SICU after tion (SaO2) of 87% on room air, which has worsened
an open total colectomy with ileostomy for ischemic over the past 4 hours. T.F. now requires 5 L of high-
colitis. On postoperative day 4, the surgery resident flow nasal cannula to maintain an SaO2 above 92%.
on your interdisciplinary ICU team notes moderate Chest radiography is notable for bilateral opacities
erythema (3 cm) and purulent drainage from the consistent with viral pneumonia. Initial laboratory
wound. The resident subsequently opens the skin tests indicate alanine aminotransferase 23 U/L and
portion of the wound and finds infected material just estimated creatinine clearance 31 mL/minute (base-
above the unaffected fascia. No systemic signs and line 68 mL/minute). Which agent is considered
symptoms are noted. Which intervention is most first-line therapy in hospitalized patients with coro-
appropriate? navirus disease 2019 (COVID-19) and increasing
A. Continue dressing changes and patient oxygen requirement?
assessment. A. Dexamethasone.
B. Initiate empiric vancomycin for postoperative B. Hydroxychloroquine.
wound infection. C. Remdesivir.
C. Initiate empiric antibiotic therapy targeted D. Tocilizumab.
against skin and colonic flora.
D. Initiate broad-spectrum antibiotic therapy for
suspected necrotizing fasciitis.

ACCP Updates in Therapeutics® 2022: Critical Care Pharmacy Preparatory Review and Recertification Course

778
Infectious Diseases I

BPS Critical Care Pharmacy Specialist Examination Content Outline

This chapter covers the following sections of the Critical Care Pharmacy Specialist Examination Content Outline:
1. Domain 1: Clinical Knowledge and Application
a. Task 1: 3
b. Task 2: 3, 4, 6
c. Task 3: 1-7
d. Task 4: 1-6, 8
e. Task 5: 1, 2, 5-7
f. Task 6: 1-4
g. Task 7: 1-3, 5-7
h. Task 8: 3, 4
2. Domain 2: Practice Management, Policy, and Quality Improvement
a. Task 1: 2, 3
b. Task 4: 2, 3
3. Domain 3: Evidence-Based Medicine, Scholarship, Education, and Professional Development
a. Task 1: 3
b. Task 3: 1

ACCP Updates in Therapeutics® 2022: Critical Care Pharmacy Preparatory Review and Recertification Course

779
Infectious Diseases I

I.  VENTILATOR-ASSOCIATED PNEUMONIA

A. Epidemiology
1. About 90% of hospital-acquired pneumonia episodes in critically ill patients occur during mechanical
ventilation. Mechanical ventilation is an independent risk factor for pneumonia.
2. Between 9% and 27% of mechanically ventilated patients develop pneumonia.
3. The incidence of pneumonia increases with the duration of mechanical ventilation, with the highest risk
during the first 5 days. Individual risk factors for VAP include underlying chronic lung disease, acute
lung injury, aspiration, coma, trauma (e.g., chest trauma; traumatic brain injury), burns, reintubation,
and overall severity of illness.
4. VAP accounts for more than 50% of antibiotic use in critically ill patients and has an attributable cost
of more than $40,000 per episode.
5. VAP has an estimated attributable mortality of 10%–50%, which varies among critically ill populations.

B. Classification and Etiology


1. VAP is a distinct type of hospital-acquired pneumonia that occurs more than 48 hours after endotracheal
intubation. VAP is further classified by the Centers for Disease Control and Prevention (CDC) as a
ventilator-associated event and infection-related ventilator-associated complication. Updated guidelines
from the IDSA for the diagnosis and management of VAP were published in 2016. (Please see Table 2
at the end of this section for summary and comparison with 2005 guidelines.)
2. VAP is usually caused by bacterial pathogens; may be monomicrobial or polymicrobial, but is rarely
caused by viral or fungal pathogens.
3. The prevalence of specific bacterial pathogens varies between critically ill patient populations,
geographic region, and local antibiotic usage patterns. Risk factors for multidrug-resistant organisms
(MDROs) are summarized in Box 1.
4. Classification of VAP as early onset (within the first 2–4 days of hospitalization) or late onset (after 5
days or more of hospitalization) to differentiate between suspected pathogens is no longer recommended.
However, longer duration (e.g., 5 days) of hospitalization before the onset of VAP is associated with
nosocomial and MDRO pathogens.

Box 1. Risk Factors for MDROs Causing VAP


Prior intravenous antibiotic therapy in preceding 90 days
Acute renal replacement therapy before VAP onset
5 or more days of hospitalization before the occurrence of VAP
Septic shock at the time of VAP
ARDS preceding VAP
MDRO = multidrug-resistant organism.

C. Prevention
1. VAP is considered a reportable and preventable complication of endotracheal intubation and mechanical
ventilation.
2. Data are emerging on effective prevention strategies.
3. Recommended best practices for preventing VAP include:
a. Avoid or limit the duration of endotracheal intubation.
b. Minimize duration and deep levels of sedation to promote assessment of readiness to extubate.
c. Maintain and improve physical conditioning while mechanically ventilated.
d. Minimize pooling of secretions above the endotracheal tube cuff.

ACCP Updates in Therapeutics® 2022: Critical Care Pharmacy Preparatory Review and Recertification Course

780
Infectious Diseases I

e. Elevate head of bed at least 30 degrees.


f. Maintain integrity of mechanical ventilator circuit.

D. Diagnosis
1. The diagnostic approach for VAP includes (1) determining whether clinical signs and symptoms are
caused by pneumonia and (2) if pneumonia is present, identifying the causative pathogen(s), preferably
using lower respiratory tract culture.
2. About half of mechanically ventilated patients with a clinical suggestion of pneumonia will have
bacteriologically confirmed pneumonia.
3. Clinical signs and symptoms of pneumonia include new or changing infiltrate on chest radiograph and
at least two of the following:
a. Elevated WBC
b. Fever (e.g., temperature greater than 100.4°F [38°C])
c. Macroscopically purulent sputum production
d. Impaired or worsening oxygenation
4. Lower respiratory tract cultures can be obtained through noninvasive or invasive techniques and
reported as qualitative, semiquantitative, or quantitative.
a. Quantitative (expressed as CFU/mL) and semiquantitative (expressed as rare/few to many) cultures
using recommended diagnostic growth thresholds are more specific than qualitative cultures for
identifying the causative pathogen.
b. Noninvasive techniques: Tracheal aspirate from endotracheal or tracheostomy tube – proximal to
distal sampling (depending on depth of sample) of upper airway secretions; usually semiquantitative
c. Invasive techniques
i. Blind, catheter-directed, or bronchoscopic BAL – Distal sampling of lung lobe/segment using
saline lavage; significant quantitative growth threshold above 10,000 or 100,000 CFU/mL
ii. Bronchoscopic protected specimen brush (PSB) – Distal sampling of specific bronchial
segment; significant quantitative growth threshold above 1000 CFU/mL
5. According to the current IDSA guidelines, the suggested diagnostic strategy for VAP includes clinical
suspicion and use of noninvasive sampling with semiquantitative cultures to diagnose VAP, rather than
invasive sampling or noninvasive sampling with quantitative cultures.
a. If an invasive strategy with quantitative cultures is used, it is suggested that antibiotics be withheld
rather than continued if quantitative culture results are below the diagnostic threshold for VAP (i.e.,
PSB less than 1000 CFU/mL; BAL less than 10,000 CFU/mL).

E. Treatment
1. Antibiotic therapy for VAP is empiric or definitive.
2. Antibiotic selections should include intravenous agents able to achieve relevant pulmonary tissue
concentrations related to pathogen minimum inhibitory concentration (MIC) dosed optimally using
evidence-based pharmacokinetic and pharmacodynamic principles.
3. Empiric antibiotic therapy should be initiated in patients with clinical suspicion for VAP (Table 1).
Patients with septic shock should receive antibiotics within 1 hour from onset of hypotension (see
Sepsis chapter).
a. Inappropriate empiric antibiotic therapy for VAP (i.e., delay in or absence of antibiotics active
against identified causative pathogen[s]) is associated with increased mortality.
b. To decrease the likelihood of inappropriate therapy, empiric antibiotic selections should be based on:
i. Presence of MDRO risk factor
ii. Local VAP pathogen prevalence, particularly MRSA
iii. Local antibiotic susceptibility patterns (i.e., ICU-specific antibiogram)

ACCP Updates in Therapeutics® 2022: Critical Care Pharmacy Preparatory Review and Recertification Course

781
Infectious Diseases I

c. All patients with suspected VAP should receive an antibiotic active against MSSA, P. aeruginosa,
and other gram-negative bacilli.
i. Suggest prescribing two antipseudomonal antibiotics from different classes in patients with
any of the following:
(a) MDRO risk factor
(b) ICU antibiogram with greater than 10% of gram-negative isolates resistant to an agent
being considered for monotherapy (i.e., multidrug-resistant [MDR] P. aeruginosa)
(c) ICU where local antimicrobial susceptibility rates unavailable
d. Suggest including an agent active against MRSA in patients with any of the following:
i. MDRO risk factor
ii. ICU MRSA prevalence greater than 10%–20% of S. aureus. (Note: MRSA rates vary between
centers, ICUs, and populations, with general rates in U.S. hospitals greater than 20% [https://
cddep.org/tool/mrsa_infection_rates_country/].)
iii. Prevalence of MRSA is not known
e. Combination antibiotic therapy using agents with similar bacterial spectra but different mechanisms
of action may be necessary to increase the likelihood of appropriate empiric antibiotic therapy for
other gram-negative pathogens.

Table 1. VAP Classifications and Recommended Empiric Antibiotic Therapy Based on 2016 IDSA VAP Guidelines
Likely Pathogens Recommended Empiric Antibiotic(s)a
Patients without MDRO risk factor: Single antipseudomonal agent with 90% or greater empiric activity on
local ICU antibiogram, and local MRSA prevalence is less than 10-20%
P. aeruginosa
S. pneumoniae
Haemophilus influenzae
α- and β-hemolytic Streptococcus spp. Cefepime, imipenem, levofloxacin, meropenem,
Methicillin-sensitive S. aureus (MSSA) piperacillin/tazobactam
Antibiotic-sensitive enteric gram-negative bacilli
(GNB) (e.g., E. coli; Klebsiella spp.; Enterobacter spp.;
Proteus spp.; Serratia spp.)
Patient without MDRO risk factor: Single antipseudomonal agent with less than 90% empiric activity on
local ICU antibiogram and/or local MRSA prevalence greater than 10-20%b
Double Antipseudomonal Coverage
Antipseudomonal β-lactamc
P. aeruginosa (aztreonam, cefepime, ceftazidime, imipenem,
Methicillin-resistant S. aureus (MRSA) meropenem, piperacillin/tazobactam)
S. pneumoniae +
Haemophilus influenzae Antipseudomonal fluoroquinolone (ciprofloxacin or
α- and β-hemolytic Streptococcus spp. levofloxacin)
Methicillin-sensitive S. aureus (MSSA) OR
Antibiotic-sensitive enteric gram-negative bacilli aminoglycosided
(GNB) (e.g., E. coli; Klebsiella spp.; Enterobacter spp.; (amikacin, gentamicin, or tobramycin)
Proteus spp.; Serratia spp.)
MRSA
Linezolid or vancomycin

ACCP Updates in Therapeutics® 2022: Critical Care Pharmacy Preparatory Review and Recertification Course

782
Infectious Diseases I

Table 1. VAP Classifications and Recommended Empiric Antibiotic Therapy Based on 2016 IDSA VAP Guidelines
(continued)
Likely Pathogens Recommended Empiric Antibiotic(s)a
Patients with MDRO risk factor
Antipseudomonal β-lactamc
(aztreonam, cefepime, ceftazidime, imipenem,
meropenem, piperacillin/tazobactam)
P. aeruginosa
+
Methicillin-resistant S. aureus (MRSA)
Antipseudomonal fluoroquinolone (ciprofloxacin or
Acinetobacter spp.
levofloxacin)
Antibiotic-resistant enteric GNB (e.g., extended-
OR
spectrum β-lactamase [ESBL]-producing organisms)
aminoglycosided
Stenotrophomonas maltophilia
(amikacin, gentamicin, or tobramycin)
AND
Linezolid or vancomycin
a
Agents/classes are listed in alphabetic order.
b
There may be situations in which a patient has no MDRO risk factors but may meet the recommendation to receive dual antipseudomonal therapy and/or MRSA
coverage.
c
Doripenem is no longer available in the United States. Before being discontinued, doripenem had a U.S. Food and Drug Administration (FDA) warning against use for
treatment of ventilator-associated bacterial pneumonia.
d
Guidelines suggest avoiding empiric aminoglycoside if alternative agents with activity are available.

f. Lower respiratory tract cultures should be obtained before initiation of antibiotic therapy to increase
the likelihood of identifying causative pathogen(s). Inappropriate delays in initiation of antibiotic
therapy should be avoided in unstable patients (e.g., patients with septic shock).
g. Empiric antibiotic therapy should be de-escalated to definitive therapy, depending on identified
pathogen(s) and antibiotic susceptibilities.
4. Definitive antibiotic therapy should be focused on the causative pathogen(s) identified on lower
respiratory tract culture.
a. Pathogen-specific definitive antibiotic choices should be based on antibiotic susceptibilities and, if
possible, available evidence supporting efficacy and safety in patients with VAP.
i. MRSA: Vancomycin or linezolid; daptomycin is not indicated for pneumonia because of direct
inhibition by pulmonary surfactant.
ii. P. aeruginosa
(a) Monotherapy with a β-lactam listed below and found to be susceptible on final culture
results is recommended in immunocompetent patients not in septic shock and not at a
high risk of dying. Concomitant aminoglycoside or antipseudomonal fluoroquinolone is
recommended in immunocompromised patients and those in septic shock or at a high risk
of dying. Combination therapy can be de-escalated to monotherapy β-lactam once septic
shock resolves.
(b) β-Lactams: Cefepime, ceftazidime, piperacillin/tazobactam, imipenem, or meropenem
(c) Aminoglycosides: Amikacin, gentamicin, or tobramycin
(d) Fluoroquinolones: Ciprofloxacin or levofloxacin
iii. Acinetobacter spp.: Imipenem or meropenem; ampicillin/sulbactam (sulbactam is active agent)
as alternative option
iv. Stenotrophomonas maltophilia: Sulfamethoxazole/trimethoprim
v. Extended-spectrum β-lactamase (ESBL)-producing organisms: Carbapenem (e.g., ertapenem;
imipenem; meropenem)

ACCP Updates in Therapeutics® 2022: Critical Care Pharmacy Preparatory Review and Recertification Course

783
Infectious Diseases I

vi. Since the 2016 IDSA guidelines, cefiderocol, ceftolozane/tazobactam, ceftazidime/avibactam,


and imipenem/cilastatin/relebactam have been FDA approved for the treatment of VAP. These
agents can be considered for certain multidrug-resistant (MDR) pathogens (e.g., ESBL; specific
carbapenemases) in concert with an antimicrobial stewardship program.
b. Duration of definitive antibiotic therapy is recommended to be 7 days for all patients rather than
longer durations. Pathogen-based recommendations for duration of definitive antibiotic are no
longer indicated in VAP guidelines.
i. The PneumA trial was a pivotal noninferiority study that evaluated 8 days (7 full treatment
days) versus 15 days (14 full treatment days) for duration of definitive antibiotic therapy in
patients with bacteriologically diagnosed VAP. The primary noninferiority outcome was VAP
recurrence. All patients included in the study received appropriate empiric antibiotic therapy
(JAMA 2003;290:2588-98). Major study findings were:
(a) Definitive antibiotic therapy for 8 days was noninferior to 15 days in patients with VAP
not caused by non–lactose-fermenting gram-negative bacilli (e.g., P. aeruginosa).
(b) Patients with VAP caused by non–lactose-fermenting gram-negative bacilli in the 8-day
group more often had VAP recurrence.
(c) The rate of MDR pathogens during a recurrent VAP episode was higher in patients in the
15-day group.
ii. Two meta-analyses that included the results of the PneumA trial suggest limited benefit of
prolonged duration of definitive antibiotic therapy for treating VAP (Chest 2013;144:1759-67;
Cochrane Database Syst Rev 2015;8:CD007577). Synopsis includes:
(a) Any increase in VAP recurrence rate is small;
(b) Mortality and clinical cure do not appear to be affected by shorter durations;
(c) Evidence for recurrence from subgroup analyses has important limitations.
iii. Guidelines acknowledge that situations may exist in which shorter or longer durations of
definitive antibiotic therapy may be indicated, depending on the rate of clinical and radiologic
improvement. A single-center, retrospective study suggested that patients with “possible VAP”
who had minimal and stable ventilator settings at diagnosis are appropriate for short-course
therapy (i.e., 1-3 days) (Clin Infect Dis 2017;64:870-6). However, application of these findings
is limited, given that only 39% of short-course patients had an identified pathogen on tracheal
aspirate or BAL culture, bringing into question the likelihood of confirmed pneumonia.
5. Response to antibiotic therapy should be assessed serially using clinical signs and symptoms of infection
and status of VAP-related organ dysfunction.
a. Lack of response in signs and symptoms of VAP beyond treatment day 3 necessitates reassessment
of diagnosis, causative pathogen(s), antibiotic regimen, and presence of pneumonia-related
complications (e.g., ARDS).
b. Patients thought to have persistent VAP should undergo repeat lower respiratory tract culture and
receive empiric antibiotic therapy considering previous pathogen(s) and antibiotic exposure.

Table 2. Summary of Key Changes in 2016 IDSA VAP Guidelines


Characteristic 2005 Guidelines 2016 Guidelines
GRADE criteria to identify “recommended”
Expert opinion based on level of evidence (strong) or “suggested” (weak) guidance based
Methodology
ranging from Level I (high) to III (low) on level of evidence categories “very low,” “low,”
“moderate,” and “high quality”

ACCP Updates in Therapeutics® 2022: Critical Care Pharmacy Preparatory Review and Recertification Course

784
Infectious Diseases I

Table 2. Summary of Key Changes in 2016 IDS VAP Guidelines (continued)


Characteristic 2005 Guidelines 2016 Guidelines
Suggest clinical suspicion with noninvasive
sampling and semiquantitative cultures

Diagnosis Clinical strategy or bacteriologic strategy If invasive sampling is used, suggest that
antibiotics be withheld rather than continued
if quantitative culture results are below the
diagnostic threshold for VAP
No longer differentiate VAP episodes based on
time from hospital admission
Classification Early onset or late onset Emphasis on local risk assessment for P.
aeruginosa and MRSA occurring early in
hospitalization
• Antibiotic therapy in preceding 90 days
• Chronic dialysis within 30 days
• Prior intravenous antibiotic therapy in
• Current hospitalization of 5 days or more
preceding 90 days
• Home chronic wound care
• Acute renal replacement therapy before VAP
• Home infusion therapy
onset
MDRO Risk • Hospitalization in the preceding 90 days
• 5 or more days of hospitalization before the
Factors • Immunosuppressive disease and/or
occurrence of VAP
therapy
• Septic shock at the time of VAP
• Known contact or colonization with
• ARDS preceding VAP
MDROs
• Residence in a nursing home or
extended-care facility
Recommend coverage for MSSA, P. aeruginosa,
and other gram-negative bacilli in all empiric
regimens.
Suggest including an agent active against MRSA
if any of the following:
Categorized based on early onset or late • MDRO risk factor for antimicrobial resistance
onset and presence of MDRO risk factor • ICU MRSA prevalence greater than 10%-20%
• Early onset without MDRO risk factor: of all S. aureus
Empiric Therapy no antipseudomonal or MRSA coverage • Prevalence of MRSA is not known
• Late onset or early onset with MDRO
Suggest prescribing dual antipseudomonal antibi-
risk factor: dual antipseudomonal and
otics from different classes if any of the following:
MRSA coverage
• MDRO risk factor for antimicrobial resistance
• ICUs with greater than 10% of gram-negative
isolates are resistant to an agent being
considered for monotherapy
• ICU where local antimicrobial susceptibility
rates unavailable
Suggest avoiding if alternative agents with
Aminoglycosides No specific recommendation
adequate gram-negative activity are available
Suggest avoiding if alternative agents with
Colistin No specific recommendation
adequate gram-negative activity are available

ACCP Updates in Therapeutics® 2022: Critical Care Pharmacy Preparatory Review and Recertification Course

785
Infectious Diseases I

Table 2. Summary of Key Changes in 2016 IDS VAP Guidelines (continued)


Characteristic 2005 Guidelines 2016 Guidelines
In patients who received appropriate Recommend 7 days rather than longer durations
Duration of empiric antibiotic therapy: regardless of pathogen
Definitive • Non–lactose-fermenting Gram negative Suggest using procalcitonin plus clinical criteria
Therapy bacilli: 14 days to guide discontinuation of antibiotic therapy,
• All other pathogens: 7 days rather than clinical criteria alone

Patient Cases

1. C.T. is a 48-year-old man sustaining a 40% TBSA burn to his left side with likely inhalational injury
requiring mechanical ventilation. C.T. has been in the ICU for 7 days. Overnight, C.T. had a temperature of
102.1°F (39°C), a WBC of 18 x 103 cells/mm3, and an increase in macroscopically purulent sputum produc-
tion; C.T. is hemodynamically stable with no signs of new-onset organ dysfunction. C.T.’s chest radiograph
is difficult to assess, given his inhalational injury. The respiratory therapist was asked by the fellow to do
a catheter-directed BAL for suspected VAP. The culture was sent to the microbiology laboratory. Which
would be best to do at this point?
A. Initiate broad-spectrum empiric antibiotic therapy for suspected MDR VAP.
B. Use Gram stain results to determine empiric antibiotic regimen.
C. Await preliminary quantitative culture results before initiating empiric antibiotic therapy.
D. Send blood and urine cultures, and await their results before initiating empiric antibiotic therapy.

2. C.P. is a 67-year-old woman with a medical history significant for chronic obstructive pulmonary disease
(COPD), type 2 diabetes, and coronary artery disease. She is readmitted to the ICU with respiratory fail-
ure requiring intubation secondary to severe COPD. C.P. had a 10-day hospitalization 12 days ago, during
which time she received intravenous azithromycin for a COPD exacerbation. On day 4 of this readmission,
a worsening infiltrate is in the left lower lung base with increased sputum production from admission, maxi-
mum temperature is 101.9°F (38.8°C), and there is worsening oxygenation, despite previous improvement
in initial COPD exacerbation. The patient is thought to have clinical VAP, and a semiquantitative tracheal
aspirate is sent to identify causative pathogen(s). The local ICU prevalence of MRSA is 30%, and the most
active β-lactam antibiotic against P. aeruginosa on the antibiogram has 80% activity. Which empiric anti-
biotic regimen is best for this patient?
A. Azithromycin plus moxifloxacin.
B. Cefepime plus tobramycin, and vancomycin.
C. Ceftriaxone plus azithromycin.
D. Linezolid plus tobramycin.

3. K.L., a 37-year-old man who presents after a motorcycle collision, has sustained several orthopedic and
chest injuries. K.L. has been mechanically ventilated for 8 days, during which time he was given a diagno-
sis of VAP caused by Klebsiella pneumoniae. K.L. received appropriate empiric antibiotic therapy and has
improved oxygenation, decreased WBC temperature curve. Which would be the best duration of definitive
antibiotic therapy for K.L.’s VAP?
A. 24 hours after resolution of clinical signs and symptoms.
B. 7 days.
C. 10 days.
D. 14 days.

ACCP Updates in Therapeutics® 2022: Critical Care Pharmacy Preparatory Review and Recertification Course

786
Infectious Diseases I

II.  CENTRAL LINE–ASSOCIATED BLOODSTREAM INFECTIONS

A. Epidemiology
1. Critically ill patients commonly require short-term (less than 14 days per catheter) or temporary (non-
tunneled or non-implanted) placement of a CVC or central line for medication administration and
hemodynamic support and monitoring.
2. More than 30,000 bloodstream infections a year are associated with central catheter placement.
The number of cases has decreased over the past decade because of increased awareness and many
systematic prevention efforts.
3. In 2012, the NHSN reported rates of CLABSI across adult critically ill populations of 0.8–3.4 episodes
per 1000 central line–days. Aggregated national data from 2016 showed an almost 50% decrease in
CLABSI rates from 2008 (0.56 vs. 1.00 cases per 1000 central line-days). Although specific rates vary
between sites and populations, all central line types and locations of insertion have an attributable risk
of bloodstream infection.
4. Established risk factors for CLABSI include:
a. Excessive manipulation of the catheter during and after insertion
b. Internal jugular or femoral insertion site
c. Microbial colonization at the insertion site or catheter hub
d. Neutropenia
e. Prolonged duration of catheterization
f. Prolonged hospitalization before catheterization
g. Total parenteral nutrition
5. The attributable cost of a CLABSI is up to $40,000 per episode.

B. Definitions
1. Central or peripheral venous catheters are defined by the location at which the distal tip of the catheter
terminates. CVCs terminating in a great vessel are considered part of the central blood circulation.
The distal tip of a CVC usually resides in the inferior, superior, or distal vena cava, right atrium, or
pulmonary artery (e.g., pulmonary artery catheter).
2. CVCs are usually inserted into the central venous system using the internal or external jugular,
subclavian, femoral, or iliac veins. There are several types of short-term CVCs:
a. Single- and multiple-lumen (e.g., triple lumen) catheters: Most commonly used CVC
b. Catheter introducer: Used for massive resuscitation or facilitation of pulmonary artery catheter
insertion
c. Peripherally inserted central catheter (PICC): Short- or medium-term central catheter inserted in a
peripheral vein (e.g., cephalic, basilic, or brachial veins)
d. Pulmonary artery catheter: A central catheter of around 100 cm used for invasive hemodynamic
monitoring
3. Primarily for surveillance, the CDC defines CLABSI as a primary laboratory-confirmed bloodstream
infection occurring no sooner than 2 calendar days from catheter placement and no later than the day
after catheter removal. Laboratory-confirmed bloodstream infection is defined as either:
a. A recognized pathogen (i.e., not a common commensal organism) cultured from one or more blood
cultures, and the organism cultured from blood is not related to an infection at another site, or
b. A common commensal organism (e.g., diphtheroids, Bacillus spp., coagulase-negative
staphylococci, viridans streptococci) cultured from two or more blood cultures drawn on separate
occasions, and the organism cultured from blood is not related to an infection at another site, and
patient has at least one of the following signs or symptoms: fever (temperature greater than 100.4°F
[38°C]), chills, or hypotension

ACCP Updates in Therapeutics® 2022: Critical Care Pharmacy Preparatory Review and Recertification Course

787
Infectious Diseases I

4. IDSA defines a catheter-related bloodstream infection as minimally bacteremia or fungemia in a patient


who has an intravascular device and:
a. More than one positive blood culture obtained from a peripheral vein. A definitive diagnosis
requires that the same organism(s) grow from at least one percutaneous blood culture and from the
CVC tip or that two blood cultures, one from a catheter hub and one from a peripheral vein, are
positive for the same organism(s).
b. Clinical manifestations of infection (e.g., fever, chills, and/or hypotension), and
c. No apparent source for bloodstream infection other than the catheter

C. Etiology
1. CLABSIs are usually monomicrobial. Pathogen prevalence is based on patient-specific risk factors and
underlying illness.
2. Common organisms responsible for CLABSIs include coagulase-negative staphylococci (e.g.,
Staphylococcus epidermidis), S. aureus, Candida spp., and enteric gram-negative bacilli (e.g., E. coli,
Klebsiella spp., Enterobacter spp.).
3. Risk factors for MDROs include critical illness, femoral catheter placement, immunosuppression, and
previous antibiotic exposure.
4. Candida spp. are more common in patients with the following risk factors: total parenteral nutrition,
prolonged exposure to broad-spectrum antibiotics, hematologic malignancy, stem cell or solid-organ
transplantation, femoral site of catheterization, or colonization owing to Candida spp. at several sites.

D. Prevention
1. CLABSI is considered a preventable complication. The NHSN, using CDC definitions, provides
population-specific event rates for institution surveillance and performance benchmarks.
2. The foundation for preventing CLABSI includes training and education, proper aseptic insertion
techniques, and active surveillance and performance improvement systems.
3. Recommended best practices for preventing CLABSI have been proposed and endorsed by the IDSA,
the Society for Healthcare Epidemiology of America (SHEA), and the Joint Commission. These
evidence-based recommendations are categorized as basic practices for all acute care hospitals or special
practices wherein basic practices are less than effective at reducing CLABSI rates. The entire document
is available (Infect Control Hosp Epidemiol 2014;35:753-71). Major recommendations include:
a. Basic practices
i. Minimize central venous line insertions and durations of insertion.
ii. Provide comprehensive education to and ensure competency for all involved with insertion,
care, and maintenance of CVC.
iii. Daily chlorhexidine baths for patients to reduce colonization
iv. Use of a systematic process or checklist at the time of insertion to ensure adherence to proper
insertion technique
v. Alcohol-based chlorhexidine skin cleanser for the site during insertion
vi. Handwashing during insertion, care, and maintenance of the catheter
vii. Cleanse catheter hubs before accessing.
viii. Routine post-insertion site care
ix. Limit prolonged use of intravenous tubing sets.
b. Special practices
i. Use antiseptic or antimicrobial-impregnated catheters.
ii. Chlorhexidine-containing site/catheter dressings
iii. Use antiseptic hub cover or port protector.
iv. Antimicrobial lock therapy

ACCP Updates in Therapeutics® 2022: Critical Care Pharmacy Preparatory Review and Recertification Course

788
Infectious Diseases I

E. Diagnosis
1. Clinical signs and symptoms of infection have poor sensitivity and specificity. Fever is the most sensitive
clinical finding, whereas inflammation or purulence at the insertion site is the most specific.
2. If a CLABSI is suspected, paired blood cultures drawn from the catheter (at least one hub/port) and
from a peripheral vein should be obtained. The individual bottles should be appropriately marked
through the culture-reporting period.
a. If a blood culture cannot be drawn from a peripheral vein, it is recommended that at least two blood
cultures be obtained through different catheter hubs/ports.
b. Blood cultures positive for S. aureus, coagulase-negative staphylococci, or Candida spp. that are
not attributable to another source should increase the suggestion of CLABSI.
c. Blood cultures should be obtained before initiation of antimicrobial therapy, as appropriate.
3. A definitive diagnosis of CLABSI requires positive percutaneous blood culture results with positive
culture of same pathogen from the catheter tip or catheter-drawn cultures. Please see the Infections
Diseases II chapter for further discussion on the pathway for regulatory reporting of CLABSI.

F. Treatment
1. Catheter removal should be considered in all patients with a confirmed CLABSI. If a CVC is still
necessary, a different anatomic site should be used. Changing to a new catheter at the same site using a
guidewire or catheter introducer should be avoided.
2. Antimicrobial therapy for CLABSI is empiric or definitive.
3. Inappropriate empiric antimicrobial therapy is associated with increased mortality, including bacterial
and fungal etiologies.
a. Empiric antimicrobial therapy should minimally include an agent active against methicillin-
resistant coagulase-negative (e.g., methicillin-resistant S. epidermidis [MRSE]) or coagulase-
positive (e.g., MRSA) staphylococci. Vancomycin or daptomycin are preferred; linezolid should
not be used for empiric management of CLABSI.
b. Pathogen-specific risk factors, documented colonization, and previous antimicrobial exposure
should be considered when choosing empiric antimicrobial therapy.
i. Patients at risk of MDROs should receive combination therapy against gram-negative bacilli
using agents from separate antibiotic classes.
ii. Use of an echinocandin (e.g., anidulafungin, caspofungin, or micafungin) should be considered
for patients at risk of candidemia. Fluconazole is reasonable in patients without recent exposure
to and low prevalence of nonsusceptible species.
c. Local antimicrobial activity should be considered to increase the probability of appropriate therapy.
d. Empiric antimicrobial therapy should be de-escalated, depending on the identified pathogen(s) and
related antimicrobial susceptibility. Antimicrobial therapy should be discontinued if a CLABSI is
not evident and there are no other sources of infection.
4. Definitive management and antimicrobial therapy should be based on whether the CLABSI is complicated
or uncomplicated (more common than complicated) and the identified pathogen(s). Ongoing clinical
trials may provide guidance to the optimal duration of antibiotic therapy for CLABSI and non–central
line-associated bacteremia (BMJ Open 2020;10:e038300).
a. The duration of antimicrobial therapy should be based on the first day of negative blood culture.
b. Complicated CLABSI
i. Endocarditis; immunosuppression (S. aureus only); diabetes (S. aureus only); chronic
intravascular hardware; osteomyelitis; positive blood cultures greater than 72 hours from
initiation of appropriate therapy; septic thrombus; thrombophlebitis
ii. Remove catheter.
iii. Treat with pathogen-targeted antimicrobial therapy for 4–6 weeks; 6–8 weeks for osteomyelitis.

ACCP Updates in Therapeutics® 2022: Critical Care Pharmacy Preparatory Review and Recertification Course

789
Infectious Diseases I

c. Uncomplicated CLABSI
i. Coagulase-negative staphylococci
(a) Consider catheter removal. If catheter is retained, consider antibiotic lock therapy in
addition to systemic antibiotic therapy for 10–14 days.
(b) Treat with systemic antibiotic therapy for 5–7 days if the catheter is removed.
ii. S. aureus
(a) Remove catheter.
(b) Treat with systemic antibiotic therapy for a minimum of 14 days.
(1) Methicillin-sensitive S. aureus (MSSA) – Penicillinase-resistant penicillin (e.g.,
nafcillin); first-generation cephalosporin (e.g., cefazolin) (Note: Vancomycin has
been shown inferior to β-lactam therapy for MSSA.)
(2) MRSA – Vancomycin; daptomycin or linezolid; sulfamethoxazole/trimethoprim
(c) Patients with catheter tip bacterial growth but negative blood cultures should receive
antibiotic therapy for 5–7 days with close monitoring for signs and symptoms of ongoing
infection and consideration for repeat blood cultures.
iii. Enterococcus spp.
(a) Remove catheter.
(b) Treat with systemic antibiotic therapy for 7–14 days.
iv. Gram-negative bacilli
(a) Remove catheter.
(b) Treat with systemic antibiotic therapy for 7–14 days.
v. Candida spp.
(a) Remove catheter.
(b) Treat with systemic antifungal therapy for at least 14 days.

Patient Cases

4. T.W. is a 47-year-old woman admitted to the MICU with respiratory failure secondary to severe 2009 H1N1
influenza infection. T.W., who requires intubation and mechanical ventilation, is given a diagnosis of septic
shock associated with influenza and a secondary MSSA pneumonia. An internal jugular vein CVC was
placed in the ED during acute resuscitation. T.W. continues to require a CVC. Although her hypotension
and fever resolved 72 hours post-admission, she has a new temperature of 101.7°F (38.7°C) with worsening
leukocytosis on ICU day 5; there is no change on her chest radiograph. Which action would be best to take
next?
A. Initiate broad-spectrum antibiotic therapy for a new sepsis episode.
B. Perform bronchoscopic BAL for suspected VAP.
C. Remove CVC.
D. Send two sets of blood cultures, one from the catheter and one from a peripheral blood sample.

ACCP Updates in Therapeutics® 2022: Critical Care Pharmacy Preparatory Review and Recertification Course

790
Infectious Diseases I

Patient Cases (continued)

Questions 5 and 6 pertain to the following case.


F.P. is a 29-year-old man admitted to the MICU from another hospital with severe, alcohol-induced, sterile acute
pancreatitis. During his 5-day stay at the outside hospital, he had multiple organ failure, including respiratory
failure requiring tracheostomy. A right internal jugular CVC was placed on the patient’s admission to the outside
hospital. On MICU day 3, F.P. has a maximum temperature of 102°F (38.9°C). Two sets of blood cultures are
obtained; the right internal jugular catheter is removed and the tip sent for culture.

5. Which regimen would be best to consider for empiric management of a suspected CLABSI?
A. Daptomycin and ceftriaxone.
B. Fluconazole.
C. Linezolid and cefepime.
D. Vancomycin, cefepime, and tobramycin.

6. F.P. is found to have K. pneumoniae CLABSI and receives appropriate empiric antibiotic therapy. He has
had a good clinical response with no persistence of bacteremia. Which represents the best duration of F.P.’s
definitive antibiotic therapy for CLABSI?
A. 5 days.
B. 14 days.
C. 21 days.
D. 4 weeks.

III.  INFLUENZA

A. Epidemiology
1. Influenza is a seasonal viral illness affecting all ages and associated with significant morbidity and
mortality. Seasonal patterns in the United States vary from year to year. For example, the 2008–2009
influenza season started in November 2008 and waned by April 2009, but then, because of influenza
A H1N1 (2009 H1N1), it resurged in May 2009, peaking in mid-October 2009 and persisting to April
2010. The 2020–2021 season spanned late September 2020, waned in late March 2021, and resurged to
match earlier rates through early September 2021.
2. According to the CDC, it is estimated that 400,000–900,000 patients a year in the United States are
hospitalized with influenza-related illness. However, during the 2020–2021 season, the reported
overall rate (cumulative rate per 100,000 population) of hospitalization for laboratory-confirmed
influenza was 0.8, notably lower than previous seasons (67 for 2019–2020 season). Age-related rates
for adult hospitalizations and ICU admissions were unavailable from the CDC as of September 2021.
As of September 2020, the CDC had insufficient data to report the ICU admission rate for hospitalized
patients. Over the immediate past five seasons, rates for adult ICU admission because of influenza
ranged from 15%–19%.
3. Risk factors for severe influenza requiring hospitalization include chronic respiratory or metabolic
illness, immunosuppression (disease or pharmacotherapy), pregnancy, and age older than 65 years.
In 2009, reemergence of the high-virulence strain influenza A H1N1 (pandemic 2009 H1N1) caused

ACCP Updates in Therapeutics® 2022: Critical Care Pharmacy Preparatory Review and Recertification Course

791
Infectious Diseases I

significant morbidity and mortality in young adults and other groups considered at a lower risk of
severe influenza. The pandemic 2009 H1N1 strain continues to be tracked and varies significantly from
year-to-year. During the 2013–2014 season, 2009 H1N1 was implicated in up to 50% of cases, while it
was associated with fewer than 2% of cases in 2014–2015. This strain was responsible for almost 54%
of influenza cases in 2015–2016, just over 2% in 2016–2017, 10.8% in 2017–2018, and just over 50% in
both 2018–2019 and 2019–2020. In contrast, 2009 H1N1 was responsible for 7.5% of reported cases in
2020–2021.
4. The most common complications of severe influenza include hypoxemic respiratory failure, bacterial
pneumonia, and ARDS. Concomitant sepsis, septic shock, multiple organ failure, encephalopathy, and
rhabdomyolysis also are associated with severe influenza.
5. Among all causes of deaths in the United States, pneumonia and influenza have surpassed the epidemic
threshold since 2009, with rates close to 10% during the peak of influenza seasons. Severe influenza
caused by the 2009 H1N1 strain has been associated with crude mortality rates of 15%–53%.

B. Etiology
1. Influenza is caused by the RNA viruses influenza A, B, or C, which usually spread through droplet
transmission. Influenza A and B viruses are the predominant causes of clinically significant illness.
Influenza virus subtypes are described by surface proteins hemagglutinin (H) and neuraminidase (N).
2. According to the CDC, influenza A has been the most prevalent cause of influenza since 2009, with
2009 H1N1 and H3 being the most prevalent influenza A subtypes. Influenza B prevalence has varied
during the same period, but it remains an important cause of illness. Overall for 2020–2021, influenza
A was isolated in 62.7% of cases. The most common specified influenza strains isolated in 2020–2021
were influenza A H3 at 10.8% followed by influenza A H1N1 at 7.5%. The most common influenza B
strain was of Victoria lineage at 5.9%.
3. Influenza viruses can cause a broad range of respiratory tract infections, ranging from mild to moderate
upper respiratory tract infections to severe pneumonia. Influenza infection has been associated with
acute viremia.

C. Prevention
1. Annual vaccination remains the primary tool for influenza prevention.
2. In-hospital and ICU outbreaks of influenza can contribute to viral transmission and associated sequelae.
3. Institutions should begin implementing influenza screening and infection control measures when
influenza viruses are confirmed to be in the local community.
4. The CDC recommends the implementation of droplet precautions for hospitalized patients with
suspected or confirmed influenza; further, these precautions are recommended for 7 days after illness
onset or until 24 hours after the resolution of fever and respiratory symptoms, whichever is longer.

D. Diagnosis
1. Patients with influenza may present with fever, myalgias, headache, malaise, dry cough, pharyngitis, and
rhinorrhea. Fever and myalgias generally last 3–5 days, whereas malaise and respiratory symptoms may
last 2 weeks or more. Patients with severe influenza may present with hypoxemic respiratory failure and
sepsis.
2. Clinical signs and symptoms of influenza are nonspecific. To confirm the diagnosis of influenza,
sampling of the upper respiratory tract using nasal washing or nasopharyngeal swab or lower respiratory
tract within 5 days of illness is preferred. Diagnostic tests obtained beyond 5 days may have false-
negative results.
3. Diagnostic tests available for influenza include viral culture, serology, rapid antigen testing, polymerase
chain reaction (PCR), and immunofluorescence assays.

ACCP Updates in Therapeutics® 2022: Critical Care Pharmacy Preparatory Review and Recertification Course

792
Infectious Diseases I

a. Viral culture and reverse transcription polymerase chain reaction (RT-PCR) are the most sensitive
and are the only tests able to identify individual influenza subtypes. These tests can be performed
using nasopharyngeal swab or respiratory tract culture (e.g., sputum, BAL).
b. Rapid diagnostic tests of nasopharyngeal swab or nasal wash have high specificity (90%–95%)
(i.e., low false-positive rate and rapid turnaround), promoting these tests as first line for general
diagnosis. Depending on the rapid diagnostic assay used, differentiation between influenza A and
influenza B is possible, although further subtype identification is not available with these tests.
Because of poor sensitivity, negative rapid diagnostic tests should be followed up with viral culture
or RT-PCR.
4. Case definitions for influenza surveillance are suspected, probable, or confirmed based on patient
presentation and laboratory assessment.
a. Suspected – Mild to severe influenzalike illness within reasonable seasonal threshold.
b. Probable – Mild to severe influenzalike illness within reasonable seasonal threshold or after recent
contact with person with probable or confirmed influenza infection and not otherwise explained.
c. Confirmed – Mild to severe influenzalike illness with confirmatory laboratory tests indicating
presence of influenza A or B.

E. Management and Treatment


1. Management of critically ill patients with influenza includes treatment of primary influenza infection,
secondary bacterial infection(s), and related noninfectious complications (e.g., respiratory failure,
ARDS, prolonged mechanical ventilation, multiple organ failure). Management of common bacterial
infections and noninfectious complications is described in other chapters.
a. Diagnostic tests for influenza should be obtained and empiric antiviral treatment of influenza
initiated during seasonal outbreaks in critically ill patients presenting with acute febrile and
respiratory illness consistent with influenza. Severely ill patients with influenza may present with
hypothermia similar to other populations with sepsis. Accompanying severe hypoxemic respiratory
failure should heighten the concern for influenza.
b. Antiviral treatment (Table 3) should be initiated within 48 hours from onset of symptoms.
Hospitalized patients receiving antiviral therapy after 48 hours from symptom onset may benefit.
The CDC recommends that treatment be initiated even 5 days after symptoms in critically ill patients.
c. Neuraminidase inhibitors continue to be the mainstay of antiviral therapy for recent influenza A
and B strains. These agents inhibit viral neuraminidase, decreasing the release of post-replication
viral particles (virions) from infected cells, limiting spread to additional tissues. Neuraminidase
inhibition may also suppress the initiation of infection after acute inoculation.
i. Oseltamivir phosphate is an oral (capsule, powder for suspension) ethyl ester prodrug converted
through hepatic ester hydrolysis to the active form oseltamivir carboxylate. Oseltamivir can
be administered to critically ill patients by orogastric or nasogastric tube. An intravenous
formulation of oseltamivir has been tested in early clinical trials. Oseltamivir is the drug of
choice for hospitalized patients with severe influenza.
ii. Zanamivir is available on the market as an orally inhaled drug delivered by a Diskhaler device.
Intravenous zanamivir was once available for patients with severe influenza through clinical
trial participation or emergency investigational new drug through the manufacturer. However,
this program has been discontinued for countries outside the European Union (internal
communication).
iii. Peramivir is an intravenous neuraminidase inhibitor with activity against influenza A and
B. Peramivir was approved in December 2014 for use in patients 18 years or older with
acute uncomplicated influenza. Peramivir is administered as a single 15-minute intravenous
infusion. Data are limited for peramivir in critically ill patients.

ACCP Updates in Therapeutics® 2022: Critical Care Pharmacy Preparatory Review and Recertification Course

793
Infectious Diseases I

d. Adamantanes inhibit the replication of influenza A viruses by preventing viral assembly.


Adamantanes also interfere with the function of the transmembrane domain of the viral M2
protein, preventing release of viral particles into host cells.
e. Baloxavir is an endonuclease inhibitor approved by the FDA in 2018 for the management of
uncomplicated influenza in individuals older than 12 years. Baloxavir inhibits endonuclease of
influenza virus A and B strains, limiting the activity of the polymerase acidic protein responsible
for viral gene transcription and virus replication. Data are limited on its safety and efficacy in
patients with severe influenza.
f. Susceptibility of influenza to available antiviral agents varies from year to year and between
strains and subtypes. Local surveillance of susceptibility patterns at the beginning and throughout
a respective season is imperative to provide appropriate therapy.
i. In the past three seasons, 100% of tested influenza A (2009 H1N1 and H3) and B viruses were
susceptible to oseltamivir, peramivir, and zanamivir.
ii. High levels of resistance to the adamantanes (amantadine and rimantadine) have persisted
among circulating influenza A viruses. The adamantanes are not effective against influenza B
viruses; therefore, they are not viable options for the management of contemporary influenza.

Table 3. Influenza-Specific Pharmacotherapy


Drug Class;
Half-life Treatment Adverse
Agent Influenza Bioavailability Comments
(hr) Dosage Effects
Activity
75 mg twice a
day for 5 daysa; Dosage
up to 150 mg adjustment
twice a day for renal
has been used impairment may
in patients be necessary;
Neuraminidase Orally or with severe Nausea, patients
Oseltamivir
inhibitor; OG/NG tube: 6–10 influenza but is vomiting, requiring
phosphate
A and B > 75% not associated delirium continuous
with improved renal
outcomes replacement
and thus is therapy should
not currently receive normal
recommended dosing
by the CDC

ACCP Updates in Therapeutics® 2022: Critical Care Pharmacy Preparatory Review and Recertification Course

794
Infectious Diseases I

Table 3. Influenza-Specific Pharmacotherapy (continued)


Influenza Half-life Treatment Adverse
Agent Bioavailability Comments
Activity (hr) Dosage Effects
Dosage
adjustment
recommended
in patients
Neuraminidase
with creatinine
inhibitor; Diarrhea,
clearance
A and B; 600 mg once for hypersensitiv-
Peramivir IV: 100% 20 below 50 mL/
limited clinical 15 minutes ity reactions,
min; should be
trial experience delirium
administered
for Influenza B
after dialysis
strains
in patients
receiving
hemodialysis
Not
recommended
in patients
with chronic
lung disease
Allergic
or severe
reactions,
Neuraminidase influenza;
Inhaled: Up to 10 mg twice a diarrhea,
Zanamivir inhibitor; 2.5–5 not available
17% day for 5 daysa nausea,
A and B for delivery
headache,
through
dizziness
mechanical
ventilator
circuit
because of
bronchospasm
Tablet, oral: Weight 40–79.9
Generally
Prodrug con- 79% kg: 40 mg orally
minimal (< 5% 93% protein
Polymerase verted almost (53%–96%); once; weight ≥
of patients): binding;
acidic completely primary 80 kg: 80 mg
Baloxavir Diarrhea, limited data for
endonuclease to active hepatic orally once; no
marboxil bronchitis, hospitalized
inhibitor; A baloxavir; metabolism information
nausea, patients with
and B bioavailability (80% of available for
sinusitis, severe influenza
not formally dose) feeding tube
headache
established administration
Nausea,
dizziness,
High resistance
insomnia,
Adamantane; 100 mg twice to current
Amantadine 70%–100% 15–17 lower seizure
A only a day influenza A
threshold,
strains
anticholinergic
effects

ACCP Updates in Therapeutics® 2022: Critical Care Pharmacy Preparatory Review and Recertification Course

795
Infectious Diseases I

Table 3. Influenza-Specific Pharmacotherapy (continued)


Influenza Half-life Treatment Adverse
Agent Bioavailability Comments
Activity (hr) Dosage Effects
High resistance
Dizziness,
to current
nausea,
Adamantane; 100 mg twice influenza
Rimantadine > 90% 24–35 vomiting;
A only a day A strains;
anticholinergic
limited market
effects
availability
a
Longer durations of up to 14 days may be needed for severely ill patients.
IV = intravenous(ly); NG = nasogastric; OG = orogastric.

2. Response to antiviral therapy should be assessed throughout treatment using clinical signs and
symptoms of infection. Development of oseltamivir resistance during therapy has been reported, but
this is rare. If suggested through local surveillance, therapy should be switched to zanamivir by the
appropriate administration route. Improvement in infectious and noninfectious complications may be
delayed, despite resolution of primary influenza infection. ARDS, in particular, may persist for days
beyond primary influenza.

Patient Cases

7. T.Y., a 32-year-old woman who has had flulike symptoms for the past 72 hours, presents to the ED from
home with severe fatigue, shortness of breath, and rigors. T.Y. has a heart rate of 120 beats/minute, mean
arterial pressure 70 mm Hg, respiratory rate 24 breaths/minute, temperature 102.7°F (39.3°C), and arte-
rial oxygen saturation (SaO2) of 85% on room air. Because it is in the middle of influenza season (high
prevalence of influenza A and B), a nasal swab is done and sent for rapid diagnostic testing for suspected
influenza infection. Shortly thereafter, T.Y. is intubated for severe respiratory failure and admitted to the
MICU. In addition to antibiotic therapy for CAP, which would best be considered next?
A. None; the patient is outside the time window to effectively treat influenza.
B. Await rapid diagnostic test results before initiating influenza-specific therapy.
C. Give amantadine.
D. Give oseltamivir.

8. G.L. is a 25-year-old woman admitted to the medical ICU in late February this year with severe, acute
hypoxemic respiratory failure requiring mechanical ventilation. G.L. is hypotensive, requiring norepineph-
rine after fluid resuscitation, and has acute kidney injury. She is awaiting a renal consult for continuous
renal replacement therapy. A rapid diagnostic test of a nasal washing suggests influenza B, consistent with
this year’s local influenza epidemiology. A nasogastric feeding tube is placed when she is admitted to the
ICU. Which would be most appropriate to treat G.L.’s severe influenza?
A. Enteral amantadine.
B. Enteral oseltamivir.
C. Inhaled zanamivir.
D. Intravenous peramivir.

ACCP Updates in Therapeutics® 2022: Critical Care Pharmacy Preparatory Review and Recertification Course

796
Infectious Diseases I

IV.  CATHETER-ASSOCIATED URINARY TRACT INFECTIONS

A. Epidemiology
1. Urinary catheters are commonly used in critically ill patients, with documented use of 50%–80% of
adult critically ill patient-days. Urinary catheters most often used are short term (less than 30 days)
temporary indwelling or intermittent urethral catheters. Other catheters include suprapubic catheters or
external collection catheters (e.g., condom catheters). Presence of an indwelling urinary catheter is an
independent risk factor for urinary tract infection (UTI).
2. CAUTIs are the most common cause of infection in critically ill patients. In 2012, the incidence (cases
per 1000 catheter-days) of CAUTIs varied among acute care critically ill populations and ranged from
1.2 for medical/surgical patients to 4.7 for burn patients. Aggregated data from 2016 show an almost
50% decrease in CAUTI incidence since 2012, likely related to organized prevention programs and
implementation of best practices for catheter use.
3. CAUTIs account for 15%–21% of hospital-acquired bacteremias.
4. The attributable mortality for CAUTI is 0%–15%.

B. Definitions
1. CDC adult surveillance definitions
a. UTI – At least one of the following signs or symptoms: Fever (temperature greater than 100.4°F
[38°C]); suprapubic tenderness; or costovertebral angle pain or tenderness and one of the following:
i. A positive urine culture of 105 CFU/mL or more of no more than two species of microorganisms
ii. At least one of the following findings: Positive dipstick for leukocyte esterase and/or nitrite,
pyuria (urine specimen with at least 10 white blood cells/mm3 of unspun urine or greater than
5 white blood cells/high-power field of spun urine), or microorganisms seen on Gram stain of
unspun urine and a positive urine culture of 103–105 CFU/mL of no more than two species of
microorganisms
b. CAUTI – A UTI in which an indwelling urinary catheter was in place for greater than 2 days and
attributable to the catheter removed no more than 1 day before infection
2. IDSA adult definitions
a. Catheter-associated asymptomatic bacteruria – Patients with indwelling urethral, indwelling
suprapubic, or intermittent catheterization (or condom catheter in a man) is defined by the presence
of 105 CFU/mL or greater of one or more bacterial species in a single catheter urine specimen in a
patient without symptoms compatible with UTI
b. CAUTI – In patients with indwelling urethral, indwelling suprapubic, or intermittent catheterization,
CAUTI is defined by the presence of symptoms or signs compatible with a UTI with no other identified
source of infection, together with 103 CFU/mL or more of one bacterial species in a single catheter
urine specimen or in a midstream-voided urine specimen from a patient whose urethral, suprapubic,
or condom catheter has been removed within the previous 48 hours. Lower colony counts are more
likely to represent significant bacteriuria in a symptomatic person than in an asymptomatic person.

C. Etiology
1. Most pathogens causing CAUTIs are acquired from the external environment, including the urethra, the
catheter collection system, and local skin flora. Most short-term catheter CAUTIs are monomicrobial.
Longer duration of an indwelling catheter is associated with the formation of biofilms within the
catheter and related system, which can promote polymicrobial infections.

ACCP Updates in Therapeutics® 2022: Critical Care Pharmacy Preparatory Review and Recertification Course

797
Infectious Diseases I

2. E. coli is the most prevalent pathogen causing CAUTIs; however, because of a wider distribution of
pathogens compared with non–catheter-associated UTIs, E. coli accounts for only about one-third of
CAUTIs. Additional bacterial pathogens include other enteric gram-negative bacilli (e.g., Klebsiella
spp.; Proteus spp.; Enterobacter spp.), non–lactose-fermenting gram-negative bacilli (e.g., P.
aeruginosa), and gram-positive cocci (e.g., Enterococcus spp.; MSSA; MRSA; MRSE). Candida spp.
may be involved in up to one-third of CAUTIs.
3. Similar to other health care–associated infections, causative pathogens are associated with local
pathogen patterns, pathogen-specific risk factors, and patient severity of illness.

D. Prevention
1. CAUTI is considered a preventable complication. The NHSN, using CDC definitions, provides
population-specific event rates for institution surveillance and performance benchmarks.
2. The most effective way to reduce the incidence of catheter-associated asymptomatic bacteruria and
catheter-associated UTI is to reduce the use of urinary catheterization by restricting its use to patients
who have clear indications and by removing the catheter as soon as it is no longer needed.
3. The foundation for preventing CAUTIs includes training and education, proper aseptic insertion
techniques, and active surveillance and performance improvement systems.
4. Recommended best practices for preventing CAUTIs have been proposed and endorsed by IDSA,
SHEA, and the Joint Commission. These evidence-based recommendations are categorized as basic
practices for all acute care hospitals or special practices wherein basic practices are less than effective at
reducing CAUTI rates. The entire document is available (Infect Control Hosp Epidemiol 2014;35:464-
79). Major recommendations include:
a. Basic practices
i. Minimize use and duration of indwelling catheters.
ii. Provide comprehensive education to and ensure competency for all involved with insertion,
care, and maintenance of urinary catheters.
iii. Use of a systematic process or checklist at the time of insertion to ensure adherence to proper
insertion technique
iv. Handwashing during insertion, care, and maintenance of the catheter
v. Proper care and maintenance of indwelling catheter and collection system
b. Special practices
i. Daily systematic assessment of continued need for indwelling catheter
ii. Development and implementation of a protocol to manage postoperative urinary retention

E. Diagnosis
1. Although clinical signs and symptoms are the mainstay for differentiating CAUTIs from catheter-
associated asymptomatic bacteruria, they are not specific for CAUTI. These include fever, rigors, altered
mental status, malaise, or lethargy with no other identified cause. Presence of flank pain, costovertebral
angle tenderness, acute hematuria, and pelvic discomfort may be more specific, but these are difficult
to assess in many critically ill patients.
2. Urine culture should be obtained in all critically ill patients with signs and symptoms of CAUTI.
Sampling should be done through the catheter port using aseptic technique. Catheters in place for
longer than 2 weeks should be replaced and a urine sample obtained from the port of the newly placed
catheter. Samples from the catheter collection system (e.g., catheter bag) should be avoided because of
the potential for colonization.
3. Urinalysis should be obtained in patients with a suspected CAUTI. Pyuria without signs and symptoms
does not indicate CAUTI; however, the absence of pyuria in a symptomatic patient suggests a diagnosis
other than CAUTI.

ACCP Updates in Therapeutics® 2022: Critical Care Pharmacy Preparatory Review and Recertification Course

798
Infectious Diseases I

F. Treatment
1. A urinary culture should be obtained before initiation of antimicrobial therapy.
2. If an indwelling catheter has been in place for longer than 2 weeks, the catheter should be removed or,
if still indicated, replaced to hasten resolution of symptoms.
3. Similar to other health care–acquired infections, empiric antimicrobial therapy should be based on local
pathogen prevalence, pathogen-specific risk factors (e.g., MDRO risk factors), previously identified
pathogens, previous antimicrobial exposure, and local antibiotic susceptibility.
4. Empiric antimicrobial therapy should be de-escalated according to the identified pathogen(s) and
respective antimicrobial susceptibility on final urine culture results. Empiric antimicrobial therapy
should be discontinued in patients without CAUTI or other sources of infection.
5. Definitive antimicrobial therapy should be based on final antimicrobial susceptibility results and
presence of concomitant infection(s). Catheter irrigation (i.e., bladder washings) is not recommended.
6. Shorter duration of antimicrobial therapy should be considered depending on clinical response.
Regardless of catheter removal, duration of antimicrobial therapy should be 7 days for patients with
resolution of signs and symptoms within 72 hours of appropriate antimicrobial therapy and up to 14
days in patients with resolution after 72 hours.
7. Patients with persistent signs and symptoms of CAUTI should receive a urologic workup to assess for
abscess or other causes of relapse.

Patient Case

Questions 9 and 10 pertain to the following case.


P.H. is a 67-year-old woman admitted from a long-term care facility to the MICU for respiratory failure secondary
to severe pneumonia. P.H. is initiated on vancomycin and levofloxacin and has a urethral catheter placed in the
emergency department. On ICU day 6, P.H. has a new fever with a temperature of 101.9°F (38.9°C) and an
elevated WBC to 18 x 103 cells/mm3, despite an initial clinical response to pneumonia therapy. Blood and urine
cultures are sent. Urinalysis reveals many bacteria and greater than 10 white blood cells/mm3.

9. Which intervention is most appropriate in P.H.?


A. Await final culture results before changing the current antibiotic regimen.
B. Continue pneumonia therapy for an extended treatment duration.
C. Initiate empiric antibiotic therapy for a suspected catheter-associated urinary tract infection (CAUTI).
D. Insert a new urinary catheter and resend a urinalysis to confirm CAUTI.

10. Given P.H.’s risk factors for multidrug-resistant pathogens, which listing of pathogen would most likely be
associated with a CAUTI?
A. E. coli, P. aeruginosa, C. albicans.
B. Enterococcus spp., C. albicans, S. maltophilia.
C. MRSA, Enterococcus spp., C. albicans.
D. P. aeruginosa, Enterococcus spp., MRSA.

ACCP Updates in Therapeutics® 2022: Critical Care Pharmacy Preparatory Review and Recertification Course

799
Infectious Diseases I

V.  COMPLICATED INTRA-ABDOMINAL INFECTION

A. Epidemiology
1. Complicated intra-abdominal infection spans prehospital and in-hospital dispositions and a variety of
pathogenic processes involving several organ systems.
2. The incidence of complicated intra-abdominal infection is difficult to estimate, given the breadth of
illness. Appendicitis is the most prevalent cause of complicated intra-abdominal infection. Recent
controlled trials and large international observational sepsis studies report abdominal infection as the
second or third most common source of sepsis, accounting for 21% of sepsis and 30% of septic shock
cases.
3. Complicated intra-abdominal infection is the second most common cause of mortality in critically ill
patients. Crude mortality for primary intra-abdominal infection is almost 30%, whereas rates exceed
50% in patients with secondary intra-abdominal infection. Those presenting with sepsis caused by
intra-abdominal infection have crude mortality above 40%.

B. Definitions
1. The IDSA and Surgical Infection Society jointly published guidelines for the management of intra-
abdominal infection in 2010. In 2017, the Surgical Infection Society updated these guidelines, including
management for complicated intra-abdominal infection, which are reflected throughout this section.
2. A complicated intra-abdominal infection is defined as infection that extends beyond the hollow viscus
of origin into the peritoneal space and is associated with either abscess formation or peritonitis. These
infections usually arise from spillage of viscus-related fluid and flora into the peritoneal cavity, causing
inflammation and injury to the peritoneal membrane. Retroperitoneal infections also are possible, but
these are related to the individual retroperitoneal organ rather than the peritoneum.
3. Peritonitis is described as primary, secondary, and tertiary.
a. Primary peritonitis, or spontaneous bacterial peritonitis, is peritonitis related to bacterial
translocation of proximal small bowel overgrowth and not peritoneal disruption or organ
perforation. Primary peritonitis is generally diffuse in nature.
b. Secondary peritonitis is caused by leakage of intraluminal fluid and microorganisms secondary to
macro- or microperforation of the GI tract. Secondary peritonitis can be diffuse or localized to an
organ, depending on the extent of peritoneal involvement. Causes of secondary peritonitis include
direct trauma, ischemia, thrombosis, ulceration, malignancy, and anastomotic leak.
c. Tertiary peritonitis is peritonitis that persists or recurs at least 48 hours after the management of
primary or secondary peritonitis. Tertiary peritonitis can represent a new intra-abdominal process
or host, anatomic, or therapeutic failure of treatment of primary or secondary peritonitis.

C. Etiology
1. Pathogens associated with complicated intra-abdominal infections and peritonitis are influenced by type/
cause of peritonitis, MDRO risk factors, previous antibiotic exposure, and patient-specific colonization.
Patients having health care–associated intra-abdominal infections more often tend to have antibiotic-
resistant, nosocomial pathogens compared with patients having community-acquired infection.
2. Primary peritonitis is usually monomicrobial and associated with translocation of organisms across the
diaphragm or proximal small bowel. The most prevalent organisms include S. pneumoniae, E. coli, and
Klebsiella spp.
3. Secondary peritonitis is typically polymicrobial related to the origin of GI tract leakage.

ACCP Updates in Therapeutics® 2022: Critical Care Pharmacy Preparatory Review and Recertification Course

800
Infectious Diseases I

a. Gastric and duodenal secretions are usually sterile or with limited inocula of gram-positive bacteria
and Candida spp.
b. Proximal small bowel is densely populated with aerobic gram-negative bacilli including E. coli,
Klebsiella spp., Proteus spp., and Enterobacter spp., as well as populations of aerobic gram-positive
bacteria such as S. aureus, streptococci, and enterococci.
c. The distal small bowel and the large bowel are populated with proximal small bowel flora in
addition to anaerobic gram-negative and gram-positive organisms, including Bacteroides fragilis
and Clostridioides spp. (usually non–C. difficile).
4. Tertiary peritonitis includes core organisms of primary and secondary peritonitis further compounded by
the influence of management strategies, including malnutrition, anatomic disruption, and antimicrobial
therapy.

D. Diagnosis
1. The diagnosis of complicated intra-abdominal infection relies on assessment of clinical signs and
symptoms, differentiation from other causes of infection, and radiographic evaluation.
2. Rapid-onset abdominal pain and tenderness with signs of peritoneal irritation on physical examination
are the most common presenting signs and symptoms. Additional symptoms of anorexia, abdominal
distention, nausea, or vomiting, with or without fever, tachycardia, or tachypnea. These are often
difficult to assess in critically ill patients; therefore, intra-abdominal infection should be considered in
patients with unexplained new-onset organ dysfunction and sepsis.
3. Radiographic evaluation is commonly performed using abdominal radiographs, ultrasonography, and
CT, including contrast CT to assess for vascular thrombosis. Contrast studies of postoperative drains or
fistulae may also help assess anastomotic integrity.
4. Culture of intra-abdominal fluid (e.g., ascitic fluid in patients with cirrhosis; intraoperative culture
of abscess fluid) associated with the primary source should be obtained in moderately to severely ill
patients. Gram stain results should be used to help guide empiric antimicrobial therapy. Pathogens
identified on final culture should guide definitive antimicrobial therapy.

E. Management and Treatment


1. Resuscitation for hemodynamic instability should be initiated and managed according to the Surviving
Sepsis Campaign guidelines.
2. Achieving primary source control of ongoing peritoneal contamination by operative diversion or
resection is recommended for patients with diffuse peritonitis. Patients with focal peritonitis should
undergo percutaneous abscess and fluid drainage.
3. Empiric antimicrobial therapy should be initiated in patients thought to have complicated intra-
abdominal infections.
a. Empiric antimicrobial regimens should be based on the source/location of intra-abdominal
infection, community-acquired or health care–associated disposition of infection, severity of
infection, local pathogen trends and susceptibilities, MDRO risk factors (e.g., hospitalization for
greater than 48 hours during current admission or in the previous 90 days; recent broad-spectrum
antimicrobial therapy; infection developing greater than 48 hours after initial source control; home
wound care or dialysis within preceding 90 days), and patient-specific colonization patterns.
b. Individual antimicrobial agents should be dosed according to available pharmacokinetic and
pharmacodynamic principles to optimize efficacy and limit toxicity.
c. Community-acquired, mild to moderate severity

ACCP Updates in Therapeutics® 2022: Critical Care Pharmacy Preparatory Review and Recertification Course

801
Infectious Diseases I

i. Routine culture is not recommended.


ii. Empiric antibiotic therapy should include agents active against aerobic and facultative enteric
gram-negative bacilli and enteric streptococci. Obligate anaerobic therapy should be initiated
in patients with distal small bowel, appendiceal, or colonic sources. Concerns with emergence
of antibiotic resistance may limit the utility of narrower-spectrum agents (e.g., cefazolin;
cefoxitin). Anti-enterococcal and antipseudomonal therapies are not recommended.
(a) Ceftriaxone or cefotaxime plus metronidazole (generally considered first line)
(b) Cefuroxime plus metronidazole
(c) Ertapenem
(d) Moxifloxacin alone; levofloxacin or ciprofloxacin plus metronidazole
(e) Tigecycline
(1) Pooled results from approval trials across all infections suggests increased mortality
in patients treated with tigecycline versus active comparator regimens. The cause of
this increase has not been established.
(2) FDA-approved label warns that the increase in all-cause mortality should be
considered when selecting among treatment options.
d. Community acquired, high severity defined as severe physiologic disturbance (e.g., septic shock);
advanced age; immunocompromised state; delay in, or high likelihood of failure to achieve,
primary source control

i. Routine culture of intra-abdominal fluid is recommended as available.


ii. Empiric antimicrobial therapy should be broadened to include MDROs, enterococci, and
obligate anaerobes. Agents active against MRSA are not recommended empirically. Although
fluoroquinolones are recommended, emergence of resistant E. coli and P. aeruginosa is a
concern.
(a) Broad-spectrum carbapenem (e.g., imipenem, meropenem)
(b) Cefepime or ceftazidime plus metronidazole (may need additional anti-enterococcal
agent)
(c) Ciprofloxacin or levofloxacin plus metronidazole (may need additional anti-enterococcal
agent)
(d) Piperacillin/tazobactam
e. Health care associated
i. Routine culture of intra-abdominal fluid is recommended as available.
ii. Broad-spectrum empiric antimicrobial therapy should include coverage for MDR Gram
negative bacilli (e.g., P. aeruginosa) and obligate anaerobes. Depending on empiric antibiotic
susceptibility rates (i.e., local antibiogram), combination therapy against aerobic gram-
negative bacilli with aminoglycoside, or, if MDROs are prevalent, ceftolozane-tazobactam
and/or polymyxin.
(a) Broad-spectrum carbapenem (e.g., imipenem, meropenem with or without anti-
enterococcal therapy)
(b) Cefepime or ceftazidime plus metronidazole
(c) Piperacillin/tazobactam
iii. Anti-enterococcal therapy should be considered in patients with recent exposure to broad-
spectrum antimicrobial therapy, septic shock, and those documented colonization with
enterococci.
iv. Anti-MRSA therapy should be considered in patients known to be colonized with MRSA or
with MDRO risk factors, including advanced age, co-morbid medical conditions, previous
hospitalization or surgery, and significant recent exposure to antibiotic agents.

ACCP Updates in Therapeutics® 2022: Critical Care Pharmacy Preparatory Review and Recertification Course

802
Infectious Diseases I

v. Antifungal therapy should be added to the regimen of patients with yeast on Gram stain, recent
evidence of heavy colonization, surgically treated pancreatitis, prolonged broad-spectrum
antibiotic therapy, or critically ill patients with an upper gastrointestinal source. Fluconazole
is the drug of choice for fluconazole-susceptible strains. Echinocandins should be used first
line in critically ill patients until final culture results are available.
vi. Empiric antimicrobial therapy should be de-escalated to final culture results and related
antimicrobial susceptibilities.
4. Definitive antimicrobial therapy for complicated intra-abdominal infection should be continued for
no more than 4 days in patients who have adequate source control. Duration of antimicrobial therapy
in patients without definitive source control should be no more than 5-7 days, depending on clinical
response. Patients with delayed or incomplete clinical response to antimicrobial therapy should be
reassessed for additional source control intervention.
5. Considerations for short-term prophylactic courses no longer than 24 hours include:
a. Acute gastric or jejunal perforation in absence of malignancy or acid-reducing pharmacotherapy
with adequate source control within 24 hours
b. Traumatic or iatrogenic bowel injuries repaired within 12 hours of injury
c. Acute appendicitis without perforation or abscess

Patient Cases

11. D.L. is a 69-year-old woman admitted from home to the SICU with severe acute abdominal pain in the left
lower quadrant. An abdominal CT reveals free air in the peritoneal cavity and evidence of distal ischemic
colitis. D.L. is taken urgently to the operating room, where she is noted to have gross contamination from
distal colonic perforation with marked peritonitis. After a partial colectomy and abdominal washout, she is
admitted to the SICU for continued resuscitation for septic shock. Which empiric antibiotic regimen would
be most appropriate for D.L.?
A. Ceftriaxone and vancomycin.
B. Ciprofloxacin and metronidazole.
C. Ertapenem.
D. Piperacillin/tazobactam.

12. K.D. is a 58-year-old man with severe peripheral vascular disease admitted from home to the SICU with
severe acute abdominal pain in the left lower quadrant. An emergency abdominal CT reveals free air in the
peritoneal cavity and evidence of distal small bowel ischemia. K.D. is taken emergently to the operating
room for an exploratory laparotomy, where he is noted to have gross peritoneal contamination with marked
peritonitis from a small bowel perforation. Intraoperative cultures were sent and are pending. After primary
repair of the distal small bowel and abdominal washout, K.D. returns to the SICU for continued resuscita-
tion for septic shock. Which empiric antibiotic regimen would be most appropriate for K.D.?
A. Piperacillin/tazobactam.
B. Ertapenem.
C. Ciprofloxacin and vancomycin.
D. Cefazolin, metronidazole, and fluconazole.

ACCP Updates in Therapeutics® 2022: Critical Care Pharmacy Preparatory Review and Recertification Course

803
Infectious Diseases I

VI.  ACUTE PANCREATITIS

A. Epidemiology
1. Acute pancreatitis is responsible for more than 200,000 acute care admissions annually in the United
States.
2. About 20% –30% of acute pancreatitis episodes are severe, have evidence of pancreatic necrosis, and
are associated with local and systemic complications (e.g., multiple organ failure). An Acute Physiology
and Chronic Health Evaluation II (APACHE II) score of 8 or higher and a Ranson’s criteria score
of 3 or greater, together with clinical presentation, have been used historically to categorize patients
as having severe acute pancreatitis. The more recent revised Atlanta classification categorizes severe
acute pancreatitis as the presence of single or multiple organ failure for 48 hours or more, whereas
the Determinant-Based Criteria (DBC) classification system defines it as persistent organ failure or
infected pancreatic or peripancreatic necrosis. The DBC adds a category of critical acute pancreatitis
for patients with persistent organ failure and infected necrosis.
3. Between 30% and 78% of patients with necrotizing pancreatitis will have concomitant organ failure,
with the highest incidence in patients having infected necrosis. Pancreatitis-associated systemic
sequelae and organ failure includes:
a. Systemic inflammatory response syndrome (SIRS)
b. Hypovolemic shock
c. Hypoxemia secondary to ARDS
d. Acute kidney injury
e. Gastrointestinal (GI) bleeding
f. Disseminated intravascular coagulation and severe metabolic disturbances can also occur.
4. Although most pancreatic necrosis is sterile and does not require treatment with antimicrobials, about
one-third of patients with necrotizing pancreatitis will have infected necrosis.
5. Overall, crude mortality for acute pancreatitis is 2%–9%; however, mortality rates for necrotizing and
infected necrotic pancreatitis with organ failure are as high as 44% and 62%, respectively.

B. Pathophysiology
1. Pancreatitis is a result of glandular autodigestion from excessive ductal and tissue exposure to amylase,
lipase, and protease caused by trypsin-related hyperstimulation, macro-ductal blockade, or micro-
ductal blockade.
2. The pathophysiology of acute pancreatitis can be organized into three phases:
a. Excessive activation or decreased inactivation of trypsin leading to activation of pancreatic
exocrine enzymes
b. Local inflammatory and immune response to pancreatic injury
c. Systemic inflammatory and immune response, including SIRS, hypovolemia, and ARDS
3. Common causes of acute pancreatitis include biliary obstruction (e.g., gallstones), direct toxicity (e.g.,
alcohol), trauma, surgery/biliary procedures, and drugs.

C. Definitions
1. Severe pancreatitis is defined as pancreatitis associated with hypovolemia, organ failure, or local
complications including necrosis, abscess, or pseudocyst. Hypovolemia may increase the risk of
pancreatic necrosis and intestinal ischemia because of tissue hypoperfusion.
2. Pancreatic necrosis is evidenced on CT scan as diffuse or focal areas of nonviable pancreatic tissue
often associated with peripancreatic fat necrosis. In general, greater than 30% of the pancreas should
be affected. Infected necrosis is defined as the presence of pathogenic microorganisms in the necrotic
tissue. The presence of retroperitoneal gas on CT scan may indicate infected pancreatitis in patients

ACCP Updates in Therapeutics® 2022: Critical Care Pharmacy Preparatory Review and Recertification Course

804
Infectious Diseases I

with severe acute pancreatitis. The timing of infected necrosis varies, but infected necrosis usually
peaks 2–4 weeks from the onset of acute pancreatitis.
3. Pancreatic pseudocyst is a non-epithelialized wall containing pancreatic excretions caused by acute or
chronic pancreatitis or pancreatic trauma. Pseudocysts are usually sterile.
4. Pancreatic abscess is an infected pseudocyst or liquefaction of pancreatic necrosis that becomes infected.
5. Additional late complications of peripancreatic necrosis include secondary pancreatic infection of
previously acute necrotic collections during the first 4 weeks after presentation and walled-off necrosis,
which may lead to pancreatic abscess that matures after 4 weeks or more.

D. Diagnosis
1. The diagnosis of acute pancreatitis requires two of the following three features:
a. Acute and constant epigastric or right upper quadrant abdominal pain with or without nausea and
vomiting;
b. Serum amylase and/or lipase greater than 3 times the upper limit of normal; and
c. Characteristic findings of acute pancreatitis on CT scan or magnetic resonance imaging.
2. Patients with severe pancreatitis may present with SIRS, hypovolemia, and new-onset organ failure,
including hypotension.

E. Management and Treatment


1. Management of severe acute pancreatitis includes acute pain management, fluid resuscitation, supportive
care of systemic complications and organ failure, and nutrition support. Surgical debridement of
infected necrosis or drainage of pancreatic abscess should be considered.
a. Initial fluid management should be goal-directed titration of isotonic, crystalloid intravenous fluids
using clinical and biochemical targets of perfusion.
b. Patients should receive nutrition within 24 hours. Patients unable to eat orally should receive enteral
nutrition by nasogastric or nasojejunal access over parenteral nutrition.
2. Antibiotic therapy for acute pancreatitis is considered empiric, definitive, or prophylactic.
a. Empiric antibiotic therapy is indicated in patients with suspected or confirmed infected necrosis
or pancreatic abscess.
i. Patients with suspected infected necrosis or pancreatic abscess should undergo radiographically
(e.g., ultrasonography, CT) guided drainage with culture of recovered material/fluid. Common
organisms associated with infected necrosis or pancreatic abscess include:
(a) E. coli
(b) Klebsiella spp.
(c) Enterobacter spp.
(d) Proteus spp.
(e) Streptococci
ii. The antibiotics of choice include:
(a) Broad-spectrum carbapenems (e.g., imipenem; meropenem)
(b) Fluoroquinolone plus metronidazole
(c) Third-generation cephalosporin plus metronidazole
iii. Antifungal therapy should be added to antibiotic therapy if yeast is identified on culture Gram
stain. Fluconazole is recommended unless there is a high suspicion for fluconazole-resistant
fungi.
iv. Empiric antibiotics should be discontinued if pancreatic culture is negative, indicating sterile
pancreatitis. Patients with persistent SIRS should undergo repeat imaging and drainage of
identified abscess or fluid.

ACCP Updates in Therapeutics® 2022: Critical Care Pharmacy Preparatory Review and Recertification Course

805
Infectious Diseases I

v. If infection is confirmed, antibiotic therapy should be de-escalated toward the identified


pathogen(s) according to antibiotic susceptibility.
vi. Definitive antibiotic therapy for confirmed infection should be continued for up to 14 days.
b. Prophylactic antimicrobial therapy in patients with sterile necrotizing pancreatitis is controversial.
Although small pilot investigations suggest benefit, a large, noninferiority, placebo-controlled
trial suggested no benefit with prophylactic meropenem. Current recommendations do not support
routine use of prophylactic antibiotic or antifungal therapy in patients with sterile necrotizing
pancreatitis. (Gastroenterology 2018;154:1096-101; Am J Gastroenterol 2013;108:1400-15)
3. Ongoing assessment of resolution of SIRS and pancreatitis-associated organ failure is imperative.
Serial assessment of serum amylase or lipase has limited value over clinical assessment and physical
examination. Elevation of serum amylase or lipase for several weeks, however, should heighten concern
for persistent pancreatic/peripancreatic inflammation, blockage of the pancreatic duct, or development
of a pseudocyst.

Patient Cases

13. R.P. is a 43-year-old man with a history of chronic alcohol abuse who presents to the emergency department
from home with acute, severe epigastric abdominal pain; heart rate 142 beats/minute; MAP 62 mm Hg;
WBC 24 x 103 cells/mm3; and serum lipase 1527 U/L. R.P. receives 3 L of normal saline intravenously over
60 minutes. An immediate abdominal CT scan is remarkable for significant pancreatic edema and greater
than 30% necrosis of the pancreas. There is no evidence of an acute fluid collection. Which antimicrobial
regimen would be most appropriate for R.P.?
A. Ceftriaxone and vancomycin.
B. Ciprofloxacin and metronidazole.
C. Meropenem.
D. None; prophylactic antibiotic therapy is not indicated in acute pancreatitis.

14. T.J. is a 27-year-old man who is admitted to the MICU from an outside hospital with alcohol-induced
acute pancreatitis and associated respiratory failure, SIRS, and blood pressure 100/55 mm Hg. Admission
abdominal CT shows severe pancreatitis with about 30% necrosis and a large focal fluid collection requiring
guided drainage. A fluid sample is sent for a Gram stain, which reveals many gram-negative rods and yeast.
Which antimicrobial regimen would best be initiated in T.J.?
A. None – low suspicion for infected pancreatitis.
B. Ciprofloxacin and metronidazole.
C. Imipenem.
D. Meropenem and fluconazole.

VII.  CLOSTRIDIOIDES DIFFICILE INFECTION

A. Epidemiology
1. C. difficile (also known as Clostridioides difficile) is a spore-forming, anaerobic, gram-positive bacillus.
Stool carriage of C. difficile has been found in up to 26% of acute care patients.
2. C. difficile is transmitted person to person through the fecal-oral route. The capability to become
dormant in spores increases the potential for environmental spread.

ACCP Updates in Therapeutics® 2022: Critical Care Pharmacy Preparatory Review and Recertification Course

806
Infectious Diseases I

3. C. difficile is pathogenic through the production and expression of two primary toxins, C. difficile toxin
A (TcdA) and C. difficile toxin B (TcdB). North America has seen a recent emergence of strains with
increased virulence (e.g., BI/NAP1 strain) through increased production of toxins A and B, production
of additional toxins, and enhanced sporification.
4. CDI is responsible for almost 30% of antibiotic-associated diarrhea and is the most common cause of
infectious diarrhea in health care settings. The estimated incidence of CDI is 3–10 cases per 10,000
patient-days. Community-acquired CDI has increased in prevalence.
5. The clinical manifestations of CDI range from asymptomatic carrier to CDI spanning mild or moderate
diarrhea to fulminant and sometimes fatal pseudomembranous colitis, toxic megacolon, or colonic
perforation. Post-colectomy small bowel enteritis and rectal pouchitis related to CDI have been reported.
Additional complications of severe CDI include:
a. SIRS
b. Hypovolemia
c. Electrolyte disturbances
d. Sepsis and septic shock
e. Multiple organ failure
6. CDI is associated with medical costs greater than $3 billion per year.
7. Attributable mortality from CDI is estimated to be below 10%. However, crude mortality is as high
as 75% in patients presenting with septic shock, colonic perforation, or toxic megacolon. Subtotal
colectomy in patients with a severe CDI is associated with an in-hospital mortality rate of up to 42%.

B. Definitions
1. CDI is defined as the presence of symptoms (e.g., diarrhea) and a stool test result positive for C.
difficile toxin, toxigenic C. difficile (e.g., DNA amplification detecting toxin-coding genes), or
pseudomembranous colitis on colonoscopic examination. The most recent guidelines for CDI diagnosis
and management include those from the American College of Gastroenterology in 2021 (Kelly CR, et
al. Am J Gastroenterol 2021;116:1124 -47), IDSA/SHEA guidelines from 2017, and a focused update
from IDSA/SHEA in 2021 related to management of CDI. The ACG 2021 guidelines are updated from
the 2013 version to harmonize CDI severity categories with IDSA/SHEA while expanding a focus on
diagnostic methods and management of patients with concomitant chronic inflammatory bowel disease.
2. Severe CDI according to expert-based guidelines is defined as stated earlier, plus:
a. IDSA 2017 and 2021; ACG 2021
i. WBC 15 x 103 cells/mm3 or greater
ii. Serum creatinine (SCr) 1.5 times or greater than the premorbid level
3. Fulminant (IDSA 2017 and 2021; ACG 2021) CDI is defined as:
a. Severe CDI plus one of the following:
i. Hypotension or evidence of shock
ii. Ileus or
iii. Megacolon
4. Recurrence is defined as the relapse of a recent infection or a reinfection after definitive therapy.
5. Additional categorizations of CDI are used for infection control surveillance and institutional/ICU
quality review. These are related to the locations of C. difficile acquisition (e.g., community acquired or
health care associated) and onset of symptoms (e.g., community onset vs. health care onset).

C. Risk Factors
1. Antibiotic therapy is the most important risk factor.
a. All antibiotic classes have been associated with CDI.

ACCP Updates in Therapeutics® 2022: Critical Care Pharmacy Preparatory Review and Recertification Course

807
Infectious Diseases I

b. Highest-risk (listed by decreasing risk) antibiotic classes include clindamycin, fluoroquinolones,


cephalosporins/carbapenems, and penicillins.
2. Gastric acid–suppressing pharmacotherapy, including proton pump inhibitors and histamine-2 receptor
antagonists
3. Age older than 65 years
4. Duration of hospitalization
5. Cancer chemotherapy
6. GI surgery
7. Previous CDI

D. Prevention – There are two global strategies to prevent CDI:


1. Decrease the risk of acquiring C. difficile.
a. Staff, patient, family, caregiver education
b. Hand hygiene to remove C. difficile spores through non–alcohol-based handwashing using soap or
chlorhexidine gluconate and water
c. Contact isolation, including full-barrier precautions (gown and gloves), single-occupancy room,
and the cohorting of patients with a CDI
d. Limit reuse or between-patient sharing, and terminal clean patient care–related equipment (e.g.,
digital thermometers, point-of-care blood glucose machines, dietary trays, intravenous infusion
pumps) and rooms (previous antibiotic use by the previous bed occupant may increase the risk of
CDI in the current occupying patient).
e. Environmental decontamination using bleach-containing cleaning solution
2. Avoid or address reversible risk factors.
a. Limit overuse of, and discontinue unnecessary, antibiotic therapy.
b. Decrease duration of hospital stay.
c. Discontinue unnecessary gastric acid–reducing pharmacotherapy.

E. Diagnosis
1. Diagnosis of CDI is based on clinical and laboratory findings.
2. Clinical findings include presence of diarrhea, defined as passage of three or more unformed stools
within 24 consecutive hours.
a. Rarely, a symptomatic patient will present with ileus and colonic distension with minimal or no
diarrhea.
b. Patients with cecal CDI or right-sided CDI colitis may have formed stools.
3. Laboratory findings include stool sample positive for toxigenic C. difficile, C. difficile toxin, or
colonoscopic or histopathologic findings showing pseudomembranous colitis. Available strategies for
detecting toxin-producing C. difficile include:
a. Testing for C. difficile should only be performed on unformed stool unless patients have ileus.
Identifying the ideal testing strategy remains difficult. Institution-specific decisions for which
test(s) to use should be evidence based and collaborative across interested parties. Moreover,
institutions should consider creating local interdisciplinary guidelines for C. difficile testing to
avoid positive results in colonized patients without infection.
b. Stool culture with detection of a toxigenic isolate through identification of neutralizable toxin
activity is considered the gold standard test. However, this process could take up to 9 days, limiting
its clinical utility.

ACCP Updates in Therapeutics® 2022: Critical Care Pharmacy Preparatory Review and Recertification Course

808
Infectious Diseases I

c. Enzyme immunoassay (EIA) tests for C. difficile toxins A and B are rapid and widely available;
however, poor sensitivity may limit their utility. Obtaining serial samples has been used to increase
sensitivity. Sensitivity may also be increased using a two-step process with initial EIA to detect
the C. difficile antigen glutamate dehydrogenase and a follow-up stool culture with detection of the
toxigenic isolate.
d. PCR testing is rapid, sensitive, and specific. Widespread availability may be limited.
e. The nucleic acid amplification test (NAAT) involves an illumigene C. difficile assay, which uses
loop-mediated isothermal DNA amplification to detect a specific genetic region responsible for
coding toxins A and B.
4. Recommended laboratory diagnostic pathways differ regarding whether an institution-specific protocol
for stool sample quality based on CDI likelihood is predefined. Briefly, the absence of predefined
criteria requires a multistep laboratory process that includes a toxin test (e.g., NAAT plus toxin assay)
rather than a NAAT alone. In contrast, the presence of predefined criteria for stool quality and clinical
presentation suggestive of CDI permits the use of NAAT alone or a multistep process over a strategy
using a toxin assay alone.
5. The same diagnostic criteria are used for recurrent CDI.

F. Management and Treatment


1. Removal of potential cause(s), as appropriate (e.g., discontinuation of associated antibiotic therapy)
2. Assessment of disease severity (mild or moderate vs. severe) and whether episode is initial or recurrent
3. Evaluation of need for surgical intervention
a. Subtotal colectomy with rectal preservation should be considered for severely ill patients.
b. Alternative colon-sparing operative surgical strategies have been described.
4. Antibiotic therapy targeted against C. difficile (Table 4)
a. Samples for diagnostic testing for C. difficile should be obtained before empiric antibiotic therapy
is initiated. Initiation of empiric antibiotic therapy before final test results should be based on
clinical assessment.
b. Antibiotic choices and respective routes of administration should be based on severity of CDI
and ability to achieve relevant intraluminal antibiotic concentrations relative to CDI. Special
considerations include:
i. Fidaxomicin, an oral, limited-bioavailability macrolide antibiotic, is indicated for treatment
of CDI. The 2021 IDSA/SHEA focused guideline update recommends (conditional
recommendation; moderate certainty of evidence) fidaxomicin as first-line therapy for first
episode, non-fulminant CDI. This recommendation is supported by systematic review and
meta-analysis of four studies noting increased sustained response to therapy at 4 weeks with
fidaxomicin compared to oral/enteral vancomycin. Initial cure, all-cause mortality, and drug
adverse events were similar between fidaxomicin and vancomycin. Data supporting fidaxomicin
use in critically ill patients are limited to small representative sample sizes and retrospective
or observational studies.
ii. Oral or enteral vancomycin continues to be an alternative for an initial episode of CDI. The
optimal enteral vancomycin dosage in critically ill patients is unknown. Although ACG and
IDSA guidelines recommend enteral vancomycin dosages of 125 mg four times daily for
uncomplicated/nonfulminant CDI (Table 4), some practices consider dosages of 250 mg four
times daily if there is lack of response or if there is concern for subclinical GI dysmotility (e.g.,
incomplete enteral nutrition tolerance, elevated gastric residuals, abdominal distention) (Int J
Antimicrob Agents 2013;42:553-8; Clin Infect Dis 2015;61:934-41).

ACCP Updates in Therapeutics® 2022: Critical Care Pharmacy Preparatory Review and Recertification Course

809
Infectious Diseases I

iii. For enema volumes in patients requiring rectal instillation of vancomycin, location of CDI-
affected area(s) and risk of colonic perforation should be considered. Patients receiving
vancomycin enemas may need a cuffed rectal delivery device/tube to facilitate retention.
iv. Patients with CDI-related colitis and proximal colonic diversion (i.e., no continuity with oral
or gastric route) may require rectal instillation of vancomycin enema.
v. Fecal transplantation has improved outcomes compared with oral vancomycin in noncritically
ill patients with recurrent CDI. Logistical challenges with product availability or stool
sample collection, fecal delivery and administration, and risk of colonization with pathogenic
organisms (e.g., E. coli) should be considered.
vi. There are no definitive recommendations for duration of CDI antibiotic therapy or prevention
of recurrence when non-CDI antibiotic therapy is continued concurrently.
c. Response to therapy should be assessed by evaluating clinical signs and symptoms, including
resolution of diarrhea, laboratory abnormalities, sepsis, and related organ failure. Results of stool
testing for C. difficile in patients with resolution of disease do not predict recurrence.

Table 4. Treatment Options for Clostridioides difficile Infection


Clinical IDSA/SHEA
ACG 2013 IDSA/SHEA 2018 ACG 2021
Definition 2021 Focused Update
Vancomycin 125
Fidaxomicin 200
mg PO/NG/FT 4 Fidaxomicin 200 mg BID
mg PO/enteral BID
times daily for 10 for 10 days
for 10 days (strong
days
recommendation;
—OR— Alternative: Vancomycin
moderate-quality
Fidaxomicin 200 125 mg PO/NG/FT 4
Metronidazole evidence)
Initial mg BID for 10 times daily for 10 days
500 mg PO/ —OR—
episode, days
NG/FT TID Vancomycin 125
nonsevere Alternative for nonsevere
for 10 days mg PO/NG/FT
Alternative: infection: Metronidazole
4 times daily for
Metronidazole 500 mg PO/NG/FT TID
10 days (strong
500 mg PO/NG/ for 10–14 days (delayed
recommendation; low-
FT TID for 10–14 response)
quality evidence)
days (delayed
response)
Vancomycin 125 mg
PO/NG/FT 4 times
Vancomycin 125
daily for 10 days (strong
mg PO/NG/FT 4
Vancomycin recommendation; low-
times daily for 10
Initial 125 mg PO/ quality evidence)
days
episode, NG/FT 4 —OR—
—OR—
severe times daily for Fidaxomicin 200 mg
Fidaxomicin
10 days PO/enteral BID for
200 mg BID for 10
10 days (conditional
days
recommendation; very
low-quality evidence)

ACCP Updates in Therapeutics® 2022: Critical Care Pharmacy Preparatory Review and Recertification Course

810
Infectious Diseases I

Table 4. Treatment Options for Clostridioides difficile Infection (continued)

Clinical IDSA/SHEA
ACG 2013 IDSA/SHEA 2018 ACG 2021
Definition 2021 Focused Update
Vancomycin
125 mg PO/
NG/FT 4
times daily + Vancomycin 500 Same as IDSA/SHEA
metronidazole mg PO/NG/FT Vancomycin 500 mg PO/ 2021
Initial 500 mg IV 4 times daily + NG/FT 4 times daily +
episode, TID metronidazole 500 metronidazole 500 mg IV Surgery: Suggest
severe mg IV TID TID total colectomy with
complicated Ileus: end ileostomy and
(ACG)/ Vancomycin Ileus: Add rectal Ileus: Add rectal stapled rectal stump
fulminant 500 mg PO/ vancomycin 500 vancomycin 500 mg/ NS or diverting loop
(IDSA) NG/FT and per mg/ NS 100 mL 100 mL instilled for 1 hr ileostomy with colonic
rectum (NS instilled for 1 hr 4 4 times daily lavage and intraluminal
500 mL) + times daily vancomycin
metronidazole
500 mg IV
TID
Vancomycin 125
mg 4 times daily
Recommend
for 10 days if
fidaxomicin for first
metronidazole was
recurrence after initial
used for initial
Fidaxomicin 200 mg course of vancomycin
episode
given BID for 10 days— or metronidazole
—OR—
OR—BID for 5 days, then (conditional
Prolonged tapered
once every other day for recommendation,
and pulsed
20 days moderate-quality
vancomycin
evidence)
regimen if
Same regimen Alternative: Vancomycin
standard regimen
as used for PO in tapered and pulsed Suggest tapering/pulsed
was used for
initial episode, regimen dose vancomycin first
First initial episode; for
unless severe; —OR— recurrence after initial
recurrence example: 125 mg
if severe, Vancomycin 125 mg course of fidaxomicin,
4 times daily for
vancomycin given 4 times daily PO for vancomycin, or
10–14 days, BID
should be used 10 days metronidazole (strong
for 1 wk, once
recommendation, very
daily for 1 wk,
Adjunctive therapy: low-quality evidence)
then every 2 or 3
Bezlotoxumab 10 mg/
days for 2–8 wk
kg IV once during Suggest bezlotoxumab
—OR—
administration of above for high risk of
Fidaxomicin
antibiotics recurrence (conditional
200 mg BID
recommendation,
for 10 days if
moderate-quality
vancomycin was
evidence)
used for initial
episode

ACCP Updates in Therapeutics® 2022: Critical Care Pharmacy Preparatory Review and Recertification Course

811
Infectious Diseases I

Table 4. Treatment Options for Clostridioides difficile Infection (continued)

Clinical IDSA/SHEA
ACG 2013 IDSA/SHEA 2018 ACG 2021
Definition 2021 Focused Update
Fidaxomicin 200 mg
given BID for 10 days—
OR—BID for 5 days, then
once every other day for
20 days
Vancomycin in
—OR— Recommend
tapered and pulsed
Vancomycin PO in fecal microbiota
regimen
tapered and pulsed transplant (FMT)
—OR—
regimen to prevent further
Vancomycin 125
—OR— recurrences (strong
mg PO 4 times
Vancomycin 125 mg 4 recommendation,
daily for 10 days,
Vancomycin times daily PO for 10 moderate-quality
then rifaximin 400
Second PO, then days, then rifaximin 400 evidence)
mg TID for 20
recurrence 28-day dosage mg TID for 20 days
days
taper —OR— Suggest bezlotoxumab
—OR—
Fecal microbiota for high risk of
Fidaxomicin 200
transplantation (preferably recurrence (conditional
mg BID for 10
after 3 antibiotic courses recommendation,
days —OR—
are attempted and moderate-quality
Fecal
completed) evidence)
microbiota
transplantation
Adjunctive therapy:
Bezlotoxumab 10 mg/
kg IV once during
administration of above
antibiotics
ACG = American College of Gastroenterology; BID = twice daily; FT = feeding tube; IDSA = Infectious Diseases Society of America; IV = intravenously; NS = normal
saline; PO = orally; SHEA = Society for Healthcare Epidemiology of America; TID = 3 times daily.
Information from: Johnson S, Lavergne V, Skinner AM, et al. Clinical practice guideline by the Infectious Diseases Society of America (IDSA) and Society for
Healthcare Epidemiology of America (SHEA): 2021 focused update guidelines on management of Clostridioides difficile infection in adults. Clin Inf Dis 2021. Available
at www.idsociety.org/globalassets/idsa/practice-guidelines/cdi-2021-focused-update.pdf. Accessed September 1, 2021; McDonald LC, Gerding DN, Johnson S, et al.
Clinical practice guidelines for Clostridium difficile infection in adults and children: 2017 update by the Infectious Diseases Society of America (IDSA) and Society for
Healthcare Epidemiology of America (SHEA). Clin Infect Dis 2018;66:987-94. Surawicz CM, Brandt LJ, Binion DG, et al. Guidelines for Diagnosis, Treatment, and
Prevention of Clostridium difficile Infections. Am J Gastroenterol 2013;108:478–98.; Kelly CR, Fischer M, Allegretti JP, et al. ACG Clinical Guidelines: Prevention,
Diagnosis, and Treatment of Clostridioides difficile Infections, Am J Gastroenterol 2021; 116: 1124-1147.

ACCP Updates in Therapeutics® 2022: Critical Care Pharmacy Preparatory Review and Recertification Course

812
Infectious Diseases I

Patient Cases

15. K.L. is a 57-year-old man admitted to the MICU with diffuse abdominal pain, temperature 101.9°F
(38.8°C), WBC 27 x 103 cells/mm3, heart rate 125 beats/minute, and mean arterial pressure 57 mm Hg. An
abdominal CT scan reveals a large amount of intestinal air, suggestive of ileus and moderate transverse and
sigmoid colonic inflammation. On review of the patient’s history, it is learned that K.L. recently completed
a 4-week course of broad-spectrum antibiotic therapy for a postoperative osteomyelitis after repair of right
comminuted femur fracture, increasing the suggestion of CDI. Which regimen would be best for empiric
management of a suspected CDI in K.L.?
A. Metronidazole 500 mg intravenously every 8 hours.
B. Vancomycin 500 mg orally/by nasogastric tube/feeding tube four times daily, metronidazole 500 mg
intravenously every 8 hours, and intracolonic vancomycin 500 mg instilled every 8 hours.
C. Metronidazole 500 mg orally or by nasogastric tube every 8 hours and vancomycin 250 mg orally or by
nasogastric tube every 6 hours.
D. Vancomycin 250 mg orally or by nasogastric tube every 6 hours.

16. E.A. is a 79-year-old man admitted to the MICU for severe pneumonia. On day 5 of treatment with broad-
spectrum antibiotic therapy despite negative culture, E.A. is noted to have several liquid bowel movements
requiring a rectal pouch (around 1.6 L of stool in past 24 hours), a low-grade fever, elevation of WBC at 27
x 103 cells/mm3, heart rate 124 beats/minute, MAP 67 mm Hg, lactate 1.6 mmol/L, and SCr 1.5 mg/dL. A
stool sample is sent for a suspected C. difficile infection (CDI). Which regimen would be the best empiric
management of a suspected CDI in E.A.?
A. Metronidazole 500 mg intravenously every 8 hours and intracolonic vancomycin 500 mg instilled
every 8 hours.
B. Metronidazole 500 mg intravenously every 8 hours.
C. Metronidazole 500 mg orally or by nasogastric tube every 8 hours and vancomycin 125 mg orally or by
nasogastric tube every 6 hours.
D. Vancomycin 125 mg orally or by nasogastric tube every 6 hours.

VIII.  WOUND INFECTION

A. Epidemiology
1. Postoperative wound infection is the most common health care–associated infection, affecting up to
5% of inpatient surgery patients. There are 150,000–300,000 cases of postoperative wound infections
annually in the United States.
2. Most postoperative wound infections are mild or moderate in severity. Major complications associated
with severe postoperative wound infections include wound dehiscence, reoperation, sepsis, and
necrotizing fasciitis.
3. Patients with postoperative wound infection have more than a 2-fold higher risk of death compared with
those without infection. Crude mortality in patients with streptococcal necrotizing fasciitis and shock
(e.g., streptococcal toxic shock) is high, ranging from 30% to 70%.

ACCP Updates in Therapeutics® 2022: Critical Care Pharmacy Preparatory Review and Recertification Course

813
Infectious Diseases I

B. Definitions
1. Superficial incisional: Infection involving only the skin or subcutaneous tissue of the incision
2. Deep incisional: Infection involving fascia and/or muscular layers
a. Deep incision primary: Wound infection in the primary incision in a patient who has had an
operation with one or more incisions
b. Deep incision secondary: Wound infection in a secondary incision in a patient who has had an
operation with more than one incision
c. Necrotizing fasciitis is an aggressive, deep incisional infection tracking along the superficial fascia,
which consists of the tissues between the skin and the underlying muscles. Necrotizing fasciitis
often results in major tissue destruction. Fournier gangrene is a variant of necrotizing fasciitis
involving the scrotum and penis or vulva.
3. Organ or space: Infection involving any space or organ, opened or manipulated during the procedure,
excluding skin incision, fascia, or muscle layers.

C. Etiology
1. Pathogens causing postoperative wound infections are often related to local flora present on the skin at
the time of incision and flora associated with organs/tissues involved in the operative procedure.
2. Prevalence of drug-resistant strains (e.g., MRSA, multidrug-resistant gram-negative bacilli) depends on
local patterns of infection and patient colonization.
a. The most common bacterial pathogens causing postoperative wound infection are skin flora,
including staphylococci and streptococci.
b. Bacterial pathogens related to anatomic location of the operation:
i. Upper GI tract (gastric, biliary, proximal small intestine)
(a) Biliary: Aerobic and anaerobic gram-negative and gram-positive organisms
(b) Non-biliary: Enteric, aerobic gram-negative bacilli
ii. Lower GI tract (distal small bowel; colon): Mixed gram-positive and gram-negative flora,
facultative and anaerobic
iii. Female genitalia: Mixed gram-positive and gram-negative flora, facultative and anaerobic
iv. Axilla: Aerobic gram-negative organisms
v. Perineum: Aerobic gram-negative and mixed anaerobic organisms
vi. Respiratory: Aerobic gram-positive and gram-negative organisms
c. Necrotizing fasciitis
i. Most infections are monomicrobial, caused predominantly by group A streptococci
(Streptococcus pyogenes), S. aureus, and anaerobic streptococci (e.g., Peptostreptococcus).
Non–C. difficile clostridia (e.g., Clostridium septicum) and Aeromonas hydrophilia also are
associated with monomicrobial infection. Additional common pathogens in patients presenting
with Fournier gangrene include E. coli and Bacteroides spp.
ii. Polymicrobial infections usually involve a broad range of pathogens, including aerobic and
anaerobic gram-positive and gram-negative organisms. S. pyogenes should be empirically
suspected in all necrotizing infections. Likelihood of polymicrobial infection is increased if
associated with:
(a) Decubitus ulcers
(b) Injection sites in illicit drug users
(c) Intestinal operation
(d) Penetrating abdominal trauma
(e) Perianal abscess
(f) Spread from genitalia (i.e., Fournier gangrene)

ACCP Updates in Therapeutics® 2022: Critical Care Pharmacy Preparatory Review and Recertification Course

814
Infectious Diseases I

D. Prevention
1. Using evidence-based guidelines can prevent up to 60% of postoperative wound infections. Wound
infections after elective operation are considered preventable and are reportable health care–associated
infections.
2. Many organizations and agencies promote prevention and monitor the prevalence of postoperative wound
infection, including the CDC, the Surgical Care Improvement Project (SCIP), the Joint Commission,
and the Centers for Medicare & Medicaid Services.
3. Recommended strategies are basic or special approaches.
a. Basic approaches include:
i. Administer preoperative weight-based antibiotic therapy according to operative site and level
of expected operative field contamination. Timing of antibiotic therapy should maximize
blood and tissue concentrations at the time of incision. Antibiotics should be redosed every 2
half-lives for prolonged procedures. Antibiotic duration should be limited to 24 hours unless
there is evidence of peritonitis or active infection during the operative procedure.
ii. Avoid use of hair removal. If necessary, use clippers or depilatory agent.
iii. Maintain glycemic control in immediate postoperative period; goal should be to avoid glucose
above 180 mg/dL.
iv. Avoid perioperative hypothermia.
v. Optimize tissue oxygenation by maintaining adequate perfusion and oxygen delivery.
vi. Interdisciplinary staff education, use of procedural checklists, and infection surveillance
b. Special approaches include:
i. Preoperative screening for S. aureus and consideration of decontamination in patients
undergoing orthopedic or cardiothoracic procedures
ii. Aseptic, intraoperative wound lavage

E. Diagnosis
1. Postoperative wound infections most commonly occur 48 hours after the procedure. Fever in the first
48 hours is usually idiopathic or from noninfectious causes.
2. Wounds should be physically examined serially until healed. Purulent material should be collected
aseptically and sent for Gram stain and culture. Cultures in patients with suspected deep tissue
infections should be obtained from deep tissues with concomitant blood cultures.
3. Signs and symptoms of superficial incisional postoperative wound infection include:
a. Purulent incisional drainage
b. Local pain or tenderness, swelling, and erythema after the incision is opened
c. Positive culture of purulent drainage
4. Necrotizing fasciitis should be suspected if the following are present:
a. Severe pain that seems disproportional to the appearance of the wound
b. Failure to respond to initial antibiotic therapy
c. Hard, wooden feel of the subcutaneous tissue, often extending beyond the area of affected skin
d. Crepitus on physical examination or radiographic (radiograph, CT scan) finding, indicating gas in
subcutaneous tissues
e. Skin necrosis or ecchymoses
f. Sepsis or septic shock

ACCP Updates in Therapeutics® 2022: Critical Care Pharmacy Preparatory Review and Recertification Course

815
Infectious Diseases I

F. Management and Treatment


1. Opening of the incision, evacuation of the infected material, and continued dressing changes are the
foundation of treatment for confirmed postoperative wound infections.
2. Antibiotic therapy targeted against likely pathogens should be initiated in patients with systemic signs
and symptoms or suspected deep tissue infection.
3. Superficial incisional infection
a. Erythema and induration less than 5 cm and minimal systemic signs of infection (no fever, no
leukocytosis, and tachycardia):
i. Serial dressing changes
ii. No antibiotic therapy necessary
iii. Continue to assess wound for resolution or progression.
b. Erythema and induration greater than 5 cm, fever, leukocytosis, and tachycardia:
i. Open suture line.
ii. Initiate empiric antibiotic therapy targeted against operative site–related suspected pathogens.
Examples include:
(a) Extremity, head, neck, or trunk site: Cefazolin or vancomycin if MRSA suspected
(b) GI, genitalia, or perineum site: Cephalosporin or levofloxacin plus metronidazole;
ertapenem
iii. Adjust antibiotics according to culture results. Continue for up to 48 hours or until infection
is resolved.
iv. Serial dressing changes
v. Continue to assess wound for resolution or progression.
4. Necrotizing fasciitis
a. Surgical debridement of necrotic tissue serially (i.e., every 24–48 hours) until no further need for
debridement
b. Antibiotic therapy is empiric or definitive.
i. Empiric antibiotic therapy should be initiated as early as possible.
ii. Polymicrobial infection should be suspected, with empiric therapy that is active against aerobic
and anaerobic gram-positive and gram-negative pathogens
(a) Vancomycin or linezolid, plus:
(b) Piperacillin/tazobactam, broad-spectrum carbapenem, or cefepime plus metronidazole
(c) Consider clindamycin to decrease pathogenic toxin and cytokine production if S. pyogenes
is suspected.
iii. Empiric antibiotic therapy should be de-escalated according to final culture results.
iv. Definitive antibiotic therapy should be based on final culture results and antibiotic susceptibility.
Infection caused by S. pyogenes should be treated with an active β-lactam or vancomycin (in
severe penicillin allergy) plus clindamycin. Use of intravenous immunoglobulin (IVIG) is
controversial. Limited evidence suggests a shorter time to no further need for debridement but
no effect on mortality.
v. Antibiotic therapy should be continued until surgical debridement is no longer necessary and
until resolution of infection-related signs and symptoms.
5. Organ or space infection: See individual chapters, sections, or guidelines for managing organ/space-
specific infection (e.g., pancreatitis, intra-abdominal infection; genitourinary tract).

ACCP Updates in Therapeutics® 2022: Critical Care Pharmacy Preparatory Review and Recertification Course

816
Infectious Diseases I

Patient Case

Questions 17 and 18 pertain to the following case.


R.J. is a 57-year-old man admitted to the SICU after a bowel resection following an acute bowel obstruction.
On postoperative day 3, R.J. has worsening tachycardia and decreased urinary output together with a maximum
temperature of 102.9°F (39.4°C) and a WBC of 18.2 x 103 cells/mm3. R.J. reports worsening pain around his
large abdominal surgical wound despite minimal erythema. On opening a few sutures of the wound, pus is
expressed on examination, and crepitus is noted in the area surrounding the wound. R.J. is emergently taken
to the operating room for exploration and is noted to require significant debridement of necrotic subcutaneous
tissue, including the involved fascia. R.J.’s wound is left open with a temporary gauze dressing.

17. Which empiric antimicrobial regimen is most appropriate for R.J.?


A. Ceftaroline and vancomycin.
B. Clindamycin, penicillin G, and vancomycin.
C. Piperacillin/tazobactam, clindamycin, and vancomycin.
D. Vancomycin.

18. Intraoperative tissue culture from the first debridement of R.J.’s wound shows many gram-positive cocci in
chains and moderate non–lactose-fermenting gram-negative bacilli. R.J. is to undergo serial intraoperative
debridements of necrotic tissue. Given the concern for S. pyogenes from the Gram stain, which pharmaco-
therapeutic intervention would be most appropriate for definitive treatment of R.J.’s necrotizing fasciitis?
A. Add synergistic gentamicin.
B. Continue or initiate adjunctive clindamycin for toxin production.
C. Ensure vancomycin is part of the regimen for concerns of S. pyogenes resistance.
D. Give intravenous immunoglobulin (IVIG).

IX.  STEVENS-JOHNSON SYNDROME AND TOXIC EPIDERMAL NECROLYSIS

A. Epidemiology
1. Severe cutaneous reactions and related syndromes are unpredictable and rare. The primary causes
of these injuries include drugs, dysregulated immune response, and acute infection. Stevens-Johnson
syndrome (SJS), toxic epidermal necrolysis (TEN also referred to as Lyell’s Syndrome), and drug
hypersensitivity syndrome, or drug rash with eosinophilia and systemic symptoms (DRESS) are the
most common presentations. There is a suggestion of genetic influence on the occurrence of SJS and
TEN.
2. SJS and TEN are the most severe of reactions, representing different points on a similar spectrum of
cutaneous injury involving epidermolysis or separation of the epidermis from the dermis.
3. Similar to thermal injury, the TBSA affected is used to describe the extent of cutaneous injury.
According to contemporary reports from U.S. burn centers, the mean TBSA involvement for patients
with TEN is greater than 60%.
4. The incidence of severe cutaneous reactions is difficult to estimate. The highest rates reported approach
20%; however, definitions include mild to moderate reactions. The estimated incidence of SJS and TEN
is 1 in 500,000–1,000,000 population. Patients infected with HIV may have a higher incidence.

ACCP Updates in Therapeutics® 2022: Critical Care Pharmacy Preparatory Review and Recertification Course

817
Infectious Diseases I

5. The severity of clinical presentation is associated with the extent of tissue and mucosal injury and
necrosis. Most patients have a prodromal fever and malaise preceding cutaneous symptoms. Clinical
presentation usually includes fever, SIRS, hypotension from cytokine-mediated vasodilation, and mild
to moderate hypovolemia from volume depletion and third spacing. Bleeding may also be present,
depending on the extent of mucosal injury.
6. Health care–associated or nosocomial complications, including pneumonia, CAUTIs, CLABSIs, and
malnutrition, are common in severely injured patients.
7. SJS and TEN are life-threatening reactions. Average crude mortality associated with SJS/TEN is 25%–
55% and can be as high as 90%.

B. Definitions
1. SJS and TEN are part of the same disease process, differing in severity. Common features of SJS and
TEN include cutaneous erythema, progressive blistering, epidermolysis, and mucosal erosions.
2. The most widely accepted classification system for SJS and TEN was proposed by Bastuji-Garin et al.
This system includes five categories:
a. Bullous erythema multiforme: Epidermal detachment involving less than 10% TBSA and localized
typical targets or raised atypical targets
b. SJS: Epidermal detachment involving less than 10% TBSA and widespread erythematous or
purpuric macules or flat atypical targets
c. SJS/TEN overlap: Epidermal detachment involving 10%–30% TBSA and widespread purpuric
macules or flat atypical targets
d. TEN with spots: Epidermal detachment involving greater than 30% TBSA and widespread purpuric
macules or flat atypical targets
e. TEN without spots: Large sheets of epidermal detachment involving greater than 10% TBSA
without purpuric macules or target lesions

C. Etiology
1. Drugs are the most common cause of SJS and TEN and are implicated in more than 90%–95% of cases.
More than 200 medications have been reported as causing SJS and TEN. The most common agents
implicated are sulfonamide antibiotics and aromatic anticonvulsants (phenytoin, phenobarbital, and
carbamazepine). Other agents/classes include:
a. Abacavir
b. Allopurinol
c. β-Lactam antibiotics
d. Lamotrigine
e. Nevirapine
f. Nonsteroidal anti-inflammatory drugs, particularly the oxicams
g. Quinolones, particularly ciprofloxacin
h. Tetracyclines
2. Vaccinations also have been associated with SJS and TEN, including measles, mumps, and rubella
(MMR).
3. Exposure to industrial chemicals and fumigants
4. Infection with Mycoplasma pneumoniae

ACCP Updates in Therapeutics® 2022: Critical Care Pharmacy Preparatory Review and Recertification Course

818
Infectious Diseases I

D. Diagnosis
1. Primary clinical signs/symptoms associated with SJS and TEN are fever and malaise, followed by
cutaneous blisters and erosive mucosal lesions of the mouth, lips, eyes, and genital area. The distribution
of cutaneous lesions is predominantly central, with mucosal involvement of usually at least two sites.
Lesions consist of widespread, flat atypical targets or purpuric macules. TBSA is used to differentiate
SJS from TEN and to categorize the severity of cutaneous and mucosal involvement. TBSA is calculated
by the following:
a. Arms – 9% each
b. Head and neck – 9%
c. Legs – 18% each
d. Perineum – 1%
e. Trunk – Anterior 18%; posterior 18%
2. The diagnosis of SJS and TEN is confirmed by histopathologic analysis of lesional tissue and is
corroborated with clinical presentation. Early lesions show scattered necrotic keratinocytes in the
epidermis, whereas late-stage lesions reveal confluent full-thickness epidermal necrosis, which leads to
formation of subepidermal bullae.
a. SCORTEN is a severity-of-illness system designed to predict mortality for TEN. It is computed
within the first 24 hours of presentation and on day 3 using the sum of seven objective clinical
variables (each item present is worth 1 point):
i. Age older than 40 years
ii. Heart rate greater than 120 beats/minute
iii. Presence of cancer or hematologic malignancy
iv. Epidermal detachment greater than 10% TBSA on day 1
v. Blood urea nitrogen greater than 28 mg/dL
vi. Glucose greater than 252 mg/dL
vii. Serum bicarbonate less than 20 mEq/L
b. Mortality prediction increases sharply with each additional point, starting at 3% for 0 or 1 point and
reaching 90% for 5 or more points. SCORTEN mortality estimates are often used as benchmark
rates to assess noncontrolled pharmacotherapy studies.

E. Management and Treatment


1. Identification, discontinuation, and avoidance of likely or suspected causes are imperative. Causative
agents with a long half-life should be identified and strategies to expedite removal considered.
2. Transfer to ICU, preferably at a certified burn center.
3. Overt assessment of mucus membranes to prevent extension of injury and related sequelae. This
includes the respiratory tract, eyes, and GI tract.
4. The cornerstones of management of SJS and TEN are:
a. Resuscitation and supportive care (Recent supportive care guidelines are outlined in J Am Acad
Dermatol 2020;82:1553-67)
i. Goal-directed fluid resuscitation should be initiated immediately to maintain:
(a) Mean arterial blood pressure above 65 mm Hg
(b) Central venous pressure 8–12 mm Hg
(c) Urine output 0.5–1 mL/kg/hour
(d) Central venous oxygen saturation above 70%
ii. Support respiratory function with respiratory therapy and continual assessment for intubation,
as appropriate
iii. Avoid using skin to anchor devices and catheters.
iv. Physical and occupational therapy when appropriate

ACCP Updates in Therapeutics® 2022: Critical Care Pharmacy Preparatory Review and Recertification Course

819
Infectious Diseases I

b. Limit debridement of necrotic epidermis. Best practice recommendations support leaving detached
epidermis in place as a biologic dressing along with coverage of affected areas with artificial or
biologic dressing. This may be done serially because progression of affected areas may occur.
Topical emollients (e.g., petroleum jelly) should be applied to entire epidermis.
c. Management of extracutaneous injuries
i. Ocular involvement
(a) Adequate ocular lubrication
(b) Consideration of topical, preservative-free ophthalmic corticosteroid drops
(c) Treatment of corneal fluorescein or ulceration
ii. Oral involvement
(a) Maintain lip barrier integrity (e.g., white paraffin)
(b) Consider antiseptic oral rinse (e.g., chlorhexidine)
(c) Consider topical corticosteroid rinse (e.g., betamethasone)
d. Nutrition support (see related chapter)
e. Avoidance and treatment of infectious complications
i. Implement infection prevention best practices, including minimizing unnecessary devices and
procedures related to health care–associated infection.
ii. Prophylactic antibiotic therapy is not recommended for SJS and TEN.
iii. Empiric antibiotic therapy should be carefully chosen and reserved for suspected infection, as
evidenced by signs and symptoms of sepsis or site-specific infection. Continuation of antibiotic
therapy should be reserved for confirmed infection, and duration should be limited according
to the specific infection.
5. Adjuvant therapies
a. Plasmapheresis – Support is derived from case series; thought to be generally safe and an effective
strategy to remove pathogenic, nondialyzable plasma factors, including some drugs, toxins,
metabolites, antibodies, immune complexes, and disease-inducing cytokines
b. Immunomodulating therapy
i. Corticosteroids – Despite some evidence of benefit, use is controversial and not universally
recommended. More recent case reports suggest that high-dose pulse therapy during the
first 3 days of presentation decreases disease progression. Associated risks (e.g., infection;
hyperglycemia, poor wound healing) may outweigh benefits.
ii. Cyclosporine – Information from individual case series suggests benefit at a dose of 3 mg/kg/
day. There are no formal recommendations for routine use.
iii. Cyclophosphamide – Early case reports suggested benefit, but cyclophosphamide is not
recommended.
iv. Colony-stimulating factor – May be used in conjunction with cyclosporine in patients with
neutropenia and TEN
v. IVIG
(a) IVIG for SJS and TEN is controversial.
(b) In vitro data support that immunoglobulin G (IgG) antibodies against Fas-FasL proteins
may decrease keratinocyte apoptosis.
(c) Many retrospective single-group and cohort studies suggest benefit (usual dosage 1 g/
kg/day for 3 days) over SCORTEN estimated mortality rates and similar control groups,
respectively.
(d) Given the rare incidence and logistical difficulty of designing a multicenter prospective
study, available prospective studies have been small and single center. Results from these
studies, however, have shown no benefit to trends of worse outcome.
(e) A systematic review and meta-analysis of use in TEN patients found no benefit over
standard of care.

ACCP Updates in Therapeutics® 2022: Critical Care Pharmacy Preparatory Review and Recertification Course

820
Infectious Diseases I

(f) The decision to administer IVIG remains clinically supported by pathophysiology-


pharmacology interactions and observational data. Centers with expertise to care for
patients with TEN should assess the utility of IVIG and develop interdisciplinary guidance
for local use.
vi. Consensus guidelines from the United Kingdom recommend immunomodulating therapies
be used under the supervision of skin failure specialists in the context of a clinical study or
registry. In contrast, consensus guidelines from India recommend (grade B recommendation)
that low-dose IVIG be considered in the first 24–48 hours in patients with HIV, children, and
pregnant women in the first trimester.

Patient Case

Questions 19 and 20 pertain to the following case.


L.H. is a 23-year-old woman with a recently diagnosed uncomplicated urinary tract infection. Two days after
starting sulfamethoxazole/trimethoprim, L.H. presents to the burn ICU from an outside hospital with severe
systemic inflammatory response and 30% total body surface area (TBSA) epidermolysis of her upper arms and
back. L.H. reportedly presented to the outside hospital 6 hours earlier with only 10% TBSA involvement. New
lesions since admission to the burn ICU include erythema of her thighs and oral cavity with TBSA now at 40%,
suggestive of toxic epidermal necrolysis (TEN). L.H.’s SCORTEN score is 3.

19. Which would be the best initial pharmacotherapeutic intervention?


A. Crystalloid resuscitation.
B. Cyclophosphamide.
C. Empiric vancomycin for wound prophylaxis.
D. High-dose systemic corticosteroids.

20. L.H. continues to worsen, with a TBSA involvement now at 50%, with worsening oral cavity involvement
and progressing acute kidney injury. You and your team consider IVIG as the next pharmacotherapeutic
intervention. Which best describes the evidence-based role of IVIG in managing TEN?
A. IVIG should be administered before the patient’s condition progresses to TEN.
B. IVIG should be reserved for specialty centers with interdisciplinary consensus protocols to guide use.
C. Multicenter pivotal trials show that IVIG is most efficacious for patients with ocular involvement.
D. Meta-analyses of the available evidence support that IVIG is standard of care in managing TEN.

X.  NOVEL SEVERE ACUTE RESPIRATORY SYNDROME CORONAVIRUS 2

A. Epidemiology
1. In late December 2019, a novel coronavirus strain causing pneumonia in a human was first identified
in Wuhan, China. This virus, first labeled 2019-nCoV, is now called novel severe acute respiratory
syndrome coronavirus 2 (SARS-CoV-2).
2. As of January 3, 2022, there have been more than 290 million cases of SARS-CoV-2 worldwide, with
over 5.5 million associated deaths. More than 55 million cases and over 800,000 deaths have been
attributed to SARS-CoV-2 in the United States (https://coronavirus.jhu.edu/).
3. Illness related to SARS-CoV-2 is called coronavirus disease 2019 (COVID-19). COVID-19 was
categorized as a worldwide pandemic on March 11, 2020.

ACCP Updates in Therapeutics® 2022: Critical Care Pharmacy Preparatory Review and Recertification Course

821
Infectious Diseases I

4. COVID-19 is characterized by a broad spectrum of illnesses, from mild upper respiratory symptoms to
life-threatening respiratory failure, sepsis, and/or multiorgan failure.
5. In general, about 80% of patients with COVID-19 present with mild illness, and 15%–20% present with
more severe illness requiring hospitalization. Between 2% and 10% of patients with COVID-19 require
ICU admission, accounting for 30%–40% of patients hospitalized with COVID-19.
6. Risk factors for COVID-19–related hospitalization span several comorbidities, including hypertension,
diabetes, cardiovascular disease, COPD, obesity, and chronic kidney disease. Patients with comorbidi-
ties are at increased risk of hospitalization, ICU admission, and mortality.
a. The most common complication of COVID-19 in ICU patients is hypoxemic respiratory failure
from viral pneumonia, usually bilateral. Cytokine release syndrome or “cytokine storm” with mul-
tiorgan failure may be the primary presentation in some patients.
b. Between 25% and 90% of patients admitted to the ICU require MV.
c. Additional acute complications: Septic shock, liver dysfunction, bleeding and coagulation dysfunc-
tion, cytokine release syndrome, lymphopenia, and acute cerebrovascular disease
d. Late complications: Myocarditis, cardiomyopathy, and ventricular arrhythmias
7. In-hospital crude mortality for COVID-19 is generally 15%–60%.
a. Mortality approaches 40% in ICU patients with COVID-19.
b. Age-related in-hospital mortality ranges from less than 5% for individuals younger than 40 to great-
er than 60% for those older than 80.

B. Etiology
1. SARS-CoV-2 is a coronavirus; coronaviruses are relatively large, enveloped, single-stranded RNA vi-
ruses. Clinically, coronaviruses are usually associated with the common cold and mild to moderate
gastrointestinal illness. Common seasonal strains include coronaviruses 229E, HKU1, NL63, and OC43.
2. SARS-CoV-2, SARS-CoV (SARS), and Middle East respiratory syndrome (MERS) are the most recent
coronaviruses to cause severe respiratory infection.
3. SARS-CoV-2 predominantly targets nasal and bronchial epithelial cells and lower respiratory tract epi-
thelial pneumocytes.
4. SARS-CoV-2 pathogenicity is mediated through binding of the viral structural spike protein (S protein)
to membrane angiotensin-converting enzyme 2 (ACE2) receptors, predominant in alveolar type II cells.
Subsequently, serine protease type 2 transmembrane serine protease (TMPRSS2) cleaves the ACE2
receptor–S protein complex, leading to activation of the viral S protein and enhanced cellular uptake.
Once intracellular, SARS-CoV-2 RNA is incorporated into the host nucleus, and viral replication en-
sues. Activation of the ACE2 receptor may also implicate the contact pathway and complement system,
perhaps through an ACE2-mediated increase in bradykinin, in the pathophysiology of COVID-19. This
has been hypothesized to be the primary mechanism of cytokine release syndrome in patients with
COVID-19.
5. Mutations in SARS-CoV-2 continue to be reported.
a. One of the first reported variants was D614G, which has a modified S protein that may increase
cellular viral uptake. Many areas worldwide are concurrently experiencing multiple SARS-CoV-2
variants.
b. To date, the Delta variant (B.1.617.2) has demonstrated increased transmissibility (about 2-fold) and
pathogenicity compared with the wild-type SARS-CoV-2 strain.

C. Prevention
1. SARS-CoV-2 is primarily transmitted from human to human through respiratory secretions, most likely
through droplets. Transmission is also possible through direct surface contact, though this is less likely.
2. Physical distancing, handwashing, and airborne pathogen precautions (i.e., mask wearing) together with
contact precautions in medical environments are recommended to decrease transmission.

ACCP Updates in Therapeutics® 2022: Critical Care Pharmacy Preparatory Review and Recertification Course

822
Infectious Diseases I

3. Clinicians caring for patients undergoing aerosol-generating procedures (e.g., endotracheal intubation;
bronchoscopy) should don respirator masks together with eye protection and full-body gowning. Nega-
tive-pressure patient rooms are preferred during aerosol-generating procedures.
4. The incubation period for SARS-CoV-2 is estimated to be up to 14 days from exposure. Information on
onset and duration of viral shedding and infectious potential is still emerging and suggests variable time
intervals of 2–18 days from symptom onset. Individuals with symptoms likely have a higher potential
for transmitting the virus.
5. Several vaccines against SARS-CoV-2 are available. As of September 2021, Pfizer-BioNTech mRNA
vaccine has been FDA-approved for those age 16 years and older, while Moderna (mRNA) and Janssen-
J&J (DNA) vaccines are available under an FDA Emergency Use Authorization (EUA). Use of the
Pfizer-BioNTech vaccine in people age 12–15 years is also available under an FDA EUA. These and
other emerging vaccines are targeting the SARS-CoV-2 S protein – the complete protein, a component,
or the genetic material of such – as the immunogenic trigger.

D. Diagnosis
1. Primary COVID-19 symptoms in hospitalized patients include fever (80%), dry cough (75%), shortness
of breath (70%), fatigue (40%), myalgias (35%), and gastrointestinal symptoms (30%). The most com-
mon reason for hospitalization is shortness of breath or hypoxemia.
2. Laboratory abnormalities in hospitalized patients with COVID-19 include:
a. Lymphopenia (related to T-cell lysis from direct infection by SARS-CoV-2)
b. Elevated inflammatory markers (e.g., C-reactive protein, erythrocyte sedimentation rate, ferritin,
interleukin [IL]-6, tumor necrosis factor alpha). May resemble cytokine release syndrome
c. Coagulation abnormalities (e.g., prolonged prothrombin time, thrombocytopenia, elevated D-di-
mer, hypofibrinogenemia)
3. Radiographic abnormalities can vary, but patients with severe disease commonly present with bilateral,
lower lobe–predominant infiltrates on chest radiography. Chest CT can reveal bilateral, ground-glass
opacities with or without consolidation, usually depending on the time course of presentation.
4. The CDC recommends that nasopharynx samples be used to detect SARS-CoV-2 in patients presenting
with COVID-19 symptoms. Lower respiratory tract samples (e.g., endotracheal aspirate, BAL) have a
higher diagnostic yield, but concerns with aerosolization and lack of tolerability in those with severe
hypoxemia may limit their usefulness.
5. COVID-19 diagnosis is confirmed using RT-PCR.

E. Management and Treatment


1. Management priorities for critically ill patients with COVID-19 include supportive care of hypoxemic
respiratory failure and septic shock; immunomodulation of the inflammatory coagulation response, in-
cluding consideration for anticoagulation; and primary antiviral therapy.
2. New information is emerging on effective management strategies and therapies. Useful resources in-
clude the National Institutes of Health COVID-19 guidelines (www.covid19treatmentguidelines.nih.
gov/about-the-guidelines/table-of-contents/) and the Surviving Sepsis Campaign (Crit Care Med
2020;48:e440-e469).
a. Hypoxemic respiratory failure
i. Supplemental oxygen to maintain SaO2 above 92%–96%; high-flow nasal cannula preferred to
noninvasive positive pressure ventilation
ii. For patients who require MV: ARDSNet lung-protective strategy
(a) Low tidal volumes
(b) Appropriate plateau pressures
(c) Higher PEEP (positive end-expiratory pressure)

ACCP Updates in Therapeutics® 2022: Critical Care Pharmacy Preparatory Review and Recertification Course

823
Infectious Diseases I

(d) Intermittent or continuous (in those for whom intermittent dosing fails) neuromuscular
blockade
(e) Proning
iii. Extracorporeal membrane oxygenation (ECMO): Venovenous ECMO in patients with refrac-
tory hypoxemia despite optimizing ventilation, rescue therapies, and proning
b. Septic shock
i. Goal blood pressure: 60–65 mm Hg
ii. Fluid resuscitation: Conservative management strategy using balanced crystalloid solutions
iii. Vasopressors: Norepinephrine as first line; vasopressin or epinephrine as second line
iv. Inotropes: Dobutamine if cardiac dysfunction or persistent hypoperfusion despite fluid resus-
citation and norepinephrine
v. Corticosteroids: See Immunomodulation section in the text that follows.
c. Immunomodulatio
i. Corticosteroids
(a) Dexamethasone (or equivalent-dose prednisone, methylprednisolone, or hydrocortisone)
6 mg per day orally or intravenously for up to 10 days or until hospital discharge is
recommended for COVID-19 treatment in hospitalized patients requiring increased
amounts of supplemental oxygen or MV or ECMO.
(1) The largest corticosteroid study to date is the RECOVERY trial, a randomized, open-
label trial comparing dexamethasone 6 mg orally or intravenously once daily for up
to 10 days with usual care alone in 6425 patients (dexamethasone 2104; usual care
4321) hospitalized with COVID-19. At baseline, 3883 patients required supplemental
oxygen, and 1007 required MV.
(A) Dexamethasone decreased 28-day mortality (primary outcome) (dexamethasone
22.9% vs. usual care 25.7%; age-adjusted rate ratio 0.83; 95% CI, 0.75–0.93;
p<0.001).
(B) Results were similar among patients who required MV (29.3% vs. 41.4%; rate
ratio 0.64; 95% CI, 0.51–0.81) and those receiving supplemental oxygen (23.3%
vs. 26.2%; rate ratio 0.82; 95% CI, 0.72–0.94). There was no difference among
patients not requiring respiratory support.
(b) A meta-analysis of seven controlled trials encompassing 1703 critically ill patients
(requiring supplemental oxygen or MV) with COVID-19, including 1007 patients (15.7%
of end enrollment) enrolled early in RECOVERY, showed that corticosteroids were
associated with lower 28-day all-cause mortality compared with placebo or usual care
(OR 0.66 [95% CI, 0.53–0.82]; p<0.001).
ii. Other immunomodulators
(a) Toclizumab: Anti-IL-6 receptor monoclonal antibody (alternative class agent: sarilumab)
(1) The COVACTA trial (N Engl J Med 2021;384:1503-16) was a randomized, placebo-
controlled trial in 438 hospitalized adults with severe COVID-19 pneumonia.
(A) Patients were randomized 2:1 to either a single intravenous infusion of tocilizumab
(8 mg/kg actual body weight) or placebo; 25% of patients received a second dose
of tocilizumab.
(B) At enrollment, 37% of patients required MV and 22.4% received concomitant
corti-costeroids.
(C) No between-group differences were observed in clinical progression or 28-day
mor-tality (tocilizumab, 19.7% vs. placebo, 19.4%; weighted difference 0.3%;
95% CI, –7.6 to 8.2).

ACCP Updates in Therapeutics® 2022: Critical Care Pharmacy Preparatory Review and Recertification Course

824
Infectious Diseases I

(2) The REMAP-CAP trial (N Engl J Med 2021;384:1491-502) randomized 803 patients
within 24 hours of ICU admission (median 1.2 days from hospitalization) to open-label
tocilizumab 8 mg/kg actual body weight (max of 800 mg) (353 patients), sarilumab
400 mg (48 patients), or standard of care (402 patients).
(A) Most patients (552 [68.7%]) were free of MV and/or ECMO at enrollment; 76%
of patients received concomitant corticosteroids.
(B) Tocilizumab was associated with reduced in-hospital mortality (28% vs. 36%;
OR 1.64; 95% CI, 1.25–2.14) and higher median days free of respiratory and
cardio-vascular organ failure (10 days vs. 0 days; OR 1.64; 95% CI, 1.25–2.14).
(C) Tocilizumab was also associated with decreased progression to MV, ECMO, or
death (OR 0.59; 95% CI, 0.41–0.86).
(3) The RECOVERY trial (Lancet 2021;397:1637-45) was a randomized, controlled,
open-label study in 4116 hospitalized patients with COVID-19 demonstrating acute
hypoxia and evidence of systemic inflammation (i.e., C-reactive protein ≥75 mg/L).
(A) Patients were randomized 1:1 ratio to usual care vs. usual care plus tocilizumab
400 mg–800 mg intravenously once; 29% of tocilizumab patients received a
second dose at provider discretion.
(B) At enrollment, 82% of patients received concomitant corticosteroids and 13.8%
re-quired MV.
(C) Tocilizumab was associated with a decreased 28-day mortality overall
(tocilizumab, 31% vs. usual care, 35%; rate ratio 0.85; 95% CI 0.76–0.94) and
across prespeci-fied subgroups based on illness severity.
(D) Patients not requiring MV at enrollment who received tocilizumab were less likely
to progress to MV or death (35% vs 42%; risk ratio 0.84; 95% CI 0.77–0.92).
(4) Tocilizumab is authorized for use under an FDA Emergency Use Authorization
(EUA) for treatment of COVID-19 in hospitalized adults and pediatric patients age 2
years and older who are receiving systemic corticosteroids and require supplemental
oxygen, non-invasive or invasive mechanical ventilation, or ECMO.
(b) Baricitinib: Janus kinase inhibitor (alternative class agent: tofacitinib)
(1) The ACTT-2 study (N Engl J Med 2021;384:795-807) was a double-blind, randomized,
placebo-controlled trial evaluating baricitinib 4 mg/day orally or via nasogastric tube
plus remdesivir (200 mg on day one, then 100 mg daily for 5-10 days) versus placebo
in 1033 hospitalized patients with COVID-19.
(A) At enrollment, 32% of patients had severe disease, with 10.7% of patients requiring
MV or ECMO. Patients receiving corticosteroids for COVID-19 were excluded.
(B) Overall, patients receiving baricitinib plus remdesivir had a shorter time to
recovery (7 days; 95% CI 6–8 vs. 8 days; 95% CI 7–9; rate ratio 1.16; 95% CI,
1.01–1.32), which was most pronounced in patients receiving high-flow oxygen
or noninvasive ventilation at enrollment (10 days vs. 18 days; rate ratio 1.51; 95%
CI, 1.10–2.08).
(C) No difference was noted in 28-day mortality (5.1% vs. 7.8%; hazard ratio 0.65;
95% CI 0.39–1.09).
(2) The COV-BARRIER trial (Lancet Respir Med 2021; www.thelancet.com/journals/
lanres/article/PIIS2213-2600(21)00331-3/fulltext) was a multinational, placebo-
controlled, randomized trial in 1525 hospitalized patients with COVID-19 pneumonia
and elevated inflammatory markers. Patients receiving MV were excluded.

ACCP Updates in Therapeutics® 2022: Critical Care Pharmacy Preparatory Review and Recertification Course

825
Infectious Diseases I

(A) Patients were randomized 1:1 to receive baricitinib 4 mg/day (renally-adjusted as


necessary) orally or placebo in addition to usual care; 92% of patients received
con-comitant dexamethasone.
(B) No between-group difference were noted in disease progression.
(C) Patients receiving baricitinib overall had lower 28-day all-cause mortality (8%
vs. 13%; hazard ratio 0.57; 95% CI 0.41–0.78).
(D) The treatment effect for mortality was most notable among the subgroup of 370
pa-tients receiving high-flow oxygen or noninvasive ventilation at baseline (17.5%
vs. 29.4%; hazard ratio 0.52; 95% CI, 0.33–0.80).
(3) Baricitinib use is supported by an FDA EUA for treatment of COVID-19 in hospitalized
adults and pediatric patients age 2 years or older requiring supplemental oxygen, non-
invasive or invasive MV, or ECMO.
(4) Baricitinib can be administered orally or via oral dispersion or gastrostomy or
nasogas-tric tube.
(c) Although preclinical models and early clinical reports have been enthusiastic, data are
insuffi-cient to support routine use of other immunomodulators in patients with COVID-19.
Ongoing clini-cal trials are anticipated. Additional drug classes/agents reported include:
(1) IL-1 inhibitors (e.g., anakinra)
(2) Interferon alfa or beta
(3) Anti–IL-6 monoclonal antibody siltuximab
(4) Bruton tyrosine kinase inhibitors (e.g., acalabrutinib, ibrutinib, zanubrutinib)
d. Antiviral therapy
i. Remdesivir
(a) Nucleotide prodrug of adenosine analog that binds to viral RNA polymerase, terminating
RNA transcription and inhibiting viral replication. Initially developed to treat MERS
(b) FDA-approved as Veklury for adult and pediatric (12 years or older and weighing at least
40 kg) patients hospitalized with COVID-19
(1) Dosage: 200 mg intravenously once, then 100 mg intravenously daily for 5 days; 10
days’ duration can be considered in patients with no clinical improvement after 5 days
(2) Precautions include patients with baseline liver dysfunction, particularly those with
elevated alanine aminotransferase, and those with a creatinine clearance less than 30
mL/minute. The precaution in renal dysfunction is related to the cyclodextrin diluent
in both available powder and solution products.
(c) Recommended in hospitalized patients who require supplemental oxygen with or without
corticosteroids
(1) Should only be used in combination with corticosteroids in patients requiring high-
flow oxygen; not recommended in patients requiring MV or ECMO because benefit is
uncertain in subgroups of more severely ill patients.
(2) If supplies are limited, prioritize use for patients not requiring high-flow oxygen; not
recommended in patients requiring MV or ECMO.
(d) To date, the largest published, placebo-controlled trial is the Adaptive COVID-19 Treatment
Trial (ACTT-1), which compared remdesivir with placebo in 1063 patients hospitalized
with COVID-19 and receiving standard of care.
(1) Primary outcome was the time to recovery within 28 days, defined as the first day
of clinical improvement to categories 1–3 on an ordinal scale ranging from 1, not
hospitalized, return to usual care; to 3, hospitalized, not requiring supplemental
oxygen; to 8, death. All patients had scores of 4–7 at enrollment. At baseline, 197
patients (18.5%) required high-flow supplemental oxygen (score of 6), and 292 (25.6%)
required MV or ECMO (score of 7).

ACCP Updates in Therapeutics® 2022: Critical Care Pharmacy Preparatory Review and Recertification Course

826
Infectious Diseases I

(2) Patients receiving remdesivir had a shorter time to recovery (median 11 days vs. 15
days; rate ratio 1.32; 95% CI, 1.12–1.55; p<0.001). Among patients with baseline
scores of 6 and 7, rate ratios for time to recovery were 1.20 (95% CI, 0.79–1.81) and
0.95 (95% CI, 0.64–1.42), respectively.
(3) Rate ratio for 14-day mortality among all patients was 0.70 (95% CI, 0.47–1.04), with
the greatest treatment effect in the largest patient subgroup (n=421) with a baseline
score of 5 (rate ratio 0.22 [95% CI, 0.08–0.58]). Corresponding rate ratios among
patients with baseline scores of 6 and 7 were 1.12 (95% CI, 0.53–2.38) and 1.06 (95%
CI, 0.59–1.92), respectively.
(4) Adverse effects were generally similar between groups.
ii. Hydroxychloroquine or chloroquine
(a) Antimalarial agents that, in vitro, increase the endosomal pH, inhibiting fusion of SARS-
CoV-2 to the host cell membranes
(b) Not recommended as definitive or prophylactic therapy because larger controlled trials
have failed to show a difference in clinical recovery or mortality in hospitalized patients
with COVID-19
(c) Combination with azithromycin reserved for clinical trial
iii. Miscellaneous agents
(a) Ivermectin: Not recommended; large, well-controlled trials are needed to determine place
in therapy
(b) Lopinavir/ritonavir and other HIV protease inhibitors: Not recommended outside clinical
trial

Table 5. National Institutes of Health Guideline Recommendations for Pharmacologic Management of Patients
Hospitalized with Coronavirus Disease 2019
Disease Severity Remdesivir Dexamethasonea Tocilizumabb Baricitinib
Neither for or against
No supplemental routine use; consider in
Against use N/A N/A
oxygen requirement patients at high risk for
worsening
Recommended
in combination
with remdesivir
Recommended in in patients with
Supplemental
patients with minimal increasing oxygen N/A N/A
oxygen requirement
oxygen requirement requirement;
may use alone
if remdesivir is
unavailable
High-flow Consider adding either agent to
Recommended as
oxygen device Recommended in dexamethasone ± remdesivir in
preferred therapy
or noninvasive combination with patients recently hospitalized with
with or without
ventilation dexamethasone rapid decline in oxygenation and
remdesivir
requirement systemic inflammation

ACCP Updates in Therapeutics® 2022: Critical Care Pharmacy Preparatory Review and Recertification Course

827
Infectious Diseases I

Table 5. National Institutes of Health Guideline Recommendations for Pharmacologic Management of Patients
Hospitalized with Coronavirus Disease 2019 (continued)
IDSA/SHEA
Clinical Definition ACG 2013 IDSA/SHEA 2018 2021 Focused ACG 2021
Update
Add to
Mechanical
dexamethasone
ventilation or
for patients
extracorporeal Recommended as
N/A admitted to N/A
membrane preferred therapy
intensive care
oxygenation
unit within past
requirement
24 hours
Alternative corticosteroids at equivalent glucocorticoid dose may be considered.
a

b
Although approximately 33% of REMAP-CAP and RECOVERY patients received a second dose of tocilizumab at the treating physician’s discretion, evidence to
support repeat dosing is insufficient.
N/A = No recommendation provided.
Information from: National Institutes of Health (NIH); COVID-19 Treatment Guidelines Panel. Coronavirus Disease 2019 (COVID-19) Treatment Guidelines. Available
at www.covid19treatmentguidelines.nih.gov/about-the-guidelines/table-of-contents/. Accessed September 7, 2021; RECOVERY Collaborative Group. Tocilizumab in
patients admitted to hospital with COVID-19 (RECOVERY): a randomised, controlled, open-label, platform trial. Lancet 2021;397:1637-45; REMAP-CAP Investigators,
Gordon AC, Mouncey PR, et al. Interleukin-6 receptor antagonists in critically ill patients with Covid-19. N Engl J Med 2021;384:1491-502.

Patient Case

21. S.B. is a 48-year-old woman with severe obesity and heart disease intubated emergently at an outside hos-
pital for refractory hypoxemic respiratory failure despite maximal MV support. S.B. is being evaluated
for venovenous ECMO. She is quickly transferred to the MICU for evaluation. S.B. is initiated on dexa-
methasone 6 mg intravenously once daily, and her analgesia-sedative regimen is modified to maintain deep
sedation in anticipation of starting continuous neuromuscular blockade. S.B. is receiving norepinephrine at
10 mcg/minute, which continues to be tapered to maintain a mean arterial pressure greater than 65 mm Hg.
S.B. is being considered for remdesivir, which is in ample supply at your institution. Which best describes
the current guideline recommendations for remdesivir in patients with COVID-19 requiring MV?
A. Shown to decrease in-hospital mortality.
B. Not recommended because of safety concerns.
C. Recommended to shorten time to clinical recovery.
D. Uncertain benefit on clinical recovery and mortality.

ACCP Updates in Therapeutics® 2022: Critical Care Pharmacy Preparatory Review and Recertification Course

828
Infectious Diseases I

REFERENCES

Ventilator-Associated Pneumonia for preventing VAP. Endorsed by many infection


1. American Thoracic Society; Infectious Diseases and acute care organizations.
Society of America. Guidelines for the management 8. Klompas M, Li L, Menchaca JT, et al. Ultra-short-
of adults with hospital-acquired, ventilator-associ- course antibiotics for patients with suspected
ated, and healthcare-associated pneumonia. Am J ventilator-associated pneumonia but minimal
Respir Crit Care Med 2005;171:388-416. Previous and stable ventilator settings. Clin Infect Dis
evidence-based multi-organizational guidelines 2017;64:870-6.
for the management of hospital-acquired pneumo- 9. Kuti EL, Patel AA, Coleman CI. Impact of inap-
nia, including VAP. Included to support transition propriate antibiotic therapy on mortality in
between guidelines. patients with ventilator-associated pneumonia and
2. Chastre J, Fagon JY. Ventilator-associated pneumo- blood stream infection: a meta-analysis. J Crit
nia. Am J Respir Crit Care Med 2002;165:867-903. Care 2008;23:91-100. Comprehensive compilation
Comprehensive review of the foundational aspects and quantitative assessment of studies evaluating
of the diagnosis and management of VAP. inappropriate empiric antibiotic therapy for VAP.
3. Chastre J, Wolff M, Fagon JY, et al.; PneumA Trial 10. Pugh R, Grant C, Cooke RP, Dempsey G. Short-
Group. Comparison of 8 vs 15 days of antibiotic course versus prolonged-course antibiotic
therapy for ventilator-associated pneumonia in therapy for hospital-acquired pneumonia in criti-
adults: a randomized trial. JAMA 2003;290:2588- cally ill adults. Cochrane Database Syst Rev 2015;
98. Pivotal study helping define the optimal 8:Cd007577.
duration of definitive antibiotic therapy for VAP.
Foundation for current guideline recommenda- Central Line–Associated Bloodstream Infections
tions for duration of antibiotic therapy. 1. Centers for Disease Control and Prevention.
4. Dimopoulos G, Poulakou G, Pneumatikos IA, et al. Central Line-Associated Bloodstream (CLABSI)
Short- vs long-duration antibiotic regimens for ven- Event. Available at www.cdc.gov/nhsn/pdfs/
tilator-associated pneumonia: a systematic review pscmanual/4psc_clabscurrent.pdf. Accessed
and meta-analysis. Chest 2013; 144:1759–67. August 20, 2018. Provides CDC surveillance defi-
5. Dudeck MA, Weiner LM, Allen-Bridson K, et nitions for CLABSI.
al. National Healthcare Safety Network (NHSN) 2. Centers for Disease Control and Prevention.
report, data summary for 2012, device-associated Healthcare-Associated Infections in the United
module. Am J Infect Control 2013;41:1148-66. States, 2006-2016: A Story of Progress. Available
Updated CDC-based surveillance data and bench- at https://www.cdc.gov/hai/surveillance/data-
marks for rates of device-related infections. reports/data-summary-assessing-progress.html.
6. Kalil AC, Metersky ML, Klompas M, et al. Accessed September 12, 2018.
Management of Adults With Hospital-acquired and 3. Dudeck MA, Weiner LM, Allen-Bridson K, et
Ventilator-associated Pneumonia: 2016 Clinical al. National Healthcare Safety Network (NHSN)
Practice Guidelines by the Infectious Diseases report, data summary for 2012, device-associated
Society of America and the American Thoracic module. Am J Infect Control 2013;41:1148-66.
Society. Clin Infect Dis. 2016 Sep 1;63(5):e61-e111. Updated CDC-based surveillance data and bench-
Most recent evidence-based multi-organizational marks for rates of device-related infections.
guidelines for the management of hospital- 4. Marschall J, Mermel LA, Fakih M, et al. Strategies
acquired pneumonia, including VAP. to prevent central line-associated bloodstream
7. Klompas M, Branson R, Eichenwald EC, et al. infections in acute care hospitals: 2014 update.
Strategies to prevent ventilator-associated pneu- Infect Control Hosp Epidemiol 2014;35:753-71.
monia in acute care hospitals: 2014 update. Infect 5. Mermel LA, Allon M, Bouza E, et al. Clinical
Control Hosp Epidemiol 2014;35:915-36. Updated practice guidelines for the diagnosis and manage-
evidence-based and expert panel recommendations ment of intravascular catheter-related infection:

ACCP Updates in Therapeutics® 2022: Critical Care Pharmacy Preparatory Review and Recertification Course

829
Infectious Diseases I

2009 update by the Infectious Diseases Society of 7. Rothberg MB, Haessler SD. Complications of
America. Clin Infect Dis 2009;49:1-45. Evidence- seasonal and pandemic influenza. Crit Care Med
based guidelines for the clinical diagnosis and 2010;38(suppl):e91-e97. Critical care–focused
management of catheter-related infections, includ- review of severe influenza-related complications.
ing CLABSI.
6. Pronovost P, Needham D, Berenholtz S, et al. An Catheter-Associated Urinary Tract Infections
intervention to decrease catheter-related blood- 1. Centers for Disease Control and Prevention.
stream infections in the ICU. N Engl J Med Catheter-Associated Urinary Tract Infection
2006;355:2725-32. Pivotal study demonstrating (CAUTI) Event. Available at https://www.cdc.
a systematic approach using interdisciplinary gov/infectioncontrol/guidelines/cauti/index.html.
education and procedural checklists to decrease Accessed May 20, 2017. CDC prevention and sur-
CLABSI in critically ill patients. veillance definitions.
2. Daneman N, Rishu AH, Pinto RL, et al.; Canadian
Influenza Critical Care Trials Group. Bacteremia Antibiotic
1. Centers for Disease Control and Prevention. Length Actually Needed for Clinical Effectiveness
Influenza Signs and Symptoms and the Role of (BALANCE) randomised clinical trial: study pro-
Laboratory Diagnostics Available at www.cdc.gov/ tocol. BMJ Open 2020;10:e038300.
flu/symptoms/index.html. Accessed September 7, 3. Dudeck MA, Weiner LM, Allen-Bridson K, et
2021. al. National Healthcare Safety Network (NHSN)
2. Centers for Disease Control and Prevention. report, data summary for 2012, device-associated
FluView: A Weekly Influenza Surveillance Report module. Am J Infect Control 2013;41:1148-66.
Prepared by the Influenza Division. Available Updated CDC-based surveillance data and bench-
at https://www.cdc.gov/flu/weekly/index.htm. marks for rates of device-related infections.
Accessed September 7, 2021. 4. Hooton RM, Bradley SF, Cardenas DD, et al.
3. Centers for Disease Control and Prevention. Diagnosis, prevention, and treatment of catheter-
Influenza Antiviral Medications: Summary for associated urinary tract infection in adults: 2009
Clinicians. Available at www.cdc.gov/flu/profes- international clinical practice guidelines from
sionals/antivirals/index.htm. Accessed September the Infectious Diseases Society of America. Clin
7, 2021. Infect Dis 2010;50:625-63.
4. de Jong MD, Ison MG, Monto AS, Metev H, Clark 5. Lo E, Nicolle LE, Coffin SE, et al. Strategies to
C, O’Neil B, Elder J, McCullough A, Collis P, prevent catheter-associated urinary tract infec-
Sheridan WP. Evaluation of intravenous peramivir tions in acute care hospitals: 2014 update. Infect
for treatment of influenza in hospitalized patients. Control Hosp Epidemiol 2014;35:464-79.
Clin Infect Dis 2014;59:e172-85. Clinical trial of
intravenous peramivir compared to placebo in Complicated Intra-abdominal Infection
hospitalized patients. 1. Dellinger RP, Levy MM, Rhodes A, et al.
5. Flannery AH, Thompson Bastin ML. Oseltamivir Surviving Sepsis Campaign: international guide-
dosing in critically ill patients with severe lines for management of severe sepsis and septic
influenza. Ann Pharmacother 2014;48:1011-8. shock: 2012. Crit Care Med 2013;41:580-637.
Descriptive review of evidence related to oselta- 2. Levy MM, Dellinger RP, Townsend SR, et al. The
mivir dosing in critically ill patients. Surviving Sepsis Campaign: results of an interna-
6. Harper SA, Bradley JS, Englund JA, et al. Seasonal tional guideline-based performance improvement
influenza in adults and children—diagnosis, program targeting severe sepsis. Crit Care Med
treatment, chemoprophylaxis, and institutional 2010;38:367-74.
outbreak management: clinical practice guidelines 3. Marshall JC, Innes M. Intensive care unit man-
of the Infectious Diseases Society of America. agement of intra-abdominal infection. Crit Care
Clin Infect Dis 2009;48:1003-32. Med 2003;31:2228-37. In-depth descriptive review
of pathophysiology and management strategies of

ACCP Updates in Therapeutics® 2022: Critical Care Pharmacy Preparatory Review and Recertification Course

830
Infectious Diseases I

complicated intra-abdominal infection in critically 9. Villatoro E, Mulla M, Larvin M. Antibiotic ther-


ill patients. apy for prophylaxis against infection of pancreatic
4. Mazuski JE, Tessier JM, May AK, et al. The necrosis in acute pancreatitis. Cochrane Database
Surgical Infection Society Revised Guidelines on Syst Rev 2012;5:CD002941.
the Management of Intra-Abdominal Infection. 10. Whitcomb DC. Acute pancreatitis. N Engl J Med
Surgical Infections 2017;18:1-76. Updated guide- 2006;354:2142-50.
lines from the Surgical Infection Society. Previous
guidelines jointly included IDSA. Clostridioides difficile Infection
5. Solomkin JS, Mazuski JE, Bradley JS, et al. 1. Apisarnthanarak A, Razavi B, Mundy LM.
Diagnosis and management of complicated Adjunctive intracolonic vancomycin for severe
intra-abdominal infection in adults and children: Clostridium difficile colitis: case series and review
guidelines by the Surgical Infection Society and of the literature. Clin Infect Dis 2002;35:690-6.
the Infectious Diseases Society of America. Clin 2. Dubberke ER, Carling P, Carrico R, et al. Strategies
Infect Dis 2010;50:133-64. to prevent Clostridium difficile infections in acute
care hospitals: 2014 update. Infect Control Hosp
Acute Pancreatitis Epidemiol 2014;35:628-45.
1. Banks PA, Bollen TL, Dervenis C, et al. 3. Johnson S, Lavergne V, Skinner AM, et al.
Classification of acute pancreatitis – 2012: revi- Clinical practice guideline by the Infectious
sion of the Atlanta classification and definitions by Diseases Society of America (IDSA) and Society
international consensus. Gut 2013;62:102-11. for Healthcare Epidemiology of America (SHEA):
2. Crockett SD, Wani S, Gardner TB, et al. American 2021 focused update guidelines on management
Gastroenterological Association Institute guide- of Clostridioides difficile infection in adults. Clin
line on initial management of acute pancreatitis. Inf Dis 2021. Available at www.idsociety.org/
Gastroenterology 2018;154:1096-101. globalassets/idsa/practice-guidelines/cdi-2021-fo-
3. Dellinger EP, Tellado JM, Soto NE, et al. Early cused-update.pdf. Accessed September 1, 2021.
antibiotic treatment for severe acute necrotizing 4. Kelly CP, LaMont JT. Clostridium difficile—more
pancreatitis: a randomized, double-blind, placebo- difficult than ever. N Engl J Med 2008;359:1932-40.
controlled Study. Ann Surg 2007;245:674-83. 5. McDonald LC, Gerding DN, Johnson S, et al.
4. Frossard JL, Steer ML, Pastor CM. Acute pancre- Clinical practice guidelines for Clostridium diffi-
atitis. Lancet 2008;371:143-52. cile infection in adults and children: 2017 update
5. Leppäniemi A, Tolonen M, Tarasconi A, et al. 2019 by the Infectious Diseases Society of America
WSES guidelines for the management of severe (IDSA) and Society for Healthcare Epidemiology of
acute pancreatitis. World J Emerg Surg 2019;14:27. America (SHEA). Clin Infect Dis 2018;66:987-94.
6. Mazuski JE, Tessier JM, May AK, et al. The 6. Owens RC. Clostridium difficile–associated
Surgical Infection Society revised guidelines on disease: an emerging threat to patient safety.
the management of intra-abdominal infection. Pharmacotherapy 2006;26:299-311.
Surgical Infections 2017;18:1-76. Additional set 7. Surawicz CM, Brandt LJ, Binion DG, et al.
of guidelines that address prophylactic antibi- Guidelines for Diagnosis, Treatment, and
otic therapy in patients with acute necrotizing Prevention of Clostridium difficile Infections. Am
pancreatitis. J Gastroenterol 2013;108:478–98.
7. Nathens AB, Curtis JR, Beale RJ, et al. 8. Zar FA, Bakkanagari SR, Moorthi KMLST, et al.
Management of the critically ill patient with severe A comparison of vancomycin and metronidazole
acute pancreatitis. Crit Care Med 2004;32:2524-36. for the treatment of Clostridium difficile–associ-
8. Tenner S, Baillie J, DeWitt J, et al. American ated diarrhea, stratified by disease severity. Clin
College of Gastroenterology guideline: manage- Infect Dis 2007;45:302-7.
ment of acute pancreatitis. Am J Gastroenterol
2013;108:1400-15.

ACCP Updates in Therapeutics® 2022: Critical Care Pharmacy Preparatory Review and Recertification Course

831
Infectious Diseases I

Wound Infection supportive care guidelines for the manage-


1. Anaya DA, Dellinger EP. Necrotizing soft-tissue ment of Stevens-Johnson syndrome/toxic
infection: diagnosis and management. Clin Infect epidermal necrolysis in adults. J Am Acad
Dis 2007;44:705-10. Dermatol 2020;82:1553-67.
2. Anderson DJ, Podgorny K, Berríos-Torres SI, et 8. Struck MF, Hilbert P, Mockenhaupt M, et al.
al. Strategies to prevent surgical site infections in Severe cutaneous adverse reactions: emergency
acute care hospitals: 2014 update. Infect Control approach to non-burn epidermolytic syndromes.
Hosp Epidemiol 2014;35:605-27. Intensive Care Med 2010;36:22-32.
3. Lappin E, Ferguson AJ. Gram-positive toxic shock
syndromes. Lancet Infect Dis 2009;9:281-90. Novel Severe Acute Respiratory Syndrome
4. May AK, Stafford RE, Bulger EM, et al. Treatment Coronavirus 2
of complicated skin and soft tissue infections: 1. Alhazzani W, Moller MH, Arabi YN, et al.
Surgical Infection Society guidelines. Surg Infect Surviving Sepsis Campaign: guidelines on the
2009;10:467-99. management of critically ill adults with corona-
5. Stevens DL, Bisno AL, Chambers HF, et al. virus disease 2019 (COVID-19). Crit Care Med
Practice guidelines for the diagnosis and manage- 2020;48:e440-e469.
ment of skin and soft tissue infections: 2014 update 2. Beigel JH, Tomashek KM, Dodd LE. Remdesivir
by the Infectious Diseases Society of America. for the treatment of COVID-19: preliminary
Clin Infect Dis 2014;59:e10-e52. report. N Engl J Med 2020;383:994.
3. Kalil AC, Patterson TF, Mehta AK, et al.
Severe Nonthermal Cutaneous Injury Baricitinib plus remdesivir for hospitalized adults
1. Bastuji-Garin S, Rzany B, Stern RS, et al. Clinical with Covid-19. N Engl J Med 2021;384:795-807.
classification of cases of toxic epidermal necroly- 4. Marconi VC, Ramanan AV, de Bono S, et al. Efficacy
sis, Stevens-Johnson syndrome, and erythema and safety of baricitinib for the treatment of hospi-
multiforme. Arch Dermatol 1993;129:92-6. talised adults with COVID-19 (COV-BARRIER):
2. Creamer D, Walsh S, Smith C, et al. U.K. guidelines a randomised, double-blind, parallel-group, pla-
for the management of Stevens-Johnson syndrome/ cebo-controlled phase 3 trial. Lancet Resp Med
toxic epidermal necrolysis in adults 2016. British 2021. Available at www.thelancet.com/journals/
Journal Dermatology 2016;174:1194-1227. lanres/article/PIIS2213-2600(21)00331-3/fulltext.
3. Huang YC, Li YC, Chen TJ. The efficacy of intra- Accessed September 4, 2021.
venous immunoglobulin for the treatment of 5. National Institutes of Health (NIH); COVID-
toxic epidermal necrolysis: a systematic review 19 Treatment Guidelines Panel. Coronavirus
and meta-analysis. British Journal Dermatology Disease 2019 (COVID-19) Treatment Guidelines.
2012;167:424-32. Available at www.covid19treatmentguidelines.
4. Kumar Gupta L, Mani Martin A, Argawai N, et nih.gov/about-the-guidelines/table-of-contents/.
al. Guidelines for the management of Stevens– Accessed September 7, 2021.
Johnson syndrome/toxic epidermal necrolysis: an 6. RECOVERY Collaborative Group; Horby P, Lim
Indian perspective. Indian J Dermatol Venereol WS, et al. Dexamethasone in hospitalized patients
Leprol 2016;82:603-25. with COVID-19N Engl J Med 2021;384:693-704.
5. Palmieri TL, Greenhalgh DG, Saffle JR, et al. A 7. RECOVERY Collaborative Group. Tocilizumab
multicenter review of toxic epidermal necrolysis in patients admitted to hospital with COVID-19
treated in U.S. burn centers at the end of the twen- (RECOVERY): a randomised, controlled, open-
tieth century. J Burn Care Rehabil 2002;23:87-96. label, platform trial. Lancet 2021;397:1637-45.
6. Pereira FA, Mudgil AV, Rosmarin DM. Toxic 8. REMAP-CAP Investigators, Gordon AC,
epidermal necrolysis. J Am Acad Dermatol Mouncey PR, et al. Interleukin-6 receptor antago-
2007;56:181-200. nists in critically ill patients with Covid-19. N Engl
7. Seminario-Vidal L, Kroshinsky D, Malachowski J Med 2021;384:1491-502.
SJ, et al. Society of Dermatology Hospitalists

ACCP Updates in Therapeutics® 2022: Critical Care Pharmacy Preparatory Review and Recertification Course

832
Infectious Diseases I

9. Rosas IO, Baru N, Waters M, et al. Tocilizumab in


hospitalized patients with severe Covid-19 pneu-
monia. N Engl J Med 2021;384:1503-16.
10. WHO Rapid Evidence Appraisal for COVID-19
Therapies (REACT) Working Group. Association
between administration of systemic corticoste-
roids and mortality among critically ill patients
with COVID-19: a meta-analysis. JAMA
2020;324:1330-41.
7. Wiersinga WJ, Rhodes A, Cheng AC, et al.
Pathophysiology, transmission, diagnosis, and
treatment of coronavirus disease 2019 (COVID-
19): a review. JAMA 2020;324:782-93.

ACCP Updates in Therapeutics® 2022: Critical Care Pharmacy Preparatory Review and Recertification Course

833
Infectious Diseases I

ANSWERS AND EXPLANATIONS TO PATIENT CASES

1. Answer: A the cause of new-onset fever and leukocytosis, given


The patient has suspicion for MDR VAP, as evidenced by the emergency placement and related duration of
the presence of clinical signs of infection, the patient’s the CVC (Answer B is incorrect). Although catheter
increased sputum production, and the patient’s having removal should strongly be considered, cultures should
been in the ICU for 5 days or longer. Empiric antibiotic be obtained before catheter removal for documentation
therapy should be initiated after obtaining a respiratory if the patient has a bloodstream infection (Answer C
culture and be based on patient-specific risk factors for is incorrect). Initiation of antibiotic therapy should be
MDROs, together with local pathogen prevalence and considered, if appropriate, but only after cultures of the
antibiotic susceptibility, to increase the likelihood of suspected source are obtained (Answer D is correct;
providing timely appropriate antibiotic therapy (Answer Answer A is incorrect).
A is correct). Gram stain results are often nonsensitive
for causative pathogens, and waiting for preliminary or 5. Answer: D
final respiratory culture results, as well as blood or urine This patient, who is thought to have a CLABSI, has risk
cultures, may cause an unacceptable delay in appropri- factors for MDROs, given that the patient was hospi-
ate antibiotic therapy (Answers B–D are incorrect). talized for 5 or more days. Empiric antibiotic therapy
choices should include agents active against MRSE
2. Answer: B and MRSA as well as P. aeruginosa (Answer D is cor-
Although this patient is suspected of having early- rect; Answer A is incorrect). Linezolid is active against
onset VAP for the current admission, a history of MRSA; however, it should be considered only for defini-
recent intravenous antibiotic therapy is a risk factor for tive therapy because its empiric use in patients with a
MDROs. Empiric antibiotic therapy for VAP in patients CLABSI is associated with worse outcomes (Answer C
with MRDO risk factors should include agents active is incorrect). Fluconazole may be considered in addition
against P. aeruginosa and MRSA. Empiric combina- to antibiotic therapy, but monotherapy is not recom-
tion therapy against P. aeruginosa is recommended to mended empirically (Answer B is incorrect).
increase the likelihood of appropriate antibiotic therapy
(Answer B is correct; Answers A and C are incorrect) 6. Answer: B
with a β-lactam antibiotic as one of the preferred agents The guideline recommendation for definitive antibiotic
(Answer D is incorrect). Atypical bacteria coverage is therapy duration is 7–14 days from the first negative
not necessary because their prevalence is low, although blood culture in patients with uncomplicated gram-
consideration should be given if there is a poor response negative CLABSI. Longer durations of therapy should
to initial therapy. be considered in patients with persistent bacteremia
or if the patient has a poor clinical response. (Answer
3. Answer: B B is correct; Answers A and C are incorrect). Patients
Based on the PneumA trial and related meta-analyses, with complicated bacteremia (e.g., endocarditis, sep-
the most recent IDSA VAP guidelines recommend tic thrombus, chronic intravascular hardware) should
definitive antibiotic therapy duration of 7 full treatment receive 4-6 weeks of therapy (Answer D is incorrect).
days for all patients. This is further emphasized in this
patient, who has VAP caused by Klebsiella spp., which 7. Answer: D
are lactose-fermenting gram-negative bacilli, and who This patient likely has severe influenza amid a local
received appropriate empiric antibiotic therapy and had seasonal outbreak. Local infection patterns suggest
an appropriate clinical response during therapy (Answer a prevalence of influenza A and B strains. Empiric
B is correct; Answers A, C, and D are incorrect). influenza-specific therapy against these strains
should be initiated in patients with severe influenza
4. Answer: D before confirmatory test results are known to avoid a
In the absence of other suspected sources (i.e., no change delay in appropriate therapy (Answer B is incorrect).
in chest radiograph), CLABSI should be suspected as Neuraminidase-based therapy is recommended for

ACCP Updates in Therapeutics® 2022: Critical Care Pharmacy Preparatory Review and Recertification Course

834
Infectious Diseases I

modern influenza A and B stains (Answer D is correct; listed in Answers B–D; however, the absence of E. coli
Answer C is incorrect). Even if they are outside 48 hours makes these answers incorrect.
from symptom onset, patients with severe influenza
have benefited from therapy initiated beyond this period 11. Answer: D
(Answer A is incorrect). This patient has complicated intra-abdominal infection
from secondary peritonitis caused by colonic perfora-
8. Answer: B tion. Although it is community acquired, the presence
Neuraminidase inhibitors are the mainstay of treat- of septic shock suggests severe classification increas-
ment because resistance to adamantanes has developed ing the risk of gram-negative MDROs (Answer C is
in contemporary influenza A strains, and these agents incorrect). The involvement of the colon also obligates
have no antiviral activity against influenza B (Answer antibiotic therapy active against anaerobes and entero-
A is incorrect). Inhaled zanamivir is not appropriate cocci (Answer B is incorrect); MRSA is an unlikely
for administration to mechanically ventilated patients pathogen (Answer A is incorrect). According to this,
(Answer C is incorrect). Intravenous peramivir also is piperacillin/tazobactam is the most appropriate agent
available; however, data are limited for its use in criti- listed (Answer D is correct).
cally ill patients with severe, complicated influenza
infection (Answer D is incorrect). Enteral oseltamivir 12. Answer: A
is the preferred therapy for current influenza B strains The approach to managing a complicated intra-
in critically ill patients with severe, complicated influ- abdominal infection includes timely source control
enza infection (Answer B is correct). Clinicians should and initiation of empiric antimicrobial therapy active
be mindful of the need for renal dosage adjustment, if against likely pathogens. This involves assessment of
necessary. the anatomic location of the source, risk factors for mul-
tidrug-resistant organisms (i.e., community- or health
9. Answer: C care–associated disposition), and severity of illness. This
The presence of new fever and an elevated WBC in patient has community-acquired secondary peritonitis
conjunction with an indwelling urinary catheter and from a distal small bowel perforation with concomi-
pyuria on urinalysis is highly suggestive of a CAUTI tant septic shock. Although his disease is community
rather than worsening pneumonia (Answer B is incor- acquired, the presence of septic shock suggests a severe
rect). Similar to other ICU-related infections, empiric classification, increasing the risk of gram-negative mul-
antibiotic therapy should be considered if there is strong tidrug-resistant organisms and involvement of the distal
clinical suspicion for infection (Answer C is correct). small bowel obligates empiric coverage for anaerobic
Waiting for final culture results before initiating antimi- pathogens and enterococci, for all of which piperacillin/
crobial therapy in patients thought to have a CAUTI may tazobactam should provide appropriate empiric cover-
delay appropriate treatment (Answer A is incorrect). age (Answer A is correct). Ertapenem would likely have
Although it is recommended to discontinue the urinary adequate empiric coverage for enteric, gram-negative
catheter and replace as necessary, a confirmatory uri- pathogens, both aerobic and anaerobic; however, lack
nalysis is not needed if the catheter has been indwelling or enterococcal coverage makes it less optimal (Answer
for less than 2 weeks (Answer D is incorrect). B is incorrect). Ciprofloxacin lacks sufficient activity
against enterococci or anaerobic pathogens, and MRSA
10. Answer: A coverage with vancomycin is unnecessary (Answer C is
The causative pathogens associated with a CAUTI, with incorrect). Cefazolin is too narrow as empiric therapy
or without multidrug-resistant risk factors, are more for a severe community-acquired complicated intra-
heterogeneous than an uncomplicated or community- abdominal infection, and fluconazole should be reserved
acquired urinary tract infection. Nonetheless, E. coli for definitive therapy for fluconazole-susceptible yeast
remains the most common bacterial pathogen together identified on final culture (Answer D is incorrect).
with other gram-negative bacilli and Candida spp.
(Answer A is correct). Except for S. maltophilia (Answer
B is incorrect), other associated pathogens include those

ACCP Updates in Therapeutics® 2022: Critical Care Pharmacy Preparatory Review and Recertification Course

835
Infectious Diseases I

13. Answer: D below 2–4 mmol/L, suggesting the absence of shock or


Acute necrotizing pancreatitis often presents with signs other defining features of fulminant CDI; thus, mono-
and symptoms consistent with systemic inflammatory therapy with enteral vancomycin is the best available
response syndrome. However, most cases of severe answer (Answer D is correct). Although the 2021 IDSA/
necrotizing pancreatitis are sterile and do not require SHEA focused guideline update recommends fidaxomi-
antibiotic therapy. The presence of a pancreatic pseu- cin for non-fulminant, first-episode CDI, oral/enteral
docyst or abscess increases the likelihood of infected vancomycin is an acceptable alternative, especially if
necrotizing pancreatitis, which thus warrants empiric fidaxomicin is unavailable. Combination therapy with
antimicrobial therapy. However, this patient has no oral/enteral vancomycin and intravenous metronidazole
evidence of these on CT scan; thus, empiric antibiotic should be reserved for a complicated CDI (Answer C
therapy is not indicated (Answers A–C are incorrect). is incorrect), and intravenous metronidazole is not rec-
Moreover, prophylactic antimicrobial therapy is not ommended (Answer B is incorrect). The combination
recommended in the recent guidelines, despite the of metronidazole and intracolonic is recommended for
presentation of systemic inflammation and pancreatic patients with a complicated CDI and concern for ileus or
necrosis (Answer D is correct). A prospective, random- inability to deliver oral/enteral vancomycin to the colon
ized, noninferiority study most strongly supports these (Answer A is incorrect).
recommendations.
17. Answer: C
14. Answer: D Necrotizing fasciitis is a severe, life-threatening infection
This patient has acute, severe pancreatitis with CT evi- that is often polymicrobial. Prompt surgical debridement
dence of pancreatic abscess. Gram stain of fluid obtained of necrotic tissue and broad-spectrum antibiotic therapy
from CT-guided drainage suggests the presence of are the mainstays of initial therapy. Agents active
gram-negative bacilli and yeast (Answer A is incorrect). against S. pyogenes, MRSA, and aerobic and anaerobic
Empiric antimicrobial therapy is indicated for suspected gram-negative bacilli should be initiated empirically,
infected pancreatitis. Extended-spectrum carbapenems, together with adjunctive clindamycin added, which
which achieve relevant pancreatic fluid concentrations, may decrease bacterial toxin production. The empiric
are effective in the management of infected pancreatitis. regimen of piperacillin/tazobactam, vancomycin, and
Addition of anti-candidal therapy is indicated, given the clindamycin is most appropriate, given the severity of
presence of yeast on the Gram stain (Answer D is cor- infection despite the absence of traditional multidrug-
rect; Answers B and C are incorrect). resistant organism risk factors (Answer C is correct).
Penicillin G is not broad enough (Answer B is incorrect),
15. Answer: B and the combination of ceftaroline and vancomycin
This patient is thought to have fulminant (IDSA 2017) does not contain clindamycin (Answer A is incorrect).
CDI, given his recent exposure to broad-spectrum Vancomycin monotherapy does not include gram-
antibiotic therapy, signs of infection, CT findings, and negative activity or adjunctive clindamycin (Answer D
the presence of hypotension. Combination therapy with is incorrect).
metronidazole and enteral vancomycin is indicated
(Answers A and D are incorrect). The presence of 18. Answer: B
ileus requires consideration of adding intracolonic The intraoperative cultures of this patient’s necrotizing
vancomycin because of possible impaired delivery to infection are of concern for S. pyogenes. S. pyogenes
the colon through enteral routes (Answer B is correct; produces exotoxin, which is associated with tissue
Answer C is incorrect). necrosis. Although bactericidal antibiotic therapy
is necessary for eradicating S. pyogenes, adjunctive
16. Answer: D clindamycin may decrease toxin production, which could
The patient in this case presents with what is likely an limit the extent of tissue necrosis (Answer B is correct).
initial-episode, severe CDI, as supported by new-onset Vancomycin should be reserved for patients with an S.
diarrhea and clinical signs and symptoms of infection. pyogenes infection who have a β-lactam allergy because
Of note, the patient is not hypotensive and has a lactate β-lactam resistance is rare (Answer C is incorrect).

ACCP Updates in Therapeutics® 2022: Critical Care Pharmacy Preparatory Review and Recertification Course

836
Infectious Diseases I

Moreover, synergistic antibiotic therapy with gentamicin available evidence regarding safety, efficacy, and
is not needed, given the effectiveness of gram-positive pharmacogenomic implications (Answer B is correct).
β-lactams against S. pyogenes (Answer A is incorrect).
The role of IVIG in streptococcal necrotizing fasciitis is 21. Answer: D
controversial. A small randomized, placebo-controlled Remdesivir is recommended for patients requiring
trial – stratified on the basis of need for surgery and supplemental oxygen. On the basis of the results of the
clindamycin treatment – showed no improvement in ACTT study showing no difference in recovery time or
survival or reduction in the time to no further progression mortality in more severe subgroups, the NIH guidelines
of necrotizing fasciitis or cellulitis in 21 patients with do not recommend for or against remdesivir in patients
streptococcal toxic shock syndrome. As such, the requiring high-flow oxygen, MV, or ECMO (Answer D
guidelines do not recommend IVIG until additional is correct; Answers A and C are incorrect). If remdesivir
studies are available (Answer D is incorrect). is initiated before advancing to MV, it is recommended
to continue and complete therapy. Available evidence
19. Answer: A guidelines do not suggest increased adverse effects with
One of the cornerstones of managing severe cutaneous remdesivir between less and more severely ill patients
injury includes volume resuscitation, preferentially with (Answer B is incorrect).
crystalloids (Answer A is correct). The usefulness of
immunomodulating therapies such as corticosteroids
and cyclophosphamide is limited by observational and
poorly controlled evidence or case reports; thus, use
of immunomodulating therapies should be reserved
for specialty centers using formal protocols with
consideration for treatment under clinical study or reg-
istry (Answers B and D are incorrect). Wound care is
imperative in these patients; however, adding unneces-
sary drugs that could confound response or worsen the
injury (e.g., antibiotics) should be avoided (Answer C
is incorrect) unless there is an objectively suspected or
confirmed infection.

20. Answer: B
Use of IVIG for Stevens-Johnson syndrome (SJS)/TEN
is controversial. Available evidence is from case reports/
series, observational cohort studies, or small, single-
center randomized trials (Answer C is incorrect). Because
of bias, limited external validity, and mixed results/
observations between publications, meta-analyses and
consensus guidelines do not broadly endorse the use of
IVIG (Answer D is incorrect). The available data analyses
that suggest that IVIG decreases the SCORTEN-related
mortality in patients with TEN and a middle to higher
SCORTEN score; there are even fewer data for the role
of IVIG in SJS (Answer A is incorrect). Given the limited
evidence supporting pharmacotherapeutic interventions
and the life-threatening severity of TEN, burn referral
centers should base the decision to use IVIG on local
protocols/guidelines developed by interdisciplinary
practitioners who have objectively evaluated the

ACCP Updates in Therapeutics® 2022: Critical Care Pharmacy Preparatory Review and Recertification Course

837
Infectious Diseases I

ANSWERS AND EXPLANATIONS TO SELF-ASSESSMENT QUESTIONS

1. Answer: D to vancomycin if there were a high suspicion for addi-


The patient described has early onset VAP without tional pathogens (Answers A and C are incorrect).
apparent updated risk factors for MDROs (Answers
A, B, and C, are incorrect). Pathogens associated with 4. Answer: D
early onset hospital-acquired pneumonia and VAP in Given the high prevalence of the 2009 H1N1 subtype of
the absence of other MDR risk factors are usually com- influenza A, oseltamivir is the empiric drug of choice. In
munity-acquired organisms, including S. pneumoniae, addition, oseltamivir has high-level activity against other
MSSA, H. influenzae, and enteric gram-negative bacilli contemporary influenza A and B subtypes (Answer D is
(Answer D is correct). However, because some epide- correct). Zanamivir also has high-level activity against
miological studies have documented the occurrence of these strains; however, inhaled therapy through the
traditionally nosocomial or late-onset pathogens within mechanical ventilator is not indicated because of insuf-
5 days from admission, guideline updates recommend ficient systemic delivery, and intravenous zanamivir is
that P. aeruginosa coverage be considered as empiric available through compassionate use and indicated only
therapy for all patients suspected of having VAP. This if oseltamivir cannot be administered (e.g., patient is
further emphasizes the importance of understanding unable to receive enteral medications, has poor absorp-
local VAP pathogen trends amid MDR risk factors and tion) (Answers B and C are incorrect). Amantadine has
time from admission to onset. Atypical bacteria are insufficient activity against most contemporary influ-
rarely associated with early onset VAP. If MDRO risk enza A and B strains (Answer A is incorrect).
factors were present, MRSA and P. aeruginosa would
also be considered. 5. Answer: B
Although a health care–associated CAUTI is caused by
2. Answer: B a more diverse spectrum of pathogens, E. coli is still the
Empiric antibiotic choices for VAP should be based on most common pathogen and is responsible for around
the likely causative pathogens, the presence of MDRO 30% of cases (Answer B is correct). The other pathogens
risk factors, and local antibiotic susceptibility patterns. listed are also possible and should be considered when
The 2016 IDSA guidelines recommend P. aeruginosa choosing empiric antibiotic therapy in patients with a
coverage as empiric therapy for all patients suspected suspected CAUTI (Answers A, C, and D are incorrect).
of having VAP, with monotherapy antipseudomonal
β-lactam sufficient in the absence of MDRO risk factors 6. Answer: D
or if the local antibiogram suggests less than 10% resis- This patient has community-acquired complicated
tance. Therefore, cefepime is preferred to ceftriaxone intra-abdominal infection involving the middle small
(Answer B is correct; Answers C and D are incorrect). intestine. Although enteric gram-negative bacilli (e.g.,
Atypical bacteria are rarely associated with early-onset E. coli, Klebsiella spp.) are the most common pathogens
VAP (Answer A is incorrect). related to this type of infection, patients with severe dis-
ease, as evidenced by concomitant septic shock, are at
3. Answer: D a higher risk of MDROs, including P. aeruginosa and
Gram-positive organisms are the most common cause enterococci. Piperacillin/tazobactam has empiric activ-
of CLABSI, including MRSE and MRSA. Vancomycin ity against these organisms, whereas the other regimens/
is the best option listed for empiric management agents listed have relevant gaps in the bacterial spec-
(Answer D is correct). Although linezolid has a suffi- trum relative to these pathogens (Answer D is correct;
cient spectrum of activity against these organisms, it is Answers A–C are incorrect).
not recommended for empiric management of CLABSI
because of concerns for worse patient outcomes (Answer 7. Answer: A
B is incorrect). The other options are inactive against This patient presents with severe acute pancreatitis
MRSE and MRSA and could be considered in addition with radiographic evidence of pancreatic necrosis.
Although the patient presents with SIRS, the absence of

ACCP Updates in Therapeutics® 2022: Critical Care Pharmacy Preparatory Review and Recertification Course

838
Infectious Diseases I

significant fluid collection or abscess suggests there is 11. Answer: A


no concomitant infection. As such, there is no indication The RECOVERY trial showed that dexamethasone
for empiric antibiotic therapy at this time (Answer A is decreases 28-day mortality compared with usual care
correct; Answer B is incorrect). Most recent evidence in hospitalized patients with COVID-19 requiring sup-
and guideline recommendations do not support prophy- plemental oxygen or MV. A meta-analysis of available
lactic antibiotic therapy for preventing infection of the trials also supports corticosteroid use (Answer A is
necrotic tissue (Answer D is incorrect). The mainstay of correct). Hydroxychloroquine has limited to no benefit
therapy for this patient is volume resuscitation and con- in patients hospitalized with COVID-19 (Answer B is
sideration of surgical debridement of pancreatic necrosis incorrect). Evidence from the ACTT study showed that
if persistent SIRS is evident (Answer C is incorrect). remdesivir may shorten the duration to clinical recovery,
but it has no difference on mortality compared with pla-
8. Answer: A cebo. Non-controlled, aggregated data have suggested
Metronidazole orally or through a feeding tube is not an association with lower mortality, but this has not
recommended for management of an initial-episode been confirmed in controlled trials; therefore, remdesi-
CDI (Answer B is incorrect). Intravenous metronida- vir should be considered in addition to dexamethasone
zole is indicated for fulminant CDI in combination with rather than monotherapy in patients with increasing oxy-
vancomycin (Answer C is incorrect) In the absence of gen requirement (Answer C is incorrect). Tocilizumab,
fulminant CDI (i.e., shock, megacolon, ileus), the 2021 an anti–IL-6 monoclonal antibody, can be considered
IDSA focused guideline update recommends fidaxomi- in addition to dexamethasone with or without remdesi-
cin over oral/enteral vancomycin for first-episode CDI vir in patients recently hospitalized and demonstrating
to increase the likelihood of complete clinical response rapid decline in oxygenation and systemic inflammation.
(Answer A is correct; Answer D is incorrect). If fidax- (Answer D is incorrect).
omicin is unavailable or cost-prohibitive, oral/enteral
vancomycin is an acceptable alternative.

9. Answer: A
This patient has a superficial incisional wound infec-
tion requiring opening and the local debridement of
infected material. Lack of systemic signs of infection
and no involvement of the fascia suggest that no anti-
biotic therapy is necessary at this time (Answer A is
correct; Answers B–D are incorrect). If the infection
extends to include these features or worsened erythema
consistent with cellulitis, empiric antibiotic therapy may
be warranted.

10. Answer: B
Similar to severe thermal cutaneous injury, the foun-
dation for managing TEN is volume resuscitation and
supportive care as well as removal of all suspected
causes (Answer B is correct). The utility of corticoste-
roids is limited and may be harmful to wound healing
(Answer A is incorrect). Addition of unnecessary drugs
that could confound response or worsen the injury (e.g.,
antibiotics) should be avoided (Answers C and D are
incorrect).

ACCP Updates in Therapeutics® 2022: Critical Care Pharmacy Preparatory Review and Recertification Course

839
Hepatic Failure/GI/Endocrine
Emergencies
Stephanie N. Bass, Pharm.D., BCCCP, BCPS
Cleveland Clinic
Cleveland, Ohio

Mollie G. Lumpkin, Pharm.D., BCCCP, BCPS


Cleveland Clinic
Cleveland, Ohio
Hepatic Failure/GI/Endocrine Emergencies

Hepatic Failure/GI/Endocrine
Emergencies
Stephanie N. Bass, Pharm.D., BCCCP, BCPS
Cleveland Clinic
Cleveland, Ohio

Mollie G. Lumpkin, Pharm.D., BCCCP, BCPS


Cleveland Clinic
Cleveland, Ohio

ACCP Updates in Therapeutics® 2022: Critical Care Pharmacy Preparatory Review and Recertification Course

843
Hepatic Failure/GI/Endocrine Emergencies

Learning Objectives TPN Total parenteral nutrition


TSH Thyroid-stimulating hormone
1. Define acute liver failure (ALF), and develop a UGIB Upper gastrointestinal bleeding
treatment strategy to help manage and reduce the
complications associated with ALF.
2. Evaluate the severity of an episode of acute pan- Self-Assessment Questions
creatitis, and construct a plan for pharmacologic, Answers and explanations to these questions can be
nutritional, and surgical management. found at the end of this chapter.
3. Identify risk factors and treatment options for
gastrointestinal fistulas, postoperative ileus, and Questions 1 and 2 pertain to the following case.
postoperative nausea and vomiting. A 25-year-old woman is brought to the emergency
4. Design a treatment plan for patients who present department (ED) after a suspected overdose of acet-
with an acute upper gastrointestinal bleed. aminophen. The time of ingestion is unknown. On
5. Differentiate between the main endocrine emergen- presentation, her acetaminophen concentration is unde-
cies in the intensive care unit, and be able to design tectable, but her alanine aminotransferase (ALT) and
a therapeutic regimen for a patient presenting with aspartate aminotransferase (AST) concentrations are
each condition. 3500 IU/L and 2500 IU/L, respectively. The patient is
markedly confused with incoherent speech, but arous-
able. Other pertinent laboratory values include bilirubin
Abbreviations in This Chapter 3.0 mg/dL and alkaline phosphatase 500 IU/L. White
blood cell count is 12 × 103 cells/mm3, platelet count is
ALF Acute liver failure 90,000/mm3, and international normalized ratio (INR) is
AP Acute pancreatitis 2.6.
BG Blood glucose
CIRCI Critical illness–related corticosteroid 1. Which option best represents the two signs or
insufficiency symptoms that would qualify this patient for a diag-
CPP Cerebral perfusion pressure nosis of acetaminophen-induced acute liver failure
CT Computed tomography (ALF)?
DILI Drug-induced liver injury A. Jaundice and encephalopathy.
DKA Diabetic ketoacidosis B. Thrombocytopenia and encephalopathy.
ED Emergency department C. Coagulopathy and encephalopathy.
ERCP Endoscopic retrograde D. Leukocytosis and encephalopathy.
cholangiopancreatography
HHS Hyperosmolar hyperglycemic state 2. Which is the most appropriate treatment for her sus-
ICP Intracranial pressure pected acetaminophen-induced ALF?
ICU Intensive care unit
A. Give intravenous acetylcysteine 21-hour regi-
INR International normalized ratio
men, continuing if necessary until signs and
MRI Magnetic resonance imaging
symptoms of ALF have resolved.
NAI-ALF Non–acetaminophen-induced acute liver
B. Acetylcysteine therapy is not indicated at this
failure
time because her acetaminophen concentration
NG Nasogastric
is undetectable.
NJ Nasojejunal
C. Give oral acetylcysteine 72-hour regimen.
NSAID Nonsteroidal anti-inflammatory drug
D. Oral acetylcysteine or intravenous acetylcys-
POI Postoperative ileus
teine may be used because the two routes are
PONV Postoperative nausea and vomiting
similarly efficacious.
PPI Proton pump inhibitor
SIRS Systemic inflammatory response syndrome
T 3 Triiodothyronine
T4 Thyroxine

ACCP Updates in Therapeutics® 2022: Critical Care Pharmacy Preparatory Review and Recertification Course

844
Hepatic Failure/GI/Endocrine Emergencies

3. A 46-year-old man presents with alcohol-induced C. Decrease alvimopan dose to 6 mg twice daily.
severe acute pancreatitis (AP). Pertinent medical D. Use is contraindicated in patients with previous
history includes alcoholic cirrhosis and several admis- myocardial infarction.
sions for aspiration pneumonia secondary to hepatic
encephalopathy. He is given intravenous fluids for ini- 6. A 40-year-old woman presents for elective abdomi-
tial volume resuscitation with lactated Ringer solution. nal surgery to remove a malignancy from her liver.
Which action would be best for the patient’s nutrition? Given the extensiveness of the surgery, the expected
A. Give nothing by mouth (i.e., kept NPO) to rest use of volatile anesthetics, and the use of periop-
the pancreas until AP is resolved. erative opioids, a multimodal plan is developed to
B. Initiate total parenteral nutrition (TPN). avoid postoperative nausea and vomiting (PONV).
C. Give enteral feeding by the nasogastric (NG) Which best describes the timing of prophylactic
route. administration with respect to surgery?
D. Give enteral feeding by the nasojejunal (NJ) A. Dexamethasone 4 mg intravenously given at
route. the end of surgery.
B. Ondansetron 4 mg intravenously given when
4. A 37-year-old woman presents after a Roux-en-Y inducing anesthesia.
gastric bypass for morbid obesity. Her postoperative C. Neurokinin-1 receptor antagonist (aprepitant
course was complicated by the formation of an entero- 40–125 mg) given at the end of surgery.
cutaneous fistula, fevers, and leukocytosis. She was D. Droperidol 0.625–1.25 mg intravenously given
initiated on broad-spectrum antibiotics, and a wound at the end of surgery.
vacuum-assisted closure was placed on her fistula site
to help with drainage and healing. Her fistula output 7. A 69-year-old woman presents to the surgical inten-
was 570 mL/day yesterday, and today, it was 250 mL/ sive care unit (ICU) at your institution with upper
day. Which statement is most accurate regarding her gastrointestinal bleeding (UGIB) caused by a gas-
fistula output between the two recordings? tric ulcer. She has lost a significant amount of blood
A. Her output would be defined as a low output on because of the bleed and currently requires blood
both days. transfusions. As part of her diagnostic workup to
B. Her output would be defined as a high output determine the etiology of her ulcer, she tests posi-
that has converted to a low output. tive for a Helicobacter pylori infection. Which best
C. Her output would be defined as a high output reflects an inappropriate treatment option for her?
that has converted to a moderate output. A. Octreotide 50 mcg bolus, followed by 50 mcg/
D. Her output would be considered moderate on hour for 72 hours.
both days. B. Treatment with a proton pump inhibitor (PPI)/
antibiotic combination for 14 days.
5. A 68-year-old man presents for a large bowel resec- C. A therapeutic endoscopy within 24 hours.
tion. Given the high incidence of postoperative ileus D. Blood transfusions to maintain hemoglobin
(POI) with this procedure, he is initiated on alvimo- greater than 7 g/dL.
pan before surgery. To reduce the cardiovascular risk
associated with alvimopan, the U.S. Food and Drug 8. Which set of laboratory abnormalities best reflects
Administration (FDA) placed a restriction on alvi- those that patients in thyroid storm typically present
mopan use that has been implemented through the with?
EASE (ENTEREG Access Support & Education) A. High thyroid-stimulating hormone (TSH),
program. Which best represents that restriction? triiodothyronine (T3), and thyroxine (T4)
A. Start continuous electrocardiogram (ECG) concentrations.
monitoring on initiation to monitor for cor- B. Low TSH, high T3, and high T4 concentrations.
rected QT (QTc) prolongation. C. Low TSH, high T3, and low T4 concentrations.
B. Use is restricted to short term with a limit of D. Low TSH, low T3, and low T4 concentrations.
15 doses.

ACCP Updates in Therapeutics® 2022: Critical Care Pharmacy Preparatory Review and Recertification Course

845
Hepatic Failure/GI/Endocrine Emergencies

BPS Critical Care Pharmacy Specialist Examination Content Outline

This chapter covers the following sections of the Critical Care Pharmacy Specialist Examination Content Outline:
1. Domain 1: Clinical Knowledge and Application
a. Task 1: 1, 3
b. Task 2: 1-3, 7
c. Task 3: 1-7
d. Task 4: 1-3, 5, 8
e. Task 5: 5-7
f. Task 6: 1-5, 7
g. Task 7: 1-7
h. Task 8: 1
2. Domain 2: Practice Management, Policy, and Quality Improvement
a. Task 1: 3: 2
3. Domain 3: Evidence-Based Medicine, Scholarship, Education, and Professional Development
a. Task 1: 3

ACCP Updates in Therapeutics® 2022: Critical Care Pharmacy Preparatory Review and Recertification Course

846
Hepatic Failure/GI/Endocrine Emergencies

I.  ACUTE LIVER FAILURE

A. Epidemiology
1. Incidence of ALF is less than 10 cases per 1 million individuals per year (2000 cases per year in the
United States), though morbidity and mortality are exceedingly high with ALF. Multiorgan failure and
death occur in as many as 50% of patients.
2. ALF accounts for less than 10% of liver transplants annually in the United States.
3. ALF can occur in any age and demographic group, though the etiology of ALF is sometimes
geographically dependent. Viral hepatitis cases are more common in developing countries, whereas
toxin-related cases (e.g., those related to acetaminophen) occur more often in developed countries.
4. The most common causes in the United States are drug induced, viral, autoimmune, and shock. Drug-
induced causes, primarily acetaminophen, account for about 50% of the ALF cases in the United States.
About 15% have no identifiable cause.
5. The mortality rate has decreased significantly during the past few decades because of advances in
medical care and early consideration for liver transplantation. Survival rates for ALF now exceed 65%,
whereas before early transplantation, survival rates were less than 15%.

B. Definitions
1. ALF is defined by the U.S. Acute Liver Failure Study Group and the American Association for the
Study of Liver Diseases as evidence of liver-induced coagulopathy, considered an INR of 1.5 or greater,
and any degree of mental alteration (encephalopathy) in a patient without preexisting liver failure and
with an illness duration less than 26 weeks.
a. ALF is in contrast to ACLF (acute-on-chronic liver failure), which is a syndrome defined on the
basis of acute decompensation of chronic liver disease and presence of multiorgan failure.
2. ALF may be further differentiated according to the time to encephalopathy after the onset of jaundice.
These intervals typically provide clues regarding the cause of the ALF; however, the differentiation
itself generally has no prognostic implications distinct from the causes themselves.
a. Hyperacute ALF: Encephalopathy occurs less than 7 days after the onset of jaundice. This subclass
of ALF is typically caused by acetaminophen toxicity or ischemic hepatitis and is associated
with higher rates of transplant-free survival. Patients tend to have high-degree encephalopathy at
presentation and a higher incidence of cerebral edema, albeit a better prognosis overall.
b. Acute ALF: Encephalopathy occurs 7–21 days after the onset of jaundice. Common causes of acute
ALF include viral hepatitis. These patients have a high incidence of cerebral edema but, unlike in
hyperacute ALF, lower rates of transplant-free survival.
c. Subacute ALF: Encephalopathy occurs more than 21 days and less than 26 weeks after the onset
of jaundice. Typical causes are drug induced or indeterminate, and this subclass is associated with
lower transplant-free survival, though patients have less marked coagulopathy and encephalopathy
at presentation.

C. Diagnosis
1. An unexplained elevated INR in a patient presenting with encephalopathy requires further evaluation
for ALF because the combination of these two symptoms is very specific to ALF.
2. In ALF, certain markers of chronic liver disease (e.g., jaundice, ascites, right upper quadrant pain,
portal hypertension) may not be present.
3. Additional physical examination, laboratory analysis, and imaging necessary for the diagnosis and
workup of ALF are shown in Table 1.
4. Patients with any degree of encephalopathy should be transferred to an ICU, ideally with contact to a
transplant center, because rapid progression can occur.

ACCP Updates in Therapeutics® 2022: Critical Care Pharmacy Preparatory Review and Recertification Course

847
Hepatic Failure/GI/Endocrine Emergencies

Table 1. Diagnostic Approach to a Patient with Suspected ALF


Assess for exposure history to viruses, drugs (e.g., acetaminophen), or toxins (e.g.,
History mushrooms)
Assess substance abuse history
Normal signs of chronic liver disease (e.g., ascites, jaundice, right upper quadrant pain)
Physical examination may not be present
Assess encephalopathy grade
Basic metabolic panel, CBC, liver function tests, coagulation tests, arterial blood gas,
acetaminophen concentrations, ammonia, toxicology screen, blood typing
Laboratory analysis
Viral serologies
Liver biopsy useful for determining autoimmune hepatitis or ALF associated with HSV
Hepatic imaging studies (computed tomography [CT], ultrasonography) may be used to
Imaging
detect a thrombus of the hepatic vein
ALF = acute liver failure; CBC = complete blood cell count; HSV = herpes simplex virus.
Adapted from: Lee WM. Acute liver failure. Semin Respir Crit Care Med 2012;33:36-45.

Table 2. Causes of ALF in the United States


Acetaminophen 46%
Indeterminate 12%
Non-acetaminophen drug induced 11%
Hepatitis B 7%
Autoimmune 7%
Ischemic 7%
Hepatitis A 1.5%
Wilson disease 1.2%
Pregnancy 1%
Budd-Chiari 0.7%
Other 7%
Adapted from: Stravitz RT, Lee WM. Acute liver failure. Lancet 2019;394:869-81.

D. Causes (Table 2)
1. Acetaminophen
a. Acetaminophen overdose is responsible for almost 50% of ALF cases in the United States.
Acetaminophen overdose is considered the primary cause of ALF in the United States and Europe,
and it was responsible for 70,000 health care encounters and 300 deaths in the United States in
2005.
b. Unlike most other types of drug-induced liver injury (DILI), acetaminophen-induced liver failure
is dose-dependent and predictable, and it is typically associated with doses above 10 g/day (or
more than 200 mg/kg/day) in 24 hours or more than 6 g/day (around 150 mg/kg/day) in 48 hours
in adults.
c. Rates of ALF caused by acetaminophen have increased during the previous 2 decades.
d. If not treated in the early stages (i.e., before the development of encephalopathy), the mortality rate
is around 20%–40%.

ACCP Updates in Therapeutics® 2022: Critical Care Pharmacy Preparatory Review and Recertification Course

848
Hepatic Failure/GI/Endocrine Emergencies

e. Acetaminophen-induced ALF typically presents as hyperacute liver failure and is defined by four
stages of progression.
i. Preclinical: Occurs within the first 24 hours of ingestion. Typically associated with minimal or
no signs or symptoms of hepatotoxicity
ii. Injury: Occurs 24–48 hours after ingestion. Associated with marked elevation in liver
transaminases
iii. Failure: Occurs 72–96 hours after ingestion. Associated with peak liver injury including
encephalopathy, coagulopathies, and jaundice
iv. Recovery: Occurs 1 week after ingestion if patient survives through failure stage
f. Additional information on background, pathophysiology, and treatment of acetaminophen overdose
can be found in the Toxicology review chapter.
2. Non–acetaminophen-induced acute liver failure (NAI-ALF)
a. DILI
i. When DILI is caused by drugs other than acetaminophen, the incidence is rare, causing about
11% of ALF cases per year.
ii. Unlike acetaminophen-induced liver failure, DILI is rarely the result of dose-related toxicity,
and most cases are idiosyncratic.
iii. DILI typically presents as a subacute ALF, with most cases occurring within the first 6 months
after drug initiation. However, some drugs (e.g., nitrofurantoin, minocycline, statins) have the
potential to cause DILI 6 months or more after initiation.
iv. Transplant-free survival is low for these patients (about 30%), and most patients will require
transplantation.
v. DILI is ultimately a diagnosis of exclusion. The American College of Gastroenterology
guidelines for the management of DILI recommend a specific workup for viral hepatitis,
autoimmune hepatitis, Wilson disease, and Budd-Chiari syndrome before diagnosis of DILI.
vi. To identify potential culprit medications, a detailed patient medication history should be
obtained, including herbal medications. Classes of drugs commonly associated with DILI
include antibiotics, nonsteroidal anti-inflammatory drugs (NSAIDs), and anticonvulsants,
which together account for almost two-thirds of those attributable to DILI.
vii. Scoring systems such as the RUCAM (Roussel Uclaf Causality Assessment Method) have
been developed to assess the causality attribution for suspected DILI. These scoring systems
give points on the basis of timing of exposure and liver function tests, risk factors for DILI,
competing medications and diagnoses, and rechallenge information. Higher scores indicate a
higher likelihood of drug cause.
viii. See Appendix 1 for a list of medications linked to DILI.
b. Viral
i. Infection with hepatitis A, B, or E is the primary etiology of ALF in the developing world and
has become a relatively infrequent cause of ALF in the United States (about 12%).
ii. Hepatitis A
(a) Accounts for about 2% of ALF cases in the United States
(b) Transmission is usually by the fecal-oral route.
iii. Hepatitis B
(a) Rates of hepatitis B–induced ALF have fallen significantly in the past few decades;
however, hepatitis B is still the cause of about 7% of ALF cases per year.

ACCP Updates in Therapeutics® 2022: Critical Care Pharmacy Preparatory Review and Recertification Course

849
Hepatic Failure/GI/Endocrine Emergencies

(b) ALF secondary to hepatitis B is often caused by reactivation of chronic or inactive


hepatitis B during times of immunosuppression (e.g., chemotherapy, high-dose steroids).
To prevent reactivation, patients who are positive for HBsAg (hepatitis B surface antigen)
and who are to begin immunosuppressive regimens require antiviral prophylaxis with
a nucleos(t)ide analog, typically lamivudine or entecavir. Treatment, which is usually
initiated before immunosuppression, continues during immunosuppressive therapy and
for 6 months thereafter.
iv. Hepatitis E
(a) Rare cause of ALF in the United States; however, hepatitis E–induced ALF is a significant
cause of ALF in countries where it is endemic, such as Russia, Pakistan, Mexico, and India.
(b) Hepatitis E–induced ALF tends to be more severe in pregnant women, and hepatitis E can
be transmitted to neonates during acute infections in pregnant women.
v. Herpes simplex virus (HSV)
(a) HSV is rarely a cause of ALF; however, cases have been reported, particularly in
immunocompromised and pregnant patients.
(b) Patients with HSV ALF can be treated with acyclovir 5–10 mg/kg intravenously every 8
hours for at least 7 days.
vi. Viral hepatitis–induced ALF generally presents as acute or subacute liver failure with an onset
of symptoms greater than 1 week after onset of jaundice.
vii. Globally, mortality rates are greater than 50% because of ALF from hepatitis A and E in the
developing world; however, mass vaccination and better public health standards have helped
reduce the incidence of viral infections in the developed world.
c. Acute ischemic injury
i. Often called “shock liver,” acute ischemic injury may lead to ALF after cardiac arrest, any
period of significant hypovolemia or hypotension, or during severe congestive heart failure.
ii. Documented hypotension is not always evident with acute ischemic injury. Drug-induced
hypotension or hypoperfusion may also cause acute ischemic injury, such as with cocaine and
methamphetamine.
iii. Typical laboratory presentation includes markedly elevated aminotransferase concentrations. In
addition, simultaneous renal dysfunction and other markers of hypoperfusion may be present.
iv. Acute ischemic injury is classified as a hyperacute ALF, which generally resolves with
resolution of the circulatory problem.
d. Mushroom poisoning
i. Typically caused by the Amanita phalloides spp. of mushrooms
ii. Mushroom poisoning is classified as a hyperacute ALF with an onset of symptoms within
24 hours after ingestion. In addition to hepatotoxicity, patients with mushroom poisoning
present with severe gastrointestinal (GI) symptoms including nausea, vomiting, diarrhea, and
abdominal cramping.
e. Wilson disease
i. Rare cause of ALF; implicated in approximately 1% of cases per year, mostly affecting young
people
ii. Wilson disease is a rare disorder that causes accumulation of copper in the body. Diagnosis is
characterized by an abrupt onset of Coombs-negative hemolytic anemia with serum bilirubin
greater than 20 mg/dL. Other diagnostic laboratory findings include low serum ceruloplasmin,
presence of Kayser-Fleischer rings, and high urinary and hepatic copper concentrations.
iii. Patients who present with fulminant liver failure secondary to Wilson disease have exceedingly
high mortality rates without transplantation.

ACCP Updates in Therapeutics® 2022: Critical Care Pharmacy Preparatory Review and Recertification Course

850
Hepatic Failure/GI/Endocrine Emergencies

f. Autoimmune hepatitis
i. Autoimmune hepatitis is considered a chronic inflammatory disease; however, patients can
still be considered to have ALF if they had rapid deterioration of symptoms. About 20% of
patients with stable disease will have ALF, which is typically instigated by an environmental
trigger.
ii. Initiation of corticosteroid therapy may be considered for some patients with early-stage
ALF without multiorgan failure. Azathioprine and mycophenolate have also been studied.
Regimens should be tailored to the patient. However, patients with progressive disease will
require liver transplantation.
g. Budd-Chiari syndrome
i. Budd-Chiari syndrome is caused by an acute hepatic vein thrombosis. Presenting symptoms
include abdominal pain, ascites, and frank hepatomegaly.
ii. Prognosis is poor if patients present with ALF, and commonly, transplantation may be required
instead of venous decompression.
h. Acute fatty liver of pregnancy/hemolysis, elevated liver enzymes, low platelets (HELLP)
i. Toward the end of pregnancy, a small percentage of women will develop rapidly progressive
hepatic failure that is generally associated with three hallmark signs: jaundice, coagulopathy,
and low platelets, also known as HELLP.
ii. HELLP is associated with increased mortality for both the fetus and the mother.
iii. Requires emergency delivery of fetus, after which symptoms should resolve

E. Complications
1. ALF affects almost every organ system in the body:
a. Neurologic
i. Cerebral edema and elevated intracranial pressures (ICPs) are the most serious complications
of ALF. Uncontrolled edema and elevated ICPs can lead to uncal herniation and are usually
fatal. Cerebral edema may also lead to tissue hypoxia, which may result in long-term neurologic
deficits.
(a) Elevated ICPs can be caused by several factors, but osmotic shifts in the brain,
inflammation, and neurotoxins are thought to be the primary causes.
(b) Because ammonia is converted to osmotically active glutamine, concentrations have
been correlated with both encephalopathy and cerebral edema. Ammonia concentrations
greater than 200 mcg/dL are associated with cerebral herniation, whereas concentrations
less than 75 mcg/dL are rarely associated with hepatic encephalopathy. In ALF, either
because of impaired hepatocyte activity or the abnormal shunting of venous flow away
from the liver, normal mechanisms to detoxify and clear ammonia are no longer effective.
ii. Encephalopathy is considered an indicator for the clinical presentation of cerebral edema.
Encephalopathy can be difficult to identify and may present initially as agitation and confusion;
however, it may progress rapidly to unresponsiveness. Table 3 gives useful guidelines for
measuring the severity of encephalopathy.
iii. Occurrence of cerebral edema and elevated ICPs is generally related to the severity of hepatic
encephalopathy. Patients with grade I and grade II encephalopathy rarely have cerebral edema,
whereas cerebral edema is present in about 30% of patients with grade III encephalopathy and
about 75% of patients with grade IV encephalopathy.

ACCP Updates in Therapeutics® 2022: Critical Care Pharmacy Preparatory Review and Recertification Course

851
Hepatic Failure/GI/Endocrine Emergencies

Table 3. Grades of Encephalopathy


Grade I Changes in behavior with minimal change in level of consciousness
Gross disorientation
Drowsiness
Grade II
Possibly asterixis
Inappropriate behavior
Marked confusion
Grade III Incoherent speech
Sleeping most of the time but arousable to vocal stimuli
Comatose
Grade IV
Unresponsive to pain, decorticate or decerebrate posturing
Adapted from: Conn HO, Leevy CM, Vlehcevic ZR, et al. Comparison of lactulose and neomycin in the treatment of chronic portal-systemic encephalopathy. A double
blind randomized trial. Gastroenterology 1977;72:573-83.

b. Cardiovascular
i. Primary hemodynamic concern in ALF is low systemic vascular resistance, similar to cirrhosis.
ii. Most patients are severely volume depleted on admission because of poor nutritional status and
third spacing into the extravascular space and will require aggressive fluid resuscitation initially.
iii. In patients with elevated ICPs, maintaining adequate perfusion becomes even more important
in order to preserve adequate perfusion to the brain, and goals for these patients include
maintaining a cerebral perfusion pressure (CPP) of at least 60 mm Hg. Higher mean arterial
pressure (MAP) targets may be needed in these patients in order to maintain CPP goals.
CPP = MAP − ICP.
c. Coagulopathy
i. ALF is defined by the presence of an elevated INR caused by decreased production, together
with increased consumption, of coagulation factors.
ii. Consumption of platelets is also seen, and thrombocytopenia (150,000/mm3 or less) is common.
iii. However, thromboelastography studies of patients with ALF have shown that overall
hemostasis in patients with ALF is maintained by compensatory mechanisms, even in patients
with elevated INR values, potentially because of a reduction in hepatic synthesis of natural
anticoagulants.
iv. Spontaneous bleeding in patients with ALF, though uncommon, is capillary-type bleeding and
usually results from mucosal bleeding in the stomach, lungs, or genitourinary system. Unlike
chronic liver failure, bleeding from esophageal varices generally does not occur.
v. Clinically significant bleeding that requires blood transfusions is rare in ALF.
d. Renal
i. Acute kidney injury with ALF is generally classified as either prerenal injury or acute tubular
necrosis.
(a) Prerenal azotemia typically is caused by vasodilatation owing to portal hypertension and
is worsened by systemic hypoperfusion, similar to hepatorenal syndrome in patients with
cirrhosis.
(b) Acute tubular necrosis typically occurs secondary to drugs or toxins, such as acetaminophen
or Amanita poisoning.
e. Infection
i. Patients with ALF are at high risk of infection because of the presence of indwelling foleys
and central venous catheters in addition to intrinsic monocyte and neutrophil dysfunction.
Infections are of particular concern because they may delay transplantation or be problematic
during the postoperative period.

ACCP Updates in Therapeutics® 2022: Critical Care Pharmacy Preparatory Review and Recertification Course

852
Hepatic Failure/GI/Endocrine Emergencies

ii. The most common infections in ALF are pneumonia, followed by urinary tract infections
and bloodstream infections. The most commonly isolated organisms are gram-positive cocci
(e.g., Staphylococcus, Streptococcus) and enteric gram-negative bacilli. Fungal infections,
particularly those caused by Candida, occur in about one-third of patients with ALF.
f. Metabolic abnormalities
i. Lack of effective glycogenolysis and gluconeogenesis caused by impaired hepatocyte function
places patients at high risk of hypoglycemia.
ii. Symptoms of hypoglycemia can often be difficult to identify in patients with severe
encephalopathy, whereas profound hypoglycemia can worsen an already altered mental state.
2. Therapy must be multimodal to support each of the organ systems affected by ALF.

F. Management of ALF
1. Antidotes
a. Acetaminophen-induced ALF (see Table 4)
i. Although most effective if given within the first hour, GI decontamination with activated
charcoal may be of benefit for up to 4 hours after ingestion and does not reduce the effect of
acetylcysteine.
ii. Administration of acetylcysteine is recommended in all ALF cases in which acetaminophen
is suspected as a cause.
(a) Acetylcysteine can be given either orally or intravenously. Studies have shown similar
outcomes between the two routes; however, in those studies, the main efficacy outcome of
interest was development of hepatotoxicity. When patients present already with symptoms
of hepatotoxicity (as in ALF), intravenous acetylcysteine is recommended. The U.S. Acute
Liver Failure Study Group recommends intravenous therapy for any of the following:
(1) Greater than grade I encephalopathy
(2) Hypotension
(3) If oral therapy cannot be tolerated (e.g., vomiting, compromised airway, ileus)
(b) Many poison centers may extend therapy beyond the recommended course if there is
a detectable acetaminophen concentration or if ALT concentrations continue to remain
elevated at the end of therapy, especially if therapy was initiated more than 8 hours after
ingestion and baseline acetaminophen concentrations were greater than 300 mcg/mL.
(c) Recent studies have evaluated alternative dosing strategies for patients with massive
overdoses, including doubling the last bag (i.e., providing 200 mg/kg over 16 hours)
Massive overdoses are considered ingestions with initial acetaminophen concentrations
greater than 300 mcg/mL; doubling the last bag has been associated with reduced
hepatotoxicity. However, these strategies need to be further evaluated.
(d) Therapy may continue until the signs and symptoms of encephalopathy or coagulopathy
resolve or until the patient receives a liver transplant.
(e) Additional information on treatment of acetaminophen-induced ALF can be found in the
Toxicology chapter.

Table 4. Acetylcysteine for Acetaminophen-Induced ALF


Route Dose
Loading dose: 140 mg/kg × 1 dose
Oral
Maintenance dose: 70 mg/kg every 4 hr × 17 doses (72 hr total)
150 mg/kg (max 15 g) over 60 min, followed by
IV
50 mg/kg (max 5 g) over 4 hr, followed by 100 mg/kg (max 10 g) over 16 hr (21 hr total)
IV = intravenous.

ACCP Updates in Therapeutics® 2022: Critical Care Pharmacy Preparatory Review and Recertification Course

853
Hepatic Failure/GI/Endocrine Emergencies

b. NAI-ALF
i. Acetylcysteine may improve oxidative stress in NAI-ALF by acting as a free radical scavenger.
In addition, acetylcysteine may improve both hepatic and systemic perfusion through its
vasodilatory effects.
ii. A multicenter randomized trial compared intravenous acetylcysteine with placebo for 72 hours
(see Table 5) for treatment of NAI-ALF. Randomized patients were stratified according to
coma grade, with most patients having a low-grade encephalopathy. There was no difference
in the primary outcome of overall survival at 3 weeks between acetylcysteine and placebo;
however, the transplant-free survival rate significantly increased with the use of acetylcysteine
(40% vs. 27%, p=0.04).
(a) The increase in transplant-free survival with acetylcysteine was mainly confined to the
subgroup of patients with encephalopathy grade I and grade II (52% vs. 30% with placebo,
p=0.01).
(b) When outcomes were compared on the basis of each etiology of NAI-ALF, patients with
DILI and hepatitis B virus had more improvement in overall survival and transplant-free
survival from acetylcysteine compared with placebo than with other causes.
iii. Intravenous acetylcysteine has also been studied for the treatment of NAI-ALF as a loading
dose and then a continuous infusion of 150 mg/kg over 24 hours until resolution of coagulopathy
(INR less than 1.3). This strategy increased transplant-free survival compared with a historical
cohort (96.4% vs. 23.3%, p<0.01); however, the strategy needs to be further evaluated.
iv. Oral acetylcysteine has also been evaluated for the treatment of NAI-ALF in a prospective
non-interventional study in which it, compared with a historical cohort, was associated with
reduced mortality. In this study, oral acetylcysteine was given as a 140-mg/kg loading dose,
followed by 70 mg/kg every 4 hours for an additional 17 doses. However, at this time, only
intravenous acetylcysteine has been studied in a randomized controlled trial for NAI-ALF.

Table 5. Acetylcysteine for NAI-ALF


Route Dose
150 mg/kg over 60 min, followed by
Intravenous 12.5 mg/kg/hr for 4 hr, followed by
6.25 mg/kg/hr for 67 hr

Patient Case

1. A 60-year-old woman with rheumatoid arthritis was initiated on azathioprine 3 months ago and now presents
with NAI-ALF secondary to DILI. On presentation, her ALT and AST concentrations are 500 IU/L and 350
IU/L, respectively. Her INR is 1.7, and she is mildly confused and drowsy. Which intervention has been
shown most effective for the treatment of NAI-ALF?
A. Intravenous acetylcysteine 21-hour regimen.
B. Intravenous acetylcysteine 72-hour regimen.
C. Oral acetylcysteine 72-hour regimen.
D. Oral glutamine supplementation.

c. Mushroom poisoning
i. Gastric lavage and activated charcoal may be beneficial for patients still having GI symptoms
indicative of a recent ingestion.

ACCP Updates in Therapeutics® 2022: Critical Care Pharmacy Preparatory Review and Recertification Course

854
Hepatic Failure/GI/Endocrine Emergencies

ii. Although data regarding efficacy are lacking, penicillin G or silibinin (silymarin or milk
thistle) can be used as an antidote to α-amanitin, a toxin released after mushroom ingestion.
Penicillin G directly competes with and inhibits the ability of the toxin to bind to plasma
protein and penetrate the liver. Recommended dose for mushroom poisoning is 300,000 - 1
million units/kg/day given intravenously.
2. Management of neurologic complications
a. Encephalopathy
i. All medications that can cause sedation or confusion should be avoided (i.e., benzodiazepines,
anticholinergics, etc.).
ii. Grade I encephalopathy can typically be managed with close monitoring and without
medication; grade II–IV encephalopathy should be treated in an ICU setting, if possible.
iii. Given its ability to decrease serum ammonia concentrations and experience with treatment of
hepatic encephalopathy in patients with cirrhosis, lactulose is recommended for patients with
ALF with low-grade encephalopathy. The recommended dose is 20–30 g three or four times
daily to produce 2 or 3 soft stools a day.
(a) Despite its proposed benefits, retrospective data analyses of patients with ALF who
receive lactulose therapy have not shown a benefit on encephalopathy or overall outcome.
(b) In addition, lactulose has the potential to cause abdominal distension, which could be
a concern for liver transplantation. Moreover, overuse of lactulose has the potential to
cause intravascular depletion, which may further contribute to hemodynamic instability.
Therefore, its effects may be harmful in the acute setting.
iv. l-Ornithine-l-aspartate up-regulates urea and glutamine synthesis, during which ammonia
is consumed. Theoretically, exogenous supplementation of l-ornithine-l-aspartate should
decrease blood ammonia concentrations. One large randomized controlled trial of ALF
compared l-ornithine-l-aspartate with placebo and found no differences in serum ammonia
concentrations, improvement in encephalopathy, or survival between groups.
v. Patients with grade III and grade IV encephalopathy should be intubated for airway protection
and treated with minimal sedation to allow for more frequent neurologic assessments. If
sedation is necessary, propofol is typically used because it can reduce cerebral blood flow and
lowers ICP.
b. Seizures
i. Seizures have the potential to increase ICP. Therefore, seizures should be controlled quickly
with short-acting benzodiazepines. If seizures persist, antiepileptic agents should be scheduled.
ii. Use of prophylactic antiepileptics is not recommended. Studies have shown that use of
prophylactic phenytoin in patients with ALF has no impact on prevention of seizures, cerebral
edema, or overall survival.
c. Elevated ICPs
i. ICP should be kept less than 20–25 mm Hg while preserving CPP of at least 60 mm Hg.
ii. Routine ICP monitoring has not been shown to reduce mortality in patients with ALF, and
routine placement of ICP monitors is not recommended in all patients. Clinicians may choose
to place an ICP monitor in patients with high-grade encephalopathy (grades III and IV) to
provide close monitoring of cerebral edema. In addition, some centers may use noninvasive
ICP monitoring strategies such as transcranial Doppler or optic nerve depth measurement.
However, these monitoring strategies should be reserved for centers with large neurocritical
care and neurosurgical experience.

ACCP Updates in Therapeutics® 2022: Critical Care Pharmacy Preparatory Review and Recertification Course

855
Hepatic Failure/GI/Endocrine Emergencies

iii. Osmotic agents are used first line for control of ICP.
(a) Mannitol has been used effectively in acutely reducing ICP in patients with ALF, though
the effect is usually transient.
(1) Mannitol is given as 0.5–1 g/kg intravenously once, which may be repeated to effect
as long as the serum osmolality is less than 320 mOsm/L; however, mannitol is
typically ineffective if the baseline ICP is greater than 60 mm Hg.
(A) Serum osmolality is estimated based on serum sodium, glucose, and blood urea
nitrogen based on the following formula: (Na x 2) + (glucose/18) + (blood urea
nitrogen/2.8).
(2) Adverse effects to consider for mannitol administration include fluid overload,
particularly in patients with renal impairment, hyperosmolarity, hypotension, and
hypernatremia.
(3) Hypertonic saline bolus (23.4% 30-mL bolus or 3% 200- to 300-mL bolus) is
an alternative to mannitol for acute reductions in ICP. Adverse effects include
hypernatremia and hyperchloremia.
(b) In patients with grade III or grade IV encephalopathy, multiorgan failure, or hemodynamic
instability, prophylactic hypertonic saline may be used to reduce the risk of cerebral
edema.
(1) In a small, randomized controlled trial, 30 patients with ALF and grade III or grade
IV encephalopathy were randomized to receive prophylactic hypertonic saline to
maintain a serum sodium of 145–155 mEq/L compared with patients maintained at
near-normal serum sodium levels (137–142 mEq/L). The primary outcome, incidence
of ICP defined as elevations greater than 25 mm Hg, was significantly decreased in
the hypertonic saline group (20% hypertonic saline vs. 46.7% control, p=0.04).
(2) Hypertonic saline in this study was administered as a 30% sodium chloride infusion
by a syringe at 5–20 mL/hour; however, many preparation and dosing strategies have
been used (e.g., 23.4% 30-mL bolus, 7.5% 2-mL/kg bolus, 3% 200- to 300-mL bolus,
or continuous infusion), and the goal should be to target a serum sodium of 145–155
mEq/L.
iv. When severe ICP elevations do not respond to other measures, barbiturates such as pentobarbital
may be used to control ICP.
(a) Profound hypotension may limit barbiturate use in ALF when patients have hemodynamic
instability at baseline. Patients may require vasopressors to maintain adequate MAP (and
CPP) while receiving barbiturates.
(b) Barbiturate clearance is significantly decreased in patients with ALF, which may limit
clinicians’ ability to perform neurological assessments for extended periods.
v. Hyperventilation to a Paco2 of 25–30 mm Hg can restore cerebral autoregulation, which results
in vasoconstriction and decreased ICP.
(a) The effects of hyperventilation on ICP appear to be short-lived. A randomized controlled
trial of prophylactic hyperventilation showed no benefit on cerebral edema and survival.
In addition, there is concern that cerebral vasoconstriction with hyperventilation may
worsen cerebral hypoxia.
(b) Thus, prophylactic hyperventilation currently plays no role; it is only used for acute
control of ICP elevations.
vi. Hypothermia (33°C–34°C) may control ICP in patients with ALF by lowering the production of
ammonia and decreasing the cerebral uptake of ammonia as well as decreasing cerebral blood
flow. However, hypothermia for patients with ALF has not been compared with normothermia
in controlled trials, and a recent retrospective cohort study showed no difference in overall and
transplant-free survival compared with normothermia. In addition, there are concerns about
coagulation disturbances and increased risk of infection with hypothermia.

ACCP Updates in Therapeutics® 2022: Critical Care Pharmacy Preparatory Review and Recertification Course

856
Hepatic Failure/GI/Endocrine Emergencies

Patient Case

2. A 33-year-old man presents with ALF secondary to acetaminophen overdose. He is now 72 hours post-inges-
tion and is profoundly encephalopathic and unresponsive to pain on examination. An ICP monitor is placed,
which shows acute elevations of 30 mm Hg. Which is most appropriate for the acute management of ICP
elevations?
A. Hypertonic saline continuous infusion to maintain serum sodium 145–155 mEq/L.
B. Mannitol 0.5 mg/kg intravenously × 1.
C. Hyperventilation to Paco2 of 25–30 mm Hg.
D. Thiopental continuous infusion.

3. Management of hemodynamic instability


a. Patients should initially be resuscitated with 0.9% sodium chloride. Hypotonic fluids including
lactated Ringer solution (273 mOsm/L) should be avoided, if possible, in patients with grade III
and grade IV encephalopathy because of the risk of cerebral edema. In addition, Plasma-Lyte (294
mOsm/L) could be used as an alternative to 0.9% sodium chloride, given that it is not hypo-osmotic
and would provide a balanced solution to avoid the risk of hyperchloremic metabolic acidosis.
b. Vasopressors should be used if fluid resuscitation fails to maintain a MAP greater than 75 mm Hg
or a CPP of at least 60 mm Hg.
i. Norepinephrine is the vasopressor of choice in patients requiring vasopressor support.
ii. Use of vasopressin is controversial for patients with ALF who have high-grade encephalopathy.
One study of six patients with grade IV encephalopathy showed an increased ICP after 1
hour of terlipressin, though systemic hemodynamics were not significantly altered; however,
a subsequent study did not show similar effects on ICP with terlipressin. Because there have
been no similar studies of vasopressin in patients with ALF, its use in patients with ALF and
high-grade encephalopathy should be cautioned. However, in the most recent supportive care
management guidelines for patients with acute liver failure, vasopressin is recommended to be
added to norepinephrine with persistent shock, despite these concerns.
4. Management of coagulopathies
a. Although the INR may be elevated in patients with ALF, overall hemostasis is maintained through
compensatory mechanisms.
b. Because traditional measures of coagulation may be unreliable, guidelines suggest use of
viscoelastic testing such as thromboelastography over measuring INR, platelets, and fibrinogen
when available. In a small randomized trial of 60 cirrhotic patients, blood product transfusion
guided by thromboelastography was significantly reduced compared with blood product transfusion
guided by traditional measures with no differences in bleeding complications. Unfortunately, these
tests are not available at many institutions and traditional measures are often still required.
c. In patients with an elevated INR without signs and symptoms of an acute bleed, INR should not
be corrected using fresh frozen plasma. Vitamin K (5–10 mg) may be administered because many
patients with ALF are deficient in vitamin K, further contributing to the coagulopathy of ALF.
Intravenous administration is usually recommended because subcutaneous administration of
vitamin K can lead to erratic absorption, and enteral absorption may also be unreliable.

ACCP Updates in Therapeutics® 2022: Critical Care Pharmacy Preparatory Review and Recertification Course

857
Hepatic Failure/GI/Endocrine Emergencies

d. If clinically significant bleeding occurs, INR correction with fresh frozen plasma is warranted.
i. Guidelines recommend an INR correction to about 1.5 for clinically significant bleeding.
ii. If a fresh frozen plasma infusion alone does not adequately lower INR, recombinant activated
factor VII (rFVIIa) may be administered. In a small nonrandomized study of 15 patients
with fulminant hepatic failure, administration of rFVIIa at a dose of 40 mcg/kg temporarily
improved coagulation parameters (i.e., INR less than 1.6) compared with patients receiving only
fresh frozen plasma (100% vs. 0%, p<0.002). Although the improvements in coagulation were
only temporary, patients in the rFVIIa group were able to have invasive procedures performed
(e.g., ICP monitors placed) more often than were patients in the control group (100% vs. 38%,
p=0.03).
iii. Data are limited on use of prothrombin complex concentrates for reversing coagulopathy in
ALF. One small study of liver disease (only four patients with ALF) found that significantly
fewer patients with liver disease achieved an INR of less than 1.5 or hemostasis than those
without liver disease.
e. To reduce the risk of spontaneous intracranial hemorrhage, platelet transfusions should be provided
if the count drops to less than 15,000–20,000/mm3 in the absence of bleeding.
i. If clinically significant bleeding occurs, patients should be transfused to a target platelet count
greater than 50,000/mm3.
ii. For invasive procedures, platelet counts should be 50,000–70,000/mm3 to prevent bleeding,
though thromboelastography data analyses suggest that a target of 100,000/mm3 is ideal.
f. Patients with ALF should be evaluated for venous thromboembolism prophylaxis based on
traditional risk assessments (e.g., thrombocytopenia, recent and/or active bleeding, etc). Venous
thromboembolism prophylaxis should not be withheld due to elevated INR because INR often
overestimates hypercoaguability in patients with ALF. Selection of agent for prophylaxis is unclear.
No studies have compared low-molecular-weight heparin with unfractionated heparin in patients
with ALF; however, most studies of patients with cirrhosis support the use of LMWH, though the
control group is no-prophylaxis.
g. Histamine-2 receptor antagonists or PPIs should be initiated in patients with ALF to reduce the
incidence of spontaneous GI bleeding.
5. Management of infectious complications
a. Antimicrobial prophylaxis is often given because infection remains the primary cause of death in
patients with ALF. However, data analyses are limited on the benefit of antimicrobial prophylaxis
in ALF. A retrospective cohort study of 1551 patients with ALF showed that prophylactic antibiotic
therapy did not reduce the incidence of bloodstream infections (12.8% in the prophylaxis group vs.
15.7% in the non-prophylaxis group, p=0.12) and did not reduce 21-day mortality.
b. Patients should be monitored closely for infection through surveillance cultures, and antimicrobials
should be initiated promptly if the patient has any signs or symptoms of a systemic infection.
Empiric administration is recommended for any of the following scenarios:
i. Positive surveillance cultures
ii. Progression to higher-grade encephalopathy
iii. Refractory hypotension
iv. Development of systemic inflammatory response syndrome (SIRS) criteria
6. Miscellaneous – Corticosteroids
a. Corticosteroid therapy may be initiated for patients with ALF caused by autoimmune hepatitis,
particularly if they have early-stage disease without multiorgan failure.
b. Typical dose is prednisone 40–60 mg/day.
c. For patients with late-stage disease (i.e., high-grade encephalopathy, multiorgan failure), decisions
regarding liver transplantation should not be delayed while awaiting a response from steroid
therapy.

ACCP Updates in Therapeutics® 2022: Critical Care Pharmacy Preparatory Review and Recertification Course

858
Hepatic Failure/GI/Endocrine Emergencies

G. Advanced Therapies for ALF


1. Liver assist devices: The molecular adsorbent recirculating system (MARS) has been studied for the
treatment of ALF. In a large propensity-matched study of 104 patients who received MARS for ALF
(matched 4:1 to 416 controls), MARS significantly increased the odds of 21-day transplant-free survival
(OR 1.9; 95% CI, 1.07–3.39; p=0.03).
2. Transplantation
a. Orthotopic liver transplantation is the only definitive treatment for patients with ALF.
b. The advances in liver transplantation have improved ALF survival to greater than 60%.

H. Prognosis
1. Predictive models of survival and need for liver transplantation are useful to identify the patients most
likely to benefit from liver transplantation.
2. The most important predictor of outcome seems to be the cause of ALF. Transplant-free survival is
50% or greater when acetaminophen, hepatitis A, acute ischemic injury, or pregnancy is the cause,
whereas other causes confer less than 25% transplant-free survival. In addition, transplant-free survival
is significantly decreased when patients present with grade III or grade IV encephalopathy compared
with grade I or grade II encephalopathy.
3. King’s College criteria, developed from a cohort of about 600 patients with ALF, incorporate parameters
such as cause of ALF and clinical parameters, including degree of encephalopathy and liver function
tests, in order to evaluate the decision to perform transplantation versus provide medical therapy.

II.  ACUTE PANCREATITIS

A. Epidemiology
1. AP is responsible for 275,000 hospitalizations per year in the United States.
2. In 2009, AP was the leading gastroenterology discharge diagnosis in the United States, with an annual
cost of $2.6 billion.
3. Incidence is estimated at 5–30 cases per 100,000.
4. AP is classified according to disease severity. Overall, about 20% of patients with AP have a severe
course with an overall mortality rate of approximately 5%.

B. Definitions
1. The clinical diagnosis of AP is based on characteristic symptoms (i.e., abdominal pain and nausea),
together with elevated serum concentrations of pancreatic enzymes. According to the Acute Pancreatitis
Classification Working Group definition, two of the following three criteria must be met for a patient to
be given a diagnosis of AP:
a. Abdominal pain consistent with AP. This is typically persistent epigastric pain, sometimes
radiating to the back.
b. Serum lipase (or amylase) concentrations greater than 3 times the upper limit of normal
c. Imaging (computed tomography [CT], magnetic resonance imaging [MRI], or transabdominal
ultrasonography) consistent with pancreatitis
2. AP is classified according to disease severity as mild, moderately severe, or severe.
a. Mild AP: Tends to be self-limiting (less than 48 hours) with no organ failure or necrosis. There is
also the absence of local and systemic complications.
b. Moderately severe AP: Characterized by local complications and organ failure that lasts less than
48 hours. Local complications for moderately severe AP resolve without intervention.

ACCP Updates in Therapeutics® 2022: Critical Care Pharmacy Preparatory Review and Recertification Course

859
Hepatic Failure/GI/Endocrine Emergencies

c. Severe AP: Characterized by persistent organ failure for more than 48 hours. Patients with severe
AP usually have one or more local complications. Mortality rates are highest for severe AP,
particularly if persistent organ failure develops within the first few days of disease.
3. AP is further classified according to the phase. There are two distinct phases in this disease: early and
late. The early phase usually lasts for the first week, and the late phase can last for weeks to months:
a. Early: Symptoms of SIRS with or without transient organ failure during the first week. Inflammatory
responses and organ failure are mainly because of a response to local pancreatic injury.
b. Late: Complications that occur after 1 week that can last weeks to months. This phase is
characterized by persistent organ failure and the presence of local complications. The late phase
occurs only in those with moderately severe or severe AP.
4. Local complications with moderately severe and severe AP may include:
a. Peripancreatic fluid collections
b. Pancreatic and peripancreatic necrosis (sterile or infected). Pancreatic necrosis is an area of
nonviable pancreatic parenchyma identified by lack of enhancement on imaging, generally
involving greater than 30% of the pancreas.
c. Pseudocysts
d. Walled-off necrosis (sterile or infected)
5. Organ failure associated with AP is typically cardiovascular, respiratory, or renal dysfunction. The
modified Marshall scoring system is recommended to identify patients with AP with organ failure. A
score of 2 or more for one of these three systems indicates organ failure.

C. Pathophysiology – AP is primarily caused by inappropriate activation of trypsinogen to trypsin. Trypsin


is the key enzyme responsible for the activation of pancreatic zymogens. When trypsin is inappropriately
formed and retained in the pancreas, activation of digestive enzymes inside the pancreas causes pancreatic
autodigestion and pancreatic injury.

D. Causes
1. Gallstones
a. Gallstone pancreatitis is the main cause of AP, responsible for 40%–70% of cases.
b. Gallstone pancreatitis occurs primarily in white women older than 60 years and in patients with
small stones (less than 5 mm in diameter).
c. Gallstone pancreatitis is usually an acute event that resolves once the gallstone has been removed
or has passed.
d. Patients may undergo a cholecystectomy to prevent further episodes in the future.
2. Alcohol use
a. Excessive alcohol use is the etiology of pancreatitis in 25%–35% of cases.
b. Symptoms can occur as an acute episode or present as a chronic pancreatitis.
c. Alcohol-induced pancreatitis is more common in men than in women.
d. Alcohol use and its association with pancreatitis is thought to have a dose-dependent relationship.
Alcohol intake for more than 5 years or greater than 50 g daily increases the risk of pancreatitis.
However, only about 5% of patients with a history of heavy alcohol consumption develop AP.
3. Hypertriglyceridemia
a. Primary and secondary hypertriglyceridemia are increasingly a cause of AP (5%–22% reported).
b. If serum triglycerides are greater than 1000 mg/dL, hypertriglyceridemia should be suspected as
the cause.
4. Certain medications can cause pancreatitis through varied mechanisms. Examples include:
a. Angiotensin-converting enzyme (ACE) inhibitors
b. Estrogens
c. HAART (highly active antiretroviral therapy) medications (e.g., didanosine)

ACCP Updates in Therapeutics® 2022: Critical Care Pharmacy Preparatory Review and Recertification Course

860
Hepatic Failure/GI/Endocrine Emergencies

d. Thiazide diuretics (e.g., hydrochlorothiazide)


e. Valproic acid
5. Malignancy – The presence of a pancreatic tumor blocking the main pancreatic duct should be suspected
in any patient older than 40 years with signs and symptoms of pancreatitis and no other apparent cause.
6. About 20% of AP cases are idiopathic.

E. Diagnosis
1. Signs and symptoms
a. Abdominal pain
i. AP pain is usually located in the epigastric region or left upper quadrant; however, it may
radiate to the back, chest, or flank. Pain is typically constant and severe.
ii. Gallstone pancreatitis–induced pain has been described as knifelike.
b. Nausea
2. Laboratory abnormalities
a. According to the Acute Pancreatitis Classification Working Group definition of AP, serum lipase
(or amylase) concentrations at least 3 times the upper limit of normal are required for diagnosis.
b. Serum lipase is preferred for diagnosis because elevations in serum lipase concentrations are more
specific to the diagnosis of AP than elevations in amylase and serum lipase concentrations remain
elevated longer than amylase concentrations.
i. Amylase concentrations rise quickly (within a few hours) in AP; however, they return to
normal within a few days.
ii. Amylase concentrations may remain normal in alcohol-induced AP and hypertriglyceridemia.
3. Imaging
a. Transabdominal ultrasonography should be done to confirm the diagnosis of AP for all patients.
b. Contrast-enhanced CT scans of the abdomen are more than 90% sensitive and specific in diagnosing
AP; however, routine use is unnecessary.
c. Contrast-enhanced CT and/or MRI should be used in patients with an unclear diagnosis or in
patients who do not improve after 48–72 hours to evaluate for complications.

F. Management
1. Hydration
a. Patients with AP need early aggressive volume resuscitation because they are volume depleted
for many reasons, including vomiting, reduced oral intake, third spacing of fluids because of
inflammatory response, and diaphoresis. In fact, it has been hypothesized that worsening pancreatic
hypoperfusion that develops from pancreatic inflammation in the setting of volume depletion leads
to pancreatic necrosis. Thus, early aggressive volume resuscitation may prevent the development
of pancreatic necrosis.
b. Early aggressive volume resuscitation (i.e., within the first 24 hours) has been shown to be more
effective than later aggressive volume resuscitation. Studies that have continuously used aggressive
hydration strategies beyond the first 24 hours of presentation of AP have not shown benefit.
i. Early aggressive volume resuscitation is defined in guidelines as either 5–10 mL/kg/hr or
250–500 mL/hr for the first 12–24 hours with either non-invasive measures (e.g., heart rate,
mean arterial pressure, urine output, etc.) or invasive measures (e.g., stroke volume variation,
etc.) to determine fluid response.

ACCP Updates in Therapeutics® 2022: Critical Care Pharmacy Preparatory Review and Recertification Course

861
Hepatic Failure/GI/Endocrine Emergencies

c. Selection of intravenous fluid for initial volume resuscitation in AP may be important, favoring
pH-balanced fluids. Low pH activates trypsinogen, implying that low-pH fluids exacerbate
inflammatory response in AP. In a randomized controlled trial comparing initial resuscitation
(within the first 24 hours) with lactated Ringer solution to 0.9% sodium chloride for patients with
AP, there was a significant reduction in SIRS after 24 hours in patients resuscitated with lactated
Ringer compared with 0.9% sodium chloride (84% reduction vs. 0%, respectively, p=0.035). The
study also showed a significant reduction in C-reactive protein concentrations with administration
of lactated Ringer solution compared with 0.9% sodium chloride. However, two recent randomized
controlled trials comparing resuscitation with lactated Ringer solution and 0.9% sodium chloride
showed similar early (within first 24 hours) reductions in SIRS with lactated Ringer solution,
but no differences in SIRS prevalence after 24 hours or clinical outcomes, including mortality.
Thus, because no study has shown meaningful clinical differences in fluid selection, the current
guidelines make no recommendation on intravenous fluid of choice.
d. Because there can be harm in over-resuscitating patients, especially those with concomitant
cardiovascular or renal failure, aggressive hydration may not be beneficial if there is no response
within the first 6–12 hours.
2. Etiology-specific treatments
a. Insulin for hypertriglyceridemia-induced AP
i. High-dose insulin infusion is the treatment mainstay for severe AP secondary to
hypertriglyceridemia.
ii. Insulin is initiated at 0.1 units/kg/hour, with dose adjusted on the basis of serum triglyceride
concentrations, which are typically checked every 12 hours. If serum triglycerides do not
decrease by at least 25%–50% in 24 hours, the insulin infusion is increased by 0.05 units/kg/
hour until the goal serum triglyceride is achieved (ideally less than 500 mg/dL).
iii. To prevent hypoglycemia with high-dose insulin, blood glucose (BG) should be monitored
hourly, and a dextrose infusion should be initiated and adjusted to maintain the BG at around
150 mg/dL.
b. Endoscopic retrograde cholangiopancreatography (ERCP)
i. Although many gallstones are passed through the duodenum and lost in the stool without causing
harm to the patient, gallstones that are not cleared can cause an obstruction in some patients in
either the biliary tree or the pancreatic duct, which can lead to severe AP or cholangitis.
ii. An ERCP, together with a sphincterotomy, may be used to extract gallstones from the pancreatic
ducts or biliary tree.
iii. There is considerable risk with ERCP, including bleeding and the potential to worsen AP
because of manipulation of the pancreas.
iv. Eight randomized controlled trials have evaluated the effectiveness of early ERCP in reducing
the risk of complications in patients with gallstone AP. A limitation of these studies is that many
did not exclude acute cholangitis, which is an indication for ERCP without AP. A systematic
review controlling for patients with cholangitis found no benefit of early ERCP (mortality OR
0.67; 95% CI, 0.26–1.75). Given these limitations and results, the current guidelines do not
recommend routine use of ERCP in patients without cholangitis.
v. AP is the most common complication of ERCP. Although the incidence of post-ERCP
pancreatitis has decreased significantly during the past few decades, post-ERCP pancreatitis
continues to occur in 2%–4% of patients.
(a) Rectal NSAIDs (i.e., 100 mg of indomethacin) after an ERCP can be given to patients at
high risk of post-ERCP pancreatitis.

ACCP Updates in Therapeutics® 2022: Critical Care Pharmacy Preparatory Review and Recertification Course

862
Hepatic Failure/GI/Endocrine Emergencies

(b) In a multicenter randomized controlled trial of 600 high-risk patients undergoing


ERCP, 100 mg of rectal indomethacin (administered as two 50-mg suppositories given
immediately post-ERCP) reduced the incidence of AP compared with placebo (9.2% vs.
16.9%, p=0.005) and, specifically, the development of moderate to severe AP (4.4% vs.
8.8%, p=0.03).
3. Infection
a. Pancreatic and non-pancreatic infections contribute to mortality in patients with AP. The primary
infectious concern with AP is infected necrotizing pancreatitis, which confers a high mortality
rate (about 30%); thus, patients with severe acute necrotizing pancreatitis are at highest risk of
pancreatic infections.
b. The role of prophylactic antibiotics is controversial in AP.
i. Early randomized trials showed a potential benefit in the reduction of infectious complications,
particularly central line–related bloodstream infections, pulmonary infections, urinary
infections, and pancreatic infections; however, the studies were unblinded, and the benefit was
mostly confined to patients with severe AP.
ii. In more recent literature with higher-quality trials, the benefit of prophylactic antibiotic is
unsupported. A systematic review of 10 randomized controlled trials showed a trend toward
reduced mortality with prophylactic antibiotics (OR 0.66; 95% CI, 0.42–1.04); however, the
mortality benefit was lost among the subgroup of recent studies (after 2002: OR 0.85; 95% CI,
0.52–1.80). Similarly, there was no difference in risk of infected necrosis in more recent trials
(OR 0.81; 95% CI, 0.44–1.49).
c. According to the available information, use of prophylactic antibiotics to prevent the development
of infected pancreatic necrosis is not recommended at this time.
d. For patients who have not improved after 7–10 days of hospitalization with pancreatic necrosis,
an infection should be suspected, and empiric antibiotics should be initiated at that time.
Because of penetration issues, only carbapenems, fluoroquinolones, metronidazole, or high-dose
cephalosporins should be used.
4. Nutrition
a. Historically, patients were kept NPO in order to rest the pancreas and prevent the further release of
pancreatic enzymes. Patients were given TPN until complete resolution of AP.
i. This practice has largely fallen out of favor because several studies have shown that early
enteral nutrition is safe and effective for patients with AP. In addition, a recent meta-analysis
of 11 randomized trials concluded that early (within 24–48 hours) feedings were not associated
with increased adverse effects and may reduce hospital length of stay, particularly for mild to
moderate AP. Enteral feeding maintains the gut mucosal barrier and helps prevent infectious
complications such as infected necrosis, which may result from bacterial translocation from
the gut.
ii. A meta-analysis of eight trials comparing enteral nutrition with TPN in patients with AP showed
decreased mortality (relative risk [RR] 0.5; 95% CI, 0.28–0.91), infectious complications
(RR 0.39; 95% CI, 0.23–0.65), multiorgan failure (RR 0.55; 95% CI, 0.37–0.81), and surgical
interventions (RR 0.44; 95% CI, 0.29–0.67) with the use of enteral nutrition. Enteral nutrition
in these trials was given by the NJ route.
iii. TPN is mainly reserved for patients unable to meet caloric demands with enteral feeding.
b. Although the NJ route for enteral feeding has been preferred because it avoids the gastric area
where pancreatic enzyme stimulation may occur, the NG route appears to be safe.
i. A recent meta-analysis of three randomized controlled trials showed no significant differences
in mortality, tracheal aspiration, or proportion meeting energy balance between the two routes;
however, the results were limited by small sample sizes.

ACCP Updates in Therapeutics® 2022: Critical Care Pharmacy Preparatory Review and Recertification Course

863
Hepatic Failure/GI/Endocrine Emergencies

ii. NG feeding is easier than NJ feeding because NJ tubes can be difficult to place, expensive, and
inconvenient. However, there is concern that NG feeding may increase the risk of aspiration
pneumonitis, particularly in patients with a history of aspiration.
5. Surgery
a. Cholecystectomy should be performed in patients with gallstones retained in the gallbladder to
prevent the recurrence of AP. For necrotizing biliary AP, cholecystectomy should be delayed until
inflammation is resolved and fluid collections are cleared in order to avoid infection.
b. Surgery is generally unnecessary for asymptomatic patients with pseudocysts and pancreatic or
extrapancreatic necrosis.
c. Surgical debridement of sterile necrosis is only necessary if gastric outlet obstruction or bile duct
obstruction is present.
d. Surgical intervention is the treatment of choice for infected necrotizing pancreatitis.
i. For stable patients with infected necrotizing pancreatitis, surgical debridement should be
delayed for at least 4 weeks to allow appropriate delineation of necrotic versus non-necrotic
tissue, and antibiotics should be tried before surgical intervention.
ii. Unstable patients with infected necrosis should undergo immediate debridement, and
necrosectomy may be required in patients who do not respond to a combination of antibiotics
and debridement.

Patient Case

3. A 44-year-old woman presents to the ED with a 2-day history of diffuse abdominal pain. Amylase is 700
IU/L, and lipase is 800 IU/L. She is given intravenous fluids with lactated Ringer solution and enteral
nutrition through an NJ tube. However, after 48 hours, she has still not improved, and CT imaging reveals
pancreatic necrosis involving 40% of the pancreas. Which interventions would best be performed at this
time?
A. Piperacillin/tazobactam should be prophylactically administered to prevent infected necrotizing
pancreatitis.
B. Surgical management of necrotizing pancreatitis is necessary.
C. Antibiotics should be deferred unless systemic signs of infection are present.
D. Meropenem should be prophylactically administered to prevent infected necrotizing pancreatitis.

III.  GASTROINTESTINAL FISTULAS

A. Epidemiology
1. About 40% of patients with Crohn disease will develop a spontaneous fistula in their lifetime, and up to
12% of patients with diverticulitis will develop a spontaneous fistula.
2. Incidence of postoperative fistula formation varies from less than 1% to about 20%, depending on the
type of abdominal surgery, with the rate of fistula increasing with more complex surgical procedures
and more complicated resections/anastomoses.
3. In the past 50 years, the mortality rate associated with fistulas has decreased significantly from greater
than 40% to currently around 20%, mainly because of improvements in supportive care and the advent
of nutritional support.

ACCP Updates in Therapeutics® 2022: Critical Care Pharmacy Preparatory Review and Recertification Course

864
Hepatic Failure/GI/Endocrine Emergencies

4. GI fistulas remain a source of considerable morbidity with prolonged hospital courses, infections, and
malnutrition.

B. Definition
1. A fistula is any abnormal connection between two epithelialized surfaces. GI fistulas involve an
abnormal connection between the GI tract and the skin, another internal organ, or an internal cavity
such as the peritoneal or pleural space.
2. Classification systems for fistulas:
a. Anatomic: Describe the fistula origin and drainage point
i. Internal: Connection to another internal organ or internal cavity (i.e., ileocolic)
ii. External: Connection to the skin (i.e., enterocutaneous)
b. Physiologic: Classified according to daily fistula output. Daily fistula output is one of the most
important determinants of morbidity and mortality, and it generally decreases before spontaneous
closure.
i. High output: Greater than 500 mL/day. High-output fistulas are associated with a mortality
rate of about 35%.
ii. Moderate output: 200–500 mL/day
iii. Low output: Less than 200 mL/day
c. Fistulas can also be described according to whether they maintain continuity with the GI tract.
i. Lateral fistulas divert off the intestines while maintaining the continuity of the intestinal tract.
With lateral fistulas, intestinal contents follow normal progression beyond the fistula.
ii. End fistulas disrupt the continuity of the intestinal tract beyond the fistula.

C. Causes
1. Postoperative fistulas
a. Most fistulas are formed after surgery (about 80%), most commonly after operations for cancer.
b. Postoperative fistulas form because of either infection or breakdown at intestinal anastomoses.
c. Postoperative fistulas are more commonly external, often because of the presence of a drain.
2. Spontaneous fistulas
a. 15%–25% of GI fistulas occur spontaneously.
b. Crohn disease and inflammatory bowel disease are the leading cause of spontaneous fistula
formation, though pancreatitis and cancer (particularly if radiation therapy is involved) can also
lead to spontaneous fistulas.
c. Spontaneous fistulas often form because of local inflammatory processes that can cause local
abscess formation or perforation.
d. Spontaneous fistulas can be either external or internal.
3. Trauma-induced fistulas
a. Some fistulas are caused by trauma (abdominal wounds or blunt trauma).
b. Trauma may lead to fistula formation by causing vascular injury.

D. Diagnosis
1. Common presenting symptoms include pain, abdominal tenderness, leukocytosis, and fever. External
fistulas are generally easier to recognize because of the presence of effluent at the drainage sites. Patients
with internal fistulas may present with nonspecific symptoms such as diarrhea, dyspnea, or SIRS.
2. In postoperative patients, the first indication of a fistula is delayed recovery, which usually occurs
within the first week after surgery.

ACCP Updates in Therapeutics® 2022: Critical Care Pharmacy Preparatory Review and Recertification Course

865
Hepatic Failure/GI/Endocrine Emergencies

3. Diagnostic workup should include fistula output volume, fistula output description (e.g., color,
consistency), biochemical evaluation of fistula content (e.g., water-electrolyte balance, amylase, lipase,
pH), microbiological evaluation of fistula content, and markers of nutritional status.
4. Radiographic contrast studies using CT or MRI are necessary to determine the anatomic aspects of the
fistula (e.g., origin, length of fistula, presence of obstruction or abscesses). Barium is generally used for
contrast because of its ability to show mucosal surfaces.

E. Treatment
1. Fluid resuscitation and electrolyte management
a. GI fistula fluid is typically iso-osmotic and rich in sodium, potassium, chloride, and bicarbonate.
High-output fistulas can result in large fluid and electrolyte imbalances leading to dehydration,
hypokalemia, hyponatremia, and metabolic acidosis.
b. Fistula output from the upper GI tract is generally more acidic and rich in potassium. Replacement
fluid should include a maintenance infusion of 0.9% sodium chloride with potassium and frequent
reassessments for potassium corrections.
c. Pancreatic and duodenal fistulas result in bicarbonate losses and may require bicarbonate
replacement.
d. Fistula fluid composition may be analyzed in order to correctly replete fluid and electrolyte deficits.
2. Drainage
a. Drainage is used to prevent the accumulation of fluid and the development of infection.
b. Most enterocutaneous fistulas will drain to skin spontaneously; however, some fluid may be
retained in the fistula tract, which may lead to infection.
c. Vacuum-assisted closure (VAC) devices administer negative pressure wound therapy and can
increase blood flow and decrease fluid collections. For enterocutaneous fistulas, a VAC system can
help protect skin and decrease fistula output. Anecdotal evidence with VAC systems has shown
increased spontaneous closure rates within 90 days.
3. Nutrition
a. Nutritional deficiencies are present in 55%–90% of patients with GI fistulas, particularly with
upper GI fistulas, because there is substantial fluid, electrolyte, and protein loss from the upper GI
tract.
b. Because of nutritional deficits, patients may need nutritional supplementation in excess of daily
demands. Patients with low-output fistulas may need additional protein, and patients with high-
output fistulas may need additional daily caloric and protein requirements (e.g., 1.5–2 times basal
daily expenditure).
c. Enteral feedings are the preferred method for nutritional supplementation because enteral feedings
provide direct stimulation to the enterocyte, which may enhance mucosal proliferation.
i. In a retrospective cohort study of 335 patients with high-output small intestine fistulas (median
output 1350 mL/day), 85% (285) were treated with enteral nutrition. Spontaneous closure rates
were acceptable.
ii. Tolerability may limit the use of enteral feeding (e.g., high gastric residuals, diarrhea).
d. TPN has the potential advantage of improving spontaneous closure rates because of reductions in
GI secretions and reduction in fistula output volumes, particularly for patients with high-output
fistulas and when combined with antisecretory agents (e.g., octreotide).
e. Fistula site may also influence the choice of enteral nutrition compared with TPN. In general, enteral
feeding is provided for fistulas from the esophagus, lower small intestine, and colon, whereas TPN
is used for fistulas from the stomach, pancreas, or upper small intestine.

ACCP Updates in Therapeutics® 2022: Critical Care Pharmacy Preparatory Review and Recertification Course

866
Hepatic Failure/GI/Endocrine Emergencies

f. Some evidence supports adding supplements to enteral feeding. These include fish oil, omega-3
fatty acids, and glutamine, which may improve immune function or increase blood flow to the
intestinal tissue. In a study of 28 patients with high-output fistulas, patients who received glutamine
supplementation (0.3 g/kg/day orally) in addition to TPN were significantly more likely to have
spontaneous fistula closure.
4. Somatostatin and octreotide
a. Somatostatin is a tetradecapeptide that is naturally found in the GI tract and the nervous system.
i. It has several biological effects, including inhibition of hormone secretion (i.e., gastrin,
cholecystokinin, secretin, insulin, glucagon), inhibition of exocrine secretory response (i.e.,
gastric acid and pancreatic secretion), inhibition of motor activity in the GI tract, inhibition of
nutrient absorption, and stimulation of water and electrolyte absorption.
ii. Octreotide is an octapeptide synthetic analog of somatostatin with similar activity.
iii. Because output volume is correlated with spontaneous closure, drugs such as somatostatin and
octreotide are used to reduce output volumes.
b. Somatostatin vs. octreotide
i. Somatostatin has a very short half-life (about 1–2 minutes), which requires a continuous
infusion, whereas octreotide has a half-life of around 2 hours, allowing for intermittent dosing
(three times daily).
ii. Effect of octreotide appears to diminish with repeated dosing, possibly because of down-
regulation of somatostatin receptors.
iii. Somatostatin is active at all somatostatin receptors, whereas octreotide has variable affinity at
the somatostatin receptors.
c. Somatostatin significantly reduces output volumes. One prospective, randomized controlled
single-center trial compared somatostatin 250 mcg/hour intravenously continuously with placebo
for patients receiving TPN. Somatostatin significantly reduced the time to achieve a 50%, 75%, and
100% reduction in fistula output compared with placebo; also, although there was no difference in
the rates of fistula closure (85% vs. 81.25%), the time to fistula closure was significantly reduced
with the use of somatostatin (13.9 days vs. 20.4 days, p<0.05).
d. Octreotide has been shown to decrease fistula output in some studies, though in other trials, it had
no effect on fistula output.
i. One small study of 14 patients showed a beneficial effect of octreotide on output volumes.
In this crossover study, octreotide at 100 mcg subcutaneously three times daily significantly
reduced fistula output compared with placebo for the first 2 days of therapy by about 400 mL/
day. When the group that was originally randomized to receive octreotide crossed over to the
placebo arm, output increased by about 250 mL/day.
ii. Two subsequent studies did not show similar results on fistula output. Possible reasons for
decreased efficacy with octreotide include diminished response with repeat dosing and
decreased activity at some somatostatin receptors.
e. Reduction in fistula output with the use of somatostatin or octreotide should occur within 48 hours.
If no noticeable response in fistula response occurs at 48 hours, treatment should be discontinued.
5. Refractory high-output fistula management
a. Acid-suppressing medications (e.g., PPIs and histamine-2 receptor antagonists) have been studied
for the treatment of refractory fistula because of their ability to decrease gastric acidity and decrease
the amount of gastric secretions. Proton pump inhibitors are more effective than histamine-2
receptor antagonists for refractory fistula output.

ACCP Updates in Therapeutics® 2022: Critical Care Pharmacy Preparatory Review and Recertification Course

867
Hepatic Failure/GI/Endocrine Emergencies

b. Antimotility agents are usually recommended (e.g., loperamide, diphenoxylate/atropine, codeine)


because of their ability to inhibit the activity of gastrointestinal tract muscles.
i. Loperamide has the greatest effect on fistula output. In studies, loperamide doses up to 12–24
mg per dose have been given safely for the management of high-output fistulas. However, due
to concerns for adverse reactions, caution should be advised with doses higher than 16 mg
per day. Additionally, administration of large quantities of the liquid form of loperamide may
increase fistula output.
6. Conservative versus surgical management
a. Conservative management is first line for most patients, with the primary goal being spontaneous
closure of the fistula. Several prognostic indicators improve the likelihood of spontaneous closure.
i. Low-output fistulas
ii. Patients younger than 40 years
iii. Fistula sites: Oropharyngeal, esophageal, duodenal, pancreatic, jejunal
iv. A long fistula tract (greater than 2 cm)
v. Intestinal continuity maintained
b. Surgery is usually indicated for fistulas that fail to close spontaneously after 30–60 days. Some
causes of fistula formation (e.g., bowel injury caused by trauma or certain surgical procedures) may
require emergency surgery to repair damage.

Patient Case

4. A 34-year-old woman with morbid obesity presents to the surgical ICU after gastric bypass surgery 1 week
prior with an enterocutaneous fistula requiring medical management. Her current output is about 600 mL/
day. For the patient’s high-output fistula, which best represents the intervention that has not been associated
with a reduction in fistula output volume?
A. Somatostatin 250 mcg/hour intravenously continuously.
B. Octreotide 100 mcg subcutaneously three times daily.
C. TPN.
D. Glutamine 0.3 g/kg/day orally.

IV.  POSTOPERATIVE ILEUS

A. Epidemiology
1. Incidence of POI can vary, depending on the type of procedure:
a. Abdominal hysterectomy: About 3%
b. Bowel resection: About 15%
2. POI can lead to a prolonged hospital stay, prolonged recovery, and increased morbidity including
increased postoperative pain, PONV, and risk of postoperative complications (e.g., aspiration
pneumonia, thromboembolism, nosocomial infection).

B. Definition
1. POI is a transient impairment of appropriate GI motility after a surgical procedure.
2. The paralytic state in POI is not caused by a mechanical obstruction, and ileus can affect the stomach,
small intestine, or large intestine.

ACCP Updates in Therapeutics® 2022: Critical Care Pharmacy Preparatory Review and Recertification Course

868
Hepatic Failure/GI/Endocrine Emergencies

3. The duration of POI is typically 2–3 days after a procedure, but POI may last up to 6 days postoperatively.
Return to normal bowel function is monitored using objective signs such as passing of flatus, active
bowel sounds, or a bowel movement.
a. The duration of POI often depends on the surgical site. Return to normal function is fastest for
the small bowel, normally within 24 hours. Paralytic state may last on average 24–48 hours in the
stomach, whereas it may take up to 3–5 days for the colon to return to normal function.
b. If POI persists beyond about 6 days, it is called a paralytic ileus.

C. Causes
1. Bowel motility is controlled by the autonomic nervous system. Parasympathetic stimulation increases
bowel motility, whereas sympathetic stimulation inhibits it.
a. Increased sympathetic output postoperatively may lead to increased ileus formation. The colon is
more dependent on the autonomic nervous system than the stomach or small intestine, which may
explain the longer recovery time postoperatively.
b. The vagal nerve is important to parasympathetic activity in the stomach. Inadvertent damage to the
vagal nerve during abdominal surgery can result in impaired emptying of the stomach.
2. During periods of fasting, upper GI tract motility is controlled by the migrating motor complex, which
moves intestinal contents distally.
a. During periods of fasting postoperatively, the contractility of the stomach and small intestine is
entirely dependent on the migrating motor complex.
b. Inflammation of the GI tract after surgery, inhibitory neural reflexes, and the release of inhibitory
neurotransmitters such as nitrous oxide, substance P, and vasoactive intestinal peptide may lead to
decreased activity of migrating motor complex.
3. Exacerbating factors
a. Anesthesia: Delayed gastric emptying has been observed with the use of halothane, enflurane, and
atropine.
b. Postoperative opioids: Opioids, through agonism at the mu2-opioid receptor, slow GI motility
mainly by decreasing colonic motility. High doses and prolonged courses postoperatively can
contribute to paralytic ileus.
c. Other medications known to decrease GI motility are often given perioperatively (e.g.,
anticholinergics).

D. Diagnosis
1. POI is typically characterized by abdominal distension, lack of bowel sounds, delayed passage of flatus
or stool, and accumulation of gas and stool in the bowels, which may lead to nausea and vomiting.
2. All patients should have a physical examination for abdominal distension, followed by plain abdominal
radiographs to identify air and dilated loops of bowel.
3. An abdominal CT scan can be used to rule out a mechanical obstruction.

E. Management
1. Use of epidural anesthesia
a. Epidural blockade may improve POI by reducing local sympathetic and inflammatory response
postoperatively and increasing splanchnic blood flow. It may also reduce postoperative opioid use.
b. Several studies of epidural anesthesia have shown reductions in POI. Most studies that showed
benefit used thoracic epidural blockade and administered epidural anesthesia for at least 48–72
hours.

ACCP Updates in Therapeutics® 2022: Critical Care Pharmacy Preparatory Review and Recertification Course

869
Hepatic Failure/GI/Endocrine Emergencies

2. Use of laparoscopic surgery: Few studies have shown reduced POI rates with laparoscopic surgery
compared with open abdominal procedures. However, studies have shown reductions in inflammatory
response (e.g., cytokine production) and reduced postoperative pain with laparoscopic procedures,
which may affect POI.
3. NG decompression
a. Historically, NG tubes were placed in most patients for gastric decompression and used until
normal GI function returned.
b. In a meta-analysis of 26 trials including 4000 patients, the use of NG tube insertion was routinely
associated with fever, atelectasis, and pneumonia, though patients treated without NG tubes did
have more abdominal pain and vomiting. The study concluded that for every patient who required
NG tube insertion, 20 patients could be treated effectively without NG tube insertion and that NG
tubes should be used selectively because of concerns for adverse effects.
4. Enteral feeding
a. Traditionally, enteral feeds were delayed postoperatively until after ileus was resolved.
b. Recent data from several randomized controlled trials show a modest improvement in POI
resolution from early enteral feedings, typically initiated within the first 24 hours postoperatively.
This effect probably results from stimulation of the bowel.
5. Sham feeding
a. Sham feeding is the process of eliciting the release of hormonal and neuronal GI activity without
regular feeding. One way by which this mechanism can occur is through the chewing of gum.
b. In a small randomized study of 19 patients, chewing gum three times a day reduced time to first
flatus (2.1 days vs. 3.2 days, p<0.01) and time to first defecation (3.1 days vs. 5.8 days, p<0.01)
compared with control.
6. Pharmacologic therapy
a. Opioid-sparing analgesia
i. NSAIDs have two potential effects on resolving POI: sparing opioids through their analgesic
effects and reducing the production of inflammatory mediators (e.g., prostaglandin). Adding
an NSAID to opioid therapy reduces the need for opioids by 20%–30%.
ii. A randomized, double-blind study of morphine patient-controlled analgesia with or without
ketorolac showed decreased morphine use and earlier first bowel movements (1.5 days vs. 1.7
days, p<0.05) in the patients who received the additional ketorolac compared with those who
did not.
iii. Use should be carefully assessed so that the benefit outweighs the risk of postoperative bleeding
caused by platelet inhibition together with the potential increased risk of anastomotic leak and
wound dehiscence.
b. Prokinetics and laxatives
i. Erythromycin
(a) Erythromycin is a macrolide antibiotic that has prokinetic activity as a motilin receptor
agonist. Motilin induces gastric contractions and migrating motor complex.
(b) In randomized controlled trials, erythromycin does not appear beneficial for POI
resolution.
ii. Metoclopramide. No studies of POI have shown a benefit; however, the antiemetic activity of
metoclopramide may be beneficial as an adjunctive therapy for patients with POI.
iii. Laxatives
(a) Laxatives should play an important role in the management of POI because of their
stimulatory action in the GI tract; however, data are limited on the use of laxatives.

ACCP Updates in Therapeutics® 2022: Critical Care Pharmacy Preparatory Review and Recertification Course

870
Hepatic Failure/GI/Endocrine Emergencies

(b) Most of the data regarding the use of laxatives for POI support the use of bisacodyl
suppositories (e.g., 10 mg rectally daily), which have reduced time to return of normal
bowel function and some evidence of reduced hospital length of stay.
iv. Gastrografin
(a) Gastrografin is an oral contrast agent with theoretical benefit in POI because it is hypertonic
and reduces gut wall edema by drawing fluid into the gut lumen and promoting peristalsis.
(b) In a randomized controlled trial of 80 patients with prolonged POI, gastrografin did not
reduce the mean duration of POI compared with placebo (83.7 vs. 101.3 hours, p=0.19).
c. Peripherally acting mu-opioid receptor antagonists
i. Alvimopan (Entereg)
(a) Alvimopan has 200-fold selectivity for the peripheral opioid receptors and has poor
absorption from the GI tract when administered orally (bioavailability about 6%),
decreasing the likelihood of systemic absorption and penetration across the blood-brain
barrier.
(b) Alvimopan was studied in four North American phase III randomized controlled trials for
POI. Each trial included adult surgical patients (generally bowel resection and abdominal
hysterectomy) who received standard postoperative care for prevention of POI. In all four
trials, patients were randomized to receive placebo, alvimopan 6 mg orally twice daily, or
alvimopan 12 mg orally twice daily beginning immediately before surgery and continuing
for 7 days (total of 15 doses).
(1) Three of the four trials showed significant reductions in time to return of normal
bowel function (i.e., tolerating solid food and passing bowel movements). The 12-mg
dose was especially beneficial in females and patients older than 65.
(2) A pooled analysis from the four phase III trials comparing outcomes in patients who
received the 12-mg dose versus placebo showed a 20-hour reduction in time to return
of normal bowel function (102 hours vs. 121.8 hours, p<0.05) and a reduction in
hospital length of stay (6.6 days vs. 7.6 days, p<0.001).
(c) Alvimopan was also associated with significantly less POI-related morbidity, including
NG tube insertion (6.6% vs. 11.5%, p<0.001), and fewer POI complications, including
paralytic ileus, as well as adverse events of nausea, vomiting, and abdominal distension
(2.9% vs. 8.8%, p<0.001).
(d) In a 12-month study of alvimopan for opioid-induced bowel dysfunction in patients with
chronic non-cancer pain, myocardial infarction rates were higher in patients treated with
alvimopan than in placebo (7 [1.3%] vs. 0 [0%]).
(1) This higher risk did not appear to be related to the therapy duration (12 months). In
addition, increased risk of cardiovascular events has not occurred in other alvimopan
studies, including POI studies, and no causal relationship has been established.
(2) However, given the concern of added risk, the FDA has since developed REMS (Risk
Evaluation and Mitigation Strategies) for the use of alvimopan. Hospitals that intend
to use alvimopan should be enrolled in the EASE (ENTEREG Access Support &
Education) program. The EASE program limits the use of alvimopan to short-term
inpatient use, and patients cannot receive more than 15 doses.
ii. Methylnaltrexone
(a) Methylnaltrexone is a peripherally acting mu-opioid receptor antagonist approved for
treatment of chronic opioid-induced constipation. Methylnaltrexone is not FDA approved
for the treatment of POI.

ACCP Updates in Therapeutics® 2022: Critical Care Pharmacy Preparatory Review and Recertification Course

871
Hepatic Failure/GI/Endocrine Emergencies

(b) Methylnaltrexone is typically given subcutaneously daily or every other day for opioid-
induced constipation in a weight-based dose of 0.15 mg/kg rounded to the nearest 2 mg
(usually 8 or 12 mg)
(c) Results from phase III trials showed no reduction in duration of POI compared with
placebo, but methylnaltrexone may be an alternative to alvimopan if oral therapy is not
an option.
iii. Naloxegol
(a) Naloxegol is another oral peripherally acting mu-opioid receptor antagonist, which is
approved for opioid-induced constipation. Dosing for opioid-induced constipation is 25 mg
orally once daily, with dosing reductions required for renal dysfunction and concomitant
CYP3A4 inhibitors.
(b) Like methylnaltrexone, naloxegol is not FDA approved for the treatment of POI.

Patient Case

5. A 40-year-old man presents for a large bowel resection. Because of the complexity of the procedure and the
anticipated use of perioperative opioids and inhalation anesthesia, the team is creating a plan to prevent POI.
Which medication would be best to recommend for reducing the incidence of POI?
A. Hydromorphone patient-controlled analgesia postoperatively instead of morphine.
B. Alvimopan 12 mg orally just before surgery and continued twice daily for 7 days.
C. Metoclopramide 5 mg intravenously every 6 hours for 7 days postoperatively.
D. Octreotide 100 mcg subcutaneously every 8 hours for 7 days postoperatively.

V.  POSTOPERATIVE NAUSEA AND VOMITING

A. Epidemiology
1. Postoperative vomiting occurs in about 30% of overall surgical patients, whereas postoperative nausea
occurs in about 50%. In high-risk patients, the incidence of PONV can be as high as 80%.
2. Uncontrolled PONV may result in a prolonged stay in the post-anesthesia care unit and, sometimes,
unplanned hospital admissions for outpatient procedures.

B. Risk Factors
1. Surgical factors that increase the risk of PONV:
a. Use of volatile anesthetics: The effect of volatile anesthetics on PONV is usually dose-dependent
and typically presents within the first 6 hours postoperatively.
b. Use of postoperative opioids: The effect of postoperative opioids similarly increases the risk of
PONV in a dose-dependent fashion.
c. Use of general anesthesia
d. Duration of anesthesia: Each 1-hour increase in duration of anesthesia increases the risk of PONV.
e. Type of surgery: Cholecystectomy, laparoscopic, gynecological are most commonly associated
with PONV.

ACCP Updates in Therapeutics® 2022: Critical Care Pharmacy Preparatory Review and Recertification Course

872
Hepatic Failure/GI/Endocrine Emergencies

2. Patient-specific factors that increase the risk of PONV:


a. Female sex
b. History of motion sickness or PONV
c. Nonsmoker
d. Younger age
3. The Apfel simplified risk score is based on four predictors: female sex, history of PONV and motion
sickness, nonsmoking status, and use of postoperative opioids. The estimated risk of PONV is 10%,
20%, 40%, 60%, and 80% when zero, one, two, three, or four of the above risk factors are present,
respectively.
4. A stratified prevention strategy is suggested according to the number of risk factors, as follows:
a. Low risk (zero risk factors): No pharmacologic prophylaxis recommended; a “wait and see”
strategy is suggested
b. Medium risk (one or two risk factors): One or two prophylactic interventions are recommended.
c. High risk (more than two risk factors): Three or more prophylactic interventions should be used.
d. Of note, new guidelines recommend that at least one pharmacologic prophylactic therapy be
administered to all patients regardless of risk factors because risk scores are not completely
predictive.

C. Prevention
1. A multimodal strategy that usually targets risk avoidance should be implemented to avoid PONV.
The IMPACT study enrolled more than 5000 surgical patients and evaluated six interventions for the
prevention of PONV in a factorial design, including avoidance of volatile anesthetics and nitrous oxide,
use of short-acting opioids in the postoperative period, and use of ondansetron, dexamethasone, and
droperidol for pharmacologic prophylaxis. Each intervention except for choice of opioid was associated
with a reduction in the incidence of PONV; however, the study showed that the interventions acted
independently of each other, and thus, a benefit with several interventions was seen.
2. Regional anesthetics should be used instead of general anesthetics, when possible, because general
anesthetics are associated with an 11-fold increase in PONV. If general anesthesia is required, propofol
is preferred to volatile anesthetics for inducing and maintaining anesthesia. In the IMPACT study, the
use of propofol compared with volatile anesthetics was associated with a 19% relative risk reduction in
the incidence of PONV (p<0.001).
3. Avoiding nitrous oxide as a carrier gas is recommended; use of other carrier gases (e.g., nitrogen,
oxygen) is associated with a 12% relative risk reduction in PONV (p=0.003).
4. Perioperative opioid use should be minimized, if possible. Guidelines recommend the use of perioperative
NSAIDs for opioid-sparing analgesia. Intravenous acetaminophen has also been studied as part of a
multimodal analgesia strategy to reduce postoperative opioid use; although oral acetaminophen may
also be used, its effect on PONV has not been well studied. According to data from the IMPACT
study, which compared remifentanil with fentanyl, the use of short-acting opioids does not appear to be
associated with the incidence of PONV.
5. Pharmacologic prophylaxis:
a. Serotonin-3 antagonists are first-line treatment for pharmacologic prophylaxis. These drugs are
most effective for the prevention of PONV when given at the end of surgery.
i. Ondansetron, when given at a prophylactic dose of 4 mg intravenously, has greater anti-
vomiting effects (number needed to treat [NNT] = 6) than antinausea effects (NNT = 7).
ii. Granisetron at doses of 0.35–3 mg intravenously is as effective as ondansetron, whereas
palonosetron 0.075 mg intravenously was more effective than ondansetron at preventing
PONV in a small study of gynecological laparoscopic surgical patients (42% PONV with
palonosetron vs. 67% with ondansetron, p<0.05).

ACCP Updates in Therapeutics® 2022: Critical Care Pharmacy Preparatory Review and Recertification Course

873
Hepatic Failure/GI/Endocrine Emergencies

b. Dexamethasone is most effective for the prevention of PONV when given at the time of induction.
i. According to data from the IMPACT study, dexamethasone 4 mg intravenously was associated
with a 26% relative risk reduction in the incidence of PONV.
ii. A recent study evaluating a higher dexamethasone dose (e.g., 8 mg intravenously) at the time
of induction for the prevention of PONV found that this high-dose strategy resulted in an 8%
absolute risk reduction in vomiting compared with standard PONV care.
c. Droperidol at prophylactic doses of 0.625–1.25 mg intravenously is effective for the prevention of
PONV when given at the end of surgery.
i. Efficacy of droperidol is similar to that of ondansetron and dexamethasone, with a relative risk
reduction of about 25% (data from the IMPACT study).
ii. FDA black box warnings for cardiovascular risk with droperidol have limited its use; however,
the doses used for preventing PONV are very low and are unlikely to be associated with
cardiovascular effects.
d. Neurokinin-1 receptor antagonists (e.g., aprepitant) should be given at the time of induction. Data
are limited on the use of neurokinin-1 receptor antagonists in PONV; however, they appear to be
as effective as ondansetron. One large randomized double-blind trial evaluated 805 abdominal
surgery patients who were randomly assigned to receive 40 mg of oral aprepitant, 125 mg of oral
aprepitant, or 4 mg of intravenous ondansetron. Although there was no difference in the primary
outcome of complete response (considered no vomiting or use of rescue therapy), aprepitant at both
doses did reduce the occurrence of vomiting compared with ondansetron.
e. Transdermal scopolamine can be applied the evening before surgery. When used with ondansetron,
adding scopolamine was associated with a 10% absolute reduction in the occurrence of PONV
within the first 24 hours postoperatively.

D. Rescue Therapy
1. When rescue therapy is needed within the first 6 hours postoperatively, an antiemetic should be selected
from a therapeutic class different from the initial prophylactic drug. Repeat doses of the same drugs that
were used for initial prophylaxis can be tried if PONV occurs more than 6 hours after surgery.
2. If patients did not receive a prophylactic agent, rescue treatment with a serotonin-3 antagonist should
be tried.

Patient Case

6. A 27-year-old woman presents for a total abdominal hysterectomy. She is a nonsmoker who has a significant
history of motion sickness. During the procedure, she is expected to receive general anesthesia with volatile
anesthetics. She will probably require high-dose opioids perioperatively. Given this patient’s risk of devel-
oping PONV, which would she best receive for prevention of PONV?
A. Patient has a high risk of PONV; she should receive transdermal scopolamine the evening before sur-
gery, dexamethasone 4 mg intravenously at the time of induction, and ondansetron 4 mg intravenously
at the end of surgery.
B. Patient has a moderate risk of PONV; she should receive dexamethasone 4 mg intravenously at the time
of induction and ondansetron 4 mg intravenously at the end of surgery.
C. Patient has a high risk of PONV; she should receive dexamethasone 4 mg intravenously at the time of
induction and ondansetron 4 mg intravenously at the end of surgery.
D. Patient has a low risk of PONV; she should receive ondansetron 4 mg intravenously at the end of
surgery.

ACCP Updates in Therapeutics® 2022: Critical Care Pharmacy Preparatory Review and Recertification Course

874
Hepatic Failure/GI/Endocrine Emergencies

VI.  UPPER GASTROINTESTINAL BLEEDING

A. Definition and Epidemiology


1. Bleeding that occurs in the esophagus, stomach, or duodenum
2. In the United States, the annual incidence is 65 hospitalizations per 100,000 adults. Incidence is twice
as high in males as in females and increases with age.
3. Accounts for 350,000 hospitalizations per year at an annual cost of $2.5 billion
4. UGIB is 4 times more common than lower GI bleeding, and the hospitalization rate is around 6-fold
higher.
5. Mortality at 28 days after hospital admission for non-variceal hemorrhage is about 13%, whereas
mortality after variceal hemorrhage is about 20%. Mortality rates after both UGIB classifications seem
to be decreasing with advances in care.

B. Etiologies
1. Nonvariceal hemorrhage
a. Gastric and/or duodenal ulcer
i. Most common cause of severe cases of UGIB, accounting for 47% of all UGIB episodes
ii. Bleeding is more common in the setting of anticoagulant use and is typically self-limited.
iii. Most commonly caused by an H. pylori infection, but may also be secondary to NSAIDs,
hyperacidity (e.g., in Zollinger-Ellison syndrome), or stress-related mucosal disease (stress-
related mucosal damage is discussed in detail in the Supportive and Preventive Medicine
chapter)
b. Esophagitis
i. Noted in around 13% of patients with UGIB
ii. Risk factors include history of gastroesophageal reflux disease, medication use (e.g., NSAIDs,
oral bisphosphonates, tetracycline), and infection (e.g., Candida, herpes simplex virus).
iii. Commonly presents with hematemesis and less commonly associated with melena
iv. Lower rebleeding and mortality rate than other sources of UGIB
c. Gastroduodenal erosions (erosive gastritis/duodenitis): Defects of the gastric/duodenal mucosal
layer that lead to inflammation without ulcer formation
i. Causes are similar to those for gastric/duodenal ulcers. Additional risk factors include excessive
alcohol consumption, radiation injury, bariatric surgery, and chronic bile reflux.
ii. Bleeding is more common in the setting of anticoagulant use and is typically self-limited.
iii. May progress to ulcer formation
d. Mallory-Weiss tear: A longitudinal mucosal laceration in the distal esophagus and proximal
stomach (an intramural dissection)
i. A sudden increase in intra-abdominal pressure leads to forceful distention of the
gastroesophageal junction and a resultant mucosal tear.
ii. Typically caused by forceful vomiting
e. Less common causes: Vascular malformation, malignant formations, aortoenteric fistulas, gastric
antral vascular ectasia, and prolapse gastropathy
2. Variceal hemorrhage
a. Relatively few severe cases of UGIB are secondary to variceal hemorrhage (< 5%), but the incidence
is higher in patients with cirrhosis.
b. Mortality rate is around 20% at 6 weeks.
c. Portal hypertension caused by the obstruction of venous blood flow through the cirrhotic liver leads
to increased pressure in the portal vein and causes the redirection of blood flow to other areas of
the body.
d. Varices may be present in any portion of the GI tract, but they are most common in the esophagus
and stomach.

ACCP Updates in Therapeutics® 2022: Critical Care Pharmacy Preparatory Review and Recertification Course

875
Hepatic Failure/GI/Endocrine Emergencies

e. Gastroesophageal varices are present in about 50% of patients with cirrhosis. About 12% of patients
with varices will develop variceal hemorrhage within 1 year of diagnosis, and the recurrence rate
for variceal hemorrhage within 1 year is about 60%.

C. Initial Assessment and Risk Stratification


1. Most patients (50%) present with both melena and hematemesis; 30% have hematemesis (either red
blood or “coffee-ground” emesis) alone, and 20% have melena alone.
2. Hematochezia (bright red blood per rectum) may also be present (in about 5% of patients), which may
represent a swift UGIB (about 1 L of blood is needed in the stomach to cause hematochezia, whereas
only 50–100 mL is needed to cause melena).
3. Bleeding ulcers may result in right upper quadrant pain. Mallory-Weiss tears may present as emesis,
retching, or coughing before hematemesis. Patients with symptoms associated with chronic liver disease
will likely have variceal bleeding.
4. Hemodynamic instability may be present in patients with significant hypovolemia. Initial care of these
patients should focus on patient stabilization.
5. Insertion of an NG tube is controversial because it has not been shown to improve clinical outcomes,
but inspection of the aspirate may be useful in patients without frank hematemesis.
a. If the aspirate contains bright red blood, urgent endoscopy is likely indicated.
b. A normal-appearing aspirate does not rule out UGIB because about 15% of patients with a normal
aspirate have high-risk lesions on endoscopy.
c. Insertion of an NG tube may be contraindicated in patients with a history of varices, particularly
those with recent endoscopic band ligation.
6. In patients with a variceal hemorrhage, a hepatic venous pressure gradient greater than 20 mm Hg is
a strong predictor of early rebleeding and death and can be used for risk stratification. Measuring this
gradient is not feasible at most centers, but more than 80% of patients with Child-Turcotte-Pugh class
C have a gradient greater than 20 mm Hg.
7. Scoring tools may help in patient risk stratification, which can aid in site of care and endoscopy timing
decisions.
a. The Blatchford scoring system uses clinical and laboratory parameters to predict the need for
clinical intervention.
b. The Rockall scoring system incorporates endoscopic findings and predicts a patient’s risk of
rebleeding and death.
c. The AIMS65 scoring system uses clinical and laboratory parameters to estimate in-hospital
mortality, hospital length of stay, and cost in patients with acute UGIB.
d. Current guidelines recommend using a Blatchford score of 1 or less to identify patients at very low
risk of rebleeding or mortality who may not require hospitalization or inpatient endoscopy. The
guidelines do not make a recommendation for or against using the Rockall score and recommend
against using the AIMS65 scoring system to identify patients at low risk of rebleeding or mortality.

D. Management
1. General measures
a. Venous access with two large-caliber (at least 18 gauge; 16 gauge preferred if hemodynamically
unstable) peripheral intravenous catheters should be achieved. Access by peripheral intravenous
catheters is preferred to central venous catheterization because of their improved ability to deliver
intravenous fluids more quickly (because of the Poiseuille law).
b. Supplemental oxygen by nasal cannula should be administered to patients with an oxygen saturation
below 90%.
c. A blood type and cross-match should be sent immediately (in preparation for possible blood
transfusion).

ACCP Updates in Therapeutics® 2022: Critical Care Pharmacy Preparatory Review and Recertification Course

876
Hepatic Failure/GI/Endocrine Emergencies

2. Hemodynamically unstable patients should be resuscitated immediately with intravenous fluids


(crystalloids) and blood transfusions (if indicated).
a. A study of patients with hemodynamic instability secondary to UGIB compared usual care with
intensive resuscitation focused on achieving hemodynamic stability, a hematocrit greater than
28%, and an INR less than 1.8. Intensive resuscitation was associated with a lower mortality rate
(2.8% vs. 11.1%, p=0.04) and a lower incidence of myocardial infarction (5.6% vs. 13.9%, p=0.04).
b. See the Shock Syndromes II: Hypovolemic, Critical Bleeding, and Obstructive chapter for further
discussion of hypovolemic shock secondary to hemorrhage.
3. Transfusion
a. Current guidelines recommend a transfusion threshold of 7 g/dL for hospitalized hemodynamically
stable patients, including those in the ICU, and a threshold of 8 g/dL in patients with preexisting
cardiovascular disease. This recommendation does not extend to patients with acute coronary
syndrome.
b. A higher hemoglobin threshold may be appropriate in patients with hypotension or who are
bleeding rapidly.
c. Patients receiving warfarin or direct oral anticoagulants may require reversal, but current guidelines
do not recommend delaying endoscopy.
d. Platelet transfusion, typically if the platelet count is less than 50,000/mm3, should be considered.
4. Endoscopy
a. Diagnostic endoscopy
i. Used to diagnose and assess the risk posed by the bleeding lesion(s). Therapeutic endoscopy
may also be used for the lesion(s) to reduce the risk of bleeding recurrence (discussed later in
the chapter).
ii. Patients with UGIB should generally have a diagnostic endoscopy within 24 hours.
iii. Patients who are hemodynamically unstable and those with a suspected variceal UGIB should
have a diagnostic endoscopy as soon as possible and no later than 12 hours after presentation.
iv. Erythromycin or metoclopramide may be used when a large amount of blood in the stomach
would hinder an endoscopy; however, routine use is not recommended. Use reduces the need
for repeated endoscopy (OR 0.55; 95% CI, 0.32–0.94) but does not alter the need for blood
products, length of hospital stay, or need for surgery.
v. Endotracheal intubation before endoscopy may be indicated to prevent aspiration, but patient
selection is controversial.
vi. Endoscopic findings predict the risk of rebleeding and guide further therapies.
(a) Nonvariceal UGIB: Stigmata of recent hemorrhage from a peptic ulcer predict the risk of
further bleeding and guide management decisions (Table 6).

Table 6. Endoscopic Findings of Bleeding Peptic Ulcers, Prevalence, and Rebleeding Rate
Risk of Stigmata of Forrest
Prevalence Rebleeding Rate
Rebleeding Recent Hemorrhage Grade
Active spurting bleeding IA
9.3% (spurting and oozing) 55% (spurting and oozing)
Active oozing bleeding IB
High
Nonbleeding visible vessel IIA 6.1% 43%
Adherent clot IIB 6.5% 22%
Flat pigmented spot IIC 13.1% 10%
Low
Clean base III 52.6% 5%
Information from: Laine L, Peterson W. Bleeding peptic ulcer. N Engl J Med 1994;331:717-27; Enestvedt BK, Gralnek IM, Mattek N, et al. An evaluation of endoscopic
indication and findings related to nonvariceal upper-GI hemorrhage in a large multicenter consortium. Gastrointest Endosc 2008;67:422-9; and Laine L, Jensen DM.
Management of patients with ulcer bleeding. Am J Gastroenterol 2012;107:345-60.

ACCP Updates in Therapeutics® 2022: Critical Care Pharmacy Preparatory Review and Recertification Course

877
Hepatic Failure/GI/Endocrine Emergencies

(b) Variceal UGIB: If active variceal hemorrhage is confirmed, endoscopic and pharmacologic
therapy should be initiated.
b. Therapeutic endoscopy
i. Can be used in conjunction with a diagnostic endoscopy to treat the source of UGIB once it
has been identified
ii. Non-variceal UGIB
(a) Peptic ulcer
(1) Endoscopic therapy should be used for lesions with active spurting, active oozing, or
a non-bleeding visible vessel.
(2) Endoscopic therapy for an adherent clot resistant to vigorous irrigation is controversial.
Patients with clinical features associated with a higher risk of rebleeding (e.g., older
age) may benefit from endoscopic therapy.
(3) Endoscopic therapy should not be used for ulcers with a flat pigmented spot or clean
base.
(4) Endoscopic therapy includes hemostatic clips, epinephrine injection, and thermal
therapy. Often, these therapies are combined. Epinephrine injection should not be
used alone but should be combined with a second modality.
(b) Other findings (e.g., erosions or Mallory-Weiss tear) are typically unamenable to
endoscopic therapy.
iii. Variceal UGIB
(a) The primary endoscopic therapy for esophageal varices is endoscopic band ligation,
which is preferred to sclerotherapy.
(b) Sclerotherapy is recommended in patients for whom endoscopic band ligation is unfeasible.
(c) Gastric varices are often unamenable to endoscopic band ligation, and rescue therapies
may be necessary (discussed later in the chapter).
5. Pharmacologic management
a. Non-variceal UGIB
i. Acid-suppressive therapy is the mainstay of pharmacologic therapy for acute non-variceal
UGIB.
ii. Gastric acid inhibits platelet aggregation, impairs clot formation, and promotes fibrinolysis.
In in vitro studies, inhibiting gastric acid and raising the intragastric pH to 6 or higher may
promote clot formation and decrease the risk of rebleeding. However, clinically, a pH of 4–5
should likely be sufficient.
iii. Histamine-2 receptor antagonists are ineffective at achieving a pH greater than 6 and have
shown inconsistent benefit; therefore, these agents are not recommended for patients with
acute ulcer-related UGIB.
iv. Adding octreotide to PPI therapy does not appear to reduce hospital or ICU length of stay,
rebleeding, or mortality in patients with nonvariceal UGIB and is not routinely recommended.
v. High-dose PPIs should be used as an adjunct to endoscopic therapy.
(a) The most recent American College of Gastroenterology guidelines do not recommend for
or against the use of pre-endoscopic PPIs in patients with UGIB. This is a change from
prior versions, which stated that pre-endoscopic PPIs could be considered to decrease the
need for endoscopic therapy.
(b) High-dose PPI is defined as 80 mg/day or more for 3 days or more. This can be administered
as a continuous infusion or as intermittent boluses.
(c) High-risk patients with UGIB because of ulcers who underwent endoscopic therapy
should receive high-dose PPI treatment for 3 days, followed by twice-daily therapy for 2
weeks after the index endoscopy.

ACCP Updates in Therapeutics® 2022: Critical Care Pharmacy Preparatory Review and Recertification Course

878
Hepatic Failure/GI/Endocrine Emergencies

b. Variceal UGIB
i. Octreotide, a somatostatin analog, should be initiated promptly when variceal UGIB is
suspected and continued for 2–5 days after the diagnosis is confirmed.
(a) Octreotide causes selective splanchnic vasoconstriction through inhibition of the release of
vasodilatory peptides (mainly glucagon). Octreotide may also have a local vasoconstrictive
effect. Splanchnic vasoconstriction decreases portal inflow, which indirectly reduces
variceal blood flow and hemorrhage volume.
(b) Typically given as a 50-mcg bolus, followed by a 50-mcg/hour continuous intravenous
infusion
(c) When used without therapeutic endoscopy, octreotide is only marginally beneficial (i.e.,
reduction of packed red blood transfusion by 0.7 unit with no benefit on rebleeding or
mortality).
(d) Compared with endoscopic therapy alone, a somatostatin analog combined with
endoscopic therapy is associated with improved initial control of bleeding (relative risk
[RR] 1.12; 95% CI, 1.02–1.23) and hemostasis at 5 days (RR 1.28; 95% CI, 1.18–1.39) with
no difference in mortality or serious adverse events.
(e) Patients should be monitored for bradycardia and hyperglycemia during octreotide
infusion.
(f) For patients in whom a transjugular intrahepatic portosystemic shunt (TIPS) is performed
successfully, octreotide can be discontinued.
ii. Vasopressin infusion is not recommended for variceal UGIB because of the high incidence
of adverse events (cardiac, peripheral, and bowel ischemia) with doses necessary to reduce
splanchnic blood flow (0.2–0.8 unit/minute).
iii. Because of the high incidence of peptic ulcer–related UGIB, a PPI bolus and continuous
infusion should be initiated, even when variceal UGIB is suspected, until the diagnosis of
variceal UGIB is confirmed.
(a) There is no evidence that high-dose PPI therapy reduces the risk of rebleeding after
endoscopic therapy for variceal UGIB.
iv. Patients with cirrhosis, with or without ascites, and UGIB (whether variceal or non-variceal)
should be initiated on short-term prophylactic antibiotics.
(a) Antibiotics are associated with a lower risk of infection, lower risk of rebleeding, shorter
length of stay, and higher survival rates.
(b) Guidelines recommend therapy with ceftriaxone 1 g daily. Ceftriaxone is preferred to
fluoroquinolones because of the high prevalence of fluoroquinolone resistance.
(c) Ceftriaxone should be considered for discontinuation when hemorrhage has resolved and
octreotide is discontinued.
(d) Prophylactic antibiotic therapy should be continued for no more than 7 days.
v. For patients in whom a TIPS is not performed, a nonselective β-blocker (propranolol, nadolol,
or carvedilol) should be initiated once octreotide is discontinued unless the patient’s heart rate
or blood pressure prohibit β-blocker therapy.
vi. Simvastatin may be added to standard therapy in patients with cirrhosis and variceal bleeding.
(a) A randomized placebo-controlled trial showed that simvastatin did not reduce the rate of
rebleeding but was associated with decreased mortality in patients with Child-Pugh class
A and B cirrhosis.
(b) Patients should be monitored closely for clinical signs and symptoms of rhabdomyolysis.
c. Treatment of H. pylori infection is beyond the scope of this chapter; however, a 14-day treatment
should be given to all patients with suspected or diagnosed infection, and eradication should be
confirmed 4 weeks after therapy.

ACCP Updates in Therapeutics® 2022: Critical Care Pharmacy Preparatory Review and Recertification Course

879
Hepatic Failure/GI/Endocrine Emergencies

6. Rescue therapies
a. For patients in whom endoscopic therapy has failed or who are not candidates for endoscopy,
angiographic intervention (typically selective arterial embolization) may be required.
b. For variceal UGIB, balloon tamponade may be used as a temporizing method (maximum 24 hours)
while definitive therapy is planned.
c. TIPS is indicated in patients in whom hemorrhage from esophageal varices cannot be controlled
or in whom bleeding recurs, despite pharmacologic and endoscopic therapy. Patients with Child-
Turcotte-Pugh class C cirrhosis or Child-Turcotte-Pugh class B with bleeding on endoscopy may
be considered for TIPS within 72 hours.

Patient Case

7. A 27-year-old man with a medical history of Crohn disease presents to your ED with frank bloody output
from his rectum. The patient has hypotension (systolic blood pressure 85 mm Hg, MAP 58 mm Hg), and an
NG lavage reveals “coffee-ground” material. The resident on call is in the process of calling the endoscopy
team to help diagnose and intervene on an UGIB with an esophagogastroduodenoscopy. Which statement is
most accurate regarding the care of this patient?
A. An endoscopy is the appropriate course of action but should not be performed until after initiation of
PPI continuous infusion.
B. A PPI continuous infusion should be initiated as soon as possible and continued for 72 hours
post-endoscopy.
C. The patient, who likely has a lower GI bleed, would benefit from a colonoscopy as opposed to an
esophagogastroduodenoscopy.
D. An endoscopy and a PPI continuous infusion should be initiated as soon as possible.

E. Resuming Antithrombotic and/or Antiplatelet Therapy After Hemorrhage


1. Patients who do not have a strong indication for continued anticoagulation at the time of the bleeding
event should have antithrombotic therapy discontinued.
2. Most patients who develop UGIB while receiving long-term antithrombotic therapy continue to be at
risk of thrombosis; these patients should be resumed on antithrombotic therapy.
a. In a retrospective cohort study of patients who had a GI hemorrhage during warfarin therapy,
those who resumed warfarin within 90 days after the hemorrhagic event had a lower adjusted risk
of death (HR 0.31; 95% CI, 0.15–0.62) and thrombosis (HR 0.05; 95% CI, 0.01–0.58), without a
significant increase in the risk of recurrent GI hemorrhage (HR 1.32; 95% CI, 0.50–3.57). In this
study, warfarin was resumed a median of 4 days (interquartile range 2, 9) after presentation for
UGIB. These data suggest that for many patients with warfarin-associated GI hemorrhage, the
benefits of resuming warfarin therapy outweigh the risks.
b. Antithrombotic therapy should be withheld at minimum until the source of bleeding is found and
controlled.
c. The patient’s short-term risk of rebleeding should be weighed against the short-term risk of
thrombosis when deciding to resume antithrombotic therapy.
d. Some experts recommend that warfarin be resumed 4–7 days after presentation for UGIB.
e. The risks, benefits, and timing of resuming target-specific oral anticoagulants are unclear.

ACCP Updates in Therapeutics® 2022: Critical Care Pharmacy Preparatory Review and Recertification Course

880
Hepatic Failure/GI/Endocrine Emergencies

3. Low-dose aspirin should be resumed in patients who develop ulcer-related UGIB as soon as the risk of
cardiovascular complication is thought to outweigh the risk of bleeding. In a small randomized trial,
continuation of low-dose aspirin after peptic ulcer–related UGIB was not noninferior to placebo for
recurrent bleeding within 30 days (risk difference 4.9%; 95% CI, -3.6% to 13.4%; noninferiority margin
10%) but was associated with a lower mortality rate (risk difference 11.6%; 95% CI, 3.7%–19.5%). These
data suggest that low-dose aspirin therapy can be resumed in most patients, with careful monitoring
for hemorrhage.

VII.  ENDOCRINE EMERGENCIES

A. Epidemiology
1. Diabetic ketoacidosis (DKA) and hyperosmolar hyperglycemic state (HHS) are the most common
diabetic emergencies.
a. DKA accounts for 500,000 hospital days and costs up to $5 billion per year in the United States.
b. Between 2009 and 2014, there was a 54% increase in hospitalization rates for DKA in the United
States, with the highest rates in patients younger than 44.
c. Most patients who develop DKA have type 1 diabetes, and DKA is considered the most important
contributor to mortality rates in children and adolescents with diabetes.
d. In-hospital mortality rates for DKA are 0.1%–2.6%, and can be as high as 5%–20% for HHS,
depending on age and other comorbidities.
2. Hyperglycemia BG above 140 mg/dL) occurs commonly in the setting of critical illness, with prevalence
rates depending on the level of hyperglycemia and patient population evaluated.
a. About 27% of critically ill patients have a BG above 200 mg/dL on ICU admission, and about 90%
of patients will have at least one BG reading above 110 mg/dL during their ICU stay.
b. Hyperglycemia has consistently been associated with increased mortality in critically ill patients,
most notably in patients without diabetes. The link between hyperglycemia and worse outcomes
seems strongest for patients with acute coronary syndrome or stroke.
c. Glucose variability (BG fluctuation over time) has also been associated with ICU mortality.
3. Hypoglycemia also occurs commonly in the ICU, both with and without intensive glucose control.
a. In a large international cohort of patients, moderate hypoglycemia (BG less than 70 mg/dL)
occurred in 37% of patients and was independently associated with mortality on multivariable
regression (OR 1.78; 95% CI, 1.39–2.27).
b. In a secondary analysis of the Normoglycemia in Intensive Care Evaluation–Survival Using
Glucose Algorithm Regulation (NICE-SUGAR) study, moderate hypoglycemia occurred in 45%
of patients (74.2% in the intensive-control group and 15.8% in the conventional-control group), and
severe hypoglycemia (BG below 40 mg/dL) occurred in 3.7% of patients (6.9% in the intensive-
control group and 0.5% in the conventional-control group). Both moderate and severe hypoglycemia
(HR 1.41; 95% CI, 1.21–1.62 and HR 2.10; 95% CI, 1.59–2.77, respectively) were independently
associated with death compared with the absence of hypoglycemia.
4. Thyroid crisis, also known as thyroid storm or critical thyrotoxicosis, is an uncommon manifestation of
hyperthyroidism known to occur in less than 10% of patients admitted for thyrotoxicosis. Thyrotoxicosis
has been associated with mortality rates of 8%–25%, if treated, and up to 100%, if untreated.
5. Myxedema coma is an uncommon severe manifestation of hypothyroidism, with mortality rates of
about 20%–25% with appropriate treatment and up to 80% without treatment.

ACCP Updates in Therapeutics® 2022: Critical Care Pharmacy Preparatory Review and Recertification Course

881
Hepatic Failure/GI/Endocrine Emergencies

6. Adrenal crisis (i.e., acute adrenal insufficiency or Addisonian crisis) is a life-threatening disorder
caused by glucocorticoid (and possibly mineralocorticoid) deficiency.
a. Adrenal crisis necessitating hospital admission has an incidence of 3.3 per 100 patient-years in
those with chronic adrenal insufficiency.
b. Critical illness–related corticosteroid insufficiency (CIRCI) is a separate entity from adrenal crisis.
The prevalence of CIRCI is about 10%–20% but depends on the definition used and the population
evaluated. Prevalence rates as high as 60% for patients with septic shock have been reported.

B. DKA and HHS


1. Clinical presentation of DKA
a. Current guidelines define DKA as a combination of hyperglycemia (BG greater than 250 mg/dL),
ketosis (positive urine or serum ketones), and acidosis (pH less than 7.30) with a serum bicarbonate
less than 18 mmol/L and anion gap greater than 10. Patients with severe DKA present with severe
acidemia (pH less than 7), a serum bicarbonate less than 10 mmol/L, and an anion gap greater than
12 together with depressed mental status (stupor or coma).
b. Serum glucose concentrations are usually less than 800 mg/dL but may be more elevated in
patients who are comatose. In the setting of starvation, pregnancy, treatment with insulin prior to
presentation, or use of sodium-glucose- cotransporter 2 inhibitors, glucose may be normal or only
mildly elevated.
c. A lack of insulin leads to reduced glucose uptake, which, together with increased counterregulatory
hormone (catecholamines, cortisol, glucagon, and growth hormone) release, leads to increased
lipolysis. The released free fatty acids are metabolized in the liver to ketone bodies (β-hydroxybutyrate
and acetoacetate), resulting in ketonemia and elevated anion gap metabolic acidosis.
d. Although symptoms may exist for a few days, ketoacidosis occurs quickly, and patients may
deteriorate rapidly.
e. Typically occurs in young, leaner patients with type 1 diabetes
f. Patients may present with symptoms of nausea and vomiting (80% of patients; more indicative of
DKA than of HHS), polyuria, polydipsia, weight loss, abdominal pain (30% of patients), and fruity
breath from acetone in the blood. Signs of DKA include tachycardia, poor skin turgor, hypotension
(because of intravascular volume loss from osmotic diuresis), and Kussmaul respirations (due to
severe acidemia).
g. Measured serum potassium may be high because hyperosmolality and lack of endogenous insulin
cause intracellular potassium ions to shift into the extracellular space. However, patients typically
have a relative deficiency of total body potassium, which can be worsened with insulin treatment.
2. Clinical presentation of HHS
a. HHS is similar to DKA and is defined as BG greater than 600 mg/dL and serum osmolality greater
than 320 mOsm/L with pH greater than 7.30 and serum bicarbonate greater than 18 mmol/L.
Typically, ketonemia or ketonuria is not present (if present, only in small amounts).
b. HHS presentation is often later in the disease course than DKA (because of the lack of ketosis),
and symptoms evolve over days to weeks. Patients also have higher degrees of dehydration owing
to osmotic diuresis.
c. Typically occurs in older patients with obesity with type 2 diabetes
d. Signs and symptoms, including electrolyte abnormalities, are similar to those in DKA. However,
confusion is much more apparent in HHS and is directly related to the serum osmolality.
e. Patients with HHS do not normally develop ketoacidosis. Although there is an overall insulin
deficiency in both DKA and HHS, there is enough insulin secretion to prevent ketogenesis in
patients with HHS.

ACCP Updates in Therapeutics® 2022: Critical Care Pharmacy Preparatory Review and Recertification Course

882
Hepatic Failure/GI/Endocrine Emergencies

f. Lack of access to water, either because of the illness itself or because of an altered thirst response
in older adult patients, can worsen the severity of dehydration in the setting of hyperglycemia.
3. Typically, DKA and HHS are caused by an insufficiency of insulin in patients with diabetes combined
with another potential trigger such as infection, medication nonadherence, myocardial infarction,
cerebrovascular accident, pancreatitis, and certain drugs (i.e., steroids, diuretics, vasopressors,
antipsychotics, cocaine, sodium-glucose cotransporter 2 inhibitors).
4. Management
a. Typically involves fluid resuscitation, correction of hyperglycemia, and electrolyte (mainly
potassium) replacement. Correction of acidemia may also be indicated.
b. Patients are often profoundly hypovolemic. Total body water deficits may be as high as 10–12 L and
should be replaced within the first 24 hours.
i. In the absence of concomitant cardiogenic shock, 0.9% sodium chloride should be administered
at a rate of 15–20 mL/kg (typically 1–1.5 L) for the first hour. Thereafter, fluid administration
is titrated to hemodynamic parameters and urine output (i.e., fluid infusion rates typically
250–500 mL/hour).
ii. Patients with mild dehydration and normal or high sodium concentrations can be changed to
0.45% sodium chloride infused at a rate of 250–500 mL/hour.
iii. Maintenance fluids can be switched to a dextrose-containing fluid (often 5% dextrose with
0.45% sodium chloride) once BG concentrations have dropped to less than 200 mg/dL in DKA
and less than 300 mg/dL in HHS.
iv. Serum sodium concentration should be considered when choosing resuscitation fluid because
rapid correction of hypernatremia may have severe consequences.
c. Insulin therapy is the main treatment modality of DKA and HHS.
i. Insulin corrects hyperglycemia and inhibits the release of free fatty acids, which decreases
ketone formation and corrects acidosis.
ii. Intravenous regular insulin is preferred to subcutaneous insulin because of its short half-life
and ease of titration.
iii. Initiate with a 0.1-unit/kg intravenous bolus, followed by a 0.1-unit/kg/hour continuous
infusion. Bolus dosing may vary by patient and institution; if bolus is given, BG should be
monitored closely to avoid hypoglycemia. An alternative approach, without an initial bolus
but initiated at a higher continuous infusion rate (0.14 unit/kg/hour), provides a time similar
to DKA resolution.
iv. The insulin infusion should subsequently be titrated on an hourly basis to decrease BG
concentrations by 50–75 mg/dL/hour.
v. DKA: Decrease dose to 0.02–0.05 unit/kg/hour once BG concentrations drop to 200 mg/dL,
and maintain a BG of 150–200 mg/dL until ketoacidosis has resolved.
vi. HHS: Decrease dose to 0.02–0.05 unit/kg/hour once BG concentrations drop to 300 mg/dL,
and maintain a BG of 200–300 mg/dL until mental status changes have resolved.
vii. Continuous insulin infusions should be continued until ketoacidosis resolves in DKA and
abnormal serum osmolality and mental status in HHS resolve; then, change to subcutaneous
insulin.
(a) Criteria for resolution of ketoacidosis include a BG less than 200 mg/dL and two of the
following: a serum bicarbonate concentration of 15 mEq/L or greater, a venous pH greater
than 7.3, and a normalization of anion gap. Per guidelines, anion gap should be 12 mEq/L
or less, but normal range may vary by institution.

ACCP Updates in Therapeutics® 2022: Critical Care Pharmacy Preparatory Review and Recertification Course

883
Hepatic Failure/GI/Endocrine Emergencies

(b) Doses of 0.5–0.8 units/kg of subcutaneous insulin per day can be used with a basal
(glargine or detemir) and bolus (lispro, aspart, or glulisine) approach and an overlap of 1-2
hours between administration of subcutaneous insulin and discontinuation of intravenous
insulin infusion.
d. Potassium should be replaced aggressively (assuming adequate renal function) with a goal serum
potassium of 4–5 mEq/L.
i. Patients with initial serum potassium less than 3.3 mEq/L should have insulin withheld and
potassium administered until their serum potassium is above 3.3 mEq/L.
ii. Patients with serum potassium of 3.3–5.2 mEq/L should have 20–30 mEq of potassium added
to each liter of intravenous fluid administered.
iii. Patients with initial serum potassium greater than 5.2 mEq/L should not receive potassium,
but they should be monitored closely (with potassium administered if the patient’s serum
falls below 4 mEq/L). Of importance, patients with DKA and initial hyperkalemia should
not receive therapy directed toward total body potassium removal (e.g., sodium polystyrene
sulfonate or furosemide). The patient’s serum potassium will typically decrease as insulin is
administered.
e. Intravenous sodium bicarbonate should not be administered routinely.
i. Acidemia will typically quickly correct as ketosis is resolved, and bicarbonate may have
deleterious effects (e.g., hypokalemia).
ii. Studies of patients with DKA with a pH of 6.9 or greater have not supported a benefit with
sodium bicarbonate administration.
iii. A recent study of sodium bicarbonate in ICU patients showed a potential mortality benefit in
patients with acute kidney injury. However, this study excluded patients with ketoacidosis.
iv. If a patient’s pH is below 6.9, sodium bicarbonate should be administered.
(a) Prospective studies of this patient population have not been reported, but this level of
acidemia may lead to severe sequelae.
(b) Guidelines recommend that 100 mEq of sodium bicarbonate be placed in 400 mL of
sterile water with 20 mEq of potassium chloride, administered at a rate of 200 mL/hour.
The infusion should be discontinued if the patient’s pH is above 7.
f. Phosphorus repletion should be initiated in patients with serum phosphate below 1 mg/dL or in
those with severe cardiac or respiratory dysfunction associated with hypophosphatemia.

C. Hyperglycemia
1. Clinical presentation
a. Because hyperglycemia is so prevalent in critically ill patients, there is no specific clinical
presentation.
b. Critically ill patients often do not have the “classic triad” of diabetes symptoms of polyuria,
polydipsia, and polyphagia.
c. Stress-induced hyperglycemia has been associated with illness severity.
2. Causes
a. Critically ill patients have increased release of “stress hormones” (e.g., cortisol and epinephrine)
and cytokines. These responses lead to both increased glucose production and insulin resistance,
which results in hyperglycemia.
b. Hyperglycemia is further exacerbated by infusions of dextrose-containing fluids, corticosteroids,
and exogenous sympathomimetic medication administration.

ACCP Updates in Therapeutics® 2022: Critical Care Pharmacy Preparatory Review and Recertification Course

884
Hepatic Failure/GI/Endocrine Emergencies

3. Management
a. Hyperglycemia was once considered a beneficial adaptive response in the critically ill population
and was not considered a treatment priority. In general, BG was only treated if it exceeded 200 mg/
dL.
b. Tighter glucose control garnered increased interest because hyperglycemia has independently been
associated with increased ICU mortality.
c. A treatment paradigm shift occurred in 2001 with the publication of the landmark “intensive
insulin therapy” study.
i. In this single-center study, surgical ICU patients who were receiving parenteral nutrition were
randomized to intensive intravenous insulin therapy (goal BG 80–110 mg/dL) or conventional
insulin therapy (goal BG 180–200 mg/dL).
ii. Patients randomized to intensive insulin therapy had a significantly lower ICU mortality
rate than did patients receiving conventional insulin therapy (4.6% vs. 8.0%, p<0.04). The
ICU mortality benefit was most pronounced in patients who stayed in the ICU (and received
intensive insulin therapy) for greater than 5 days (10.6% vs. 20.2%, p=0.005).
iii. Patients randomized to intensive insulin therapy also less commonly developed bloodstream
infections, acute kidney injury, and ICU-acquired weakness (at the time termed critical-illness
polyneuropathy).
iv. A study of medical ICU patients with an identical design from the same center was published
in 2006. The study detected no in-hospital mortality benefit with intensive insulin therapy
(37.3% vs. 40.0% with conventional insulin therapy, p=0.33). Patients in the intensive insulin
therapy arm less commonly developed acute kidney injury, had a shorter time to liberation
from mechanical ventilation, and had earlier discharge from the ICU and hospital. For patients
who stayed in the ICU for 3 days or more, in-hospital mortality was lower in the intensive
insulin therapy arm (43.0% vs. 52.5%, p=0.009), but the validity of this subgroup analysis has
been called into question because patients were not defined by a baseline characteristic.
v. In light of these findings, intensive insulin therapy was widely recommended by treatment
guidelines (including the 2008 Surviving Sepsis Campaign guidelines) and often implemented
into practice as a standard of care.
d. Multicenter trials have not confirmed the mortality benefit of intensive insulin therapy.
i. Because of concern with the single-center nature of the aforementioned studies, unblinded
design, and large relative mortality benefit, three multicenter trials were designed and
conducted.
ii. One multicenter study was terminated early because of safety concerns. Patients randomized
to intensive insulin therapy more commonly developed severe hypoglycemia (BG of 40 mg/
dL or less) than those allocated to conventional insulin therapy (17.0% vs. 4.1%, p<0.001).
Intensive insulin therapy was not associated with a benefit in 28-day mortality (24.7% vs.
26.0%, p=0.74), but the study was inadequately powered to assess this outcome.
iii. A second multicenter study was terminated early because of several protocol violations.
Intensive insulin therapy was not associated with a benefit in ICU mortality (17.2% vs. 15.3%,
p=0.41), but patients allocated to intensive insulin therapy more commonly developed severe
hypoglycemia (BG of 40 mg/dL or less; 8.7% vs. 2.7%, p<0.0001).
iv. The largest multicenter study, NICE-SUGAR, evaluated intensive insulin therapy (target BG
81–108 mg/dL) or conventional glucose control (BG of 180 mg/dL or less) in more than 3000
patients.
(a) At 90 days, intensive insulin therapy was associated with increased mortality (27.5% vs.
24.9%, p=0.02).
(b) Severe hypoglycemia (BG of 40 mg/dL or less) was more common in patients allocated to
intensive insulin therapy (6.8% vs. 0.5%, p<0.0001).

ACCP Updates in Therapeutics® 2022: Critical Care Pharmacy Preparatory Review and Recertification Course

885
Hepatic Failure/GI/Endocrine Emergencies

(c) Intensive insulin therapy was not associated with a benefit in ICU or hospital length of
stay, days of mechanical ventilation, or need for renal replacement therapy.
e. A meta-analysis that included seven randomized controlled trials and 11,425 patients found that
intensive insulin therapy was not associated with a difference in 28-day mortality (OR 1.04; 95%
CI, 0.93–1.17) but was associated with a significant increase in the incidence of hypoglycemia (OR
7.7; 95% CI, 6.0–9.9).
f. Clinical practice guidelines from the American College of Critical Care Medicine regarding insulin
infusions for the management of hyperglycemia recommend the following.
i. A BG of 150 mg/dL or greater should trigger initiation of an intravenous insulin infusion,
which should be titrated by protocol to keep BG less than 150 mg/dL for most adult ICU
patients and absolutely less than 180 mg/dL.
ii. Cardiac surgery and adult trauma patients should have a BG target less than 150 mg/dL.
iii. Patients who are admitted with a diagnosis of ischemic stroke, intraparenchymal hemorrhage,
aneurysmal subarachnoid hemorrhage, or traumatic brain injury should have BG values
absolutely less than 180 mg/dL, with avoidance of BG values less than 100 mg/dL.
iv. The insulin infusion protocol should achieve a low rate of hypoglycemia (BG less than 70 mg/
dL).
v. BG should be monitored every 1–2 hours but may be liberalized to no less often than every 4
hours given the stability of BG values within the target range for an individual patient, or if the
protocol has been shown to lead to a low frequency of hypoglycemia.
vi. Subcutaneous insulin may be used for select ICU patients.
vii. Stable ICU patients should be transitioned to subcutaneous basal/bolus insulin therapy before
the intravenous insulin infusion is discontinued.

D. Hypoglycemia
1. Clinical presentation
a. Glucose is an obligate molecule for brain function. The brain cannot produce glucose, efficiently
use alternative fuels, or store substantial amounts of glycogen, so maintenance of brain function
requires a continuous supply of glucose from the circulation.
b. Multiple mechanisms exist to maintain blood glucose within physiologic range. These complex
pathways are multifaceted and include pancreatic (decreased insulin and increased glucagon),
central nervous system (CNS) (increased norepinephrine and acetylcholine), adrenal medulla
(increased epinephrine), and liver (increased glycogenolysis and gluconeogenesis) involvement.
c. In the early stages of hypoglycemia, symptoms such as sweating, anxiety, hunger, palpitations,
tremor, and arousal are present. As hypoglycemia persists and worsens, confusion, dizziness, and
difficulty speaking develop. Severe hypoglycemia leads to seizures and hypoglycemic coma.
d. Typically, complete recovery of symptoms occurs with glucose administration, but permanent
brain damage may occur in patients with severe prolonged hypoglycemia.
e. Hypoglycemia, a reversible cause of cardiac arrest, should be considered for patients with an
unclear reason for cardiac arrest.
f. Neurologic symptoms of hypoglycemia may be masked by sedation, and the counterregulatory
response may be impaired or masked (e.g., in circulatory shock).
2. Causes
a. Hypoglycemia can be caused by excessive insulin administration, reduced intake of glucose (rarely
the cause of severe hypoglycemia in the absence of insulin administration), decreased insulin
resistance (weight loss, adrenal or pituitary insufficiency), decreased clearance of insulin (renal or
hepatic insufficiency), or other drugs (commonly sulfonylureas, meglitinides, and ethanol; possibly
pentamidine, quinine, quinolones, insulin-like growth factor 1, β-blockers, or ACE inhibitors).

ACCP Updates in Therapeutics® 2022: Critical Care Pharmacy Preparatory Review and Recertification Course

886
Hepatic Failure/GI/Endocrine Emergencies

b. In a retrospective cohort study of mixed ICU patients, independent predictors of hypoglycemia


(BG less than 45 mg/dL) included continuous venovenous hemofiltration with bicarbonate-based
replacement solution (OR 14; 95% CI, 1.8–106), a decrease of nutrition without adjustment for
insulin infusion (OR 6.6; 95% CI, 1.9–23), diabetes mellitus (OR 2.6; 95% CI, 1.5–4.7), insulin use
(OR 5.3; 95% CI, 2.8–11), sepsis (OR 2.2; 95% CI, 1.2–4.1), and inotropic support (OR 1.8; 95%
CI, 1.1–2.9).
c. Other studies of critically ill patients have identified renal insufficiency, diabetes, mechanical
ventilation, female sex, greater severity of illness, longer ICU stay, liver disease, immunocompromised
state, and medical or nonelective admission as risk factors for hypoglycemia.
d. Renal insufficiency in the setting of insulin administration should be particularly noted because
patients with renal failure have decreased clearance of insulin, which may prolong the duration of
hypoglycemia.
3. Management
a. For conscious patients without severe symptoms, oral glucose should be ingested (milk, juice, or
dextrose tablets).
i. The typical initial glucose dose is 20 g, which should be repeated in 15–20 minutes if symptoms
have not improved or if BG remains low.
ii. Because the response to this “rescue glucose” is transient (less than 2 hours), it should be
followed by more substantial glucose intake as a snack or meal.
b. Parenteral therapy is necessary for patients unable to take glucose orally or hospitalized patients
with moderate or severe hypoglycemia.
i. Glucagon 1 mg promotes hepatic glucogenesis and glycogenolysis. It is a useful therapy for
patients with type 1 diabetes (often outside a health care setting) and those without diabetes,
but it is less useful in patients with type 2 diabetes because it stimulates insulin secretion and
glycogenolysis. Nausea/vomiting is a common adverse effect of glucagon.
ii. Hospitalized patients with severe hypoglycemia (either severe symptoms or BG less than 40
mg/dL) or those receiving an insulin infusion with BG less than 70 mg/dL (less than 100 mg/
dL in neurologic injured patients) should be treated with intravenous dextrose.
(a) Insulin infusion should be discontinued, as appropriate.
(b) Administer 10–20 g of 50% dextrose solution (20–40 mL of 50% dextrose in water).
(c) The BG measurement should be repeated in 15 minutes, with additional dextrose doses
administered to achieve a BG greater than 70 mg/dL.
(d) Care should be taken to avoid excessive dextrose administration in order to avoid
iatrogenic hyperglycemia and because glucose variability has been associated with
adverse outcomes.
(e) To prevent hypoglycemia, the insulin regimen should be reassessed if BG is less than 100
mg/dL.

E. Thyroid Crisis
1. Clinical presentation
a. Thyroid crisis is characterized by severe manifestations of hyperthyroidism.
b. Diagnosis can be established on the basis of clinical presentation in a patient with laboratory
evidence of hyperthyroidism.
i. Hyperthermia and diaphoresis are cardinal manifestations of thyroid crisis. Other key
components include tachycardia out of proportion to fever and gastrointestinal dysfunction
(nausea, vomiting, diarrhea, jaundice).

ACCP Updates in Therapeutics® 2022: Critical Care Pharmacy Preparatory Review and Recertification Course

887
Hepatic Failure/GI/Endocrine Emergencies

ii. In progressive thyroid crisis, patients may develop mental status changes (e.g., agitation,
psychosis, coma), and altered mentation has been associated with higher mortality. Other
clinical manifestations include cardiac arrhythmias, heart failure, and tachypnea.
iii. Serum TSH is often below the normal range with elevated free T3 and free and total T4. Total
T3 levels may be within normal limits in patients with a precipitating illness that reduces T4 to
T3 conversion.
iv. Other laboratory abnormalities include hypercalcemia, hyperglycemia, leukocytosis or
leukopenia, and elevated liver function tests.
v. No universally accepted diagnostic criteria exist, but the Burch and Wartofsky criteria (Table
7) can be used to identify potential thyroid storm. Although this scoring system is effective, it
is not highly specific.

Table 7. Burch and Wartofsky Criteria for Thyroid Storm


Criteria Points
Temperature (°F)
• 99–99.9 5
• 100–100.9 10
• 101–101.9 15
• 102–102.9 20
• 103–103.9 25
• ≥ 104 30
Heart rate (beats/minute)
• 99–109 5
• 110–119 10
• 120–129 15
• 130–139 20
• ≥ 140 25
Atrial fibrillation
• Absent 0
• Present 10
Congestive heart failure
• Absent 0
• Mild (edema) 5
• Moderate (rales) 10
• Severe (pulmonary edema) 20
GI signs
• Absent 0
• Moderate (nausea, vomiting, diarrhea, abdominal pain) 10
• Severe (jaundice) 20
CNS disturbance
• Absent 0
• Mild (agitation) 10
• Moderate (delirium, psychosis, extreme lethargy) 20
• Severe (seizure, coma) 30

ACCP Updates in Therapeutics® 2022: Critical Care Pharmacy Preparatory Review and Recertification Course

888
Hepatic Failure/GI/Endocrine Emergencies

Table 7. Burch and Wartofsky Criteria for Thyroid Storm (continued)

Criteria Points
Precipitant history
• Positive 0
• Negative 10
Score
• > 44: Thyroid storm
• 25–44: Suggestive/impending storm
• < 25: Storm unlikely
Information from: Bahn RS, Burch HB, Cooper DS, et al. Hyperthyroidism and other causes of
thyrotoxicosis: management guidelines of the American Thyroid Association and American Association
of Clinical Endocrinologists. Thyroid 2011;21:593-646.

2. Causes
a. Thyroid crisis can develop in patients with a history of untreated hyperthyroidism but is more often
precipitated by an acute event. Precipitating events include surgery, pregnancy, and trauma. In
hospitalized patients, the most common cause is infection.
b. Drugs associated with thyroid crisis include radioactive iodine, overdose of levothyroxine
or liothyronine, cytotoxic chemotherapy, aspirin, iodinated contrast dye, amiodarone, and
organophosphate toxicity.
c. Rarely, thyroid crisis is the initial presentation for patients with undiagnosed hyperthyroidism.
3. Management
a. Treatment of thyroid crisis is complex and requires several interventions simultaneously.
b. Decrease thyroid hormone synthesis.
i. Methimazole and propylthiouracil inhibit production of new thyroid hormone.
ii. Propylthiouracil is preferred in thyroid crisis because it not only blocks new thyroid synthesis
but also decreases peripheral conversion of T4 to T3.
iii. Propylthiouracil is also preferred in the first trimester of pregnancy; after the start of the second
trimester, patients may continue propylthiouracil or transition to methimazole. Concentrations
of both agents in breast milk are low, but methimazole is generally preferred in breastfeeding
mothers due to risk of hepatotoxicity with propylthiouracil.
iv. Propylthiouracil is given as a 500- to 1000-mg loading dose, followed by 250 mg every 6
hours. Doses may be given enterally or rectally.
v. Patients receiving propylthiouracil should be monitored for adverse effects, including
agranulocytosis, allergic hepatitis, and vasculitis.
vi. If methimazole is used because of propylthiouracil allergy or intolerance, usual dose is 20–30
mg every 8 hours. Doses may be given enterally or rectally.
vii. Intravenous steroids (hydrocortisone 300-mg loading dose, followed by 100 mg every 8 hours)
should be initiated to block the conversion of T4 to T3.
c. Inhibit thyroid hormone release with inorganic iodine.
i. Propylthiouracil and methimazole inhibit thyroid hormone synthesis but do not block release
of stored hormone from the thyroid gland.
ii. Initial dose of iodine should be given at least 1 hour after the first dose of propylthiouracil or
methimazole to reduce the risk of providing more substrate for thyroid hormone production.
iii. Oral potassium iodide (Lugol solution) is administered as 3–5 drops every 6 hours. If available,
sodium iodide can be given as an intravenous infusion of 0.5–1 g over 24 hours.

ACCP Updates in Therapeutics® 2022: Critical Care Pharmacy Preparatory Review and Recertification Course

889
Hepatic Failure/GI/Endocrine Emergencies

iv. In patients who are allergic to iodine, lithium carbonate may be used as an alternative. Initial
dose should be 300 mg every 6 hours, with subsequent adjustment to maintain serum lithium
concentration of 0.8–1.2 mEq/L.
d. Reduce levels of circulating thyroid hormone.
i. Cholestyramine can be given to bind circulating T3 and T4.
ii. Salicylates should be avoided because use can decrease binding of thyroid hormones to proteins
and therefore increase concentrations of free thyroid hormones.
iii. In severe cases that do not respond to standard therapy, plasmapheresis and therapeutic plasma
exchange may be considered to decrease T4 and T3 levels.
e. Reduce heart rate.
i. Because cardiovascular collapse leads to systemic decompensation, β-blockers should be
initiated as quickly as possible.
ii. β-Blocker dose should be titrated to achieve heart rate control (typically below 90 beats/minute).
iii. Traditionally, propranolol is the preferred therapy because it may block T4 to T3 conversion at
high doses. Dosing for this indication is aggressive, starting at 60 mg orally every 4 hours with
an optional initial loading dose of 80 mg.
iv. Alternative agents include carvedilol, esmolol, and diltiazem. Diltiazem should be reserved for
patients with active bronchospasm or for those who do not tolerate β-blockers.

F. Myxedema Coma
1. Clinical presentation
a. Myxedema coma occurs in patients with advanced untreated hypothyroidism. No absolute
diagnostic criteria distinguish myxedema coma from severe hypothyroidism. Identifying telltale
signs of hypothyroidism is key to making the appropriate diagnosis in a timely manner.
b. Diagnosis is made on the basis of clinical presentation, not laboratory evidence of hypothyroidism.
i. Altered mental status and hypothermia are the most common presenting symptoms. Other
clinical features include hypotension, bradycardia, hypoglycemia, constipation, urinary
retention, puffiness of the hands and face, and pleural, pericardial, or peritoneal effusions.
ii. Patients may present with shock or arrhythmias, including prolongation of the QT interval that
can lead to torsades de pointes.
iii. If a patient presents with signs of infection without the systemic inflammatory response
syndrome, myxedema coma should be suspected.
iv. Patients typically have elevated TSH and low or undetectable T3 and T4, but TSH may be
inappropriately low or normal in cases of central hypothyroidism. The level of change in
thyroid hormones does not correlate well with disease severity.
c. Myxedema coma is more common in older women and occurs more often during winter months
because of altered temperature regulation.
d. Discussion of nonthyroidal illness syndrome (“euthyroid sick syndrome” or “SICU thyroid”) is
beyond the scope of this chapter.
2. Causes
a. Myxedema coma may be the consequence of longstanding hypothyroidism but can also be
precipitated by an acute event such as infection, myocardial infarction, surgery, or cold exposure.
b. Drugs associated with the development of myxedema coma include sedatives, anesthetics, narcotics,
lithium, immune checkpoint inhibitors, and amiodarone, as well as abrupt discontinuation of
levothyroxine.
3. Management
a. Because myxedema coma is an endocrine emergency, treatment should be initiated without waiting
for laboratory data.

ACCP Updates in Therapeutics® 2022: Critical Care Pharmacy Preparatory Review and Recertification Course

890
Hepatic Failure/GI/Endocrine Emergencies

b. General treatment measures include rewarming of the patient and treatment of the precipitating
illness. Because infection is a common precipitator of myxedema coma, all patients should undergo
a thorough infectious workup.
c. Guidelines from the American Thyroid Association recommend the following for patients with
myxedema coma:
i. Patients may have underlying adrenocorticotropic hormone deficiency, and restoration of
thyroid function can accelerate cortisol metabolism. Empiric intravenous corticosteroids at
doses appropriate for the stressed state should be administered before levothyroxine (strong
recommendation). Hydrocortisone is typically given in doses of 50 mg every 6 hours or 100
mg every 8 hours.
ii. Thyroid hormone replacement should be initiated with intravenous levothyroxine. A loading
dose of 200–400 mcg should be given, with lower doses used in patients who are smaller or
older or who have a history of cardiac disease or arrhythmia (strong recommendation).
iii. After the initial loading dose, the daily replacement dose should be 1.6 mcg/kg body weight
intravenously (strong recommendation). Daily therapy may be changed to the enteral route
after the patient improves clinically.
iv. Because T4 to T3 conversion may be decreased in myxedema coma, intravenous liothyronine
may be given in addition to levothyroxine (weak recommendation). High serum T3 during
treatment is associated with mortality, so high doses should be avoided. A loading dose of 5–20
mcg can be given, followed by 2.5–10 mcg every 8 hours. Patients who are smaller or older or
who have a history of cardiac disease or arrhythmia should receive smaller doses. Treatment
can be continued until the patient has regained consciousness and clinical parameters have
improved.
v. Patients should be monitored for improvements in mental status, cardiac function, and
pulmonary function.
vi. Measurement of thyroid hormones every 1–2 days is reasonable to ensure a favorable
trajectory. Optimal levels of TSH, T4, and T3 in this scenario have not been defined, but lack of
improvement may be an indication to increase the levothyroxine dose and/or add liothyronine.
High serum T3 can be considered an indication to decrease therapy for safety reasons (weak
recommendation).
d. Patients may need supportive care with vasoactive medications. Typically, a catecholamine agent
with β1-adrenergic receptor agonist properties (epinephrine or dopamine) is preferred to increase
the patient’s heart rate and blood pressure.

Patient Case

8. A 23-year-old man (weight 80 kg) presents to the ED with an acute mental status change and a core body
temperature of 94°F (34.4°C). He has a history of hypothyroidism, but according to his family, he had
decided to stop taking all of his thyroid medications 1 week earlier. The team has given him a diagnosis of
myxedema coma. Which is the most appropriate treatment option for this patient?
A. Intravenous levothyroxine 400 mcg × 1 followed by 125 mcg daily.
B. An insulin infusion titrated to a BG of 140–180 mg/dL.
C. Propylthiouracil 200 mg every 4 hours.
D. Propranolol 60 mg orally every 4 hours.

ACCP Updates in Therapeutics® 2022: Critical Care Pharmacy Preparatory Review and Recertification Course

891
Hepatic Failure/GI/Endocrine Emergencies

G. Adrenal Crisis
1. Clinical presentation
a. Because there is no universally accepted evidence-based definition of adrenal crisis, cases are
identified based on a pragmatic definition and clinical assessment. In adults, adrenal crisis is
defined as an acute deterioration in health status associated with absolute or relative hypotension
(SBP less than 100 mm Hg or 20 mm Hg or more below baseline) that improves within 1–2 hours
after parenteral glucocorticoid administration. Adrenal crisis is a rare cause of vasodilatory shock.
b. Patients with acute adrenal insufficiency typically present with severe hypotension, altered mental
status, acute abdominal pain, vomiting, and fever. As such, patients often receive a misdiagnosis of
an acute abdomen. Accompanying symptoms may include fatigue, lethargy, confusion, weakness,
and pain in the muscles or joints. Patients with primary adrenal insufficiency may also have salt
craving (because of mineralocorticoid deficiency secondary to a lack of aldosterone). Characteristic
skin hyperpigmentation from primary adrenal insufficiency may also be present.
c. Laboratory findings will include low serum cortisol (i.e., cortisol level below 18 mcg/dL in an
acutely stressed patient is suggestive of adrenal insufficiency); hyponatremia, hyperkalemia,
hypoglycemia, and normocytic anemia may also be present.
d. Critical illness–related corticosteroid insufficiency (CIRCI) refers to impairment of the
hypothalamic-pituitary-adrenal axis during critical illness as a complication of an underlying
disease. Patients most commonly have septic shock but may also have acute respiratory distress
syndrome, cardiac arrest, trauma, or burns.
i. Patients with CIRCI often have hypotension refractory to fluids that requires addition of
vasopressors. Laboratory assessment may show hypoglycemia and eosinophilia; hyponatremia
and hyperkalemia are uncommon.
ii. Historically, CIRCI was diagnosed according to either a low serum cortisol concentration
or an inadequate increase in cortisol after a cosyntropin stimulation test. These tests have
significant limitations in critically ill patients, such as poor reproducibility of the cosyntropin
stimulation test and lack of agreement of commercially available cortisol assays with analytic
standards.
iii. Despite these limitations, an increase in cortisol after a cosyntropin stimulation test less than
9 mg/dL is the best predictor of adrenal insufficiency (as determined by metyrapone testing in
patients with severe sepsis/septic shock). A serum cortisol value less than 10 mg/dL has a high
positive predictive value for adrenal insufficiency but a low sensitivity.
iv. Corticosteroids should only be given for patients with septic shock and suspected CIRCI who
do not achieve resuscitation goals despite fluid administration and moderate- to high-dose
vasopressors.
v. Of importance, clinical practice guidelines for the treatment of CIRCI recommend against
using cortisol-based testing in determining a patient’s candidacy for corticosteroid therapy. As
such, using cortisol assays or a cosyntropin stimulation test to diagnose CIRCI and influence
treatment decisions is not recommended.
2. Causes
a. Adrenal crisis may result from either primary adrenal failure or secondary adrenal disease because
of impairment of the hypothalamic-pituitary axis
b. Acute-onset primary adrenal insufficiency is usually caused by adrenal hemorrhage or infarction,
whereas acute-onset secondary adrenal insufficiency is usually caused by pituitary apoplexy,
pituitary or hypothalamic surgery, or traumatic brain injury. Patients receiving corticosteroids who
have acute stress (e.g., infection or surgery) will be unable to increase their cortisol appropriately
because of suppression of corticotropin-releasing hormone and corticotropin.

ACCP Updates in Therapeutics® 2022: Critical Care Pharmacy Preparatory Review and Recertification Course

892
Hepatic Failure/GI/Endocrine Emergencies

c. Bacterial infections commonly precipitate adrenal crisis in adults. In children, viral infections are
more common precipitating events.
d. Several medications may contribute to inadequate serum cortisol concentrations, including
etomidate and ketoconazole. Adrenal crisis has been reported after vaccinations and zoledronic
acid infusions. Immune checkpoint inhibitor therapy may also cause adrenal insufficiency.
e. An abrupt decrease in dose or cessation of corticosteroids in patients receiving moderate
glucocorticoid doses (equivalent to prednisone 7.5 mg daily or above) for 3 weeks or more may
precipitate adrenal crisis.
f. CIRCI is caused by inadequate circulating cortisol concentrations combined with tissue resistance.
3. Management
a. Acute adrenal crisis
i. Patients with acute adrenal crisis should receive intravenous fluids and vasopressors (typically
isotonic sodium chloride and norepinephrine) for the treatment of shock (if present). After
initial fluid bolus, dextrose may be added for treatment of hypoglycemia.
ii. Corticosteroids should be administered, but the optimal dose is unclear. Typically, intravenous
hydrocortisone at 200–300 mg/day (50 mg every 6 hours or 100 mg every 8 hours) is initiated.
Some experts have advocated that lower doses are sufficient.
iii. Although hydrocortisone is preferred because of its protein binding, tissue distribution, and
balanced glucocorticoid-mineralocorticoid effects, other parenteral glucocorticoids may be
used. Dosing is based on glucocorticoid potency relative to hydrocortisone. Mineralocorticoid
replacement is usually unnecessary in the acute setting because its effect on sodium retention
takes several days to develop.
iv. After the patient is clearly improved (e.g., shock resolution), the corticosteroid dose can be
tapered and may be transitioned to oral therapy.
b. Patients receiving chronic corticosteroids who have an acute stressor should receive an increased
glucocorticoid dose (“stress dose”) of corticosteroids to prevent adrenal crisis.
i. The corticosteroid dose recommended in this setting depends on the level of stress.
ii. In febrile patients, those undergoing major dental procedures, and those undergoing invasive
diagnostic procedures (e.g., colonoscopy), a doubling or tripling of the maintenance dose for
1–3 days may be sufficient.
iii. Patients experiencing severe infection, undergoing major surgery, or presenting with shock
should have intravenous hydrocortisone at 200–300 mg/day (50 mg every 6 hours or 100 mg
every 8 hours) initiated. This dose should be tapered to the patient’s home regimen after the
patient clinically improves or after surgery.
c. Treatment of CIRCI in the setting of septic shock is discussed in the Shock Syndromes I:
Introduction, Vasodilatory, and Sepsis chapter. Use of corticosteroids for the treatment of acute
respiratory distress syndrome is discussed in the Pulmonary Disorders I chapter.

ACCP Updates in Therapeutics® 2022: Critical Care Pharmacy Preparatory Review and Recertification Course

893
Hepatic Failure/GI/Endocrine Emergencies

REFERENCES

Acute Liver Failure pressure in patients with acute liver failure.


1. Acharya SK, Bhatia V, Sreenivas V, et al. Efficacy Hepatology 2004;39:464-70.
of L-ornithine L-aspartate in acute liver failure: 13. Nanchal R, Subramanian R, Karvellas C, et al.
a double-blind, randomized, placebo-controlled Guidelines for the management of adult acute and
study. Gastroenterology 2009;136:2159-68. acute-on-chronic liver failure in the ICU: cardio-
2. Bernal W, Wendon J. Acute liver failure. N Engl J vascular, endocrine, hematologic, pulmonary,
Med 2013;369:2525-34. and renal considerations. Crit Care Med 2020;
3. Chalasani NP, Hayashi PH, Bonkovsky HL, et al. 48:e173-91.
ACG clinical guideline: the diagnosis and manage- 14. O’Grady J, Schalm SW, Williams R. Acute
ment of idiosyncratic drug-induced liver injury. liver failure: redefining the syndromes. Lancet
Am J Gastroenterol 2014;109:950-66. 1993;342:273-5.
4. European Association for the Study of the Liver. 15. Shami VM, Caldwell SH, Hespenheide EE, et al.
EASL clinical practice guidelines on the manage- Recombinant activated factor VII for coagulopa-
ment of acute (fulminant) liver failure. J Hepatol thy in fulminant hepatic failure compared with
2017;99:1047-81. conventional therapy. Liver Transpl 2003;9:138-43.
5. Heard KJ. Acetylcysteine for acetaminophen poi- 16. Shawcross DL, Davies NA, Mookerjee RP, et al.
soning. N Engl J Med 2008;359:285-92. Worsening of cerebral hypoxia by the administra-
6. Karvellas CJ, Cavazos J, Battenhouse H, et tion of terlipressin in acute liver failure with severe
al. Effects of antimicrobial prophylaxis and encephalopathy. Hepatology 2004;39:471-5.
blood stream infections in patients with acute 17. Stravitz RT, Kramer AH, Davern T, et al. Intensive
liver failure: a retrospective cohort study. Clin care of patients with acute liver failure: recom-
Gasteroenterol Hepatol 2014;12:1942-9. mendations of the U.S. Acute Liver Failure Study
7. Karvellas CJ, Stravitz RT, Battenhouse H, et al. Group. Crit Care Med 2007;35:2498-508.
Therapeutic hypothermia in acute liver failure: a 18. Stravitz RT, Lee WM. Acute liver failure. Lancet
multicenter retrospective cohort analysis. Liver 2019;394:869-81.
Transplant 2015;21:4-12. 19. Terziroli B, Mieli-Vergani G, Vergani D.
8. Larson AM, Polson J, Fontana RJ, et al. Acute Liver Autoimmune hepatitis: standard treatment and
Failure Study Group. Acetaminophen-induced systematic review of alternative treatments. World
liver failure: results of a United States multicenter, J Gastroenterol 2017;23:6030-48.
prospective, study. Hepatology 2005;42:1367-72. 20. Yarema MC, Johnson DW, Berlin RJ, et al.
9. Lee WM, Hynan LS, Rossaro L, et al. Intravenous Comparison of the 20-hour intravenous and
N-acetylcysteine improves transplant-free survival 72-hour oral acetylcysteine protocols for the treat-
in early stage non-acetaminophen acute liver fail- ment of acute acetaminophen poisoning. Ann
ure. Gastroenterology 2009;137:856-64. Emerg Med 2009;54:606-14.
10. Lee WM, Larson AM, Stravitz RT. American
Association for the Study of Liver Diseases. Acute Pancreatitis
AASLD Position Paper: The Management of 1. Al-Omran M, Al Balawi CH, Tashkandi MF,
Acute Liver Failure: Update 2011. Available at et al. Enteral versus parenteral nutrition for
http://aasld.org/sites/default/files/guideline_docu- acute pancreatitis. Cochrane Database Syst Rev
ments/alfenhanced.pdf. Accessed August 2020. 2010;1:CD002837.
11. MacDonald AJ, Subramanian RM, Olson JC, et al. 2. Banks PA, Bollen TL, Dervenis C, et al.
Use of the molecular adsorbent recirculating sys- Classification of acute pancreatitis—2012: revi-
tem in acute liver failure. Crit Care Med 2021. doi: sion of the Atlanta classification and definition by
10.1097/CCM.0000000000005194 international consensus. Gut 2013;62:102-11.
12. Murphy N, Auzinger G, Bernel W, et al. The effect 3. Chang YS, Fu HQ, Xiao YM, et al. Nasogastric
of hypertonic sodium chloride on intracranial or nasojejunal feeding in predicted severe

ACCP Updates in Therapeutics® 2022: Critical Care Pharmacy Preparatory Review and Recertification Course

894
Hepatic Failure/GI/Endocrine Emergencies

acute pancreatitis: a meta-analysis. Crit Care fistulas: a systematic review and meta-analysis.
2013;17:R118. World J Surg 2012;36:1016-29.
4. Choosakul S, Harinwan K, Chirapongsathorn S, 2. de Aguilar-Nascimento J. Oral glutamine in addi-
et al. Comparison of normal saline versus lactated tion to parenteral nutrition improves mortality
Ringer’s solution for fluid resuscitation in patients and the healing of high-output fistulas. Nutr Hosp
with mild acute pancreatitis: a randomized con- 2007;22:672-6.
trolled trial. Pancreatology 2018;18:507-12. 3. Falconi M, Pederzoli P. The relevance of gastroin-
5. Dellinger EP, Tellado JM, Soto NE, et al. Early testinal fistulae in clinical practice: a review. Gut
antibiotic treatment for severe acute necrotizing 2002;49:iv2-10.
pancreatitis: a randomized, double-blind, placebo- 4. Gonzalez-Pinto I, Moreno Gonzalez E. Optimising
controlled study. Ann Surg 2007;245:674-83. the treatment of upper gastrointestinal fistulae.
6. Elmunzer BJ, Scheiman JM, Lehman GA, et al. Gut 2002;49:iv21-28.
A randomized trial of rectal indomethacin to 5. Hesse U, Ysebaert D, de Hemptinne B. Role of
prevent post-ERCP pancreatitis. N Engl J Med somatostatin-14 and its analogues in the manage-
2012;366:1414-22. ment of gastrointestinal fistulae: clinical data. Gut
7. Jiang K, Huang W, Yang XN, et al. Present and 2002;49:iv11-20.
future of prophylactic antibiotics for severe acute 6. Levy E, Frileux P, Cugnenc PH, et al. High-output
pancreatitis. World J Gastroenterol 2012;18:279-84. external fistulae of the small bowel: manage-
8. Lee A, Ko C, Buitrago C, et al. Lactated Ringers ment with continuous enteral nutrition. Br J Surg
vs normal saline resuscitation for mild acute pan- 1989;76:676-9.
creatitis: a randomized trial. Gastroenterology 7. Nubiola-Calonge P, Badia JM, Sancho J, et al.
2021;160:955-7. Blind evaluation of the effect of octreotide, a
9. Tenner S, Baillie J, DeWitt J et al. American somatostatin analogue, on small-bowel fistula out-
College of Gastroenterology guideline: manage- put. Lancet 1987;2:672-4.
ment of acute pancreatitis. Am J Gastroenterol 8. Parli S, Pfeifer C, Oyler DR, et al. Redefining
2013;108:1400-15. “bowel regimen”: Pharmacologic strategies and
10. Vaughn VM, Shuster D, Rogers MAM, et al. Early nutritional considerations in the management of
versus delayed feeding in patients with acute pan- small bowel fistulas. Am J Surg 2018;216:351-8.
creatitis: a systematic review. Ann Intern Med 9. Polk TM, Schwab CW. Metabolic and nutri-
2017;166:883-92. tional support of the enterocutaneous fistula
11. Vege SS, DiMagno MJ, Forsmark CE, et al. Initial patient: a three-phase approach. World J Surg
medical treatment of acute pancreatitis: American 2012;36:524-33.
Gastroenterological Association Institute techni- 10. Schecter WP. Management of enterocutaneous fis-
cal review. Gastroenterology 2018;154:1103-39. tulas. Surg Clin North Am 2011;91:481-91.
12. Working Group IAP/APA Pancreatitis Guidelines. 11. Schecter WP, Hirschberg A, Chang DS, et al.
IAP/APA evidence-based guidelines for the man- Enteric fistulas: principles of management. J Am
agement of acute pancreatitis. Pancreatology Coll Surg 2009;209:484-91.
2013;13:e1-15. 12. Torres AJ, Landa JI, Moreno-Azcoita M, et al.
13. Wu BU, Banks PA. Clinical management of Somatostatin in the management of gastroin-
patients with acute pancreatitis. Gastroenterology testinal fistulas: a multicenter trial. Arch Surg
2013;144:1272-81. 1992;127:97-100.
14. Wu BU, Hwang JQ, Gardner TH, et al. Lactated
Ringer’s solution reduces systemic inflammation Postoperative Ileus and Nausea/Vomiting
compared with saline in patients with acute pan- 1. Apfel CC, Korttila K, Abdalla M, et al. IMPACT
creatitis. Clin Gastroenterol Hepatol 2011;9:710-7. Investigators. A factorial trial of six interventions
for the prevention of postoperative nausea and
Gastrointestinal Fistulas vomiting. N Engl J Med 2004;350:2441-51.
1. Coughlin S, Roth L, Lurati G, et al. Somatostatin 2. Asao T, Kuwano H, Nakamura J, et al. Gum chew-
analogues for the treatment of enterocutaneous ing enhances early recovery from postoperative

ACCP Updates in Therapeutics® 2022: Critical Care Pharmacy Preparatory Review and Recertification Course

895
Hepatic Failure/GI/Endocrine Emergencies

ileus after laparoscopic colectomy. J Am Coll Surg randomised controlled trial (DREAMS trial). BMJ
2002;195:30-2. 2017;357:j1455.
3. Bell TJ, Poston SA, Kraft MD, et al. Economic 13. Park SK, Cho EJ. A randomized, double-blind
analysis of alvimopan in North American phase trial of palonosetron compared with ondansetron
III efficacy trials. Am J Health Syst Pharm in preventing postoperative nausea and vomiting
2009;66:1362-8. after gynaecological laparoscopic surgery. J Int
4. Cheatham ML, Chapman WC, Key SP, et al. A Med Res 2011;39:399-407.
meta-analysis of selective versus routine nasogas- 14. Person B, Wexner S. The management of postop-
tric decompression after elective laparotomy. Ann erative ileus. Curr Probl Surg 2006;43:12-65.
Surg 1995;221:469-78. 15. Skolnik A, Gan TJ. Update on the management
5. Chen JY, Wu GJ, Mok MS, et al. Effect of of postoperative nausea and vomiting. Curr Opin
adding ketorolac to intravenous morphine Anesthesiol 2014;27:605-9.
patient-controlled analgesia on bowel function in 16. Vather R, Josephson R, Jaung R, et al. Gastrografin
colorectal surgery patients—a prospective, ran- in prolonged postoperative ileus: a double-
domized, double-blind study. Acta Anaesthesiol blinded randomized controlled trial. Ann Surg
Scand 2005;49:546-51. 2015;262:23-30.
6. Delaney CP, Wolff BG, Viscusi ER, et al. 17. Vather R, Trivedi S, Bissett I. Defining postopera-
Alvimopan, for postoperative ileus following tive ileus: results of systematic review and global
bowel resection: a pooled analysis of phase III survey. J Gastrointest Surg 2013;17:962-72.
studies. Ann Surg 2007;245:355-63. 18. Wolff BG, Michelassi F, Gerkin TM, et al.
7. De Oliveira GS, Santana Castro-Alves LJ, Ahmed Alvimopan, a novel, peripherally acting µ opioid
S, et al. Dexamethasone to prevent postoperative antagonist: results of a multicenter, randomized,
nausea and vomiting: an updated meta-analysis double-blind, placebo-controlled, phase III trial of
of randomized controlled trials. Anesth Analg major abdominal surgery and postoperative ileus.
2013;116:58-74. Ann Surg 2004;240:728-35.
8. Gan TJ, Apfel CC, Kovac A, et al. A random- 19. Wolff BG, Viscusi ER, Delaney CP, et al. Patterns
ized, double-blind comparison of the NK1 of gastrointestinal recovery after bowel resection
antagonist, aprepitant, versus ondansetron for the and total abdominal hysterectomy: pooled results
prevention of nausea and vomiting. Anesth Analg from the placebo arms of alvimopan phase III
2007;104:1082-9. North American clinical trials. J Am Coll Surg
9. Gan TJ, Belani KG, Bergese S, et al. Fourth 2007;205:43-51.
consensus guidelines for the management of post- 20. Wolff BG, Weese JL, Ludwig KA, et al.
operative nausea and vomiting. Anesth Analg Postoperative ileus-related morbidity profile in
2020;131:411-48. patients treated with alvimopan after bowel resec-
10. Gan TJ, Sinha AC, Kovac AL, et al. A random- tion. J Am Coll Surg 2007;204:609-16.
ized, double-blind, multicenter trial comparing 21. Yu CS, Chun HK, Stambler N, et al. Safety and
transdermal scopolamine plus ondansetron to efficacy of methylnaltrexone in shortening the
ondansetron alone for the prevention of postopera- duration of postoperative ileus following seg-
tive nausea and vomiting In the outpatient setting. mental colectomy: results of two randomized,
Anesth Analg 2009;108:1498-504. placebo-controlled phase 3 trials. Dis Colon
11. Gero D, Gie O, Hubner M, et al. Postoperative Rectum 2011;54:570-8.
ileus: in search of an international consensus on
definition, diagnosis and treatment. Langenbecks Upper Gastrointestinal Bleeding
Arch Surg 2017;402:149-58. 1. Abraldes JG, Villanueva C, Aracil C, et al. Addition
12. Magill L; DREAMS Trial Collaborators and West of simvastatin to standard therapy does not reduce
Midlands Research Collaborative. Dexamethasone rebleeding but increases survival in patients with
versus standard treatment for postoperative nau- cirrhosis. Gastroenterology 2016;150:1160-70.
sea and vomiting in gastrointestinal surgery:

ACCP Updates in Therapeutics® 2022: Critical Care Pharmacy Preparatory Review and Recertification Course

896
Hepatic Failure/GI/Endocrine Emergencies

2. Alaniz C, Mohammad RA, Welage LS. Continuous for the Study of Liver Diseases. Hepatology
infusion of pantoprazole with octreotide does not 2017;65:310-35.
improve management of variceal hemorrhage. 14. Guntipalli P, Chason R, Elliott A, et al. Upper
Pharmacotherapy 2009;29:248-54. gastrointestinal bleeding caused by severe esoph-
3. Bai Y, Guo JF, Li ZS. Meta-analysis: erythromycin agitis: a unique clinical syndrome. Dig Dis Sci
before endoscopy for acute upper gastrointestinal 2014;59:2997-3003.
bleeding. Aliment Pharmacol Ther 2011;34:166-71. 15. Holster I, Kuipers EJ. Management of acute
4. Bañares R, Albillos A, Rincon D, et al. Endoscopic nonvariceal upper gastrointestinal bleeding: cur-
treatment versus endoscopic plus pharmacologic rent policies and future perspectives. World J
treatment for acute variceal bleeding: a meta- Gastroenterol 2012;18:1202-7.
analysis. Hepatology 2002;305:609-15. 16. Kim J. Management and prevention of upper GI
5. Baradarian R, Ramdhaney S, Chapalamadugu R, bleeding. In: Richardson M, Chessman K, Chant
et al. Early intensive resuscitation of patients with C, et al., eds. Pharmacotherapy Self-Assessment
upper gastrointestinal bleeding decreases mortal- Program, 2012 Book 11. Gastroenterology and
ity. Am J Gastroenterol 2004;99:619-22. Nutrition I. Lenexa, KS: American College of
6. Barkun AN, Almadi M, Kuipers EJ, et al. Clinical Pharmacy, 2012:7-26.
Management of nonvariceal upper gastrointesti- 17. Laine L. Clinical Practice. Upper gastrointesti-
nal bleeding: guideline recommendations from the nal bleeding due to a peptic ulcer. N Engl J Med
international consensus group. Ann Intern Med 2016;374:2367-76.
2019;171:805-22. 18. Laine L, Barkun AN, Saltzman JR, et al. ACG
7. Barkun AN, Bardou M, Martel M, et al. Prokinetics clinical guideline: upper gastrointestinal and ulcer
in acute upper GI bleeding: a meta-analysis. bleeding. Am J Gastroenterol 2021;116:899-917.
Gastrointest Endosc 2010;72:1138-45. 19. Laine L, Peterson W. Bleeding peptic ulcer. N
8. Chey WD, Wong BC, Practice Parameters Engl J Med 1994;331:717-27.
Committee of the American College of 20. Lau JK, Leung WK, Wu JCY, et al. Omeprazole
Gastroenterology. American College of before endoscopy in patients with gastrointestinal
Gastroenterology guideline on the management of bleeding. N Engl J Med 2007;356:1631-40.
Helicobacter pylori infection. Am J Gastroenterol 21. Lo EA, Wilby KJ, Ensom MH. Use of proton pump
2007;102:1808-25. inhibitors in the management of gastroesophageal
9. Colantino A, Jaffer AK, Brotman DJ. Resuming varices: a systematic review. Ann Pharmacother
anticoagulation after hemorrhage: a practical 2015;49:207-19.
approach. Cleve Clin J Med 2015;82:245-56. 22. Odutayo A, Desborough MJ, Trivella M, et al.
10. Crooks C, Card T, West J. Reductions in 28-day Restrictive versus liberal blood transfusion for
mortality following hospital admission for upper gastrointestinal bleeding: a systematic review
gastrointestinal hemorrhage. Gastroenterology and meta-analysis of randomised controlled trials.
2011;141:62-70. Lancet Gastroenterol Hepatol 2017;2:354.
11. Enestvedt BK, Gralnek IM, Mattek N, et al. An 23. Riha HM, Wilkinson R, Twilla J, et al. Octreotide
evaluation of endoscopic indication and findings added to a proton pump inhibitor versus a
related to nonvariceal upper-GI hemorrhage in a proton pump inhibitor alone in nonvariceal
large multicenter consortium. Gastrointest Endosc upper-gastrointestinal bleeds. Ann Pharmacother
2008;67:422-9. 2019;53:794-800.
12. Garcia-Tsao G, Bosch J. Management of varices 24. Sachar H, Vaidya K, Laine L. Intermittent vs con-
and variceal hemorrhage in cirrhosis. N Engl J tinuous proton pump inhibitor therapy for high-risk
Med 2010;362:823-32. bleeding ulcers: a systematic review and meta-
13. Garcia-Tsao G, Abraides JG, Berzigotti A, et al. analysis. JAMA Intern Med 2014;174:1755-62.
Portal hypertensive bleeding in cirrhosis: risk 25. Saltzman JR, Tabak YP, Hyett BH, et al. A simple
stratification, diagnosis, and management: 2016 risk score accurately predicts in-hospital mortality,
practice guidelines by the American Association length of stay, and cost in acute upper GI bleeding.
Gastrointest Endosc 2011;74:1215-24.

ACCP Updates in Therapeutics® 2022: Critical Care Pharmacy Preparatory Review and Recertification Course

897
Hepatic Failure/GI/Endocrine Emergencies

26. Srygley FD, Gerardo CJ, Tran T, et al. Does this an endocrine society clinical practice guideline. J
patient have a severe upper gastrointestinal bleed? Clin Endocrinol Metab 2009;94:709-28.
JAMA 2012;370:1072-9. 10. Devereaux D, Tewelde SZ. Hyperthyroidism
27. Sung JJ, Lau JY, Ching JY, et al. Continuation of and thyrotoxicosis. Emerg Med Clin North Am
low-dose aspirin therapy in peptic ulcer bleeding: 2014;32:277-92.
a randomized trial. Ann Intern Med 2010;152:1-9. 11. Dubbs SB, Spangler R. Hypothyroidism: causes,
28. Villaneuva C, Colomo A, Bosch A, et al. killers, and life-saving treatments. Emerg Med
Transfusion strategies for acute gastrointestinal Clin North Am 2014;32:303-17.
bleeding. N Engl J Med 2013;368:11-21. 12. Jaber S, Paugam C, Futier E, et al. Sodium bicar-
29. Witt DM, Delate T, Garcia DA. Risk of thrombo- bonate therapy for patients with severe metabolic
embolism, recurrent hemorrhage, and death after acidaemia in the intensive care unit (BICAR-ICU):
warfarin therapy interruption for gastrointestinal a multicentre, open-label, randomised controlled,
tract bleeding. Arch Intern Med 2012;172:1484-91. phase 3 trial. Lancet 2018;392:31-40.
30. Wuerth BA, Rockey DC. Changing epidemiol- 13. Jacobi J, Bircher N, Krinsley J, et al. Guidelines
ogy of upper gastrointestinal hemorrhage in the for the use of an insulin infusion for the manage-
last decade: a nationwide analysis. Dig Dis Sci ment of hyperglycemia in critically ill patients.
2018;63:1286-93. Crit Care Med 2012;40:3251-76.
14. Jonklaas J, Bianco AC, Bauer AJ, et al. Guidelines
Endocrine Emergencies for the treatment of hypothyroidism: prepared by
1. American Diabetes Association (ADA). Diabetes the American Thyroid Association Task Force
care in the hospital, nursing home, and skilled nurs- on Thyroid Hormone Replacement. Thyroid
ing facility. Diabetes Care 2015;38(suppl):S80-5. 2014;24:1670-751.
2. Annane D, Pastores SM, Rochwerg B, et al. 15. Kavanagh BP, McCowen KC. Glycemic control in
Guidelines for the diagnosis and management of the ICU. N Engl J Med 2010;363:2540-6.
critical illness-related corticosteroid insufficiency 16. Kitabchi AE, Miles JM, Umpierrez GE.
(CIRCI) in critically ill patients (part I): Society Hyperglycemic crises in adult patients with
of Critical Care Medicine (SCCM) and European diabetes. Diabetes Care 2009;32:1335-43.
Society of Intensive Care Medicine (ESICM) 2017. 17. Kitabchi AE, Murphy MB, Spencer J, et al. Is a
Crit Care Med 2017;45:2078-88. priming dose of insulin necessary in a low-dose
3. Bornstein SR. Predisposing factors for adrenal insulin protocol for the treatment of diabetic keto-
insufficiency. N Engl J Med 2009;360:2328-39. acidosis? Diabetes Care 2008;31:2081-5.
4. Brunkhorst FM, Engel C, Bloos F, et al. Intensive 18. Klubo-Gwiezdzinska J, Wartofsky L. Thyroid
insulin therapy and pentastarch resuscitation in emergencies. Med Clin North Am 2012;96:385-403.
severe sepsis. N Engl J Med 2008;358:125-39. 19. Krinsley JS, Preiser JC. Time in blood glucose
5. Burch HB, Cooper DS. Management of Grave’s range 70 to 140 mg/dl >80% is strongly associated
disease: a review. JAMA 2015;314:2544-54. with increased survival in non-diabetic critically
Erratum in JAMA 2016;9:315:614. ill adults. Crit Care 2015;19:179.
6. Cooper DS. Antithyroid drugs. N Engl J Med 20. Krinsley JS, Schultz MJ, Spronk PE, et al. Mild
2005;352:905-17. hypoglycemia is independently associated with
7. Cooper MS, Steward PM. Corticosteroid insuf- increased mortality in the critically ill. Crit Care
ficiency in acutely ill patients. N Engl J Med 2011;15:R173.
2003;348:727-34. 21. Kwaku MP, Burman KD. Myxedema coma. J
8. Cryer PE. Mechanisms of hypoglycemia- Intensive Care Med 2007;22:224-31
associated autonomic failure in diabetes. N Engl J 22. Marik PE, Preiser JC. Toward understanding tight
Med 2013;369:362-72. glycemic control in the ICU: a systematic review
9. Cryer PE, Axelrod L, Grossman AB, et al. Evaluation and metaanalysis. Chest 2010;137:544-51.
and management of adult hypoglycemic disorders: 23. Murad MH, Coto-Yglesias F, Wang AT, et al.
Clinical review: drug-induced hypoglycemia:

ACCP Updates in Therapeutics® 2022: Critical Care Pharmacy Preparatory Review and Recertification Course

898
Hepatic Failure/GI/Endocrine Emergencies

systematic review. J Clin Endocrinol Metab


2009;94:741-5.
24. NICE-SUGAR Study Investigators, Finfer S,
Chittock DR, et al. Intensive versus conventional
glucose control in critically ill patients. N Engl J
Med 2009;360:1283-97.
25. NICE-SUGAR Study Investigators, Finfer S, Liu
B, et al. Hypoglycemia and risk of death in criti-
cally ill patients. N Engl J Med 2012;367:1108-18.
26. Preiser JC, Devos P, Ruiz-Santana S, et al. A pro-
spective randomized multi-centre controlled trial
on tight glucose control by intensive insulin ther-
apy in adult intensive care units: the Glucontrol
study. Intensive Care Med 2009;35:1738-48.
27. Ross DS, Burch HB, Cooper DS, et al. 2016
American Thyroid Association guidelines for
diagnosis and management of hyperthyroid-
ism and other causes of thyrotoxicosis. Thyroid
2016;26:1343-1421.
28. Rushworth RL, Torpy DJ, Falhamar H. Adrenal
crisis. New Engl J Med 2019;381:852-61.
29. Salvatori R. Adrenal insufficiency. JAMA
2005;294:2481-8.
30. Van den Berghe G, Wilmer A, Hermans G, et al.
Intensive insulin therapy in the medical ICU. N
Engl J Med 2006;354:449-61.
31. Van den Berghe G, Wouters P, Weekers F, et al.
Intensive insulin therapy in critically ill patients.
N Engl J Med 2001;345:1359-67.
32. Vriesendrop TM, van Santen S, DeVries JH, et
al. Predisposing factors for hypoglycemia in the
intensive care unit. Crit Care Med 2006;34:96-101.

ACCP Updates in Therapeutics® 2022: Critical Care Pharmacy Preparatory Review and Recertification Course

899
Hepatic Failure/GI/Endocrine Emergencies

ANSWERS AND EXPLANATIONS TO PATIENT CASES

1. Answer: B 4. Answer: D
Intravenous acetylcysteine increases transplant-free Somatostatin significantly decreases fistula output
survival rates for patients with NAI-ALF, particu- compared with placebo (Answer A is incorrect). Total
larly for patients with low-grade encephalopathy. Oral parenteral nutrition increases spontaneous closure rates
acetylcysteine has not been studied in a randomized by reducing GI secretions (Answer C is incorrect),
controlled trial for NAI-ALF (Answer C is incorrect), and octreotide had a beneficial effect on fistula output
and the dosing strategy for NAI-ALF is different from in one small study (Answer B is incorrect). Although
the 21-hour intravenous regimen for acetaminophen glutamine has been associated with spontaneous rates
overdose (Answer A is incorrect). The dosing strategy of fistula closure, it has been not been associated with
for NAI-ALF is a 72-hour regimen with a 150-mg/kg reduced fistula output (Answer D is correct).
bolus, followed by a 12.5-mg/kg/hour dose for 4 hours
and then a 6.25-mg/kg/hour dose for 67 hours (Answer 5. Answer: B
B is correct). Oral glutamine is not used for NAI-ALF; Of the possible answers, only alvimopan has been shown
it has been studied to aid in the healing of GI fistulas to reduce the incidence of ileus postoperatively (Answer
(Answer D is incorrect). B is correct). Metoclopramide has shown mixed results,
though the antiemetic properties may be beneficial as
2. Answer: B adjunctive therapy in POI, and octreotide is not used in
Osmotic agents are first-line treatment for control of the prevention of POI (Answers C and D are incorrect).
ICP. Although hypertonic saline prevents ICP eleva- All opioids can contribute equally to POI (Answer A is
tions, the continuous infusion is not used for acute incorrect).
control (Answer A is incorrect). For acute control of ICP
elevations, mannitol boluses are used first line provided 6. Answer: A
serum osmolaity is less than 320 (Answer B is cor- According to the simplified Apfel risk score criteria,
rect). Hyperventilation and barbiturates are only used this patient has four risk factors for developing PONV
to control ICP elevations when other options have failed (female sex, nonsmoker, history of motion sickness, and
(Answers C and D are incorrect). perioperative opioids). These risk factors place her at high
risk of developing PONV, estimated at greater than 80%
3. Answer: C (Answers B and D are incorrect). Patients with a high risk
This patient has severe acute necrotizing pancreatitis of PONV should receive more than two pharmacologic
because she has not improved after the first 48 hours, interventions to prevent PONV (Answer A is correct;
and her CT reveals pancreatitic necrosis involving more Answer C is incorrect).
than 30% of her pancreas. There appears to be no benefit
with using prophylactic antibiotics for patients with nec- 7. Answer: D
rotizing AP, in reducing either mortality rates or rates of Frank bloody output from the rectum is more indicative
pancreatic and extrapancreatic infections, particularly of a lower GI bleed than an UGIB. However, patients
in more recent studies (Answers A and D are incorrect). with a brisk UGIB may present with bright red blood per
Surgical management for sterile necrosis is only recom- rectum. Although a lower GI bleed is more likely, the
mended if patients have gastric outlet obstruction and/ patient should be initiated on a PPI infusion and undergo
or bile duct obstruction (Answer B is incorrect). For an esophagogastroduodenoscopy as soon as possible
patients with severe acute necrotizing pancreatitis, it is (Answer D is correct). After the esophagogastroduo-
recommended to defer antibiotics unless there is sug- denoscopy, the patient should have a colonoscopy. The
gestion of infection or if patients have not improved priority in this patient’s case is to evaluate for an UGIB
within 7–10 days (Answer C is correct). because the finding of “coffee-ground” material on
NG lavage suggests that the bleeding source is UGIB
(Answer C is incorrect). Although previous guidelines
recommended initiating PPIs before endoscopy, the

ACCP Updates in Therapeutics® 2022: Critical Care Pharmacy Preparatory Review and Recertification Course

900
Hepatic Failure/GI/Endocrine Emergencies

most recent version does not recommend for or against


this practice (Answer A is incorrect). High-dose PPIs
only need to be continued if a high-risk lesion is identi-
fied on endoscopy (Answer B is incorrect).

8. Answer: A
The patient, who presented with signs and symptoms
of myxedema coma, was given this diagnosis by the
treating team. Of the possible answers listed, only levo-
thyroxine is appropriate for the treatment of myxedema
coma (Answer A is correct). Insulin, propylthiouracil,
and propranolol play no role in the treatment of myx-
edema coma (Answers B–D are incorrect).

ACCP Updates in Therapeutics® 2022: Critical Care Pharmacy Preparatory Review and Recertification Course

901
Hepatic Failure/GI/Endocrine Emergencies

ANSWERS AND EXPLANATIONS TO SELF-ASSESSMENT QUESTIONS

1. Answer: C 4. Answer: C
The most recent guidelines from the American Fistula output is defined as high if the output is greater
Association for the Study of Liver Diseases and the than 500 mL/day, moderate if it is 200–500 mL/day, and
U.S. Acute Liver Failure Study Group define ALF as low if it is less than 200 mL/day. This patient’s fistula
a coagulopathy, usually an INR of 1.5 or more, with output has decreased significantly (Answers A and D
any degree of encephalopathy in patients without pre- are incorrect) from 570 mL/day to 250 mL/day, but it
existing liver disease (Answer C is correct). Although is still not enough to classify her fistula output as low
jaundice, thrombocytopenia, and leukocytosis can occur (Answer B is incorrect). Her output has decreased from
in patients with ALF, they are not currently defined as high to moderate (Answer C is correct).
hallmark signs of the disease that exist in all patients
(Answers A, B, and D are incorrect). 5. Answer: B
In a long-term study of alvimopan for opioid-induced
2. Answer: A bowel dysfunction, alvimopan was associated with
Administration of acetylcysteine is recommended in all higher rates of myocardial infarction than was placebo.
ALF cases when acetaminophen is suspected as a cause, To mitigate this risk, the FDA has limited the use of
regardless of the acetaminophen concentration (Answer alvimopan to short-term, inpatient use, and patients can-
B is incorrect). Although oral and intravenous formula- not receive more than 15 doses (Answer B is correct);
tions have efficacy for the treatment of acetaminophen however, its use is not contraindicated in patients with
overdose, the intravenous formulation is recommended a history of myocardial infarction (Answer D is incor-
when patients have greater than grade I encephalopa- rect). The FDA-approved dose is 12 mg twice daily, and
thy or hypotension or when they cannot tolerate oral there is no requirement for QTc monitoring with alvi-
therapy (Answers C and D are incorrect). Intravenous mopan (Answers A and C are incorrect).
acetylcysteine is recommended for most patients who
present with liver failure and can be extended beyond 6. Answer: D
the 21-hour regimen, especially if therapy was initiated Dexamethasone and aprepitant should be given before
more than 8 hours after ingestion or baseline concen- inducing anesthesia for the prevention of PONV
trations were greater than 300 mg/dL (Answer A is (Answers A and C are incorrect). Ondansetron and other
correct). serotonin-3 antagonists are most effective if given at the
end of surgery (Answer B is incorrect). Droperidol is
3. Answer: D effective for the prevention of PONV when given at the
Patients with AP should not be kept NPO (Answer A end of surgery (Answer D is correct).
is incorrect). Studies that have compared TPN with
enteral feeding in AP have shown that enteral feeding is 7. Answer: A
associated with reduced mortality and infectious com- H. pylori is a recognized carcinogen and should be
plications. Thus, enteral feeding is recommended over eradicated using a 14-day PPI/antibiotic combination
TPN for AP, if it is tolerated (Answer B is incorrect). (Answer B is incorrect). Patients with an acute UGIB
Enteral feeding can be given by either the NJ or the NG who present with a high-risk bleed should have a diag-
route for AP, though the NG route may increase the risk nostic and therapeutic endoscopy within 24 hours of
of aspiration. Because this patient has several admis- admission (Answer C is incorrect). Blood transfusions
sions for aspiration, it is safer to use the NJ route over should be administered to keep the hemoglobin con-
the NG route; also, there are more data on the NJ route centration greater than 7 g/dL (Answer D is incorrect).
of enteral feeding in AP because it prevents stimulation Therefore, the only inappropriate treatment option is
of pancreatic enzymes (Answer C is incorrect, Answer using octreotide (Answer A is correct).
D is correct).

ACCP Updates in Therapeutics® 2022: Critical Care Pharmacy Preparatory Review and Recertification Course

902
Hepatic Failure/GI/Endocrine Emergencies

8. Answer: B
Thyroid storm is an uncommon but deadly manifesta-
tion of hyperthyroidism; therefore, TSH will be low,
whereas T4 and T3 will be high (Answer B is correct).
Myxedema coma is a manifestation of hypothyroidism;
therefore, patients will typically have high TSH and low
T4/T3. When TSH is high, both T3 and T4 are typically
low (Answer A is incorrect). Conversely, if TSH is low,
both T3 and T4 are typically high (Answers C and D are
incorrect).

ACCP Updates in Therapeutics® 2022: Critical Care Pharmacy Preparatory Review and Recertification Course

903
Hepatic Failure/GI/Endocrine Emergencies

Appendix 1. Most Common or Well-Described Drugs Associated with DILI and the Patterns of Liver Injury
Drug Class Typical Pattern of Injury
Antibiotics
•  Amoxicillin/clavulanate •  Cholestatic injury
•  Isoniazid •  Hepatocellular injury
•  Trimethoprim/sulfamethoxazole •  Cholestatic injury, can also be hepatocellular injury
•  Fluoroquinolones •  Hepatocellular, cholestatic, or mixed injury
•  Macrolides •  Hepatocellular injury
•  Nitrofurantoin •  Hepatocellular injury
•  Minocycline •  Hepatocellular injury
Antiepileptics
•  Phenytoin •  Hepatocellular, cholestatic, or mixed injury
•  Carbamazepine •  Hepatocellular, cholestatic, or mixed injury
•  Lamotrigine •  Hepatocellular injury
•  Valproate •  Hepatocellular injury, may also cause hyperammonemia
Nonsteroidal anti-inflammatory drugs •  Hepatocellular
Immune modulators
•  Interferon •  Hepatocellular injury
•  Anti-TNF agents •  Hepatocellular injury
•  Azathioprine •  Cholestatic injury, can also be hepatocellular injury
Herbal medications and dietary supplements
•  Green tea extract •  Hepatocellular injury
•  Flavocoxid •  Hepatocellular, cholestatic, or mixed injury
Miscellaneous
•  Methotrexate •  Fatty liver
•  Allopurinol •  Hepatocellular or mixed injury
•  Amiodarone •  Hepatocellular, cholestatic, or mixed injury
•  Androgen-containing steroids •  Cholestatic injury
•  Inhaled anesthetics •  Hepatocellular injury
•  Sulfasalazine •  Hepatocellular, cholestatic, or mixed injury
•  Proton pump inhibitors •  Hepatocellular injury
TNF = tumor necrosis factor.
Information from: Chalasani NP, Hayashi PH, Bonkovsky HL, et al. ACG clinical guideline: the diagnosis and management of idiosyncratic drug-induced liver injury.
Am J Gastroenterol 2014;109:950-66.

ACCP Updates in Therapeutics® 2022: Critical Care Pharmacy Preparatory Review and Recertification Course

904
Practice Administration
and Development:
Protocol Development and
Quality Improvement
Lisa Hall Zimmerman, Pharm.D., FCCP, FCCM, BCPS,
BCNSP, BCCCP
Beaumont Hospital - Royal Oak
Department of Pharmaceutical Services
Royal Oak, Michigan

Jaime Robenolt Gray, Pharm.D., FCCM, BCCCP


Temple University Health System
Department of Pharmacy
Philadelphia, Pennsylvania
Practice Administration and Development: Protocol Development and Quality Improvement

Practice Administration
and Development:
Protocol Development and
Quality Improvement
Lisa Hall Zimmerman, Pharm.D., FCCP, FCCM, BCPS,
BCNSP, BCCCP
Beaumont Hospital - Royal Oak
Department of Pharmaceutical Services
Royal Oak, Michigan

Jaime Robenolt Gray, Pharm.D., FCCM, BCCCP


Temple University Health System
Department of Pharmacy
Philadelphia, Pennsylvania

ACCP Updates in Therapeutics® 2022: Critical Care Pharmacy Preparatory Review and Recertification Course

907
Practice Administration and Development: Protocol Development and Quality Improvement

Learning Objectives Self-Assessment Questions


Answers and explanations to these questions may be
1. Determine the steps involved in the development of found at the end of this chapter.
a guideline and/or policy.
2. Demonstrate how to perform a medication use eval- 1. As the clinical coordinator/manager, you have
uation (MUE) and list high-risk medications and received a request to add the critical care pharma-
medication-related processes that are suited for a cist’s review of all intensive care unit (ICU) patients.
MUE. You recognize that you lack the necessary person-
3. Describe different quality improvement tools and nel to accomplish this request. Which justification
identify opportunities to optimize outcomes in a method would offer the most significant financial
critically ill patient population. impact on the institution to justify the position?
4. Identify processes or quality improvement initia- A. Developing protocols in the critically ill patient.
tives that would benefit from a gap analysis. B. Reducing prescribing errors.
5. Explain types of pharmacotherapeutic interventions C. Reducing medication administration errors.
and documentation processes, and justify the value D. Implementation of an anticoagulation reversal
of clinical pharmacy services. stewardship program.

2. You are the critical care pharmacist implementing


Abbreviations in This Chapter a delirium screening tool in critically ill patients
according to the Society of Critical Care Medicine
ADE Adverse drug event guidelines. Which would best be described as a
ASHP American Society of Health-System practice environment barrier to implementing delir-
Pharmacists ium screening in your ICU?
CMS Centers for Medicare & Medicaid Services
A. The nursing standard work does not include
CPOE Computerized provider order entry
delirium screening.
CPS Clinical pharmacy services
B. Screening process takes up valuable nursing
DUE Drug use evaluation
time.
EBM Evidence-based medicine
C. Providers are unfamiliar with how to perform
EHR Electronic health record
delirium screening.
ICU Intensive care unit
D. Providers believe their knowledge of bedside
LOS Length of stay
delirium is adequate and does not require a
MUE Medication use evaluation
screening tool.
PAI Practice Advancement Initiative
PI Performance improvement
3. The pharmacy and therapeutics (P&T) committee
P&T Pharmacy and therapeutics (committee)
would like to evaluate the use of pharmacotherapy
QA Quality assurance
in stress ulcer prophylaxis (SUP) in the ICU. Which
QI Quality improvement
is the best method for making this evaluation?
SUP Stress ulcer prophylaxis
TJC The Joint Commission A. Perform a medication use evaluation (MUE).
B. Administer a performance improvement (PI)
initiative.
C. Review adverse drug event (ADE) data from
the ICU.
D. Review medication error data from the ICU.

ACCP Updates in Therapeutics® 2022: Critical Care Pharmacy Preparatory Review and Recertification Course

908
Practice Administration and Development: Protocol Development and Quality Improvement

4. Which medication use process is best suited for an 8. As the clinical pharmacist in the ICU, you are
MUE? asked to evaluate the quality initiatives specific to
A. Pharmacist verification times for routine orders critically ill patients. Which best describes a Centers
in the ICU. for Medicare & Medicaid Services (CMS) quality
B. Management of warfarin-induced metric?
hypoprothrombinemia. A. Sepsis management bundle and venous throm-
C. Drug interaction warnings on the computerized boembolism prophylaxis.
provider order entry (CPOE) system. B. Thrombolytic therapy for stroke and SUP.
D. Duplicate warnings on the CPOE system. C. Heart failure readmission and pneumonia vac-
cination rates.
5. You have been asked to develop a quality improve- D. Ventilator bundles and acute coronary syn-
ment (QI) strategy for your ICU regarding when drome readmission rates.
to institute targeted temperature management in
patients after cardiac arrest. The current process has
been defined. You identify key stakeholders in this
patient population to collaborate with you in this
process improvement. Which is the next best step in
developing the QI program?
A. Determine which patients with cardiac
arrest should receive targeted temperature
management.
B. Evaluate the current clinical guidelines associ-
ated with targeted temperature management.
C. Determine the current process of targeted tem-
perature management.
D. Evaluate the data associated with patients
receiving targeted temperature management.

6. Using the Design, Measure, Analyze, Improve, and


Control method, which best represents what is opti-
mized in the Analyze phase?
A. Data are collected.
B. Methods are established.
C. Change process is implemented.
D. Data are analyzed.

7. Which metric best quantifies a critical care pharma-


cist’s activities?
A. Weighing each variable to quantify measured
activities.
B. Number of cardiac arrest codes attended each
month.
C. Number and severity of hospital-acquired
Clostridioides difficile infections each year.
D. Number of in-services provided each year.

ACCP Updates in Therapeutics® 2022: Critical Care Pharmacy Preparatory Review and Recertification Course

909
Practice Administration and Development: Protocol Development and Quality Improvement

BPS Critical Care Pharmacy Specialist Examination Content Outline

This chapter covers the following sections of the Critical Care Pharmacy Specialist Examination Content Outline:
1. Domain 2: Practice Management, Policy, and Quality Improvement
a. Task 1: 1
b. Task 3: 2, 4
c. Task 5: 1
d. Task 2: 1
e. Task 6: 3
f. Task 4: 1
2. Domain 3: Evidence-Based Medicine, Scholarship, Education, and Professional Development
a. Task 3: 1

ACCP Updates in Therapeutics® 2022: Critical Care Pharmacy Preparatory Review and Recertification Course

910
Practice Administration and Development: Protocol Development and Quality Improvement

I.  POLICY AND GUIDELINE DEVELOPMENT

Objective: Critical care pharmacists should participate in developing ICU and institutional policies, procedures, and
education.

A. Policy and Procedure


1. Policy – A course or plan of action; the existence of written policies and procedures establishes standards
of practice or quality/compliance measures and protects against error. Most policies are mandatory and
may result in disciplinary action if not followed. Statement that clearly and unambiguously describes
the organization’s guiding principles and views about a particular matter
a. Benefits
i. Improves consistency and efficiency of work processes
ii. Provides a training tool during a new employee’s orientation
iii. Provides reference material for education and practice
iv. Minimizes practice variations
v. Reduces the organizational risk by mandating compliance
b. Development and implementation should involve all relevant team members to ensure all concerns
are addressed and to promote input from all stakeholders.
2. Procedure – A series of actions or tasks performed in a specific way or sequence that is intended to
achieve a desired result or accomplish a task in a consistent manner
a. Requires approval by the organizational committee
b. Describes each team member’s responsibility
c. Defines the expected outcome(s)
d. Procedures may be described in the following forms:
i. Written instructions
ii. Flowcharts
iii. Checklists
e. Can be used as a QI tool or a source of measures

B. Clinical Guidelines/Protocol
1. Supports clinical decision-making by defining best practice
2. Uses evidence-based and standardized treatment options
3. Developed by examining the evidence and gaining consensus among practitioners
4. Interdisciplinary team: Patient, physician, nurse, respiratory therapist, pharmacist, clinical dietitian,
physical therapist, social worker and/or case manager, hospital administrators, and patient’s family
5. Voluntary – Allows for deviation depending on clinical scenario
6. Protocol can be an extension of a clinical policy or procedure.
7. Clinical guidelines and protocols are often kept separate from institutional policies and procedures.
8. Examples of disease management guidelines include:
a. Pulmonary embolism management
b. Pain, Agitation, Delirium, Immobility, and Sleep (PADIS) management
c. Spontaneous awakening and breathing trials
d. Anticoagulation reversal
9. Critical care pathways
10. Formulary proposals
11. Medication reconciliation for admission and transfer within the health care system
12. Clinical decision support

ACCP Updates in Therapeutics® 2022: Critical Care Pharmacy Preparatory Review and Recertification Course

911
Practice Administration and Development: Protocol Development and Quality Improvement

a. Consider ways to incorporate guidelines/protocols into CPOE to facilitate ordering and monitoring
in conjunction with the guideline/protocol.

C. Framework of a Policy and/or Procedure


Critical care pharmacists should lead, or be actively involved with, the assessment of medication-related
guidelines, protocols, practice changes, or PI initiatives in the ICU.
1. Purpose – For whom the policy is intended, why the policy is needed, and the desired goals
2. Definitions – Provide the reader with a clear understanding of the language
3. Policy – Intent of the policy
4. Provisions
5. Procedure
6. Resources/references
7. Owner
8. Date created/modified
9. Approvals

D. How to Develop Policies


1. Planning
a. Define and prioritize policy goals and objectives.
b. Identify key stakeholders and include them in the policy development process.
c. Appoint a project facilitator/leader.
d. Clearly delineate each team member’s roles and responsibilities in the policy development process.
e. Develop a strategy for tracking and managing each group’s progress.
2. Consultative process for developing a particular policy
a. Solicits input from organizational or external stakeholders
b. Promotes good relations between sites and service providers
c. Encourages participation and feeling of ownership in the process
d. Provides new ideas and expertise
e. Alternative viewpoint may identify inconsistency, ambiguity, and/or duplication.
3. General steps in drafting
a. Write a first draft.
b. Distribute the draft, and consult across the same organization/stakeholders/experts for comments.
c. Review feedback, both written and verbal, and amend/revise the draft.
d. Redistribute the draft for additional/final feedback.
e. Write the final draft.
f. Document the date that the policy was ratified and then subsequent review dates.
g. Committee review and approval.
h. Incorporate the new policy into the policy and procedures.
i. Communicate the new policy to all relevant people.
4. Identify policy review process.
a. Formal process may be required within the institution (i.e., standing policy and procedures
committee, which develops and reviews the policy and provides recommendations and decisions).
b. Involvement and approval from the P&T and critical care committees or other oversight committees

E. Education
1. Create education for the key stakeholders (e.g., physicians, advanced practice providers, nurses,
pharmacists, dietitian, respiratory therapists).
2. Disseminate education through newsletters, intranet posts, in-services, huddles, quarterly updates,
grand rounds, one-on-one direct education, online learning modules, etc.

ACCP Updates in Therapeutics® 2022: Critical Care Pharmacy Preparatory Review and Recertification Course

912
Practice Administration and Development: Protocol Development and Quality Improvement

3. Assessment of education through annual competency examination, skills assessment, monthly


simulation modules, etc.

F. Review of existing policies. Continuous QI requires documentation and training for pharmacists and staff.
Many state boards of pharmacy are requiring reportable events to be identified and documented. Policies
and procedures can also be developed for analyzing the data collected to assess causes and contributing
factors so that findings can be used to improve outcomes.
1. Every policy should regularly be reviewed for relevance and appropriateness (e.g., every 1–3 years),
depending on organizational structure/standards.
2. Monitor and evaluate compliance with, and effect on, policies and guidelines.
3. Evaluate the policy on a routine schedule
a. Determine need for updates to the policy.
b. Evaluation strategies
c. Ongoing monitoring
d. Presentation of data
e. Consumer feedback
f. Stakeholder feedback (e.g., physician, pharmacy, nursing, policy steering committee, critical care
committee)
g. Planning day agenda
h. Forms: Data collection sheets, data reporting format
i. Key stakeholder questionnaires
4. Evidence-based critical care literature and clinical practice guidelines in designing a patient-specific
plan of care
a. Definition: “The conscientious, explicit and judicious use of current best evidence in making
decisions about the care of individual patients. The practice of evidence-based medicine means
integrating individual clinical expertise with the best available external clinical evidence from
systematic research.” (BMJ 1996;312:71)
b. Pros and cons of evidence-based medicine (EBM)
i. Advantages
(a) Critical appraisal skills of the literature improve with the review of EBM.
(b) Wasteful, outdated, or harmful practices can be abandoned.
(c) Presupposes that we keep up-to-date; ideally, a systematic process of incorporating new
EBM is included
(d) Makes the decision-making process transparent to colleagues and patients
(e) Leads to greater appreciation of the evidence for our practice as well as the inherent
uncertainties
ii. Disadvantages
(a) Time-consuming
(b) Sometimes impossible (when there is no published literature on a question)
(c) Useful papers may be disregarded because of minor blemishes (rescue bias).
(d) Contradictory study findings leading to differences in interpretation of benefit
(e) No science to tell us how robust the evidence must be for it to be incorporated into clinical
practice
(f) External validity is subjective, and evidence can be misapplied.
(g) Easy-to-prove techniques more favored in literature
(h) It is never “up-to-date.”
(i) Tends to emphasize the priority of randomized controlled trials (which have inherent flaws)
to the exclusion of other study designs (which may be appropriate in certain settings)

ACCP Updates in Therapeutics® 2022: Critical Care Pharmacy Preparatory Review and Recertification Course

913
Practice Administration and Development: Protocol Development and Quality Improvement

(j) May underemphasize patient values and interests


(k) Publications with favorable results are more likely to be published than are those without
favorable results (publication bias).
5. Barriers to implementing evidence
a. Practice environment (organizational context)
i. Financial disincentives – Lack of reimbursement
ii. Organizational constraints – Lack of time; insufficient leadership or administrative support;
staffing, equipment, or resource constraints
iii. Perception of liability – Risk of formal complaint
iv. The patient’s expectations – Expressed wishes related to care
b. Prevailing opinion (social context)
i. Standard work – Usual routine
ii. Opinion leaders – Key individuals not in agreement with evidence
iii. Medical training – Obsolete knowledge
iv. Advocacy – By pharmaceutical companies
c. Knowledge, skills, and attitudes (professional context)
i. Clinical uncertainty – Necessary test for vague symptoms
ii. Sense of competence – Self-confidence in skills
iii. Compulsion to act – Need to do something
iv. Information overload – Inability to appraise evidence

G. Checklist to Follow When Developing Guidelines


1. Scope and purpose
a. Objectives are specifically described.
b. Health questions covered by the guideline are specifically described.
c. The patient population to which the guideline applies is specifically described.
2. Stakeholder involvement
a. Guideline development group includes individuals relevant to the guideline.
b. Target population’s views and preferences have been identified.
c. Target users of the guideline are defined.
3. Development of guideline
a. Literature search is systematic.
b. Criteria for selecting evidence are clearly defined.
c. Strengths and limitations of evidence are described.
d. Health benefits, adverse effects, and risks are considered when developing recommendations.
e. Link between recommendations and supporting evidence is provided.
f. Guideline is externally reviewed by experts.
g. Defined times for guideline updates are provided.
4. Clarity of recommendations
a. Recommendations are clear and unambiguous.
b. Different options are clearly presented.
c. If involving medication therapy, dosing is clear, including initial starting rate and titrations for
continuous infusions.
5. Applicability
a. Guideline describes facilitators and barriers to the application.
b. Guideline provides tools on how it should be applied to practice.
c. Guideline presents monitoring and/or auditing criteria.
6. Monitoring of adherence to the guidelines through MUEs to assess compliance or need for revisions.

ACCP Updates in Therapeutics® 2022: Critical Care Pharmacy Preparatory Review and Recertification Course

914
Practice Administration and Development: Protocol Development and Quality Improvement

Patient Case

1. As the critical care pharmacist, you have been asked to develop a guideline for reversal agents used in the
setting of bleeding associated with anticoagulation therapy. Which is the best example of a component that
should be involved in establishing the clinical pathway?
A. Evaluate the closed-loop technology to support a clinical pathway, and establish a physician champion.
B. Use EBM supporting the clinical pathway and agreement among practitioners.
C. Develop a clinical protocol, and obtain agreement among practitioners.
D. Evaluate formulary proposals and the EBM supporting the clinical practice.

II.  GAP ANALYSIS

A. A gap analysis is an assessment of a practice model that may be within your health system or pharmacy
and that is compared with a best practice model (actual vs. potential performance). Within pharmacy, a gap
analysis may focus on pharmacy services, pharmacy technology, or a specific medication or medication
process. A gap analysis can also be used to analyze gaps in processes and between the existing outcome and
the desired outcome. This process can be summarized as follows.

B. Goals of a Gap Analysis – To provide the project team with an understanding of the differences between
current and best practices. An assessment must be made of the barriers that exist before best practice can
be implemented successfully.
1. Review systems.
2. Develop requirements.
3. Comparisons
4. Implications
5. Recommendations

C. Gap Analysis
1. Identify the existing process or gap analysis questions.
2. Identify the existing outcome.
3. Identify the desired outcome.
4. Identify the process for achieving the desired outcome.
5. Assess the response to the level of compliance or implementation (e.g., fully, partly, in progress, no
activity, or noncompliant).
6. Develop an action plan to fill the gap.
7. Develop and prioritize the requirements, and develop a timeline for completion to bridge the gap.

D. Best Practice Evaluation


1. Review of the primary literature
2. Survey practitioners – Feedback from practitioners (e.g., ICU pharmacists, nurses, physicians)
3. Benchmarking
4. Survey strategies
a. Have a single well-defined objective.

ACCP Updates in Therapeutics® 2022: Critical Care Pharmacy Preparatory Review and Recertification Course

915
Practice Administration and Development: Protocol Development and Quality Improvement

b. Keep the survey short.


c. Assure respondents that survey responses will be kept anonymous (if possible).
d. Survey a large-enough sample to ensure meaningful results.
e. Explain the purpose of the survey, and how you intend to use the results, in your invitation/greeting
page.
f. Design the survey for easily measurable results.
g. Ask one thing per question.
h. Avoid biasing the response.
i. Limit the number of required questions.
j. Question order matters – The first question or two should be easy and interesting.
k. Create a logical flow to the questions.
l. Test the survey.
m. Spell out the time expectations in your invitation and on the greeting page.
n. Survey the appropriate people.
o. Share the results and actions with the respondents.

E. A gap analysis for medication safety should include strategies for prevention and mitigation, assessment
and detection, therapeutic use, critical thinking and knowledge, and education. A gap analysis should also
include policies, procedures, protocols, guidelines, competencies, patient care models, educational methods,
and other key components to maximize efficacy and prevent medication errors and harm.

F. Examples of a Gap Analysis Include:


1. Centers for Disease Control and Prevention (CDC): The Core Elements of Hospital Antibiotic
Stewardship Programs: Antibiotic Stewardship Program Assessment Tool (excerpt) (Appendix 1)
2. Opioid ADE prevention
3. Anticoagulation agent ADE prevention
4. Aminoglycoside ADE prevention
5. Intravenous-to-oral conversion of antihypertensives
6. Medication labeling
7. Medication barcoding and scanning
8. Narcotic diversion prevention
9. Sterile admixture services
10. Extemporaneous compounding services
11. Pharmacist activities during a code or rapid response
12. Pharmacy patient care models – Not enough ICU pharmacists to see all ICU patients, based on workload,
documentation requirements
13. American Society of Health-System Pharmacists (ASHP) Practice Advancement Initiative (PAI) to
optimize care through pharmacist-provided comprehensive medication management
14. ICU liberation practices, including ABCDEF bundle
15. Antibiotic streamlining
16. Criteria-based antimicrobials and non-antimicrobials for high-cost/misused drugs
17. Departmental professional development education
18. Institute for Safe Medication Practices (ISMP) Targeted Medication Safety Best Practices for Hospitals
19. The Joint Commission (TJC) Medication Management Standards

ACCP Updates in Therapeutics® 2022: Critical Care Pharmacy Preparatory Review and Recertification Course

916
Practice Administration and Development: Protocol Development and Quality Improvement

III.  QUALITY ASSURANCE, QUALITY/PERFORMANCE IMPROVEMENT

A. Overview
1. Quality improvement (QI) consists of systematic and continuous actions that lead to measurable
improvement in health care services and the health status of targeted patient groups.
2. An organization’s quality is based on the current system (e.g., how things are currently done).
3. Health care performance is defined by an organization’s efficiency, outcome of care, and level of patient
satisfaction. Using benchmarks may help with measurements/goals/outcomes.
4. To achieve a different level of performance (i.e., results) and improve quality, an organization’s current
system needs to be challenged/evaluated.
5. Key components of a successful QI program:
a. QI works as systems and processes.
b. Keeps the patient as the central focus
c. Embraces a team approach
d. Uses data to guide decision-making
6. Improvement strategies
a. Understand the delivery system and key processes.
b. Recognize that resources (inputs) and activities carried out (processes), including the work around
for the actual defined processes, are addressed together to ensure or improve the quality of care
(outputs/outcomes).
7. Quality management departments within a health care institution often share data with the risk
assessment department.
8. QI programs within an institution
a. Executive steering committee
b. Various departments involved in QI initiatives:
i. Pharmacy department
ii. Nursing
iii. Medical staff
iv. Respiratory
v. Quality department
vi. Medical ethics committee
vii. P&T committee
viii. Data reporting
ix. Medication safety committee
x. Compliance department
xi. Critical care/clinical specialty committee
9. Analyzing the quality assurance (QA)/QI program
a. A normal level (upper and lower control limit) should be established for a process to operate.
b. The process is evaluated, and the results are compared with the normal level expected. Control
charts can show the variance of the output of a process over time. The process is considered
in control if the variance between measurements is the normal random variation considered
inherent in the process. If the variance falls outside the limits or has a run of non-natural points,
the process is considered out of control.
c. Example: Established process for daily sedative interruption for mechanically ventilated patients.
The preestablished expected level of daily sedative interruption was established when the protocol
was considered initiated. The QA data collected evaluate the frequency by which the assessments
are being performed over a time interval.

ACCP Updates in Therapeutics® 2022: Critical Care Pharmacy Preparatory Review and Recertification Course

917
Practice Administration and Development: Protocol Development and Quality Improvement

B. Critical Care Pharmacist’s Role in QI – Metrics for evaluating the quality of critical care pharmacy services
1. ICU LOS
2. Hospital LOS
3. ICU readmissions
4. Impact on mortality or morbidity
5. Impact on disease identification, disease prevalence, or clinical outcome
6. Evaluation of infectious diseases within the ICU
7. Cost of care versus diagnosis-related group (DRG) reimbursement
8. Duration of mechanical ventilation
9. Medication management procedures need to be continuously monitored and improved because of their
complexity.
10. Medication safety
11. Direct costs (medication costs, technological upgrades, and software)
12. Indirect costs (salaries, power for building)
13. Data collection, analysis
14. Implement processes to reduce medication costs and waste.

C. National Quality Initiatives


1. The Institute of Medicine – Chartered in 1970. Published reports titled “The Urgent Need to Improve
Health Care Quality,” “Crossing the Quality Chasm,” “To Err Is Human”
2. The Institute for Healthcare Improvement (IHI) – Founded in 1991
a. 100,000 Lives Campaign, 5 Million Lives Campaign (December 12, 2006 – December 9, 2008)
b. No needless list
i. No needless deaths
ii. No needless pain or suffering
iii. No helplessness in those served or serving
iv. No unwanted waiting
v. No waste
vi. No one left out
c. Critical care initiatives
i. Acute myocardial infarction
ii. Catheter-associated urinary tract infections
iii. Central line–associated bloodstream infections
iv. Health care–associated infections
v. Sepsis detection and initial management
vi. Medication reconciliation to prevent ADEs
vii. Pain, agitation/sedation, delirium, immobility, and sleep (PADIS) (i.e., the ABCDEF bundle)
viii. Surgical site infections
ix. Ventilator-associated pneumonia
x. Rapid response teams
xi. High-alert medication safety
xii. Pressure ulcer care
d. The five rights of medication administration - the right patient, the right drug, the right dose, the
right route, and the right time
e. Ask “Why” five times to get to the root cause - key to solving a problem
3. National Quality Forum – Created in 1999
a. More than 300 measures, indicators, events, practices, and other products to help assess quality.
Has been endorsed to become the gold standard of measuring health care quality

ACCP Updates in Therapeutics® 2022: Critical Care Pharmacy Preparatory Review and Recertification Course

918
Practice Administration and Development: Protocol Development and Quality Improvement

b. Patient safety–endorsed measures notable for ICU care (2015)


i. ADEs
ii. Catheter-associated urinary tract infections
iii. Central line–associated bloodstream infections
iv. Surgical site infections
v. Ventilator-associated pneumonia
vi. Ventilator-associated events
vii. Stroke
viii. Acute myocardial infarction
ix. Falls
x. Pressure ulcer rate
4. The Leapfrog Group – Launched in 2000
a. Hospital quality and safety survey – Voluntary survey of hospitals rating themselves on quality
and safety practices
b. Reported at www.leapfroggroup.org
c. ICU-related measures, health care–associated infections
i. Catheter-associated urinary tract infections
ii. Catheter-associated bloodstream infections
iii. Inpatient hospital-onset MRSA infections
iv. Inpatient hospital-onset C. difficile infections
v. Surgical site infections
vi. Specially trained physicians care for ICU patients (this is from Leapfrog report).
5. The Joint Commission (TJC) – Founded in 1951
a. One of three CMS accreditation organizations, TJC is an independent, not-for-profit organization
that sets the standards for accreditation in health care
b. Tracer methodology – Method of evaluation done during an on-site survey that traces the health
care experiences of a patient while in the hospital
c. Identifies, tests, and specifies standardization of performance measures
d. 2021 National Patient Safety Goals - effective January 2021
i. Identify patients correctly.
(a) Use at least two ways to identify patients. For example, use the patient’s name and date
of birth. This is done to make sure that each patient receives the correct medication and
treatment.
ii. Improve staff communication: Send important test results to the right staff person on time.
iii. Use medications safely.
(a) Before a procedure, label medications that are not labeled (e.g., medications in syringes,
cups, and basins). Do this where medications and supplies are set up.
(1) Unless an authorized staff member prepares a medication, takes it directly to the
patient, and administers it without a break in the process, the medication must be
labeled.
(b) Medication or solution labels should include name, strength, concentration, diluent and
volume, and expiration date/time.
(c) Reduce the likelihood of patient harm associated with the use of anticoagulant therapy
(d) Record and pass along correct information about a patient’s medications at admission,
transfer, and discharge. Find out what medications the patient is taking. Compare these
medications with new medications given to the patient. Make sure patients know which
medications to take when they are at home. Tell patients it is important to bring their up-
to-date list of medications each time they visit a physician.

ACCP Updates in Therapeutics® 2022: Critical Care Pharmacy Preparatory Review and Recertification Course

919
Practice Administration and Development: Protocol Development and Quality Improvement

iv. Use alarms safely. Make improvements to ensure alarms on medical equipment are heard and
responded to on time.
v. Prevent infection.
(a) Use the hand cleaning guidelines from the Centers for Disease Control and Prevention or
the World Health Organization. Set goals for improving hand cleaning. Use the goals to
improve hand cleaning.
vi. Identify patient safety risks: Reduce the risk of suicide.
vii. Prevent mistakes in surgery using pre-procedural checklist:
(a) Make sure that the correct surgery is done on the correct patient and at the correct place
on the patient’s body.
(b) Mark the correct place on the patient’s body where the surgery is to be done.
(c) Have a time-out before the surgery to make sure that a mistake is not being made and
document the time-out.
e. Chart-abstracted measures for hospitals. Note that beginning with 01/01/2020 discharges, the
emergency department (ED), immunization (IMM), substance use (SUB), tobacco treatment (TOB),
and venous thromboembolism (VTE) measures previously in the aligned CMS/TJC specifications
manual have been moved to the Specifications Manual for Joint Commission National Quality
Measures for hospital use.
i. Chart-abstracted measures from the hospital inpatient quality reporting (IQR) program;
however, the hospital may still voluntarily report (critical care-focused measures)
(a) Venous thromboembolism - VTE-6-Hospital-acquired, potentially preventable venous
thromboembolism
(b) Immunization - IMM-2 influenza immunization
(c) Surgical Care Improvement Project - SCIP-Inf-4 cardiac surgery patients with controlled
postoperative blood glucose
ii. Required chart-abstracted core measures from the Specifications Manual for Joint Commission
National Quality Measures, version 2022A, posted 7/30/2021 (critical care–focused measures)
(a) Comprehensive stroke - CSTK-04 Procoagulant reversal agent initiation for intracerebral
hemorrhage (ICH)
(b) Comprehensive stroke - CSTK-05a Hemorrhage transformation for patients treated with
intravenous alteplase therapy only
(c) Comprehensive stroke - CSTK-05b Hemorrhage transformation for patients treated with
intravenous alteplase therapy or mechanical endovascular reperfusion therapy
(d) Comprehensive stroke - CSTK-06 Nimodipine treatment administered
(e) Comprehensive stroke - CSTK-08 Thrombolysis in cerebral infarction (TICI) post-
treatment reperfusion grade
(f) Emergency department - ED-1a - Median time from ED arrival to ED departure for
admitted ED patients
(g) Emergency department - ED-2a - Admit decision time to ED departure time for admitted
patients
(h) Stroke - STK-01 Venous thromboembolism (VTE) prophylaxis
(i) Stroke - STK-02 Discharged on antithrombotic therapy
(j) Stroke - STK-03 Anticoagulation therapy for atrial fibrillation/flutter
(k) Stroke - STK-04 thrombolytic therapy
(l) Stroke - STK-05 antithrombotic therapy by end of hospital day 2
(m) Stroke - STK-06 discharged on statin medication
(n) Stroke - STK-08 stroke education

ACCP Updates in Therapeutics® 2022: Critical Care Pharmacy Preparatory Review and Recertification Course

920
Practice Administration and Development: Protocol Development and Quality Improvement

6. Det Norske Veritas (DNV) – Established in 2008:


a. One of three CMS-approved hospital accreditation organizations.
b. Over 500 hospitals have converted from TJC to DNV.
c. DNV does not set any standards for quality, but allows the hospital leadership to develop quality
initiatives (e.g., marking the surgical site is a TJC standard).
d. If the institution is accredited by the DNV, the hospital leadership establishes these standards.
7. Healthcare Facilities Accreditation Program (HFAP)
a. One of three CMS-approved hospital accreditation organizations, the HFAP was originally created
in 1945 to conduct objective review of services provided by osteopathic hospitals.
b. Provides accreditation to all hospitals, ambulatory care/surgical facilities, mental health facilities,
physical rehabilitation facilities, clinical laboratories and critical access hospitals. HFAP also
provides certification reviews for primary stroke centers.
c. Standards are written and cross-walk written tied directly to the Medicare requirements.
8. AHRQ (Agency for Healthcare Research and Quality) – The health services research arm of the U.S.
Department of Health and Human Services. Sponsors the National Quality Measures Clearinghouse –
A “public repository for evidence-based quality measures and measure sets”
9. Hospital Quality Alliance – Created in 2002
a. Hospital Compare was created through the efforts of Medicare and the Hospital Quality Alliance
b. 2004 – Hospitals could voluntarily report data on 10 “starter-set” quality performance measures
and receive incentive payment.
c. 2005 – Quality-of-care data expanded to 21 measures
d. 2008 – Hospital Consumer Assessment of Healthcare Providers and Systems (HCAHPS) – First
national, standardized, publicly reported survey of patients’ perspectives of hospital care
e. Reporting initiatives sharpens the focus on QI.
10. CMS
a. Mission – To “ensure effective up-to-date health care coverage and to promote quality care for
beneficiaries”
b. Quality initiative (established in 2001) empowers consumers with quality-of-care information and
encourages providers to improve the quality of care.
c. Hospital quality initiative (established in 2003) – Hospitals must submit data measures or accept a
reduction in payment.
i. Pay-for-performance measures associated with quality measures
ii. CMS has removed many previously reportable core measures; however, the institution can
choose which previously required measures to report.
iii. FY 2022 Payment Determination Hospital IQR Program Measures (selected measures)

Table 1. CMS Quality Improvement Program Measures for Acute Care Hospitals
Topic Measure
Stroke Measure (STK) Set
• Discharged on antithrombotic therapy (NQF #0435)
• Anticoagulation therapy for atrial fibrillation/flutter (NQF #0436)
• Antithrombotic therapy by the end of hospital day 2 (NQF #0438)
• Discharged on statin medication (NQF #0439)
Venous Thromboembolism (VTE) Measure Set
• Venous thromboembolism prophylaxis (NQF #371)
Sepsis Measure
• Severe sepsis and septic shock: Management bundle (Composite Measure) (NQF #0500)

ACCP Updates in Therapeutics® 2022: Critical Care Pharmacy Preparatory Review and Recertification Course

921
Practice Administration and Development: Protocol Development and Quality Improvement

Table 1. CMS Core Measures (continued)


Topic Measure
Mortality Measures
• Hospital 30-Day, All-cause, Risk-Standardized Mortality Rate (RSMR) following acute myocardial
infarction (AMI) mortality (NQF #0230)
• Hospital 30-Day, All-cause, Risk-Standardized Mortality Rate (RSMR) following heart failure (HF)
hospitalization (NQF #0229)
• Hospital 30-Day, All-cause, Risk-Standardized Mortality Rate (RSMR) following pneumonia
hospitalization (NQF #0468)
• Hospital 30-Day, All-cause, Risk-Standardized Mortality Rate (RSMR) following chronic obstructive
pulmonary disease (COPD) hospitalization (NQF #1893)
• Hospital 30-Day, All-cause, Risk-Standardized Mortality Rate (RSMR) following coronary artery
bypass graft (CABG) surgery (NQF #2558)
Readmission Measures
• Hospital 30-Day All-cause, Risk-Standardized Readmission Rate (RSRR) following acute myocardial
infarction (AMI) hospitalization (NQF #0505)
• Hospital 30-Day, All-cause, Risk-Standardized Readmission Rate (RSRR) following heart failure
hospitalization (NQF #0330)
• Hospital 30-Day, All-cause Risk-Standardized Readmission Rate (RSRR) following pneumonia
hospitalization (NQF #0506)
• Hospital-level 30-Day, All-cause, Risk-Standardized Readmission Rate (RSRR) following elective
primary total hip arthroplasty (THA) and/or total knee arthroplasty (TKA) (NQF #1551)
• Hospital-wide All-cause Unplanned Readmission Measure (HWR) (NQF #1789)
• Hospital 30-Day, All-cause, Risk-Standardized Readmission Rate (RSRR) following chronic
obstructive pulmonary disease (COPD) hospitalization (NQF #1891)
• Hospital 30-Day, All-cause, Unplanned, Risk-Standardized Readmission Rate (RSRR) following
coronary artery bypass graft (CABG) surgery (NQF #2515)
Healthcare-Associated Infections (HAI) Measures
• National Healthcare Safety Network (NHSN) Central Line–Associated Bloodstream Infection
(CLABSI) outcome (NQF #0139)
• American College of Surgeons – Centers for Disease Control and Prevention (ACS-CDC) Harmonized
Procedure Specific Surgical Site Infection (SSI) Outcome Measure (NQF #0753) – SSI following
colon surgery
• National Healthcare Safety Network (NHSN) Catheter-Associated Urinary Tract Infection (CAUTI)
Outcome Measure (NQF #0138)
• National Healthcare Safety Network (NHSN) Facility-wide Inpatient Hospital-Onset Methicillin-
Resistant Staphylococcus aureus (MRSA) Bacteremia Outcome Measure (NQF #1716)
• National Healthcare Safety Network (NHSN) Facility-wide Inpatient Hospital-Onset Clostridioides
difficile Infection (CDI) Outcome Measure (NQF #1717)
• Influenza Vaccination Coverage Among Health Care Personnel (HCP) (NQF #0431)
Emergency Department (ED) Throughput Measures 
• Admit Decision Time to ED Departure Time for Admitted Patients (NQF #0497)
Information from: Centers for Medicare & Medicaid Services (CMS). CMS Measures Inventory Tool. Available at https://www.qualityreportingcenter.com/globalassets/
iqr_resources/iqr-resources-for-fy-2022-pymt-determination/cms_qualityprogram_measures_comparison_fy2022_hqr_vfinal508.pdf.

ACCP Updates in Therapeutics® 2022: Critical Care Pharmacy Preparatory Review and Recertification Course

922
Practice Administration and Development: Protocol Development and Quality Improvement

11. Institute for Safe Medication Practices (ISMP) – Established 1975


a. Nonprofit organization responsible for targeting medication error prevention and safe medication
use; a certified patient safety organization
b. Based on a nonpunitive approach and system-based solutions
c. Five key areas of focus: Knowledge, analysis, education, cooperation, and communication
d. Medication Errors Reporting Program – Practitioner self-reporting program
e. ISMP Targeted Medication Safety Best Practices for Hospitals (2020–2021)
i. Worksheet can be used to perform a gap analysis of an institution’s performance with medication
safety issues that continue to cause fatal and harmful errors to patients.
ii. The interactive worksheet can be found at https://www.ismp.org/resources/worksheet-ismp-
targeted-medication-safety-best-practices-hospitals.
12. The Choosing Wisely initiative of the American Board of Internal Medicine Foundation is a collaboration
of critical care societies including the American Association of Critical Care Nurses, American College
of Chest Physicians, American Thoracic Society, and Society of Critical Care Medicine.
a. Do not order diagnostic tests at regular intervals (e.g., every day), but in response to specific clinical
questions.
b. Do not transfuse red blood cells in hemodynamically stable, non-bleeding ICU patients with a
hemoglobin concentration greater than 7 g/dL.
c. Do not use parenteral nutrition in adequately nourished critically ill patients within the first 7 days
of an ICU stay.
d. Do not deeply sedate mechanically ventilated patients without a specific indication and without
daily attempts to lighten sedation.
e. Do not continue life support for patients at high risk of death or severely impaired functional
recovery without offering patients and their families the alternative of care focused entirely on
comfort.

D. The goal of the ASHP Practice Advancement Initiative (PAI), formerly known as the Pharmacy Practice
Model Initiative, is to significantly advance the health of patients by supporting futuristic practice models
that support the most effective use of pharmacists as direct patient care providers. The initiative aims to assist
leaders and practitioners in creating a framework, determining services, identifying emerging technology,
developing templates, and implementing change. The ASHP Pharmacy Practice Model Initiative started in
2010. See Figure 1 for the Practice Advancement Initiative 2030 recommendations.

E. Local Quality Initiatives (from ASHP: The ASHP Discussion Guide on the Pharmacist’s Role in Quality
Improvement)
1. Every accredited hospital must participate in QI initiatives.
2. The overall goals of the PI programs are to maintain and support the delivery of safe, quality care.
3. Each program should include the following:
a. Adherence to standards of care
b. Opportunities for improvement, with action plans to implement change strategies
c. Strategies for the effectiveness of change strategies
d. Involvement of multidisciplinary teams in process improvement
4. PI plans should:
a. Articulate the commitment to PI
b. Delineate the goals of the PI process
c. Specify the authority and responsibilities for PI
d. Describe the organizational structure and process related to the PI program
e. Describe the method for improving organizational performance
f. Describe the communication and recognition of PI activities

ACCP Updates in Therapeutics® 2022: Critical Care Pharmacy Preparatory Review and Recertification Course

923
The goal of PAI 2030 is to significantly advance the health and well‑being of people by supporting patient‑centered care
delivery models that optimize the most effective use of pharmacists as direct patient care providers.

PRACTICE-FOCUSED RECOMMENDATIONS

Pharmacy Technician Leadership in


Patient-Centered Pharmacist Role, Technology
Role, Education, & Medication Use &
Care Education, & Training & Data Science
Training Safety

A1. Pharmacists should collaborate B1. All pharmacists should C1. Pharmacists should use health D1. Pharmacy technicians E1. Pharmacists should
with patients, families, and have an individualized information technologies to should participate in advance the use of
caregivers to ensure that treatment continuing professional advance their role in patient care advanced roles in all pharmacogenomic
plans respect patients’ beliefs, development plan. and population health. practice settings to promote information for personalized
values, autonomy, and agency. efficiency and improve access medication treatment.
to patient care.
B2. Pharmacists should C2. Pharmacy practice leaders
A2. The pharmacy workforce should leverage and expand their should foster the development E2. Pharmacists should
lead medication reconciliation scope of practice, including and application of advanced D2. Pharmacy technicians assume leadership roles
processes during care transitions prescribing, to optimize analytics (e.g., machine learning should have complete in medication stewardship
(e.g., emergency department, patient care. and artificial intelligence) in responsibility for advanced activities at the local, state,
upon admission and discharge, activities such as risk assessment, technical and supporting and national levels.
ambulatory care setting, community monitoring performance metrics, activities (e.g., order
pharmacy, long term care). B3. Pharmacists should identifying patients in need of fulfillment, tech‑check‑tech,
participate in and assume key pharmacist care, optimizing regulatory compliance,
roles on emergency response medication use, and business supply chain management,
A3. The pharmacy workforce teams. management. diversion prevention, revenue

924
should collaborate with patients, cycle management, patient
caregivers, payers, and healthcare assistance programs).
professionals to establish consistent C3. Pharmacy practice
and sustainable models for leaders should be engaged in
seamless transitions of care. assessing emerging patient
care technologies (e.g., mobile
applications, monitoring devices,
A4. Pharmacist documentation digital wearables or ingestables,
related to patient care must be blockchain technology) to
available to all members of the support optimal medication‑use
healthcare team, including patients, outcomes.
in all care settings.

C4. The pharmacy workforce


A5. The pharmacy workforce should be competent in
should partner with patients and health information technology
the interprofessional care team to (including but not limited to
identify, assess, and resolve barriers analytics, automation, and clinical
to medication access, adherence, applications of technology) with
and health literacy. ongoing education and training
embedded at all stages of career
development.
Practice Administration and Development: Protocol Development and Quality Improvement

A6. Patients must have access to a


pharmacist in all settings of care.

ACCP Updates in Therapeutics® 2022: Critical Care Pharmacy Preparatory Review and Recertification Course
Figure 1. ASHP Practice Advancement Initiative 2030.
Used with permission from: American Society of Health-System Pharmacists (ASHP). PAI 2030 Recommendations. Available at https://www.ashp.org/Pharmacy-Practice/PAI/PAI-Recommendations. Accessed September 3, 2021.
ORGANIZATION-FOCUSED RECOMMENDATIONS

Pharmacist Role, Pharmacy Technician Leadership in


Patient-Centered Technology
Education, & Role, Education, & Medication Use &
Care & Data Science
Training Training Safety

A7. The pharmacy workforce, in all B4. Health systems C5. Virtual pharmacy D3. All newly hired E3. Pharmacy must be an active and accountable
care settings, must have access to should require services (e.g., technicians should have partner in the financial stewardship (e.g.,
complete patient medical records completion of ASHP‑ telepharmacy) should completed an ASHP/ACPE‑ minimizing waste, using cost‑effective therapies,
and related health information. accredited residency be deployed to accredited technician managing the supply chain) of care delivered in
training as a minimum optimize operational education and training all settings.
credential for new and clinical services program.
A8. The pharmacy enterprise pharmacist practitioners. that extend patient
should be integrated and modeled care services and E4. Pharmacy practice leaders should ensure
to provide patient‑centered care enhance continuity of D4. Health systems should evidence‑based medication use by continually
across the continuum (e.g., home B5. Pharmacists care. require technicians to be analyzing and reporting use patterns and
and outpatient infusion, specialty should participate in certified by the Pharmacy outcomes.
pharmacy, community pharmacy, organization‑based Technician Certification
acute care). credentialing and C6. The pharmacy Board.
privileging processes enterprise must have E5. Health systems should support
to ensure competency sufficient resources to interprofessional innovation centers designed to
A9. The pharmacy workforce within their scope of develop, implement, D5. Pharmacy departments pursue breakthroughs in areas such as patient
should lead medication education practice. and maintain should foster the experience, medication use, clinical outcomes,
for patients and caregivers that technology‑related development of professional operational efficiency, technology, and revenue
optimize outcomes, including in medication‑use safety career paths for pharmacy generation.
care transitions. B6. Pharmacy practice standards. technicians.
leaders should ensure
that their workforce has E6. Health systems should support the well‑being

925
A10. Pharmacists should play the necessary knowledge C7. Pharmacy and resiliency of their staffs.
an active role in ensuring that and competency to departments should
ethical principles drive clinical adapt to emerging have access to an
and business decisions related to E7. Pharmacy departments should strive to
healthcare needs. analytics resource,
medication use. achieve equity, diversity, and inclusion in all
such as a data scientist,
technical, clinical, and leadership roles.
to collect, aggregate,
B7. Pharmacists measure, visualize,
A11. Health systems must provide practicing in specialty and disseminate E8. The pharmacy enterprise should engage,
24/7 pharmacy services with areas should be board‑ data related to the employ, or develop expertise in areas such as
advanced clinical capability. certified through the financial and clinical finance, analytics, business management, quality
Board of Pharmacy performance of assurance, informatics, human resources, payer
Specialties or other pharmacists.
A12. Health systems should relations, and supply chain management.
appropriate body.
support innovative models for
providing a safe and appropriate
C8. Pharmacy E9. Health systems should have a pharmacist
level of pharmacy services for
departments should executive leader, with a reporting structure
small and rural hospitals or other
use technology consistent with other executive leaders, to
alternative practice settings.
to ensure the safe oversee and influence enterprise‑wide decision
compounding of sterile making related to medication use and technology.
A13. Pharmacy departments should products.
take responsibility for appropriate
Practice Administration and Development: Protocol Development and Quality Improvement

medication use in the structuring E10. The pharmacy workforce should assess and
of external partnerships. mitigate risk in medication‑use systems across all
settings.

ACCP Updates in Therapeutics® 2022: Critical Care Pharmacy Preparatory Review and Recertification Course
Figure 1. ASHP Practice Advancement Initiative 2030. (continued)
Used with permission from: American Society of Health-System Pharmacists (ASHP). PAI 2030 Recommendations. Available at https://www.ashp.org/Pharmacy-Practice/PAI/PAI-Recommendations. Accessed September 3, 2021.
PROFESSION-FOCUSED RECOMMENDATIONS

Pharmacist Role, Pharmacy Technician Leadership in


Patient-Centered Technology
Education, & Role, Education, & Medication Use &
Care & Data Science
Training Training Safety

A14. Pharmacists should B8. Pharmacy education, C9. Pharmacy should employ D6. A scope of practice E11. Pharmacists should lead the
lead and advocate for residency training, and high‑reliability principles when including core competencies development, implementation,
comprehensive medication continuing education should designing and selecting health should be developed and and evaluation of medication‑
management in all cover healthcare reimbursement, information technology. defined for pharmacy related national quality
healthcare settings. payment, and business technicians in acute‑care and indicators and accountability
management in all areas of ambulatory‑care settings. measures.
practice. C10. Pharmacy should advocate
for information technology that
is interoperable and transparent E12. Pharmacists should be
B9. Pharmacists in all care with respect to usability, leaders in federal and state
settings should be included security, and functionality legislative and regulatory
as integral members of the across the continuum of care. policy development related
healthcare team and share to improving individual and
accountability for patient population health outcomes.
outcomes and population health. C11. Pharmacy should establish
standards for the application
of artificial intelligence (AI) E13. Pharmacy should
B10. The pharmacy workforce in the various steps of the partner with interprofessional
should be knowledgeable and medication‑use process, organizations to define and
have the resources to care for including prescribing, reviewing delineate practice advances
patients with behavioral and medication orders, and into state and federal laws and
mental health disorders. assessing medication‑use regulations to optimize patient

926
patterns in populations. care.

B11. Training and credentialing


should exist to develop and E14. Pharmacy should leverage
recognize pharmacists who healthcare models that
specialize in health information acknowledge pharmacist value
technology. and align payment with quality
of outcomes.

B12. Credentialed ambulatory‑


care pharmacists should be
considered primary care
providers.

B13. The profession should


champion multi‑state or national
licensure for pharmacists.

B14. Pharmacists, in collaboration


with other key stakeholders,
Practice Administration and Development: Protocol Development and Quality Improvement

must work to increase


public, regulatory, and health
professional understanding of
pharmacists’ roles and value.

ACCP Updates in Therapeutics® 2022: Critical Care Pharmacy Preparatory Review and Recertification Course
Figure 1. ASHP Practice Advancement Initiative 2030. (continued)
Used with permission from: American Society of Health-System Pharmacists (ASHP). PAI 2030 Recommendations. Available at https://www.ashp.org/Pharmacy-Practice/PAI/PAI-Recommendations. Accessed September 3, 2021.
Practice Administration and Development: Protocol Development and Quality Improvement

F. QI Tools
1. Six Sigma – DMAIC process: Goal to reduce variation around process defects and test with statistical
methods
a. DEFINE the problem
b. MEASURE current state through process mapping, or Failure Modes and Effects Analysis (FMEA)
c. ANALYZE relationship between input/output
d. IMPROVE future state processes
e. CONTROL to sustain the gain
2. Failure Modes and Effects Analysis (FMEA): Proactive evaluation to determine possible failures in
processes in order to prevent them from occurring on initiation
a. Evaluate the steps in the process
b. What could go wrong? (Failure modes)
c. Why would the failure happen? (Failure causes)
d. What would be the consequences of each failure? (Failure effects)
3. LEAN principles: How do we do more with less? Ideally, create a high-quality service/product with less
resources or a shorter lead time
a. Value - Activity that enhances service from the customer’s viewpoint
i. Waste – opposite of value with goal to eliminate waste
ii. Types of waste: Defects, Overproduction, Waiting, Not using employees’ creativity, Transport,
Inventory, Motion, Extra Processing (“DOWNTIME”)
b. Value stream – all actions required to bring service to the customer (both value and non-value
added)
i. Value stream mapping is a LEAN tool that can be used to “walk the line” by experiencing the
process and defining the current state.
c. Flow – flow of processes without interruption
i. Process mapping to outline the workflow
d. Pull – trigger flow from demand
e. Perfection – continuous improvement
4. Plan-do-study-act (PDSA): Model for promoting quality improvement through real-time evaluation of
standard work
a. What are we trying to accomplish? – An aim or project goal must be developed (example of specific
aim: time to appropriate antibiotic therapy in patients seen in the ED with presumed sepsis).
i. Describe the process to be improved.
ii. Set a target for improvement that extends beyond current performance.
iii. Secure necessary resources in the process.
b. How will we know that a change is an improvement?
i. Establish baseline measurement – Select and gather data.
ii. Compare baseline data with the target and identify key causes and sources of variation.
c. What changes can we make that will result in improvement?
i. Broad, general ideas and thoughts about change (“change concepts”)
ii. Select the change.
d. Testing change
i. Scientific method
e. Plan-do-study-act cycle:
i. Plan – to carry out the test or observation
(a) Questions are asked and predictions are made.
(b) Details are described.
(c) Who will make the test?

ACCP Updates in Therapeutics® 2022: Critical Care Pharmacy Preparatory Review and Recertification Course

927
Practice Administration and Development: Protocol Development and Quality Improvement

(d) What exactly will they do?


(e) When will they do it?
(f) Where will they do it?
(g) How long will they do it?
(h) Predict what will happen.
(i) Determine which data to collect.
ii. Do – try it out
(a) Change is tested following the “plan.”
(b) Collect data on the change.
(c) Unexpected problems and observations are documented.
iii. Study – analyze data, study results
(a) Study the effect of the test change on the single measure.
(b) Compare data with predictions.
(c) Summarize what was learned.
iv. Act – refine the change on the basis of what was learned
(a) Adapt, adopt, or abandon change idea for next cycle.
(b) Develop an implementation plan.
(c) Share final reflections.

G. Data Generation
1. Benchmarking – Compares your performance with that of another health care system
2. Provider concerns
3. Nursing reports/concerns
4. Patient concerns or comments: Reports from third-party payers and regulatory agencies
5. Incident reporting
6. Root-cause analysis reports
7. Medication error reports
8. Patient care evaluation studies
9. Patient satisfaction survey results

H. Analysis Tool Kit


1. Affinity diagrams
2. Cause-and-effect diagrams
3. Decision matrixes
4. Root-cause analysis
5. Apparent cause analysis
6. Process maps
7. Run chart, control chart, or flowcharts
8. Force-field analysis
9. Histograms, scatterplots
10. Relationship diagrams
11. Supplier, input, process, output, customer
12. Voice of the customer
13. Five Whys
14. Fishbone diagram
15. FMEA

ACCP Updates in Therapeutics® 2022: Critical Care Pharmacy Preparatory Review and Recertification Course

928
Practice Administration and Development: Protocol Development and Quality Improvement

I. Publishing QI Results
1. Important to publish QI data to promote improvement efforts in health care
2. Reduces the chances of the same mistakes being repeated
3. Shares your work by spreading important and useful information
4. Standards for Quality Improvement Reporting Excellence (SQUIRE) – Provides a framework for
reporting new knowledge about how to improve health care. Available at www.squire-statement.org/

Patient Cases

2. As a critical care pharmacist, you are involved in a QI program. Which TJC measure is best known to be
affected by the critical care pharmacist?
A. ED median time from ED arrival to ED departure for admitted ED patients.
B. Procoagulant reversal agent initiation for intracerebral hemorrhage (ICH).
C. Hemorrhagic transformation for patients receiving intravenous alteplase therapy or mechanical endo-
vascular reperfusion therapy.
D. Severe sepsis and septic shock management bundle.

3. As part of a QI initiative, you identify an increase in the monthly rate of catheter-related bloodstream
infections in your ICU. You decide to evaluate the current practice and use LEAN to determine where the
practice issue is. Which is the best example of a component of LEAN?
A. Define the problem.
B. Ask your nurses why they think a spike in catheter-related bloodstream infections has occurred.
C. Identify transportation barriers.
D. Describe the past process.

IV.  MEDICATION USE EVALUATION

A. TJC requires drug use evaluations (DUEs) to be completed to monitor the safety of medications. ASHP
published guidelines on performing a MUE in 2020.

B. ASHP Guidelines on MUEs: Include a performance improvement framework to improve safety, efficacy,
quality, and efficiency in patient care.
1. 1. An example of a performance improvement framework is FOCUS-PDCA:
a. Find the process to be targeted for improvement.
b. Organize the team that knows the process.
c. Clarify current knowledge of the process.
d. Understand causes of process variation.
i. Five Whys
e. Select process improvement.
i. FACES (feasibility, acceptability, cost/benefit, effectiveness, sustainability) tool
f. Plan: Develop a solution.
g. Do: Implement improvements.
h. Check: Evaluate the results.
i. Act: Determine the changes needed moving forward and implement them.

ACCP Updates in Therapeutics® 2022: Critical Care Pharmacy Preparatory Review and Recertification Course

929
Practice Administration and Development: Protocol Development and Quality Improvement

C. MUEs and DUEs are often used synonymously. Both DUEs and MUEs are PI and QA methods that ensure
optimal medication therapy management and improve patient safety and outcomes.

D. A DUE is drug- or disease-specific, focusing specifically on the criteria-based assessment of the medication
use process and prescribing, whereas an MUE provides a broader scope that focuses on a drug or class of
drugs, the process(es), and the outcome, with a specific emphasis on improving patient outcomes. MUEs
focus on several elements of the medication process/use such as prescribing, pharmacist medication order
validating or verifying, dispensing, preparing, administering, monitoring, patient education, and outcomes.

E. The sample size of the MUE depends on the type of medication data being analyzed—usually, a sample of
20–30 patients is sufficient; however, a sample of 100 or more patients may be required to analyze patient
outcomes.

F. Performance improvement framework, such as the FOCUS-PDSA model, can be used to outline the MUE
process.

G. Data collection for specific criteria can use either a yes/no format (with a section for comments) or open-
ended questions. Using CPOE and electronic medical records, the processes of data retrieval, monitoring,
and generating specialized reports have become easier.

H. The type and number of MUEs should be determined by the risk mitigated when using a medication. Broad
categories for MUEs include:
1. Promote optimal medication therapy
2. Improve patient safety
3. Standardize to reduce unnecessary variation
4. Optimize drug therapy
5. Assess value of innovative practices
6. Meet quality or regulatory standards
7. Minimize costs
a. These should be considered together with the impact of the medication or medication use process on
patients and the medication use system as a whole to prioritize the highest-impact MUEs.

I. Medications selected for an MUE may be based on the following:


1. High risk
2. High volume
3. ADEs
4. Preventable ADEs
5. Near-miss and harmful medication errors
6. Formulary review, including addition (non-formulary use), retention, or deletion
7. Pharmacy intervention data
8. Treatment failures
9. Physician or nurse identification or request
10. Patient concerns
11. Off-label use
12. High cost
13. Beneficial when used a specific way
14. Drug shortages, recalls, or safety alerts

ACCP Updates in Therapeutics® 2022: Critical Care Pharmacy Preparatory Review and Recertification Course

930
Practice Administration and Development: Protocol Development and Quality Improvement

J. Examples of medications and medication use processes in critically ill patients that may be selected for an
MUE can be found in Box 1 and Box 2.

Box 1. Examples of Medications Suited for a MUE in Critically Ill Patients

Alteplase, tenecteplase IV acetaminophen


Andexanet alfa IV ketorolac
Angiotensin II IV pantoprazole, esomeprazole
Argatroban, bivalirudin IV valproic acid
Cangrelor IV verapamil
Ceftazidime/avibactam, ceftolozane/tazobactam, Ketamine
meropenem/vaborbactam, and imipenem/ Linezolid
relebactam Liposomal bupivacaine
Conivaptan and tolvaptan Meropenem, imipenem, ertapenem
Daptomycin Midazolam, lorazepam, diazepam
Darbepoetin alfa, erythropoietin Naloxone
Desmopressin Neostigmine
Dexmedetomidine Norepinephrine, epinephrine, dopamine,
Digoxin dobutamine, phenylephrine, vasopressin
Direct oral anticoagulants Octreotide
Enoxaparin, dalteparin, fondaparinux Phenytoin, fosphenytoin
Fidaxomicin Polymyxin, colistin
Haloperidol Propofol
Heparin Recombinant factors (rfVIIa, AT-III, 4F-PCC)
Hydromorphone, fentanyl, methadone, or other Sugammadex
opioids Tirofiban
Idarucizumab Tranexamic acid and aminocaproic acid
Inhaled epoprostenol and nitric oxide Vasopressin
Insulin Vecuronium, cisatracurium, rocuronium
Isoproterenol
AT-III = antithrombin III; 4F-PCC = four-factor prothrombin complex concentrate; IV = intravenous; MUE = medication use evaluation; rfVIIa = recombinant activated
human factor II.

ACCP Updates in Therapeutics® 2022: Critical Care Pharmacy Preparatory Review and Recertification Course

931
Practice Administration and Development: Protocol Development and Quality Improvement

Box 2. Examples of High-Risk Medication-Related Processes Suited for an MUE in Critically Ill Patients

PADIS (ABCDEF bundle) protocols


Use of neuromuscular blocking agents
Alcohol withdrawal management
Management of supratherapeutic anticoagulation (e.g., warfarin, direct oral anticoagulants, heparin, argatroban)
Use of β-blockers in myocardial infarction
Monitoring for dysrhythmias with QTc-prolonging drugs
Antihypertensive IV-to-PO switch therapy
Antihypertensive use in acute stroke
Fluid resuscitation
Management of gastrointestinal bleeding
Use of total parenteral nutrition
Use of albumin
Hyperkalemia management guidelines
Hypomagnesemia management guidelines
Management of status epilepticus
Management of hyponatremia
Use of IV sodium bicarbonate
Vancomycin dosing and ordering serum concentrations
Aminoglycoside dosing and ordering serum concentrations
Intermittent infusions of antimicrobials (e.g., carbapenems, piperacillin/tazobactam)
Evaluation of antimicrobial dosing in CRRT or hemodialysis
Surgical prophylaxis guidelines
Antimicrobial IV-to-PO switch therapy
Management of Clostridioides difficile diarrhea
Vaccine administration
IV push medication guidelines (rate of administration and medication preparation - diluted or undiluted)
Stress ulcer prophylaxis
Insulin infusions
Hypoglycemic protocols
CRRT = continuous renal replacement therapy; PO = oral(ly); QTc = corrected QT (interval).

K. Ideally, MUEs should be performed proactively and used to identify any gaps in practice or patient safety.
1. An interventional MUE is completed concurrently or prospectively, and if the criteria are not met, an
intervention by the pharmacist can and should be made to improve the use of the medication and patient
outcomes.
2. A noninterventional MUE is completed concurrently or retrospectively by a medical record review, and
data are collected but criteria are not met. The review is retrospective, and there is no pharmacist-to-
prescriber interaction during the review process.

L. MUEs should have specific criteria set – These criteria are best determined by a multidisciplinary team of
medication or disease experts.

M. MUE criteria may be approved by the P&T committee. The P&T committee may create an MUE
subcommittee. Other hospital committees such as the QI committee may also request that an MUE be
performed. Other names for the MUE committees may include formulary, drug safety, therapeutic
assessment, medication safety, and drug use review committees.

ACCP Updates in Therapeutics® 2022: Critical Care Pharmacy Preparatory Review and Recertification Course

932
Practice Administration and Development: Protocol Development and Quality Improvement

N. The MUE subcommittee should be multidisciplinary and may be composed of the following:
1. Clinical pharmacists
2. Physicians, nurse practitioners, and physician assistants
3. Nurses
4. Administrators
5. PI/QA representatives
6. Risk management representatives
7. Respiratory therapists, nutritionists, and other ancillary support staff

O. Given the pharmacist’s expertise in medication management, pharmacists often chair or co-chair the MUE
subcommittee and perform the MUE.

P. The MUE subcommittee can recommend drugs and drug processes that require an MUE to the P&T
committee; alternatively, the P&T committee can request MUEs from the MUE subcommittee.

Q. The MUE process includes reviewing the findings and developing plans of improvement. The results and
conclusions of the MUE should be reported to the P&T committee and department chairs.

R. After plans of for improvement are identified and actions taken, a follow-up MUE should be completed to
document that improvement has occurred.

S. Periodic MUEs in the same area should be performed every 3–6 months for 1 year to ensure that the
improvements made remain effective and are sustained. If any new changes have occurred in the medication
use process, the MUE criteria should be reassessed and the new criteria incorporated.

T. Policies and educational initiatives should be developed to improve opportunities identified from the MUE,
such as:
1. In-service lectures
2. Newsletter publications
3. Medication alerts
4. Guideline development
5. Protocols
6. Policy and procedures
7. CPOE pathways, prescribing guides, or information or pop-up warnings

Patient Case

4. Which medication use process is best suited for an MUE?


A. Review of pharmacist notes in the electronic health record (EHR).
B. Review of accuracy of expiration dates placed on intravenous admixture products.
C. Review of vancomycin dosing and ordering of vancomycin blood concentrations.
D. Review of frequency of drug interaction warnings on the CPOE system.

ACCP Updates in Therapeutics® 2022: Critical Care Pharmacy Preparatory Review and Recertification Course

933
Practice Administration and Development: Protocol Development and Quality Improvement

V.  EDUCATION

A. Educate health care professionals and other stakeholders concerning issues related to the care of critically
ill patients.
1. Provides informal instruction to pharmacists and other ICU health care professionals
2. Participates in the training of pharmacy and medical students, residents, and fellows through experiential
critical care rotations
3. Provides critical care pharmacology and therapeutic didactic lectures to health care professionals,
including students, residents, and fellows
4. Implements pharmacist and pharmacy technician training programs for ICU personnel
5. Provides accredited continuing education sessions
6. Develops educational materials such as pocket guides or tip sheets related to the care of critically ill
patients
7. Educates lay group community medical personal about the role of the ICU pharmacist
8. Coordinates or directs internships; experiential training; traineeships, residency, or fellowship programs
9. Teaches advanced cardiac life support

B. Communication Strategies
1. 7% of the things you say are the words themselves, 38% are tone of voice and inflection, and 55% are
body language.
2. Evaluate the audience.
a. Patient/general public
b. Health care providers
c. Learners (e.g., students, residents)
d. Legislators
e. The media
3. Communicate (see Table 2).

Table 2. Communication Strategies


Strategy Questions to ask regarding the communication
Credibility Is your messenger credible? Is the messenger a trusted and respected source
of information with your audience?
Context Is your message in context with reality and the environment in which your
audience is located?
Content Is your message relevant to your audience? Is the audience interested in the
information?
Clarity Is your message straightforward? How far will it travel and how long will it
last? Do not use abbreviations
Continuity and consistency Repeat your message for audience penetration
Channels What channels/tools of communication are you using? What value are they
bringing to your audience?
Customer benefits What is in it for me?
Caring, compassion, and concern Does your audience know that you care?
Capability of audience Is your audience capable of understanding the message? Will the audience
take the time to read, watch, or listen to it?
Call to action What is your audience supposed to do now?

ACCP Updates in Therapeutics® 2022: Critical Care Pharmacy Preparatory Review and Recertification Course

934
Practice Administration and Development: Protocol Development and Quality Improvement

4. Sharing the message/tools to communicate


a. Pharmacy departmental newsletter
b. Hospital newsletter
c. Electronic screensavers providing information
d. Best practice advisory methods
e. Local/regional communication newsletter/e-mail list
f. Electronic message
g. Periodically repeat educational content over time.

VI.  VI. DOCUMENTATION PROCESSES USED FOR CRITICAL CARE PHARMACY SERVICES

A. Importance of Documentation
1. Evidence of economic benefit and improvement in patient safety for clinical pharmacy services (CPS)
and critical care pharmacy services is well established. The Society of Critical Care Medicine supports
an ICU pharmacist as an essential component of the ICU team.
2. Clinical pharmacy interventions that affect patient care should be documented in the electronic
medical record. Pharmacist involvement directly decreases drug-related costs, prevents adverse effects,
improves quality and efficacy of care, reduces mortality, shortens length of stay (LOS), and lowers
overall patient care costs.
3. Clinical pharmacy interventions of the critical care pharmacist include:
a. Reduced ADEs in the critically ill population
b. Reduced order-prescribing errors
c. Optimization of the correct drug for the correct disease process
d. Safe and effective use of medication
e. Decreased medication use
f. Decreased LOS
g. Reduced ICU admissions
h. Reduced medication costs and overall lower cost of care
i. Reduced medication administration errors
j. Reduced inappropriate use of antibiotics
k. Protocol development
l. Guideline development
m. Performance of MUEs
4. Documenting CPS is important not only to quantify CPS and workload, but also to provide justification
for maintaining and expanding services.
5. CPS should be evaluated for cost impact and cost outcomes savings and should include the following:
a. Weighted metric for each variable to quantify measured activities
b. Pharmacotherapy improvement
c. Cost savings
d. Antibiotic stewardship
e. Provider education
f. Quality/safety improvement (value-based purchasing, HCAHPS)
g. Participation in emergency response (cardiac arrest, stroke, sepsis)
h. Prospective chart review
i. Rounding with health care providers
j. Formal pharmacy consults

ACCP Updates in Therapeutics® 2022: Critical Care Pharmacy Preparatory Review and Recertification Course

935
Practice Administration and Development: Protocol Development and Quality Improvement

6. Collaborative drug therapy management (CDTM) or collaborative practice agreements (CPAs) – When
a prescriber and a pharmacist establish written guidelines or protocols authorizing the pharmacist to
initiate, modify, or continue drug therapy for a specific patient. CDTM is a type of pharmacotherapeutic
intervention that should be documented and reported. Examples relevant to the practice of critical care
pharmacy include parenteral nutrition prescribing, antimicrobial stewardship programs, and analgesia/
sedation/delirium screening and management.

B. Documentation Process
1. The pharmacy department should have a policy and procedure for documenting clinical pharmacy
interventions. In addition, a dashboard should be developed that incorporates these clinical interventions
(Appendix 2).
2. In general, pharmacist interventions are the result of either formal or informal consultations or
unsolicited interventions.
3. Documenting the pharmacotherapeutic intervention in the EHR using the electronic medical record
or using handwriting on paper in the medical chart provides transparency between all health care
professionals – physicians, nurses, pharmacists, dietitians, respiratory therapists, and social workers.
4. Pharmacists can also document interventions in the pharmacy profile; however, this method generally
allows for review only by pharmacy personnel who have access to the pharmacy computer system such
as other pharmacists and pharmacy interns, students, and technicians.
5. Both methods should be available for documenting pharmacotherapeutic interventions, and criteria
should be established to determine which method is most appropriate.
6. Pharmacists should have the authority to document pharmacotherapeutic interventions in the EHR.
7. Pharmacists should be trained and educated to document in the EHR. The ASHP Clinical Skills
Certificate program is a tool that can be used to train pharmacists to document in the EHR.
8. Documentation methods may include standard format documentation methods such as:
a. SOAP (subjective, objective, assessment, plan)
b. TITRS (title, introduction, text, recommendation, signature)
c. FARM (findings, assessment, resolution, monitoring)
9. Wording for solicited consultations should be direct. Unsolicited pharmacist interventions should be
documented subtly, allowing the primary provider to decline the recommendation without incurring
liability. Phrases that can be used include:
a. “May consider”
b. “Suggest”
c. “May recommend”
10. When feasible, written notes by pharmacists should be documented in the EHR after an oral
communication with the clinician; this allows for any patient data discrepancies to be corrected and
for agreement and confirmation between the prescriber and the pharmacist to execute the intervention.
11. Pharmacists should follow up on their patient interventions daily and provide follow-up notes that
include patient progress or new interventions, when needed.
a. Pharmacists should provide their contact information.
b. Co-signatures should be required for pharmacy residents (until deemed competent according to
pharmacy department standards), interns, and students.
c. Continual physical presence during and after direct patient care rounds should be provided to
support the physicians, advanced practice providers, and nursing team with triaging medication-
related questions.

ACCP Updates in Therapeutics® 2022: Critical Care Pharmacy Preparatory Review and Recertification Course

936
Practice Administration and Development: Protocol Development and Quality Improvement

12. Many web-based or handheld electronic systems are available that can be used to document and report
pharmacotherapeutic interventions and cost savings. Ideally, these are documented in the patient’s EHR
for the reporting of weighted metrics.

C. Evaluation of CPS
1. Continuous QI should include quality indicators and periodic reviews of pharmacist-written
documentation and consultations.
2. These reporting systems allow for collecting, aggregating, and benchmarking data against data from
other hospitals and bed size. They increase the credibility of data collection methods and results when
evaluated by health care administrators (Appendix 2).
3. To document raw drug cost savings from changing to less expensive medications, the following method
may be applied: subtract the cost of the originally prescribed drug therapy (drug daily cost multiplied
by the number of days prescribed) from the cost of the less expensive drug therapy (drug daily cost
multiplied by the number of days prescribed). Using this method, the cost of intravenous diluents and
admixture fluids and syringes used in the preparation process may be included. Medication costs can
also be affected by factors such as shortages and FDA granting using a New Drug Application.
4. Value of pharmacist interventions should be determined as well. Value may be assigned utilizing cost-
avoidance associated with types of pharmacist interventions or with the Overhage and Lukes scale.
5. Documentation of interventions for reporting to other hospital committees such as the P&T committee
should include the following:
a. Date, time
b. Type of intervention
c. Drug(s) involved
d. Prescriber name, service, and type of health care provider
e. Duration of time spent completing the intervention
f. Whether the intervention was accepted or denied, or clarification was achieved
g. Value of the intervention (e.g. cost avoidance, Overhage & Lukes scale)
6. Documentation of services should show diversity, effectiveness, cost, and outcomes of activities.
a. Policy development
b. Research
c. Resource use
d. Management
e. Leadership
f. Education
g. Health care professionals (e.g., physicians, nurses, respiratory therapists)
i. Pharmacy students
ii. Pharmacy residents (PGY1, PGY2) and pharmacy fellows
iii. Pharmacy personnel
7. Outcomes of documentation
a. Establish additional clinical services.
b. Expand roles of existing services.
c. Assess new processes or practices (prescriber privileges or provider reimbursement).
d. Provide data for QA or research initiatives.
e. Accreditation purposes
f. Promotional reasons
g. Financial impact can be analyzed and used to justify time spent in that area.
h. Generate a business plan to expand clinical services:

ACCP Updates in Therapeutics® 2022: Critical Care Pharmacy Preparatory Review and Recertification Course

937
Practice Administration and Development: Protocol Development and Quality Improvement

i. Background and description


ii. Market and competitor analysis
iii. Operational structure and processes
iv. Financial projections
v. Milestones, schedules, and action plan
vi. References
vii. Supportive documents
viii. Financial pro forma statements
ix. Letters of support
8. Reports
a. Statistical interpretations of services
b. Satisfaction surveys of their services
c. Risk reduction
d. Peer review
e. Publication
f. Return on investment data
g. Future initiatives
9. Pharmacotherapeutic intervention data should be presented and emphasized at as many committee
opportunities as possible – this increases the visibility and corroborates the importance of the pharmacy
department and the critical care pharmacy services. Box 3 lists examples of pharmacotherapeutic
interventions that can be reported. Pharmacotherapeutic intervention data should be presented to the
following:
a. ICU committee
b. P&T committee
c. QA committee
d. Use review committee
e. Medical executive committee – Engages with potential expansion of pharmacy services or cost
avoidance
10. Pharmacotherapeutic intervention data may be presented quarterly (Figure 2). These data should
include the type of intervention (Figure 3) and both the raw drug cost savings derived from changing to
less expensive drugs and the outcome cost savings such as preventing an adverse event and decreasing
the LOS (Figure 4).

ACCP Updates in Therapeutics® 2022: Critical Care Pharmacy Preparatory Review and Recertification Course

938
Practice Administration and Development: Protocol Development and Quality Improvement

Box 3. Examples of Pharmacotherapeutic Interventions

1.  Discontinue drug 22.  Medication reconciliation


2.  Switch drug 23. Transfer medication reconciliation
3.  Switch drug for less expensive but equally safe 24. Discharge medication reconciliation
and effective drug 25.  Discharge counseling
4.  Drug dosing 26.  Written patient education provided
5.  Drug dosage form 27.  Patient adherence to medications
6.  Drug route 28.  Adverse drug reaction
7.  Pharmacokinetic consultation 29. Overdosage
8.  Pharmacotherapy consultation 30.  Subtherapeutic dose
9.  IV-to-PO switch therapy 31.  Medication error
10. Contraindication 32.  Monitoring for efficacy
11.  Duplicate therapy 33.  Monitoring for toxicity
12.  Nonformulary switch to formulary 34.  Ordering laboratory tests for monitoring drug
13.  Nonformulary approval efficacy and toxicity
14.  Drug unavailable 35.  Clarification of medication order
15.  IV drug incompatibility 36.  Untreated indication
16.  Drug-drug interaction 37.  Failure to receive medication
17.  Drug-food interaction 38.  Immunization recommendation
18.  Drug-laboratory test interaction 39.  Immunization administered
19.  Pharmacogenomics interaction 40.  Health risk assessment
20.  Therapeutic interchange 41.  Rapid response interventions
21. Allergy prevention 42.  Code interventions
43.  CDTM interventions
CDTM = collaborative drug therapy management.

Clinical Interventions
8000 7298
6000 5748 5015
4000 3451
2000
0
1st Quarter 2nd Quarter 3rd Quarter 4th Quarter
Clinical Interventions
Figure 2. Clinical interventions.

ACCP Updates in Therapeutics® 2022: Critical Care Pharmacy Preparatory Review and Recertification Course

939
Practice Administration and Development: Protocol Development and Quality Improvement

Intervention Type
Consult Pharmacy Dosing
Order Clarification
Allergy
Drug Interaction
Non-Formulary
Duplicate Therapy
IV to PO
Emergency Response
Medication Reconcilliation
Drug Monitoring
Dose Check
Discontinue/Switch Drug
Other
0 2000 4000 6000 8000 10000 12000 14000 16000

Intervention Type
Figure 3. Pharmacotherapy intervention types.

Quarterly Cost Savings


$800,000 $681,978 $652,145 $701,542
$600,000 $477,867
$400,000
$200,000
$0
1st Quarter 2nd Quarter 3rd Quarter 4th Quarter
Cost Savings
Figure 4. Raw and outcome drug cost savings

11. Benchmarking is important in annualizing the total number of pharmacotherapeutic interventions with
previous years (Figure 5) and the cost savings with previous years (Figure 6) and in providing rational
explanations for any discrepancies noted.

12. Pharmacotherapeutic interventions that are reported may be prioritized according to their clinical impact on
patient safety and cost savings. Examples of high-priority pharmacotherapeutic interventions that should be
reported include the following:
1. Allergy prevented
2. Contraindication prevented
3. Drug dosing adjustments
4. Duplicate therapy avoided
5. Drug interactions avoided
6. Medication reconciliation intervention
7. Ordering laboratory tests for monitoring of drug safety and efficacy
8. Switching drug for less expensive but equally safe and effective drug

ACCP Updates in Therapeutics® 2022: Critical Care Pharmacy Preparatory Review and Recertification Course

940
Practice Administration and Development: Protocol Development and Quality Improvement

Total No. of Clinical Interventions


50000
40000 35459 38496
30000 24500
20945 23452
20000
10000
$0
2017 2018 2019 2020 2021
Clinical Interventions

Figure 5. Annual pharmacotherapy interventions 2017–2021.

Total Annual Cost Savings


$3,000,000
$2,516,542
$2,000,000 $1,954,654

$1,000,000 $846,245 $950,471 $904,254

$0
2017 2018 2019 2020 2021
Cost Savings

Figure 6. Annual raw drug and outcome cost savings 2017–2021.

ACCP Updates in Therapeutics® 2022: Critical Care Pharmacy Preparatory Review and Recertification Course

941
Practice Administration and Development: Protocol Development and Quality Improvement

REFERENCES

Policy and Procedure 5. Golden SH, Hager D, Gould LJ, et al. A gap analysis
1. American Society of Health-System Pharmacists needs assessment tool to drive a care delivery and
(ASHP). ASHP Guidelines on Medication Use research agenda for integration of care and sharing
Evaluation. Available at https://www.ashp.org/-/ of best practices across a health system. Jt Comm J
media/assets/pharmacy-informaticist/docs/sopit- Qual Patient Saf 2017;43:18-28.
formulary-guideline-medication-use-evalutation.
ashx?la=en. Accessed September 3, 2021. Quality Improvement
2. Ciccarello C, Leber MB, Leonard MC, et al. ASHP 1. American Society of Health-System Pharmacists
guidelines on the pharmacy and therapeutics (ASHP). The ASHP Discussion Guide on the
committee and the formulary system. Am J Health Pharmacist’s Role in Quality Improvement.
Syst Pharm 2021 Apr 13:zxab080. Available at ashp.org/-/media/assets/pharmacy-
3. American Society of Health-System Pharmacists practice/resource-centers/leadership/
(ASHP). Guidelines on the Pharmacy and leadership-of-profession-phar macists-role-
Therapeutics Committee and the Formulary quality-improvement-guide Accessed September
System. Am J Health Syst Pharm 2021;78:907-18. 3, 2021
Available at https://www.ashp.org/-/media/assets/ 2. American Society of Health-System Pharmacists
policy-guidelines/docs/guidelines/gdl-pharmacy- (ASHP) Practice Advancement Initiative
therapeutics-committee-formulary-system.ashx. (PAI). 2030 Available at https://www.ashp.
Accessed September 3, 2021. org/Pharmacy-Practice/PAI/About-PAI-2030.
4. Grol R, Grimshaw J. From best evidence to best Accessed September 3, 2021..
practice: effective implementation of change in 3. American Society of Health-System Pharmacists
patients’ care. Lancet 2003;362:1225-30. (ASHP). The consensus of the Pharmacy Practice
5. Sackett DL, Rosenberg WMC, Gray JAM, et al. Model Summit. Am J Health Syst Pharm
Evidence-based medicine: what it is and what it 2011;68:1148-52.
isn’t. BMJ 1996;312:71. 4. American Society of Health-System Pharmacists
(ASHP). Applying LEAN to the Medication
Gap Analysis Use Process. Available at https://www.
1. Kane-Gill SL, Dasta JF, Buckley MS, et al. Clinical ashp.org/-/media/assets/phar macy-practice/
practice guideline: safe medication use in the ICU. r e s o u r c e - c e n t e r s /q u a l i t y - i m p r o v e m e n t /
Crit Care Med 2017;45:e877-e915. learn-about-quality-improvement-applying-lean-
2. Devlin JW, Skrobik Y, Gelinas C, et al. Clinical medication-use.ashx. Accessed September 3, 2021.
practice guidelines for the prevention and 5. Herring SR, Jones BR, Bailey BP. Idea generation
management of pain, agitation/sedation, delirium, techniques among creative professionals. In:
immobility, and sleep disruption in adult patients in Proceedings of the 42nd Hawaii International
the ICU. Crit Care Med 2018;46:e825-73. Conference on System Sciences – 2009. Available
3. Centers for Disease Control and Prevention at http://orchid.cs.illinois.edu/publications/HICSS-
(CDC). The Core Elements of Hospital idea-generation-2009.pdf. Accessed September 3,
Antibiotic Stewardship Programs 2019: Antibiotic 2021.
Stewardship Program Assessment Tool. Available 6. Joint Commission: Performance Measurement.
at https://www.cdc.gov/antibiotic-use/healthcare/ Available at https://manual.jointcommission.org/
pdfs/hospital-core-elements-H.pdf. Accessed pub/Manual/ReleaseNotesArchive/TJC_v2021B.
September 3, 2021. pdf. Accessed September 3, 2021.
4. ASHP Foundation. Pharmacy Forecast 2021: 7. U.S. Department of Health and Human
Strategic Planning Advice for Pharmacy Resources. Quality Improvement. Available at
Departments in Hospitals and Health Systems. https://www.hrsa.gov/sites/default/files/quality/
Available at Am J Health Syst Pharm 2021;78:472-97.

ACCP Updates in Therapeutics® 2022: Critical Care Pharmacy Preparatory Review and Recertification Course

942
Practice Administration and Development: Protocol Development and Quality Improvement

t oolbox /508p d fs /qu al it y i mprove me nt.p d f. 5. Horn E, Jacobi J. The critical care clinical pharma-
Accessed September 3, 2021. cist: evolution of an essential team member. Crit
8. Choosing Wisely. Five Things Physicians Care Med 2006;34:S46-51.
and Patients Should Question. Available at 6. Kane SL, Weber RJ, Dasta JF. The impact of
w w w.cho osi ng w isely.org /cl i n icia n-l ist s /# critical care pharmacists on enhancing patient out-
keyword=critical_care. Accessed September 3, comes. Intensive Care Med 2003;29:691-8.
2021. 7. Lat I, Paciullo C, Daley MJ, et al. Position paper on
9. Varkey P, Reller MK, Resar RK. Basics of quality critical care pharmacy services: 2020 update. Crit
improvement in health care. Mayo Clin Proc Care Med 2020;48:e813-e834.
2007;82:735-9. 8. Leape LL, Cullen DJ, Clapp MD, et al. Pharmacist
10. Institute for Healthcare Improvement (IHI). participation on physician rounds and adverse
Failure Modes and Effects Analysis (FMEA) drug events in the intensive care unit. JAMA
Tool. Available at www.ihi.org/resources/Pages/ 1999;282:267-70.
Tools/FailureModesandEffectsAnalysisTool.aspx. 9. Montazeri M, Cook DJ. Impact of a clinical phar-
Accessed September 8, 2021. macist in a multidisciplinary intensive care unit.
11. ISMP Targeted Medication Safety Best Practices Crit Care Med 1994;22:1044-8.
for Hospitals (2020-2021). Available at https://www. 10. Ovehage JM, Lukes A. Practical, reliable, compre-
ismp.org/resources/worksheet-ismp-targeted- hensive method for characterizing pharmacists’
medication-safet y-best-practices-hospitals. clinical activities. Am J Health Syst Pharm
Accessed September 8, 2021. 1999;56:2444-50.
11. Patanwala AE, Narayan SW, Haas CE, et al.
Medication Use Evaluation Proposed guidance on cost-avoidance studies
1. American Society of Health-System Pharmacists in pharmacy practice. Am J Healty Syst Pharm
(ASHP). ASHP Guidelines on Medication-Use 2021;78:1559-67.
Evaluation. Available at https://www.ashp.org/-/ 12. Pawloski P, Cusick D, Amborn L. Development
media/assets/policy-guidelines/docs/guidelines/ of clinical pharmacy productivity metrics. Am J
m e d ic a t io n - u s e - e v a lu a t io n - c u r r e nt . a s h x . Health Syst Pharm 2012;69:49-54.
Accessed August 25, 2021. 13. Prelaski CR, Lat I, Poston J. Pharmacist contri-
butions as members of the multidisciplinary ICU
Documentation of Pharmacy Services team. Chest 2013;144:1687-95.
1. Andrawis M, Ellison C, Riddle S, et al.
Recommended quality measures for health-sys-
tem pharmacy: 2019 update from the Pharmacy Acknowledgment
Accountability Measures Work Group. Am J I would like to sincerely thank Dr. Henry Cohen for
Health Syst Pharm 2019;76:874-87. his contribution to this chapter. His willingness to col-
2. Brilli RJ, Spevetz A, Branson RD, et al. Critical laborate with me as we combined chapters for the 2016
care delivery in the intensive care unit: defining ACCP SCCM Critical Care Preparatory Course is
clinical roles and the best practice model. Crit Care greatly appreciated. Portions of this chapter contain our
Med 2001;29:2007-19. combined work during this collaboration.
3. Cassat S, Massey L, Buckingham S, et al.
Development of health-system inpatient phar-
macy clinical metrics. Am J Health Syst Pharm
2019;76:1958-64.
4. Dager W, Bolesta S, Brophy G, et al. An opinion
paper outlining recommendations for training,
credentialing, and documenting and justifying
critical care pharmacy services. Pharmacotherapy
2011;31:135e-75e.

ACCP Updates in Therapeutics® 2022: Critical Care Pharmacy Preparatory Review and Recertification Course

943
Practice Administration and Development: Protocol Development and Quality Improvement

ANSWERS AND EXPLANATIONS TO PATIENT CASES

1. Answer: B drug or disease, such as the review of a pharmacist’s


Using evidence-based practice helps provide support in notes in the EHR, review of the accuracy of expiration
creating a clinical pathway with support and agreement dates placed on intravenous admixture products, and
among practitioners (Answer B is correct). Using closed- review of the frequency of drug interaction warnings on
loop technology is important when implementing the CPOE system (Answers A, B, and D are incorrect).
a clinical pathway, as is establishing a physician
champion (Answer A). Developing a clinical protocol
and pathway comes after the evidence is evaluated
(Answer C). Formulary proposals are not a primary
factor in implementing a clinical pathway (Answer D).

2. Answer: B
Current data show that the critical care pharmacist
affects many areas associated with TJC measures.
According to TJC’s current measures, the critical
care pharmacist affects procoagulant reversal agent
initiation for ICH (Answer B is correct). The ED median
time from ED arrival to ED departure for admitted ED
patients is not affected by the pharmacist (Answer A
is incorrect). Hemorrhage transformation for patients
treated with intravenous alteplase therapy or mechanical
endovascular reperfusion therapy is also not affected by
the pharmacist (Answer C is incorrect). Although the
pharmacist would have impact on the severe sepsis and
septic shock management bundle, this is not currently a
required measurement (Answer D is incorrect).

3. Answer: A
Defining the problem is the first step in LEAN (Answer
A is correct). The LEAN strategy involves attempting to
remove the team members’ opinions so their emotions do
not influence the process. Hence, asking the nurses their
opinion for the spike in catheter-related bloodstream
infections influences their emotions (Answer B is
incorrect). Identifying operational barriers is a LEAN
process, however a transportation barrier is not related
to catheter related blood stream infections (Answer C is
incorrect). Describing the past process is not a LEAN
evaluation component (Answer D is incorrect).

4. Answer: C
An MUE is drug- or disease-specific and is best suited
for reviewing vancomycin dosing and the ordering of
vancomycin blood concentrations (Answer C is correct).
Quality assurance surveys are best suited for monitoring
the medication use process that may not be specific to a

ACCP Updates in Therapeutics® 2022: Critical Care Pharmacy Preparatory Review and Recertification Course

944
Practice Administration and Development: Protocol Development and Quality Improvement

ANSWERS AND EXPLANATIONS TO SELF-ASSESSMENT QUESTIONS

1. Answer: D therapy management and improve patient safety and


Financial justification for a critical care pharmacist has outcomes for drug-related processes—in this case,
been shown by the impact of clinical pharmacists on pharmacotherapy in SUP. Although one component
critically ill patients. The critical care pharmacist can of an MUE is PI, a review of the quality should occur
reduce adverse events in the critically ill population, before determining whether a PI project is necessary
reduce order prescribing errors, optimize the correct (Answer B is incorrect). An interventional MUE
drug for the correct disease process, decrease the LOS, incorporates a review of quality in the form of making
reduce medication administration errors, and reduce a pharmacotherapeutic intervention—PI. Although
inappropriate antibiotic use. One of the main financial reviewing ADE data and medication error data from the
impacts on critically ill patients is antimicrobial therapy. ICU is helpful in detecting and determining problems
Using an anticoagulation reversal stewardship program associated with the use of pharmacotherapy in SUP,
will most affect the institution (Answer D is correct). these are isolated events and are reporter-dependent,
Developing protocols is important in streamlining and a lack of reports does not ensure that the use of
practice in the critically ill population and can reduce pharmacotherapy in SUP is appropriate (Answers
health care costs (Answer A). Making prescribing C and D are incorrect). Only an MUE is a robust
errors (Answer B) and medication administration errors and comprehensive method of evaluating the use of
(Answer C) can result in adverse events to the patient, pharmacotherapy in SUP.
but these would not most significantly affect finances.
4. Answer: B
2. Answer: B Evaluating the management of warfarin-induced
Practice environment constraints include financial hypoprothrombinemia is best suited for an MUE. The
disincentives (not enough staff to support delirium goal of an MUE is to ensure optimal medication therapy
screening), organizational constraints (screening management and improve patient safety and outcomes
process takes up valuable nursing time), perception for drug-related processes; an MUE is drug-, drug
of liability (if the screen is conducted and the test is class–, or disease-specific—in this case, management of
positive, the provider will be blamed), and patient warfarin-induced hypoprothrombinemia (Answer B is
expectations (Answer B is correct). Another barrier correct). Quality assurance is a process for monitoring
to implementing delirium screening includes the key the effectiveness and safety of the medication use process
stakeholders’ standard or routine work of the process, that includes prescribing, dispensing, and administering
which is based on opinion, not evidence (Answer A medications. Evaluating pharmacist verification times
is incorrect). A third barrier involves the professional for routine orders in the ICU, drug interaction warnings
context and includes clinical uncertainty. Practitioners on the CPOE system, and duplicate warnings on the
may believe they have adequate skills to act. In addition, CPOE system is not necessarily drug- or disease state–
information overload occurs with the vast amount of specific and is best suited for a QA review (Answers A,
literature being published. An example of a knowledge C, and D are incorrect).
barrier includes providers being unfamiliar with how
to perform delirium screening (Answer C is incorrect). 5. Answer: A
A fourth barrier is when the key stakeholders’ opinion The next best step is determining which patients with
does not agree with screening patients for delirium cardiac arrest should receive targeted temperature
because of changes in evidence-based medicine and management. (Answer A is correct). The next step
possible knowledge gaps (Answer D is incorrect). after determining the patient population is to evaluate
the current clinical guidelines associated with targeted
3. Answer: A temperature management. (Answer B is incorrect).
Evaluating the use of pharmacotherapy in SUP in the Finally, data will be collected on the current process of
ICU is best suited for an MUE (Answer A is correct). managing targeted temperature management (Answer
The goal of an MUE is to ensure optimal medication

ACCP Updates in Therapeutics® 2022: Critical Care Pharmacy Preparatory Review and Recertification Course

945
Practice Administration and Development: Protocol Development and Quality Improvement

D is incorrect), which will be evaluated to determine


the areas for improvement in the outcome desired.
The current process has already been determined, as
provided in the question (Answer C is incorrect).

6. Answer: D
Data are analyzed in the analyze phase (Answer D is
correct). Measuring includes collecting data (Answer A
is incorrect). Establishing methods is part of the design
(Answer B is incorrect). The change process being
implemented is the improvement phase (Answer C is
incorrect).

7. Answer: A
Using weighted metrics for each critical care pharmacist
activity – quantified over time and annualized – more
accurately quantifies a critical care pharmacist’s
activities (Answer A is correct). The number of cardiac
arrest codes attended each month, number and severity
of hospital-acquired Clostridioides difficile infections
each year, and the number of in-services provided each
year are important, but do not quantify a comprehensive
list of a critical care pharmacist’s activities individually
(Answers B, C, and D are incorrect).

8. Answer: A
Sepsis management bundle and VTE prophylaxis are
quality measures reported to CMS (Answer A is correct).
Thrombolytic therapy for patients with stroke is correct,
but stress ulcer prophylaxis is not reported (Answer B
is incorrect). Heart failure readmissions are reported,
but pneumonia vaccination rates are not (Answer C is
incorrect). Use of ventilator bundles is not reported,
but rates of acute coronary syndrome readmission are
reported (Answer D is incorrect).

ACCP Updates in Therapeutics® 2022: Critical Care Pharmacy Preparatory Review and Recertification Course

946
Practice Administration and Development: Protocol Development and Quality Improvement

Appendix 1. Centers for Disease Control and Prevention (CDC) – The Core Elements of Hospital Antibiotic
Stewardship Programs: Antibiotic Stewardship Program Assessment Tool (excerpt)

Exert from Source: Centers for Disease Control and Prevention (CDC). The Core Elements of Hospital Antibiotic Stewardship Programs: Antibiotic Stewardship Program
Assessment Tool. Reference to specific commercial products, manufacturers, companies, or trademarks does not constitute its endorsement or recommendation by the U.S.
Government, Department of Health and Human Services, or Centers for Disease Control and Prevention. Available at the following website at no charge. https://www.cdc.
gov/antibiotic-use/healthcare/pdfs/assessment-tool-P.pdf. Accessed September 3, 2021.

ACCP Updates in Therapeutics® 2022: Critical Care Pharmacy Preparatory Review and Recertification Course

947
Practice Administration and Development: Protocol Development and Quality Improvement

Appendix 2. Example of Interventions and Clinical Pharmacy Services Cost Savings

Pharmacy Department
Data

Measure Benchmark 1st Quarter 2nd Quarter 3rd Quarter 4th Quarter
Pharmacotherapeutic
None 3451 5748 5015 7298
interventions
Outcomes cost and raw
drug cost savings from
None $477,867 $681,967 $652,145 $701,542
pharmacotherapeutic
interventions

Findings/Conclusion: For 2017, the total number of pharmacotherapeutic interventions remained the same from the
previous year with 20,940 interventions, realizing $846,245 in outcomes and raw drug cost savings. For 2018, the total
number of pharmacotherapeutic interventions remained similar to the prior year with 24,500 interventions, realizing
$950,471 in outcomes and medication use savings. For 2019, the total number of pharmacotherapeutic interventions
remained similar to the prior year with 23,452 interventions, realizing $904,254 in outcomes and raw drug cost savings.
For 2020, the total number of pharmacotherapeutic interventions increased with 35,459 interventions documented,
realizing $1,954,654 in outcomes and raw drug cost savings.

Pharmacotherapeutic interventions performed by clinical pharmacists consist of making downward and upward
dosing adjustments of medications, providing pharmacokinetic consultations, avoiding drug-drug and drug-
food interactions, avoiding toxic medications, avoiding drug-disease contraindications, avoiding drug-allergy
interactions, approving and dosing restricted antibiotics, switching patients from intravenous medications to oral
medications, initiating more effective or safer drug therapies, initiating equally efficacious but less expensive
medications, discontinuing unnecessary and duplicate medications, changing dosage forms according to patient
tolerance, switching nonformulary to formulary medications, and making recommendations to monitor for efficacy
and toxicity.

Clinical pharmacists review and respond to abnormal drug blood concentration assays and laboratory values such
as serum chemistry and coagulation profiles as they pertain to medication management. Notes documenting the
interventions are placed in the pharmacy profile. Depending on the quantity of pharmacotherapeutic interventions
and the resulting cost savings, the Pharmacy Department’s efforts to document clinical interventions, ensure
medication safety, and contain medication-related costs have been very effective.

Action Indicated: No additional actions are required. The Pharmacy Department will continue to perform and
document clinical interventions and evaluate the resulting cost savings.

ACCP Updates in Therapeutics® 2022: Critical Care Pharmacy Preparatory Review and Recertification Course

948

You might also like